Вы находитесь на странице: 1из 764

FIRST AID

Q&A FOR THE


USMLE
STEP 2 CK
Second Edition
SENIOR EDITORS EDITORS
TAO LE, MD, MHS HERMAN SINGH BAGGA, MD
Assistant Clinical Professor Resident
Chief, Section of Allergy and Clinical Immunology Department of Urology
Department of Medicine University of California, San Francisco Medical Center
University of Louisville
THOMAS L.H. HOCKER, MD
KRISTEN VIERREGGER, MD Resident
Resident Department of Dermatology
Department of Pathology Mayo Clinic
University of California, Irvine Medical Center
CHRISTOPHER R. KINSELLA, JR., MD
Research Fellow
University of Pittsburgh Medical Center

MATTHEW O'ROURKE, MD
Resident
Morgan Stanley Children's Hospital of New York
Presbyterian
Columbia University Medical Center

JOHN RHYNER, MD
Resident
Department of Internal Medicine
University of Michigan Medical Center

SADE CLARKE UDOETUK, MD


Resident
Department of Psychiatry
Baylor College of Medicine

New York / Chicago / San Francisco / Lisbon / London / Madrid / Mexico City
Milan / New Delhi / San Juan / Seoul / Singapore / Sydney / Toronto
Copyright 2010, 2008 by Tao Le. All rights reserved. Except as permitted under the United States Copyright Act of 1976, no part of this publication may be repro-
duced or distributed in any form or by any means, or stored in a database or retrieval system, without the prior written permission of the publisher.

ISBN: 978-0-07-162930-0

MHID: 0-07-162930-0

The material in this eBook also appears in the print version of this title: ISBN: 978-0-07-162571-5, MHID: 0-07-162571-2.

All trademarks are trademarks of their respective owners. Rather than put a trademark symbol after every occurrence of a trademarked name, we use names in an
editorial fashion only, and to the benefit of the trademark owner, with no intention of infringement of the trademark. Where such designations appear in this book, they
have been printed with initial caps.

McGraw-Hill eBooks are available at special quantity discounts to use as premiums and sales promotions, or for use in corporate training programs. To contact a
representative please e-mail us at bulksales@mcgraw-hill.com.

Medicine is an ever-changing science. As new research and clinical experience broaden our knowledge, changes in treatment and drug therapy are required. The authors
and the publisher of this work have checked with sources believed to be reliable in their efforts to provide information that is complete and generally in accord with the
standards accepted at the time of publication. However, in view of the possibility of human error or changes in medical sciences, neither the authors nor the publisher nor
any other party who has been involved in the preparation or publication of this work warrants that the information contained herein is in every respect accurate or
complete, and they disclaim all responsibility for any errors or omissions or for the results obtained from use of the information contained in this work. Readers are
encouraged to confirm the information contained herein with other sources. For example and in particular, readers are advised to check the product information sheet
included in the package of each drug they plan to administer to be certain that the information contained in this work is accurate and that changes have not been made
in the recommended dose or in the contraindications for administration. This recommendation is of particular importance in connection with new or infrequently used
drugs.

TERMS OF USE

This is a copyrighted work and The McGraw-Hill Companies, Inc. (McGraw-Hill) and its licensors reserve all rights in and to the work. Use of this work is subject to
these terms. Except as permitted under the Copyright Act of 1976 and the right to store and retrieve one copy of the work, you may not decompile, disassemble, reverse
engineer, reproduce, modify, create derivative works based upon, transmit, distribute, disseminate, sell, publish or sublicense the work or any part of it without McGraw-
Hills prior consent. You may use the work for your own noncommercial and personal use; any other use of the work is strictly prohibited. Your right to use the work may
be terminated if you fail to comply with these terms.

THE WORK IS PROVIDED AS IS. McGRAW-HILL AND ITS LICENSORS MAKE NO GUARANTEES OR WARRANTIES AS TO THE ACCURACY, ADEQUA-
CY OR COMPLETENESS OF OR RESULTS TO BE OBTAINED FROM USING THE WORK, INCLUDING ANY INFORMATION THAT CAN BE ACCESSED
THROUGH THE WORK VIA HYPERLINK OR OTHERWISE, AND EXPRESSLY DISCLAIM ANY WARRANTY, EXPRESS OR IMPLIED, INCLUDING BUT
NOT LIMITED TO IMPLIED WARRANTIES OF MERCHANTABILITY OR FITNESS FOR A PARTICULAR PURPOSE. McGraw-Hill and its licensors do not
warrant or guarantee that the functions contained in the work will meet your requirements or that its operation will be uninterrupted or error free. Neither McGraw-Hill
nor its licensors shall be liable to you or anyone else for any inaccuracy, error or omission, regardless of cause, in the work or for any damages resulting therefrom.
McGraw-Hill has no responsibility for the content of any information accessed through the work. Under no circumstances shall McGraw-Hill and/or its licensors be liable
for any indirect, incidental, special, punitive, consequential or similar damages that result from the use of or inability to use the work, even if any of them has been advised
of the possibility of such damages. This limitation of liability shall apply to any claim or cause whatsoever whether such claim or cause arises in contract, tort or
otherwise.
D E D I C AT I O N

To the contributors to this and future editions, who took time to share their
knowledge, insight, and humor for the benet of residents and clinicians.

and

To our families, friends, and loved ones, who supported us in the task of
assembling this guide.
This page intentionally left blank
CONTENTS

Authors vii

Faculty Reviewers ix

Preface xi

Acknowledgments xiii

How to Contribute xv

S EC T IO N I O R G A N SYST E M S 1

Chapter 1 Cardiovascular 3

Chapter 2 Dermatology 47

Chapter 3 Endocrinology 57

Chapter 4 Epidemiology and Preventive Medicine 91

Chapter 5 Ethics and Legal Issues 117

Chapter 6 Gastrointestinal 127

Chapter 7 Hematology/Oncology 167

Chapter 8 Infectious Disease 211

Chapter 9 Musculoskeletal 261

Chapter 10 Neurology 285

Chapter 11 Obstetrics 321

Chapter 12 Gynecology 351

Chapter 13 Psychiatry 375

Chapter 14 Pulmonary 409

Chapter 15 Renal/Genitourinary 441

S EC T IO N I I F U LL-LE N G T H E X A M I NAT I O N S 471

Test Block 1 473

Test Block 2 507

Test Block 3 539

v
Test Block 4 573

Test Block 5 607

Test Block 6 641

Test Block 7 675

Test Block 8 709

About the Authors 743

vi
AUTHORS

CYNTHIA ADAMS, MD LAURA MEINTS


Resident Vanderbilt University School of Medicine
Boston Combined Residency Program in Pediatrics Class of 2009
Children's Hospital Boston
ROBERT MICHELETTI, MD
CLARISSA BARNES, MD Resident
Resident Departments of Internal Medicine and Dermatology
Department of Internal Medicine Hospital of the University of Pennsylvania
Johns Hopkins University School of Medicine
DANIEL L. MILLER
RACHEL BORTNICK, MPHIL
Johns Hopkins University School of Medicine
Medical Scientist Training Program Class of 2009
Harvard Medical School Johns Hopkins Bloomberg School of Public Health
Class of 2009
CHRISTOPHER CHAPMAN, MD
Resident DEEPIKA NEMANI
Department of Internal Medicine University of Pennsylvania School of Medicine
University of Chicago Medical Center Class of 2009

LIA CLATTENBURG, MD, MPH


TIMOTHY NIESSEN
Resident
Johns Hopkins University School of Medicine
Department of Internal Medicine
Class of 2009
Union Memorial Hospital
Johns Hopkins Bloomberg School of Public Health
Baltimore, Md.
Class of 2009
GILLIAN DIERCKS
HOWARD O'ROURKE, MD
Columbia University College of Physicians and Surgeons
Class of 2009 Resident
Department of Diagnostic Radiology
VAHID ENTEZARI, MD University of Pittsburgh Medical Center

Postdoctoral Research Fellow


NISHANT PATEL
Orthopedic Biomechanics Laboratory
Beth Israel Deaconess Medical Center Johns Hopkins University School of Medicine
Class of 2009
JOHNATHAN ETHRIDGE
Johns Hopkins University School of Medicine ROHITH PIYARATNA, MD
Class of 2009 Resident
Department of Anesthesiology
ERICA Y. FAN Stanford University Medical Center
St. Louis University School of Medicine
Class of 2009 ANTHONY PRINCE, MD
Resident
CARL ERIK FISHER Department of Otolaryngology
Columbia University College of Physicians and Surgeons Mount Sinai Medical Center
Class of 2009
FIORELLA SAPONARA, MD
SATTAR GOJRATY, MD Resident
Resident Transitional Program
Department of Internal Medicine Maimonides Medical Center
Hospital of the University of Pennsylvania New York City

NILAY KAVATHIA, MD ASHA JAYENDRAKUMAR SHAH, MD


Resident Intern
Department of Internal Medicine Department of Internal Medicine
Thomas Jefferson University Hospital Emory University School of Medicine

vii
MONICA E. SHUKLA BRANT W. ULLERY, MD
Vanderbilt University School of Medicine Resident
Class of 2009 Department of Surgery
Hospital of the University of Pennsylvania
MEGHAN SISE
Columbia University College of Physicians and Surgeons KELLY VRANAS, MD
Class of 2009 Resident
Department of Internal Medicine
D'MITRI SOFIANOS, MD Hospital of the University of Pennsylvania
Resident
Department of Orthopedic Surgery DAVID WEI
University of Utah Medical Center Columbia University College of Physicians and Surgeons
Class of 2009
ANNA E. TEETER
Duke University School of Medicine ZACHARY ZAVODNI
Class of 2009
Duke University School of Medicine
Class of 2009
JOSHUA D. UDOETUK, MD
Transitional Intern
University of Texas-Houston Medical School

ASSOCIATE AUTHORS

MARINA FRIMER, MD SUNIL SHETH


Resident Harvard Medical School
Department of Obstetrics and Gynecology and Women's Health Class of 2009
Albert Einstein College of Medicine
LEANNE STANLEY
MARK J. MANN, MD Duke University School of Medicine
Resident Class of 2009
Department of Urology
State University of New York Upstate Medical University TIAN ZHANG
Syracuse, N.Y. Harvard Medical School
Class of 2009
PARIN J. PATEL, MD
Resident
Department of Internal Medicine
Hospital of the University of Pennsylvania

viiiviii
FACULTY REVIEWERS

JONATHAN W. BRESS, MD KERILYN MORGAN, MD


Nephrologist Associate Program Director
Philadelphia Hypertension & Nephrology Consultants Department of Internal Medicine
Banner Good Samaritan Medical Center
RACHEL CHONG, MD Phoenix, Ariz.
Endocrinologist
Lakeridge Health Corporation RINI BANERJEE RATAN, MD
Assistant Clinical Professor
PETER DANYI, MD, MPH, MBA Department of Obstetrics and Gynecology
Instructor, Hospitalist Program Columbia University College of Physicians & Surgeons
Division of General Internal Medicine
Johns Hopkins University School of Medicine ELIZABETH SASTRE, MD
Assistant Professor
ROBIN GIRDHAR, MD Vanderbilt University Medical Center
Vice Chairperson and Director of Quality Assurance Attending Physician
Division of Cardiology Medical Service, Primary Care
Shadyside Hospital Tennessee Valley Healthcare Veterans Administration Hospital
University of Pittsburgh School of Medicine
JERRY D. SMILACK, MD
ANDREW J. LENNEMAN, MD Emeritus Associate Professor of Medicine
Clinical Fellow Mayo College of Medicine
Cardiovascular Medicine Division Retired Consultant
Vanderbilt University School of Medicine Department of Internal Medicine
Mayo Clinic
ESTER C. LITTLE, MD
MYRA J. WICK, MD, PHD
Associate Director
Banner Liver Disease Center Department of Medical Genetics
Clinical Assistant Professor of Medicine Mayo Clinic
University of Arizona College of Medicine
APRIL ZHU, MD
MARCUS A. MCFERREN, MD, PHD The Permanente Medical Group
Department of Dermatology Santa Clara, Calif.
Yale-New Haven Hospital

ix
This page intentionally left blank
PREFACE

With First Aid Q&A for the USMLE Step 2 CK, we continue our commit-
ment to providing students with the most useful and up-to-date preparation
guides for the USMLE Step 2 CK. This addition to the First Aid series repre-
sents an outstanding effort by a talented group of authors and includes the fol-
lowing:

1,000 high-yield USMLE-style questions based on the top-rated


USMLERx Qmax Step 2 CK Test Bank (www.usmlerx.com)
Concise yet complete explanations to correct and incorrect answers
Organized as a perfect complement to First Aid for the USMLE Step 2 CK
Eight full-length test blocks simulate the actual exam experience
High-yield images, diagrams, and tables complement the questions and
answers
Timely updates and corrections at www.rstaidteam.com

We invite you to share your thoughts and ideas to help us improve First Aid
Q&A for the USMLE Step 2 CK. See How to Contribute, p. xv.

Louisville Tao Le
Irvine Kristen Vierregger

xi
This page intentionally left blank
ACKNOWLEDGMENTS

This has been a collaborative project from the start. We gratefully acknowl-
edge the thoughtful comments and advice of the medical students, residents,
international medical graduates, and faculty who have supported the authors
in the development of First Aid Q&A for the USMLE Step 2 CK.

Additional thanks to Neil Busis and Hey Chong for their review of the manu-
script.

For support and encouragement throughout the process, we are grateful to


Thao Pham, Selina Franklin, Louise Petersen, Jonathan Kirsch, and Vikas
Bhushan. Thanks to our publisher, McGraw-Hill, for the valuable assistance
of their staff. For enthusiasm, support, and commitment to this challenging
project, thanks to our editor, Catherine Johnson. For outstanding editorial
work, we thank Steve Freedkin, Isabel Nogueira, and Emma D. Underdown.
A special thanks to Rainbow Graphics for remarkable production work.

For contributions, corrections, and surveys we thank Juan F. Alvarez, M.R.


Brenz, Ericka Li Fuentes, Katherine Kline, Solomon Onyenkachukwu, Mat-
thew Swenson, and Jennifer Turley.

Louisville Tao Le
Irvine Kristen Vierregger

xiii
This page intentionally left blank
HOW TO CONTRIBUTE

To continue to produce a high-yield review source for the USMLE Step 2 CK exam, we invite you to
submit any suggestions or corrections. We also offer paid internships in medical education and publish-
ing ranging from three months to one year (see below). Please send us your suggestions for

Corrections or enhancements to existing questions and explanations


New high-yield questions
Low-yield questions to remove

For each entry incorporated into the next edition, you will receive a $10 gift certicate, as well as per-
sonal acknowledgment in the next edition. Diagrams, tables, partial entries, updates, corrections, and
study hints are also appreciated, and signicant contributions will be compensated at the discretion of
the authors.

The preferred way to submit entries, suggestions, or corrections is via our blog at:
www.rstaidteam.com
Otherwise, please send entries, neatly written or typed or on disk (Microsoft Word), to:
First Aid Q&A for the USMLE Step 2 CK , Second Edition
914 North Dixie Avenue, Suite 100
Elizabethtown, KY 42701

All entries become property of the authors and are subject to editing and reviewing. Please verify all data
and spellings carefully. In the event that similar or duplicate entries are received, only the rst entry re-
ceived will be used. Include a reference to a standard textbook to facilitate verication of the fact. Please
follow the style, punctuation, and format of this edition if possible.

I N T E R N S H I P O P P O RT U N I T I E S

The First Aid Team is pleased to offer part-time and full-time paid internships in medical education and
publishing to motivated medical students and physicians. Internships may range from three months (e.g., a
summer) up to a full year. Participants will have an opportunity to author, edit, and earn academic credit on
a wide variety of projects, including the popular First Aid and USMLERx series. Writing/editing experi-
ence, familiarity with Microsoft Word, and Internet access are desired. For more information, submit a r-
sum or a short description of your experience along with a cover letter to rstaidteam@yahoo.com.

xv
This page intentionally left blank
SECTION I

Organ Systems

Cardiovascular

Dermatology

Endocrinology

Epidemiology and
Preventive Medicine

Ethics and Legal Issues

Gastrointestinal

Hematology/Oncology

Infectious Disease

Musculoskeletal

Neurology

Obstetrics

Gynecology

Psychiatry

Pulmonary

Renal/Genitourinary

1
This page intentionally left blank
CHAPTER 1

Cardiovascular

3
HIGH-YIELD SYSTEMS
4 Section I: Organ Systems Questions

Q U E ST I O N S

1. A 66-year-old retired carpenter presents with (A) Blood glucose reduction


chronic shortness of breath upon exertion. He (B) Blood pressure reduction
has smoked one pack of cigarettes per day for (C) Serum cholesterol reduction
the past 5 years and drinks alcohol regularly. (D) Smoking cessation
Physical examination reveals a displaced point (E) Weight loss
of maximal impulse and hepatosplenomegaly.
His medications include pantoprazole for gas- 3. A 36-year-old man presents to the clinic with
troesophageal reux and sertraline for depres- complaints of a genital sore. The patient is a
sion. Echocardiogram reveals an ejection frac- sexually active heterosexual involved with
tion of 30% and dilated left and right ventricles. three partners and practices unprotected inter-
Laboratory tests show: course. Fours days ago he noted a painless sore
on his penis. He is afebrile, with a heart rate of
Cardiovascular

Na+: 129 mEq/L 80/min and blood pressure of 120/77 mm Hg.


K+: 5.2 mEq/L Physical examination reveals a solitary ulcer-
Cl: 101 mEq/L ated lesion located on the lateral aspect of his
Blood urea nitrogen: 45 mg/dL penis. The lesion is nontender and associated
Creatinine: 1.3 mg/dL with bilateral inguinal lymphadenopathy. Phys-
Glucose: 134 mg/dL ical examination is otherwise normal. If left
Aspartate aminotransferase: 220 U/L untreated, this man is at increased risk for
which of the following?
Alanine aminotransferase: 140 U/L
Alkaline phosphatase: 280 U/L (A) Ascending aortic aneurysm
(B) Coronary artery aneurysm
Which of the following is the most likely cause (C) Endocarditis
of his cardiac ndings? (D) Mitral valve stenosis
(A) Borrelia burgdorferi (E) Rupture of ventricular free wall
(B) Cigarette smoking
(C) Coxsackie B virus 4. An 81-year-old man is hospitalized for acute
(D) Ethanol onset of shortness of breath and lower extrem-
(E) Pantoprazole toxicity ity edema. Although he lives by himself, it is
(F) Trypanosoma cruzi very difcult for him to move around his apart-
ment without experiencing fatigue. He has not
2. A 52-year-old man presents to his primary care seen his physician in years but was told in the
physicians ofce for routine care. He has hy- past that he had high blood pressure. On physi-
pertension, hypercholesterolemia, and type 2 cal examination his jugular venous pulse is pal-
diabetes mellitus, and has smoked one pack of pated 9 cm above his sternal notch, inspiratory
cigarettes per day for the past 30 years. Medica- crackles are heard at his lung bases, and there
tions include hydrochlorothiazide, atorvastatin, is 3+ lower extremity edema. Which of the fol-
and glipizide. There is a family history of myo- lowing will conrm the most likely diagnosis?
cardial infarction in the maternal grandfather (A) Cardiac angiography
at age 60. The patient has undergone screen- (B) Echocardiography
ing for colon and prostate cancer. Physical ex-
(C) Electrocardiogram
amination reveals a pleasant, obese man who is
(D) Endomyocardial biopsy
175 cm (5 9) tall and weighs 108 kg (238 lb).
(E) Pulmonary function tests
His blood pressure is 155/81 mm Hg, heart rate
(F) X-ray of the chest
is 78/min, respiratory rate is 14/min, and tem-
perature is 36.8C (98.3F). What one action
would most reduce the patients stroke risk?
HIGH-YIELD SYSTEMS
Chapter 1: Cardiovascular Questions 5

5. A 42-year-old man presents to the clinic for


routine evaluation. His medical history is sig-
nicant for gallstones. The patient denies
smoking and drinks alcohol occasionally. His
mother had a heart attack at the age of 63
years. His blood pressure is 134/77 mm Hg.
The patient is overweight with well-healed lap-
aroscopic cholecystectomy scars. Fasting labo-
ratory tests show:
Aspartate aminotransferase: 37 U/L
Alanine aminotransferase: 28 U/L
Alkaline phosphatase: 88 U/L Reproduced, with permission, from PEIR Digital Library
Total cholesterol: 268 mg/dL (http://peir.net).

Cardiovascular
LDL cholesterol: 183 mg/dL (A) Aortic dissection
HDL cholesterol: 46 mg/dL (B) Exacerbation of chronic obstructive pul-
Triglycerides: 166 mg/dL monary disease
(C) Myocardial infarction
What is the most appropriate next step in man- (D) Pleural effusion
agement? (E) Pulmonary embolus
(A) A trial of lifestyle modication alone (diet,
exercise, and weight loss) 7. A 72-year-old man with coronary artery disease
(B) A trial of lifestyle modication combined and hypertension is hospitalized after suffering
with statin and niacin therapy a myocardial infarction 5 days ago. He sud-
(C) A trial of lifestyle modication combined denly complains of severe chest pain. His
with statin therapy blood pressure is 90/60 mm Hg and heart rate
(D) Niacin therapy is 65/min. Auscultation reveals no murmurs or
(E) Statin therapy rubs. An ECG reveals sinus rhythm with an
acute ST-segment elevation in the anteroseptal
6. Two and a half weeks after coronary artery by- area. Urgent bedside echocardiography showed
pass grafting, a 63-year-old man returns to the anteroseptal, lateral, and apical akinesis, mild
emergency department acutely short of breath. left ventricular systolic dysfunction, and severe
The patient states that he began having chest pericardial effusion. Within 20 minutes he is
pain and shortness of breath approximately 1 unconscious with undetectable pulses and
hour earlier. He has a history of hypertension, blood pressure. What is the most likely cause
diabetes, and two myocardial infarctions. On of the patients sudden decompensation?
examination he is hypoxic with an oxygen satu- (A) Free wall rupture
ration of 86% on room air. Other vital signs (B) Left ventricular thrombus
and results of a physical examination are nor- (C) Mitral regurgitation
mal. ECG shows no interval change from his (D) Pericarditis
most recent ECG. CT of the chest is shown in (E) Ventricular septal rupture
the image. What is the most likely etiology of
this patients shortness of breath?
HIGH-YIELD SYSTEMS
6 Section I: Organ Systems Questions

8. A 56-year-old woman was recently started on The duration of symptoms is now approxi-
medication for high blood pressure. At her next mately 30 minutes. What is the most appropri-
ofce visit her hypertension is under good con- ate treatment for this patient at this time?
trol, but she now complains of feeling strange
(A) Calcium channel blocker
since she started the medication. On further
(B) Intravenous angiotensin-converting en-
questioning, she reports feeling chest tightness
zyme inhibitor
several times over the past 2 weeks, and has
(C) Intravenous -blocker
also noticed pain in her elbows and knees. Her
(D) Magnesium sulfate
blood pressure is 124/78 mm Hg (146/82 mm
(E) Tissue plasminogen activator
Hg on last visit), heart rate is 102/min, and re-
spiratory rate is 14/min. Her examination is no-
11. A 70-year-old woman presents to the emer-
table for several erythematous plaques on the
gency department complaining of dizziness.
malar distribution of the face, arms, and upper
She is disoriented to the date and her location
torso. What medication was she most likely
and it is difcult to gather an accurate history.
Cardiovascular

started on during her last visit?


Her pulse is 48/min, blood pressure is 84/60
(A) Captopril mm Hg, and respiratory rate is 12/min. On
(B) Furosemide examination her extremities are cool and
(C) Hydralazine clammy. Her capillary rell time is 5 seconds.
(D) Metoprolol What is the most appropriate therapy?
(E) Verapamil
(A) Adenosine
(B) Amiodarone
9. A 19-year-old woman was attacked while com-
(C) Atropine
ing home from a party and is brought to the
(D) Isoproterenol
emergency department. She recalls being
(E) Metoprolol
punched in the side of the head and stabbed in
the left ank. Her speech is slow and she com-
12. A 77-year-old man, complaining of abdominal
plains of a bad headache. Her pulse is 110/
pain, anorexia, and nausea and vomiting over
min, blood pressure is 90/50 mm Hg, and re-
the past 24 hours, presents to the clinic with
spiratory rate is 25/min. On examination she
his son. The son reveals that his father has also
has a stab wound at the left costal margin in
complained of blurred vision. The patients vi-
the midaxillary line. Two large-bore intrave-
tal signs are stable and his abdomen is soft, but
nous lines are inserted, and after infusion of
he appears to be somewhat confused. He is
2 L of lactated Ringers solution her blood
currently taking metoprolol, digoxin, and hy-
pressure rises to 95/55 mm Hg. What is the
drochlorothiazide for ischemic congestive
most appropriate next step in management?
heart failure. His son says that sometimes his
(A) Abdominal ultrasound father confuses his medications. The patient
(B) Diagnostic peritoneal lavage also has renal insufciency with a baseline se-
(C) Exploratory laparotomy rum creatinine of 2.6 mg/dL. The ECG reveals
(D) Noncontrast CT of the head a widened QRS complex and a new rst-degree
(E) Peritoneal laparoscopy heart block. Which of the following is the most
likely cause of this patients symptoms?
10. A 48-year-old man presents to the emergency
(A) Digoxin toxicity
department complaining of crushing subster-
(B) Gastroenteritis
nal chest pain. He is diaphoretic, anxious, and
(C) Hypocalcemia
dyspneic. His pulse is 110/min, blood pressure
(D) Hypovolemia secondary to thiazide di-
is 175/112 mm Hg, respiratory rate is 30/min,
uretic overuse
and oxygen saturation is 94%. Aspirin, oxygen,
(E) Myocardial infarction
sublingual nitroglycerin, and morphine are
given, but they do not relieve his pain. ECG
shows ST-segment elevation in leads V2 to V4.
HIGH-YIELD SYSTEMS
Chapter 1: Cardiovascular Questions 7

13. A 35-year-old woman presents to the clinic be- A peripheral blood smear is shown in the im-
cause of visual problems. She states that she age. Which of the following is the most likely
has always had difculty looking up, and over diagnosis?
the past few years her overall vision has be-
come blurry. Review of symptoms is notable
for several recent episodes of near fainting.
She takes no medication and has no other
medical history, and has not seen a physician
for 7 years. Because she was adopted as a child,
she does not know her family history, but her
son has required special tutoring at school. The
patient also remarks that her son seems to have
been dropping objects lately. Physical exami-
nation reveals bilateral ptosis. Her extraocular

Cardiovascular
movements are intact and the pupils are equal,
round, and reactive. Her corrected visual acu- Reproduced, with permission, from Lichtman MA, Beutler E,
ity is 20/100 in the right eye and 20/120 in the Kipps TJ, Seligsohn U, Kaushansky K, Prchal JT. Williams
left eye. The view of the fundus is obscured. Hematology, 7th edition. New York: McGraw-Hill, 2006: Plate
III-2.
On ambulation she raises her knees and makes
a slapping sound on the oor as she walks. (A) Disseminated intravascular coagulation
ECG indicates heart block. What is the patho- (B) Factor V Leiden
genesis of this patients disorder? (C) Immune thrombocytopenic purpura
(A) Borrelia burgdorferi infection (D) Protein C deciency
(B) Deletion mutation in dystrophin (E) Thrombotic thrombocytopenic purpura
(C) Frameshift mutation in dystrophin
15. A 60-year-old man with coronary artery disease,
(D) Trinucleotide repeat expansion
peptic ulcer disease, and gout presents to the
(E) X-linked emerin deciency
emergency department with a 24-hour history
14. A college sophomore is found by his roommate of abdominal pain. The pain, which is most in-
to be poorly responsive and brought to the tense in the upper abdomen, was sudden in
emergency department. After resuscitation, the onset and has become progressively more se-
man complains of a severe headache and pho- vere. Free air in the abdomen is detected on
tophobia that is accompanied by dizziness, x-ray lms. The patient is in an agitated state.
nausea, vomiting, and neck pain. Physical ex- His extremities are cool and capillary rell
amination is noteworthy for positive Kernigs time is 3 seconds. His blood pressure is 80/40
and Brudzinskis signs as well as petechiae on mm Hg and heart rate is 130/min. The neck
the trunk and mucocutaneous bleeding. Labo- veins are at and the lungs are clear to auscul-
ratory studies show: tation. His hemoglobin is 13.8 g/dL. A urinary
catheter is inserted and 10 mL of urine is
WBC count: 17,000/mm drained. What is the most appropriate treat-
Hemoglobin: 11 g/dL ment for this patient at this time?
Platelet count: 70,000/mm (A) Broad-spectrum antibiotics for presumed
Bleeding time: 10 min sepsis
Prothrombin time: 17 sec (B) Infusion of isotonic uid
Activated partial thromboplastin time: 47 sec (C) Infusion of norepinephrine
Thrombin time: 18 sec (D) Inotropic support with dopamine, vaso-
pressin, or dobutamine
(E) Transfuse with 1 unit packed RBCs
HIGH-YIELD SYSTEMS
8 Section I: Organ Systems Questions

16. A 29-year-old woman presents to the emer- is performed. A gross view of the patients heart
gency department with a 3-week history of be- is shown in the image. Which of the following
ing awakened by a dull, prolonged chest pain is a risk factor for the type of lesion pictured?
that occurs 34 times a week. She is a smoker
but has never suffered a myocardial infarction
(MI) or had chest pain before and has no fam-
ily history of early MI. Results of a 12-lead
ECG are normal. Her rst set of cardiac en-
zyme measurements (creatine kinase, creatine
kinase-MB fraction, troponin I) are negative. If
coronary angiography were taken at the time of
her chest pain, which of the following ndings
is most like?
(A) Coronary artery spasm
Cardiovascular

(B) Greater than 80% stenosis in at least two


Reproduced, with permission, from Fauci AS, Braunwald E,
coronary arteries Kasper DL, Hauser SL, Longo DL, Jameson LJ, Loscalzo J,
(C) No abnormal ndings eds. Harrisons Online. New York: McGraw-Hill, 2008: Figure
(D) Plaque rupture and thrombosis 118-1.

17. A 42-year-old man presents to the emergency (A) Coronary artery disease
department with a complaint of increasing (B) Hypertension
shortness of breath when walking to get his (C) Mitral valve prolapse
newspaper, difculty breathing while lying at, (D) Prolonged bedrest
and a 4.5-kg (10-lb) weight gain over the past (E) Prosthetic valve replacement
month. He is afebrile, his pulse is 75/min, and
his blood pressure is 98/50 mm Hg. On exami- 19. A 28-year-old man with a history of intravenous
nation he smells of alcohol and has 2+ pitting drug abuse presents to the emergency depart-
edema in the lower extremities and a third ment with a 2-day history of fever, chills, and
heart sound. X-ray of the chest reveals cardio- shortness of breath. On physical examination
megaly. What additional ndings must be pres- the patient has a new heart murmur, small reti-
ent to conrm this mans underlying diagnosis? nal hemorrhages, and subungual petechiae.
Which of the following is the most likely caus-
(A) Hepatojugular reux and pulmonary con- ative organism?
gestion
(B) Left ventricular dilation and aortic insuf- (A) Group A Streptococcus
ciency (B) Mycobacterium tuberculosis
(C) Left ventricular dilation and systolic dys- (C) Staphylococcus aureus
function (D) Staphylococcus epidermidis
(D) Myocardial thickening and diastolic dys- (E) Streptococcus viridans
function
20. A boy is delivered at 37 weeks gestation via
(E) Pulmonary congestion and diastolic dys-
spontaneous vaginal delivery. He is the prod-
function
uct of a normal pregnancy and was delivered
18. A 69-year-old man with rheumatic heart dis- without complications. Prenatally the mother
ease presents to the emergency department was blood type B and was rubella immune and
complaining of a fever and weakness on his left negative for Rh antibody, group B streptococci,
side. On physical examination the patient is rapid plasma reagin, hepatitis B surface anti-
weak in his left upper extremity and he draws gen, gonorrhea, and chlamydia. The patient
only the right half of a clock. Shortly after his appears cyanotic. He is breathing at a rate of
presentation, the patient dies, and an autopsy 60/min and his heart rate is 130/min. He has a
HIGH-YIELD SYSTEMS
Chapter 1: Cardiovascular Questions 9

normal S1 and S2. There is a harsh holosys- 23. A 91-year-old woman presents to the emer-
tolic murmur that is loudest at the left lower gency department with a chief complaint of
sternal border. His examination reveals palpa- shortness of breath over the past 2 days. She
ble nonbounding peripheral pulses bilaterally. has a history of hypertension and coronary ar-
Which of the following is the most likely diag- tery bypass surgery 25 years earlier. Her blood
nosis? pressure is 178/92 mm Hg and she has jugular
venous distension, hepatomegaly, and 3+ lower
(A) Coarctation of the aorta
extremity edema. ECG is remarkable for left
(B) Dextraposed transposition of the great ar-
ventricular hypertrophy, no ST-segment eleva-
teries
tions or depressions, no Q waves, and no T-
(C) Patent ductus arteriosus
wave abnormalities. Echocardiogram reveals
(D) Tetralogy of Fallot
an ejection fraction of 60% and left atrial dila-
(E) Truncus arteriosus
tation. There is universal left ventricular thick-
21. A 32-year-old man is stabbed in the left chest ening. No valvular regurgitation or stenosis was

Cardiovascular
noted. Which of the following underlying con-
and presents to the emergency department in
ditions is the most likely cause of this patients
distress. His pulse is 130/min, blood pressure is
symptoms?
70/50 mm Hg, and respiratory rate is 39/min.
The stab wound is in the left fth intercostal (A) Hypertensive heart disease
space in the midaxillary line. On examination (B) Hypertrophic obstructive cardiomyopathy
his trachea is deviated to the right, jugular (C) Ischemic heart disease
veins are distended bilaterally, and he has ab- (D) Mitral valve prolapse
sent breath sounds and hyperresonance to per- (E) Myocarditis
cussion on the left side. Subcutaneous emphy-
sema is palpated on the left thoracic wall. 24. A 39-year-old white man with essential hyper-
What is the best next step in management? tension presents for a routine health mainte-
nance visit. He has no complaints and reports
(A) Chest tube thoracotomy
compliance with his hydrochlorothiazide. His
(B) Diagnostic peritoneal lavage
pulse is 70/min, blood pressure is 145/92 mm
(C) Needle thoracostomy
Hg, and respiratory rate is 16/min. His body
(D) Pericardiocentesis
mass index is 24 kg/m. His physical examina-
(E) Surgical exploration
tion is within normal limits. For which condi-
22. A 75-year-old man comes into the emergency tion is the patient at increased risk?
department with a 10-minute history of crush- (A) End-stage renal disease
ing substernal chest pain radiating to his left (B) Hypercholesterolemia
arm. This man is well known to the staff due to (C) Hypertrophic cardiomyopathy
his long history of chest pain. His creatine (D) Second-degree Mobitz I atrioventricular
phosphokinase level is elevated and his tro- block
ponin T level is 0.4 ng/mL. Which of the fol- (E) Type 2 diabetes mellitus
lowing is the most likely diagnosis?
(A) Acute myocardial infarction
(B) Hypochondriasis
(C) Prinzmetals angina
(D) Stable angina
(E) Unstable angina
HIGH-YIELD SYSTEMS
10 Section I: Organ Systems Questions

25. An 83-year-old woman is being evaluated for


confusion. She was admitted 3 days ago after
having an acute MI. Her hospital course has
been complicated by narrow-complex ventricu-
lar tachycardia, which has nally been stabi-
lized on an antiarrhythmic medication. She
was also started on a post-MI protocol and an
antidepressant. One day after beginning these
medications, she begins to develop confusion
and slurred speech. Her temperature is 36.7C
(98.1F), blood pressure is 138/60 mm Hg,
pulse is 88/min, and respiratory rate is 14/min.
She is alert and oriented to person, but she Reproduced, with permission, from Hall JB, Schmidt GA,
does not realize she is in the hospital. Addition- Wood LDH. Principles of Critical Care, 3rd edition. New
York: McGraw-Hill, 2005: Figure 28-8.
Cardiovascular

ally, she exhibits difculty with word articula-


tion, although she speaks uently, and she (A) Cardiac tamponade
demonstrates a mild resting tremor. The re- (B) Decompensated congestive heart failure
mainder of her examination is normal. Which (C) Panic attack
of the following medications is most likely to (D) Pericarditis
cause these central nervous system effects? (E) Tension pneumothorax
(A) Aspirin
27. An elderly man presents to the emergency de-
(B) Enalapril
partment with chest pain. He has a history of
(C) Fluoxetine
stable angina and recent onset diabetes melli-
(D) Lidocaine
tus, but now the chest pain comes on with less
(E) Metoprolol
exertion and takes longer to go away. An ECG
26. A 43-year-old woman presents to the emer- and cardiac enzymes are ordered. If this man
gency department because of chest pain, short- has unstable angina, what are the expected
ness of breath, and worsening fatigue for the ndings on ECG and cardiac enzyme testing?
past day. The chest pain initially worsened with (A) Delta waves on the ECG and elevated car-
lying down and improved with leaning for- diac enzyme levels
ward, but now it seems equal in intensity over (B) Low voltage ECG and elevated cardiac en-
all positions. On physical examination she has zyme levels
labored, fast breathing and appears to be in (C) No changes on ECG and elevated cardiac
pain. She has jugular venous distention. She is enzyme levels
tachycardic, has a regular rhythm, and has dis- (D) ST-segment depressions on ECG and nor-
tant heart sounds with a friction rub. Her lungs mal cardiac enzyme levels
are clear to auscultation bilaterally, her abdom- (E) ST-segment elevations with Q waves and
inal examination is benign, and she has no pe- normal cardiac enzyme levels
ripheral edema. Her temperature is 39.0C
(102.2F), pulse is 126/min, blood pressure is 28. A 19-year-old man complains of chest pain
89/66 mm Hg, respiratory rate is 32/min, and while playing basketball on his high school
oxygen saturation is 98% on room air. X-ray of team. Paramedics are called and he is rushed
the chest is shown in the image. Which of the to the hospital. Physical examination reveals
following is the most likely diagnosis? moderate mitral regurgitation and a crescendo-
decrescendo systolic ejection murmur that gets
louder with Valsalva maneuver. Echocardiog-
raphy reveals thickened left ventricular walls
and dynamic left ventricular outow tract ob-
struction. What is the best rst step in manage-
ment?
HIGH-YIELD SYSTEMS
Chapter 1: Cardiovascular Questions 11

(A) -Blockers nauseous. His temperature is 37.5C (99.5F),


(B) Calcium channel blockers pulse is 112/min, blood pressure is 142/85 mm
(C) Partial excision of the interventricular sep- Hg, and respiratory rate is 22/min. He tests pos-
tum itive for MI by serial cardiac enzymes. He is
(D) Warfarin started on the appropriate therapy and is ready
for discharge the following evening. What is
29. A 47-year-old woman who is 2 weeks post triple the number one preventive measure this pa-
bypass surgery presents to the emergency de- tient can take to decrease his immediate risk
partment with a chief complaint of sudden on- for a second MI?
set, sharp chest pain for several hours. She is
(A) Decrease the amount of cholesterol in his
fatigued and short of breath. On physical ex-
diet
amination she has distended neck veins that
(B) Exercise three times a week
grow more distended on inspiration. Mufed
(C) Lower his blood pressure to the 120/80
heart sounds are heard. Her temperature is
mm Hg range

Cardiovascular
37.0C (98.6F), pulse is 133/min, blood pres-
(D) Lower his blood sugar levels to achieve a
sure is 70/50 mm Hg, respiratory rate is 30/
hemoglobin A1c level <7%
min, and oxygen saturation is 100% on room
(E) Quit smoking
air. An echocardiogram shows a large pericar-
dial effusion and chamber collapse; therefore,
31. A 37-year-old woman with sarcoidosis presents
pericardiocentesis is performed. Although a
to her primary care physician complaining of
large amount of blood is aspirated, the patients
progressive fatigue and shortness of breath over
clinical picture acutely worsens. Her pain level
the past 3 months. She also reports that her
increases substantially; pulse is 150/min, blood
socks and shoes do not t the way they used to
pressure is 60/41 mm Hg, respiratory rate is 30/
and that she fainted a few weeks ago for the
min, and oxygen saturation is 100%. Repeat
rst time in many years. She denies any recent
echocardiography shows an even larger peri-
illness and only takes medications to control
cardial effusion with chamber collapse. Which
her sarcoid. She states that she is more com-
complication of pericardiocentesis is most
fortable sitting than lying down. She has jugu-
likely in this patient?
lar venous distension, which increases with in-
(A) Acute left ventricular failure with pulmo- spiration. Her blood pressure is 134/87 mm
nary edema Hg, respiratory rate is 17/min, pulse is 96/min,
(B) Aspiration of 10 mL air into the pericar- and temperature is 37.2C (98.9F). She also
dium has decreased breath sounds bilaterally at the
(C) Laceration of a coronary vessel bases. ECG shows decreased QRS voltage. An
(D) Pneumothorax echocardiogram shows a thick left ventricle.
(E) Puncture of the left ventricle Which of the following is the most likely diag-
nosis?
30. A 57-year-old man presents to the emergency
(A) Aortic stenosis
department with worsening substernal chest
(B) Cardiac tamponade
pain occurring over the past 20 minutes. He
(C) Hypertensive heart disease
has a medical history signicant for a 2-pack-
(D) Pericarditis
per-day smoking history, gout, obesity, hyper-
(E) Restrictive cardiomyopathy
cholesterolemia, hypertension, osteoarthritis of
both knees, inammatory bowel disease, and
recently diagnosed type 2 diabetes mellitus that
is well controlled on oral antiglycemics (hemo-
globin A1c of 7.8%). On physical examination
he is in moderate distress, diaphoretic, and
HIGH-YIELD SYSTEMS
12 Section I: Organ Systems Questions

32. A 64-year-old man in the surgical intensive care was 179/97 mm Hg. Since then she has ad-
unit goes into rapid atrial brillation on postop- hered to a low-fat diet and exercises regularly.
erative day one after a decortication for a locu- On repeat measurement 1 month later, her
lated pulmonary empyema. He is given an ap- blood pressure was still elevated, despite her
propriate loading dose of digoxin, but 4 hours compliance with the prescribed hydrochloro-
after his second dose, the patient complains of thiazide and lisinopril. She has no complaints
increased palpitations and dizziness. The patient and denies headaches, chest pain, or mental
is conscious and hemodynamically stable. STAT status changes. On physical examination she is
serum blood tests show a potassium level of 5.0 a slender woman in no apparent distress. An
mEq/L; all other electrolytes, including diva- abdominal bruit that lateralizes to the left is
lents, are in the normal range. The digitalis level heard. Her blood pressure is 178/99 mm Hg in
is above the therapeutic range at 4 ng/mL (thera- her left arm and 181/95 mm Hg in her right
peutic range 0.5-2 ng/mL). Results of cardiac te- arm. A basic metabolic panel and complete
lemetry are shown in the image. Which of the blood count are within normal range. Which
Cardiovascular

following should be administered immediately? of the following is the most appropriate next
step in patient care?
(A) Add a statin to the patients current drug
regimen to decrease fatty arterial plaques
(B) Admit patient to the hospital and start in-
travenous nitroprusside
Reproduced, with permission, from USMLERx.com. (C) Increase the dosage of her antihypertensive
(A) Calcium regimen
(B) Furosemide (D) Order duplex imaging of the renal arteries
(C) Magnesium and proceed to percutaneous transluminal
(D) Potassium angioplasty if renal artery stenosis is found
(E) Sodium polystyrene sulfonate (E) Order duplex imaging of the renal arteries
and proceed to surgical revascularization if
33. A 1-week-old infant presents to her general pe- renal artery stenosis is found
diatricians ofce for a well-child visit. She was
born at 37 weeks gestation without complica- 35. A 49-year-old man presents to the clinic for a
tions. Her temperature is 37.0C (98.6F), health maintenance visit. He has no com-
pulse is 130/min, blood pressure is 72/54 mm plaints, but he requests a prescription for his
Hg, and respiratory rate is 28/min. She is cur- pressure pills, as he lost his original prescrip-
rently at the 50th percentile for weight and tion. On physical examination his blood pres-
75th percentile for height. She is acyanotic and sure is 220/130 mm Hg. Physical examination
has a wide, xed split S2, with a 2/6 systolic is otherwise within normal limits. Laboratory
ejection murmur at the left upper sternal bor- tests show:
der. The remainder of the examination is unre- Na+: 142 mEq/L
markable. Which of the following is the most
K+: 3.8 mEq/L
likely diagnosis?
Cl: 105 mEq/L
(A) Atrial septal defect Carbon dioxide: 25 mEq/L
(B) Coarctation of the aorta Blood urea nitrogen: 20 mg/dL
(C) Dextratransposition of the great arteries
Creatinine: 1.0 mg/dL
(D) Tetralogy of Fallot
(E) Ventricular septal defect Glucose: 133 mg/dL
Urinalysis is within normal limits, and his
34. A 26-year-old white nonsmoking woman re- ECG is normal. Which of the following is the
turns for a follow-up appointment with her pri- most effective management?
mary care provider. At a routine health mainte-
nance visit 8 months earlier, her blood pressure
HIGH-YIELD SYSTEMS
Chapter 1: Cardiovascular Questions 13

(A) Administer intravenous nitroprusside for his primary care provider for a routine health
management of hypertensive emergency maintenance visit. He reports that he feels
(B) Administer intravenous nitroprusside for better than ever and explains that his daugh-
management of hypertensive urgency ter made him come in for his annual visit. He
(C) Administer oral furosemide for manage- takes prazosin daily and occasionally some
ment of hypertensive emergency acetaminophen. He has no drug allergies and
(D) Administer oral metoprolol for manage- denies smoking, drinking, or the use of illicit
ment of hypertensive urgency drugs. His physical examination is within nor-
(E) Administer sublingual nifedipine for man- mal range except for his rectal examination,
agement of hypertensive emergency which revealed an enlarged prostate. His tem-
perature is 36.8C (98.2F), respiratory rate is
36. A 58-year-old man is admitted to the coronary 13/min, pulse is 82/min, and blood pressure is
care unit for telemetric monitoring after an ep- 138/86 mm Hg. Which of the following is the
isode of bradycardia. While in the unit, he sud- most likely adverse effect of 1-adrenergic

Cardiovascular
denly loses consciousness. His pulse is unde- blockade in this patient?
tectable and his blood pressure drops to 40
(A) Decreased urine ow
mm Hg. His airway is clear and patent, and he
(B) Increased blood pressure
is still breathing on his own. An ECG shows
(C) Increased sexual drive
electrical activity. Chest compressions are
(D) Irritability
started and he is quickly given a bolus of intra-
(E) Orthostatic hypotension
venous sodium bicarbonate and atropine.
When his tracing does not improve, the bo-
38. A 2-year-old girl is referred to the hospital for
luses are repeated twice, and nally his tracing
evaluation of her inability to gain weight. She
returns to normal sinus rhythm. Moments
is well fed by her parents, but appears to tire
later, when he regains consciousness, he com-
during feedings and has been losing weight de-
plains of a dry mouth, blurred vision, and feel-
spite frequent high-calorie meals. There is no
ing ushed. What is the most appropriate next
family history of developmental delay or short
step in the management of this patient?
stature. She is well dressed, her hair is brushed,
(A) This patient has atropine toxicity and re- and she is playful but tires quickly. Her tem-
quires urgent administration of a cholin- perature is 36.5C (97.7F), pulse is 110/min,
ergic agonist blood pressure is 90/50 mm Hg, and respira-
(B) This patient has atropine toxicity and re- tory rate is 24/min. She has a harsh 2/6 holosys-
quires urgent administration of a musca- tolic murmur that is best heard at the left ster-
rinic agonist nal border, which is unchanged and has been
(C) This patient has bicarbonate toxicity and present since birth. Which of the following is
requires urgent administration of calcium the most appropriate next step in manage-
citrate ment?
(D) This patient is experiencing transient ad-
(A) Continue to monitor the patient for in-
verse effects of atropine and requires only
creased weight loss and increased shunting
supportive measures
(B) pH probe for gastroesophageal reux dis-
(E) This patient is experiencing transient ad-
ease
verse effects of bicarbonate and requires
(C) Refer for evaluation and possible closure
only supportive measures
of ventricular septal defect
(D) Skeletal survey
37. A 62-year-old man with a history of benign pros-
(E) Stool culture
tatic hyperplasia and hypertension presents to
HIGH-YIELD SYSTEMS
14 Section I: Organ Systems Questions

39. A 72-year-old man presents with shortness of drug use and states that he has been feeling
breath and increased home oxygen require- well until recently, when he began to sleep
ment. The patient has coronary artery disease, with more pillows and to become out of breath
he has had two previous myocardial infarc- when walking. His blood pressure is 143/89
tions, and he has a history of chronic obstruc- mm Hg, respiratory rate is 21/min, pulse is
tive pulmonary disease requiring 2 L of contin- 112/min, and he is afebrile. On physical exam-
uous home oxygen. The patient has a ination he has a laterally displaced point of
45-pack-year history of smoking. He is unable maximal impulse and an S3 gallop, as well as
to walk more than a block and the swelling in rales over his right lung base. X-ray of the chest
his legs has worsened. The physician suggests shows cardiomegaly and a pleural effusion.
measuring a brain natriuretic peptide (BNP) Echocardiogram reveals an ejection fraction of
level to distinguish a cardiac from a pulmonary 25%. Which of the following is the most likely
cause of his symptoms. Which of the following diagnosis?
statements regarding BNP is true?
(A) Asthma exacerbation
Cardiovascular

(A) BNP acts to decrease venous capacitance (B) Delirium tremens


and increase preload (C) Dilated cardiomyopathy
(B) BNP is decreased in the setting of left ven- (D) Endocarditis
tricular dysfunction (E) Hypothyroidism
(C) BNP is secreted by the cardiac atria
(D) BNP is secreted in response to hypovo- 42. A cardiologist is called to consult on the care of
lemia a 2-day-old girl delivered at 33 weeks gesta-
(E) BNP levels cannot differentiate systolic tion. The infant is lying supine in her isolette.
and diastolic dysfunction She is acyanotic, but has a heart rate of 192/
(F) BNP secretion results in pressure overload min and a respiratory rate of 60/min. She has a
nonradiating continuous machinery murmur
40. A 77-year-old man with a history of hyperten- at the left upper sternal border that remains the
sion, hypercholesterolemia, chronic obstruc- same with compression of the ipsilateral, then
tive pulmonary disease, and a 90-pack-year contralateral jugular veins. S1 and S2 are nor-
smoking history presents to the emergency de- mal. Her peripheral pulses are bounding.
partment with lethargy and abdominal pain. Which of the following is the most likely diag-
His temperature is 36.9C (98.5F), blood nosis?
pressure is 82/54 mm Hg, pulse is 125/min,
(A) Aortic stenosis with aortic regurgitation
and respiratory rate is 16/min. A pulsatile ab-
(B) Patent ductus arteriosus
dominal mass is palpable just superior to the
(C) Systemic arteriovenous stula
umbilicus. There is diffuse abdominal tender-
(D) Venous hum
ness, although rebound tenderness and guard-
(E) Ventricular septal defect
ing are absent. There is also slight skin discol-
oration noted in the left lower back. Which of
43. A 64-year-old white man with type 2 diabetes
the following is the most likely diagnosis?
mellitus presents to the emergency department
(A) Aortic dissection after passing out. While climbing the stairs,
(B) Mesenteric ischemia he felt dyspneic and lost consciousness as he
(C) Perforated gastric ulcer reached the top. Before the event he experi-
(D) Ruptured abdominal aortic aneurysm enced no palpitations or change in vision.
(E) Stroke When he awoke he felt alert and called for
help immediately. He reports dyspnea on exer-
41. A 52-year-old African-American man with a tion for the past year. He has no history of chest
history of smoking and asthma presents to the pain, seizures, or recent illnesses. Medications
emergency department complaining of short- include calcium and vitamin D supplements,
ness of breath. He has alcohol on his breath glyburide, and synthroid. He is allergic to peni-
and admits to drinking 34 beers each night cillin. Family history is noncontributory. He
plus an occasional mixed drink. He denies drinks wine socially and does not smoke or use
HIGH-YIELD SYSTEMS
Chapter 1: Cardiovascular Questions 15

illicit substances. His blood pressure is 136/92 ining her life. Which of the following is the
mm Hg, heart rate is 88/min, respiratory rate is most likely complication if the patients condi-
14/min, and temperature is 36.5C (97.7F). tion remains untreated?
Physical examination reveals a systolic crescendo-
(A) Cardiac tamponade
decrescendo murmur best heard in the second
(B) Decreased jugular venous pressure
right intercostal space with a soft S2. ECG shows
(C) Mitral regurgitation
nonspecic ST-segment changes and left ven-
(D) Restrictive cardiomyopathy
tricular hypertrophy with a normal heart rate
(E) Septic shock
and rhythm. Which of the following interven-
tions will most likely reveal the cause of this syn-
45. A 60-year-old man with a history of congestive
copal event?
heart failure presents to his physician. He has a
(A) ECG 5-year history of excessive daytime sleepiness
(B) Echocardiography and snoring. He also admits to three drinks of
(C) Electroencephalography alcohol per day. His temperature is 36.6C

Cardiovascular
(D) Exercise stress test with echocardiogram (98.0F), pulse is 85/min, blood pressure is
(E) Sublingual nitroglycerin and serial cardiac 138/82 mm Hg, respiratory rate is 14/min, and
enzymes oxygen saturation is 99% on room air. His body
(F) Tilt test mass index is 31 kg/m. Physical examination
is signicant for macroglossia and a short neck.
44. A 56-year-old woman with a history of chronic Polysomnography is performed and is signi-
renal disease presents to the emergency depart- cant for multiple nocturnal episodes of airow
ment because of severe, sharp, retrosternal cessation at the nose and mouth, despite evi-
chest pain that radiates to her jaw. The pain dence of continuing respiratory effort. Which
worsens when the patient lies down, and she is of the following is the most effective manage-
most comfortable leaning forward and hugging ment for this patient?
her knees. She takes erythropoietin, furo-
(A) Avoidance of alcohol
semide, calcitriol, and sodium polystyrene sul-
(B) Avoidance of supine posture
fonate. She is scheduled for dialysis three times
(C) Nasal continuous positive airway pressure
per week, but she admits to sometimes missing
(D) Uvulopalatopharyngoplasty
sessions. She stopped drinking and smoking 20
(E) Weight reduction
years ago, and she has no family history of
heart or renal problems. Auscultation of the
46. During a screening physical examination for
heart reveals a friction rub. Laboratory tests
participating in high school sports, a 14-year-
show:
old girl is found to have a late apical systolic
WBC count: 12,000/mm3 murmur preceded by a click. The rest of the
Hemoglobin: 10.0 g/dL examination is unremarkable. Echocardiogra-
Hematocrit: 30.0% phy shows superior displacement of the mitral
leaets of >2 mm during systole into the left
Platelet count: 150,000/mm3
atrium, with a thickness of at least 8 mm. In
Na+: 141 mEq/L addition, she states that her father also has
K+: 4.8 mEq/L some type of heart murmur, but she knows
Cl: 101 mEq/L nothing else about it. Which of the following is
HCO3: 22 mEq/L the most appropriate management at this time?
Blood urea nitrogen: 63 mg/dL (A) Digoxin
Creatinine: 3.2 mg/dL (B) Instruct the patient to avoid all forms of
Glucose: 111 mg/dL strenuous activity
The emergency medicine physician urges the (C) Metoprolol
patient to be more compliant with her dialysis, (D) Mitral valve replacement
but the patient complains that she is too tired (E) Prophylactic antibiotics for dental proce-
to go to dialysis all of the time and that it is ru- dures
HIGH-YIELD SYSTEMS
16 Section I: Organ Systems Questions

47. A 20-year-old woman arrives at the emergency 49. A 70-year-old man comes to his primary care
department actively seizing with QRS prolon- physician for his annual check-up. He has a
gation on ECG per paramedics. The patients history of hypertension, hyperlipidemia, and
roommate called emergency medical services coronary artery disease, and had coronary ar-
after the patient collapsed, was not responsive tery bypass grafting 6 years ago. On examina-
to questioning, and began having clonic jerks tion the physician notes a right carotid bruit.
bilaterally in her upper extremities. The pa- Which of the following is the most appropriate
tients roommate denies any knowledge of the next step?
patient consuming alcohol or illicit drugs. She
(A) Carotid duplex ultrasound
does not believe the patient had any plan of
(B) Carotid endarterectomy
harming herself, but does acknowledge that
(C) ECG
the patient has seemed down lately and was
(D) Referral to a neurologist
recently prescribed medication for generalized
(E) Transthoracic echocardiography
anhedonia. Which of the following is the most
(F) Warfarin therapy
Cardiovascular

appropriate rst-line treatment?


(A) Activated charcoal 50. A 32-year-old man is brought to the emergency
(B) Diazepam department by paramedics after being found
(C) Flumazenil wandering downtown, apparently delirious and
(D) Physostigmine agitated. During transport to the hospital the
(E) Sodium bicarbonate and diazepam patient becomes diaphoretic and tremulous
and has a blood pressure to 163/100 mm Hg,
48. A 78-year-old woman presents to a nursing pulse of 102/min, and temperature of 39C
home physician complaining of palpitations (102.2F). On examination the patient has di-
over the past several months. Her episodes are lated pupils and ulcerations of his nasal sep-
not associated with any chest pain, dizziness, tum mucosa with the residue of a white pow-
or loss of consciousness. The patient reports der along the nasal alae in addition to his
that she spent several weeks in the hospital as a tachycardia, hypertension, hyperthermia, and
child with rheumatic fever. ECG is shown in agitation. Which of the following is the reason
the image. Which of the following is the most why nonselective -blockers should be avoided
likely diagnosis? in this patient?
(A) Increased risk of late vasospasm
(B) Risk of acutely worsening hypertension
through vasoconstriction
(C) Risk of causing acute hypotension
(D) Risk of causing dyspnea
(E) Risk of ventricular arrhythmia

51. A 59-year-old man presents to his internist for a


routine visit. He has no complaints, and review
of symptoms is negative. His past medical his-
Reprinted, with permission, from Crawford MH. Current Di- tory is signicant for poorly controlled hyper-
agnosis & Treatment in Cardiology, 2nd edition. New York: tension for 15 years due to noncompliance
McGraw-Hill, 2003: Figure 20-1. with antihypertensive medications. He takes
hydrochlorothiazide 25 mg orally four times a
(A) Atrial brillation
day. His family history is signicant for hyper-
(B) Atrial utter
tension, heart failure, and stroke. He has a
(C) Multifocal atrial tachycardia
30-pack-year smoking history and drinks two
(D) Paroxysmal atrial tachycardia
beers a day. On physical examination he is a
(E) Paroxysmal supraventricular tachycardia
mildly obese man in no acute distress. He has
HIGH-YIELD SYSTEMS
Chapter 1: Cardiovascular Questions 17

a normal jugular venous pressure. He has a but no other medical history. The patient is not
prominent point of maximum impulse, regular able to relate any meaningful history. Blood
rate and rhythm, normal S1, loud S2, and au- pressure is 80/40 mm Hg, heart rate is 126/
dible S4 with no murmurs. His lungs are clear min, respiratory rate is 20/min, and oxygen sat-
to auscultation bilaterally, and he has no signs uration is 99% on room air; he is afebrile. His
of edema. His abdominal and neurologic ex- heart rate is irregularly irregular with no mur-
aminations are within normal limits. His tem- murs, clicks, or rubs. Respiratory examination
perature is 37.0C (98.6F), pulse is 81/min, is unremarkable. X-ray of the chest shows no
respiratory rate is 12/min, blood pressure is acute disease. ECG shows no discernible P
165/96 mm Hg, and oxygen saturation is 100% waves and an irregularly spaced QRS response.
on room air. His ECG shows normal sinus Which of the following is the best rst step in
rhythm with large amplitude of the S wave in management?
V1 and V2 and of the R wave in V5 and V6.
(A) Administration of adenosine
Also present are diffuse ST segment/T wave
(B) Cardiac catheterization and stent place-

Cardiovascular
changes, widened bid P waves, and prolonged
ment
QRS waveforms. Which of the following is the
(C) Cardioversion to sinus rhythm
most likely diagnosis?
(D) Carotid massage
(A) Acute myocardial infarction (E) Placement of dual lead pacemaker
(B) Cerebrovascular accident
(C) Dilated cardiomyopathy E X T E N D E D M ATC H I N G
(D) Left ventricular hypertrophy
(E) Pericarditis
The response options for the next 2 items are
52. A 60-year-old woman is transferred to a physi- the same. Select one answer for each item
cian from an outside hospital following a mo- in the set.
tor vehicle collision. Her medical history is no-
table for Osler-Weber-Rendu syndrome. She is For each patient with chest pain, select the most
otherwise healthy. Which of the following tri- likely diagnosis.
ads is most likely to characterize her medical
history prior to the collision?
(A) Acute aortic dissection
(A) Hypertension, bradycardia, and irregular (B) Acute myocardial infarction
respirations (C) Angina pectoris
(B) Jaundice, fever, and right upper quadrant (D) Cardiac tamponade
pain (E) Compression fracture of the spine
(C) Symptoms of hypoglycemia, low blood (F) Coronary vasospasm
sugar, and relief with increase in blood (G) Esophageal spasm
sugar (H) Myocarditis
(D) Telangiectasia, recurrent epistaxis, and (I) Panic disorder
positive family history (J) Pericarditis
(E) Venous stasis, hypercoagulability, and en- (K) Pneumonia
dothelial damage (L) Pulmonary embolus
(M)Rib fracture
53. A 65-year-old man presents to the emergency (N) Tension pneumothorax
department following the acute onset of palpi-
tations. His wife states that he was eating din-
ner when he noticed the palpitations, light-
headedness, and shortness of breath. The
patient has a history of treated hypertension,
HIGH-YIELD SYSTEMS
18 Section I: Organ Systems Questions

54. A 47-year-old man is brought to the emergency 57. A 75-year-old retired anesthesiologist with a
department via ambulance. He was found un- history of two previous myocardial infarctions
conscious and bleeding from a 4-cm penetrat- presents because of extreme fatigue upon exer-
ing wound over his lateral left chest. On admis- tion. He is unable to walk more than two
sion he is in respiratory distress and he is blocks and requires three pillows to sleep com-
tachycardic and hypotensive, with a blood pres- fortably at night. Physical examination reveals
sure of 68/43 mm Hg. The jugular venous jugular venous distension. He has previously
pulse is elevated and heart sounds are difcult been unable to tolerate enalapril due to exces-
to auscultate. sive coughing.

55. A 66-year-old woman presents to the emer-


The response options for the next 3 items are
gency department with chief complaints of
the same. Select one answer for each item
nausea, vague abdominal pain, and epigastric
in the set.
discomfort. The pain began while she was
Cardiovascular

climbing stairs earlier in the day and increased


For each patient with pericardial disease, select the
gradually. It was relieved after 10 minutes of
most effective management.
sitting down, but she remains concerned. Her
pulse is 105/min and blood pressure is 146/82 (A) Antibiotics
mm Hg. (B) Chest tube placement
(C) Colchicine
(D) Corticosteroids
The response options for the next 2 items are
(E) Emergency cardiac catheterization
the same. Select one answer for each item
(F) Emergent pericardiocentesis
in the set.
(G) Intravenous uids
(H) Loop diuretics
For each of the following patients with fatigue, se-
(I) Morphine
lect the most appropriate pharmacologic interven-
(J) Multidrug antituberculous therapy
tion.
(K) Nitroglycerin
(A) Candesartan (L) Nonsteroidal anti-inammatory drugs
(B) Digoxin (M)Renal dialysis
(C) Erythropoietin
(D) Folate 58. A 45-year-old man with chronic renal disease
(E) Heparin on dialysis presents with a chief complaint of
(F) Isoniazid sharp chest pain for several days that has not
(G) Lisinopril improved with acetaminophen. He also notes
(H) Metoprolol increasing fatigue and dyspnea over the past
(I) Warfarin few days with a bothersome cough. He nor-
mally is compliant with dialysis, but reports
56. A 36-year-old woman at 18 weeks gestation that he has missed his last three dialysis treat-
presents with a chief complaint of fatigue. Her ments. His temperature is 37.5C (99.5F),
history is signicant for leukemia 7 years ear- pulse is 85/min, blood pressure is 100/72 mm
lier that was treated successfully with a course Hg, respiratory rate is 20/min, and oxygen satu-
of doxorubicin chemotherapy. On physical ex- ration is 99% on room air. On physical exami-
amination she has bilateral rales throughout nation he appears in mild distress. He has
her lung elds and 3+ pitting edema. Echocar- slightly distended neck veins. His heart sounds
diography reveals a dilated left ventricular are mufed and a pericardial friction rub is
chamber and an ejection fraction of 40%. heard. An echocardiogram shows a large peri-
cardial effusion.
HIGH-YIELD SYSTEMS
Chapter 1: Cardiovascular Questions 19

59. A 58-year-old man with a history of angina and sharp, localized to the left side of her chest,
a positive stress test presents for cardiac cathe- and radiating to her jaw and neck. The pain
terization. During the procedure, one of his worsens when she lies down and improves on
coronary arteries is lacerated. The patient de- leaning forward. Her temperature is 37.5C
velops tachycardia and becomes hypotensive. (99.5F), pulse is 81/min, blood pressure is
The anesthesiologist notices that his systolic 139/81 mm Hg, respiratory rate is 15/min, and
pressure falls even further on inspiration. A oxygen saturation is 100% on room air. Physi-
bedside echocardiogram is performed and cal examination is signicant for a soft pericar-
shows a small pericardial effusion. dial friction rub. X-ray of the chest shows car-
diomegaly, and echocardiography shows a
60. An otherwise healthy 17-year-old girl presents moderate pericardial effusion.
to the emergency department because of 2
weeks of chest pain. She describes the pain as

Cardiovascular
HIGH-YIELD SYSTEMS
20 Section I: Organ Systems Answers

AN S W E R S

1. The correct answer is D. Dilated cardiomy- risk reduction for cerebrovascular accident as
opathy (DCM) is a common cause of conges- reducing hypertension.
tive heart failure (CHF). It is usually due to
Answer C is incorrect. Reducing serum cho-
causes such as ischemic heart disease or hyper-
lesterol does impact stroke risk but does not
tension, but in this case, it is likely due to the
have a greater impact on stroke risk than re-
toxic effects of chronic alcohol consumption.
ducing hypertension. Stroke prevention studies
The liver function tests and physical examina-
have implicated hypertension as the greatest
tion results are consistent with chronic alcohol-
contributing risk factor to stroke.
ism and alcoholic cirrhosis.
Answer D is incorrect. Smoking cessation
Answer A is incorrect. Lyme disease, caused
would improve the patients overall health and
by Borrelia burgdorferi, can induce DCM, but
longevity and should be encouraged. Cerebro-
Cardiovascular

there is nothing in the question stem to indi-


vascular accident can be multifactorial, but be-
cate this is the most likely etiology.
cause hypertension is believed to have the larg-
Answer B is incorrect. Cigarette smoking is a est effect on stroke risk, any stroke reduction
risk factor for coronary artery disease (CAD) program in this patient must effectively control
that can lead to ischemic cardiomyopathy. his hypertension.
However, his smoking history is not likely sig-
Answer E is incorrect. Weight loss would im-
nicant enough to lead to this series of events.
prove this patients glycemic control, as well
Answer C is incorrect. Coxsackie B virus is a as overall cardiovascular health, but would not
common cause of myocarditis, which can also have a larger impact on stroke risk than reduc-
lead to diastolic cardiomyopathy and CHF. ing hypertension.
This should be suspected in younger individu-
als without underlying medical problems. 3. The correct answer is A. The patient presents
with primary syphilis. The lesion is typically a
Answer E is incorrect. Pantoprazole is not as-
single painless papule that rapidly becomes
sociated with toxicity to the heart.
eroded and indurated. Chancres are usually lo-
Answer F is incorrect. Chagas disease, caused cated on the penis in heterosexual males but in
by Trypanosoma cruzi, can be responsible homosexual males may be found in the anal
for heart failure and dysrhythmias, but this is canal, mouth, or external genitalia. In females
highly unlikely given that the distribution of they may be seen on the cervix or labia. Serol-
the pathogen is mostly in Latin America. ogy or dark eld microscopy can be used to
conrm the diagnosis. If left untreated, the pa-
2. The correct answer is B. Hypertension is the tient may progress to secondary and tertiary
most important controllable risk factor for syphilis. Tertiary syphilis causes disruption of
stroke, and the stroke risk attributable to this the vasa vasorum or the aorta and consequent
patients high blood pressure is larger than any dilation of the aorta, often involving the aortic
other factor. The other answers, although im- root or ascending aorta. This can result in an-
portant for improving the patients health and eurysm of the ascending aorta and aortic valve
longevity, are less tightly correlated to reducing incompetence.
stroke risk.
Answer B is incorrect. Coronary artery aneu-
Answer A is incorrect. Blood glucose reduc- rysms are a sequela of Kawasakis disease. This
tion would lessen the patients risk for diabetic disease is seen more commonly in children.
complications, including retinopathy, neuropa- Symptoms include fever, congested conjunc-
thy, and nephropathy, but is not as signicant a tiva, lymphadenopathy, and changes in the lips
or oral mucosa.
HIGH-YIELD SYSTEMS
Chapter 1: Cardiovascular Answers 21

Answer C is incorrect. Complications of en- terol is 160 mg/dL or less. Therapeutic lifestyle
docarditis include chordae rupture, glomerulo- changes in the form of a 12-week trial of diet,
nephrits, pericarditis, and distal emboli. exercise, and weight loss should be attempted
given his current LDL cholesterol level.
Answer D is incorrect. Mitral valve stenosis is
seen as a late sequelae of rheumatic heart dis- Answer B is incorrect. Combination therapies
ease. may prove necessary for management of his
cholesterol at some later point. As this is his
Answer E is incorrect. This complication may
rst presentation, therapeutic lifestyle changes
be seen 410 days after a myocardial infarction
should be instituted as rst-line therapy.
(MI).
Answer C is incorrect. Combination therapies
4. The correct answer is B. The patient most may prove necessary for management of his
likely has an acute CHF exacerbation with the cholesterol at some later point. As this is his
underlying etiology being hypertension. rst presentation, therapeutic lifestyle changes
Echocardiography is an essential test in all pa-

Cardiovascular
should be instituted as rst-line therapy.
tients with newly diagnosed heart failure and is
Answer D is incorrect. Niacin is a cheap and
an excellent, noninvasive method of assessing
effective cholesterol-adjusting medication, par-
chamber size, function, and ejection fraction.
ticularly in raising levels of HDL cholesterol.
Answer A is incorrect. Cardiac angiography Adverse effects include ushing and pruritus
can determine cardiac pressures, but given the and this drug is often poorly tolerated. As with
invasive nature of the procedure, it is best re- the statins, drug therapy is not necessary given
served for patients with CHF in whom CAD is his lipid prole and risk factors.
suspected as the underlying cause.
Answer E is incorrect. While statin therapy is
Answer C is incorrect. ECG in heart failure effective in lowering LDL cholesterol, current
may show left ventricular hypertrophy (LVH) guidelines suggest that rst priority should be
in this patient, but is unlikely to be helpful if given to a trial of therapeutic lifestyle changes.
acute ischemia is not suspected. Drug therapy should be initiated at an LDL
Answer D is incorrect. Biopsy does play a role cholesterol level of 190 mg/dL or higher. He-
in the evaluation of heart failure, but is gen- patic dysfunction occurs in a small percentage
erally reserved for cases of unknown origin or of patients on statins, mostly within the rst
acute cases in young patients. Certain causes few months of treatment. The normal liver
function tests presented here are reassuring if
of heart failure such as inltrative disease can
statin therapy is instituted at a later time.
be conrmed by biopsy, but it is not likely to
be useful here.
6. The correct answer is E. Recent surgery and
Answer E is incorrect. Pulmonary function likely limited mobility in the postoperative pe-
tests have a role in the evaluation of dyspnea. riod are two risk factors for pulmonary em-
However, they are not necessary if a cardiac bolus. The enhanced CT scan of the chest
cause is strongly suspected. shows a lling defect within the right pulmo-
nary artery consistent with pulmonary em-
Answer F is incorrect. X-ray of the chest in
bolus. The patient should be treated with anti-
heart failure may show pulmonary congestion
coagulation. Other common risk factors for
or cardiomegaly, but these ndings are rela-
deep venous thrombosis and pulmonary em-
tively nonspecic.
bolus include malignancy, pregnancy, and hy-
5. The correct answer is A. You should be famil- percoagulable states.
iar with the goals of cholesterol-adjusting ther- Answer A is incorrect. There is no evidence of
apies. This patient has only one risk factor aortic dissection on the image.
(family history) and his goal of LDL choles-
HIGH-YIELD SYSTEMS
22 Section I: Organ Systems Answers

Answer B is incorrect. The patient does not Answer D is incorrect. Pericarditis would
have chronic obstructive pulmonary disease likely result in a pericardial rub on physical ex-
(COPD), and a COPD exacerbation would amination and it usually causes ECG changes.
cause wheezes on exam. Therefore this diag- The acute nature of this decompensation does
nosis is unlikely. not suggest pericarditis.
Answer C is incorrect. The patient is at an Answer E is incorrect. Although septal rupture
increased risk for an MI given his history of could result in acute decompensation, a new
diabetes and two prior MIs. However, this is harsh murmur could likely be picked up on
not the most likely diagnosis given the lack of physical examination. The electromechanical
change in his ECG and the presence of a pul- dissociation is more consistent with free wall
monary embolus on CT. rupture.
Answer D is incorrect. A pleural effusion is
8. The correct answer is C. This patient displays
certainly present in the CT scan. However, it
symptoms of angina, tachycardia, rash, and
Cardiovascular

does not appear large enough to cause such


joint pains. This lupus-like syndrome is a well-
acute chest pain, shortness of breath, and hy-
described adverse effect of hydralazine therapy.
poxemia. The effusion may be secondary to
The vasodilatory action of hydralazine can re-
the patients recent thoracic surgery.
sult in reex tachycardia and decreased oxygen
delivery to the myocardium in patients with ex-
7. The correct answer is A. Myocardial rupture is
isting CAD. Other agents known to cause a sys-
a sudden postinfarction complication that typi-
temic lupus erythematosus-like syndrome in-
cally occurs 510 days after an MI (peak at 7
clude isoniazid, procainamide, and phenytoin.
days). During this time the integrity of the car-
diac wall is compromised by macrophage and Answer A is incorrect. ACE inhibitors are
mononuclear inltration, brovascular re- most commonly associated with a dry cough
sponse, and other inammatory mediators, as (10%20% of people). Other important adverse
they replace necrotic tissue with scar tissue. effects include hyperkalemia (due to the block-
Old age, rst MI, and a history of hypertension ade of aldosterone secretion), angioedema, and
are risk factors. The clinical manifestations, as renal failure, especially in patients with known
seen here, are a sudden loss of heart rate, blood kidney disease.
pressure, and consciousness, while the ECG
Answer B is incorrect. In addition to the more
continues to show a sinus rhythm. Measures to
common adverse effects of hypotension and
prevent cardiac rupture include the administra-
hypokalemia, furosemide can lead to ototoxic-
tion of -blockers, angiotensin-converting en-
ity. It is not a known cause of tachycardia or a
zyme (ACE) inhibitors, and the avoidance of
lupus-like syndrome.
steroidal and nonsteroidal anti-inammatory
agents such as ibuprofen and indomethacin. Answer D is incorrect. -Blockers are associ-
ated with depression and erectile dysfunction.
Answer B is incorrect. An LV thrombus can
They can also facilitate hypoglycemia through
occur as a post-MI complication. Although an
their adrenergic blockade and lead to hyper-
embolus could result in stroke and subsequent
kalemia by similar mechanisms. Another im-
mental status change, this would not cause a
portant adverse effect of -blockers is an in-
sudden loss of pulses and blood pressure.
crease in pulmonary reactivity; as a result, they
Answer C is incorrect. Papillary muscle rup- are contraindicated in patients with asthma
ture will lead to sudden pulmonary edema, and chronic obstructive pulmonary disease.
shortness of breath, effusions and crackles, and Furthermore, -blockers lead to bradycardia,
a new mitral regurgitation murmur on chest not tachycardia.
examination.
Answer E is incorrect. Calcium channel block-
ers such as verapamil act as reverse chron-
HIGH-YIELD SYSTEMS
Chapter 1: Cardiovascular Answers 23

otropes, and thus can lead to bradycardia and Answer A is incorrect. Calcium channel
even atrioventricular block. Other common blockers have not been shown to affect mor-
adverse effects include gingival hyperplasia tality after MI. Reex sympathetic activation,
and constipation. There is no known associa- tachycardia, and hypotension associated with
tion with lupus. these drugs may even be harmful in some pa-
tients.
9. The correct answer is C. A stab wound in a
Answer B is incorrect. ACE inhibitors are im-
patient who is hemodynamically unstable re-
portant once the patient is stable. They limit
quires immediate exploratory laparotomy. This
infarct expansion and improve structural re-
patient is in shock, and the source of bleeding
modeling in the days following an acute MI.
should be found.
Answer C is incorrect. -Blockers are recom-
Answer A is incorrect. If this patient were he-
mended to all patients with an ST-segment el-
modynamically stable, this would have been
evation after MI to decrease myocardial oxygen
a viable option. However, there should be no

Cardiovascular
consumption and mortality.
delay in getting this patient to the operating
room. Answer D is incorrect. Magnesium is not rou-
tinely used in acute MI.
Answer B is incorrect. This would have been
another viable option if the patient was hemo-
11. The correct answer is C. This patient has symp-
dynamically stable. However, shock is a con-
tomatic bradycardia as evidenced by her altered
traindication to diagnostic peritoneal lavage,
mental status and hypoperfusion. In an elderly
and no time should be spared getting the pa-
patient, it is most likely caused by an inferior
tient to the operating room.
wall MI or sick sinus syndrome, but certain
Answer D is incorrect. Although a head in- medications like nitroglycerin, -blockers, ACE
jury is likely, the proper ow in management inhibitors or barbiturates can mimic a shock-like
should entail the ABCDs (Airway, Breath- state. Atropine is the drug of choice for symp-
ing, Circulation, and Disability). Controlling tomatic bradycardia.
hemorrhage that is manifesting as shock takes
Answer A is incorrect. Adenosine is not part of
priority over dealing with a concomitant head
the bradycardia algorithm. It is used for atrio-
injury.
ventricular nodal reentrant tachycardia such as
Answer E is incorrect. Hemorrhage control Wolff-Parkinson-White syndrome.
cannot be adequately achieved with laparos-
Answer B is incorrect. Amiodarone is not part
copy. An expeditious laparotomy is indicated
of the bradycardia algorithm. It is a class III
in the setting of hypovolemic shock.
antiarrhythmic agent used for wide-complex
tachycardia such as atrial brillation.
10. The correct answer is E. This patient is pre-
senting with a classic acute MI, and he has ful- Answer D is incorrect. While isoproterenol is
lled all indications for brinolytic therapy: a part of the advance cardiac life support brady-
acute chest pain suggesting MI, time to ther- cardia algorithm, it is not a rst-line agent. The
apy <12 hours, and ST-segment elevation >23 mnemonic for the algorithm is All Trained
mm in the chest leads and 1 mm in the limb Dogs Eat Iams: Atropine, Transcutaneous
leads. Contraindications to brinolytic therapy, pacing, Dopamine, Epinephrine, and Isopro-
however, must still be ruled out; these include terenol, given in that order.
a history of intracranial hemorrhage, acute
Answer E is incorrect. Metoprolol is not part
ischemic stroke within the past 3 months, cere-
of the bradycardia algorithm. It is a -blocker
brovascular malformation or brain metastasis,
and would therefore slow the heart rate, exac-
suspicious aortic dissection, active internal
erbating the problem.
bleeding or bleeding diathesis, and signicant
head trauma within the past 3 months.
HIGH-YIELD SYSTEMS
24 Section I: Organ Systems Answers

12. The correct answer is A. Digoxin toxicity often Cardiac abnormalities, particularly disease of
presents with vague abdominal complaints, ac- the conduction system, can lead to syncopal epi-
companied by neurologic (headache, delir- sodes or even sudden death. Heart block can be
ium) complaints; visual (altered color percep- progressive, sometimes necessitating placement
tion, scotomata) complaints; and, most notably, of a pacemaker. The hallmark of disease is myo-
cardiac arrhythmias. This patient may have tonia, which is tonic spasm of affected muscles.
taken too many digoxin pills. Measurement of Pharmacologic therapy for myotonia includes
the plasma digoxin level will help conrm the phenytoin, procainamide, or quinine sulfate,
diagnosis (therapeutic range is 0.52 ng/mL). but this does not improve the weakness, cardiac
However, toxicity can also exist at normal lev- abnormalities, or overall course of the disease.
els, particularly in persons who are elderly. Be-
Answer A is incorrect. Borrelia burgdorferi is
cause digoxin is renally excreted, the patient
the causative organism of Lyme disease, which
may have acute renal failure precipitating his
causes a more rapidly progressive heart block
toxicity; this must be investigated. Note that
than seen with myotonic dystrophy, occurring
Cardiovascular

digoxin levels taken within 68 hours of inges-


over weeks to months, not years. It is also char-
tion do not reect the steady state and are not
acterized by rash, joint pains, and individual
reliable predictors of prognosis. Antidigoxi-
nerve palsies, not muscle weakness and myoto-
genin antibody Fab fragments are rst-line
nia.
therapy in the setting of life-threatening ar-
rhythmia. Answer B is incorrect. Beckers muscular
dystrophy is caused by deletion mutations in
Answer B is incorrect. Although gastroenteritis
>70% of patients. Although the two X-linked
can present with abdominal pain and nausea,
diseases are closely related, patients with Beck-
the most likely diagnosis is digoxin toxicity be-
ers muscular dystrophy have a structurally
cause of the ECG changes.
abnormal form of the protein, compared with
Answer C is incorrect. Although hypercal- Duchennes complete absence of dystrophin.
cemia may present with abdominal pains, hy- Not surprisingly, the Beckers phenotype is
pocalcemia does not present in this fashion. much less severe than Duchennes, with many
patients walking well into adult life.
Answer D is incorrect. Hypovolemia would
not explain the clinical symptoms or the ECG Answer C is incorrect. Frameshift mutations
ndings. are present in >90% of patients with Duch-
ennes muscular dystrophy, an X-linked reces-
Answer E is incorrect. Although patients who
sive disorder. It often presents in early child-
are elderly have atypical presentations of MI,
hood with proximal muscle weakness and/or
the ECG changes are not suggestive of an
calf hypertrophy. Treatment is supportive and
acute MI.
death usually occurs by the third decade of life
13. The correct answer is D. This patient is suffer- as a result of pulmonary infections or respira-
ing from myotonic dystrophy, a muscular dys- tory compromise.
trophy caused by multiple CTG nucleotide re- Answer E is incorrect. Emery-Dreifuss mus-
peats within the myotonin protein kinase gene. cular dystrophy is another X-linked muscular
With successive generations, the number of dystrophy caused by deciencies in the nuclear
CTG sequences increases parallel with earlier membrane protein emerin. Onset of weakness
onset of symptoms and more severe disease (ge- varies from childhood to late adulthood. A dis-
netic anticipation). This accounts for the likely tinguishing feature consists of early contrac-
onset of weakness already in the patients son. tures of the elbow exors, neck extensors, and
Patients often present with ptosis and weakness posterior calf muscles. The cardiac conduction
of the facial muscles, giving a characteristic system is affected in Emery-Dreifuss muscular
hatchet face appearance. Another common dystrophy, but heart problems are usually ac-
presentation is weakness and atrophy of the in- companied by a severe cardiomyopathy.
trinsic hand muscles and forearm extensors.
HIGH-YIELD SYSTEMS
Chapter 1: Cardiovascular Answers 25

14. The correct answer is A. Disseminated intra- the clinical history, laboratory ndings, or pe-
vascular coagulation (DIC) is a consumptive ripheral blood smear.
coagulopathy that has been associated with a
Answer E is incorrect. Thrombotic thrombo-
number of clinical conditions, including bacte-
cytopenic purpura (TTP) involves vascular ag-
rial infections such as meningococcemia. DIC
gregation of platelets that leads to thrombocy-
involves activation of the coagulation pathways,
topenia and mechanical injury to RBCs. TTP
excessive brin formation, and platelet activa-
may be caused by drugs, HIV, pregnancy, auto-
tion. Subsequent bleeding results because of
immune disease, familial causes, or can be id-
the depletion of coagulation factors and plate-
iopathic; TTP has not been associated directly
lets in the circulation. Because of the con-
with meningococcemia. Although one may
sumption of coagulation factors and activation
see thrombocytopenia with prolonged bleed-
of platelets, patients present with prolonged
ing time in the presence of schistocytes, the
bleeding time, prothrombin time, activated
prothrombin time, activated partial thrombo-
partial thromboplastic time, and thrombin
plastin time, and thrombin time would not be

Cardiovascular
time; thrombocytopenia; and schistocytes on
prolonged in the case of TTP.
peripheral blood smear. Although not specic,
the presence of D-dimer and brinogen degra- 15. The correct answer is B. This patient is most
dation products supports a diagnosis of DIC. likely suffering from hypovolemic shock sec-
Answer B is incorrect. Factor V Leiden in- ondary to perforation of a peptic ulcer, which
volves a genetic mutation that renders factor V is conrmed by the nding of free air in the
of the coagulation pathway resistant to degrada- abdomen. Initial resuscitation requires rapid
tion by activated protein C. Factor Va therefore reexpansion of the effective blood volume
does not get cleaved, but rather remains active, along with interventions to control ongoing
leading to a hypercoagulable state. Thrombo- losses. This is best accomplished with a rapid
cytopenia and bleeding are not manifestations infusion of isotonic saline or lactated Ringers
of factor V Leiden and therefore are not sup- solution through two large-bore intravenous
ported by the clinical history, laboratory nd- lines.
ings, or peripheral blood smear. Answer A is incorrect. Antibiotics are indi-
Answer C is incorrect. Immune thrombo- cated in the setting of septic shock. Given the
cytopenic purpura (ITP) refers to immune- patients age, it is possible that a perforated di-
mediated platelet destruction by autoantibod- verticulum could cause septic shock and have
ies. Although ITP may present with petechiae a similar presentation. However, a perforated
and thrombocytopenia, the peripheral smear ulcer is more likely because his pain was epi-
often shows larger, younger platelets that are gastric, whereas diverticular pain tends to lo-
produced to compensate for the increased calize to the lower abdomen. Furthermore,
platelet destruction. ITP may present with even if this patient were in septic shock, vol-
spontaneous bleeding if the platelet count is ume resuscitation would still be the initial step
<10,000/mm. Like DIC, ITP may present in treatment.
with increased bleeding time due to a decrease Answer C is incorrect. Infusing norepineph-
in the number of platelets available. ITP, how- rine can increase arterial pressure by raising
ever, is not associated with a prolonged pro- peripheral resistance. However, this is inappro-
thrombin time, activated partial thromboplas- priate other than as a temporizing measure in
tin time, or thrombin time as seen in DIC. severe shock while the effective blood volume
Answer D is incorrect. Protein C deciency is being reexpanded with uid resuscitation.
causes a procoagulant-anticoagulant imbal- Answer D is incorrect. Inotropic support is
ance. Without protein C, there is increased particularly important in cardiogenic shock.
propensity to form clots and decreased antico- The at jugular veins suggest hypovolemic
agulation. Thrombocytopenia and increased shock in this case. In the presence of severe or
bleeding are not manifestations of protein C prolonged hypovolemia, inotropic support may
deciency and therefore are not supported by
HIGH-YIELD SYSTEMS
26 Section I: Organ Systems Answers

be needed to maintain adequate cardiac out- tients usually present with symptoms of heart
put, but this should occur after effective blood failure, arrhythmias, or even sudden death.
volume has been restored. Fifty percent of cases are of idiopathic etiology
but the most common known causes are isch-
Answer E is incorrect. Resuscitation should
emic cardiomyopathy due to CAD, myocardi-
begin with crystalloid solutions. Banked blood
tis, and inltrative disease. Alcohol is a poorly
products are hyperkalemic and acidotic, which
understood but signicant risk factor for DCM,
can worsen reperfusion injury as the periph-
and abstinence can result in remarkable recov-
eral reperfusion ushes products of anaerobic
ery of cardiac function.
metabolism back to the heart. This can lead
to transient myocardial depression. It should Answer A is incorrect. Although hepatojugular
be noted that although the patients hemoglo- reux and pulmonary congestion are both of-
bin is normal, it takes time to equilibrate, even ten seen in heart failure due to DCM, neither
during an acute bleed. He may require RBCs specically is required for a diagnosis of DCM.
in the future.
Cardiovascular

Answer B is incorrect. Ventricular dilation is


a key component of DCM. However, aortic
16. The correct answer is A. This patient most
insufciency is not usually found. Further-
likely suffers from Prinzmetals angina, which
more, this patient does not have any physical
is caused by coronary artery spasm. This type
examination ndings suggestive of aortic in-
of angina usually occurs in smokers younger
sufciency, such as a diastolic murmur, Corri-
than those with unstable angina (UA) due to
gans pulse (water-hammer pulse), de Mussets
atherosclerotic disease. The pain is intermit-
sign (head-bobbing), Traubes sign (pistol-shot
tent and can wake them up in the morning.
sound over the femoral artery), Duroziezs sign
During chest pain, an ECG will show multi-
(to-and-fro murmur over the femoral artery),
lead ST-segment elevations that can resolve
Quinckes pulse (capillary pulsation in the nail
with the administration of nitroglycerin.
beds), or Hills sign (popliteal artery pressure
Answer B is incorrect. Stenosis of this type greatly increased over brachial pressure).
would result in persistent stable angina for the
Answer D is incorrect. Dilation of the
patient, while she describes more intermittent
ventricle(s) leads to impaired contraction of
chest pain. Therefore, it is unlikely that she
the myocardium and eventually to hypotrophy
has such an extent of arterial stenosis, and the
of the myocardium. Diastolic dysfunction is
picture does not demonstrate any blockages.
not a dening feature of DCM.
Answer C is incorrect. The patient may have
Answer E is incorrect. Pulmonary congestion
normal coronary arteries in between episodes,
is a result of heart failure and a common oc-
but the chest pain is brought on by coronary
currence in patients with DCM. However, the
artery spasm so that is what should be seen on
pathogenesis of DCM lies in systolic dysfunc-
angiography. Spasm can be provoked by intra-
tion, not diastolic dysfunction. Diastolic heart
coronary acetylcholine or hyperventilation.
failure is characterized by symptoms of heart
Answer D is incorrect. Plaque rupture and failure in the context of normal LV systolic
thrombosis leads to acute MI, which would not function (normal LV ejection fraction) and
present as chronic anginal pain of 34 weeks diastolic dysfunction (abnormal or elevated LV
duration. lling pressures). Common causes of diastolic
heart failure include restrictive cardiomyopathy
17. The correct answer is C. The patient has (RCM) that results in decreased ability for LV
DCM, a diagnosis that requires evidence of LV lling, chronic hypertension with LVH, hyper-
dilation and systolic dysfunction with LV ejec- trophic cardiomyopathy (HCM), and ischemic
tion fraction (LVEF) <40% on echocardiogra- heart disease.
phy. Dilation of the LV results in decreased
ability to contract and eject blood from the 18. The correct answer is C. The photograph de-
chamber, resulting in a decreased LVEF. Pa- picts a vegetative growth on a native mitral
HIGH-YIELD SYSTEMS
Chapter 1: Cardiovascular Answers 27

valve. Mitral valve prolapse (MVP), particu- antibiotics and surgery for valve repair or re-
larly as a complication of rheumatic heart dis- placement, if necessary.
ease, is a risk factor for native valve infective
Answer A is incorrect. Infection with Group
endocarditis. This is because altered blood ow
A Streptococcus, if untreated, can lead to rheu-
around a damaged valve provides the opportu-
matic fever, damaging heart valves and muscle.
nity for a clot to develop and harbor bacteria,
Rheumatic fever can predispose the patient
which gain access to the blood through a
to subsequent development of infective endo-
wound, dental work, surgery, or intravenous
carditis, but it does not cause the disease ini-
drug use. It can be prevented by replacing the
tially.
valve with a prosthetic valve.
Answer B is incorrect. Tuberculosis can cause
Answer A is incorrect. CAD could lead to val-
cardiac disease, but in this patient, especially
vular disease that would increase the risk of in-
with the history of intravenous drug abuse,
fective endocarditis, but CAD does not directly
Staphylococcus aureus is the most likely organ-
increase the risk of developing infective endo-

Cardiovascular
ism.
carditis.
Answer D is incorrect. If the patient has a
Answer B is incorrect. Hypertension can lead
prosthetic valve, coagulase-negative staphylo-
to strain on the heart and its valves if poorly
coccus is the predominant organism. Within
controlled, but it does not directly increase the
2 months of surgery, the most common organ-
risk of developing infective endocarditis.
isms are nosocomial: coagulase-negative staph-
Answer D is incorrect. Prolonged bedrest can ylococcus, Staphylococcus aureus, facultative
lead to the development of deep venous blood gram-negative bacilli, diphtheroids, and fungi.
clots. This could lead to pulmonary embolism Twelve months after surgery, the organisms are
as the clot travels in the venous return and similar to those that cause community-acquired
through the right side of the heart. It would be endocarditis.
extremely rare for the clot to nd a path from
Answer E is incorrect. Streptococcus viridans is
the right side of the heart to the left-sided mi-
the most common pathogen for left-sided sub-
tral valve. Even if this occurred, the concern-
acute bacterial endocarditis.
ing complication would be embolus to the
brain, not lodging of the clot in the valve to
20. The correct answer is B. Dextraposed transpo-
serve as a site for bacterial growth.
sition of the great arteries (D-TGA) is the most
Answer E is incorrect. While an articial valve common cause of cyanotic heart disease in neo-
is a risk factor for infective endocarditis, it does nates. It accounts for 5% of congenital heart
not contribute to native valve endocarditis. defects. With this defect, the aorta arises from
the right ventricle and the pulmonary artery
19. The correct answer is C. In patients with in- arises from the LV. This leads to pulmonary
fective endocarditis and a history of intrave- and systemic circuits that are in parallel as op-
nous drug abuse, Staphylococcus aureus is the posed to in series. The deoxygenated blood is
causative agent in the vast majority of cases therefore recirculated through the body in the
and is more likely to cause acute rather than systemic circulation, while the oxygenated
subacute endocarditis. If the patient has a pros- blood only ows through the pulmonary circu-
thetic valve, then coagulase-negative staphylo- lation. A lesion, such as an atrial septal defect
coccus is the predominant organism. Bacterial (ASD), ventricular septal defect (VSD), or pa-
endocarditis is an infectious process of the en- tent ductus arteriosus (PDA), is therefore re-
dothelial surface of the heart. Symptoms in- quired for mixing of the systemic and pulmo-
clude fever, fatigue, malaise, vascular phenom- nary circulations for survival. D-TGA usually
ena such as Janeway lesions, and immunologic presents at birth with cyanosis and tachypnea.
phenomena such as Osler nodes. Diagnosis is Plain lm radiographs demonstrate an egg-
usually based on the Duke criteria, and labora- shaped silhouette due to the absent main pul-
tory studies include blood culture and echocar- monary artery stem and small heart base. Pros-
diography. Treatment is through intravenous taglandin E1 is used to keep the PDA open and
HIGH-YIELD SYSTEMS
28 Section I: Organ Systems Answers

increase mixing of deoxygenated and oxygen- trophy, and an overriding aorta. TOF usually
ated blood. Balloon atrial septostomy can also presents in infancy with cyanotic spells and
be used if necessary. An arterial switch surgical agitation. The spells either resolve spontane-
procedure is used to repair the defect. ously or can lead to hypoxia, metabolic acido-
sis, and death. On physical examination, these
Answer A is incorrect. Coarctation of the
infants have a normal S1, single S2, and a
aorta is a cause of acyanotic heart disease. It
harsh systolic crescendo-decrescendo ejection
accounts for 8% of congenital heart defects,
murmur (due to right ventricular outow tract
with a 2:1 male:female predominance. Coarc-
obstruction), which is loudest over the left up-
tation of the aorta may be associated with a bi-
per sternal border and radiates to the back. A
cuspid aortic valve in 80% of cases. CHF may
boot-shaped heart may be seen on x-ray of the
develop in infancy in approximately 10% of
chest and there are increased pulmonary vas-
patients. It manifests as a systolic ejection mur-
cular markings. TOF is corrected surgically.
mur at the left upper sternal border, radiating
to the interscapular region. Physical examina- Answer E is incorrect. Truncus arteriosus is a
Cardiovascular

tion is also remarkable for weak and delayed rare congenital heart defect that causes cyanotic
femoral pulses relative to the upper extremity and congestive heart disease. It is characterized
pulses, and hypertension in the upper extremi- by a single great arterial trunk arising from the
ties. A reverse 3 sign may be seen on x-ray of ventricles, branching into the pulmonary arter-
the chest due to constriction of the aorta at the ies distal to the coronary arteries and proximal
coarctation and dilation of the aorta pre- and to the rst brachiocephalic branch of the aor-
postcoarctation. Notching of the ribs may also tic arch. There is always an associated VSD for
be seen due to collateral circulation through complete mixing of pulmonary and systemic
the intercostal arteries, and erosion of the ribs blood, resulting in identical oxygen saturation
by the collaterals. The defect may be repaired in the pulmonary arteries and aorta. Thirty
via angioplasty or surgery. percent of these infants have a right-sided aor-
tic arch. A harsh holosystolic murmur is pres-
Answer C is incorrect. PDA causes acyan-
ent at the left lower sternal border due to the
otic congenital heart disease. PDAs account
VSD, and there is a prominent single S2. Arte-
for 10% of congenital heart disease, occurring
rial pulses are bounding and there is a widened
with a high incidence in preterm infants and
pulse pressure. Cardiomegaly and increased
a 2:1 female:male predominance. Small PDAs
pulmonary vascular markings are present on
are asymptomatic, while large ones may cause
x-ray of the chest. Digoxin and diuretics may
CHF, failure to thrive, and recurrent lower
be used in the initial management, and the de-
respiratory tract infections. On physical ex-
fect may then be repaired with surgery.
amination, a continuous machinery murmur
heard best at the left upper sternal border, and
21. The correct answer is C. This patient pre-
bounding peripheral pulses may be present.
sented with a tension pneumothorax, which re-
There may also be a prominent apical impulse
sults from a parenchymal wound that acts as a
and a thrill. X-ray of the chest may show car-
one-way valve that allows free air into the pleu-
diomegaly and increased pulmonary vascular
ral space but prevents its escape, causing col-
markings. PDAs usually close within the rst
lapse of the lung on the affected side. It is a
month of life. In preterm infants, indometha-
medical emergency, as the building pressure in
cin may be administered to close the PDA.
the pleural space causes shifting or displace-
PDAs may also be surgically ligated or coil em-
ment of the mediastinum to the contralateral
bolized if necessary.
side and subsequently compromises cardiopul-
Answer D is incorrect. Tetralogy of Fallot monary function. Compression of the opposite
(TOF) is the third most common cause of cy- lung impairs proper gas exchange while im-
anotic heart disease in neonates, after D-TGA pingement on the heart impairs proper cardiac
and hypoplastic left heart syndrome. It is com- function. The most common mechanisms of
posed of four defects: right ventricular outow
tract obstruction, VSD, right ventricular hyper-
HIGH-YIELD SYSTEMS
Chapter 1: Cardiovascular Answers 29

this type of injury are blunt or penetrating in- are part of the continuum of acute coronary
juries, or secondary to medical procedures. A syndromes, in which plaque rupture and coro-
large-bore needle should be inserted in the sec- nary thrombosis compromise blood ow to a
ond intercostal space in the midclavicular line region of viable myocardium. In UA and
to facilitate decompression and reestablish car- NSTEMI, ST-segment elevation and patho-
diopulmonary function. The needle is left in logic Q waves are absent. They are treated with
place until a thoracostomy tube can be in- medical management (antiplatelet therapy, ni-
serted. troglycerin, -blockade, and morphine), and
considered for revascularization.
Answer A is incorrect. Although tube thora-
cotomy is also needed, tension pneumothorax Answer B is incorrect. This man has the signs,
is a medical emergency and demands urgent symptoms, and laboratory test results of some-
decompression. A large release of air after nee- body undergoing MI, so hypochondriasis can
dle insertion conrms the diagnosis, and can be ruled out.
then be followed by a thoracostomy. The chest

Cardiovascular
Answer C is incorrect. Prinzmetals angina
tube is then attached to a vacuum to continu-
does not produce elevated cardiac enzyme lev-
ously remove air from the pleural cavity until
els. It is due to coronary vasospasm, and can be
the parenchyma heals.
treated with medical therapy (calcium channel
Answer B is incorrect. The location of the blockers).
wound in the fth intercostal space implies
Answer D is incorrect. Stable angina does not
that abdominal injury is possible. However,
produce elevated cardiac enzyme levels. It is
in following the ABCs as part of the Advanced
dened as chest pain that is reproducible with
Trauma Life Support protocol, breathing must
a certain degree of exertion. These patients
be adequately treated before evaluating circula-
may be treated with medical management if
tion and proceeding to a diagnostic peritoneal
low-risk, or with revascularization if high-risk
lavage.
(i.e., left main artery disease or three-vessel dis-
Answer D is incorrect. Pericardiocentesis is ease), or if they have refractory symptoms.
the treatment of choice for cardiac tampon-
Answer E is incorrect. UA does not increase
ade, which can also present with tachycardia,
cardiac enzyme levels. However, the initial
jugular venous distention, and decreased blood
management of UA and NSTEMI are simi-
pressure. However, the location of the wound,
lar (medical management with nitroglycerin,
the tracheal deviation, absent breath sounds,
-blockade, antiplatelet/anticoagulation, and
hyperresonance to percussion, and severity of
consideration of angiography and revascular-
dyspnea suggest tension pneumothorax.
ization).
Answer E is incorrect. Exploratory thoracotomy
is the treatment for massive hemothorax. How- 23. The correct answer is A. You should be able to
ever, tension pneumothorax is best managed recognize diastolic dysfunction as a cause of
with needle thoracostomy followed by tube tho- heart failure. In diastolic heart failure, LVEF is
racotomy. normal (>50%). Heart failure results from an
inability of the LV to ll during diastole rather
22. The correct answer is A. This man has a car- than an inability of the LV to eject blood into
diac enzyme leak of specic myocardial mark- systemic ow. Hypertensive heart disease is one
ers that suggests acute MI. UA is dened as ei- of the most common causes of diastolic heart
ther rest angina >20 minutes in duration, failure. This patients medical history of hyper-
new-onset angina, or increasing angina that is tension, in-ofce measurement indicating high
more frequent, longer in duration, or occurs blood pressure, and the ECG showing LVH is
with less exertion than previous angina. UA consistent with this diagnosis.
and non-ST-segment elevation MI (NSTEMI)
HIGH-YIELD SYSTEMS
30 Section I: Organ Systems Answers

Answer B is incorrect. Hypertrophic obstruc- percholesterolemia increase a patients risk of


tive cardiomyopathy (HOCM) may mani- cardiovascular disease.
fest with hypertension and an ECG showing
Answer C is incorrect. Hypertension does not
LVH. However, echocardiography of patients
increase a patients risk of HCM. HCM is a
with HOCM typically shows asymmetric sep-
disease in which the heart muscle thickens and
tal wall thickening (septum:ventricular wall
therefore has impaired functioning. It is typi-
thickness ratio >1.3). Subaortic stenosis often-
cally an inherited disorder due to autosomal
times results from septal wall thickening, caus-
dominant gene mutations. This disease may
ing obstruction across the aortic outow tract.
cause angina and dyspnea, but often the rst
HOCM is often familial and is manifest earlier
symptom of HCM is sudden cardiac death.
in life (childhood through young adulthood).
Patients rarely present in the ninth decade of Answer D is incorrect. Hypertension does
life. not increase a patients risk of atrioventricular
block. The most common etiologies of second-
Answer C is incorrect. Ischemic heart disease
Cardiovascular

degree Mobitz I atrioventricular block are


is one of the most common causes of diastolic
drugs, acute inferior MI, and enhanced vagal
dysfunction and must always be included on
tone.
the differential. However, an ECG showing
lack of ST elevation (indicating acute infarct) Answer E is incorrect. Hypertension does not
or depression (indicating ischemia), Q waves, increase a patients risk of type 2 diabetes mel-
or T-wave inversions (indicating previous in- litus; however, both hypertension and diabetes
farct) makes this diagnosis less likely. increase a patients risk of cardiovascular dis-
ease.
Answer D is incorrect. This patients normal
LVEF is more consistent with diastolic heart
25. The correct answer is D. Lidocaine is an im-
failure. MVP results in systolic heart failure in-
portant agent used for the acute treatment of
stead of diastolic heart failure. Increased regur-
ventricular arrhythmias, especially those after
gitation over the mitral valve results in an in-
acute MI. It is a class IB antiarrhythmic agent
creased preload and inability of the ventricle to
and thus acts on the ventricular myocardium
eject this increased volume (decreased LVEF).
by mildly blocking sodium channels, slowing
Furthermore, no regurgitation was noted on
repolarization in Purkinje cells, and increasing
Doppler ow on echocardiography, making
the ring threshold in pacemaker cells. It has
this answer choice unlikely.
relatively few serious adverse effects; however,
Answer E is incorrect. Myocarditis can be as- patients taking lidocaine may experience sig-
sociated with diastolic heart failure, but is less nicant neurologic complications. This patient
likely in this case given no recent viral illness. is displaying the classic neurologic adverse ef-
This patients age, past medical history, and fects of lidocaine, including slurred speech and
ECG are all consistent with hypertensive heart confusion. Other common adverse effects in-
disease. clude tremor, personality and mood changes,
and hallucinations. These effects are entirely
24. The correct answer is A. Hypertension is a risk reversible with the removal of therapy.
factor for both chronic renal insufciency and
Answer A is incorrect. Salicylates are not asso-
end-stage renal disease. Hypertension can di-
ciated with central nervous system adverse ef-
rectly cause renal disease and accelerate the
fects with normal dosing. With normal dosing,
progression of underlying renal pathology. In
they may cause renal disease, bleeding, and
addition, hypertension increases a patients risk
gastrointestinal upset. In toxic quantities, how-
of premature cardiovascular disease, heart fail-
ever, they can lead to an altered mental status
ure, stroke, and intracerebral hemorrhage.
and tinnitus.
Answer B is incorrect. Hypertension does
Answer B is incorrect. ACE inhibitors are
not increase a patients risk of hypercholester-
commonly associated with hyperkalemia, renal
olemia; however, both hypertension and hy-
HIGH-YIELD SYSTEMS
Chapter 1: Cardiovascular Answers 31

failure, and dry cough. More rarely, they cause cannot explain her clinical picture. Her peri-
angioedema. They do not commonly cause cardial effusion has progressed to cause cardiac
central nervous system disturbances. tamponade. Her distended neck veins, mufed
heart sounds, and hypotension are all signs of
Answer C is incorrect. Selective serotonin
cardiac tamponade and do not occur with sim-
reuptake inhibitors are commonly associated
ple pericarditis.
with sexual dysfunction and anxiety. Even in
overdose, they have few adverse effects, which Answer E is incorrect. Although a tension
are mostly limited to the gastrointestinal tract. pneumothorax can lead to tachypnea, tachy-
cardia, and hypotension, the patients x-ray of
Answer E is incorrect. -Blockers are mainly
the chest would have shown a pneumothorax
associated with depression and sexual adverse
had one existed.
effects. They can also produce sleep distur-
bances but are not associated with more severe
27. The correct answer is D. UA is acute myocar-
central nervous system effects.
dial ischemia without evidence of myocardial

Cardiovascular
necrosis, manifesting as angina that is new-on-
26. The correct answer is A. This patient has the
set, crescendos, or occurs at rest. It can present
characteristic symptoms and signs of cardiac
with or without ECG changes, including ST-
tamponade. She complains of chest pain, fa-
segment depressions, but by denition cardiac
tigue, and dyspnea, all characteristic of tam-
enzyme levels are normal. Treatment involves
ponade. On physical examination, she has
relief of ischemic pain, assessment of hemody-
Becks triad, a group of signs characterized by
namic status, and antithrombotic therapy if
hypotension, distant heart sounds, and dis-
necessary. If this patient were having an acute
tended neck veins. The uid accumulation
MI, we would expect to see ST-segment eleva-
around the heart decreases the ventricular ll-
tions and elevated cardiac enzyme levels.
ing pressure, which decreases cardiac output.
She also has tachycardia and tachypnea, both Answer A is incorrect. Delta waves are asso-
found in patients with cardiac tamponade. Evi- ciated with Wolff-Parkinson-White syndrome
dence of cardiomegaly is due to a large pericar- and would be only an incidental nding in this
dial effusion, and the echocardiogram would patient, as opposed to a nding produced by
show a large pleural effusion with chamber his angina. Also, UA should not produce car-
collapse, a characteristic echocardiographic diac enzyme elevations.
sign of tamponade.
Answer B is incorrect. A low-voltage ECG can
Answer B is incorrect. Although the patients be caused by poorly placed leads, hemoperi-
x-ray of the chest showed cardiomegaly, this cardium, emphysema, or massive obesity. UA
was secondary to a large pericardial effusion would not cause a low-voltage ECG and would
leading to tamponade, not an enlarged heart not cause elevated cardiac enzyme levels.
as seen in heart failure. It would be unlikely
Answer C is incorrect. Elevated cardiac en-
to see such an acute onset of decompensated
zyme levels with no ECG changes can occur
heart failure in this otherwise healthy, young
but are not indicative of UA. This scenario can
woman. In addition, the patient has the classic
be seen days after a cardiac event, as enzyme
physical ndings of cardiac tamponade.
levels can remained elevated for many days,
Answer C is incorrect. Although panic at- or as a result of an enzyme leak secondary to a
tacks can present as chest pain, tachypnea, and cardiac insult.
tachycardia, this patient has several ndings on
Answer E is incorrect. The presence of Q
physical examination and diagnostic studies
waves (in addition to ST-segment elevations)
that support the diagnosis of cardiac tampon-
indicates previous insult to the myocardium, as
ade.
Q waves do not appear until days after the in-
Answer D is incorrect. Although this patient sult. Therefore it is unlikely that we would see
most likely has pericarditis, this diagnosis alone Q waves associated with this patients UA.
HIGH-YIELD SYSTEMS
32 Section I: Organ Systems Answers

28. The correct answer is A. Idiopathic hypertro- Answer D is incorrect. Pneumothorax is a rare
phic subaortic stenosis (IHSS) is the most com- complication of pericardiocentesis. This pa-
mon cause of sudden death in young athletes tients lung examination and x-ray of the chest
in the United States. IHSS results in LV out- are not consistent with the diagnosis of pneu-
ow tract obstruction, impaired LV unloading, mothorax.
LVH, and diastolic dysfunction (impaired LV
Answer E is incorrect. Puncture of the LV dur-
relaxation and lling). Patients may be asymp-
ing pericardiocentesis rarely causes signicant
tomatic, but can present with dizziness, dysp-
bleeding. Although a small pericardial bleed
nea, angina, and sudden death. Physical exam-
may be painful, it typically resolves on its own
ination often reveals mitral insufciency, an
and does not lead to hemodynamic compro-
S4 gallop (due to a stiffened LV), and a cre-
mise.
scendo-decrescendo ejection murmur that
gets louder with Valsalva maneuver (decreased
30. The correct answer is E. The patient can best
preload). Both echocardiography and ECG
decrease his risk of a second MI by quitting
Cardiovascular

show signs of LVH, and x-ray of the chest may


smoking. In some studies, patients who already
reveal left atrial dilation due to mitral insuf-
had an initial coronary event and subsequently
ciency. The rst step in management is symp-
quit smoking decreased their risk of a second
tomatic control with -blockade.
coronary event by 50%.
Answer B is incorrect. Calcium channel
Answer A is incorrect. Secondary prevention
blockers are second-line agents in the symp- of CAD by treating hyperlipidemia has shown
tomatic control of LVH. a risk reduction for a second MI in the 20%
Answer C is incorrect. Partial excision of the 35% range in 5-year follow-up.
septum is a successful treatment for patients Answer B is incorrect. Exercise has been
with septal hypertrophy, but it is not the rst shown to offer only mild to moderate reduc-
step in management. tion in risk for CAD. In this case, the patient
Answer D is incorrect. Anticoagulation plays should undergo a symptom-limited stress test
no role in the initial management of LVH. within 412 weeks and start an exercise pro-
gram monitored by a physician.
29. The correct answer is C. Laceration of a coro-
Answer C is incorrect. This patient is barely in
nary vessel is the most dangerous complication
the hypertensive range (>140/90 mm Hg) and
of pericardiocentesis. It can lead to worsened
so better control of his hypertension will not
cardiac tamponade, MI, and even death. This
bring as dramatic an effect as modication of
patient has worsening chest pain and hemody-
his smoking habit.
namics during pericardiocentesis, the most
likely cause of which is laceration of a coronary Answer D is incorrect. Tight glycemic control
vessel. has not been shown to dramatically decrease
the risk of CAD.
Answer A is incorrect. Acute LV failure with
pulmonary edema is a rare complication of 31. The correct answer is E. This is a classic de-
pericardiocentesis. This patients lungs are scription of RCM. RCM is almost always asso-
clear to auscultation bilaterally after the proce- ciated with inltrative diseases such as amyloi-
dure, which makes this complication unlikely. dosis, sarcoidosis, or hemochromatosis. These
Answer B is incorrect. Aspiration of air into conditions restrict LV lling, causing decreased
the pericardium typically does not cause clini- output and compliance, and increased lling
cal signs or symptoms, unless the amount of pressure. Consequently, patients begin to expe-
air aspirated into the pericardium exceeds the rience CHF symptoms. Here, this patient com-
amount of uid withdrawn.
HIGH-YIELD SYSTEMS
Chapter 1: Cardiovascular Answers 33

plains of dyspnea (positional and with exer- 32. The correct answer is C. Nearly any dysrhyth-
tion), syncope, and peripheral edema. She also mia may be precipitated by acute digitalis tox-
has the classic Kussmauls sign (increased jug- icity, but atrial tachycardia with 2:1 block, ac-
ular venous distension with inspiration) that, celerated junctional rhythm, and bidirectional
although it is not specic for this condition, ventricular tachycardia (torsade de pointes) are
contributes to making the diagnosis. The com- frequently seen, due to the combination of de-
bination of the echocardiogram and ECG creased atrioventricular node conduction and
signs listed are also classic for making the diag- increased automaticity. Magnesium sulfate is
nosis. Treatment of RCM is to control the un- the drug of choice for treating torsades de
derlying cause (e.g., iron chelation for hemo- pointes, including in the setting of digitalis tox-
chromatosis), diuretics, ACE inhibitors, and icity, since it decreases calcium inux and thus
nitrates. reduces the early afterdepolarizations that per-
petuate this dysrhythmia. A therapeutic level is
Answer A is incorrect. Aortic stenosis (AS) is
45 mEq/L. Additional treatment for torsades
important because it is currently the leading

Cardiovascular
in the setting of digoxin overdose include anti-
indication for valve replacement. AS usually
digitalis Fab fragments, lidocaine, and direct
presents in people 60 years old and older, and
current cardioversion.
features the classic triad of angina, syncope,
and heart failure. Physical examination reveals Answer A is incorrect. Calcium gluconate is
pulsus parvus et tardus (small and slowly rising used for prophylaxis against dysrhythmia in
carotid pulse). the setting of hyperkalemia. However, it is
not recommended in the setting of digitalis
Answer B is incorrect. Cardiac tamponade
toxicity, since intracellular calcium levels are
usually presents as subacute dyspnea, fatigue,
already high. Also, the patients potassium is
or anxiety that waxes and wanes. It is often as-
high-normal.
sociated with end-stage renal disease or other
conditions that may involve the pericardium. Answer B is incorrect. A loop diuretic would
Physical examination is characterized by Becks not be indicated in this setting, since it could
triad (jugular venous distension, hypotension, exacerbate electrolyte abnormalities and con-
and mufed heart sounds). It can be caused by tribute to hypotension.
pericarditis.
Answer D is incorrect. Potassium adminis-
Answer C is incorrect. This patients blood tration is important in the setting of digitalis
pressure is within normal limits, and there is low toxicity if serum potassium is low, since hy-
suspicion for hypertensive heart disease. How- pokalemia slows repolarization of the myocyte
ever, hypertensive heart disease can manifest as and may perpetuate dysrhythmias such as tor-
concentric and eventually dilated heart failure. sades de pointes. The patients potassium is
On echocardiography, a dilated heart with an currently high-normal so potassium adminis-
elevated end-diastolic volume and low ejection tration would not be indicated; however, this
fraction would be detected. Arrhythmias and an- should be carefully monitored, as the extracel-
gina may accompany a hypertensive crisis. lular potassium shift caused by digitalis may
mask low intracellular potassium levels.
Answer D is incorrect. Pericarditis is most of-
ten confused with RCM. To differentiate these Answer E is incorrect. Sodium polystyrene
two conditions, rst look at the history. Patients sulfonate (Kayexalate) is a potassium-binding
with pericarditis will likely have had a viral in- resin used in the treatment of hyperkalemia in
fection 12 weeks preceding the complaints. renal failure and other settings. It should not
Physical examination is also helpful because be used in the setting of digitalis toxicity, since
pericarditis patients will often have a pericar- hyperkalemia in that case is generally due to
dial knock or rub and a prominent S4 heart an extracellular shift, not increased whole-body
sound. On biopsy, pericarditis samples will be potassium.
normal and RCM will be abnormal.
HIGH-YIELD SYSTEMS
34 Section I: Organ Systems Answers

33. The correct answer is A. ASDs are often as- 34. The correct answer is D. This patient most
ymptomatic and cause acyanotic heart disease. likely has bromuscular dysplasia leading to re-
As described in this patient, physical examina- nal artery stenosis. She has early onset hyper-
tion may be remarkable for a wide, xed, split tension that is refractory to pharmacotherapy.
S2 with a systolic ejection murmur at the left In addition, she has an abdominal bruit, sug-
upper sternal border. Some patients also have a gestive of renal artery stenosis. Young women
mid-diastolic rumble at the left lower sternal with early onset of hypertension refractory to
border. Both murmurs represent increased pharmacotherapy are the most common pa-
blood ow across the pulmonic and tricuspid tient population for bromuscular dysplasia.
valves. Fibromuscular dysplasia can be diagnosed by
duplex imaging of the renal arteries, and per-
Answer B is incorrect. Coarctation of the
cutaneous transluminal angioplasty is the treat-
aorta may produce a systolic ejection murmur
ment of choice in young patients with this dis-
at the left upper sternal border, radiating to the
ease and refractory hypertension.
interscapular region. Physical examination is
Cardiovascular

also remarkable for weak and delayed femoral Answer A is incorrect. This patients renal
pulses relative to upper extremity pulses, and artery stenosis is most likely due to bromus-
hypertension in the upper extremities. cular dysplasia, not atherosclerosis. Adding a
statin may decrease her cholesterol, but there
Answer C is incorrect. Transposition of the
is no reason to suspect that her cholesterol is
great arteries occurs when the aorta arises from
elevated or that decreasing her cholesterol will
the right ventricle and the pulmonary artery
affect her blood pressure.
rises from the LV. D-TGA describes the form in
which the aorta is anterior and to the right of the Answer B is incorrect. Nitroprusside is the
pulmonary artery. It usually presents with cyano- treatment of choice in a hypertensive emer-
sis and tachypnea within hours of birth because gency. Although this patient has stage 2 hyper-
the systemic and pulmonary circulations are par- tension, she has no signs of end-organ damage;
allel rather than serial. Oxygenated blood circu- therefore, she is not in a hypertensive emer-
lates through the lungs repeatedly, while deoxy- gency, and nitroprusside is not indicated.
genated blood is pumped to the body. D-TGA is
Answer C is incorrect. Although therapy with
incompatible with life unless there are other de-
a thiazide diuretic and an ACE inhibitor may
fects present, such as ASDs or VSDs or a PDA,
be sufcient to control hypertension from renal
that allow for mixing between the two parallel
artery stenosis in some individuals, these medi-
circulations and delivery of oxygen to the body.
cations are not achieving goal pressures in this
Answer D is incorrect. TOF is composed of young woman. In addition, it is important to
four defects: right ventricular outow tract ob- establish a diagnosis in this patient, rather than
struction, VSD, right ventricular hypertrophy, treating her hypertension without knowing its
and an overriding aorta. TOF usually presents etiology. ACE inhibitors can cause an elevation
in infancy, with cyanotic spells, a normal S1, in creatinine in patients with bilateral renal ar-
single S2, and a harsh systolic crescendo-decre- tery stenosis because the glomerular ltration
scendo ejection murmur (due to right ventric- depends largely on angiotensin II in the setting
ular outow tract obstruction). This murmur of decreased renal blood ow.
is heard best over the left upper sternal border
Answer E is incorrect. Although duplex im-
and radiates to the back. It is also possible to
aging of the renal arteries is an appropriate
have a holosystolic murmur at the left lower
diagnostic step, percutaneous transluminal
sternal border from the VSD and a continuous
angioplasty is the treatment of choice for bro-
machinery murmur if a PDA is present.
muscular dysplasia. It has a high success rate, a
Answer E is incorrect. VSDs are the most low restenosis rate, and minimal risk; therefore,
common congenital heart defect. They are angioplasty would be preferred over more inva-
usually associated with a holosystolic murmur sive surgical revascularization in this setting.
that is loudest at the left lower sternal border.
HIGH-YIELD SYSTEMS
Chapter 1: Cardiovascular Answers 35

35. The correct answer is D. This patient is in hy- disorder such as pulmonary embolism, tam-
pertensive urgency: he has a diastolic blood ponade, or severe acidosis. Advanced cardiac
pressure >130 mm Hg but has no signs of end- life support recommends initial basic life sup-
organ damage. His ECG, laboratory values, port stabilization of a patient with demon-
and physical examination results are all nor- strated PEA. Once airway and breathing have
mal. Initial management of hypertensive been secured, several medications can be ad-
urgency involves administration of an oral anti- ministered to address the PEA. These include
hypertensive. -blockers, loop diuretics, ACE several doses each of empiric bicarbonate, epi-
inhibitors, or calcium channel blockers are nephrine, and atropine. After the administra-
recommended, and two agents should be tion of each medication, reassessment of the
started if initial treatment fails to lower pres- ECG is important. Atropine works on the heart
sures to a safe level in 36 hours. Therefore by antagonizing muscarinic receptors and re-
oral metoprolol would be appropriate manage- leasing the vagal stimulus on the heart. As an
ment for this patient in hypertensive urgency. anticholinergic agent, it has several character-

Cardiovascular
istic and uncomfortable adverse effects. Al-
Answer A is incorrect. The patient is not in
though they are rarely life threatening, they
hypertensive emergency because he does not
can be quite concerning to a patient experienc-
have end-organ damage. Intravenous nitro-
ing them. They include blurry vision (blind as
prusside is rst-line treatment in hypertensive
a bat), cutaneous ushing (red as a beet, hot as
emergency, but not hypertensive urgency.
a hare), confusion (mad as a hatter), and dry
There is no proven benet from rapid reduc-
mucous membranes (dry as a bone). Because
tion in blood pressure in asymptomatic patients
these adverse effects are uncomfortable, they
who have no evidence of end-organ damage.
should be treated supportively.
Answer B is incorrect. Although this patient
Answer A is incorrect. This patient does not
does meet requirements for hypertensive ur-
need to be treated. Therefore, a cholinergic
gency, intravenous nitroprusside is a poor
agonist is not needed.
choice, as a rapid reduction in blood pressure
may lead to end-organ damage. Answer B is incorrect. No medication is
needed to treat these adverse effects of atro-
Answer C is incorrect. While diuretics may
pine.
lower the circulating blood volume and there-
fore the blood pressure, -blockers, calcium Answer C is incorrect. This patient is not ex-
channel blockers, and ACE inhibitors tend to periencing bicarbonate toxicity. Symptoms of
be more effective. Loop diuretics may be used bicarbonate toxicity include respiratory depres-
as an adjunct if initial treatment fails. sion, hypernatremia, hypokalemia, and edema.
Answer E is incorrect. The patient does not Answer E is incorrect. The patient is not expe-
have signs or symptoms of end-organ damage, riencing transient adverse effects of bicarbonate.
such as headaches, confusion, visual changes,
chest pain, papilledema, retinal hemorrhages, 37. The correct answer is E. When patients stand,
or hematuria; therefore he is not in hyperten- baroreceptors in the carotids and aorta typi-
sive emergency. In addition, sublingual nifedi- cally sense decreased blood ow as blood pools
pine works quickly and may decrease the blood in the venous system due to gravity. This leads
pressure into a range in which hypoperfusion to a sympathetic response which stimulates 1-
and ischemia may occur. Therefore it should adrenergic receptors, causing a reexive vaso-
generally be avoided in hypertensive emer- constriction and increased resistance in order
gency or urgency. to maintain blood pressure. If a patient is tak-
ing an 1-adrenergic blocking agent, this re-
36. The correct answer is D. Pulseless electrical exive vasoconstriction and increased resis-
activity (PEA) is an important reversible brady- tance is blocked, and patients often experience
arrhythmia. It often represents an underlying orthostatic hypotension.
HIGH-YIELD SYSTEMS
36 Section I: Organ Systems Answers

Answer A is incorrect. Prazosin is used to her well-kept appearance, and no physical evi-
treat benign prostatic hyperplasia (BPH). 1- dence of abuse such as bruises or other scars,
adrenergic blockade leads to decreased tone in it is less likely that abuse is the cause of her
the smooth muscle of the prostate and urethra, weight loss. A skeletal survey is not necessary.
thereby improving urine ow in patients with
Answer E is incorrect. This patient does not
BPH.
have a fever or elevated WBC count. Further-
Answer B is incorrect. Prazosin decreases more, the childs symptoms and history are
blood pressure by 1-adrenergic blockade. more consistent with failure to thrive due to
Stimulation of 1-adrenergic receptors causes VSD rather than to infection.
vasoconstriction and increased resistance,
which leads to an increase in blood pressure. 39. The correct answer is E. BNP levels are in-
If stimulation of these receptors is blocked with creasingly utilized in the management of heart
prazosin, then there is vasodilation, decreased failure. It is often used in the setting of short-
resistance, and a decrease in blood pressure. ness of breath when differentiating between a
Cardiovascular

pulmonary and a cardiac etiology. Levels can


Answer C is incorrect. 1-Adrenergic block-
predict heart failure from a systolic and dia-
ade is not known to change sexual drive.
stolic cause with approximately equal accu-
Answer D is incorrect. 1-Adrenergic block- racy; however, BNP cannot differentiate be-
ade is not known to cause irritability. tween the two. Note that N-pro-BNP (a peptide
cleaved in BNP release) is often tested because
38. The correct answer is C. Feeding is a strenu- levels rise higher in patients with LV dysfunc-
ous activity for many infants and can lead to tion.
failure to thrive in children with congenital
Answer A is incorrect. The cardiac natriuretic
heart disease. Most small VSDs close sponta-
peptides have numerous effects and act as
neously during the rst few years of life; how-
counterregulatory hormones in heart failure.
ever, this childs VSD is still present given the
The main physiologic effects are diuresis, hy-
unchanged murmur, and is leading to weight
potension, and antagonism of the renin-angio-
loss. A high-calorie intake has not led to resolu-
tensin system. This results in increased venous
tion of the failure to thrive. Therefore referral
capacitance and decreased preload.
for possible closure of the VSD is the correct
choice. Answer B is incorrect. One of the main uses
of the plasma BNP level is in the diagnosis
Answer A is incorrect. The child is currently
and prognosis of CHF. Numerous studies have
below the fth percentile for weight with an
shown an increase (not decrease) in BNP lev-
identiable etiology. As delayed head growth
els in the setting of LV dysfunction.
may also prevent normal brain development,
this child should be referred for possible clo- Answer C is incorrect. As the name suggests,
sure of her VSD. BNP was initially discovered in the brain but is
mainly secreted by stretch of the cardiac ven-
Answer B is incorrect. Though a pH probe
tricles, not the atria. Atrial natriuretic peptide
could be placed, there are no signs of gastro-
is secreted by the cardiac atria.
esophageal reux disease (GERD) to support
this decision. The child has an uncorrected Answer D is incorrect. BNP is secreted in re-
VSD, which is the more likely cause of failure sponse to volume overload (not hypovolemia),
to thrive in this case. Frequent spitting up or which has a similar stretch effect on the ven-
vomiting and crying or irritability after feed- tricles as pressure overload.
ings would be typical presenting symptoms of
Answer F is incorrect. BNP does not cause an
GERD.
increase in pressure, but rather is secreted in
Answer D is incorrect. Given the appropriate response to pressure overload, which manifests
interaction between the child and her parents, as stretch of the cardiac ventricles.
HIGH-YIELD SYSTEMS
Chapter 1: Cardiovascular Answers 37

40. The correct answer is D. This patient presents dium leads to decreased contractility and the
with the classic triad of symptoms for the diag- symptoms of CHF. The physical exam and di-
nosis of a ruptured abdominal aortic aneurysm agnostic test ndings are classic for this mal-
(AAA): abdominal pain, pulsatile abdominal ady.
mass, and hypotension. In addition, this patient
Answer A is incorrect. Although this patient
has several risk factors for an AAA rupture in-
has a history of asthma, this episode involves
cluding hypertension and chronic obstructive
his cardiovascular system. He also lacks most
pulmonary disease. The skin discoloration
of the signicant signs and symptoms typically
along the left lower back may be due to a retro-
associated with asthma (such as wheezing and
peritoneal hematoma that is associated with a
change in breath sounds).
ruptured AAA. Patients with AAA diameters be-
tween 57 cm have a 5-year risk of rupture of Answer B is incorrect. Delirium tremens (DT)
about 33%. A ruptured AAA can be lethal and is important to consider whenever a patient has
demands immediate surgical attention. When a history of excessive alcohol use. In this case,

Cardiovascular
ruptured AAA is highly suspected, he patient however, the patient still has alcohol on his
should be taken immediately to the operating breath, indicating recent use. DT is caused by
room for surgical repair without further diag- withdrawal of alcohol and should be especially
nostic tests. suspected if a patient has been admitted to the
hospital for a number of days and then begins
Answer A is incorrect. Aortic dissections may
to undergo changes associated with DT.
present with acute onset abdominal pain and/
or back pain and are associated with many of Answer D is incorrect. There are three types of
the same risk factors as AAA, including hyper- endocarditis: native valve, prosthetic valve, and
tension and hypercholesterolemia. Patients that related to intravenous drug use. There is
with aortic dissections do not typically present no mention here of a replacement valve. The
with a pulsatile abdominal mass. other two etiologies are possible in this case,
but the patient is not suffering from fever or
Answer B is incorrect. Although patients with
chills, two prominent symptoms in endocardi-
mesenteric ischemia may present with acute
tis. Anorexia, weight loss, and malaise are other
onset severe abdominal pain with negative ab-
clues that would point to this condition as the
dominal physical exam ndings, this patient
cause, and they are absent in this case.
is more likely to have a ruptured AAA due to
the presence of hypotension and a pulsatile ab- Answer E is incorrect. Hypothyroidism may
dominal mass that are not explained by mesen- masquerade in a variety of ways. However,
teric ischemia. there are many prominent signs and symptoms
that are classic for this condition that are not
Answer C is incorrect. Patients with a perfo-
mentioned here, including weakness, men-
rated gastric ulcer often present with severe ab-
strual irregularities, lethargy, cold intolerance,
dominal pain and distension with free intraperi-
constipation, loss of hair, skin changes, and hy-
toneal air that can be seen on an upright x-ray
pothermia.
of the chest. A diagnosis of a perforated gastric
ulcer would not explain the hypotension or 42. The correct answer is B. PDA is a vascular
pulsatile abdominal mass seen in this patient.
connection that exists between the aorta and
Answer E is incorrect. The absence of neuro- main pulmonary artery. It causes acyanotic
logic abnormalities, in conjunction with the congenital heart disease. PDAs account for
patients presentation and history, support a di- 10% of congenital heart disease, occurring with
agnosis of ruptured AAA rather than stroke. a high incidence in preterm infants and a 2:1
female:male predominance. Small PDAs are
41. The correct answer is C. DCM may be caused asymptomatic, while large ones may cause
by a number of factors, one of the most impor- CHF, failure to thrive, and recurrent lower re-
tant of which is alcohol. Other causes include spiratory tract infections. On physical examina-
chronic CAD, myocarditis, doxorubicin toxic- tion, a continuous machinery murmur heard
ity, and viral infection. The dilated pericar- best at the left upper sternal border, and
HIGH-YIELD SYSTEMS
38 Section I: Organ Systems Answers

bounding peripheral pulses may be present. bicuspid) and thus more prone to stenosis.
There may also be a prominent apical impulse Rheumatic endocarditis tends to cause fusion
and a thrill. X-ray of the chest may show cardi- of the commissures, forming a bicuspid valve;
omegaly and increased pulmonary vascular then the valve broses and calcies. AS of
markings. PDAs usually close within the rst rheumatic origin is highly associated with mi-
month of life. In preterm infants, indometha- tral valve disease. When AS develops gradually,
cin may be administered to close the PDA. the LV responds with concentric hypertrophy
PDAs also may be surgically ligated or coil em- to maintain cardiac output, and thus the trans-
bolized if necessary. valvular pressure gradient increases. Because of
this compensation, AS can exist for years with-
Answer A is incorrect. The murmur of AS with
out symptoms. When the valve orice de-
aortic regurgitation is loudest over the right
creases to <1 cm symptoms may develop. Pa-
sternal border and radiates to the neck and
tients present with dyspnea on exertion
apex. The murmur is usually preceded by an
(because of increased pulmonary capillary
ejection click heard best at the left lower ster-
Cardiovascular

wedge pressure), angina (due to the increased


nal border.
myocardial oxygen requirements), and exer-
Answer C is incorrect. Systemic arteriovenous tional syncope (because exercising muscles va-
stulas are usually extracardiac in location and sodilate and arterial pressure drops in the face
can produce continuous murmurs. However, of a xed carbon dioxide level). Echocardio-
the murmur would not be expected to occur in gram reveals LVH and calcications (if pres-
the location described. The rest of the physical ent). Doppler echocardiogram also allows esti-
examination ndings are also most consistent mation of the transaortic valvular gradient and
with a PDA. of the valvular orice, which are measures of
disease severity. Other valve function can be
Answer D is incorrect. The continuous mur-
assessed concurrently, and HCM causing ob-
mur of venous hum is usually loudest on the
struction can be ruled out.
right (but can also be present on the left) su-
pra- and infraclavicular areas. It usually is in- Answer A is incorrect. ECG in AS usually
audible with the patient in the supine position, shows LVH and can show LV strain, but there
but intensies when the patient is sitting. It is no close correlation between ECG ndings
also disappears with compression of the ipsilat- and severity of disease.
eral jugular vein. It is an innocent murmur that
Answer C is incorrect. This patients history
is produced by alteration in blood ow through
does not suggest seizure.
veins.
Answer D is incorrect. Strenuous physical ex-
Answer E is incorrect. VSDs are the most
ercise should be avoided in patients with severe
common congenital heart defects. They usu-
AS, even if asymptomatic, because their carbon
ally are associated with a holosystolic murmur
dioxide is xed.
that is loudest at the left lower sternal border.
Answer E is incorrect. Although nitroglycerin
43. The correct answer is B. The most likely diag- is helpful in ameliorating angina pectoris, the
nosis is AS. Approximately 80% of patients who vasodilation can precipitate a fall in blood pres-
are symptomatic with AS are male. In the sure that cannot be compensated.
United States, senile calcic stenosis is the
Answer F is incorrect. Tilt test evaluates pa-
most common cause of AS and presents be-
tients for vasovagal syncope. The patients pre-
tween 60 and 80 years of age. Risk factors for
sentation makes this diagnosis less likely.
age-related degenerative calcic AS are the
same as those for atherosclerosis (age, male,
44. The correct answer is A. This patient has
smoking, hypertension, decreased HDL, diabe-
symptoms consistent with pericarditis, or in-
tes mellitus), and the pathology of the valves is
ammation of the pericardial sac. The condi-
similar to that of atherosclerotic vessels. Aortic
tion is most often idiopathic (84% of cases),
valves can also be congenitally abnormal (e.g.,
HIGH-YIELD SYSTEMS
Chapter 1: Cardiovascular Answers 39

with other major causes being neoplasia, tuber- 45. The correct answer is C. The denitive event
culosis, infection, and collagen vascular dis- in obstructive sleep apnea (OSA) is closure of
eases. Uremia in patients with chronic renal the patients upper airway, usually at the level
failure can cause pericarditis, and uremic peri- of the oropharynx. OSA can occur at any age
carditis is an indication for emergent dialysis. but is more common in men 3060 years old.
Pericarditis can be complicated by cardiac tam- The denitive investigation is polysomnogra-
ponade, recurrent pericarditis, or pericardial phy, and the key diagnostic nding is episodes
constriction if left untreated. In patients with of airow cessation at the nose and/or mouth,
cardiac tamponade, ventricular lling is lim- despite evidence of continuing respiratory ef-
ited, which leads to decreased cardiac output fort. Several approaches to the management of
and can lead to shock. Pericarditis is diagnosed OSA have been outlined. For patients with
by the presence of two of the following three ischemic heart disease or CHF who also have
factors: pleuritic chest pain, pericardial friction OSA, treatment should begin with nasal con-
rub, or widespread ST-segment elevation on tinuous positive airway pressure (CPAP). This

Cardiovascular
ECG. Ten to thirty percent of patients with is the only treatment modality that has been
acute pericarditis may go on to develop recur- tested and proved efcacious for patients with
rent or incessant disease, termed chronic auto- ischemic heart disease or CHF and OSA. Na-
reactive pericarditis. sal CPAP prevents upper airway occlusion by
propping the airway open with positive pres-
Answer B is incorrect. If the patients peri-
sure delivered via face mask, with the air acting
carditis was complicated by cardiac tampon-
as a pneumatic stent.
ade, jugular venous pressure would be in-
creased, not decreased, due to elevated right Answer A is incorrect. Alcohol is a frequent
heart lling pressures. contributor to the pathogenesis of OSA be-
cause alcohol has a depressant effect on the
Answer C is incorrect. Mitral regurgitation is
muscles of the upper airway. The structural in-
not a common complication of pericarditis, al-
tegrity of the airway is compromised, and thus
though it could possibly occur in the setting of
the airway is predisposed to occlusion. Avoid-
myocardial ischemia leading to acute papillary
ance of alcohol should be advocated in pa-
muscle rupture.
tients with mild to moderate OSA and a history
Answer D is incorrect. Pericardial constriction of alcohol consumption. However, in patients
and RCM can have similar signs and symptoms. with CHF, nasal CPAP is the only treatment
RCM is dened as impaired ventricular lling that has been proven efcacious and is there-
due to decreased compliance, and is caused in fore the treatment of choice.
some cases by inltrative diseases, such as amy-
Answer B is incorrect. Avoidance of supine
loidosis. Pericardial constriction also impairs
posture should be attempted in patients with
ventricular lling, but it is caused by scarring
mild to moderate OSA. Although it would be
from pericarditis with loss of elasticity of the
reasonable to attempt weight reduction and
pericardial sac. Although pericardial constric-
avoidance of alcohol and supine posture in our
tion is a possible complication of pericarditis,
patient, the best management, given his history
RCM is not. Therefore, RCM is an incorrect
of CHF, is nasal CPAP.
answer choice.
Answer D is incorrect. Uvulopalatopharyn-
Answer E is incorrect. Shock can occur sec-
goplasty is a last-resort surgical procedure that
ondary to cardiac tamponade, but an infec-
increases the airway lumen by reducing the
tious etiology would be unlikely. Furthermore,
quantity of redundant soft tissue. This proce-
the hemodynamics of septic shock and shock
dure results in a long-term cure in <50% of pa-
secondary to tamponade are different. In septic
tients with OSA.
shock, cardiac lling pressures are low, while
there are elevated lling pressures in tampon- Answer E is incorrect. Mild to moderate OSA
ade. can often be managed by moderate weight re-
HIGH-YIELD SYSTEMS
40 Section I: Organ Systems Answers

duction. However, in patients with CHF, na- 47. The correct answer is E. Tricyclic antidepres-
sal CPAP is the only treatment that has been sant (TCA) overdose presents with sedation,
proven efcacious and is therefore the treat- coma, anticholinergic effects, seizures, and ar-
ment of choice. rhythmias. First-line treatment includes so-
dium bicarbonate for QRS prolongation, diaz-
46. The correct answer is E. MVP occurs when epam or lorazepam for seizures, and careful
the leaets of the mitral valve bulge backwards cardiac monitoring for arrhythmias. Sodium bi-
into the left atrium during systole, resulting in carbonate is indicated for QRS widening >100
a characteristic mid- to late-systolic murmur, msec, ventricular arrhythmias, and/or hypoten-
often associated with a preceding click. It is sion. The benet of sodium bicarbonate is due
caused by an underlying structural abnormality both to an increase in serum pH and extracel-
of the mitral valve, and seems to be more com- lular sodium. Elevated serum pH lowers free
mon in adolescent girls. Consider antibiotic drug concentrations and favors the neutral
prophylaxis in patients with a mid-systolic click form of the drug, making less available to bind
Cardiovascular

and late-systolic mitral regurgitation murmur, to sodium channels. Increasing extracellular


including those with increased leaet thicken- sodium increases the electrochemical gradient
ing or redundancy, left atrial enlargement, and across cardiac cell membranes, thus attenuat-
LV dilatation, even in the absence of corre- ing TCA-induced blockade of rapid sodium
lated clinical ndings. Due to the transient channels.
bacteremia following dental procedures, pro-
Answer A is incorrect. Activated charcoal is
phylactic antibiotics are recommended since
important for patients who do not have neu-
the prolapsed valve can serve as a nidus for col-
rologic or cardiovascular components to their
onization.
presentation. For patients who are neurologi-
Answer A is incorrect. Digoxin does not play a cally depressed, an endotracheal airway must
role in the management of MVP. rst be established prior to giving charcoal.
Answer B is incorrect. MVP, unlike hyper- Activated charcoal shows the greatest benet
trophic cardiomyopathy, does not have an in- when administered within 1 hour of ingestion.
creased risk of sudden death and therefore does Some agents that are not absorbed well by ac-
not require restriction of activity. tivated charcoal include heavy metals such as
lead and mercury, inorganic ions such as cal-
Answer C is incorrect. -Blockers are not in- cium or potassium, acids or alkali, hydrocar-
dicated since the patient is asymptomatic and bons, and alcohols such as acetone or ethylene
without a history of known cardiac disease. glycol.
Answer D is incorrect. Surgical valvular re- Answer B is incorrect. Diazepam will control
placement is reserved for more advanced cases the seizure activity but not the cardiac or anti-
of valvular disease in which the patient has re- cholinergic components. TCAs are thought to
fractory symptoms. Various symptoms (includ- induce seizure by decreasing central GABAer-
ing atypical chest pain, exertional dyspnea, gic tone. Benzodiazepines are the preferred an-
palpitations, syncope, and anxiety) and clini- tiseizure agents due to the resulting increase in
cal ndings (including low blood pressure and GABAergic tone. Antiseizure medications that
electrocardiographic repolarization abnormali- block sodium channels, such as phenytoin,
ties) have been associated with MVP and have should be avoided, as they promote ventricular
been termed MVP syndrome. Valve surgery is arrhythmias by potentiating the sodium chan-
clearly warranted for patients with symptom- nel blockade already present from the TCA
atic severe mitral regurgitation, mitral regur- toxicity.
gitation accompanied by atrial brillation or
Answer C is incorrect. Flumazenil is used for
pulmonary hypertension, and those who are as-
benzodiazepine sedation reversal but has been
ymptomatic but have LV enlargement or even
associated with onset of ventricular arrhythmias
mildly reduced systolic function.
and seizures.
HIGH-YIELD SYSTEMS
Chapter 1: Cardiovascular Answers 41

Answer D is incorrect. Physostigmine is the at risk for atherosclerosis of the carotid arteries.
antidote for anticholinergic overdose and The carotid bruit arises from turbulent blood
should be avoided in patients with a TCA over- ow caused by an atherosclerotic plaque. The
dose secondary to association with seizures and next appropriate step for this patient would be
cardiac arrest. a diagnostic test to assess the severity of carotid
artery stenosis. Although a traditional carotid
48. The correct answer is A. This patient has per- arteriogram remains the gold standard, it
sistent atrial brillation. Her risk for the disor- comes with costs and risks that make it a less
der is greatly increased by her history of rheu- likely rst choice for asymptomatic carotid ar-
matic fever and presumed rheumatic heart tery stenosis screening.
disease and is varied depending on the severity
Answer B is incorrect. Carotid endarterectomy
of valvular disease. The ECG shown is charac-
is the procedure indicated in asymptomatic pa-
teristic of atrial brillation, showing an irregu-
tients with 60%99% carotid artery stenosis or
lar baseline, no clear P waves, and irregular
symptomatic patients with >50% stenosis. This

Cardiovascular
and varied QRS complexes. Rheumatic heart
patients extent of stenosis is not yet known, so
disease can cause deformity of the valve cusps,
further work-up is indicated before surgery.
fusion of the commissures, and shortening and
fusion of the chordae tendineae. Stenosis of Answer C is incorrect. An ECG is useful in
the mitral valve is present in 50%60% of the assessing for cardiac disease, but does not play
cases, and combined aortic/mitral valve lesions a role in evaluation of carotid artery disease. If
occur about 20% of the time. Therefore an as- carotid endarterectomy is indicated in this pa-
tute clinician should auscultate for a mitral tient, an ECG would likely be needed in the
stenosis murmur (an diastolic opening snap fol- preoperative evaluation.
lowed by a late rumbling diastolic murmur).
Answer D is incorrect. Although a referral to
The increased pressure in the left atrium from
a neurologist is appropriate, if the patient has
the mitral stenosis can cause atrial brillation
symptomatic coronary artery stenosis with tran-
(which will need to be anticoagulated). This
sient ischemic attacks, then he needs imaging
patient should also receive prophylactic antibi-
to assess the degree of coronary artery stenosis.
otics for dental, urologic, and surgical proce-
dures to prevent endocarditis. Answer E is incorrect. Transthoracic echocar-
diography is useful to assess for heart function
Answer B is incorrect. The ECG of a pa-
and valvular disease, but does not play a role in
tient with atrial utter would reveal a regular
evaluation of carotid artery disease.
rhythm and P waves in a sawtooth pattern.
Answer F is incorrect. Antiplatelet therapy
Answer C is incorrect. The presence of mul-
has been demonstrated to decrease the rate of
tiple pacemakers or reentrant pathways in mul-
stroke in patients with carotid artery disease. In
tifocal atrial tachycardia leads to multiple (>3),
contrast, warfarin therapy is used for cardiac
varied P-wave morphologies on ECG.
disease.
Answer D is incorrect. An unusual P wave be-
fore each normal QRS complex would be seen 50. The correct answer is B. Cocaine toxicity pro-
in paroxysmal atrial tachycardia, in which an duces a hyperadrenergic state which is identi-
ectopic atrial pacemaker overrides the native ed by hypertension, tachycardia, tonic-clonic
rhythm of the heart. seizures, dyspnea, and ventricular arrhythmias.
Cocaine produces vasoconstriction in coronary
Answer E is incorrect. An atrioventricular
arteries, resulting in hypertension and bradycar-
nodal reentry circuit in paroxysmal supraven-
dia. Nonselective -adrenergic blockade may
tricular tachycardia leads to P waves hidden
worsen this effect, suggesting that this vasocon-
within normal QRS complexes on ECG.
striction may be mediated by -adrenergic re-
ceptors and antagonized by -adrenergic recep-
49. The correct answer is A. This patients history
tor-mediated vasodilatation.
of hypertension and hyperlipidemia places him
HIGH-YIELD SYSTEMS
42 Section I: Organ Systems Answers

Answer A is incorrect. Cocaine withdrawal Answer C is incorrect. Hypertension can lead


produces a late, dopamine-related vasospasm. to LVH and then eventually to DCM if al-
However, -blockers do not worsen this effect. lowed to progress. After the ventricle hypertro-
phies from excess workload, it can later dilate
Answer C is incorrect. Hypertension is a hy-
and its ejection fraction can decrease. How-
peradrenergic effect of cocaine toxicity. Using
ever, this patient has no signs or symptoms that
a -blocker will cause vasoconstriction which
would suggest DCM, which makes this diag-
may exacerbate hypertension and bradycardia,
nosis unlikely.
but will not cause hypotension.
Answer E is incorrect. This patient has no
Answer D is incorrect. Dyspnea is not directly
signs (pericardial friction rub) or symptoms
related to cocaine toxicity. Pulmonary com-
(pleuritic chest pain, dyspnea, cough, or fever)
plications associated with cocaine ingestion
that would suggest pericarditis. In addition, if
include pulmonary hemorrhage, barotrauma,
the patient had pericarditis, one would expect
asthma exacerbation, pneumonitis, and pul-
to see diffuse ST-segment elevation on ECG.
monary edema. -blocker toxicity can be asso-
Cardiovascular

Although this patients ECG shows ST/T


ciated with pulmonary complications such as
changes, it does not show diffuse ST-segment
bronchospasm and wheezing.
elevation.
Answer E is incorrect. Ventricular arrhyth-
mias are associated with cocaine toxicity but 52. The correct answer is D. Osler-Weber-Rendu
not specically a result of -blocker treatment syndrome, also known as hereditary hemor-
of cocaine induced hypertension. Cocaine can rhagic telangiectasia, is an autosomal domi-
cause dysrhythmias, myocarditis, myocardial nant brovascular dysplasia in which vascular
ischemia, infarction and cardiomyopathy. lesions (telangiectasias, arteriovenous malfor-
mations, and aneurysms) are found throughout
51. The correct answer is D. LVH occurs as a con- the body, particularly in the lungs, brain, and
sequence of uncontrolled hypertension. In- gastrointestinal tract.
creased pressures lead to increased workload
Answer A is incorrect. Hypertension, bradycar-
for the heart and cardiac muscle hypertrophy.
dia, and irregular respirations constitute Cush-
Patients with LVH may have a prominent point
ings triad, which is suggestive of increased in-
of maximum impulse, loud S2, and audible S4
tracranial pressure. Although the patient may
from an atrial kick. In addition, there may be
have suffered a head injury in the motor vehi-
signs of LVH on ECG; however, LVH can of-
cle collision, there is no reason to suspect that
ten be detected with echocardiography far ear-
she had increased cranial pressure prior to the
lier than it can be detected on an ECG. On
collision.
this patients ECG, the large amplitude of the
S in V1 and V2, of the R in V5 and V6, the dif- Answer B is incorrect. Jaundice, fever and
fuse ST/T wave changes, the widened bid P right upper quadrant pain constitute Charcots
waves (indicating left atrial enlargement), and triad, which is suggestive of ascending cholan-
the prolonged QRS all suggest LVH. gitis. Osler-Weber-Rendu syndrome can result
in hepatomegaly and right upper quadrant
Answer A is incorrect. Acute MI is unlikely in
pain secondary to hepatic atrial ventricular s-
this patient because he is asymptomatic. This
tulas, but fever would not be present in such
patient has ST/T changes on his ECG, but
patients.
these changes are displacements in the op-
posite direction to the major deection of the Answer C is incorrect. Symptoms of hypogly-
QRS complex, which is more suggestive of cemia, documented low blood sugar and relief
LVH, not acute MI. of symptoms with increase in blood sugar con-
stitute Whipples triad, the gold standard for di-
Answer B is incorrect. Although hypertension
agnosing hypoglycemia.
increases a patients risk of cerebrovascular ac-
cident, his patient has no signs or symptoms Answer E is incorrect. Venous stasis, hyperco-
that would support this diagnosis. agulability, and endothelial damage constitute
HIGH-YIELD SYSTEMS
Chapter 1: Cardiovascular Answers 43

Virchows triad, which lists possible etiologies vignette suggests that angina is a more likely
for thrombosis. cause.
Answer E is incorrect. A compression fracture
53. The correct answer is C. The patient presents
of the spine can often be diagnosed by palpa-
with atrial brillation and hemodynamic insta-
tion of the spinal column. This is common in
bility. He is hypotensive and tachycardic and
elderly patients.
has altered mental status. The appropriate
treatment is immediate cardioversion. Answer F is incorrect. Coronary vasospasm of-
ten occurs idiopathically in young women or is
Answer A is incorrect. Adenosine is indicated to
often seen in the setting of cocaine use. ECG
treat atrioventricular nodal reentry tachycardia.
may reveal transient ST-segment changes.
The heart rate in this condition is 150250/min,
and the P wave is often buried in the QRS or Answer G is incorrect. Be aware that esopha-
shortly after. geal spasm can be very similar to cardiac chest
pain. In both cases described there is another
Answer B is incorrect. Ischemia is a cause of

Cardiovascular
more likely etiology.
atrial brillation. However, in the acute setting
cardioversion should be performed on a hemo- Answer H is incorrect. Myocarditis has a wide
dynamically unstable patient with atrial bril- range of presentations from chest pain to sud-
lation. den death, depending on the etiology. It often
manifests as fatigue, decreased exercise toler-
Answer D is incorrect. Carotid massage and
ance, or heart failure in a previously healthy
the Valsalva maneuver are two tools that may
individual.
be used instead of adenosine to treat AVNRT.
Answer I is incorrect. Panic disorder is a fre-
Answer E is incorrect. This patient may even-
quent cause of chest pain. However, in both
tually require a pacemaker depending on the
cases presented there is another more likely
underlying nature of his disease. However, the
etiology.
most appropriate treatment of a hemodynami-
cally unstable patient is cardioversion. Answer J is incorrect. Pericarditis is often man-
ifest by severe chest pain, a rub on cardiac ex-
amination, and diffuse ST-segment changes on
Questions 54 and 55 ECG.
54. The correct answer is D. The patient presents Answer K is incorrect. Pneumonia can pre-
with the classic triad of hypotension, jugular sent as chest pain, but other signs of infection
venous distension, and distant heart sounds would be expected.
(Becks triad), indicative of cardiac tamponade.
There is also a plausible mechanism given the Answer L is incorrect. Pulmonary embolus of-
traumatic left chest wound. ten presents as chest pain associated with short-
ness of breath and hemoptysis. Risk factors for
55. The correct answer is C. Symptoms of cardiac deep venous thrombosis (hypercoagulability,
ischemia are often atypical in women, the el- malignancy, elderly, nonambulatory, and re-
derly, and patients with diabetes. Note that the cent surgery) would likely be present.
usual duration of angina is a few minutes to Answer M is incorrect. A rib fracture can ob-
half an hour, with pain gradually increasing viously result in chest pain but usually in the
and improved with rest. setting of trauma. Nothing in either vignette
Answer A is incorrect. Acute aortic dissection suggests a rib fracture.
can be catastrophic and often presents as tear- Answer N is incorrect. A tension pneumotho-
ing chest pain radiating to the back. rax occurs when there is a one-way valve cre-
Answer B is incorrect. Acute MI can present ated by an abnormal connection between the
atypically in the elderly and women. The tran- lungs and the pleural space. It manifests as re-
sient nature of the symptoms in the second spiratory distress, deviated trachea, hypoxemia,
HIGH-YIELD SYSTEMS
44 Section I: Organ Systems Answers

jugular venous distension, and distant breath Answer G is incorrect. Lisinopril and other
sounds on the affected side. Treatment is with ACE inhibitors are considered teratogenic be-
immediate decompression. cause they cause renal failure in neonates, re-
nal tubular dysgenesis, and decreased skull os-
sication, and should be avoided in pregnancy.
Questions 56 and 57 An ACE inhibitor would be considered one of
56. The correct answer is H. This patient likely the rst-line treatments for CHF, as it has been
suffered DCM as a result of doxorubicin toxic- shown to decrease mortality. However, in the
ity, which is now being exacerbated by the car- patient in the second vignette an ACE inhibi-
diopulmonary stress of pregnancy. Any medical tor is making the patient cough, so an ARB
intervention should take into account the pos- would be preferred.
sible teratogenic effects on the child. A Answer I is incorrect. Warfarin is a teratogen
-blocker such as metoprolol would be an ap- causing skeletal and central nervous system
propriate treatment for heart failure in this defects, and should be avoided in pregnancy.
Cardiovascular

case. Of note, this patient would also most Warfarin is not a treatment for CHF.
likely benet from a loop diuretic given her
present volume status.
Questions 58, 59, and 60
57. The correct answer is A. ACE inhibitors and
58. The correct answer is M. The patient has car-
angiotensin receptor blockers (ARBs) are the
diac tamponade secondary to uremic pericardi-
most effective treatments for CHF. Since many
tis. The usual treatment for cardiac tamponade
patients suffer from cough as an idiopathic ad-
is pericardiocentesis. However, in relatively sta-
verse effect of ACE inhibitors, one should be
ble patients with cardiac tamponade secondary
familiar with ARBs as a possible alternative.
to uremic pericarditis, the treatment of choice
ARBs can have serious potential adverse effects
is to initiate or intensify dialysis.
on a fetus.
Answer B is incorrect. Digoxin is an effective 59. The correct answer is F. The patient has car-
symptomatic treatment for heart failure, but diac tamponade secondary to a hemopericar-
has no proven effect on mortality or disease dium. If uid is rapidly introduced into the
progression. pericardium, it cannot stretch to accommodate
the additional volume; therefore, the intraperi-
Answer C is incorrect. Erythropoietin would
cardial pressure rises quickly. A very small vol-
be useful in treating certain types of anemia,
ume of uid in the pericardium can lead to
which can cause fatigue. However, since the
tamponade if introduced rapidly enough, as
fatigue described in second case is most likely
seen in this patient. Emergent pericardiocente-
due to CHF, this medication would be inap-
sis is recommended in the case of cardiac tam-
propriate.
ponade in a hemodynamically unstable pa-
Answer D is incorrect. Folate would be indi- tient.
cated in patients with signs and symptoms of
folate deciency as a cause of fatigue. 60. The correct answer is L. The patient has signs
and symptoms of viral pericarditis. Although
Answer E is incorrect. Heparin is safe and ef-
she has a moderate pericardial effusion seen on
fective during pregnancy but requires intrave-
x-ray of the chest and echocardiography, her
nous administration. However, neither patient
pericardium has stretched to accommodate the
is in need of anticoagulation.
excess uid, so she is not experiencing cardiac
Answer F is incorrect. Isoniazid is a treatment tamponade. High-dose nonsteroidal anti-inam-
for tuberculosis, another possible cause of fa- matory drugs (NSAIDs) are rst-line therapy in
tigue. However, tuberculosis is not suggested viral or idiopathic pericarditis. Her effusion
in either vignette. should resolve over time without pericardiocen-
tesis.
HIGH-YIELD SYSTEMS
Chapter 1: Cardiovascular Answers 45

Answer A is incorrect. Antibiotics are useful be helpful in cardiac tamponade if the patient
for treating bacterial pericarditis. Patients with is hypovolemic at baseline, but aggressive intra-
bacterial pericarditis, also known as purulent venous uid administration can worsen or pre-
pericarditis, are typically acutely ill with sepsis- cipitate cardiac tamponade in patients who are
like symptoms. There usually is a concurrent euvolemic or hypervolemic at baseline.
infection such as pneumonia, meningitis, or
Answer H is incorrect. Loop diuretics are used
osteomyelitis. Although the incidence is low
to treat CHF, among other disease states, but
since the onset of antibiotic use, the most com-
are not indicated in the treatment of pericardi-
mon organism causing bacterial pericarditis is
tis or cardiac tamponade.
Staphylococcus aureus.
Answer I is incorrect. Morphine is used to
Answer B is incorrect. Chest tubes are not in-
treat myocardial ischemia or infarction.
dicated in pericarditis, pericardial effusion, or
cardiac tamponade. Answer J is incorrect. Multidrug antitubercu-
lous therapy is used to treat tuberculous peri-

Cardiovascular
Answer C is incorrect. Colchicine is often
carditis. Pericardial involvement can occur
used in combination with NSAIDs to treat
with any bacterial infection; however, the most
pericarditis, but NSAIDs are rst-line treat-
frequent pathogens are Staphylococcus, Pneu-
ment for pericarditis.
mococcus, Streptococcus (rheumatic pancardi-
Answer D is incorrect. Corticosteroids can be tis), Haemophilus, and Mycobacterium tuber-
used to treat pericarditis but are recommended culosis.
only in recurrent pericarditis refractory to
Answer K is incorrect. Nitroglycerin is used
NSAIDs and colchicine or in immune-mediated
to treat myocardial ischemia or MI. Pericardi-
pericarditis.
tis and effusion are common events following
Answer E is incorrect. Emergency cardiac acute MI. Acute inammation associated with
catheterization is not indicated in the treat- the infarct can cause early post-MI pericardial
ment of pericarditis or cardiac tamponade. disease. Pericardial disease that occurs weeks
to months post-MI is related to immunologic
Answer G is incorrect. Intravenous uids are
mechanisms.
often given in cardiac tamponade. They can
This page intentionally left blank
CHAPTER 2

Dermatology

47
HIGH-YIELD SYSTEMS
48 Section I: Organ Systems Questions

Q U E ST I O N S

1. A 15-year-old boy presents to his physician (A) Atopic dermatitis


complaining of a rash. He rst noticed a dry, (B) Contact dermatitis
red patch on his back. Then smaller patches (C) Erythema infectiosum
started to appear on his shoulders, and over the (D) Impetigo
past week, the rash has spread to his trunk. The (E) Seborrheic dermatitis
rash itches, but he otherwise feels well. He had
a slightly sore throat 2 weeks ago but denies fe- 3. A 42-year-old man presents for evaluation of a
ver, cough, or other symptoms of upper respi- mole on his back that his wife noticed has
ratory infection. He is sexually active with his changed in size. She is not present but told
girlfriend and they use condoms. He denies him that it used to be smaller. He is fair
travel or exposure to tick bites. On examination skinned and admits to never using sunscreen.
there are erythematous scaly papules and He has always had numerous freckles and
Dermatology

plaques as shown in the image. There are no moles but has no personal or family history of
other signicant ndings on examination. skin cancer. Examination shows the lesion
Rapid plasma reagin test is negative. Which of seen in the image. He also has approximately
the following is the most likely diagnosis? 30 other small, round nevi on his arms and
back. Which of the following features is most
predictive of poor outcome in this case?

Reproduced, with permission, from Wolff K, Johnson RA,


Surmond D. Fitzpatricks Color Atlas & Synopsis of Clini-
cal Dermatology, 5th edition. New York: McGraw-Hill,
2005: Figure 7-1. Reproduced, with permission, from PEIR Digital Library
(http://peir.net).
(A) Guttate psoriasis
(B) Lyme disease (A) Asymmetric shape
(C) Pityriasis rosea (B) Diameter >6 mm
(D) Secondary syphilis (C) Irregular borders
(E) Tinea corporis (D) Tumor thickness
(E) Variation in color
2. A 14-year-old girl presents to her primary phy-
sician with an intensely pruritic rash on both 4. A 26-year-old man presents to the emergency
hands and on her right cheek. The rash con- department with burns on his chest. He had a
sists of vesicles on erythematous plaques ar- ght with his girlfriend and she threw boiling
ranged linearly with slight crusting. She denies water at him, splashing his chest and arms. The
fever or sore throat and otherwise feels well. burns occurred about an hour ago, and are dis-
She does not have a history of eczema or sick tributed on the upper third of his left anterior
contacts. Which of the following is the most trunk and cover most of his left proximal arm.
likely diagnosis? The patients temperature is 37.4C (99.4F),
blood pressure is 127/74 mm Hg, pulse is 80/
min, respiratory rate is 18/min, and oxygen satu-
HIGH-YIELD SYSTEMS
Chapter 2: Dermatology Questions 49

ration is 99% on room air. The burns are quite (A) Cosyntropin stimulation test
painful, swollen, and erythematous, with blister (B) Fasting blood glucose
formation. The application of pressure produces (C) Fasting lipid panel
blanching and is quite painful. Which of the (D) Serum electrolytes
following is the most appropriate management? (E) Thyroid-stimulating hormone
(A) Admission to hospital and intravenous an-
6. A 47-year-old man with a history of recent sub-
tibiotic administration
clinical hepatitis C infection presents com-
(B) Cleaning and dressing of the burns, and
plaining of rash and mouth pain for the past
analgesics as needed
week. The rash is pruritic. Examination reveals
(C) Lubricant application and analgesics as
lesions on his wrists, ankles, and scalp; the le-
needed
sions are shiny, violaceous, sharply demar-
(D) Referral to a burn center
cated, conuent papules containing ne white
(E) Surgical evaluation for debridement and
lines in a lacy pattern on their surfaces. Exami-
grafting
nation of his oropharynx reveals an erosion on

Dermatology
the left buccal mucosa with the same ne
5. A 47-year-old man with no signicant past
white reticulation. Which of the following is
medical history presents to the ofce complain-
the most likely diagnosis?
ing of a new rash in his armpits. He rst no-
ticed it 1 month ago, and since then it has (A) Erythema multiforme
grown darker, thicker, and larger. He reports (B) Hypersensitivity vasculitis
that it is occasionally mildly pruritic. His vital (C) Lichen planus
signs are normal. Upon examination he has hy- (D) Secondary syphilis
perpigmented thick plaques in both axillae (E) Viral exanthema
(see image). Physical examination is otherwise
unremarkable. Which of the following is the 7. A 19-year-old woman presents to her primary
most appropriate laboratory test to order? care physician complaining of excessive bruis-
ing on her legs for the past 3 days. She denies
injury. She was treated for streptococcal throat
infection 10 days ago and recently completed
antibiotic therapy. She has had some cramping
abdominal pain, but she is premenstrual and
says the pain is similar to her usual cramps.
She took ibuprofen for the pain, with good re-
lief. She has a boyfriend but is not sexually ac-
tive. She denies previous history of bruising or
bleeding easily. Inspection of her legs reveals
diffuse tender, erythematous, indurated patches
and nodules over the anterior aspects of her
tibias bilaterally. Which of the following is the
most likely cause of her symptoms?
(A) Domestic violence
(B) Erythema nodosum
(C) Henoch-Schnlein purpura
(D) Idiopathic thrombocytopenic purpura
(E) Secondary syphilis

Reproduced, with permission, from Wolff K, Johnson RA,


Surmond D. Fitzpatricks Color Atlas & Synopsis of Clini-
cal Dermatology, 5th edition. New York: McGraw-Hill,
2005: Figure 5-1.
HIGH-YIELD SYSTEMS
50 Section I: Organ Systems Questions

8. A 25-year-old HIV-positive man presents to his


primary physician because he has been ex-
posed to herpes. He is concerned because he
had a friend with AIDS who developed a fatal
disseminated herpes infection and is afraid the
same thing might happen to him. The expo-
sure occurred 2 days ago when he shared an
ice cream bar with his niece, whom he noticed
afterward had an oral lesion. To his knowledge
he has never had an oral lesion. Neither he nor
his partner has ever had cold sores. Which of
the following is the most appropriate approach
to this patient?
(A) Admit to hospital for initiation of intrave-
Dermatology

nous anti-herpes simplex virus immuno-


Reproduced, with permission, from Wolff K, Johnson RA,
globulin therapy
Surmond D. Fitzpatricks Color Atlas & Synopsis of Clini-
(B) Admit to hospital for intravenous acyclovir cal Dermatology, 5th edition. New York: McGraw-Hill,
therapy 2005: Figure 10-21.
(C) Follow patient closely for development of
complications, but it is too late to initiate (A) Local extension and tissue destruction
acyclovir therapy (B) Progression to basal cell carcinoma
(D) Prescribe oral acyclovir, ve times daily for (C) Progression to malignant melanoma
7 days and follow closely for clinical dis- (D) Progression to squamous cell carcinoma
ease (E) This lesion does not have malignant
(E) Reassure him that he is unlikely to develop potential and further evaluation is not
severe disease and that he will probably necessary
contract herpes simplex virus-1 sooner or
later 10. A 55-year-old woman presents to her gynecolo-
gist complaining of vaginal discomfort. It rst
9. A 68-year-old man presents to his primary care started approximately 6 months ago and has
physician for evaluation of scabs on his ear uctuated in intensity, most recently causing
that have failed to resolve over the past year. itching and some slight pain when she has in-
He is a retired vineyard manager and has tercourse with her husband. She otherwise has
worked outside for most of his career. He does been well. She has been postmenopausal for 3
not have any family or personal history of skin years. Her older sister recommended an estro-
cancer. On examination he has two macular, gen cream, which she has used consistently for
scaling lesions on his face, 2 and 4.5 mm in di- >3 months without any change in symptoms.
ameter. They are hyperkeratotic with surround- On examination the introital mucosa and labia
ing erythema (see image). These lesions should minora are whitish-pink, with abnormal wrin-
be biopsied and observed carefully to prevent kling and a small ssure on the right labia mi-
which of the following? nora. Which of the following is the most likely
diagnosis?
(A) Candidiasis
(B) Estrogen deciency
(C) Lichen planus
(D) Lichen sclerosus
(E) Sexual abuse
HIGH-YIELD SYSTEMS
Chapter 2: Dermatology Answers 51

AN S W E R S

1. The correct answer is C. Pityriasis rosea may Answer E is incorrect. While tinea corpo-
be preceded by a prodrome of headache, mal- ris may be confused with the herald patch of
aise, and/or sore throat, but is most often as- pityriasis rosea, tinea corporis does not typically
ymptomatic. The diagnosis is made based on result in the numerous lesions seen in this pa-
history and physical examination. The rash is tient. Extensive eruptions of tinea corporis may
pruritic and begins with a herald patch. It then occur in immunocompromised patients such
classically spreads downward or centrifugally as those with diabetes mellitus or HIV infec-
on the trunk and proximal extremities. The tion, but there is no history to suggest that this
herald patch is erythematous, round, and clears is the case here.
centrally with a peripheral scale. The following
lesions are oval or oblong, with the long axes 2. The correct answer is B. Contact dermatitis
aligned with skin cleavage lines. The rash re- causes an acute eczematous rash and results

Dermatology
solves spontaneously within 23 months. The from a type IV hypersensitivity reaction to an
etiology of pityriasis rosea is not well under- allergen. In this case, the allergen was likely
stood but is thought to be virally mediated, pos- poison ivy resin because she presents with a
sibly secondary to a reactivation of human her- characteristic rash. In addition, the lesions are
pesvirus-7. arranged perfectly linearly, suggesting that the
cause is external to the patient (i.e., a plant),
Answer A is incorrect. Guttate psoriasis usu-
rather than an internal dermatologic disease.
ally occurs as an acute eruption of small (110
mm) salmon-pink papules without central Answer A is incorrect. Patients with eczema
clearing, primarily involving the trunk. Erup- typically manifest symptoms by ages 57 years.
tion may occur after a streptococcal infection Lesions in atopic dermatitis are not character-
(usually pharyngitis) in a child or young adult. ized by linear distributions. In adolescents and
Though this patient describes a recent sore adults, the most common sites of involvement
throat, the rash described is not typical for gut- are exural surfaces, wrists/forearms, and face.
tate psoriasis.
Answer C is incorrect. Erythema infectiosum,
Answer B is incorrect. Erythema migrans sec- or fth disease, typically causes the slapped
ondary to Lyme disease is an erythematous oval cheek rash (edematous, erythematous
lesion with central clearing that may be con- plaques) in a febrile patient. Parvovirus B19 is
fused with the herald patch of pityriasis rosea. the infectious agent.
The classic bulls-eye occurs in only a minor-
Answer D is incorrect. Impetigo is an epider-
ity of cases. Erythema migrans may even itch. mal infection with staphylococci or strepto-
However, this lesion typically occurs near the cocci that typically occurs on the face, neck, or
tick bite, in warm areas of the body. The loca- extremities. The rash is vesicular with honey-
tion of the herald patch in this case and the colored crusting and can be intensely pruritic,
subsequent appearance of multiple lesions are but it is not typically described as linear.
very atypical for erythema migrans.
Answer E is incorrect. Seborrheic dermatitis
Answer D is incorrect. The rash of second- can resemble the rash described but occurs
ary syphilis is typically symmetric and charac- in a distribution consistent with the locations
teristically involves the palms and soles. The of sebaceous glands. Thus, although the eye-
lesions are red and often scaled. While sore brows, nasolabial folds, and scalp are common
throat may occur as part of secondary syphilis, locations, the hands are not.
other symptoms including headache, fever, an-
orexia, myalgias, and diffuse lymphadenopathy 3. The correct answer is D. The lesion pictured
are not present in this case. A negative rapid is a malignant melanoma. Melanomas are rec-
plasma reagin test effectively rules out second- ognizable by the ABCDEs (Asymmetric shape,
ary syphilis as a possible etiology. Borders irregular, Color variation, Diameter
HIGH-YIELD SYSTEMS
52 Section I: Organ Systems Answers

>6 mm, and Enlargement or Evolution of the to adjacent areas of partial-thickness burn. The
lesion). This mnemonic is a useful way for skin may be charred or white in color, with vis-
both patients and physicians to recognize when ible blood vessels. Healing is slower than with
a previously ordinary mole should be evaluated less severe burns, because sweat glands and
for melanoma. However, this mnemonic is not hair follicles (the source of skin stem cells) are
useful for determining the prognosis for sur- destroyed. Fourth-degree burns involve under-
vival of a patient with primary melanoma. Tu- lying muscle and/or bone.
mor thickness (or Breslow depth) determined
Answer A is incorrect. Intravenous antibiotics
on biopsy has been shown to be the most pow-
are indicated for treatment of infected burns.
erful prognostic factor in primary melanomas.
There is no proven benet associated with pro-
Answer A is incorrect. Asymmetry can be a phylactic intravenous or oral antibiotic treat-
worrisome feature in nevi, but this is not a use- ment of burn patients.
ful prognostic feature once the lesion is identi-
Answer C is incorrect. Analgesics (and anti-
ed as malignant.
histamines for pruritus) are used to symptom-
Dermatology

Answer B is incorrect. A lesion >6 mm in di- atically treat supercial or rst-degree burns.
ameter should be considered malignant un- These burns can be distinguished from super-
til proven otherwise (by biopsy), but it is the cial partial-thickness or second-degree burns
depth of the lesion that will best predict sur- by the absence of blister formation. Lubricants
vival. and moisturizers (including aloe vera) may also
provide symptomatic relief; however, they are
Answer C is incorrect. Absence of a round,
recommended only for rst-degree burns and
clear-cut border (whether raised or at) raises
second-degree burns smaller than 23 inches
suspicion for the presence of a malignant le-
in diameter.
sion but is not useful for predicting survival.
Answer D is incorrect. Criteria for referral to
Answer E is incorrect. Variation of color
a burn center include a partial-thickness burn
within the lesion is suspicious for malignancy
covering at least 20% total BSA in an adult, at
but does not correlate with prognosis.
least 10% in a child <10 years old, or at least
10% in an adult at least 50 years old; or a full-
4. The correct answer is B. The burns described
thickness burn of at least 5% BSA at any age.
here are supercial partial-thickness burns (also
Special circumstances indicating referral to a
called second-degree burns, affecting the epi-
burn center include inhalational injury; suspi-
dermis and portions of the dermis), involving
cion of abuse; signicant burns to face, genita-
approximately 10% of his body surface area
lia, or joints; and signicant associated injuries
(BSA) according to the rule of nines (anterior
(i.e., fractures). This patient does not meet
trunk represents 18% BSA total, so one-third =
these criteria.
6%, and each arm represents 9%, so one-half =
4.5%). Pain, swelling, and blistering helps dis- Answer E is incorrect. Surgical evaluation for
tinguish partial-thickness burns. These burns debridement and grafting is indicated if burns
can be managed in the ambulatory setting, with become infected, or if necrotic tissue is pres-
appropriate cleansing, debridement if necessary, ent.
dressing, and appropriate pain management.
These burns should heal in 1 to 3 weeks with 5. The correct answer is B. Acanthosis nigricans
minimal scarring, but may potentially result in is a dermatologic nding characterized by hy-
pigmentation changes. First-degree burns, such perpigmented and thickened patches that are
as the typical sunburn, affect the epidermis only. most often found in the axillae or on the back
Tissue is erythematous and blanches to pres- of the neck, but that are occasionally found in
sure, and damage is minimal. Healing occurs other skin fold areas or on the hands. The nd-
spontaneously. Third-degree burns, or full-thick- ing is most commonly associated with diabetes
ness burns, affect the entire epidermis and der- mellitus or insulin resistance; thus, the most
mis. The area of the burn itself is painless,
though surrounding tissue is usually tender due
HIGH-YIELD SYSTEMS
Chapter 2: Dermatology Answers 53

appropriate laboratory test would be a fasting pruritus can be present as well. Mucous mem-
blood sugar level. brane involvement is common.
Answer A is incorrect. A cosyntropin stimula- Answer B is incorrect. Hypersensitivity vasculi-
tion test would be a conrmatory test for ad- tis is typically drug-induced but may also result
renal insufciency after checking morning from hepatitis B or C infection. It manifests as
cortisol levels. However, this disorder does not palpable purpura and skin biopsy shows a leu-
cause acanthosis nigricans. kocytoclastic vasculitis.
Answer C is incorrect. High cholesterol levels Answer D is incorrect. The rash of secondary
can cause xanthomas, which are subcutaneous syphilis may consist of mucous membrane and
plaques of cholesterol that are often present on genital lesions with a more diffuse rash (usually
the ears or heels. These lesions are small and erythematous and macular) affecting the palms
nodular, and they present in patients with fa- and soles. Wickhams striae are absent.
milial hypercholesterolemia.
Answer E is incorrect. Though recent viral

Dermatology
Answer D is incorrect. Electrolyte levels are infection can result in rashes of widely varying
not likely to be imbalanced in a patient who morphology, these most commonly occur in
is in a state of insulin resistance, unless the pa- children and are typically more erythematous
tient is signicantly hyperglycemic. This would and macular, as opposed to the violaceous and
not be the most appropriate laboratory test. papular lesions described here.
Answer E is incorrect. Hypothyroidism is a po-
7. The correct answer is B. Pretibial erythema-
tential cause of hyperpigmentation, but it does
tous, tender nodules in a young woman is a
not produce this specic pattern.
classic presentation of erythema nodosum
(EN), which is caused by inammation of sub-
6. The correct answer is C. This is a classic de-
cutaneous fat. Most cases of EN are idiopathic.
scription of lichen planus; remember the 5
The second most common cause of EN is strep
Ps: Purple, Polygonal, Pruritic, and Planar
pharyngitis, and other known causes include
Papules and Wickhams striae (the characteris-
hypersensitivity reaction secondary to drugs
tic light grey or white lines or dots seen on the
(e.g., oral contraceptives and nonsteroidal anti-
surface of lichen planus). Lichen planus is an
inammatory drugs), sarcoidosis, tuberculosis,
uncommon disease of unclear though possibly
and inammatory bowel disease.
autoimmune etiology. It affects middle-aged
adults and may be associated with hepatitis C Answer A is incorrect. This is a very atypical
infection and/or drug exposure (including distribution for bruises or injury secondary to
-blockers, penicillamines, angiotensin-convert- violence. Domestic violence may present with
ing enzyme inhibitors, and sulfonylureas). Li- unusual bruising, lacerations, or fractures, but
chen planus is often self limited, resolving more often is seen in the context of vague so-
within 812 months. Antihistamines and topical matic complaints, anxiety, and depression. Fur-
corticosteroids are recommended for milder thermore, there is a more likely explanation for
cases. Systemic steroids (e.g., intramuscular tri- this patients skin ndings and no reason to sus-
amcinolone every 3 months) or oral psoralen pect domestic violence.
with ultraviolet A light therapy may be effective
Answer C is incorrect. The combination of
for managing severe symptoms; however, the
abdominal pain and lower-extremity rash is
patient should be made aware of the increased
concerning for Henoch-Schnlein purpura
adverse effects.
(HSP), a small-vessel vasculitis. HSP classically
Answer A is incorrect. The rash of erythema occurs on the buttocks and/or lower extremi-
multiforme typically involves the extremities, ties. The tetrad of HSP includes arthralgias, ab-
palms, and soles, and may consist of erythema- dominal pain, purpuric rash, and renal disease.
tous or purpuric plaques and bullae with cen- Although HSP can follow an upper respiratory
tral clearing. Painful lesions are typical, though tract infection, it most commonly affects boys
between the ages of 2 and 11 years.
HIGH-YIELD SYSTEMS
54 Section I: Organ Systems Answers

Answer D is incorrect. Idiopathic thrombo- invade locally. AKs often present as erythema-
cytopenic purpura may cause petechiae, non- tous scaly macules (as in this case), or they may
palpable purpura, or ecchymoses, but not nod- be hyperkeratotic plaques or papules closely re-
ules. Mucosal bleeding resulting in epistaxis sembling squamous cell carcinoma.
and menorrhagia are also common. The le-
Answer B is incorrect. An AK is not a risk fac-
sions in this case do not appear to be bruises,
tor for development of basal cell carcinoma
and there are no petechiae or purpuric lesions
(BCC). Risk factors for BCC include sun ex-
present.
posure, arsenic exposure, immunosuppression,
Answer E is incorrect. The rash of secondary a history of nonmelanoma skin cancer, and
syphilis is characteristically papular and in- inherited disorders such as xeroderma pigmen-
volves palms and soles. tosum, nevoid basal cell carcinoma syndrome,
and Bazex syndrome.
8. The correct answer is D. From the history, it is
Answer C is incorrect. Though sun exposure
possible that this is the patients rst exposure
(particularly a history of severe sunburns in
Dermatology

to herpes simplex virus (HSV). In patients with


childhood) is also a risk factor for development
HIV, a 7-day course of oral acyclovir has been
of malignant melanoma, the lesion shown is
shown to reduce the duration and morbidity
an example of AK, which is not a precursor of
associated with HSV infection, and there may
malignant melanoma. Dysplastic nevi are pre-
also be a role for acyclovir in prophylaxis if ad-
cursor lesions for melanoma.
ministered soon after exposure.
Answer E is incorrect. Though AK is not a ma-
Answer A is incorrect. This is not an available
lignant lesion, a small proportion of cases may
therapy for HSV type 1 infection.
undergo malignant transformation and should
Answer B is incorrect. If the patient develops therefore be evaluated by biopsy.
severe disease, this is appropriate therapy. How-
ever, he is asymptomatic at this time, and this 10. The correct answer is D. Lichen sclerosus
treatment plan may be more harmful (line in- (also called lichen sclerosus et atrophicus) is
fection, nosocomial infection) than benecial. most common in postmenopausal women, and
causes itching of the anogenital region. How-
Answer C is incorrect. Acyclovir therapy is ef-
ever, it can occur at all ages and in both sexes
fective if initiated within 72 hours of infection.
and can be found anywhere on the skin.
This patient is still within the window for ini-
Though the etiology is unknown, chronic in-
tiation of therapy.
ammation is thought to play a role in causing
Answer E is incorrect. This would be an in- the labia to become white, wrinkled, and frag-
appropriate course of action. This patient is ile. The tissue may be so fragile that minor
immunosuppressed and is therefore at risk for trauma may cause petechial bleeding or s-
developing severe disease. sures, as seen in this case. The patient de-
scribed here has early disease. More advanced
9. The correct answer is D. This is a case of ac- disease may cause loss of labial distinction and
tinic keratosis (AK), which can be differenti- fusing of the prepuce, obscuring the urethra
ated from seborrheic keratosis by the presence and clitoris. Denitive diagnosis is made by bi-
of an erythematous base. In addition, sebor- opsy, and treatment is with an ultrapotent topi-
rheic keratoses are typically darker, ranging cal corticosteroid applied daily for several
from brownish-pink to black. The primary risk weeks and then less frequently in the long-
factor for development of AK is sun exposure. term.
The risk of an AK progressing to squamous cell
carcinoma (SCC) is small; however, approxi- Answer A is incorrect. Candidiasis can cause
mately half of cutaneous SCCs arise from AK. white plaques, intense itching, and some pain
with ssure formation. However, the surround-
Answer A is incorrect. AKs may enlarge and ing mucosa is typically intensely erythematous
progress to SCC, but AKs themselves do not and may be edematous.
HIGH-YIELD SYSTEMS
Chapter 2: Dermatology Answers 55

Answer B is incorrect. Estrogen deciency dyspareunia, and irritation. If the disease pro-
may cause atrophy of the vaginal mucosa and gresses it leads to adhesion formation and loss
many of the same symptoms as seen in lichen of architecture. Patients also can suffer from
sclerosus, such as atrophic changes and loss oral lesions, rashes elsewhere on the body, and
of architecture, dyspareunia, and labial adhe- alopecia. Treatment consists of highly potent
sion. However, estrogen deciency should re- topical corticosteroids and supportive thera-
spond to application of estrogen cream within pies, but no agent is universally effective and
2 weeks. treatment can be difcult.
Answer C is incorrect. Lichen planus is char- Answer E is incorrect. Sexual abuse may man-
acterized by inammatory mucocutaneous ifest as whitish scarring and disruption of archi-
eruptions involving oral mucous membranes, tecture, and ssure formation is suggestive of
the vulva, and hair-bearing skin and scalp. trauma. However, in a postmenopausal woman
Typically, vulvar lesions have a reticulated pat- the intense itching and white, wrinkled mu-
tern on the labia and perineum. Vulvar lichen cosa are suggestive of other diagnoses.

Dermatology
planus presents with pruritis, contact bleeding,
This page intentionally left blank
CHAPTER 3

Endocrinology

57
HIGH-YIELD SYSTEMS
58 Section I: Organ Systems Questions

Q U E ST I O N S

1. A 41-year-old woman presents to the emer- 3. A 26-year-old man presents with increased
gency department with palpitations. On ques- thirst, urinary frequency, and nocturia over the
tioning she notes heat intolerance, nervous- past several months. Physical examination is
ness, and insomnia. On physical examination unremarkable. Twenty-four-hour urine osmo-
the physician notes a ne tremor, diffuse non- larity is <300 mOsm/L. A uid deprivation test
pitting edema of the anterior lower leg, and does not result in an increased urine osmolar-
bulging of both of her eyes. What nding on ity. Administration of 0.03 g/kg of desmopres-
blood test would conrm the diagnosis? sin results in a urine osmolarity of 450
mOsm/L after 2 hours. Which of the following
(A) Anti-thyroid-stimulating hormone receptor
is the most likely diagnosis?
antibodies
(B) Decreased thyroid-stimulating hormone (A) Central diabetes insipidus
Endocrinology

levels (B) Diabetes mellitus


(C) Increased creatine kinase-myocardial (C) Nephrogenic diabetes insipidus
bound (D) Psychogenic polydipsia
(D) Increased thyroid-stimulating hormone (E) Syndrome of inappropriate secretion of
levels ADH
(E) Positive antinuclear antibody
4. A 6-year-old boy is brought to his pediatrician
2. A 17-year-old girl presents to the clinic because for a routine check-up. He has not been seen by
she has not yet menstruated and does not have a physician for the past 3 years. Recently, he has
signicant breast development. Family history is developed some patchy areas of hair loss on his
signicant only for some cousins who are color scalp. The mother also notes he has had many
blind. The patient denies ethanol, tobacco, and colds over the past year. She says he has devel-
illicit drug use and sexual activity. Physical ex- oped normally, although he started walking later
amination reveals a normal-appearing girl in no than her other two children. On physical exami-
acute distress with minimal breast development nation his wrists appear enlarged, and he has
and a lack of pubic hair. She is 168 cm (5'6") bowing of the forearms and legs. X-ray of the
tall and weighs 61.2 kg (135 lb). Cardiac exami- boys legs is shown in the image. Laboratory tests
nation reveals no murmurs, rubs, or gallops, show a calcium level of 7.1 mg/dL, phosphate
with point of maximal impulse at the left mid- of 1.8 mg/dL, and intact parathyroid hormone
clavicular line between the third and fourth in- of 130 pg/mL (normal: 1065 pg/mL). Vitamin
tercostal space. Gynecologic examination re- D level is normal. Treatment with vitamin D
veals a vagina without rugae and a cervix that is does not correct the patients hypocalcemia.
easily visualized. There is no discharge. A urine Which of the following disorders best explains
test is negative for -human chorionic gonado- this patients ndings?
tropin. Which of the following is the most likely
diagnosis?
(A) Androgen insensitivity syndrome
(B) Gonadal dysgenesis
(C) Kallmanns syndrome
(D) Kartageners syndrome
(E) Pregnancy
HIGH-YIELD SYSTEMS
Chapter 3: Endocrinology Questions 59

6. A 13-year-old boy is brought to the pediatrician


by his mother because of increasing body hair.
Several months earlier he had been diagnosed
with 17-hydroxylase deciency and treated
with hydrocortisone. Physical examination re-
veals an overweight boy with a moderate
amount of both chest and genital hair, and
some facial hair growth. His physical examina-
tion is otherwise unremarkable. Which of the
following is the best treatment for this patient?
(A) Add cosyntropin
(B) Add dexamethasone
(C) Add spironolactone
(D) Increase hydrocortisone

Endocrinology
(E) Keep the current dose of hydrocortisone

7. A 72-year-old man with atrial brillation pre-


sents with complaints of fatigue and feeling
cold. He also notes constipation and dry skin.
Reproduced, with permission, from Skinner HB. Current His daughter states he has seemed more forget-
Diagnosis & Treatment in Orthopedics, 4th edition. New ful over the past several months. His tempera-
York: McGraw-Hill, 2006: Figure 11-3. ture is 37.3C (99.1F), heart rate is 48/min,
(A) Dietary vitamin D deciency and blood pressure is 130/82 mm Hg. Cardiac
(B) Hypoalbuminemia examination shows bradycardia but normal
(C) Primary hyperparathyroidism rhythm, and normal S1 and S2 with no mur-
(D) Pseudohypoparathyroidism murs; the lungs are clear to auscultation bilat-
(E) Vitamin D-resistant rickets erally and the abdomen is soft and nontender.
The patients extremities are cool and puffy
5. A 53-year-old woman presents to the clinic with dry, coarse skin. Laboratory studies show a
with complaints of headache and blurred vi- thyroid-stimulating hormone level of 32 U/L,
sion for the past several months. She also says free thyroxine of 0.3 ng/dL, and total tri-
her family has commented that her face looks iodothyronine of 30 ng/dL. What medication is
different, and her nose is bigger than it used the patient likely taking for his atrial brilla-
to be. In addition, she says her shoes feel tion?
tighter. On physical examination she has (A) Amiodarone
coarse facial features with a prominent mandi- (B) Flecainide
ble and widely spaced incisors. MRI of the (C) Lithium
brain reveals a mass in the pituitary. This pa- (D) Methimazole
tient may be at increased risk of developing (E) Sotalol
which of the following malignancies?
(A) Colon cancer
(B) Hepatocellular carcinoma
(C) Lung cancer
(D) Malignant brain tumor
(E) Pancreatic adenocarcinoma
HIGH-YIELD SYSTEMS
60 Section I: Organ Systems Questions

8. A 45-year-old woman with chronic alcohol 10. A 28-year-old woman presents to her gynecolo-
abuse admitted 3 days ago for nausea and se- gist for her annual examination. She mentions
vere diarrhea now complains of perioral and that she and her husband have been trying to
nger tingling. She was admitted for hydration conceive for 9 months without success and that
after 1 week of severe watery diarrhea. She has her menstrual cycles have become irregular.
been receiving intravenous hydration and dex- Her gynecologist suggests that she and her hus-
trose but has not been able to take oral nutri- band continue to try to conceive and that the
tion secondary to continued nausea. Her blood woman return in 3 months for some laboratory
pressure is 130/74 mm Hg, pulse is 68/min, studies if she still has not become pregnant. In
and respiratory rate is 16/min. She is afebrile. the interim, a routine visit to the ophthalmolo-
Physical examination is signicant for facial gist reveals bitemporal hemianopsia. Which of
twitching on percussion of her facial nerve just the following is the most likely cause of this
anterior to the ear, as well as the induction of womans infertility?
carpal spasm after the ination of a blood pres-
(A) Ectopic endometrial tissue
sure cuff on her arm. Which of the following is
Endocrinology

(B) Failure of implantation


most likely to have caused these ndings?
(C) Hostile cervical mucus
(A) Azotemia (D) Ovarian unresponsiveness to gonadotro-
(B) Hypernatremia pins
(C) Hypomagnesemia (E) Suppression of ovulation
(D) Hypophosphatemia
(E) Hypouricemia 11. A 4-year-old boy is brought to the pediatrician
by his worried mother. She notes that he uri-
9. A 17-year-old girl has never had a menstrual nates 10 times a day and is always drinking wa-
period. On physical examination, she has mini- ter. She also reports that despite eating more
mal breast development and no axillary or pu- than either of his brothers did at the same age,
bic hair. She is color blind and has had a di- he is not gaining any weight. Which of the fol-
minished sense of smell since birth. Laboratory lowing human leukocyte antigen (HLA) types
evaluation would most likely reveal which of is associated with the most likely diagnosis for
the following? this child?
(A) HLA-B27
GONADOTROPIN- FOLLICLE- (B) HLA-B51
RELEASING STIMULATING LUTEINIZING (C) HLA-D11
CHOICE HORMONE HORMONE HORMONE (D) HLA-DR2
(E) HLA-DR3
A
12. A 48-year-old woman presents to her primary
B care physician because of 2 weeks of neck pain.
The pain is constant and sharp (rated at 10 of
C 10) and is felt in the anterior portion of her
neck. She also notes several weeks of loose
D stools and fatigue. Past medical history is signif-
icant for a viral upper respiratory infection
E
about 1 month ago. She has a temperature of
37.9C (100.2F), heart rate of 96/min, and
(A) A blood pressure of 136/82 mm Hg. On neck ex-
(B) B amination there is diffuse enlargement of the
(C) C thyroid and it is exquisitely tender to even mild
(D) D palpation. Laboratory tests show a total tri-
(E) E iodothyronine level of 280 ng/dL, total thyrox-
ine of 25 g/dL, and thyroid-stimulating hor-
mone of 2 U/mL (normal: 0.44 U/L).
HIGH-YIELD SYSTEMS
Chapter 3: Endocrinology Questions 61

Which of the following is the most likely diag- (E) Intravenous uids, loop diuretic, and po-
nosis? tassium
(A) Acute infectious thyroiditis
15. A 56-year-old woman presents to the outpatient
(B) Drug-induced thyroiditis
clinic for a routine visit. On physical examina-
(C) Hashimotos thyroiditis
tion a 1-cm nodule is palpated in her thyroid.
(D) Riedels thyroiditis
Her physical examination is otherwise unre-
(E) Subacute granulomatous thyroiditis
markable. Her heart rate is 70/min and regular,
blood pressure is 126/82 mm Hg, and tempera-
13. A 74-year-old woman is brought to the emer-
ture is 36.7C (98.0F). Which of the follow-
gency department by her daughter. The daugh-
ing is a poor prognostic indicator for the thy-
ter states that her mother lives alone and has
roid nodule?
no signicant medical problems. She says that
she last saw her mother a month prior, before (A) Female gender
she left on an extended business trip. When (B) Hoarseness

Endocrinology
she returned, she found her mother appeared (C) Palpitations
restless and very nervous. She also appeared to (D) Patient age of 56 years
have lost a noticeable amount of weight. The (E) Slow growth of nodule
patient told her daughter that she had been (F) Tender nodule
having increased frequency of bowel move-
ments, and felt like her heart was beating fast 16. A 26-year-old man with a history of kidney
and funny, and that she felt like she might be stones presents with 1 week of severe burning
coming down with a cold. Initial evaluation in epigastric pain. He also notes several days of di-
the emergency department reveals sinus tachy- arrhea and nausea but denies emesis or fever.
cardia and a painful, enlarged thyroid. Which His family history is remarkable for a paternal
of the following is the most likely etiology of uncle with pancreatic cancer. His temperature
her symptoms? is 37C (98.6F), heart rate is 88/min, respira-
tory rate is 16/min, and blood pressure is
(A) Autoimmune thyroiditis
125/85 mm Hg. Abdominal examination is sig-
(B) Graves disease
nicant for tenderness in the mid-epigastrium.
(C) Medication-induced hyperthyroidism
Upper endoscopy reveals a 1-cm ulceration in
(D) Subacute granulomatous thyroiditis
the rst part of the duodenum. This is the third
(E) Toxic multinodular goiter
episode of conrmed peptic ulcers in this pa-
tient. Laboratory studies show:
14. An 18-year-old woman presents to the emer-
gency department with acute mental status Na+: 140 mEq/L
changes, rapid and deep breathing, abdominal K+: 4.9 mEq/L
pain, and vomiting. On examination she is tac- Cl: 105 mEq/L
hypneic and tachycardic, her abdomen is soft HCO3: 25 mEq/L
and nontender, and her mucous membranes Ca2+: 12.0 mg/dL
are dry. Laboratory values are notable for a po- PO4: 1.4 mg/dL
tassium level of 5.5 mEq/L, bicarbonate of 12 Mg2+: 2.0 mg/dL
mEq/L, and serum glucose of 400 mg/dL. Blood urea nitrogen: 10 mg/dL
Which of the following is the most appropriate Creatinine: 1.0 mg/dL
strategy during the rst 24 hours? Glucose: 87 mg/dL
(A) Diuresis and ventilatory support Which of the following is most likely to be
(B) Diuresis, strict potassium restriction, and found in this patient?
insulin
(A) Medullary thyroid carcinoma
(C) Intravenous uids, insulin, and potassium
(B) Papillary thyroid carcinoma
(D) Intravenous uids, insulin, and strict potas-
(C) Pheochromocytoma
sium restriction
(D) Prolactinoma
(E) Squamous cell lung cancer
HIGH-YIELD SYSTEMS
62 Section I: Organ Systems Questions

17. An obese 18-year-old woman is brought to the and is frustrated at the fact that she often has a
emergency department by her mother, who blood sugar level in the 120s at night, followed
noted that she had been lethargic all day, and by a level in the 170s to 180s the following
suffered a brief, seizure-like episode. One morning. The patients primary care physician
month earlier, the patient had been started on increased her nightly dose of neutral protamine
medication for type 2 DM. Lactic acid levels Hagedorn insulin 1 month ago, but her morn-
are normal. Which of the following medica- ing glucose levels have only become more ele-
tions most likely played a role in the patients vated. She has recently begun to limit her car-
current presentation? bohydrate intake at night, with no effect. This
patients morning hyperglycemia might most
(A) A statin
likely be alleviated by which of the following?
(B) A sulfonylurea
(C) A thiazolidinedione (A) Decreasing neutral protamine Hagedorn
(D) An -glucosidase inhibitor insulin at night
(E) Metformin (B) Increasing neutral protamine Hagedorn
Endocrinology

insulin at night
18. A 75-year-old woman is brought to the emer- (C) Increasing neutral protamine Hagedorn
gency department after being found unrespon- insulin in the morning
sive at her home. She was last spoken to by her (D) Increasing regular insulin at night
daughter on the phone 24 hours earlier, at (E) Increasing regular insulin in the morning
which time she complained of chills, lethargy,
and weakness. The woman has had a heart at- 20. A 26-year-old G1P0 woman at 12 weeks gesta-
tack in the past, she has high blood pressure, tion presents to her obstetrician for her rst
and she had a total thyroidectomy performed a visit. Her pregnancy thus far has been notable
decade ago for cancer. The daughter had re- only for some mild nausea and vomiting that
turned from several months out of town, and is lasted throughout her rst trimester. She re-
unsure if the patient was taking her medica- ports feeling overly tired lately and very weak.
tions. Her temperature is 34.9C (94.9F), Her past medical history is signicant for perni-
pulse is 48/min, blood pressure is 110/65 mm cious anemia. On physical examination she is
Hg, oxygen saturation is 99% on 100% oxygen, an anxious-appearing, thin woman. Her blood
and glucose is 85 mg/dL. On examination the pressure is 130/85 mm Hg, heart rate is 115/
patient is unresponsive, obese, and edematous min, and respiratory rate is 18/min. Fetal heart
with periorbital edema. Her cardiac and pul- tones are present at 135/min. The uterine fun-
monary examinations are normal. CT of the dus is at 12 cm. The woman has a diffuse, non-
head reveals no signs of trauma or increased tender goiter, a resting tremor, and poor global
intracranial pressure, and ECG demonstrates muscle strength. Which is the most likely
no acute ischemic changes. Blood is drawn for mechanism underlying this womans condi-
laboratory testing. Which of the following is tion?
most appropriate for treating the patients men-
(A) Autoantibodies against thyroid-stimulating
tal status change?
hormone receptor
(A) Aspirin (B) Iodine overdose
(B) Glucagon (C) The mechanism of this disease is unknown
(C) Hemodialysis (D) Uncontrolled cell growth
(D) Levothyroxine (E) Viral infection
(E) Metoprolol
(F) Norepinephrine 21. A 60-year-old woman recently diagnosed with
type 2 DM complains of daily headaches and
19. A 52-year-old African-American woman with double vision that have gradually worsened
type 2 diabetes mellitus (DM) presents to her over the previous month. An MRI shows a
physicians ofce and states that she has been large pituitary adenoma. Which of the follow-
feeling lousy in the morning. She notes that ing is most likely being secreted by this tumor?
she reliably checks her blood glucose levels,
HIGH-YIELD SYSTEMS
Chapter 3: Endocrinology Questions 63

(A) ACTH Plasma renin activity is also decreased. Which


(B) Growth hormone of the following is most likely to be increased?
(C) Luteinizing hormone
(A) Aldosterone
(D) Prolactin
(B) Anion gap
(E) Thyroid-stimulating hormone
(C) Carcinoembryonic antigen
(D) Prostate-specic antigen
22. A 14-year-old boy presents at the pediatric
(E) Troponin
clinic for a routine check-up. The patient had
developed end-stage renal disease over the pre-
24. A 28-year-old patient with known Addisons dis-
vious 2 years, and was successfully treated with
ease presents with abdominal pain and is hy-
a renal transplant 6 months prior. Since his op-
potensive to a systolic pressure of 88 mm Hg.
eration, he has developed purple striae on his
He has a 2-week history of progressively worse
back and arms, central obesity, and an increas-
nonproductive dry cough, sore throat, malaise,
ingly round face. During the subsequent blood
and headache. He has not checked his temper-
analysis, which of the following results would

Endocrinology
ature at home but complains of constant chills.
be most likely?
Which of the following is the best initial man-
agement?
CHOICE ACTH URINARY FREE CORTISOL
(A) Azithromycin
A (B) Check serum glucose
(C) Hydrocortisone
B normal (D) Intravenous uids
(E) X-ray of the chest
C
25. A generally healthy 74-year-old woman who re-
D cently moved into the area visits the physicians
ofce for her rst well-visit. She states that her
E
previous doctor had been treating her with pro-
pylthiouracil (PTU) for subclinical hyperthy-
(A) A roidism, but that her prescription ran out sev-
(B) B eral months ago. Laboratory studies reveal that
(C) C her free thyroxine and triiodothyronine levels
(D) D are normal, but her thyroid-stimulating hor-
(E) E mone is depressed. PTU therapy is most im-
portant in this patient to prevent the develop-
23. A 64-year-old man presents to the emergency ment of which disorder?
department after a motor vehicle crash and re- (A) Cardiac dysrhythmias
ceives a CT of the abdomen that shows a nd- (B) Hypothyroidism
ing of a unilateral mass in the left adrenal (C) Pretibial myxedema
gland. He is unharmed from the accident, feels (D) Thyroid cancer
well, and has never smoked. His blood pressure (E) Thyroid storm
is 155/90 mm Hg, deep tendon reexes are
3/4, and muscle strength is 4/5. Laboratory
studies show:
Na+: 150 mEq/L
K+: 3.0 mEq/L
Cl: 105 mEq/L
HCO3: 36 mEq/L
HIGH-YIELD SYSTEMS
64 Section I: Organ Systems Questions

26. A 58-year-old woman presents to her physician (A) Acetaminophen


because of neck discomfort and difculty swal- (B) Ibuprofen
lowing. She rst began to have difculty with (C) Levothyroxine
swallowing solids 2 years ago, but the problem (D) Prednisone
is getting progressively worse. She denies he- (E) Radioactive iodine
moptysis, hematemesis, abdominal pain, or
change in bowel habits. She is a nonsmoker 29. A 24-year-old woman presents to her gynecolo-
and past medical history is signicant only for gists ofce because of irregular menstrual cy-
mild hypertension. Her temperature is 36.9C cles. She is otherwise healthy and takes no
(98.4F), heart rate is 72/min, and blood pres- medication. She began menstruating at the age
sure is 132/78 mm Hg. She has an asymmetri- of 12 years and has never had regular intervals
cally enlarged thyroid gland that is particularly between cycles, which range from 5 weeks to 3
rm on the right, with poorly palpable borders. months. She is not sexually active. On physical
Laboratory evaluation reveals a free thyroxine examination she is overweight with moderate
level of 4.1 ng/dL and thyroid-stimulating hor- acne on her forehead and chin. Her blood
Endocrinology

mone of 5 U/mL. Based on the results of a ra- pressure is 115/85 mm Hg, heart rate is 95/
dioisotope scan and a ne needle aspiration bi- min, and respiratory rate is 18/min. Pelvic ex-
opsy, the physician decides to perform surgery. amination reveals a smooth, nontender, appro-
For which of the following is the patient at in- priately sized uterus and slightly enlarged ova-
creased risk postoperatively? ries bilaterally. Laboratory studies are ordered.
Which of the following are the most likely lab-
(A) Bone metastases
oratory results?
(B) Hypercalcemia
(C) Hypocalcemia
FOLLICLE-
(D) Hypophosphatemia
STIMULATING LUTEINIZING
(E) Pheochromocytoma
CHOICE HORMONE HORMONE ESTROGEN
27. A 49-year-old woman presents to her physi-
A
cians ofce with a long-standing history of
polydipsia, polyuria, central obesity, and hyper-
B
lipidemia. She is currently taking metformin, a
sulfonylurea, and an angiotensin-converting C
enzyme (ACE) inhibitor. ACE inhibitors are
most benecial in preventing or slowing the D
progression of which of the following diabetic
complications?
(A) A
(A) Diabetic ketoacidosis (B) B
(B) Diabetic nephropathy (C) C
(C) Diabetic neuropathy (D) D
(D) Diabetic retinopathy
(E) Peripheral vascular disease 30. A 61-year-old obese man with a history of
chronic alcohol abuse is diagnosed with type 2
28. A 42-year-old woman presents to her physician DM. In addition to diet modication and exer-
with complaints of fever (38.2C [100.8F]) cise, his physician recommends he begin ther-
and mild-to-moderate anterior neck pain. On apy with a hyperglycemic agent. Several days
examination the physician nds her to be after starting therapy, his wife comes home
tachycardic and sweating, and to have an ex- from work and nds him sitting on the couch
quisitely tender thyroid gland. Her blood work staring into space and breathing rapidly. When
shows a depressed thyroid-stimulating hormone
level and increased free thyroxine. Which of
the following is the most appropriate treatment
at this time?
HIGH-YIELD SYSTEMS
Chapter 3: Endocrinology Questions 65

she speaks to him, she nds he is quite con- (A) Bromocriptine


fused, and immediately takes him to the emer- (B) Cortisol
gency department. Arterial blood gas analysis (C) Methyldopa
shows: (D) Metoclopramide
(E) Octreotide
pH: 7.2
HCO3: 19 mEq/L
33. A 42-year-old woman with no signicant past
Partial carbon dioxide pressure: 32 mm Hg
medical history presents for a routine health
Partial oxygen pressure: 80 mm Hg
maintenance visit. On physical examination a
Lactate: 6 mmol/L
solitary nodule is palpated in the thyroid. She
Which of the following drugs is most likely re- denies pain, dysphagia, or hoarseness. She also
sponsible for this patients symptoms? denies fatigue, weight change, heat or cold in-
tolerance, diarrhea, or constipation. There is
(A) Acarbose
no family history of thyroid cancer. Her serum
(B) Glipizide
thyroid-stimulating hormone level is normal.

Endocrinology
(C) Insulin
Which of the following is the most appropriate
(D) Metformin
next step in evaluation?
(E) Rosiglitazone
(A) Fine-needle aspiration
31. A moderately overweight 34-year-old woman (B) MRI
presents to the emergency department with ex- (C) Radionuclide scan
cessive sweating, ushing, tachycardia, and (D) Thyroid lobectomy
nervousness. Presuming that she might be suf- (E) Ultrasonography
fering from thyrotoxicosis, the physician checks
her blood levels of thyroid hormones, and nds 34. A 60-year-old man presents to his primary care
that her free thyroxine and triiodothyronine physician for routine medical care. He has no
levels are elevated, while her thyroid-stimulat- complaints, takes no medications, and has a
ing hormone is decreased. Her radioactive io- family history of DM. Examination is unre-
dine uptake test shows a complete absence of markable. A screening laboratory test reveals a
iodine uptake. Which of the following is the fasting blood glucose level of 152 mg/dL. One
most likely diagnosis? week later the test is repeated and a value of
144 mg/dL is obtained. Which of the following
(A) Factitious thyrotoxicosis
is the most likely cause of these ndings?
(B) Graves disease
(C) Thyroid-stimulating hormone-secreting pi- (A) Autoimmune destruction of pancreatic is-
tuitary tumor let cells
(D) Toxic adenoma (B) Pancreatitis
(E) Toxic multinodular goiter (C) Patients ndings represent normal labora-
tory values
32. A 38-year-old woman presents with several (D) Peripheral insulin resistance
months of decreased libido and a 4.5-kg (10-lb) (E) Surreptitious insulin injection
weight gain. She has not had her menstrual pe-
riod for the past 3 months. Physical examina-
tion is unremarkable except that a small
amount of white discharge is manually ex-
pressed from the nipples bilaterally. The serum
prolactin level is 300 ng/mL. Which of the fol-
lowing is the most appropriate rst-line treat-
ment?
HIGH-YIELD SYSTEMS
66 Section I: Organ Systems Questions

35. A 65-year-old man presents with a 1-day history given 3 L of oxygen via nasal cannula. How-
of hematuria and sharp ank pain (rated 10 of ever, she admits that after discharge she was
10) radiating toward the groin on the right side. having continued difculty breathing and did
Past medical history is signicant for three not follow the taper of the corticosteroids. The
prior episodes of nephrolithiasis over the past 5 patient has smoked one pack of cigarettes per
years, all of which presented with a similar day for the past 51 years. Which of the follow-
clinical picture. He is not taking any medica- ing is the appropriate rst step in the manage-
tion. There is no family history of renal calculi, ment of this patient?
renal disease, or endocrine disorders. His tem-
(A) 3 L of oxygen via nasal cannula
perature is 36.9C (98.5F), heart rate is 125/
(B) CT scan of the chest
min, and blood pressure is 132/86 mm Hg. He
(C) Intravenous uids
is in obvious distress and cannot sit still on the
(D) Restart corticosteroids and follow a strict
bed. Physical examination is signicant for a
taper
soft, nontender abdomen and extreme costo-
(E) X-ray of the chest
vertebral angle tenderness on the right. Labo-
Endocrinology

ratory values show: 37. An obese patient with a long-standing history


Na+: 142 mEq/L of type 2 DM presents to his primary care phy-
K+: 4.8 mEq/L sician. On examination he has decreased sen-
Cl: 104 mEq/L sation in both lower extremities. Upon ques-
HCO3: 24 mEq/L tioning of his compliance with his prescribed
Ca2+: 11.0 mg/dL medications, he reports that he has stopped
PO4: 1.4 mg/dL taking one medication because it gave him
Mg2+: 2.0 mg/dL atulence and abdominal pain. Which of the
Blood urea nitrogen: 12 mg/dL following did this man most likely stop taking?
Creatinine: 1.0 mg/dL
(A) An -glucosidase inhibitor
Glucose: 118 mg/dL
(B) Meglitinide
Intact parathyroid hormone: 300 pg/mL
(C) Metformin
Which of the following is the most likely diag- (D) Sulfonylurea
nosis? (E) Thiazolidinedione
(A) Malignancy
38. A 19-year-old G1P0 woman at 32 weeks gesta-
(B) Milk-alkali syndrome
tion presents for scheduled prenatal appoint-
(C) Primary hyperparathyroidism
ment. The pregnancy has been uncomplicated
(D) Sarcoidosis
to date. However, she mentions that she re-
(E) Secondary hyperparathyroidism
cently noticed a hard lump on her neck. She
denies pain or difculty swallowing, speaking,
36. A 72-year-old woman presents to her physician
or breathing. Physical examination reveals a
complaining of fatigue, malaise, weight loss,
rm, nontender, immobile, solitary nodule on
and salt cravings. The patient has chronic ob-
the left hemithyroid. Ultrasound reveals a solid
structive pulmonary disease and is intermit-
2-cm mass. There is no cervical lymphadenop-
tently treated with corticosteroids but is not us-
athy. Thyroid function tests reveal a thyroid-
ing home oxygen. Her oxygen saturation is
stimulating hormone level of 1.2 U/mL and
97% on room air with a blood pressure of
free thyroxine level of 0.9 ng/dL. Results of
115/65 mm Hg, which is signicantly lower
ne-needle aspiration biopsy are shown in the
than her baseline of 125/78 mm Hg. On aus-
image. Which of the following is the best next
cultation she has good breath sounds bilater-
step in management?
ally without wheeze, although the expiratory
phase is slightly prolonged. Five weeks ago she
received a corticosteroid treatment for an acute
chronic obstructive pulmonary disease exacer-
bation, for which she was hospitalized and
HIGH-YIELD SYSTEMS
Chapter 3: Endocrinology Questions 67

disorder, and has been taking carbamazepine


for the past 5 years. On physical examination
he has dry skin and coarse, brittle hair with
patchy alopecia. Tapping his right cheek causes
contraction of the muscles at the corner of his
mouth, nose, and eye on the right side. Which
of the following could best have prevented the
development of the patients current problem?
(A) Magnesium supplementation
(B) Parathyroidectomy
(C) Thyroid hormone
(D) Vitamin C supplementation
Image courtesy of the Centers for Disease Control and Pre- (E) Vitamin D supplementation
ventions Public Health Image Library; content provider Dr.

Endocrinology
Edwin P. Ewing, Jr. 41. A 16-year-old obese Hispanic girl presents to
(A) Left hemithyroidectomy her physicians ofce complaining of ugly skin
(B) Monitor until after delivery around my neck and having to wear turtle-
(C) Radioablation therapy necks. On examination the physician notes
(D) Start methimazole darkening and thickening of the skin, which
(E) Start propylthiouracil has a velvety appearance. Which of the follow-
ing is the most appropriate course of action?
39. A 6-year-old boy presents to his pediatrician for (A) Obtain a CT scan of the abdomen
a routine physical examination. His mother re- (B) Obtain a punch biopsy of the affected skin
ports no problems over the past year except (C) Obtain a serum glucose test
that he seems to be shorter than the other boys (D) Obtain a shave biopsy of the affected skin
in his class. His mother is 163 cm (5'4") tall (E) Obtain liver function tests
and experienced menarche at age 12 years,
and his father is 178 cm (5'10") tall and went 42. A 24-year-old woman comes into the emer-
through puberty at approximately age 14 years. gency department with recurrent episodes of
On his growth curve, the boys height was at palpitations, headache, and tremor. Her blood
the 10th percentile at birth, at the sixth percen- pressure is 155/95 mm Hg, heart rate is 135/
tile by age 3 years, and at the third percentile min, temperature is 37.9C (100.2F), and re-
now. His weight is currently at the 25th per- spiratory rate is 12/min. A CT of the abdomen
centile. Which of the following is most impor- shows a suprarenal mass. After conrming the
tant in this patients evaluation? diagnosis with a laboratory test, the physician
(A) Chromosomal analysis informs the patient that she will require imme-
(B) Colonoscopy diate therapy and surgical resection of the mass
(C) Growth hormone level within the next few weeks. In order to achieve
(D) Insulin-like growth factor-1 level short-term control of her blood pressure, which
(E) No further evaluation is necessary of the following agents is most appropriate?
(A) Furosemide
40. A 32-year-old man presents to his primary care (B) Hydralazine
physician complaining of diffuse muscle weak- (C) Phenelzine
ness, dry and puffy skin, and patchy areas of (D) Prazosin
hair loss on his scalp. He also notes numbness (E) Propanolol
around his mouth and a tingling sensation in
his hands and feet. He has a history of seizure
HIGH-YIELD SYSTEMS
68 Section I: Organ Systems Questions

43. A 49-year-old man presents to the clinic for a lymphadenopathy. Relevant laboratory nd-
health maintenance visit. He has a family his- ings include a total triiodothyronine level of
tory of type 2 DM. His medical history is sig- 400 ng/dL, free thyroxine of 6.8 ng/dL, and
nicant for gastroesophageal reux disease, for thyroid-stimulating hormone of 0.01 U/mL
which he takes omeprazole and over-the-coun- (normal: 0.44 U/L). Results of a thyroid-
ter antacids. He smokes one pack of cigarettes stimulating hormone-receptor antibody test are
per day and drinks an average of two beers per positive. Which of the following is the most ap-
night. The patients body mass index is 32 kg/ propriate therapy for this patient?
m. Which of the following most greatly
(A) High-dose iodine therapy
reduce(s) the patients risk of future coronary
(B) Methimazole
artery disease, renal failure, and retinopathy?
(C) Propylthiouracil
(A) Alcohol avoidance (D) Radioiodine ablation
(B) Daily multivitamin (E) Surgical resection
(C) Diet rich in fruit and vegetables
Endocrinology

(D) Smoking cessation 46. A 32-year-old woman undergoes a cesarean


(E) Weight loss and exercise section because of failure of labor to progress,
and delivers a healthy baby boy. The proce-
44. A 72-year-old man with end-stage renal disease dure is complicated by signicant intraopera-
secondary to hypertension presents with several tive blood loss and hypotension, but the patient
months of back pain. He denies fever, weight is successfully resuscitated. Postoperatively she
loss, difculty walking, altered sensation in his experiences dull, aching, non-localized ab-
legs, or incontinence. He was diagnosed with dominal pain and nausea, but denies head-
renal disease 20 years ago and was managed ache, visual changes, or abnormal edema. On
medically for many years. However, 2 years ago postoperative day three she is passing atus and
he began hemodialysis because of a progressive remains afebrile, but becomes hypotensive to
decline in renal function. There is no family 90100 mm Hg systolic and 4050 mm Hg di-
history of renal disease or malignancy. Physical astolic. She has not begun lactating despite her
examination is unremarkable. X-ray of the attempts to breast-feed her infant. Laboratory
chest shows ill-dened bands of increased bone values indicate that she is hyponatremic and
density adjacent to the vertebral endplates. mildly hyperkalemic. Urinalysis and liver en-
What laboratory abnormalities is most likely in zymes are normal. Which of the following is
this patient? the most likely cause of her symptoms?
(A) Bence-Jones protein in urine (A) Appendicitis
(B) Decreased parathyroid hormone (B) HELLP syndrome
(C) Decreased phosphate (C) Postoperative infection
(D) Elevated bone-specic alkaline phos- (D) Sheehans syndrome
phatase (E) Toxic shock syndrome
(E) Elevated parathyroid hormone
47. A 52-year-old man presents to the primary care
45. A 32-year-old G2P1 woman at 16 weeks ges- clinic for the rst time. He states that he has
tation presents to her obstetrician complain- been in good health throughout his life and
ing of fatigue, anxiety, and palpitations. She takes no medications. He was once athletic but
says she has been feeling warm, even in her has noted a dramatic decrease in his muscle
air-conditioned home, and has been having strength and exercise tolerance over the past
three or four loose stools per day, as compared year. On examination the patient is moderately
to one or two prior to her pregnancy. She has hypertensive, with a tanned, round, plethoric
a temperature of 37.1C (98.9F), heart rate face; large supraclavicular fat pads; and signi-
of 105/min, and blood pressure of 128/76 mm cant truncal obesity. He has no focal cardiovas-
Hg. Neck examination reveals mild diffuse cular, pulmonary, or neurologic ndings. His
enlargement of the thyroid gland with no fasting blood sugar is 200 mg/dL. Which of the
HIGH-YIELD SYSTEMS
Chapter 3: Endocrinology Questions 69

following is the most common etiology of this examination reveals short stature, coarse facial
condition? features, a protruding tongue, and an easily re-
ducible umbilical hernia. The girl has dif-
(A) ACTH-secreting pituitary adenoma
culty walking and knows six words, although
(B) Adrenal tumor
she is unable to form a sentence. Her mother
(C) Ectopic ACTH-secreting tumor
reports no health problems and an uncompli-
(D) Primary adrenal hyperplasia
cated pregnancy. What is the most likely cause
(E) Small cell lung cancer
of the patients condition?
48. A 48-year-old high school teacher with no prior (A) Congenital hypothyroidism
medical history presents to his primary care (B) Cushings syndrome
physician after feeling extremely fatigued for (C) Neuroblastoma
>1 month. Previously an avid runner, he has (D) Phenylketonuria
recently experienced dyspnea on moderate ex- (E) Turners syndrome
ertion. Although he denies vomiting, he admits

Endocrinology
to intermittent episodes of diarrhea. His blood 50. A 3010-g (6.6-lb) boy was born to a 37-year-old
pressure is 73/37 mm Hg and he is afebrile. primagravida by spontaneous vaginal delivery
On physical examination his skin is warm and after an uncomplicated pregnancy. On exami-
erythematous, and his jugular venous pressure nation he has cyanotic extremities and a signif-
is elevated. Cardiac examination reveals a sys- icant right precordial heave, a single S2, and a
tolic murmur near the right border of the ster- harsh systolic ejection murmur along the ster-
num that is accentuated with inspiration. nal border. He also has a prominent squared
Which of the following is most consistent with nose and cleft palate. An echocardiogram is
these ndings? subsequently performed and demonstrates te-
tralogy of Fallot. Corrective surgery is per-
(A) Elevated urinary excretion of 5-hydroxyin-
formed without complications. At 2 months of
doleacetic acid
age the infant is diagnosed with Pneumocystis
(B) Elevated urinary excretion of vanillylman-
jiroveci pneumonia, and at 3 months he is diag-
delic acid
nosed with fungal septicemia. Additional work-
(C) Peaked T waves on ECG
up of this child should include which of the
(D) Pseudomonas species grown from blood
following tests?
cultures
(E) Severe pulmonary congestion on x-ray of (A) Hemoglobin electrophoresis
the chest (B) Nitroblue tetrazolium
(C) Quantitative immunoglobulin levels
49. A 3-year-old girl is brought to the pediatricians (D) Renal ultrasound
ofce because of an abdominal mass. Physical (E) Serum calcium
HIGH-YIELD SYSTEMS
70 Section I: Organ Systems Answers

AN S W E R S

1. The correct answer is A. Anti-thyroid-stimulat- renal abnormalities, cryptorchidism, and neu-


ing hormone (anti-TSH) receptor antibodies are rologic abnormalities such as mirror move-
pathognomonic for Graves disease, as is sug- ments. Multiple mechanisms of inheritance
gested by this patients symptoms of thyrotoxico- have been observed, including autosomal re-
sis with exophthalmos and pretibial myxedema. cessive, autosomal dominant, and X-linked.
These antibodies mediate the disease by provok- Treatment is with oral contraceptives. To be-
ing a continuous and inappropriate release of come pregnant, patients with Kallmanns syn-
thyroid hormone, which results in the clinical drome require further treatment with a GnRH
picture described. Common modalities of treat- pump. This syndrome would account for this
ment include surgical removal of the gland, ra- patients symptoms of amenorrhea and lack of
dioactive iodine gland ablation, and antithyroid secondary sexual characteristics.
medication such as propylthiouracil.
Endocrinology

Answer A is incorrect. Androgen insensitiv-


Answer B is incorrect. While thyroid-stimu- ity syndrome is characterized by the presence
lating hormone (TSH) levels are decreased in of breasts and lack of axillary hair growth.
Graves disease, they are also decreased in all This occurs when a patient is genetically XY,
other forms of hyperthyroidism, so a decreased but has female external genitalia. Usually un-
TSH level would be suggestive of, but not diag- descended testes are present, which generally
nostic for, Graves disease. must be removed due to the increased risk of
testicular cancer in these patients. This diagno-
Answer C is incorrect. An increased creatine
sis would account neither for the lack of breast
kinase-myocardial bound fraction is sometimes
development nor for the presence of a cervix.
seen in conjunction with hypothyroidism. In
this case, this patient most likely has Graves Answer B is incorrect. Gonadal dysgenesis, or
disease, a form of hyperthyroidism. Turners syndrome, is a possibility in this pa-
tient with amenorrhea. It would account for
Answer D is incorrect. This patient most likely
both the amenorrhea and the lack of second-
has Graves disease, and as a form of hyperthy-
ary sexual characteristics; although growth hor-
roidism, her TSH level will be decreased, not
mone (GH) is present, if there is no response
increased.
from the ovaries, then estrogen and testoster-
Answer E is incorrect. Antinuclear antibody one are not released in sufcient amounts to
is a nonspecic test that generally indicates affect puberty. However, the patient lacks the
autoimmune disease. It is commonly used in dysmorphic features classically seen in patients
conjunction with other diagnostic and clinical with Turners syndrome, including short stat-
tests in the diagnosis of systemic lupus erythe- ure, square-shaped chest, webbed neck, high
matosus, Sjgrens syndrome, and rheumatoid palate, and short fourth metacarpal. Patients
arthritis. While Graves disease in an autoim- with Turners syndrome can also have autoim-
mune disease, it is not commonly associated mune diseases including chronic autoimmune
with antinuclear antibody. thyroiditis; morphologic defects of cardiovascu-
lar, urologic, and bone structure; and hearing
2. The correct answer is C. Kallmanns syn- loss.
drome is a disorder of gonadotropin-releasing
Answer D is incorrect. Kartageners syndrome
hormone (GnRH) synthesis and is associated
is an autosomal recessive defect of cilial move-
with primary amenorrhea without secondary
sexual characteristics due to the lack of pulsa- ment. It has been known to cause bronchiecta-
tile GnRH release, which is the initiating event sis, situs inversus, chronic sinusitis, and infertil-
of puberty. It is associated with anosmia or hy- ity in men due to defective sperm movement.
posmia due to olfactory bulb agenesis or hy- Answer E is incorrect. Pregnancy causes
poplasia. It is also associated with color blind- amenorrhea, but this patient denies sexual ac-
ness, optic atrophy, nerve deafness, cleft palate, tivity and has a negative pregnancy test.
HIGH-YIELD SYSTEMS
Chapter 3: Endocrinology Answers 71

3. The correct answer is A. Central diabetes in- 4. The correct answer is E. This patients presen-
sipidus (DI) is a deciency of production of tation is consistent with rickets. Rickets is a dis-
ADH in the posterior pituitary. ADH acts in order of bone mineralization that can be due
the distal nephron and collecting tubule of the to hypocalcemia or hypophosphatemia. Hy-
kidney to concentrate the urine and reabsorb pocalcemic rickets is typically due to a de-
water. Central DI can be a primary condition ciency of vitamin D from dietary insufciency,
due to a genetic disorder or may be idiopathic; lack of exposure to sunlight, lack of enzymes to
it can also be a secondarily acquired disorder convert vitamin D to active metabolites, or
due to trauma, neoplasm, infection, inamma- end-organ resistance to vitamin D. In this pa-
tory conditions, and toxins. A deciency in tient with normal vitamin D levels, it is one of
ADH leads to decreased water reabsorption in the later etiologies, and treatment with exoge-
the kidney that results in hypernatremia and nous vitamin D will not correct the hypocalce-
increased volumes of dilute urine. mia. Clinical presentation may include tetany,
convulsions, alopecia, and skeletal abnormali-
Answer B is incorrect. DM also causes in-
ties. Skeletal ndings include widened growth

Endocrinology
creased thirst, increased urinary frequency, and
plates, frontal bossing, enlargement of the
nocturia. However, the mechanism is through
wrists, bowing of the distal forearm, lateral
a solute (glucose) diuresis, not an inability to
bowing of the femur and tibia, and delay in
concentrate the urine. The urine osmolarity in
closure of the fontanelles. Children may pre-
DM will be elevated. The fact that the urine
sent with dental enamel hypoplasia, delay in
osmolarity is decreased excludes the diagnosis
motor milestones, and frequent infectious dis-
of DM.
eases. Laboratory ndings will include hypocal-
Answer C is incorrect. Nephrogenic DI has cemia, hypophosphatemia, and secondary hy-
the same clinical presentation as central DI. perparathyroidism.
However, patients with nephrogenic DI se-
Answer A is incorrect. Dietary deciency of
crete a normal amount of ADH from the pi-
vitamin D can also cause the clinical presenta-
tuitary, but they have a mutation in the recep-
tion seen in this patient. However, the vitamin
tor for ADH in the kidney. Desmopressin (or
D level would be decreased, and the disorder
DDAVP) is a synthetic form of ADH. If admin-
would correct with exogenous administration
istration of desmopressin increases urine osmo-
of vitamin D.
larity by 50% or more in 12 hours, the diag-
nosis of central DI is conrmed, as occurred in Answer B is incorrect. Hypoalbuminemia
this patient. If administration of desmopressin is a common cause of hypocalcemia in mal-
fails to increase urine osmolarity, this indicates nourished adults. Normally 40% of calcium
a nephrogenic source of DI, because even is bound to albumin. When a patient has hy-
though ADH levels are increased, the kidney poalbuminemia, he or she has decreased total
cannot respond. serum calcium, even though there is a normal
amount of free calcium. The correction is that
Answer D is incorrect. Psychogenic polydipsia
a 1-g/dL reduction in albumin will decrease
is a condition in which the patient consumes
total serum calcium by 0.8 mg/dL. In an oth-
excessive amounts of water. This also leads
erwise healthy pediatric patient, this is an ex-
to increased urinary frequency and nocturia.
tremely unlikely etiology.
However, in patients with psychogenic poly-
dipsia, the urine osmolarity should normalize Answer C is incorrect. Primary hyperpara-
after a water deprivation test. thyroidism will cause hypophosphatemia and
elevated parathyroid hormone (PTH), but pa-
Answer E is incorrect. Syndrome of inappro-
tients will have hypercalcemia, not hypocal-
priate ADH secretion is a condition in which
cemia. Furthermore, the bony abnormalities
the pituitary secretes excessive amounts of
seen with hyperparathyroidism include osteitis
ADH. This leads to hyponatremia and concen-
brosa cystica and brown tumors, not rickets.
trated urine.
These patients develop subperiosteal bone re-
sorption due to excessive osteoclast activity,
HIGH-YIELD SYSTEMS
72 Section I: Organ Systems Answers

which classically affects the clavicle, phalan- elevated secondary to decreased cortisol levels.
ges, and vertebral bodies. The goal of treatment with hydrocortisone is
not only to replenish cortisol, but also to sup-
Answer D is incorrect. Pseudohypoparathy-
press ACTH secretion. The reason for this is
roidism is a disorder caused by end-organ resis-
that if ACTH remains high, it will stimulate
tance to PTH. Patients present with signs of hy-
androgen production and lead to premature vi-
poparathyroidism, including hypocalcemia, but
rilization and growth plate ossication. Nightly
the PTH level will be elevated, not decreased,
dexamethasone treatments may be necessary to
as would be expected with true hypoparathy-
more completely suppress ACTH secretion in
roidism. Unlike in this patient, patients with
adolescents.
pseudohypoparathyroidism have hyperphos-
phatemia. Answer A is incorrect. Cosyntropin stimulates
ACTH production, and cosyntropin levels are
5. The correct answer is A. This patient has acro- used to diagnose 21-hydroxylase deciency.
megaly, a condition caused by excessive levels of
Answer C is incorrect. Spironolactone is a po-
Endocrinology

GH, most commonly due to a pituitary ade-


tassium-sparing diuretic and would not affect
noma. Blurred vision is due to compression of
ACTH levels.
the optic chiasm by the pituitary mass, and pa-
tients may also exhibit frontal bossing, mandibu- Answer D is incorrect. Increased hydrocor-
lar growth, coarsened facial features, and in- tisone alone may not be sufcient to suppress
creased hand and foot size due to bony ACTH secretion in this patient.
overgrowth and soft tissue swelling. Patients with
Answer E is incorrect. This individual is exhib-
acromegaly may be at increased risk for develop-
iting signs of increased virilization and likely
ing colonic polyps and colonic malignancy. Pa-
needs further suppression of ACTH secretion
tients with acromegaly are also at increased risk
to prevent premature skeletal maturation.
for coronary artery disease, cardiomyopathy, hy-
pertension, DM, and sleep apnea.
7. The correct answer is A. The patient is pre-
Answer B is incorrect. There is no association senting with signs and symptoms consistent
between GH hypersecretion and hepatocellu- with hypothyroidism which include fatigue,
lar carcinoma. weakness, cold intolerance, dry skin, constipa-
tion, bradycardia, coarse hair and skin, and
Answer C is incorrect. Although there are rare
puffy, cool extremities. Hypothyroidism can be
cases of lung tumors producing ectopic GH,
a primary disorder of the thyroid gland, a sec-
this patient has hypersecretion of GH due to
ondary disorder due to decient production of
a pituitary adenoma. These patients are not at
TSH by the pituitary, or a tertiary disorder due
increased risk of developing lung malignan-
to decient production of thyrotropin-releasing
cies.
hormone by the hypothalamus. This patients
Answer D is incorrect. Pituitary adenomas are laboratory results conrm a diagnosis of pri-
usually benign tumors that do not result in in- mary hypothyroidism because the thyroid hor-
creased risk of other malignant brain tumors. mone levels are low in the presence of an ele-
vated TSH. Amiodarone, an antiarrhythmic
Answer E is incorrect. There are rare cases
agent commonly used in atrial brillation, can
of excessive ectopic GH secretion by tumors
cause hypothyroidism by inhibiting production
of the pancreas. However, this patient has evi-
of triiodothyronine, by direct toxicity to thyroid
dence of a pituitary adenoma, which is the
follicular cells, and by effects due to amio-
source of the GH hypersecretion. These pa-
darones iodine content.
tients are not at increased risk of developing
pancreatic cancer. Answer B is incorrect. Flecainide is a class IC
antiarrhythmic agent sometimes used to treat
6. The correct answer is B. In individuals with atrial brillation; however, it is not known to
17-hydroxylase deciency, ACTH secretion is cause hypothyroidism.
HIGH-YIELD SYSTEMS
Chapter 3: Endocrinology Answers 73

Answer C is incorrect. Lithium is used primar- Answer E is incorrect. Urate levels do not af-
ily to treat bipolar disorder. Hypothyroidism is fect calcium homeostasis.
a common adverse effect of lithium; however,
it is not used to treat atrial brillation. 9. The correct answer is A. The patients ndings
are consistent with Kallmanns syndrome, a con-
Answer D is incorrect. Methimazole is an
genital deciency of GnRH synthesis in the hy-
agent used to treat hyperthyroidism. If given
pothalamus. Women present with primary
in excessive amounts, it can cause iatrogenic
amenorrhea and failure to develop secondary
hypothyroidism. However, methimazole is not
sexual characteristics. Laboratory evaluation will
used to treat atrial brillation.
show low or absent levels of GnRH. Because
Answer E is incorrect. Sotalol is a -blocker GnRH is required for release of gonadotropins
used to maintain sinus rhythm in patients with from the anterior pituitary, there will also be di-
atrial brillation, but it is not known to cause minished levels of luteinizing hormone (LH)
thyroid toxicity. and follicle-stimulating hormone (FSH). This
syndrome is associated with anosmia or hypos-

Endocrinology
8. The correct answer is C. This patient is display- mia due to hypoplasia or agenesis of the olfac-
ing classic signs of hypocalcemia, including hy- tory bulb, color blindness, cleft palate, renal dis-
perexcitability of her facial nerve (Chvosteks orders, and nerve deafness.
sign), induced carpal spasm (Trousseaus sign),
Answer B is incorrect. In Kallmanns syn-
and tingling of the extremities and lips. Calcium
drome, both LH and FSH are decreased. A
homeostasis is a complicated process involving
situation in which both GnRH and LH are in-
PTH, vitamin D, albumin, and numerous elec-
creased, but FSH may be decreased, is polycys-
trolytes. Acquired hypoparathyroidism is the
tic ovary syndrome.
most common form of true hypocalcemia, most
often occurring transiently after thyroid surgery Answer C is incorrect. In Kallmanns syn-
or after the removal of a parathyroid adenoma. drome, both LH and FSH are decreased. A sit-
Occasionally, hypomagnesemia can produce uation in which GnRH is decreased, but LH/
hypocalcemia by decreasing both the bodys FSH are increased, is one in which there is a
production of PTH and its sensitivity to the hor- mutation constitutionally activating the GnRH
mone. In this case, it is likely that the patient receptor in the anterior pituitary.
became magnesium depleted from her course
Answer D is incorrect. In Kallmanns syn-
of watery diarrhea, likely baseline poor nutri-
drome, there is a deciency (not an excess) of
tional status, and alcohol abuse.
GnRH. One situation in which there would be
Answer A is incorrect. Azotemia is not a cause elevated GnRH but decreased gonadotropins
of hypocalcemia. Chronic kidney disease, how- would be a mutation in the receptor for GnRH
ever, can lead to hypocalcemia in the setting of in the anterior pituitary.
secondary hyperparathyroidism, but there is no
Answer E is incorrect. Kallmanns syndrome
evidence of renal failure in this patient.
is a deciency of GnRH. A situation in which
Answer B is incorrect. Fluid balance (hyper- one might see amenorrhea with elevated gon-
or hyponatremia) does not play a role in cal- adotropins would be primary ovarian failure.
cium homeostasis.
10. The correct answer is E. This woman has a pi-
Answer D is incorrect. Hypophosphatemia is
tuitary prolactinoma, which is associated with
not a cause of hypocalcemia. Actually, hypocal-
amenorrhea, infertility, and galactorrhea. Pro-
cemia often leads to hyperphosphatemia second-
lactin inhibits the secretion of gonadotropins
ary to increased PTH-mediated bone resorption.
and suppresses ovulation. Dopamine is used to
Elevations in phosphate may also contribute to
suppress prolactin and thus restore fertility.
hypocalcemia by complexing with circulating
calcium and suppressing conversion of 25-OH Answer A is incorrect. Ectopic endometrial tis-
to 1,25-OH vitamin D. sue, as seen with endometriosis, is a common
HIGH-YIELD SYSTEMS
74 Section I: Organ Systems Answers

cause of infertility. Patients experience severe 12. The correct answer is E. Painful thyroiditis
dysmenorrhea and irregular menstrual cycles. limits the differential to subacute granuloma-
Infertility typically results from adhesions caused tous thyroiditis, acute infectious thyroiditis, and
by scarring of the ectopic endometrial tissue. palpation- or trauma-induced thyroiditis. In
However, endometriosis is not likely to cause this case the diagnosis is subacute granuloma-
the visual eld defects seen in this patient. tous thyroiditis, otherwise known as de Quer-
vains thyroiditis, which typically follows an
Answer B is incorrect. This woman has a
acute viral illness, typically an upper respira-
prolactinoma and is therefore most likely not
tory infection. Inammation leads to destruc-
ovulating. As a result, failure of implantation
tion of thyroid follicles, causing release of thy-
cannot be the cause of her infertility because a
roid hormone stores, leading to a transient
zygote cannot be formed in the rst place.
period of hyperthyroidism until the stores are
Answer C is incorrect. At the beginning of exhausted. There may be a transient period of
the menstrual cycle, the cervical mucus is vis- hypothyroidism that follows, but as inamma-
cous and very cellular. Around ovulation, the tion subsides, the follicles will regenerate and
Endocrinology

salt and water concentration of the mucus in- the patient will return to a euthyroid state. Be-
creases, making it thinner and allowing sperm cause of the increased thyroid hormone in the
to penetrate it. If this fails to occur, infertility serum, the TSH level will be low due to nega-
can result due to the hostile cervical mucus. tive feedback on the pituitary.
However, this would not be associated with this
Answer A is incorrect. Acute infectious thy-
patients visual symptoms.
roiditis is a cause of painful thyroiditis. How-
Answer D is incorrect. Prolactin inhibits the ever, it can be differentiated from subacute
secretion of gonadotropins and suppresses granulomatous thyroiditis by the clinical pre-
ovulation. The ovaries are still responsive to sentation. Acute infectious disease is typically
gonadotropins, as demonstrated by treatment unilateral as opposed to diffuse as seen in this
with dopamine, which suppresses prolactin patient. Patients will have acute onset of pain
and restores ovulation. Because the patient is with fever and chills. Many will have a unilat-
still menstruating, although irregularly, her eral neck mass which may be uctuant. In ad-
ovaries are still functioning. dition, thyroid function is typically normal in
patients with acute infectious thyroiditis.
11. The correct answer is E. Given that type 1
Answer B is incorrect. Drugs can cause pain-
DM is most likely an autoimmune disease, it is
less, not painful, thyroiditis. Examples include
not surprising that it is associated with certain
lithium, interferon-alfa, interleukin-2, and ami-
HLA types. HLA-DR3 is associated with type 1
odarone.
DM and is found in about 4% of patients. It is
also associated with systemic lupus erythemato- Answer C is incorrect. Hashimotos thyroiditis
sus and Graves disease. is a chronic autoimmune disorder causing hy-
pothyroidism. Hashimotos very rarely causes a
Answer A is incorrect. HLA-B27 is strongly as-
painful thyroid gland. In addition, serum thy-
sociated with ankylosing spondylitis, as well as
roid hormone would be low and TSH elevated.
Reiters syndrome (arthritis, urethritis, and con-
Patients will have symptoms of hypothyroidism
junctivitis).
including weight gain, cold intolerance, and
Answer B is incorrect. HLA-B51 is associated constipation.
with Behets syndrome.
Answer D is incorrect. Riedels thyroiditis is
Answer C is incorrect. HLA-D11 is associated a nonpainful type of thyroiditis. Patients with
with Hashimotos disease. this disorder have extensive brosis of the thy-
Answer D is incorrect. HLA-DR2 is associated roid, causing the gland to become enlarged,
with Goodpastures syndrome and multiple hard, and xed. Patients may complain of neck
sclerosis. discomfort, dysphagia, and hoarseness due to
HIGH-YIELD SYSTEMS
Chapter 3: Endocrinology Answers 75

compression of adjacent structures, but they roidism would have been a likely cause of her
do not have tenderness to palpation. Typically thyroid dysfunction.
they are euthyroid, although some patients
Answer E is incorrect. Toxic multinodular
may be hypothyroid.
goiter is another cause of hyperthyroidism. It
generally presents with a nonpainful goiter,
13. The correct answer is D. This patient is pre-
and thus is less likely to be the cause of this
senting with the symptoms of subacute granu-
patients symptoms. Toxic multinodular goiter
lomatous thyroiditis. Thyroiditis usually pres-
is caused by diffuse hyperplasia of thyroid fol-
ents initially with symptoms of hyperthyroidism
licular cells that are no longer under the regu-
or thyrotoxicosis, which is then followed by hy-
lation of TSH, most likely due to mutation of
pothyroidism. Some patients also complain of
the TSH receptor in those cells.
malaise or symptoms of an upper respiratory
tract infection. This patient is presenting with
14. The correct answer is C. This patient is in a
several of the symptoms of hyperthyroidism,
state of DKA. Lack of insulin is the primary
notably nervousness, palpitations, weight loss,

Endocrinology
disorder, and insulin administration will allow
and increased frequency of bowel movements.
glucose to enter cells and reverse the meta-
Other symptoms include heat intolerance and
bolic starvation that is driving the production
insomnia. Signs of hyperthyroidism include
of ketoacids. It is also important to immediately
warm, moist skin, the presence of a goiter, car-
administer intravenous uids because the pa-
diac arrhythmia (sinus tachycardia or atrial -
tient is severely dehydrated. Finally, even
brillation), and hyperactive reexes. The key to
though she is hyperkalemic, her potassium lev-
understanding the etiology of this patients hy-
els will decrease rapidly once the insulin is
perthyroidism is that she has a tender, enlarged
given because it will cause the potassium to
thyroid.
enter the cells. Therefore, it is important to
Answer A is incorrect. Autoimmune, or lym- give additional potassium to ensure that the pa-
phocytic, thyroiditis also presents with an initial tient does not become hypokalemic.
period of hyperthyroidism followed by hypothy-
Answer A is incorrect. The patient is dehy-
roidism. However, in autoimmune thyroiditis
drated, and therefore needs additional uid;
the thyroid gland is not painful, thus subacute
diuresis would only make her DKA worse.
granulomatous thyroiditis is a more likely diag-
Ventilatory support is usually not needed be-
nosis.
cause the tachypneic Kussmaul breathing seen
Answer B is incorrect. Graves disease is the in DKA is driven by metabolic acidosis, not
most common cause of primary hyperthyroid- hypoxia. Correction of the metabolic acidosis
ism. It, however, does not generally present through insulin administration will reduce the
with a painful gland, and is not likely to be need for this respiratory compensation.
the cause of this patients thyroid dysfunction.
Answer B is incorrect. As insulin is adminis-
Graves disease is caused by activating TSH
tered, potassium shifts from extracellular to
receptor antibodies, which results in increased
intracellular, exposing what had been a con-
production of thyroid hormone. Other signs of
cealed whole body hypokalemia. Potassium re-
Graves disease include exophthalmos, pretib-
striction would make the ensuing drop in the
ial edema, and thyroid bruits.
measured extracellular potassium even more
Answer C is incorrect. Certain medications, pronounced and dangerous, given the possi-
notably amiodarone, can cause hyperthyroid- bility of fatal arrhythmias in hypokalemia. Di-
ism. Amiodarone can lead to hyperthyroidism. uresis would also be inappropriate in a patient
This patient does not have any signicant med- with DKA, who is likely dehydrated.
ical history, and thus is unlikely to be taking
Answer D is incorrect. Potassium restriction
amiodarone. If she had had a history of atrial
would make the patient likely to become hy-
brillation that was treated with an oral medi-
pokalemic once the insulin is administered be-
cation, then amiodarone-induced hyperthy-
cause he will probably have very low intracellu-
HIGH-YIELD SYSTEMS
76 Section I: Organ Systems Answers

lar potassium. If potassium levels are extremely causes recurrent peptic ulcers due to excessive
high, calcium gluconate can be administered gastrin secretion by a gastrinoma, either in the
to prevent hyperkalemia-induced arrhythmias. pancreas or elsewhere in the gastrointestinal
tract. Hyperparathyroidism causes hypercalce-
Answer E is incorrect. The patient is dehy-
mia, hypophosphatemia, and elevated levels of
drated, so not only will diuresis make the situ-
serum PTH. The most common pituitary tumor
ation worse, but the loop diuretic would also
found in MEN type 1 is prolactinoma, but other
cause additional depletion of whole body po-
tumors include ACTH-secreting and GH-secret-
tassium levels, which are low in DKA.
ing adenomas.
15. The correct answer is B. Hoarseness generally Answer A is incorrect. Medullary thyroid can-
implies vocal cord impairment due to tumor cer is found in association with type IIA and
involvement of the recurrent laryngeal nerve. type IIB MEN, and familial medullary thyroid
This suggests a malignant tumor that has ex- cancer. It is not found in patients with type I
tended beyond the thyroid and invaded local MEN.
Endocrinology

structures. This is a poor prognostic indicator.


Answer B is incorrect. Papillary thyroid can-
Local invasion is particularly common with
cer is the most common type of thyroid cancer.
papillary carcinoma.
However, it is not found in association with the
Answer A is incorrect. Female gender is a inherited MEN syndromes.
good prognostic indicator because thyroid nod-
Answer C is incorrect. Pheochromocytomas
ules in men have a higher risk of malignancy.
are found in patients with type IIA MEN (med-
Answer C is incorrect. Palpitations may be a ullary thyroid cancer, pheochromocytoma, and
symptom of hyperthyroidism. Typically, func- hyperparathyroidism) and type IIB MEN (med-
tioning nodules that produce thyroid hormone ullary thyroid cancer, pheochromocytoma, and
are more likely to be benign. These lesions are mucosal neuromas).
seen as hot on radionuclide scans, whereas
Answer E is incorrect. Squamous cell lung
malignant lesions tend to be nonfunctioning
cancer can cause hypercalcemia and hypo-
and therefore are seen as cold.
phosphatemia by secretion of PTH-related pro-
Answer D is incorrect. Age is an important tein (PTHrP). However, lung cancer is rare in
prognostic indicator. Nodules are more likely such a young patient and there is no associa-
to be malignant in patients >60 years old and tion with recurrent peptic ulcer disease.
<30 years old. So a patient at age 50 years has a
better chance of having a benign nodule. 17. The correct answer is B. Sulfonylureas (glipi-
zide, glyburide) treat type 2 DM by increasing
Answer E is incorrect. Slow growth is a good
the amount of insulin secretion. The obvious
prognostic factor because malignant nodules
potential adverse effect of this is hypoglycemia.
tend to grow rapidly.
Weight gain is another adverse effect.
Answer F is incorrect. Malignant thyroid nod-
Answer A is incorrect. HMG CoA-reductase
ules are classically painless. A tender nodule
inhibitors (statins) will reduce cholesterol
usually suggests a benign disorder such as sub-
levels, and therefore may be helpful as an ad-
acute viral thyroiditis or hemorrhage into a be-
junct treatment to oral hyperglycemic medica-
nign cyst.
tions for patients with type 2 DM. Hypoglyce-
mia is not an adverse effect of statins.
16. The correct answer is D. This patients current
symptoms and past medical and family histo- Answer C is incorrect. The thiazolidinediones
ries are highly suspicious for type I multiple (glitazones) increase peripheral insulin sen-
endocrine neoplasia (MEN type 1), an auto- sitivity. Adverse effects include weight gain,
somal dominant condition consisting of pan- edema, and potential hepatotoxicity.
creatic tumors, hyperparathyroidism, and pitu-
Answer D is incorrect. -Glucosidase inhibi-
itary adenomas. Zollinger-Ellison syndrome
tors (acarbose) work by decreasing intestinal
HIGH-YIELD SYSTEMS
Chapter 3: Endocrinology Answers 77

absorption of carbohydrates. Hypoglycemia is 19. The correct answer is A. This patients morn-
not a common adverse effect, but atulence ing hyperglycemia may be her bodys reaction
and abdominal pain are often complaints. to nocturnal hypoglycemia. Reactive hypergly-
cemia following hypoglycemia is known as the
Answer E is incorrect. Metformin inhibits he-
Somogyi effect. To approach this question, the
patic gluconeogenesis and increases peripheral
actions of regular and neutral protamine Hage-
sensitivity to insulin, and although hypoglyce-
dorn (NPH) insulin should be understood.
mia is a possible adverse effect, it is rare and
Compared to NPH, regular insulin has a
mild. More common adverse effects include
shorter duration of onset, a peak action at 3 to
weight loss, gastrointestinal upset, and meta-
4 hours (vs. 68 hours for NPH), and a shorter
bolic lactic acidosis.
overall duration (68 hours for regular insulin
vs. 1820 hours for NPH). If the dose of NPH
18. The correct answer is D. It is highly likely that
insulin given at night causes the morning glu-
this patient has myxedema coma. She has a
cose to be too low, then the body may release
history of thyroidectomy, making her depen-
stress hormones in response. The release of

Endocrinology
dent on thyroid hormone supplementation,
these stress hormones then causes the morning
and she may have run out of medications.
glucose to be high. The correct response is to
Myxedema coma most often presents as de-
decrease insulin at night.
pressed mental status and hypothermia, and
can also involve bradycardia, hypotension, hy- Answer B is incorrect. Although morning hy-
poglycemia, and hyponatremia. It is often perglycemia may be secondary to insufcient
brought on by a precipitating illness, ischemic insulin at night, the patients history suggests
insult, or administration of sedatives. Manage- that this is not the case. Her morning hyper-
ment includes blood thyroid function tests glycemia did not resolve after attempts at diet
prior to administration of levothyroxine and tri- modication or after increasing her nighttime
iodothyronine. insulin dose. Further increasing her nighttime
insulin would only exacerbate the problem.
Answer A is incorrect. Aspirin would be given
if an acute thrombotic event were suspected, Answer C is incorrect. Increasing morning
such as a stroke or myocardial infarction. insulin would not help decrease this patients
morning glucose.
Answer B is incorrect. Glucagon can be used
in cases of refractory hypoglycemia, but this pa- Answer D is incorrect. Increasing nighttime
tient has an acceptable blood glucose level. insulin would worsen the Somogyi effect.
Answer C is incorrect. The indications for ur- Answer E is incorrect. Increasing morning
gent hemodialysis include demonstrated renal insulin would not help decrease this patients
failure plus metabolic Acidosis, life-threatening morning glucose.
Electrolyte abnormalities, toxic Ingestion, uid
Overload, or symptoms of Uremia (remember 20. The correct answer is A. This woman likely has
AEIOU). Periorbital edema may be seen Graves disease, an autoimmune disease in
in renal failure, but hypothyroidism is a more which antibodies against the TSH receptor acti-
likely cause in this patient. vate the thyroid into overproduction of thyrox-
ine. It is associated with other autoimmune dis-
Answer E is incorrect. Metoprolol would be
orders such as pernicious anemia. Her fetus is at
rst-line therapy if an acute myocardial infarc-
risk of developing thyrotoxicosis because thy-
tion were suspected, but this is less likely given
roid-stimulating autoantibodies can cross the
the ECG results.
placenta and activate the fetal thyroid. Hence,
Answer F is incorrect. Norepinephrine is a pe- fetal heart rate and maternal thyroid-stimulating
ripheral venoconstrictor used in the treatment immunoglobulin levels should be monitored
of hypotension. While norepinephrine may be during the pregnancy.
used to treat this patients hypotension, espe-
Answer B is incorrect. Ingestion of iodine can
cially if it worsens, it will not treat the underly-
result in hyperthyroidism, but given this wom-
ing cause of the myxedema coma.
HIGH-YIELD SYSTEMS
78 Section I: Organ Systems Answers

ans history of pernicious anemia, Graves dis- this child is currently taking corticosteroids as
ease is more likely. a means of immunosuppression. Exogenous
corticosteroids such as prednisolone and dex-
Answer C is incorrect. This woman likely
amethasone have similar effects to those of cor-
has Graves disease, an autoimmune disease
tisol, and the high levels used in posttransplant
in which antibodies against the TSH receptor
immunosuppression cause feedback suppres-
activate the thyroid into overproduction of thy-
sion of ACTH release from the pituitary, as
roxine.
well as cushingoid symptoms. ACTH suppres-
Answer D is incorrect. Thyroid adenomas can sion causes suppression of endogenous cortisol
cause hyperthyroidism but usually do not result production from the adrenal. This explains the
in a uniform goiter. seemingly paradoxical laboratory ndings of
hypothalamic-pituitary-adrenal axis suppres-
Answer E is incorrect. Viral hyperthyroidism
sion in the setting of hyperglucocorticoid
is seen in acute and subacute thyroiditis. How-
symptoms (although note that specic exoge-
ever, the thyroid is usually tender and is ac-
nous glucocorticoids occasionally may cross-
Endocrinology

companied by or follows an upper respiratory


react with some cortisol assays).
tract infection.
Answer B is incorrect. Cortisol levels are un-
21. The correct answer is B. GH has both direct likely to be normal in a Cushingoid patient,
effects, and effects mediated by release of insu- since cortisol would be elevated if the cause
lin-like growth factor-1 (IGF-1) from the liver. were endogenous, and suppressed if the cause
Most of the long-term growth promoting ef- were exogenous steroid administration, as in
fects of GH are mediated by IGF-1. GH itself this case.
acts as an insulin antagonist, which may ac-
Answer C is incorrect. These ndings would
count for the existence of a feedback mecha-
occur in a state of primary adrenal hyperfunc-
nism that inhibits GH release in response to a
tion, such as in adrenal carcinoma. Primary ad-
high-glucose meal in healthy persons. It also
renal hyperfunction is uncommon, and there
explains the high incidence of DM in patients
is a far more likely explanation for this patients
with GH-secreting pituitary tumors, such as
symptoms. This laboratory prole could t with
this patient.
the patient described, but exogenous steroids do
Answer A is incorrect. ACTH tumors cause not reliably cross-react in a urinary cortisol assay,
Cushings disease, which is marked by truncal and the urinary cortisol level is more likely to be
obesity, moon facies, abdominal striae, hyper- calculated as depressed than elevated.
tension, and hyperpigmentation.
Answer D is incorrect. An increased level of
Answer C is incorrect. LH-producing tumors ACTH and decreased level of urinary free cor-
may cause symptoms similar to polycystic ovar- tisol could be seen in a situation of adrenal in-
ian syndrome, such as amenorrhea, hirsutism, sufciency (the increased ACTH is a feedback
and obesity. response to decreased glucocorticoid levels).
Answer D is incorrect. Prolactinomas cause This patients symptoms suggest high, not low
galactorrhea, amenorrhea, and decreased li- levels of glucocorticoids.
bido. Answer E is incorrect. Increased ACTH and
Answer E is incorrect. TSH-secreting tumors increased urinary free cortisol would be seen
produce symptoms of hyperthyroidism includ- in conditions where the body releases an in-
ing tachycardia, tremor, diaphoresis, nervous- creased amount of ACTH, such as in an
ness, weight loss, and increase blood pressure. ACTH-secreting pituitary adenoma (Cushings
disease) or an ectopic ACTH-secreting tumor,
22. The correct answer is A. Given his history of such as small cell carcinoma of the lung or a
recent renal transplantation and the symptoms carcinoid tumor. Exogenous steroid adminis-
of Cushings syndrome, it is most likely that tration is a more likely explanation for this pa-
tients symptoms.
HIGH-YIELD SYSTEMS
Chapter 3: Endocrinology Answers 79

23. The correct answer is A. The patient is pre- mycoplasma pneumonia or walking pneumo-
senting signs and symptoms of primary hyper- nia commonly seen in adolescents and young
aldosteronism: hypertension, hyperreexia, adults. X-ray of the chest would help to con-
weakness, hypernatremia, hypokalemia, alkalo- rm this diagnosis. Although the adrenal crisis
sis, and decreased plasma renin suggest the in this scenario was likely triggered by infec-
unilateral mass seen on CT is an adrenal ade- tion and the patient could not mount a normal
noma hypersecreting mineralocorticoids. glucocorticoid stress response, his more urgent
need is uid resuscitation for hypotension.
Answer B is incorrect. Anion gap is calculated
as: sodium (chloride + bicarbonate). When Answer B is incorrect. Although DM may ini-
this formula is used, the normal anion gap is tiate an adrenal crisis, in this instance the pa-
812 mEq/L. For this patient, using this equa- tient has a clear source of infection which is
tion yields a normal anion gap of 12 mEq/L: likely the cause of the crisis. Therefore, check-
150 (105 + 33). This patient is alkalemic as a ing serum glucose ultimately has no diagnostic
consequence of primary aldosteronism. or therapeutic benet in this scenario.

Endocrinology
Answer C is incorrect. Carcinoembryonic Answer C is incorrect. Since the patient is in
antigen (CEA) could be elevated in smokers, adrenal crisis due to primary adrenal insuf-
as well as patients with liver disorders or ad- ciency, hydrocortisone should be administered
enocarcinomas of the colon, pancreas, breast, expediently. However, although the steroids ad-
lung, or ovary. CEA levels are not used for di- dress the issue of primary adrenal insufciency,
agnostic purposes; instead they are usually used and the patients hypotension will eventually
to monitor the recurrence of cancer. be reversed, their effect is not evident imme-
diately. The hypotension requires immediate
Answer D is incorrect. Increased prostate-
treatment with intravenous uids.
specic antigen (PSA) levels may be seen in a
variety of conditions that affect the prostate, in- Answer E is incorrect. X-ray of the chest
cluding prostatitis and prostate cancer. An in- would be helpful to determine if the patient
creased PSA would not cause hypernatremia, had a pneumonia or respiratory infection, but
hypokalemia, or increased blood pressure. given this patients past medical history it is im-
Look for elevated PSA levels in elderly men portant to address the hypotension and begin
with urinary symptoms. management before imaging.
Answer E is incorrect. Troponin would be ele-
25. The correct answer is A. Patients with subclin-
vated during an acute cardiac event. Troponins
ical hyperthyroidism who are >60 years old
T and I are most specic for cardiac tissue.
have a three- to vefold increased risk of devel-
Look for elevated troponin in patients present-
oping atrial brillation. There is some contro-
ing with acute substernal chest pain and car-
versy regarding the best way to treat subclinical
diac risk factors.
hyperthyroidism, but antithyroid medications
such as PTU are commonly used. In addition
24. The correct answer is D. Intravenous uids
to cardiac dysrhythmias, there is an increased
should rst be administered prior to any other
risk of bone density abnormalities in patients
treatment. Although hydrocortisone is indi-
who are noted to have subclinical hyperthy-
cated to prevent an adrenal crisis in patients
roidism.
with primary adrenal insufciency, Airway,
Breathing, and Circulation (the ABCs) must Answer B is incorrect. Although hypothyroid-
rst be addressed. As the patient is able to ism can subsequently follow hyperthyroidism
speak comfortably, his airway and breathing (subclinical or otherwise), antithyroid medica-
are intact and stable. However, the systolic tions such as PTU are more likely to lead to
blood pressure of 88 mm Hg requires uid re- hypothyroidism than they are to prevent it.
suscitation.
Answer C is incorrect. Pretibial myxedema is
Answer A is incorrect. Azithromycin or a a common symptom of Graves disease, and al-
macrolide antibiotic is indicated in atypical though subclinical hyperthyroidism can prog-
HIGH-YIELD SYSTEMS
80 Section I: Organ Systems Answers

ress to Graves disease, the most common con- roidism leads to elevated levels of serum phos-
cerns are in regard to cardiac dysrhythmias and phate.
bone density abnormalities.
Answer E is incorrect. Pheochromocytoma
Answer D is incorrect. Thyroid cancer is not is associated with medullary thyroid cancer as
commonly associated with hyperthyroidism. part of MEN type II. Despite her lack of family
history or other clues to this diagnosis, she may
Answer E is incorrect. Thyroid storm is an ex-
still have MEN type II. Thyroidectomy, how-
acerbation of hyperthyroidism, and would be
ever, does not increase her risk of developing a
seen much more commonly in patients with
pheochromocytoma.
hyperthyroidism with increased levels of tri-
iodothyronine and free thyroxine than it would
27. The correct answer is B. ACE inhibitors such
in patients with subclinical hyperthyroidism.
as captopril have been shown to decrease blood
pressure and prevent and slow the progression
26. The correct answer is C. This patients presen-
of diabetic nephropathy in patients with DM.
tation, clinical examination, and laboratory
Endocrinology

It is believed that ACE inhibitors play a reno-


ndings are consistent with thyroid carcinoma,
protective role by reducing glomerular ltra-
likely follicular type. She should be treated
tion rate and reducing macroproteinuria.
with a thyroidectomy, followed by radioactive
iodine a few months later to destroy any re- Answer A is incorrect. DKA is a serious, poten-
maining tumor cells. The anatomic proximity of tially fatal complication of type 1 DM, but it is
the parathyroid glands and their blood supply to rarely seen in type 2 DM. Adequate control of
the thyroid gland puts her at risk of iatrogenic hy- blood glucose levels, as well as avoiding physi-
poparathyroidism after her thyroidectomy, lead- ologic stressors such as infections, trauma, and
ing to hypocalcemia and hyperphosphatemia. alcohol, are important in preventing the onset
She should be monitored for this outcome and of DKA.
treated appropriately.
Answer C is incorrect. Diabetic neuropathy
Answer A is incorrect. Lung and bone metas- is a common complication of DM and is best
tases are present at diagnosis in approximately prevented by adequate control of blood glu-
10%15% of patients with follicular thyroid cose levels.
carcinoma; however, they are not a postopera-
Answer D is incorrect. Diabetic retinopathy is
tive risk.
a complication of long-standing DM, and al-
Answer B is incorrect. After thyroidectomy though blood pressure control with drugs such
(for thyroid carcinoma or another thyroid as ACE inhibitors can slow the onset of retinop-
disorder), patients can develop hypoparathy- athy, ACE inhibitors are specically renal pro-
roidism due to excision of or trauma to the tective, even in normotensive patients.
parathyroid glands. Consequently, they can
Answer E is incorrect. Peripheral vascular dis-
develop hypocalcemia, not hypercalcemia.
ease is a common complication of DM, and is
PTH normally acts to increase serum calcium
the underlying cause of diabetic neuropathy and
through increased bone resorption, increased
other cardiovascular complications. Although
gut absorption via accelerated production of
adequate blood pressure control is important,
active metabolites of vitamin D, and increased
ACE inhibitors are most effective in preventing
resorption of calcium in the kidney. Therefore,
diabetic nephropathy.
hypoparathyroidism will lead to decreased se-
rum calcium. 28. The correct answer is B. This patient presents
Answer D is incorrect. Postoperative hypopara- with the classic symptoms of subacute (de Quer-
thyroidism will cause hyperphosphatemia, not vains) thyroiditis, which is typically tender to
hypophosphatemia. PTH normally acts on the palpation. Also, increased TSH and thyroxine
proximal tubule of the kidney to increase ex- indicate that the patient also has primary hyper-
cretion of phosphate. Therefore, hypoparathy-
HIGH-YIELD SYSTEMS
Chapter 3: Endocrinology Answers 81

thyroidism. It is most commonly due to viral Estrogen-secreting tumors can be found in the
infection of the thyroid gland and is self-lim- ovaries, testicles, and pituitary.
ited in 90% of cases. Other causes of hyperthy-
Answer C is incorrect. This hormone prole
roidism, such as Graves disease, do not lead to
represents the ndings in ovarian failure. Al-
anterior neck pain. Nonsteroidal anti-inam-
though FSH and LH levels are both elevated,
matory drugs are the most appropriate treat-
the estrogen level remains low due to the resis-
ment at this time.
tance of the ovaries to secreted gonadotropins.
Answer A is incorrect. Although acetamino-
Answer D is incorrect. This hormone prole
phen would likely reduce the patients temper-
would be the result of increased gonadotropin
ature, subacute thyroiditis is an inammatory
secretion, leading to a high-estrogen state. This
process, and acetaminophen has analgesic and
may be seen in some liver cancers and in hy-
antipyretic properties, but not anti-inamma-
perprolactinemic hypothyroidism.
tory ones.
Answer C is incorrect. Although the patient is 30. The correct answer is D. This patient was most

Endocrinology
currently hyperthyroid (based on both her symp- likely started on metformin, which is the rst-
toms and her decreased TSH level), patients line medication used to treat type 2 DM in the
with subacute thyroiditis can uctuate between absence of contraindications. Metformin is a
hyper- and hypothyroidism. If her symptoms good choice for obese patients because unlike
and TSH levels indicated that she was currently other hyperglycemic agents, which can lead to
hypothyroid, then administration of levothyrox- weight gain, metformin is not associated with
ine would be appropriate. weight gain. It works by inhibiting hepatic glu-
coneogenesis and sensitizes peripheral tissues to
Answer D is incorrect. The patients fever and
insulin. One of the adverse effects of metformin,
pain are relatively mild. If either were more se-
however, is lactic acidosis, which is what this pa-
vere, prednisone would be an appropriate ad-
tient has. He is at increased risk for this rare
dition, but her current presentation does not
event because of his history of alcohol abuse.
warrant the prednisone, and it would never be
Other patients in whom metformin should be
appropriate as a sole treatment.
avoided or used with care because of the risk of
Answer E is incorrect. Radioactive iodine is lactic acidosis include elderly patients and those
the appropriate rst-line treatment for condi- with renal insufciency, liver disease, or heart
tions such as a solitary toxic adenoma, but not disease.
in subacute thyroiditis, which is self-limited in
Answer A is incorrect. Acarbose is an
90% of cases.
-glucosidase inhibitor that decreases the in-
testinal absorption of carbohydrates. It is rarely
29. The correct answer is B. This woman displays
used, however, because it commonly causes
several characteristics of polycystic ovarian syn-
atulence and diarrhea.
drome (PCOS), including high estrogen and
androgen levels, resulting in virilization (hair Answer B is incorrect. Glipizide is a sulfony-
growth and acne) and menstrual irregularities. lurea. Sulfonylureas increase insulin secretion
Increased levels of androgens lead to high estro- from pancreatic cells. However, they can
gen levels, which suppress FSH and lead to in- cause weight gain and would not be an ideal
creased LH levels. PCOS is commonly associ- medication for an obese patient.
ated with peripheral insulin resistance, which
Answer C is incorrect. Insulin is not a rst-line
may improve with the use of metformin.
agent for the treatment of type 2 DM. Oral hy-
Answer A is incorrect. This hormone pro- perglycemic agents should be tried rst. Insu-
le would be seen in the case of an estrogen- lin is rst-line treatment for patients with type
secreting tumor, with synthesis of gonadotro- 1 DM, and can be used in patients with type
pin suppressed due to the high estrogen levels. 2 DM whose serum glucose levels are difcult
to manage.
HIGH-YIELD SYSTEMS
82 Section I: Organ Systems Answers

Answer E is incorrect. Rosiglitazone is a thi- 200 ng/mL, other causes of hyperprolactinemia


azolidinedione and acts by increasing tissue such as drugs, hypothyroidism, and renal fail-
insulin sensitivity. It can cause weight gain ure should be considered. Bromocriptine is a
and thus would not be an ideal choice for this dopamine agonist that can decrease both pro-
obese patient. It can also cause hepatotoxicity, lactin secretion and the size of the adenoma.
so given this patients history of alcohol abuse
Answer B is incorrect. Cortisol is used to treat
and likely underlying liver damage, it should
patients with a deciency of ACTH. Although
probably be avoided.
a prolactinoma can suppress the secretion of
other hormones in the anterior pituitary, in-
31. The correct answer is A. Factitious thyrotoxi-
cluding ACTH, the patient is not currently
cosis involves the administration of exogenous
presenting with evidence of hypocortisolemia.
thyroid hormone, commonly in an attempt to
Therefore, cortisol would not be considered as
lose weight. The distinguishing factor between
rst-line therapy for this patient.
factitious thyrotoxicosis and the other choices
is the result of the radioactive iodine uptake Answer C is incorrect. Methyldopa is an
Endocrinology

test. By exhibiting no uptake, it shows that the -adrenergic antagonist used in the treatment
administration of exogenous thyroxine and tri- of hypertension. Because it is known to elevate
iodothyronine has downregulated uptake by serum prolactin levels, methyldopa would
the thyroid gland. The other conditions involve therefore worsen this patients signs and symp-
primary or secondary overactivity of the thyroid toms.
gland, thereby resulting in increased levels of
Answer D is incorrect. Metoclopramide is an
triiodothyronine and thyroxine.
antiemetic and gastrointestinal prokinetic drug
Answer B is incorrect. The TSH receptor an- that is known to increase prolactin levels and
tibodies in Graves disease continually activate would therefore worsen this patients signs and
the thyroid gland, resulting in hyperthyroidism symptoms.
with increased activity on the radioactive io-
Answer E is incorrect. Octreotide is a soma-
dine uptake test.
tostatin analog used to decrease GH levels in
Answer C is incorrect. A TSH-secreting pitu- patients with gonadotropin hormone-secreting
itary tumor will produce unregulated amounts pituitary adenomas.
of TSH, thereby stimulating the thyroid gland
and resulting in hyperthyroidism with in- 33. The correct answer is A. Palpable thyroid nod-
creased activity in the radioactive iodine up- ules are more common in women, older pa-
take test. tients, and people in iodine-decient parts of
the world. The initial step in evaluation should
Answer D is incorrect. A toxic adenoma will
be measuring the TSH level, which is usually
most commonly show a hot nodule on the
normal. However, in a functional nodule
radioactive uptake test with increased uptake
that is producing excess hormone, it can be de-
in the area of the nodule, and commonly an
creased. If the TSH is decreased, the next step
absence of uptake in the unaffected lobe.
is a radionuclide scan to determine if the nod-
Answer E is incorrect. A toxic multinodular ule is hot (i.e., absorbs the radioactive iodide
goiter will show increased uptake on a radioac- readily). If it is hot, the risk of malignancy is
tive iodine uptake test and is thereby differenti- very low. If the TSH is normal, as in this case,
ated from factitious thyrotoxicosis. the next step is ne-needle aspiration of the
nodule. Those nodules found to be malignant
32. The correct answer is A. This patient is pre- should be surgically removed. Radionuclide
senting with signs and symptoms consistent scans can be helpful to identify cold nodules,
with hyperprolactinemia. The most likely which should be removed surgically due to in-
cause is a prolactin-secreting pituitary ade- creased risk of malignancy or simply to help lo-
noma because the serum prolactin level is calize the nodule prior to surgery.
>200 ng/mL. When the prolactin level is 20
HIGH-YIELD SYSTEMS
Chapter 3: Endocrinology Answers 83

Answer B is incorrect. MRI is not typically Answer E is incorrect. Insulin injections


used in the work-up of thyroid nodules. would be expected to lower the patients blood
glucose levels, not to raise them.
Answer C is incorrect. Radionuclide scan
is the next step in a patient with a decreased
35. The correct answer is C. This patients condi-
TSH level, as there is an increased likelihood
tion is consistent with primary hyperparathy-
that the patient has a functioning nodule that
roidism, most commonly due to a parathyroid
is unlikely to be malignant. However, in a
adenoma. Patients may present with recurrent
patient with a normal TSH, ne-needle aspi-
renal calculi, mental status changes, or abdom-
ration should be performed rst and a radio-
inal pain, but many asymptomatic patients are
nuclide scan performed only if the biopsy is
diagnosed incidentally by ndings of elevated
indeterminate.
serum calcium. Laboratory ndings include el-
Answer D is incorrect. Surgical excision evated intact PTH, hypercalcemia, and hypo-
should be reserved for patients with docu- phosphatemia. The elevated level of PTH
mented malignancy on biopsy or evidence of a causes hypercalcemia via increased bone re-

Endocrinology
cold nodule on radionuclide scanning. sorption, increased distal tubular reabsorption
of calcium in the kidney, and stimulation of re-
Answer E is incorrect. Ultrasound can provide
nal hydroxylation of 25-hydroxyvitamin D,
anatomic detail about the nodule but does not
which increases dietary calcium absorption in
give information on the functional status of the
the gastrointestinal tract. Elevated PTH causes
nodule. Ultrasound is frequently used to assist
hypophosphatemia by inhibiting proximal tu-
with ne-needle aspiration, but alone is not
bular reabsorption of phosphate in the kidney.
considered the next step in the work-up of a
thyroid nodule. Answer A is incorrect. Malignancy can cause
hypercalcemia and hypercalciuria by many
34. The correct answer is D. The nding on two mechanisms, including local resorption of
separate occasions of a fasting blood glucose bone induced by metastases, production of
level of 126 mg/dL or higher indicates that the humoral osteoclast activators such as PTHrP,
patient has type 2 DM. The etiology of type 2 or increases in calcitriol production in pa-
DM includes relative paucity of insulin secre- tients with some lymphomas. In patients with
tion, often in the presence of increased body humorally mediated hypercalcemia, PTHrP
weight, peripheral insulin resistance, and im- mimics the action of PTH, and patients will
paired regulation of gluconeogenesis in the have hypercalcemia and hypophosphatemia,
liver. but PTH levels will not be elevated.
Answer A is incorrect. Autoimmune destruc- Answer B is incorrect. Milk-alkali syndrome
tion of pancreatic islet cells is the pathophysi- is a type of hypercalcemia due to excess intake
ologic mechanism of type 1 DM, which is not of calcium and absorbable antacids. However,
a likely diagnosis in a previously healthy man PTH will not be elevated.
of this age.
Answer D is incorrect. Sarcoidosis can
Answer B is incorrect. Pancreatitis is incor- cause hypercalcemia as a result of excess
rect. Although hyperglycemia of 200 mg/dL 1,25-dihydroxyvitamin D synthesis by mac-
or higher in the setting of pancreatitis can be rophages in granulomas. However, PTH will
a poor prognostic sign on admission, this pa- not be elevated.
tient is not presenting with abdominal pain, as
Answer E is incorrect. Secondary hyperpara-
would be expected in either acute or chronic
thyroidism is caused by chronic renal disease.
pancreatitis.
Diminished ability to excrete phosphate leads
Answer C is incorrect. This answer is incorrect to hyperphosphatemia. Excess phosphate binds
because the nding on two separate occasions calcium, causing hypocalcemia. The parathy-
of a fasting blood glucose level of 126 mg/dL
or higher is diagnostic of DM.
HIGH-YIELD SYSTEMS
84 Section I: Organ Systems Answers

roid reacts by undergoing hyperplasia to in- Answer B is incorrect. Meglitinides are short-
crease release of PTH. Therefore, laboratory acting insulin secretagogues. They can cause
values will show elevated PTH, low calcium, hypoglycemia, and are generally better toler-
and high phosphate. In addition, there would ated, from a gastrointestinal standpoint, than
be an elevated creatinine in a patient with end- -glucosidase inhibitors.
stage renal disease.
Answer C is incorrect. Metformin is an oral
hyperglycemic that works by inhibiting hepatic
36. The correct answer is D. Patients who take
gluconeogenesis and increasing peripheral sen-
corticosteroids may develop secondary hypo-
sitivity to insulin. Though it can cause some
adrenalism and may become unable to mount
gastrointestinal upset, this is usually transient
an appropriate response to ACTH. This can re-
at the beginning of treatment and is not asso-
sult in renal failure, hypotension, and hypona-
ciated with atulence. A serious adverse effect
tremia. Thus, restarting steroids is the rst pri-
of metformin is lactic acidosis. Metformin does
ority in this patient. Gradually tapering off of
not cause hypoglycemia.
steroids allows the suppressed adrenals time to
Endocrinology

return to full function. Answer D is incorrect. Sulfonylureas are insu-


lin secretagogues: they increase the amount of
Answer A is incorrect. The patient is currently
insulin that is secreted. This class of drugs can
well saturated on room air and there is no in-
commonly cause hypoglycemia.
dication for oxygen supplementation. Before
starting a patient with chronic obstructive pul- Answer E is incorrect. Thiazolidinediones
monary disease on oxygen, one should obtain (glitazones) increase peripheral insulin sen-
an arterial blood gas so not to suppress their sitivity. They cause edema and weight gain and
ventilatory drive. typically do not cause gastrointestinal distress.
Answer B is incorrect. Radiologic imaging
38. The correct answer is B. The image shows
is appropriate to rule out lung pathology as a
well-differentiated papillary carcinoma. Papil-
contributor to this patients symptoms, but she
lary carcinoma is the most common type of
should have an x-ray of the chest before pro-
thyroid cancer, and has an excellent prognosis.
ceeding to a CT of the chest.
A higher incidence is seen in women <20 or
Answer C is incorrect. Although the rst steps >70 years old, as well as individuals with a his-
in management are always the ABCs (Air- tory of head and neck radiation. Treatment is
way, Breathing, and Circulation), and uids surgical excision. However, in pregnant women
are an important component of maintaining surgery is most safely performed during the
circulation, the patient is not currently hy- second trimester, and this patient is already
potensive enough to require resuscitation. In- into her third trimester. The mass is small and
travenous uids are important if the patient is not compressing surrounding structures to
develops hypotension as a component of the cause dysphagia, dysphonia, or airway compro-
adrenal insufciency, but is not the current mise. The absence of cervical lymphadenopa-
rst step in management. thy suggests that the cancer has not metasta-
sized. Papillary carcinoma is generally slow
Answer E is incorrect. X-ray of the chest to
growing, and in the absence of metastasis may
rule out lung pathology as the cause of possible
be followed for the remaining weeks of the
constitutional symptoms may be appropriate in
pregnancy before treatment is initiated.
this patient, but is not necessary immediately.
Answer A is incorrect. Surgical resection of a
37. The correct answer is A. -Glucosidase inhibi- thyroid neoplasm in a pregnant woman is in-
tors are medications that reduce the amount of dicated only during the second trimester. This
carbohydrates absorbed from the intestine. patient is already in her third trimester of preg-
They are not commonly used because of the nancy. Although this neoplasm will require re-
bothersome adverse effect of gastrointestinal section, papillary carcinoma is generally slow
upset and atulence.
HIGH-YIELD SYSTEMS
Chapter 3: Endocrinology Answers 85

growing, and thus may be followed for the re- produced by the liver under GH stimulation
maining weeks of the pregnancy before treat- and is a more reliable measurement of GH lev-
ment is initiated. els.
Answer C is incorrect. Radioablation therapy Answer A is incorrect. Chromosomal analysis
is the primary treatment for hyperthyroidism. is an important tool in evaluating females with
This patient does not have hyperthyroidism, as short stature, because Turners syndrome is a
her thyroid hormone levels are within the nor- common cause of short stature in girls. These
mal range. In patients with thyroid neoplasms, patients will typically be short from birth but
radioablation is often performed after surgical will continue to grow along their growth curve,
resection. Radioactive iodine is contraindicated as opposed to having attenuated growth, as
during pregnancy, as it crosses the placenta seen in this patient.
and results in destruction of the fetal thyroid.
Answer B is incorrect. Colonoscopy can be
Answer D is incorrect. Methimazole is a treat- used to evaluate for inammatory bowel dis-
ment for hyperthyroidism but its use is not ease, a frequent cause of attenuated growth.

Endocrinology
recommended during pregnancy. This drug However, this patient has no complaints of di-
crosses the placenta and has been remotely as- arrhea, bloody stool, or abdominal pain. In ad-
sociated with aplasia cutis and esophageal and dition his weight is normal. Patients with short
choanal atresia. This patient does not have hy- stature due to malabsorption will typically be
perthyroidism, as her thyroid hormone levels underweight for their height. Patients with en-
are within the normal range. Methimazole is docrine disease, such as GH deciency, will
not indicated as a treatment for papillary car- typically be overweight for their height.
cinoma.
Answer C is incorrect. Because GH is secreted
Answer E is incorrect. Propylthiouracil is the in a pulsatile manner, a random GH level is
preferred drug to treat hyperthyroidism during not a reliable indicator as to whether a patient
pregnancy, as it crosses the placenta less read- has GH deciency.
ily compared to methimazole. This patient
Answer E is incorrect. Because this patient
does not have hyperthyroidism, as her thyroid
has continued to fall off his growth curve, fur-
hormone levels are within the normal range.
ther evaluation should be performed. It is not
However, neither propylthiouracil nor methim-
safe to assume that his height is due to familial
azole is a correct treatment for papillary carci-
short stature, in which both parents are typi-
noma, which must be surgically resected.
cally short and the patient starts off low on the
growth chart, but maintains a normal curve
39. The correct answer is D. The most concerning
and growth rate. Nor is it safe to assume that
type of short stature is attenuated growth, in
his height is due to constitutional delay, in
which the patient starts off at a normal height
which the child will eventually attain a normal
but falls off of his growth curve as he gets older,
adult height but develops later than other chil-
as this pattern is always pathologic and requires
dren. Parents will typically be of average height
further evaluation. Causes of attenuated growth
but report a delayed age of puberty.
include renal disease, hypothyroidism, Crohns
disease, cancer, glucocorticoid therapy, and GH
40. The correct answer is E. This man has signs
deciency. GH deciency is typically idiopathic,
and symptoms that indicate hypocalcemia. His
but can also be caused by tumors, particularly
complaints of weakness, dry skin, alopecia, cir-
craniopharyngiomas, or rarely a genetic muta-
cumoral numbness, and paresthesias are all
tion in the GH-releasing hormone receptor in
consistent with hypocalcemia. These patients
the pituitary. To diagnose GH deciency, serum
can also develop cataracts, myocardial dysfunc-
IGF-1 levels should be measured as opposed to
tion, osteomalacia, and seizures. This patient
GH levels, because GH is secreted in a pulsatile
has a positive Chvosteks sign, which is ipsilat-
fashion and a random measurement is not a re-
eral contraction of the facial muscles following
liable indicator of GH status, whereas IGF-1 is
HIGH-YIELD SYSTEMS
86 Section I: Organ Systems Answers

tapping the facial nerve. The most likely cause Answer B is incorrect. A punch biopsy would
of this patients hypocalcemia is vitamin D de- be appropriate for diagnosis of a primary skin
ciency due to therapy with carbamazepine. disorder. Acanthosis nigricans is usually a der-
Carbamazepine and other medications includ- matologic manifestation of a systemic disease.
ing phenytoin, rifampin, and theophylline in- It is the underlying disease, not the skin lesion,
crease the activity of cytochrome P450 en- that merits further investigation.
zymes in the liver that inactivate vitamin D.
Answer D is incorrect. A shave biopsy would
Decient vitamin D leads to decreased absorp-
be appropriate for diagnosis of a primary skin
tion of calcium from the gut.
disorder. Acanthosis nigricans is usually a der-
Answer A is incorrect. Hypomagnesemia can matologic manifestation of a systemic disease.
be a cause of hypocalcemia, but this patients It is the underlying disease, not the skin lesion,
hypocalcemia is more likely due to vitamin that merits further investigation.
D deciency, as he is on a medication, carba-
Answer E is incorrect. Liver function tests may
mazepine, that is known to suppress vitamin D
be elevated but are not diagnostic of a particu-
Endocrinology

levels.
lar condition.
Answer B is incorrect. Parathyroidectomy may
be used to treat patients with hyperparathyroid- 42. The correct answer is D. The symptoms of
ism due to a parathyroid adenoma. Resection headache, palpitations, and tremor are all con-
of the parathyroids would cause hypocalcemia, sistent with a pheochromocytoma, as is the CT
which would worsen this patients clinical pre- nding of a suprarenal mass. The conrming
sentation. laboratory test was likely an elevated urinary
catecholamine level; also, serum calcium and
Answer C is incorrect. Hypothyroidism can
glucose may well have been high. A pheochro-
cause dry skin and alopecia. However, patients
mocytoma secretes excessive amounts of epi-
with hypothyroidism do not typically have signs
nephrine and norepinephrine, resulting in
of tetany as seen in this patient. Therefore thy-
both peripheral vasoconstriction (-mediated
roid hormone administration would not have
effect) and increased cardiac contractility
prevented this condition.
(-mediated effect). An -adrenergic blocker
Answer D is incorrect. This patient has symp- such as prazosin is the principal means of re-
toms of vitamin D deciency, not vitamin C lieving hypertension in these patients.
deciency. Vitamin C deciency leads to im-
Answer A is incorrect. Furosemide, a loop
paired collagen synthesis. This causes scurvy,
diuretic, will decrease blood pressure by de-
which is characterized by mucosal bleeding,
creasing electrolyte and uid resorption in
bruising, petechiae, arthralgias, and vasomotor
the thick ascending tubule and resulting in
instability.
diuresis. While diuretics are commonly used
to treat primary hypertension, they are a poor
41. The correct answer is C. Acanthosis nigricans
choice in pheochromocytoma since these pa-
is a velvety dark thickening of the skin around
tients are generally already volume-contracted.
the neck, axillae, and groin areas. It is related
In fact, patients should be encouraged to take
to DM, malignancy, obesity, drugs, and various
in ample salt and water to restore volume once
endocrine disorders, and is not uncommonly
-blockade is established.
found in young Hispanic women. DM should
be suspected in this case, as well as in other Answer B is incorrect. Hydralazine, a direct
conditions associated with insulin resistance vasodilator, does not address the underlying
such as polycystic ovarian syndrome. cause of hypertension in pheochromocytoma
and is not commonly used in these patients.
Answer A is incorrect. Acanthosis nigricans is
associated with visceral malignancies, particu- Answer C is incorrect. Although monoamine
larly gastric cancer, in adults. However, this is oxidase inhibitors such as phenelzine can reduce
very unlikely in a 16-year-old patient, so a CT blood pressure in normal persons, they are not
is probably not warranted.
HIGH-YIELD SYSTEMS
Chapter 3: Endocrinology Answers 87

typically used as antihypertensives. In particular, to normalize serum calcium. Over time this can
they are contraindicated in pheochromocytoma, lead to the pathologic condition osteitis brosa
since inhibition of monamine oxidase can lead cystica. Patients develop subperiosteal bone re-
to increased levels of adrenergic monoamines sorption, which classically affects the clavicle,
and lead to a hypertensive crisis. phalanges, and vertebral bodies. X-ray of the
chest may show a classic rugby jersey (striped
Answer E is incorrect. -Adrenergic blockade
like a rugby jersey) spine due to ill-dened
is useful for controlling tachycardia and pal-
bands of increased bone density adjacent to the
pitations in pheochromocytoma patients, but
vertebral endplates.
only after -adrenergic blockade has been es-
tablished, since -blockade will exacerbate Answer A is incorrect. Bence-Jones protein is
peripheral vasoconstriction, thereby worsen- found in the urine of patients with multiple
ing hypertension in the setting of unopposed myeloma. Multiple myeloma is a malignancy
-adrenergic stimulation. of plasma cells in which patients develop lytic
lesions of the skeleton. Radiographs show the

Endocrinology
43. The correct answer is E. Reduction in weight classic nding of multiple punched-out, round
by just 7% and incorporating 30 minutes of lesions within the skull, spine, and pelvis.
daily activity reduced risk of DM by 58% in a
Answer B is incorrect. Patients with end-stage
landmark study. According to the AMerican
renal disease develop secondary hyperparathy-
Heart Association, diabetes is an independent
roidism, not decreased levels of PTH. Hypopara-
risk factor for coronary artery disease. There-
thyroidism is usually acquired after surgical re-
fore, lowering the risk of diabetes will, in turn,
section of the parathyroid glands for adenoma
decrease the risk of coronary artery disease
or inadvertently during thyroidectomy. Patients
Answer A is incorrect. Alcohol avoidance will can also have genetic deciencies of PTH.
not lower the patients risk of DM; although al-
Answer C is incorrect. Patients with end-stage
cohol excess can lead to either acute or chronic
renal disease tend to have elevated serum phos-
pancreatitis, it is not necessary to remove alco-
phate levels due to decreased excretion by the
hol from the diet completely.
kidney. The elevated phosphate contributes to
Answer B is incorrect. Use of a daily multivi- the development of secondary hyperparathy-
tamin is not necessary if ones diet is adequate, roidism.
but it can serve as an insurance policy for vi-
Answer D is incorrect. Elevated bone-specic al-
tamin and mineral intake. Its use, however, is
kaline phosphatase is found in patients with Pag-
not linked to DM risk reduction.
ets disease, a disorder of increased bone turnover.
Answer C is incorrect. Dietary changes, includ- Patients have excessive bone resorption and new
ing incorporation of fruits and vegetables, are bone formation. Radiographic ndings include
important, as they may contribute to weight loss, a cotton-wool appearance of the skull due to
but it is the weight loss that reduces DM risk by multifocal areas of sclerosis. In the spine, verte-
reducing the bodys total insulin requirements. brae can develop a thickened cortical margin,
causing a picture-frame appearance.
Answer D is incorrect. Smoking cessation,
while important for overall health, has not
45. The correct answer is C. This patient has clini-
been shown to reduce the incidence of DM.
cal symptoms and laboratory evidence consis-
tent with hyperthyroidism and the presence of
44. The correct answer is E. This patient has radio-
TSH-receptor antibodies is consistent with
graphic ndings consistent with bone disease
Graves disease, which is the most common
caused by secondary hyperparathyroidism. Pa-
cause of hyperthyroidism in both normal pa-
tients with end-stage renal disease have impaired
tients and pregnant women. If left untreated,
excretion of phosphate. Excess phosphate com-
maternal hyperthyroidism can lead to spontane-
plexes with calcium, leading to a secondary in-
ous abortion, premature labor, preeclampsia,
crease in PTH secretion. PTH acts on the bone
and maternal heart failure. The thionamides are
to increase osteoclastic resorption in an attempt
HIGH-YIELD SYSTEMS
88 Section I: Organ Systems Answers

considered rst-line therapy in pregnant Answer A is incorrect. The patient is experi-


women, including methimazole and propylthio- encing a dull aching abdominal pain that is
uracil. Although both cross the placenta and not localized and accompanied by nausea, but
can cause fetal goiter and hypothyroidism, pro- she remains afebrile and has a nonsurgical ab-
pylthiouracil is the preferred drug because it has domen on examination. While appendicitis is
a lower risk of severe congenital anomalies. still a possibility, the patients electrolyte abnor-
malities and inability to lactate suggest another
Answer A is incorrect. Although low-dose io-
cause.
dine may be safe for pregnant women with
mild hyperthyroidism, high-dose therapy is as- Answer B is incorrect. HELLP syndrome
sociated with an increased risk of fetal goiter, is dened by Hemolysis, Elevated Liver en-
so it should not be used in pregnant patients. zymes, and Low Platelets. Other important
diagnostic criteria include hypertension, visual
Answer B is incorrect. Methimazole, along
disturbance, headache, 2+ proteinuria, and se-
with the other thiocarbamide drugs, blocks
vere edema. HELLP syndrome is seen in pa-
thyroid hormone synthesis by inhibiting thy-
Endocrinology

tients experiencing eclampsia. Denitive treat-


roid peroxidase. It has been associated with a
ment is delivery. This patient has none of the
higher risk of aplasia cutis, a rare fetal scalp
classic signs and is hypotensive.
disorder. There has also been a suggested as-
sociation with tracheoesophageal stulas and Answer C is incorrect. Postoperative infec-
choanal atresia. Therefore propylthiouracil is tion should always remain on the differential,
typically chosen over methimazole in pregnant but this patient is afebrile and the surgical site
patients. is unremarkable. Additionally her inability to
lactate and electrolyte abnormalities cannot be
Answer D is incorrect. Radioiodine ablation is
explained by infection.
absolutely contraindicated in pregnancy. Be-
cause fetal thyroid tissue is present by 1012 Answer E is incorrect. Toxic shock syndrome
weeks gestation, radioiodine therapy can de- would be unlikely in this case, as the patient is
stroy fetal thyroid tissue. afebrile and has no rash. Staphylococcus aureus
is the most common pathogen associated with
Answer E is incorrect. Surgical resection may
toxic shock syndrome, and this is a common
be necessary in a pregnant patient with hyper-
postoperative pathogen, but the patient would
thyroidism, but surgery is associated with an in-
likely present with fever and erythema, edema,
creased risk of spontaneous abortion and pre-
and warmth around the surgical incision site.
mature delivery. As with nonpregnant patients,
medical therapy should be attempted rst. If
47. The correct answer is A. This patient has the
the patient cannot tolerate thionamide therapy,
classic symptoms of Cushings syndrome,
then surgery may be the next treatment option.
which most directly results from an excess of
cortisol. ACTH is secreted by the anterior pitu-
46. The correct answer is D. Sheehans syndrome
itary and stimulates the adrenal cortex to se-
is postpartum pituitary necrosis, usually in the
crete cortisol. The most common cause of
setting of obstetric hemorrhage and circulatory
Cushings syndrome is exogenously adminis-
collapse. Sheehans syndrome is a secondary
tered corticosteroids; however, the patient was
cause of hypoadrenalism, which would explain
not on any medication, and the most common
the patients hyponatremia. Also common are
noniatrogenic etiology is Cushings disease, an
abdominal pain, weakness, fatigue, and hy-
ACTH-secreting pituitary adenoma. In addi-
potension. The patient with Sheehans syn-
tion, hyperpigmentation with hypercortisolism
drome will often have an inability to breast-feed
is only seen in Cushings disease, due to in-
secondary to decient prolactin production and
creased production of melanocyte-stimulating
have other endocrine abnormalities associated
hormone induced by the tumor.
with loss of anterior pituitary hormone produc-
tion (the posterior pituitary is usually preserved), Answer B is incorrect. Although adrenal tu-
such as hypothyroidism. mors can potentially lead to oversecretion of
HIGH-YIELD SYSTEMS
Chapter 3: Endocrinology Answers 89

cortisol, they are not as common as ACTH- Answer E is incorrect. Carcinoid syndrome
secreting pituitary adenomas. typically affects the right heart (except in the
case of a primary carcinoid tumor in the bron-
Answer C is incorrect. An ectopic ACTH-
chus), and congestion, if present, is usually
secreting tumor will present in a manner simi-
mild. X-ray of the chest may reveal a promi-
lar to an ACTH-secreting pituitary adenoma
nence of the right heart border.
because it is secreting ACTH out of the con-
trol of the hypothalamus-pituitary-adrenal axis.
49. The correct answer is A. The patient has con-
It is not, however, as common as the ACTH-
genital hypothyroidism, which usually results
secreting pituitary adenoma.
from thyroid gland dysgenesis. It may also occur
Answer D is incorrect. Adrenal hyperplasia, from inborn errors in thyroid hormone metabo-
which is most commonly bilateral, can lead lism or from the presence of TSH receptor anti-
to oversecretion of cortisol and Cushings syn- bodies. Infants with congenital hypothyroidism
drome. Primary disease (in the absence of ele- often appear normal at birth, although some de-
vated ACTH) is extremely rare and is most ap- velop jaundice and hypotonia. Children with

Endocrinology
propriately treated by bilateral adrenalectomy. hypothyroidism may also present with protrud-
ing tongue and abdomen, umbilical hernias,
Answer E is incorrect. Small cell lung can-
and failure to thrive. Permanent neurologic
cers can be ACTH secreting and are a possible
damage results if treatment is delayed. The
cause of Cushings syndrome, although not the
child in this case exhibits the physical features
most common.
of congenital hypothyroidism. She also exhibits
48. The correct answer is A. Increased levels of profound motor and cognitive developmental
urine 5-hydroxyindoleacetic acid (5-HIAA) are delay. Her neurologic decits are most likely ir-
a byproduct of serotonin metabolism and are reversible at this stage, but treatment with exog-
consistent with carcinoid syndrome. Carcinoid enous thyroid hormone should be initiated to
tumors often affect the right heart due to - prevent other sequelae of hypothyroidism, such
brous deposits on the right-sided valves that as myxedema coma. Neurologic abnormalities
can induce right-sided heart failure. The carci- could have been prevented with neonatal
noid syndrome includes ushing, diarrhea, and screening, which evaluates TSH or levothyrox-
hypotension. The murmur is likely due to tri- ine levels. Early intervention with exogenous
cuspid regurgitation, although pulmonary levothyroxine and close TSH level monitoring
valve involvement is also common. Patients can result in normal IQ.
with cardiac involvement have higher levels of Answer B is incorrect. Cushings syndrome
plasma serotonin and urine 5-HIAA. can result from pituitary overproduction of
Answer B is incorrect. Urinary catecholamines ACTH, or from overproduction of cortisol by
and metanephrine are found in association the adrenal glands. Elevated levels of cortisol
with pheochromocytomas, which would pre- in Cushings syndrome result in a stereotypic
sent with hypertension, not hypotension. Cushingoid appearance, which involves ab-
dominal obesity, moon-like facies, and purple-
Answer C is incorrect. Peaked T waves are a colored striae. Cushings syndrome is not asso-
common nding in hyperkalemia but are not ciated with developmental abnormalities. The
associated with carcinoid heart disease. The patients appearance is more consistent with
ECG in carcinoid may show low QRS voltage. congenital hypothyroidism, and the abdominal
Answer D is incorrect. Sepsis can cause hy- mass is identied as an umbilical hernia, not
potension and would be associated with fatigue an adrenal mass.
(and a murmur if due to endocarditis). How- Answer C is incorrect. Neuroblastomas are
ever, the time course is unusual given an oth- tumors of neural crest cell origin that often
erwise healthy individual, and Pseudomonas occur in children <2 years and may develop
bacteremia would be unlikely outside the hos- anywhere in the body. They are associated
pital setting.
HIGH-YIELD SYSTEMS
90 Section I: Organ Systems Answers

with Hirschsprungs disease and neurobroma- fects. Abnormal facies, cleft palate, congenital
tosis. Abdominal neuroblastomas may present heart defects, thymic aplasia, and parathyroid
as nontender abdominal masses and may cross hypoplasia with hypocalcemia characterize Di-
the midline but are not associated with umbili- Georges syndrome. Patients are identied by
cal herniation. Neuroblastomas are associated physical examination, tetany in the neonatal
with elevated urinary catecholamine levels period, and frequent infections, especially with
(e.g., vanillylmandelic acid). The childs his- fungus and/or P. jiroveci. Evaluation should
tory and physical examination are more consis- include measurement of serum calcium,
tent with congenital hypothyroidism. echocardiogram, and absolute lymphocyte
count. The diagnosis is made by detection of
Answer D is incorrect. Phenylketonuria
the deletion on uorescent in situ hybridiza-
(PKU) is an autosomal recessive disorder that
tion.
results from reduced activity of phenylalanine
hydroxylase, which normally converts pheny- Answer A is incorrect. Hemoglobin electro-
lalanine to tyrosine. Accumulation of pheny- phoresis is diagnostic for sickle cell disease and
Endocrinology

lalanine inhibits myelin production and the thalassemia. These disorders are not associ-
synthesis of neurotransmitters, resulting in se- ated with congenital cardiac anomalies, facial
vere mental retardation if untreated within the dimorphisms, or cleft palate. Children with
rst 3 weeks of life. Children with PKU appear sickle cell disease are at increased risk of infec-
normal at birth, but develop microcephaly, sei- tion from encapsulated organisms because of
zures, hyperactivity, and mental retardation if functional asplenism from either dysfunction
untreated. PKU is also associated with hypo- and/or autoinfarction of the spleen, which can
pigmentation, a mousy odor, and eczema. present with life-threatening infection(s) as
Treatment involves phenylalanine restriction early as 4 months of age.
and dietary supplementation of tyrosine. Chil-
Answer B is incorrect. This test is used to de-
dren whose mothers are affected by PKU may
tect chronic granulomatous disease (CGD).
also develop mental retardation from high se-
Recurrent mucous membrane infections with
rum levels of phenylalanine if the maternal
catalase-positive organisms, abscesses, and
diet is not adequately controlled. Although the
poor wound healing characterize CGD. CGD
child in this case exhibits mental retardation
is not associated with cardiac anomalies, facial
similar to children with PKU, her physical ap-
anomalies, or cleft palate.
pearance is more consistent with the diagnosis
of congenital hypothyroidism. Answer C is incorrect. This test is useful in diag-
nosing B-lymphocyte deciencies. B-lymphocyte
Answer E is incorrect. Turners syndrome is a
deciencies rarely present in patients <6 months
genetic disorder associated with gonadal dys-
because maternally derived antibodies are pro-
genesis and a 45X karyotype. Girls affected by
tective against infection until they clear from the
Turners syndrome have a stereotypic appear-
infants circulation at approximately 6 months of
ance involving short stature, web neck, shield
age. They are not associated with congenital car-
chest, short fourth metacarpal, and microg-
diac malformations or cleft palate.
nathia. Infants may also present with cardiac
abnormalities, including coarctation of the Answer D is incorrect. Congenital cardiac
aorta, and horseshoe kidney. Umbilical her- anomalies can be associated with renal anoma-
nias, protruding tongue, and mental retarda- lies as part of the VACTERL syndrome, which
tion are not commonly associated with Turn- includes anomalies of the Vertebrae, Anus,
ers syndrome. Cardiovascular tree, Trachea, Esophagus, Re-
nal system, and Limb buds. However, this pa-
50. The correct answer is E. This child has Di- tient does not have signs or symptoms of the
Georges syndrome, with deletion of chromo- other anomalies, and the VACTERL syndrome
some 22q11. This chromosomal anomaly does not explain his frequent infections, facial
causes the third and fourth pharyngeal pouches anomalies, or cleft palate.
to develop abnormally, resulting in midline de-
CHAPTER 4

Epidemiology
and Preventive
Medicine

91
HIGH-YIELD SYSTEMS
92 Section I: Organ Systems Questions

Q U E ST I O N S

1. A 53-year-old woman presents to the clinic for (A) The culture may grow strains of bacteria
her yearly physical. She is concerned about which are not detected by the rapid test
heart disease because her mother recently had (B) The rapid streptococcal test and culture
a myocardial infarction at age 74 years and her both have high sensitivity
father passed away from heart disease at the age (C) The rapid streptococcal test and culture
of 63 years. She is a high school teacher who both have high specicity
smokes a half-pack of cigarettes per day. Her (D) The rapid streptococcal test has a high sen-
blood pressure during the last two ofce visits sitivity, while the culture has high specic-
was 122/69 and 128/73 mm Hg. Her last mea- ity
sured HDL cholesterol level was 63 mg/dL. (E) The rapid streptococcal test has a high
On physical examination she is close to her specicity, while the culture has high sen-
ideal body weight, and there are no abnormali- sitivity
Epidemiology

ties. Which of the following is an appropriate


goal for this patients LDL cholesterol level? 4. A 34-year-old woman presents to a surgeon to
discuss the possibility of an elective splenec-
(A) 100 mg/dL or less
tomy. Several weeks ago she was diagnosed
(B) 130 mg/dL or less
with idiopathic thrombocytopenic purpura. At
(C) 160 mg/dL or less
that time her platelet count was 24,000/mm.
(D) 200 mg/dL or less
Her primary care doctor started her on oral
(E) Direct therapy at raising the HDL choles-
prednisone, 50 mg/daily. At presentation today
terol level, not reducing LDL cholesterol
she continues to have lower extremity pete-
chiae along with mucosal bleeding. Her repeat
2. Investigators in a multisite trial are attempting
platelet count is 29,000/mm. Infection with
to determine how many patients are needed
which of the following is most likely to result
before they break the blinding on their study
in postsplenectomy sepsis?
and analyze their data. They want to ensure
that there are enough patients to detect a dif- (A) Haemophilus inuenzae
ference between treatment and placebo if one (B) Moraxella catarrhalis
is actually there. It is a very costly trial, and the (C) Pseudomonas aeruginosa
study sponsors do not want to pay for more (D) Staphylococcus aureus
work than is needed to prove a difference. (E) Viridans streptococci
What calculation do the researchers need to
perform to determine how much funding to re- 5. An investigator wishes to assess whether vacci-
quest from their sponsoring agency? nation increases the incidence of developmen-
tal diseases such as autism. He asks parents of
(A) Condence interval
recently diagnosed cases about their childs
(B) P value
vaccination history and compares their re-
(C) Power
sponses to those of parents of children who
(D) Type I error
have not been diagnosed with autism. What is
(E) Type II error
the most signicant type of potential bias pres-
ent in this study design?
3. A 3-year-old boy is brought to the clinic by his
mother. She is concerned because the patient (A) Confounding bias
has complained of a sore throat over the past 4 (B) Lead-time bias
days. He is not in day care and has had no (C) Length bias
known sick contacts. His 7-year-old sister is (D) Measurement bias
currently healthy. The physician does a throat (E) Recall bias
swab for a rapid streptococcal test and sends a
second specimen for culture. Why does the 6. A 65-year-old woman presents with 3 months
physician perform both tests? of unintentional weight loss, jaundice, and up-
HIGH-YIELD SYSTEMS
Chapter 4: Epidemiology Questions 93

per abdominal pain that radiates to her back. (C) Cancer/polyp > diverticulosis > angiodys-
Her gallbladder is palpable on physical exami- plasia
nation, and an ultrasound demonstrates dilated (D) Diverticulosis > angiodysplasia > cancer/
bile ducts with no visible stones. Which of the polyp
following is a known risk factor for this patients (E) Diverticulosis > cancer/polyp > angiodys-
condition? plasia
(A) Chronic gastritis
9. A hypothetical study is created to examine the
(B) Diabetes insipidus
effect of cigarette smoking on the development
(C) Diabetes mellitus
of lung cancer. Patients who smoke at least one
(D) History of cholecystitis
pack per day are matched with an appropriate
(E) Smoking
group of nonsmokers. Ten years later, informa-
tion on the development of lung cancer is col-
7. A 21-year-old woman comes to the clinic for a
lected, and the following data are observed.
health check-up. She has been sexually active
What is the relative risk (risk ratio) of develop-

Epidemiology
since she was 17 years old and has never seen a
ing lung cancer for cigarette smokers?
gynecologist before. The physician discusses
the importance of health screening with the (+) Lung cancer (-) Lung cancer
patient and performs a Papanicolaou (or Pap) Exposure to cigarettes (no. of pts.) (no. of pts.)

smear during the visit. Which of the following


is the most appropriate screening schedule rec- History of exposure a = 20 b = 80

ommendation? No history of exposure c=1 d = 100

(A) Pap tests every 3 years beginning approxi- Total patients 21 180
mately 3 years after rst intercourse or at
21 years of age Reproduced, with permission, from USMLERx.com.
(B) Yearly Pap test and human papillomavirus (A) 1 / 20
DNA testing beginning approximately 3 (B) 19 / 100
years after rst intercourse or at 21 years of (C) 1
age (D) 20
(C) Yearly Pap tests beginning approximately 3 (E) 20 / (80 / 99)
years after rst intercourse or at 21 years of
age 10. A study is performed to assess the relationship
(D) Yearly Pap tests beginning at menarche of dietary cholesterol and myocardial infarc-
and ending at menopause tion. Participants are assigned to either an ex-
(E) Yearly Pap tests once a patient is sexually perimental group, which eats a low-cholesterol
active or at 21 years of age diet, or a control group, which eats a standard
American diet. Dietary analysis later reveals
8. A 78-year-old woman in otherwise good health that the group assigned a low-cholesterol diet
schedules a visit with her primary care physi- ended up also consuming less fat than the con-
cian because she has been experiencing blood trol group. In analyzing the relationship be-
in her stool that she describes as maroon. tween dietary cholesterol and myocardial in-
Her past medical history is signicant for psori- farction, the amount of dietary fat consumed
asis and mild degenerative joint disease. In represents what type of study characteristic?
thinking about the possible causes of her lower
gastrointestinal bleed, which of the following (A) Confounding variable
lists is in order of most common to least com- (B) Enrollment bias
mon? (C) Measurement bias
(D) Recall bias
(A) Angiodysplasia > cancer/polyp > diverticu- (E) Self-selection bias
losis
(B) Angiodysplasia > diverticulosis > cancer/
polyp
HIGH-YIELD SYSTEMS
94 Section I: Organ Systems Questions

11. A 5-year-old boy is brought to his pediatrician (B) Women >30 years old who have had three
because he has been complaining of fatigue consecutive conventional cytology nega-
and headache for several weeks. On examina- tive Pap smears
tion he has enlarged lymph nodes and is sent (C) Women >60 years old who are positive for
for a bone marrow biopsy. The biopsy shows human papillomavirus DNA
large lymphoblasts with prominent nucleoli (D) Women who have had a hysterectomy and
and light blue cytoplasm, and the patient is di- documented cervical intraepithelial neo-
agnosed with acute lymphoid leukemia. He is plasia II/III
treated with high-dose glucocorticoids, intrave- (E) Women with 2 or more negative conven-
nous and intrathecal methotrexate, cyclophos- tional cytologic studies
phamide, doxorubicin, and vincristine. He
achieves a full remission. Which of the follow- 14. A 53-year-old man presents to his physicians
ing accurately matches the causative drug with ofce in tears. A close friend of his was recently
the possible adverse effect that he may experi- diagnosed with lung cancer, so the patient is
ence now or in the future? concerned because he has smoked a pack of
Epidemiology

cigarettes daily for 39 years and his father died


(A) Cyclophosphamide: hearing loss
of lung cancer. He recalls that he had a pro-
(B) Glucocorticoids: cataracts
ductive cough 2 months prior that resolved af-
(C) Glucocorticoids: peripheral neuropathy
ter several weeks. He is concerned that he
(D) Methotrexate: cognitive decit
might have lung cancer or might develop it in
(E) Methotrexate: pulmonary brosis
the future. He asks whether there is a test he
can undergo on a regular basis to catch the
12. A 72-year-old man presents to his primary phy-
cancer early. Which of the following is the
sician with complaints of fatigue, weight loss,
most appropriate screening test for this patient?
dyspnea on exertion, abdominal pain, and dark
blood in the stool. Although the patient had a (A) Annual x-ray of the chest
negative sigmoidoscopy on routine examina- (B) Biannual low-dose helical CT scan
tion 6 months ago, colon cancer is strongly sus- (C) Biannual sputum cytology
pected. Which is the best diagnostic modality (D) Bronchoscopy
to use in this patient? (E) None; advise the patient to quit smoking
and monitor for clinical signs of lung can-
(A) Colonoscopy
cer
(B) CT of the abdomen
(C) Double contrast barium enema
15. A 34-year-old man was diagnosed with type 2
(D) Sigmoidoscopy
diabetes mellitus as a child and has been pre-
(E) Upper gastrointestinal series
senting for annual examinations since he was
in his early 20s. Which of the following screen-
13. A 23-year-old woman presents to her gynecolo-
ing tests is indicated more than once a year?
gist for a rell of her birth control prescription.
She denies recent sexual activity, although she (A) Hemoglobin A1c
has been sexually active in the past and is cur- (B) Lipid prole
rently using oral contraceptives to regulate her (C) Microalbuminuria
menstrual cycle and control acne. Her last Pap (D) Ophthalmologic examination
smear was 13 months prior. She says that her (E) Podiatry examination
31-year-old sister just went for her annual ex- (F) Serum fructosamine
amination and was told that she did not need
an annual Pap smear. In which of the follow- 16. Laboratory researchers are developing a sali-
ing populations may an annual Pap smear be vary test for systemic cocaine. The manufac-
deferred? turer has requested that their ideal test will
miss very few, if any, people who have used co-
(A) HIV-positive women with no documented
caine recently. Which of the following best de-
history of cervical pathology
HIGH-YIELD SYSTEMS
Chapter 4: Epidemiology Questions 95

scribes what the manufacturer wants in epide-


miologic terms?
(A) Of cocaine users, the probability of testing
positive is very high
(B) Of people with a negative test, the proba-
bility of having used cocaine is very low
(C) Of people with a positive test, the probabil-
ity of having used cocaine is very high
(D) Of those not using cocaine, the probability
of testing negative is very low

17. The cut-off value of normal prostate-specic Reproduced, with permission, from Kasper DL, Braun-
antigen levels is 4.0 ng/dL. If this value were to wald E, Fauci AS, Hauser SL, Longo DL, Jameson LJ, Is-
be decreased to 3.7 ng/dL, the sensitivity and selbacher KJ, eds. Harrisons Online. New York: McGraw-

Epidemiology
specicity of the test for detecting prostate can- Hill, 2005: Figure 272-14.
cer will change. How would lowering the nor- (A) His son should be screened with a colonos-
mal value of prostate-specic antigen in cancer copy every 12 years starting now
screening alter the number of false-positives (B) His son should be screened with a colonos-
and false-negatives? copy every 5 years starting at age 40 years
(A) Decrease both false-positives and false- (C) His son should be screened with a colonos-
negatives copy every 5 years starting at age 50 years
(B) Decrease the number of false-negatives (D) His son should be screened with a colonos-
and increase the number of false-positives copy every 5 years starting now
(C) Increase both false-positives and false- (E) His son should be screened with a fecal
negatives occult blood test every year starting at age
(D) Increase the number of false-negatives and 50 years
decrease the number of false-positives
(E) It would not change either parameter 19. A study to evaluate the relationship of caffeine
consumption and gastric cancer is conducted.
18. A 57-year-old man comes to his primary care Patients with gastric cancer are enrolled in the
physician complaining of increasing fatigue in study, and researchers seek to match each pa-
the past few months. He is unable to walk up tient with an individual similar to him or her
to his third oor apartment without becoming in as many ways as possible, except that the
short of breath. His WBC count is 5000/mm, control subject does not have a diagnosis of
hemoglobin is 10.2 g/dL, hematocrit is 37%, gastric cancer. Both case and control subjects
platelet count is 221,000/mm, and mean cor- are then surveyed to examine possible expo-
puscular volume is 70 fL. Additionally, the pa- sures, including caffeine exposure. Of 500 pa-
tients stool is guaiac-positive. The physician tients, 250 are cases and an equal number are
sends the patient for colonoscopy, which is controls. Of cases, 150 met the minimum caf-
shown in the image. With the new diagnosis, feine consumption limit set by the study,
the patient is concerned about his 35-year-old whereas only 50 controls did. What is the odds
son. What recommendations should the physi- ratio dening the relationship of exposure and
cian make regarding the son? disease in this study?
(A) 1/6
(B) 0.375
(C) 1.7
(D) 2.7
(E) 3
(F) 6
HIGH-YIELD SYSTEMS
96 Section I: Organ Systems Questions

20. A 4-year-old girl is brought by her mother to (A) Case-control study


the pediatrician because of a rash. She had a (B) Cohort study
fever and headache yesterday, and this morn- (C) Cross-sectional survey
ing broke out in a rash on her torso. The le- (D) Meta-analysis
sions are erythematous macules and vesicles in (E) Randomized controlled clinical trial
different stages, distributed diffusely over her
trunk but most prominent on her left upper 23. A 34-year-old G4P3 woman presents to the
chest and right ank. A diagnosis of chicken- clinic with her husband desiring contracep-
pox is made. Of note, there is a 16-year-old sib- tion. She has a past medical history of endo-
ling in the household who has not been vacci- metriosis, for which she was prescribed oral
nated and has never had chickenpox. Which contraceptive pills as a teenager. Her past sur-
of the following interventions will most suc- gical history includes an exploratory laparo-
cessfully prevent infection of susceptible tomy with laser ablation and resection of an
household contacts? endometrioma at the age of 26 years as part of
an infertility work-up. She subsequently deliv-
Epidemiology

(A) Administration of intravenous acyclovir to


ered three healthy children with no obstetric
the patient
complications. She denies any history of sexu-
(B) Administration of oral acyclovir to the pa-
ally transmitted disease and is sexually active
tient and household contacts
with her husband only. She states that she and
(C) Administration of vaccine and intramuscu-
her husband do not desire any more children.
lar varicella zoster immunoglobulin to
Which of the following is the most effective
household contacts
contraceptive option and will therefore have
(D) Administration of varicella vaccine to
the lowest failure rate for this couple?
household contacts
(E) No prophylaxis is necessary for household (A) Bilateral tubal ligation
contacts under age 25 years (B) Bilateral vasectomy
(C) Combination oral contraceptive pill
21. A researcher wants to examine the relationship (D) Diaphragm
of childhood cigarette exposure and subse- (E) Intrauterine device
quent asthma diagnosis. She designs a study in (F) Male condom
which telephone interviewers contact ran- (G) Progestin-only oral contraceptive pill
domly chosen study participants with questions
regarding their childhoods and whether they 24. A researcher wants to study outcomes after ex-
have asthma. Which type of study is this re- posure to a particular drug. Participants will be
searcher conducting? randomized into an exposed group and an un-
exposed group, and observations will be made
(A) Case-control study
and data collected regarding the outcome of
(B) Cohort study
each study participant. What does it mean for
(C) Cross-sectional study
the researcher to single-blind the study?
(D) Metaanalysis
(E) Randomized controlled clinical trial (A) Neither participants nor researchers know
whether a participant is assigned to the
22. A researcher studying the impact of drug A on study group or the control group
hypertension assigns patients randomly to ei- (B) Participants do not know if they are as-
ther a treatment group or a control group. The signed to the study group or the control
treatment group receives drug A, and the con- group
trol group receives the current standard medi- (C) Researchers do not know if a participant is
cation. Neither the study staff nor the patients assigned to the study group or the control
know what drug a given patient is receiving. group
What type of study is the researcher conduct- (D) Researchers do not know if the drug will
ing? be efcacious
(E) Researchers do not know if the drug will
be safe
HIGH-YIELD SYSTEMS
Chapter 4: Epidemiology Questions 97

25. A 27-year-old primigravida at 35 weeks gesta- (D) Smoking cigarettes increases the risk for
tion presents to her obstetrician because of a vascular dementia
long bump on her leg. She denies ever having (E) Smoking increases the risk for vascular de-
had a problem like this before. Her pregnancy mentia, but alcohol has no effect on the
has been without complication. Prior to her risk
pregnancy she used only barrier protection.
Age of menarche was 12 years. She denies any 27. A cardiologist wishes to compare outcomes of
travel history and has been mainly in bed medical management and cardiac catheteriza-
since 1 week ago. Her pulse is 78/min, respira- tion for myocardial infarct presentation to the
tory rate is 10/min, and temperature is 37.2C emergency department. She conducts a litera-
(99F). On physical examination she in no ture search and statistically combines the data
acute distress. Her lungs are clear to ausculta- from seven published studies. Which of the fol-
tion bilaterally, and her extremities are warm lowing is an advantage of this type of study?
and well perfused. Examination of the extremi-
(A) Able to overcome poor study design in in-
ties reveals a long, rm, cord-like mass on the

Epidemiology
dividual studies
lateral aspect of her left leg. It is subcutaneous,
(B) Less bias than other types of studies
erythematous, and tender. Compression ultra-
(C) May allow detection of small differences
sonography of the lower extremity shows nor-
between outcomes
mal compressibility and normal Doppler ow.
(D) Pooled data already represent sufciently
Which of the following is the most appropriate
similar populations and interventions
management?
(E) Statistical analysis of pooled data is
(A) Advise the patient to take heparin if the straightforward
mass does not resolve
(B) Advise the patient to take warfarin if the 28. A 26-year-old G2P1 woman at 19 weeks gesta-
mass does not resolve tion comes into the clinic for her scheduled
(C) Advise the patient to use warm compresses prenatal visit. She complains only of mild fa-
and elevate the affected area tigue. Past obstetric history is notable for a
(D) Reassure the patient that ibuprofen is safe prior pregnancy 2 years ago. Although that
in pregnancy pregnancy was without complications, the
child has signicant developmental delay. The
26. A group of researchers at the state university patient takes no medications other than prena-
conducts a study investigating the relationship tal vitamins. Her family history is signicant
between cigarette smoking and vascular de- for two maternal aunts with mental retarda-
mentia. Retrospective data are obtained from a tion. The patient is an only child. Physical ex-
cross-sectional survey on smoking habits and amination reveals a well-developed gravid fe-
later neurologic symptoms as observed by close male in no acute distress. The uterus is
relatives. Researchers nd that smokers are appreciated well above the pubic symphysis. A
four times as likely as nonsmokers to suffer triple screen test is ordered. What other test is
from vascular dementia and that they also con- most appropriate to order at this visit?
sume twice as much alcohol as nonsmokers in
(A) -Fetoprotein level
their study. Which of the following statements
(B) Amniocentesis
could be accurately made on the basis of the
(C) Biophysical prole
available information?
(D) Chorionic villus sampling
(A) Both alcohol and cigarettes independently (E) Nonstress test
increase the risk for vascular dementia (F) Percutaneous umbilical blood sampling
(B) Drinking alcohol increases the risk for vas- (G) Ultrasound
cular dementia
(C) Smokers are more likely to drink than non-
smokers
HIGH-YIELD SYSTEMS
98 Section I: Organ Systems Questions

29. A 55-year-old G2P2 woman presents to the gy- 31. A 17-year-old girl with a history of refractory
necology clinic for a routine check-up; her last B-lymphocyte lymphoma undergoes a bone
check-up was 3 years ago. Both of her children marrow transplant from a matched unrelated
were delivered by spontaneous vaginal delivery donor. On day 19 posttransplant an erythema-
without complications. She is currently sexu- tous, maculopapular rash is noted on her trunk
ally active with her husband and has no other and extremities. Within 3 days she has devel-
partners. She says she has an occasional glass oped diffuse bullae and severe diarrhea. She
of wine with dinner and has smoked about half requires vigorous hydration and narcotic pain
a pack of cigarettes per day for the past 25 medication. Jaundice is noted, and her serum
years. She has no family history of cancer, total bilirubin is 10 mg/dL. Which of the fol-
heart disease, or diabetes. Breast examination lowing interventions gives the patient the best
reveals no masses. Pelvic and bimanual exami- chance of survival?
nations are likewise within normal limits. A
(A) Hepatic transplantation
sample is taken for a Pap smear without caus-
(B) The patient has little chance of survival;
ing signicant cervical bleeding. Mammogra-
Epidemiology

donor T lymphocytes should have been re-


phy from 3 years ago shows no suspicious
moved from the stem cells prior to infu-
masses. A Pap smear from 3 years prior showed
sion
no atypical cells. Laboratory tests show:
(C) Total body irradiation to remove the graft
WBC count: 7500/mm (D) Treatment with monoclonal antibodies di-
Hemoglobin: 12.4 g/dL rected at T lymphocytes
Platelet count: 220,000/mm (E) Treatment with thalidomide
Total cholesterol: 195 mg/dL
LDL cholesterol: 125 mg/dL 32. A 52-year-old man comes to the clinic worried
HDL cholesterol: 45 mg/dL because he saw a television program on the in-
creased risk of thyroid cancer in people ex-
What is the most appropriate next step in man-
posed to radiation as a child. He says that when
agement?
he was a boy, his dermatologist used radiation
(A) Endometrial biopsy therapy to treat chronic cystic acne. After the
(B) Mammography history and physical examination are com-
(C) Measure cancer antigen-125 level pleted, the physician shows the patient a jour-
(D) Transvaginal ultrasound nal article that details a case-control study
(E) X-ray of the chest about the risk of radiation exposure and subse-
quent thyroid cancer. A table from the study is
30. A 48-year-old man is admitted to the hospital shown in the image. What conclusions can be
with fulminant hepatitis leading to liver failure drawn from this study?
following consumption of raw oysters. He is
(+) Acute coronary (-) Acute coronary
treated with immediate liver transplantation, Exposure to rofecoxib syndromes syndromes
which is performed successfully. Following a (no. of pts.) (no. of pts.)

prolonged hospital stay he is discharged in ex- History of exposure a = 10 b=5


cellent condition. In addition to his antirejec-
No history of exposure c = 10 d = 15
tion medications, which of the following is rec-
ommended for prophylaxis in this patient? Total patients 20 20

(A) Acyclovir Reproduced, with permission, from USMLERx.com.


(B) Ketoconazole
(C) Niacin (A) The odds of having been exposed to radia-
(D) Nystatin swish and swallow tion as a child are 0.33 times greater for
(E) Trimethoprim-sulfamethoxazole someone with thyroid cancer than for
someone without thyroid cancer
HIGH-YIELD SYSTEMS
Chapter 4: Epidemiology Questions 99

(B) The odds of having been exposed to radia- his rst hepatitis B vaccination at birth. Assum-
tion as a child are 1.5 times greater for ing this child is developing normally, which
someone with thyroid cancer than for vaccines should this infant receive today?
someone without thyroid cancer
(A) Hepatitis B and diphtheria-tetanus-pertussis
(C) The odds of having been exposed to radia-
only
tion as a child are 3 times greater for some-
(B) Hepatitis B, diphtheria-tetanus-pertussis,
one with thyroid cancer than for someone
and inactivated poliomyelitis
without thyroid cancer
(C) Hepatitis B, diphtheria-tetanus-pertussis,
(D) The relative risk of having been exposed to
Haemophilus inuenzae type b, inactivated
radiation as a child is 1.5 times greater for
poliomyelitis, and pneumococcal conju-
someone with thyroid cancer than for
gate
someone without thyroid cancer
(D) Hepatitis B only
(E) The relative risk of having been exposed to
(E) None; schedule a return visit when the
radiation as a child is 3 times greater for
childs nasal congestion subsides, and ad-
someone with thyroid cancer than for

Epidemiology
minister the immunizations then
someone without thyroid cancer
35. A 28-year-old G1P0 woman is 37 weeks preg-
33. A 75-year-old man presents to his physician
nant. Her maternal prenatal screening was un-
with a 4-week history of exertional shortness of
remarkable, and her pregnancy has been un-
breath. He worked as a pipe tter for 45 years,
eventful thus far. She has several questions
retiring 5 years ago. He denies chest pain, pal-
regarding what to expect for herself and her
pitations, swelling in his legs, cough, hemopty-
child during the rst few days after delivery.
sis, and weight loss. He smokes 1 pack per day
Which of the following vaccines is indicated
and has done so for the past 45 years. His tem-
during the postnatal period?
perature is 36.7C (98F), pulse is 85/min,
blood pressure is 120/80 mm Hg, respiratory (A) Diphtheria, pertussis, and tetanus vaccine
rate is 14/min, and oxygen saturation is 99% on (B) Haemophilus inuenzae B vaccine
room air. Physical examination is unremark- (C) Hepatitis B vaccine
able. Pulmonary function testing shows a mild (D) Inactivated polio vaccine
restrictive pattern with a normal diffusing ca- (E) Measles, mumps, and rubella vaccine
pacity. X-ray of the chest shows linear opacities (F) Varicella vaccine
at the lung bases and pleural plaques. Which
of the following is the most appropriate inter- 36. A 29-year-old man approaches a physician to
vention? ask about his target cholesterol level. He has
no personal or family history of cardiac disease.
(A) High-resolution CT of the chest He does not smoke, denies high blood pressure
(B) Pulmonary function testing and diabetes, and has a body mass index of 26
(C) Repeat sputum cytology every 6 months kg/m. The man says he has heard there are
(D) Repeat x-ray lms of the chest every 6 multiple types of cholesterol, but wants to
months know just one marker that will be the best for
(E) Smoking cessation him to follow. The physician advises that fol-
lowing the LDL cholesterol would be a reason-
34. A male infant is brought to his pediatricians of-
able option. At what LDL cholesterol level
ce for a well-child visit. He has a temperature
should pharmacologic therapy be considered?
of 37.2C (99F) and clear nasal drainage.
Physical examination is otherwise unremark- (A) LDL cholesterol >100 mg/dL
able. The infant is able to lift his head when (B) LDL cholesterol >130 mg/dL
prone, track past the midline, and coo. His im- (C) LDL cholesterol >160 mg/dL
munization record indicates that he received (D) LDL cholesterol >190 mg/dL
(E) LDL cholesterol >200 mg/dL
HIGH-YIELD SYSTEMS
100 Section I: Organ Systems Questions

37. A 22-year-old woman from sub-Saharan Africa E X T E N D E D M ATC H I N G


comes to the clinic to be tested for HIV. She
denies illicit drug use and has not been sexu-
ally active since she moved to the United The response options for the next 2 items are
States 3 years ago. Her initial enzyme-linked the same. Select one answer for each item
immunosorbent assay is positive. Which of the in the set.
following would best conrm the diagnosis of
HIV? Select the mechanism of action of the antihyper-
tensive drug that would be the best choice to treat
(A) A positive Western blot test each of the following patients.
(B) A repeat enzyme-linked immunosorbent
assay (A) Antagonizes aldosterone receptors
(C) Clinical evidence of an opportunistic in- (B) Antagonizes 1-adrenergic receptors
fection (C) Antagonizes 1- and 2-adrenergic recep-
(D) History of unprotected sex with an HIV- tors
(D) Blocks angiotensin II formation
Epidemiology

positive partner
(E) Lymphopenia (E) Blocks calcium inux into vascular smooth
muscle and myocardium
38. A 24-year-old man presents with notably (F) Blocks distal convoluted tubule sodium
slurred speech, stating, I dont want to live and chloride reabsorption
anymore, I just want to take pills and make the (G) Blocks loop of Henle sodium and chloride
pain go away. A psychiatric history reveals that reabsorption
the patient has suffered from major depression (H) Directly relaxes smooth muscle
for 9 years, has had numerous psychiatric ad- (I) Stimulates 2-adrenergic receptors
missions for violent behavior, and has at- (J) Stimulates aldosterone formation
tempted suicide twice by overdose. The patient (K) Stimulates angiotensin II formation
currently has a blood alcohol level of 0.012
mg/mL. Which of the following is this patients 39. A 52-year-old man with diabetes and hyperten-
most signicant risk factor for a completed sui- sion presents to a new primary care provider.
cide attempt? He currently uses insulin to control diabetes
mellitus and hydrochlorothiazide to control
(A) Antisocial behavior hypertension. On physical examination he is
(B) Depression an overweight man with no concerning nd-
(C) Male gender ings, except for a blood pressure of 171/93 mm
(D) Past suicide attempts Hg. His creatine level is 1.7 mg/dL with a uri-
(E) Substance abuse nalysis that shows proteinuria. His physician
wants to prescribe another antihypertensive
agent in addition to hydrochlorothiazide.

40. A 63-year-old woman with a history of osteopo-


rosis presents for a second opinion regarding
hypertension management because she is wor-
ried about starting any medication that might
worsen her osteoporosis. Her only medication
is calcium supplements. She has smoked 1
pack of cigarettes per day for 15 years. Her
physical examination is normal except for a
blood pressure of 165/90 mm Hg. Laboratory
tests show a total serum calcium level of 7.0
mg/dL and mild hypercalciuria.
HIGH-YIELD SYSTEMS
Chapter 4: Epidemiology Answers 101

AN S W E R S

1. The correct answer is C. According to the that shows the size of the effect and the statis-
most recent report from the National Choles- tical power. For example, the 95% condence
terol Education Program, the goal LDL cho- interval is the interval that is 95% likely to con-
lesterol level should take into account the indi- tain the true value.
viduals risk factors. This patients smoking
Answer B is incorrect. The P value is related
history is her only risk factor. In addition, her
to but not exactly the same as a condence in-
relatively high HDL cholesterol level is protec-
terval, in that it estimates the statistical signi-
tive. Therefore, she should be considered in
cance of an answer. It expresses the probabil-
the group with a zero to one risk factor, and
ity that differences could have happened by
her optimal LDL cholesterol level is 160 mg/
chance alone, and is often set to a 95% thresh-
dL or less.
old as well, which is expressed as P <0.05.

Epidemiology
Answer A is incorrect. The lowest goal for
Answer D is incorrect. Type I error is com-
LDL cholesterol level is reserved for those with
monly understood as a false-positive result.
known coronary heart disease or equivalent risk
It is the probability of saying there is a differ-
factors. This includes all patients with diabetes
ence in treatment effects when in fact there is
mellitus.
not. Generally, type I error is understood to be
Answer B is incorrect. Patients with 2 or more more calculable than type II error, although it
risk factors should be treated with at least life- is not used in power calculations.
style modication at an LDL cholesterol level
Answer E is incorrect. Type II error is dened
of 130 mg/dL or less. Risk factors include age
as the probability of saying that there is no dif-
(men >45 years, women >55 years), smoking,
ference in treatment effects when in fact there
low HDL cholesterol (<40 mg/dL), hyperten-
is (i.e., a false-negative). It is related to power
sion, and family history of premature coronary
by the following equation: type II error +
heart disease (rst-degree female relatives <65
power = 1.
years old or rst-degree male relatives <55
years old). 3. The correct answer is D. Screening tests, such
Answer D is incorrect. Individuals with no as the rapid streptococcal test, have high sensi-
risk factors are still recommended to undergo tivity and low specicity. If the rapid strepto-
lifestyle modications at any LDL cholesterol coccal test is positive, this indicates but does
>160 mg/dL. not prove that the patients pharyngitis may be
due to group A streptococcal infection. If the
Answer E is incorrect. Although HDL cho-
rapid streptococcal test is negative, then no fur-
lesterols an important risk factor for coronary
ther testing is needed, because sensitive tests
artery disease, both HDL and LDL cholesterol
effectively rule out disease. The throat culture
levels should be considered when formulating
is highly specic and thus is able to conrm
a treatment plan. This is because HDL choles-
(rule in) the disease. It may also indicate ex-
terol is a negative risk factor for heart disease.
actly which organism is involved and narrow
treatment options.
2. The correct answer is C. Power is the proba-
bility that a study will nd a statistically signi- Answer A is incorrect. Because detection of
cant difference when one is actually there. It -hemolytic streptococci is the goal of this test-
relates directly to the number of subjects. A ing series, other bacteria which may be inci-
power calculation is required in many grant dentally identied through culture are gener-
applications. ally not signicant.
Answer A is incorrect. The condence inter- Answer B is incorrect. The critical property
val is a way of expressing statistical signicance of culture is its high positive predictive value
HIGH-YIELD SYSTEMS
102 Section I: Organ Systems Answers

resulting from the high specicity of the test, However, this organism is not encapsulated
combined with the fact that most cultured pa- and has not been associated with overwhelm-
tients have tested positive on the rapid test and ing postsplenectomy sepsis.
thus are already likely to have the disease. Sen-
Answer E is incorrect. Viridans streptococci
sitivity is less important since most uninfected
are part of the normal ora of the mouth. Viri-
individuals will have already been identied by
dans streptococci are involved in subacute bac-
the rapid test.
terial endocarditis and can cause dental caries
Answer C is incorrect. The critical property of but not overwhelming postsplenectomy sepsis
the rapid streptococcal test is its high sensitiv- in asplenic patients.
ity; a negative result is considered denitive,
while a positive result requires culture to con- 5. The correct answer is E. Bias is any process
rm. A conrmatory test will have high speci- that causes results to systematically differ from
city, such as the culture. the truth. Recall bias occurs when cases and
controls remember information differently, that
Answer E is incorrect. As a general rule, a
Epidemiology

is, when a recent diagnosis may have prompted


rapid screening test will be highly sensitive,
them to search their memory for recent events
while a conrmatory test will have a high spec-
that they feel may have contributed to the diag-
icity and positive predictive value.
nosis. A false association may be found because
cases are more likely to remember recent vacci-
4. The correct answer is A. Asplenic patients are
nations, not because of any pathological pro-
at an increased risk of developing infections
cess.
compared to those patients with functional
spleens. These patients may develop over- Answer A is incorrect. A confounder is a dis-
whelming postsplenectomy sepsis in which en- ease risk factor that causes disease through a
capsulated organisms (including Streptococcus different causal pathway than the exposure of
pneumoniae, Neisseria meningitidis, and Hae- interest, but is also associated with the exposure
mophilus inuenzae) may cause fever, lethargy, of interest. For example, a researcher may nd
or upper respiratory infection that leads to an association between smoking and liver dis-
coma and death in up to 50% of patients. The ease, but the relationship could be confounded
capsule that surrounds these organisms is an by a separate association between smoking and
antiphagocytic factor that permits these organ- other substance use that drives the association
isms to survive and cause bacteremia. It is vital far more than the smoking itself.
for these patients to receive vaccination against
Answer B is incorrect. Lead-time bias occurs
these encapsulated organisms to prevent the
when a disease is identied earlier, giving an
development of postsplenectomy sepsis.
appearance of prolonged survival when the
Answer B is incorrect. Moraxella catarrhalis natural course is not actually altered. Assessing
frequently causes otitis media in pediatric pa- a population for a disease may cause a slow,
tients. It has not been associated with postsple- insidious disease to be diagnosed earlier than
nectomy sepsis in asplenic patients. usual, falsely prolonging survival.
Answer C is incorrect. Pseudomonas aerugi- Answer C is incorrect. Length bias occurs
nosa causes pneumonia, external otitis media, when a screening test detects a disproportion-
urinary tract infection, folliculitis, and wound ate number of slowly progressive diseases but
and burn infections. This organism has not misses rapidly progressive ones. This overesti-
been associated with the development of over- mates the benets of the screen by biasing the
whelming postsplenectomy sepsis in asplenic more slowly progressing diseases.
patients.
Answer D is incorrect. Measurement bias oc-
Answer D is incorrect. Staphylococcus aureus curs when data-gathering methods differ in dif-
has been associated with food poisoning, skin ferent groups of patients, that is, if symptoms
infections, organ abscesses, pneumonia, toxic in cases versus controls were assessed using dif-
shock syndrome, and scalded skin syndrome. ferent scales, or different labs processed blood
HIGH-YIELD SYSTEMS
Chapter 4: Epidemiology Answers 103

tests for the two different groups. This can dis- had their hysterectomy due to cervical carci-
tort the data in a systematic way that could ei- noma or preinvasive changes, or if they had a
ther obscure a real association or suggest a false hysterectomy without removal of the cervix.
association.
Answer A is incorrect. Pap tests in 3-year inter-
vals should only occur in women >30 old who
6. The correct answer is E. Patients with pancre-
have had three or more normal consecutive
atic cancer can present with weight loss, jaun-
test results. In women <30 years old, Pap tests
dice, abdominal pain, dark urine, acholic
should be performed annually.
stools, and pruritus. On physical examination
the gallbladder or other abdominal mass is pal- Answer B is incorrect. Conventional Pap test
pable. Diagnosis is usually made with ultra- plus human papillomavirus (HPV) DNA test
sound with ndings of dilated bile ducts or visi- every 3 years is only an acceptable option for
ble mass, or CT scan which demonstrates the women >30 years old who have had three or
pancreatic mass. The associated risk factors for more normal consecutive Pap test results.
pancreatic cancer include smoking, chronic

Epidemiology
Answer D is incorrect. Yearly Pap tests should
pancreatitis, a rst-degree relative with pancre-
begin within 3 years of becoming sexually ac-
atic cancer, and high-fat diet.
tive, or by the of age 21 years.
Answer A is incorrect. Chronic pancreatitis,
Answer E is incorrect. Women should start
not chronic gastritis, is a risk factor for pancre-
screening approximately 3 years after begin-
atic cancer.
ning to have vaginal intercourse, but at no later
Answer B is incorrect. Diabetes insipidus is than age 21 years.
not a pathology of the pancreas and is not a
risk factor for pancreatic cancer. 8. The correct answer is E. Diverticulosis ac-
counts for 42%55% of lower gastrointestinal
Answer C is incorrect. Diabetes mellitus can
bleeding, followed by cancer/polyps account-
result from neoplasm in the body of the pan-
ing for 8%26%, and bleeding and angiodys-
creas, but it is not a risk factor for pancreatic
plasia which accounts for 3%12%. Other
cancer.
causes include inammatory bowel disease
Answer D is incorrect. History of cholecystitis (2%8%), anorectal disease (3%9%), small
is not a risk factor for pancreatic cancer. Post- bowel disease (3%5%), infectious colitis (1%
operative biliary stricture should be on the dif- 5%), radiation colitis (1%5%), and vasculitis
ferential diagnosis in a patient with a history of (13%).
cholecystectomy who presents with symptoms
Answer A is incorrect. The correct order is di-
suggestive of pancreatic neoplasm.
verticulosis, cancer/polyp, and then angiodys-
plasia.
7. The correct answer is C. The American Can-
cer Society and the US Preventive Services Answer B is incorrect. The correct order is di-
Task Force recommend that cervical cancer verticulosis, cancer/polyp, and then angiodys-
screening take place with yearly Pap tests ap- plasia.
proximately 3 years after beginning to have
Answer C is incorrect. The correct order is di-
vaginal intercourse, but no later than age 21
verticulosis, cancer/polyp, and then angiodys-
years. Women >30 years old who are at low risk
plasia.
may then undergo Pap testing every 23 years
if they have had at least three normal consecu- Answer D is incorrect. The correct order is di-
tive Pap tests prior to this time. Women >70 verticulosis, cancer/polyp, and then angiodys-
years old with three or more normal consecu- plasia.
tive Pap tests and no abnormalities within the
past 10 years may choose to stop having cervi- 9. The correct answer is D. This is an example of
cal cancer screening. Furthermore, women a prospective cohort study. Using the letters
who have had a total hysterectomy may stop from the table, the relative risk is calculated as
having cervical cancer screening unless they [a / (a + b)] / [c / (c + d)] = 20 (rounded to the
HIGH-YIELD SYSTEMS
104 Section I: Organ Systems Answers

nearest whole number), so smokers have 20 negative consequences may be more likely to
times the risk of nonsmokers of developing remember the exposure than those who experi-
lung cancer. enced no problems afterward.
Answer A is incorrect. The reciprocal of the Answer E is incorrect. Self-selection bias re-
relative risk is 1 / 20, and it represents the rela- fers to the decision of the participant whether
tive risk of nonsmokers developing lung can- to enroll in the study. Those who choose to
cer as compared to that of smokers developing participate may have different characteristics as
lung cancer. a group than those who choose not to enroll.
Answer B is incorrect. The attributable risk
11. The correct answer is B. Chronic use of high-
(or risk difference), calculated as the risk in the
dose steroids, as those used to treat acute lym-
exposed population minus the risk in the un-
phoid leukemia, may result in formation of bi-
exposed population for a particular risk factor.
lateral cataracts. Patients should be monitored
This would be [a / (a + b)] [c / (c + d)] =
closely, particularly those that are school-aged,
19/100 (rounded to the nearest hundredth).
Epidemiology

because they may suffer from visual defects.


Answer C is incorrect. The relative risk would
Answer A is incorrect. Cyclophosphamide is
be 1 if there was no difference between smok-
not implicated in hearing loss; however, cispla-
ers and nonsmokers, which is not the case.
tin and carboplatin have been associated with
Answer E is incorrect. The odds ratio, which this effect. Common adverse effects of cyclo-
is typically calculated for case-control studies phosphamide use include alopecia, infertility,
as (a / c) / (b / d), is 20 / (80 / 99). It is close to nausea, vomiting, leukopenia, and potentially
the relative risk when the prevalence of a dis- fatal acute hemorrhagic cystitis.
ease is very small.
Answer C is incorrect. No evidence suggests
that glucocorticoids cause peripheral neu-
10. The correct answer is A. A confounding vari-
ropathy. Common adverse effects of gluco-
able is a characteristic associated with both the
corticoids include cataracts, cushingoid body
exposure of interest and the disease or condi-
habitus, increased incidence of cardiovascular
tion being studied that may have an indepen-
disease, gastric ulcer, and osteoporosis, among
dent effect on the relationship between the ex-
many others.
posure and disease of interest. In the study,
dietary cholesterol may inuence the risk of Answer D is incorrect. Systemic methotrexate
myocardial infarction, but dietary fat may also is not implicated in the development of cogni-
inuence myocardial infarction risk and thus it tive decits. Common toxicities of methotrex-
is difcult to tell what proportion of decreased ate use include nausea, loose stools, stomati-
risk is due to which dietary factor. tis, alopecia, and rash. Methotrexate can also
cause pulmonary, renal, and hepatic toxicity.
Answer B is incorrect. Enrollment bias occurs
when group assignment of participants is not Answer E is incorrect. Methotrexate is not as-
random, for example, when a sicker set of par- sociated with an increased risk of pulmonary -
ticipants on average is assigned to the experi- brosis. Common toxicities of methotrexate use
mental group than to the control group. include nausea, loose stools, stomatitis, alope-
cia, and rash. Methotrexate can also cause pul-
Answer C is incorrect. Measurement bias re-
monary, renal, and hepatic toxicity.
fers to systematic (whether intentional or unin-
tentional) error in data collection such that the
12. The correct answer is A. Colonoscopy is the
true relationship between the variables under
method of choice in this individual because the
study is distorted.
lesion is most likely right sided, as suggested by
Answer D is incorrect. Recall bias refers to a negative sigmoidoscopy 6 months ago. With
bias introduced by relying on participants colonoscopy, the entire large bowel can be visu-
memory in retrospective studies. For example, alized, and it also allows the biopsy of lesions or
participants who believe a given exposure had removal of polyps found on examination.
HIGH-YIELD SYSTEMS
Chapter 4: Epidemiology Answers 105

Answer B is incorrect. CT of the abdomen is protect against HPV transmission because any
not the diagnostic method of choice in diag- skin-to-skin contact with an infected individual
nosing right-sided colon cancers, although CT may lead to infection.
colonoscopy (high-resolution CT also known
Answer C is incorrect. Women >70 years with
as virtual colonoscopy) is currently being ex-
no abnormal Pap smears in the past 10 years
plored. This is because current resolution
and no new risk factors may elect to cease hav-
may miss some polyps and other lesions. Also,
ing a Pap smear done. However, any patient
colonoscopy allows for biopsy of potential le-
with positive HPV DNA would benet from
sions.
continued screening. Of note, screening does
Answer C is incorrect. Double contrast bar- not need to continue in patients who have a
ium enema can be used to make a radiographic limited life expectancy or who would be un-
diagnosis of colonic cancer if it is not possible able to tolerate cervical cancer therapy.
to perform a colonoscopy (e.g., tortuosity of the
Answer D is incorrect. Women who have had
colon). Colonoscopy is the preferred method.
a hysterectomy without removal of the cervix

Epidemiology
Answer D is incorrect. Sigmoidoscopy would should continue to be screened. In women
not be helpful here because the patient had a with documented cervical intraepithelial neo-
negative sigmoidoscopy 6 months ago, suggest- plasia II/III, a screening should be done re-
ing that the lesion is not on the left side of the gardless of whether the patient had a total hys-
colon. To visualize the right side of the colon, terectomy. The cytologic screening of women
a colonoscopy should be performed. after total hysterectomy should include the
vaginal, vulvar, and perianal epithelium at 4-
Answer E is incorrect. The lesion is most
to 6-month intervals until she has three con-
likely a right-sided colonic adenocarcinoma,
secutive negative results, after which point she
and this area would not be well visualized by
can be screened annually.
an upper gastrointestinal series.
Answer E is incorrect. Current recommenda-
13. The correct answer is B. Current recommen- tions are annual screening if conventional cy-
dations are that a Pap smear be done once ev- tology is used and biannual screening if liquid
ery 13 years based on risk factors and cytology cytology is used in healthy patients without risk
method. In 2003 the American College of Ob- factors or prior disease. The American Cancer
stetricians and Gynecologists recommended Society and American College of Obstetricians
that women younger than 30 years be screened and Gynecologists support the use of liquid-
annually, regardless of liquid or conventional based cytology because it is more sensitive and
culture method. However, the US Preventative increases the number of adequate Pap smear
Services Task Force recommends that women readings, while reducing the false-negative rate
of any age extend time between screening to among patients with HIV and chronic cervi-
every 23 years if they have no risk factors and citis, obscuring blood and repetitive atypical
have had three or more consecutive negative squamous cells of undetermined signicance
cytologic studies by conventional method. Risk (or ASCUS) Pap reports.
factors include diethylstilbestrol exposure in
utero, prior cervical intraepithelial neoplasia 14. The correct answer is E. Lung cancer is not
II/III or abnormal Pap smear, smoking, or im- hereditary but is based on environmental expo-
munocompromised state. A gonorrhea and sures to radon, asbestos, or cigarette smoke.
chlamydia screen may be recommended if the There is evidence to suggest that a patients risk
patient is sexually active. of developing lung cancer from tobacco expo-
sure decreases somewhat after the patient quits
Answer A is incorrect. All women should have
smoking, depending on the patients smoking
a Pap once they are determined to be HIV pos-
history and medical status. The patient should
itive because immunocompromised states are
be counseled about smoking cessation, and ap-
a risk factor for HPV. Of note, condoms do not
propriate adjustments to medications should
HIGH-YIELD SYSTEMS
106 Section I: Organ Systems Answers

be made if the patient is started on nicotine years, a microalbuminuria will help assess kid-
gum or patch. In 2004 the U.S. Preventive Ser- ney function. This should be checked annually.
vices Task Force determined that the evidence Current studies have shown that an angiotensin-
is insufcient to recommend for or against converting enzyme (ACE) inhibitor is bene-
screening asymptomatic persons for lung can- cial in reversing or delaying renal impairment
cer with either low dose computed tomogra- in diabetics who have no renal artery stenosis.
phy, plain chest radiographs, sputum cytology,
Answer D is incorrect. Approximately 30%
or a combination of these tests.
40% of patients with type 1 diabetes mellitus
Answer A is incorrect. X-rays of the chest are not will eventually develop loss of vision, so it is
recommended as a screening measure. X-rays important to screen these patients annually to
of the chest or CT scans of the lungs are neces- assess for vision loss and possible retinal tear.
sary in the diagnosis of lung cancer and should
Answer E is incorrect. A podiatry examination
be pursued once the index of suspicion for lung
is important for screening associated diabetic
cancer increases.
neuropathies. Patients with type 1 diabetes
Epidemiology

Answer B is incorrect. Studies have shown mellitus are also at increased risk for peripheral
low-dose spiral CT of the chest screening of vascular disease, so peripheral pulses should
asymptomatic patients with at least a 10-pack- be checked often. These screens should be
year smoking history found more malignant checked at the patients annual visit unless he
nodules than have plain lms (2.7% vs. 0.7% is developing signs or symptoms.
in one study). However, many of the detected
Answer F is incorrect. This indicates the pa-
nodules were ultimately found to be benign
tients glycemic control over the previous 2
(20.6%), and no survival benet of this screen-
weeks. This test would be helpful in the case
ing has been proven.
of abnormal hemoglobins or to determine gly-
Answer C is incorrect. Studies show lung cemic control at time of conception in women
cancer screening with sputum cytology can with diabetes. This test is not necessary on a
detect cancer at an earlier stage than in the regular basis.
unscreened population but had no effect on
mortality. 16. The correct answer is A. This answer describes
sensitivity. A highly sensitive test will rarely
Answer D is incorrect. Bronchoscopy and/
miss people with the disease, and is thus a good
or ne-needle aspiration may be necessary for
rule-out if negative. It is calculated mathe-
further diagnosis of suspected lung nodules but
matically by dividing the number of those with
have no role in screening.
disease (or in this case, condition of interest)
and a positive test by the total number of peo-
15. The correct answer is A. A hemoglobin A1c
ple with disease (or condition).
level should be checked every 3 months to as-
sess the average serum glucose levels over that Answer B is incorrect. This answer describes
time period. This is important in adjusting med- specicity. There is necessarily a trade-off be-
ications and to follow the disease progression. tween sensitivity and specicity. A highly spe-
cic test will rarely give a false-positive result
Answer B is incorrect. Patients with types 1 or
(i.e., it will rarely determine that someone has
2 diabetes mellitus are at increased risk for hy-
the disease if he or she does not). A test with
perlipidemia and should be screened on an an-
the high sensitivity desired by the manufac-
nual basis. Patients with type 1 diabetes mellitus
turer would not also have a high specicity.
should be screened during puberty if they have
a positive family history of coronary artery dis- Answer C is incorrect. This answer describes
ease in order to prevent silent ischemic damage. positive predictive value. While a high sensitiv-
ity may lead to a high positive predictive value,
Answer C is incorrect. Approximately 30%
the latter is dependent on disease prevalence
40% of patients with type 1 diabetes mellitus
and thus it is more accurate to speak in terms
will develop renal failure. In patients >10 years
of sensitivity.
old who have had diabetes for more than 3
HIGH-YIELD SYSTEMS
Chapter 4: Epidemiology Answers 107

Answer D is incorrect. This answer describes Answer A is incorrect. Since the patient was
negative predictive value, which often results diagnosed with colon cancer at the age of 57
from a high specicity, not sensitivity. years, the son should be screened at the age of
40, since that is >10 years younger than his fa-
17. The correct answer is B. By lowering the cut-off thers diagnosis. Furthermore, every 12 years
value, the number of people who will be diag- is too frequent for screening.
nosed as having prostate cancer will increase.
Answer C is incorrect. Screening colonoscopy
This will decrease the number of false-negatives
beginning at age 50 years is recommended for
(those who are told they do not have the disease
average-risk individuals and should be repeated
when they in fact do), and increase the number
every 10 years.
of false-positives (those told they have the dis-
ease when they in fact do not). Answer D is incorrect. Since the patient was
diagnosed with colon cancer at the age of 57
Answer A is incorrect. False-positives and false-
years, the son should be screened at the age of
negatives could both decrease if the test were
40, since that is >10 years younger than his fa-

Epidemiology
made more valid, perhaps through averaging
thers diagnosis.
the results from several samples taken on dif-
ferent days, but this question asks what would Answer E is incorrect. Fecal occult blood test-
happen if the cut-off value were lowered. ing beginning at age 50 years is recommended
for average-risk individuals and should be re-
Answer C is incorrect. False-positives and
peated every year.
false-negatives could both rise if the test were
made less precise, perhaps through laboratory
19. The correct answer is F. The odds ratio can be
errors, but this question asks what will happen
calculated as (number exposed who have dis-
if a cut-off value is lowered.
ease/number not exposed who have disease)/
Answer D is incorrect. The number of false- (number exposed without disease/number not
positives would decrease and false-negatives in- exposed without disease), which in this sce-
crease if the cut-off value were raised, but the nario yields (150/100)/(50/200) = 6. Odds ratios
cut-off value was lowered. are used in studies employing a case-control de-
sign (typically retrospective) and can be used as
Answer E is incorrect. This is not correct, be-
an approximation of the relative risk only when
cause lowering the cut-off value will decrease
the disease being studied is very rare in the over-
the number of false-negatives and increase the
all population from which the controls are
number of false-positives.
drawn.
18. The correct answer is B. The patient in the Answer A is incorrect. Reversing the relation-
question has colon cancer. His presentation ship of the variables by mistake yields 1/6.
and blood tests are consistent with microcytic
Answer B is incorrect. Incorrect assignment of
anemia. The colonoscopy reveals an exophytic
the variables to cells yields this erroneous an-
mass in the colon consistent with adenocarci-
swer.
noma. The current American Gastroenterolog-
ical Association guidelines state that people Answer C is incorrect. Erroneously calculat-
with rst-degree relatives diagnosed with colon ing the relative risk for this retrospective case-
cancer or adenomatous polyps diagnosed at control study yields 1.7. Relative risk is a statis-
<60 years of age should be screened with a tical measure of the inuence of an exposure
colonoscopy every 5 years beginning at age 40, on the risk of a disease. It is used in studies em-
or 10 years younger than the earliest diagnosis ploying a cohort design (typically prospective).
in the family, whichever comes rst. Since the
Answer D is incorrect. Incorrect assignment
patient was diagnosed with colon cancer at the
of the variables to cells yields this erroneous
age of 57 years, the son should be screened at
answer.
the age of 40 since that is >10 years younger
than his fathers diagnosis.
HIGH-YIELD SYSTEMS
108 Section I: Organ Systems Answers

Answer E is incorrect. Incorrect assignment of ships, although these can be estimated with
the variables to cells yields this erroneous an- chi-square analysis.
swer.
Answer A is incorrect. Case-control studies in-
volve identifying two populations: one of cases,
20. The correct answer is D. Varicella develops in
which are persons with the disease or condi-
approximately 90% of susceptible household
tion of interest, and another population of con-
contacts with signicant exposure. The vac-
trols, which are individuals similar to the case
cine, if given in the rst 3 days postexposure,
subjects in relevant parameters other than the
has been shown to prevent or modify the
disease or condition under study. Potential ex-
course of illness in susceptible individuals. Sev-
posures or etiologic factors are then examined
enty-eight percent of adults seroconvert after
to determine if there is a common denomina-
the rst dose; a cumulative 99% have con-
tor in the background of the cases that the con-
verted after the second dose.
trols tend not to share.
Answer A is incorrect. In an otherwise healthy
Answer B is incorrect. Cohort studies involve
Epidemiology

child, treatment with oral acyclovir may be in-


identifying a group of subjects, or cohort, as
dicated under special circumstances (such as
well as a matched control group, that are fol-
in the case of a child with susceptible house-
lowed for a period of time to assess whether the
hold contacts at increased risk of serious dis-
disease or condition of interest develops.
ease), but treatment with intravenous acyclovir
is appropriate only in cases of severe infection Answer D is incorrect. Using existing indi-
or immunocompromised hosts. vidual studies, but combining their data statis-
tically to maximize power and minimize study
Answer B is incorrect. Postexposure oral acy-
limitations, is known as a metaanalysis.
clovir may be appropriate as prophylaxis in-
stead of varicella zoster immunoglobulin or Answer E is incorrect. Randomized controlled
vaccine, but there are insufcient data to sup- clinical trial is a study in which subjects are as-
port routine use. Oral acyclovir, if adminis- signed to either control or treatment groups and
tered within the rst 3 days of infection, has then treatment groups are exposed to some fac-
been shown to lessen the duration and sever- tor that control groups do not experience. Out-
ity of disease in healthy children, but is only comes are measured in both groups to deter-
recommended in cases involving household mine the effects of the exposure or treatment.
contacts who are at increased risk for severe
disease (e.g., immunosuppression, extremes of 22. The correct answer is E. The researcher is
age, and pregnant women). conducting a randomized controlled clinical
trial. Patients have been assigned randomly to
Answer C is incorrect. Administration of vari-
minimize bias. In this instance, the control
cella zoster immunoglobulin to adults after ex-
group is not receiving placebo; this does not
posure is indicated for pregnant women, health
mean that this study is not a randomized con-
care professionals with signicant exposure,
trolled clinical trial. For diseases and condi-
and immunocompromised individuals.
tions where the patient would be unduly
Answer E is incorrect. Because individuals harmed by remaining untreated for the study
>13 years old are at risk for more serious dis- duration, it is customary to compare the study
ease, prophylaxis with varicella vaccine is indi- drug or procedure to the current standard of
cated and safe for the older sibling. care for that disease or condition and not to
placebo. In this scenario, the study is also dou-
21. The correct answer is C. Cross-sectional study ble blinded because neither the study staff nor
involves assessing variables at one given point patients know who receives what treatment.
in time. Exposure and outcome are measured
Answer A is incorrect. A case-control study
simultaneously. It is useful for estimating dis-
involves matching each person in a study with
ease prevalence and distribution within a pop-
another patient who is similar in all disease-
ulation, but cannot establish causal relation-
relevant aspects aside from the factor under
HIGH-YIELD SYSTEMS
Chapter 4: Epidemiology Answers 109

study. These studies are not randomized and Answer D is incorrect. The diaphragm has an
thus are subject to a variety of biases; however, actual failure rate of 12%18% and thus would
they are useful for initial hypothesis testing. not be a good option for this patient. The dia-
phragm should be used in patients who are un-
Answer B is incorrect. A cohort study involves
able to tolerate more effective surgical or medi-
following a group of patients and a group of
cal contraception.
controls over time to see whether a given dis-
ease or condition aficts one group dispropor- Answer E is incorrect. An intrauterine device
tionately. (IUD) would be a good contraceptive option
for this patient, given that she is monogamous
Answer C is incorrect. A cross-sectional survey
with her husband and does not have a history
is an assessment of a given population where
of sexually transmitted diseases. A copper IUD
information on the disease or condition of in-
could be placed at an ofce visit, and would
terest and possible past risk factors or events is
last for approximately 10 years (progesterone
collected simultaneously.
IUDs last for approximately 5 years). However,

Epidemiology
Answer D is incorrect. A meta-analysis, or lit- failure rates for these devices are higher (0.8%
erature review, is the mathematical combina- actual failure rate for the copper IUD and
tion of prior studies and does not involve the 2.0% for the progesterone IUD) than those for
collection of new data. sterilization, making this option less effective.

23. The correct answer is B. Bilateral male steril- Answer F is incorrect. The male condom has
ization has theoretical and actual failure rates a 14% actual failure rate. The benet to the
of 0.15% and 0.1%, respectively, the lowest fail- male condom over other forms of contracep-
ure rates of any contraceptive option other tion is its protection from sexually transmitted
than complete abstinence. Unlike nonsurgical diseases; the male condom is thus particularly
contraceptive options, however, sterilization is useful in high-risk populations and would not
a permanent intervention and thus must be be necessary in this couple.
preceded by sufcient patient counseling. This Answer G is incorrect. Both the combination
couple states that they do not desire any fur- and the progestin-only oral contraceptive pill
ther children, and thus while other contracep- have actual failure rates of 5% and thus are less
tive options could be prescribed, male steriliza- effective than the more invasive options.
tion remains the most effective option for
them. 24. The correct answer is B. A blinded or sin-
gle-blinded study is one in which the partici-
Answer A is incorrect. Female sterilization
pant does not know whether he or she is an ex-
has failure rates only slightly higher than male
perimental subject or a control subject.
sterilization, with both theoretical and actual
Double blinded means that neither the par-
failure rates of 0.5%. Like male sterilization,
ticipants nor the researchers know who is as-
this option is also permanent and must be per-
signed to which group.
formed only after sufcient patient counsel-
ing. Additionally, female sterilization in this Answer A is incorrect. If neither the study par-
couple is less desirable, as surgical interven- ticipants nor the researchers know the group
tions should be minimized in a patient with a assignments, the study is double blinded.
previous history of abdominal surgery, as she is
Answer C is incorrect. Single blinding refers
already at increased risk for adhesions.
to blinding study participants. Blinding re-
Answer C is incorrect. Both the combination searchers would limit observational bias but
and the progestin-only oral contraceptive pill would not limit reporting bias on the part of
have actual failure rates of 5% and thus are less the participants if they know whether they are
effective than the more invasive options. How- the experimental group or the control group.
ever, if this patient were symptomatic from Sometimes it is not possible to double-blind
her endometriosis, she could be started on the a study; for example, a study comparing out-
combination pill for symptom relief. comes of two surgical procedures could not
HIGH-YIELD SYSTEMS
110 Section I: Organ Systems Answers

blind the surgeons, who would have to know to know which is responsible for the observed
which procedure they were to perform on difference in vascular dementia incidence.
which participant. What can be denitively stated is that, among
the study population, smokers are more likely
Answer D is incorrect. Phase II and III drug
to be drinkers than nonsmokers.
trials assess drug efcacy, but this is a separate
issue from blinding. Answer A is incorrect. Smoking and alcohol
have not been evaluated independently in this
Answer E is incorrect. Phase I, II, and III drug
study.
trials assess safety of drugs under study, but this
is a separate study characteristic from blinding. Answer B is incorrect. Alcohol may be the
Phase II and III studies focus on efcacy of cause for the increase in vascular dementia,
drugs shown to be safe in phase I studies. but unless the smoking disparity is controlled
for by study design or multivariate analysis, this
25. The correct answer is C. This patient is suffering statement cannot be conclusively made.
from a supercial thrombosis, which does not re-
Epidemiology

Answer D is incorrect. Smoking may be the


quire anticoagulation. This is a non-life-threaten-
cause for the increase in vascular dementia,
ing condition and it does not increase the risk of
but unless the disparity in alcohol consump-
pulmonary embolus. First-line treatment is warm
tion is controlled for by study design or multi-
compresses applied so the clot does not extend to
variate analysis, this statement cannot be con-
the deep veins. Treatment should last for 57
clusively made.
days.
Answer E is incorrect. Although the observed
Answer A is incorrect. Anticoagulation is not
result may be due to smoking, with no effect
necessary for supercial vein thrombosis. A
from alcohol consumption, this statement can-
deep vein thrombosis would have shown de-
not be conclusively made. The presence of
creased Doppler ow and lack of compressibil-
the confounding variable, in this case, alcohol
ity on ultrasonography.
consumption, precludes a denitive answer as
Answer B is incorrect. Warfarin may cause to which variable accounts for the increased
birth defects such as stippling of bone epiphy- incidence of vascular dementia.
ses, intrauterine growth restriction, nasal hypo-
plasia, and mental retardation. The sensitive 27. The correct answer is C. The type of study de-
period for birth defects due to warfarin is in scribed is known as a meta-analysis. Combin-
the rst trimester, but nevertheless it should ing results from several studies enhances statis-
be avoided in this case since anticoagulation is tical precision and may allow a statistically
contraindicated. signicant result from the combined analysis
that did not reach a signicant level when each
Answer D is incorrect. Ibuprofen is a non-
smaller study was analyzed alone.
steroidal anti-inammatory drug with analgesic
properties. These drugs may be used as therapy Answer A is incorrect. Meta-analyses are lim-
for a supercial vein thrombosis in conjunc- ited by the quality of individual studies and do
tion with warm compresses and elevation. not resolve aws in those studies, although the
When used in the rst two trimesters of preg- impact of any single awed study may be less-
nancy, ibuprofen has not been linked to any ened.
congenital defects. During the third trimester
Answer B is incorrect. Performing a meta-
of pregnancy, however, it has been linked to
analysis does not remove or decrease the bias
persistent pulmonary hypertension of the new-
inherent in each study, and may have a com-
born. In addition, it has tocolytic properties.
pounding effect of each studys individual bias.
26. The correct answer is C. Without controlling Answer D is incorrect. Since there is usually
for the difference in alcohol consumption be- signicant variation in the populations, meth-
tween smokers and nonsmokers, it is difcult ods, and interventions used in separately con-
ducted studies, researchers must strike a dif-
HIGH-YIELD SYSTEMS
Chapter 4: Epidemiology Answers 111

cult balance between including the greatest growth restriction. A biophysical prole is a
number of studies and excluding those whose combination of a real-time ultrasound used to
differences would cloud the overall validity of assess four parameters of fetal well-being in ad-
the analysis. dition to a nonstress test. The four ultrasound
parameters are fetal tone, breathing, movement,
Answer E is incorrect. Statistical analysis of
and amniotic uid volume, each with a 2-point
multiple studies is inherently more complex
maximum. A reassuring biophysical prole
than analysis of a single study.
has a score of 810. A score of 02, however, is
28. The correct answer is G. Ultrasound may be grounds for immediate delivery.
done at 1820 weeks gestation to determine Answer D is incorrect. Chorionic villus sam-
gestational age if it cannot be determined by pling is performed at 1012 weeks of gestation
last menstrual period. It may also be done to for an earlier assessment of chromosomal ab-
survey fetal anatomy, amniotic uid volume, normalities, with diagnostic accuracy compa-
and placental location. As she gave a history rable to that of amniocentesis.

Epidemiology
signicant for possible birth defects, ultrasound
Answer E is incorrect. A nonstress test (NST)
is likely to be helpful in this patient. She would
is a measurement of the fetal heart rate by ex-
also benet from a visit to a genetic counselor
ternal Doppler in correlation with spontaneous
in addition to the ultrasound.
fetal movement as reported by the mother. A
Answer A is incorrect. Maternal serum normal response is heart rate accelerations of
-fetoprotein (MSAFP) is already part of the >15/min over baseline for at least 15 seconds.
triple screen test. It is used to screen for neu- A normal NST shows two such accelerations
ral tube defects (including anencephaly and in a 20-minute period. An NST is performed
spina bida), abdominal wall defects (includ- in two clinical scenarios. It may be performed
ing gastroschisis and omphalocele), and is el- at 3234 weeks gestation in high-risk patients
evated in multiple gestation, fetal death, and as described above, or in pregnant patients
chromosomal defects (trisomy 13, 18, and 21). presenting in acute distress as part of the triage
The triple screen test of -human chorionic evaluation of fetal well-being. An NST can be
gonadotropin, estriol, and MSAFP provides in- done as early as 28 weeks gestation since 85%
creased sensitivity for chromosomal abnormali- of fetuses will be reactive at this time.
ties compared to MSAFP alone.
Answer F is incorrect. Percutaneous umbili-
Answer B is incorrect. In general, amniocen- cal blood sampling is performed by puncturing
tesis is indicated in conjunction with a positive the umbilical cord vessels, which are not large
triple screen in patients who are 35 years old or enough for the procedure until the late second/
older at the time of delivery. The risk of chro- early third trimesters. This technique is useful
mosomal abnormality in children of women in the evaluation of fetal anemia that occurs in
over the age of 35 is 1 in 200. Amniocentesis erythroblastosis fetalis late in pregnancy.
carries a similar risk of about 1 in 200 for fetal
loss and so is usually considered too risky for 29. The correct answer is B. The recommenda-
a 26-year-old patient. This patient is <35 years tions for mammography are screening every
old, but has a history of possible chromosomal year when the patient is 50 years old or older.
abnormalities, which may make her a candi-
Answer A is incorrect. Endometrial biopsy is
date for amniocentesis. Amniocentesis would
not recommended as a screening test. An en-
still follow ultrasound evaluation and the re-
dometrial biopsy would be performed on a
sults of her triple screen, making ultrasound
patient presenting with postmenopausal bleed-
evaluation the correct answer.
ing.
Answer C is incorrect. A biophysical prole is
Answer C is incorrect. Cancer antigen-125
performed in the third trimester of a high-risk
(CA-125) is a serum tumor marker associated
pregnancy. This would include patients who are
with ovarian, lung, breast, uterine tube, and
post-dates, have diabetes, experience decreased
gastrointestinal cancers. Once proposed as a
fetal movement, or with known intrauterine
HIGH-YIELD SYSTEMS
112 Section I: Organ Systems Answers

possible screening tool for ovarian cancer, it 31. The correct answer is D. The patient is experi-
has since been determined to be ineffective in encing severe acute graft-versus-host disease
detecting ovarian carcinoma in asymptomatic (GVHD), caused by allogeneic T lymphocytes
women. CA-125 levels may be elevated in non- that were transferred with the donors stem
cancerous states such as infection (e.g., perito- cells reacting with antigenic targets on her own
nitis and pleuritis), menstruation, pregnancy, cells. Acute GVHD is usually experienced
endometriosis, and liver disease. CA-125 test- within 4 weeks of the initial transplant. It is
ing would not therefore be appropriate for this characterized by rash, diarrhea, and decreased
patient. liver function, and is graded from I through IV.
Signicant GVHD develops in up to 60% of
Answer D is incorrect. Transvaginal ultra-
patients receiving stem cells from unrelated
sound is used to screen patients who are at
donors, and 30% of those receiving transplants
high risk of ovarian cancer.
from siblings. It is usually treated with gluco-
Answer E is incorrect. In spite of this patients corticoids, antithymocyte globulin, or mono-
smoking history, x-ray of the chest is not rec- clonal antibodies targeted against T lympho-
Epidemiology

ommended as a screening test. cytes.

30. The correct answer is E. Trimethoprim-sulfame- Answer A is incorrect. Hepatic transplantation


thoxazole is indicated as prophylactic treatment might be considered if the patients overall con-
against Pneumocystis jiroveci pneumonia in im- dition improved and lasting effects of GVHD
munosuppressed patients for the duration of im- left her with impaired hepatic function. How-
munosuppression. ever, a recent bone marrow transplant patient
would not be considered a candidate for he-
Answer A is incorrect. Raw oysters have been patic transplantation.
associated with various viral and bacterial in-
fections. Hepatitis A virus is the likely cause of Answer B is incorrect. Although removal of
this patients fulminant hepatitis. This cause of T lymphocytes from the stem cell inoculum
liver failure does not necessarily increase the has been proposed as a method of preventing
patients risk for posttransplant viral infections. GVHD, it is associated with increased inci-
dence of graft failure, as well as increased risk
Therefore, without history of another underly-
of tumor recurrence. No evidence exists sug-
ing disease, prior cytomegalovirus infection,
gesting that this method should be instituted.
prior HIV infection or risks, or underlying mal-
nourished state, this middle-age man is a low- Answer C is incorrect. Total body irradiation
risk posttransplant patient. Antiviral therapy may be performed prior to the transplant. It is
is not indicated for prophylaxis in low-risk pa- not indicated as therapy for GVHD, as it would
tients but would be indicated for certain high- most likely be fatal, and at the very least would
risk posttransplant patients and treatment of vi- leave the patient requiring a second transplant.
ral infections in posttransplant patients.
Answer E is incorrect. Thalidomide is an ex-
Answer B is incorrect. Antifungal therapy is perimental therapy for GVHD. It is not indi-
not indicated for prophylaxis in this case. An cated as therapy for acute GVHD.
indication for antifungal therapy in transplant
patients would be candidiasis. 32. The correct answer is C. In a case-control
study, an odds ratio is used to compare the ef-
Answer C is incorrect. Niacin is generally not
fects of exposure between two groups. An odds
used as a prophylactic drug and has been as-
ratio is a measure of the odds that a person
sociated with hepatotoxicity. Therefore, niacin
with a specic condition has been exposed to a
would not be indicated.
certain risk factor divided by the odds that the
Answer D is incorrect. Nystatin is used to treat control group has been exposed. To calculate
oral infection by Candida albicans. Although the odds ratio, take the number exposed to the
this condition may arise in patients who are disease divided by the number not exposed to
immunosuppressed, prophylaxis is generally the disease, divided by the number exposed
not indicated. without the disease divided by the number not
HIGH-YIELD SYSTEMS
Chapter 4: Epidemiology Answers 113

exposed without the disease. In this case, this Answer C is incorrect. Unfortunately, surveil-
equates to (100 / 50) / (200 / 300), which lance studies such as sputum cytology repeated
equals 3. at intervals of 46 months have not been
shown to improve early detection or prolong
Answer A is incorrect. Although the statement
lung cancer survival in high-risk individuals.
choice is correct in regard to its being a case-
The most appropriate intervention is smoking
control study, 0.33 is not the correct calcula-
cessation.
tion of the odds ratio.
Answer D is incorrect. Unfortunately, surveil-
Answer B is incorrect. Although the statement
lance studies such as x-ray of the chest repeated
choice is correct in regard to its being a case-
at intervals of 46 months have not been
control study, 1.5 is not the correct calculation
shown to improve early detection or prolong
of the odds ratio.
lung cancer survival in high-risk individuals.
Answer D is incorrect. Relative risk and attrib- The most appropriate intervention is smoking
utable risk are used in cohort studies. In case- cessation.

Epidemiology
control studies, the odds ratio is calculated to
compare the two groups. 34. The correct answer is C. The developmental
milestones this infant has reached suggest that
Answer E is incorrect. Relative risk and attrib-
the child is 2 months old. A mild illness and
utable risk are used in cohort studies. In case-
low-grade fever are not contraindications to re-
control studies, the odds ratio is calculated to
ceiving immunizations. The standard immuni-
compare the two groups.
zations given at the 2-month visit are hepatitis
33. The correct answer is E. Squamous cell carci- B (Hep B) vaccine, diphtheria-tetanus-pertussis
noma and adenocarcinoma are the most com- (DTaP) vaccine, Haemophilus inuenzae type
mon cancers associated with asbestos exposure, b (HiB) vaccine, inactivated poliomyelitis vac-
which is associated with a sixfold increase in cine (IPV), and pneumococcal conjugate vac-
lung cancer. This risk increases to 59-fold with cine (PCV). The Hep B vaccine is a hepatitis
a concurrent smoking history. Therefore, the B surface antigen. The DTaP vaccine is a com-
most appropriate intervention in this patient is bined vaccine against diphtheria, tetanus, and
smoking cessation. The average time between pertussis. The diphtheria and tetanus portions
the exposure to asbestos and the development are toxoids, whereas the pertussis is an acellu-
lar pertussis component (DTaP vs. DTP). The
of lung cancer is 15 years. The patient has
HiB vaccine contains killed portions of the
signs and symptoms of asbestosis (shortness of
H. inuenzae type B bacterium. IPV contains
breath, restrictive pattern on pulmonary func-
three forms of inactivated polio virus. PCV is
tion testing, and a chest radiograph showing
the pneumococcal conjugate vaccine contain-
linear opacities and pleural plaques). Specic
ing seven strains of Streptococcus pneumoniae.
therapy, such as steroids or immunosuppressive
or antibrotic agents, may also be warranted. Answer A is incorrect. There is no time when
the child would just receive the Hep B and
Answer A is incorrect. To date, no randomized
DTaP vaccines.
trial has conrmed the benet of lung cancer
screening using high-resolution CT to reduce Answer B is incorrect. There is no time when
lung cancer mortality. the child would just receive the Hep B, DTaP,
and IPV vaccines.
Answer B is incorrect. Pulmonary function test-
ing in patients with asbestosis shows a restrictive Answer D is incorrect. A child would only
pattern with a decrease in lung volumes (de- receive the Hep B vaccine at birth and at 1
creased FEV1; normal or increased FEV1:FVC month.
ratio; and decreased FVC, vital capacity, and to-
Answer E is incorrect. A mild illness and low-
tal lung capacity). Pulmonary function tests play
grade fever are not contraindications to receiv-
no role in lung cancer screening. They are war-
ing immunizations.
ranted in patients with lung cancer when surgi-
cal resection is considered.
HIGH-YIELD SYSTEMS
114 Section I: Organ Systems Answers

35. The correct answer is C. Hepatitis B vaccine positive rate of approximately 2%, the results
is the only childhood vaccine that is to be need to be conrmed with a Western blot anal-
given upon delivery. This vaccine is given ysis.
again when the infant is 2 months old.
Answer B is incorrect. If the enzyme-linked
Answer A is incorrect. Diphtheria, pertussis, immunosorbent assay test is positive, it will
and tetanus vaccine is given to children when likely be positive again, regardless of whether
they are 2, 4, 6, and 1518 months old, and the positive is false or true. The diagno-
again when they are 46 years old. sis should be conrmed by an independent
method.
Answer B is incorrect. Haemophilus inuen-
zae B vaccines are given to children when they Answer C is incorrect. Clinical evidence of
are 2, 4, 6, and 1215 months old. an opportunistic infection would support the
diagnosis, but is not used as part of the deni-
Answer D is incorrect. Inactivated polio vac-
tion of HIV status. Opportunistic infections are
cine is given when children are 2, 4, and 6
part of the denition of AIDS, but may also be
Epidemiology

months old, and again when they are 46 years


indicative of immunosuppression from another
old.
source.
Answer E is incorrect. Measles, mumps, and
Answer D is incorrect. While unprotected sex
rubella vaccine is given to children when they
is a signicant risk factor for HIV infection, this
are 1215 months and 46 years old.
history alone is not diagnostic of active HIV in-
Answer F is incorrect. Varicella vaccine is fection in the patient.
given when the child is 1215 months old.
Answer E is incorrect. While a patient with
36. The correct answer is D. The National Cho- an untreated HIV infection will develop a low
lesterol Education Program guidelines recom- CD4+ cell count, a number of other condi-
mend that those with low risk for cardiac dis- tions may lead to generalized lymphopenia, in-
ease (01 risk factor) maintain an LDL cluding congenital immunodeciencies, bone
cholesterol level <160 mg/dL, but pharmaco- marrow disorders, and hematologic malignan-
logic treatment is recommended only if LDL cies.
cholesterol levels go above 190 mg/dL. An
38. The correct answer is D. Many studies have
LDL cholesterol level between 160 and 190
found that the greatest risk factor for a com-
mg/dL can be treated with therapeutic lifestyle
pleted suicide attempt in any patient is whether
changes. Risk factors include older age, heart
they have attempted suicide in the past. In fact,
disease, diabetes, hypertension, and smoking.
some research shows that past attempts may in-
Answer A is incorrect. This level of LDL cho- crease the risk of suicide by 22 times. It is also
lesterol does not require pharmacologic ther- important to assess whether the patient has a
apy. plan for suicide, and whether the suicide at-
tempt was by lethal means.
Answer B is incorrect. This level of LDL cho-
lesterol does not require pharmacologic ther- Answer A is incorrect. Antisocial behavior in-
apy. creases the risk of suicide, but signicantly less
so than previous suicide attempts.
Answer C is incorrect. This is the recom-
mended level above which therapeutic life Answer B is incorrect. Depression increases
style changes are recommended. suicide more than antisocial behavior, but sig-
nicantly less than previous suicide attempts.
Answer E is incorrect. This level is too high to
be ideal. Answer C is incorrect. Male gender is a risk
factor for completed suicide, but is not as sig-
37. The correct answer is A. Because the enzyme- nicant a risk factor as previous attempts.
linked immunosorbent assay test has a false-
HIGH-YIELD SYSTEMS
Chapter 4: Epidemiology Answers 115

Answer E is incorrect. Substance abuse is a Answer B is incorrect. Selective -blockers an-


risk factor for violence and suicide, but it is not tagonize 1-adrenergic receptors, thus blocking
as signicant as previous suicide attempts. Sub- the positive chronotropic and inotropic effects
stance abuse may increase the risk of suicide of endogenous catecholamines.
by a factor of eight.
Answer C is incorrect. Nonselective -blockers
antagonize 1- and 2-adrenergic receptors.
Questions 39 and 40 The former are located mainly in the heart and
kidneys; the latter in the lungs, gastrointestinal
39. The correct answer is D. First-line treatment tract, liver, uterus, vascular smooth muscle,
for hypertension in diabetics with proteinuria is and skeletal muscle.
an ACE inhibitor such as captopril and enal-
april. ACE inhibitors decrease mortality and Answer E is incorrect. Calcium channel
the risk for stroke in this patient population. blockers inhibit calcium inow into vascular
ACE inhibitors also slow the progression of re- smooth muscle and myocardium, thus exerting
a negative inotropic effect.

Epidemiology
nal disease in diabetic hypertensive patients
with proteinuria. ACE inhibitors act by block- Answer G is incorrect. A loop diuretic blocks
ing ACE, the enzyme necessary for converting sodium and chloride reabsorption in the loop
angiotensin I to angiotensin II. Angiotensin II of Henle. These diuretics increase calcium ex-
causes increased aldosterone production, in- cretion. Loop diuretics do not have the mortal-
creased retention of sodium and water, in- ity benet of ACE inhibitors in patients with
creased output of the sympathetic nervous sys- diabetes, nor do they slow the progression of
tem, and vasoconstriction of vascular smooth renal disease in the instance of proteinuria.
muscle, all leading to increased blood pressure.
Answer H is incorrect. Hydralazine directly
Therefore, if angiotensin II synthesis is
relaxes smooth muscle via a cGMP-mediated
blocked, this leads to a decrease in blood pres-
mechanism, thus lowering blood pressure.
sure.
However, the ensuing baroreceptor reex tends
40. The correct answer is F. First-line treatment to lead to sympathetic stimulation of the heart.
for hypertension in patients with osteoporosis is Answer I is incorrect. Centrally acting adren-
a thiazide diuretic such as hydrochlorothia- ergic agonists stimulate 2-adrenergic recep-
zide. Thiazide diuretics act by inhibiting a so- tors. Stimulating these presynaptic receptors
dium/chloride cotransporter in the distal tu- decreases norepinephrine release. Clonidine
bule, thereby decreasing sodium reabsorption, and methyldopa are examples.
which leads to decreased blood pressure. Thi-
Answer J is incorrect. ACE inhibitors block,
azide diuretics increase the secretion of sodium
not stimulate, aldosterone formation.
and potassium and the reabsorption of cal-
cium. Therefore, patients with hypocalcemia, Answer K is incorrect. ACE inhibitors block,
hypercalciuria, and osteoporosis may benet not stimulate, angiotensin II formation. Stimu-
from thiazide diuretics in terms of calcium bal- lation of angiotensin II formation would lead
ance. to increased effects of the oligopeptide, includ-
ing sympathetic activity, tubular sodium and
Answer A is incorrect. Spironolactone com-
chloride reabsorption leading to increased wa-
petitively inhibits aldosterones effect in the
ter retention, aldosterone secretion, arteriolar
collecting duct. Thus, it decreases potassium
vasoconstriction, and ADH secretion.
excretion. Spironolactone has a signicant
mortality and morbidity benet for patients
with severe New York Heart Association class
III or IV congestive heart failure.
This page intentionally left blank
CHAPTER 5

Ethics
and Legal Issues

117
HIGH-YIELD SYSTEMS
118 Section I: Organ Systems Questions

Q U E ST I O N S

1. A 65-year-old man presents to the physicians been in contact with the medical team only by
ofce with his daughter, who is a resident in phone, arrives stating that she has power of at-
internal medicine. She encouraged her father torney and asking that her mother be taken off
to come in because he has experienced a 4.5- the ventilator. Which of the following is the
kg (10-lb) weight loss in 2 months, as well as best course of action?
decreased appetite and occasional night sweats.
(A) Discuss with the daughter her reasons for
She thinks his eyes appear somewhat icteric.
withdrawing care
After the physician conducts the interview, the
(B) Maintain current management based on
physician and the patients daughter leave so
the patients prognosis
the patient can undress for the examination.
Ethics and Legal Issues

(C) Obtain a court order mandating continua-


Once in the hallway, the daughter says to the
tion of ventilatory support
physician in a low voice, I am really afraid
(D) Obtain an ethics consultation
that he might have pancreatic cancer, and he
(E) Withdraw ventilatory support based on the
would be devastated to nd out. Can you
daughters power of attorney
please discuss his test results with me rst so
that we can decide what to tell him together?
3. A 45-year-old woman is scheduled to undergo
Which of the following is the physicians most
elective bilateral tubal ligation. The gynecol-
appropriate response?
ogy resident, covering for a colleague, greets
(A) Dont worry, I doubt that he has pancre- the patient in the preoperative area with the
atic cancer consent paper, which outlines the nature and
(B) I am ashamed of you, an internal medi- indications of intervention, risks and benets,
cine resident should know better than to and potential alternatives. The physician, how-
ask me that ever, nds the patient to be a Laotian-speaking
(C) I appreciate your concern, but your father woman with limited English skills, and she
has a right to full disclosure, and it would does not seem to understand the residents ex-
be inappropriate for me to withhold infor- planations. The patients husband, who speaks
mation from him some English, offers to translate the physicians
(D) I would be happy to do that, but rst we explanation and appears to be eager for his
have to have a full family meeting and dis- wife to undergo the procedure. How should
cuss together what would be best for your the resident proceed to obtain consent?
father
(A) Allow the patients husband to translate
(E) If this is really important to you, I can tell
(B) Ask the husband to be a surrogate decision
you the results before I tell your father maker and sign the consent
(C) Ask the husband to step out and again ex-
2. A 74-year-old woman with dementia has been
plain, slowly and clearly, the informed
in the intensive care unit (ICU) for 14 days fol-
consent with the patient
lowing acute respiratory decompensation and
(D) Draw an X next to the signature line and
renal failure. She is receiving continuous he-
ask the patient to sign the form
modialysis, and her electrolytes have been sta-
(E) Obtain a translator or translation services
ble. Her oxygen saturation has been main-
for the patient and conduct proper discus-
tained on mandatory mechanical ventilation.
sion for informed consent
She remains in critical condition, but the ICU
team believes that she has a reasonable chance
of recovering. A living will shows that she has
agreed to intubation and resuscitation if neces-
sary. However, her daughter, who is her desig-
nated health care proxy and who until now has
HIGH-YIELD SYSTEMS
Chapter 5: Ethics and Legal Issues Questions 119

4. An 87-year-old man with prostate cancer, Glea- 6. An 85-year-old mentally competent patient is
son grade 9, is now receiving palliative care fol- brought to the emergency department in respi-
lowing failure of hormonal therapy and che- ratory distress. No living will is brought with the
motherapy. He has severe bone pain from patient, and before contact is made with the
multiple metastases, despite receiving both bis- family, the patient requires intubation and pres-
phosphonate and radiation therapy. His oxy- sors and is transferred to the ICU. Broad-spec-
genation is borderline on room air, and he has trum antibiotics are started for a presumed
required supplemental oxygen by face mask. pneumonia. The family, including the patients
The patient and his family are distraught that wife and children, arrive the next day. A discus-
he is in so much pain, and they ask the physi- sion is initiated with the family regarding the pa-
cian to make the pain stop. The physician ex- tients wishes as they relate to Do Not Resusci-
plains that raising the dose of opiates will sup- tate (DNR) and Do Not Intubate (DNI) orders.
press the patients breathing, but the family During this time the patients wife is declared

Ethics and Legal Issues


repeats their request. Which of the following is the surrogate, and she states that she would like
the best course of action? the antibiotics continued, but her husband
should be DNR. The patient begins to recover
(A) Give strong nonsteroidal anti-inamma-
and is able to be extubated on day 3 of his hos-
tory drugs to avoid respiratory depression,
pital care. On CT scan of the chest to evaluate
even if the pain is not well controlled
the extent of his pneumonia, he is found to have
(B) Increase the dose of bisphosphonate
a pulmonary embolism. A recommendation is
(C) Intubate the patient
made by the hospital staff that he receive an in-
(D) Medicate the patient with sufcient opi-
ferior vena cava lter. In this situation, under
oids to control his pain, regardless of respi-
whose authority can the lter be placed?
ratory response
(E) Obtain an ethics consultation (A) A court of law
(B) The patient
5. A patient in the clinic has a positive HIV test, (C) The patients children
rst by enzyme-linked immunosorbent assay (D) The patients doctor
antibody assay and then conrmed by Western (E) The patients spouse
blot. She is married, has three children, and is
sexually active with her husband. After explain- 7. An 89-year-old woman is diagnosed with meta-
ing what the results mean, she is advised that it static breast cancer to the lungs, bone, and
is important for the husband to be informed of brain. Four months after diagnosis, the patient
these test results. She appears stunned and is admitted to the hospital with excruciating
frightened by this suggestion. What is the most bone pain and changes in her mental status.
appropriate course of action? What term refers to the administration of mor-
phine to relieve this pain with the incidental
(A) Call up the husband and suggest he stop
consequence of causing respiratory depression
in for an appointment so that he can be in-
and death?
formed of the results of the test
(B) Contact the public health department and (A) Indirect euthanasia
ask that they inform the husband (B) Involuntary active euthanasia
(C) Continue persuading the patient of the im- (C) Nonvoluntary active euthanasia
portance and necessity of informing her (D) Passive euthanasia
husband and offer support and resources (E) Voluntary active euthanasia
to enable her to have this discussion
(D) Send an anonymous letter informing the
husband of the exposure
(E) Tell her that it is illegal for her not to in-
form her husband and other sexual con-
tacts who have been potentially exposed
HIGH-YIELD SYSTEMS
120 Section I: Organ Systems Questions

8. A 14-year-old boy arrives for an athletic check- (A) Attempt to save the child, but honor the
up. Physical examination reveals needle tracks childs wishes and give no blood
on his arms. On questioning, the patient ad- (B) Attempt to save the child, but honor the
mits to recent heroin use. When asked if he parents wishes and give no blood
would consider treatment, he says yes, but only (C) Get a court order, then give the child
if he can tell his parents that he is going to an blood
academic camp. What is the most appropriate (D) Give the child blood immediately, as
course of action? needed
(A) Detain the boy and admit him to treat- (E) Refuse to treat the child with those restric-
ment under minor law tions placed
(B) Inform the boys parents and leave the de-
11. A 32-year-old woman presents to her primary
cision about treatment up to them
care physicians ofce complaining of dysuria,
Ethics and Legal Issues

(C) Inform the boys parents and refer him to a


urgency, and frequency. The physician quickly
treatment center
scribbles a prescription for antibiotics, and the
(D) Refer him to a treatment center only
patient takes it to the pharmacist, saying, Darn
(E) Refer the patient to another pediatrician
urinary tract infection again. A few hours later,
9. A 93-year-old man is transferred to the emer- the pharmacist contacts the physicians ofce
gency department after staff at the long-term because she notices that the amount of antibiot-
care facility note confusion and agitation. He ics the physician prescribed is 10 times the usual
takes many medications, including insulin and dose given for urinary tract infection. The physi-
a glipizide for diabetes and a -blocker for hy- cian admits that she was distracted by having so
pertension. On examination the patient mum- many patients waiting to be seen and must have
bles incoherently when not spoken to and yells accidentally added an extra zero. What is the
at the speaker when directly addressed. His most appropriate next step for the physician?
temperature is 38.2C (100.8F), respiratory (A) Ask the pharmacist to explain the error to
rate is 28/min, blood pressure is 135/88 mm the patient
Hg, pulse is 58/min, and oxygen saturation is (B) Call the patient, explain what happened,
72% on room air. A hospital staff member re- and apologize for the error
minds the physician that the patients chart (C) Make a note to be more careful about
contains a signed DNR order. Which of the double-checking prescriptions in the future
following is the most appropriate next step in (D) Thank the pharmacist and return to seeing
management? patients; she can tell the patient about the
(A) Culture of blood, urine, and sputum error next time she sees her because no
(B) Haloperidol administration harm was done
(C) No intervention out of respect for the (E) Thank the pharmacist and return to seeing
DNR order patients; the pharmacist will tell the pa-
(D) Oxygen by nasal cannula tient about the error
(E) Serum glucose measurement
12. A 75-year-old man is in a persistent vegetative
10. A 10-year-old boy is brought into the emer- state following a large intracranial bleed sec-
gency department after an injury and is bleed- ondary to an arteriovenous malformation. Two
ing profusely. His parents arrive soon afterward, advance directives are in the chart. One is a
and state that they are Jehovahs Witnesses and living will stating that the patient requests with-
do not consent to giving blood to their child, drawal of life-sustaining treatment if he were to
even though to fail to do so threatens the ever be in a vegetative state. The other is a du-
childs life. The child, awake and receiving rable power of attorney form stating that his
pain medications, agrees with his parents and brother is the legally designated surrogate
requests that he receive no blood. Which of health care decision maker. After reading these
the following is the next step in treatment? forms, the brother approaches the physician
and requests that the medical team continue to
HIGH-YIELD SYSTEMS
Chapter 5: Ethics and Legal Issues Questions 121

treat, feed, and hydrate the patient. How a novel mechanism of action to regulate blood
should the medical team proceed? glucose levels in diabetes. He decides to invest
in the company that makes this drug (and oth-
(A) Medical team should consult the hospital
ers) by purchasing approximately $2,000 worth
ethics committee
of stock in it. His total stock portfolio is worth
(B) Medical team should continue to feed and
approximately $75,000. He is not on the pay-
hydrate the patient but not provide addi-
roll of this company, nor has he received any
tional care (e.g., antibiotics if the patient
consulting fees or speaker fees for seminars
develops an infection)
sponsored by this company. What level of dis-
(C) Medical team should continue to treat,
closure is most appropriate for this potential
feed, and hydrate the patient
conict of interest?
(D) Medical team should let a court decide
how to proceed (A) No disclosure is necessary

Ethics and Legal Issues


(E) Medical team should seek out the opinion (B) Telling all his patients that he owns this
of the next closest family member to re- stock
solve the issue (C) Telling only diabetic patients for whom he
(F) Medical team should withdraw all treat- decides to prescribe the drug that he owns
ment this stock
(D) Telling only his diabetic patients that he
13. A primary care physician is caring for a patient owns this stock
with stage IV ovarian cancer. The woman, who
has chosen to participate in hospice care, asks 15. A 46-year-old man with advanced pancreatic
the physician to give her something to end it cancer is hospitalized following a pancreatic
all. Which of the following is the most appro- duct stent placement. He has recently been
priate next step for the physician? told that he is not a candidate for a Whipple
procedure. His previously marked jaundice has
(A) Call the patients daughter and explain her
improved, but he is experiencing ongoing 8 of
mothers request
10 abdominal pain. The patient was divorced 6
(B) Discuss with the patient her feelings and
years ago, and his parents are his only family.
identify why she is asking for life-ending
Since his procedure, the patient has asked sev-
medication
eral members of the medical team to help him
(C) Provide the patient with a prescription for
end his life. Which of the following is the most
narcotics
appropriate next step?
(D) Refer the patient to a psychiatrist
(E) Tell the patient that her request is shock- (A) Admit this patient to hospice care
ing and such medication will not be pro- (B) Call patients family and inform them of
vided patients request
(C) Consult the psychiatry service about pa-
14. An academic internal medicine physician does tients ability to make medical decisions
nephrology research while maintaining a small (D) Evaluate pain management
outpatient private practice. In the course of his (E) Increase opioid dosage with the intent of
reading he learns of a promising new drug with causing respiratory depression
HIGH-YIELD SYSTEMS
122 Section I: Organ Systems Answers

AN S W E R S

1. The correct answer is C. Patients have a right care, it may be necessary to obtain a court or-
to full disclosure of their medical status, and der to maintain support.
family members cannot request that the physi-
Answer D is incorrect. Although an ethics
cian withhold information from a patient. Fur-
consultation may eventually be necessary, the
thermore, without the patients explicit permis-
daughters motives must be delineated rst.
sion, the physician cannot discuss the patients
health status with anyone else. Answer E is incorrect. Durable power of at-
torney allows a surrogate to make decisions on
Answer A is incorrect. Without the patients
the patients behalf in cases of incapacity. How-
explicit permission, the physician cannot dis-
ever, because this patient has a living will that
Ethics and Legal Issues

cuss the patients health status with family


contradicts the choice made by the daughter,
members.
the daughters choice should not be automati-
Answer B is incorrect. Rather than being cally accepted.
dismissive, the physician should address the
daughters concerns, but remind her gently 3. The correct answer is E. The patient must be
that his obligation is to the patient whose right informed of all components of an informed
to full disclosure must be respected. consent in a language that she understands.
The only appropriate way to discuss the in-
Answer D is incorrect. A family meeting may
formed consent is through a translator.
be appropriate some time in the future, but
rst and foremost the patient should be asked Answer A is incorrect. Although it would be
what information he wants to hear himself and convenient to ask the husband to translate, the
if he would like to involve his family in deci- husband is untrained in translation. Due to his
sion making. intimate relationship with the patient and po-
tentially biased views, an independent, profes-
Answer E is incorrect. Without the patients
sional translator should be used.
explicit permission, the physician cannot dis-
cuss the patients health status with family Answer B is incorrect. The patient is not in-
members. capacitated but needs proper translation to
understand the situation and the risks and ben-
2. The correct answer is A. Before any decision ets of the procedure.
regarding this patients care is undertaken, the
Answer C is incorrect. The patient does not
daughters reasoning must be elucidated. Al-
understand English and therefore a repeated
though she has durable power of attorney, the
discussion will be of no additional benet.
daughters decision clearly disagrees with the
Translator service is needed for proper in-
patients living will. Patients reserve the right to
formed consent.
change their decision within a given set of cir-
cumstances. However, when a proxy does so, it Answer D is incorrect. The patient needs to
must be determined whether the patient might be informed properly before signing the con-
have made the same decision. This determina- sent paperwork.
tion is difcult but must be done to rule out
any conict of interest or ulterior motives by 4. The correct answer is D. Treating a patients
the daughter. pain with the risk of hastening death has been
addressed and supported by both the U.S. Su-
Answer B is incorrect. Maintaining cur-
preme Court and the Catholic Church. In
rent management without rst addressing the
Vacco v. Quill (1997), Justice OConnor stated
daughters concerns is inappropriate.
that a patient has no legal barrier to receiving
Answer C is incorrect. If, after questioning, relief from pain even to the point of causing
any conicts of interest are found and the unconsciousness and hastening death. The
daughter continues to insist on withdrawal of Catholic Church has adopted the principle of
HIGH-YIELD SYSTEMS
Chapter 5: Ethics and Legal Issues Answers 123

double effect, whereby the hastening of death the primary patient to disclose this information
as an adverse effect of pain relief is morally ac- to his or her sexual contacts.
ceptable if there was no intention of doing so
Answer B is incorrect. The ideal situation is
by the treating physician.
one in which the patient informs her husband,
Answer A is incorrect. Providing inadequate and so the most appropriate response is to
pain relief to prevent respiratory depression is continue persuading the patient to reveal this
inappropriate unless it is the desire of the pa- information. However, if the patient does not
tient. feel comfortable revealing this information,
especially in settings of domestic violence, a
Answer B is incorrect. The rst step in reliev-
public health agency may be asked, with con-
ing chronic pain is to attempt to remove or
sent of the patient, to intervene and attempt to
lessen the inciting factor. However, because
contact the spouse.
that treatment has failed, the physician should

Ethics and Legal Issues


directly relieve the sensation of pain. In this Answer D is incorrect. If the patient refuses to
case, the patient is already on palliative care. inform her husband, then depending on the
state there might be other avenues by which
Answer C is incorrect. Although intubating
sexual partners may be contacted. Sending an
the patient would secure the airway and allow
anonymous letter informing the husband of
full use of opioids, it would entail taking the
the exposure, however, would likely damage
patient off palliative care, a decision that must
the doctor-patient and husband-wife relation-
be made rst by either the patient or the fam-
ships.
ily.
Answer E is incorrect. As a rule, partner noti-
Answer E is incorrect. In this case, both the
cation is condential and voluntary. Disease
law and moral consensus support the principle
intervention specialists cannot inform third
of double effect, which holds that a physician
parties without the consent of the infected per-
may relieve pain even if it hastens death. The
sons. Contacts are also typically not informed
case is clear enough that an ethics consultation
of the identity of the source.
would probably not be necessary.
6. The correct answer is B. Whenever possible,
5. The correct answer is C. The patient should
consent for any treatment or procedure should
be convinced to disclose this information per-
be obtained from the patient. In this situation,
sonally to her husband and the practitioner
although the patients wife was the surrogate,
should not directly contact the husband; this is
her decision-making capacity ended when the
the best way to maintain a trusting and open
patient was extubated and was able to speak for
doctor-patient relationship. Support should be
himself.
made available to the patient so that she can
inform the husband. Disease intervention spe- Answer A is incorrect. A court of law is only
cialists or HIV partner counseling and referral utilized when a disagreement occurs among
services, operating out of the department of members of a patients family and/or the physi-
health, may serve as valuable resources to help cian is in charge of the patients care.
patients contact their partners. These services
Answer C is incorrect. The patients children
vary from state to state, but they can be offered
are not in a position to make decisions on be-
to patients to facilitate this process.
half of the patient because the patient has the
Answer A is incorrect. Going behind the pa- capacity to make his own decisions and his
tients back is inappropriate; however, in some wife is his surrogate.
states there are laws that protect the physician
Answer D is incorrect. The patients doctor
from legal liability for breach of condentiality
can suggest the best treatment, but when fam-
if they do inform third parties of HIV exposure.
ily members are present they make decisions
The physician is protected, however, only after
for the patient if the patient does not have the
efforts by the physician have failed to convince
ability to make decisions himself.
HIGH-YIELD SYSTEMS
124 Section I: Organ Systems Answers

Answer E is incorrect. The patients spouse, communicate with the patient and allow him
although she is the surrogate decision maker, to make decisions for himself.
can no longer make decisions when the patient
is extubated and able to communicate for him- 9. The correct answer is D. Attention in the
self. emergency department must rst be directed
to the ABCs (Airway, Breathing, Circula-
7. The correct answer is A. The administration of tion). Because he is conversant, the patients
morphine or other medications to relieve pain airway is intact. His respiratory rate indicates
with the incidental consequence of causing re- some distress, and his oxygen saturation is low.
spiratory depression leading to death is known Oxygen supplementation via mask or nasal
as indirect euthanasia and is legal everywhere cannula is in order, and it is possible that this
in the United States. may be sufcient to begin to improve the pa-
tients mental status while other problems are
Ethics and Legal Issues

Answer B is incorrect. Involuntary active eu-


sought and addressed. DNR does not mean
thanasia involves intentionally giving medica-
do not treat. Taken literally, a DNR order
tions or interventions to cause a patients death
only applies after cardiac arrest has occurred,
when the patient is competent to consent but
although in practice some actions that may be
does not.
undertaken prior to cardiac arrest (e.g., endo-
Answer C is incorrect. Nonvoluntary active tracheal intubation) may be considered resus-
euthanasia involves providing these medica- citative. DNR orders may sometimes be writ-
tions or interventions to a patient who is inca- ten as DNR/DNI, if that is the patients wish,
pable of consent. to help ease confusion.
Answer D is incorrect. Passive euthanasia is Answer A is incorrect. Multiple cultures may
the withholding or withdrawing of life-sustain- reveal an underlying infection, but atten-
ing treatments to let a patient die. tion must rst be paid to the ABCs (Airway,
Breathing, Circulation).
Answer E is incorrect. Voluntary active eu-
thanasia is dened as the intentional admin- Answer B is incorrect. Haloperidol adminis-
istration of medication or of an intervention tration may be necessary later in the patients
to cause a patients death with their informed treatment, but in this patient attention to and
consent. stabilization of the ABCs should be ad-
dressed rst.
8. The correct answer is D. Just as minors are ex-
Answer C is incorrect. The patient requires
empt from parental consent or involvement
evaluation and treatment, not resuscitation.
when they are treated for sexually transmitted
Therefore, the DNR order is not yet in force.
diseases and pregnancy counseling, they may
also undergo drug rehabilitation without pa- Answer E is incorrect. Serum glucose mea-
rental knowledge. surement would be warranted and could be
instructive, but attention must rst be paid to
Answer A is incorrect. Patients may not be
the ABCs.
forcibly detained except when they are an im-
mediate risk to themselves or others.
10. The correct answer is D. Treatment can be
Answer B is incorrect. The boy has the right initiated on the basis of legal precedent. This
to condentiality when seeking drug rehabili- patient is 10 years old and is not emancipated.
tation. Legally, he does not have the ability to refuse
Answer C is incorrect. The boy has the right treatment. His parents cannot refuse treatment
to condentiality when seeking drug rehabili- for him if that refusal will pose a serious health
tation. risk.

Answer E is incorrect. Referring the patient to Answer A is incorrect. Not giving blood is
another pediatrician would be unethical and withholding life-saving treatment. Legally, a
nonbenecial for this patient. It is important to physician can give blood to a minor against
HIGH-YIELD SYSTEMS
Chapter 5: Ethics and Legal Issues Answers 125

his and his parents wishes when to not do so 12. The correct answer is F. A living will is a legal
would be hastening the childs death. document written by the patient dictating the
patients wishes about withholding or withdraw-
Answer B is incorrect. Not giving blood is
ing life-sustaining treatment in the event of a
withholding life-saving treatment. Legally, a
terminal disease or a persistent vegetative state.
physician can give blood to a minor against
In this case, the patient meets criteria for per-
his and his parents wishes when to not do so
sistent vegetative state and therefore the medi-
would be hastening the childs death.
cal team should follow the patients written re-
Answer C is incorrect. In a less life-threaten- quest and withdraw life-sustaining treatment. If
ing situation, a court order should be obtained the living will were not present, the surrogate
to treat the minor against his parents wishes. health care decision maker would have had au-
Under these emergent circumstances, it is nec- thority to dictate how to proceed with treat-
essary to treat the patient rst, and is legal to ment. Although surrogates should typically

Ethics and Legal Issues


do so. make decisions consistent with the stated
Answer E is incorrect. Refusing to treat would wishes of the patient, this case highlights the
be to give in to the parents demands, thus has- importance of patients making their wishes
tening the childs death. known in a format such as the living will.
Answer A is incorrect. The hospital ethics
11. The correct answer is B. The physician has an committee does not need to be consulted be-
obligation to fully disclose any errors made in cause the living will clearly states the appropri-
patient care. The most effective and responsi- ate course of action for the given clinical situ-
ble way to do this is to call the patient immedi- ation.
ately, explain what happened, and apologize.
This is particularly easy in this case because no Answer B is incorrect. According to the liv-
harm was done. Furthermore, acknowledg- ing will, all life-sustaining treatment should be
ment of an error and an apology can help the withdrawn.
physician avoid litigation. Answer C is incorrect. In this case, the pa-
Answer A is incorrect. It is the physicians re- tient meets criteria for persistent vegetative
sponsibility to disclose the error directly to the state and therefore the medical team should
patient. follow the patients written request and with-
draw life-sustaining treatment.
Answer C is incorrect. Although this is a good
practice, the physician should also inform the Answer D is incorrect. The court does not
patient of the error. need to be involved because the living will
clearly states the appropriate course of action
Answer D is incorrect. Even though no harm for the given clinical situation.
was done with this error, it is appropriate and
responsible to inform the patient about the er- Answer E is incorrect. Although a close family
ror. By doing this, the physician can also enlist member should be consulted for decision mak-
the help of the patient in preventing further er- ing if a living will or designated health care
rors because the patient can be more vigilant proxy is not present, both are available and the
about her care and detect future errors. Medi- living will in this case dictates care.
cal errors due to human faults are inevitable,
13. The correct answer is B. Taking time to un-
and it is appropriate to do anything possible to
derstand a patients emotions and reasons for
help prevent errors in the future.
asking for life-ending measures is the correct
Answer E is incorrect. It is the physicians re- answer. Understanding why the patient has
sponsibility to disclose the error directly to the made such a request will help address the un-
patient; she should not rely on someone else to derlying problem.
do so.
Answer A is incorrect. Calling the womans
daughter without her permission violates pa-
HIGH-YIELD SYSTEMS
126 Section I: Organ Systems Answers

tient privacy rights and should not be the phy- is still present, and the physician must make
sicians next step. The physician should discuss the appropriate disclosures.
the problem with the patient and call family
Answer C is incorrect. The company makes
members only with the patients permission.
more than this diabetes medication, so conict
Answer C is incorrect. Physician-assisted sui- of interest disclosures should be made to all pa-
cide is unethical and illegal throughout the tients.
United States, except in the state of Oregon.
Answer D is incorrect. The physician may
The physician should not provide the patient
have been attracted to this companys stock
with medication that is intended to be life end-
because of the medication for diabetes, but we
ing.
are told that the company makes other drugs as
Answer D is incorrect. Before immediately well; thus all patients must hear about this con-
referring the patient to another physician, it is ict of interest.
Ethics and Legal Issues

most appropriate to understand her emotional


state and identify her underlying problem. 15. The correct answer is D. Inadequate pain con-
trol and comorbid depression are the two most
Answer E is incorrect. Passing judgment on
common causes of such a request by a patient,
the patient is not the most appropriate step and
and both should be evaluated in this situation.
should be avoided. The physician should gain
an understanding of the womans position in a Answer A is incorrect. Although admission to
nonthreatening, nonjudgmental manner. hospice care may be necessary for this patient
to control pain and provide adequate support
14. The correct answer is B. A central principle of for the patient, such major decisions should
bioethics is that the physician has a duciary not be made in the setting of poor pain man-
relationship with patients, meaning that physi- agement.
cians have a responsibility to act in a patients
Answer B is incorrect. The patients family
best interest. This responsibility extends to dis-
should not be informed of this unless the pa-
closures of conicts of interest, in which pro-
tient has made an active attempt on his life
fessional obligations are inuenced by personal
and a medical decision that the patient is not
interest. In this example the physician owns
able to make is necessary. It is illegal to comply
stock in a pharmaceutical company that makes
with a patients requests for euthanasia.
a number of drugs. Disclosures should be
made to any and all affected parties, including Answer C is incorrect. The patient may be
his patients. Because conicts of interest may temporarily unable to make medical decisions
affect the quality of research reported, disclo- but psychiatrys involvement should be focused
sures must be made to institutions and groups on the patients ability to cope with his diagno-
involved with research such as journals review- sis.
ing investigators manuscripts and scientic Answer E is incorrect. Pain control should be
meetings at which one might present. appropriate to the disease process. It is ethical
Answer A is incorrect. Disclosure is certainly to provide palliative care even if it may hasten
necessary. While the American Medical As- the patients death, but it should not be the
sociation code of ethics denes a limit to the cause of death. Physician-assisted suicide in-
monetary value of gifts received, it does not set volves a physician writing a prescription for a
a similar limit on other potential conicts of drug that will allow the patient to end his or
interest such as stock. Even though this stock her own life. Currently, this is legal only in Or-
represents a small portion of the physicians egon and only after specic legal procedures
overall portfolio, a potential conict of interest have been followed.
CHAPTER 6

Gastrointestinal

127
HIGH-YIELD SYSTEMS
128 Section I: Organ Systems Questions

Q U E ST I O N S

1. A 54-year-old man presents to his primary care (C) Perform an esophagoscopy


provider with the complaint of upper abdomi- (D) Schedule him for elective esophagectomy
nal fullness and pain. He states that he has lost (E) Schedule him for emergent Nissen fun-
2.3-4.6 kg (5-10 lb), but denies other symp- doplication
toms. Physical examination reveals a rm mass
in the epigastric area. Ultrasonography reveals 3. The physician on call is paged to the well-baby
a mass in the gastric antrum. A salivary gland nursery because a full-term, 3-hour-old boy has
biopsy reveals the pathology shown in the im- had green emesis twice, once after each of his
age. Which of the following therapies is ex- feedings. He is being breast-fed. He was born
pected to be part of his treatment plan? by spontaneous vaginal delivery following a
pregnancy complicated by polyhydramnios.
Gastrointestinal

His Apgar scores were 8 and 9 at 1 and 5 min-


utes, respectively. His temperature is 37C
(98.6F), blood pressure is 70/50 mm Hg,
pulse is 150/min, and respiratory rate is 24/
min. His upper abdomen is distended, soft, and
without palpable masses. Air is visualized in
the duodenum and the stomach on x-ray.
Which of the following is the most likely diag-
nosis?
(A) Duodenal atresia
Reproduced, with permission, from Lichtman MA, Beut- (B) Hirschsprungs disease
ler E, Kipps TJ, Seligsohn U, Kaushansky K, Prchal JT. (C) Intussusception
Williams Hematology, 7th edition. New York: McGraw-
Hill, 2006: Plate XXII-21.
(D) Malrotation with volvulus
(E) Pyloric stenosis
(A) Antibiotic therapy
(B) Bone marrow transplantation 4. A 68-year-old African-American man presents
(C) Gene therapy to his primary care physician for a check-up.
(D) Liver transplantation He has not been to the physicians ofce in
(E) Multiagent chemotherapy over 15 years. He reports that he is ne but that
(F) Resection of mass and gastric antrum his wife keeps telling him that he has to go see
the doctor. He says he has never been sick, de-
2. A 55-year-old white man with a 20-year history spite smoking three packs of cigarettes per day
of gastroesophageal reux visits the clinic for for over 40 years. He also says that he drinks
worsening reux symptoms over the past 18 23 beers a night but never had a problem with
months. His last visit was 7 years ago and he that either. Hes as healthy as a bull, he says.
claims to be otherwise in good health. He has His wife is in the room and says that he re-
been compliant with his antireux medica- cently has had some problems swallowing food
tions, including an H2-blocker and a proton and that he is losing weight. He laughs and
pump inhibitor. Which of the following is the says, I just need to chew more and eat more.
best next step in management? His vitals signs are normal, as are his laboratory
(A) Double the dose of his H2-blocker and values. The physician is concerned and orders
schedule him for follow-up in 4 weeks an endoscopy, which reveals a biopsy positive
(B) Double the dose of his proton pump in- for squamous cell carcinoma of the esophagus.
hibitor and schedule him for follow-up in Which of the following most likely could have
4 weeks prevented this condition?
HIGH-YIELD SYSTEMS
Chapter 6: Gastrointestinal Questions 129

(A) Avoiding fruits and vegetables 6. A full-term 6-day-old boy presents to a physi-
(B) Eating more meats, especially smoked cians ofce for routine care. He is tolerating
meats breast milk well. He is urinating, defecating,
(C) Eliminating smoking and alcohol con- and sleeping normally. Physical examination
sumption reveals an alert newborn with mild eczema,
(D) Getting a colonoscopy every 5 years good skin turgor, normal reexes, and a musty
(E) Taking proton pump inhibitors regularly odor. His newborn laboratory screen is notable
for phenylketones in the urine. What is the
5. A 58-year-old man comes to the emergency de- best advice to give his parents regarding the
partment complaining of colicky abdominal boys diet?
pain over the past 3 days that suddenly became
(A) Increase iron
more severe and constant over the past 6 hours.
(B) Increase niacin
A contrast study is performed and results are
(C) Increase phenylalanine
shown in the image. What is the rst-line treat-
(D) Increase tyrosine

Gastrointestinal
ment after uid resuscitation and nasogastric
(E) Increase vitamin D
tube placement?
7. A 59-year-old woman with renal cell carcinoma
presents to the emergency department with se-
vere right upper quadrant (RUQ) pain. She is
afebrile, acutely tender in the RUQ, and has
shifting dullness and a palpable liver edge.
Murphys sign is negative. Laboratory studies
show:
Na+: 138 mEq/L
K+: 3.6 mEq/L
Glucose: 80 mg/dL
Aspartate aminotransferase: 50 U/L
Alanine aminotransferase: 43 U/L
Alkaline phosphatase: 138 U/L
Total protein: 6.4 g/dL
Reproduced, with permission, from Chen MYM, Pope
Albumin: 3.8 g/dL
TL, Ott DJ. Basic Radiology. New York: McGraw-Hill, Total bilirubin: 1.1 mg/dL
2004: Figure 8-29. Imaging demonstrates a spider web of collat-
(A) Colonoscopy eral veins in the liver. Although extensive mea-
(B) Hemicolectomy sures are taken, the patient dies 6 hours after
(C) Proximal colostomy with delayed resection arriving. Which of the following was the most
(D) Sigmoid colectomy likely initial treatment?
(E) Sigmoidoscopy (A) -Blocker followed by lactulose
(B) Cholecystectomy
(C) Endoscopic retrograde cholangiopancre-
atography with dilation of the common
bile duct
(D) Exploratory laparotomy
(E) Tissue plasminogen activator followed by
anticoagulation
HIGH-YIELD SYSTEMS
130 Section I: Organ Systems Questions

8. A 65-year-old man presents to his physician examination is signicant for scleral icterus,
complaining of difculty swallowing, occa- ankle edema, and a distended and tense abdo-
sional chest pain, and regurgitation of food. men. Laboratory studies show:
Over the past 2 months he has lost about 7 kg
Hemoglobin: 7 g/dL
(15 lb). Results of a barium swallow study are
Reticulocyte count: 7%
shown in the image. What test should be per-
Total bilirubin: 3.1 mg/dL
formed to look for possible causes of his condi-
Aspartate aminotransferase: 84 U/L
tion?
Alanine aminotransferase: 92 U/L
Ceruloplasmin: 5 mg/dL (normal:
2045 mg/dL)
Results of a Coombs test are negative. Which
of the following is an appropriate preventive
management step after chelation therapy?
Gastrointestinal

(A) Blood protein electrophoresis


(B) Colonoscopy
(C) ECG
(D) Schilling test
(E) Upper endoscopy

11. A full-term 5-day-old African-American girl is


taken to the pediatrician because her eyes
look yellow. She is being exclusively formula-
Reproduced, with permission, from Lalwani AK. Current
fed with an iron-rich formula. She has six wet
Diagnosis & Treatment in OtolaryngologyHead & Neck diapers a day and stools twice a day. The preg-
Surgery, 2nd edition. New York: McGraw-Hill, 2008; Fig- nancy was uncomplicated and she was deliv-
ure 35-3. ered by spontaneous vaginal delivery. Her Ap-
gar scores were 9 and 10 at 1 and 5 minutes,
(A) 24-hour pH monitoring
respectively. Her temperature is 37C (98.6F),
(B) Esophageal manometry
her head circumference is in the 50th percen-
(C) Serum gastrin level measurement
tile, and her weight is 3420 g (3 g below her
(D) Upper endoscopy
birth weight). Her sclerae are icteric. There is
(E) Urease breath test
no hepatomegaly or splenomegaly. Her total
bilirubin is 9 mg/dL and her conjugated biliru-
9. A 60-year-old man with no past medical history
bin is 0.2 mg/dL. Hemoglobin is 15 g/dL.
undergoes upper endoscopy and biopsy for an
Which of the following is the most likely diag-
upset stomach that is worsened by eating. He is
nosis?
found to have inammation predominantly in
the antrum of the stomach. Which of the fol- (A) 1-Antitrypsin deciency
lowing is the most likely etiology of this condi- (B) Biliary atresia
tion? (C) Dubin-Johnson syndrome
(A) Alcohol abuse (D) Physiologic jaundice
(B) Cigarette smoking (E) Rotor syndrome
(C) Iatrogenic
12. A 76-year-old man who has had multiple epi-
(D) Infection
sodes of pancreatitis presents to his physicians
(E) Spicy foods
ofce with mild epigastric pain and 9.1-kg
10. A 21-year-old man presents to the clinic feeling (20.0-lb) weight loss over the past 6 months.
tired and generally unwell. He has fallen sev- The patient also describes daily foul-smelling
eral times over the past month and has devel- stools that oat in the toilet bowl. The physi-
oped a slight tremor in both hands. Physical cian pulls up his electronic medical record and
nds that the patient presented to the emer-
HIGH-YIELD SYSTEMS
Chapter 6: Gastrointestinal Questions 131

gency department last week for the same symp- 14. A 55-year-old white woman with a history of
toms. During that visit he had a CT of the ab- iron deciency anemia has had intermittent
domen (see image). Which of the following is trouble swallowing solids for the past few years.
the most appropriate treatment? She denies alcohol or tobacco use. Her vital
signs are stable. Her iron level is 40 g/dL and
total iron binding capacity is 500 g/dL. Other
laboratory tests are within normal limits.
Which of the following is the most likely diag-
nosis?
(A) Achalasia
(B) Barretts esophagus
(C) Esophageal carcinoma
(D) Mallory-Weiss syndrome
(E) Plummer-Vinson syndrome

Gastrointestinal
(F) Reux esophagitis
(G) Symptomatic diffuse esophageal spasm
Reproduced, with permission, from Chen MYM, Pope
TL, Ott DJ. Basic Radiology. New York: McGraw-Hill,
2004: Figure 11-62. 15. A premature newborn is being treated in the
neonatal intensive care unit. On the sixth day of
(A) Endoscopic retrograde cholangiopancre- life he is noted to be lethargic and in mild respi-
atography ratory distress. His heart rate is 162/min, blood
(B) Pancreatic enzyme replacement pressure is 55/38 mm Hg, and respiratory rate is
(C) Pancreaticogastrostomy 56/min. In addition to a distended abdomen, he
(D) Surgical resection of pancreas has guaiac-positive stools. X-ray of the abdomen
(E) Whipple procedure shows gas bubbles within the bowel wall. From
what potentially life-threatening condition is this
13. A 62-year-old woman is transferred to the med- patient most likely suffering?
ical service with an appendiceal mass serendip-
itously picked up at the edge of an x-ray taken (A) Bowel obstruction
of a broken femur in the emergency depart- (B) Intussusception
ment. Otherwise, the patient has no signicant (C) Meconium ileus
past medical history and no current symptoms. (D) Meningitis
Which of the following studies is most likely to (E) Necrotizing enterocolitis
be useful?
(A) Arterial blood gas
(B) CT of the chest and abdomen
(C) Immediate ECG
(D) MRI of the chest and abdomen
(E) Room air oxygen saturation
HIGH-YIELD SYSTEMS
132 Section I: Organ Systems Questions

16. A 45-year-old white man is brought to the (A) Acute mesenteric ischemia
emergency department by ambulance. He is (B) Colon cancer
waving wildly, trying to hit the ying bats (C) Diverticulitis
that are all around him. He is very agitated and (D) Infectious colitis
smells strongly of alcohol. The ambulance (E) Inammatory bowel disease
crew said they found the patient bleeding from
the mouth outside a bar. They could not nd 18. A 70-year-old man with a history of constipa-
any laceration on his mouth or lips and believe tion has been experiencing intermittent left-
that the bleeding is internal. The patient sided abdominal pain and fevers for 2 days. He
screams that he will not stand for this maltreat- came to the emergency department immedi-
ment any longer and tries to stand up, at which ately after he noticed blood in his toilet this
point he begins to vomit. Blood pours out of morning. His heart rate is 110/min, blood pres-
his mouth, and the patient says, Here we go sure is 90/50 mm Hg, respiratory rate is 18/
again. The ambulance crew tells the physi- min, and oxygen saturation is 95% on room air.
Gastrointestinal

cian that there was also a lot of vomit at the bar On physical examination the physician notes
where he was found. The physician is able to copious amounts of bright red blood per rec-
subdue the patient to obtain his vital signs. His tum. The physician immediately places two
blood pressure is 118/78 mm Hg, pulse is 98/ large bore intravenous lines, administers uid,
min, respiratory rate is 22/min, and tempera- and sends blood for type and screen. Which of
ture is 37.2C (98.9F). The physician is un- the following is the best next step in manage-
able to obtain a history on the patient or con- ment?
tact any relatives or friends. No signs of obvious (A) Arteriography
trauma are observed. Which of the following is (B) Colonoscopy
the best next step in diagnosis? (C) Endoscopy
(A) Barium swallow (D) Nasogastric tube aspiration
(B) Electrocardiogram (E) Surgical consultation
(C) Endoscopy
(D) Esophageal manometry 19. A 3-year-old boy is brought to the pediatrician
(E) X-ray of the chest because his mother noticed a reddish-purple
rash on his buttocks and thighs (see image).
17. A 65-year-old man comes to the emergency de- She notes that he has not seemed well since he
partment complaining of left lower abdominal had a mild cold 2 weeks earlier; he has been
pain that began the prior morning. He became complaining of aches and pains in his legs and
concerned when he developed bloody diarrhea a stomach ache. Urinalysis shows 1020 RBCs/
overnight. He has experienced similar pain, al- mm and 2+ proteinuria. Which of the follow-
though to a lesser degree, over the past 2 ing is associated with this patients disease pro-
months, especially after eating. The pain usu- cess?
ally resolved within 12 hours, and he never
had bloody diarrhea. His past medical history is
signicant for coronary artery disease and hy-
pertension. He has smoked one pack of ciga-
rettes per day for the past 30 years. On physical
examination he is afebrile, heart rate is 90/min,
and blood pressure is 135/85 mm Hg. He is vis-
ibly uncomfortable but in no apparent distress.
His abdominal examination is signicant for
left lower quadrant tenderness but no guarding
or rebound. Which of the following is the most
likely diagnosis?
HIGH-YIELD SYSTEMS
Chapter 6: Gastrointestinal Questions 133

(A) Blood smear


(B) CT scan of the abdomen
(C) Endoscopic retrograde cholangiopancre-
atography
(D) Fasting transferrin saturation levels
(E) Liver biopsy

21. A 50-year-old woman undergoes screening


colonoscopy at her primary care physicians
recommendation. She has no family history of
colorectal cancer. A single lesion is removed
during the procedure and sent for pathologic
examination. Which of the following ndings
carries the greatest risk of malignancy?

Gastrointestinal
(A) Lymphoid polyp
(B) Peutz-Jeghers polyp
(C) Tubular adenoma
Reproduced, with permission, from Lichtman MA, Beut- (D) Tubulovillous adenoma
ler E, Kipps TJ, Seligsohn U, Kaushansky K, Prchal JT. (E) Villous adenoma
Williams Hematology, 7th edition. New York: McGraw-
Hill, 2006: Figure XXV-15. 22. A 61-year-old man in previously excellent
(A) Hemoptysis health presents to his physician with com-
(B) High antistreptolysin O titer plaints of hematochezia, tenesmus, and rectal
(C) Impaired glucose tolerance pain. On work-up the physician discovers that
(D) Intussusception he has a rectal tumor that is 5 cm (2.0 in) from
(E) Malar rash the anal verge. Which of the following is the
most appropriate treatment?
20. A 63-year-old man with diabetes is called by his (A) Abdominoperineal resection
primary care physician because of abnormal (B) Imatinib
liver function test results, as follows: (C) Low anterior resection
Aspartate aminotransferase: 85 U/L (D) Radiation alone
Alanine aminotransferase: 102 U/L (E) Radiation plus chemotherapy
Alkaline phosphatase: 180 U/L
Total bilirubin: 1.9 mg/dL
On physical examination his liver is enlarged.
His skin has a slightly yellow hue, especially on
his face. The review of symptoms is signicant
for some weight loss, weakness, arthritis in his
hands, and inability to achieve an erection.
What test would generate the most likely diag-
nosis and should be done rst?
HIGH-YIELD SYSTEMS
134 Section I: Organ Systems Questions

23. A 45-year-old HIV-positive woman comes to (A) Breast cancer relapse


her primary care physician complaining of a (B) Diffuse esophageal spasm
2-day history of bloody diarrhea. She states that (C) Esophageal cancer
she has been feeling well until 2 days ago, (D) Myocardial infarction
when she developed abdominal pain. She de- (E) Nutcracker esophagus
nies fevers, chills, night sweats, nausea, or vom-
iting. She admits to feeling tired over the last 25. A 32-year-old man with Crohns disease presents
couple of weeks and has had a 2.3-kg (5-lb) to the emergency department with acute-onset
weight loss over the past 2 weeks. Her stool diffuse abdominal pain and emesis. The patient
sample shows WBCs and RBCs. Her Gram states these symptoms are different than his
stain is shown in the image. Her CD4+ cell usual Crohns disease are-ups. The pain is se-
count is 201/mm. Which of the following is vere (10/10) and is cramping in nature. He says
the most likely cause of this womans symp- his abdomen feels larger than usual. His Crohns
toms? disease has been well managed on 6-mercap-
Gastrointestinal

topurine for the past 6 months. The patient de-


nies any recent sick contacts or eating underpre-
pared foods. He states he had a bowel
movement and atus since the abdominal pain
began. In addition to Crohns disease, the pa-
tient had appendicitis for which he underwent
an appendectomy 12 years ago. His temperature
is 37.1C (98.7F), blood pressure is 135/86 mm
Hg, pulse is 84/min, and respiratory rate is 14/
min. On physical examination the abdomen is
distended and diffusely tender with high-pitched
bowel sounds. There is rebound tenderness
Image courtesy of the Centers for Disease Control and throughout the abdomen along with guarding.
Preventions Public Health Image Library. The remainder of the physical examination is
(A) Escherichia coli noncontributory. An x-ray of the abdomen
(B) Kaposis sarcoma shows dilated small loops of bowel along with
(C) Legionella absence of gas in the colon. What is the best
(D) Mycobacterium avium complex next step in management?
(E) Mycobacterium tuberculosis (A) Bowel rest only
(B) Intravenous uids and antibiotics only
24. A 64-year-old white woman presents to her pri- (C) Laparotomy
mary care physician complaining of difculty (D) MRI of the abdomen
and pain with swallowing, as well as occasional (E) Ultrasound
chest pain. She has a history of breast cancer
treated with lumpectomy and radiation, hyper- 26. A 39-year-old Japanese-American woman with
tension, high cholesterol, and ovarian polyps. insulin-dependent diabetes and asthma pre-
She indicates that her current problem started sents to her primary care physician complain-
with liquids, but has progressed to solids, and ing of trouble swallowing for the past few
that the food just gets stuck in my throat. The months. She explains that it started with solids,
chest pain was once so bad that she took one of and then progressed to liquids. She states it
her husbands nitroglycerin pills and the pain now is hard even to swallow water and that she
subsided, but it has since occurred many times. is often very thirsty. She says she has lost about
The physician orders an x-ray of the chest, but 3.2 kg (7 lb), but says she is working out fre-
it is not diagnostic. Manometry is conducted, quently. Her blood pressure is 118/76 mm Hg,
and it shows uncoordinated contractions. pulse is 86/min, respiratory rate is 16/min, and
Which of the following is the most likely diag- temperature is 37.2C (98.9F). Laboratory
nosis? tests show
HIGH-YIELD SYSTEMS
Chapter 6: Gastrointestinal Questions 135

Na+: 144 mEq/L An ultrasound of her abdomen shows a


K+: 4.0 mEq/L shrunken and nodular liver. A liver biopsy us-
Cl: 100 mEq/L ing Perls Prussian blue stain is shown in the
Carbon dioxide: 22 mmol/L image. Which of the following is the most
Blood urea nitrogen: 18 mg/dL likely complication of her disease?
Creatinine: 1.0 mg/dL
Glucose: 88 mg/dL
Her hemoglobin A1c level, measured 3 months
earlier, was 6.1%. A barium swallow is per-
formed, which reveals a dilated esophagus, es-
pecially distally, that ares out near the lower
esophageal junction. Still not completely sure
of the diagnosis, esophageal manometry is per-
formed, which reveals abnormal peristalsis and

Gastrointestinal
increased lower sphincter pressure. Which of
the following is the most appropriate manage-
ment? Reproduced, with permission, from PEIR Digital Library
(http://peir.net).
(A) Cholinergic agents
(B) Glucose pharmacotherapy (A) Acute pancreatitis
(C) Instructions to elevate the bed, avoid fatty (B) Amyloidosis
foods, and consider a histamine blocker (C) Bone marrow failure
(D) Pneumatic dilation (D) Hepatocellular carcinoma
(E) Surgery to remove diverticula (E) Splenomegaly

27. A 75-year-old woman with a history of diabetes 28. A term boy with Apgar scores of 9 and 9 at 1
and coronary heart failure presents to the and 5 minutes has failed to pass meconium at
emergency department because of increasing 72 hours. He has had no episodes of emesis,
abdominal girth. In recent months she has and his abdomen is only mildly distended to
been feeling increasingly fatigued, and al- palpation. The patients mother reports that
though she has had decreased appetite, she has her older son had the same problem at birth.
gained weight. Her heart rate is 100/min and A plain radiograph of the abdomen shows a
blood pressure is 112/70 mm Hg. She has small bowel obstruction with numerous air-
scleral icterus; the skin over her face, neck, and lled loops of bowel. The patient is treated
lower legs is slightly bronze in color; she has with a diatrizoate meglumine (Gastrogran)
palmar erythema; and she has numerous ec- enema, with good results. Which of the fol-
chymoses over her body. Her abdominal exam- lowing is the most likely mechanism for this
ination is signicant for ascites. Laboratory tests infants acute intestinal problem?
show:
(A) Congenital aganglionosis of the colon
Aspartate transaminase: 102 U/L (B) Deciency of pancreatic enzymes
Alanine transaminase: 97 U/L (C) Intussusception of the large bowel
Alkaline phosphatase: 300 U/L (D) Total absence of the small bowel
Total bilirubin: 1.9 mg/dL (E) Volvulus of the transverse colon
Albumin: 2.9 g/dL
Prothrombin time: 22 sec
Partial thromboplastin time: 42 sec
HIGH-YIELD SYSTEMS
136 Section I: Organ Systems Questions

29. A 75-year-old woman comes to the emergency (A) Emergent appendectomy and postopera-
department with complaints of nausea and tive antibiotics
nonbilious, nonbloody vomiting over the past (B) Give nothing by mouth with intravenous
4 days. The patient reports that both the nau- hydration
sea and vomiting come in waves; that is, sev- (C) Percutaneous drainage and interval appen-
eral hours will pass during which she feels well dectomy
before the vomiting suddenly recurs. A detailed (D) Serial abdominal examinations
history reveals that the woman was told several (E) Urgent ECG and cardiac enzymes
months ago that she has stones in her gall-
bladder, but she has been too frightened to 31. A 40-year-old man with a recent history of ex-
undergo surgery. She has not had a bowel ploratory laparotomy for a stabbing injury pre-
movement for 7 days. Her temperature is sents to the emergency department with diffuse
38.4C (101.1F) and the abdomen is dis- cramping abdominal pain for 1 day, accompa-
tended with high-pitched bowel sounds. nied by nausea, multiple episodes of brown-
Gastrointestinal

Which of the following is the most appropriate colored vomitus, and lack of stool, but he re-
initial test for a patient with suspected gallstone ports some atulence. He denies any fever. On
ileus? physical examination, the patient has stable vi-
tal signs, and there is diffuse distention in the
(A) Abdominal ultrasound
abdomen with guarding and tenderness but no
(B) Diagnostic laparoscopy
rebound, as well as high-pitched bowel sounds.
(C) Endoscopic retrograde cholangiopancre-
Rectal examination reveals no fecal impaction
atography
in the rectal vault, and the stool was guaiac-
(D) Hepatic iminodiacetic acid scan
negative. Complete blood cell count reveals no
(E) Plain X-ray of the abdomen
signicant abnormalities and serum chemistry
shows a mild metabolic alkalosis. CT demon-
30. A 62-year-old woman with a history of diabetes
strates a noticeable difference in the diameter
mellitus presents to the emergency department
of proximal and distal small bowel. Which of
complaining of severe abdominal pain for the
the following is the most appropriate manage-
past 12 hours, rst beginning as dull pain near
ment?
the umbilicus but now localized to the right
lower quadrant. She initially thought she was (A) Broad-spectrum antibiotics
suffering from heartburn, but decided to come (B) Colonoscopy
to the hospital because of the unrelenting pain. (C) Exploratory laparotomy with lysis of adhe-
The patient reports that just prior to examina- sions
tion by the physician, she experienced a sud- (D) Give the patient nothing by mouth, insert
den decrease in intensity of pain, but she re- a nasogastric tube, and perform intrave-
mains feeling very uncomfortable and must nous correction of electrolyte abnormali-
remain on the stretcher. On examination the ties
patient appears in distress secondary to pain, (E) Serial abdominal examinations
tachycardic, slightly hypotensive, and febrile at
39C (102F). She has a diffusely tender abdo- 32. An 84-year-old woman with coronary artery dis-
men with point tenderness over her right lower ease, congestive heart failure, peripheral vascu-
quadrant, accompanied by guarding and re- lar disease, and atrial brillation presents to the
bound. Laboratory values showed a leukocyto- emergency department with dizziness, weak-
sis of 20,000/mm with 95% polymorphonu- ness, and sudden-onset crampy periumbilical
clear lymphocytes. After conrming the pain. The pain is associated with one episode
diagnosis with imaging, which of the following of diarrhea and one episode of emesis. The pa-
is the most appropriate management? tient notes she has been having similar pain af-
HIGH-YIELD SYSTEMS
Chapter 6: Gastrointestinal Questions 137

ter meals for several months but never this


severe. Her temperature is 37.2C (98.9F),
heart rate is 135/min, blood pressure is 96/60
mm Hg, and respiratory rate is 16/min. Physi-
cal examination is notable for a slightly dis-
tended abdomen that is extremely tender to
palpation with diminished bowel sounds.
There is no rigidity or rebound tenderness
noted on the abdominal examination. In addi-
tion, the patient has heme-positive stool. Her
WBC count is 19,500/mm, hemoglobin is
10.9 g/dL, and platelet count is 159,000/mm.
Liver function testing results are normal. After
stabilizing the patient, what is the best next

Gastrointestinal
step in management?
(A) Barium enema
(B) Colonoscopy
Reproduced, with permission, from Tintinalli JE, Kelen
(C) Laparotomy GD, Stapczynski S, Ma OJ, Cline DM. Tintinallis Emer-
(D) Obstruction series gency Medicine: A Comprehensive Study Guide, 6th edi-
(E) Warfarin therapy tion. New York: McGraw-Hill, 2004: Figure 127-2.

33. A 2-year-old boy is brought to the emergency (A) Cystic brosis


department. His mother reports that the pa- (B) Enterocolitis
tient had been well until 3 days ago, when he (C) Henoch-Schnlein purpura
developed a fever and nasal congestion. He (D) Idiopathic intussusception
was diagnosed with otitis media in his right ear, (E) Meckels diverticulum
and was started on amoxicillin with clavulanic (F) Pyloric stenosis
acid by his pediatrician. He appeared to be im-
proving until this morning, when he began to 34. A 59-year-old man presents for his routine
complain of abdominal pain. The pain has colonoscopy and during his visit he has numer-
been intermittent, with episodes occurring ev- ous large adenomas removed from his colon.
ery 20 minutes for several minutes each time. Which of the following is the most effective
However, the episodes appear to be worsening strategy for follow-up of this patient?
and lasting longer with increasing pain. Thirty (A) Elective colectomy
minutes ago he had an episode of nonbloody, (B) Repeat colonoscopy in 10 years
nonbilious emesis that was followed by passage (C) Repeat colonoscopy in 3 years
of blood- and mucus-stained stools. He is cur- (D) Sigmoidoscopy in 10 years
rently in no acute distress, and his vital signs (E) Urgent colectomy
are normal. A rm sausage-shaped mass is pal-
pable in the RUQ of his abdomen. A rectal ex-
amination yields bloody mucus. He does not
have any skin lesions or rashes. X-ray of the ab-
domen is shown in the image. Which of the
following is the most likely diagnosis?
HIGH-YIELD SYSTEMS
138 Section I: Organ Systems Questions

35. A 67-year-old woman is currently postoperative down. X-ray of the chest is shown in the image.
day 8 after an emergent laparoscopic cholecys- Which of the following is the most likely diag-
tectomy for acute cholecystitis. On postopera- nosis?
tive day 2 she spiked a temperature of 40C
(101.4F) and began to complain of some
shortness of breath. X-ray of the chest revealed
right lower lobe pneumonia, and the patient
was started on clindamycin. Today she is expe-
riencing multiple episodes of foul-smelling,
watery diarrhea that is green tinged but non-
bloody. She also complains of lower abdominal
cramping. Her temperature is 37.8C (100F),
pulse is 90/min, respiratory rate is 15/min, and
blood pressure is 110/70 mm Hg. Which of the
Gastrointestinal

following is the most likely explanation for


these ndings?
(A) Ingestion of preformed enterotoxins, cyto-
toxins, and/or neurotoxins
(B) Production of cytotoxins within the gastro- Reproduced, with permission, from Brunicardi FC, An-
intestinal tract dersen DK, Billiar TR, Dunn DL, Hunter JG, Matthews
(C) Production of enterotoxins and cytotoxins JB, Pollock RE, Schwartz SI. Schwartzs Principles of Sur-
within the gastrointestinal tract gery, 8th edition. New York: McGraw-Hill, 2005: Figure
(D) Production of enterotoxins within the gas- 38-10.
trointestinal tract (A) Congenital diaphragmatic hernia
(E) Viral invasion and damage of villous epi- (B) Pyloric stenosis
thelial cells within the gastrointestinal tract (C) Respiratory distress syndrome
(D) Tracheoesophageal stula
36. A male infant is delivered at 37 weeks gesta- (E) Transient tachypnea of the newborn
tion via cesarean section for breech presenta-
tion. The pregnancy was complicated by poly- 37. The physician on call is called to the well-baby
hydramnios. The 34-year-old mother is rubella nursery because a full-term, African-American
immune and has blood type B. She is negative boy who is 49 hours old has not passed meco-
for Rh antibody, Group B streptococci, rapid nium. The pregnancy was uncomplicated. The
plasma reagin, hepatitis B surface antigen, gon- neonates blood pressure is 70/50 mm Hg,
orrhea, and Chlamydia. At delivery there is no heart rate is 140/min, and respiratory rate is 36/
meconium. He has a birth weight of 2.7 kg (6 min. The neonate is crying but is easily consol-
lb). The baby has a weak cry and is pale and able. His abdomen is markedly distended. A
frothing at the nose and mouth. He has nasal barium enema is ordered, which shows dilated
aring and retractions, with a respiratory rate proximal bowel and a narrowed distal segment.
of 56/min. Heart rate is 140/min and he has a Which of the following would provide a deni-
regular rhythm and a harsh 2/6 holosystolic tive diagnosis in this child?
murmur that is best heard at the left sternal
border. On auscultation he has ne diffuse (A) Absent ganglion cells on rectal biopsy
crackles in his lungs bilaterally. The infant is (B) Absent ligament of Treitz on upper gastro-
missing both thumbs and has fusion of the re- intestinal series
maining digits of his upper extremities bilater- (C) Air bubbles in the stomach and duodenum
ally. The pediatric resident is able to suction on x-ray lm of the abdomen
secretions from the patients nasopharynx and (D) Positive sweat test
oropharynx; however, she is unable to pass a (E) Telescoping of bowel on air contrast bar-
nasogastric or orogastric tube more than 10 cm ium enema
HIGH-YIELD SYSTEMS
Chapter 6: Gastrointestinal Questions 139

38. A 73-year-old man comes to his primary care (A) Bacterial enterocolitis
physician for his yearly check-up. His medical (B) Bowel ischemia
history is signicant for obesity, new onset dia- (C) Colon carcinoma
betes mellitus, and a remote history of tobacco (D) Mallory-Weiss tear
use. The patient has noticed that his stool has (E) Ulcerative colitis
been darker for the past 3 months, although he
has only seen gross blood in his stool once, a 41. A 47-year-old woman presents to the emer-
week ago. He also complains of recent fatigue gency department with an 8-day history of left
and occasional light-headedness when stand- lower quadrant pain and semi-formed stools.
ing up from sitting. On examination the pa- Starting this afternoon, she has noticed blood
tient has fecal occult blood and a hematocrit in her stool as well as dizziness when she gets
of 32%. Colonoscopy and upper gastrointesti- up from sitting. She denies fever, nausea, vom-
nal endoscopy reveal no obvious pathology. iting, weight loss, and night sweats. Her tem-
Which of the following is the best diagnostic perature is 37C (98.6F), heart rate is 104/

Gastrointestinal
test to locate this patients hemorrhage? min, blood pressure is 120/82 mm Hg supine
and 103/63 mm Hg when she sits up, and re-
(A) Angiography
spiratory rate is 18/min. Physical examination
(B) Barium enema
reveals no peritoneal signs and is remarkable
(C) Colonoscopy
only for fecal occult blood on rectal examina-
(D) CT scan with contrast
tion. Laboratory results reveal a WBC count of
(E) Tagged RBC scan
13,000/mm and hematocrit of 29%. Results of
an x-ray of the abdomen are shown in the im-
39. A pregnant 16-year-old girl with no prior pre-
age. What is the best next step in manage-
natal care presents to the emergency depart-
ment?
ment in labor. A male infant is delivered pre-
cipitously. Prenatal laboratory test results are
unknown. There is no meconium. He has a
birth weight of 3 kg (6 lb 10 oz). He is pink
and is crying, heart rate is 130/min, and respi-
ratory rate is 36/min, with good respiratory ef-
fort. The emergency medicine resident notices
the infant has ascites and a membrane-covered
anterior abdominal mass at the base of his um-
bilical cord. Which of the following is the most
likely diagnosis?
(A) Duodenal atresia
(B) Gastroschisis
(C) Hirschsprungs disease
(D) Meckels diverticulum
(E) Omphalocele Reproduced, with permission, from Chen MYM, Pope
TL, Ott DJ. Basic Radiology. New York: McGraw-Hill,
40. A 24-year-old woman presents to her primary 2004: Figure 10-54.
care provider because of bloody diarrhea for
several months and uveitis. Complete blood (A) Angiography with embolization
cell count shows mild anemia but a normal (B) Immediate surgery for partial colectomy
WBC count. The erythrocyte sedimentation (C) Intravenous hydration and blood transfu-
rate and the C-reactive protein level are not el- sion
evated. Which of the following is the most (D) Nothing by mouth, nasogastric tube, and
likely diagnosis? broad-spectrum antibiotics
(E) Place the patient on a high-ber diet
HIGH-YIELD SYSTEMS
140 Section I: Organ Systems Questions

42. A 66-year-old woman presents to her physician 44. A 74-year-old man presents to the emergency
because of recurrent painless bleeding on defe- department with abdominal pain. The pain is
cation over the past month. She has regular, deep and aching and is localized to the left
soft bowel movements and no history of consti- lower quadrant. The man reports multiple epi-
pation or diarrhea. A recent diagnosis of aortic sodes of diarrhea over the preceding week. He
stenosis was an incidental nding on echocar- also reports having multiple similar episodes of
diogram. She takes calcium and vitamin D abdominal pain in the past. On physical exam-
supplements daily. Her last colonoscopy (at age ination he is febrile and has tenderness to pal-
60) was normal. Heart rate is 82/min, blood pation of the left lower quadrant. His WBC
pressure is 133/72 mm Hg, respiratory rate is count is 23,000/mm. Results of CT are shown
12/min, and temperature is 36.6C (97.8F). in the image. Which of the following is the
Physical examination reveals her conjunctivae most likely diagnosis?
are pink and mucosa is moist. She has no ab-
dominal tenderness or palpable masses and no
Gastrointestinal

hemorrhoids or ssures. Stool is hemoccult


positive. Colonoscopy shows a spider-like le-
sion in the ascending colon. Which of the fol-
lowing is the most likely diagnosis?
(A) Angiodysplasia
(B) Crohns disease
(C) Diverticulosis
(D) Ischemic colitis
(E) Peptic ulcer disease
Reproduced, with permission, from USMLERx.com.
(F) Ulcerative colitis
(A) Angiodysplasia
43. A 61-year-old woman is brought to the emer- (B) Carcinoid syndrome
gency department drowsy and disoriented, able (C) Carcinoma of the colon
only to follow simple commands. On examina- (D) Diverticulitis
tion her abdomen is distended and nontender, (E) Infectious colitis
her skin has a yellow hue, and there are multi-
ple spider nevi on her chest. In her purse, the 45. A 62-year-old woman presents to her physician
physician nds prescriptions for peginterferon with complaints of heartburn, fatigue, and in-
and ribavirin. When asked to raise her hands, termittent upper abdominal pain. The pain is
the physician notices a coarse tremor. Labora- often worse after meals and especially with
tory tests show: spicy foods. She reports no recent nausea, vom-
Blood urea nitrogen: 17 mg/dL iting, weight loss, dysphagia, or bright-red
Creatinine kinase: 1.1 mg/dL blood per rectum. However, her stools are
Aspartate aminotransferase: 89 U/L darker than normal. Her last colonoscopy 2
Alanine aminotransferase: 93 U/L years ago was unremarkable. Stool guaiac test
Total bilirubin: 3.1 mg/dL result is positive. An initial complete blood cell
Ammonia: 124 g/dL count reveals:

Which of the following is the most likely diag- Hemoglobin: 10.1 g/dL
nosis? Hematocrit: 33.2%
Mean corpuscular volume: 74.6/mm
(A) Bleeding esophageal varices Mean corpuscular hemoglobin concentration:
(B) Hepatic encephalopathy 25.8%
(C) Hepatocellular carcinoma WBC count: 9200/mm
(D) Hepatorenal syndrome Platelet count: 176,000/mm
(E) Spontaneous bacterial peritonitis Ferritin: 11 ng/mL
HIGH-YIELD SYSTEMS
Chapter 6: Gastrointestinal Questions 141

The patient undergoes upper endoscopy, guarding. Barium enema reveals colonic dila-
which reveals erosive gastritis. She is started on tation of 8 cm. Stool is sent for Gram stain and
omeprazole, 40 mg twice a day and oral iron analysis for fecal leukocytes, fecal occult blood,
sulfate supplementation, 325 mg three times a and Clostridium difcile toxin. Which of the
day with meals. Six months into treatment, ab- following is most likely present in the stool
dominal pain and heartburn are resolved, but sample?
the patient still has fatigue and is pale. Repeat
(A) Clostridium difcile toxin
laboratory tests show:
(B) Gram-negative rods
Hemoglobin: 9.9 g/dL (C) Gram-positive cocci
Hematocrit: 30.2% (D) No fecal occult blood
Mean corpuscular volume: 74.2/mm (E) Spores and hyphae
Mean corpuscular hemoglobin concentration:
25.1% 47. A 40-year-old Asian woman presents to the
WBC count: 9800/mm emergency department complaining of inter-

Gastrointestinal
Platelet count: 198,000/mm mittent epigastric pain. The pain is severe, lasts
Ferritin: 10 ng/mL for a few hours, and is sometimes accompanied
by nausea and vomiting. Her bowel move-
Repeat upper endoscopy is negative for bleed-
ments have been normal. Her temperature is
ing and erosive gastritis. Stool guaiac test result
38.3C (100.9F), pulse is 100/min, blood
is negative. Which of the following is the next
pressure is 150/80 mm Hg, and respiratory rate
best step in management?
is 22/min. Physical examination reveals moder-
(A) Bone marrow biopsy ate obesity and mildly icteric sclerae. Bowel
(B) Determine blood type and screen for trans- sounds are normal, with an abrupt halt of in-
fusion spiration upon palpation of the RUQ, and
(C) Discontinue omeprazole therapy guarding is noted. Laboratory values reveal a
(D) Initiate darbepoetin therapy WBC count of 13,000/mm, total bilirubin of
(E) Initiate parenteral iron therapy 3.3 mg/dL, and normal liver enzymes and alka-
line phosphatase levels. Which of the follow-
46. A 82-year-old woman is in the surgical inten- ing is the rst diagnostic imaging study that
sive care unit after a carotid endarterectomy. should be performed?
She has been taking clindamycin and cipro-
(A) CT
oxacin for the past 13 days. On postoperative
(B) Flat and upright plain x-rays of the abdo-
day 2 the patient is febrile and tachycardic with
men
a high WBC count and a low RBC count. She
(C) Hepatobiliary iminodiacetic acid scan
is also noted to be dehydrated and hypotensive.
(D) MRI
On physical examination she is distended and
(E) Ultrasound of the RUQ
has abdominal tenderness with rebound and
HIGH-YIELD SYSTEMS
142 Section I: Organ Systems Questions

E X T E N D E D M ATC H I N G 48. A 25-year-old Asian man who recently immi-


grated presents to the clinic because of watery
The response options for the next 3 items are diarrhea, cramping, and atulence after attend-
the same. Select one answer for each item ing a picnic, where he ate corn on the cob and
in the set. ice cream cake. Stool guaiac is negative. One
day after the picnic, his symptoms resolve.
For each patient with diarrhea, select the most
likely etiology. 49. A 2-year-old boy presents with a history of
smelly, bulky diarrhea and poor weight gain. A
(A) Bacillus cereus small bowel biopsy shows villous atrophy.
(B) Campylobacter jejuni
(C) Carcinoid 50. A 79-year-old woman presents to the clinic
(D) Celiac disease with 2 days of fever, dehydration, and bloody
(E) Clostridium difcile diarrhea. She was recently hospitalized and
Gastrointestinal

(F) Crohns disease nished a course of antibiotics for pneumonia.


(G) Entamoeba histolytica Stool sample is positive for WBCs.
(H) Escherichia coli
(I) Giardia
(J) Irritable bowel syndrome
(K) Lactose intolerance
(L) Ulcerative colitis
(M)Vibrio parahaemolyticus
(N) Whipples disease
HIGH-YIELD SYSTEMS
Chapter 6: Gastrointestinal Answers 143

AN S W E R S

1. The correct answer is A. The patients biopsy noma. Surgical procedures used to treat MALT
shows a proliferation of lymphocytes consistent lymphomas generally include only resection of
with a mucosa-associated lymphoid tissue the tumor itself, not the gastric antrum.
(MALT) lymphoma. This type of small cell
lymphoma makes up approximately 8% of non- 2. The correct answer is C. One-time esophagos-
Hodgkins lymphoma, and may occur in the copy is recommended in patients 50 years old
stomach, orbit, intestine, lung, thyroid, or sali- and older to evaluate for the development of
vary gland; however, it most often arises in the esophageal metaplasia (Barretts esophagus).
stomach. Patients generally present with com- There is a direct correlation between the devel-
plaints of abdominal pain or fullness. Ninety- opment of Barretts esophagus and history of
ve percent of gastric MALToma cases are as- gastroesophageal reux disease (GERD). If

Gastrointestinal
sociated with Helicobacter pylori infection and metaplasia is present, these patients are 30125
the majority of cases achieve remission with times more likely to develop adenocarcinoma.
simple antibiotic eradication of H. pylori. Pa- Esophagoscopy and biopsy could help discern
tients with very extensive disease are occasion- whether this patient has a Barretts esophagus
ally treated with single-agent chemotherapy or has already progressed to an early stage can-
with chlorambucil. cer that has yet to cause dysphagia and weight
loss.
Answer B is incorrect. Bone marrow trans-
plantation is generally not required to treat Answer A is incorrect. Although doubling the
MALT lymphomas. Even widespread disease is medication dose may provide symptomatic
generally responsive to much more mild che- relief of GERD, it does not tell us anything
motherapy. about the damage that has been done to the
esophagus as a result of the condition. To prop-
Answer C is incorrect. Although certain ge-
erly manage the disease, it is necessary to eval-
netic alterations have been identied in MALT
uate for metaplasia.
lymphomas, such as t(11;18) and trisomies of
chromosomes 3, 7, 12, and 18, gene therapy is Answer B is incorrect. Changing the medica-
not a standard of care for any cancer currently. tion dose may provide further symptomatic
In addition, because cure rates are so high with relief of GERD, but this patient is in need of
known therapies, any physician should be re- esophagoscopy to evaluate for metaplasia in or-
luctant to propose that a patient with this dis- der to properly manage the disease.
ease be exposed to the inherent risks of gene
Answer D is incorrect. Esophagectomy is
therapy.
typically indicated for the removal of long
Answer D is incorrect. Liver transplantation segments of metaplasia in patients with Bar-
would not be required in a patient with MALT retts esophagus. In this patient, it is unknown
lymphoma that would most likely be respon- if there is any metaplasia, so esophagoscopy
sive to mild therapy. Even in patients with should be performed rst.
metastatic cancer to the liver, transplantation
Answer E is incorrect. There is no indication
is generally not indicated because of the co-
for any emergent surgical procedure. Nis-
morbidities associated with major surgery and
sen fundoplication is typically used for active
immunosuppression in cancer patients.
GERD that is refractory to medical manage-
Answer E is incorrect. Multiagent chemother- ment.
apy is not required to treat MALT lymphoma.
3. The correct answer is A. Duodenal atresia is
Answer F is incorrect. Patients with gastric
the most likely diagnosis because it presents
carcinoma may be treated with resection of the
with bilious vomiting following any feeding in
mass and gastric antrum. However, the pathol-
the rst few hours of life. Constipation often
ogy of this patients tumor is clearly not a carci-
develops after meconium is passed. Physical
HIGH-YIELD SYSTEMS
144 Section I: Organ Systems Answers

examination is notable for abdominal distension 4. The correct answer is C. Smoking and alcohol
of predominantly the upper abdomen. Polyhy- consumption are the two most prominent risk
dramnios is often the earliest sign. Infants with factors for esophageal squamous cell carci-
Downs syndrome are at increased risk of having noma, which can present with dysphagia and
duodenal atresia. A double-bubble sign, air bub- weight loss.
bles in the stomach and duodenum on x-ray of
Answer A is incorrect. To the contrary, anec-
the abdomen, is diagnostic of duodenal atresia.
dotal evidence suggests that these foods actu-
Answer B is incorrect. Hirschsprungs disease ally have a protective effect against squamous
usually presents with delayed passage of meco- cell carcinoma.
nium and abdominal distention. Other symp-
Answer B is incorrect. Smoked meats are a
toms include poor feeding and bilious vomit-
risk factor for carcinoma and one of the lead-
ing. Patients with Hirschsprungs usually pass
ing reasons why this disease is so prevalent in
meconium after rectal stimulation. A barium
other countries. Patients should stay away from
enema that shows dilated proximal bowel
Gastrointestinal

smoked foods.
and a narrowed distal segment is highly sug-
gestive of Hirschsprungs disease. The lack of Answer D is incorrect. While it is true that
ganglion cells on rectal biopsy is diagnostic of colonoscopies are suggested as regular preven-
Hirschsprungs disease. tive measures, this test would not have helped
detect the esophageal changes he was undergo-
Answer C is incorrect. Intussusception is a
ing.
common cause of bowel obstruction in infants
<2 years. It presents with bouts of irritability, Answer E is incorrect. Squamous cell carci-
colicky abdominal pain, and vomiting. If it is noma is not due to reux. Barretts esophagus
not diagnosed at this point, infants become is more likely due to this problem and results
lethargic and pass red currant jelly-like stools. in adenocarcinoma.
Physical examination is notable for a palpable
right lower quadrant mass. Air contrast barium 5. The correct answer is B. The image shows a
enema showing telescoping of the bowel is di- cecal volvulus. Cecal volvulus typically dis-
agnostic for intussusception and often thera- plays a classic kidney bean appearance. After
peutic as well. uid resuscitation and large bowel decompres-
sion have been achieved, the rst-line therapy
Answer D is incorrect. A malrotation with is hemicolectomy with second-line treatment
volvulus is the most critical cause of bowel being colonoscopy.
obstruction. It presents in the rst week of life
with bilious emesis and constipation. Rectal Answer A is incorrect. Colonoscopy is the
hemorrhage and peritonitis can occur. An up- second-line treatment for cecal volvulus, after
per gastrointestinal (GI) series with an absent hemicolectomy.
or abnormal position of the ligament of Treitz Answer C is incorrect. This is the second-line
conrms the diagnosis of malrotation. treatment for splenic exure volvulus. It is only
Answer E is incorrect. Pyloric stenosis pre- indicated if extended hemicolectomy cannot
sents with nonbilious projectile vomiting dur- be performed.
ing the rst weeks to months of life. Physical Answer D is incorrect. The obstruction here
examination is classically notable for a palpa- involves the cecum, not the sigmoid. Sigmoid
ble olive-shaped, mobile, nontender epigastric volvulus may present as abdominal distension
mass. Laboratory values reveal a hypochlore- with minimal tenderness or severe pain, vomit-
mic-hypokalemic metabolic alkalosis. Barium ing, and absolute constipation. Sigmoid colec-
study shows a narrow pyloric channel, which is tomy is the removal of the sigmoid section of
sometimes called a string sign or a pyloric the colon. It may be performed open or laparo-
beak. scopically. Sigmoid colectomy is the second-
line treatment for sigmoid volvulus, after sig-
moidoscopy.
HIGH-YIELD SYSTEMS
Chapter 6: Gastrointestinal Answers 145

Answer E is incorrect. The obstruction in- thrombosis is a cause of postsinusoidal portal


volves the cecum, not the sigmoid. Sigmoid hypertension, and acute thrombosis will cause
volvulus may present as abdominal distension RUQ pain, hepatomegaly, and ascites. Liver
with minimal tenderness or severe pain, vomit- function tests may be slightly elevated, but lab-
ing, and absolute constipation. Sigmoidoscopy oratory values are otherwise typically normal.
refers to the insertion of a scope into the colon Ultrasound will often show collateral vessels in
to examine the rectum, the sigmoid, and de- a spider web pattern and decreased hepatic ve-
scending portions of the colon. The procedure nous blood ow. Mortality is high at 40% to
is indicated for temporary decompression of 90%. Initial medical treatment is thrombolysis
sigmoid volvulus (recurrence of the volvulus is followed by anticoagulation, especially in pa-
common without prompt surgery). tients presenting with acute symptoms. How-
ever, the TIPS procedure (transjugular intrahe-
6. The correct answer is D. This child has phe- patic portosystemic shunt) or hepatic
nylketonuria (PKU). In PKU, phenylalanine transplantation is a more denitive treatment.

Gastrointestinal
cannot be converted to tyrosine. It is screened
Answer A is incorrect. -Blocker therapy is
for at birth, and is detected by phenylketones
used to medically treat portal hypertension due
(phenylacetate, phenyllactate, and phenylpyru-
to its effect on the systemic and splanchnic cir-
vate) in the urine. Classic features of PKU are
culations, whereas lactulose is used to treat he-
fair skin, eczema, musty body odor, and men-
patic encephalopathy associated with liver dys-
tal retardation. Mental retardation can be pre-
function. Neither are appropriate treatments
vented by reducing phenylalanine (not one of
for Budd-Chiari syndrome.
the answer choices) and increasing tyrosine in
the diet. Answer B is incorrect. Emergent cholecys-
tectomy should be performed if the patient is
Answer A is incorrect. Symptoms of iron de-
suffering from acute cholecystitis. However,
ciency include pallor and lethargy. Iron de-
signs of portal hypertension, negative Murphys
ciency is an acquired deciency; it does not
signs, and evidence of collateral venous ow
result from an inborn error of metabolism.
on imaging suggest hepatic vein thrombosis,
Answer B is incorrect. Niacin deciency, also not cholecystitis.
known as pellagra, is characterized by derma-
Answer C is incorrect. Nonsurgical treatment
titis, diarrhea, and dementia. In children, this
for common bile duct stones includes endo-
occurs most commonly in developing coun-
scopic retrograde cholangiopancreatography
tries where the diet is high in corn. Niacin de-
with dilation of the common bile duct, re-
ciency is acquired; it does not result from an
moval of existing stones, and stent placement.
inborn error of metabolism.
This is not an appropriate treatment for this
Answer C is incorrect. In PKU, phenylalanine patient.
needs to be reduced rather than increased.
Answer D is incorrect. An exploratory laparo-
Answer E is incorrect. Vitamin D deciency tomy would be performed for various reasons,
leads to rickets. Rickets is characterized by including whether a bowel obstruction was
bowlegs, a pigeon-breast deformity (in which suspected or whether debulking of tumor was
the sternum projects forward), kyphosis, sco- necessary. However, the information provided
liosis, and enlargement of wrist joints. Vitamin suggests hepatic vein thrombosis, which would
D is an acquired deciency; it does not result not require an exploratory laparotomy.
from an inborn error of metabolism.
8. The correct answer is D. This patient has
7. The correct answer is E. This patient suffers achalasia, as conrmed by the narrowing of the
from Budd-Chiari syndrome, a condition gastroesophageal junction (the birds beak
caused by hepatic vein thrombosis that is typi- appearance). Secondary causes include malig-
cally secondary to a hypercoagulable state such nancy and Chagas disease. Malignancies that
as cancer, pregnancy, oral contraceptive use, or can cause achalasia include gastric carcinoma
hematologic disease. Chronic hepatic vein
HIGH-YIELD SYSTEMS
146 Section I: Organ Systems Answers

and lymphoma. Upper endoscopy and associ- Answer C is incorrect. Although there is some
ated biopsies can help to make the diagnosis of erythema associated with endoscopy, gastritis is
the secondary causes. dened as the histologic appearance of inam-
mation of the gastric mucosa.
Answer A is incorrect. 24-hour pH monitoring
is useful in the diagnosis of GERD. Answer E is incorrect. Spicy foods may lead to
increased gastric acid production, but would
Answer B is incorrect. Manometry helps to
not be the most likely cause of the inamma-
diagnose the patients achalasia, but does not
tion in this case.
help to determine the cause.
Answer C is incorrect. Measuring the serum 10. The correct answer is E. This patients presen-
gastrin level would be useful in diagnosing tation and laboratory tests all point to Wilsons
Zollinger-Ellison syndrome. disease, a disease characterized by copper ac-
cumulation in multiple organs, especially the
Answer E is incorrect. The urease breath test
liver and brain, due to defective excretion of
Gastrointestinal

is useful in diagnosing Helicobacter pylori in-


this metal. Along with basal ganglia dysfunc-
fection.
tion and hemolytic anemia, this patient is
9. The correct answer is D. Infection with Heli- showing signs of portal hypertension due to cir-
cobacter pylori is a cause of type B, or antral- rhosis caused by copper accumulation in the
liver. This increase in pressure in the portal
dominant, chronic gastritis. The most common
circulation leads to variceal formation at the
cause of chronic type B gastritis is use of non-
junctions of the portal and systemic circula-
steroidal anti-inammatory drugs, but our pa-
tions. An upper endoscopy should be per-
tient lacks this history. The incidence of
formed to monitor for esophageal varices that
chronic gastritis increases with age and the his-
could be treated prophylactically or closely
tologic appearance improves with the treat-
monitored to prevent life-threatening bleeding.
ment of H. pylori. Symptoms of chronic gastri-
tis may include pain, nausea, vomiting, Answer A is incorrect. A protein electrophore-
anorexia, and upper GI bleeding; however, this sis should be performed if multiple myeloma,
condition is frequently asymptomatic. Diagno- a neoplastic proliferation of plasma cells, is
sis is conrmed by H. pylori antibody test and suspected. Although this patient has evidence
breath urease test, as well as a direct gastric bi- of anemia, the other clinical and laboratory
opsy and culture. Treatment of H. pylori gastri- ndings clearly support the diagnosis of Wil-
tis is with triple therapy, which includes two sons disease, making a protein electrophoresis
antibiotics (metronidazole and clarithromy- unnecessary.
cin), bismuth compound, and a proton pump
Answer B is incorrect. While a colonoscopy is
inhibitor. Type A gastritis is autoimmune-me-
an important screening measure for all individ-
diated, affecting the fundus, and is associated
uals starting at age 50 years, this test would not
with vitamin B12 deciency.
be particularly revealing in a patient suffering
Answer A is incorrect. Severe alcohol abuse from Wilsons disease. However, hemorrhoids,
may have negative effects on the gastric mu- a condition associated with portal hyperten-
cosa, including the development of acute sion, should be considered in such individuals.
(stress) gastritis; however, in a patient with no
Answer C is incorrect. Although cardiac dys-
history of alcoholism and with inammation
function should be monitored in diseases in-
conned to the antrum, alcohol abuse would
volving abnormal accumulation of substances,
not be the most likely cause of gastritis.
such as hemochromatosis, this is not the case
Answer B is incorrect. Cigarette smoking may in Wilsons disease.
promote atrophic gastritis in H. pylori-positive
Answer D is incorrect. In the Schilling test ra-
patients, but it has not been shown to cause in-
dioactive vitamin B12 is given orally with a pa-
ammation of the gastric mucosa by itself.
renteral dose of unlabeled vitamin B12 in order
to measure the ability of the enteric tract to ab-
HIGH-YIELD SYSTEMS
Chapter 6: Gastrointestinal Answers 147

sorb vitamin B12 in the setting of megaloblastic patient will need to take pancreatic enzymes
anemia. Hemolytic anemia, not megaloblastic with each meal for the remainder of his life.
anemia, is found in Wilsons disease. Therefore
Answer A is incorrect. Endoscopic retrograde
the Schilling test would be unnecessary.
cholangiopancreatography or endoscopic ret-
rograde pancreaticoduodenostomy is used to
11. The correct answer is D. This is a typical pre-
diagnose stones and strictures in the biliary
sentation of physiologic jaundice. Physiologic
tract. It can also be therapeutic for choledo-
jaundice affects most newborns; it results in a
cholithiasis.
mild unconjugated hyperbilirubinemia after
the third day of life and resolves by 1 week in Answer C is incorrect. A pancreaticogastros-
full-term neonates and 2 weeks in preterm neo- tomy can be used to internally drain pancreatic
nates. Notably, the conjugated bilirubin is al- pseudocysts. Another option is percutaneous
ways normal and total bilirubin is always <14 surgical drainage.
g/dL in physiologic jaundice.
Answer D is incorrect. Surgical resection of

Gastrointestinal
Answer A is incorrect. 1-Antitrypsin de- the pancreas is neither necessary nor helpful
ciency can present as conjugated hyperbiliru- for this patients pancreatic insufciency.
binemia. This child has an unconjugated hy-
Answer E is incorrect. A Whipple procedure is
perbilirubinemia and no known family history,
a pancreaticoduodenectomy and reanastomosis
making this an incorrect diagnosis.
of the digestive tract to itself and with the pan-
Answer B is incorrect. Biliary atresia is the creas. It is performed for pancreatic tumors and
most common structural abnormality of the tumors of the ampulla of Vater. The Whipple
biliary tree in infants, and one of the two most is theoretically curative if further spread of the
common causes of conjugated hyperbiliru- tumor has not yet occurred; however, 5-year
binemia. Lack of a gallbladder and presence of postsurgical survival rate is a mere 5%.
a triangular cord sign on ultrasound are sugges-
tive of biliary atresia. However, this child has 13. The correct answer is B. The most common
an unconjugated hyperbilirubinemia, making tumors of the appendix are carcinoid tumors.
this diagnosis incorrect. Remembering the rule of thirds, 1/3 metas-
tasize, 1/3 are accompanied by a second malig-
Answer C is incorrect. Dubin-Johnson syn-
nancy, and 1/3 present with multiple carcinoid
drome is an inborn disorder of bilirubin me-
tumors. Since treatment will be based on tu-
tabolism that leads to a conjugated hyperbiliru-
mor location, tumor size, and metastases, a CT
binemia. Since this child has an unconjugated
of both the chest and abdomen will give you
hyperbilirubinemia, this diagnosis is incorrect.
most of the information you need. This imag-
The inborn disorders of bilirubin metabolism
ing modality will also possibly reveal a second
that cause an unconjugated hyperbilirubine-
malignancy.
mia are Gilberts syndrome and Crigler-Najjar
syndrome. Answer A is incorrect. With an asymptomatic
tumor, there should be no pulmonary com-
Answer E is incorrect. Rotor syndrome is an
promise secondary to late carcinoid heart syn-
inborn disorder of bilirubin metabolism that
drome, so an arterial blood gas study would not
leads to a conjugated hyperbilirubinemia.
be helpful. Patients with carcinoid heart syn-
Since this child has an unconjugated hyperbil-
drome may develop shortness of breath from
irubinemia, this diagnosis is incorrect.
right heart failure, secondary to tricuspid valve
stenosis.
12. The correct answer is B. This patient has
chronic pancreatitis; the image shows an atro- Answer C is incorrect. Carcinoid tumors may
phic, calcied pancreas. He now has exocrine cause carcinoid syndrome, which presents with
insufciency, which leads to malabsorption, ushing, diarrhea, bronchospasm, and hy-
fatty stools with diarrhea, and weight loss. This
HIGH-YIELD SYSTEMS
148 Section I: Organ Systems Answers

potension. This syndrome progresses to carci- Barretts esophagus. It is characterized by dys-


noid heart syndrome in up to 50% of patients, phagia of solids but would not present with the
characterized by endocardial plaques in the iron studies given and with no identiable risk
right heart that can lead to right heart failure. factors.
Late carcinoid heart syndrome can produce
Answer D is incorrect. Mallory-Weiss syn-
diffuse low-voltage ECG changes. Since this
drome is secondary to heavy vomiting caus-
patients tumor was asymptomatic, there is no
ing a tear in the lining of the esophagus. It is
need to suspect that it has damaged the right
a source of upper GI bleeding that should be
side of her heart enough to produce ECG
considered when the clinical history involves
changes. While there is no need for an imme-
a recent history of retching. Diagnosis is made
diate ECG, she should get a standard admis-
by endoscopy.
sion ECG when the nursing staff is able to get
her one. Answer F is incorrect. Reux esophagitis is
caused by reux of gastric contents into the
Answer D is incorrect. An MRI is not the
Gastrointestinal

esophagus. Patients present with a classic pre-


imaging modality appropriate for this type of
sentation of heartburn that is worse when
screening for solid tumors.
recumbent. It is typically managed rst with
Answer E is incorrect. Since the tumor is asymp- proton pump inhibitor or histamine blocker
tomatic, there should be no pulmonary compli- therapy.
cations, so room air oxygen saturation would not
Answer G is incorrect. Symptomatic diffuse
be helpful.
esophageal spasm usually will present with
both solid and liquid dysphagia. The diagnosis
14. The correct answer is E. Plummer-Vinson
is made by manometry, and initial treatment
syndrome is the name given for the association
generally involves anticholinergics, nitroglyc-
of dysphagia, upper esophageal webs, and iron
erin, and long-acting nitrates. Surgical inter-
deciency anemia. Its etiology is uncertain,
vention is often ultimately required.
and it is a rare phenomenon in the United
States. Diagnosis is made by laboratory assess-
15. The correct answer is E. Necrotizing entero-
ment, esophageal imaging, and the exclusion
colitis (NEC) is the most common life-threat-
of other competing explanations. Its compo-
ening condition of the newborn GI tract. Risk
nents, anemia and esophageal anatomic irregu-
of NEC is signicantly increased for premature
larities, are treated in the same manner they
infants. Symptoms usually begin within the
would if they presented independently.
rst 2 weeks of life, and initial signs often in-
Answer A is incorrect. Achalasia is commonly clude abdominal distention with delayed gas-
associated with dysphagia of both solid and tric emptying. Bloody stools can be seen, and
liquids and is not associated with iron de- patients may develop respiratory distress as the
ciency. It is a rare disease of unclear etiology illness progresses. On x-ray of the abdomen,
that is truly a functional pathology: unlike the hepatic portal venous gas and a bubbly appear-
anatomic impairment of Plummer-Vinson syn- ance of pneumatosis intestinalis (gas within the
drome, achalasia is due to a failure of the lower bowel wall) are pathognomonic for NEC.
esophageal sphincter to work properly. Treatment of NEC includes nasogastric suc-
tion, but surgical resection of the affected
Answer B is incorrect. Barretts esophagus is
bowel may be required in infants that develop
dened by columnar epithelium replacing
pneumoperitoneum, obstruction, or other seri-
squamous epithelium due to continuous in-
ous complications. The overall survival of in-
ammation associated with reux. It is an in-
fants aficted with NEC is approximately 70%
dication for surgical intervention, which gener-
to 80%.
ally means a Nissen fundoplication, as it carries
an elevated risk of esophageal carcinoma. Answer A is incorrect. Bowel obstruction
could produce the abdominal distention and
Answer C is incorrect. Carcinoma is usu-
delayed gastric emptying. However, the x-ray
ally due to smoking, alcohol consumption, or
HIGH-YIELD SYSTEMS
Chapter 6: Gastrointestinal Answers 149

ndings are most consistent with the diagnosis scribe symptoms consistent with abdominal an-
of NEC. gina (e.g., postprandial abdominal pain that re-
solves in a few hours). Its presentation can be
Answer B is incorrect. Although intussuscep-
either acute or chronic, with abdominal pain
tion would cause guaiac-positive stools, the
and bloody diarrhea usually developing 24
infant lacks apparent risk factors or clinical
hours after the abdominal pain. Early on, the
manifestations (i.e., vomiting). Also, because
pain is severe with frequent passage of bloody
intussusception results in a loss of gas, the x-ray
stools. The blood loss is not signicant enough
ndings are most consistent with the clinical
to necessitate a transfusion. As the ischemia
picture of NEC.
continues, the pain diminishes, and the abdo-
Answer C is incorrect. Meconium ileus often men becomes more tender and distended. Fi-
presents in a manner similar to bowel obstruc- nally, the bowel may become gangrenous with
tion, and vomiting is often present. X-ray nd- uid and protein leaking through the damaged
ings for meconium ileus include varying width mucosa, causing shock and metabolic acidosis.

Gastrointestinal
loops of bowel with an uneven gas lling pat- At this point rapid surgical intervention is nec-
tern. essary.
Answer D is incorrect. Meningitis is not likely Answer B is incorrect. In the absence of con-
given the abdominal ndings in addition to the stitutional symptoms and due to the acute na-
x-ray results. ture of his abdominal pain and bloody diar-
rhea, the diagnosis of colon cancer is unlikely.
16. The correct answer is C. This patient is suffer- Although the possibility of colon cancer can-
ing from a Mallory-Weiss tear, which is caused not be excluded in this patient, it is not the
by excessive vomiting. Hematemesis is very most likely diagnosis.
common at presentation and may also be asso-
Answer C is incorrect. Bleeding is usually
ciated with endoscopy and hiatal hernias,
more consistent with diverticulosis than diver-
which predispose to this condition. The bleed-
ticulitis. Although the presence of diverticular
ing is often self-limited but may persist, recur,
disease cannot be ruled out in this patient, it
and be life threatening. It is important to diag-
is unlikely to be the cause of his current symp-
nose it quickly, preferably with endoscopy. Al-
toms.
though endoscopy may cause hematemesis, it
will not exacerbate it. Answer D is incorrect. The chronicity of this
patients symptoms does not support a diagno-
Answer A is incorrect. A barium swallow
sis of infectious colitis. Infectious colitis pres-
should not be completed because it will inter-
ents with mild to moderate diarrhea but is not
fere with the endoscopy procedure and has a
bloody. Other symptoms include acute onset
low diagnostic yield.
of fever, malaise, and abdominal pain.
Answer B is incorrect. ECG might help if the
Answer E is incorrect. Postprandial pain that
patient was more hypotensive/tachycardic and
resolves in 12 hours is not commonly de-
showing signs of failing because fatal arrhyth-
scribed in patients with inammatory bowel
mias may occur, but it is a better test for treat-
disease. Additionally, the acute onset of this
ment than for diagnosis.
presentation is not consistent with inamma-
Answer D is incorrect. Manometry would be tory bowel disease, which typically presents
of no value in this case. in much younger patients and causes chronic
Answer E is incorrect. X-ray of the chest does symptoms.
not help in the diagnosis.
18. The correct answer is D. The patient in the
17. The correct answer is A. Acute mesenteric question has symptoms consistent with a lower
ischemia is often seen in individuals who are GI bleed in the context of diverticulitis. Al-
elderly (>60 years old) and who have a history though the symptoms described are suggestive
of atherosclerotic disease. The patients may de- of a lower GI bleed, a massive upper GI bleed
can also present as rectal bleeding. Therefore
HIGH-YIELD SYSTEMS
150 Section I: Organ Systems Answers

it is important to perform nasogastric tube aspi- Answer C is incorrect. Impaired glucose tol-
ration and rule out an upper GI bleed. erance would be expected in a patient with
diabetes mellitus. Patients with long-standing,
Answer A is incorrect. Arteriography should
poorly controlled diabetes often have glomeru-
be done if colonoscopy is not possible because
lar disease with proteinuria; however, they do
of the severity of the bleeding or if colonoscopy
not present with the symptoms of HSP such as
was not able to identify the site of the bleed.
purpuric rash and abdominal pain.
This technique, however, is limited, as the pa-
tient must be bleeding briskly during the scan Answer E is incorrect. A malar rash would be
in order to localize the site of the bleeding. seen in systemic lupus erythematosus and may
be associated with lupus nephritis.
Answer B is incorrect. Colonoscopy should be
performed once an upper GI bleed has been
20. The correct answer is D. The diagnosis of
ruled out. Colonoscopy not only allows you to
hemochromatosis should be suspected when a
localize the site of the bleeding, but can also
patient presents with diabetes, hepatomegaly,
Gastrointestinal

be a therapeutic intervention.
skin pigmentation changes, arthritis, and hy-
Answer C is incorrect. Endoscopy should be pogonadism. Three tests used in diagnosis of
performed if the nasogastric tube yields frank iron overload states include serum iron, serum
blood. This nding suggests that the source of ferritin, and transferrin concentration. An in-
the bleeding could be in the upper GI tract. crease in iron stores in normal subjects leads to
That suspicion can be conrmed and possibly an increase in plasma iron levels, a rise in
treated with endoscopy. plasma transferrin saturation (ratio of iron to
transferrin), and a rise in the concentration of
Answer E is incorrect. Surgical consultation
ferritin. A fasting transferrin saturation of 60%
should be obtained if the patient remains un-
or higher in men or 50% or greater in women
stable despite aggressive resuscitation.
will detect about 90% of patients with homozy-
19. The correct answer is D. This patient has the gous hypogonadotropic hypogonadism. Often
characteristic tetrad of Henoch-Schnlein pur- a lower cutoff will be used to minimize the
pura (HSP), including the purpuric rash, arth- number of patients with the condition who are
ralgias (aches and pains), abdominal pain, missed.
and glomerular renal involvement. HSP is a Answer A is incorrect. Many diseases of the
systemic vasculitis most commonly seen in blood can cause iron overload syndromes. In
children. It is associated with numerous GI such disease a blood smear can lead to a di-
complications, most classically intussusception agnosis, but without any other symptoms the
(typically ileo-ileal in HSP versus the more possibility of a blood disorder is unlikely in our
common terminal ileum, ileocecal valve, and patient.
ascending colon involvement seen otherwise),
Answer B is incorrect. CT scan of the abdo-
but also pancreatitis and cholecystitis.
men can show increased density of the liver
Answer A is incorrect. Hemoptysis would due to iron deposition, but this test is not used
be expected in Wegeners granulomatosis or to diagnose hemochromatosis.
Goodpastures syndrome, both inammatory
Answer C is incorrect. Endoscopic retrograde
diseases presenting with symptoms of nephritic
cholangiopancreatography is used in the diag-
syndrome such as edema and hypertension.
nosis of choledocholithiasis, tissue sampling in
Answer B is incorrect. Patients with postin- patients with pancreatic or biliary cancer, diag-
fectious glomerulonephritis often have a high nosing ampullary cancer, and many others. It
antistreptolysin O titer due to association with is not used in the diagnosis of hemochromato-
recent Group A -hemolytic streptococcal in- sis.
fection. These patients present with oliguria,
Answer E is incorrect. A liver biopsy is the
edema, hypertension, and brown urine.
only reliable method for establishing or exclud-
HIGH-YIELD SYSTEMS
Chapter 6: Gastrointestinal Answers 151

ing the presence of liver cirrhosis in the setting gery (even when followed by chemotherapy) it
of hemochromatosis. This is an invasive test is insufcient treatment.
and should not be used as the rst test in the
diagnosis of hemochromatosis. 23. The correct answer is A. The Gram stain
shows gram-negative rods, which make Escheri-
21. The correct answer is E. These large, broad- chia coli the only possible correct answer. The
based polyps carry the highest potential for ma- patients symptoms are characteristic for en-
lignant change. terohemorrhagic E. coli infection. The illness
develops 35 days after infection and is charac-
Answer A is incorrect. Lymphoid polyps are
terized by bloody diarrhea without fever.
inammatory polyps and are non-neoplastic.
Answer B is incorrect. Kaposis sarcoma le-
Answer B is incorrect. Hamartomatous polyps,
sions can be found in the GI tract, where they
such as Peutz-Jeghers polyps, are non-neoplas-
can be asymptomatic or cause symptoms such
tic, although the Peutz-Jeghers syndrome in
as abdominal pain, weight loss, hemorrhage,

Gastrointestinal
which they appear does confer additional risk
and diarrhea. Since Gram stain revealed gram-
for malignancy in other organs including the
negative organisms, Kaposis sarcoma is a less
stomach, breasts, and ovaries.
likely cause of the patients symptoms.
Answer C is incorrect. Tubular adenoma is in-
Answer C is incorrect. Legionella generally
correct, as these polyps less frequently undergo
appears as small coccobacilli that stain weakly
malignant change than villous adenomas do.
gram-negative, and the infection presents clini-
Answer D is incorrect. Tubulovillous ad- cally as a pneumonia with high fevers and ac-
enomata have less malignant potential than companying GI symptoms.
their solely villous counterparts.
Answer D is incorrect. Mycobacterium avium
22. The correct answer is A. Given that the lesion complex (MAC) infection can involve the
is <10.0 cm (<3.9 in) from the anal verge, it is large bowel and cause symptoms similar to this
not possible to preserve the anal sphincter, and patients. MAC infection, however, is unlikely
surgical treatment requires an abdomino- with a CD4+ cell count >200/mm; it is more
perineal resection, which involves resection of common with a CD4+ cell count <50/mm.
the rectum and anus with placement of a per- Answer E is incorrect. Mycobacterium tuber-
manent colostomy. culosis infection can involve the GI tract in pa-
Answer B is incorrect. Imatinib, an inhibitor tients with AIDS. This is unlikely, however, in
of certain oncogenic tyrosine kinases, is useful patients with CD4+ cell counts >200/mm.
for GI stromal tumors. A rectal lesion of this
24. The correct answer is B. Spasms of the esopha-
type requires surgical resection.
gus are characterized by both solids and liquids,
Answer C is incorrect. If the lesion had been causing odynophagia and dysphagia, as well as
>10.0 cm (<3.9 in) from the anal verge, it noncardiac angina. Globus, or the feeling of
would have likely been possible to perform a food stuck in ones throat, is also very common.
low anterior resection, which would preserve Nitroglycerin may actually confuse the diagno-
the anus and distal rectum, allowing for resec- sis because it acts to relax the smooth muscle,
tion and primary anastomosis, thereby avoiding thereby relieving the pain. X-rays may be help-
the need for a colostomy. ful in diagnosis by showing what is known as a
Answer D is incorrect. Radiation is a useful corkscrew formation of the esophagus. The
adjunct to surgical resection in rectal cancer anatomy of the esophagus may be divided into
but is not adequate as the only treatment mo- three parts, and when these three do not con-
dality. tract in a uniform manner as with spasms, then
a food bolus may become trapped and cause
Answer E is incorrect. Radiation is used as an pain. Manometry establishes the diagnosis by
adjuvant to surgical resection, but without sur- showing these uncoordinated contractions.
HIGH-YIELD SYSTEMS
152 Section I: Organ Systems Answers

Answer A is incorrect. Although this patient nous uids and a nasogastric tube for bowel de-
has had cancer in the past, she does not seem compression), this patient has peritoneal signs
to be suffering from it again. Her symptoms do and should therefore be taken to the operating
not indicate any such etiology. room for surgical management.
Answer C is incorrect. Usually, cancer will Answer D is incorrect. The clinical history,
cause dysphagia only for solids and may not physical examination, and radiographic nd-
cause the pain this patient is experiencing un- ings are diagnostic of a small bowel obstruc-
less it has spread beyond the walls of the esoph- tion. Further work-up with an MRI is not
agus. indicated at this time. Since this patient has de-
veloped peritoneal signs, he should be taken to
Answer D is incorrect. A myocardial infarc-
the operating room for surgical management.
tion will not present with the given history of
progressive dysphagia. Remember that chest Answer E is incorrect. The clinical history,
pain does not always translate into a cardiac physical examination, and radiographic nd-
Gastrointestinal

etiology. ings are diagnostic of a small bowel obstruc-


tion. Further work-up with an ultrasound is not
Answer E is incorrect. Very similar to spasms,
indicated at this time. Since this patient has de-
nutcracker esophagus differs in the fact that it
veloped peritoneal signs, he should be taken to
is characterized by continuous, coordinated
the operating room for surgical management.
contractions on manometry. This difference is
important with treatment because spasms may
26. The correct answer is D. Dysphagia is the
ultimately be treated with a myotomy, while
number one symptom of achalasia, which is
nutcracker cannot.
characterized by a neuronal decit that results
in incomplete lower esophageal sphincter re-
25. The correct answer is C. This patient presents
laxation. It affects both solids and liquids. Her
with symptoms of a small bowel obstruction
weight loss is almost assuredly due to this prob-
with peritoneal signs and thus needs surgical
lem. The birds-beak appearance seen on
management. His presenting symptoms along
barium swallow is due to the dilated distal
with the radiographic ndings support this di-
esophagus. Manometry is the denitive study
agnosis. Small bowel obstruction is the most
and the results are as described in the question.
common indication for surgery in a patient
Pneumatic dilation works in 80%90% of pa-
with Crohns disease. Although patients with
tients, though it may cause perforation. Botuli-
incomplete small bowel obstructions can be
num toxin also may be given, as can anticho-
initially treated conservatively with bowel rest,
linergic agents, calcium channel blockers, and
a nasogastric tube, and intravenous uids, pa-
prostaglandins.
tients who develop peritoneal signs with a
small bowel obstruction should be taken to the Answer A is incorrect. Cholinergic agents
operating room for surgical management. such as pilocarpine and bethanechol are not
indicated for treatment of achalasia. In fact,
Answer A is incorrect. Like all patients with
there is minimal evidence that anticholinergic
small bowel obstruction, this patient should
agents may be of some benet, though this is
be put on bowel rest, resuscitated with intra-
largely anecdotal. Few patients are managed
venous uids, and have a nasogastric tube
successfully with exclusively medical manage-
placed. However, since he has developed peri-
ment.
toneal signs, he should be taken to the operat-
ing room for surgical management. Answer B is incorrect. Although this patient
has a history of diabetes, dysphagia is rarely a
Answer B is incorrect. Although some patients
symptom. Her hemoglobin A1c level is also be-
with small bowel obstructions can be initially
low the cut-off.
treated with conservative measures (intrave-
Answer C is incorrect. These are all remedies
for reux esophagitis, not achalasia. Patients
with reux typically present with complaints
HIGH-YIELD SYSTEMS
Chapter 6: Gastrointestinal Answers 153

of indigestion, regurgitation, an acidic taste in are suspected. Treatment is typically support-


the mouth, or chronic cough. Progressive dys- ive, involving food restriction, uid resuscita-
phagia should raise suspicions of an anatomic tion, and further investigation into underlying
or functional defect in peristalsis and lower causes such as alcoholism or cholic sources.
esophageal sphincter function. Manometry in
Answer B is incorrect. Amyloidosis is not a
reux generally shows decreased lower esopha-
complication of hemochromatosis. Amyloido-
geal sphincter tone.
sis can present in a wide variety of ways, de-
Answer E is incorrect. This patient does not pending on which organ system is involved.
have diverticuli, which would be seen on bar- However, the most common complications
ium swallow. An uncommon esophageal di- are renal, so symptoms of renal failure such as
verticulum that is nonetheless worth knowing peripheral edema typically present rst. Other
for the Step 1 exam is Zenkers diverticulum, symptoms include coronary heart failure, pe-
which is an outpouching of the esophageal ripheral neuropathy, arthritis, and carpal tun-
mucosa between the cricopharyngeus and the nel syndrome. Denitive diagnosis is typically

Gastrointestinal
lower inferior constrictor. It forms after years through biopsy of the involved organ system,
of high intrapharyngeal pressure during swal- although CT imaging may also show some
lowing (possibly secondary to upper esopha- hints of disease.
geal sphincter dysfunction) and usually doesnt
Answer C is incorrect. Bone marrow failure
present until the patients seventh decade. It
is not a known complication of hemochroma-
predisposes patients to aspiration pneumonia,
tosis. Patients with bone marrow failure pre-
and should be treated surgically.
sent with low blood counts. The types of cell
counts that are low depend on which cell lines
27. The correct answer is D. Hemochromatosis is
have failed. Symptoms of fatigue and malaise
the pathologic accumulation of iron in paren-
typically follow from red cell aplasia, fever and
chymal organs. The patient in question has as-
other signs of infection from neutropenia, and
cites, a coagulopathy (evident from the exten-
bruising and bleeding from thrombocytopenia.
sive bruising and elevated prothrombin time
Diagnosis can be done through laboratory tests
and partial thromboplastin time), and a nodu-
such as a complete blood count with differen-
lar, shrunken liver on ultrasound giving her a
tial, but the gold standard test is bone marrow
diagnosis of liver cirrhosis. Additionally, with
biopsy. Treatment is with bone marrow trans-
her history of diabetes, coronary heart failure,
plantation, although treatment with antithy-
and skin pigmentation changes, the diagnosis
mocyte globulin or antilymphocyte globulin
of hemochromatosis is likely. Other potential
can be used for severe aplastic anemia.
symptoms include hypogonadism, fatigue, and
arthralgias. The diagnosis of hemochromatosis Answer E is incorrect. Although several blood
is conrmed by Perls Prussian blue stain of a diseases that result in iron overload disease can
liver biopsy. Hepatocellular carcinoma is a late cause splenomegaly, hemochromatosis does
sequela in these patients, especially if they are not cause splenomegaly.
cirrhotic on presentation. Treatment is through
phlebotomy and restriction of iron intake. 28. The correct answer is B. Meconium ileus is
nearly pathognomonic for cystic brosis (CF)
Answer A is incorrect. Acute pancreatitis is
but can less commonly be caused by volvulus,
not a likely complication of hemochromato-
intestinal pseudoobstruction, or other rarer
sis, although some reports show that hemo-
causes of pancreatic insufciency. CF is espe-
chromatosis can cause chronic pancreatitis.
cially likely in this case with a positive family
Acute pancreatitis typically presents as acute
history for a sibling with meconium ileus.
epigastric pain radiating to the back, often as-
Meconium ileus occurs in CF because of a de-
sociated with nausea, emesis, and fever. Diag-
ciency in pancreatic secretions, which causes
nosis is usually through abnormal amylase and
the meconium to become mucilaginous and
lipase levels, although CT imaging can also be
viscid, sticking to the walls of the intestine.
useful, particularly if abscesses or pseudocysts
HIGH-YIELD SYSTEMS
154 Section I: Organ Systems Answers

Answer A is incorrect. Congenital agangli- used to assess the gallbladder and biliary tree
onosis of the colon (Hirschsprungs disease) for obstruction. It is most useful in acute chole-
typically presents with constipation in the rst cystitis and would not provide information
month of life following the normal passage of about the patients suspected bowel obstruc-
meconium. tion, although it might demonstrate a biliary-
enteric stula.
Answer C is incorrect. Intussusception occurs
in infants 510 months old and presents with
30. The correct answer is A. For perforated appen-
intermittent, colicky abdominal pain. Radio-
dicitis with no clear loculations, the recom-
graphs typically show small bowel obstruction
mended management is emergent appendec-
with absence of gas in the right colon.
tomy with postoperative intravenous antibiotics
Answer D is incorrect. Total absence of the and delayed primary closure.
small bowel is a rare condition typically associ-
Answer B is incorrect. Bowel rest with noth-
ated with volvulus and gastroschisis.
ing-by-mouth status and intravenous hydration
Gastrointestinal

Answer E is incorrect. Volvulus may occur are appropriate in the setting of acute pancrea-
shortly after birth but presents with bilious titis, not acute appendicitis.
vomiting and poor feeding.
Answer C is incorrect. Percutaneous drainage
and interval appendectomy are appropriate for
29. The correct answer is E. Gallstone ileus is an
perforated appendicitis with a well-loculated
unusual complication of cholelithiasis and oc-
abscess in patients who may need temporiz-
curs when a gallstone enters the bowel through
ing measures or have contraindications such as
a biliary-enteric stula and creates an obstruc-
cardiovascular instability. Our patient is stable
tion, usually at the ileocecal valve. In all pa-
and does not have a loculated abscess, and
tients with suspected small bowel obstruction,
therefore percutaneous drainage is unlikely to
rapidly obtainable supine and erect plain lms
be the primary management for this patient.
are the rst diagnostic test of choice. They may
demonstrate air under the diaphragm due to Answer D is incorrect. Serial abdominal ex-
perforation, air in the biliary tree, or signs of aminations along with antimicrobial treatment
partial or complete obstruction. Direct visual- are appropriate management steps for divertic-
ization of the gallstone tends to occur only ulitis, not appendicitis.
15% of the time because most gallstones are ra-
Answer E is incorrect. The patients pain is
diolucent. CT of the abdomen is a useful con-
more typical of acute appendicitis and not an
rmatory test in these patients and would pro-
acute myocardial infarction, thus ECG and
vide better anatomic detail prior to surgery.
cardiac enzymes would not be appropriate.
Answer A is incorrect. Abdominal ultrasound
may demonstrate enterobiliary stulas but is 31. The correct answer is D. Patients with partial
limited due to reection from bowel gas. small bowel obstruction may complain of
crampy abdominal pain, with or without pas-
Answer B is incorrect. The diagnosis of gall-
sage of feces or atus. Those with malignan-
stone ileus is made preoperatively in about
cies, hernias, or previous intra-abdominal sur-
50% of patients and is often made at laparo-
geries may be prone to bowel obstructions.
tomy or laparoscopy. However, noninvasive
Presentation may include fever, tachycardia,
diagnostic tests, such as plain x-ray and CT,
hypotension, dry mucous membranes, high
should be attempted rst.
pitched or hypoactive bowel sounds, and ab-
Answer C is incorrect. Endoscopic retrograde dominal tenderness or distention. For a partial
cholangiopancreatography is only useful in small bowel obstruction, supportive care may
demonstrating a stula if the gallbladder is able be enough under close supervision. Nasogas-
to be lled at the time of diagnosis. tric suction should be used for decompression,
and the patient should be maintained on noth-
Answer D is incorrect. Hepatic iminodiacetic
ing by mouth with intravenous hydration and
acid scan is a nuclear medicine study that is
correction of electrolyte abnormalities.
HIGH-YIELD SYSTEMS
Chapter 6: Gastrointestinal Answers 155

Answer A is incorrect. The patient does not gut. Mesenteric ischemia may result from either
have signs of infection or necrotic bowel. With thrombosis of the mesenteric vessels, embolism,
infection or peritonitis, the patient will pre- or nonexclusive causes (such as shock). This di-
sent with abdominal wall tenderness and rigid- agnosis may result in life-threatening intestinal
ity, rebound tenderness, and decreased bowel infarction and should be treated as a surgical
sounds. These patients will most likely mini- emergency. The patient should be stabilized
mize their movement to decrease pain. Mes- with intravenous uid resuscitation along with
enteric ischemia, which can lead to necrotic heparin and antibiotics and be taken to the op-
bowel, will present with acute abdominal pain erating room for an exploratory laparotomy so
and pain out of proportion to examination re- the nonviable bowel can be resected.
sults. Look for risk factors such as atrial bril-
Answer A is incorrect. Mesenteric ischemia is
lation, congestive heart failure, peripheral vas-
a surgical emergency that must be diagnosed
cular disease, or hypercoagulability. Diagnosis
and treated immediately. Barium enema is
is made with an angiogram, which identies
useful in identifying colon pathology. There

Gastrointestinal
vascular compromise.
is no utility in performing a barium enema in
Answer B is incorrect. Colonoscopy may be the diagnosis of mesenteric ischemia. Because
helpful in a stable patient with large bowel ob- this patients clinical presentation and medical
struction, but it is inappropriate for evaluating history support a diagnosis of mesenteric isch-
small bowel obstructions. Clinical symptoms emia, colon pathology leading to GI bleed and
are similar to those of small bowel obstruction, diarrhea/vomiting is less likely.
but x-ray of the abdomen may demonstrate di-
Answer B is incorrect. This patient presents
lated loops of small bowel.
with a surgical emergency. Mesenteric ischemia
Answer C is incorrect. Complete small bowel has a mortality rate of 80%, and therefore early
obstruction will present with vomiting and diagnosis and intervention must be undertaken.
crampy abdominal pain with or without pas- Mesenteric ischemia often involves the superior
sage of feces or atus, since 1224 hours are mesenteric artery and therefore compromises
usually required before the colon has become blood ow to the small intestine. A colonoscopy
vacant. Surgery is appropriate for complete would not be useful in diagnosing mesenteric
small bowel obstruction, small bowel obstruc- ischemia because the small intestine cannot be
tion with vascular compromise such as necrotic evaluated. Once the patient is stabilized, she
bowel, or symptoms lasting at least 3 days with- should be taken to the operating room for a lap-
out resolution. arotomy and resection of necrotic bowel.
Answer E is incorrect. While serial abdominal Answer D is incorrect. Mesenteric ischemia
examinations are useful to mark the progress may lead to small bowel obstruction. An ob-
of patients recovering from partial small bowel struction series is not usually obtained in a pa-
obstruction, such examinations alone are inap- tient who presents with a highly suspicious di-
propriate for management because the bowels agnosis of mesenteric ischemia. This diagnosis
still need to be decompressed and relieved of is a surgical emergency and needs immediate
volume load by nasogastric suction and nothing- intervention to reduce mortality.
by-mouth status.
Answer E is incorrect. Patients presenting with
mesenteric ischemia due to an embolus may
32. The correct answer is C. This patient has a
be started on heparin therapy. Initiation of war-
classic presentation of mesenteric ischemia.
farin is associated with a transient hypercoagu-
This diagnosis should remain high on the differ-
lable state and therefore would not be useful in
ential for a patient who presents with abdominal
emergently treating mesenteric ischemia.
pain out of proportion to physical ndings along
with GI tract bleeding, a history of atrial brilla-
33. The correct answer is D. Intussusception is the
tion or vascular disease. This patient also pres-
most common abdominal emergency in chil-
ents a history of intestinal angina to further
dren <2 years old, and the second most com-
the possibility of occlusive arterial disease to the
mon cause of intestinal obstruction (pyloric
HIGH-YIELD SYSTEMS
156 Section I: Organ Systems Answers

stenosis is rst). Intussusception occurs most sue within the diverticulum. Intussusception
commonly near the ileocolic junction, and 75% associated with Meckels is often seen in older
of these cases are idiopathic. Mucosal bleeding boys.
leads to bloody, sometimes mucus-containing,
Answer F is incorrect. Pyloric stenosis often
stool known as currant-jelly stools. The onset
presents at age 36 weeks with nonbilious pro-
is typically abrupt with severe, recurrent, colicky
jectile vomiting immediately after meals. Pa-
abdominal pain. Episodes typically last 1520
tients exhibit a characteristic desire to be re-fed
minutes and the child may act normally be-
soon after (hungry vomiter). On physical ex-
tween episodes. Nonbloody nonbilious emesis
amination, an olive-like mass may be palpated
may also be observed. As the intussusception
in the mid-epigastrium. Laboratory tests may
worsens, the emesis may become bilious.
show a hypochloremic metabolic alkalosis due
Answer A is incorrect. CF is a fatal autosomal to a loss of gastric hydrochloric acid and uid
recessive disease that is more common in white from vomiting.
people. CF may have respiratory, GI, sinus,
Gastrointestinal

and/or musculoskeletal manifestations and can 34. The correct answer is C. It is recommended
cause infertility. Pancreatic insufciency affects that if a patient has numerous large adenomas
85% of patients with CF, and leads to steator- removed from the colon, he or she should re-
rhea with the passage of bulky, foul-smelling, turn for colonoscopy in 3 years.
oating stools. This thick stool can act as a lead
Answer A is incorrect. Those who have had
point for intussusception in patients with CF. ulcerative colitis for >10 years or those with
Answer B is incorrect. Enterocolitis is inam- familial adenomatous polyposis are candidates
mation of the mucosa of the small and large for prophylactic colectomy. This patients situ-
intestine, which may result from infection, ation does not warrant such treatment.
diet, or other etiologies. Intussusception may Answer B is incorrect. Ten years is the regular
complicate enterocolitis; however, there is usu- interval for routine screening colonoscopies. In
ally diarrhea and less severe, less regular pain this situation, with the presence of numerous
associated with enterocolitis. The child also large adenomas, waiting 10 years is too long,
remains noticeably ill between episodes of ab- since that would be enough time for cancer to
dominal pain. develop and then spread.
Answer C is incorrect. HSP is a systemic vas- Answer D is incorrect. Ten years is too long to
culitic syndrome characterized by the deposi- wait in this situation. Moreover, the less sensi-
tion of IgA-containing immune complexes in tive nature of sigmoidoscopies leads to their use
tissues. There is a classic tetrad of rash (palpa- in screening more often than every 10 years.
ble purpura), arthralgias, abdominal pain (of-
ten colicky, associated with emesis), and renal Answer E is incorrect. This is not the correct
disease that can manifest over days to several answer, as this intervention is not appropriate
weeks in any order. Intussusception is a rare for this patients presentation. Urgent colec-
complication of HSP that may result from in- tomy is generally performed in cases of acute
testinal hemorrhage and hematoma formation colitis secondary to ulcerative colitis, Crohns
serving as a lead point. colitis, colonic inertia, and familial adenoma-
tous polyposis. Colonic ischemia and perfora-
Answer E is incorrect. A Meckels diverticu- tion are also potential indications.
lum is a vestigial remnant of the embryonic
yolk sac, typically seen as an outpouching of 35. The correct answer is C. Clostridium difcile
the ileum. Intussusception may also compli- colitis should be suspected in hospitalized pa-
cate Meckels diverticulum; however, these di- tients receiving antibiotic therapy who present
verticuli typically manifest with painless rectal with watery, nonbloody diarrhea and abdomi-
bleeding within the rst 2 years of life, second- nal cramping. Antibiotic therapy, especially
ary to mucosal ulceration of ectopic gastric tis- with cephalosporins, ampicillin/amoxicillin,
HIGH-YIELD SYSTEMS
Chapter 6: Gastrointestinal Answers 157

and clindamycin, may cause a disruption of Answer E is incorrect. Viral invasion and dam-
the normal bacterial ora of the colon, allow- age of villous epithelial cells is the mechanism
ing for colonization with C. difcile. C. difcile of action for some viral causes of diarrhea.
produces two types of toxins. Toxin A is an en- Common pathogens include both the rotavirus
terotoxin that can bind to mucosal cells and and the Norwalk virus. This is usually also a
cause hypersecretion of isotonic uid at a rate self-limiting process for which supportive care
greater than the colon is able to reabsorb. with appropriate uid and electrolyte replace-
Toxin B is a cytotoxin that can cause severe ment is necessary.
mucosal damage and produce a pseudomem-
branous appearance. In mild cases of colitis, 36. The correct answer is D. Tracheoesophageal
cessation of the causative antibiotics may be stula (TEF) is a relatively common congeni-
sufcient to allow for reconstitution of normal tal anomaly involving the respiratory tract. It
colonic ora. For more persistent or severe occurs in approximately 1 in 4000 live births.
cases, oral metronidazole or vancomycin is TEF occurs with esophageal atresia (EA) in

Gastrointestinal
equally effective. >90% of cases. In the most common form of
TEF and EA, the esophagus ends in a blind
Answer A is incorrect. Ingestion of preformed
pouch, and the TEF is connected to the distal
enterotoxins, cytotoxins, and/or neurotoxins
esophagus. TEF and EA result from a defect in
may occur in cases of food poisoning. Bacteria
lateral septation of the foregut into the esopha-
such as Staphylococcus aureus (which is com-
gus and trachea. TEF and EA often occur with
mon in foods like potato salad and custards)
the VACTERL (Vertebral, Anorectal, Cardio-
and Bacillus cereus (which is often found in
vascular, Tracheal, Esophageal, Renal, and
reheated rice) produce preformed toxins with
Limb abnormalities) syndrome, as seen in this
short incubation periods of 26 hours. As a re-
patient with the holosystolic murmur of a ven-
sult, infection with these organisms is charac-
tricular septal defect and limb abnormalities.
terized by immediate, prominent vomiting in
In cases with EA, there is polyhydramnios in
addition to diarrhea.
utero. These babies present with excessive se-
Answer B is incorrect. Intra-abdominal pro- cretions that cause frothing at the mouth and
duction of cytotoxins is the mechanism of ac- nose, drooling, choking, cyanosis, and respira-
tion for certain types of destructive colitis. tory distress immediately after birth. If the tra-
Bacteria such as Shigella dysenteriae produce chea is connected to the distal esophagus, gas-
soluble factors known as cytotoxins, which di- tric distention can occur. The symptoms of
rectly destroy mucosal epithelial cells. Like respiratory distress are exacerbated by feeding.
other toxin-induced diarrheas, these tend to In the presence of EA, a naso- or orogastric
have short incubation periods and brief dura- tube can be passed only 1015 cm down the
tions. airway and cannot be passed into the stomach.
As shown in the image, the tube coils in the
Answer D is incorrect. Intra-abdominal pro-
upper esophageal pouch. With a distal TEF, a
duction of enterotoxins is the mechanism of
distended gas-lled stomach is seen on x-ray of
action for certain secretory diarrheas. Bacteria
the chest, and the TEF may be seen on a lat-
such as Vibrio cholerae and enterotoxigenic
eral view. Fluoroscopy can conrm the diagno-
Escherichia coli produce exotoxins that bind to
sis of EA.
the mucosal cells of the gut and cause hyper-
secretion of copious amounts of isotonic uid Answer A is incorrect. Congenital diaphrag-
leading to what can be massive intestinal uid matic hernia (CDH) results from a defect in
losses, resulting in saline depletion, base decit the diaphragm that allows herniation of ab-
acidosis, and potassium deciency. The disease dominal contents into the thorax. Approxi-
is usually self-limited, and vigilant uid and mately 85% of the defects occur on the left. In-
electrolyte replacement during this time is of testinal malrotation and pulmonary hypoplasia
vital importance. are associated with almost all cases of CDH.
Most affected infants present within the rst
few hours of life with severe respiratory distress.
HIGH-YIELD SYSTEMS
158 Section I: Organ Systems Answers

They also have a scaphoid abdomen, absence lung uid in the fetus. It is more common in
of breath sounds, and shifted heart sounds. In infants of mothers who are diabetic or asth-
left-sided CDH, bowel sounds are heard over matic. Transient tachypnea of the newborn
the left hemithorax. CDH is usually diagnosed presents with an early onset of tachypnea, usu-
in utero by prenatal ultrasound. X-ray of the ally within 2 hours of delivery, which may be
chest postnatally shows the intestine passing associated with grunting and retractions. Oc-
through the diaphragm and conrms the diag- casionally, cyanosis also occurs but is relieved
nosis. with administration of >40% oxygen. Mild hy-
poxemia, hypercapnia, or acidosis may be pres-
Answer B is incorrect. Pyloric stenosis often
ent on laboratory evaluation. X-ray of the chest
presents at age 36 weeks with nonbilious pro-
shows perihilar streaking in the interlobular
jectile vomiting immediately after meals and a
ssures due to excess lung uid lling up the
desire to be fed again soon after (hungry vom-
alveoli and spilling over into the extraalveolar
iter). On physical examination, an olive-like
interstitium, especially the perivascular tissue
mass may be palpated in the midepigastrium.
Gastrointestinal

and interlobar ssures.


Laboratory values can show a hypochloremic
metabolic alkalosis due to a loss of gastric hy-
37. The correct answer is A. It is considered
drochloric acid and uid from vomiting. The
pathologic for a full-term infant not to pass
diagnosis of pyloric stenosis can be conrmed
meconium within the rst 48 hours of life.
by ultrasound or upper GI study.
Hirschsprungs disease, or congenital megaco-
Answer C is incorrect. Neonatal respiratory lon, usually presents with delayed passage of
distress syndrome (NRDS) most often occurs meconium and abdominal distention. This is a
in premature infants with an incidence that is condition that arises when neural crest cells
inversely proportional to gestational age and fail to migrate to the colon during embryonic
birth weight. Infants of diabetic mothers, deliv- development, leaving the patient without peri-
ery at <37 weeks gestation, delivery via cesar- stalsis. A barium enema that shows dilated
ean section, precipitous delivery, asphyxia, and proximal bowel and a narrowed distal segment
multigestational pregnancies are associated is highly suggestive of Hirschsprungs disease.
with an increased incidence of NRDS. NRDS The lack of ganglion cells on rectal biopsy is
results from surfactant deciency that leads to also diagnostic of Hirschsprungs disease.
high surface tension, collapse of alveoli, and
Answer B is incorrect. A volvulus with mal-
less compliant lungs. NRDS presents within
rotation is the most critical cause of bowel
minutes of birth with tachypnea (respiratory
obstruction. It presents in the rst week of life
rate >60/min), grunting, nasal aring, retrac-
with bilious emesis and constipation. An upper
tions, shallow rapid breathing, and cyanosis.
GI series with an absent or abnormal position
The cyanosis is progressive and is minimally
of the ligament of Treitz conrms the diagnosis
responsive to oxygen therapy. Tubular breath
of malrotation.
sounds may be present, and rales may be heard
at the lung bases on inspiration. Laboratory Answer C is incorrect. A double-bubble sign
tests initially show a respiratory and metabolic (air bubbles in the stomach and duodenum) is
acidosis. Hypoxemia and hypercapnia may diagnostic of duodenal atresia, a condition clas-
develop. X-ray of the chest shows lung paren- sically associated with Downs syndrome. Duo-
chyma with a reticulogranular ground-glass denal atresia can cause constipation, but that is
appearance and air bronchograms, especially usually after meconium has been passed. Duo-
in the left lower lobe because of the cardiac denal atresia presents clinically with bilious
shadow. emesis within the rst few hours of life. Physi-
cal examination is notable for abdominal dis-
Answer E is incorrect. Transient tachypnea of
tension of predominantly the upper abdomen.
the newborn is a benign self-limited condition
that usually occurs after a normal vaginal or Answer D is incorrect. A positive sweat test
cesarean delivery of a preterm or term infant. is the denitive way to diagnose CF. Delayed
It is believed to result from slow absorption of passage of meconium for >48 hours can be an
HIGH-YIELD SYSTEMS
Chapter 6: Gastrointestinal Answers 159

initial sign of CF, but there are no dilated or ve percent of these defects are accurately di-
narrowed loops of bowel in patients with CF. It agnosed on prenatal ultrasound. Ten percent
is important to know that most patients (50%) of infants with omphalocele have Beckwith-
usually present with failure to thrive or respi- Wiedemann syndrome (exophthalmos, macro-
ratory compromise. Additionally, CF is most glossia, gigantism, hyperinsulinemia, and hy-
prevalent in white people. poglycemia). The exposed intestine should be
wrapped sterilely to minimize insensible uid
Answer E is incorrect. Intussusception is a
losses and loss of heat. An oro- or nasogastric
common cause of bowel obstruction in infants
tube should be inserted to decompress the
<2 years old. It presents with bouts of irritabil-
stomach, the airway should be stabilized, and
ity, colicky abdominal pain, and vomiting. If it
peripheral intravenous access should be estab-
is not diagnosed at this point, infants become
lished for initiation of uids and antibiotics.
lethargic and pass red currant jelly-like stools.
The omphalocele is emergently repaired with
Physical exam is sometimes notable for a pal-
surgery.
pable right lower quadrant mass. Air contrast

Gastrointestinal
barium enema showing telescoping of the Answer A is incorrect. Duodenal atresia is
bowel is both diagnostic and therapeutic for in- complete obstruction of the lumen of the duo-
tussusception. denum. It is a cause of bowel obstruction dur-
ing the neonatal period. Up to 30% of patients
38. The correct answer is E. Technetium-labeled with duodenal atresia have Downs syndrome.
RBCs can detect bleeding rates as low as 0.1 Other congenital abnormalities have also been
mL/min, although higher rates provide for op- associated with duodenal atresia. It typically
timal localization. No other method can detect presents with bilious emesis on the rst day of
bleeding rates lower than this. After administra- life. Polyhydramnios in utero is associated with
tion of the tagged RBCs, the body is scanned 50% of cases, and one third of the cases are as-
to detect the location of the bleeding. sociated with jaundice. A double-bubble sign
is seen on x-ray of the abdomen due to gas-
Answer A is incorrect. Angiography can detect
eous distention of the stomach and proximal
bleeding rates as low as 0.5 mL/min. This is a
duodenum. Initial management includes with-
higher rate than a tagged RBC scan can detect,
holding feedings, decompressing the stomach
so this would not be the ideal diagnostic test.
by the placement of a naso- or orogastric tube
Answer B is incorrect. Barium enema can to suction, starting intravenous uid replace-
show the architecture of the colon, but would ment, and antibiotics. The duodenal atresia is
not be expected to show colonic bleeding. then repaired surgically.
Answer C is incorrect. Colonoscopy can de- Answer B is incorrect. Gastroschisis is also a
tect gross bleeding but would not pick up the congenital defect that involves herniation of
low level of bleeding that this patient has. abdominal viscera through the abdominal wall
Answer D is incorrect. CT is not the modality next to the umbilicus. There is no sac cover-
used to detect colonic bleeding. ing the herniated viscera. It is typically located
to the right of a normally positioned umbilical
39. The correct answer is E. Omphalocele is a cord. It usually presents with polyhydramnios
congenital defect in which there is herniation in utero and is usually detectable on prenatal
of abdominal viscera through the abdominal ultrasound. The affected infant is typically de-
wall into the base of the umbilical cord. Un- livered spontaneously, preterm, and has mal-
like an umbilical hernia, the viscera herniate rotation, stenosis, or atresia of the GI tract. An
into a sac that is covered by amniotic mem- oro- or nasogastric tube should be inserted to
brane and peritoneum but no skin. Omphalo- decompress the stomach, the airway should be
cele typically presents with polyhydramnios in stabilized, and peripheral intravenous access
utero, and the infant is often born prematurely should be established for initiation of uids
and has other GI and cardiac defects. Ninety- and antibiotics. Emergency surgery is neces-
sary to correct the defect.
HIGH-YIELD SYSTEMS
160 Section I: Organ Systems Answers

Answer C is incorrect. Hirschsprungs disease, monly presents with bloody diarrhea and some-
also known as congenital aganglionic megaco- times is associated with weight loss or abdomi-
lon, results from abnormal innervation of the nal cramping. Extraintestinal manifestations
bowel. It is the most common cause of obstruc- include uveitis, a redness of the lateral sclera
tion of the lower intestine in neonates. It is of- seen in 45% of patients with ulcerative colitis.
ten associated with other congenital defects. It In mild cases, erythrocyte sedimentation rate,
typically presents in full-term infants with de- C-reactive protein, and WBC count may not be
layed passage of meconium (meconium is nor- elevated. Treatment includes aminosalicylates,
mally passed within the rst 48 hours of birth corticosteroids, immunosuppressive drugs, and
in full-term babies), bilious emesis, and ab- tumor necrosis factor inhibitors.
dominal distention. Some affected infants may
Answer A is incorrect. Bacterial enterocolitis
pass meconium normally during the neonatal
is another potential cause of bloody diarrhea. A
period, and then present later with a history of
number of pathogens can cause bloody stools
chronic constipation and failure to thrive. Pa-
including Salmonella, Shigella, and Escheri-
Gastrointestinal

tients with Hirschsprungs have normal anal


chia coli. Other symptoms include fever and
tone on rectal examination, and the examina-
abdominal cramping, and laboratory values
tion is often followed by explosive release of
should show an elevated erythrocyte sedimen-
gas and feces (blast sign). Hirschsprungs may
tation rate, C-reactive protein level, and WBC
be diagnosed by rectal manometry if rectal dis-
count. Mild anemia can be noted if blood loss
tention results in a failure of rectal pressure to
has been signicant enough. These diseases
drop or a paradoxical rise in pressure. Barium
are typically self-limiting and would not be on-
enema may reveal a cone-shaped transition
going for a month.
zone between the normal dilated proximal in-
testine and the smaller-caliber obstructed dis- Answer B is incorrect. Bowel ischemia oc-
tal colon, which results from nonrelaxation of curs when there is a lack of proper blood ow
the aganglionic segment. Absence of ganglion to the intestines; this can be due to a variety of
cells on rectal biopsy is the gold standard for causes. It typically presents with diffuse abdom-
diagnosis. Hirschsprungs disease is treated with inal pain, and late in the course, bloody diar-
surgery. rhea and peritoneal signs from bowel infarction
and necrosis. It is far more common in elderly
Answer D is incorrect. Meckels diverticu-
patients or people with other comorbities; this
lum is a vestigial remnant of the embryonic
patient is young and presumably otherwise
yolk sac, also known as the vitelline or ompha-
healthy. Furthermore, extraintestinal mani-
lomesenteric duct, which is typically seen as
festations are not noted and the time course
an outpouching of the ileum. These diverticuli
would be one of days, not months, as this is a
typically manifest as painless rectal bleeding
deadly disorder if not treated immediately.
within the rst 2 years of life, secondary to mu-
cosal ulceration of ectopic gastric tissue within Answer C is incorrect. Colon carcinoma can
the diverticulum. They can also present with present with bloody diarrhea, but is usually as-
diverticulitis or with intestinal obstruction from sociated with a signicant weight loss, and it
intussusception or volvulus. It is the most com- would be very rare in a person so young.
mon congenital anomaly of the GI tract and
Answer D is incorrect. A Mallory-Weiss tear
has been associated with the rule of 2s: 2
occurs in the esophagus, often as a result of
times as many boys are affected, it often occurs
violent vomiting. It can cause black tarry stools
2 feet from the ileocecal valve, 2 types of mu-
or bloody emesis as the blood is exposed to the
cosa (gastric and pancreatic) may be located
low pH of the stomach contents before passing
within the diverticuli, and it occurs in 2% of
through the anus. Portal hypertension is un-
infants.
likely in this young otherwise healthy woman.
40. The correct answer is E. Ulcerative colitis is
41. The correct answer is C. This patient presents
an inammatory bowel disease that involves
with typical signs and symptoms of diverticular
only the rectum and large intestine. It com-
bleeding. She has signs of hypovolemia and
HIGH-YIELD SYSTEMS
Chapter 6: Gastrointestinal Answers 161

blood loss. The best treatment is to rehydrate dominal pain, anorexia, diarrhea, fatigue, and
the patient and transfuse her to bring her he- fever. The diagnosis is made and conrmed by
matocrit up. Most diverticular bleeding will biopsies obtained by colonoscopy. Exacerba-
stop on its own. tions of Crohns disease are managed medically
with bowel rest, nasogastric suctioning, intrave-
Answer A is incorrect. If the patient has per-
nous uids, and immunosuppressive agents.
sistent bleeding that does not stop on its own,
Surgery is generally not indicated.
this is a reasonable option. But before mov-
ing to this invasive procedure, it is preferable Answer C is incorrect. The leading cause
to treat by hydration and transfusion and see if of painless lower GI bleeding in patients >60
the bleeding will spontaneously stop. years old is diverticulosis. However, this pa-
tients colonoscopy does not show any divertic-
Answer B is incorrect. If there is signicant
ula. Diverticulosis is most often asymptomatic,
bleeding that does not self-resolve and is not
but can present with associated tenesmus, con-
able to be embolized, surgery is the next op-
stipation, or diarrhea. Patients with bleeding

Gastrointestinal
tion. The patient in this question has not gone
from diverticulosis should be stabilized and
through a trial of conservative treatment yet of
evaluated by colonoscopy. Angiography can
rehydration and blood transfusion, so it is in-
usually control bleeding; in the case of massive
correct to go straight to surgery.
bleeding, surgery may be necessary.
Answer D is incorrect. This is appropriate
Answer D is incorrect. Ischemic colitis pre-
treatment for diverticulitis, but this patient
sents typically in older patients with atheroscle-
does not appear to have diverticulitis, as she
rotic disease. It usually occurs spontaneously
is afebrile with no nausea or vomiting, no
and causes hematochezia with crampy pain.
marked abdominal cramping pain, and only a
Ischemic colitis also occurs in approximately
relatively mild leukocytosis. When treating for
5% of patients who undergo surgery for ileoaor-
diverticulitis, the choice of antibiotics would
tic or abdominal aortic aneurysms.
include metronidazole and a second- or third-
generation cephalosporin or uoroquinolone. Answer E is incorrect. Peptic ulcer disease
usually causes epigastric, burning pain, which
Answer E is incorrect. This is a treatment used
can either worsen or improve with food, de-
for people with uncomplicated diverticular
pending on the ulcers location. Bleeding is in
disease. This patient is showing signs of dehy-
the upper GI tract, can be acute or chronic,
dration and anemia and should be treated for
and typically does not present with hematoche-
those conditions. After the patient is stable, she
zia. If this patients history were more sugges-
can be discharged with instructions to eat a
tive (abdominal pain, history of nonsteroidal
high-ber diet.
anti-inammatory drug use, or alcohol use),
peptic ulcer disease could be considered in the
42. The correct answer is A. Angiodysplasia is the
differential diagnosis.
second leading cause of painless lower GI
bleeding in patients >60 years old. The lesion Answer F is incorrect. Ulcerative colitis often
is usually fern-like or spider-like with a ten- leads to hematochezia and is often accompa-
dency to bleed, ranging from occult blood to nied by diarrhea, abdominal pain, tenesmus,
melena to frank hematochezia. Lesions are and urgency. The rectum is almost always in-
typically in the cecum or ascending colon and volved. Neither colonoscopy nor the patients
can be associated with aortic stenosis. Patients history revealed ndings consistent with ulcer-
should rst be stabilized, then treated with ative colitis.
electrocoagulation or embolization through
angiography. 43. The correct answer is B. This patient suffers
from hepatic encephalopathy secondary to
Answer B is incorrect. Crohns disease does
hepatitis C cirrhosis. Hepatic encephalopathy
not typically cause severe bleeding, but frank
is a neuropsychiatric condition caused by the
blood is seen in approximately 30% of patients.
inability of the liver to lter nitrogenous com-
Affected patients are more likely to have ab-
pounds and other toxins out of the systemic cir-
HIGH-YIELD SYSTEMS
162 Section I: Organ Systems Answers

culation. Ammonia is believed to play a role in 83% of patients, respectively. Treatment in-
pathogenesis, but several other toxins have also cludes intravenous antibiotics and uid resus-
been implicated. Symptoms include disturbed citation.
intellectual function, consciousness, and neu-
Answer A is incorrect. Although angiodyspla-
romuscular function (asterixis, hyperreexia,
sia is the most common cause of lower GI tract
and myoclonus). Management strategies in-
bleeding in the elderly, there is no evidence of
clude reducing protein consumption, as well
an arteriovenous malformation on the CT im-
as treating with lactulose and metronidazole.
age. Bleeding from an arteriovenous malforma-
Answer A is incorrect. Because this patient tion tends to be self-limited and painless, with-
shows signs of portal hypertension secondary to out signicant leukocytosis.
hepatitis C virus cirrhosis, esophageal varices
Answer B is incorrect. Carcinoid syndrome
must be considered. However, although all pa-
may present with chronic diarrhea, but fever
tients with signs of portal hypertension should
and leukocytosis would not likely be present.
be monitored for variceal formation, this pa-
Gastrointestinal

tients acute clinical presentation is consistent Answer C is incorrect. Carcinoma of the co-
with hepatic encephalopathy. lon is unlikely to present acutely with diarrhea,
fever, and high leukocytosis.
Answer C is incorrect. Although this patients
likely hepatitis C diagnosis puts her at risk for Answer E is incorrect. Infectious colitis is a
hepatocellular carcinoma, the acute clinical possibility, but given the CT scan showing in-
presentation is most consistent with hepatic amed diverticuli, it is a less likely diagnosis.
encephalopathy.
45. The correct answer is C. GI malabsorption of
Answer D is incorrect. Hepatorenal syndrome, iron is the reason for the patients continued
a diagnosis of exclusion, is dened as a liver iron deciency anemia. Oral iron therapy is
disease, such as cirrhosis, complicated by func-
nonheme iron in the ferric state and needs to
tional renal failure. Glomerular ltration rate
be converted to the ferrous state for proper ab-
is decreased and blood urea nitrogen:creatinine
sorption in the duodenum. As evidenced by
ratio is typically elevated. Although this diagno-
iron deciency anemia in hypochlorhydric
sis must be ruled out in the setting of decom-
states, such as partial gastrectomy, vagotomy,
pensated cirrhosis, this patients presentation is
and atrophic gastritis, gastric acid is necessary
more consistent with hepatic encephalopathy.
for proper iron absorption. Studies suggest that
Answer E is incorrect. Spontaneous bacterial a hypochlorhydric state induced by prolonged
peritonitis is an infection of the ascitic uid, omeprazole use may result in difcult-to-treat
typically with Enterobacteriaceae, pneumo- iron deciency anemia. Since there is no lon-
coccus, or gram-negative bacteria. It typically ger evidence of GI disease, removal of omepra-
occurs in the setting of cirrhosis. Physical ex- zole therapy is the best initial step in manage-
amination ndings can include fever and ab- ment.
dominal pain and tenderness, or the infection
Answer A is incorrect. Bone marrow biopsy
may be silent clinically. The clinical presenta-
could provide the denitive diagnosis of iron
tion, along with an ascitic uid polymorphonu-
deciency anemia by revealing depleted iron
clear neutrophil count >250/mm, is diagnos-
body stores. However, this procedure is inva-
tic of spontaneous bacterial peritonitis.
sive, and a trial of discontinuation of omepra-
zole therapy should be attempted rst.
44. The correct answer is D. The patients CT
scan demonstrates multiple diverticuli of the Answer B is incorrect. Since the patient is no
sigmoid colon. The sigmoid colon is the most longer having evidence of bleeding, it is un-
common site for diverticuli due to the nar- necessary at this time to obtain blood typing
rower diameter of the colon in that region. Left and screening in preparation for transfusion.
lower quadrant abdominal pain is present in
Answer D is incorrect. Darbepoetin ther-
>90% of patients with diverticulitis, while fever
apy is a synthetic form of erythropoietin that
and leukocytosis occur in more than 50% and
HIGH-YIELD SYSTEMS
Chapter 6: Gastrointestinal Answers 163

stimulates the production of new RBCs. It is most likely toxic megacolon caused by Clostrid-
typically used in patients with chronic kidney ium difcile, not a fungus. Candida has been
disease or with chemotherapy in oncology pa- found in stool samples of patients on chronic
tients. antibiotics experiencing diarrhea, though such
patients would not experience the other toxic
Answer E is incorrect. Parenteral iron therapy
effects noted above.
would resolve the issues surrounding malab-
sorption; however, the risk of complications
47. The correct answer is E. This patient is pre-
such as fever, chills, myalgias, and anaphylaxis
senting with signs and symptoms that should
is much greater. A trial of omeprazole discon-
direct the physicians attention to the biliary
tinuation is warranted before switching to par-
tree and, more specically, to the gallbladder.
enteral iron.
She has three of the 4 Fs (Fat, Forty, Fe-
male, and Fertile) for typical cholelithiasis pa-
46. The correct answer is A. This patient has been
tients, and her pain as described is suspicious
on an extended course of antibiotics and is

Gastrointestinal
for biliary colic. On physical examination,
most likely suffering from toxic megacolon.
mild icterus, low fever, positive Murphys sign,
This is diagnosed by three of the following:
and tenderness/guarding in the RUQ should
anemia, an elevated WBC count, tachycardia,
prompt evaluation for acute cholecystitis. The
and a fever. There needs to also be one of the
imaging study of choice in initial evaluation of
following present: change in mental status, de-
acute cholecystitis is an ultrasound of the
hydration, hypotension, and electrolyte abnor-
RUQ. This has been demonstrated to be 95%
malities. A stool sample will most likely reveal
sensitive for stones of any type in the gallblad-
Clostridium difcile toxin, which is the most
der. It can detect stones as small as 3 mm in di-
common culprit in situations of toxic megaco-
ameter, and is excellent at detecting inamma-
lon such as this in which a patient has been ex-
tion in the gallbladder and dilation of intra- and
posed to a long course of an antibiotic active
extrahepatic ducts.
against the gram-negative bacteria in commen-
sal intestinal ora. Answer A is incorrect. CT scanning is not
the rst-line choice for imaging in suspected
Answer B is incorrect. While several gram-
cholecystitis due to its lower sensitivity (vs. ul-
negative organisms including Klebsiella and
trasound), its higher cost, and the involvement
Proteus can cause changes in bowel habits,
of radiation exposure. CT is useful in the eval-
usually diarrhea, this clinical picture is most
uation of neoplasms in the pancreas and hepa-
likely associated with toxic megacolon brought
tobiliary tree.
on by Clostridium difcile.
Answer B is incorrect. Plain x-rays of the ab-
Answer C is incorrect. The most likely cul-
domen are rarely diagnostic for stones because
prit is Clostridium difcile, and its toxin
only 10%15% are radiopaque. This imaging
is most readily picked up on stool sample.
modality is useful in the evaluation of acute
Gram-positive cocci including enterococci
emphysematous cholecystitis, pneumobilia
may prove problematic in the colon in the
secondary to a GI-biliary stula, and small
context of diverticulitis. This condition may
bowel obstruction. However, in this case, these
present with abdominal pain, tenderness in
diagnoses are not likely.
the left lower quadrant, fever, nausea, vomit-
ing, chills, cramping, and constipation. Answer C is incorrect. Hepatobiliary imino-
diacetic acid scanning (radionuclide biliary
Answer D is incorrect. Patients with toxic
scanning) is sensitive and specic for detec-
megacolon often have bloody diarrhea, so we
tion of acute cholecystitis and for detection of
would expect to see blood in the stool. Intesti-
a bile leak after surgery. It is a good technique
nal bleeding results in anemia, one of the diag-
for imaging of the gallbladder; however, it is
nostic criteria for toxic megacolon.
more expensive and time consuming than an
Answer E is incorrect. Spores and hyphae are ultrasound and is therefore used if ultrasound
consistent with a fungal infection, but this is is inconclusive.
HIGH-YIELD SYSTEMS
164 Section I: Organ Systems Answers

Answer D is incorrect. MRI is not commonly hours after consumption and is limited to
used in the evaluation of acute cholecystitis, about 24 hours duration. Emetic illness causes
due to its high cost and the presence of other vomiting within 0.56 hours of consumption
highly effective imaging for this purpose. that lasts <24 hours. Diagnosis rests on identi-
cation of disease-causing serotypes of B. cereus
in the suspect food (classically reheated rice).
Questions 48, 49, and 50
Answer B is incorrect. Campylobacter jejuni
48. The correct answer is K. Lactose intolerance is a curved gram-negative rod that is the lead-
is most prominent in Asians and African- ing cause of bacterial diarrheal illness in the
Americans, and the incidence is lowest in United States. It is often contracted from raw
whites of northern European descent. The chicken or milk and from nonchlorinated wa-
symptoms described occur after ingestion of ter. Symptoms of the illness include watery or
milk or milk-containing foods and reliably stop mucoid diarrhea, which may contain occult
with a lactose-free diet. They are caused by an blood, along with fever, nausea, abdominal
Gastrointestinal

osmotic diarrhea secondary to the inability to pains, and headache. The illness is usually self-
break down lactose (a protein found in milk). limiting and passes within 710 days. C. jejuni
Laboratory ndings may include low fecal pH is identied in stool samples through Gram
or presence of reducing substances in the stool. stain and culture on special media.
However, diagnosis should be conrmed by a
positive lactose breath hydrogen test. Answer C is incorrect. Carcinoid syndrome
is a constellation of symptoms resulting from
49. The correct answer is D. Celiac disease is clas- hormone-producing tumors most commonly
sically a disease of infancy, which presents with found in the gut. The carcinoid cells produce
failure to thrive and malabsorption, associated serotonin derivatives, which are normally
with ingestion of wheat gluten. However, due metabolized to an inactive form by the liver.
to evolving recommendations of when to intro- Carcinoid syndrome occurs when the tu-
duce wheat-containing foods during infancy mor metastasizes to the liver so the hormonal
and early childhood, celiac disease can also products can bypass hepatic metabolism. The
present later in childhood. Histopathology serotonin-like compounds enter the systemic
shows villous atrophy. circulation and cause a number of symptoms,
most commonly secretory diarrhea, ushing,
50. The correct answer is E. Although the presen- and wheezing. Diagnosis is based on urinary
tation of pseudomembranous colitis following 5-hydroxyindoleacetic acid, a breakdown prod-
usage of clindamycin is classic, many different uct of serotonin.
classes of antibiotics can alter the intestinal Answer F is incorrect. Crohns disease is a
ora and lead to overgrowth of Clostridium dif- chronic inammatory disease that affects any
cile. Furthermore, most antibiotic-associated portion of the GI tract. It causes mucoid non-
diarrhea is osmotic in nature and does not re- bloody diarrhea that is recurrent, as well as
sult from C. difcile infection. The mechanism constitutional symptoms and abdominal pain.
involves decreased breakdown of carbohydrates It commonly affects individuals in their 20s
in the gut due to reduced bacterial counts. with another peak in the sixth and seventh de-
This causes a greater concentration of complex cades of life. Intestinal biopsy shows transmural
carbohydrates within the intestinal lumen, inammation with noncaseating granulomas.
leading to higher osmotic pressure and watery
diarrhea. Answer G is incorrect. Entamoeba histolytica
is a protozoan that exists in two life stages: the
Answer A is incorrect. Bacillus cereus is gram- trophozoite and cyst. The trophozoites infect
positive rod that produces exotoxins that are humans and cause disease, while the cysts sur-
very heat stable. It can lead to both diarrheal vive outside the host in water and soil and on
and emetic illness, which are caused by dis- foods. When cysts are swallowed, the protozoa
tinct exotoxins. The diarrheal illness produces
a watery diarrhea with abdominal cramps 615
HIGH-YIELD SYSTEMS
Chapter 6: Gastrointestinal Answers 165

excyst and progress to the trophozoite phase. tion and hyperactivity of N-methyl-D-aspartate
Infection is often subclinical but can present glutamate receptors. Diagnosis is by exclusion of
with bloody mucoid stools. Diagnosis depends other causes of chronic diarrhea.
on nding cysts in three separate stool samples.
Answer L is incorrect. Ulcerative colitis is
Answer H is incorrect. Escherichia coli is a a type of inammatory bowel disease that is
gram-negative bacillus that is part of the nor- limited to the colon and produces a chronic
mal bacterial ora of the gut. There are four bloody diarrhea with crampy abdominal pain.
different classes of E. coli that are recognized Colonic involvement is continuous, involves
to cause disease in humans, each with a differ- the rectum, and progresses proximally. Biopsy
ent mechanism: enteroinvasive, enterohemor- shows a friable mucosa with supercial mi-
rhagic, enterotoxigenic, and enteropathogenic. croulcerations and crypt abscesses.
Disease presents as either watery or bloody di-
Answer M is incorrect. Vibrio parahaemolyti-
arrhea with crampy abdominal pain. Diagnosis
cus is a free-living bacterium that is found in
rests on identifying disease-causing serotypes in

Gastrointestinal
saltwater and is contracted by eating raw sea-
the stool.
food. Gastroenteritis is the most common pre-
Answer I is incorrect. Giardia, similar to En- sentation, with crampy abdominal pain and
tamoeba, has both cystic and trophoblastic often guaiac-positive stool. The disease is gen-
phases of its life cycle, with the trophozoites erally self-limiting and treatment is supportive,
causing disease in humans. The protozoan is with antibiotics considered only for severe in-
contracted through drinking contaminated fection.
water and sometimes through ingestion of raw
Answer N is incorrect. Whipples disease is a
vegetables. Giardia causes a watery nonbloody
systemic illness linked to a gram-positive ba-
diarrhea that is often limited to 12 weeks but
cillus, Tropheryma whippelii. GI manifesta-
may establish itself as a chronic infection in
tions include a malabsorptive syndrome, with
some individuals. Diagnosis depends on visual-
cachexia, distended abdomen, and signs of
ization of the organism in stool samples.
vitamin deciency such as glossitis and night
Answer J is incorrect. Irritable bowel syndrome blindness. Other organ systems may be affected
is a functional GI disorder that can cause ab- such as the central nervous system, heart,
dominal pain and changes in bowel habits. It joints, and skin. Biopsy of the small bowel
commonly occurs in individuals <35 years, and shows expanded villi containing macrophages
postulated etiologic factors include enteric infec- that stain positively with periodic acid-Schiff.
This page intentionally left blank
CHAPTER 7

Hematology/Oncology

167
HIGH-YIELD SYSTEMS
168 Section I: Organ Systems Questions

Q U E ST I O N S

1. A 56-year-old man presents to his physician agulability. What is the underlying reason for
complaining of severe fatigue. He began to feel this?
increasingly tired about 6 months ago, but be-
(A) There is an initial increase in vitamin
lieves that his fatigue has been worsening over
K-dependent coagulation factors
the past 3 weeks. He also notes he has had a
(B) Venous valvular insufciency is exacer-
nonproductive cough for about 2 weeks and
bated during the rst 3 days of warfarin
has experienced several episodes of drenching
therapy
night sweats. On examination he has several
(C) Warfarin causes a more rapid drop in the
large bruises on his extremities but recalls no
levels of proteins C and S than factors II,
injuries. Abdominal examination reveals mas-
Hematology/Oncology

VII, IX, and X


sive enlargement of both the liver and the
(D) Warfarin induces resistance of factor V to
spleen, without any lymphadenopathy. Labora-
degradation by activated protein C
tory studies show:
(E) Warfarin leads to an initial increase in
WBC count: 1200/mm3 platelet aggregation
Neutrophils: 58%
Eosinophils: 7% 3. A 23-year-old woman is diagnosed with stage
Lymphocytes: 30% IIIA Hodgkins lymphoma after an extensive
Monocytes: 0% work-up. As her physician is explaining the rec-
Basophils: 5% ommended treatment regimen for her disease,
RBC count: 3.0/mm3 the patient confesses she has been hoping to
Hemoglobin: 7.5 mg/dL become pregnant in the next few years. How-
Platelet count: 18,000/mm3 ever, she states she is willing to wait until she
has completed her treatment. She wants to
Peripheral blood smear reveals irregular nuclei
know the consequences of both her primary
and cell membranes, as well as cytoplasmic
disease of Hodgkins lymphoma and her treat-
projections. Which of the following is the most
ment regimen, as well as what effects they may
likely diagnosis?
have on a potential pregnancy. Which of the
(A) Acute lymphocytic leukemia following complications is the least likely?
(B) Hairy cell leukemia
(A) An increased risk of second malignancy
(C) Idiopathic thrombocytopenic purpura
(B) Ovarian failure that will preclude preg-
(D) Infectious mononucleosis
nancy
(E) Nodular sclerosing Hodgkins lymphoma
(C) Pericarditis that may cause congestive
heart failure during the pregnancy
2. A 42-year-old woman who smokes tobacco is
(D) Recurrence of the patients disease due to
found to have acute-onset respiratory distress
increased estrogen
and tachycardia 4 hours after a non-emergent
(E) Restrictive lung disease that may be exac-
cholecystectomy. She is subsequently treated
erbated by pregnancy
for a symptomatic pulmonary embolism. She
is begun on low-molecular-weight heparin and
4. A 97-year-old man was admitted to the hospital
warfarin while in the hospital, and is supplied
because of weakness and rectal bleeding for
with subcutaneous doses of low-molecular-
the previous 24 hours. Questioning reveals a
weight heparin to take at home for a total
decrease in stool caliber over the past several
course of 5 days, in addition to the warfarin
months. Examination reveals a nontender mass
that she will take for at least 6 months. The ini-
in the left lower quadrant. Stool heme is posi-
tial 5 days of overlap of both heparin and war-
tive. Serum hemoglobin is 8.8 g/dL. Further
farin is necessary because at the beginning of
work-up reveals colon cancer, Duke stage D.
treatment, warfarin actually leads to hyperco-
HIGH-YIELD SYSTEMS
Chapter 7: Hematology/Oncology Questions 169

The patients spouse and only child are no lon- 6. A 69-year-old woman has been in the intensive
ger living, but the patients 50-year-old grand- care unit for 7 days following complicated hip
daughter pleads with the physician not to in- replacement surgery. The patient is currently
form her grandfather of the diagnosis, stating receiving heparin subcutaneously and wears
that he is an anxious man and that she wants intermittent pneumatic compression devices
him to enjoy his remaining time. Which of the on her lower extremities bilaterally. The pa-
following is the most appropriate next step? tient has developed new-onset right calf pain,
edema, tenderness, and a positive Homans
(A) Agree not to reveal the diagnosis to the pa-
sign. A Doppler ultrasound revealed a deep
tient
vein thrombosis. Her platelet count is 78,000/
(B) Ask the granddaughter to obtain other fam-
mm, and there has been no evidence of spon-
ily members opinions and try to reach
taneous bleeding. Which of the following will
consensus
best help prevent future complications?

Hematology/Oncology
(C) Ask the patient how much information he
wants to know (A) Beginning warfarin therapy
(D) Request a consult from the hospitals eth- (B) Discontinuation of bilateral pneumatic
ics team compression devices
(E) Request psychiatric evaluation of the pa- (C) Discontinuation of heparin
tient (D) Performing venography
(E) Placing an inferior vena cava lter
5. A 10-year-old girl is brought to the pediatrician (F) Transfusing platelets
due to a limp. She has been complaining of fa-
tigue and pain in her right leg for months. 7. A 62-year-old woman has an abnormal Papani-
Physical examination reveals multiple bruises colaou (Pap) smear nding of low-grade
on different sites of her body, enlarged anterior squamous intraepithelial lesion (LSIL) on rou-
cervical and axillary lymph nodes, and hepa- tine screening. She does not have a signicant
tosplenomegaly. A blood smear is shown in the past medical history, and she is immunocom-
image. Which of the following is the best next petent. Her last Pap smear 3 years ago was nor-
step? mal. She has never had an abnormal Pap
smear and has always complied with obtaining
routine screening tests. She is entirely asymp-
tomatic and postmenopausal. She is referred
for colposcopy, and biopsy of a satisfactory
specimen conrms LSIL. What is the most ap-
propriate next step in the management?
(A) Endocervical curettage
(B) Loop electrosurgical excision procedure
(C) Observation consisting of a repeat Pap
smear in 1 year
(D) Observation consisting of colposcopy and
Reproduced, with permission, from Lichtman MA, Beut- repeat Pap smears every 4 months for up to
ler E, Kipps TJ, Seligsohn U, Kaushansky K, Prchal JT. a year
Williams Hematology, 7th edition. New York: McGraw-
Hill, 2006: Plate XX-11.
(E) Repeat colposcopy in 1 year, even if repeat
Pap smear is normal
(A) Call child protective services for suspected
child abuse
(B) Obtain a bone marrow biopsy
(C) Obtain a liver biopsy
(D) Perform a test for serum heterophile anti-
bodies
(E) Recommend a diet higher in vitamin C
HIGH-YIELD SYSTEMS
170 Section I: Organ Systems Questions

8. A 32-year-old woman develops a progressive id- 10. A 58-year-old man with a history of psoriasis
iopathic cardiomyopathy and eventually un- presents for his annual examination. He was
dergoes a cardiac transplant. She is placed on a rst diagnosed with psoriasis 3 years ago and
rigorous protocol for immunosuppression. Two has shown little improvement despite numer-
years later she develops right-sided sinus full- ous therapeutic regimens. His plaques are pri-
ness, and imaging reveals a mass lling the marily distributed around his waist and but-
right paranasal sinus. She is treated with a re- tocks and have recently begun to itch. Skin
duction in immunosuppression and acyclovir. biopsy reveals atypical lymphocytes aligned
She achieves a full recovery. Why is acyclovir along the basal layer and epidermal microab-
part of her therapeutic regimen? scesses (see image). Which of the following is
the most likely diagnosis?
(A) Acyclovir has also been shown to have an-
tineoplastic properties
Hematology/Oncology

(B) Acyclovir is necessary for prophylaxis


against cytomegalovirus in any immuno-
suppressed patient
(C) Acyclovir will reduce the risk of infection
due to radiation of the mucous membranes
and subsequent breakdown
(D) Eradication of Epstein-Barr virus may con-
tribute to resolution of the paranasal sinus
mass
(E) Since the patients immunosuppressive
regimen must be decreased, protection
against viruses that might infect the pa-
tients new heart is particularly important

9. An 18-year-old college freshman presents to


the emergency department after suffering a sei- Image courtesy of Armed Forces Institute of Pathology.
zure at a party. She is accompanied by her (A) Contact dermatitis
friend, who states that the patient had been (B) Eczema
consuming alcohol and had complained of se- (C) Inverse psoriasis
vere abdominal pain and vomited several times (D) Mycosis fungoides
before she began convulsing. The patient is ad- (E) Pityriasis rosea
mitted for observation and work-up of her
symptoms. She has no further seizures and ap- 11. A 47-year-old man presents to his primary care
pears to be recovering well. The next morning physician complaining of fatigue for the past
an observant medical student notes that the several months. He says he has had poor sleep
urine collecting in her Foley catheter bag is due to sweating at night and has lost 4.5 kg (10
reddish-brown. The student double-checks the lb) recently, although he has not been trying to
patients laboratory results and nds that she lose weight. In addition, he has been suffering
has a positive urine toxicology screen that from severe headaches and blurry vision re-
could explain her symptoms. Which of the fol- cently. On examination he is pale and thin,
lowing drugs most likely contributed to pa- with multiple ecchymoses. Cardiac examina-
tients symptoms? tion is signicant for a II/VI systolic ow mur-
(A) Cocaine mur. He has an enlarged spleen on abdominal
(B) Heroin examination. Laboratory tests show:
(C) Marijuana WBC count: 95,000/mm
(D) Methamphetamine 15% blasts
(E) Phenobarbital 15% bands
47% polymorphonuclear cells
HIGH-YIELD SYSTEMS
Chapter 7: Hematology/Oncology Questions 171

7% basophils on her palms and soles and is diagnosed with


10% lymphocytes graft-versus-host disease, which requires long-
Hemoglobin: 7.2 g/dL term therapy. She is discharged home with
Platelet count: 90,000/mm nursing care on trimethoprim-sulfamethox-
azole, glucocorticoids, and tacrolimus. On day
Which of the following is the best next step in
45 she develops spiking fevers and rigors. She
establishing a diagnosis?
states she has been compliant with her pre-
(A) Coagulation studies scribed medications. Given her history, which
(B) Cytogenetic studies of the following describes the most prominent
(C) Detection of serum leukocyte alkaline infectious risk at this time?
phosphatase levels
(A) Acute bacterial infection with Staphylococ-
(D) Iron studies
cus aureus 6 weeks following transplant
(E) No further studies are necessary for estab-

Hematology/Oncology
(B) Fungal infection due to prolonged immu-
lishing the diagnosis
nosuppression
(C) Gram-negative bacteremia in the rst 6
12. A 28-year-old man presents to the clinic com-
plaining of heaviness in his testicle for 2 weeks. weeks following transplant
He says he feels as though his testicle is en- (D) Pneumocystis jiroveci after being treated for
larged. The man has a temperature of 37.2C hematogenous malignancy
(98.9F), heart rate of 60/min, and blood pres- (E) Reactivation of herpes simplex virus in the
sure of 115/70 mm Hg. He has a normal ab- posttransplant period
dominal examination with no palpable masses. (F) Salmonella infection, given patient history
The right testicle is noticeably larger than the of promyelocytic leukemia
left testicle. There are no discrete nodules. Tes-
14. A 45-year-old man presents to his primary care
ticular ultrasound is performed, followed by an
physician with a complaint of patchy dry areas
orchiectomy. He is found to have a seminoma
on his skin. After several trials of skin emol-
and a retroperitoneal lymph node that is en-
lients and corticosteroid therapies without im-
larged at 1.8 cm. He is given a diagnosis of
provement, the lesions are biopsied by a der-
stage IIA testicular seminoma (T2N1M0).
matologist. Pathologic examination reveals a
What additional treatment is needed?
population of atypical mononuclear cells inl-
(A) Contralateral orchiectomy trating the dermis and epidermis. A complete
(B) Platinum-based chemotherapy and bilat- blood cell count is normal, and no atypical
eral orchiectomy cells are observed on peripheral blood smear.
(C) Prophylactic mediastinal radiation The patient states that he feels quite well.
(D) Retroperitoneal lymph node dissection Which of the following is most appropriate as
(E) Retroperitoneal radiation recommended by the patients dermatologist?

13. A 28-year-old woman with a history of refrac- (A) Anticipation of bone marrow transplant
tory promyelocytic leukemia is admitted to the (B) Observation alone
hospital to undergo a bone marrow transplant (C) Systemic chemotherapy
from a matched unrelated donor. In the week (D) Systemic corticosteroids
prior to the transplant she receives myeloabla- (E) Treatment of the affected areas of skin with
tive chemotherapy. She remains stable on anti- topical nitrogen mustard
bacterial prophylaxis for 9 days following her (F) Treatment of the entire skin surface via to-
transplant. On day 10 she develops large bullae tal skin electron beam therapy
HIGH-YIELD SYSTEMS
172 Section I: Organ Systems Questions

15. A 32-year-old woman presents to the emer- 17. A 55-year-old woman with a history of alcohol-
gency department with edema and pain of the ism and chronic pancreatitis (last exacerbation
right lower extremity that began after a 6-hour 2 years ago) presents to her primary care physi-
car ride. A Doppler ultrasound was completed cian with weight loss, pruritus, anorexia, dark
in which a deep vein thrombosis (DVT) was urine, jaundice, yellow sclerae, and vague ab-
noted. The patient has no prior history of DVT dominal pain. Which of the following physical
or pulmonary emboli. The patient has been ndings would most strongly support her most
taking oral contraceptive pills for the past 2 likely diagnosis?
years and is currently compliant with her med-
(A) Chandelier sign
ication. Her family history is signicant for a
(B) Courvoisiers sign
maternal grandmother, mother, and sister with
(C) Cullens sign
recurrent DVT. Her temperature is 36.2C
(D) Grey-Turners sign
(97.2F), blood pressure is 112/78 mm Hg,
Hematology/Oncology

(E) Positive Murphys sign


heart rate is 86/min, and respiratory rate is 14/
min. There is no clinical evidence indicating a 18. A 51-year-old woman undergoes a successful
pulmonary embolism. Which of the following bone marrow transplant from a matched unre-
is the most likely cause of her DVT? lated donor for refractory Hodgkins disease.
(A) Antithrombin deciency She is discharged from the hospital on no med-
(B) Coagulation factor V gene mutation ications and is feeling well. At an appointment
(C) Protein C excess 6 months posttransplant she is well with no evi-
(D) Protein S deciency dence of malignancy. Three weeks later she
(E) Prothrombin gene mutation travels to Florida with her family. She is cau-
tious of the sun but develops sunburn on her
16. Which of the laboratory ndings in the table face, despite wearing sunscreen and a protec-
below are characteristic of hemophilia B? PT tive hat. When she returns from her trip 5 days
refers to prothrombin time and aPTT refers to later she presents with persistent erythema of
activated partial thromboplastin time. her face. She also states her wrists and hands
have been sore for the past 23 weeks. On ex-
(A) A
amination her face is mildly tender to touch,
(B) B
and a rash is present. Her hands are diffusely
(C) C
swollen. She is afebrile, and the remainder of
(D) D
her physical examination is benign. What is
(E) E
the most likely diagnosis?

BLEEDING TIME PLATELET COUNT


CHOICE (min) PT (sec) APTT (sec) (/mm3) FACTOR IX FACTOR VIII

A 3 17 27 202,000 abnormal normal

B 4 12 42 190,000 abnormal normal

C 4 12 45 230,000 normal abnormal

D 8 13 27 58,000 normal normal

E 9 20 28 143,000 normal abnormal


HIGH-YIELD SYSTEMS
Chapter 7: Hematology/Oncology Questions 173

(A) Graft-versus-host disease WBC count: 1200/mm3


(B) Hypersensitivity to sunlight due to the pa- Hemoglobin: 8 g/dL
tients antirejection regimen Platelet count: 19,000/mm3
(C) New-onset systemic lupus erythematosus Activated partial thromboplastin time: 25 sec
(D) Rosacea Prothrombin time: 23 sec
(E) Staphylococcal skin infection acquired Alanine aminotransferase: 16 U/L
during travel Aspartate aminotransferase: 14 U/L
(F) Sun poisoning Urine vanillylmandelic acid: Elevated
D-dimer: Normal
19. A 33-year-old man presents to the clinic with
Which of the following is the most likely diag-
complaints of headaches and blurred vision for
nosis?
the past 3 months. He also notes decreased li-
bido. Past medical history is signicant for pep- (A) Acute lymphocytic leukemia

Hematology/Oncology
tic ulcer disease, for which he takes omepra- (B) Aplastic anemia
zole. Family history is signicant for a mother (C) Child abuse
with recurrent peptic ulcers and an uncle who (D) Neuroblastoma
died of a pancreatic tumor. Physical examina- (E) Pheochromocytoma
tion is unremarkable. MRI of the brain reveals
a mass in the pituitary. In addition to manag- 21. A 27-year-old white woman presents to the gy-
ing the pituitary mass, which of the following necologist because of pelvic pain. She has ex-
studies is most appropriate in this patient? perienced painful periods since beginning
menses at age 12 years. The pain is sometimes
(A) Calcitonin
present when she is not menstruating, and has
(B) Parathyroid hormone
become progressively worse over the years. She
(C) Renal ultrasound
experienced some improvement during the 4
(D) Thyroid ultrasound
years she took oral contraceptives, which she
(E) Urinary metanephrines
stopped taking 2 years ago when she got mar-
ried. She and her husband have been trying to
20. A 14-month-old boy with no known medical his-
conceive for the past 18 months without suc-
tory presents to his pediatrician because of a
cess. Her current menstrual cycle is 2530
black eye. He has had the injury for at least a
days, with heavy periods lasting 57 days. Her
week, but his mother is uncertain as to when or
last menstrual period was 3 weeks ago and was
how it happened, although she does report that
accompanied by severe cramps that were unre-
he has had a low-grade fever and has been sleep-
sponsive to ibuprofen and prevented her from
ing all the time for the past 6 days. On exami-
going to work for 2 days. The pain usually im-
nation he is pale and ill appearing with a tem-
proves with onset of menses, and is aggravated
perature of 38C (100.4F), blood pressure of
by pain with intercourse and bowel move-
120/72 mm Hg, heart rate of 86/min, and respi-
ments. Her -human chorionic gonadotropin
ratory rate of 22/min. He is in the 10th percen-
test is negative. Which of the following is the
tile for weight, the 10th percentile for height,
best test for diagnosing her underlying condi-
and the 50th percentile for head circumference.
tion?
There is periorbital bruising and proptosis of the
right eye, petechiae on his legs, and cervical (A) Diagnostic laparoscopy with biopsies
lymphadenopathy. There is no hepatomegaly, (B) Endometrial biopsy
splenomegaly, or palpable masses. Laboratory (C) Hysterosalpingography
tests show: (D) Measurement of prostaglandin F2 level
(E) Pelvic ultrasound
Blood urea nitrogen: 7 mg/dL
Creatinine: 0.4 mg/dL
HIGH-YIELD SYSTEMS
174 Section I: Organ Systems Questions

22. A 59-year-old man presents to the clinic with a and an 11.3-kg (25-lb) unintentional weight
6.8-kg (15-lb) weight loss over 2 months, with loss, both occurring within the past 6 months.
occasional night sweats. He notices some ab- In addition, the patient notes that over the past
dominal fullness but no pain, and his last 3 months she has become increasingly lethar-
colonoscopy was negative for polyps. He has gic and experienced bouts of nausea. She has
never smoked tobacco, but he has had prior ra- smoked two packs per day for the past 50 years.
diation exposure as an x-ray technician. Physi- She denies a history of heart failure or liver cir-
cal examination is signicant for splenomegaly, rhosis. She currently takes no medications. Her
with no signs of jaundice. Fecal occult blood is temperature is 36.7C (98.1F), blood pressure
negative. X-ray of the chest is clear. His WBC is 125/85 mm Hg, pulse is 78/min and regular,
count is 126,000/mm, and a peripheral smear respiratory rate is 15/min, and oxygen satura-
shows a leukocytosis with all stages of matura- tion is 99% on room air. Examination reveals
tion seen and 3% blasts. Which of the follow- crackles at the left lower lung eld; no lower
Hematology/Oncology

ing tests would conrm the diagnosis? extremity edema is present. Laboratory test
show:
(A) Bone marrow biopsy with hypercellularity
and elevated proportion of blasts WBC count: 6000/mm3
(B) Cytogenetic identication of t(9;22) Hemoglobin: 14.7 g/dL
(C) Elevated -fetoprotein level Platelet count: 210,000/mm3
(D) Lymph node biopsy showing Reed- Na+: 125 mEq/L
Sternberg cells K+: 4 mEq/L
(E) Repeat colonoscopy showing colonic Cl: 102 mEq/L
polyps CO2: 24 mmol/L
Blood urea nitrogen: 8 mg/dL
23. A 65-year-old previously healthy Ashkenazi Creatinine: 1 mg/dL
Jewish woman presents with complaints of ten- Glucose: 120 mg/dL
derness throughout her ribs and spine. She has Urine osmolality: 125 mOsm/kg
noted increasing fatigue and weight loss over Urinary Na+: 35 mEq/L
the past 6 months, and has missed several days
X-ray of the chest shows a focal 5-cm mass le-
of work due to minor infections. Physical ex-
sion in the right lower lung that is corroborated
amination is within normal limits aside from
by CT. Which of the following is the most
point tenderness near her posterior ribs and
likely histologic type of lung cancer present in
thoracic spine. Laboratory studies reveal a
this patient?
WBC count of 8400/mm, hemoglobin level
of 7.9 g/dL, platelet count of 200,000/mm, (A) Adenocarcinoma
and 3+ protein in the urine. X-ray of the chest (B) Bronchoalveolar cell carcinoma
reveals lytic lesions in several areas of the ribs (C) Large cell carcinoma
and spine. Bone scan is within normal limits. (D) Small cell carcinoma
What is the most likely pathology behind her (E) Squamous cell carcinoma
fractures?
25. A 68-year-old man has been treated for non-
(A) Anorexia nervosa
Hodgkins lymphoma for the past 3 weeks, and
(B) Bony infarcts resulting from Gauchers dis-
now complains of weakness and fatigue for the
ease
past day. His temperature is 38.5C (101.3F),
(C) Congenital osteoporosis
blood pressure is 140/88 mm Hg, and heart
(D) Domestic abuse
rate is 78/min. The remainder of the examina-
(E) Increased osteoclastic activity secondary to
tion is normal. Laboratory tests show:
multiple myeloma
(F) Metastatic disease to bone from an un- Na+: 136 mEq/L
identied breast cancer K+: 5.8 mEq/L
Cl: 99 mEq/L
24. A 67-year-old woman is seen in the doctors of- Ca2+: 7.9 mEq/L
ce for a cough productive of bloody sputum HCO3: 25 mEq/L
HIGH-YIELD SYSTEMS
Chapter 7: Hematology/Oncology Questions 175

Blood urea nitrogen: 9 mg/dL 28. A 63-year-old postmenopausal woman is re-


Creatinine: 0.9 mg/dL ferred to the gynecologic clinic by her primary
Urate: 9.1 mg/dL care physician for evaluation of genital pruritus
of 1 months duration. She has a remote his-
What is the most likely cause of this patients
tory of human papillomavirus infection. She
hyperkalemia?
denies changes in vaginal discharge or vaginal
(A) Increased gastrointestinal absorption bleeding. She also denies constitutional symp-
(B) Lysis of neoplastic cells toms or weight loss. On examination she is in
(C) Metabolic acidosis no apparent distress and heart rate is 70/min,
(D) Pseudohyperkalemia blood pressure is 100/58 mm Hg, and respira-
(E) Renal failure tory rate is 10/min. Genital examination re-
veals an ulcerative white lesion approximately
26. A 68-year-old woman presents to her primary 1 cm in diameter on her labia majora. What is

Hematology/Oncology
care physician for a routine visit. She is gener- the most appropriate next step in manage-
ally in good health, and her only medications ment?
are hydrochlorothiazide and metoprolol for
high blood pressure. She has had no recent (A) Obtain a biopsy of the lesion
changes in her health, and review of systems is (B) Prescribe estrogen cream to be applied to
negative. Examination shows a few enlarged area
cervical and inguinal lymph nodes. The liver (C) Treat with acyclovir
(D) Treat with cryotherapy
and spleen are not enlarged. An electrolyte
(E) Treat with uconazole
panel is within normal limits, a peripheral
(F) Treat with metronidazole
smear shows smudge cells, and a complete
blood cell count shows a WBC count of
29. A 9-year-old African-American boy is brought
47,000/mm (with 89% lymphocytes), hemo-
to the clinic by his mother after she noticed a
globin level of 12.9 g/dL, and platelet count of
droop on the left side of his face and difculty
213,000/mm. Which of the following is the
moving his left arm. There was no preceding
most appropriate management?
head trauma or aura, and he was conscious
(A) Chemotherapy throughout. The decit lasted for about 3
(B) Imatinib therapy hours and he returned to baseline activity ex-
(C) No treatment is indicated at this time cept for some residual weakness of the left
(D) Radiation therapy hand. He has a maternal grandfather who had
(E) Splenectomy a heart attack at age 63 years, and the childs
father has hypertension. Review of systems is
27. A 69-year-old man with congestive heart failure signicant for easy fatiguing with exercise, but
and chronic obstructive pulmonary disease has negative for fevers, loss of appetite, weight loss,
palpable adenopathy in the neck and inguinal vomiting, or diarrhea. Signicant ndings on
region. He feels perfectly well and has normal examination include mild pallor and modestly
blood counts. Lymph node biopsy shows follic- decreased left hand strength. Laboratory stud-
ular small cleaved lymphocytic lymphoma. ies show his total cholesterol level is 140 mg/
Bone marrow biopsy shows peritrabecular inl- dL. Which of the following diagnoses could
trates of small cleaved lymphocytes. Which of unify these ndings?
the following is the best management?
(A) Familial hypercholesterolemia
(A) Arrange for bone marrow transplant (B) Hypertension
(B) Arrange for immediate hospice care (C) Multiple myeloma
(C) Begin palliative radiation therapy to in- (D) Sickle cell disease
volved lymph node beds (E) Simple partial seizure
(D) Initiate an aggressive multiagent chemo-
therapy regimen
(E) Manage symptoms conservatively with
close follow-up
HIGH-YIELD SYSTEMS
176 Section I: Organ Systems Questions

30. A 30-year-old woman presents to a primary that she and her partner have been attempting
care physician for a new patient visit. She re- to become pregnant for almost a year. She says
ports recurrent episodes of pneumonia, bron- that she has no other complaints, and says that
chitis, and otitis over the past 4 years. Although she has never felt better. Her pulse is 78/min,
her vaccinations were up to date, she devel- blood pressure is 110/70 mm Hg, and she is
oped tetanus following a foot laceration last afebrile. Pelvic examination is within normal
year. Her lymph nodes and tonsils are en- limits. The physician notices, however, an ab-
larged. Laboratory testing reveals low IgG, IgA, normality on breast examination, which reveals
and IgM levels. After referral to a hematologist, a 3-cm mass palpated just above the nipple.
who rules out other acquired and genetic The mass is mobile and well circumscribed.
causes of her hypogammaglobulinemia, she is The patient says that she noticed it before, and
diagnosed with common variable immunode- that it has gotten larger recently. However, she
ciency. This woman is at the highest risk of de- didnt think it was important because it didnt
Hematology/Oncology

veloping which of the following condition? hurt. Which of the following is the most likely
diagnosis?
(A) Cardiovascular disease
(B) Lymphoma (A) Carcinoma of the breast
(C) Miscarriage (B) Fibroadenoma
(D) Renal disease (C) Fibrocystic change
(E) Splenic autoinfarction (D) Galactocele
(E) Mammary duct ectasia
31. A 72-year-old man with a 40-pack-year smoking
history presents with a 9-kg (20-lb) weight loss 33. A 6-year-old boy presents to his pediatrician
and fatigue. He has no other complaints. He is with 5 days of nosebleeds and a rash. The nose-
not taking any medication. His physical exami- bleeds have increased in frequency to three
nation and vital signs are unremarkable. Labo- times a day. The mother reports no fevers at
ratory tests show: home, though he did have 45 days of cough
and runny nose 2 weeks ago. His past medical
Na+: 138 mEq/L
history is unremarkable and he takes no medi-
K+: 4.6 mEq/L
cations. On physical examination he is afebrile
Cl: 101 mEq/L
with a petechial rash and some crusted blood
HCO3: 24 mEq/L
on the nares. There is no splenomegaly. A
Ca2+: 11.2 mg/dL
complete blood cell count shows a platelet
PO4: 1.6 mg/dL
count of 127,000/mm and normal hemoglo-
Mg2+: 2.0 mg/dL
bin. A peripheral blood smear shows megath-
Blood urea nitrogen: 11 mg/dL
rombocytes. Initial treatment should consist of
Creatinine: 1.1 mg/dL
which of the following?
Glucose: 94 mg/dL
Parathyroid hormone: 12 pg/mL (A) Corticosteroids
(B) Dialysis
Which of the following is the best next step in
(C) Intravenous gamma-globulin
the evaluation of this patient?
(D) Observation
(A) CT of the chest (E) Platelet transfusion
(B) MRI of the brain
(C) Parathyroid ultrasound 34. A 22-year-old man who has recently moved to
(D) Renal biopsy the United States from eastern Africa presents
(E) Serum vitamin D levels to the emergency department with severe ab-
dominal pain. He states that the pain began
32. A 29-year-old woman presents to an obstetrics 3040 minutes earlier and that it is currently
clinic asking for a pregnancy test. Her last 10 of 10 on the pain scale. He denies vomiting
menstrual period was 7 weeks ago. She took a or diarrhea. While in the emergency depart-
home pregnancy test and it was positive. She ment, he passes a bloody stool. Examination
says that this is a highly desired pregnancy, and demonstrates an acute abdomen and the pa-
HIGH-YIELD SYSTEMS
Chapter 7: Hematology/Oncology Questions 177

tient is taken to the operating room. On lapa- (D) Response to a short course of high-dose
rotomy an intussusception caused by tremen- steroids
dous swelling of the intestinal wall is (E) Response to increased chronic immuno-
discovered. A large section of intestine is re- suppression
sected. Pathologic examination of frozen sec-
tions reveals lymphoid tissue. Which of the fol- 37. A 3-year-old patient with sickle cell anemia is
lowing is the most likely diagnosis? brought to the emergency department com-
plaining of pain in his lower arms for 3 days.
(A) Burkitts lymphoma
On physical examination both arms are swol-
(B) Chronic lymphocytic leukemia
len, tender, and erythematous. He is febrile,
(C) Chronic myelogenous leukemia
and complains of chills. Leukocytosis is seen
(D) Ewings sarcoma
on blood count with prominent neutrophils.
(E) Szary syndrome
Which of the following is the most appropriate

Hematology/Oncology
treatment?
35. A 65-year-old, otherwise healthy woman pre-
sents to the clinic with shortness of breath. (A) Calcitonin
Physical examination reveals jugular venous (B) Ceftriaxone
distention and bilateral lower extremity edema. (C) Ciprooxacin
Echocardiography shows enlarged left ventric- (D) Etidronate
ular end-systolic and end-diastolic dimensions, (E) Vitamin D
an ejection fraction of 45%, and increased left
ventricular wall thickness. Which of the fol- 38. A 4-year-old boy presents to his physician with
lowing is the most likely cause? increased fatigue over the past 4 months. He
also has intermittent fevers that resolve on their
(A) Amyloidosis
own and progressively more frequent nose-
(B) Chagas disease
bleeds, up to three times a week. His mother
(C) Doxorubicin
notes that he has generally stopped walking,
(D) Idiopathic hypertrophic subaortic stenosis
but when he tries to walk he seems to favor his
(E) Ischemic cardiomyopathy
left leg. Examination shows pallor, cervical
lymphadenopathy, hepatomegaly, splenomeg-
36. A 53-year-old woman with a history of polycys-
aly, and crusted blood on the nares. His WBC
tic kidney disease undergoes a renal transplant
count is 28,000/mm, with a hemoglobin level
from a cadaveric donor. She experiences sev-
of 8.3 g/dL and a decreased platelet count.
eral bouts of acute rejection that are controlled
Bone marrow biopsy shows increased cellular-
by appropriate increases in immunosuppres-
ity and lymphoblast predominance. Which of
sion during the rst year posttransplant. Dur-
the following is true of this patients likely diag-
ing her fourth posttransplant year her creati-
nosis?
nine is rising gradually. Transcutaneous biopsy
reveals architectural distortion and sclerosis of (A) Children with trisomy 21 are at increased
the renal tubules. What is the most likely out- risk of developing this disease
come of the patients condition? (B) Presence of the Philadelphia chromosome
means that there is no current treatment
(A) Development of lymphoproliferative tu-
(C) Survival with chemotherapy is approxi-
mors requiring decreased immunosuppres-
mately 20%30% in children
sion
(D) The central nervous system is usually
(B) Gradual onset of renal failure and return
spared from metastatic spread
to dialysis
(E) Tumor lysis syndrome is uncommon
(C) Requirement of chronic antibiotic prophy-
laxis against urinary tract infection
HIGH-YIELD SYSTEMS
178 Section I: Organ Systems Questions

39. A 66-year-old postmenopausal white woman 41. A 63-year-old African-American man with no
presents to her physician with a 1-month his- past medical history presents with a recent in-
tory of painless gross hematuria. She works as a crease in prostate-specic antigen from 2.2 to
hairdresser, is an avid coffee drinker, and has a 4.3 ng/mL. Digital rectal examination reveals a
50-pack-year smoking history. Results of physi- 0.5-cm nodule on the left lobe of prostate.
cal examination are normal. Which of the fol- Which of the following is the most appropriate
lowing would best conrm the diagnosis? next step in the evaluation of this patient?
(A) CT of the pelvis (A) CT scan and bone scan
(B) Cystoscopy (B) Prostate biopsy
(C) Serum creatine kinase levels (C) Radical prostatectomy
(D) Ultrasound (D) Treat patient with a 7-day course of antibi-
(E) Urine culture otics and recheck prostate-specic antigen
Hematology/Oncology

level in 3 months
40. A 53-year-old woman presents with diffuse (E) Urinalysis
back and rib pain that is worsened with move-
ment. She states her periods have recently 42. A 37-year-old woman gives birth to her third
stopped, but denies experiencing hot ashes or healthy child. He is discharged from the hospi-
other menopausal symptoms. She also notes a tal with her at 2 days of life. For the rst 3
6.8-kg (15-lb) weight gain over the past 34 months of his life, however, his parents note
months. On further questioning she states she poor weight gain, chronic diarrhea, and persis-
has noted tingling in her ngers and toes for tent diaper rash. Complete blood count reveals
the past 7 months, and has recently begun trip- a paucity of lymphocytes, and antibody stain-
ping frequently and having trouble opening ing demonstrates a deciency of the X-linked
doors and buttoning her shirts. Physical exami- interleukin-2 receptor chain. The baby is diag-
nation reveals diffuse lymphadenopathy and nosed with X-linked severe combined immu-
abdominal examination reveals hepatomegaly. nodeciency (SCID). His parents are told that
Her skin is warm and dry; hyperpigmentation the standard of care for him is a bone marrow
is noted in the regions of the neck and shoul- transplant from a major histocompatibility
ders, and her digits are noted to be clubbed. complex-matched related donor. His two sisters
Laboratory testing and peripheral blood smear are tested and are not matches. His parents are
results reveal large numbers of plasma cells; interested in exploring the option of gene ther-
additional laboratory ndings are: apy. Which of the following concerns is unique
to the use of gene therapy for the treatment of
WBC count: 13,000/mm3
SCID?
Hemoglobin: 9.8 g/dL
Platelet count: 340,000/mm3 (A) Graft-versus-host disease
Erythrocyte sedimentation rate: 64 mm/hr (B) Increased risk of failure
Blood urea nitrogen: 35 mg/dL (C) Increased susceptibility to viral illness in
Creatinine: 2.1 mg/dL the future
Thyroid-stimulating hormone: 25 U/mL (D) Inefcient response to vaccines in the fu-
Blood glucose level: 384 mg/dL ture
Ca2+: 15.4 mg/dL (E) Leukemia
Which of the following is most likely to con-
43. A 6-year-old African-American girl presents to
rm the patients underlying diagnosis?
her pediatrician with intense pain in both feet
(A) Complete thyroid evaluation after jumping in a cold swimming pool. There
(B) Dexamethasone suppression test was no trauma reported, and the pain seems to
(C) Hemoglobin A1c decrease with warming of the feet. Physical ex-
(D) Pulse oximetry amination shows scleral icterus and spleno-
(E) Urine and serum protein electrophoresis megaly. There is no point tenderness of the an-
(F) WBC differential kles or feet, but the toes are cool and appear
dusky. A complete blood cell count shows he-
HIGH-YIELD SYSTEMS
Chapter 7: Hematology/Oncology Questions 179

moglobin of 10.2 g/dL, and a liver panel shows 45. A 30-year-old man presents to his physician for a
increased indirect bilirubin. A peripheral routine physical. On questioning he comments
smear is shown in the image. Which of the fol- that his father was diagnosed with colon cancer at
lowing long-term treatments is needed to mini- 45 years of age. The patient has never had polyps
mize the recurrence of her condition? and does not suffer diarrhea, constipation, or
bloody stools. The patient is nervous about
screening and wants to delay as long as possible.
According to current recommendations, when
should he have his rst colonoscopy?
(A) 30 years old
(B) 35 years old
(C) 40 years old

Hematology/Oncology
(D) 45 years old
(E) 50 years old

46. A 2-year-old girl presents to the pediatrician be-


Reproduced, with permission, from Lichtman MA, Beut-
cause her mother noticed a mass while dress-
ler E, Kipps TJ, Seligsohn U, Kaushansky K, Prchal JT.
Williams Hematology, 7th edition. New York: McGraw-
ing her. Aside from the mass, the childs
Hill, 2006: Plate III-5. mother has no concerns. She reports that the
child is otherwise well and denies any fever,
(A) Avoidance of sulfa drugs and fava beans weight loss, or fatigue. The childs temperature
(B) Corticosteroids is 37C (98.6F), blood pressure is 100/60 mm
(C) Hematopoietic stem cell transplant Hg, heart rate is 80/min, and respiratory rate is
(D) Hydroxyurea 25/min. A smooth, rm, left-sided nontender
(E) Serial transfusions ank mass is palpated. There is no hepatomeg-
aly, no splenomegaly, and no lymphadenopa-
44. A 28-year-old woman is diagnosed with acute thy. A CT scan reveals only an encapsulated
promyelocytic leukemia (APL) after experienc- left-sided mass and no other abnormalities.
ing severe fatigue and malaise for several weeks. Laboratory studies show:
She and her parents have done extensive read-
ing on the Internet and expect that she will be Blood urea nitrogen: 8 mg/dL
treated with intensive multiagent chemotherapy. Creatinine: 0.4 mg/dL
To their surprise, her physician suggests an oral Alanine aminotransferase: 20 U/L
agent that is not used to treat other forms of Aspartate aminotransferase: 20 U/L
acute leukemia. Why might the physician be Total bilirubin: 0.4 mg/dL
concerned about using standard intensive mul- Alkaline phosphatase: 70 U/L
tiagent chemotherapy in this patient? Urine vanillymandelic acid: Negative
WBC count: 6200/mm3
(A) A desire to allow the patient to preserve Hemoglobin: 14 g/dL
her fertility Platelet count: 190,000/mm3
(B) A higher incidence of chemotherapy- Urine RBCs: 1+
related death in patients with APL as com- Urine protein: 0
pared to other leukemias Urine WBC esterase: Negative
(C) A personal preference to treat APL with
homeopathic agents Which of the following is the most appropriate
(D) An observation that APL does not respond therapy?
to conventional chemotherapeutics (A) Chemotherapy only
(E) An understanding that APL is so sensitive (B) Local excision of tumor without nephrec-
to oral agents that intravenous chemother- tomy
apy is simply not required in its treatment (C) Nephrectomy and chemotherapy
(D) Nephrectomy only
(E) Radiation
HIGH-YIELD SYSTEMS
180 Section I: Organ Systems Questions

47. A 55-year-old woman presents to her primary locations on her body. She does not have jugu-
care physician with complaints of pain in her lar venous distention. No masses are palpable
hands in the cold. On further questioning she on breast examination. Heart and lung exami-
says she notices that her hands have a bluish nation are within normal limits. Abdominal ex-
hue when she is outside, and that they become amination is signicant for hepatosplenomeg-
pale and then quite red when warm. Her physi- aly. Her WBC count is 35,000/mm. Her
cian, concerned that Raynauds phenomenon WBCs are myeloperoxidase positive. Which of
may indicate the presence of an autoimmune the following statements regarding patients car-
illness, initiates an autoimmune work-up that is rying this diagnosis is true?
entirely negative. Three weeks later the patient
(A) Incidence decreases with increasing age
returns to the ofce with reports of visual oat-
(B) Previous chemotherapy does not affect risk
ers. She states she feels ne otherwise. Her
for developing the disease
physical examination is negative aside from sev-
Hematology/Oncology

(C) Prognosis is not affected by the cytogenet-


eral large bruises on her legs. Which of the fol-
ics of the disease
lowing is the most appropriate management?
(D) Retinoic acid was once used to treat this
(A) Begin a course of prednisone to alleviate disease but is no longer part of the current
the patients autoimmune symptoms in the treatment
absence of concrete diagnosis (E) They are at increased risk for stroke
(B) Discuss the possibility of domestic vio-
lence with the patient 50. A 67-year-old man presents to his physician
(C) Order plasma electrophoresis with u-like symptoms and cough. He also re-
(D) Reassure the patient and reevaluate in ports a 2.3-kg (5-lb) weight loss over the past 2
3 months months. The patient has a 40-pack-year history
(E) Resend the patients serum for antinuclear of smoking but quit 10 years ago. His past med-
antibody tests ical history and physical examination are unre-
markable. Complete blood cell count and liver
48. A 60-year-old man presents to the clinic with function tests are normal. His stool is negative
dysphagia, facial ushing, and diarrhea. Physi- for occult blood. X-ray of the chest revealed a
cal examination reveals a goiter and mild cervi- 3.5-cm right lobe mass and widening of the
cal lymphadenopathy. Based on thyroid func- mediastinum. CT scan shows a right middle
tion test results, he is referred to a surgeon for lobe mass, and in addition two mediastinal
total thyroidectomy, which reveals medullary lymph nodes are seen that measure 1.5 cm and
carcinoma. He is currently being treated with 2 cm in diameter, respectively. Biopsies of the
thyroid hormone suppression therapy. Which lung nodule obtained by bronchoscopy indi-
of the following is the most signicant risk fac- cate malignancy. Which of the following is the
tor for this form of thyroid cancer? best next best step in management?
(A) Alcohol (A) Mediastinoscopy
(B) Genetic risk factor (B) Pulmonary function tests
(C) Iodine exposure (C) Thoracentesis
(D) Radiation to the neck for enlarged tonsils (D) Ventilation-perfusion lung scan
as a child (E) Video-assisted thoracoscopic surgery
(E) Smoking
(F) Viral infection 51. A 7-year-old boy presents to his pediatrician be-
cause of increased gingival bleeding after brush-
49. A 55-year-old woman who was successfully ing his teeth. The patients mother denies a his-
treated with chemotherapy for breast cancer tory of easy bruising or prolonged bleeding. The
several years ago presents to her primary care boy also reports an episode of prolonged and
physician complaining of 2 months of easy painful knee swelling after a fall in which he hit
bruising, bleeding gums, fatigue, and shortness his knee. The patients family history is signicant
of breath. On physical examination she has a for a maternal grandfather who died of a massive
low-grade fever and several bruises in different hemorrhage after a minor surgical procedure. On
HIGH-YIELD SYSTEMS
Chapter 7: Hematology/Oncology Questions 181

examination there are no ecchymoses or pete- 54. A 12-year-old boy presents with pain and swell-
chiae. His conjunctivae are pink and a full physi- ing of his left thigh, and is diagnosed with oste-
cal examination is noncontributory. His activated osarcoma. He has no other medical problems;
partial thromboplastin time is 63 sec, prothrom- however, his family history reveals that his
bin time is 12 sec, bleeding time is 4 min, and mother has been treated for bilateral breast
coagulation time is prolonged. Which of the fol- cancer and that his sister and two uncles died
lowing additional laboratory results is most likely of brain tumors. He is treated with multi-
to be seen in this patient? agent chemotherapy and recovers from his dis-
ease. Several years later, a complete blood cell
(A) Decreased factor VIII concentrations
count sent on routine follow-up reveals abnor-
(B) Decreased platelet concentrations
malities consistent with myelodysplastic syn-
(C) Decreased WBC count
drome. Within weeks he has developed a re-
(D) Increased factor V concentrations
fractory form of acute myelogenous leukemia

Hematology/Oncology
(E) Increased hemoglobin
that is rapidly fatal. Which of the following
syndromes is most likely affecting this patient?
52. A 64-year-old woman with a history of cardiac
disease, multiple strokes, and progressive os- (A) Ataxia telangiectasia
teoarthritis is admitted to the hospital for bilat- (B) Familial Li-Fraumeni syndrome
eral total knee replacement surgery. The morn- (C) Von Hippel-Lindau syndrome
ing after surgery her platelet count is 9000/ (D) Wiskott-Aldrich syndrome
mm. Which of the following medications is (E) Xeroderma pigmentosa
most likely responsible for this nding?
(A) Aspirin
(B) Clopidogrel
(C) Heparin
(D) Streptokinase
(E) Warfarin

53. Which of the laboratory ndings in the table be-


low are characteristic of von Willebrands dis-
ease? PT refers to prothrombin time and aPTT
refers to activated partial thromboplastin time.
(A) A
(B) B
(C) C
(D) D
(E) E

BLEEDING TIME PLATELET COUNT


CHOICE (min) PT (sec) APTT (sec) (/mm3) FACTOR IX FACTOR VIII

A 3 13 32 202,000 abnormal normal

B 10 15 42 140,000 abnormal normal

C 4 12 45 230,000 normal abnormal

D 10 13 27 58,000 normal normal

E 9 12 35 143,000 normal abnormal


HIGH-YIELD SYSTEMS
182 Section I: Organ Systems Questions

55. A 37-year-old woman presents to her physician 57. A 49-year-old man with a history of several dys-
with 3 days of fever, fatigue, and rash. Her plastic nevi presents for an annual examina-
medications include oral contraceptives. Her tion. He mentions that one of the moles on his
temperature is 38.7C (101.7F); she also has a left shoulder seems larger than usual. It has
nonpalpable petechial rash and splenomegaly. also started to itch. He denies anorexia, weight
Coagulation studies and brinogen levels are changes, or malaise, though he admits to feel-
normal, but a complete blood cell count shows ing somewhat tired in the past couple of
a hemoglobin level of 9.7 g/dL and platelet months. On examination he is well appearing
count of 135,000/mm. A peripheral blood and has no lymphadenopathy. The mole is
smear shows schistocytes. Which of the follow- dark brown, 7 mm in diameter, nodular, and
ing is the most likely diagnosis? round with an irregular shape. There is no ul-
ceration or bleeding. Which of the following is
(A) Disseminated intravascular coagulation
the most appropriate next step in manage-
Hematology/Oncology

(B) HELLP syndrome


ment?
(C) Henoch-Schnlein purpura
(D) Idiopathic thrombocytopenic purpura (A) Excision of the lesion
(E) Thrombotic thrombocytopenic purpura (B) Excision of the lesion with sentinel lymph
node biopsy
56. A 56-year-old woman with diabetes and protei- (C) Local radiation therapy
nuria presents to the emergency department (D) Oral antihistamines for symptomatic relief
complaining of right groin pain and decreased (E) Reassurance, with follow-up in 6 months
urine output for the past day. She denies
trauma, fever, or a history of kidney stones or 58. A 23-year-old African-American man presents
urinary tract infections. Her temperature is to the emergency department with complaints
37.2C (99.0F), blood pressure is 146/90 mm of abdominal and bilateral lower extremity
Hg, pulse is 104/min, and respiratory rate is 16/ pain as well as chest pain that is worse on in-
min. Heart and lung examinations are normal; spiration. He notes a frequent and long history
she has no visible rashes and no costovertebral of similar episodes throughout his life. His
angle tenderness. There is trace ankle edema temperature is 38.3C (101F), respiratory rate
bilaterally. Laboratory tests show: is 25/min, and heart rate is 100/min. Lung ex-
amination reveals bronchial breath sounds and
Na+: 134 mEq/L
egophony at the left lung base. Oxygen satura-
K+: 4.9 mEq/L
tion is 94% on 2-L nasal cannula. Laboratory
Cl: 100 mEq/L
tests show:
HCO3: 24 mEq/L
Blood urea nitrogen: 38 mg/dL WBC count: 12,000/mm
Creatinine: 1.6 mg/dL Hemoglobin: 12 g/dL
Albumin: 2.2 g/dL Hematocrit: 38%
Urinalysis: 3+ blood, 4+ protein, no WBCs, no Platelet count: 200,000/mm
stones Reticulocyte count: 2.1%
What is the most appropriate next step in man- What prophylactic treatment would have pre-
agement of this patients condition? vented this patients acute chest pain?
(A) Analgesics and angiotensin-converting en- (A) Diphenyhydramine
zyme inhibitors (B) Hydroxyurea
(B) Kidney biopsy and high-dose corticoster- (C) Pneumococcal vaccination
oids (D) Prophylactic penicillin
(C) Renal vein ultrasound and heparin (E) Splenectomy
(D) Thiazides and spiral CT of the pelvis
(E) Urine culture and oral quinolones
HIGH-YIELD SYSTEMS
Chapter 7: Hematology/Oncology Questions 183

E X T E N D E D M ATC H I N G 59. A 34-year-old G2P2 woman presents to her


physician because she noticed some scant
bloody discharge from her left nipple several
The response options for the next 2 items are
days ago. She does not recall experiencing re-
the same. Select one answer for each item
cent trauma to her breast, although she re-
in the set.
cently stopped breast-feeding her youngest
child. Her vital signs are normal and she does
For each patient, select the most appropriate next
not have any palpable breast masses or breast
step.
tenderness. A small drop of bloody uid is ex-
(A) A course of dicloxacillin pressed from the left nipple.
(B) Breast ultrasound
(C) Chemotherapy with doxorubicin 60. A 67-year-old woman comes to an outpatient
(D) CT of the chest surgical clinic for further evaluation of her

Hematology/Oncology
(E) Excisional biopsy mammogram, which reveals microcalcica-
(F) Fine-needle aspiration biopsy tions and irregular density. On physical exami-
(G) Follow-up visit in 6 months nation there are no palpable breast masses but
(H) Incision and drainage lymphadenopathy is present.
(I) Lumpectomy with lymph node dissection
(J) Modied radical mastectomy
(K) Radiation therapy
(L) Simple mastectomy
(M)Stereotactic core biopsy
HIGH-YIELD SYSTEMS
184 Section I: Organ Systems Answers

AN S W E R S

1. The correct answer is B. Hairy cell leukemia not a common nding, and lymphadenopathy
is a rare malignancy of B lymphocytes. It is is almost universally found. In addition, the
most common in patients 5060 years old, and peripheral blood smear is generally normal in
shows a 5:1 male predominance. Patients gen- Hodgkins disease.
erally present with the symptoms of pancytope-
nia, namely fatigue, easy bruising, and recur- 2. The correct answer is C. Along with the coag-
rent infection, as well as splenomegaly (and ulation factors (II, VII, IX, and X), the antico-
hepatomegaly in 50% of patients). Mycobacte- agulant proteins C and S are also vitamin K-
rial infections are common due to a striking dependent, and therefore affected by warfarin,
monocytopenia that is nearly universal in these which is a vitamin K analog. Proteins C and S
Hematology/Oncology

patients. Lymphadenopathy is rare. Examina- have shorter half-lives than the coagulation fac-
tion of the spleen shows inltration of the red tors, and therefore a faster turnover in the body.
pulp by these cells, which demonstrate the Thus, for a brief window of approximately 3
characteristic hairy cell cytoplasmic projec- days after the initiation of warfarin therapy, the
tions and are typically tartrate-resistant acid body has normal function of its coagulation
phosphatase positive. Although patients may factors, but loss of anticoagulant proteins C
demonstrate markedly abnormal blood cell and S, rendering the patient hypercoagulable.
counts at presentation, >90% of patients will Negative consequences of this are avoided by
live >10 years with treatment. administering heparin with warfarin for the
rst 5 days.
Answer A is incorrect. Acute lymphocytic leu-
kemia (ALL) affects children much more often Answer A is incorrect. Warfarin inhibits the
than adults. It may result in pancytopenia, as is synthesis of the vitamin K-dependent factors,
seen in this patient, but would not cause such including factors II, VII, IX, and X, and pro-
a striking monocytopenia. Immature blasts are teins C and S; it never increases the amount of
evident on the peripheral smear of ALL pa- the factors, but the shorter half-life of the anti-
tients. coagulants protein C and S lead to the initial
hypercoagulable state.
Answer C is incorrect. Idiopathic thrombo-
cytopenic purpura is an antibody-mediated Answer B is incorrect. Venous valvular insuf-
thrombocytopenia caused by antibody response ciency is a common factor that contributes to
to platelets. It does not affect other cell lines, venous stasis, one of the three elements of Vir-
so is an unlikely cause of this patients illness. chows triad (along with endothelial injury and
a hypercoagulable state). Warfarin, however,
Answer D is incorrect. Infectious mono-
has no effect on venous valvular insufciency.
nucleosis (Epstein-Barr virus infection) may
affect patients of any age, although it is most Answer D is incorrect. Factor V Leiden is the
common in adolescents. It may cause spleno- most common inherited hypercoagulable state
megaly and a peripheral smear not unlike that and results from a point mutation that causes
shown. It does not, however, cause pancytope- resistance of factor V to protein C degradation.
nia, a hallmark of hairy cell leukemia. Warfarin does not induce a condition similar
to factor V Leiden.
Answer E is incorrect. Although the possibil-
ity of other lymphoid malignancies must be Answer E is incorrect. Warfarin is a vitamin K
considered in any patient with leukemia or analog, and therefore inhibits the production
lymphoma, this patients presentation is not of the vitamin K-dependent factors (both co-
consistent with Hodgkins lymphoma, nor agulants and anticoagulants). Therefore it has
are his laboratory ndings. Fatigue and night no effect on platelets.
sweats may affect patients with newly diag-
nosed Hodgkins disease, but splenomegaly is
HIGH-YIELD SYSTEMS
Chapter 7: Hematology/Oncology Answers 185

3. The correct answer is D. The patient is pre- cuss the diagnosis with the patient include the
senting with stage IIIA disease, which implies previous request of the patient to not be in-
that her disease has spread to nodal sites on formed or the belief of the physician that dis-
both sides of her diaphragm. This disease stage closure would severely harm the patient.
will require therapy with both systemic chemo-
Answer A is incorrect. It would be unethical
therapy and radiation to the mediastinum, and
to acquiesce to the familys request. If nondis-
potentially the abdomen and pelvis. This pa-
closure is the patients request as well, then the
tient should be concerned about many effects
physician will be able to honor it.
of her disease and its treatment that may pre-
vent or complicate her pregnancy; however, Answer B is incorrect. Even if the entire fam-
there is no evidence that the hormonal milieu ily wants to keep the diagnosis from the pa-
of pregnancy promotes recurrence of Hodg- tient, if the patient him- or herself wants to be
kins disease. Importantly, this type of evidence informed, the physician is obligated to tell the

Hematology/Oncology
does exist in breast cancer, where increased es- patient the diagnosis.
trogen may promote the growth of tumor cells.
Answer D is incorrect. Rarely will consultation
Answer A is incorrect. The risk of a second be a correct answer on a USMLE examination.
cancer is substantially increased in patients The physician has the information needed to
with Hodgkins lymphoma, particularly if sys- act and should do so.
temic chemotherapy is required as treatment.
Answer E is incorrect. Psychiatric evaluation
The risk of leukemia may be elevated almost
is not warranted at this time. All physicians can
100-fold in this case.
judge decision-making capacity, and if the pa-
Answer B is incorrect. Ovarian failure is a risk tient has the capacity to decide whether he or
for this patient who will receive systemic che- she wants to be informed of the diagnosis, the
motherapy. In some cases, ovarian tissue can obligation then exists to abide by that wish.
be removed surgically and frozen to preserve
a patients fertility. Should the patient need to 5. The correct answer is B. This childs symp-
undergo pelvic irradiation, her risk of ovarian toms of bone pain, easy bruising, and fatigue,
failure would be almost 100%. as well as examination ndings of hepatosple-
nomegaly and lymphadenopathy, suggest a ma-
Answer C is incorrect. Mediastinal irradiation lignancy, which is conrmed by the presence
that is required to treat Hodgkins lymphoma of blasts on the blood smear. The most com-
may cause pericarditis that can result in bro- mon leukemia in children is ALL, and this di-
sis and congestive heart failure. Because preg- agnosis is supported by the lymphocytes on the
nancy requires increased cardiac output, heart smear. A bone marrow biopsy is recommended
failure or worsening of underlying heart failure for all patients with ALL.
may result.
Answer A is incorrect. Child abuse should
Answer E is incorrect. Patients treated with always be suspected in cases of children with
radiation to the mediastinum are at risk for re- multiple bruises and/or bone pain. However,
strictive lung disease that may follow radiation this childs other systemic symptoms and blood
pneumonitis. During pregnancy, when the dia-
smear results suggest another etiology for her
phragm is displaced superiorly and breathing
bruises.
is somewhat restricted, asymptomatic lung dis-
ease may become clinically important. Answer C is incorrect. Although the patient
does have signs of liver dysfunction, includ-
4. The correct answer is C. With rare exceptions, ing hepatomegaly and easy bruising, her smear
patients are entitled to know their diagnosis suggests a hematologic malignancy.
and prognosis, even if they are old, their condi-
Answer D is incorrect. A test for serum hetero-
tion is probably terminal, and they have anx- phile antibodies is helpful in diagnosing infec-
ious personalities and/or psychiatric diagnoses. tious mononucleosis, which would explain this
Exceptions to the physicians obligation to dis-
HIGH-YIELD SYSTEMS
186 Section I: Organ Systems Answers

patients lymphadenopathy, fatigue, and hepa- of venography is not indicated in this case and
tosplenomegaly. However, given the patients would not prevent further complications.
easy bruising and blood smear, mononucleosis
Answer E is incorrect. Although patients with
is not high on the list of differential diagnoses.
a history of DVT may be at increased risk for
Answer E is incorrect. Although vitamin C de- pulmonary embolism, the rst step in manage-
ciency does cause easy bruising, it would not ment is cessation of heparin exposure rather
explain the patients bone pain, hepatospleno- than placement of an inferior venal cava lter.
megaly, lymphadenopathy, or blood smear.
Answer F is incorrect. With a platelet count
of 78,000/mm and absence of spontaneous
6. The correct answer is C. The patient in this clin-
bleeding, a platelet transfusion is not indicated.
ical scenario most likely has heparin-induced
Cessation of heparin leads to the rapid recov-
thrombocytopenia (HIT), which commonly oc-
ery of the platelet count and decreased risk of
Hematology/Oncology

curs within 410 days after the initiation of un-


further thrombosis.
fractionated heparin treatment. Unlike other
drug-induced thrombocytopenia, HIT is associ-
7. The correct answer is D. Recent evidence sug-
ated with thrombosis due to platelet activation
gests that most low-grade squamous intraepi-
rather than bleeding. Because platelet counts typ-
thelial lesions will regress spontaneously. Cur-
ically remain >20,000/mm, spontaneous bleed-
rent guidelines indicate observation of a
ing is rarely seen. Pathophysiology of this disorder
biopsy-conrmed LSIL is appropriate manage-
includes the presence of platelet antibodies that
ment for a reliable patient that is also immuno-
cause limb- and life-threatening thrombosis, of-
competent.
ten at unusual sites including arteries. The rst
intervention in cases of HIT is immediate cessa- Answer A is incorrect. Endocervical curettage
tion of any heparin and immediate initiation of a is reserved for abnormal Pap smear ndings of
direct thrombin inhibitor such as argatroban or high-grade squamous intraepithelial lesion.
lepirudin. Answer B is incorrect. Loop electrosurgical
Answer A is incorrect. Warfarin therapy should excision procedure is reserved for abnormal
be avoided in patients with HIT until the plate- Pap smear ndings of high-grade squamous in-
let count rises to >100,000/mm. Initiation of traepithelial lesion.
warfarin therapy may cause a transient hyper- Answer C is incorrect. Repeat Pap smear in 1
coagulable state due to a decline in protein C year following a biopsy-conrmed LSIL is not
levels, thereby increasing the risk of limb isch- an acceptable form of management.
emia.
Answer E is incorrect. Repeat colposcopy in 1
Answer B is incorrect. Patients with HIT are year is not an acceptable form of management.
at an increased risk for thrombosis. The non-
involved contralateral leg is at increased risk 8. The correct answer is D. This patient has de-
for developing a DVT. Discontinuation of veloped a posttransplant lymphoproliferative dis-
the pneumatic compression devices bilater- order (PTLD). Any patient who is chronically
ally might lead to an increase in mortality and immunosuppressed is at risk for development of
would not prevent further complications. lymphoproliferative malignancy, and a cardiac
Answer D is incorrect. Ultrasound has an ac- transplant patient is at intermediate risk (4.6%
curacy rate of 80%90% in the detection of incidence) compared to kidney (1.0%) or liver
DVT. Therefore Doppler ultrasound in con- (3.0%) transplant patients. PTLDs generally oc-
junction with the physical symptoms and pre- cur in extranodal locations, and may respond
sentation of pain, edema, tenderness, and a completely to reduction in immunosuppression;
positive Homans sign (calf pain on dorsiex- however, since immunosuppression is necessary
ion of the foot) are sufcient to make a diagno- for protection of grafted organs, radiotherapy is
sis of DVT. A further DVT work-up consisting also often used. In addition, most PTLDs are re-
HIGH-YIELD SYSTEMS
Chapter 7: Hematology/Oncology Answers 187

lated to primary or reactivation infection with 450 (CYP450) inducers, such as barbiturates,
Epstein-Barr virus. Some evidence demonstrates because they increase the demand for heme
that the use of acyclovir during treatment for synthesis. After acute attacks urine can appear
PTLD contributes to eradication of the virus to be brown or reddish. Many patients with AIP
and resolution of the malignancy caused by it. have a family history of the disorder.
The majority of centers thus employ acyclovir as
Answer A is incorrect. Cocaine does not in-
an adjuvant therapy to radiotherapy/surgery cou-
duce the CYP450 system, and is not a trigger
pled with a reduction in immunosuppression.
for an AIP attack.
Answer A is incorrect. The purpose of acyclo-
Answer B is incorrect. Heroin does not induce
vir in treatment of PTLD is to eliminate the
the CYP450 system, and is not a trigger for an
Epstein-Barr virus that is causative in the de-
AIP attack.
velopment of the malignancy, not to eradicate

Hematology/Oncology
the neoplastic cells themselves; this is left to Answer C is incorrect. Marijuana does not in-
the host immune system and radiation therapy, duce the CYP450 system, and is not a trigger
if used. for an AIP attack.
Answer B is incorrect. Although severe infec- Answer D is incorrect. Methamphetamine
tion with cytomegalovirus is a risk for any im- does not induce the CYP450 system, and is not
munosuppressed patient, antiviral therapy is a trigger for an AIP attack.
generally not used in patients undergoing can-
cer therapy. Acyclovir prophylaxis is used in 10. The correct answer is D. The arrow in the image
some bone marrow transplant centers. indicates an epidermal cluster of atypical lym-
phocytes (Pautriers abscess). This is pathogno-
Answer C is incorrect. Radiation to mucous monic for mycosis fungoides, a subtype of cuta-
membranes will indeed cause inammation neous T-lymphocyte lymphoma (CTCL) that is
and decreased barrier protection. This is gen- unrelated to fungal infection in spite of the name.
erally not held to be an indication for antiviral This disease is most frequently found in men in
prophylaxis, however. their mid-fties and is often misdiagnosed for
Answer E is incorrect. If anything, decreasing years. Skin lesions may progress from patch phase
the patients immunosuppressive regimen will to plaque phase to cutaneous tumor, and subcu-
assist her immune system in preventing viral taneous deposits in the face may lead to the clas-
infection. sic (though rarely seen) leonine facies. Szary
syndrome is mycosis fungoides with erythro-
9. The correct answer is E. This patient is pre- derma, lymphadenopathy, and the presence of
senting with classic symptoms of acute inter- atypical T lymphocytes in the blood.
mittent porphyria (AIP): gastrointestinal distress
Answer A is incorrect. While contact dermati-
and neurological symptoms following ingestion
tis may develop in this distribution in reaction
of alcohol and a barbiturate, followed by pro-
to clothing or local irritants, the biopsy points
duction of brown or reddish urine. AIP is a se-
to another diagnosis.
vere autosomal dominant form of porphyria,
which results from a partial deciency of por- Answer B is incorrect. This is a very late and
phobilinogen deaminase activity, which is the atypical presentation for eczema and should
third enzyme in the pathway of heme synthe- not be the primary consideration in this case.
sis. Attacks occur when intermediates in the Answer C is incorrect. Inverse psoriasis typi-
heme synthetic pathway build up, and result in cally presents as erythematous plaques in in-
a variety of symptoms including gastrointestinal tertriginous areas. It has minimal scaling in
distress, tachycardia, hypertension, peripheral comparison to the more common plaque pso-
neuropathy, seizures, and psychiatric symp- riasis and tends to spare extensor surfaces. Like
toms. Attacks can be triggered by stress, alco- plaque psoriasis, it is a chronic, relapsing disor-
hol consumption, the premenstrual period, der with an inammatory basis.
and consumption of drugs that are cytochrome
HIGH-YIELD SYSTEMS
188 Section I: Organ Systems Answers

Answer E is incorrect. Pityriasis rosea is charac- minimal gastrointestinal side effects. Chemo-
terized by pink, aky papules typically found on therapy can be used as salvage therapy for pa-
the trunk, inner thighs, and inner arms. One- tients who relapse following irradiation. The
fourth of patients have pruritic lesions. There staging of seminomas is as follows: stage I: dis-
is no treatment and the disease is self-limited. ease conned to the testis; stage II: retroperito-
Irritants (e.g., soaps) should be avoided, and in neal nodal involvement (IIA if <2 cm and IIB
severe cases steroid therapy can help. Biopsy if >2 cm); stage III: supradiaphragmatic nodal
generally shows a mild perivascular and intersti- involvement or involvement of the viscera.
tial lymphocytic inltrate. This does not t the
Answer A is incorrect. At this point there is no
description given in the case because pityriasis
need to remove the mans other testicle.
rosea is self-limited and does not have clusters of
atypical lymphocytes on histologic analysis. Answer B is incorrect. Platinum-based che-
motherapy is used to treat patients that have
Hematology/Oncology

11. The correct answer is B. Given this patients metastatic nonseminomatous germ cell tumors
clinical picture and laboratory results, he likely after treatment by orchiectomy or with bulky
has chronic myelogenous leukemia (CML). retroperitoneal disease. In the case of unilateral
The differential diagnosis includes acute my- testicular cancer, a bilateral orchiectomy is not
elogenous leukemia and myelodysplastic syn- warranted.
drome. A denitive diagnosis can be made by
Answer C is incorrect. Prophylactic mediasti-
cytogenetic studies that reveal the Philadelphia
nal radiation had been practiced in the past
chromosome (9,22 translocation), which is
but it is no longer used. The myelosuppression
pathognomonic for CML. Ninety percent of
that may result can compromise the patients
patients with CML will demonstrate this trans-
ability to receive chemotherapy at a later date.
location, and in the 10% who do not a bone
marrow biopsy will be denitive. Answer D is incorrect. Retroperitoneal lymph
node dissection is used in treating low-stage
Answer A is incorrect. Coagulation studies
nonseminomatous germ cell tumors and has
are helpful in assessing for liver disease or in
signicant morbidity. This technique often
bruising of unknown etiology; however, this
damages the sympathetic innervation of the
patients easy bruising is likely due to thrombo-
seminal vesicle, which can lead to retrograde
cytopenia, which in turn is likely due to CML.
ejaculation and infertility. The case presented
Answer C is incorrect. While low levels of leu- here is a seminoma and does not warrant this
kocyte alkaline phosphatase are seen in CML, surgical procedure.
this information alone cannot make a deni-
tive diagnosis. 13. The correct answer is B. Fungal infections be-
come increasingly important in bone marrow
Answer D is incorrect. Iron studies are helpful
transplant after the rst 7 days posttransplan-
in evaluating microcytic anemia of unknown
tion. Patients with graft-versus-host disease
etiology; however, this patients anemia is likely
(GVHD) who require long-term immunosup-
due to CML.
pression are at high risk of infection by organ-
Answer E is incorrect. While CML is the sus- isms such as Candida and Aspergillus. These
pected diagnosis based on the clinical picture, infections can occur even after engraftment
further studies are needed to conrm this diag- and resolution of neutropenia (which would be
nosis. expected at posttransplant day 45). Other infec-
tions that are important during this time frame
12. The correct answer is E. Seminoma is a tumor are cytomegaloviral pneumonia, parasitic in-
type that is extremely sensitive to radiation. fection, and human herpes virus 6 infection of
Stages I or IIA disease can be successfully the skin and mucous membranes.
treated with orchiectomy and irradiation of the
Answer A is incorrect. The risk of bacterial
retroperitoneum. The low dose of radiation
infection is high for bone marrow transplant
that is required is usually well-tolerated with
patients in the rst 2030 days following trans-
HIGH-YIELD SYSTEMS
Chapter 7: Hematology/Oncology Answers 189

plant. This is due to the 14 weeks of neutro- The tumor phase may follow the plaque stage,
penia that follows the transplantation and ren- and involves deep involvement of the dermis
ders patients susceptible to aerobic bacteria with growth in height, forming discrete tumors.
found in the gut and on the skin. Infection The nal and most severe stage of disease is a
with Nocardia is a concern during the 30- to systemic illness generally called Szarys syn-
90-day posttransplant period. Infections with drome. During this phase the entire skin is in-
Staphylococcus aureus are generally limited to volved with lymphoma cells, and telltale Szary
the preengraftment period in the 13 weeks cells are found in the circulation. This is the
posttransplant. This type of infection would only phase of CTCL that affects the systemic
not be expected in a patient 45 days posttrans- circulation. This patients disease is consistent
plant. with the plaque stage of disease, since the der-
mis is invaded by lymphomatous cells. Several
Answer C is incorrect. Patients are most at risk
treatments exist for CTCL and vary according to

Hematology/Oncology
for gram-negative bacteremia in the 13 weeks
stage. Early (patch or plaque) stage disease may
following transplant. These infections are usu-
be treated with topical nitrogen mustards on an
ally caused by intestinal ora such as Escheri-
outpatient basis. More advanced disease may be
chia coli, Pseudomonas, and Klebsiella.
treated with conventional radiation therapy or
Answer D is incorrect. Patients being treated total skin electron beam therapy.
for hematogenous malignancy are indeed at
Answer A is incorrect. Bone marrow trans-
increased risk of Pneumocystis jiroveci pneumo-
plant may be considered in cases of Szarys
nia. However, they are treated with trimetho-
syndrome. It would not be anticipated in a pa-
prim-sulfamethoxazole prophylaxis, which
tient with plaque stage disease because of the
prevents 100% of cases when taken correctly.
relatively small chance that the disease might
Provided this patient has been compliant with
progress to systemic illness.
her medications as stated, her risk of acquiring
this disease is quite low. Answer B is incorrect. Although observation is
appropriate for certain patients, it is rarely re-
Answer E is incorrect. The risk of herpes sim-
lied upon because of the real chance that dis-
plex virus (HSV) reactivation is highest in the
ease might progress to disgurement or death.
2 weeks following transplant. It causes severe
Because topical treatments have relatively be-
mucositis that has been shown to occur with
nign adverse effect proles, they are generally
lower incidence if HSV-seropositive patients
used for even mild early stage disease.
are treated with prophylactic acyclovir.
Answer C is incorrect. Occasionally, systemic
Answer F is incorrect. Patients with Hodgkins
chemotherapy is required to treat CTCL; how-
disease are at an unexplained increased risk for
ever, it is generally used only after the onset of
Salmonella infection. No such known risk ex-
systemic disease (Szarys syndrome).
ists for patients with promyelocytic leukemia.
Answer D is incorrect. Systemic corticoster-
14. The correct answer is E. The patient is present- oids are not generally used to treat CTCL.
ing with cutaneous T-cell lymphoma (CTCL),
Answer F is incorrect. More advanced or re-
or mycosis fungoides. This disease is a T-lympho-
fractory CTCL may be treated with total skin
cyte lymphoma that is generally indolent. CTCL
electron beam therapy. This method of treat-
may affect patients in several different stages, al-
ment is generally reserved for patients with in-
though one stage does not necessarily lead to an-
volvement of large areas of skin, or with disease
other. Patients may live for 20+ years without pro-
refractory to less expensive and more conve-
gression of disease. The phases of disease are as
nient therapies.
follows: The phase of least severity is a patch
stage, involving only the epidermis in discrete ar-
15. The correct answer is B. Although DVTs in
eas of the skin. This phase may or may not prog-
this population are often attributed to the use
ress to a plaque stage, affecting discrete skin areas
of oral contraceptives, there is a strong family
but invading the dermis as well as the epidermis.
history of recurrent DVTs and therefore we
HIGH-YIELD SYSTEMS
190 Section I: Organ Systems Answers

must consider hereditary disorders of the coag- 16. The correct answer is B. This prole is consis-
ulation system. Factor V Leiden is an auto- tent with hemophilia B. Hemophilia B (Christ-
somal dominant disorder in which coagulation mas disease) is an X-linked recessive coagulop-
factor Va (the activated form of coagulation athy involving a deciency of coagulation
factor V) is resistant to degradation by activated factor IX that may lead to mild to severe bleed-
protein C. Factor Va remains active and allows ing, depending on the factor activity. A pro-
for the conversion of prothrombin to thrombin, longed activated partial thromboplastin time is
thereby promoting a hypercoagulable state in seen with hemophilia. Prothrombin time,
patients. Factor V Leiden accounts for 40% bleeding time, and platelet count are expected
50% of venous thromboemboli in cases in to be normal in a patient with hemophilia.
whom a genetic predisposition is found. A pa-
Answer A is incorrect. This is a normal prole
tient with a heterozygous mutation of the gene
and is not consistent with the diagnosis of he-
encoding factor V carries a relative risk of 7 for
Hematology/Oncology

mophilia.
DVTs, while a patient with the mutation in
conjunction with oral contraceptive use carries Answer C is incorrect. This prole is consis-
a relative risk of 35 for DVTs. tent with hemophilia A. Hemophilia A is an
X-linked recessive coagulopathy involving a
Answer A is incorrect. Factor V Leiden is the
deciency in coagulation factor VIII that may
most common cause of inherited thrombo-
lead to mild to severe bleeding, depending on
philia. Antithrombin is an inhibitor of throm-
the factor activity. A prolonged activated partial
bin and of coagulation factors IXa and Xa.
thromboplastin time is seen with hemophilia.
Although antithrombin deciency would lead
Prothrombin time, bleeding time, and platelet
to a hypercoagulable state, a mutation in the
count are expected to be normal in a patient
gene encoding coagulation factor V is more
with hemophilia.
common and thus is more likely to be the un-
derlying cause of a venous thrombotic event Answer D is incorrect. This prole is consis-
than is antithrombin deciency. tent with thrombocytopenia, as seen with disor-
ders such as idiopathic thrombocytopenic pur-
Answer C is incorrect. Protein C excess would
pura (ITP) or HIT.
lead to inactivation of coagulation factors Va
and VIII, and decreased propensity for the for- Answer E is incorrect. This prole is consistent
mation of thrombi. with von Willebrands disease (vWD). vWD is
an autosomal dominant genetic defect in von
Answer D is incorrect. Factor V Leiden is
Willebrand factor (vWF), which both mediates
the most common cause of inherited throm-
platelet binding to damaged endothelium and
bophilia. Protein S is a cofactor of the protein
binds/stabilizes factor VIII. Patients with vWD
C system in which factor Va and factor VIIIa
typically present with a prolonged bleeding
are inactivated and clot formation is thereby
time, a prolonged activated partial thromboplas-
inhibited. Although protein S deciency leads
tin time, and a factor VIII deciency.
to a severely hypercoagulable state, a mutation
in the gene encoding coagulation factor V is
17. The correct answer is B. Courvoisiers sign is a
more common and thus is more likely to be
palpable nontender gallbladder as a result of
the underlying cause of a venous thrombotic
common bile duct obstruction/compression by
event than is protein S deciency.
pancreatic adenocarcinoma at the head of the
Answer E is incorrect. Factor V Leiden is the pancreas. Biliary obstruction below the level of
most common cause of inherited thrombo- the cystic duct is unlikely to be caused by stone
philia. The increased relative risk of develop- disease.
ing venous thrombosis with factor V Leiden is
Answer A is incorrect. Chandelier sign refers
7, whereas that of a prothrombin gene muta-
to the pain on manipulation of the cervix and
tion is 2.8.
uterus during a pelvic examination in a patient
with pelvic inammatory disease.
HIGH-YIELD SYSTEMS
Chapter 7: Hematology/Oncology Answers 191

Answer C is incorrect. Cullens sign refers to Answer E is incorrect. Staphylococcal skin in-
ecchymosis in the periumbilical region sug- fections include impetigo and abscesses. The
gestive of retroperitoneal hemorrhage in severe patients rash does not resemble either of these.
acute pancreatitis. In addition, she feels well and is afebrile.
Answer D is incorrect. Grey-Turners sign re- Answer F is incorrect. The patient may sim-
fers to ecchymosis in the ank that suggests ret- ply have sunburn. From her account, however,
roperitoneal hemorrhage accompanying severe she avoided much direct sunlight and applied
acute pancreatitis. sunscreen. Regardless, a patient at risk for
chronic GVHD deserves a full work-up when
Answer E is incorrect. Murphys sign refers to
presenting with these symptoms. In addition, it
the inspiratory arrest during deep palpation of
seems that her hands and wrists were sore and
the right upper quadrant in patients with acute
inamed, leading away from the diagnosis of
cholecystitis.

Hematology/Oncology
sun poisoning.
18. The correct answer is A. Twenty to 50% of pa-
19. The correct answer is B. This patient has a pi-
tients with a history of allogeneic bone marrow
tuitary mass causing headaches and blurred vi-
transplant develop GVHD. GVHD is caused
sion due to compression of the optic chiasm. It
by a proliferation of grafted donor T lympho-
is most likely a prolactin-secreting adenoma
cytes that ultimately reject the hosts foreign
because the patient is complaining of de-
proteins, leading to end-organ damage. This
creased libido, a common nding in men with
disease is a separate entity from acute GVHD
hyperprolactinemia, along with possible impo-
and resembles an autoimmune disorder. It may
tence, infertility, gynecomastia, and galactor-
result in macular-papular rash, as seen in this
rhea. Given his history of peptic ulcer disease
patient; sicca syndrome; arthritis; and bile duct
as well as a family history of peptic ulcer dis-
degeneration, leading to jaundice. The chronic
ease and pancreatic tumors, type I multiple en-
form of GVHD may occur months or even
docrine neoplasia (MEN) should be consid-
years after transplant and is likely due to vascu-
ered in this patient since the autosomal
lar damage by the proliferated T lymphocytes.
dominant disorder consists of Pituitary tumors,
Patients who are >40 years at the time of trans-
Pancreatic tumors, and Primary hyperparathy-
plant are at the greatest risk of developing
roidism (the 3 Ps). Parathyroid hormone
chronic GVHD. This complication is irrevers-
(PTH) as well as serum calcium and phosphate
ible, but prednisone is the standard of care.
levels should be checked to evaluate for hyper-
Answer B is incorrect. Following bone mar- parathyroidism and resultant hypercalcemia
row transplant, patients are not placed on an and hypophosphatemia.
immunosuppressive regimen in the absence
Answer A is incorrect. Calcitonin is a hor-
of GVHD. It is stated that the patient was dis-
mone secreted by thyroid C cells. Serum lev-
charged on no medications 6 months prior to
els may be elevated in patients with medullary
her trip.
thyroid cancer. This malignancy is associated
Answer C is incorrect. Although the patient with type II MEN syndromes, not type I MEN.
certainly could be suffering from new-onset
Answer C is incorrect. Renal ultrasound
systemic lupus erythematous, in a patient with
would not be indicated in this patient, as there
a history of bone marrow transplant, chronic
is no association of renal pathology with any of
GVHD is much more likely.
the MEN types.
Answer D is incorrect. Rosacea is a possible
Answer D is incorrect. Thyroid ultrasound
diagnosis for this patient, particularly because
could be used to evaluate for a mass if there was
rosacea is often exacerbated by sun exposure.
concern for thyroid malignancy. Although med-
However, chronic GVHD is much more likely.
ullary thyroid cancer is found in patients with
type II MEN syndromes, it is not associated with
pituitary adenomas and pancreatic tumors.
HIGH-YIELD SYSTEMS
192 Section I: Organ Systems Answers

Answer E is incorrect. Plasma and/or urinary Shwachman-Diamond syndrome. Acquired


metanephrines are used to evaluate for pheo- causes are more common, including a wide va-
chromocytoma, an adrenal tumor that pro- riety of drugs and chemicals, ionizing radiation,
duces catecholamines. Pheochromocytomas and viruses. Petechiae, pallor, fatigue, and fe-
are found in association with type IIA MEN, ver are consistent with this diagnosis. However,
which also includes medullary thyroid cancer aplastic anemia would not cause hypertension,
and hyperparathyroidism, and type IIB MEN, lymphadenopathy, periorbital bruising, propto-
which also may include medullary thyroid can- sis, or elevated urinary catecholamines.
cer, mucosal neuromas, intestinal ganglioneu-
Answer C is incorrect. Child abuse should be
romas, and marfanoid habitus. Pheochromocy-
considered if a child sustains injuries incom-
toma is not typically associated with pancreatic
patible with normal activities. His periorbital
and pituitary tumors.
bruise might resemble a bruise from an eye
Hematology/Oncology

being punched by a st. However, proptosis


20. The correct answer is D. This child has meta-
would not occur in this situation. Child abuse
static neuroblastoma, which is common in
could explain failure to thrive if physical abuse
children <2 years old. Primary neuroblastoma
was accompanied by neglect. However, it
most commonly presents as a painless abdomi-
would not explain any of his other symptoms,
nal mass, hypertension, respiratory distress,
his hypertension, or his laboratory values.
Horners syndrome, or cord compression. Of-
ten, however, neuroblastoma is not diagnosed Answer E is incorrect. Pheochromocytomas
until after it has metastasized. Tumor inltra- is a catecholamine-secreting neoplasm of
tion of the bone marrow causes pancytopenia. chromafn cells that causes intermittent hy-
Tumor inltration of the periorbital bones pertension and can result in elevated urinary
causes periorbital ecchymoses and proptosis, catecholamines. Other symptoms of a pheo-
an appearance often referred to as raccoon chromocytoma include headache, perspira-
eyes. Low-grade fever, fatigue, and failure to tion, palpitations, pallor, and diaphoresis. A
thrive (which is evidenced by his small size) pheochromocytoma does not cause lethargy,
occur with metastatic disease. Urinary cate- abnormal complete blood count, fever, lymph-
cholamines are elevated in neuroblastoma, but adenopathy, or periorbital bruising, and is thus
denitive diagnosis requires biopsy. Treatment unlikely in this child.
involves chemotherapy and surgical excision of
local disease. Prognosis varies with age at pre- 21. The correct answer is A. Endometriosis is a
sentation: infants with metastatic disease re- condition in which endometrial tissue abnor-
spond favorably to treatment, while most chil- mally implants at sites outside the uterus. Ova-
dren >1 year with advanced neuroblastoma ries are the most common site of involvement,
will die from progressive disease despite inten- but implantation in extrapelvic sites such as
sive multimodality therapy. the umbilicus, lungs (pretty rare, but docu-
mented), and surgical scars has also been doc-
Answer A is incorrect. ALL is the most com-
umented. Most cases occur in women 2030
mon form of cancer in children. ALL can
years old, and it is rarely seen after menopause.
cause pancytopenia, lymphadenopathy, fever,
Endometriosis may be asymptomatic, but it
failure to thrive, petechiae, and pallor. ALL
may also cause menorrhagia, severe pelvic
does not cause periorbital ecchymoses, prop-
pain, and infertility, as seen in this patient. The
tosis, hypertension, or elevated urinary cat-
amount of pain is often out of proportion to
echolamines.
the number of extrauterine implants. Like en-
Answer B is incorrect. Aplastic anemia, char- dometrial tissue in the uterine cavity, extrauter-
acterized by pancytopenia and hypocellular ine endometrial tissue is affected by hormonal
marrow, is usually due to injury to the pluri- cycles and symptoms may improve with use of
potent stem cell. The initial insult may be oral contraceptive pills. Pathologic review of
congenital or acquired. Congenital causes are biopsied lesions is required for diagnosis, and
uncommon, including Fanconis anemia and the gold standard for doing so is laparoscopy.
HIGH-YIELD SYSTEMS
Chapter 7: Hematology/Oncology Answers 193

Classic lesions appear as powder burns or are egaly. Diagnosis is most common in the fourth
mulberry- or raspberry-colored; 30%40% of to sixth decades of life, and irradiation in-
women with endometriosis will experience de- creases the risk of developing CML. Because it
creased fertility. Endometriosis is commonly is a proliferation of mature myeloid cells, pe-
found in the work-up of women presenting ripheral smear should show predominant leu-
with infertility, where infertility is dened as kocytosis with cells in all stages of maturation.
the inability to conceive after 1 year of unpro- Denitive diagnosis relies on identication of
tected intercourse. Women with endometriosis the Philadelphia chromosome or the bcr-abl
may have female relatives who are also af- fusion product. However, about 5% of patients
fected. have atypical CML without the presence of
bcr-abl and show a poor response to therapy.
Answer B is incorrect. Endometriosis results
when endometrial tissue implants outside the Answer A is incorrect. A bone marrow biopsy

Hematology/Oncology
uterus. Thus, an endometrial biopsy sampling may be part of the work-up of leukemia. CML
of uterine tissue would not examine ectopic may show hypercellularity of the bone marrow,
tissue and would be of little help in identifying as well as differentiated cells rather than blasts.
this patients underlying condition. A bone marrow biopsy will not, however, dif-
ferentiate the type of leukemia. The diagnosis
Answer C is incorrect. Hysterosalpingography
of CML depends on cytogenetics.
(HSG) is performed by retrograde injection of
dye into the cervix. As the dye lls the uterus Answer C is incorrect. -Fetoprotein is a
and spills out of the fallopian tubes into the marker for liver cancer, which normally pre-
pelvic cavity, HSG yields an outline of these sents with jaundice, elevated liver enzymes,
structures and provides information about ana- and abdominal fullness. It is most common in
tomical defects. HSG should not be performed individuals with cirrhosis, either from hepatitis
in cases of suspected endometriosis, as endo- infection or alcohol use.
metriosis is theorized to occur by retrograde
Answer D is incorrect. Reed-Sternberg cells
migration of endometrial tissue into the pelvic
can be seen in Hodgkins lymphoma, which
cavity, and HSG could potentially cause the
typically presents as lymphadenopathy rather
implantation of extrauterine endometrial tis-
than an elevated WBC count.
sue.
Answer E is incorrect. Colon cancer may be
Answer D is incorrect. While increased levels
consistent with the constitutional symptoms of
of endometrial prostaglandin F2 are impli-
this patient but would not explain the hemato-
cated in primary dysmenorrhea, the prosta-
logic ndings.
glandin F2 level is not routinely measured in
gynecology. Furthermore, increased levels are
23. The correct answer is E. The patients most
not diagnostic of endometriosis, which requires
likely diagnosis is multiple myeloma. This ma-
direct visualization of lesions by laparoscopy.
lignancy of plasma cells leads to proliferation
Answer E is incorrect. Pelvic ultrasound is of a single plasma cell clone. This plasmacyto-
used to identify structural defects within the sis leads to secretion of osteoclast-activating
abdominal and pelvic cavities. It provides in- factor by myeloma cells, increasing osteoclastic
formation about the size and composition activity and causing lytic lesions. The plasma-
(solid or cystic) of a mass, but cannot provide cytosis also contributes to immunosuppression
information about a lesions gross appearance. (because one antibody clone dominates rather
Diagnosis of endometriosis requires direct vis- than the hundreds of thousands that are found
ualization with biopsy (and pathologic conr- in a normal patient) and anemia. The prolifer-
mation) of extrauterine endometrial tissue. ation of plasma cells also causes a proliferation
of antibody clones that spill into the urine,
22. The correct answer is B. CML, in its chronic known as Bence Jones proteins, and may even-
phase, presents with constitutional symptoms tually cause renal failure.
along with abdominal fullness due to splenom-
HIGH-YIELD SYSTEMS
194 Section I: Organ Systems Answers

Answer A is incorrect. Anorexia nervosa may euvolemic hyponatremia. Diagnosis is based


cause increased susceptibility to fracture on urine osmolality >50100 mOsm/kg with
through osteopenia. However, this would not concurrent serum hyposmolality in the ab-
cause lytic lesions on radiograph. sence of a physiologic reason for increased
ADH (chronic heart failure or cirrhosis). Urine
Answer B is incorrect. Type 1 Gauchers dis-
sodium >20 mEq/L is used to show that the pa-
ease is due to a genetic deciency of gluco-
tient is not hypovolemic. Of the bronchogenic
cerebrosidase, and can cause osteopenia, os-
carcinomas of the lung, small cell carcinoma
teolytic lesions, and pathologic fractures. It is
carries the worst prognosis and is assumed to
usually diagnosed in childhood or, rarely, early
be metastatic at time of diagnosis.
adulthood. Gauchers disease is most often
found in the Ashkenazi Jewish population, but Answer A is incorrect. Adenocarcinomas typi-
it is a rare disease that is much less common cally cause the paraneoplastic syndromes of
Hematology/Oncology

than multiple myeloma. A patient with Gauch- digital clubbing, hypertrophic pulmonary os-
ers disease would not be at increased suscep- teoarthropathy, thrombophlebitis, and nonbac-
tibility to infection, and would normally have terial verrucous endocarditis. Adenocarcinoma
splenomegaly and growth delay in addition to and the other non-small cell lung cancers carry
the bone symptoms. a better prognosis than small cell.
Answer C is incorrect. Congenital osteoporo- Answer B is incorrect. Bronchoalveolar cell
sis would cause multiple fractures, but would carcinoma is most often associated with mul-
be expected to cause diffuse low bone density, tiple nodules on imaging studies, interstitial
not lytic lesions. inltration, and prolic sputum production. It
may be confused with interstitial pneumonia
Answer D is incorrect. Domestic abuse should
on x-ray of the chest. Bronchoalveolar cell car-
always be considered in cases of multiple frac-
cinoma and the other non-small cell lung can-
tures; however, it does not account for the pa-
cers carry a better prognosis than small cell.
tients other symptoms or her lytic lesions.
Answer C is incorrect. Large cell carcinoma
Answer F is incorrect. One method of differ-
is a very uncommon type of lung cancer that
entiating multiple myeloma from other ma-
is associated with gynecomastia. Large cell car-
lignancies is through bone scan. Osteoclastic
cinoma carries a slightly worse prognosis than
metastases from breast cancer can appear as
the other non-small cell lung cancers.
hot spots on bone scan, although osteoblastic
lesions would be invisible. Malignancy is less Answer E is incorrect. Squamous cell car-
likely in the setting of increased protein in the cinomas typically cause the paraneoplastic
urine, especially if there is a high M spike on syndrome of hypercalcemia via production of
protein electrophoresis. A biopsy can be done PTH-related peptide (PTHrP). Squamous cell
on the lytic lesion to conrm the diagnosis if carcinoma and the other non-small cell lung
there are few Bence Jones proteins. cancers carry a better prognosis than small
cell.
24. The correct answer is D. An 11.3-kg (25-lb)
unintentional weight loss, hemoptysis, lesion 25. The correct answer is B. Although symptoms
visible on CT scan, and a signicant smoking of hyperkalemia are rare and when present are
history should raise concern for lung cancer. nonspecic, they include lethargy and weak-
The patient is suffering from euvolemic hy- ness. In the context of normal kidney function
ponatremia due to syndrome of inappropriate as well as hyperuricemia and hypocalcemia,
ADH secretion (SIADH), a paraneoplastic syn- this hyperkalemia is likely the result of the che-
drome seen in small cell carcinoma of the motherapeutic effect on his tumor. This phe-
lung. Small cell carcinoma, along with sarcoi- nomenon, known as tumor lysis syndrome,
dosis, pneumonia, head injury, and antipsy- presents shortly after commencement of che-
chotics or antidepressants can cause nonosmot- motherapy with elevations in serum urate lev-
ically stimulated release of ADH, resulting in els, and can progress to urate-induced kidney
HIGH-YIELD SYSTEMS
Chapter 7: Hematology/Oncology Answers 195

failure. Potassium may climb to dangerously Answer B is incorrect. Imatinib is an inhibitor


high levels as well. of the bcr-abl gene product that is the hallmark
of CML and would not be indicated in this pa-
Answer A is incorrect. Potassium is readily ab-
tient.
sorbed from ingested food in the gastrointesti-
nal tract. However, increased potassium intake Answer D is incorrect. Radiation therapy is
is rarely a cause of signicant hyperkalemia in reserved for patients with CLL with symptoms
the setting of normal renal function. secondary to mass effects of large lymphoid
masses.
Answer C is incorrect. Acidosis can lead to ex-
tracellular potassium shifts, causing a transient Answer E is incorrect. Splenectomy is useful
increase in plasma potassium levels. The pa- when there is marked splenomegaly and ane-
tients electrolyte panel reveals a normal anion mia or thrombocytopenia refractory to medical
gap, and acidosis would not explain his uric treatment.

Hematology/Oncology
acid and calcium abnormalities.
27. The correct answer is E. The patient has been
Answer D is incorrect. Pseudohyperkalemia
diagnosed with follicular lymphoma, a low-grade
is due to in vitro hemolysis of RBCs, causing
disease of B-lymphocyte origin that is a result of a
them to release their potassium-rich contents.
t(14;18) translocation. Such disease is almost al-
It occurs most frequently when there is a long
ways stage III (involved nodes above and below
delay between specimen collection and analy-
the diaphragm) or stage IV(disseminated disease)
sis. This patient displays symptoms of true hy-
at diagnosis, as was the case for this patient.
perkalemia, making pseudohyperkalemia less
Rarely, low-grade lymphomas are curable with
likely.
intensive chemotherapy; however, they are gen-
Answer E is incorrect. Renal failure is a cause erally indolent, with most patients surviving 7 to
of hyperkalemia, especially when ltration lev- 10 years with no therapy. The watch and wait
els fall or the distal tubule fails to function nor- approach has become the standard of care for pa-
mally due to nephrotoxins or decreased distal tients with low-grade lymphomas, with palliative
delivery of ltrate. This patient, however, dis- chemotherapy started only on progression of the
plays a normal creatinine and is unlikely to be disease. Histologic transformation may occur in
experiencing signicant renal failure. 30%50% of follicular lymphomas to a large
B-lymphocyte lymphoma. Median survival is <1
26. The correct answer is C. This patient most year after transformation occurs.
likely has chronic lymphocytic leukemia
Answer A is incorrect. In a patient who is el-
(CLL), which mainly affects individuals >60
derly with other medical problems, bone mar-
years old and accounts for 30% of all leukemias
row transplant would not be indicated for a
in the United States. The patients are often as-
low-grade lymphoma.
ymptomatic at presentation, and the disease is
only detected on laboratory tests. In this case, Answer B is incorrect. This patient will most
the patient has an increased WBC count con- likely survive for several years before his disease
sisting predominantly of lymphocytes. This pa- progresses. Hospice care is generally reserved
tient has a predicted median life expectancy of for terminally ill patients with a life expectancy
7 years. Early treatment does not improve the of <6 months. It is not indicated for this pa-
outcome, and management is palliative rather tients current health status.
than aimed at a cure. Because this patient is as-
Answer C is incorrect. Because the natural
ymptomatic, no treatment is indicated at this
history of follicular lymphoma is such that the
time.
patient may live for 710 years without disease
Answer A is incorrect. Chemotherapy is used in progression, palliative radiation is not indicated
the setting of advanced stage disease, progressive in the patient who is asymptomatic. It would
disease, and recurrent infections, among others. be considered only in the presence of severe
It is not curative but symptomatic. pain or obstructive symptoms.
HIGH-YIELD SYSTEMS
196 Section I: Organ Systems Answers

Answer D is incorrect. The CHOP protocol var involvement it results in diffuse erythema,
(Cyclophosphamide, Hydroxydaunomycin, On- not discrete ulcerative lesions.
covorin [vincristine], and Prednisone) is an in-
Answer F is incorrect. Metronidazole is used
tensive multiagent chemotherapeutic regimen
to treat bacterial vaginosis, which is an infec-
that is often used to treat intermediate- and high-
tion of the vaginal canal, not the external geni-
grade lymphomas. It would not be indicated in
talia. Women typically complain of a shy odor
treatment of low-grade disease because it has a
and increased vaginal discharge. Diagnosis is
severe adverse effect prole with low success in
made via microscopic examination of vaginal
indolent disease.
secretions obtained by speculum examination,
which will show the characteristic clue cells,
28. The correct answer is A. This is a common pre-
epithelial cells with cocci attached to their sur-
sentation of vulvar carcinoma. Typically it pres-
face.
ents in women 6570 years of age, and the most
Hematology/Oncology

common presenting complaint is vulvar pruri-


29. The correct answer is D. A stroke or transient
tus. Previous human papillomavirus (HPV) in-
ischemic attack in a child should prompt a
fection is a major risk factor for the development
search for causes other than those commonly
of vulvar carcinoma. Denitive diagnosis re-
seen in adults. Sickle cell disease is often
quires a biopsy. Spread is by local invasion and
thought of in African-Americans, but should
then to regional lymph nodes, so treatment in-
also be considered in individuals from Central
cludes wide surgical excision and regional
or South America. Approximately 11% of chil-
lymph node excision.
dren with sickle cell disease have a stroke be-
Answer B is incorrect. Topical estrogen treats fore the age of 20 years, and transient ischemic
atrophic vaginitis, which results from a reduc- attacks can often be the rst sign of an impend-
tion of endogenous estrogens in postmeno- ing stroke. The pallor and fatiguing can also be
pausal women. Symptoms include vaginal clues pointing toward anemia. Chronic red
soreness, dyspareunia, and burning. On exam- blood cell transfusion in sickle cell patients has
ination the vaginal mucosa is thin, with a de- been shown to reduce the risk of cerebral
crease in the number of vaginal folds. thrombotic events as well as pain crises and
acute chest syndrome.
Answer C is incorrect. Acyclovir is used to
treat HSV infection. Primary infection is char- Answer A is incorrect. Familial hypercholes-
acterized by malaise and systemic symptoms, terolemia would certainly raise the possibility
whereas genital involvement results in painful of developing strokes, but this would be un-
vesicular eruptions that progress to painful ul- usual given the negative family history and lack
cers. of other physical ndings such as xanthomas
(lipid deposits in soft tissues).
Answer D is incorrect. Cryotherapy is used
to treat condyloma acuminata, or anogeni- Answer B is incorrect. Hemorrhagic strokes
tal warts. Diagnosis of condyloma acuminata secondary to hypertension occur only after an
can usually be made by clinical inspection. individual has had the disease for many years.
Lesions are typically exophytic and not ulcer- It is unlikely that a 9-year-old boy would al-
ative. Although they are caused by HPV infec- ready have a complication of hypertension.
tions (types 6 and 11), they do not cause vulvar
Answer C is incorrect. Multiple myeloma can
pruritus.
cause both anemia due to myelophthisis and
Answer E is incorrect. Fluconazole is used to stroke secondary to hyperviscosity syndrome.
treat infections with Candida albicans. Can- However, this patient is much too young to
didal vulvovaginitis can present with genital consider this diagnosis, which usually pre-
pruritus, as well as burning, dysuria, and dys- sents after age 40 years with a median age of
pareunia. A thick cottage cheese-like odorless 66 years.
discharge is characteristic. When there is vul-
Answer E is incorrect. Simple partial seizures
may cause transient focal decits without
HIGH-YIELD SYSTEMS
Chapter 7: Hematology/Oncology Answers 197

change in consciousness, but would not explain true in this patient who has a signicant smok-
the clinical picture of anemia. Additionally, sei- ing history and weight loss.
zures do not leave residual neurologic decits.
Answer B is incorrect. MRI of the brain may
be used to evaluate patients with suspected
30. The correct answer is B. Common variable
central nervous system malignancy. However,
immunodeciency (CVID) is a syndrome
brain tumors are less likely to secrete PTHrP.
characterized by hypogammaglobulinemia in
Lung cancer is a much more common cause
combination with phenotypically normal B
of tumor-induced hypercalcemia, especially in
lymphocytes. The disease presents suddenly in
a patient who has a signicant smoking history.
the third or fourth decade after recurrent infec-
tions and lack of response to vaccines. It is a di- Answer C is incorrect. A parathyroid ultra-
agnosis of exclusion, and its pathogenesis is not sound may be used to localize a parathyroid
well understood. Women in their fth and adenoma in a patient with suspected primary

Hematology/Oncology
sixth decades of life with CVID are at a 438- hyperparathyroidism. These patients will also
fold increased risk of developing lymphoma. In present with hypercalcemia and hypophos-
addition to malignancy, patients with CVID phatemia, but they will have elevated levels of
are at increased risk of chronic lung disease, PTH.
autoimmune phenomenon, and chronic diar-
Answer D is incorrect. Chronic renal disease
rhea.
can lead to phosphate retention, which binds
Answer A is incorrect. Patients with CVID are calcium and leads to a secondary hyperpara-
not at increased risk of cardiovascular disease, thyroidism. Patients would have an elevated
although they are at greater risk for chronic creatinine level, hyperphosphatemia, and el-
lung disease (most commonly bronchiectasis). evated PTH levels. End-stage renal disease
would not be expected to show hypercalcemia
Answer C is incorrect. The risk of miscarriage
and hypophosphatemia. Renal tumors rarely
is not increased in patients with CVID.
produce PTHrP. Thus a renal biopsy would
Answer D is incorrect. There is no association not be helpful in identifying the source of this
between CVID and renal disease. patients problem.
Answer E is incorrect. Twenty-ve percent Answer E is incorrect. Hypercalcemia with
of patients with CVID have splenomegaly. low to normal PTH can be due to vitamin D
However, there is no evidence that patients intoxication as a result of excessive ingestion of
with CVID are more likely to autoinfarct their vitamin D. The vitamin D metabolites calci-
spleens. diol and calcitriol should be measured if there
is no obvious source of malignancy and nei-
31. The correct answer is A. This patient has hyper- ther PTH nor PTHrP is elevated. In a patient
calcemia and hypophosphatemia. This is consis- who is not taking any medication, vitamin D
tent with hyperparathyroidism; however, the intoxication is less likely. In addition, vitamin
PTH level is low to normal. In addition to para- D intoxication would cause elevated phospho-
thyroid adenomas, malignancy can cause hyper- rus levels.
calcemia by the release of PTHrP. PTHrP mim-
ics the action of PTH, and causes hypercalcemia 32. The correct answer is B. A broadenoma is a
by increased bone resorption and increased dis- benign and slow-growing breast tumor with epi-
tal tubular reabsorption of calcium in the kid- thelial and stromal components. It is the most
ney. It causes hypophosphatemia by inhibiting common lesion in women <30 years old. This
proximal tubular reabsorption of phosphate. patients presentation of a sharply demarcated,
Malignancy (particularly lung cancer, breast mobile, nontender mass is classic for broade-
cancer, and multiple myeloma) should be sus- noma. In addition, these lesions often get larger
pected in patients with unexplained hypercalce- with pregnancy. These tumors are usually re-
mia and low or normal PTH. This is especially moved when they reach 24 cm in diameter.
HIGH-YIELD SYSTEMS
198 Section I: Organ Systems Answers

Answer A is incorrect. Carcinoma of the breast Answer B is incorrect. Dialysis has been used
is unlikely in this patient. Although the mass is to treat ITP, but it is not a rst-line treatment.
nontender, which is typical of carcinoma, her
Answer C is incorrect. Intravenous gamma-
age and the masss mobility and location make
globulin is the next step for relapse of disease
this diagnosis less likely.
after a course of steroids. It is not used as an
Answer C is incorrect. Although brocys- initial treatment.
tic changes are common in premenopausal
Answer E is incorrect. Platelet transfusion is
women, patients normally experience bilateral
indicated for active bleeding, particularly if
breast swelling and pain. Also, there is usually
platelet levels fall below 20,000/mm.
uctuation in symptoms with the menstrual
cycle.
34. The correct answer is A. The presence of in-
Answer D is incorrect. Galactocele is also a tussusception in an adult or older child should
Hematology/Oncology

dilatation of the duct in which a cyst becomes raise suspicion for Burkitts lymphoma. This
lled with a viscous milky uid. However, it disease is more common in patients living in
usually presents during or shortly after lacta- Africa, an association believed to be due to the
tion. high rate of infection with Epstein-Barr virus.
The African form tends to involve the maxilla
Answer E is incorrect. Mammary duct ecta-
and mandible rather than the abdomen. How-
sia usually occurs in patients in their 40s, and
ever, the African form of Burkitts lymphoma
presents as a new breast mass, with nipple dis-
can affect the Peyers patches of the intestine,
charge and breast tenderness. These symptoms
causing tremendous swelling that leads to foci
are caused by blockage and subsequent dilata-
for intussusception. The 8;14 translocation,
tion of the mammary duct.
which is associated with the c-myc oncogene,
is commonly found in Burkitts lymphoma.
33. The correct answer is D. ITP is a disorder of
This disease is a high-grade lymphoma that is
primary immune platelet destruction. The
very responsive to chemotherapy.
clinical presentation is of insidious onset of
mucocutaneous bleeding. The diagnosis is one Answer B is incorrect. The median age for
of exclusion; a complete blood cell count usu- presentation of CLL is 60 years. When symp-
ally shows isolated thrombocytopenia, and tomatic, this condition classically presents with
large platelets may be apparent on peripheral lymphadenopathy, hepatosplenomegaly, and
smear. Other tests may include a bone marrow lymphocytosis. Additionally, CLL rarely in-
biopsy to exclude aplastic anemia or drug-in- volves the gastrointestinal mucosa. Therefore,
duced suppression of megakaryocytes. In chil- CLL is not the most likely diagnosis.
dren, this disease is usually self-limiting and re-
Answer C is incorrect. CML typically affects
quires no treatment, but adults generally
patients older than 50 years. CML is a myelo-
require medical (through immunosuppression
proliferative disorder that is often discovered
or dialysis) or surgical (through splenectomy)
on routine blood tests where leukocytosis and
management. Children with chronic ITP have
thrombocytosis are discovered. When symp-
platelet counts from 20,00075,000/mm and
tomatic, fatigue, weight loss, malaise, abdomi-
typically do not require treatment. Pulse or
nal discomfort, and bleeding due to platelet
short-course corticosteroids may be used in re-
dysfunction are common. The t(9;22) translo-
fractory cases, but long-term daily steroid use
cation creates the Philadelphia chromosome,
should be avoided.
which is often seen in CML. Because CML is
Answer A is incorrect. Corticosteroids are the less likely to cause intussusception or to affect
initial treatment of choice of ITP if the disease a patient in his or her 20s, this answer is not
proves not to be self-limiting. Fifty to seventy- the best answer.
ve percent of adults will respond, but <20% of
Answer D is incorrect. Ewings sarcoma may
those patients will achieve long-term remission.
develop in any bone or soft tissue, but is most
HIGH-YIELD SYSTEMS
Chapter 7: Hematology/Oncology Answers 199

common in the at and long bones. This con- of gradual decline in renal function. The un-
dition is characterized by localized pain and derlying immune mechanism of chronic rejec-
swelling rather than intestinal involvement, so tion is poorly understood but is assumed to be
this is not the most likely diagnosis. humoral in origin. Unfortunately, chronic re-
jection is unresponsive to steroids and other
Answer E is incorrect. Szarys syndrome
immunosuppressives, and it is the single most
is a term used to describe a systemic form of
important cause of late graft failure. Factors
CTCL, during which Szary cells are visible
implicated in chronic rejection are cadaveric
in the peripheral blood and the entire skin sur-
source, past episodes of acute rejection, isch-
face is affected by erythematous lesions. This
emic injury to the donor kidney, prolonged
patients presentation is not consistent with
warm ischemic time prior to the transplant of
Szarys syndrome, the most advanced stage of
the kidney, and poor compliance with drug
CTCL. Szary syndrome usually occurs after
therapy.

Hematology/Oncology
the more localized stages (patch, plaque, and
tumor) have persisted for several years. Szary Answer A is incorrect. PTLD is an uncom-
cells are recognizable for their convoluted or mon condition of B-cell lymphoma occurring
cerebriform nuclei and scant cytoplasm. in immunosuppressed patients following organ
transplantation. Any transplant patient is at risk
35. The correct answer is A. The patient presents for PTLD, which presents with a mass com-
with symptoms of right-sided heart failure. prised of B lymphocytes that rarely causes a
Echocardiographic results are consistent with a rise in creatinine levels. However, this patient
restrictive cardiomyopathy, which leads to im- shows no sign of PTLD.
paired elasticity. Causes of restrictive cardio-
Answer C is incorrect. Although this patient is
myopathy include amyloidosis, hemochroma-
at an increased risk for urinary tract infection,
tosis, and sarcoidosis.
chronic antibiotic prophylaxis is not indicated.
Answer B is incorrect. Chagas disease is a
Answer D is incorrect. Acute rejection would
known cause of dilated cardiomyopathy. This
be characterized by a rapid rise in creatinine
patient presents with a restrictive cardiomyopa-
levels and histologically is characterized by in-
thy.
ammation without brosis. Chronic rejection
Answer C is incorrect. Doxorubicin is a known does not respond to high-dose steroids in the
cause of dilated cardiomyopathy. way that acute rejection does.
Answer D is incorrect. Idiopathic hypertrophic Answer E is incorrect. Chronic rejection is not
subaortic stenosis is a congenital form of hyper- resolved by increased immunosuppression.
trophic cardiomyopathy and a common cause
of sudden death among young athletes. Hyper- 37. The correct answer is B. In patients with sickle
trophic cardiomyopathy, unlike restrictive car- cell anemia, the primary agents of osteomyelitis
diomyopathy, results from impaired relaxation are Salmonella paratyphi and Staphylococcus
and on echocardiography shows decreased left aureus. Osteomyelitis is a common complica-
ventricular chamber size. tion of sickle cell disease, as sickle cell crises
may stop blood ow to portions of bones, allow-
Answer E is incorrect. Ischemic cardiomyop-
ing those segments of bone to become foci for
athy is a common cause of secondary dilated
infection. Any antibiotic regimen will need to
cardiomyopathy. Echocardiography demon-
cover both organisms. A third-generation cepha-
strates increased left ventricular size and di-
losporin, such as ceftriaxone, is an acceptable
minished ejection fraction. On the other hand,
choice.
wall thickness is decreased in dilated versus re-
strictive cardiomyopathy. Answer A is incorrect. Calcitonin is a hor-
mone involved in bone metabolism. It can be
36. The correct answer is B. The patient is experi- used for osteoporosis, Pagets disease, and some
encing chronic rejection of her graft, a process cancers. The patients bone lesions are not as-
HIGH-YIELD SYSTEMS
200 Section I: Organ Systems Answers

sociated with hormonal abnormalities, but Answer E is incorrect. Tumor lysis syndrome
rather with sickle cell osteomyelitis. is an oncologic emergency commonly seen in
patients with ALL, due to the fast turnover of
Answer C is incorrect. The patient is 3 years
leukemic cells.
old; therefore, while uoroquinolones are very
effective against Salmonella osteomyelitis, their
39. The correct answer is B. The presence of other-
use in a child may cause problems with bone
wise unexplained hematuria in an individual
development, and should be substituted for
over 40 denotes bladder cancer until proven
less morbid, but still effective, medications.
otherwise. The patient also has numerous risk
Answer D is incorrect. Etidronate is a bispho- factors that raise suspicion for bladder cancer,
sphonate useful in the treatment of Pagets dis- such as her smoking history, possible exposure
ease, osteoporosis, myeloma, and metastatic to dyes, and coffee drinking. Cystoscopy is the
disease to bone (from breast and prostate), mainstay of diagnosis and staging of bladder
Hematology/Oncology

among others. The patient in this question cancer. A cystoscope is inserted into the bladder,
most likely has an infectious process, given the and any tumor is then biopsied and resected.
history of sickle cell disease, fever, and leuko-
Answer A is incorrect. A CT scan of the pelvis
cytosis with left shift, and will require an anti-
is neither sensitive nor specic in the diagnosis
biotic to cure it.
of bladder cancer. However, this is important
Answer E is incorrect. Vitamin D is a vitamin in staging the cancer after diagnosis has been
with a pivotal role in the prevention of rickets made.
and osteomalacia; however, the diagnosis in this
Answer C is incorrect. Serum creatine ki-
case is more likely to be osteomyelitic lesions.
nase level is the most important test in evalu-
ating for rhabdomyolysis, which may lead to
38. The correct answer is A. ALL is a disease of
myoglobinuria that mimics hematuria. This
hyperproliferative immature lymphocytes and
patient, however, has no indication that she is
occurs mainly in children, with a second peak
breaking down muscle tissue (such as muscle
in persons who are elderly. It is associated with
pain, aches, or a history of recent trauma).
a number of other diseases, including Downs
syndrome, Fanconis anemia, and ataxia telang- Answer D is incorrect. Ultrasound may be
iectasia. useful in evaluating the upper tracts for renal
parenchymal disease, hydronephrosis, or a soft
Answer B is incorrect. Immunohistochemistry
tissue mass. However, it cannot determine the
usually shows positive terminal deoxynucleoti-
depth of invasion and is not useful in the diag-
dyl transferase in 95% of patients with ALL,
nosis of bladder cancer.
but a subset of adults have the Philadelphia
chromosome on cytogenetics. These patients Answer E is incorrect. Urine culture may help
are able to be treated with imatinib mesylate to establish infection as the cause of hematu-
(Gleevec), which is effective in decreasing the ria; however, the lack of pyuria, fever, and pain
burden of ALL. makes a urinary tract infection an unlikely
cause of gross hematuria.
Answer C is incorrect. In children, ALL is
treated with combination chemotherapy, in-
40. The correct answer is E. This patient is pre-
cluding daunorubicin, vincristine, prednisone,
senting with POEMS syndrome, a constella-
and L-asparaginase, and survival rates exceed
tion of symptoms associated with osteosclerotic
75%. However, adults who have ALL have a
multiple myeloma. Symptoms include Poly-
lower survival rate of 35%40%.
neuropathy, Organomegaly, Endocrinopathy,
Answer D is incorrect. The central nervous Multiple myeloma, and Skin changes. Poly-
system is actually a sanctuary site for ALL, neuropathies are generally progressive senso-
and any patient with this disease should have rimotor decits. Hepatosplenomegaly is gener-
a lumbar puncture to rule out metastasis to the ally seen, in contrast to other myeloma
central nervous system. patients. Endocrine manifestations include
HIGH-YIELD SYSTEMS
Chapter 7: Hematology/Oncology Answers 201

amenorrhea, diabetes mellitus, hypothyroid- death in men. However, most men diagnosed
ism, and adrenal insufciency. Skin changes with prostate cancer do not die from the disease
are diverse, but may include hyperpigmenta- and in the early stages most men have no symp-
tion, hypertrichosis, thickening, and digital toms. Later signs and symptoms can include
clubbing. Although this patients symptoms nocturia, dysuria, hesitancy, sexual dysfunction,
seem unrelated, her peripheral blood smear or bone pain. Once diagnosed, more tests and
demonstrates a large number of plasma cells, analyses are performed to determine the staging
as are seen in multiple myeloma. In addition, and grading of the disease (using the Gleason
her laboratory work reveals anemia, hyperuri- system, which assigns a score of 15 for the two
cemia, and hypercalcemia, all consistent with most common patterns of tumor found in the
a diagnosis of multiple myeloma. This diagno- biopsy). Treatment can include watchful wait-
sis can be conrmed by the presence of an M ing, surgery, radiation therapy, high-intensity fo-
spike on protein electrophoresis of urine and cused ultrasound, chemotherapy, cryosurgery,

Hematology/Oncology
serum. hormonal therapy, or some combination of
these. Ultimately the plan is based on stage of
Answer A is incorrect. The patient has symp-
the disease, the Gleason score, the PSA level,
toms of hypothyroidism, and her elevated thy-
the mans age, his general health, and his com-
roid-stimulating hormone level supports this di-
fort about potential treatments.
agnosis. However, her other symptoms prompt
a multisystem evaluation. Answer A is incorrect. Although CT scan and
bone scan are important components in evalua-
Answer B is incorrect. The dexamethasone
tion of the disease, these tests are ordered after a
suppression test is used to assess the presence
prostate biopsy conrms that cancer is present.
of Cushings syndrome. Although this patient
has some of the manifestations of Cushings Answer C is incorrect. Radical prostatectomy
syndrome, including weight gain and hypergly- is one treatment for prostate cancer. This
cemia, her other symptoms and abnormal pe- would be one of the options to consider if a bi-
ripheral blood smear are indicative of another opsy is positive.
process.
Answer D is incorrect. The patient displays
Answer C is incorrect. A hemoglobin A1c level no signs and symptoms of prostatitis, which
would allow the clinician to assess how long presents with signs of infection and pain dur-
the patient has been hyperglycemic. It would ing DRE. Getting a recheck of the PSA level
not, however, aid in the diagnosis of multiple in 3 months would be inappropriate with this
myeloma. patient, who is highly at risk of prostate cancer.
Answer D is incorrect. Although the patients Answer E is incorrect. An urinalysis may help
clubbed digits might promote a pulmonary in distinguishing between urinary tract infec-
evaluation, pulse oximetry would do nothing tion or prostatitis, which may increase PSA lev-
to conrm a diagnosis of multiple myeloma. els, and prostate cancer. This patient, however,
has both a positive DRE and increased PSA,
Answer F is incorrect. A differential of the pa-
and no signs of infection. Urinalysis, therefore,
tients WBCs might aid in the diagnosis of a
is not needed to rule out infection.
malignancy. It would not conrm the diagno-
sis, however.
42. The correct answer is E. Patients with X-linked
SCID and no matched related donor are very
41. The correct answer is B. Patients with a recent
good candidates for stem cell gene therapy.
elevation of prostate-specic antigen (PSA) lev-
The defect in the X-linked interleukin-2 recep-
els and a positive digital rectal examination
tor chain gene is known, making this disease a
(DRE) are at high risk for prostate cancer. The
prototype for gene treatment. The rst gene
next appropriate step is a biopsy to assess
therapy in patients with SCID was performed
whether cancer is present. Prostate cancer is
in 1991, and involved transduction with a ret-
the most common cancer in men in the U.S.
and the second largest cause of cancer-related
HIGH-YIELD SYSTEMS
202 Section I: Organ Systems Answers

roviral vector with clinical improvements doc- Answer B is incorrect. Patients with autoim-
umented in all patients. Trials have continued mune hemolytic anemia would have sphero-
and several patients have been successfully cytes on a peripheral blood smear and a posi-
treated in this fashion. A signicant number, tive Coombs test. These patients would benet
however, have developed a rapidly fatal leuke- from a course of corticosteroids, but this treat-
mia, a transformation thought to result from ment has no role in management of sickle cell
the gene insertion into patient cells with onco- disease.
genic action.
Answer C is incorrect. Allogeneic hematopoi-
Answer A is incorrect. GVHD is not a risk in etic stem cell transplant is an emerging therapy
gene therapy patients, as they are not undergo- for patients with severe refractory sickle cell
ing bone marrow transplant. disease.
Answer B is incorrect. Both gene therapy and Answer E is incorrect. Serial transfusion is a
Hematology/Oncology

bone marrow transplant carry a 30%40% risk of treatment for the thalassemias, a family of ge-
failure. Gene therapy is much less studied, but netic disorders of the blood found in people of
has had similar success rates to bone marrow Mediterranean descent. Thalassemia usually
transplant in the small population in question. presents with anemia and small, pale RBCs on
a peripheral smear.
Answer C is incorrect. No difference in sus-
ceptibility to illness has been demonstrated
44. The correct answer is B. Although APL is gen-
with the use of gene therapy in treating SCID.
erally responsive to the chemotherapeutics
Answer D is incorrect. Patients treated with used to treat acute myelogenous leukemia
gene therapy have been shown to respond well (AML), cytarabine and daunorubicin, approxi-
to clinical vaccinations, as have those treated mately 10% of APL patients treated on this reg-
with bone marrow transplant. imen die from disseminated intravascular coag-
ulation. This phenomenon occurs when dying
43. The correct answer is D. Sickle cell anemia is leukemic promyelocytes release their granule
a recessive genetic disease that often presents components. Modern protocols use tretinoin,
with painful microocclusive crises due to sick- an oral drug that promotes maturation of leu-
ling of RBCs in capillaries. Chronic hemolysis kemic cells bearing the t(15;17) abnormality,
is evident with jaundice and elevated unconju- the best known genetic marker for APL. Treti-
gated bilirubin. Blood smear shows sickle- noin is often used in combination with anthra-
shaped RBCs and nuclear fragments called cycline chemotherapy, with a period during
Howell-Jolly bodies. Acute treatment is symp- which tretinoin is used alone.
tomatic pain control, increasing oxygen ten-
Answer A is incorrect. Few data are available on
sion to reduce future sickling (oxygen), and re-
the effects of tretinoin on long-term fertility; how-
ducing the occlusive crisis through hydration.
ever, as it is a derivative of retinoic acid, it is a ma-
Long-term treatment includes hydroxyurea.
jor teratogen and could be devastating to a fetus
Hydroxyurea stimulates the production of fetal
during maternal treatment. Women of childbear-
hemoglobin, which will not sickle with low ox-
ing age being treated with tretinoin are strongly
ygen tension.
urged to avoid pregnancy for this reason.
Answer A is incorrect. Glucose-6-phosphate
Answer C is incorrect. No physician should
dehydrogenase (G6PD) deciency is a genetic
recommend homeopathic agents as rst-line
disorder common in individuals of African and
therapy for a potentially treatable cancer. Tretin-
Mediterranean descent. Hemolysis is induced
oin has been tested in multiple clinical trials for
by exposure to certain drugs or foods, includ-
safety and efcacy.
ing sulfa compounds and fava beans, and by
infections. These substances have no effect on Answer D is incorrect. APL is a subtype of
individuals with sickle cell anemia. In G6PD AML, and is generally responsive to the che-
deciency, a peripheral smear will show bite
cells and Heinz bodies (oxidized hemoglobin).
HIGH-YIELD SYSTEMS
Chapter 7: Hematology/Oncology Answers 203

motherapeutics used to treat AML (cytarabine Answer E is incorrect. If the patient had no
and daunorubicin). Older protocols did in- family history of colon cancer, 50 years would
volve the use of these medications; however, be the recommended age for his rst colonos-
the emergence of tretinoin as a treatment op- copy.
tion has allowed a decrease in the risk of che-
motherapy-associated disseminated intravascu- 46. The correct answer is C. This child has
lar coagulation. Wilms tumor. Wilms tumor usually presents
between 2 and 4 years of age with a painless
Answer E is incorrect. In fact, clinical trials
ank mass that does not cross the midline.
have shown that tretinoin is not sufcient as a
Some children, such as this one, also present
single agent to treat APL. For this reason cur-
with hypertension and microscopic hematuria.
rent protocols involve the use of tretinoin in
Others can present with fever, weight loss,
combination with intravenous anthracycline
polyuria, dysuria, bone pain, nausea, and vom-

Hematology/Oncology
chemotherapy (daunorubicin or doxorubicin).
iting. The diagnosis is made by abdominal CT.
Although the understanding of the sensitivity
Stage 1 is treated with nephrectomy and che-
of cells with t(15;17) to tretinoin has allowed
motherapy. If metastasis is present, radiation
major treatment advances and increased the
therapy is added. This child has no evidence of
therapeutic:toxicity ratio in APL, tretinoin has
metastasis.
not proven to produce durable remission when
used alone. Answer A is incorrect. Chemotherapy without
excision is palliative therapy for advanced, bi-
45. The correct answer is B. The current recom- lateral, unresectable Wilms tumor.
mendations are for patients to receive their rst
Answer B is incorrect. Excising the tumor is
colonoscopy at age 50 years if there is no family
appropriate therapy for localized neuroblas-
history of colorectal cancer. If there is a family
toma. Neuroblastoma is commonly confused
history of cancer in a rst-degree relative or mul-
with Wilms tumor on physical examination
tiple second-degree relatives, colonoscopy is rec-
because it can present with a painless abdomi-
ommended at 40 years of age, or 10 years before
nal mass and hypertension. However, neuro-
the age when the youngest rst-degree relative
blastoma causes elevated urine catecholamines
was diagnosed with colon cancer (whichever is
and does not cause hematuria. Most impor-
earlier). Repeat colonoscopy should be per-
tant, on imaging neuroblastoma does not origi-
formed every 10 years. Patient nervousness
nate from the kidney.
should be dealt with by educational counseling
by the doctor and does not change the recom- Answer D is incorrect. Chemotherapy must
mended screening age. also be used in treating Wilms tumor.
Answer A is incorrect. Although the patient Answer E is incorrect. Radiation is used as ad-
has a family history of colorectal cancer, it juvant therapy for treatment of more advanced
would not be recommended that he have his Wilms tumor but has no role in this patient
rst colonoscopy until 10 years before his fa- without evidence of metastasis.
ther was diagnosed, or 35 years of age.
47. The correct answer is C. This patient has
Answer C is incorrect. If his father had been Waldenstrms macroglobulinemia, a low-
50 years old or older at the time of his col- grade malignant lymphoma of plasmacytoid
orectal cancer diagnosis, it would be recom- lymphocytes that secrete excessive amounts of
mended that the patient in question have his IgM. Most patients with this disease present
rst colonoscopy at 40 years. with symptoms of hyperviscosity (Raynauds
Answer D is incorrect. The age of the patient phenomenon and visual disturbances), and
relative at the time of his diagnosis was 45 protein-protein interaction (platelet dysfunc-
years. It is recommended that patients with a tion). Diagnosis requires the demonstration of
family history of colorectal cancer have their an IgM spike on plasma electrophoresis, as
rst colonoscopy 10 years before the age when well as inltration of bone marrow with plas-
the family member was diagnosed. macytoid lymphocytes. Plasmapheresis is used
HIGH-YIELD SYSTEMS
204 Section I: Organ Systems Answers

to control hyperviscosity initially because re- tion. Mutations in the ret proto-oncogene are
sponse to chemotherapy can take several responsible for approximately >90% of cases of
weeks. Treatment regimens involve the same MEN type IIA. Papillary and anaplastic carcino-
chemotherapeutics as those used for multiple mas have not been found to have a genetic risk
myeloma, including alkylating agents and factor to date.
prednisone. Overall mean survival is 5 years.
Answer A is incorrect. Although it is difcult
Answer A is incorrect. Beginning a course of to separate the effects of alcohol from those of
prednisone without a nal diagnosis could be tobacco as risk factors in head and neck can-
deadly to this patient. Beginning low-dose ste- cer, neither has been directly linked to thyroid
roids may initially cause her disease to respond, cancer. Alcohol is a risk factor for upper aerodi-
but treatment outside a chemotherapeutic regi- gestive tract cancers, with greatest risk at inges-
men puts her at risk for relapse with refractory, tion rates greater than 50 g/d.
Hematology/Oncology

rapidly fatal disease.


Answer C is incorrect. Iodine exposure prior
Answer B is incorrect. Domestic violence to radiation exposure can be preventative for
is an important issue of which to be mindful thyroid cancer.
when treating any patient, specically one
Answer D is incorrect. Radiation to the neck
with repeated, vague complaints and suspi-
for enlarged tonsils is a risk factor for all types
cious lesions. However, this patients symptoms
of thyroid cancer. Between 1920 and 1960 ra-
indicate a disease that requires diagnosis. Her
diation was used to treat several benign diseases
safety should be assessed by her physician, but
including enlarged tonsils, enlarged lymph
domestic violence is unlikely to account for
nodes in the neck, enlarged thymus, acne, and
this constellation of symptoms.
scalp ringworm. However, this is not specic to
Answer D is incorrect. The patient has symp- medullary carcinoma of the thyroid.
toms that require an active work-up and diag-
Answer E is incorrect. Thus far, there has
nosis. They are unlikely to resolve without
been a negative correlation between smoking
treatment, and waiting to diagnose and treat
and thyroid cancer. There has been specula-
her disease will increase the likelihood of a fa-
tion as to whether or not this may be secondary
tal outcome.
to the higher incidence of thyroid carcinoma
Answer E is incorrect. Although the possibility among female patients.
of autoimmune disease exists, it would be rare
Answer F is incorrect. Chronic viral infections
following a complete and extensive work-up
such as HSV, HIV, Epstein-Barr, and HPV are
with no abnormalities. Generally, resending
risk factors for head and neck squamous cell
tests in anticipation of laboratory error is futile
carcinoma. This is likely secondary to suppres-
without an indication of abnormality.
sion of the tumor suppressor gene.
48. The correct answer is B. Medullary carcinoma
49. The correct answer is E. This patient presents
accounts for about 5% of all thyroid cancers
with a thrombocytopenic picture and with leu-
and is familial in up to 25% of cases. MEN
kocytosis. Her positive myeloperoxidase test is
consists of three syndromes featuring tumors of
diagnostic for AML. Complications of this dis-
endocrine glands, each with its own character-
ease include leukostasis leading to hyperviscos-
istic pattern. Specically, medullary thyroid
ity retinopathy, transient ischemic attacks/cere-
cancer is associated with MEN type IIA, as
brovascular accidents, tumor lysis syndrome
well as pheochromocytoma and parathyroid
(while undergoing treatment), gingival hyper-
hyperplasia/tumor causing hyperparathyroid-
plasia, central nervous system involvement, re-
ism. In MEN type IIA and familial medullary
current infection, and disseminated intravascu-
thyroid carcinoma, most patients present in the
lar coagulation (especially with M3 subtype).
third decade of life, but screening of family
members is important at an early age so pa- Answer A is incorrect. The incidence of AML
tients can inform their children of the condi- increases with age. Persons with AML who are
>60 years old have a poor prognosis.
HIGH-YIELD SYSTEMS
Chapter 7: Hematology/Oncology Answers 205

Answer B is incorrect. Environmental risk Answer E is incorrect. For solitary pulmonary


factors for developing leukemia include radia- nodules with intermediate probability of ma-
tion, chemotherapy (especially with alkylating lignancy, video-assisted thoracoscopic surgery
agents and topoisomerase inhibitors), benzene offers a more aggressive approach to diagnosis.
exposure, and smoking. In this case, a bronchoscopy has already been
performed and indicated malignancy.
Answer C is incorrect. Cytogenetics and sub-
type are extremely important in determining
51. The correct answer is A. This patient has he-
prognosis and effectiveness of treatment in
mophilia. This diagnosis is based on a history of
AML. Poor prognostic factors include mono-
mucosal bleeding and hemarthrosis along with
somies of chromosomes 5 or 7, deletion of 5q,
a family history of a bleeding disorder that is
and complex karyotype.
consistent with an X-linked recessive inheri-
Answer D is incorrect. Retinoic acid will often tance pattern (the patients mother is a carrier of

Hematology/Oncology
help induce remission in the promyelocytic the disease). Patients with hemophilia do not
form of AML (M3 subtype). present with ecchymoses or petechiae as may be
seen in other bleeding disorders such as vWD.
50. The correct answer is A. This patient is symp- Hemophilia presents with an increased acti-
tomatic, a smoker, and has a right lobe mass vated partial thromboplastin time because of de-
and mediastinal lymph node involvement that ciencies in components of the intrinsic path-
require immediate evaluation. Nodes <2 cm in way. The prothrombin time is normal in these
diameter are typically benign nodes, whereas patients because their extrinsic coagulation
nodes >2 cm are more likely to be malignant. pathway remains intact. Finally, bleeding time
The status of his nodes would provide critical is normal in these patients because neither their
information necessary for this patients man- platelet concentration nor function is altered.
agement. Therefore, the next appropriate step
Answer B is incorrect. Patients with hemo-
would be to evaluate the nodes by mediastinos-
philia do not typically present with low plate-
copy. If the nodes are benign, the patient
let concentrations. One would expect that a
would be a good candidate for surgical resec-
patient with a decreased platelet concentration
tion. However, if the nodes are malignant,
may have prolonged bleeding time and may
other modalities of treatment would be indi-
have spontaneous bleeding. Low platelet con-
cated.
centration would not explain a prolonged ac-
Answer B is incorrect. Pulmonary function tivated partial thromboplastin time, as seen in
tests are used to evaluate the risk of periopera- this patient.
tive complications following lung resection.
Answer C is incorrect. Leukopenia, or a de-
Patients considered for lung surgery undergo
creased WBC count, is not a nding associated
spirometry in order to provide important infor-
with hemophilia. This may be seen with im-
mation regarding the management of the pa-
munodeciencies.
tient. Preoperative testing would not be a pri-
ority until a staging procedure has established Answer D is incorrect. Factor V Leiden is a dis-
the patients candidacy for surgery. order in which coagulation factor V is resistant
to degradation by activated protein C. Increased
Answer C is incorrect. Thoracentesis is used
factor V concentrations are not associated with
to evaluate pleural effusions. This procedure
hemophilia, but rather with an increased ten-
would not be used in this case.
dency to clot, leading to strokes, pulmonary em-
Answer D is incorrect. Ventilation-perfusion bolism, and frequent miscarriages.
(V/Q) lung scans are used to detect pulmonary
Answer E is incorrect. Hemophilia is not as-
emboli, evaluate lung function for advanced
sociated with increased hemoglobin. The de-
cases of chronic obstructive pulmonary dis-
ciency that leads to hemophilia involves co-
ease, and detect the presence of shunts in pul-
agulation factors (either VIII or IX).
monary blood vessels. V/Q scans would not be
used to evaluate mediastinal lymph nodes.
HIGH-YIELD SYSTEMS
206 Section I: Organ Systems Answers

52. The correct answer is C. HIT is a well-estab- 53. The correct answer is E. vWD is a bleeding
lished adverse effect of treatment and may oc- disorder in which there is a deciency in vWF.
cur in as many as 20% of patients receiving vWF stabilizes coagulation factor VIII and al-
heparin therapy. Thrombocytopenia most com- lows for normal platelet function to occur. A
monly occurs after 410 days of treatment. The deciency in vWF leads to increased degrada-
more severe form of HIT, type II HIT, is tion of coagulation factor VIII and increased
caused by antibodies against the heparin-plate- bleeding time. Thrombocytopenia is not usu-
let factor 4 complex. Patients on chronic warfa- ally seen with vWD because the platelet dys-
rin therapy (e.g., those with a history of cardiac function is usually qualitative. Deciency of
disease and multiple strokes) are transitioned to coagulation factor VIII affects the intrinsic co-
the shorter-acting heparin prior to major sur- agulation pathway and therefore leads to a pro-
gery, thus explaining the time course of this pa- longed aPTT. vWD has no effect on the extrin-
tients reaction. sic coagulation pathway and therefore a normal
Hematology/Oncology

PT is expected to be seen.
Answer A is incorrect. Aspirin is a nonsteroi-
dal anti-inammatory drug that also inhibits Answer A is incorrect. This set of values is
platelet activation, although it does not cause most consistent with hemophilia B. Unlike
thrombocytopenia. Aspirin may cause bleed- hemophilia A, hemophilia B results in a de-
ing, gastrointestinal ulcer, or dyspepsia. Aspirin ciency of factor IX. This results in a prolonged
is commonly used as an antiplatelet agent in aPTT because factor IX is involved in the in-
patients at risk for stroke or myocardial infarc- trinsic pathway. Bleeding time is normal be-
tion and for pain and inammatory relief in cause it is factor VIII, not factor IX, that is in-
arthritis. volved with platelet activation and aggregation.
Answer B is incorrect. Although clopidogrel Answer B is incorrect. This set of values is
does interfere with platelet activation, it does most consistent with disseminated intravascu-
not cause thrombocytopenia. The main ad- lar coagulation. Coagulation factors (includ-
verse effects of clopidogrel are mild bleeding, ing platelets) are consumed, leading to the lab
rash, and dyspepsia. Clopidogrel is commonly prole seen here. Deciency of vWF leads to
used as an antiplatelet agent in patients at risk increased factor VIII degradation and has no
for stroke or myocardial infarction. effect on factor IX. In addition, vWD is associ-
ated with a prolonged bleeding time and pro-
Answer D is incorrect. Streptokinase is a
longed aPTT.
thrombolytic agent that is not associated with
thrombocytopenia. The main adverse effects of Answer C is incorrect. This set of values is
streptokinase are hypotension, shivering, and most consistent with hemophilia A. vWD is as-
occasional fever. There is no apparent indica- sociated with a prolonged bleeding time in the
tion for streptokinase in this patient, although context of a normal platelet count along with
thrombolytic agents may be used in the acute factor VIII deciency and prolonged aPTT.
setting for the treatment of acute stroke, myo- Hemophilia A is not associated with prolonged
cardial infarction, pulmonary embolism, or bleeding time.
deep vein thrombosis in selected patients.
Answer D is incorrect. This set of values is
Answer E is incorrect. Warfarin is a common most consistent with thrombocytopenia. A de-
anticoagulant used, for example, in patients ciency in vWF leads to increased degradation
with deep venous thromboses, history of atrial of coagulation factor VIII with an increased
brillation, or articial heart valves. It does not aPTT. Although bleeding time is increased
cause thrombocytopenia. Its major adverse he- in vWD due to decreased platelet function,
matologic effect is hemorrhage; other major thrombocytopenia is not necessarily a nding
adverse effects include hypersensitivity reac- of vWD.
tion and skin necrosis.
HIGH-YIELD SYSTEMS
Chapter 7: Hematology/Oncology Answers 207

54. The correct answer is B. Li-Fraumeni syn- cytopenia and microangiopathic hemolysis;
drome is a syndrome characterized by a muta- peripheral smear shows schistocytes.
tion in one copy of the p53 tumor suppressor
Answer A is incorrect. Disseminated intravas-
gene. It is inherited in an autosomal dominant
cular coagulation can also present with throm-
fashion, and predisposes patients to sarcomas,
bocytopenia and hemolysis due to massive
central nervous system tumors, and carcino-
thrombosis, and peripheral smears may also
mas. Patients are at a 100-fold risk of acquiring
show schistocytes. Coagulation studies, how-
treatment-related AML following treatment
ever, should show an elevated prothrombin
with cytotoxic drugs. Treatment-related AML
time, and the brinogen level should be low.
is universally refractory to known therapies.
Answer B is incorrect. HELLP syndrome is a
Answer A is incorrect. Ataxia telangiectasia is
constellation of ndings, including Hemolysis,
an autosomal recessive disorder that is charac-
Elevated Liver enzymes, and Low Platelets. It

Hematology/Oncology
terized by progressive ataxia, telangiectasias of
is a variant of preeclampsia that occurs during
the conjunctiva and ears, and predisposition to
pregnancy.
malignancy.
Answer C is incorrect. Henoch-Schnlein pur-
Answer C is incorrect. Von Hippel-Lindau
pura is the most common systemic vasculitis
syndrome is an autosomal dominant disease
in children and is caused by immune complex
characterized by hemangioblastomas of the
deposition in vessel walls. The clinical presen-
retina and central nervous system. Bilateral
tation includes palpable purpura beginning on
renal cysts are also common. It is caused by a
the extensor surfaces and buttocks, polyarthral-
mutation in the tumor suppressor gene VHL,
gia of the lower extremity joints, colicky abdomi-
located on chromosome 3p25, the same gene
nal pain, and nephritis. Platelet count is normal,
that is mutated in sporadic renal cell carci-
whereas serum IgA levels are usually increased.
noma. Forty to seventy percent of patients de-
velop renal cell carcinoma at some point. Answer D is incorrect. ITP also presents with
a petechial rash and thrombocytopenia. It is
Answer D is incorrect. Wiskott-Aldrich syn-
more common in children than thrombotic
drome is an X-linked disorder of T cells that
thrombocytopenic purpura, but it does not
presents as immune deciency. Patients have an
present with fever or splenomegaly.
increased risk of leukemia and/or lymphoma.
Answer E is incorrect. Xeroderma pigmentosum 56. The correct answer is C. This patient has dia-
is an autosomal recessive disorder that results betes with proteinuria and likely has nephrotic
from a defect in DNA repair mechanisms. Pa- syndrome. Among the many sequelae of neph-
tients are at very high risk for basal and squamous rotic syndrome is hypercoagulability secondary
cell carcinomas, as well as melanoma. to the loss of antithrombotic proteins in the
urine (proteins C and S and antithrombin).
55. The correct answer is E. Thrombotic throm- These patients are at increased risk for arterial
bocytopenic purpura is a disorder of increased and venous thrombotic complications, includ-
platelet aggregation leading to mucocutaneous ing myocardial infarction, pulmonary embo-
bleeding (petechiae or purpura) and hemoly- lism, deep vein thrombosis, and, as is the case
sis. Most cases are idiopathic, but the condition with this patient, unilateral renal vein thrombo-
is associated with certain medications such as sis. Classically, patients with renal vein throm-
cyclosporine and clopidogrel, oral contracep- bosis present with ank pain (secondary to dis-
tive use, HIV infection, and autoimmune dis- tention of the renal capsule), hematuria, and
ease. The classic pentad is thrombocytopenia, evidence of renal impairment. Timely diagnosis
microangiopathic hemolytic anemia, mental via renal ultrasound, CT, or direct venography
status changes, fever, and renal failure, al- as well as treatment with heparin are crucial, as
though the last three may not be present. Diag- the disruption of renal blood ow can lead to
nosis is conrmed with unexplained thrombo- permanent damage.
HIGH-YIELD SYSTEMS
208 Section I: Organ Systems Answers

Answer A is incorrect. Analgesics and angio- agnosis of the lesion should be obtained prior
tensin-converting enzyme inhibitors would do to the initiation of therapy.
little to address the underlying pathology in
Answer D is incorrect. Because this lesion
this patient. Moreover, angiotensin-converting
is concerning for malignant transformation,
enzyme inhibition may worsen this patients
symptomatic treatment alone is not appropri-
renal failure by decreasing the contralateral
ate.
kidneys hyperltration.
Answer E is incorrect. While dysplastic nevi
Answer B is incorrect. These modalities are
may be irregular in shape and color, the en-
important tools for the diagnosis and treatment
larging size and patients complaint of itching
of nephrotic syndrome. High-dose corticoster-
are concerning for malignancy and should be
oids are used to treat minimal change disease,
further evaluated immediately.
among other causes of nephrotic syndrome,
Hematology/Oncology

but they will do little to address this patients


58. The correct answer is B. This patient is suffer-
acute complication.
ing from acute chest syndrome as a result of
Answer D is incorrect. Spiral CT is useful in sickle cell anemia. This is caused by sickling of
diagnosing renal calculi, and administration of his RBCs as a result of a substitution of valine
a thiazide diuretic is helpful in treating chronic for glutamine in the -globin chain of hemo-
stone formation by decreasing hypercalciu- globin. Sickling is prompted by hypoxia, acido-
ria. This patient does not have ndings that sis, dehydration, or variations in body tempera-
are consistent with nephrolithiasis; thus, these ture. Acute chest syndrome is a vasoocclusive
treatments would not be helpful. crisis within the lungs associated with infection
of pulmonary infarction. Affected patients pre-
Answer E is incorrect. Urine culture and an-
sent with hypoxia, fever, dyspnea, chest pain,
tibiotics are appropriate diagnostic and treat-
and progressive pulmonary inltrates seen on
ment modalities to address acute pyelonephri-
x-ray studies. Such episodes can be prevented
tis. This patient is afebrile, lacks costovertebral
by hydroxyurea, which decreases the incidence
angle tenderness, and does not have dysuria or
of sickle cell crises. Hydroxyurea acts as an in-
pyuria.
hibitor of deoxynucleotide synthesis, which re-
duces the production of RBCs containing a
57. The correct answer is A. Any description of a
high level of sickle hemoglobin and favors the
changing mole is concerning for occurrence of
production of RBCs containing a high fetal he-
melanoma. The ABCDE mnemonic is often
moglobin level.
used to remember the key changes that indi-
cate development of malignancy (Asymmetry, Answer A is incorrect. Diphenyhydramine
irregular Borders, Color changes or irregular- is used for allergic reactions. The mainstay of
ity, Diameter increasing, and Evolution or treatment for pain crisis is hydration and pain
changing of the mole). Several features of this control with nonsteroidal anti-inammatory
mole are particularly concerning, including drugs and narcotics. Acute chest syndrome in
the size (>6 mm) and presence of itching. The particular is treated with oxygen, analgesia, and
treatment of choice is excisional biopsy, with antibiotics. Sometimes exchange transfusions
margins determined by apparent depth of the are also necessary, especially if the patient is
lesion. suffering from aplastic crisis, severe hypoxia,
and cerebral thrombosis (patients with stroke).
Answer B is incorrect. Identication of the
However, this is not preventive treatment.
sentinel lymph node with excisional biopsy is
indicated in patients with clinical evidence of Answer C is incorrect. Pneumococcal vaccine
lymph node involvement or with deep lesions. reduces the rates of infection by encapsulated
organisms. However, painful sickle crises con-
Answer C is incorrect. Radiation therapy is
tinue to occur in these patients.
not an accepted treatment for malignant mela-
noma. Furthermore, biopsy and pathologic di-
HIGH-YIELD SYSTEMS
Chapter 7: Hematology/Oncology Answers 209

Answer D is incorrect. Prophylactic penicillin Streptococcus species. Neither of these patients


is recommended, since patients with sickle cell has evidence of mastitis.
anemia tend to get more infections by encap-
Answer B is incorrect. Ultrasound is useful in
sulated organisms such as Streptococcus pneu-
discerning solid from cystic lesions, especially
moniae and Haemophilus inuenzae. For the
when a suspicious mass is not clinically malig-
same reason, patients with sickle cell disease
nant on examination or mammography. Ultra-
are also at greater risk for osteomyelitis with
sound is not useful for diagnosing an intraduc-
Salmonella species. However, penicillin does
tal papilloma.
not reduce the incidence of painful crises but
rather reduces infection rates. Answer C is incorrect. While chemotherapy
is a useful modality in treating breast cancer, a
Answer E is incorrect. Autoinfarction of the
histologic diagnosis of breast cancer must rst
spleen can occur during early childhood sec-
be established in order to select the most ap-

Hematology/Oncology
ondary to microvascular obstruction by sickled
propriate treatment plan.
RBCs. As a consequence of infarction and -
brosis, the spleens immunologic capacity is re- Answer D is incorrect. CT of the chest and/
duced, and hence the patient is at greater risk or abdomen may be utilized in staging breast
for pneumonia, sepsis, and meningitis by en- cancer and planning a treatment regimen. CT,
capsulated organisms. Splenectomy does not however, serves no role in the initial diagnosis
reduce the incidence of vasoocclusive crises. of breast cancer.
Answer F is incorrect. Fine-needle aspiration
Questions 59 and 60 biopsy is useful in obtaining a histologic diag-
nosis for palpable breast lesions, but has signi-
59. The correct answer is E. Unilateral serous or cant false-negative rates for nonpalpable breast
serosanguineous nipple discharge from a single ndings. Therefore, stereotactic core needle
duct is most commonly caused by an intraduc- biopsy is a better choice for evaluating nonpal-
tal papilloma. Other less likely causes include pable breast ndings.
breast cancer or mammillary dysplasia with
Answer G is incorrect. All suspicious masses
ectasia. The diagnosis of intraductal papilloma
warrant an immediate work-up; breast cancer
is made upon excision of the affected duct,
must be diagnosed and treated. Adopting a
which is also a curative procedure and allows
wait-and-watch approach is medically negli-
for histologic examination of the lesion to rule
gent and is not appropriate in this situation.
out carcinoma.
Answer H is incorrect. Incision and drainage
60. The correct answer is M. This patients physi- would be the procedure of choice if the pa-
cal exam ndings (axillary lymphadenopathy) tient suffered from a breast abscess; neither of
and mammogram (microcalcications and ir- these patients has evidence of having a breast
regular density) are highly suspicious for ductal abscess.
carcinoma in situ. Before any therapeutic op-
Answer I is incorrect. While lumpectomy of
tions are discussed, a denitive tissue biopsy
the involved lesion along with nodal dissection
must be obtained. Stereotactic core needle bi-
is a useful modality in treating breast cancer, a
opsy is the preferred method in most centers,
histologic diagnosis of breast cancer must rst
especially for nonpalpable lesions with micro-
be established in order to select the most ap-
calcications on mammography.
propriate treatment and surgical plan.
Answer A is incorrect. A course of dicloxacil-
Answer J is incorrect. While modied radi-
lin would be the treatment of choice if the pa-
cal mastectomy is a useful modality in treating
tient suffered from mastitis, which commonly
breast cancer, a histologic diagnosis of breast
occurs in the rst weeks of breast feeding and
cancer must rst be established in order to se-
is usually caused by Staphylococcus aureus or
lect the most appropriate treatment and surgi-
cal plan.
HIGH-YIELD SYSTEMS
210 Section I: Organ Systems Answers

Answer K is incorrect. While radiation is a Answer L is incorrect. While simple mastec-


useful modality in treating breast cancer, a his- tomy is a useful modality in treating breast
tologic diagnosis of breast cancer must rst be cancer, a histologic diagnosis of breast cancer
established in order to select the most appro- must rst be established in order to select the
priate treatment plan. most appropriate treatment and surgical plan.
Hematology/Oncology
CHAPTER 8

Infectious Disease

211
HIGH-YIELD SYSTEMS
212 Section I: Organ Systems Questions

Q U E ST I O N S

1. An 18-month-old girl presents to the emer- have Pneumocystis jiroveci pneumonia and his
gency department with a fever. She was the current CD4+ cell count is <170/mm. The
product of a normal pregnancy. Her parents patient is intubated and intravenous uids are
state that the patient has had a cough and rhi- started. The patients brother arrives and states
norrhea for the past 24 hours. She has also has that the patient has expressed wishes to with-
a decreased appetite and has been fussy. She hold treatment at this stage of his disease. Ac-
had been afebrile until this morning, when she cording to the brother, the patient had created
appeared ushed and uncomfortable and had an advance directive during his last hospitaliza-
a temperature of 40C (104F). In the emer- tion. The patient has not taken his medication
gency department the patient begins to have regimen for the past month. Assuming the pa-
generalized tonic-clonic movements of her tient remains unconscious, which of the fol-
Infectious Disease

face, neck, and extremities for 1 minute. She is lowing would compel the physician to with-
conscious but drowsy immediately after the sei- hold medical treatment?
zure. Her temperature is 40.5C (104.9F), re-
(A) A signed advance directive stating that the
spiratory rate is 24/min, pulse is 120/min, and
patient wishes to refuse all treatment
blood pressure is 100/54 mm Hg. Her pupils
(B) A statement by the patients brother indi-
are round and equally reactive to light. Her
cating that the patient wishes to refuse
mucous membranes are moist, and her phar-
treatment
ynx is mildly erythematous. She is able to move
(C) A statement by the patients primary care
her extremities spontaneously and has normal
physician indicating that the patient wishes
tone throughout. Kernigs and Brudzinskis
to refuse treatment
signs are absent. The patient has no prior his-
(D) An acquaintance to whom the patient
tory of seizures. Which of the following is the
granted power of attorney for the duration
most appropriate next step in management?
of his previous hospital stay requests that
(A) Acetaminophen interventions be withheld
(B) Electroencephalogram (E) Initial paperwork for an advance directive
(C) Lumbar puncture in conjunction with the brothers state-
(D) Phenobarbital ment
(E) Valproic acid (F) The brothers statements alone are suf-
cient to withhold new interventions, but
2. A 36-year-old man comes to his primary care ventilation and uid treatment should con-
physician with a rapidly expanding rash under tinue
his right arm. The rash is round with central
clearing. Upon questioning the man admits 4. A girl is born via vertex vaginal delivery to a
that he has been feeling somewhat lethargic 32-year-old G3P2 woman at 36 weeks gesta-
and achy lately, but had attributed this to fa- tion. The mother received no prenatal care
tigue after a recent hike in the Shenandoah and reports an uncomplicated pregnancy. Four
Valley in Virginia. Which of the following is weeks after birth, the infant is brought to the
the most likely diagnosis? clinic because of a crusty exudate from both
eyes. Physical examination reveals bulging an-
(A) Infection with Staphylococcus aureus
terior and posterior fontanelles and a swollen
(B) Lyme disease
cranium. CT imaging shows multiple intracra-
(C) Meningitis
nial opacications. Lost to follow-up, the child
(D) Myocarditis
dies spontaneously at 6 weeks. A histologic
(E) Rocky Mountain spotted fever
sample of the childs myocardium on autopsy
3. A 33-year-old man who has AIDS is brought to is shown in the image. What treatment after
the emergency department after he collapses delivery would most likely have saved this
outside his apartment. The patient is found to childs life?
HIGH-YIELD SYSTEMS
Chapter 8: Infectious Disease Questions 213

lobe pneumonia. Laboratory testing shows a


blood urea nitrogen level of 35 mg/dL and cre-
atinine of 4.3 mg/dL. Her blood urea nitrogen
and creatinine levels the day before had been
23 mg/dL and 1.5 mg/dL, respectively. WBC
count is 25,000/mm and blood cultures grow
Pseudomonas aeruginosa. There is tenderness
upon palpation of the costovertebral angle re-
gion. Which of the following mechanisms
would most likely account for the worsening
kidney function?

Reproduced, with permission, from Brooks GF, Carroll (A) Contrast administration during the course
KC, Butel JS, Morse SA. Jawetz, Melnick, & Adelbergs of her treatment in the hospital

Infectious Disease
Medical Microbiology, 24th edition. New York: McGraw- (B) Diabetic kidney failure secondary to angio-
Hill, 2007: Figure 45-21. tensin II-induced vasoconstriction
(A) Acyclovir (C) Hematogenous spread of the pulmonary
(B) Ceftriaxone infection to the kidney leading to pyelone-
(C) Parenteral penicillin G phritis
(D) Pyrimethamine (D) Insufcient intravascular volume second-
(E) Supportive care alone ary to discontinuation of maintenance in-
travenous uids
5. An 18-year-old woman presents to her obstetri- (E) Respiratory acidosis secondary to pulmo-
cian/gynecologist with dysuria and frequency. nary infection
Urine culture demonstrates Escherichia coli,
and the patient is given a 3-day course of cipro- 7. A 2-week-old infant is brought to emergency
oxacin. Which of the following adverse department with fevers, increased irritability,
reaction(s) occurs most commonly with this anorexia, and decreased responsiveness. The
class of medication? infant did not improve after administration of
the appropriate dose of infant acetaminophen.
(A) Allergic reactions and rashes The infant is centrally pink, lethargic, and has
(B) Gastrointestinal upset with anorexia, nau- shallow respirations. The fontanelle is soft and
sea, and vomiting at, and heart sounds, breath sounds, and ab-
(C) Mild headache and dizziness dominal examination are all normal. Capillary
(D) QT prolongation of ECG rell is adequate. Neurologic examination
(E) Tendonitis and tendon rupture shows decreased tone and a weak, intermittent
cry. During the examination the mother men-
6. A 45-year-old white woman with type 2 diabe- tions that she delivered the baby at home with
tes mellitus originally admitted for a pulmo- a midwife and did not receive prenatal care.
nary embolism develops a cough and fever Lumbar puncture reveals a pleocytosis with a
while in the hospital. She was admitted 4 days neutrophil predominance, and a low glucose
ago and had started warfarin the day prior to and high protein content. Which of the follow-
the onset of her cough and fever. On day two ing is the most likely cause of the infants symp-
she was advanced to a normal diet and mainte- toms?
nance intravenous uids were discontinued.
The nurses say she has been eating and drink- (A) Group B Streptococcus
ing normally. On examination she is febrile, (B) Inuenza
her neck veins are at, her skin is warm to the (C) Overdose of acetaminophen
touch, and her blood pressure is 80/40 mm Hg. (D) Varicella-zoster virus
Urine output was 20 mL per hour for the last 3 (E) Viridans streptococci
hours. A CT is ordered and shows a right lower
HIGH-YIELD SYSTEMS
214 Section I: Organ Systems Questions

8. A 3-month-old girl presents to her general pe- 10. A 34-year-old African-American man with dia-
diatricians ofce with a rash. She is the prod- betes is brought to the emergency department
uct of a normal uncomplicated pregnancy and after collapsing on the bus on his way home
was born at 40 weeks gestation. Her mother from work. His blood sugar level was 20 mg/dL
reports that the girl was well until this morn- on admission. After resuscitative measures, his
ing, when she suddenly developed a fever to blood sugar is 90 mg/dL; he is awake, alert, ori-
41C (105.8F) with no other symptoms. In ented, and appears stable. His insulin dosage
the pediatricians ofce, the patient begins to had recently been adjusted, and physical ex-
seize and is rushed to a nearby hospital. She is amination reveals white patches in his mouth.
hospitalized and continues to have seizures Results of a contrast-enhanced CT study are
whenever her temperature spikes above 40C normal. Which of the following is the most ap-
(104F). After 3 days her fever breaks and she propriate next step to manage his condition
develops a maculopapular rash on her trunk and prevent morbidity?
that then spreads to her face and extremities.
Infectious Disease

(A) Administer amphotericin B


Which of the following is the most likely
(B) Administer glucose
pathogen?
(C) Administer mannitol
(A) Human herpesvirus-6 (D) Do nothing; this represents normal oral
(B) Measles virus ora
(C) Parvovirus B19 (E) Manage blood sugar levels and insulin
(D) Rubella virus medication
(E) Varicella-zoster virus
11. A concerned father takes his 6-year-old son to
9. A 4-month-old infant born at 32 weeks gesta- the pediatricians ofce after he notices a tar-
tion is taken to her pediatricians ofce for eval- get-shaped rash on his sons back several days
uation of a respiratory illness. The patients after returning home from a camping trip. The
mother says her daughter developed a low- boy also has had a low-grade fever (tempera-
grade fever (38.5C [101F]) with sneezing ture 38.3C [100.9F]) for the past 3 days, re-
and a runny nose 3 days prior to this visit, after current headaches, and generalized malaise.
her 5-year-old brother had a similar illness. While talking with the pediatrician, the father
Now her symptoms include wheezing, cough, recalls he noticed a black bump where his
and irritability. The infant is hemodynamically sons rash began, which he scraped off at the
stable but tachypneic with a respiratory rate of time. Suspicious of Lyme disease, the pediatri-
68/min. Head, ears, eyes, nose, and throat ex- cian prescribes a course of amoxicillin. The
amination is notable for clear rhinorrhea with boys father questions why the pediatrician did
nasal aring. There is no fullness or erythema not prescribe doxycycline, which he was pre-
of the tympanic membranes bilaterally. Lung scribed when he had Lyme disease 2 years ear-
auscultation demonstrates wheezing bilater- lier. What potential adverse effect of doxycy-
ally, with prolongation of the expiratory phase. cline is the pediatrician avoiding by prescribing
Which of the following is the best next step in amoxicillin for this patient?
management?
(A) Cartilage damage
(A) Antibiotic administration and release to (B) Ototoxicity
home (C) Red man syndrome
(B) Chest radiograph and WBC count with (D) Renal toxicity
differential (E) Tooth discoloration
(C) Hospitalization with supportive care
(D) Palivizumab administration and release to 12. An 8-year-old boy is brought to the emergency
home department with a fever, myalgias, and a rash
(E) Respiratory syncytial virus intravenous im- involving his limbs and trunk (see image). His
munoglobulin administration and release temperature is 38.9C (102F). His mother
to home
HIGH-YIELD SYSTEMS
Chapter 8: Infectious Disease Questions 215

states that the trunk was the last place to be in- (A) Coxsackie A
volved by the rash. The patient states that (B) Human herpesvirus-6
about a week ago he found a stray dog and re- (C) Paramyxovirus
turned it to its owner after he saved the dog (D) Parvovirus B19
from many vampire ticks. Which of the fol- (E) Varicella-zoster virus
lowing is the most likely diagnosis?
14. A 10-month-old girl is brought to her pediatri-
cians ofce because she has been febrile and
irritable. She is the product of a normal preg-
nancy and has been healthy since birth. Yester-
day, her parents noticed that she was fussy, less
active, and did not eat as much as usual. Over-
night she developed a temperature of 38.5C
(101.3F) and was awake crying for most of the

Infectious Disease
night. Her parents report that she has also been
tugging on her left ear. Her temperature is
39C (102.2F), respiratory rate is 24/min, and
pulse is 140/min. She is not toxic appearing
Reproduced, with permission, from Wolff K, Johnson RA, but continues to be irritable. Her pupils are
Surmond D. Fitzpatricks Color Atlas & Synopsis of Clini- equally reactive to light bilaterally. She has
cal Dermatology, 5th edition. New York: McGraw-Hill, moist mucous membranes, and her pharynx is
2005: Figure 24-4.
nonerythematous. The left tympanic mem-
(A) Ehrlichiosis brane is bulging, erythematous, and immobile
(B) Lyme disease on pneumatic otoscopy. The right tympanic
(C) Rocky Mountain spotted fever membrane is mobile and nonerythematous.
(D) Tick-borne encephalitis She does not have a rash or a history of ear in-
(E) Tularemia fections. Her immunizations are up to date.
What is the most appropriate next step in man-
13. A 3-year-old girl complaining of fever is agement?
brought to the emergency department by her (A) Observation
parents. The girl has had a tactile fever for the (B) Refer the patient to a surgeon for myringo-
past 24 hours and is now complaining of a tomy and placement of tympanostomy
painful mouth. She has no signicant past tubes
medical history, and her social, family, and (C) Refer the patient to the emergency depart-
travel histories are noncontributory. Her par- ment of a tertiary care facility
ents deny that she has been coughing or has (D) Start 90 mg/kg/day of amoxicillin and 6.4
had any upper respiratory infection symptoms. mg/kg/day of clavulanate for 10 days
Her heart rate is 89/min, blood pressure is (E) Start amoxicillin at 8090 mg/kg/day for
96/58 mm Hg, respiratory rate is 22/min, and 10 days
temperature is 38.2C (100.8F). On physical (F) Start erythromycin and sulsoxazole at 50
examination her oropharynx is inamed, with 150 mg/kg/day for 10 days
multiple vesicles scattered throughout her
tongue, buccal mucosa, posterior pharynx, pal-
ate, and lips. Additionally, she has maculopap-
ular and vesicular lesions on both of her hands,
buttocks, and groin. What viral agent is most
likely responsible for this clinical presentation?
HIGH-YIELD SYSTEMS
216 Section I: Organ Systems Questions

15. A 65-year-old man comes to the doctor for 17. A 23-year-old patient with a past medical his-
evaluation of a recent skin infection (as shown tory of type 1 diabetes and recent outpatient
in the image). He is otherwise healthy and has treatment for pelvic inammatory disease with
never had a similar episode. He has a noncon- oral antibiotics returns to the emergency de-
tributory social and family history. During the partment with a chief complaint of fevers and
interview he mentions he really enjoys working lower abdominal pain for 1 day. Her tempera-
out at the community gym each day. On physi- ture is 39.6C (103.37F), pulse is 125/min,
cal examination he has no symptoms other blood pressure is 88/48 mm Hg, and respira-
than a localized infection that is pruritic and tory rate is 18/min. She conrms she took her
mildly malodorous. A potassium hydroxide entire course of oral antibiotic therapy and has
preparation of scrapings from the lesion con- regularly been taking her insulin. On examina-
rms the diagnosis. Which of the following is tion her skin is warm to the touch and her ab-
the most likely diagnosis? domen is rigid and diffusely tender to light pal-
pation. Laboratory tests are ordered and show a
Infectious Disease

hemoglobin level of 13.4 g/dL, WBC count of


24,000/mm, platelet count of 247,000/mm,
and plasma glucose level of 224 mg/dL. A
-human chorionic gonadotropin test and uri-
nalysis are negative. What is the most likely
underlying cause of her symptoms?
(A) Acute cholecystitis
(B) Diabetic ketoacidosis
Reproduced, with permission, from Wolff K, Johnson RA, (C) Gonorrheal cervicitis after reinfection by
Surmond D. Fitzpatricks Color Atlas & Synopsis of Clini- an untreated sexual partner
cal Dermatology, 5th edition. New York: McGraw-Hill, (D) Ruptured ectopic pregnancy
2005: Figure 23-5. (E) Ruptured tubo-ovarian abscess
(A) Contact dermatitis
(B) Hand-foot-and-mouth disease 18. A 27-year-old woman presents to her primary
(C) Scabies care physician with a complaint of several days
(D) Stevens-Johnson syndrome of dysuria, vaginal pruritus, and increased vagi-
(E) Tinea pedis nal discharge. She has never experienced simi-
lar symptoms. A pregnancy test is negative. She
16. An 18-year-old college woman comes to the is currently sexually active with one male part-
clinic complaining of fever, cough, and a re- ner and uses barrier contraception. Gyneco-
cent rash on her face and chest with pruritus. logic examination reveals a thick white dis-
Physical examination reveals multiple papules charge and a vaginal pH of 4. A 10% potassium
and vesicles in varying stages of development. hydroxide preparation reveals branching hy-
She states that she lives in a dormitory and has phae with budding yeast. Which of the follow-
a roommate. X-ray of the chest shows an inl- ing oral medication is the most appropriate
trate. Which of the following is the most appro- treatment regimen?
priate treatment? (A) Azithromycin
(A) Acyclovir (B) Ciprooxacin
(B) Famciclovir (C) Clindamycin
(C) Foscarnet (D) Fluconazole
(D) Ganciclovir (E) Metronidazole
(E) Triuridine
19. A 4-year-old boy presents to his pediatricians
ofce. He has had a cough and clear nasal dis-
charge for the past 23 days, with fevers rang-
ing from 38C40C (100.4F104F) over
the past 48 hours. On examination he is febrile
HIGH-YIELD SYSTEMS
Chapter 8: Infectious Disease Questions 217

and appears tired but responsive. He has small chemotherapy is brought to the emergency de-
red spots with grey centers on his buccal mu- partment with a history of worsening head-
cosa, as well as an erythematous, blanching, aches for 5 weeks; blurred vision, double vision
maculopapular rash on his face that appeared and persistent hearing loss for 3 days; and mild
within the past hour. His examination is other- confusion on the morning of admission. His
wise unremarkable. The patient and his brother, who brought him in, said the patient
11-month-old sister have not received any im- also had difculty walking earlier that morn-
munizations since birth because, after his ing. On admission he has a fever of 38.3C
mother had an adverse reaction to the inu- (101F). The physician is worried about an in-
enza vaccine, the parents consider immuniza- tracranial infection. Which of the following is
tions to be more harmful than helpful. Which the most appropriate initial step in diagnosis?
of the following should be administered to the
(A) Administer tissue plasminogen activator
11-month-old sister as soon as possible?
(B) Central nervous system imaging

Infectious Disease
(A) Diphtheria-tetanus-pertussis vaccine (C) Urgent lumbar puncture and culture
(B) Haemophilus inuenzae type b vaccine (D) Urgent lumbar puncture and India ink
(C) Inuenza vaccine stain
(D) Measles-mumps-rubella vaccine (E) Urgent lumbar puncture and serology
(E) Varicella vaccine
22. A 36-year-old woman brings her 2-month-old
20. A 30-year-old man begins an antibiotic regi- child for a pediatric check-up. The baby has
men for treatment of pulmonary tuberculosis been developing normally. The woman has a
that includes isoniazid. One month after initia- 29-year-old brother with autism. She is very
tion of therapy, the man visits his primary care concerned that her baby will develop this con-
physician with complaints of fatigue, anorexia, dition as well. She has read many alarming re-
and nausea. Laboratory tests show: ports that vaccinations have been linked to
higher incidence of autism in the United
WBC count: 8000/mm
States and refuses to put her child at risk.
Hemoglobin: 15.2 g/dL
Which of the following is the most appropriate
Hematocrit: 49.7%
next step?
Aspartate aminotransferase: 240 U/L
Alanine aminotransferase: 285 U/L (A) Ask the mother to step out of the room to
Alkaline phosphatase: 405 U/L speak further about the issue and have the
nurse administer the vaccine to the child
Which of the following places this patient at
while no one else is in the room
increased risk for developing this complication
(B) Contact the babys father to ask him to
of isoniazid therapy?
convince his wife that immunizations are
(A) Active injection drug use necessary
(B) Age <35 years (C) Educate the mother about the benets of
(C) First-time treatment with isoniazid vaccines and the lack of evidence linking
(D) Male gender autism and vaccinations, but leave the de-
(E) White race cision to her
(D) Obtain a court order to administer the vac-
21. A 55-year-old man with Hodgkins lymphoma cine
receiving CHOP (cyclophosphamide, doxoru- (E) Report the mother for child endangerment
bicin, Oncovin [vincristine], and prednisone)
HIGH-YIELD SYSTEMS
218 Section I: Organ Systems Questions

23. A 15-year-old girl presents to her pediatrician (A) Aplastic anemia


with a complaint of dull upper abdominal pain (B) Endemic Burkitts lymphoma
and a sore throat, both lasting for about 1 week. (C) Orchitis
She has no major medical history and reports (D) Progressive multifocal leukoencephalopa-
that she is not sexually active. Her temperature thy
is 38.1C (100.6F), blood pressure is 118/80 (E) Subacute sclerosing panencephalitis
mm Hg, pulse is 65/min, and respiratory rate is
12/min. Her spleen is enlarged, and she has 25. A 23-year-old medical student rushes to his se-
signicant cervical and periauricular lymph- nior physician on duty to report a needlestick.
adenopathy. Which of the following is the most He reports that he was injecting anesthetic into
appropriate diagnostic test? the nger of an HIV-positive homeless intrave-
nous drug user in preparation to suture a lacer-
(A) CT of the abdomen
ation. In the process of doing so, he sustained a
(B) Exploratory laparotomy
deep, percutaneous needlestick injury with the
Infectious Disease

(C) Fine-needle aspiration of a lymph node


contaminated hollow-bore needle. The student
(D) Flow cytometry on peripheral lymphocytes
is anxious and has washed out the wound sev-
(E) Latex agglutination assay
eral times before coming to ask his senior phy-
sician what he should do next. Records from
24. A 5-year-old previously healthy child is taken
the HIV-positive patient indicate his last CD4+
from his day care to the pediatrician because
cell count was 169/mm with a viral load of
his mother is concerned that the child has
400,000/mL. What is the most appropriate next
been fussy for the past few days and now has a
step in management of this needlestick injury?
rash on his face and torso (see image). The
mother also says the boy told her his head hurt, (A) Begin single-drug antiretroviral therapy
and she thinks he may have a low-grade fever. for 4 weeks and draw a blood sample after
On examination the child is afebrile and not 6 months
ill-appearing. He has a rash extending over his (B) Begin three-drug antiretroviral therapy
entire body except for the palms and soles. The for 4 weeks and draw a blood sample after
infectious agent causing this disease can lead 6 months
to what complication in immunocompromised (C) Draw a blood sample for baseline HIV se-
hosts? rology and begin single-drug antiretroviral
therapy for 4 weeks
(D) Draw a blood sample for baseline HIV se-
rology and begin three-drug antiretroviral
therapy for 4 weeks
(E) Draw a blood sample for baseline HIV se-
rology and begin three-drug antiretroviral
therapy only if serology is positive

Reproduced, with permission, from Wolff K, Goldsmith


LA, Katz SI, Gilchrest BA, Paller AS, Leffell DJ. Fitzpat-
ricks Dermatology in General Medicine, 7th ed. New
York: McGraw-Hill, 2008: Figure 1925.
HIGH-YIELD SYSTEMS
Chapter 8: Infectious Disease Questions 219

26. A 33-year-old man with HIV infection and a 27. A 20-year-old woman presents to the emer-
CD4+ cell count of 54/mm presents to the gency department with complaints of 1 day of
emergency department with a nonproductive headache and fever with associated nausea,
cough, fever, and exertional dyspnea. X-ray of vomiting, and a diffuse red rash. She reports
the chest is shown in the image. Which of the she has been on spring break for the past week,
following is the most appropriate pharmaco- has been drinking heavily, and has been spend-
therapy? ing most of each day in the sun, but she has
used SPF 30 sunblock and does not remember
getting a sunburn. Her last menstrual period
began 3 days ago. She denies any recent sexual
activity. Examination is notable for a diffuse
macular erythematous rash resembling sun-
burn that covers the majority of her body, in-
cluding her palms and soles. Her neck is sup-

Infectious Disease
ple and she does not exhibit any photophobia
or signs of intoxication. Twenty minutes after
presentation she becomes lethargic, and her
temperature is 39.6C (103.3F), heart rate is
115/min, blood pressure is 84/48 mm Hg, and
respiratory rate is 18/min. Which of the follow-
ing is the most likely diagnosis?
(A) Alcohol poisoning
(B) Heatstroke
(C) Neisseria meningococcemia
(D) Rocky Mountain spotted fever
(E) Toxic shock syndrome
Reproduced, with permission, from Hall JB, Schmidt 28. A 22-year-old man with cystic brosis presents to
GA, Wood LDH. Principles of Critical Care, 3rd edition. the emergency department with the acute onset
New York: McGraw-Hill, 2005: Figure 48-1.
of fever, chills, and dyspnea. Blood cultures are
(A) Amphotericin B positive for Pseudomonas aeruginosa, and the
(B) Ceftriaxone patient is started on a multiple-antibiotic regi-
(C) Fluconazole men of a -lactam antibiotic and gentamicin.
(D) Metronidazole Ten days later the patients bacteremia has re-
(E) Nystatin swish and swallow solved but his creatinine level is 2.0 mg/dL. Uri-
(F) Pyrazinamide nalysis demonstrates muddy brown granular
(G) Trimethoprim-sulfamethoxazole casts, and the fractional excretion of sodium in
his urine is 2.5%. Which of the following is the
most likely mechanism of this patients renal dis-
ease?
(A) Decreased perfusion of the kidney second-
ary to volume depletion
(B) Deposition of antibody and/or antibody-
antigen complexes on the glomerular base-
ment membrane
(C) Direct toxic effect on renal tubule cells
(D) Mesangial deposition in glomerular capil-
lary walls
(E) Renal arterial and arteriolar vasoconstric-
tion
HIGH-YIELD SYSTEMS
220 Section I: Organ Systems Questions

29. A concerned mother brings her 7-day-old son each) on the left neck with two 3-cm lymph
to his pediatrician after he develops a rash. Ex- nodes in the anterior cervical chain. What or-
amination reveals a cluster of erythematous ganism is most likely responsible for this infec-
vesicles localized to the childs mouth. The tion?
childs vital signs are all within normal limits,
(A) Bartonella henselae
and aside from the rash and mild irritability he
(B) Brucella
appears healthy. A sample of tissue is taken
(C) Clostridium botulinum
from the base of a ruptured vesicle and sub-
(D) Francisella tularensis
jected to the Tzanck test (see image). The
(E) Staphylococcus aureus
child is treated with the most commonly pre-
scribed agent for this condition. Which of the 31. A 3-year-old boy is brought to his family pedia-
following is a serious adverse effect associated trician for evaluation of chronic low-grade fe-
with this drug? vers, decreased appetite, and nonproductive
Infectious Disease

cough. He has previously been healthy, with


no past similar episodes. The pediatrician
learns that the patient has been visiting his
grandparents in a retirement living facility on
weekends. The physician wants to rule out all
possibilities and orders an x-ray of the chest,
which appears to be normal. A puried protein
derivative test is performed and 48 hours later
demonstrates 12-mm induration. The diagnosis
of pulmonary tuberculosis is made, and the
child is started on isoniazid, rifampin, and
Image courtesy of the Centers for Disease Control and pyrazinamide. Which of the following is a
Preventions Public Health Image Library; content pro- known adverse reaction of rifampin that should
vider Joe Miller. be monitored during this childs treatment?
(A) Agranulocytosis (A) Angioedema
(B) Fulminant liver failure (B) Hepatotoxicity
(C) Osteopenia (C) Hyperuricemia
(D) Pulmonary edema (D) Peripheral neuritis
(E) Renal toxicity (E) Stevens-Johnson syndrome

30. A 7-year-old boy is brought to the emergency 32. A 21-year-old HIV-positive man with a CD4+
department for evaluation of neck pain. His cell count of 42/mm presents to his ophthal-
mother reports he began complaining of the mologist with blurry vision and oaters in his
pain 3 days ago, when she noted a mass on the right eye. He denies pain. Which of the follow-
left neck. The child has been healthy previ- ing is the most important disease to rule out?
ously and the mother denies any previous epi-
(A) Blepharitis
sodes. His immunizations are up to date. Fam-
(B) Conjunctivitis
ily history is noncontributory. The mother
(C) Cytomegalovirus retinitis
denies recent travel or sick contacts. There are
(D) Open-angle glaucoma
no pets in the house, but the mother does re-
port noticing several small scratches on the
boys face after he played with the neighbors
cats a few weeks ago. His heart rate is 102/min,
blood pressure is 96/62 mm Hg, and respira-
tory rate is 22/min. Physical examination re-
veals three red papules (approximately 3 mm
HIGH-YIELD SYSTEMS
Chapter 8: Infectious Disease Questions 221

33. A 21-year-old G0P0 woman with no signicant 35. A 33-year-old smoker is being treated for
past medical history presents to the emergency chronic cavitary histoplasmosis. He lives in ru-
department with a 2-day history of bilateral ral Missouri and works at a farm where he
lower abdominal pain and vaginal discharge. cleans chicken coops. He also owns a small
She began having unprotected sexual inter- farm where he plants and harvests corn. He
course with her new boyfriend 4 weeks ago. makes occasional long trips to Baltimore to sell
Her temperature is 39.1C (102.4F), blood his corn and buy more seeds. He states that his
pressure is 131/73 mm Hg, heart rate is 98/ only hobby is swimming and he swims in local
min, and respiratory rate is 13/min. During the public swimming pools. Which of the follow-
bimanual examination she has adnexal tender- ing most likely could have prevented this pa-
ness, purulent cervical discharge, and extreme tients condition?
pain. Which diagnostic test is most useful in
(A) Avoiding cigarette smoke
guiding the treatment of her condition?
(B) Avoiding corn elds

Infectious Disease
(A) -Human chorionic gonadotropin test (C) Avoiding swimming in public pools
(B) Diagnostic laparoscopy (D) Avoiding the chicken droppings
(C) DNA probes for Chlamydia trachomatis (E) Avoiding travel to Baltimore
and Neisseria gonorrhoeae
(D) Gram stain of discharge 36. A 45-year-old veteran is seen in the clinic be-
(E) WBC count cause of poor impulse control. Over the past
month he has spent all of his savings on busi-
34. A patient with HIV infection and a CD4+ cell ness ventures and is currently estranged from
count <100/mm who is a cat lover developed his wife. He admits a decreased need for sleep,
lesions on his hands and arms about 2 weeks a new sense of grandiosity, and increased en-
ago (see image). Biopsy of the lesions reveals ergy spent on researching projects to make
tiny clumps of bacilli on Warthin-Starry silver money. He has no prior psychiatric history, and
stain. Which of the following agents is effective his only medical history includes treatment for
in treating this mans lesion? gonorrhea during his military service and an
appendectomy at age 36 years. Physical exami-
nation reveals a thin, middle-age man with
pressured speech. His pupillary examination is
normal, except that the pupils are reactive to
convergence but not to light. Infection with
which of the following organisms is the most
likely etiology of his symptoms?
(A) Chlamydia trachomatis
(B) Herpes simplex virus
Reproduced, with permission, from Kasper DL, Braun-
(C) Neisseria gonorrhoeae
wald E, Fauci AS, Hauser SL, Longo DL, Jameson LJ, Is-
selbacher KJ, eds. Harrisons Online. New York: McGraw- (D) Toxoplasma
Hill, 2005: Figure 144-1. (E) Treponema pallidum
(A) Acyclovir
(B) Dexamethasone cream
(C) Erythromycin
(D) Ganciclovir
(E) Highly active antiretroviral therapy
HIGH-YIELD SYSTEMS
222 Section I: Organ Systems Questions

37. A 56-year-old man is hospitalized with an in- has never had this before. Further examination
fection for which intravenous antibiotics are reveals extension of thrush into the esophagus.
required. Upon questioning, it is revealed that Which of the following conditions does the pa-
he has a penicillin allergy. Which of the fol- tient most likely have?
lowing statements regarding the choice of anti-
(A) Cardiac chest pain
biotics in this situation is correct?
(B) HIV
(A) A history of anaphylaxis to penicillin pre- (C) Mallory-Weiss tear
cludes the use of cephalosporins (D) Nothing; healthy patients can get candidal
(B) An allergy to penicillin means that other esophagitis
drugs in its class (penicillins) should not (E) Severe combined immunodeciency syn-
be prescribed, but that antibiotics from drome
other classes are safe to use (F) Vaginal candidiasis
(C) For patients with anaphylaxis to penicillin,
Infectious Disease

aztreonam may be a worthwhile alterna- 40. A 2.5-year-old girl was diagnosed with a viral
tive upper respiratory infection last week and now
(D) If the allergy occurred when the patient presents to her physician with a brassy cough
was a child, the risk of having another re- and purulent airway secretions. Her parents be-
action remains the same throughout adult- lieved that she may have croup and attempted
hood mist therapy without success. The patient has
(E) Imipenem can safely be prescribed to a pa- been febrile to 39.0C (102.2F) for the past
tient with a skin-test-proven allergy to peni- 24 hours and has had difculty breathing dur-
cillin ing the past hour. Her review of systems is oth-
(F) The rate of allergic reaction to other erwise negative, and the mother denies that the
-lactam antibiotics is much higher among patient has had any drooling or difculty/pain
HIV-positive patients when swallowing. Her heart rate is 92/min,
blood pressure is 92/60 mm Hg, respiratory
38. A 22-year-old woman presents to the emer- rate is 21/min, and temperature is 38.9C
gency department after being found unrespon- (102F). A Gram stain of secretions obtained
sive with diffuse erythema involving her palms. in the ofce is shown in the image. What or-
Her temperature is 39.4C (103F) and blood ganism is most likely responsible for this pa-
pressure is 90/55 mm Hg. Her medical history tients condition?
is noncontributory. Physical examination re-
veals diffuse erythema involving the palms but
is otherwise unremarkable. Her boyfriend
mentions a week-long camping trip approxi-
mately 1.5 weeks ago and notes that both he
and the patient are outdoor enthusiasts. What
is the most likely diagnosis?
(A) Lyme disease
(B) Measles
(C) Rheumatic fever
(D) Rocky Mountain spotted fever
(E) Toxic shock syndrome
Reproduced, with permission, from Brooks GF, Carroll
KC, Butel JS, Morse SA. Jawetz, Melnick, & Adelbergs
39. A 27-year-old sexually active African-American Medical Microbiology, 24th edition. New York: McGraw-
woman presents to the emergency department Hill, 2007: Figure 14-1.
complaining of low-grade fevers, chills, night
sweats, and burning chest pain that is worse (A) Anaerobic organisms
with swallowing. The physician immediately (B) Moraxella catarrhalis
notices white patches consistent with oral can- (C) Nontypeable Haemophilus inuenzae
didiasis on the patients tongue. She says she
HIGH-YIELD SYSTEMS
Chapter 8: Infectious Disease Questions 223

(D) Pseudomonas aeruginosa 43. A 49-year-old quarry worker undergoes her


(E) Staphylococcus aureus yearly employee physical. Screening x-ray of
the chest reveals a 1.5-cm (0.6-in) subpleural
41. A 2-year-old boy is brought to the clinic for parenchymal lesion in the lateral aspect of the
evaluation of acne. His mother also describes right lung, as well as enlarged hilar nodes, egg-
a recent onset of pain in his left foot and right shell hilar calcications, and multiple small
hand. She says he has been hospitalized once nodules scattered throughout the upper lung
for lymphadenitis, but he has been otherwise elds. The patient denies cough, weight loss,
healthy. His review of systems is otherwise neg- and night sweats. In fact, she states that she
ative. He has two maternal uncles who died in feels perfectly well. Sputum culture is positive
childhood from pneumonia. The child is afe- for acid-fast bacilli. Exposure to which of the
brile, with a heart rate of 64/min, blood pres- following substances increased this patients
sure of 108/70 mm Hg, and respiratory rate of chance of contracting tuberculosis?
15/min. Physical examination reveals multiple

Infectious Disease
(A) Asbestos
scarred lesions on his skin, two of which are 1
(B) Beryllium
cm in diameter and lled with purulent mate-
(C) Carbon
rial. Pain is elicited by palpation of the puru-
(D) Coal
lent lesions, and they are somewhat warm to
(E) Silica
the touch. From which immunodeciency dis-
order is this patient most likely suffering?
44. A 4-year-old Asian boy presents to the emer-
(A) Ataxia-telangiectasia gency department with a fever of 39.4C
(B) Chdiak-Higashi syndrome (103F). His mother reports that he rst devel-
(C) Chronic granulomatous disease oped a sore throat 10 days ago, his fever has
(D) Hyper IgE syndrome persisted for 8 days, he developed an erythema-
(E) Wiskott-Aldrich syndrome tous maculopapular rash in his groin that
started 6 days ago that has now spread to the
42. A 30-year-old woman comes to her primary rest of his body, and he had red, watery eyes 3
care physician complaining of episodes of an days ago. Signicant physical ndings include
irregular heartbeat, muscle weakness, joint an erythematous tongue, red swollen feet, cer-
swelling, and headaches. The symptoms began vical adenopathy, a diffuse maculopapular
in early October and have lasted for approxi- rash, and mildly injected sclera. Administration
mately 5 weeks; she was unable to get an ear- of which of the following would most likely
lier appointment. The headaches and irregular have prevented this condition?
rhythm uctuated throughout the day. Now
(A) Acetaminophen
the arrhythmia has resolved, but she is still
(B) Aspirin and intravenous immunoglobulin
bothered by the weakness and headaches. She
(C) Corticosteroids
had no precipitating illness, condition, or un-
(D) Metoprolol
usual exposures. Her only recent travel was a
(E) Penicillin
camping trip with her daughter in West Vir-
ginia this past summer. Which of the following
is the most likely diagnosis?
(A) Gonococcal disease
(B) Gout
(C) Lyme disease
(D) Migraine
(E) Sjgrens syndrome
(F) Systemic lupus erythematosus
HIGH-YIELD SYSTEMS
224 Section I: Organ Systems Questions

45. A 50-year-old man presents to the emergency 47. A 67-year-old man is brought to the emergency
department with a fever, productive cough, department by his daughter, who reports that
dyspnea, and pleuritic chest pain. He states her father has been acting strangely. She in-
that the symptoms started suddenly yesterday dicates that he is usually quiet and considerate.
night. X-ray of the chest demonstrates a conu- However, for the past day he has been argu-
ent left lower lobe. The border of the minor s- mentative, irresponsible, and increasingly con-
sure is visible. Quellung reaction identies the fused. She decided to bring him to the emer-
infection as pneumococcal pneumonia. Sus- gency department when an elderly neighbor
ceptibility and culture show resistance to peni- called to complain that he had made overt sex-
cillin. Which of the following is the most ap- ual advances toward her. The patient is alert
propriate treatment? but disoriented and uncooperative. His tem-
perature is 38.2C (100.8F), and he is sensi-
(A) Aztreonam
tive to light. There are no focal neurologic def-
(B) Ceftriaxone
icits, but cerebrospinal uid contains 50 WBCs
Infectious Disease

(C) Doxycycline
with normal protein and normal glucose levels.
(D) Gentamicin
Which of the following would most likely con-
(E) Metronidazole
rm the diagnosis?
46. A 4-year-old child presents to the clinic for (A) Draw blood to check blood alcohol level
evaluation of a fever (39.0C [102.1F]), head- (B) Obtain CT of the head with contrast
ache, generalized malaise, and decreased ap- (C) Perform herpes simplex virus DNA poly-
petite that began 2 days earlier. He has now de- merase chain reaction of the cerebrospinal
veloped a generalized pruritic rash all over his uid
body, as shown in the image. Which of the fol- (D) Perform urinalysis and begin empiric anti-
lowing is the most likely diagnosis? biotics
(E) Repeat lumbar puncture to measure open-
ing pressure

48. A 43-year-old man with HIV infection who has


declined antiretroviral therapy is undergoing a
corticosteroid taper for Pneumocystis jiroveci
pneumonia. He presents to his primary care
physician with the lesions shown in the image
and complains of nausea, abdominal pain, and
foul-smelling stools. The patients skin lesions
are not pruritic or painful. What treatment
could best have prevented this development?

Reproduced, with permission, from Wolff K, Johnson RA,


Surmond D. Fitzpatricks Color Atlas & Synopsis of Clini-
cal Dermatology, 5th edition. New York: McGraw-Hill,
2005: Figure 25-39.

(A) Chickenpox
(B) Contact dermatitis
(C) Hand-foot-and-mouth disease
(D) Herpes zoster
(E) Smallpox
(F) Stevens-Johnson syndrome
HIGH-YIELD SYSTEMS
Chapter 8: Infectious Disease Questions 225

has been particularly irritable and has been


slurring her speech since yesterday. She has
otherwise been in good health and only relates
Infectious Disease

a history of an oral blister a week ago. He de-


nies any remote travel history. Her temperature
is 40C (104F), blood pressure is 150/78 mm
Hg, pulse is 98/min, and respiratory rate is 20/
min. She is confused and disoriented, but the
rest of the neurologic examination is unre-
markable. There is no associated papilledema
on funduscopy. Kernigs and Brudzinskis signs
are negative. CT of the head is normal. A lum-
Reproduced, with permission, from Wolff K, Johnson RA, bar puncture is performed and cerebrospinal
uid analysis shows:

Infectious Disease
Surmond D. Fitzpatricks Color Atlas & Synopsis of Clini-
cal Dermatology, 5th edition. New York: McGraw-Hill,
2005: Fig. 1e-HIV6.
Protein: 82 mg/dL
Glucose: 63 mg/dL
(A) Dapsone WBC count: 150/mm
(B) Highly active antiretroviral therapy Gram stain: negative
(C) Trimethoprim-sulfamethoxazole
(D) Zidovudine/lamivudine combination ther- Which of the following is the most likely diag-
apy nosis?
(E) Zidovudine monotherapy (A) Bacterial meningitis
(B) Herpes simplex virus encephalitis
49. A 26-year-old sexually active woman who re- (C) Neurocysticercosis
cently spent a year in East Africa presents to (D) Rabies encephalitis
her gynecologist with bilateral nontender in- (E) Subacute HIV encephalomyelitis
guinal lymphadenopathy. Serology conrms (F) Tabes dorsalis
the diagnosis of lymphogranuloma venereum.
Which of the following is the most appropriate 51. A 25-year-old primigravida presents to the ob-
pharmacotherapy? stetricians ofce for her rst prenatal visit.
(A) Acyclovir Which of the following titers is routinely ob-
(B) Ceftriaxone tained at the rst prenatal visit?
(C) Doxycycline (A) Cytomegalovirus and hepatitis B
(D) Gentamicin (B) Cytomegalovirus and rubella
(E) Penicillin (C) Rubella and syphilis
(D) Rubella, syphilis, HIV, hepatitis B, cyto-
50. A 37-year-old woman was brought in by para- megalovirus, and toxoplasmosis
medics after suffering a seizure at her home. (E) Toxoplasmosis and HIV
Per her husband, she was feeling ill and con- (F) Toxoplasmosis, syphilis, and rubella
fused for the past couple of days, complaining
of fever and headaches. He also states that she
HIGH-YIELD SYSTEMS
226 Section I: Organ Systems Questions

52. A 65-year-old man with poorly controlled dia- (C) Recommend treatment of her boyfriend
betes mellitus presents to the emergency de- (D) Report the case to the county department
partment complaining of headache, nasal con- of health
gestion, and purulent nasal discharge. On (E) Screen the patient for diabetes
physical examination he is febrile. He has sinus
tenderness, periorbital and facial swelling, and 55. A 47-year-old HIV-positive woman is brought
black eschar in the mucosa of his left naris. Ac- in by ambulance after her husband called the
companying family members state that he has paramedics claiming that his wife was having
been somewhat confused and disoriented difculty breathing. Per her husband, she has
lately. Once this mans diagnosis is conrmed, been experiencing a nonproductive cough with
what is the most important next step in his fever, chills, fatigue, and difculty catching her
care? breath over the past couple weeks. Her last
CD4+ cell count was 174/mm, despite being
(A) Aggressive surgical debridement of the in-
on highly active antiretroviral therapy with zi-
Infectious Disease

volved tissues
dovudine, didanosine, and indinavir. Her tem-
(B) Broad-spectrum antibiotic coverage with
perature is 38.7C (101.6F), blood pressure is
vancomycin and piperacillin-tazobactam
150/79 mm Hg, pulse is 94/min, and respira-
(C) High-dose treatment with liposomal am-
tory rate is 29/min. Arterial blood gas analyses
photericin B
show a partial arterial oxygen pressure of 59
(D) Treatment with amoxicillin-clavulanate
mm Hg, partial arterial carbon dioxide pressure
(E) Treatment with the broad-spectrum anti-
of 29 mm Hg, and pH 7.56. Which of the fol-
fungal agent posaconazole
lowing is the best next step in management?
53. An 18-year-old man arrives at the emergency (A) Aerosolized pentamidine
department. He is a swimmer, and is complain- (B) Amphotericin B
ing of ear pain and discharge for the past 2 (C) Dapsone
days. He denies any fever or headache, but (D) Intravenous vancomycin
states that he does feel some pressure in his (E) Trimethoprim-sulfamethoxazole with cor-
head, and pulling on his pinna elicits pain. ticosteroids
Which of the following is the most appropriate
next step in management? 56. A 22-year-old college student is brought to the
emergency department by her roommate, who
(A) CT of the head
states that the patient has been feeling sick for
(B) Intravenous antibiotics for Naegleria fowl-
the past 36 hours. The patient initially com-
eri
plained of a headache, fever, nausea, and vom-
(C) MRI of the head
iting, then later developed multiple petechiae
(D) Surgery
on her arms and legs. Her roommate decided
(E) Topical medication and cleaning
to take the patient to the hospital when she be-
came difcult to arouse. Which of the follow-
54. A 14-year-old girl presents to her family practi-
ing is the most appropriate prophylaxis for the
tioner with a 2-day history of profuse vaginal
patients roommate?
discharge and pruritus. She admits that she has
recently become sexually active with her boy- (A) Acyclovir
friend. The physician examines a potassium (B) Ampicillin
hydroxide mount of the vaginal discharge and (C) Cefotaxime
sees many WBCs, but no hyphae. The pH of (D) Rifampin
the discharge is higher than normal. What is (E) Vancomycin
the physician obligated to do next?
(A) Inform the patients mother that she is en-
gaging in unsafe sex
(B) Recommend abstaining from sexual inter-
course for 2 weeks
HIGH-YIELD SYSTEMS
Chapter 8: Infectious Disease Questions 227

57. A 25-year-old woman presents to her physician (A) Bacillus cereus


because of the development of a low-grade fe- (B) Salmonella
ver, cough, nasal congestion, and a distinct (C) Shigella
rash on her tongue. A review of her chart re- (D) Staphylococcus aureus
veals that she has been previously very healthy (E) Vibrio cholerae
but did not receive many immunizations as a (F) Vibrio parahaemolyticus
child because of the familys religious beliefs.
On examination the patient is afebrile with an 59. A 5-year-old boy presents to pediatric clinic
enanthem on her buccal mucosa and tongue complaining of a cough. The childs mother
(see image). Which of the following is the most reports that he had a temperature to 38.6C
appropriate next step in treatment? (101.5F) at home and has not been able to
sleep for several nights due to a constant, pro-
ductive cough. The cough began 5 days ago
and has gradually worsened. The child also

Infectious Disease
had rhinorrhea for several days prior to onset of
the cough. The child has no vomiting, diar-
rhea, abdominal pain, or difculty urinating.
The childs mother reports that his immuniza-
tions are up to date. His temperature is 38.2C
(100.8F), heart rate is 92/min, respiratory rate
is 32/min, and oxygen saturation is 96% on
room air. The child is alert and playful, cough-
ing frequently. Auscultation of lungs reveals
Image courtesy of the Centers for Disease Control and diffuse rhonchi and mild wheezing with equal
Preventions Public Health Image Library. breath sounds bilaterally. X-ray of the chest
(A) Acyclovir shows diffuse inltrates with no focal disease.
(B) Amantadine What pathogen is most likely responsible for
(C) Corticosteroids this childs illness?
(D) Immune globulin against the causative (A) Adenovirus
agent (B) Cytomegalovirus
(E) Supportive therapy only (C) Group B Streptococcus
(D) Haemophilus inuenzae type B
58. A 22-year-old woman comes to the emergency (E) Streptococcus pneumoniae
department accompanied by her boyfriend.
She is complaining of nausea, vomiting, ab-
dominal pain, and cramping over the past
hour. She denies fever, headaches, or watery or
bloody stools. She states that she was feeling
ne until an hour ago, when she began experi-
encing the onset of her symptoms. She has
been on a no-carbohydrate diet for the past 3
weeks. Her last meal was an egg salad for lunch
approximately 3 hours ago. She denies any
travel in her history. Her physical examination
is normal except for mild abdominal tender-
ness with palpation. Stool testing is guaiac neg-
ative and reveals no fecal leukocytes. Which of
the following is the most likely pathogen?
HIGH-YIELD SYSTEMS
228 Section I: Organ Systems Questions

60. A 3-year-old boy is brought to the emergency 61. A 12-year-old boy presents to the emergency
department because of neck stiffness and torti- department because of a fever of 39C (102F),
collis. He developed a sore throat 3 days prior myalgias, headaches, and petechial rash after
with a fever of 38.8C (101.8F), increasingly returning from Boy Scout camp in western
became irritable, and had a decreased oral in- North Carolina. His mother states that the rash
take. His pulse is 98/min, blood pressure is started at his feet and hands and then spread to
90/58 mm Hg, respiratory rate is 20/min, and his trunk.
temperature is 38.9C (102F). The patients
oropharynx is clear but his tonsils are mildly 62. A 2-year-old boy who attends day care has a fe-
displaced to the left. Contrast CT scan of the ver for 3 days. The fever nally subsides, but
head shows a ring-enhancing lesion of the right then a maculopapular pink rash develops over
lateral neck. Which of the following statements his trunk and spreads to his arms and face. His
regarding retropharyngeal abscesses is true? mother has not noticed the child scratching at
the rash, but says that he does seem more irri-
Infectious Disease

(A) Antibiotics play no role in the treatment of


table.
retropharyngeal abscesses
(B) Incision and drainage with culture of ab-
63. A 9-year-old boy is brought to his doctor be-
scess material provides the most denitive
cause of headache, fever, vomiting, and ex-
diagnosis and treatment of retropharyngeal
tremely sore throat. Physical examination re-
abscesses
veals a bright pink tongue and erythematous
(C) Infection often originates in the superior
oropharynx with white patches on his tonsils.
mediastinum and migrates superiorly to
There is also a rash on his trunk.
the retropharyngeal lymph nodes
(D) Respiratory distress is a well known com-
plication of lateral pharyngeal abscesses The response options for the next 3 items are
but not retropharyngeal abscesses the same. Select one answer for each item
(E) The causative pathogen is almost exclu- in the set.
sively Staphylococcus aureus
For each of the following patients, select the most
likely sexually transmitted disease.
E X T E N D E D M ATC H I N G
(A) Candida albicans
(B) Chlamydia trachomatis
The response options for the next 3 items are
(C) Escherichia coli
the same. Select one answer for each item
(D) Haemophilus ducreyi
in the set.
(E) Herpes simplex virus
(F) HIV
For each of the following patients, select the most
(G) Human papillomavirus
appropriate treatment.
(H) Neisseria gonorrhea
(A) Acyclovir (I) Proteus
(B) Amantadine (J) Treponema pallidum
(C) Amphotericin B
(D) Dapsone 64. A 28-year-old African-American man presents
(E) Doxycycline to the physician because of penile discharge,
(F) Fluconazole frequent urination, and pain on urination. His
(G) Foscarnet left knee is swollen, red, and tender. He was re-
(H) Gentamicin cently sexually active without using protection.
(I) No treatment is necessary
(J) Penicillin
HIGH-YIELD SYSTEMS
Chapter 8: Infectious Disease Questions 229

65. A 28-year-old African-American man presents


The response options for the next 2 items are
to the physician because of penile discharge,
the same. Select one answer for each item in
frequent urination, and pain on urination. He
the set.
was recently sexually active without protection.
Gram stain of a urethral specimen shows no For each patient with urinary symptoms, select the
gram-negative diplococci. most likely diagnosis.
66. A 28-year-old African-American man presents (A) Acute cystitis
to the physician because of penile discharge, (B) Acute pyelonephritis
frequent urination, and pain on urination. (C) Acute urinary retention
Physical examination reveals multiple vesicles (D) Carcinoma of the bladder
on the patients penis. (E) Diabetes insipidus
(F) Diabetes mellitus
(G) Hypercalcemia

Infectious Disease
The response options for the next 2 items are
(H) Interstitial cystitis
the same. Select one answer for each item
(I) Neurogenic bladder
in the set.
(J) Psychogenic polydipsia
(K) Renal cell carcinoma
For each of the following patients, select the most
(L) Urethral stricture
appropriate prophylactic and/or treatment regimen
to institute.
69. A 19-year-old woman presents to her primary
(A) Hepatitis B vaccine care provider because of urinary urgency, hesi-
(B) Isoniazid and pyridoxine tancy, frequency, painful urination, and in-
(C) No new treatment or prophylaxis is needed complete voiding. Abdominal and pelvic ex-
at this time aminations are unremarkable. The patient
(D) Papanicolaou smears every year recently became sexually active. Gram stain of
(E) Pneumocystis jiroveci pneumonia prophy- a centrifuged midvoid urine specimen shows a
laxis coagulase-negative, gram-positive coccus resis-
(F) Suppressive acyclovir tant to novobiocin.
(G) Toxoplasmosis prophylaxis
(H) X-ray of the chest 70. A 41-year-old woman presents to the clinic
with a 1-day history of dysuria, moderate back
67. A 34-year-old HIV-positive man has a CD4+ pain, nausea, and vomiting. Her temperature is
cell count of 196/mm3. 38.4C (101.1F). Abdominal examination re-
veals costovertebral angle tenderness, and pel-
68. A 26-year-old HIV-positive woman has had two vic examination is unremarkable. A urine
outbreaks of painful genital lesions in the past -chorionic gonadotropin test is negative.
6 months.
HIGH-YIELD SYSTEMS
230 Section I: Organ Systems Answers

AN S W E R S

1. The correct answer is A. The patient has had a Answer E is incorrect. Valproic acid is an an-
simple febrile seizure, which is a brief, general- tiepileptic medication that is effective in de-
ized, tonic-clonic seizure associated with a fe- creasing recurrent febrile seizures. However,
brile illness (temperature >38C [100.4F]), the risks and potential adverse effects out-
without an underlying serious infection or neu- weigh the benets. Valproic acid can lead to
rologic cause. Febrile seizures occur within 24 valproate-induced hepatotoxicity, especially in
hours of the onset of fever and can be simple or children <2 years old. There are also no data
complex. In contrast to complex febrile seizures, to suggest that preventing recurrent febrile sei-
simple febrile seizures last <15 minutes, have no zures decreases the risk of developing afebrile
focal features during the seizure or postictal pe- seizures.
riod, and, if they occur in series, do not last >30
Infectious Disease

minutes total. Febrile seizures are most com- 2. The correct answer is B. This patients rash,
mon in the 12- to 18-month age group. Febrile erythema chronicum migrans, is suggestive of
seizures may occur with viral infections, such as Lyme disease, caused by the pathogen Borrelia
human herpesvirus (HHV) 6 infection (roseola), burgdorferi, which is transmitted by the bite of
upper respiratory tract infections, and acute oti- the Ixodes tick. It is endemic in the northeast-
tis media (OM), and after immunization with ern United States. Treatment is with oral doxy-
diphtheria-tetanus-pertussis or measles-mumps- cycline, amoxicillin, or cefuroxime axetil. Pa-
rubella vaccine. Antipyretics such as acetamino- tients with certain neurologic or cardiac forms
phen and sponging with tepid water can be used of the illness may require intravenous treat-
to control the fever. A short-acting benzodiaz- ment with drugs such as ceftriaxone or penicil-
epine such as lorazepam can be given intrave- lin.
nously if the seizure lasts >5 minutes.
Answer A is incorrect. Although Staphylococ-
Answer B is incorrect. An electroencephalo- cus aureus may superinfect a skin lesion such
gram is unnecessary in the setting of a simple as a tick bite, it generally produces an ulcerat-
febrile seizure and cannot predict the likeli- ing lesion with a more evenly erythematous
hood of recurrent febrile seizures or develop- surrounding area. Erythema chronicum mi-
ment of other types of seizures. grans is thus inconsistent with this infection.
Treatment is generally with penicillin or van-
Answer C is incorrect. A lumbar puncture
comycin, though resistance patterns for this
should be performed in children <12 months
agent are evolving rapidly.
old after their rst febrile seizure. It should also
be considered if the seizure occurs after the Answer C is incorrect. Meningitis typically
second day of illness and if the clinician can- presents with a headache and can sometimes
not condently rule out meningitis. Because include a rash, typically with meningococcus,
this patient has no focal neurologic symptoms, but this is more prevalent in children. Also,
meningismus, or Brudzinskis or Kernigs signs, the history of recent hiking in an area endemic
and has an upper respiratory infection that ex- for Lyme disease is highly suggestive of a tick-
plains her fever, meningitis is very unlikely. borne illness. Ceftriaxone is generally the anti-
biotic of choice for staphylococcal infections.
Answer D is incorrect. Phenobarbital is an-
other antiepileptic medication that is effective Answer D is incorrect. Myocarditis is inam-
in decreasing recurrent febrile seizures. The mation of the myocardium with myocyte ne-
risks and potential adverse effects, however, crosis. It often follows viral infection and can
outweigh the benets, and it is not recom- present like an upper respiratory tract infec-
mended by the American Academy of Pediat- tion, featuring fever, dyspnea, and chest pain.
rics. Only rarely would it present with such a local-
ized rash. Treatment is focused on reducing
HIGH-YIELD SYSTEMS
Chapter 8: Infectious Disease Answers 231

the demands placed on the myocardium. Im- holding and withdrawing life-supporting care
munosuppression and anti-inammatory drug from a patient are currently considered ethi-
use is controversial. cally identical.
Answer E is incorrect. Rocky Mountain spot-
4. The correct answer is D. This child suffered
ted fever (RMSF), like Lyme disease, presents
from the classic triad of chorioretinitis, hydro-
with a headache and rash, but the rash de-
cephalus, and intracranial calcications that
scribed in the stem is more typical of Lyme
indicates congenital toxoplasmosis infection.
disease. The rash suggestive of RMSF is more
This diagnosis is conrmed by the histologic
diffuse, appearing on the wrists, ankles, soles,
image, which shows tachyzoites of Toxoplasma
and palms, rather than in one isolated location.
gondii, the pathogenic parasite of toxoplasmo-
This disease is endemic to the southeastern
sis, contained in a pseudocyst inside a myo-
United States. Treatment is with doxycycline
cyte. Maternal infection occurs via contact
and chloramphenicol.
with the oocysts present in the feces of an in-

Infectious Disease
fected house cat or by consuming under-
3. The correct answer is A. A patient and family
cooked meat of infected animals. If untreated,
have the right to withhold treatment for venti-
toxoplasmosis can lead to blindness, retarda-
lation, antibiotics, or uids. There is no dis-
tion, or death. Diagnosis is made via a combi-
tinction between withholding and withdrawing
nation of clinical assessment and polymerase
life-supporting treatment. However, when the
chain reaction or serologic laboratory testing
patient is not conscious it is important to con-
on infant tissue. The most widely accepted
rm the patients wishes through prior legal
regimen for treatment of infected neonates is
documentation and to ensure that the relative
pyrimethamine, sulfadiazine, and leucovorin.
is aware of the most recent decisions of the pa-
tient. Answer A is incorrect. Acyclovir is the treat-
ment of choice for congenital herpes simplex
Answer B is incorrect. In the absence of a
virus (HSV) infection; its administration has
signed legal document, the patients family is
been shown to improve long-term survival
the rst line of contact for decision making re-
along with associated morbidities. Initial mani-
garding the patients management.
festations of HSV are usually subtle or absent
Answer C is incorrect. Although the patients at birth. The three common patterns of presen-
primary care physician may be called upon to tation are (1) localized skin, eye, and mouth
help make choices in management after family involvement; (2) localized central nervous sys-
members have been contacted, the physicians tem disease; and (3) disseminated systemic ill-
prior interactions with the patient alone are ness. The disease most commonly makes itself
not enough to withdraw care. known in the rst month after birth. Diagnosis
is generally clinical with support from poly-
Answer D is incorrect. Power of attorney can
merase chain reaction analysis of infected tis-
be granted by the patient to any person for a
sue. Prognosis is worst for disseminated disease,
specic period of time, or permanently (du-
which carries a mortality rate >50%.
rable power of attorney). A person previously
granted power of attorney holds no special de- Answer B is incorrect. Ceftriaxone is a third-
cision-making rights once that specic period generation cephalosporin that has no role in
of time (previous hospitalization) has ended. the treatment of congenital toxoplasmosis. It
has strong gram-negative coverage but is of no
Answer E is incorrect. The advance directive
utility against parasitic infection. It is most use-
does not become a legal document until it is
ful for the treatment of congenital Chlamydia
signed. Any prior paperwork may be used to
infections, especially involving the eyes.
help a power of attorney.
Answer C is incorrect. Parenteral penicillin G
Answer F is incorrect. The patient has the
is the treatment of choice for congenital syphi-
right to refuse life-supporting treatment. With-
lis. Though its presentation can resemble that
HIGH-YIELD SYSTEMS
232 Section I: Organ Systems Answers

of toxoplasmosis, the tachyzoites pictured in Answer A is incorrect. Contrast nephropathy


the image are diagnostic of toxoplasmosis. The may be contributing, as it can occur a few days
most common initial symptoms of congenital after the administration of contrast and is more
syphilis are a rash on the palms and soles, he- likely in patients with diabetic nephropathy,
patosplenomegaly, jaundice, and snufes. The but it would not explain the clinical picture
gold standards of diagnosis are direct spiro- of fever and low blood pressure. Had hypoten-
chete visualization with darkeld microscopy sion and decreased urine output immediately
or direct uorescent antibody tests of tissue. followed the administration of intravenous
contrast, it would be characterized as an ana-
Answer E is incorrect. Supportive care is all
phylactoid reaction due the activation of mast
that can be offered to infants aficted with
cells.
congenital rubella infection and other diseases
caused by viral pathogens. This would be an Answer B is incorrect. Diabetic nephropathy
inadequate course of treatment in a child in- causes a more chronic renal failure. Given the
Infectious Disease

fected with toxoplasmosis, as antimicrobial patients baseline creatinine of 1.5 mg/dL, it is


intervention has a proven mortality benet. likely that diabetic nephropathy is the etiology
Congenital rubella syndrome typically presents of her underlying chronic renal insufciency.
with developmental delay, hearing loss, growth This patient presents with acute renal failure.
retardation, and cardiac anomalies.
Answer D is incorrect. Insufcient intravas-
cular volume may cause acute renal failure
5. The correct answer is B. Up to 17% of patients
(prerenal azotemia). Although maintenance
taking uoroquinolones experience some de-
uids had been discontinued, the patient was
gree of gastrointestinal upset.
not restricted to nothing by mouth and pre-
Answer A is incorrect. Allergic reactions, in- sumably was eating and drinking normally.
cluding rash occur in <2% of patients taking While still a possible explanation, this is less
uoroquinolones. likely, as there is an infectious source as well as
Answer C is incorrect. Neurologic manifesta- clinical ndings compatible with pyelonephri-
tions such as mild headache and dizziness are tis. Furthermore, classically, the blood urea
the second most common adverse reaction of nitrogen:creatinine ratio in prerenal azotemia
patients taking uoroquinolones, affecting up is >20; here the ratio is <10. Further tests to
to 11% of patients. determine the etiology of the renal failure
would be urine osmolality, urine sodium, and
Answer D is incorrect. QT prolongation is fractional excretion of sodium.
also a rare complication of treatment and very
uncommonly leads to ventricular arrhythmias. Answer E is incorrect. Acidosis will not ex-
plain the costovertebral angle tenderness or
Answer E is incorrect. Tendonitis and Achilles the entire clinical picture. Furthermore, the
tendon rupture are exceedingly rare in patients most common acid-base disturbance in sepsis
taking uoroquinolones but have increased in- is combined metabolic acidosis and respiratory
cidence in children and pregnant women. alkalosis.

6. The correct answer is C. Patients with urinary 7. The correct answer is A. Group B Streptococ-
tract infections and hospital-acquired pneumo- cus (GBS) is the most common cause of neo-
nias can quickly become bacteremic. With he- natal meningitis, transmitted vertically from
matogenous spread, the infecting agents can the mother to the patient. Late-onset GBS in-
potentially infect other organs. In this case, the fection usually occurs between postpartum
sudden onset of fever, bacteremia, x-ray nd- weeks 2 and 3. This mother did not receive
ings, and acute kidney failure are suggestive of prenatal care and was most likely GBS-positive
sepsis. The costovertebral angle tenderness also but this was not recognized. Normally at 36
suggests pyelonephritis. weeks gestation, many institutions perform
HIGH-YIELD SYSTEMS
Chapter 8: Infectious Disease Answers 233

screening for GBS, and patients who have a days later. A high fever >40C (>104F) typi-
positive culture receive intravenous penicillin cally accompanies a blanching, maculopapu-
when they present in labor. lar rash that begins on the head and then de-
scends over the rest of the body. It can become
Answer B is incorrect. Meningitis due to in-
conuent and fades in a cranial-to-caudal fash-
uenza is rare in neonates compared to rates
ion, leaving behind desquamation and brown-
of GBS infection. Though the examination
ish discoloration that lasts 710 days.
would likely be similar for infection by either
agent, the cerebrospinal uid (CSF) glucose Answer C is incorrect. Parvovirus B19 infec-
level would likely be normal to increased in tion causes erythema infectiosum (fth dis-
inuenza infection. ease). The prodrome may include low-grade
fever, headache, and symptoms of upper re-
Answer C is incorrect. The child received an
spiratory tract infection. There is a slapped-
appropriate dose of acetaminophen based on
cheek erythematous rash on the face, which
the history.

Infectious Disease
rapidly spreads as a diffuse, erythematous, mac-
Answer D is incorrect. Varicella-zoster virus is ulopapular rash to the trunk and proximal ex-
not the most likely cause, as neither the classic tremities. There is then central clearing of the
stigmata of skin rash/skin scar or eye abnormal- lesions, resulting in a lacy reticular rash. The
ities (chorioretinitis and cataracts) is present. rash worsens with fever, exposure to the sun,
Answer E is incorrect. Viridans streptococci exercise, and/or stress.
are the etiologic agent of subacute bacterial Answer D is incorrect. Rubella causes an ery-
endocarditis, not meningitis in neonates. Sys- thematous, tender, maculopapular rash that
temic infection with this organism presents begins on the face and descends over the rest
most commonly with fever and chills. Physical of the body. Tender cranial lymphadenopathy
examination is notable for petechiae, splinter precedes the rash by at least 24 hours. The rash
hemorrhages, Osler nodes, Janeway lesions, usually lasts 3 days but evolves rapidly. As it be-
and Roth spots. comes visible on the trunk, it may have faded
from the face. Discrete maculopapules and
8. The correct answer is A. Roseola (or exanthema large areas of ushing are often present. The
subitum or sixth disease) is most commonly rash may coalesce, especially on the face, and
caused by HHV-6, and less commonly HHV-7. may become pinpoint, especially on the trunk.
It is a mild febrile illness that occurs predomi- There may be low-grade fever.
nantly in infants. There may be mild symptoms
such as mild rhinorrhea and mild erythema of Answer E is incorrect. The prodromal stage of
the pharynx and conjunctivae. The acute onset varicella (chickenpox) manifests with fever of
of high fever, often >40C (>104F), heralds 37.8C38.9C (100F102F), anorexia, mal-
clinical illness and may be accompanied by sei- aise, and headache that occurs 2448 hours
zures. The fever resolves abruptly after 35 days, before the rash. The rash begins on the scalp,
and a rose-colored maculopapular rash with dis- face, or trunk, and spreads peripherally. It is a
crete 2- to 5-mm lesions begins as the fever generalized, pruritic, macular rash that evolves
breaks. The rash starts on the trunk and then into clear, uid-lled, teardrop-shaped vesicles.
spreads to the face and extremities. It typically The vesicles become cloudy and umbilicated
remains nonconuent and disappears within 24 and then crust over, and appear at different
hours. stages.

Answer B is incorrect. Measles is character- 9. The correct answer is C. This childs presenta-
ized by a 3-day prodrome of low-grade to mod- tion is characteristic for bronchiolitis. Acute
erate fever, Cough, Coryza, Conjunctivitis (the bronchiolitis is commonly a viral disease with
3Cs). Kopliks spots, small red spots with >50% of cases being caused by respiratory syn-
grey centers on the buccal mucosa, occur 23 cytial virus (RSV). This patient has presented
in respiratory distress with tachypnea (normal
HIGH-YIELD SYSTEMS
234 Section I: Organ Systems Answers

respiratory rate for an infant of this age is 30 ing severe RSV disease. Some studies have
40/min) and nasal aring. Bronchiolitis is gen- demonstrated an increase in mortality when
erally self-limited, although in severe cases or given to symptomatic cyanotic congenital heart
in high-risk patients complications include ap- disease patients. Given her respiratory distress,
nea and severe respiratory distress, requiring release to home is not advisable.
hospitalization for further monitoring. Guide-
lines for hospitalization for infants with bron- 10. The correct answer is E. The patient has acute
chiolitis are not xed but usually include toxic pseudomembranous candidiasis, which can be
appearance; oxygen saturation <95%; atelecta- precipitated by changes in immune status, an-
sis or consolidation on chest radiograph; tac- tibiotics, and abnormal glucose levels in dia-
hypnea >70/min or other signs of respiratory betics. In the setting of abnormal blood sugars,
distress, such as nasal aring, intercostal retrac- thrush is likely to resolve with improved man-
tion, or cyanosis; and gestational age <34 agement of his blood sugars.
weeks or age <3 months. Treatment is largely
Infectious Disease

Answer A is incorrect. Amphotericin B is an


supportive because common interventions
effective antifungal, but it has several serious
(e.g., epinephrine, bronchodilators, corticoster-
adverse effects such as nephrotoxicity, fever,
oids) have not been proven effective in ran-
and rigors, that reserve its use for more serious
domized controlled trials.
fungal infections.
Answer A is incorrect. Because most cases of
Answer B is incorrect. This patients blood
bronchiolitis are viral in nature, antibiotics
sugar level has been normalized and insulin
have no role in the treatment of this condition.
levels adjusted. Adding sugar would actually
Given the ineffectiveness of antibiotics in this
feed the yeast, and create further instability of
patients condition in addition to her respira-
the blood sugar levels, further complicating
tory distress, the patients release to home after
this patients thrush.
antibiotic administration would be unwise.
Answer C is incorrect. Mannitol is used as a
Answer B is incorrect. Chest radiography
diuretic, especially in the setting of increased
would reveal hyperinated lungs with patchy
intracranial pressure from cerebral edema,
atelectasis. The WBC count and differential
classically after traumatic head injury and asso-
are usually normal in cases of bronchiolitis,
ciated epidural bleed.
and neither test would be denitive for a diag-
nosis or necessary for determining a course of Answer D is incorrect. While Candida albi-
treatment. cans can be a normal inhabitant of the oral
mucosa, the patient has an overgrowth of
Answer D is incorrect. Palivizumab (Synagis)
C. albicans, as evidenced by the abnormal
is a monoclonal antibody to the RSV F protein
white patches in his mouth.
that is given intramuscularly and is effective in
preventing severe RSV disease. It is most effec- 11. The correct answer is E. The rash described is
tive when given to high-risk infants before and classic for a case of Lyme disease. Patients typi-
during the RSV season. Palivizumab should cally present in the early stage with erythema
be given to children <2 years old with chronic migrans with or without systemic symptoms,
lung disease (e.g., bronchopulmonary dyspla- including fever, myalgia, malaise, and/or head-
sia) or prematurity. Although prophylactic ad- ache. Early-stage Lyme disease is treated with
ministration of palivizumab would have been one of a number of antibiotics, including tetra-
a reasonable action for this child prior to ill- cycline, doxycycline, amoxicillin, and cefu-
ness, administration during an acute illness is roxime. Doxycycline (as well as other tetracy-
not recommended and release to home is not clines) should be avoided in children <8 years
reasonable. old because of the risk that permanent tooth
Answer E is incorrect. RSV intravenous im- discoloration may occur.
munoglobulin (RSV-IVIG) is a pooled hyper- Answer A is incorrect. Cartilage damage is an
immune IVIG that is also effective in prevent- adverse effect associated with quinolones.
HIGH-YIELD SYSTEMS
Chapter 8: Infectious Disease Answers 235

Answer B is incorrect. Ototoxicity is an ad- Hands and ngers are affected more often than
verse effect of aminoglycosides. feet, and most commonly it is the dorsal sur-
faces that demonstrate the lesions of the disor-
Answer C is incorrect. Red man syndrome is
der.
an adverse effect attributed to vancomycin.
Answer B is incorrect. HHV-6 is the virus asso-
Answer D is incorrect. Renal toxicity can be
ciated with roseola infantum, which manifests
observed as an adverse effect of aminoglyco-
with a prodrome of an acute-onset high fever
sides and cyclosporine, among other drugs.
(>40C [104F]) preceding a maculopapular
rash that appears as the fever breaks. The rash
12. The correct answer is C. The American dog
is classically described as manifesting on the
tick is one of the vectors for RMSF. RMSF
trunk with a rapid spread to the face and ex-
classically has the rash described in the stem,
tremities. There are no associated oropharyn-
starting on the hands and feet and spreading to
geal lesions.
the trunk.

Infectious Disease
Answer C is incorrect. Paramyxovirus is re-
Answer A is incorrect. Also transmitted by the
sponsible for measles, which presents with a
dog tick, the presentation of ehrlichiosis is very
low-grade fever prodrome with Cough, Coryza,
similar to that of RMSF, as it is transmitted by
and Conjunctivitis (the 3 Cs) and later de-
a dog tick bite and symptoms include high fe-
velopment of an erythematous, maculopapular
ver and headache. It rarely has a rash, however.
rash that spreads from the head to the feet.
Answer B is incorrect. Lyme disease is trans-
Answer D is incorrect. Parvovirus B19 is as-
mitted by the deer tick or white-footed mouse
sociated with erythema infectiosum (fth dis-
tick and presents with fever, fatigue, and a ery-
ease), which manifests with a characteristic
thema chronicum migrans, a target-like rash.
slapped cheek rash in addition to a pruritic,
Consider it in persons who have been camping
erythematous, maculopapular rash that mani-
or otherwise outdoors in Connecticut, even if
they dont report a tick bite, as only a minority fests on the arms and spreads to the trunk and
of patients give a history of tick bite. legs.

Answer D is incorrect. There is no encephali- Answer E is incorrect. Varicella-zoster virus


tis in RMSF. Tick-borne encephalitis is a viral is responsible for causing varicella (chicken-
(Flavivirus) disease carried by ticks that mani- pox), characterized by the classical teardrop
fests as meningitis. There is no specic treat- vesicular rash with vesicles in multiple stages
ment for tick-borne encephalitis. of healing; the virus also causes zoster, which
manifests with a pruritic teardrop vesicular
Answer E is incorrect. Tularemia is usually rash distributed in a dermatomal fashion.
acquired by the handling of infected rabbits,
and from the bites of ticks and deeries. Initial 14. The correct answer is E. OM is an infection
presentation is an acute febrile illness that pro- of the middle ear. Children have a shorter,
gresses to pneumonia, but it may also lead to more horizontal Eustachian tube than adults,
pharyngitis, pleural effusion, hilar lymphade- which predisposes them to OM. The peak age
nopathy, and pneumonitis. range for OM is 618 months. The most com-
mon pathogens are Streptococcus pneumoniae,
13. The correct answer is A. This vignette is a clas- Haemophilus inuenzae, and Moraxella ca-
sic presentation of hand-foot-and-mouth dis- tarrhalis. OM can also occur with viral infec-
ease caused by coxsackie A virus. Patients typi- tions. It may present with ear pain, hearing
cally present with a mild illness that may or loss, and vertigo. On otoscopic examination,
may not have the presence of fever. The the tympanic membrane is erythematous due
oropharynx is inamed, with vesicles scattered to inammation and bulges due to uid in the
on the tongue, buccal mucosa, posterior phar- middle ear. Pneumatic otoscopy reveals an im-
ynx, gums, palate, and lips. Hands and ngers, mobile tympanic membrane because the mid-
feet, buttocks, and groin may also demonstrate dle ear is lled with uid instead of air. Swell-
maculopapular, vesicular, or pustular lesions. ing around the ear suggests that the mastoid is
HIGH-YIELD SYSTEMS
236 Section I: Organ Systems Answers

involved. Amoxicillin, a -lactam, is the initial will reveal branching hyphae. Treatment in-
antimicrobial treatment of choice in acute volves the application of topical antifungals.
OM. The usual course of treatment is 10 days.
Answer A is incorrect. Although contact derma-
Even when treated with the appropriate anti-
titis often presents as a localized pruritic rash, a
microbial therapy, uid can still persist in the
potassium hydroxide preparation of scrapings
patients ears for months. Other possible com-
from the lesion would not yield any ndings.
plications include perforation of the tympanic
membrane from pressure of the uid on the Answer B is incorrect. This presentation is not
membrane, acute mastoiditis, petrositis, and consistent with hand-foot-and-mouth disease,
labyrinthitis from contiguous spread of infec- which typically presents with constitutional
tion. symptoms and white-grey maculopapular ves-
icles with a characteristic peripheral distribu-
Answer A is incorrect. Observation is reserved
tion on the hands, feet, buttocks, and buccal
for patients who are >2 years old who have
mucosa/palate/tongue/tonsils (herpangina).
Infectious Disease

nonsevere illness and close follow-up. Antimi-


Coxsackie A virus is the most common caus-
crobial therapy may then be initiated if the pa-
ative agent and would not be visible on a po-
tient worsens or does not improve over 4872
tassium hydroxide preparation.
hours.
Answer C is incorrect. Scabies (Sarcoptes
Answer B is incorrect. Myringotomy, with
scabiei) is a mite that burrows in the skin and
placement of tympanostomy tubes to drain
causes characteristic pruritic lesions (small ery-
middle ear uid and prevent abscess forma-
thematous papules with excoriations and blood
tion, is reserved for treatment of chronic or re-
crusts). When the feet are affected, the scabies
current acute OM.
typically burrow on the lateral/posterior aspects
Answer C is incorrect. The patient is not toxic of the feet. The diagnosis can be made by iden-
appearing and does not have meningismus. tifying mites or eggs in scrapings from the le-
Her fever can be explained by the acute OM. sions. Treatment involves topical permethrin
It is unlikely that she has meningitis, sepsis, or cream.
a more serious infection that would require re-
Answer D is incorrect. This presentation is
ferral to a tertiary care facility.
not classic for Stevens-Johnson syndrome,
Answer D is incorrect. Signs of illness usually which typically presents as a diffuse, desqua-
resolve in 2448 hours with appropriate anti- mating rash (erythema multiforme) involving
microbial therapy. Amoxicillin-clavulanate is the mucous membranes and/or conjunctivae.
reserved for patients who fail to show improve- In most cases, Stevens-Johnson syndrome is
ment after 4872 hours on amoxicillin therapy drug induced, although infection (especially in
and still have symptoms such as persistent ear children) may also cause disease. A potassium
pain, fever, or bulging tympanic membrane. hydroxide preparation of scrapings from the le-
sion would not yield any ndings.
Answer F is incorrect. Erythromycin plus
sulsoxazole is a macrolide combination used
16. The correct answer is A. Oral acyclovir should
in patients who have had anaphylaxis or urti-
be considered for immunocompetent adults
caria with previous -lactam therapy.
and for children >12 years who have an in-
creased risk of severe varicella infections. Oral
15. The correct answer is E. Tinea pedis (athletes
acyclovir therapy in this population, beginning
foot) is a common cutaneous fungal infection
within 24 hours of symptom onset, has been
that is often acquired secondary to occlusive
shown to decrease the duration of lesions and
footwear. It is often transmitted in swimming
fever and diminish the symptoms and duration
pool and shower facility areas. Although rare in
of disease. Treatment with oral acyclovir 2572
younger children, it is common in preadoles-
hours after the rash appears has no effect on
cents and adolescents. The infection often af-
the course of the illness. Immunocompromised
fects the lateral toe webs and the subdigital
patients should receive intravenous acyclovir
crevices. A potassium hydroxide preparation
HIGH-YIELD SYSTEMS
Chapter 8: Infectious Disease Answers 237

therapy because of the possibility of life- treated PID, which subsequently ruptured. Pa-
threatening complications. tients who have a TOA will often be febrile,
nauseated, tachycardic, and complain of ab-
Answer B is incorrect. Famciclovir is a prod-
dominal and pelvic pain.
rug of penciclovir, which is a nucleoside ana-
log similar to acyclovir. Famciclovir has dem- Answer A is incorrect. Acute cholecystitis
onstrated efcacy in the treatment of herpes presents with fever and pain in the upper right
zoster, but it has not been extensively studied quadrant of the abdomen, especially after fatty
for use in primary varicella infection of healthy meals. However, this patient is presenting with
populations. lower abdominal pain and symptoms of septic
shock, which would be uncommon in chole-
Answer C is incorrect. Acyclovir-resistant
cystitis. Rupture of the gallbladder is possible
strains of varicella zoster virus have been re-
with cholecystitis, and is more common in
ported in patients with AIDS. Foscarnet, an
patients with diabetes, but has not usually oc-
inorganic pyrophosphate analog that is a selec-

Infectious Disease
curred at the time of initial development of
tive inhibitor of viral DNA polymerases and
cholecystitis symptoms.
reverse transcriptases, is a potentially efca-
cious drug in patients with acyclovir-resistant Answer B is incorrect. Diabetic ketoacido-
varicella zoster virus strains, since it is a DNA sis (DKA) occurs in diabetics who have inad-
chain inhibitor of phosphorylation. In this pa- equate insulin levels or after a precipitating
tient, acyclovir would be rst-line treatment. event such as an infection has led to a high-
catecholamine state. Patients present with hy-
Answer D is incorrect. Ganciclovir is used
perglycemia, polydipsia, polyuria, abdominal
primarily to treat cytomegalovirus (CMV), a
pain, and lethargy. Fever would be unlikely
member of the same family as varicella (Her-
to result purely from DKA; a fever could be
pesviridae). CMV, however, is usually a mild
caused by an underlying infectious precipitant
infection in immunocompetent individuals,
such as PID or TOA. Additionally, it would be
typically manifesting as a mononucleosis syn-
expected for a patient in DKA to have urine
drome. Like acyclovir, ganciclovir is a nucleo-
positive for ketone bodies. Thus, while a TOA
side analog that inhibits DNA synthesis. How-
would make this diabetic patient at risk for
ever, CMV does not contain thymidine kinase.
DKA, it is unlikely that DKA is the primary
Although ganciclovir has some activity against
cause of this patients illness.
varicella zoster virus, other nucleoside analogs
are preferred. Answer C is incorrect. Reinfection from an
untreated sexual partner is a signicant risk
Answer E is incorrect. Triuridine is a thymi-
after treatment for any sexually transmitted dis-
dine analog used topically to treat HSV infec-
ease (STD). However, this patient has symp-
tion. It works by inhibiting viral replication by
toms of vasodilatory shock, which would be in-
incorporating into viral DNA in place of thy-
consistent with gonorrheal cervicitis.
midine. However, the clinical presentation of
this patient is more consistent with varicella Answer D is incorrect. Ruptured ectopic preg-
zoster virus than HSV, and triuridine would nancy often presents with vaginal bleeding,
not be an appropriate therapy. abdominal pain, and hypotension. However,
this patient has a negative -human chorionic
17. The correct answer is E. This patient was re- gonadotropin test, indicating that she is not
cently treated with outpatient antibiotics for pregnant at this time. In addition, a ruptured
pelvic inammatory disease (PID). Given her ectopic pregnancy would present with hypo-
immunocompromised state secondary to her volemic shock and cool, clammy skin rather
diabetes mellitus, more aggressive treatment as than vasodilatory shock with warm skin.
an inpatient with intravenous antibiotics would
have been more appropriate. It is most proba- 18. The correct answer is D. The constellation of
ble that this patient developed a tubo-ovarian this patients symptoms, normal vaginal pH,
abscess (TOA) as a result of her inadequately and hyphae on potassium hydroxide prepara-
tion likely indicates that the patient has vaginal
HIGH-YIELD SYSTEMS
238 Section I: Organ Systems Answers

candidiasis, a fungus. Most antibiotics are not to be administered as soon as possible to the pa-
effective against fungus, which needs to be tients sister.
treated with antifungal medications. Because
Answer B is incorrect. The Haemophilus in-
this is presumably her rst infection by history,
uenzae type b vaccine is a standard childhood
she may be treated with a single 150-mg dose
immunization that children should receive.
of uconazole, the drug of choice for uncom-
However, it does not need to be administered
plicated vaginal candidiasis infections.
as soon as possible to the patients sister.
Answer A is incorrect. Azithromycin is a mac-
Answer C is incorrect. The patient has mea-
rolide antibiotic that inhibits RNA synthesis by
sles; therefore, the inuenza vaccine is not the
binding to the 50S ribosomal RNA subunit. It
vaccine that is needed as soon as possible.
is used to treat Chlamydia infections but is not
therapeutic for vaginal candidiasis. Answer E is incorrect. Varicella is not usually
associated with cough, coryza, conjunctivitis,
Answer B is incorrect. Ciprooxacin is a uo-
Infectious Disease

Kopliks spots, or high fever; thus, the varicella


roquinolone with a wide spectrum of antibiotic
vaccine is not indicated at this time.
activity. It is a rst-line agent for the treatment
of gonorrhea but is not effective against Can-
20. The correct answer is A. This patient suffers
dida.
from isoniazid-induced hepatitis, a complication
Answer C is incorrect. Clindamycin is a 50S that has varying rates in different population
ribosome inhibitor that is effective against groups but can be fatal. Treatment with isoniazid
gram-positive bacteria and many anaerobes. It more commonly causes mild, asymptomatic he-
is largely ineffective against gram-negative bac- patotoxicity characterized by minor elevations in
teria. It is used to treat bacterial vaginosis and transaminases (<100 U/L). Active intravenous
has no activity against Candida species. drug use (largely via viral hepatitis infection), as
well as chronic alcohol use, increases the risk of
Answer E is incorrect. Metronidazole is an an-
developing symptomatic isoniazid-induced hepa-
tibiotic effective in treating anaerobic bacteria
titis. Each of the other answer choices decreases
and some protozoa. It is used to treat bacterial
the risk for this complication. Also of note, isoni-
vaginosis and Trichomonas infections but is in-
azid inhibits the cytochrome P450 hepatic en-
effective against Candida.
zyme system, thus increasing the effective serum
concentrations of medications such as warfarin,
19. The correct answer is D. The child has mea-
phenytoin, theophylline, and carbamazepine.
sles, which present with a 3-day prodrome of
low-grade fever; Cough, Coryza, Conjunctivi- Answer B is incorrect. Older age is associated
tis (the 3 Cs); and Kopliks spots (small red with an increased rate of isoniazid-induced
spots with grey centers on the buccal mucosa), hepatitis.
which last for 1224 hours followed by an ery-
Answer C is incorrect. Previous intolerance of
thematous, blanching, maculopapular rash
isoniazid is associated with an increased rate of
that begins on the face and then descends over
isoniazid-induced hepatitis.
the rest of the body. Complications include
OM, pneumonia, laryngotracheitis, and rarely Answer D is incorrect. Female gender is as-
subacute sclerosing panencephalitis. Treat- sociated with an increased rate of isoniazid-
ment of measles is supportive and mainly con- induced hepatitis.
sists of antipyretics and hydration; however, Answer E is incorrect. African-American
unimmunized contacts should receive the people experience increased rates of isoniazid-
measles-mumps-rubella vaccine to prevent the induced hepatitis.
spread of the disease.
Answer A is incorrect. The diphtheria-tetanus- 21. The correct answer is B. The most serious
pertussis vaccine is an important standard child- complication of lumbar puncture is cerebral
hood immunization. However, it does not need
HIGH-YIELD SYSTEMS
Chapter 8: Infectious Disease Answers 239

herniation. In patients with increased risk fac- dia ink is highly suggestive of cryptococcal dis-
tors for herniation, cranial imaging in the form ease, denitive diagnosis is based on culture of
of a CT scan should be performed prior to organisms from CSF.
lumbar puncture. These include patients sus-
Answer E is incorrect. A CT scan would be
pected to have increased cranial pressure (focal
the rst test in this patient, but further analy-
neurologic decits or papilledema), patients
sis of CSF from a lumbar puncture would be
with impaired cellular immunity, patients with
helpful in discerning the etiology of the pa-
altered mental status, and patients with recent
tients symptoms. Analysis of cells, protein, and
seizures. MRI is another imaging option, but
glucose in CSF can narrow down the differ-
CT is most often performed due to the acuity
ential. The presence of WBCs is not specic
of these situations.
for infection, but the absence of WBCs makes
Answer A is incorrect. Thrombolytic therapy infection an unlikely diagnosis. The presence
(such as tissue plasminogen activator [t-PA]), of RBCs could indicate subarachnoid hemor-

Infectious Disease
if given within 3 hours of an ischemic stroke, rhage (or a traumatic tap). A low CSF glucose
greatly improves functional outcome. The pa- supports the diagnosis of bacterial meningitis
tients symptoms developed well over 3 hours over viral meningitis. A high CSF protein sug-
ago. While all the patients symptoms could gests bacterial meningitis over viral meningitis.
indicate a cerebrovascular accident, the time Finally, cytology can be sent if malignancy is
course is not consistent with this diagnosis. suspected.
Embolic and hemorrhagic stroke symptoms
occur acutely, while ischemic stroke symp- 22. The correct answer is C. The documented de-
toms may occur over the course of a few hours crease in life-threatening childhood infections
or even a few days. However, stroke symptoms is ample justication for routine immuniza-
rarely develop over the course of weeks. Fur- tions. Several large studies have failed to nd
thermore, t-PA should not be administered evidence that autism is increased in immu-
without cranial imaging to exclude hemor- nized children. The physician must ensure that
rhagic stroke. t-PA is strictly contraindicated for a parent is well informed about the benets of
hemorrhagic strokes. immunization.
Answer C is incorrect. Urgent lumbar punc- Answer A is incorrect. Although it may seem
ture and culture would be the test of choice for that the physician is acting in the best interest
diagnosing bacterial, fungal, mycobacterial, or of the child, it is the parents right to withhold
viral meningitis and is warranted in this immu- treatment that is not necessary in the immedi-
nocompromised patient, but lumbar puncture ate term to keep the child alive. Acting without
should await cranial imaging in patients with the parents consent is extremely unethical.
risk factors for herniation. Though this de-
Answer B is incorrect. This is not an appropri-
lays the lumbar puncture, medical treatment
ate or effective response.
should not be withheld in cases with a high in-
dex of suspicion for infectious etiology. Answer D is incorrect. The court will not au-
thorize forced immunization. Parents have the
Answer D is incorrect. Urgent lumbar punc-
right to withhold treatment that is not neces-
ture and India ink stain would be the quickest
sary in the immediate term to keep the child
method for evaluating a patient for cryptococ-
alive.
cal meningoencephalitis. It is most common
in patients with AIDS with CD4+ cell counts Answer E is incorrect. Refusal to vaccinate a
<100/mm. Symptoms (fever, malaise, and child does not fall under the category of child
headache) are indolent and begin over the endangerment.
course of 12 weeks, consistent with this pa-
tients presentation. However, given his im- 23. The correct answer is E. This patient presents
munocompromised state, a CT scan should be with a history and physical examination nd-
done prior to lumbar puncture to reduce the ings classic for infectious mononucleosis
chances of herniation. While staining with In- caused by the Epstein-Barr virus (EBV). EBV
HIGH-YIELD SYSTEMS
240 Section I: Organ Systems Answers

may remain in saliva for months after the dis- cheeks. The rest of the body, especially the ex-
ease is contracted, and a high percentage tensor surfaces, develops a macular rash with
(25%50%) of patients exposed to EBV may central clearing giving it a lacy appearance. In
carry the virus without experiencing clinically immunocompromised children (such as those
signicant disease. The monospot test used with AIDS) parvovirus B19 has been linked to
in most clinics is a latex agglutination assay, causing aplastic anemia.
and is both sensitive and specic. Treatment is
Answer B is incorrect. Endemic Burkitts lym-
essentially supportive, using acetaminophen
phoma is the most common childhood cancer
and nonsteroidal anti-inammatory drugs to
in several parts of the world including East Af-
treat fever and pain. Patients with splenomeg-
rica and New Guinea. It often involves the jaw
aly should avoid contact sports due to the risk
and is characterized by a chromosomal translo-
of splenic rupture.
cation of the c-myc proto-oncogene that leads
Answer A is incorrect. The patients history to constitutive transcription of the gene. EBV
Infectious Disease

and examination point to mononucleosis as a has been associated with this disease, not par-
cause of her splenomegaly and dull abdominal vovirus B19.
pain. If her latex agglutination assay was nega-
Answer C is incorrect. Orchitis can be a com-
tive and her pain increased in severity or per-
plication of mumps virus or coxsackie B virus
sisted without explanation, an imaging study
infection. Mumps virus is the most common
would be reasonable.
cause of orchitis, which is rare in prepubescent
Answer B is incorrect. Infectious mononucle- males, but can occur in 14%35% of adoles-
osis is a common and plausible t for this pa- cents and adults. Infertility is rare even when
tients presentation. Opening her abdomen in both testes are involved.
search of the cause of her dull upper abdomi-
Answer D is incorrect. Progressive multifocal
nal pain would be hasty, and without an im-
leukoencephalopathy is a progressive disease
aging study in this non-emergent case, would
leading to demyelination of central nervous
probably constitute malpractice.
system neurons as the oligodendroglial cells
Answer C is incorrect. Fine-needle aspiration are destroyed. It is caused by the JC virus, not
is a relatively noninvasive technique for diag- the parvovirus B19 virus.
nosing diseased tissue, most often malignan-
Answer E is incorrect. Subacute sclerosing pa-
cies. It would be overly aggressive in this pa-
nencephalitis is a chronic encephalitis caused
tient whose clinical picture is more consistent
by persistent measles virus infection. The on-
with an acute infection than with a developing
set is usually marked by subtle changes in be-
cancer.
havior, but as the disease progresses it leads
Answer D is incorrect. Flow cytometry is use- to frank dementia. Other neurologic signs
ful in diagnosing diseases in which specic cell develop including repetitive myoclonic jerks.
identity needs to be determined, e.g., lympho- Most patients survive 13 years after diagnosis,
mas, leukemias, and various other malignan- with a mean survival of 18 months.
cies. It has no role in the diagnosis of mono-
nucleosis, and would be a long way down the 25. The correct answer is D. The average risk of
road in this patient with a plausible (and inex- seroconversion following a needlestick injury is
pensively-diagnosed) etiology. about 3 per 1000 (<1%) without prophylaxis,
and is reduced further, by 80%, when postex-
24. The correct answer is A. This child has ery- posure prophylaxis is administered in a timely
thema infectiosum or fth disease, which is fashion. Considerations in determining high-
caused by the B19 parvovirus. The prodromal risk versus low-risk situations include viral load
phase of the disease is mild and consists of (>50,000/mm), advanced AIDS, or prior use
headache, low-grade fever, and sometimes a of antiretroviral agents. Recommended man-
mild upper respiratory illness. The characteris- agement of a high-risk exposure includes two
tic rash follows and includes an erythematous nucleoside reverse transcriptase inhibitors
facial ushing described as resembling slapped (NRTIs) plus a protease inhibitor, while a low-
HIGH-YIELD SYSTEMS
Chapter 8: Infectious Disease Answers 241

risk exposure involves prophylaxis with the Answer C is incorrect. Fluconazole is an anti-
NRTIs alone. HIV serology should be tested at fungal used for mild fungal disease or menin-
baseline, 6 weeks, 12 weeks, and 6 months fol- gitis. It can also be used for maintenance ther-
lowing the exposure. apy. It acts through inhibition of the ergosterol
synthesis pathway, as do all of the azoles.
Answer A is incorrect. Baseline HIV status
should be obtained with an initial blood draw Answer D is incorrect. Metronidazole is an
immediately following HIV exposure but be- antiprotozoal drug commonly used against
fore beginning drug therapy. agellated protozoans. It works well against
anaerobic bacteria, among others; however,
Answer B is incorrect. An initial blood draw
trimethoprim-sulfamethoxazole is the rst-line
should be obtained to establish an HIV base-
treatment for P. jiroveci pneumonia. Metro-
line before beginning postexposure drug ther-
nidazole acts by disrupting the helical structure
apy. Subsequent serology testing should occur
of DNA, causing it to fragment. It is metabo-
at 6 weeks, 12 weeks, and 6 months.

Infectious Disease
lized into an active form by the enzyme ferre-
Answer C is incorrect. The 2005 Centers for doxin in bacteria.
Disease Control and Prevention guidelines for
Answer E is incorrect. Nystatin swish and
management of HIV exposure recommends
swallow is an antifungal predominantly used
antiretroviral therapy with two NRTIs for a
for oral candidal infections that appear as
low-risk exposure (i.e., HIV status unknown or
white plaques and occur primarily in the im-
low viral load), and the addition of a protease
munocompromised population. Nystatin acts
inhibitor for higher-risk exposures.
by altering cell membrane permeability.
Answer E is incorrect. Postexposure prophy-
Answer F is incorrect. Pyrazinamide is used
laxis should be started immediately with a goal
for treatment of tuberculosis (TB) in com-
to start within a couple of hours following ex-
bination with isoniazid and rifampin. It acts
posure to achieve the minimum risk of sero-
through inhibition of fatty acid synthase I. TB
conversion.
classically presents as a prolonged cough as-
sociated with hemoptysis, fever, rigors, night
26. The correct answer is G. The most likely diagno-
sweats, and weight loss. TB usually occurs in
sis is Pneumocystis jiroveci pneumonia. P. jiroveci
a population with a history of exposure (i.e.,
is presently classied as a fungus. This condition
those who live in endemic areas).
occurs almost exclusively in the immunocompro-
mised (primarily AIDS) population. The classic
27. The correct answer is E. This patient is suffer-
triad of nonproductive cough, fever, and exer-
ing from the effects of toxic shock syndrome
tional dyspnea is seen in about 50% of cases. Di-
(TSS), which is most often caused by Staphy-
agnosis is usually through imaging, either x-ray or
lococcus aureus or Group A streptococci. Fac-
CT of the chest when x-ray is inconclusive. The
tors that increase the risk of TSS include high-
rst-line chemotherapeutic and chemoprophy-
absorbency tampons and using a single tampon
lactic agent is trimethoprim-sulfamethoxazole.
for multiple days. Nonmenstrual causes of TSS
Answer A is incorrect. Amphotericin B is a include a variety of infections, as well as both vag-
potent antifungal with serious adverse effects inal and cesarean childbirth. Symptoms of TSS
that is used in the treatment of severe fungal include high fevers, headaches, gastrointestinal
disease. Amphotericin B functions by binding distress, neurologic symptoms, hypotension, and
to membrane sterols in fungal cells, disrupting erythematous macular rash. Treatment is sup-
the membrane, and causing lysis. portive, with admission to an intensive care unit,
removal of any foreign material such as tampons,
Answer B is incorrect. P. jiroveci pneumonia
and pressure support with uids and vasopressors
is caused by what is presently classied as a
as needed. There is no rm evidence supporting
fungus. Ceftriaxone is a -lactam class drug for
the use of antibiotics, IVIG, or corticosteroids in
use against bacterial infections.
TSS, although these treatment measures are of-
ten employed.
HIGH-YIELD SYSTEMS
242 Section I: Organ Systems Answers

Answer A is incorrect. Although this patient is is not the mechanism behind aminoglycoside
clearly at risk for alcohol poisoning as a result nephrotoxicity.
of binge drinking, her symptoms are worsen-
Answer D is incorrect. This choice describes
ing after several hours of abstinence from al-
the pathophysiology of membranoproliferative
cohol use. Although alcohol could account for
glomerulonephritis, which is typically idio-
her change in mental status, it would not be
pathic, autoimmune, or infectious in etiology.
expected to cause a febrile illness or a diffuse
erythematous rash. Answer E is incorrect. This choice describes
another prerenal etiology that would result in a
Answer B is incorrect. Heatstroke is a condi-
fractional excretion of sodium <1%. Renal ar-
tion in which the bodys core temperature rises
terial and arteriolar vasoconstriction occurs in
above 40.6C (105F), with associated neuro-
hepatorenal syndrome and with medications
logic dysfunction. Other ndings in heatstroke
such as nonsteroidal anti-inammatories, but
include cutaneous vasodilation, disseminated
not with the use of aminoglycosides.
Infectious Disease

intravascular coagulation, rhabdomyolysis,


and seizures. Macular rash is not expected.
29. The correct answer is E. This childs clinical
Although this patient is febrile, her core body
presentation is suspicious for congenital HSV
temperature does not meet the requirements
infection, a diagnosis conrmed by the giant
for heatstroke.
cells with multiple nuclei visualized on the
Answer C is incorrect. Neisseria meningococ- positive Tzanck test shown in the image. Al-
cemia is another febrile illness that can rap- though varicella may cause a similar Tzanck
idly progress to shock. However, it is usually smear, the clinical presentation is more sugges-
accompanied by meningitis and meningeal tive of herpetic lesions. Furthermore, the treat-
signs such as photophobia and a stiff neck. In ment would be the same for a severe varicella
addition, the rash of meningococcemia is pete- infection. The treatment of choice for HSV in-
chial, not diffuse and macular. fection is acyclovir, which has the unfortunate
quality of crystallizing within the nephron dur-
Answer D is incorrect. RMSF is another fe-
ing the process of renal excretion. Acyclovir is
brile illness with a rapid course. However, in
poorly soluble in urine, and crystalluria devel-
RMSF, the rash typically appears several days
ops in conditions of low urine ow; this leads
after the onset of the fever. In addition, the rash
to azotemia, renal tubular obstruction, and ul-
is petechial and begins at the distal extremities
timately renal failure. To avoid this, administer
before moving centripetally.
acyclovir in relatively low, long-term doses and
avoid rapid boluses. Maintaining adequate hy-
28. The correct answer is C. This patient suffers
dration and a high glomerular ltration rate
from aminoglycoside nephrotoxicity secondary
are helpful in avoiding this adverse event. The
to treatment with gentamicin. Nephrotoxicity
prognosis for localized congenital HSV infec-
from aminoglycosides most commonly mani-
tion is excellent, especially if treated early and
fests as acute tubular necrosis, with toxic dam-
appropriately. Mortality is high for infants with
age and death of the proximal tubule cells, re-
disseminated disease.
sulting in exposed gaps and exposure of the
basement membrane. Answer A is incorrect. Agranulocytosis is not
a reported adverse effect of acyclovir, which
Answer A is incorrect. This choice describes a
is more likely to cause renal failure or neuro-
prerenal etiology of renal failure, which would
toxicity. Agranulocytosis is an expected effect
result in a fractional excretion of sodium <1%.
of cytotoxic agents such as cyclophosphamide,
Answer B is incorrect. This choice describes and a frightening adverse reaction to numer-
the pathology of several renal diseases, includ- ous agents, such as clozapine, sulfasalazine,
ing rapidly progressive glomerulonephritis and ticlopidine, and thionamide-class antithyroid
post-streptococcal glomerulonephritis, but it medications.
HIGH-YIELD SYSTEMS
Chapter 8: Infectious Disease Answers 243

Answer B is incorrect. Fulminant liver fail- nopathy and fever, with signicant numbers
ure is not a reported adverse effect of acyclo- of patients developing ulcers/eschars/papules,
vir, which is more likely to cause renal failure pharyngitis, myalgias, nausea/vomiting, and
or neurotoxicity. This reects the fact that the hepatosplenomegaly. F. tularensis may be
drug is cleared renally, not hepatically. Fulmi- transmitted by cat scratch or bite, although it
nant hepatic failure is an adverse reaction as- is classically transmitted by handling rabbits.
sociated with acetaminophen (most common), However, overall, tularemia is less common
in addition to amiodarone, halothane, isoni- than cat scratch disease, so F. tularensis is less
azid, ketoconazole, phenytoin, rifampin, and likely to be responsible for this infection. Sero-
valproate. logic testing would be useful to rule out tulare-
mia.
Answer C is incorrect. Osteopenia is not a
reported adverse effect of acyclovir, which is Answer E is incorrect. Staphylococcus aureus
more likely to cause renal failure or neurotoxic- is responsible for large numbers of osteomyeli-

Infectious Disease
ity. Bone loss has been attributed (notoriously) tis and cellulitis cases, in addition to other in-
to glucocorticoids, in addition to heparin, cy- fectious illnesses. However, it is not implicated
closporine, warfarin, and synthetic thyroid hor- commonly in cases of lymphadenopathy.
mone.
31. The correct answer is B. Rifampin is used in
Answer D is incorrect. Pulmonary edema
combination with isoniazid (for 6 months) and
is not a reported adverse effect of acyclovir,
pyrazinamide (for the rst 2 months) for treat-
which is more likely to cause renal failure or
ing TB. The multidrug approach is used to
neurotoxicity. Noncardiogenic pulmonary
minimize the likelihood of development of
edema has been attributed to opiates and sali-
secondary drug resistance during therapy. Ad-
cylates.
verse effects of rifampin include orange discol-
oration of the urine and tears, gastrointestinal
30. The correct answer is A. The patient described
disturbances, and hepatotoxicity, which often
has cat scratch disease, caused by Bartonella
manifests as an asymptomatic elevation in
henselae. Patients typically present after an in-
transaminase levels.
cubation period of 712 days with one or more
red papules at the site of inoculation. Lymph- Answer A is incorrect. Angioedema is not a
adenopathy often manifests after a period of recognized adverse reaction to rifampin.
14 weeks. In addition to the clinical ndings,
Answer C is incorrect. Hyperuricemia is an
the patients exposure to neighborhood cats
adverse effect that is well recognized in adults
should provide a hint to the diagnosis.
taking pyrazinamide. In such patients, the in-
Answer B is incorrect. Brucella species are re- creased uric acid levels often manifest as gout.
sponsible for a variety of zoonotic diseases that Children taking pyrazinamide who develop
cause a classic triad of fever, arthritis/arthralgia, hyperuricemia are often asymptomatic. Other
and hepatosplenomegaly. They can be acquired adverse effects of pyrazinamide include arth-
from cattle, goats, sheep, swine, and dogs. This ralgias and arthritis.
patient, however, has no such exposure.
Answer D is incorrect. Peripheral neuritis is a
Answer C is incorrect. Clostridium botulinum principal toxic effect of isoniazid. Most com-
is responsible for botulism, which results in a monly, it manifests as tingling or numbness in
symmetric, descending, accid paralysis that the hands or feet. Vitamin B6 (pyridoxine) is
can be life threatening if not diagnosed and often administered concurrently to prevent the
treated promptly. Lymphadenopathy is not symptoms of peripheral neuropathy. It is im-
typical. portant to mention that hepatotoxicity is also a
common toxic effect of isoniazid.
Answer D is incorrect. Francisella tularensis is
the microorganism responsible for tularemia. Answer E is incorrect. Stevens-Johnson syn-
This disease often manifests with lymphade- drome often occurs in the setting of begin-
ning a new medication and results in a severe,
HIGH-YIELD SYSTEMS
244 Section I: Organ Systems Answers

desquamating rash (erythema multiforme) that of PID to be much lower on the differential.
involves the mucous membranes and/or con-
Answer B is incorrect. Diagnostic laparoscopy
junctiva. Although clinical responses to vari-
is useful in establishing a denitive diagnosis
ous drugs will differ and some patients may de-
of PID. However, laparoscopy is specic but
velop a mild rash or experience pruritus while
not sensitive for the diagnosis of PID. Laparos-
taking rifampin, these are uncommon adverse
copy is usually indicated in patients who re-
effects and rarely progress to the severity of Ste-
main symptomatic after treatment for PID or
vens-Johnson syndrome. However, as with the
in whom another diagnosis, such as appendici-
initiation of any new medications, any change
tis, is likely.
in the patients condition, including the devel-
opment of a rash, should be monitored and re- Answer C is incorrect. DNA probes for Chla-
ported to the physician. mydia trachomatis and Neisseria gonorrhoeae
are useful as a conrmatory test for PID. How-
32. The correct answer is C. In this untreated ever, empiric therapy is often started before the
Infectious Disease

HIV-positive patient, it is important to rule out results of these tests are available, and therefore
CMV retinitis, as it can lead to blindness if left they are rarely useful in guiding therapy.
untreated. A polymerase chain reaction assay is
Answer D is incorrect. Gram stain of cervical
the test of choice, as serologic tests could be
discharge is useful only when positive for Neis-
unreliable in this patient. An ophthalmologic
seria gonorrhoeae, which makes a diagnosis of
examination would reveal a pizza-pie pattern
PID signicantly more likely. A negative test is
retinopathy with exudates and perivascular
not useful in ruling out PID.
hemorrhage. Therapy for CMV retinitis con-
sists of oral valganciclovir, intravenous ganci- Answer E is incorrect. WBC counts are of lim-
clovir, or intravenous foscarnet, with cidofovir ited utility in the diagnosis of PID, and would
as an alternative. not be expected to change the management.
Fewer than half of patients with PID have an
Answer A is incorrect. Blepharitis is infection
increased WBC count.
of the eyelids, characterized by swelling, red-
ness, and dry crusting, but it is not typically
34. The correct answer is C. The disease is bacil-
painful. Instead, it often presents with a foreign
lary angiomatosis, caused by Bartonella hense-
body sensation. Also, if left untreated, it will
lae infection. It is caused by vascular prolifera-
not lead to vision loss.
tion and neutrophilic inammatory response
Answer B is incorrect. Conjunctivitis is char- to the bacteria and can look similar to Kaposis
acterized by discharge, irritation (but not frank sarcoma (KS). Biopsy of the lesions often shows
pain), and itching. tiny clumps of bacilli on Warthin-Starry silver
stain, which are diagnostic. It can be treated
Answer D is incorrect. Open-angle glaucoma
with antibiotics; erythromycin and doxycycline
is not an ophthalmologic emergency; acute
are effective. For cutaneous bacillary angioma-
closed-angle glaucoma is an emergency. Open-
tosis, 24 months of therapy is recommended;
angle glaucoma has no symptoms in the early
if there are other manifestations, including os-
stages with a gradual loss of peripheral vision
teomyelitis, splenitis, or endocarditis, therapy
over a period of years resulting in tunnel vi-
should be extended for 46 months.
sion.
Answer A is incorrect. Acyclovir is used to
33. The correct answer is A. This patient likely treat herpes zoster reactivation, commonly
has PID. When pregnant, patients with PID known as shingles, in HIV-positive patients.
are managed as inpatients. Empiric outpatient These lesions are not typical of shingles in that
therapy for PID is recommended only when they cross the midline and occupy more than
physical examination and risk factors are con- one dermatome.
sistent with PID and a pregnancy test is nega-
Answer B is incorrect. Corticosteroids are not
tive. PID is rare in pregnancy, and thus a posi-
useful in Bartonella infection, which is caus-
tive pregnancy test would cause the likelihood
HIGH-YIELD SYSTEMS
Chapter 8: Infectious Disease Answers 245

ing this patients bacillary angiomatosis. Also Answer E is incorrect. Histoplasmosis is not
steroids should not be given to patients with common in Baltimore, so avoiding travel to
compromised immune systems. Baltimore will not reduce the risk of infection.
Answer D is incorrect. Ganciclovir is an anti-
36. The correct answer is E. Argyll Robertson pu-
viral medication effective against both herpes
pil in the presence of acute mania is suggestive
viruses and CMV. CMV commonly causes oc-
of neurosyphilis in this patient. Other central
ular disease in immunocompromised patients,
nervous system manifestations, such as gait ab-
and rarely presents with only cutaneous mani-
normalities (i.e., tabes dorsalis) are now rare. A
festations.
remote history of sexually transmitted disease
Answer E is incorrect. Highly active antiretro- and a solitary presentation of a manic episode
viral therapy should be considered in eligible without a prior psychiatric history are also im-
patients with CD4+ cell counts <350/mm, portant in this case. Often patients may have
and treatment with antiretroviral drugs can been infected years earlier yet do not present

Infectious Disease
prevent bacillary angiomatosis. To treat bacil- to the physician until this stage of infection.
lary angiomatosis, however, antibiotic therapy Conrmation can be done with a Venereal
is needed. Disease Research Laboratory test on the pa-
tients CSF.
35. The correct answer is D. Exposure to bird and
Answer A is incorrect. Patients with Chla-
bat droppings or soil contaminated with bird or
mydia trachomatis infections usually present
bat droppings is a well known cause of histoplas-
with urethritis, epididymitis, and urethral dis-
mosis. Histoplasmosis is also more common in
charge.
the vast areas that drain into the Mississippi
River. Histoplasmosis is an indolent fungal in- Answer B is incorrect. HSV can cause altered
fection, and the acute lung form of the disease mental status, seizures, and meningitis in
usually presents with chest pain, dyspnea, and young children but is unlikely in an immune-
dry cough. The chronic form of the disease usu- competent adult host.
ally occurs in those who already have lung dis-
Answer C is incorrect. Neisseria gonorrhoeae
ease and may present similarly to TB. Symp-
may cause monarticular arthritis and genital
toms occur within 10 days of exposure.
symptoms, but is unlikely to result in acute
Immunocompromised patients may develop a
psychiatric symptoms such as in this patient.
potentially fatal disseminated form of the dis-
ease. Treatment is with ketoconazole or voricon- Answer D is incorrect. Toxoplasmosis is usu-
azole. In disseminated disease, particularly men- ally transmitted by contact with cat feces and
ingitis, patients may be treated with intravenous is also unusual in an immunocompetent host.
amphotericin followed by long-term suppression It is unlikely to result in this patients clinical
with itraconazole. ndings.
Answer A is incorrect. Avoiding cigarette
37. The correct answer is C. The most common
smoke will not reduce the risk of histoplasmo-
medication allergy is to penicillin. However,
sis infection.
although 10% of patients report a penicillin al-
Answer B is incorrect. Avoiding corn elds lergy, the vast majority of those are able to tol-
will not reduce the risk of histoplasmosis infec- erate it. In some cases no allergy ever truly ex-
tion. isted, and the rash or other reaction was
actually due to the underlying disease. In addi-
Answer C is incorrect. Avoiding swimming in
tion, there is a tendency for patients with real
public pools would not reduce the risk of his-
type I allergies to lose penicillin-specic IgE
toplasmosis infection. There is no person-to-
antibodies over time such that an allergy can
person transmission of the infecting agent, His-
disappear with age. Skin testing can help deter-
toplasma capsulatum.
mine whether a true allergy exists. Cepha-
HIGH-YIELD SYSTEMS
246 Section I: Organ Systems Answers

losporins share with penicillin a common if therapy with these medications is necessary
-lactam ring as well as R-group side chains in in a penicillin-allergic patient.
some cases. Otherwise, they have a different
Answer D is incorrect. Penicillin-specic IgE
chemical structure and are less allergenic.
antibodies tend to be lost over time. For this
Crossreactivity between penicillins and cepha-
reason, patients with a history of a penicillin al-
losporins occurs in 3%8% of patients, and it is
lergy in childhood may outgrow their allergy.
recommended that patients who require ceph-
Penicillin skin testing may be performed to
alosporin administration undergo penicillin
conrm the presence of an allergy.
skin testing (no standardized skin test reagents
are available for cephalosporins). Patients with Answer E is incorrect. Although the precise
conrmed penicillin allergies who need cepha- incidence of penicillin-carbapenem crossreac-
losporin therapy should be given an alternate tivity is unknown, it is not insignicant. In a
drug or a graded challenge to the cephalosporin, patient with a skin-test-proven penicillin allergy
or should be desensitized. Aztreonam, an antibi- who requires carbapenem therapy, desensitiza-
Infectious Disease

otic from the monobactam family that contains tion or graded challenge to carbapenems may
a monocyclic ring structure, is less immuno- be attempted.
genic than penicillin and the cephalosporins Answer F is incorrect. Patients with HIV infec-
and has no crossreactivity with those drugs, ex- tion are at a greatly increased risk of cutaneous
cept in the case of ceftazidime, with which it reactions when exposed to sulfonamide antibi-
shares an R-group side chain. Thus, penicillin- otics such as trimethoprim-sulfamethoxazole.
or cephalosporin-allergic patients (except those The incidence of skin reaction to trimetho-
allergic to ceftazidime) can receive aztreonam prim-sulfamethoxazole in uninfected individu-
safely, and vice versa. Carbapenem antibiotics als is only 3.3%, but is 40%80% in patients
such as imipenem have crossreactivity with pen- with HIV infection.
icillin, but good clinical data are not available
on the subject. Because the incidence of cross- 38. The correct answer is E. TSS often presents as
reactivity may be signicant, penicillin skin test- a diffuse erythema involving the palms followed
ing is recommended. As with cephalosporins, by desquamation after 710 days with fever and
desensitization or graded challenge should be hypotension. The pathogen cultured is Staphy-
used in penicillin-allergic patients who require lococcus aureus, which releases a toxin, TSST-1.
carbapenem therapy. Fever >38.9C (102F), hypotension, and mul-
Answer A is incorrect. Cross-reactivity be- tiorgan dysfunction are common. TSS is often
tween penicillins and cephalosporins is ap- caused by a tampon placed for a long period in
proximately 3%8%. Patients with a history of a menstruating woman. However, it can also be
anaphylaxis to penicillin should not generally due to wound or skin infections. The treatment
be given a cephalosporin without further test- of choice is penicillin or oxacillin plus clin-
ing to conrm the presence or absence of a damycin. The withdrawal of highly absorbed
penicillin allergy, and should not receive ce- tampons from the market partially explains the
phalosporins except as part of a careful attempt decrease in menstrual-related cases.
to desensitize or expose the patient to a graded Answer A is incorrect. Lyme disease presents
challenge. A history of anaphylaxis to penicillin with a papule that expands to an annular le-
in and of itself does not necessarily preclude sion with a central clearing known as erythema
the use of cephalosporins in a given patient. migrans. Usually the rash is present at the belt
Answer B is incorrect. Penicillin has cross- line or axilla. Clinical features include initial
reactivity with cephalosporin and carbapenem viral-like syndrome followed by cardiac and
antibiotics that may make them unsafe to use neurologic symptoms without appropriate
in patients with proven penicillin allergies. De- treatment. Polyarticular arthritis may develop
sensitization or graded challenge can be used
HIGH-YIELD SYSTEMS
Chapter 8: Infectious Disease Answers 247

weeks to months later. First-line treatment is Answer C is incorrect. Mallory-Weiss tear is


doxycycline. Amoxicillin may also be used. a condition producing hematemesis follow-
The etiologic agent is Borrelia burgdorferi. ing esophageal mucosal tear. It is predisposed
by vomiting, coughing, and retching. It is cer-
Answer B is incorrect. Measles presents as dis-
tainly in the differential for esophagitis, but it
crete lesions that become conuent as the rash
is not the correct answer here.
proceeds from the hairline down, but spares
the palm and soles. Cough, conjunctivitis, co- Answer D is incorrect. Candidiasis does not
ryza, and Kopliks spots are visible. Treatment infect the esophagus in immunocompetent
includes supportive care and antibiotics for hosts.
otitis and pneumonia. The etiologic agent is
Answer E is incorrect. Severe combined im-
paramyxovirus.
munodeciency syndrome (SCID) is a condi-
Answer C is incorrect. Rheumatic fever pre- tion in which children have a total deciency
sents as nonpruritic, erythematous papules of T and B lymphocytes, resulting in immu-

Infectious Disease
occurring in polycyclic waves over the truck nocompromise from birth. This patient could
and sparing the face. Clinical features include have SCID; however, this condition is less
streptococcal pharyngitis, migratory polyarthri- likely than HIV and would have affected the
tis, carditis, rheumatic heart disease, chorea, patient from birth.
and subcutaneous nodules. Treatment in-
Answer F is incorrect. Candidal esophagitis
cludes anti-inammatory agents (aspirin), cor-
and vaginal candidiasis are separate entities.
ticosteroids for carditis if present, and penicillin
The former is seen only in immunocompro-
for pharyngitis. The etiologic agent is Group A
mised hosts, while the latter can be seen in
streptococcus.
healthy females.
Answer D is incorrect. RMSF presents as a
rash on wrists and ankles and spreads centrip- 40. The correct answer is E. Bacterial tracheitis is
etally. It appears on the palms and soles in a an acute bacterial infection of the upper airway
later stage. Headache, myalgias, and abdomi- that can cause a potentially fatal airway ob-
nal pain are common with a 40% mortality struction. The condition affects patients who
rate if left untreated. First-line treatment is are generally <3 years old and is often seen in
doxycycline followed by tetracycline as the the setting of a recent viral respiratory infec-
second-line treatment. The etiologic agent is tion. Bacterial tracheitis is characterized by a
Rickettsia rickettsii. brassy cough with high fever and possible re-
spiratory distress. Treatments aimed at croup
39. The correct answer is B. Oral candidiasis most (mist and racemic epinephrine) are ineffective
often presents as white patches in the oral mu- against bacterial tracheitis. The most common
cosa. Candidiasis is present only in the immu- pathogen responsible for bacterial tracheitis is
nocompromised, which would make HIV the Staphylococcus aureus. The image shows gram-
most likely comorbidity in this case. Treatment positive cocci in clusters and is consistent with
is usually simple nystatin swish and swallow. Staphylococcus aureus infection.
Prognosis is excellent and candidal thrush usu-
Answer A is incorrect. Anaerobic organisms
ally resolves without complications as long as
are grouped with Moraxella catarrhalis and
the condition is appropriately diagnosed and
nontypeable Haemophilus inuenzae as signi-
treated.
cant contributors to cases of bacterial trache-
Answer A is incorrect. Candidal esophagitis itis, but all three pathogens are less common
is not associated with cardiac chest pain. Both than Staphylococcus aureus.
may present with chest pain, but the history
Answer B is incorrect. Moraxella catarrhalis is
states that the patients chest pain is retroster-
a pathogen commonly associated with bacterial
nal and worse with swallowing, suggesting
tracheitis but is not as common of a cause of
esophagitis. No other symptoms are suggestive
the condition as S. aureus. Gram stain of M. ca-
of cardiac chest pain.
tarrhalis would show gram-negative diplococci.
HIGH-YIELD SYSTEMS
248 Section I: Organ Systems Answers

Answer C is incorrect. Nontypeable Haemo- caused by a defect in neutrophil chemotaxis


philus inuenzae has been associated with bac- resulting from mutations in the LYST gene
terial tracheitis cases but trails behind Staphy- required for sorting of lysosomal granule con-
lococcus aureus as the most common cause of tents. Patients have albinism due to abnormal
the condition. H. inuenzae is a gram-negative melanosomes and inappropriate delivery of
rod. melanin in the skin.
Answer D is incorrect. Pseudomonas aerugi- Answer D is incorrect. Hyper IgE syndrome is
nosa is largely implicated in ventilator-associ- characterized by recurrent bacterial infection
ated pneumonias, central nervous system in- and cavitating pneumonia. It is typically associ-
fections, osteomyelitis/septic arthritis, urinary ated with impaired inammation giving rise to
tract infections, and primary and secondary so-called cold abscesses. Although abscesses
skin infections. However, it is not a common are found in hyper IgE syndrome, granulomas
pathogen in cases of bacterial tracheitis. In ad- are generally not. Furthermore, HIES occurs
Infectious Disease

dition, Gram stain would show gram-negative in an autosomal dominant or autosomal reces-
rods. sive pattern of inheritance and not in the X-
linked pattern suggested by the case history.
41. The correct answer is C. The patient de-
Answer E is incorrect. Wiskott-Aldrich syn-
scribed in the vignette has chronic granuloma-
drome is an X-linked disorder with B- and
tous disease. Although the presentation of this
T-lymphocyte dysfunction. Affected patients
disease is variable, the diagnosis should be sus-
typically present with eczema and thrombocy-
pected in patients with recurrent lymphadeni-
topenia. Although patients have abnormalities
tis, multiple-site osteomyelitis, cutaneous ab-
of their skin, these are heralded by petechiae
scesses (confused with acne), hepatic
(from the thrombocytopenia) and eczema. The
abscesses, a family history of recurrent infec-
disease results from mutations of the Wiskott-
tions, or unusual infections with catalase-posi-
Aldrich syndrome protein (WASp) gene on the
tive organisms such as Staphylococcus aureus
X chromosome. WASp is important for rearrang-
or Aspergillus. In many of these cases, the pa-
ing the actin cytoskeleton in immune cells and
tient is asymptomatic or with much fewer signs
thus enables responsiveness.
and symptoms (e.g., lack of fever) than would
otherwise be expected given the infectious bur-
42. The correct answer is C. The history is typical
den. Chronic granulomatous disease is caused
of disseminated Lyme disease. After the acute
by mutations in genes encoding proteins in-
phase of the illness, cardiac, neurologic, and
volved in generating a respiratory burst. One of
musculoskeletal symptoms become prominent.
these, phox91, is on the X chromosome, pro-
Typical cardiac symptoms include uctuating
viding an X-linked pattern of inheritance as
second- or third-degree atrioventricular block,
demonstrated by the family history.
which resolves completely after a few weeks,
Answer A is incorrect. Ataxia-telangiectasia is and myocarditis. Neurologic complaints can
a T-lymphocyte disorder that manifests as ocu- include meningitis, cranial neuritis, and rarely,
locutaneous telangiectasias and progressive encephalitis. Musculoskeletal symptoms in-
cerebellar ataxia. It is caused by a DNA repair clude intermittent joint swelling and some-
defect resulting from a mutation in the ATM times chronic arthritis.
gene encoding a protein critical for repair of
Answer A is incorrect. Gonococcal disease can
DNA breaks. Ataxia generally begins when the
cause joint swelling, but the cardiac and neu-
child learns to walk, and the telangiectasias
rologic symptoms are not commonly caused by
generally become apparent between 3 and 6
Neisseria gonorrhoeae.
years of age.
Answer B is incorrect. Gout can cause joint
Answer B is incorrect. Chdiak-Higashi syn-
pain and swelling, but the neurologic and car-
drome includes neutropenia, neuropathy, and
diac symptoms cannot be attributed to gout.
oculocutaneous albinism. It is inherited in an
Also, the patient is not in a demographic group
autosomal-recessive manner. The disorder is
in which gout is usually seen; she is too young.
HIGH-YIELD SYSTEMS
Chapter 8: Infectious Disease Answers 249

Answer D is incorrect. The headaches are not occurs in young children. It is more common
typical of migraines. They have an acute onset in children of Asian descent. Patients initially
and last longer than typical migraines, uctu- have nonspecic respiratory or gastrointestinal
ating throughout the day, rather than being complaints. Aside from fever, patients have at
steadily progressive and then resolving. Also, least four of the following symptoms: conjunc-
the muscle aches and arrhythmia are not asso- tivitis, rash, cervical adenopathy, strawberry
ciated with migraines. tongue, and swollen hands and feet. If Kawasa-
kis disease is suspected, ECGs should be ob-
Answer E is incorrect. Sjgrens syndrome is
tained to rule out acute myocardial infarction,
possible, but again, the recent travel to an en-
and echocardiogram should be obtained to
demic area and the time course of the symp-
conrm the diagnosis. Dilation of the coronary
toms makes Lyme disease the more likely diag-
artery lumen is present in 50% of patients by
nosis.
the 10th day. High-dose aspirin is given to con-
Answer F is incorrect. While the patients trol inammation and fever, but has no effect

Infectious Disease
symptoms could be caused by systemic lupus on aneurysm formation. Twenty-ve percent of
erythematosus, the recent travel to an area en- patients will develop coronary aneurysms by
demic for Lyme disease and the time course of day 13 if aspirin is given alone. Combined
the symptoms makes it the more likely diagno- therapy with aspirin and IVIG reduces risk of
sis. coronary aneurysm to 4%5%. Administration
of aspirin and IVIG early in his disease would
43. The correct answer is E. Patients with silicosis have greatly reduced the incidence of the coro-
are particularly susceptible to TB infection, nary aneurysm shown in this patients angio-
and the risk increases with increasing lung dust gram.
burden. In one study, miners with silicosis
Answer A is incorrect. Aspirin and IVIG are
were 30 times more likely to be infected with
rst-line treatment for Kawasakis disease.
TB than miners without diagnosable silicosis.
While acetaminophen is another popular anti-
Regular tuberculin screening is especially im-
pyretic in the pediatric population, it has not
portant for patients with silicosis.
been shown to be effective against the fever of
Answer A is incorrect. Asbestos exposure can Kawasakis disease. Furthermore, it is the IVIG
lead to bronchogenic carcinoma and malig- that reduces risk of coronary aneurysms.
nant mesothelioma, and the risk of cancer is
Answer C is incorrect. There has been debate
further increased in smokers.
about the use of steroids in the treatment of
Answer B is incorrect. Beryllium inhalation Kawasakis disease, but it is currently believed
causes berylliosis, which does not increase the that corticosteroids may actually increase the
risk of TB infection. risk of aneurysm formation in patients with Ka-
Answer C is incorrect. Exposure to carbon wasakis disease. As a result, it is actually con-
dust causes anthracosis, a usually harmless pig- traindicated in these patients.
mentation of the lungs noted at autopsy among Answer D is incorrect. Metoprolol and other
city dwellers and, to a much greater extent, in -blockers have been shown to have a protec-
smokers. tive effect in adults with coronary artery dis-
Answer D is incorrect. Coal workers pneu- ease, but have not been used in children with
moconiosis results from coal dust inhalation coronary aneurysms due to Kawasakis disease.
and is not linked to an increased risk of TB. Answer E is incorrect. Since this patient
Coal dust contains both carbon and silica and reported initial upper respiratory infection
causes more cellular reaction (and thus more symptoms, this patient could have streptococ-
pathology) than carbon dust alone. cal pharyngitis. Penicillin is often given in pa-
tients with acute rheumatic fever to prevent
44. The correct answer is B. The patient most the development of valvular rheumatic heart
likely has Kawasakis disease, a vasculitis that disease. Fever, rash, and a strawberry tongue
HIGH-YIELD SYSTEMS
250 Section I: Organ Systems Answers

could occur with both streptococcal pharyn- used for atypical pneumonia and would cover
gitis (with subsequent acute rheumatic fever) Streptococcus pneumoniae as well. Doxycycline
and Kawasakis disease, but the time course can be used for atypical pneumonia but would
(symptoms occurring all at once instead of not cover S. pneumoniae.
upper respiratory infection followed by other
Answer D is incorrect. Gentamicin is effec-
symptoms in a few weeks) is more consistent
tive against gram-negatives and even covers
with Kawasakis disease. Symptoms of swollen
pseudomonads; it is usually combined with
extremities and conjunctivitis are more consis-
a penicillin-based drug for broader-spectrum
tent with Kawasakis disease, and coronary an-
coverage. Gentamicin is used for treatment of
eurysms have not been known to be associated
pneumonia when Staphylococcus aureus is iso-
with acute rheumatic fever. Antibiotics have
lated. S. aureus is easy to grow in culture, but
not been shown to have any inuence on the
oftentimes represents contamination.
clinical course of Kawasakis disease.
Answer E is incorrect. Metronidazole is used
Infectious Disease

45. The correct answer is B. Streptococcal pneu- as an alternative regimen for pneumonia
monia is the most common cause of commu- caused by anaerobes such as Bacteroides fra-
nity-acquired pneumonia in this age group. In gilis, Peptostreptococcus, and Fusobacterium
those over 40 years of age, the most common (rst-line therapy is clindamycin or amoxicil-
causes are Streptococcus pneumoniae, Haemo- lin-clavulanate). These agents are more com-
philus inuenzae, anaerobes, viruses, and My- monly indicated in aspiration pneumonia
coplasma spp. Streptococcal pneumonia causes caused by anaerobes that are part of the nor-
a lobar pneumonia, but diagnosis cannot be mal ora of the mouth and nasopharynx. Aspi-
made on radiographic ndings alone. Diagno- ration pneumonia is most common in patients
sis depends on sputum Gram stain and cul- with decreased consciousness and those with
ture. Macrolides (erythromycin and clarithro- dysphagia due to neurologic decits or upper
mycin) are rst-line empiric treatment for gastrointestinal disorders. Patients present with
unidentied community-acquired pneumonia, typical symptoms of cough, fever, purulent
but in this patient with rapid-onset symptoms sputum, and dyspnea that occurs over days to
and lobar consolidation characteristic of pneu- weeks (instead of hours). Sputum is character-
mococcal infection, oral ampicillin would be istically putrid.
used in penicillin-sensitive strains. If the strain
is resistant to penicillin, a -lactam (such as 46. The correct answer is A. Chickenpox (vari-
ceftriaxone) can be used. cella) is an acute febrile illness associated with
a rash that is common in children who have
Answer A is incorrect. Aztreonam is ineffec-
not been immunized. The illness has a wide
tive against the gram-positives and would be
spectrum of clinical severity but is generally
ineffective against streptococcal pneumonia.
self-limited. The prodromal phase often con-
Aztreonam is a monobactam used against
sists of fever, malaise, headache, and anorexia,
gram-negative bacteria, and is indicated in hos-
which usually manifest 12 days before the
pital-acquired pneumonia.
outbreak of the rash. The rash of varicella often
Answer C is incorrect. Doxycycline is mostly rst appears on the scalp, face, and trunk. Pru-
used for Chlamydia pneumoniae, Mycoplasma ritic erythematous macules comprise the pri-
pneumoniae, Rickettsia, and Brucella. Patients mary exanthem, with progression through the
affected by these microbes are relatively as- papular stage to form clear, uid-lled vesicles.
ymptomatic and disease usually manifests as New areas of lesions develop while initial le-
gradual onset of headache, malaise, and low- sions are crusting. The characteristic rash of
grade fever. M. pneumoniae is most common varicella has lesions in multiple stages of evolu-
in school-aged children, military recruits, and tion as opposed to smallpox, in which all le-
college students in fall and winter. C. pneu- sions advance through development at the
moniae is most common in elderly patients. same rate. Children are now routinely vacci-
Azithromycin and erythromycin are commonly nated against varicella infection.
HIGH-YIELD SYSTEMS
Chapter 8: Infectious Disease Answers 251

Answer B is incorrect. This presentation is Answer B is incorrect. This case is inconsis-


not consistent with contact dermatitis, which tent with a large stroke that could be visualized
is typically pruritic, has a pattern that suggests on CT of the head. Instead, the patient may
external causes (i.e., a linear distribution), and have had a frontal lobe stroke or be experienc-
does not have an associated viral prodrome. ing multi-infarct dementia, but this does not
An inciting exposure can usually be identied explain the WBCs in the patients CSF.
from the patients history.
Answer D is incorrect. The patient is not quite
Answer C is incorrect. This presentation is not old enough or sick enough to be at a high risk
consistent with hand-foot-and-mouth disease, of developing urosepsis. Urosepsis also does
which typically presents with fever and con- not cause appearance of WBCs in the CSF.
stitutional symptoms, and then as white-grey
Answer E is incorrect. This could be hydro-
maculopapular vesicles with a characteristic
cephalus; however, the onset is too acute, and
peripheral distribution on the hands, feet, but-
WBCs should not be in the CSF with hydro-

Infectious Disease
tocks, and buccal mucosa/palate/tongue/tonsils
cephalus. The patient is also not in a high-risk
(herpangina). Coxsackie A virus is the most
demographic. Hydrocephalus is an increase
common causative agent.
in the volume of CSF within the central ner-
Answer D is incorrect. This presentation is vous system. Symptoms of hydrocephalus in
not normal for herpes zoster, which typically the adult population include cognitive distur-
presents as vesicular lesions distributed along bances, headaches, nausea, difculty walking,
a dermatome, usually in patients who are im- blurred vision, and incontinence. Denitive
munocompromised or elderly. Herpes zoster treatment is usually surgical, with shunt place-
occurs after reactivation of latent virus in the ment required in all but 25% of cases.
dorsal root ganglia.
48. The correct answer is B. KS is a low-grade
Answer E is incorrect. This presentation is
vascular tumor associated with infection by
not consistent with smallpox infection (variola
HHV-8. There are four forms of KS, but the
major), which typically presents as a conuent
one most common in HIV-positive patients is
rash with discrete, rm vesicles and nodules
AIDS-related KS, an opportunistic infection
that are characteristically all at the same stage
that aficts patients with AIDS 20,000 times
of development.
more commonly than the U.S. population in
Answer F is incorrect. This presentation is not general. Cutaneous involvement is common,
classic for Stevens-Johnson syndrome, which but the disease can cause extracutaneous
typically presents as a diffuse, desquamating manifestations, including disease in the oral
rash (erythema multiforme) involving the mu- cavity, gastrointestinal tract, and the respira-
cous membranes and/or conjunctiva. In most tory tract. The lesions depicted here are clas-
cases, Stevens-Johnson syndrome is drug in- sic for KS. The development and progression
duced, although infection (especially in chil- of KS can be halted with highly active anti-
dren) may also cause disease. retroviral therapy.
Answer A is incorrect. Dapsone is an alter-
47. The correct answer is C. The patient has HSV
native drug for prophylaxis of Pneumocystis
encephalitis, characterized by bizarre behavior,
jiroveci pneumonia, given as second-line ther-
hypersexuality, fever, and aseptic CSF. He
apy to patients allergic to sulfa drugs.
should be started on intravenous acyclovir after
diagnosis is conrmed with a cerebrospinal Answer C is incorrect. Trimethoprim-sulfame-
herpes simplex virus DNA polymerase chain thoxazole is useful in prophylaxis of Pneumo-
reaction. cystis jiroveci pneumonia, but will not prevent
KS.
Answer A is incorrect. The presence of WBCs
in the patients CSF is not consistent with alco- Answer D is incorrect. Zidovudine/lamivudine
hol intoxication. combination therapy can be a useful part of a
HIGH-YIELD SYSTEMS
252 Section I: Organ Systems Answers

highly active antiretroviral therapy regimen, Later stages of syphilis include cardiac and neu-
but they would not be recommended without rologic manifestations if left untreated.
a third antiretroviral drug.
50. The correct answer is B. The patient is exhib-
Answer E is incorrect. Zidovudine, or AZT,
iting signs of HSV encephalitis. The clinical
was the rst available antiretroviral drug, but
syndrome is characterized by fever, headache,
current recommendations do not advise mono-
seizures, focal neurologic signs, and impaired
therapy with AZT for KS.
consciousness. It is the most common cause of
fatal sporadic encephalitis in the United States,
49. The correct answer is C. Lymphogranuloma
mostly occurring in children and adolescents.
venereum is caused by Chlamydia trachomatis
Central nervous system invasion occurs via the
serotypes L1, L2, and L3. It is uncommon in
trigeminal nerve following an episode of pri-
the United States, but is endemic in parts of
mary HSV-1 of the oropharynx. CSF analysis
East and West Africa, India, Southwest Asia,
typically shows lymphocytic pleocytosis, in-
Infectious Disease

and the Caribbean. HIV seropositivity is the


creased erythrocytes, elevated protein, and nor-
strongest risk factor. Primary infection with
mal glucose. The patients history of a recent
lymphogranuloma venereum is characterized
oral ulcer favors HSV as the etiology of this pa-
by a genital ulcer that heals within a few days.
tients encephalitis.
Inguinal lymphadenopathy occurs 26 weeks
later, and represents direct extension of the in- Answer A is incorrect. Bacterial meningitis
fection to inguinal lymph nodes. Diagnosis is presents with a classic triad of fever, nuchal ri-
usually made through serologic testing in the gidity, and change in mental status, including
setting of a compatible clinical scenario. Doxy- skin lesions consisting of petechiae and pur-
cycline is the recommended therapy for both pura. CSF analysis reveals pleocytosis, elevated
lymphogranuloma venereum and other sero- protein, and decreased glucose.
types of C. trachomatis.
Answer C is incorrect. Neurocysticercosis is
Answer A is incorrect. Acyclovir is an anti- caused by the larval stage of Taenia solium, the
viral therapy, and is the treatment of choice pork tapeworm. The majority of neurocysticer-
for genital herpes. Herpes is characterized by cal infections are asymptomatic. CSF analysis
painful genital ulcers and is diagnosed using typically shows mild pleocytosis, with normal
polymerase chain reaction testing of the ulcer- protein and glucose levels.
ated tissue.
Answer D is incorrect. Rabies encephalitis is
Answer B is incorrect. Ceftriaxone is the rec- an unlikely diagnosis in the absence of a his-
ommended treatment for gonorrhea. In women, tory of a dog or bat bite. Most patients pre-
gonorrhea is often asymptomatic, but it may sent with hydrophobia, aerophobia, pharyn-
present as vaginal pruritus with a mucopurulent geal spasms, and hyperactivity. Cerebrospinal
discharge. If left untreated, gonorrhea may de- uid shows pleocytosis, normal glucose, and a
velop into PID and concomitant infertility. mildly elevated protein.
Answer D is incorrect. Gentamicin is an Answer E is incorrect. The patients history
aminoglycoside associated with nephrotoxicity was not suggestive of active HIV disease and
and ototoxicity. Doxycycline is a safer therapy thus makes subacute HIV encephalomyelitis
for lymphogranuloma venereum. unlikely.
Answer E is incorrect. Penicillin remains the Answer F is incorrect. Tabes dorsalis presents
standard of care for syphilis. The clinical presen- with paresthesias, abnormal gait, and shoot-
tation of syphilis begins as a painless chancre in ing pains of the extremities or trunk. Physical
the primary infection, later progressing to a sys- ndings also include diminished peripheral re-
temic infection that includes a maculopapular exes and altered position and vibration sense,
rash involving the palms and soles, raised white- as well as Argyll Robertson pupils. CSF is often
grey lesions of the mucous membranes termed normal.
condyloma lata, and diffuse lymphadenopathy.
HIGH-YIELD SYSTEMS
Chapter 8: Infectious Disease Answers 253

51. The correct answer is C. The ToRCHeS in- setting, the clinician should consider this diag-
fections include Toxoplasmosis, Rubella, Cyto- nosis and can conrm it by culture or histopa-
megalovirus, Herpesvirus/HIV, and Syphilis. thology. Treatment involves a combination of
Infections in utero or during birth are associ- surgical debridement and antifungal therapy,
ated with signicant fetal and neonatal mor- as well as elimination of predisposing factors
bidity and mortality. These infections have such as hyperglycemia. The most important
been grouped together because they feature single therapy, however, is surgical debride-
similar presentations, including a rash and oc- ment to remove all necrotic tissue. This should
ular ndings. The American College of Obste- be undertaken as soon as the diagnosis is sus-
tricians and Gynecology currently recom- pected, as it provides the best hope for cure.
mends that rubella and syphilis serologies be
Answer B is incorrect. Vancomycin and piper-
checked during the rst prenatal visit.
acillin-tazobactam provide broad-spectrum an-
Answer A is incorrect. CMV is not routinely tibacterial coverage. However, these antibiotics

Infectious Disease
checked during prenatal visits. would be ineffective in treating invasive fungal
infection.
Answer B is incorrect. CMV is not routinely
checked during prenatal visits. Answer C is incorrect. High-dose treatment
with liposomal amphotericin B is the preferred
Answer D is incorrect. The utility of checking
antifungal therapy for mucormycosis. How-
the ToRCHeS titers during routine prenatal
ever, surgical debridement is the most impor-
care has not been cost effective in controlled
tant rst step in treating rhino-orbital-cerebral
studies. Diagnosis is more accurately made
zygomycosis.
with rubella and syphilis titers, CMV urine
cultures, and fetal ultrasound. Answer D is incorrect. Amoxicillin-clavulan-
ate is commonly prescribed to treat bacterial
Answer E is incorrect. Toxoplasmosis is not
sinus infection in outpatients. It would have no
routinely checked during prenatal visits.
activity against the invasive mucormycosis in
Answer F is incorrect. Toxoplasmosis is not this case.
routinely checked during prenatal visits.
Answer E is incorrect. Posaconazole is a
broad-spectrum oral antifungal agent that is ac-
52. The correct answer is A. This patient most
tive against zygomycetes. It is used in patients
likely has mucormycosis. Mucormycosis, also
who fail or cannot tolerate standard therapy,
called zygomycosis, is an angioinvasive fungal
and as an oral step-down therapy in those who
infection that can cause devastating rhino-
have responded to amphotericin B.
orbital-cerebral and pulmonary infection. Zy-
gomycetes are fungi with broad, irregularly
53. The correct answer is E. Pain elicited by gen-
branched, nonseptate hyphae that are found
tly pushing/pulling the pinna is classic for otitis
on decaying vegetation and soil. Although hu-
externa. In diabetic patients, otitis externa is
man exposure is frequent, infection is rare. In-
usually due to Pseudomonas, and can be
dividuals who are immunocompromised, in-
chronic in patients with seborrhea. The treat-
cluding those with diabetes mellitus, AIDS,
ment of choice is antibiotic ear drops.
and organ transplants and those taking gluco-
corticoids, are at the greatest risk of infection Answer A is incorrect. CT is a relatively costly
following spore inhalation. Mucormycosis is test that may do nothing to alter the treatment
generally a fast-spreading infection that pre- plan. The patient is afebrile, and it is unlikely
sents as acute sinusitis with fever, nasal dis- that the infection has spread into the bone to
charge, sinus pain, and headache. Spread become osteomyelitis.
throughout the sinuses and to the contiguous
Answer B is incorrect. Naegleria fowleri is an
structures of the palate, orbit, and brain can re-
unrelated infection in which people swimming
sult in tissue necrosis with eschar, facial numb-
in colonized ponds get a cerebral infection via
ness, blindness, altered mental status, and ulti-
the nose. It can be fatal after about a week.
mately death. In the appropriate clinical
HIGH-YIELD SYSTEMS
254 Section I: Organ Systems Answers

Answer C is incorrect. MRI is a fairly costly test impaired oxygenation. Examination reveals tac-
that may do nothing to alter the treatment plan. hypnea, crackles, and rhonchi. X-ray of the
The patient is afebrile, and it is unlikely that chest reveals diffuse bilateral interstitial or alveo-
the infection has spread past the external ear. If lar inltrates, in addition to pneumothoraces,
osteomyelitis (malignant otitis externa) was sus- cysts, nodules, or pleural effusions. Immunou-
pected an imaging study would be appropriate. orescence and silver staining of induced sputum
is essential for the denitive diagnosis of P.
Answer D is incorrect. Surgical debridement
jiroveci pneumonia. Intravenous trimethoprim-
would be necessary only if the patient had ma-
sulfamethoxazole is the initial drug of choice for
lignant otitis externa. Malignant otitis externa
treatment. In addition, corticosteroids are indi-
is almost exclusively seen in immunocompro-
cated in patients with a partial arterial oxygen
mised patients. If this patient had a fever or
pressure <70 mm Hg or an alveolar-arterial gra-
signs of systemic toxicity, then more concern
dient 35 mm Hg or higher.
would be raised, and further investigation
Infectious Disease

would be warranted. Answer A is incorrect. Aerosolized pentami-


dine is used as prophylaxis for P. jiroveci pneu-
54. The correct answer is C. This patient likely monia, as well as in those patients with atypical
has Trichomonas infection. This is distin- manifestations or extrapulmonary presentation.
guished from candidiasis by the lack of hyphae
Answer B is incorrect. Amphotericin B is a
on the potassium hydroxide wet mount and in-
potent antifungal agent considered the treat-
creased pH of the vaginal uid. The disease
ment of choice for mucormycosis and for most
classically bounces between partners as they
central nervous system fungal infections, or for
reinfect each other, making treatment of the
fungal infections considered to be severe or
partner simultaneously an important aspect of
life-threatening. However, the patients presen-
management.
tation is most consistent with that of P. jiroveci
Answer A is incorrect. As a minor seeing her pneumonia.
physician because of an STD, the patient has
Answer C is incorrect. Dapsone is primarily
the right to condentiality. However, fully edu-
used as a prophylactic agent for toxoplasmosis
cating the patient about effective strategies for
and P. jiroveci pneumonia in immunocompro-
STD and pregnancy prevention, including ab-
mised patients, or it can be used in combina-
stinence, would be appropriate.
tion with trimethoprim for treatment. How-
Answer B is incorrect. This is not necessary for ever, trimethoprim-sulfamethoxazole is the
effective treatment. rst-line agent.
Answer D is incorrect. Reportable STDs are Answer D is incorrect. Vancomycin is the drug
chlamydia, gonorrhea, and syphilis. The pa- of choice for methicillin-resistant Staphylococcus
tient may have one of these conditions and aureus (MRSA), streptococcal infections, staphy-
should be tested for them, but Trichomonas is lococcal enterocolitis, or antibiotic-associated
not necessary to report per se. pseudomembranous colitis produced by Clostrid-
Answer E is incorrect. Candida infection can ium difcile. The patients presentation is not typ-
be a presenting symptom of diabetes. However, ical for that of a MRSA infection.
the lack of hyphae in the vaginal discharge
56. The correct answer is D. The patient most
makes this diagnosis unlikely.
likely has meningitis caused by Neisseria men-
55. The correct answer is E. The patients presenta- ingitidis. Such patients typically present with
tion is most consistent with that of Pneumocystis fever, malaise, headache, stiff neck, photopho-
jiroveci pneumonia. It is frequently the rst pre- bia, altered mental status, nausea/vomiting, or
senting manifestation of patients with AIDS and seizures. A diffuse petechial rash may occur in
a CD4+ cell count <200/mm. Patients present the setting of meningococcal meningitis. Diag-
with fever, cough, chest pain, weight loss, and nosis is based on clinical presentation, blood
cultures, and lumbar puncture, which reveals
HIGH-YIELD SYSTEMS
Chapter 8: Infectious Disease Answers 255

a positive CSF Gram stain and culture, ele- polymerase in affected cells. It is used in treat-
vated protein levels, and decreased glucose lev- ment and prophylaxis for the following viral in-
els. CT of the head should be performed be- fections, listed in order of decreasing efcacy:
fore lumbar puncture to rule out mass effect HSV, varicella zoster virus, EBV.
and prevent herniation. Empiric treatment for
Answer B is incorrect. Amantadine is used in
suspected meningitis is vancomycin and ceftri-
prophylaxis for inuenza A and in the treat-
axone or cefotaxime; ampicillin may be added
ment of Parkinsons disease. It has a number of
to cover Listeria infection if the patient is im-
adverse effects including ataxia, slurred speech,
munosuppressed, very young, or an older adult.
and dizziness, because it can penetrate the
Prophylaxis with rifampin, ciprooxacin, or
blood-brain barrier. Rimantadine is a deriva-
ceftriaxone should be given to close contacts of
tive with fewer adverse effects because it does
patients with meningococcal meningitis.
not penetrate the blood-brain barrier.
Answer A is incorrect. Acyclovir is used to
Answer C is incorrect. Corticosteroids are

Infectious Disease
treat herpes encephalitis. However, it is not
used in many different types of diseases to re-
used as a prophylactic agent for close contacts
duce inammation and the signs and symp-
of patients with meningococcal meningitis.
toms related to inammation. They would not
Answer B is incorrect. Ampicillin is used to be indicated in treatment of measles.
treat meningitis in patients who are very young,
Answer D is incorrect. Immune globulin
an older adult, or immunosuppressed (if Lis-
against the measles virus is available but must
teria is the suspected causative organism).
be administered within 6 days of exposure.
However, it is not used as a prophylactic agent
Since the incubation period is 1012 days be-
for close contacts of patients with meningococ-
fore the prodromal stage, and this woman is
cal meningitis.
already exhibiting signs and symptoms of the
Answer C is incorrect. Cefotaxime may be used prodrome, immune globulin would not be
as an alternative to ceftriaxone in the treatment helpful. It can be given to household and other
of suspected meningitis. However, it is not used contacts who have not received the measles-
as a prophylactic agent for close contacts of pa- mumps-rubella vaccine.
tients with meningococcal meningitis.
58. The correct answer is D. The patient is pre-
Answer E is incorrect. Vancomycin is used
senting with symptoms of acute food poisoning
to treat meningitis. However, it is not used as
with Staphylococcus aureus enterotoxin. S. au-
a prophylactic agent for close contacts of pa-
reus toxicity is usually due to eating ham, poul-
tients with meningococcal meningitis.
try, potato or egg salad, or other dairy products.
Symptoms usually begin 16 hours following
57. The correct answer is E. The patient in this
ingestion of the contaminated food product.
case has measles based on the pathognomonic
Vomitus or food can be tested for the entero-
enanthem of the oral mucosa, called Kopliks
toxin, but the diagnosis is usually clinical.
spots. Kopliks spots along with a low-grade fe-
ver, dry cough, coryza, and conjunctivitis are Answer A is incorrect. Bacillus cereus toxic-
part of the prodromal phase which lasts 35 ity often follows the ingestion of fried or un-
days. The nal stage involves high fever and a dercooked rice. The enterotoxin is heat stable
maculopapular rash over the entire body. This and produces symptoms of nausea and profuse
woman is at risk of infection because she did vomiting. The diagnosis is clinical and the ill-
not receive the measles-mumps-rubella vaccine ness is self-limiting.
usually given to children at 1215 months and
Answer B is incorrect. Salmonella poisoning
then again at 46 years of age. The only treat-
follows the ingestion of infected beef, poultry,
ment for these patients includes antipyretics
eggs, or dairy products, and causes a watery
for fever, bedrest, and uids.
diarrhea. The incubation time for Salmonella
Answer A is incorrect. Acyclovir is an antivi- is usually 13 days and the diagnosis is made
ral agent that preferentially inhibits DNA viral with culture of stool.
HIGH-YIELD SYSTEMS
256 Section I: Organ Systems Answers

Answer C is incorrect. Shigella can be present Answer D is incorrect. Haemophilus inuen-


in potato or egg salad, lettuce, or raw vegeta- zae type B is now a rare cause of illness in chil-
bles. However, it results in dysentery and can dren due to the Hib conjugate vaccine. The
be diagnosed by stool culture. vaccine is given in four doses at 2, 4, 6, and 12
months of age. The vaccine is responsible for a
Answer E is incorrect. Vibrio cholerae toxicity
large reduction in morbidity and mortality due
occurs hours to days following the consump-
to H. inuenzae infections in children.
tion of contaminated shellsh. Symptoms in-
clude watery stools with or without nausea or Answer E is incorrect. Streptococcus pneumo-
vomiting. Grams staining of the patients stool niae is a common cause of bacterial pneumonia
may show sheets of curved gram-negative rods. among children 58 years old. Bacterial pneu-
Treatment relies mainly on hydration therapy. monia often presents with fever, chills, tachyp-
nea, malaise, and shortness of breath. Physical
Answer F is incorrect. Vibrio parahaemolyti-
examination and radiologic studies often reveal
cus toxicity can result after eating raw shellsh,
Infectious Disease

unilateral, focal, or lobar areas of consolidation.


particularly mollusks and crustaceans. Symp-
Bacterial pneumonia is treated with antibiotics
toms commence 48 hours after consumption
appropriate to pathogen sensitivity.
of the infected food product and results in ab-
dominal pain, vomiting, and dysentery. Diag-
60. The correct answer is B. CT scan with con-
nosis is made by culture.
trast is useful in identifying a retropharyngeal
abscess and may reveal a ring-enhancing node
59. The correct answer is A. Community-acquired
or scalloping of the nodal wall. Soft tissue neck
pneumonia is commonly caused by respiratory
lms taken during inspiration may show an air-
viruses such as adenovirus among children of
uid level in the retropharyngeal space. How-
this age group. Viral pneumonias often present
ever, the only denitive diagnosis and treat-
with cough, wheezing, and low-grade fever.
ment is an incision and drainage procedure
Other common viral agents causing commu-
with culture of drained material.
nity-acquired pneumonia in pediatric patients
include parainuenza, inuenza, and RSV. In Answer A is incorrect. Most retropharyngeal
the absence of complications, children with vi- abscesses are successfully managed with in-
ral community-acquired pneumonia may be travenous antibiotics with or without surgical
treated with supportive care as outpatients. drainage. A third-generation cephalosporin in
combination with anaerobic coverage, such
Answer B is incorrect. CMV is one of the
as clindamycin, is appropriate. Approximately
ToRCHeS Infections (Toxoplasmosis, Rubella,
50% of cases will require surgical drainage.
Cytomegalovirus, Herpesvirus/HIV, Syphilis),
which crosses the placenta and causes intrauter- Answer C is incorrect. The retropharyngeal
ine infection. ToRCHeS infections may cause space is bounded by the pharynx and the cervi-
intrauterine growth restriction and neonatal cal vertebrae and extends into the superior me-
sepsis. CMV may cause afebrile pneumonia in diastinum. Infection of lymph nodes within this
an infant <3 months old and is associated with deep neck space is usually due to bacterial ex-
immunodeciency in an older child. tension from the oropharynx. Localized oropha-
ryngeal infection may be caused by dental infec-
Answer C is incorrect. GBS is a major cause
tion or trauma. Vertebral osteomyelitis may also
of illness in neonates. GBS is often found in
extend to the retropharyngeal lymph nodes.
the female vaginal canal and is transmitted
vertically during delivery. GBS causes bacte- Answer D is incorrect. Lateral pharyngeal
remia, pneumonia, and sepsis in newborns. lymph nodes are also within a deep neck space
Preventive measures include treatment of the bounded by the pharynx. Large retropharyn-
mother with antibiotics at least 4 hours prior to geal and lateral pharyngeal infections may
delivery to allow time for transport across the impinge on the upper airway causing respira-
placenta. tory distress. Other complications of deep neck
lymph node infection include aspiration pneu-
monia and posterior mediastinitis.
HIGH-YIELD SYSTEMS
Chapter 8: Infectious Disease Answers 257

Answer E is incorrect. Retropharyngeal infec- (mucocutaneous and genital herpes lesions),


tions are usually polymicrobial and include but is also effective against varicella-zoster vi-
pathogens found in the oropharynx such as rus and EBV.
anaerobic bacteria, group A Streptococcus and
Answer B is incorrect. Amantadine was ini-
Staphylococcus aureus. Other less common
tially developed to treat Parkinsons disease,
pathogens include Haemophilus inuenzae
but was found to have antiviral activity and is
and Klebsiella.
used for prophylaxis of inuenza A. It works
by blocking viral penetration and uncoating,
Questions 61, 62, and 63 but has many adverse effects including ataxia,
slurred speech, and depression, since it can
61. The correct answer is E. RMSF is caused by in- cross the blood-brain barrier.
fection with Rickettsia rickettsii, an organism car-
ried by the American dog tick. Patients with ex- Answer C is incorrect. Amphotericin B is an
antifungal agent used to treat a wide spectrum

Infectious Disease
posure to the wilderness and tick bites in the
Midatlantic region are at highest risk of infec- of systemic mycoses. It binds ergosterol, a com-
tion. RMSF is characterized by a rash that be- pound unique to fungi, and forms pores that
gins on the wrists and ankles and spreads cen- allow leakage of electrolytes.
trally. Headache, fever, and malaise can also be Answer D is incorrect. Dapsone inhibits bac-
present along with disseminated intravascular co- terial synthesis of dihydrofolic acid and is used
agulation or altered mental status in severe cases. to treat leprosy and other skin conditions. It
Treatment should be initiated as soon as possible is also sometimes used as prophylaxis against
with doxycycline, even in younger patients, since Pneumocystis jiroveci pneumonia in patients
the condition is rapidly fatal if untreated. Short with HIV (if trimethoprim-sulfamethoxazole is
courses of doxycycline to treat RMSF do not not tolerated) and to treat idiopathic thrombo-
cause signicant dental staining. cytopenic purpura.

62. The correct answer is I. Roseola, also called Answer F is incorrect. Fluconazole is an an-
sixth disease, is caused by HHV-6. It usually af- tifungal agent that inhibits the synthesis of er-
fects children 6 months to 3 years old and gosterol. It is used to treat systemic mycoses
manifests as a rosy rash after 3 days of high fe- including cryptococcal meningitis in patients
vers. It is a self-limited disease requiring no with AIDS and candidal infections (oral thrush
treatment. and yeast infections).
Answer G is incorrect. Foscarnet inhibits the
63. The correct answer is J. Scarlet fever is caused pyrophosphate binding site on viral DNA poly-
by GBS and can occur if streptococcal pharyn- merases. It is used to treat HSV types 1 and 2
gitis is left untreated. It is characterized by nd- and CMV retinitis.
ings such as strawberry tongue and a sandpa-
per-like rash on the trunk that desquamates Answer H is incorrect. Gentamicin is an
after a few days. Treatment is with penicillin to aminoglycoside, a bactericidal class of antibiot-
prevent rheumatic fever and its complications, ics that inhibits the initiation complex, thereby
which can include irreversible damage to the causing misreading of mRNA transcripts. It is
heart and its valves. used for treating severe gram-negative rod in-
fections along with -lactam antibiotics, which
Answer A is incorrect. Acyclovir is an antivi- help break down the cell walls, allowing the
ral agent that preferentially inhibits viral DNA aminoglycoside to be effective.
polymerase. It is most effective against HSV
HIGH-YIELD SYSTEMS
258 Section I: Organ Systems Answers

Questions 64, 65, and 66 counts. Urinary tract infection due to E. coli
rarely causes penile discharge and would be
64. The correct answer is H. Septic monoarthritis
much more common in women than in men.
with penile discharge and dysuria is gonorrhea
Treatment is through antibiotics.
until proven otherwise. Gonorrhea typically
presents with urethral discomfort, dysuria, and Answer D is incorrect. Haemophilus ducreyi,
discharge. Risk factors include unprotected sex a gram-negative rod and relative of Haemophi-
with multiple partners. It is treated by antibiot- lus inuenzae, causes chancroid characterized
ics such as third-generation cephalosporins or by painful genital ulcers. It must be distin-
ciprooxacin and usually fully resolves with guished from syphilis, HSV, and lymphogranu-
treatment. loma venereum. Syphilis and lymphogranu-
loma venereum are painless, but misdiagnosis
65. The correct answer is B. Nongonococcal ure- with HSV is possible because both conditions
thritis is the most common of the STDs. Be- cause painful blisters. Because HSV is far more
Infectious Disease

cause the Gram stain does not show the pres- common than chancroid in the United States,
ence of gonorrhea, chlamydia remains the it is the usual diagnosis of painful genital blis-
most likely cause. Although chlamydia can ters. If syphilis serology and HSV cultures are
present asymptomatically, it can also present as negative, chancroid is the diagnosis of exclu-
dysuria, urethral discomfort, discharge, scrotal sion. Chancroid is treated through antibiotics,
swelling in men, and PID in women. Risk fac- particularly macrolides.
tors include unprotected sex with multiple
Answer F is incorrect. HIV is a lentivirus, a
partners. Treatment is with antibiotics such as
subgroup of the retroviruses. It is particularly
doxycycline or azithromycin.
known for its latency (it may manifest several
years after infection) and its persistent viral ti-
66. The correct answer is E. Herpes infection is a
ter. It has a high afnity for CD4+ cells and
viral infection that can be caused by HSV type
weakens the hosts immune system. Treatment
1 or type 2, and its spectrum of diseases in-
is through antiretroviral medications.
cludes gingivostomatitis, keratoconjunctivitis,
encephalitis, genital disease, and newborn in- Answer G is incorrect. Human papillomavirus
fection. Genital herpes typically presents with can cause epithelial tumors of the skin and mu-
multiple vesicles that can erupt, dysuria, ure- cosa. It can also cause cervical cancer, so rou-
thral discharge, and cervicitis in women. Risk tine Papanicolaou smears are recommended in
factors include unprotected sex with multiple sexually active women. Treatment is through
partners. Suppressive treatment is through an- immune-modifying medications such as imi-
tiviral medications such as acyclovir. Symp- quimod or surgical resection.
tomatic relapses are common throughout the
Answer I is incorrect. Proteus is a member
patients life.
of the family Enterobacteriaceae, the enteric
Answer A is incorrect. Candida albicans is a family of bacteria. It can cause a urinary tract
fungus that can cause yeast infections. It is rel- infection, and the clinical picture is one of dy-
atively common in women and is not an STD. suria, including frequent urination, burning on
It can cause itching and white discharge. Treat- urination, fever, and high WBC counts. Penile
ment is through antifungal medications. discharge would be uncommon. It is also asso-
ciated with kidney stones. Treatment is through
Answer C is incorrect. Escherichia coli is a
antibiotics such as -lactams.
member of the family Enterobacteriaceae, the
enteric family of bacteria. It can cause a uri- Answer J is incorrect. Treponema pallidum is
nary tract infection, and the clinical picture is associated with the STD syphilis. It is a mem-
one of dysuria, including frequent urination, ber of the Spirochaetaceae family of bacteria.
burning on urination, fever, and high WBC Symptoms depend on the stage of the disease,
HIGH-YIELD SYSTEMS
Chapter 8: Infectious Disease Answers 259

but syphilis initially presents as a painless geni- Questions 69 and 70


tal ulceration. Treatment is through antibiot-
69. The correct answer is A. Dysuria, hesitancy,
ics, particularly penicillin.
urgency, and frequency are the hallmarks of
urinary tract infection. So-called honeymoon
Questions 67 and 68 cystitis caused by Staphylococcus saprophyti-
cus is common when acquiring a rst or new
67. The correct answer is E. Primary prophylaxis sexual partner, or when intercourse is frequent.
against Pneumocystis jiroveci pneumonia is indi- Microscopic examination reveals bacteriuria in
cated for patients with CD4+ cell counts <200/ addition to pyuria.
mm. The prophylaxis of choice is one double-
strength tablet of trimethoprim-sulfamethoxazole 70. The correct answer is B. Symptoms of systemic
per day. illness, including fever, nausea, and vomiting,
differentiate pyelonephritis from uncomplicated

Infectious Disease
68. The correct answer is F. Any patient with HIV urinary tract infection. The classic triad of costo-
and recurrent genital herpes outbreaks should vertebral angle tenderness, fever, and nausea/
begin taking 400 mg acyclovir twice a day to vomiting suggests pyelonephritis.
prevent further sequelae.
Answer C is incorrect. Acute urinary retention
Answer A is incorrect. Any patient who has a typically presents with suprapubic pain on pal-
CD4+ cell count <100/mm3 and is seronega- pation and without symptoms of infection.
tive for hepatitis B should be given the vaccine
series. Answer D is incorrect. The classic presenta-
tion of bladder cancer is painless gross hema-
Answer B is incorrect. Any patient with a turia.
PPD test nding that is >5 mm, but who is
found not to have active infection (by x-ray of Answer E is incorrect. Diabetes insipidus is
the chest and three negative cultures), should marked by polyuria, nocturia, and polydipsia
receive this TB prophylactic regimen for 6 and is caused by decreased ADH secretion or
months to prevent reactivation. ADH resistance.

Answer C is incorrect. Each of these patients Answer F is incorrect. Diabetes mellitus is


requires a change in management. marked by polyuria, polydipsia, and polyphagia
and is caused by autoimmune destruction of
Answer D is incorrect. All HIV-positive women cells in pancreas or insulin resistance.
should undergo a Pap smears every 6 months
due to the increased risk of cervical cancer Answer G is incorrect. The classic presenta-
caused by their immunocompromised state. tion of hypercalcemia includes stones, bones,
abdominal groans, and psychic overtones
Answer G is incorrect. Patients with anti- (nephrolithiasis, joint and muscle aches, ab-
Toxoplasma antibodies are at risk for reactivated dominal pain, nausea and vomiting, constipa-
toxoplasmosis when their CD4+ cell counts tion, and depression). Polyuria may also be
are <100/mm3. The prophylaxis of choice is present.
one double-strength tablet of trimethoprim-
sulfamethoxazole per day. Patients with CD4+ Answer H is incorrect. Interstitial cystitis pre-
cell counts in this range should already be tak- sents with symptoms similar to those of uncom-
ing trimethoprim-sulfamethoxazole for P. jiroveci plicated urinary tract infection, but no bacteria
pneumonia prophylaxis. are found on urinalysis, and the symptoms do
not respond to antibiotic therapy.
Answer H is incorrect. Any patient with a PPD
test nding that is >5 mm, or who is consid- Answer I is incorrect. Neurogenic bladder
ered to be at high risk for TB exposure, should may present with either urinary overow leak-
undergo x-ray of the chest. age or urinary retention.
HIGH-YIELD SYSTEMS
260 Section I: Organ Systems Answers

Answer J is incorrect. Psychogenic polydipsia Answer L is incorrect. Urethral stricture pre-


involves drinking excess water and may present sents with obstructive symptoms, including drib-
with polyuria and nocturia. Diagnosis is via a bling, intermittency, incomplete emptying of
water deprivation test. the bladder, and weak urine stream.
Answer K is incorrect. Renal cell carcinoma
often presents with ank pain, palpable ab-
dominal mass, and hematuria.
Infectious Disease
CHAPTER 9

Musculoskeletal

261
HIGH-YIELD SYSTEMS
262 Section I: Organ Systems Questions

Q U E ST I O N S

1. A 48-year-old woman presents to her primary warm bilaterally with decreased exion. Neu-
care physician because of swelling of her hands rologic examination is nonfocal. Laboratory
for the past 3 months. She says her skin feels evaluation reveals a positive antinuclear anti-
rm and tight. For the past 5 years, she notes body titer. Which of the following medications
her hands turn very pale when she goes outside used to treat Wolff-Parkinson-White syndrome
in cold weather, then they turn blue, and then is most likely causing this patients symptoms?
return to a red color. She has a history of gas-
(A) Amiodarone
troesophageal reux disease and is taking
(B) Hydralazine
omeprazole. Recently she has begun having
(C) Isoniazid
pain while swallowing solid foods. On physical
(D) Procainamide
examination she has bilaterally swollen ngers
(E) Sotalol
Musculoskeletal

and hands. The overlying skin appears very


smooth, but on palpation the hands are rm
3. A 4-year-old boy is brought to the pediatrician
and indurated. On the nger pads there are
for evaluation of frequent falls. His mother says
several subcutaneous hard nodules. Which of
that for the past 6 months he seems to be mov-
the following will most likely be elevated in
ing more slowly than usual, and he cant run as
this patient?
quickly and cant climb stairs. The boy was
(A) Anti-centromere autoantibody born at 39 weeks gestation via normal sponta-
(B) Antineutrophil cytoplasmic autoantibody neous vaginal delivery with no complications.
(C) Anti-Scl-70 autoantibody He has been otherwise healthy and achieved
(D) Anti-Smith autoantibody all of his appropriate motor and speech mile-
(E) Anti-SSA autoantibody stones. Family history is signicant for a mater-
nal uncle who died at age 20 years from respi-
2. A 32-year-old man with a history of Wolff- ratory failure. On physical examination the
Parkinson-White syndrome presents to his pri- patient has hyperlordosis of the spine. His
mary care physician because of a rash on his calves are very prominent bilaterally. When
face and joint aches. These complaints started asked to lie on his back and stand up, he rst
about 1 month ago, but got much worse after rolls over onto his stomach and then uses his
spending a long day at the beach a few days hands to climb up his legs until he is standing.
ago. He also has had pain, warmth, and swell- He has 3/5 strength in his shoulders and thighs
ing of his knees for the past few weeks, as well bilaterally, but 5/5 strength in his hands, calves,
as low-grade fevers to 37.938.3C (100.2 and feet. The rest of the neurologic examina-
100.9F) daily for the past 2 weeks. He is tak- tion is unremarkable. Serum creatine kinase
ing a medication to prevent atrial brillation. level is 1500 U/L. How was this boys disorder
Family history is negative for any rheumato- most likely inherited?
logic disorders or malignancy. At present he
(A) Autosomal dominant
has a temperature of 38.3C (100.9F), heart
(B) Autosomal recessive
rate of 84/min, and blood pressure of 118/76
(C) Autosomal trisomy
mm Hg. He has an area of erythema on his
(D) Mitochondrial
cheeks bilaterally that extends across his nasal
(E) X-linked recessive
bridge. His knees are edematous, red, and
HIGH-YIELD SYSTEMS
Chapter 9: Musculoskeletal Questions 263

4. A 7-year-old boy is brought to the pediatricians feels stiff in the morning for at least half an
ofce by his mother. She rst noticed a limp hour. She has trouble getting dressed because
several days ago. The child is in no discomfort it is difcult to put on a shirt or pull up her
and has no history of trauma or recent infec- pantyhose. On physical examination she has
tion. He is otherwise in good health and has decreased active range of motion of her shoul-
reached all developmental milestones on time. ders and hips. She has no muscle tenderness.
Vital signs are within normal limits. Physical Strength is 5/5 in all four extremities. Neuro-
examination reveals limited abduction and in- logic examination is nonfocal. Laboratory tests
ternal rotation of his left leg. The left hip joint show:
shows no evidence of instability or localized
WBC count: 8600/mm
pain. Deep tendon reexes are brisk, pulses are
Hemoglobin: 10.4 g/dL
normal, and muscle strength testing shows 5/5
Platelet count: 610,000/mm
strength bilaterally. A radiograph of the childs
Erythrocyte sedimentation rate: 120 mm/hr
pelvis is shown in the image. What is the most
Creatine kinase: 52 U/L

Musculoskeletal
appropriate treatment for this childs condi-
tion? A short course of prednisone leads to almost
complete resolution of symptoms. Which of
the following is the most likely diagnosis?
(A) Fibromyalgia
(B) Osteoarthritis
(C) Polymyalgia rheumatica
(D) Polymyositis
(E) Rheumatoid arthritis

6. A 15-year-old soccer player runs into a goal


post, protecting his body with an outstretched
left hand. After the game he complains to his
father that his wrist hurts and they stop by the
Reproduced, with permission, from Chen MYM, Pope TL,
local emergency department. The boy screams
Ott DJ. Basic Radiology. New York: McGraw-Hill, 2004: Fig-
ure 7-48. in pain when the emergency physician pal-
pates the oor of the anatomic snuffbox. X-ray
(A) Chemotherapy alone of the left wrist and hand shows no fracture.
(B) Gentle closed reduction What is the best next step in management?
(C) Hospital admission and intravenous antibi-
otics (A) Operative exploration for the source of
(D) Observation and limited weight bearing pain
(E) Surgical intervention with adjuvant che- (B) Removable plaster splint and physical ther-
motherapy apy
(C) Rest, ice, and nonsteroidal anti-inammatory
5. A 69-year-old woman presents to her primary medications
care physician complaining of fatigue and ach- (D) Short arm cast and physical therapy in 46
iness in her shoulders and hips. She says that weeks
over the past 2 months she has lost 4.5 kg (10 (E) Thumb spica cast and repeat x-ray lms of
lb) and feels tired all the time. Her whole body the left wrist and hand in 23 weeks
HIGH-YIELD SYSTEMS
264 Section I: Organ Systems Questions

7. A 24-year-old man presents to the ophthalmol- (A) Decreased ratio of FEV1:FVC


ogist complaining of blurred vision. He notes (B) Increased residual volume
that his left eye has been red and painful for (C) Increased total lung capacity
the past 3 days, and he has not been able to (D) Normal or increased ratio of FEV1:FVC
look at bright lights. On further questioning he and decreased carbon monoxide diffusion
also says he has had pain and swelling of the capacity
right knee for about 1 week, and for the past (E) Normal or increased ratio of FEV1:FVC
month his hips have felt very stiff, especially in and normal carbon monoxide diffusing ca-
the morning. On physical examination his vital pacity
signs are normal, the conjunctiva of his left eye
is severely injected, and his right knee is edem- 9. A 72-year-old man presents to the emergency
atous, erythematous, and warm to touch, with department with pain in his right big toe. The
limited exion and extension of the leg. Addi- pain started this morning and is getting pro-
tionally, his hips are tender to palpation, and gressively worse, and he is unable to bear
Musculoskeletal

he has limited forward and lateral exion of weight on that foot. This is his fth presenta-
the spine. Genitourinary examination reveals tion to the emergency department in the past 7
the man is uncircumcised. Upon retraction of years for similar complaints. He says his doctor
the foreskin, there is a 1 1 cm shallow non- gave him a medication to prevent the attacks,
tender ulcer with well-dened margins adja- but he does not remember the name and has
cent to the urethral meatus. Which of the fol- not taken it in many months. He is febrile, has
lowing will conrm the most likely diagnosis? a pulse of 90/min, and has a blood pressure of
136/86 mm Hg. The rst metatarsophalangeal
(A) Measure serum rheumatoid factor
joint of the right foot is swollen, warm, and ery-
(B) Serum Venereal Disease Research Labora-
thematous. It is exquisitely tender to palpation
tory test
and there is decreased movement. He also has
(C) Stool bacterial culture
several nontender nodules on the medial as-
(D) Urethral swab for gonococcus
pect of the big toes bilaterally, alongside the
(E) Urine polymerase chain reaction for Chla-
rst metacarpophalangeal joints bilaterally,
mydia trachomatis
and on his ears. Which of the following is the
best agent for the acute management of this
8. A 57-year-old man presents to his primary care
patients condition?
physician complaining of shortness of breath
that has been worsening over the past 6 (A) Acetaminophen
months. Initially he had dyspnea on exertion, (B) Allopurinol
but this evolved into dyspnea at rest, as well as (C) Indomethacin
a nonproductive cough for the last month. He (D) Intravenous colchicines
denies weight loss, fevers, or night sweats. On (E) Probenecid
further review of systems he complains of pain
and swelling in his hands. He also notes that 10. A 57-year-old woman presents to her primary
his body feels stiff in the morning. He has no care physician complaining of shortness of
other signicant past medical history and is breath. These symptoms rst began after stren-
taking no medication except for occasional uous exercise approximately 6 months ago, but
ibuprofen for his hand pain. He has never over the past month she has felt short of breath
smoked. On physical examination he has after climbing 10 steps and sometimes at rest.
crackles at the lung bases bilaterally. He has She has no known medical problems, but has
enlarged metacarpophalangeal joints bilater- not seen a physician in 5 years. She is not tak-
ally. They are red, warm, and tender to palpa- ing any medication and is a nonsmoker. Car-
tion. He also has slight ulnar deviation of the diac examination is notable for a loud P2 com-
ngers. What will pulmonary function tests ponent of S2 but no murmurs. Her ngers are
with this patient most likely show? shiny, swollen, and indurated. She also has sev-
eral telangiectasias on her hands and hard,
HIGH-YIELD SYSTEMS
Chapter 9: Musculoskeletal Questions 265

subcutaneous nodules on her nger pads. Lab- 13. A 23-year-old man with new-onset back and
oratory evaluation is signicant for elevated an- buttock pain presents to his primary care physi-
ticentromere antibody but normal antitopoi- cian for evaluation. He states he has morning
somerase I antibody. Which of the following is stiffness in his back that resolves over the
the most likely diagnosis? course of the day. Further testing is negative
for rheumatoid factor and positive for HLA-
(A) Limited cutaneous systemic sclerosis
B27 surface antigen. For which of the follow-
(B) Rheumatoid arthritis
ing conditions is the patient at greatest risk?
(C) Systemic lupus erythematosus
(D) Wegeners granulomatosis (A) Aortitis
(B) Splenomegaly
11. A physician suspects that his patient might (C) Thrombocytopenia
have gouty arthritis. To conrm his clinical (D) Uveitis
suspicion, the physician orders a microscopic (E) Xerostomia
evaluation of the joint uid for the presence of

Musculoskeletal
negatively birefringent, needle-shaped crystals. 14. A 55-year-old white man presents to the emer-
This is known to be a highly specic test. Rela- gency department complaining of diffuse, con-
tive to the physicians clinical diagnosis alone, stant bone pain for the past 3 months. He is a
a highly specic test will greatly reduce which schoolteacher, and denies any recent history of
of the following? trauma or infection. He does note that his fa-
vorite tted baseball hat no longer ts. His
(A) False-negative results
temperature is 37.6C (99.7F), blood pressure
(B) False-positive results
is 120/75 mm Hg, pulse is 85 min, and respira-
(C) Prevalence
tory rate is 22/min. Physical examination re-
(D) True-negative results
veals no focal points of tenderness. Mild fron-
(E) True-positive results
tal bossing and bilateral tibial bowing are
12. A 26-year-old woman presents to her primary noted. X-ray of the skull is shown in the image.
care physician complaining of left lower leg Which of the following is the most likely diag-
pain when she exercises. She states that when nosis?
she is not jogging she is pain free, but when
she jogs >3 miles she begins to note pain and
tightness in her left lower leg. She states that
she often has concurrent numbness and tin-
gling in the top of her foot during these epi-
sodes of pain. When she ceases strenuous activ-
ity, all symptoms slowly subside over the course
of half an hour. Physical examination is unre-
markable and demonstrates a neurovascularly
intact left lower extremity with supple compart-
ments, and no focal areas of tenderness. Plain
lms of the left knee, tibia, and bula are simi-
Reproduced, with permission, from Fauci AS, Braunwald E,
larly unremarkable. Which of the following is Kasper DL, Hauser SL, Longo DL, Jameson LJ, Loscalzo J,
the most likely diagnosis? eds. Harrisons Online. New York: McGraw-Hill, 2008: Figure
(A) Acute compartment syndrome 349-2.
(B) Exertional compartment syndrome (A) Ankylosing spondylitis
(C) Left knee osteoarthritis (B) Pagets disease
(D) Left sided patellofemoral pain syndrome (C) Pituitary adenoma
(E) Left tibial stress fracture (D) Rickets
(E) Scurvy
HIGH-YIELD SYSTEMS
266 Section I: Organ Systems Questions

15. A 45-year-old woman presents to her primary 17. A 75-year-old white man presents to his pri-
care physician complaining of difculty sleep- mary care physician complaining of lower back
ing and extreme fatigue with minimal exertion. and leg pain. The pain is brought on by walk-
These symptoms have been persistent over the ing, has been steadily worsening over the past
past 3 months. She describes her pain as a stiff few years, and is not relieved by nonsteroidal
ache throughout her back and shoulders. Al- anti-inammatory drugs. In the past the pain
though the stiffness typically improves over the was conned only to his lower back and upper
course of the day, any major form of exertion leg/thigh, but in recent months it has started to
leaves her muscles aching. She denies any re- shoot past his knees. He states that the pain is
cent history of trauma or infection. Physical ex- worst with walking, and he denies any recent
amination reveals points of increased tender- trauma or illness. On physical examination his
ness over the medial aspect of the upper border lower extremities are neurovascularly intact,
of the trapezius muscle bilaterally and above with full 5/5 strength, intact sensation, and 2+
the medial border of the scapular spine. X-ray dorsalis pedis and posterior tibial pulses.
Musculoskeletal

lms of the shoulder are normal. Which of the Which is the most appropriate next step in di-
following is the most likely diagnosis? agnosis?
(A) Fibromyalgia (A) CT of the spine
(B) Frozen shoulder syndrome (B) Electromyography of the lower extremities
(C) Malingering (C) Lumbar spine plain lms
(D) Osteoarthritis (D) MRI of the lower back
(E) Rheumatoid arthritis (E) Whole body positron emission tomo-
graphic scan
16. A 35-year-old woman presents to her primary
care physician with complaints of progressive 18. A 62-year-old white woman recently started
weakness over the past month. She says she has pharmacologic therapy for osteoporosis and
noticed an increasing amount of difculty as- now presents complaining of epigastric burn-
sociated with getting out of chairs, climbing ing and dysphagia. From the clinical interview,
stairs, brushing her hair, and lifting her 3-year- it is clear that she is not following dosing in-
old son out of his playpen, an activity that she structions. She is afebrile and results of an ab-
was able to do without difculty prior to the dominal examination are normal. What class
onset of her weakness. She admits to missing 3 of medication is most likely causing the pa-
days of work approximately 6 weeks ago due to tients symptoms?
a bad cold. Physical examination is remark-
(A) Bisphosphonate
able for symmetric proximal muscle weakness.
(B) Calcitonin
Several tests are ordered to establish a diagno-
(C) Oral calcium
sis. Which of the following most strongly sup-
(D) Recombinant parathyroid hormone
ports a diagnosis of polymyositis?
(E) Selective estrogen receptor modier
(A) -Amyloid deposits noted on muscle bi-
opsy
(B) Increased creatine kinase levels
(C) Normal creatine kinase levels
(D) Perifascicular atrophy noted on muscle
biopsy
(E) Positive rheumatoid factor
HIGH-YIELD SYSTEMS
Chapter 9: Musculoskeletal Questions 267

19. A 75-year-old man presents to his family physi- (A) Application of an external xation device
cian with a 3-day history of joint pain. The pa- (B) Exploratory laparotomy with packing of
tient has had various aches and pains in his the pelvis
joints over the years, but in the past 3 days he (C) Open reduction and internal xation of
has noticed that his rst metatarsophalangeal the fracture
(MTP) joint in his left foot has become very (D) Pelvic CT with contrast
painful. He describes the pain as a dull, con- (E) Surgical exploration of pelvic hematoma
stant ach, 7 of 10 in severity. He denies any re-
cent trauma or exposure to infectious agents. 21. A 28-year-old woman presents to her primary
The remainder of his musculoskeletal exami- care physician complaining of a rash and hair
nation is normal, with full range of motion and loss. She says that while on a cruise last month,
no focal tenderness in any of his other joints. she got a bad sunburn on her cheeks and nose,
The rst MTP in the left foot is warm, ery- but the redness has not subsided. She also
thematous, and extremely tender to touch. notes that she has been losing large clumps of

Musculoskeletal
Synovial uid is clear with no WBCs or RBCs, hair in the shower for the past several weeks.
and a Gram stain is negative. Results of bone On further review of systems she notes feeling
biopsy are shown in the image. Which of the very fatigued for the past few months and has
following is the most likely diagnosis? lost 4.5 kg (10 lb). She has a temperature of
38.3C (100.9F), heart rate of 92/min, and
blood pressure of 110/72 mm Hg. Her muscles
are diffusely tender and her shoulders, knees,
and ankles have decreased passive range of mo-
tion secondary to pain and stiffness. She has 1+
pitting edema in her legs up to the mid-calf bi-
laterally. Laboratory work-up reveals positive
antinuclear antibody and positive anti-double-
stranded DNA antibody. Which of the follow-
ing would most likely be found in this patient?
(A) Anticentromere antibodies
(B) Antineutrophil cytoplasmic antibodies
Image courtesy of PEIR Digital Library (http://peir.net).
(C) Leukopenia
(A) Acute gout (D) Macrocytic anemia
(B) Osteoarthritis (E) Thrombocytosis
(C) Rheumatoid arthritis
(D) Septic arthritis 22. A 26-year-old woman was diagnosed with rheu-
(E) Systemic lupus erythematosus matoid arthritis 1 month ago. She returns to
her primary care physician for a follow-up visit
20. A 26-year-old woman was hit by a car and and is interested in learning how rheumatoid
brought to the trauma bay with a blood pres- arthritis might affect her, other than the joint
sure of 62/40 mm Hg. On examination she is destruction it can cause. The patient is most
unresponsive, her pupils are equally round and likely to develop which of the following?
reactive, her distal pulses are weak, and her ex-
(A) Myocarditis
tremities are cold. X-ray of the chest is normal.
(B) Nodules
Peritoneal lavage shows no blood in the abdo-
(C) Pericarditis
men. After receiving 2 L of lactated Ringers
(D) Renal failure
solution, her blood pressure increases to 71/46
(E) Splenomegaly
mm Hg. The pelvis is unstable with compres-
sion, and an anteroposterior x-ray of the pelvis
conrms a pelvic fracture. What is the best
next step in management?
HIGH-YIELD SYSTEMS
268 Section I: Organ Systems Questions

23. A 9-year-old girl presents to her pediatrician and blood pressure is 119/96 mm Hg. The pa-
with a 3-week history of spiking fevers at night tient is extremely tender over her temples, and
up to 39.4C (102.9F). The fevers usually abnormally thickened temporal arteries are
subside with acetaminophen or ibuprofen but palpable bilaterally. Laboratory tests demon-
eventually return. She has also been complain- strate an erythrocyte sedimentation rate of 110
ing of pain in her legs. Furthermore, she has mm/hr. What is the most appropriate initial
developed a salmon-colored rash on her chest step in treatment?
and arms that ares every time she has a fever
(A) Acetazolamide
spike. Her temperature is 38.3C (100.9F),
(B) Atenolol
heart rate is 96/min, and blood pressure is
(C) Methotrexate
104/56 mm Hg. Bilateral anterior cervical and
(D) Nifedipine
axillary lymphadenopathy is present. Cardiac
(E) Oral prednisone
examination reveals a regular rate and rhythm
with no murmurs or rubs. The abdomen is soft
Musculoskeletal

and nontender, and her liver is palpable 3 cm E X T E N D E D M ATC H I N G


below the costal margin. Her right knee and
left ankle are swollen and warm, with de- The response options for the next 2 items are
creased range of motion. Which of the follow- the same. Select one answer for each item
ing is the most likely diagnosis? in the set.
(A) Acute rheumatic fever
(B) Juvenile rheumatoid arthritis For each of the following patients with joint com-
(C) Parvovirus B19 infection plaints, identify the type of arthritis.
(D) Reiters syndrome (A) Ankylosing spondylitis
(E) Septic arthritis (B) Fibromyalgia
(C) Gonococcal arthritis
24. A 40-year-old man presents to his primary care (D) Gout
physician with a chief complaint of lower back (E) Osteoarthritis
pain. He is not particularly concerned about (F) Polymyalgia rheumatica
the pain, as it is not causing him signicant (G) Psoriatic arthritis
distress. Which of the following would be most (H) Reactive arthritis
reassuring to the physician? (I) Reiters syndrome
(A) Age >50 years (J) Rheumatoid arthritis
(B) Duration of symptoms >1 month (K) Systemic lupus erythematosus
(C) History of prostate cancer
(D) History of recurrent urinary tract infection 26. A 65-year-old man presents to the emergency
(E) Relief of pain with nonsteroidal anti- department with severe pain (rated 10 of 10) in
inammatory drugs his right rst toe. The pain started suddenly
last night with no precipitating trauma. His toe
25. A 74-year-old woman presents to her primary is extremely tender to the touch, and is very
care physician with a 1-month history of tem- warm and swollen.
poral headaches, fevers, and general malaise.
The headaches are described as a throbbing 27. A 58-year-old woman has pain and stiffness in
pain, especially localized to her temporal re- her hands that increases throughout the day.
gions bilaterally. She denies any recent trauma Physical examination shows bony enlargement
or exposure to infectious agents. Her tempera- of the distal interphalangeal joints. X-rays of
ture is 38.0C (100.4F), heart rate is 82/min, the hands show joint space narrowing with sub-
chondral sclerosis.
HIGH-YIELD SYSTEMS
Chapter 9: Musculoskeletal Questions 269

The response options for the next 3 items are


28. A 27-year-old man presents to the clinic be-
the same. Select one answer for each item
cause of several weeks of malaise, fatigue, and
in the set.
low-grade fever. He mentions that he has had
some testicular and abdominal pain after eat-
Match each of the following clinical scenarios with ing. He also states that he has been tripping
the most appropriate diagnosis. over his feet, and a neurologic examination
demonstrates a right foot drop. His history is
(A) Behets syndrome signicant for hepatitis B virus infection.
(B) Churg-Strauss vasculitis
(C) Disseminated intravascular coagulation 29. A 52-year-old woman with recurrent sinusitis
(D) Giant cell arteritis and a chronic cough develops hemoptysis.Uri-
(E) Hemolytic-uremic syndrome nalysis shows microhematuria, and a kidney bi-
(F) Henoch-Schnlein purpura opsy demonstrates granulomatous inamma-
(G) Idiopathic thrombocytopenic purpura tion within the arterial walls.

Musculoskeletal
(H) Kawasakis disease
(I) Microscopic polyangiitis 30. A 36-year-old man with asthma and chronic al-
(J) Polyarteritis nodosa lergies presents to his physician with monon-
(K) Takayasus arteritis europathy of his ulnar nerve and mild pleuritic
(L) Thromboangiitis obliterans chest pain. His differential demonstrates 11.7%
(M)Thrombotic thrombocytopenic purpura eosinophils. X-ray of the chest shows a right-
(N) Wegeners granulomatosis sided pulmonary inltrate.
HIGH-YIELD SYSTEMS
270 Section I: Organ Systems Answers

AN S W E R S

1. The correct answer is A. This patient has symp- Answer D is incorrect. Anti-Smith autoantibod-
toms and signs suggestive of systemic sclerosis, ies are elevated in patients with systemic lupus
or scleroderma. The swollen, indurated ngers erythematosus (SLE). SLE is an autoimmune
and hands are a classic nding in cutaneous sys- disorder that most commonly affects young
temic sclerosis. The fact that the skin ndings women. It can cause widespread systemic dis-
are limited to the hands and do not extend prox- ease including renal failure, central nervous
imally indicates that this woman has limited cu- system disturbances, and pulmonary disease.
taneous systemic sclerosis (LCSS), as opposed There is a wide range of clinical manifestations
to diffuse disease. LCSS is typically associated of SLE, but common presenting symptoms in-
with the CREST syndrome: Calcinosis, Ray- clude fever, fatigue, weight loss, malar rash,
nauds phenomenon, Esophageal dysmotility, photosensitivity, alopecia, arthritis, myalgias,
Musculoskeletal

Sclerodactyly, and Telangiectasias. This patient pleuritic chest pain, and neurologic changes.
demonstrates many of the components of Patients with SLE may also have elevated anti-
CREST, including nodules in the nger pads, double-stranded DNA autoantibodies.
which represent calcic deposits in the subcuta-
Answer E is incorrect. Anti-SSA is elevated
neous tissue. She also has Raynauds phenome-
in patients with Sjgrens syndrome, which is
non, or episodic vasoconstriction of the small ar-
characterized by dry eyes, dry mouth, and en-
teries in the ngers, causing pallor and cyanosis
larged salivary glands. It may also be elevated
when patients are exposed to cold temperatures
in patients with SLE, but is not associated with
or emotional stress, and rubor on rewarming.
systemic sclerosis.
She also has a history of reux and dysphagia,
which are signs of esophageal dysmotility. LCSS
2. The correct answer is D. This patient is pre-
is typically associated with elevated levels of
senting with a lupus-like syndrome. He has
anti-centromere and/or anti-nucleolar autoanti-
malar rash, fevers, and arthritis, all signs that
bodies.
are seen in patients with lupus. Patients with
Answer B is incorrect. Antineutrophil cytoplas- drug-induced lupus usually have positive anti-
mic autoantibody (ANCA) is typically found in nuclear antibodies, but tend to have a less se-
patients with Wegeners granulomatosis, a vas- vere presentation compared to patients with
culitis that affects medium- and small-sized ves- SLE. The syndrome usually resolves several
sels. Wegeners classically affects the lungs and weeks after discontinuing the medication. Pro-
kidneys. Pulmonary symptoms include cough, cainamide is a class IA antiarrhythmic drug
dyspnea, and hemoptysis. Renal involvement that is used in some patients with Wolff-Parkin-
is usually a pauci-immune glomerulonephritis. son-White (WPW) syndrome to prevent atrial
ANCA may also be positive in microscopic poly- brillation, and can cause a drug-induced lu-
angiitis, Churg-Strauss syndrome, and several pus-like syndrome. Other drugs that cause it
other vasculitides, but it is not associated with include hydralazine, isoniazid, minocycline,
systemic sclerosis. propylthiouracil, lithium, carbamazepine, and
phenytoin.
Answer C is incorrect. Anti-Scl-70 is fre-
quently elevated in patients with diffuse cu- Answer A is incorrect. Amiodarone is a class
taneous systemic sclerosis (DCSS). These III antiarrhythmic agent that is sometimes used
patients will have skin ndings extending prox- to treat patients with WPW syndrome. Its major
imally past the wrists. They will also lack other use is to prevent orthodromic atrioventricular re-
signs of CREST syndrome that are seen in this entrant tachycardia. Amiodarone can be a very
patient. Patients with DCSS are at increased toxic medication, but it is not typically associ-
risk of pulmonary brosis, the leading cause ated with a drug-induced lupus-like syndrome.
of death in patients with scleroderma. Patients Adverse effects of amiodarone include pulmo-
with DCSS may also have cardiac abnormali- nary toxicity, thyroid dysfunction, liver toxicity,
ties and renal disease. ocular disease, and gastrointestinal upset.
HIGH-YIELD SYSTEMS
Chapter 9: Musculoskeletal Answers 271

Answer B is incorrect. Hydralazine is an an- be a carrier of the mutation. Therefore, she


tihypertensive medication that is frequently as- would also be expected to have the disease. Au-
sociated with an increased risk of developing tosomal dominant disorders include Hunting-
drug-induced lupus. However, it is not a drug tons disease and neurobromatosis.
that is used to manage arrhythmias in WPW
Answer B is incorrect. Although this family
syndrome.
history might suggest an autosomal recessive
Answer C is incorrect. Isoniazid is an anti- pattern of inheritance, DMD is always inher-
biotic that is associated with causing a drug- ited as an X-linked recessive condition. Com-
induced lupus-like syndrome. However, it is mon autosomal recessive disorders include cys-
not used to treat WPW syndrome. Isoniazid tic brosis and sickle cell disease.
is most commonly used in the treatment and
Answer C is incorrect. Autosomal trisomies in-
prophylaxis of tuberculosis.
clude trisomy 13, 18, and 21. These disorders
Answer E is incorrect. Sotalol is a class III an- present with many congenital anomalies at

Musculoskeletal
tiarrhythmic agent that may be used to treat birth, including cardiac defects, limb abnor-
patients with WPW syndrome; however, it does malities, and craniofacial dysmorphia.
not cause a lupus-like syndrome. The most
Answer D is incorrect. Mitochondrial disor-
common adverse effect of sotalol is cardiac tox-
ders are inherited from the mother. The mi-
icity, including bradycardia and arrhythmias.
tochondrial myopathies can also affect muscle
strength, but the mother and sister would also
3. The correct answer is E. This patient is pre-
be affected, because all offspring of affected fe-
senting with signs and symptoms suggestive of
males show signs of disease.
Duchennes muscular dystrophy (DMD).
DMD is present at birth, but typically does not
4. The correct answer is D. This child is suffering
begin to manifest until age 35 years. DMD
from Legg-Calv-Perthes disease. This condition
classically affects the proximal musculature, so
consists of idiopathic avascular necrosis of the
patients will have difculty climbing stairs.
femoral head as seen in the radiograph. It most
This patient is displaying a positive Gowers
commonly affects boys 410 years old and pre-
sign. However, patients with proximal muscle
sents as a painless limp when symptomatic. The
weakness will roll to the prone position and use
disease is usually self-limiting with symptoms
their arms to climb up their legs, as opposed to
lasting <18 months. Observation and limited
using truncal muscles to get into a sitting posi-
weight bearing are the treatment choices for this
tion and then thigh muscles to rise to standing.
condition. Management by an experienced or-
Patients with DMD develop hyperlordosis and
thopedist is appropriate.
scoliosis because of weakness of truncal mus-
cles. The chest deformity, along with muscle Answer A is incorrect. This patient has a clas-
weakness, impairs respiratory function, and pa- sic case of Legg-Calv-Perthes disease, which
tients often require tracheostomy. Calves are does not require chemotherapy.
enlarged due to pseudohypertrophy. Creatine Answer B is incorrect. Gentle closed reduc-
kinase (CK) is elevated, indicating a primary tion is appropriate management for acutely
muscle disorder as opposed to a neurologic dis- displaced slipped capital-femoral epiphysis.
order. Death occurs in the third decade due to The child is not suffering from this condition,
respiratory failure or cardiomyopathy. DMD is which typically presents as a painful limp and
inherited in an X-linked recessive fashion. inability to bear weight on the affected side in
Clues in the family history include a maternal an overweight or obese boy 1113 years old
uncle who died of respiratory failure in his 20s, and is commonly seen in the African-American
and normal, unaffected females. population.
Answer A is incorrect. In autosomal dominant Answer C is incorrect. Hospitalization and in-
disorders, the disease will affect any individual travenous antibiotic therapy is the treatment of
carrying a single mutation. Unless this boy has choice for osteomyelitis. Although osteomyeli-
a new, spontaneous mutation, his mother must tis has an insidious onset and is often seen in
HIGH-YIELD SYSTEMS
272 Section I: Organ Systems Answers

children <5 years old, several signs are absent tory pain, which is most severe in the morning,
in this patient that would point to such a diag- after an extended period of inactivity. Patients
nosis, such as constitutional symptoms, pain, with OA would not be expected to have an el-
and localized swelling. evated erythrocyte sedimentation rate or rapid
response to steroids.
Answer E is incorrect. Surgical intervention
with adjuvant chemotherapy is appropriate Answer D is incorrect. Polymyositis is another
treatment for osteosarcoma. This boy is un- rheumatologic disorder that typically presents
likely to have osteosarcoma, given his age and with proximal muscle weakness. Patients may
symptoms. Osteosarcoma is typically seen in also complain of difculty getting dressed,
men 2030 years old, is located typically in the but this is due to actual muscle weakness as
distal femur, and often causes constitutional opposed to joint stiffness, as seen in PMR.
symptoms and pain that is worse at night. Patients with polymyositis typically do not
complain of pain in the hip girdle. On exami-
5. The correct answer is C. This patient has a nation, they will have muscle tenderness and
Musculoskeletal

clinical presentation suggestive of polymyalgia diminished muscle strength, but should have
rheumatica (PMR). PMR is a disorder that is full range of motion of all joints including the
linked to temporal (giant cell) arteritis, and al- shoulders. Laboratory ndings include elevated
most exclusively occurs in patients >50 years muscle enzymes such as CK.
old. PMR typically presents as subacute or
Answer E is incorrect. Rheumatoid arthritis
chronic onset of symmetrical pain and morn-
(RA) may present with similar symptoms as
ing stiffness in large proximal joints including
PMR, including symmetric polyarthritis and
the shoulder, hip girdle, and neck. Patients
morning stiffness. However, RA tends to in-
may report trouble getting dressed because of
volve more joints than PMR, and also involves
stiffness in these joints. Patients may also have
smaller joints including the hands and feet,
systemic symptoms including weight loss and
which are typically spared in PMR. In addi-
fatigue. When present, the pain of PMR is due
tion, RA is usually a difcult disease to man-
to synovitis and bursitis of the joints, rather
age and requires high-dose steroids or other
than actual muscle tenderness as the name im-
disease-modifying antirheumatic drugs, so this
plies. The classic laboratory nding in both
patients rapid response to low-dose steroids is
PMR and temporal arteritis is elevated erythro-
unlikely to be seen in a patient with RA.
cyte sedimentation rate, usually <40 mm/hr
and sometimes exceeding 100 mm/hr. The
6. The correct answer is E. The scaphoid is the
rapid response to prednisone further conrms
most commonly fractured carpal bone. It artic-
the diagnosis, as both PMR and temporal ar-
ulates with the distal radius, trapezium, and
teritis typically have a dramatic response to
capitate, and is restricted from motion during
even low-dose steroids.
radial deviation and dorsiexion (outstretched
Answer A is incorrect. Patients with bromy- arm). Thus, any forceful stress exerted on the
algia tend to be <50 years old and complain scaphoid in this position results in fracture.
of diffuse musculoskeletal pain. Physical ex- Tenderness, swelling, or bruising in the ana-
amination typically reveals muscle tenderness. tomic snuffbox, a triangular depression on the
Laboratory work-up is unremarkable, includ- dorsal aspect of the hand bordered by the exten-
ing a normal erythrocyte sedimentation rate. sor and abductors of the thumb, can be highly
Fibromyalgia would not rapidly resolve after a indicative of a scaphoid fracture. Initial plain
short course of steroids. lm radiographs do not always detect scaphoid
fractures, especially in nondisplaced fractures.
Answer B is incorrect. Osteoarthritis (OA) is a
Thus, as a general rule, a patient with a clini-
very common cause of joint pain in older pa-
cally suspected scaphoid fracture but negative
tients. OA commonly involves the hips, knees,
initial x-ray is treated with a short arm thumb
spine, and hands. The pain associated with
spica splint and reevaluated in 2 weeks. Repeat
OA tends to worsen throughout the day with
increasing activity, as opposed to inamma-
HIGH-YIELD SYSTEMS
Chapter 9: Musculoskeletal Answers 273

x-ray lms are important to detect fractures of 7. The correct answer is E. This patient is pre-
the proximal third of the scaphoid, which are senting with the signs and symptoms of classic
associated with avascular necrosis. Failure to reactive arthritis, which is one of the spondy-
accurately diagnose and treat a scaphoid frac- loarthropathies. Reactive arthritis is character-
ture may result in a variety of adverse outcomes ized by a triad of uveitis, urethritis, and arthritis
including nonunion, delayed union, decreased (cant see, cant pee, cant climb a tree). This
grip strength, decreased range of motion, and patients complaints are consistent with uveitis
OA of the radiocarpal joint. (blurred vision, photophobia, and pain) and
peripheral inammatory arthritis (symmetric,
Answer A is incorrect. Surgery is not indicated
of the large joints of the lower limbs). The mu-
at this time since the initial x-ray lm did not
cosal ulceration seen at the urethral meatus is
reveal a fracture. Open reduction is indicated
characteristic of circinate balanitis, a lesion
for displaced fractures and nonunions.
caused by Chlamydia trachomatis, the organ-
Answer B is incorrect. A plaster splint is made ism implicated in the pathogenesis of reactive

Musculoskeletal
from various crinoline-type material strips im- arthritis. Although some patients may report a
pregnated with plaster that crystallizes or sets history of penile discharge, the infection can
when water is added. It provides temporary also be asymptomatic, especially in uncircum-
immobilization to improve pain and discom- cised males, who may not notice the nontender
fort, minimize blood loss, decrease the risk of lesion. Females may have asymptomatic chla-
fat emboli, and prevent further neurovascular mydial infection that precedes the onset of re-
injury associated with fractures. However, in active arthritis.
this case a fracture should be assumed and full-
Answer A is incorrect. Rheumatoid factor is a
time immobilization applied until a scaphoid
marker found in many patients with RA as well
fracture can be ruled out weeks later.
as several other rheumatologic conditions. Al-
Answer C is incorrect. Soft-tissue injuries not though RA can present with morning stiffness,
associated with bony fractures include sprains, the arthritis is typically symmetric and affects
strains, contusions, tendinitis, and bursitis. many joints. Typically it involves small joints
Traumatic injury or repeated overuse can lead of the hands as opposed to a single large joint
to this type of injury. Treatment for most soft- of the lower extremity. Also, RA typically does
tissue injuries includes rest for a certain time not cause back pain or limited movement of
period, ice therapy, and nonsteroidal anti- the spine. The combination of ndings in this
inammatory drugs (NSAIDs). The patients patient is much more suggestive of reactive ar-
mechanism of injury and physical examination thritis than RA.
ndings are highly suspicious for a scaphoid
Answer B is incorrect. The Venereal Disease
fracture and thus warrant splint immobiliza-
Research Laboratory test is used to test for
tion. Ice and NSAIDs can be used as adjunc-
syphilis. Primary syphilis can cause a painless
tive therapy.
ulceration on the genitalia called a chancroid.
Answer D is incorrect. A short arm cast is ap- However, early syphilis is not associated with
plied below the elbow to the hand and does the other ndings seen in this patient, such as
not adequately immobilize the scaphoid. It is uveitis, sacroiliitis, and peripheral arthritis.
primarily indicated for forearm or wrist frac-
Answer C is incorrect. Reactive arthritis pre-
tures, or to immobilize the forearm or wrist
sents similarly to classic reactive arthritis, with
muscles and tendons in place following sur-
inammatory back pain and asymmetric oli-
gery. In addition, repeat x-ray lms are needed
goarthritis affecting the large joints. Common
sooner than 4 weeks to determine whether
pathogens implicated in reactive arthritis in-
there is a scaphoid fracture, so that appropriate
clude Shigella, Campylobacter, Yersinia, and
measures can be undertaken in a timely fash-
Salmonella. However, the lack of diarrhea and
ion to prevent complications arising from this
the presence of a genital ulcer and uveitis are
type of injury.
more suggestive of reactive arthritis due to
chlamydial infection.
HIGH-YIELD SYSTEMS
274 Section I: Organ Systems Answers

Answer D is incorrect. Gonococcal infection Answer E is incorrect. Normal or increased ra-


can cause urethritis and arthritis. The arthritis tio of FEV1:FVC with a normal carbon mon-
caused by gonococcus involves large joints. oxide diffusion capacity can be seen in patients
However, gonococcus does not typically cause with restrictive lung disease due to extrinsic dis-
inammatory back pain, and the genital lesion orders. Disorders of the chest wall or pleura, or
seen on this patient is more consistent with neuromuscular disorders, can decrease TLC.
chlamydial infection. Gonococcus is more As with intrinsic lung disorders that cause a
likely to cause a purulent urethritis. restrictive pattern, the FEV1:FVC in patients
with extrinsic disorders will be normal to in-
8. The correct answer is D. This patient is pre- creased. This is because both FEV1 and FVC
senting with classic signs and symptoms of RA. are reduced in relation to an overall decreased
The combination of RA with dyspnea and TLC. However, the diffusion capacity will be
crackles is suggestive of RA-associated intersti- normal, because the lung tissue itself is not in-
tial lung disease, the most common manifesta- volved in the pathology.
Musculoskeletal

tion of rheumatoid lung disease. Patients with


RA-associated interstitial lung disease present 9. The correct answer is C. This patient is pre-
with a clinical picture similar to idiopathic pul- senting with acute gouty arthritis. In the absence
monary brosis, including dyspnea, nonpro- of contraindications, the initial drug of choice in
ductive cough, crackles, and clubbing in late a patient with acute gouty arthritis of less than
disease. Lung disease is most common in pa- 24 hours duration is a potent NSAID such as
tients 5060 years old and is more common in indomethacin. Indomethacin can rapidly im-
men. Interstitial lung disease causes a restric- prove pain and swelling and prevent progressive
tive pattern on pulmonary function testing. To- inammation. Another anti-inammatory agent
tal lung capacity (TLC), functional residual that can be used is oral colchicine, but it is
capacity, and residual volume are all de- sometimes reserved for patients who are intoler-
creased. FEV1 and FVC are also decreased, ant of NSAIDs or those who have had prior suc-
but these reductions are in relation to a de- cess with colchicine, because the drug has many
creased total lung capacity, so the ratio of adverse effects, including diarrhea, at therapeu-
FEV1:FVC is either normal or slightly in- tic levels. Other options include intra-articular
creased. The carbon monoxide diffusion ca- steroid injections or oral prednisone, but pa-
pacity is reduced due to inammation and tients may have rebound inammation after
scarring of the lung tissue. withdrawal of oral steroids.
Answer A is incorrect. Decreased ratio of Answer A is incorrect. Acetaminophen is not
FEV1:FVC is found in patients with obstruc- used in treating acute gouty arthritis because it
tive lung disease, most commonly due to does not decrease inammation.
chronic obstructive pulmonary disease. Al-
Answer B is incorrect. Allopurinol decreases
though there are a variety of obstructive lung
the production of uric acid by inhibiting the
diseases associated with rheumatoid arthritis,
enzyme xanthine oxidase. It is very effective
rheumatoid arthritis-associated interstitial lung
in decreasing uric acid levels in patients who
disease is much more common. Although pa-
have hyperuricemia secondary to overproduc-
tients with chronic obstructive pulmonary
tion. However, allopurinol should never be
disease may present with cough and dyspnea,
started during an acute gouty attack. Initiation
this diagnosis is less likely in a patient who has
of therapy with uric acid-lowering drugs during
never smoked.
an acute attack can lead to a more intense and
Answer B is incorrect. Patients with restrictive prolonged episode. Allopurinol should not be
lung disease have decreased residual volumes. started until a few weeks after the acute attack
resolves, and it should be given with colchicine
Answer C is incorrect. Patients with interstitial
to prevent recurrence.
lung disease have a decreased, not increased,
TLC.
HIGH-YIELD SYSTEMS
Chapter 9: Musculoskeletal Answers 275

Answer D is incorrect. Intravenous colchicine tions, including pulmonary hypertension, in-


can be very effective in treating acute gouty ar- terstitial brosis, pneumonitis, pleurisy, and
thritis. However, there is a high rate of toxic- alveolar hemorrhage. SLE typically affects
ity including gastrointestinal upset, myopathy, young women and can cause widespread sys-
agranulocytosis, and even sudden death. Its use temic disease, including fever, fatigue, weight
is limited to hospitalized patients with polyar- loss, malar rash, photosensitivity, alopecia, ar-
ticular gout who cannot tolerate NSAIDs or thritis, myalgias, neurologic changes, and renal
oral medications. It is contraindicated in pa- failure. This patient does not have any other
tients with leukopenia, hepatic disease, or renal clinical signs suggestive of SLE. In addition,
insufciency. It is not rst-line therapy. patients with SLE have elevated antinuclear,
anti-double-stranded DNA, and/or anti-Smith
Answer E is incorrect. Probenecid is a drug
autoantibodies, not anticentromere antibody.
that decreases serum uric acid by inhibiting
urate reabsorption in the kidney. Probenecid Answer D is incorrect. Wegeners granuloma-
may be used for long-term prophylaxis in pa- tosis is a systemic vasculitis of medium- and

Musculoskeletal
tients with hyperuricemia due to decreased small-size vessels that classically affects the
excretion. Like allopurinol, probenecid should lungs and kidneys. Pulmonary symptoms in-
not be started during an acute attack because it clude cough, dyspnea, and hemoptysis. X-ray
can intensify and prolong the episode. of the chest may show cavitating nodules, al-
veolar inltrates, and diffuse hazy opacities.
10. The correct answer is A. This patient has signs Patients with Wegeners do not typically have
suggestive of systemic sclerosis, or scleroderma. pulmonary hypertension. Wegeners is associ-
Pulmonary disease occurs in up to 70% of pa- ated with elevated levels of ANCA.
tients with scleroderma, and it is the leading
cause of mortality in these patients. Pulmonary 11. The correct answer is B. Specicity is dened
hypertension is a more rare complication of as the proportion of disease-free persons appro-
scleroderma, but patients with it may present priately classied by the screening test as nega-
with dyspnea and x-ray evidence of an enlarged tive. A test with high specicity is useful in con-
pulmonary artery. She most likely has LCSS, rming a diagnosis because a highly specic test
which is limited to sclerosis distal to the wrists, will have few results that are false-positive. In
and is often associated with CREST syndrome this case, the joint uid evaluation is used to
(Calcinosis, Raynauds phenomenon, Esopha- conrm the physicians clinical suspicion and
geal dysmotility, Sclerodactyly, and Telangi- decrease the likelihood that a patient without
ectasias) and anticentromere antibodies. gouty arthritis will mistakenly receive that diag-
nosis.
Answer B is incorrect. RA can also cause pul-
monary disease that presents with dyspnea. Answer A is incorrect. False-negatives are dis-
The most common manifestation of RA lung eased persons who are misclassied as nega-
disease is interstitial pulmonary brosis, but pa- tive. This parameter does not decrease with a
tients may also have pleural disease, lung nod- highly specic test. High sensitivity, however,
ules, and vasculitis. Patients with RA present would by denition reduce the proportion of
with painful swelling of the joints and morning false negatives.
stiffness that classically affects the metacarpo-
Answer C is incorrect. Specicity of a test is
phalangeal and proximal interphalangeal joints
a property of the screening test and is not as-
of the hands. Patients with RA do not have
sociated with the prevalence of the disease in a
shiny, rm skin. In addition, anticentromere
population.
antibody is not typically elevated in patients
with RA. Answer D is incorrect. True-negatives are
healthy persons who are correctly classied as
Answer C is incorrect. Patients with SLE can
being without disease. This parameter would
have a wide range of pulmonary manifesta-
increase with a highly specic test.
HIGH-YIELD SYSTEMS
276 Section I: Organ Systems Answers

Answer E is incorrect. True-positives are dis- cal injuries. Patients suffering from any of the
eased persons who are correctly classied as patellofemoral knee pain syndromes usually
positive. This parameter does not decrease complain of anterior knee pain, and do not
with a highly specic test, although it would present in the manner described in the stem.
increase with a highly sensitive test.
Answer E is incorrect. Stress fractures are tiny
cracks in bone that result from overuse. Fa-
12. The correct answer is B. Exertional compart-
tigued muscles and increasing the amount or
ment syndrome is a condition in which the pa-
intensity of an activity too rapidly may cause
tient experiences pain over the anterior lower
these cracks. Most stress fractures occur in the
leg caused by a pressure build-up within the
weight-bearing bones of the lower leg and the
muscles of the leg. Patients typically complain
foot. Treatment consists of ice, NSAIDs, and
of pain after a period of activity or exercise, and
rest for 68 weeks which allows ample time for
it is quickly relieved by rest. As blood ow to
healing. Evidence of fracture may never ap-
the muscle increases with activity, the muscle
pear on plain radiographs or may not appear
Musculoskeletal

swells and becomes constricted by the encom-


for 210 weeks after symptom onset, although
passing fascia. Pain results from the ensuing
triple-phase nuclear bone scans are more sensi-
ischemia. There may also be associated numb-
tive in the detection of stress fractures early in
ness in the dorsum of the foot or weakness on
the clinical course. A patient with a tibial stress
dorsiexion at the ankle. The diagnosis is made
fracture would not be pain-free within 30 min-
by measuring pressures within the leg at rest
utes of cessation of activity, nor would the pa-
followed by a reading after some exercise.
tient likely experience a temporary loss of sen-
Treatment consists of a surgical fasciotomy,
sation over the dorsum of the foot.
which involves release of the tight fascia.
Answer A is incorrect. Acute compartment 13. The correct answer is D. The patients demo-
syndrome differs from exercise-induced or graphic information, presentation, and study
chronic compartment syndrome in that the results are most consistent with a diagnosis of
former occurs secondary to a traumatic injury ankylosing spondylitis (AS). AS is a chronic in-
such as a fracture of one of the long bones or ammation of the sacroiliac joints and the
a crush injury. The patient would present with spine, which can eventually lead to fusion of
severe pain and clinically tight compartments the vertebrae. Patients experience back pain
at the time of examination with associated par- and morning stiffness. Systemic ndings are as-
esthesias. Treatment involves immediate fas- sociated with the disease, including uveitis in
ciotomy to prevent cell death. approximately 30% of affected patients. Other
associated conditions include cataracts, sec-
Answer C is incorrect. Pain associated with
ondary glaucoma, aortic insufciency, and
knee OA would not likely resolve within a half
prostatitis in men. Patients are generally man-
hour of cessation of exercise. Patients typically
aged with NSAIDs and specic exercise regi-
experience pain, soreness, and swelling with
mens, and may require hip replacements or
activity that does not necessarily resolve im-
spinal surgery later in the course of disease.
mediately after rest. Treatment includes ice,
NSAIDs, and limited activity. In addition, the Answer A is incorrect. Aortitis, or inamma-
patients age and lack of x-ray ndings make tion of the aorta, can be infectious or autoim-
this diagnosis unlikely. mune. It is associated with AS but affects <5%
of patients with the disease. Affected patients
Answer D is incorrect. Patellofemoral knee
are at risk for a variety of complications of di-
pain most commonly arises from an imbalance
minished aortic ow, including central hy-
or irregularity of patellar movement or track-
pertension from renal artery stenosis, vision
ing. Conditions that predispose to these abnor-
changes, neurologic decits, and claudication.
malities include an imbalance in quadriceps
strength, patella alta, recurrent patellar sublux-
ation, direct trauma to the patella, and menis-
HIGH-YIELD SYSTEMS
Chapter 9: Musculoskeletal Answers 277

Answer B is incorrect. Splenomegaly occurs in ndings (including the bilateral bowing of the
about 10% of patients with SLE but is not com- tibias) above are most consistent with Pagets
monly found in patients with AS. In patients disease.
with SLE splenomegaly is associated with nau-
Answer D is incorrect. Rickets, or osteomala-
sea, diarrhea, and vague abdominal pain.
cia, is the weakening and softening of bones
Answer C is incorrect. Thrombocytopenia is that is frequently associated with decreased vi-
associated with a wide variety of systemic ill- tamin D or calcium intake. Though it can oc-
nesses but is not commonly associated with AS. cur in adults, it occurs most frequently in early
Autoimmune etiologies for thrombocytopenia childhood. This is particularly common in
exist but are not associated with AS. developing countries as a result of famine and
malnourishment.
Answer E is incorrect. Xerostomia (dry
mouth) is commonly associated with Sjgrens Answer E is incorrect. Scurvy is a disorder of
syndrome, not AS. Patients with Sjgrens syn- collagen synthesis typically associated with vita-

Musculoskeletal
drome have sicca symptoms: dryness of the min C deciency. Common manifestations of
oral mucosa, eyes, and other mucosal surfaces scurvy include perifollicular hemorrhages, peri-
of the body (e.g., skin, nose, and vagina). How- follicular hyperkeratotic papules, petechiae and
ever, they generally do not experience progres- purpura, splinter hemorrhages, bleeding gums,
sive back pain and stiffness. hemarthroses, and subperiosteal hemorrhages.

14. The correct answer is B. Pagets disease is char- 15. The correct answer is A. This patient has a
acterized by an increased rate of bone turnover, classic presentation of bromyalgia. Patients
causing both excessive resorption as well as ex- often present with generalized musculoskeletal
cessive formation. The history of diffuse bone aching, with stiffness and fatigue that improves
pain and increasing head size in conjunction over the course of the day. Pain and tightness
with the physical examination ndings of fron- in the neck and upper back, along with areas
tal bossing and tibial bowing are suspicious for of point tenderness, are common symptoms.
Pagets disease. The skull x-rays reveal the clas- Patients may complain of muscle pain and fa-
sic cotton wool appearance found in patients tigue after minimal exertion, poor sleep, awak-
with Pagets disease. Additionally, these patients ing frequently over the course of the night, and
will have an elevated alkaline phosphatase with having difculty falling back to sleep. Physical
normal calcium and phosphate. A majority of examination and radiologic studies do not typi-
those with Pagets disease are asymptomatic and cally demonstrate any joint abnormalities.
do not require treatment. Treatment may be in-
Answer B is incorrect. A patient with frozen
dicated in order to treat symptoms or to prevent
shoulder syndrome would have signicant
complications. Treatment includes calcitonin
decits in range of motion that would be ex-
and bisphosphonates to slow bone resorption.
posed during physical examination. Adhesive
Answer A is incorrect. This disorder is associ- capsulitis, or frozen shoulder, is a progressive
ated with ankylosis/fusion of joints, and does disorder in which patients suffer from shoulder
not t well with the description and ndings pain and decreased range of motion without
above. AS typically presents in the late teens the presence of truly intrinsic shoulder disease.
and early 20s with hip pain and lower back This disorder often follows bouts of bursitis,
pain that is worst in the morning and improves and is characterized by a thickened capsule,
with activity. with mild inammatory changes, leading to
decreased range of motion and increased pain.
Answer C is incorrect. Hormone imbalances
associated with pituitary adenomas may mani- Answer C is incorrect. Patients with bromyal-
fest in a number of ways including acromega- gia are often dismissed as malingerers because
lic bony changes. However, the history and their symptoms are often nonfocal and vague
in nature. It is important not to dismiss cases
of bromyalgia as malingering. Fibromyalgia is
HIGH-YIELD SYSTEMS
278 Section I: Organ Systems Answers

a real medical entity with viable treatment op- heart, and presents with skin ndings such as
tions, including amitriptyline, uoxetine, chlor- heliotrope rash and Gottron papules. Patients
promazine, or cyclobenzaprine. experience proximal muscle weakness, joint
swelling, and occasional muscle tenderness.
Answer D is incorrect. In OA, pain is usu-
ally limited to joints, and radiologic changes, Answer E is incorrect. Positive rheumatoid
including joint space narrowing, osteophytes, factor is not associated with polymyositis, but
and subchondral cysts, are likely to be found. instead with rheumatoid arthritis. Patients be-
The lack of proper history or x-ray ndings gin to experience fatigue, stiffness after waking
goes against the diagnosis of OA. up, diffuse myalgias, and weakness. Joint pain
and swelling follows, particularly in the wrist,
Answer E is incorrect. The diffuse nature of
elbow, ngers, knees, and toes. Rheumatoid ar-
the pain (not limited to joints) and absence of
thritis classically affects the proximal joints of
radiographic ndings make RA less likely. In
the hand.
RA, pain usually affects joints, not muscle. In
Musculoskeletal

addition, one might expect to nd radiologic


17. The correct answer is D. Patients with lumbar
changes, including joint space narrowing in
spinal stenosis present when the narrowed spi-
RA. Therefore, the diagnosis of RA is unlikely.
nal canal and degenerative joint changes com-
press nerve roots and become symptomatic.
16. The correct answer is B. Polymyositis, derma-
The best diagnostic conrmation of this disor-
tomyositis, and inclusion body myositis are all
der is lumbar spine MRI, which may demon-
idiopathic inammatory arthropathies. Al-
strate bulging/protrusion of the intervertebral
though the etiology behind these disorders is
disks, osteophytes at the facet joints, or hyper-
largely unknown, they typically present with
trophy of the ligamentum avum. Because this
progressive bilateral muscle weakness. Diag-
patient experiences neurogenic claudication
nostic distinction between the three entities is
(ischemia of compressed nerves while walk-
made on muscle biopsy. In addition, CK levels
ing), he would be a candidate for decompres-
may provide further distinction between the
sion surgery.
disorders. Polymyositis is always associated with
increased CK levels. CK levels in dermatomyo- Answer A is incorrect. CT scan would not ad-
sitis and inclusion body myositis can be nor- equately image the intervertebral disks, which
mal, and therefore cannot be used to rule out can play an essential role in the development
these two diseases. of lumbar spinal stenosis. Like plain lms,
spine CT is best for diagnosing bony abnor-
Answer A is incorrect. -Amyloid deposits are
malities such as fractures.
a classic biopsy nding in inclusion body myo-
sitis. It is the most common acquired myopa- Answer B is incorrect. Electromyography
thy in patients >50 years old. Patients develop would not provide any direct imaging data,
progressive, asymmetric weakness of both prox- which is essential to the diagnosis of lumbar
imal and distal muscles and generally do not spinal stenosis. Although electromyography
respond to immunosuppressive treatment. can detect nerve compression, results are most
useful in detecting motor unit pathology such
Answer C is incorrect. CK levels are in-
as that found in myasthenia gravis or amyo-
creased, not normal, in polymyositis. Patients
trophic lateral sclerosis.
with dermatomyositis and inclusion body myo-
sitis can have normal CK levels, so the distinc- Answer C is incorrect. Plain lms do not ad-
tion between the disease processes must be equately image the soft tissues that may be
made with biopsy ndings. involved with the etiology of lumbar spinal
stenosis, including bulging/protrusion of the
Answer D is incorrect. Perifascicular atrophy
intervertebral disks, or hypertrophy of the liga-
is a classic biopsy nding in dermatomyosi-
mentum avum. Plain lms are useful in eval-
tis. It affects the joints, esophagus, lungs, and
uating bony abnormalities such as OA.
HIGH-YIELD SYSTEMS
Chapter 9: Musculoskeletal Answers 279

Answer E is incorrect. Positron emission to- shows extracellular and intracellular monoso-
mography (PET) scan would not adequately dium urate crystals. These needle- and rod-
image the soft tissues or bony structures that shaped, negatively birefringent crystals are
must be examined in this case of lumbar spinal pathognomonic for gout. In addition, the
stenosis. PET scan is useful in assessing tissue monoarticular acute presentation without his-
perfusion and detecting malignancy. tory of trauma or infection is consistent with
this diagnosis.
18. The correct answer is A. Bisphosphonates
Answer B is incorrect. OA is joint degen-
such as alendronate may cause upset stomach,
eration that is usually secondary to overuse or
inammation, and erosion of the esophagus.
trauma. The acute onset of this patients toe
This can usually be prevented by remaining
pain and lack of supporting history make OA
upright for 3060 minutes after taking the
unlikely.
medication. The current recommendation is
to discontinue the use of bisphosphonates if Answer C is incorrect. RA is an autoimmune

Musculoskeletal
dysphagia, odynophagia, retrosternal pain, or joint disorder that normally presents with sym-
worsening heartburn occurs. metric joint degeneration and periarticular
pain and stiffness. Synovial uid in RA shows a
Answer B is incorrect. Calcitonin is taken ei-
predominance of neutrophils and high protein.
ther via the intranasal, subcutaneous, or intra-
In addition, the history and physical examina-
muscular route. It inhibits osteoclast activity.
tion ndings are more consistent with gout
Adverse effects include hypocalcemia and na-
than with RA.
sal irritation but not dysphagia or heartburn.
Answer D is incorrect. Aspiration of a septic
Answer C is incorrect. For women who are
joint would most likely yield synovial uid with
postmenopausal and other persons at risk for
increased WBC counts and a 75% chance of
osteoporosis, the recommendation is 1500 mg
visualizing the offending organism itself.
of calcium daily as well as 400800 IU/day of
vitamin D. Calcium, however, is not associ- Answer E is incorrect. SLE is an autoimmune
ated with dysphagia and epigastric burning and inammatory disorder that affects a wide vari-
does not require the patient to remain upright ety of organs. The sign and symptoms of SLE
after ingesting the medication. can be remembered using the mnemonic
DOPAMINE RASH (Discoid rash, Oral ul-
Answer D is incorrect. Recombinant parathy-
cers, Photosensitivity, Arthritis, Malar rash,
roid hormone works primarily to stimulate new
Immunologic criteria, Neurologic symptoms,
bone by increasing the number and activity of
Elevated erythrocyte sedimentation rate, Re-
osteoblasts. It is a subcutaneous medication.
nal disease, Antinuclear antibody positive, Se-
Patients taking this medication may complain
rositis, and Hematologic abnormalities). The
of orthostatic hypotension, dizziness, or leg
patients ndings do not correlate well with the
cramps. They also should be monitored for hy-
diagnosis of SLE, but rather are more represen-
percalcemia. Upper gastrointestinal symptoms
tative of an acute gout are.
are not associated with its use.
Answer E is incorrect. Selective estrogen re- 20. The correct answer is A. An exsanguinating
ceptor modiers include raloxifene. Raloxifene hemorrhage is likely when hypotension and
acts as an estrogen agonist on bone, decreasing shock are present in the setting of a pelvic frac-
bone resorption. This is not associated with ture. Patients with evidence of unstable frac-
dysphagia or worsening heart burn. tures of the pelvis associated with hypotension
should be considered for some form of external
19. The correct answer is A. Gout is a disease as- pelvic stabilization, which has been shown to
sociated with an abnormally high level of urate decrease mortality in these patients. The exact
in the serum. This disorder often results in re- mechanism by which early pelvic stabilization
curring acute monoarticular arthritis, classi- is effective in promoting hemodynamic stabil-
cally affecting the rst MTP joint. The image
HIGH-YIELD SYSTEMS
280 Section I: Organ Systems Answers

ity in patients with unstable pelvic fractures is LCSS. The CREST syndrome consists of Cal-
not completely understood. However, some be- cinosis, Raynauds phenomenon, Esophageal
lieve that reducing the pelvis back to its normal dysmotility, Sclerodactyly, and Telangiectasias.
conguration reduces pelvic volume, and
Answer B is incorrect. ANCA is a common
therefore limits the amount of blood loss to the
nding in patients with Wegeners granuloma-
retroperitoneal pelvic hematoma. Thus, keep-
tosis, which is a systemic vasculitis that classi-
ing the pelvic volume small may promote tam-
cally affects the lung and kidney. ANCA may
ponade of the bleeding sources in the pelvis.
also be positive in microscopic polyangiitis,
Answer B is incorrect. Laparotomy is not war- Churg-Strauss syndrome, and several other vas-
ranted without evidence of gross blood in the culitides, but is not associated with SLE.
abdomen or evidence of intestinal perforation.
Answer D is incorrect. Anemia is a very com-
The diagnostic peritoneal lavage is a reliable
mon nding in patients with SLE, but it is
diagnostic test for this purpose. In addition,
typically a normochromic, normocytic anemia
patients with evidence of unstable pelvic frac-
Musculoskeletal

due to chronic disease. Rarely, patients with


tures who warrant laparotomy should receive
SLE can have a hemolytic anemia. Macrocytic
external pelvic stabilization prior to any inci-
anemia is typically caused by folate or vitamin
sions.
B12 deciency and can also be seen in patients
Answer C is incorrect. Internal xation should with alcohol abuse, liver disease, and hypothy-
be considered only if the patient is hemody- roidism.
namically stable.
Answer E is incorrect. Patients with SLE typi-
Answer D is incorrect. Patients who are he- cally develop thrombocytopenia, not thrombo-
modynamically unstable should not be sent to cytosis. Patients may present with purpura or
the CT scanner. Once stable, a patient may be easy bruising and bleeding.
taken to the scanner. Patients with evidence of
arterial extravasation of intravenous contrast in 22. The correct answer is B. RA is an autoim-
the pelvis via CT should be considered for pel- mune disorder with potential for multiple or-
vic angiography and possible embolization. gan system involvement. The most common
presentation of RA is that of an inammatory
Answer E is incorrect. A pelvic hematoma
symmetric synovitis of the peripheral joints.
should never be explored due to the risk of un-
The disease is also associated with several extra-
controllable bleeding.
articular manifestations, including nodules,
splenomegaly, interstitial brosis, and vasculi-
21. The correct answer is C. This woman is pre-
tis. Rheumatoid nodules are present in 20%
senting with signs and symptoms suggestive of
35% of patients with RA and usually form on
SLE. It can cause widespread systemic disease
pressure points on the body.
including renal failure, central nervous system
disturbances, and pulmonary disease. Antinu- Answer A is incorrect. Myocarditis is a possible
clear antibody is positive in almost all patients extraarticular manifestation of RA; however, it
with SLE, but it is not specic. Anti-double- is rare.
stranded DNA and anti-Sm antibodies are
Answer C is incorrect. Pericarditis is a possible
highly specic for lupus, so their presence con-
extra-articular manifestation of RA that occurs
rms the diagnosis of SLE. Patients with SLE
in <10% of patients with RA over their lifetime.
frequently have hematologic abnormalities.
However, 30% of patients may have evidence
Leukopenia is a very common nding. It is
of pericardial effusions on echocardiography
usually lymphopenia as opposed to neutrope-
that are of no clinical signicance.
nia, so patients are not at signicantly in-
creased risk of infection. Answer D is incorrect. Patients with RA can
develop renal disease, including focal glomeru-
Answer A is incorrect. Anticentromere anti-
bodies are specic for CREST syndrome, or
HIGH-YIELD SYSTEMS
Chapter 9: Musculoskeletal Answers 281

lonephritis or toxicity from RA treatments, but Answer D is incorrect. Classic Reiters syn-
this is very rare. drome is characterized by a classic triad of
uveitis, urethritis, and arthritis (cant see, cant
Answer E is incorrect. Splenomegaly (as part
pee, cant climb a tree). Reiters syndrome is
of Feltys syndrome with granulocytopenia) is
typically caused by Chlamydia trachomatis. It
a possible extra-articular manifestation of RA;
is one of the spondyloarthropathies in which
however, it is uncommon.
patients have axial and sacroiliac inammatory
arthritis. Patients may also have asymmetric
23. The correct answer is B. The combination of
involvement of large peripheral joints, particu-
several weeks of high-spiking fevers, arthritis,
larly of the lower limbs.
and salmon-colored rash is highly suggestive of
systemic-onset juvenile rheumatoid arthritis Answer E is incorrect. Septic arthritis is a
(JRA). The salmon-colored macular rash is ac- common disorder in patients in this age group.
tivated by heat and is frequently found in warm However, the history of several weeks of spiking
areas of the body such as the axilla and the fevers and rash is less consistent with an acute

Musculoskeletal
waist. The rash is commonly triggered by a fe- bacterial infection. In addition, acute bacterial
ver spike, but subsides when the temperature septic arthritis typically affects a single joint.
returns to normal. Physical examination nd-
ings in systemic-onset JRA include lymphade- 24. The correct answer is E. Low back pain is a
nopathy, hepatomegaly, and splenomegaly. very common complaint seen by the primary
There is no specic test for JRA, so the diagno- care physician. The most common origin of
sis is typically made by clinical presentation. low back pain is musculoskeletal, such as strain
of paraspinal muscles or injury to ligaments or
Answer A is incorrect. Acute rheumatic fever
fascia. However, back pain can be the present-
(ARF) is a complication of Group A strepto-
ing symptom of a more serious underlying dis-
coccal pharyngitis. It is characterized by ve
order such as malignancy. In the initial work-
major criteria: migratory arthritis of the large
up of a patient with back pain, a detailed
joints, carditis, rash, chorea, and subcutaneous
history is essential in determining which pa-
nodules. The rash of ARF is erythema margi-
tients can be managed conservatively and
natum, a pink, nonpruritic rash with sharply
which need further diagnostic evaluation. Pain
dened borders and central clearing that pre-
relieved by NSAIDs is unlikely to be due to
dominantly occurs on the trunk. Fever is one
malignancy or infection.
of the minor criteria of ARF, along with ar-
thralgias and previous rheumatic heart disease. Answer A is incorrect. Age >50 years in a pa-
Diagnosis of ARF by Jones criteria requires tient with back pain is associated with a greater
the presence of two major, or one major and risk of malignancy. More benign causes of low
two minor, criteria in addition to evidence of a back pain, such as musculoskeletal pain, tend
recent streptococcal infection. to occur in younger patients.
Answer C is incorrect. Parvovirus can also Answer B is incorrect. Pain that lasts >1
cause fever, rash, and arthritis, and is often month is concerning for a more serious condi-
considered in the differential diagnosis of tion such as malignancy or infection. Ninety
systemic-onset JRA. However, viral infections percent of patients with low back pain will re-
rarely cause the spiking fevers that are seen in cover spontaneously within 4 weeks. As a re-
patients with systemic-onset JRA. In addition, sult, symptoms lasting >1 month are an indica-
the rash of parvovirus is classically a slapped- tion for further evaluation.
cheek appearance, as opposed to the rash
Answer C is incorrect. Any patient with a his-
shown above. Viral exanthems tend to persist
tory of malignancy presenting with low back
regardless of temperature, as opposed to the
pain should have further radiographic evalu-
rash of JRA that uctuates with fever.
ation to rule out metastatic disease. A history
HIGH-YIELD SYSTEMS
282 Section I: Organ Systems Answers

of prostate cancer is particularly concerning, affected joint. A classic location of acute gouty
given the propensity of prostate cancer to me- arthritis is the rst metatarsophalangeal joint, a
tastasize to the lumbar spine. condition known as podagra. Gout most com-
monly occurs in middle-age to elderly men
Answer D is incorrect. History of recurrent uri-
and is caused by hyperuricemia. Acute gouty
nary tract infection can be an indication that
arthritis typically has a sudden onset, often at
the patient has back pain due to pyelonephri-
night, and tends to last for 310 days.
tis. Further work-up including complete blood
cell count, erythrocyte sedimentation rate, and
27. The correct answer is E. This patient has a clin-
radiographic imaging should be performed.
ical presentation suggesting OA. The dull, ach-
ing pain caused by OA worsens with activity and
25. The correct answer is E. The patient suffers
may resolve with rest. Patients may also com-
from temporal arteritis. Temporal arteritis, also
plain of stiffness and limited mobility. Multiple
known as giant cell arteritis, is characterized by
joints are typically involved. In the hand, the
inammation of large to medium-size arteries,
Musculoskeletal

proximal and distal interphalangeal joints are


especially the temporal artery. Patients often
classically affected, while the metacarpophalan-
present with headaches, fever, anemia, ele-
geal joints are spared. Enlargement of the distal
vated erythrocyte sedimentation rate, and
interphalangeal and proximal interphalangeal
marked tenderness to palpation over the tem-
joints results in Heberdens and Bouchards
poral regions. Occasionally, a palpably thick-
nodes, respectively. The major radiographic
ened temporal artery is noted. This disorder
signs of OA include joint space narrowing, sub-
generally responds well to steroid therapy. Cur-
chondral sclerosis, osteophytes, and subchon-
rent guidelines dictate that initial treatment of
dral cysts.
temporal arteritis should be 4060 mg of oral
prednisone per day for approximately 1 month, Answer A is incorrect. AS is a spondyloar-
followed by a gradual taper. This decreases the thropathy that typically affects young men. It
risk of the long-term sequelae of the disease, primarily causes inammatory back pain and
namely, visual loss, aortic aneurysm, and aortic sacroiliitis. Other manifestations include ante-
dissection. rior uveitis, aortic regurgitation, bowel ulcer-
ations, and IgA nephropathy.
Answer A is incorrect. Acetazolamide is a car-
bonic anhydrase inhibitor and does not play a Answer B is incorrect. Patients with bromy-
role in the initial treatment of temporal arteritis. algia fulll two criteria: (1) chronic pain for at
least 3 months in all four quadrants of the body
Answer B is incorrect. Atenolol is a -blocker
and along the axial skeleton and (2) tenderness
and does not play a role in the initial treatment
in 11 of 18 specic anatomic points. They of-
of temporal arteritis.
ten have multiple associated conditions, such
Answer C is incorrect. Methotrexate is an as irritable bowel syndrome, tension head-
immunosuppressive agent (like prednisone); aches or migraines, anxiety disorders, thyroid
however, it is not indicated for the treatment of dysfunction, sleep disorders, depression, and
temporal arteritis because studies examining its dysmenorrhea.
effects have been inconclusive.
Answer C is incorrect. Gonococcal arthritis
Answer D is incorrect. Nifedipine is a calcium typically causes monarticular arthritis, tenosyn-
channel blocker and does not play a role in the ovitis, and dermatitis. Patients usually have an
initial treatment of temporal arteritis. acute onset of arthritis, often accompanied by
systemic symptoms such as fever and chills. Pa-
tients may have pain in the right upper quad-
Questions 26 and 27 rant due to perihepatitis, a condition known as
26. The correct answer is D. Acute gouty arthritis Fitz-Hugh-Curtis syndrome.
is typically monarticular, presenting with se-
vere pain, redness, swelling, and warmth of the
HIGH-YIELD SYSTEMS
Chapter 9: Musculoskeletal Answers 283

Answer F is incorrect. PMR is a rheumatic Questions 28, 29, and 30


syndrome that consists of aching pain in the
28. The correct answer is J. Polyarteritis nodosa is
neck, shoulders, and pelvis. Pain is most severe
a vasculitis of small or medium-size arteries
in the proximal muscles. There are no signs of
that can cause aneurysms secondary to inltra-
atrophy, but patients do have a reduced range
tion and destruction of the blood vessel wall by
of motion because of their pain.
neutrophils. Polyarteritis is also associated with
Answer G is incorrect. Psoriatic arthritis is a hepatitis B seropositivity in 30% of patients and
spondyloarthropathy that causes inammatory with perinuclear ANCA (P-ANCA). The most
back pain and oligoarthritis of large peripheral common manifestations are constitutional
joints. The pain is usually insidious in onset symptoms, myalgias, and arthralgias, but it may
and is worst in the morning or after a long pe- also involve the central nervous system, gastroin-
riod of inactivity. Patients must have cutaneous testinal tract, and kidneys. This patients foot
signs of psoriasis to be diagnosed with psoriatic drop is secondary to inltration of the artery sup-
arthritis. plying the common peroneal nerve, and his

Musculoskeletal
postprandial pain is secondary to intestinal an-
Answer H is incorrect. Reactive arthritis is
gina of one of the mesenteric vessels. Treat-
another spondyloarthropathy that typically fol-
ment is with corticosteroids and cyclophosph-
lows a gastrointestinal infection. Common
amide if refractory.
pathogens implicated in reactive arthritis in-
clude Shigella, Campylobacter, Yersinia, and
29. The correct answer is N. Wegeners granulo-
Salmonella.
matosis is caused by inammatory inltration
Answer I is incorrect. Classic Reiters syn- of small and medium-size arteries. The disease
drome is also a spondyloarthropathy that typi- tends to affect the sinuses, upper and lower air-
cally occurs after infection with Chlamydia tra- ways, and kidneys. Symptoms may include
chomatis. The classic triad of ndings includes cough, hemoptysis, pleuritis, and dyspnea. Al-
uveitis, urethritis, and arthritis (cant see, cant though only a minority of patients present with
pee, cant climb a tree). Other ndings may glomerulonephritis early on, approximately
include circinate balanitis and keratoderma three-fourths of patients eventually develop this
blennorrhagicum. complication. Most patients with Wegeners
granulomatosis are positive for circulating
Answer J is incorrect. RA causes symmetric
ANCA (C-ANCA). Initial treatment is with cy-
polyarticular inammation that commonly af-
clophosphamide.
fects the hands, feet, knees, and ankles. Pain is
classically most severe in the morning and im-
30. The correct answer is B. Churg-Strauss vascu-
proves throughout the day. Patients also com-
litis affects small and medium-size arteries (but
plain of morning stiffness that prevents them
typically smaller arteries than those affected by
from getting out of bed. In contrast to OA,
Wegeners granulomatosis). The vasculitis is
patients with RA typically have inammation
seen in the setting of asthma and allergic rhini-
of the metacarpophalangeal and proximal in-
tis. Asthma usually precedes evidence of vascu-
terphalangeal joints, with sparing of the distal
litis by up to 20 years. The criteria for diagnosis
interphalangeal joints.
include the presence of asthma, eosinophilia,
Answer K is incorrect. SLE is an autoimmune mono- or polyneuropathy, pulmonary inl-
disorder dened by the presence of antinuclear trates, paranasal sinus abnormalities, and ex-
antibodies. The arthritis of SLE is typically a travascular eosinophils.
symmetric polyarthritis that is nondeforming
Answer A is incorrect. Behets disease is a sys-
and nonerosive. It generally affects the meta-
temic vasculitis that affects arteries and veins of
carpophalangeal and proximal interphalangeal
all sizes. Clinical manifestations include recur-
joints of the hands, wrists, and knees.
rent oral aphthous and genital ulcers, eye le-
sions, skin lesions, and a positive pathergy test
(a pinprick that turns into a pustule).
HIGH-YIELD SYSTEMS
284 Section I: Organ Systems Answers

Answer C is incorrect. Disseminated intravas- Answer H is incorrect. Kawasakis disease is a


cular coagulation is seen in many different clini- vasculitis of childhood affecting small and me-
cal scenarios, including sepsis, postdelivery (e.g., dium-size muscular arteries (especially the cor-
with amniotic embolus), and trauma, as well as onary arteries). Diagnosis requires a fever for
in the setting of neoplasia. It is characterized by at least 5 days and four of the following: rash,
diffuse bleeding from venipuncture sites, mu- conjunctival injection, erythematous mucous
cosal bleeding, and hypotension. Prothrombin membranes, cervical lymphadenopathy, and
time and partial thromboplastin time are both changes in the hands or feet (e.g., desquama-
increased, as is bleeding time. A positive D-di- tion).
mer test and increased brin split products with
Answer I is incorrect. Microscopic polyangi-
decreased circulating brinogen levels are in-
itis is a vasculitis of small and medium-size
dicative of a diffuse coagulation process.
arteries and veins that can cause mononeuri-
Answer D is incorrect. Giant cell arteritis is tis multiplex, necrotizing glomerulonephritis,
a vasculitis that typically affects the cranial and pulmonary capillaritis, leading to alveolar
Musculoskeletal

branches of the aortic arch. Patients can pre- hemorrhage and hemoptysis. In contrast to pol-
sent with headache, scalp pain, constitutional yarteritis nodosa, it tends to involve the lung. It
symptoms, or absent temporal artery pulsations. differs from Wegeners granulomatosis in that
It is associated with PMR (pain and weakness patients with microscopic polyangiitis do not
of the shoulder and pelvic girdle). Disease ac- exhibit granulomatous inammation. Patients
tivity is associated with an elevated erythrocyte tend to be P-ANCA positive.
sedimentation rate. Temporal artery biopsy
Answer K is incorrect. Takayasus arteritis, also
demonstrates mononuclear cell or granuloma-
known as pulseless disease, is a vasculitis that
tous inammation, usually with multinucle-
affects young women and demonstrates vascu-
ated giant cells.
litis of the aorta and its branches. Patients can
Answer E is incorrect. Hemolytic-uremic syn- present with constitutional ndings and clau-
drome is associated with a mild viral illness or dication of the extremities. Clinical ndings
gastroenteritis with Escherichia coli O157:H7. include an absent or decreased brachial artery
The characteristic presentation involves anemia, pulse, subclavian bruit, pain along the vessels,
thrombocytopenia, and acute renal failure. and unequal pulses.
Answer F is incorrect. Henoch-Schnlein pur- Answer L is incorrect. Thromboangiitis oblit-
pura occurs most frequently in children and erans (also known as Buergers disease) is an in-
causes palpable purpura unrelated to thrombo- ammatory disease of the small and medium-
cytopenia along with arthritis, abdominal pain, size arteries, veins, and nerves of the distal
and glomerulonephritis. The disease typically extremities. Biopsy demonstrates a highly cellu-
follows an upper respiratory tract infection and lar and inammatory thrombus that spares the
is diagnosed by demonstrating IgA deposition vessel wall. Clinical ndings typically involve
in the cutaneous blood vessel wall. ischemia of the digits. There are no laboratory
anomalies specic for the disease. The typical
Answer G is incorrect. Idiopathic thrombocy-
patient is a male smoker <45 years old.
topenic purpura is characterized by petechiae,
purpura, ecchymoses, and mucosal bleeding. Answer M is incorrect. Thrombotic throm-
A complete blood cell count shows a very low bocytopenic purpura is a microangiopathic
platelet count, and a peripheral blood smear hemolytic anemia characterized by fever, ane-
demonstrates large platelets with the absence mia, acute renal failure, thrombocytopenia,
of schistocytes. Idiopathic thrombocytopenic and neurologic abnormalities. A peripheral
purpura can resolve spontaneously, but treat- blood smear demonstrates thrombocytopenia
ment in adults usually includes corticosteroids; and the presence of schistocytes. Indirect bili-
intravenous immunoglobulin and anti-Rho(D) rubin and lactate dehydrogenase levels will be
are reserved for uncontrollable hemorrhage or elevated secondary to hemolysis.
extremely low platelet counts (i.e., <10,000/
mm3).
CHAPTER 10

Neurology

285
HIGH-YIELD SYSTEMS
286 Section I: Organ Systems Questions

Q U E ST I O N S

1. A 4-year-old boy is discovered to have an IQ of 3. The parents of a 6-month-old child bring him
60 and is referred to a pediatrician. His mother to a pediatrician because they are concerned
reports he was born 5 weeks premature by he might be having seizures. They describe the
spontaneous vaginal delivery after an uncom- events as convulsions lasting under a minute
plicated pregnancy. His developmental history and occurring in clusters of increasing fre-
is notable for initially sitting at 9 months, say- quency and severity. They also report that the
ing his rst word at 16 months, walking at 19 child can no longer sit on his own. On exami-
months, and climbing up stairs at 3.5 years. He nation the child initially seems healthy and is
plays cooperatively with the other children. On afebrile, but experiences a series of ve exten-
physical examination he is cooperative with sor-type spasms of the neck and trunk. Which
the examiner, responding in three-word sen- of the following tests can conrm the diagnosis
tences. He walks on his tiptoes with a scissored of Wests syndrome?
Neurology

gait. His legs are hypertonic bilaterally with


(A) CT of the head
brisk patellar reexes; he has ankle clonus and
(B) ECG
upgoing toes bilaterally. Which of the follow-
(C) Electroencephalography
ing is the most likely diagnosis?
(D) Electromyography
(A) Autism (E) Lumbar puncture
(B) Cerebral palsy
(C) Downs syndrome 4. A 21-year-old woman presents to her obstetri-
(D) Duchennes muscular dystrophy cian because of urinary problems. She has
(E) Fragile X syndrome been stressed because she is working hard to
do well on her nal examinations at college.
2. A 42-year-old man presents to urgent care with Over the past 2 weeks she nds herself contin-
back pain for 2 days that began while he was ually rushing to the bathroom because I feel
moving furniture during home remodeling. like I need to go, but I cant. This has never
The pain is sharp, travels down his left leg, and happened to her before, and she has begun
feels like an electric shock. It is relieved with wearing pads just in case. She denies fevers
rest and aggravated with walking. His tempera- and dysuria. She is otherwise healthy, exercises
ture is 36.9C (98.5F), blood pressure is regularly, and takes a multivitamin daily. Her
122/70 mm Hg, heart rate is 68/min, and respi- physical examination is normal, as is her uri-
ratory rate is 12/min. The patient appears un- nalysis. Cystometry is performed, and her de-
comfortable, and when asked to describe the trusor contraction to bethanechol chloride is
location of the pain, places his entire hand at greatly exaggerated. What is the most likely eti-
on the lumbar region to the left of the spine. ology of this womans urinary urgency?
Straight leg raise on the left is positive for repli-
(A) Acute cystitis
cation of the pain. Reexes are decreased in
(B) Chronic bladder distention
the left lower extremity. Which of the follow-
(C) Detrusor muscle inammation
ing is the most appropriate next step in man-
(D) Multiple sclerosis
agement?
(E) Pelvic oor damage
(A) CT of the spine
(B) Lumbar series x-ray 5. A 1-year-old boy is brought to the pediatric
(C) MRI of the spine neurologist for writhing movements. These
(D) Neurosurgical consultation movements started one week ago and are quick
(E) Rest and nonsteroidal anti-inammatory jerking movements of the arms, hands, and
drugs legs with slow writhing movements inter-
spersed. He has not pulled himself to standing
HIGH-YIELD SYSTEMS
Chapter 10: Neurology Questions 287

yet but can crawl. His birth history is unre- ease. Upon arrival he has a decrease in level of
markable and he has been otherwise healthy. consciousness. Which of the following should be
On review of systems his mother reports seeing performed rst in order to conrm a diagnosis?
him try to hit himself with his sts. His vital
(A) Comprehensive metabolic panel and com-
signs are stable. His neurological examination
plete blood cell count
is notable for the movements described above;
(B) Electroencephalography
cranial nerves IIXII are intact and he has
(C) Lumbar puncture
gross hypertonia. Laboratory tests show:
(D) Neuroimaging with CT
Na+: 142 mEq/L (E) Neuroimaging with MRI
K+: 4.1 mEq/L (F) Positron emission tomography scan
Cl: 102 mEq/L
HCO3: 23 mEq/L 7. A 2.5-month-old boy is brought to the emergency
Blood urea nitrogen: 28 mg/dL department for evaluation of fever. Two days prior
Creatinine: 2.0 mg/dL to admission the patient developed a fever to
39.1C (102.3F) and became irritable with na-
Which of the following is the most likely diag-

Neurology
sal discharge and decreased oral intake. His birth
nosis?
history is unremarkable. He lives at home with
(A) Galactosemia his mother and 6-year-old brother, who attends
(B) Lesch-Nyhan syndrome elementary school where several kids have been
(C) Maple syrup urine disease absent recently for illness. His heart rate is 137/
(D) Phenylketonuria min, blood pressure is 72/48 mm Hg, respiratory
(E) Wilsons disease rate is 35/min, and rectal temperature is 39.3C
(102.7F). Neurologic examination is remarkable
6. A 55-year-old man presents to the emergency de- for a lethargic-appearing child who is responsive
partment because he experiencing a headache to stimulation. His anterior fontanel is open and
while lifting heavy boxes. The headache was bulging. The resident caring for the patient is cer-
acute in onset, but gradually worsened over sev- tain that he has acute bacterial meningitis and
eral hours. It was associated with nausea. No orders a lumbar puncture. Which of the cerebro-
over-the-counter analgesics alleviated the symp- spinal uid results from the lumbar puncture in
toms. His past medical history is signicant for a the table below would be most consistent with
recent diagnosis of adult polycystic kidney dis- acute bacterial meningitis?
(A) A
(B) B
(C) C
(D) D
(E) E

PRESSURE WBC COUNT (/mm3)


CHOICE (mm H2O) [predominant cells] PROTEIN (mg/dL) GLUCOSE (mg/dL)

A 60 2 [lymphocytes] 30 70

B 228 3780 [polymorphonuclear 327 36


leukocytes]

C 115 980 [mononuclear cells] 123 62

D 123 300 [mononuclear cells] 194 42

E 97 415 [lymphocytes] 138 69


HIGH-YIELD SYSTEMS
288 Section I: Organ Systems Questions

8. An 80-year-old white man is brought to the could have been predicted by which of the fol-
clinic by his son, who is concerned that his fa- lowing results on a triple marker screen?
ther has become more forgetful over the past
year, cannot follow instructions, is unable to MATERNAL -HUMAN
drive his car or manage his checkbook, and has SERUM CHORIONIC
had a decline in being able to manage activi- CHOICE -FETOPROTEIN ESTRADIOL GONADOTROPIN
ties of daily living. He has no signicant medi-
cal history other than diabetes, and has no past A
psychiatric history. His Mini-Mental State Ex-
amination (MMSE) score is 17. Which of the B
following criteria would help establish a diag-
nosis of Alzheimers disease? C

(A) A high level of protein 14-3-3 in the cere- D


brospinal uid
(B) Focal neurologic ndings and evidence of E
Neurology

prior strokes
(C) Gait dysfunction and urinary incontinence
(D) Gradual onset of symptoms and continued (A) A
cognitive decline (B) B
(E) Lewy bodies in the brain stem on autopsy (C) C
(D) D
9. A 79-year-old man who was diagnosed with late (E) E
onset bipolar disorder is hospitalized after sus-
taining a fracture. During admission, he is alert 11. A 2-year-old boy is brought to his pediatrician
and fully oriented, and receives pain medica- because of delay in milestones. He is able to sit
tion. Two days after surgery to reduce the frac- and has started to walk and babble. His past
ture, the patient was verbally unresponsive, dis- medical history is unknown as he was adopted
oriented, displayed a clouding of consciousness, from Eastern Europe when he was 9 months
and could only utter short phrases. An MMSE old. Vital signs are stable. Other than his
was administered at that time, and his score global developmental delays, the patient ap-
was 10 of 30. A day later, the patient displayed pears healthy. Laboratory tests show a phenyla-
good eye contact and normal psychomotor ac- lanine level of 20 mg/dL (normal: 1 mg/dL)
tivity. His score on the MMSE was 24. Which and plasma tyrosine of 0.7 mg/dL (normal: 1
of the following is the most appropriate man- mg/dL). Which of the following is the best
agement? therapeutic option?

(A) Chlordiazepoxide, thiamine, folic acid, (A) Administer phenylalanine hydroxylase


and a multivitamin (B) Increase dietary phenylacetate
(B) Discontinue unnecessary medications, (C) Increase dietary phenylalanine
pan-culture, and check for electrolyte ab- (D) Restrict dietary phenylalanine
normalities (E) Restrict dietary tyrosine
(C) Donepezil
12. A 67-year-old man is found on the oor in his
(D) Lithium and a sedative drug
home unresponsive and unarousable. On phys-
(E) Watchful waiting
ical examination the patient is breathing spon-
10. A 36-year-old woman who received no prenatal taneously and has bounding radial pulses bilat-
care gave birth to a full-term 3200-g (7.1-lb) erally. He does not open his eyes, makes no
girl. The infants physical examination is nota- sounds, and withdraws to sternal rub. Which of
ble for a at nasal bridge, upslanting palpebral the following is the next step in the manage-
ssures, prominent epicanthal folds, and a pan- ment of this patients condition?
systolic murmur. Karyotype analysis reveals an
abnormality. In utero, the infants condition
HIGH-YIELD SYSTEMS
Chapter 10: Neurology Questions 289

(A) Administer naloxone, thiamine, and dex-


trose
(B) Check brain stem reexes
(C) Check serum electrolytes
(D) Intubate
(E) Undress the patient

13. A homeless man is brought to the emergency


department in a confused state following a sei-
zure in public. Witnesses report the seizure as
occurring suddenly and lasting at least 23
minutes. A rst responder on the scene says,
he swung his arms and his entire body shook.
On examination the patients tongue is un-
harmed, although his pants are covered in Reproduced, with permission, from Wolff K, Johnson RA,
fresh urine. He is oriented to person but not

Neurology
Surmond D. Fitzpatricks Color Atlas & Synopsis of Clini-
place or time and has no recollection of what cal Dermatology, 5th edition. New York: McGraw-Hill,
happened, but is aware he has some sort of 2005: Figure 15-29.
seizure disorder for which he occasionally
(A) Multiple neuroendocrine neoplasia type I
takes an unnamed medication that he is not
(B) Neurobromatosis type I
carrying. More in-depth questioning reveals
(C) Neurobromatosis type II
that he has been using increasing amounts of
(D) Tuberous sclerosis
cocaine and heroin since he was kicked out of
(E) Von Hippel-Lindau syndrome
a group home and rarely takes his seizure med-
ication as a result. An electroencephalogram 15. A 16-month-old toddler is brought to the clinic
(EEG) of this patient during the episode would because of worsening coordination. The par-
most likely show which of the following? ents say she was developing well until the age
(A) A 3-Hz spike and wave pattern of 5 months. Prior to then, the patient had an
(B) A triphasic discharge pattern in sporadic appropriate social smile, recognized caregivers,
fashion and cooed. At the age of 11 months she could
(C) Generalized seizure activity affecting one say a few words, but over the past 3 months has
hemisphere not learned any new words and is not speaking
(D) Seizure activity involving both hemi- her rst words anymore. She was walking with
spheres assistance at 12 months, but now has lost coor-
(E) Seizure activity localized to a discrete por- dination and cannot walk. Her parents note
tion of one hemisphere that she has developed a peculiar behavior of
wringing her hands for long periods. This dis-
14. During a routine eye examination in an order is almost exclusively found in which of
18-year-old patient, an ophthalmologist notices the following populations?
pigmented hamartomas of the iris. Not know-
(A) Ashkenazi Jews
ing what to make of this, the doctor does a full
(B) Females
physical examination, which is unremarkable
(C) Males
except for the skin ndings shown in the im-
(D) Patients with fragile X syndrome
age. The patient notes a family history of simi-
(E) Patients with trisomy 21 genotype
lar skin ndings. Which of the following condi-
(F) Southeast Asians
tions is most likely?
HIGH-YIELD SYSTEMS
290 Section I: Organ Systems Questions

16. A 38-year-old man with prior head trauma (A) Brain tumor
from a motor vehicle accident and no signi- (B) Cluster headache
cant past medical history presents with epi- (C) Epidural hemorrhage
sodes of dizziness. These episodes are induced (D) Migraine headache
by rolling over to one side while lying down in (E) Subdural hemorrhage
bed, last <1 minute, and are associated with in- (F) Tension headache
tense nausea. These episodes have occurred
periodically for the past 2.5 months. Nothing 19. A 45-year-old patient with a newly diagnosed
alleviates the symptoms. Which of the follow- brain tumor presents with a speech impedi-
ing is the rst step in diagnosing the patients ment. He is not able to repeat anything that is
condition? said to him or to name objects. However, he
has preserved uency of speech and good com-
(A) Brain stem evoked audiometry
prehension of spoken and written language.
(B) Dix-Hallpike maneuver
He has no other symptoms or neurologic de-
(C) Electronystagmography
cits, and his neurologic examination is other-
(D) Lumbar puncture
wise unremarkable. Which of the following is
Neurology

(E) Neuroimaging
the most likely diagnosis?
17. A 13-year-old boy with a previous history of sei- (A) Brocas aphasia
zures is brought to the physicians ofce by his (B) Conduction aphasia
parents for staring spells that occur in the (C) Global aphasia
morning. His parents report that his epilepsy (D) Transcortical sensory aphasia
has been well controlled, and it has been sev- (E) Wernickes aphasia
eral years since his last seizure. However, they
say that he has trouble with spastic movements 20. A 48-year-old auto mechanic presents to the
of his arms and legs almost every morning. An clinic with complaints of many years of pins
EEG taken during one of these events would and needles in his left hand that initially oc-
most likely show which of the following? curred only while working but have worsened
substantially. He claims the pain wakes him al-
(A) Beta buzz evolving into diffuse
most every night. Physical examination reveals
polyspikes
marked weakness and wasting of the left hand
(B) A 3-Hz spike-and-wave pattern
muscles. Which of the following is the most
(C) Generalized seizure activity affecting one
likely diagnosis?
hemisphere
(D) Seizure activity involving both hemi- (A) Amyotrophic lateral sclerosis
spheres (B) Angina
(E) Seizure activity localized to a discrete por- (C) Carpal tunnel syndrome
tion of one hemisphere (D) Multiple sclerosis
(E) Myasthenia gravis
18. A 30-year-old patient presents to his physician
because of a gradually increasing headache, 21. A 56-year-old white man is diagnosed with sub-
which she describes as being a dull aching arachnoid hemorrhage following rupture of a
feeling at the top of her head. Despite having cerebral aneurysm in the anterior circulation.
no previous history, she describes having suf- The hemorrhage is conrmed on CT of the
fered uncontrollable jerking movements yester- head, but the decision is made for the patient
day, which subsided only after 30 seconds. She to undergo endovascular coiling rather than
states that the worst aspect of the headache is surgical clipping. Roughly 1 week after the
that it reaches a crescendo in the morning, original bleed, the patient suffers an acute de-
making it difcult for her to get to work on cline in his mental status, with a conrmed
time. Which of the following is the most likely new infarct on MRI. Based on the history and
diagnosis? time course, what was the likely etiology of the
new infarct?
HIGH-YIELD SYSTEMS
Chapter 10: Neurology Questions 291

(A) Atherosclerosis with her grandchildren. During the past 3


(B) Cardiogenic thromboembolism weeks, her husband has taken over most of the
(C) Hydrocephalus household chores. When the physician begins
(D) Vasospasm the examination, the patient becomes tearful
and reluctant to answer questions, frequently
22. A 65-year-old African-American man presents saying, I dont know. Her MMSE score is 24,
with new-onset aphasia and left-sided homony- losing points for short-tem recall. She also per-
mous hemianopsia. Results of CT of the head forms poorly on simple executive tasks. Which
are shown in the image. Which vascular terri- of the following is the most likely diagnosis?
tory is involved?
(A) Alzheimers dementia
(B) Delirium
(C) Normal aging
(D) Pseudodementia
(E) Vascular dementia

Neurology
24. A 24-year-old woman with Graves disease pre-
sents complaining of blurry vision during her
afternoon commute driving home from work.
Administration of edrophonium leads to tran-
sient resolution of her clinical symptoms. The
physician diagnoses her with myasthenia gravis.
In discussing the patients treatment options
Reproduced, with permission, from Kasper DL, Braun- with her, which of the following can the physi-
wald E, Fauci AS, Hauser SL, Longo DL, Jameson LJ, Is- cian describe as potentially curative?
selbacher KJ, eds. Harrisons Online. New York: McGraw-
Hill, 2005: Figure 349-12. (A) Intravenous immunoglobin
(B) Neostigmine
(A) Anterior cerebral artery (C) Plasmapheresis
(B) Basilar artery (D) Prednisone
(C) Lacunar territories (E) Thymectomy
(D) Middle cerebral artery
(E) Posterior cerebral artery 25. A 50-year-old man suffers from headaches.
They occur in periodic cycles, only several
23. A 72-year-old woman presents to her physician months out of the year, with 12 headache at-
complaining of poor sleep and decreased appe- tacks per day. The headaches are severe; lo-
tite. She is a retired high school teacher who cated behind the right eye; and are associated
lives with her husband. Their children live in with a red, tearing, right eye and nasal stuf-
the neighborhood and visit on a regular basis. ness on the same side. Which of the following
The patient has no history of psychiatric disor- is the best rst step in management?
ders or major medical illnesses aside from hy-
pertension well controlled on a diuretic. For (A) -Blockers
the past month, the patient has been less inter- (B) Anticonvulsants
ested in many activities that she previously en- (C) Avoiding factors that precipitate these
joyed. In particular, she no longer reads books headaches (e.g., alcohol or physical stress)
and has become reluctant to garden or play (D) Nonsteroidal anti-inammatory drugs
(E) Tricyclic antidepressants
HIGH-YIELD SYSTEMS
292 Section I: Organ Systems Questions

26. A 54-year-old white man presents to the local (A) Benzodiazepines


emergency department with altered mental (B) Carbamazepine
status and largely unintelligible speech. On (C) Lamotrigine
further examination he has considerable gait (D) Phenytoin
difculties and eye movement abnormalities. (E) Topiramate
During the few moments that he is somewhat
lucid, he cannot recall where he lives, where 29. A 3-day-old boy presents to the emergency de-
he was born, or what he does for a living. The partment after his mother found him rhythmi-
patients past history includes alcoholism, and cally shaking his extremities for 1 minute. He
his current glucose level is 50 mg/dL. Which was born at term via spontaneous vaginal deliv-
of the following is the most appropriate imme- ery after an uncomplicated pregnancy. His Ap-
diate treatment? gar scores were 8 and 9 at 1 and 5 minutes, re-
spectively. His birth weight is 3100 g (6.8 lb).
(A) Folate
His family history is notable for a brother who
(B) Glucose
died at age 6 years from mycobacterial pneu-
(C) Thiamine
monia. His temperature is 36.5C (97.7F),
Neurology

(D) Vitamin B6
blood pressure is 70/50 mm Hg, respiratory
(E) Vitamin B12
rate is 30/min, and heart rate is 110/min. He
has a squared nose, cleft palate, and a harsh
27. A 65-year-old African-American man presents
systolic ejection murmur along the sternal bor-
to the emergency department after 2 hours of
der. Laboratory studies show:
increasing headache, nausea, and vomiting.
His medical history includes hypertension and WBC count: 6200/mm3
diabetes. His heart rate is 41/min, blood pres- Hemoglobin: 15 g/dL
sure is 150/74 mm Hg, respiratory rate is 12/ Platelet count: 190,000/mm3
min, and temperature is 36.9C (98.5F). The Ca2+: 6.2 mg/dL
patient is alert and protecting his airway. Neu- 25-Hydroxyvitamin D: 50 ng/dL
rological examination shows marked left-sided PO4: 4 mg/dL
weakness and dilated right pupil. After order- Mg2+: 2 mg/dL
ing the appropriate imaging studies, what is the
Which of the following is the most likely diag-
rst priority in caring for this patient?
nosis?
(A) Administer antiemetics
(A) Beckwith-Wiedemann syndrome
(B) Administer thrombolytic agents within 3
(B) Chronic granulomatous disease
hours
(C) DiGeorges syndrome
(C) Aggressive blood pressure management
(D) Multiple endocrine neoplasia type 1
(D) Frequent blood glucose level measure-
(E) Niemann-Pick disease
ments
(E) Neurosurgical consultation
30. A 34-year-old woman is seen by a neurologist
complaining of a 3-month history of head-
28. A 27-year-old woman was recently diagnosed
aches. The neurologist promptly orders MRI of
with partial seizures and started on a medica-
the brain, which reveals a tumor that has a
tion for this condition. After 2 months the pa-
large central lesion in the posterior parietal
tient complains that her coworkers have noted
lobe, along with multiple, smaller satellite le-
that she has become mentally sluggish. Addi-
sions around it. The tumor is resected, and re-
tionally, the patient was in the emergency de-
sults of a biopsy are shown in the image.
partment last weekend with horrible back pain
Which of the following is the most likely diag-
and was diagnosed with a kidney stone. Which
nosis?
of the following is most likely responsible for
this patients symptoms?
HIGH-YIELD SYSTEMS
Chapter 10: Neurology Questions 293

33. A 4-year-old boy is brought to the local emer-


gency department by his mother because he is
not acting right. His mother, who is preg-
nant, reports that he seemed well last night,
but this morning he was uncharacteristically
groggy and appeared ushed. He fell over
twice while walking around the house and
passed out for 20 seconds while eating break-
fast. Although he did not hit his head, he did
complain of a headache. On the way to the
hospital he vomited once; it was nonbloody
Image courtesy of PEIR Digital Library (http://peir.net).
and nonbilious. There are no sick contacts in
the household, but his father awoke with an
(A) Acoustic schwannoma unusually severe headache this morning. The
(B) Cavernous malformation family is vacationing at a nearby mountain re-
(C) Glioblastoma multiforme

Neurology
sort noted for its rustic log cabins with wood-
(D) Medulloblastoma burning replaces. His temperature is 37C
(E) Meningioma (98.6F), blood pressure is 90/50 mm Hg,
pulse is 130/min, and respiratory rate is 26/
31. The parents of a 6-year-old boy bring their min. The patient is minimally cooperative; he
child to the physician for episodes too numer- refuses to walk, preferring to nap in his moth-
ous to count of the child staring off into ers arms. There is no evidence of head trauma
space while uttering his eyes. His attention and his physical examination is otherwise un-
returns after 510 seconds, and he is never remarkable. Further testing would most likely
aware of the lapses in his attention. Which an- reveal which of the following abnormalities?
tiepileptic is the rst-line agent for children
with this disorder? (A) Blood glucose level of 60 mg/dL
(B) Low partial pressure of arterial oxygen
(A) Carbamazepine measured on arterial blood gas testing
(B) Ethosuximide (C) Metabolic acidosis with increased anion
(C) Gabapentin gap
(D) Tiagabine (D) Pulmonary effusion on x-ray of the chest
(E) Valproate (E) Pulse oximetry reading of 89%

32. A young girl hops and skips her way into her 34. A 75-year-old man with a history of carotid
pediatricians ofce. She takes a seat beside her stenosis presents with acute left-sided hemipa-
mother and is able to copy crosses in the work- resis and aphasia. He was brought to the emer-
book the nurse hands her but is not able to gency department from his nursing home by
print her name. She then counts to 10 and be- an ambulance, which was held up in trafc for
gins to sing a song she was taught by her older at least half an hour. He remains somewhat re-
sister. How old is she? sponsive with stable vital signs but his neuro-
(A) 2 years old logic decits are unchanged. The physician
(B) 3 years old also notes that his clothing is soiled with urine
(C) 4 years old and feces, although he is known to be conti-
(D) 5 years old nent at baseline. Which of the following is the
(E) 6 years old most appropriate initial imaging study?
(A) Carotid ultrasound
(B) Cerebral angiography
(C) Conventional radiography
(D) CT of the head without contrast
(E) MRI of the brain
HIGH-YIELD SYSTEMS
294 Section I: Organ Systems Questions

35. A 36-year-old woman presents to the emer- to have a large laceration over the occipital
gency department after experiencing a seizure area of his head. He is stabilized, given ade-
in her apartment 3 hours earlier. She has no quate lactated Ringers solution and blood
known prior medical problems, and takes only transfusions for hypovolemia due to acute
aspirin for some recent headaches. She is cur- blood loss, and taken to the intensive care unit.
rently lethargic but responsive and coherent. Two days later he is lethargic and has a serum
Her temperature is 37.2C (98.9F), blood sodium level of 118 mEq/L. What is this pa-
pressure is 117/68 mm Hg with no orthostatic tients expected plasma osmolality, urine osmo-
changes, and pulse is 70/min. Physical exami- lality, and clinical volume status?
nation reveals anicteric sclerae with reactive
pupils and moist mucous membranes. Labora- CHOICE PLASMA URINE CLINICAL STATUS
tory studies show:
Na+: 124 mEq/L A hypotonic hypertonic euvolemic
Ca2+: 9.8 mg/dL
Cl: 100 mEq/L B hypotonic hypertonic hypervolemic
Neurology

HCO3: 19 mEq/L
Blood urea nitrogen: 9.2 mg/dL C isotonic hypertonic hypovolemic
Creatinine: 0.9 mg/dL
Glucose: 110 mg/dL
D hypertonic hypotonic hypovolemic
Liver function tests, amylase, lipase, and biliru-
bin are normal. Urine electrolytes reveal an el- E hypertonic hypotonic euvolemic
evated urine sodium. What is the most likely
cause of this patients hyponatremia?
(A) A
(A) Bartters syndrome
(B) B
(B) Brain tumor
(C) C
(C) Hyperglycemic crisis
(D) D
(D) Primary biliary cirrhosis
(E) E
(E) Subarachnoid hemorrhage
38. A 20-year-old college student is brought to the
36. A 58-year-old African-American man with hy-
emergency department after she was hit on the
pertension and diabetes presents to the oph-
head by a stray baseball. The patient was con-
thalmology clinic for a routine examination.
scious when the rst responders arrived at the
The patient has no visual complaints. Visual
scene, but she began complaining of a severe
acuity is 20/20 in both eyes. Funduscopic ex-
headache and then lost consciousness en route
amination reveals enlarged optic nerve head
to the hospital. Initial CT scan shows a large
cupping with signicant rim pallor in both
epidural hematoma that requires emergent
eyes. Gonioscopy reveals open angles, and ap-
evacuation. The patient has not regained con-
planation tonometry reveals borderline intraoc-
sciousness, and the clerk has not yet found her
ular pressures. Which of the following agents
emergency contact information. Which of the
could be used to treat this patients condition?
following is the most appropriate next step in
(A) Corticosteroids management?
(B) Intravenous mannitol
(A) Ask the patients dormitory housemaster to
(C) Isoproterenol
sign a consent form for emergency treat-
(D) Phenoxybenzamine
ment
(E) Pilocarpine
(B) Infuse mannitol to decrease intracranial
pressure while waiting for the patients par-
37. A 27-year-old pedestrian is brought to the
ents to arrive
emergency department by ambulance follow-
(C) Perform emergent evacuation of epidural
ing an accident in which he was struck by a
hematoma
moving car. Among other injuries he is found
HIGH-YIELD SYSTEMS
Chapter 10: Neurology Questions 295

(D) Perform serial neurologic examinations (A) Death by age 45 years


while waiting for the clerk to obtain the in- (B) Future organomegaly and easy bruising
formation about the patients emergency (C) Hyperactivity if left untreated
contact (D) Opisthotonos
(E) Wait for the parents to arrive before pro- (E) Spasticity and rigidity in type A of this con-
ceeding with any treatment dition

39. A middle-aged man is brought to the emer- 41. A 22-year-old previously healthy woman pre-
gency department by ambulance after develop- sents to the emergency department complain-
ing acute-onset hemiparesis and aphasia while ing of the worst headache of her life, which
at work. Brain imaging identies a large ische- started suddenly 3 hours ago. She denies
mic area in the right anterior cerebral artery/ photo- or phonophobia, but complains of
middle cerebral artery distribution. Bedside ca- blurry vision associated with the onset of the
rotid duplex shows complete occlusion of the headache. CT scan of the head without con-
right internal carotid artery just distal to the bi- trast reveals a Fisher grade III subarachnoid
furcation. Laboratory tests show a platelet hemorrhage. The patient is transported back to

Neurology
count of 250,000/mm, blood glucose of 110 the trauma bay, at which time she loses con-
mg/dL, and normal coagulation times. His sciousness and seizes. Which of the following
blood pressure is 140/85 mm Hg and he is afe- is the rst step in management of this patient
brile. Due to confusion and aphasia, it is al- after the seizure?
most impossible to obtain a history from the
(A) Antiepileptic and mannitol
patient. The physician is planning on adminis-
(B) Cervical spine evaluation/immobilization
tering thrombolytics, but needs certain key in-
(C) Intubate the patient
formation beforehand. Suddenly, the patients
(D) Start oxygen by nasal cannula
wife arrives in the emergency department.
(E) Trauma survey
Which of the following is a contraindication to
thrombolytic therapy? 42. A 58-year-old woman has been suffering from
(A) Bruise on the leg daily headaches for months. They are not asso-
(B) Documented carotid stenosis ciated with aura, nausea, vomiting, or photo-
(C) Myocardial infarction 10 years ago phobia. They last for 6 hours per day and oc-
(D) Peptic ulcer when he was 16 years old, cur about half the month. Her medical history
treated is signicant only for arthritis, for which she
(E) Stroke 2 months ago has been taking high doses of nonsteroidal
anti-inammatory drugs and acetaminophen-
40. A 6-month-old boy of Ashkenazi Jewish de- based analgesics daily for the past several
scent presents to his pediatricians ofce. His months. She has been gradually escalating her
mother is concerned that he isnt developing analgesic dose to manage her condition. Her
right. At 2 months he was following objects as physical examination is within normal limits.
they moved across the midline and lifting his Which of the following is most likely to be ef-
head. However, for the past month he has been fective in preventing her daily headaches?
less alert, startles more easily, and has not been
(A) Behavioral modication therapies
lifting his head. Physical examination is unre-
(B) Controlling her analgesic use
markable, but ophthalmologic examination
(C) Prophylactic treatment with -blockers
shows a bright red macula surrounded by a
(D) Treatment with selective serotonin
whitish ring. What is this patients clinical
reuptake inhibitors
prognosis?
(E) Treatment with tricyclic antidepressants
HIGH-YIELD SYSTEMS
296 Section I: Organ Systems Questions

43. A 12-month-old boy is brought to his pediatri- 44. A 76-year-old man with chronic hypertension
cian by his parents. He had a normal, healthy presents to the emergency department with al-
childhood until 1 month ago. His parents com- tered mental status and elevated blood pres-
plain that over the past month he has been sure in the 230/140 mm Hg range as a result of
vomiting frequently, and also appears to be in- noncompliance with blood pressure medica-
creasingly irritable. He is now taking more tion. The physician suspects a bleed based on
naps during the day and is playing less than CT of the head. Where does bleeding typically
usual. His parents also think there is a problem occur in this patient population?
with his sight, as he often does not appear to
(A) Basal ganglia/thalamus
see toys and furniture that are directly in his
(B) Brain stem
line of vision. The child is afebrile with a pulse
(C) Epidural space
of 140/min, blood pressure of 90/56 mm Hg,
(D) Ventricles
and respiratory rate of 24/min. Results of oph-
thalmologic examination are shown in the im-
45. An 8-year-old girl is brought to the emergency
age. Which of the following is the most appro-
department for evaluation of a seizure. She
Neurology

priate next step in managing this patient?


rst complained of stomach pain and soon af-
terward smacked her lips for 10 minutes. Dur-
ing that time she was unresponsive, and she
does not remember the event. There is no his-
tory of head trauma or a history of seizures.
Her mother had seizures that resolved in pu-
berty and is distraught about her daughters sei-
zure. Her temperature is 37.8C (100.0F),
blood pressure is 90/60 mm Hg, heart rate is
70/min, and respiratory rate is 22/min. She is
drowsy but is able to be aroused. Her neck is
supple, and the remainder of the neurologic
examination is normal. A complete metabolic
panel, complete blood cell count and liver
function tests, is normal except for a slightly el-
evated prolactin level. CT scan of the head
with contrast is negative. An EEG shows slow
waves throughout, with sharp waves over the
left temporal region. What is the most appro-
priate therapy for this child?
Reproduced, with permission, from Ropper AH, Brown (A) Acetaminophen
RH. Adams and Victors Principles of Neurology, 8th edi- (B) Aspirin
tion. New York: McGraw-Hill, 2005: Figure 13-9. (C) Cefotaxime and vancomycin
(A) Administer mannitol (D) Ethosuximide
(B) Administer timolol (E) Phenobarbital
(C) CT of the head without contrast
(D) Lumbar puncture
(E) MRI of the brain
HIGH-YIELD SYSTEMS
Chapter 10: Neurology Questions 297

46. A 65-year-old woman is brought to the physi- 48. A 64-year-old male smoker presents to the
cian by her son because he is concerned about clinic with gradual onset distortion of straight
her memory loss over the past year. Yesterday, lines, blurring of small objects, and distur-
after going for a walk, she could not remember bances in color vision. He has decreased visual
her way back home, even though she was only acuity in the central visual elds of both eyes.
two blocks away from the house. Although the Funduscopy shows drusen accompanied by a
patient denies any major problems, her son choroidal neovascular membrane, subretinal
says she is forgetful and becomes confused. hemorrhages, subretinal brosis, and retinal
There is a family history of dementia. She has hemorrhage. Which of the following is the
no history of drug or alcohol abuse. Her vital most appropriate next step in management?
signs are within normal limits. She is oriented
(A) Atropine
to person, place, and time. She can recall past
(B) Laser photocoagulation
events quite well but her recent memory is im-
(C) Timolol
paired. She has difculty recalling the names
(D) Topical steroids
of common objects. Her physical examination,
(E) Trabeculectomy

Neurology
laboratory test results, and thyroid tests indicate
no abnormalities. Which of the following is the
49. A 79-year-old woman is in the early stages of
most appropriate next step in management?
Alzheimers disease. She has some mild recent
(A) Bupropion memory loss and difculty with everyday mo-
(B) Donepezil tor tasks, which has been progressive over 2
(C) Doxepin years. She has no living will and has not desig-
(D) Gingko biloba nated a durable power of attorney in the event
(E) Thiamine that she becomes incapacitated. Which of the
(F) Vitamin E following statements regarding advance direc-
tives in patients with early Alzheimers disease
47. After a sailing accident in which a man falls is true?
overboard, he arrives at the emergency depart-
(A) Family members should be allowed to de-
ment with a temperature of 25C (77.0F), no
cide on future treatment plans
spontaneous respirations or cardiac activity,
(B) It is impossible for these patients to make
and a depressed skull fracture. Rewarming is
decisions regarding their health care due
begun, the man is intubated and mechanically
to loss of self-identity
ventilated, and cardiotropic medications are
(C) Patients generally do not have a sufcient
administered, resulting in a faint, palpable,
level of executive function to designate a
pulse. The dolls-eyes reex is absent. There
health care proxy
is no deviation of the eyes in response to irriga-
(D) Substituted judgment should be applied to
tion of the ear canal with ice water. His Glas-
determine the patients proxy
gow Coma Scale score is 3/15. Body tempera-
(E) The physician should discuss the course
ture after CT is 28.1C (82.5F). Which of the
and prognosis of the disease with the pa-
following characteristics would prevent brain
tient
death from being declared in this patient at
this time?
(A) Articial ventilation
(B) Body temperature
(C) Cardiac activity
(D) Glasgow Coma Scale score of 3
(E) Skull fracture
HIGH-YIELD SYSTEMS
298 Section I: Organ Systems Questions

50. A 50-year-old man presents to his primary care Aspartate aminotransferase: 20 U/L
physician with difculty achieving an erection. Alkaline phosphatase: 50 U/L
He states that he has noticed a decrease in his Blood urea nitrogen: 18 mg/dL
sex drive during the past 23 months. The man Creatinine: 0.9 mg/dL
has a history of diet-controlled hypertension Cortisol (fasting): 11 mg/dL
and had a left total knee replacement 1 year Prolactin: 38 ng/mL
ago. He is in a monogamous relationship with Thyroid-stimulating hormone: 2.8 mg/dL
his wife and has been for 22 years. His blood Total testosterone: 190 ng/dL
pressure is 130/90 mm Hg, temperature is Free testosterone: 40 pg/mL
37.2C (98.9F), and heart rate is 86/min. On
Which of the following is the most appropriate
examination his confrontational visual elds
diagnostic test at this point?
reveal a small bitemporal superior visual eld
loss. The rest of the mans cranial nerves are (A) Mammogram
normal. His abdominal, genital, and prostate (B) Measurement of -human chorionic go-
examinations are unremarkable. Laboratory nadotropin level
(C) Monitor and repeat blood work in 6
Neurology

tests show:
months
WBC count: 5600/mm3
(D) MRI of the brain
Hematocrit: 40%
(E) Ultrasonography of scrotum
Platelet count: 156,000/mm3
HIGH-YIELD SYSTEMS
Chapter 10: Neurology Answers 299

AN S W E R S

1. The correct answer is B. This child has cere- Answer E is incorrect. Fragile X syndrome is
bral palsy (CP), which is the most prevalent the most common cause of mental retardation
nonprogressive motor disorder in children. The in male patients. It is characterized by abnor-
most common type of CP is pyramidal, account- mal facies (macrocephaly, elongated face, and
ing for 75% of the cases. Mental retardation is prominent ears), macroorchidism, and stereo-
present in 90% of cases of pyramidal CP. Pyra- typed behavior rather than movement prob-
midal CP is nonprogressive and characterized lems.
by spasticity, hyperreexia, slow effortful volun-
tary movements, and impaired ne motor func- 2. The correct answer is E. The patients symp-
tion. Delayed developmental milestones, persis- toms are consistent with a herniated nucleus
tence of infantile reexes (e.g., Babinskis pulposus. Spontaneous improvement is the
reex), contractures, and weakness or underde- rule; >90% of patients have resolution within 6
velopment of affected limbs are common. Chil- weeks of pain onset. Conservative treatment in-

Neurology
dren typically walk on their toes and have a scis- volves rest: 1 or 2 days of bedrest (longer is
sor gait. The cause of CP is usually not clearly counterproductive as deconditioning begins)
dened; however, risk factors include prematu- followed by 2 to 3 weeks of activity limitation
rity, perinatal asphyxia, intrauterine growth re- (reduce lifting, twisting) with reassessment
striction, infection, trauma, brain malformation, prior to lifting activity restriction. Nonsteroidal
and hemorrhage. anti-inammatory drugs, moist heat, and gen-
tle exercise to promote joint mobility are all
Answer A is incorrect. Autism is characterized
typically helpful.
by poor verbal and nonverbal communication
skills, repetitive stereotyped behaviors, mental Answer A is incorrect. Imaging is reserved for
retardation, abnormal responses to the environ- patients with systemic symptoms (fever, weight
ment, and poor social skills. These symptoms loss) when cancer or infection is high on the
are present before the age of 3 years. Spasticity, differential, in patients with trauma (i.e., motor
hyperreexia, voluntary movement disorders, vehicle accident), and in patients who do not
and impaired ne motor function are not typi- improve with more conservative measures.
cally part of this condition.
Answer B is incorrect. Imaging is reserved for
Answer C is incorrect. Downs syndrome is patients with systemic symptoms (fever, weight
the most common cause of mental retardation. loss) when cancer or infection is high on the
It is characterized by abnormal facies (at fa- differential, in patients with trauma (i.e., motor
cial prole and prominent epicanthal folds), vehicle accident), and in patients who do not
a simian crease, and cardiac malformations. improve with more conservative measures.
However, Downs syndrome does not cause
Answer C is incorrect. Imaging is reserved for
spasticity, hyperreexia, or other movement
patients with systemic symptoms (fever, weight
complaints described in this patient.
loss) when cancer or infection is high on the
Answer D is incorrect. Duchennes muscular differential, in patients with trauma (i.e., motor
dystrophy (DMD) is the most common heredi- vehicle accident), and in patients who do not
tary neuromuscular disease and is sometimes improve with more conservative measures.
associated with mental retardation. There is a
Answer D is incorrect. Surgical decompres-
progressive loss of muscle strength. The classic
sion of the nerve root would be reserved for
sign is a Gowers maneuver, in which a child
cases intractable to more conservative mea-
pushes off the oor and walks his hands up his
sures.
legs to assume a standing position. DMD typi-
cally affects the limb girdle muscles initially 3. The correct answer is C. The child has a clas-
and is progressive. sic clinical presentation of Wests syndrome,
HIGH-YIELD SYSTEMS
300 Section I: Organ Systems Answers

which is characterized by infantile spasms, ar- Answer E is incorrect. Pelvic oor damage,
rest of psychomotor development and a hypsar- often secondary to trauma such as childbirth,
rhythmia pattern on EEG. leads to sphincteric insufciency and stress, or
in severe cases, total incontinence. Spasticity
Answer A is incorrect. Although inferior to
would be an unlikely presentation. Further-
MRI, CT would be useful if one suspected
more, the response to bethanechol chloride
seizures secondary to a central nervous system
would not be expected to be increased.
(CNS) neoplasm.
Answer B is incorrect. ECG would not be use- 5. The correct answer is B. Lesch-Nyhan disease
ful in this context, but is easily confused with is an X-linked recessive disorder and is caused
EEG. by a deciency of hypoxanthine-guanine phos-
horibosyltransferase. This deciency causes a
Answer D is incorrect. Electromyography
build-up of uric acid (hence the increase in
would not be useful in this context, but is often
blood urea nitrogen) and can lead to renal fail-
confused with EEG and is useful in the diag-
ure and kidney stones. Children are healthy
nosis of many neurodegenerative syndromes in
until their st year of life, when they show cho-
Neurology

adults.
reothetosis, self-mutilation, psychomotor retar-
Answer E is incorrect. Lumbar puncture (LP) dation, and hypertonia. Allopurinol, a xanthine
would be more helpful in diagnosing infec- oxidase inhibitor, is a treatment for the renal
tious causes of meningitis. On rare occasion, manifestations; however, no treatment exists
LP might reveal a metabolic cause of Wests for the neurological sequela.
syndrome such as nonketotic hypoglycinemia.
Answer A is incorrect. Galactosemia is caused
4. The correct answer is D. Bethanechol chlo- by a lack of galactose-1-phosphate uridyltrans-
ride is a parasympathomimetic drug that is ferase. Symptoms include mental retardation,
used during cystometry to aid in diagnosis. cataracts, and hepatosplenomegaly.
Lack of a contraction to bethanechol chloride Answer C is incorrect. Maple syrup urine dis-
suggests detrusor muscle damage and an in- ease is caused by a block in the breakdown of
creased response suggests upper motor neuron branched chain amino acids (i.e., isoleucine,
dysfunction. Multiple sclerosis (MS) is an au- leucine, and valanine). Increased ketoacids
toimmune disease that is the result of CNS de- in the blood lead to mental retardation and
myelination. It most commonly affects young death, but do not lead to an increase in the
women and can be exacerbated by stress. Blad- blood urea nitrogen level or choreothetosis
der spasticity and urinary retention can be the with self mutilation.
presenting symptoms.
Answer D is incorrect. Phenylketonuria (PKU)
Answer A is incorrect. Although acute cystitis is an autosomal recessive disorder character-
is common in young women, it is often accom- ized by increased levels of phenylalanine ke-
panied by dysuria and RBCs, WBCs, and bac- tones caused by a deciency in phenylalanine
teria in the urine. The response to bethanechol hydroxylase. Presenting symptoms are mental
chloride should not be inuenced by cystitis. retardation and a musty body odor.
Answer B is incorrect. Chronic bladder disten- Answer E is incorrect. Wilsons disease is an
tion, often secondary to outow obstruction, autosomal recessive disorder caused by abnor-
presents with overow incontinence, not spas- mal metabolism of copper leading to copper ac-
ticity, and would not be expected to lead to an cumulation in the brain, liver, eyes, and bone.
increased response to bethanechol chloride. Clinical signs and symptoms include hepato-
Answer C is incorrect. The response to splenomegaly, anemia, hepatitis, jaundice, and
bethanechol chloride should be decreased in a deterioration of neurologic functioning.
cases of detrusor dysfunction due to muscle
damage.
HIGH-YIELD SYSTEMS
Chapter 10: Neurology Answers 301

6. The correct answer is D. Neuroimaging with <5 WBCs/mm are present with 75% or more
CT is the rst essential step in conrming a di- lymphocytes. Normal protein levels are 2045
agnosis of cerebral hemorrhage, which is sus- mg/dL and normal glucose levels are >50 mg/
pected given the patients history of presenta- dL, or approximately 75% serum glucose.
tion and the acuity of the symptoms. Patients
Answer C is incorrect. These CSF ndings
with adult polycystic kidney disease are at in-
are typical of viral meningitis. In such patients,
creased risk for developing berry aneurysms.
pressure is normal or slightly elevated (80150
Answer A is incorrect. Testing the patients mm H2O), and there are rarely >1000 WBCs/
blood has no immediate role in the establish- mm present. Polymorphonuclear leukocytes
ment of a diagnosis of cerebral hemorrhage. predominate early in the course of the illness,
and mononuclear cells predominate through-
Answer B is incorrect. While useful for diag-
out the illness. Protein is often elevated to 50
nosing seizure disorders, EEG has no utility in
200 mg/dL and glucose levels are often normal
diagnosing acute cerebral hemorrhage.
but may be decreased, depending on the viral
Answer C is incorrect. Although an LP may pathogen.

Neurology
have diagnostic value in order to assess the
Answer D is incorrect. This is representative
presence of RBCs or the presence of xantho-
of fungal meningitis. Pressure is often elevated,
chromia, it should not be performed without
and 5500 WBCs/mm are present, with poly-
having had a prior CT/MRI, because surround-
morphonuclear leukocytes predominating
ing edema may cause shifts in brain content,
early in the course, and mononuclear cells
and brain herniation into the spinal canal can
predominate throughout the majority of the
occur, leading to death.
illness. Protein levels are often 25500 mg/dL
Answer E is incorrect. Although MRI can be and glucose levels are generally <50 mg/dL.
helpful to determine the presence of blood
Answer E is incorrect. Acute syphilis and lep-
or other abnormalities, scanning may take up
tospirosis will provide CSF ndings consistent
to an hour. This patient has a possible acute
with those seen in this option. Pressure is el-
bleed and needs to be assessed quickly.
evated at 50500 mm H2O and WBCs are pre-
Answer F is incorrect. A positron emission to- dominantly present, 50200 mg/dL of protein
mography scan has no role in the acute diag- is found, and glucose levels are usually nor-
nosis of a cerebral hemorrhage. mal.

7. The correct answer is B. Acute bacterial men- 8. The correct answer is D. According to the Di-
ingitis in an infant often manifests with signs agnostic and Statistical Manual of Mental Dis-
and symptoms similar to the vignette (i.e., fe- orders, Fourth Edition, Text Revision, gradual
ver, increased irritability or lethargy, decreased onset of symptoms and continued cognitive de-
oral intake, papilledema, and a bulging fonta- cline are criteria for diagnosing Alzheimers
nel). Laboratory evaluation of cerebrospinal disease (AD). Other criteria include impair-
uid (CSF) will demonstrate an elevated open- ment of recent memory (inability to learn new
ing pressure (usually 100300 mm H2O) with information) and at least one of the following:
10010,000 WBCs/mm, most of which will disturbance of language; inability to execute
be polymorphonuclear leukocytes. Protein lev- skilled motor activities in the absence of weak-
els are elevated (100500 mg/dL), and glucose ness; disturbances of visual processing; or dis-
is decreased to <50% of serum glucose levels. turbances of executive function (including ab-
Acute bacterial meningitis is a life-threatening stract reasoning and concentration). Behavioral
condition that must be rapidly diagnosed and problems are common in patients with AD;
treated in affected patients to reduce morbidity personality changes (ranging from progressive
and mortality. passivity to open hostility) may precede the
Answer A is incorrect. This is typical of CSF cognitive impairments. AD is the most com-
ndings in a normal individual. Normal open- mon form of dementia in the elderly, account-
ing pressure is 5080 mm H2O, and generally ing for 60%80% of cases. The brains of indi-
HIGH-YIELD SYSTEMS
302 Section I: Organ Systems Answers

viduals with AD are characterized by mon form of neurodegenerative dementia af-


extracellular deposition of amyloid- protein, ter AD. The core clinical features of DLB are
intracellular neurobrillary tangles, and loss of a gradually progressive dementia; uctuations
neurons. in cognitive function; persistent, well-formed
visual hallucinations; and spontaneous motor
Answer A is incorrect. A high level of pro-
features of parkinsonism. Supportive features
tein 14-3-3 in the CSF is associated with
of DLB include repeated falls, syncope, sen-
Creutzfeldt-Jakob disease (CJD). CJD is the
sitivity to neuroleptic medications, delusions,
most frequent of the human prion diseases.
hallucinations in nonvisual senses, rapid eye
Rapidly progressive mental deterioration and
movement sleep behavior disorders, and de-
myoclonus are the two cardinal clinical mani-
pression. The diagnosis is based on history and
festations of CJD. Myoclonus, especially pro-
clinical features.
voked by startle, is present in more than 90%
of patients at some point during the illness.
9. The correct answer is B. This patient meets
The gold standard of diagnosis involves brain
the diagnostic criteria for delirium. Character-
biopsy. The etiology of CJD can be familial or
istics of delirium include rapid onset, a uctu-
Neurology

iatrogenic (from corneal transplantation or du-


ating and cloudy level of consciousness, im-
ral grafts). There is no effective treatment for
paired orientation, disordered thinking, and
CJD, which is uniformly fatal.
usually complete reversibility of the condition
Answer B is incorrect. Focal neurologic nd- (as with this patient who returned to normal
ings and evidence on physical examination functioning a day later). Postoperative delirium
consistent with prior strokes is associated with is common in patients who are elderly, regard-
vascular dementia. Vascular dementia would less of preexisting psychopathology. The proper
also be suggested if a patient has an abrupt approach is to identify and correct underlying
onset of symptoms followed by stepwise dete- causes such as dehydration, electrolyte imbal-
rioration, and has infarcts on cerebral imaging. ance, infection, and polypharmacy.
Although diabetes is associated with vascular
Answer A is incorrect. The combination of
disease and the risk factors of stroke, there are
chlordiazepoxide (Librium), thiamine, folic
no focal neurologic decits or a prior history of
acid, and a multivitamin is used to treat alco-
strokes.
hol withdrawal. There is no mention of alco-
Answer C is incorrect. A triad of gait dysfunc- hol use and dependence in this patient.
tion, urinary incontinence, and cognitive dys-
Answer C is incorrect. Donepezil is the treat-
function help establish a diagnosis of normal
ment of choice for patients with AD. The pa-
pressure hydrocephalus. This patient does not
tients condition can be clinically differentiated
have these symptoms. Furthermore, CT scan
from dementia on the basis that dementia is a
of the head may show dilated cerebral ventri-
chronic disease, has an insidious onset, and is
cles. Normal pressure hydrocephalus occurs
chronically progressive. Level of consciousness
in elderly patients and is due to an increased
and orientation is intact initially, but the ma-
subarachnoid space volume that does not ac-
jority of cases of dementia are not reversible.
company increased ventricular volume. Some
Therefore, this answer is incorrect.
believe that the initial event is diminished ab-
sorption of CSF at the arachnoid villi. Clinical Answer D is incorrect. Lithium and a seda-
symptoms result from distortion of the central tive drug are used in cases of acute manic epi-
portion of the corona radiata by the distended sodes. Although the patient in this case has a
ventricles. Dementia results from distortion of previous diagnosis of bipolar disorder, there is
the periventricular limbic system. no evidence that he is having an acute manic
episode that would involve an expansive or irri-
Answer E is incorrect. Lewy bodies in the
table mood lasting at least 1 week, grandiosity,
brainstem on autopsy are seen in dementia
pressured speech, or ight of ideas.
with Lewy bodies (DLB) associated with Par-
kinsons disease. DLB is the second most com-
HIGH-YIELD SYSTEMS
Chapter 10: Neurology Answers 303

Answer E is incorrect. Delirium is a medical the enzyme that converts phenylalanine to ty-
urgency, and its cause must be identied as rosine. Therefore, in patients with PKU pheny-
soon as possible. Therefore, watchful waiting is lalanine is not converted to tyrosine, causing a
not the correct answer. build-up of phenylketones. These phenylke-
tones are toxic and cause mental retardation
10. The correct answer is B. The infant has and a musty body odor. The only therapy to
Downs syndrome; classic physical ndings in- date is the dietary restriction of phenylalanine
clude at facial prole, prominent epicanthal and an increase of tyrososine.
folds, upslanting palpebral ssure, clinodactyly,
Answer A is incorrect. While phenylalanine
simian crease, and congenital heart disease. If
hydroxylase is the enzyme missing in PKU, its
dating of conception is reliable, at 1518 weeks
administration is not currently the standard of
of gestation a triple marker screen can be per-
care.
formed. This is the classic prole seen in tri-
somy 21. An amniocentesis is offered to women Answer B is incorrect. Phenylacetate is a ke-
who have abnormal triple screens or those who tone of phenylalanine and is increased in PKU.
will be >35 years old at the time of delivery. Therefore, decreasing the amounts of ketones

Neurology
Karyotyping is performed on amniotic uid by restricting phenylalanine is the treatment
and allows a denitive diagnosis to be made. for PKU.
The infant was at increased risk of having
Answer C is incorrect. Increasing phenylala-
Downs syndrome because her mother is >35
nine in the diet will only exacerbate PKU, as
years old.
phenylalanine ketones will build up and caus-
Answer A is incorrect. These results are classi- ing more neurological impairment.
cally seen in fetuses with trisomy 18. Trisomy
Answer E is incorrect. Restricting tyrosine
18, or Edwards syndrome, classically presents
from the diet will only exacerbate the symp-
with rocker-bottom feet, low-set ears, micro-
toms of PKU as tyrosine levels are already low
gnathia, prominent occiput, clenched hands,
from the lack of phenylalanine hydroxylase.
and congenital heart disease.
Therefore, therapy involves increasing the
Answer C is incorrect. These results are abnor- amount of tyrosine in the diet.
mal but are not specic for any particular con-
dition. The -fetoprotein (AFP) and -human 12. The correct answer is D. The rst step in the
chorionic gonadotropin (-hCG) levels suggest diagnosis and clinical assessment of a patient
Downs syndrome; however, the estradiol level with altered consciousness is to perform a pri-
is trending in the wrong direction. mary survey that provides rapid identication
of potentially fatal conditions. This patient was
Answer D is incorrect. These results are ab-
found down and his provider should have a
normal but are not specic for any particular
high degree of suspicion for injury. The pri-
condition. Although an elevated AFP level is
mary survey can be remembered as the AB-
seen in neural tube defects, -hCG and estra-
CDEs: Airway, Breathing, Circulation, Dis-
diol levels are not markers for this condition.
ability, and Exposure. This patient is breathing,
Answer E is incorrect. These results are ab- but this does not mean the airway is clear! The
normal but are not specic for any particular breathing could be labored or difcult due to
condition. Although the estradiol and -hCG impending airway compromise or obstruction.
levels suggest Downs syndrome, the AFP level Thus, the rst step in management would be
is trending in the wrong direction. establishing a denite airway by intubation.
Once the ABCs have been examined, disability
11. The correct answer is D. This childs labora- should be ascertained by using the Glasgow
tory tests are diagnostic of PKU, an autosomal Coma Scale (GCS). This patient has a GCS
recessive disease usually screened for on neo- score of 6. Any patient with a GCS score <8
natal blood work but was probably missed due should be intubated to establish a denitive
to foreign adoption. PKU is caused by de- airway. (Notice that we did not need the pa-
creased levels of phenylalanine hydroxylase, tients GCS in order to arrive at our rst man-
HIGH-YIELD SYSTEMS
304 Section I: Organ Systems Answers

agement step, but the GCS conrmed our de- Answer A is incorrect. Absence seizures have
cision.) a characteristic 3-Hz spike and wave pattern.
These are typically very brief, lasting only a few
Answer A is incorrect. A patient may experi-
seconds, and occur many times per day in af-
ence obtundation secondary to a number of
fected children. Focal seizures, whether simple
factors, including pharmaceutical sedation or
or complex, do not result in loss of conscious-
comorbid disease processes. Although revers-
ness and typically involve a discrete region in
ing common reversible causes with naloxone,
one cerebral hemisphere. While it is possible
thiamine, and dextrose is a reasonable step, the
for a focal seizure to generalize into a grand
rst priority is the primary survey, which can
mal seizure, there is no evidence that this oc-
be remembered as the ABCDEs.
curred in this patient.
Answer B is incorrect. Checking brainstem
Answer B is incorrect. This EEG pattern is
reexes is an important part of the neurologic
typical for CJD.
examination of a patient with altered mental
status; however, this should occur after airway, Answer C is incorrect. Partial seizures cause
breathing, and circulation are properly main- activity in only one hemisphere as opposed to
Neurology

tained. both hemispheres. In this case, symptoms may


not be bilateral.
Answer C is incorrect. A patient can have an
altered mental status secondary to electrolyte Answer E is incorrect. Focal seizures cause ac-
imbalances and comorbid disease processes; tivity located to a discrete portion of one hemi-
although a serum electrolyte panel should be sphere and may not cause tonic-clonic reac-
examined, diagnostic testing belongs in the tions.
secondary survey. The rst step in clinical as-
sessment is to complete the primary survey, 14. The correct answer is B. The skin ndings in
which can be remembered as the ABCDEs. the photograph are caf-au-lait spots, and the
eye nding is an accurate description of a
Answer E is incorrect. Exposure of the patient
Lisch nodule, all consistent with neurobro-
is the last step in the primary survey. Removal
matosis (NF) type I (an autosomal dominant
of clothing from the patient aids the primary
disease caused by a gene found on chromo-
survey and is necessary to complete the sec-
some 17).
ondary survey. It should be performed after
the ABCs are managed (Airway, Breathing, Answer A is incorrect. Neither Lisch nod-
and Circulation), and the patients disability ules nor caf-au-lait spots are found in mul-
level has been determined, but before the sec- tiple neuroendocrine neoplasia (MEN) type I.
ondary survey is started. MEN type I is typied by the 3 Ps: Pituitary
(usually prolactinoma), Parathyroid (hypercal-
13. The correct answer is D. This patient with an cemia), and Pancreatoma (gastrinoma). MEN
underlying seizure disorder has most likely suf- type I is autosomal dominant and tends to pre-
fered a grand mal (tonic-clonic) seizure involv- sent in middle age.
ing both cerebral hemispheres and resulting in
Answer C is incorrect. Lisch nodules are con-
complete loss of consciousness. These types of
sistent with NF type I, not NF type II. NF type
seizures begin suddenly with tonic extension of
II is associated with acoustic neuromas as well
the back and extremities and typically last for
as multiple intracranial meningiomas. The
several minutes. Patients often experience in-
gene for NF type II is located on chromosome
continence and/or tongue biting during the
22.
seizure. The post-ictal period is marked by
confusion as the patient slowly returns to con- Answer D is incorrect. Neither Lisch nod-
sciousness. In this patient the most likely un- ules nor caf-au-lait spots are found in tuber-
derlying cause is low serum antiepileptic levels ous sclerosis. Tuberous sclerosis is associated
secondary to medication noncompliance. with hypopigmented macules (ash leaf spots),
HIGH-YIELD SYSTEMS
Chapter 10: Neurology Answers 305

shagreen spots (leathery cutaneous thickening mon form is benign paroxysmal positional ver-
of the skin), facial hamartomas, seizures, and tigo (BPPV). The rst step in diagnosis, before
mental retardation. Tuberous sclerosis is inher- more complex tests are undertaken, is to perform
ited in an autosomal dominant fashion, and it a thorough neurologic examination, including
presents in childhood as mental retardation, the Dix-Hallpike maneuver. With the patient sit-
epilepsy, and facial hamartoma. ting, the neck is extended and turned to one
side. The patient is then placed supine rapidly,
Answer E is incorrect. Neither Lisch nodules
so that the head hangs over the edge of the bed,
nor caf-au-lait spots are found in von Hippel-
and is observed for nystagmus. The patient is
Lindau syndrome. The syndrome presents with
then returned to upright, observed for another
several hemangiomas in many organs. It is asso-
30 seconds for nystagmus, and the maneuver is
ciated with a renal cell cancer and polycythemia
repeated with the head turned to the other side.
(due to increased erythropoietin).
If the patient has BPPV, the Dix-Hallpike ma-
neuver will provoke upward beating torsional ny-
15. The correct answer is B. Retts syndrome is a
stagmus, with the upper poles of the eye beating
genetic neurodegenerative disease found al-

Neurology
towards the oor, usually lasting 30 seconds or
most exclusively in females. Patients have nor-
less.
mal physical, mental, and social development
until about the age of 5 months and then begin Answer A is incorrect. Brainstem evoked audi-
to regress in development. Language and coor- ometry is often used to diagnose sensorineural
dination are the most common functions that hearing loss and entails measuring responses
are adversely affected. in brain waves that are stimulated by a clicking
sound to check the central auditory pathways
Answer A is incorrect. Retts syndrome has not
been shown to have an increased prevalence in of the brainstem. The test is about 90% sensi-
Ashkenazi Jews. tive for diagnosing acoustic neuromas.

Answer C is incorrect. This disorder occurs Answer C is incorrect. With typical BPPV,
only very rarely in males. The MECP2 gene further testing is not necessary. Electronystag-
responsible for Retts syndrome is located on mography is indicated only if there is preexist-
the X chromosome. The mutation is often ing vestibular disease.
sporadic but has been found to more often oc- Answer D is incorrect. There is no indication
cur in the paternally derived X chromosome, for an LP for diagnosing a patient with classic
which combined with the often lethal pheno- symptoms of BPPV.
type seen in males might explain the extreme
Answer E is incorrect. Neuroimaging is neces-
female predominance. However, the exact
sary when the nystagmus is incongruent with
mechanism for the gender predominance re-
the classic posterior canal presentation of BPPV.
mains unknown.
Answer D is incorrect. Fragile X is a syndrome 17. The correct answer is D. This patient suffers
in males and is associated with mental retarda- from myoclonic seizures as a result of juvenile
tion. myoclonic epilepsy. Myoclonic seizures in-
volve both cerebral hemispheres but rarely
Answer E is incorrect. Trisomy 21 or Downs
cause a loss of consciousness or postictal con-
syndrome is characterized by mental retarda-
fusion. Juvenile myoclonic epilepsy typically
tion, but the patients do not have a normal
occurs in adolescents with a history of general-
phase of development as seen in Retts patients.
ized seizures and responds well to anticonvul-
Answer F is incorrect. Retts syndrome has not sant medications.
been associated with any ethnic or racial group.
Answer A is incorrect. Tonic seizures consist
16. The correct answer is B. The patients symp- of sudden onset tonic extension or exion of
toms and clinical history indicate the presence the head, trunk, and/or extremities for several
of positional vertigo, of which the most com- seconds. They typically occur in relation to
drowsiness and are often associated with other
HIGH-YIELD SYSTEMS
306 Section I: Organ Systems Answers

neurologic abnormalities. EEG shows an dural hematomas present with a lucid interval
electrodecremental response, which is a high- prior to a precipitous decline in level of con-
frequency electrographic discharge in the sciousness, although this only occurs in 10%
frequency (beta buzz) with relatively low am- 30% of cases. Other symptoms include focal
plitude compared to the background rhythm. neurologic signs, seizures, nausea and vomit-
The pattern can evolve into slow spike-and- ing, and headache.
wave complexes or diffuse polyspikes.
Answer D is incorrect. The lack of a family
Answer B is incorrect. Absence seizures have history, the lack of an aura, and the age of on-
a characteristic 3-Hz spike-and-wave pattern on set point away from migraines.
EEG and are brief, 2- to 3-s seizures that may
Answer E is incorrect. Some history of trau-
occur many times per day in affected children.
matic head injury would be expected in the
However, absence seizures typically present in
development of a subdural hemorrhage. Any
children 48 years old and are not associated
focal neurologic signs following blunt head
with a loss of postural muscle tone. This pa-
trauma or a GCS score <15 after head trauma
tient is not suffering from absence seizures.
warrants investigation for subdural or epidural
Neurology

Answer C is incorrect. The denition of a hematoma.


generalized seizure is one that involves both
Answer F is incorrect. Tension headache is a
cerebral hemispheres. Involvement of only one
diagnosis of exclusion; here, the focal neuro-
hemisphere would by denition not be a gen-
logic signs clearly point away from this diagno-
eralized seizure.
sis.
Answer E is incorrect. Focal seizures involve
seizure activity localized to a discrete portion 19. The correct answer is B. The patient suffers
of one hemisphere. Whether simple (aware- from conduction aphasia, which is character-
ness unaffected) or complex (behavior, mem- ized by problems with repeating what is said
ory, or awareness modied), focal seizures do and naming, but preserved uency and com-
not result in loss of consciousness. Although it prehension. Anatomically the lesion occurs at
is possible for a focal seizure to generalize into the arcuate fasciculus between Brocas and
a grand mal seizure, there is no evidence that Wernickes area.
this occurred in this patient. Answer A is incorrect. Brocas aphasia is char-
acterized by preserved comprehension, but
18. The correct answer is A. Brain tumors often
problems with language production. These pa-
present with a headache that is worse in the
tients speak nonuently, haltingly, and without
morning and gets better with vomiting. This is
much intonation. Naming and repetition are
because lying down all evening while asleep
also impaired. They are aware of their prob-
causes increased intracranial pressure (ICP),
lem and may often become depressed. Inter-
which when further exacerbated by a mass le-
estingly, some patients can maintain uency
sion causes a headache in the morning that is
when singing. Brocas area includes the poste-
relieved with vomiting. In addition, other pre-
rior part of the inferior frontal gyrus and a sur-
senting features can include seizures, focal
rounding rim of prefrontal heteromodal cortex.
neurologic decits, and signs and symptoms of
increased ICP. Answer C is incorrect. This is caused by a
large lesion in the dominant hemisphere, lead-
Answer B is incorrect. The patients headache
ing to problems with language production,
does not show ipsilateral conjunctival injection
comprehension, and repetition. It can be con-
and tearing, as would be expected in cluster
ceptualized as a combination of Brocas and
headaches.
Wernickes aphasia.
Answer C is incorrect. Some history of trau-
Answer D is incorrect. This type of aphasia is
matic head injury would be expected in the
characterized by impaired comprehension of
development of an epidural hemorrhage. Epi-
language, but preserved repetition and uency.
HIGH-YIELD SYSTEMS
Chapter 10: Neurology Answers 307

Answer E is incorrect. Wernickes aphasia rep- Answer E is incorrect. Myasthenia gravis is


resents a problem with language comprehen- an autoimmune disease caused by antibod-
sion, which affects both language output and ies against the acetylcholine receptors of the
input. Language output is paraphasic and cir- postsynaptic cleft. It is most common in young
cumlocutious. Repetition, naming, reading, women, although elderly men can be affected.
and writing are also impaired although certain It typically presents with double vision that is
axial tasks such as opening ones eyes may be worse as the day progresses and/or difculty
preserved. Wernickes area includes the pos- swallowing and proximal muscle weakness.
terior third of the superior temporal gyrus Unilateral muscle wasting is not characteristic
and a surrounding rim of inferior parietal and of myasthenia gravis.
midtemporal cortex.
21. The correct answer is D. Vasospasm typically
20. The correct answer is C. The mechanic has occurs 314 days after rupture of an aneurysm
signs and symptoms of carpal tunnel syndrome, and has a mortality rate approaching 12%.
which is entrapment of the median nerve char- Roughly 70%90% of all patients will experi-
acterized by pain and paresthesias in the me- ence vasospasm after an aneurysmal rupture,

Neurology
dial portion of the palm that may radiate proxi- and about one-half will result in an ischemic
mally down the forearm. Carpal tunnel stroke. Given such high odds, this is the most
syndrome often presents as a work-related ill- likely etiology of a stroke in this patient.
ness in persons whose jobs require prolonged
Answer A is incorrect. Atherosclerosis can in-
use of the hand and wrist. If left uncorrected,
duce blood clots and lead to ischemic events.
carpal tunnel syndrome can lead to severe neu-
The most common locations in the brain for
ropathy and muscle wasting distal to the point
thrombotic events include the intracerebral
of median nerve entrapment.
arches, the branches of the circle of Willis, and
Answer A is incorrect. Amyotrophic lateral the posterior circulation. Again, the history of
sclerosis is an idiopathic disease that results in recent aneurysmal rupture leads one to suspect
chronic, progressive degeneration of neurons vasospasm over other causes.
in the spinal cord, brain stem, and motor cor-
Answer B is incorrect. Twenty percent to 30%
tex. In this disease, both upper and lower mo-
of all ischemic strokes occur due to a cardio-
tor neuron signs and involvement of more than
genic thromboembolism and may be caused
one extremity are expected to be seen.
by a number of pathogenic factors. Common
Answer B is incorrect. Angina is typically de- causes include stasis due to atrial brillation or
scribed as acute, localized squeezing/burning myocardial infarction and endothelial abnor-
chest pain that may radiate to the arm, neck, malities due to valvular diseases. In this case,
jaw, or abdomen. Angina is typically transient however, the recent history of aneurysmal rup-
and is worse on exertion; it is improved by rest ture leads one to suspect vasospasm.
or vasodilators such as nitroglycerine. This pa-
Answer C is incorrect. Although hydrocepha-
tients symptoms seem more neuropathic and
lus is a common complication of subarachnoid
are conned to an extremity with clear signs of
hemorrhage (SAH), the time course makes this
neuropathy, making angina a much less likely
unlikely to be the cause of a new infarct. After
cause.
SAH, roughly 20% of patients can expect to de-
Answer D is incorrect. Currently believed to velop hydrocephalus.
have an autoimmune etiology, multiple scle-
rosis (MS) is a chronic progressive neurologic 22. The correct answer is D. Middle cerebral ar-
disease that results in demyelinating lesions tery strokes typically present with aphasia
of various portions of the CNS. MS is twice (dominant hemisphere), neglect (nondomi-
as common in females. To be diagnosed with nant hemisphere), contralateral hemiparesis,
MS, a patient must present with multiple neu- gaze preference, or homonymous hemianop-
rologic complaints distinct in time and space. sia. The CT scan demonstrates ischemic stroke
To the physicians knowledge, this patient only
has one chronic neurologic complaint.
HIGH-YIELD SYSTEMS
308 Section I: Organ Systems Answers

of the middle cerebral artery territories with Answer C is incorrect. Normal aging is associ-
ensuing edema. ated with a decreased ability to learn new ma-
terial and slowing of cognitive processes.
Answer A is incorrect. Anterior cerebral artery
strokes present with leg paresis, amnesia, per- Answer E is incorrect. Vascular dementia is
sonality changes, foot drop, gait dysfunction, or the second most common cause of dementia.
cognitive changes. Patients with vascular dementia usually have a
medical history of hypertension and/or stroke.
Answer B is incorrect. Basilar strokes present
Also, vascular dementia usually progresses in a
with coma, cranial nerve palsies, apnea, visual
stepwise fashion with each recurrent infarct.
symptoms, drop attacks, or dysphagia.
Answer C is incorrect. Lacunar strokes are 24. The correct answer is E. Myasthenia gravis is
caused by the occlusion of single deep pen- an autoimmune disease caused by antibodies
etrating arteries resulting in small subcortical against postsynaptic acetylcholine receptors at
infarcts <15 mm that appear as small lesions the neuromuscular junction. Thirty percent to
(the word lacune derives from their lake-like 50% of patients with thymoma have myasthe-
Neurology

radiologic appearance) on CT or MRI. They nia gravis. If a thymoma is present, surgical re-
account for some 15%25% of strokes in the section can often be curative.
United States and typically present with pure
Answer A is incorrect. Intravenous immuno-
motor or sensory decits, dysarthria, or ataxic
globulin (IVIG) is a pooled product of human
hemiparesis.
gamma-globulin. In severe cases, IVIG can be
Answer E is incorrect. Posterior cerebral artery used to provide temporary relief, such as dur-
strokes present with homonymous hemianop- ing a myasthenia crisis when patients are at risk
sia, memory decits, or dyslexia/alexia. of respiratory failure. The exact mechanism of
how IVIG works in myasthenia gravis is not
23. The correct answer is D. Pseudodementia is a understood. Although this therapy may provide
condition by which depression in persons who some symptomatic relief, it is not curative. In-
are elderly may present as symptoms of cogni- fusions are required every few weeks, take mul-
tive impairment. Pseudodementia can be dif- tiple hours to administer, and are quite expen-
ferentiated from other causes of dementia by sive.
the time of onset, its tendency to occur for days
Answer B is incorrect. Neostigmine is an ace-
to weeks, cognitive changes, personality
tylcholinesterase inhibitor used to increase the
changes, past medical history positive for mood
concentration of acetylcholine at the neuro-
disorder, and patient awareness and distress of
muscular junction in patients with myasthenia.
the cognitive changes.
Although this therapy may provide some symp-
Answer A is incorrect. Alzheimers dementia tomatic relief, it is not curative. The medicine
typically has a vague onset. The patient is often must be administered for life to prevent recur-
unaware or unconcerned with their cognitive rence of symptoms.
decits. Additionally, patients tend to exhibit
Answer C is incorrect. Plasmapheresis in-
prominent cognitive impairment, and CT re-
volves exchanging the patients plasma for
sults may be abnormal.
fresh frozen plasma. It is often used in prepara-
Answer B is incorrect. Delirium is mainly tion for surgery, as short-term management of
differentiated from dementia based on the pa- an exacerbation, or to provide temporary relief
tients level of consciousness and orientation. during a crisis when the patient is at risk of re-
Patients with delirium who reveal a uctuat- spiratory failure. Weakness improves within
ing level of consciousness and orientation may days, but the improvement is not curative and
be impaired in comparison to patients with it lasts only 68 weeks.
dementia. Other factors include rapid onset,
Answer D is incorrect. Prednisone is used to
duration, psychomotor retardation, and revers-
downregulate the immune system in patients
ibility.
with myasthenia. Although this therapy pro-
HIGH-YIELD SYSTEMS
Chapter 10: Neurology Answers 309

vides temporary relief, it is not curative, and symptoms, particularly amnesia, and may be
the disease will continue without further ste- warded off by proper treatment of Wernickes
roid treatment. syndrome.
Answer A is incorrect. Folate deciency does
25. The correct answer is C. This patient is suffer-
not produce neurologic decits, with the ex-
ing from cluster headaches. Treatment is pri-
ception of some memory loss and personality
marily focused on avoiding factors that exacer-
changes. Most commonly, folate deciency
bate these headaches, particularly alcohol and/
presents with gastrointestinal symptoms, such
or strenuous physical exercise. This should be
as a swollen and red tongue, nausea, emesis,
the rst step in management before pharmaco-
diarrhea, and abdominal pain after meals.
therapy is initiated. If therapy is indicated, pa-
tients will typically require both prophylactic Answer B is incorrect. Glucose should not be
and abortive therapy for relief of their symp- given in a patient with suspected thiamine de-
toms. Abortive therapy includes oxygen admin- ciency because high-dose glucose administra-
istration and the use of triptans. Prophylactic tion can precipitate symptoms of thiamine de-
therapy includes the use of prednisone, lith- ciency. This is because even more substrate

Neurology
ium, and calcium channel blockers. is given for biochemical pathways that do not
have proper substrates.
Answer A is incorrect. -Blockers are good
prophylactic medications for migraine head- Answer D is incorrect. Vitamin B6 deciency
aches. Their role in cluster headaches is less would not present in this manner. Such de-
well dened. ciency usually presents with bilateral distal
extremity numbness, weakness, sideroblastic
Answer B is incorrect. Anticonvulsants such
anemia, and skin changes, such as erythema-
as valproate are good prophylactic medications
tous itching and burning, blisters, and hyper-
against migraine headaches, less so for cluster
pigmentation with thickening.
headaches.
Answer E is incorrect. The patient does not
Answer D is incorrect. Nonsteroidal anti-
have sensory and motor decits consistent with
inammatory drugs are helpful in stopping a
vitamin B12 deciency. The classic triad of vi-
headache once it has begun. However, it has
tamin B12 deciency includes weakness, sore
a greater abortive role in migraine and tension-
tongue, and paresthesias. However, the chief
type headaches.
symptoms are usually weight loss and anorexia,
Answer E is incorrect. Tricyclic antidepres- malaise, and neurologic symptoms such as par-
sants are good prophylactic medications against esthesias and gait abnormalities.
migraine headaches and have been used in
tension headaches, but their role in cluster 27. The correct answer is E. This patient has symp-
headaches is less well dened. toms consistent with elevated ICP. His vital
signs show a Cushing reex (hypertension and
26. The correct answer is C. The patients presen- bradycardia) in response to rising ICP. In any
tation corresponds to the classic presentation patient presenting with stroke-like symptoms,
of thiamine deciency, with the patient per- the rst imaging study to obtain is non-contrast-
haps suffering from permanent Korsakoffs syn- enhanced CT of the head to evaluate for hem-
drome as evidenced by amnesia. However, thi- orrhage. The treatment of hemorrhage involves
amine should be administered in the hope that neurosurgical consultation for possible decom-
the patient simply has reversible Wernickes pression, if necessary. Additionally, the physician
encephalopathy. Both disorders are caused by should elevate the head of the bed. This hemor-
the same pathophysiology of thiamine de- rhage may lead to fatal herniation of the brain.
ciency. Wernickes encephalopathy refers to an Mannitol, hyperventilation, and occasionally
acute presentation of the symptoms described hypertonic saline are also used to decrease ICP.
in the question. Korsakoffs or Wernicke-Korsa- Antiseizure medication may be given if there
koff syndrome refers to the persistence of these was a witnessed seizure or cortical bleed.
HIGH-YIELD SYSTEMS
310 Section I: Organ Systems Answers

Answer A is incorrect. Antiemetics would pro- Answer D is incorrect. Phenytoin blocks use-
vide symptomatic relief but are certainly not dependent sodium channels and is used to
the rst priority. treat grand mal seizures. It has many adverse
effects including gingival hyperplasia, anemia,
Answer B is incorrect. Thrombolytic agents
teratogenicity, hirsutism, and nystagmus.
are indicated in the treatment of ischemic
stroke if the patient is seen within 3 hours
29. The correct answer is C. This child has Di-
since the rst symptoms. To be a candidate for
Georges syndrome, a disease caused by a dele-
thrombolytic therapy, the patient must have an
tion of chromosome 22q11. This chromosomal
International Normalized Ratio <1.7, a platelet
anomaly causes abnormal development of the
count >100,000/mm, and no surgery with the
third and fourth pharyngeal pouches, resulting
past 14 days. In the setting of cerebral hemor-
in midline defects. Abnormal facies, cleft pal-
rhage, thrombolytics are contraindicated as
ate, congenital heart defects, thymic aplasia,
they may worsen the pathological process.
and parathyroid hypoplasia characterize Di-
Answer C is incorrect. The patients blood Georges syndrome. Patients often present in
pressure and mental status should be watched the neonatal period with tetany or seizures sec-
Neurology

closely as he is treated for his hemorrhage, but ondary to hypocalcemia. Hypocalcemia can be
his blood pressure is within a relatively safe detected on physical examination by positive
range presently and is not the top priority. Trousseaus and Chvosteks signs. These chil-
dren are also susceptible to infection due to
Answer D is incorrect. Given his history of
thymic aplasia and a decreased number of T
diabetes, this patient should have tight glu-
lymphocytes.
cose control and likely needs adjustments in
his regimen while he is an inpatient. However, Answer A is incorrect. Beckwith-Wiedemann
the most appropriate priority is a neurosurgical syndrome is characterized by perinatal growth
consultation. acceleration, hemihypertrophy, macroglossia,
linear ear creases, abdominal wall defects, ex-
28. The correct answer is E. Topiramate enhances ophthalmos, and transient neonatal hypogly-
GABA-activated chloride channels and inhibits cemia. Patients with Beckwith-Wiedemann
excitatory neurotransmission, through actions syndrome are at increased risk of developing
on kainate and AMPA receptors. The drug has Wilms tumor.
been linked to short-term memory loss and
Answer B is incorrect. Chronic granuloma-
word-nding difculties. Additionally, the drug
tous disease is characterized by recurrent mu-
may rarely cause renal stones (incidence
cous membrane infections, abscesses, and poor
1.5%).
wound healing. This disease is a result of a de-
Answer A is incorrect. Benzodiazepines are fect in neutrophil-mediated phagocytosis due
used to treat seizures and anxiety. They may to a lack of NADPH activity. Chronic granu-
cause sedation and mental slowness. However, lomatous disease is usually diagnosed between
they do not cause kidney stones. 38 years of age after multiple infections. It is
not associated with hypocalcemia, abnormal
Answer B is incorrect. Carbamazepine stabi-
facies, or cardiac defects.
lizes the inactivated state of sodium channels.
It is used to treat simple, partial, and tonic- Answer D is incorrect. MEN type 1 causes
clonic seizures. The drug is also effective for a predisposition to developing tumors of the
treating trigeminal neuralgia. Its adverse effects parathyroid, pituitary, and pancreas. Parathy-
include diplopia, ataxia, induction of cyto- roid tumors cause hyperparathyroidism and
chrome P450, liver toxicity, and blood dyscra- consequently hypercalcemia. They are not as-
sias. sociated with hypocalcemia, abnormal facies,
cardiac defects, or predisposition to infections.
Answer C is incorrect. Lamotrigine is an anti-
seizure medication that is associated with Ste- Answer E is incorrect. Niemann-Pick disease
vens-Johnson syndrome. results from deciency of sphingomyelinase,
HIGH-YIELD SYSTEMS
Chapter 10: Neurology Answers 311

leading to build up of sphingomyelin and cho- Answer A is incorrect. Carbamazepine has


lesterol. The infantile form presents with hepa- been reported to exacerbate absence seizures
tosplenomegaly, failure to thrive, and rapidly and is contraindicated in such patients.
progressive neurodevelopmental regression. It
Answer C is incorrect. Gabapentin has been
is not associated with cardiac malformations
reported to exacerbate absence seizures and is
or immunodeciency and does not cause hy-
contraindicated in such patients.
pocalcemia.
Answer D is incorrect. Tiagabine has been
30. The correct answer is C. Astrocytomas are the reported to exacerbate absence seizures and is
most common tumor type, and of these, contraindicated in such patients.
glioblastoma multiforme is the most common
Answer E is incorrect. Valproate can be use-
primary brain tumor. Imaging can reveal a tu-
ful for absence patients but is normally used
mor mass that often crosses the corpus callosum
as second-line therapy in those who have had
into the contralateral cerebral hemisphere, or a
inadequate relief of symptoms from ethosux-
multicentric appearance with satellite lesions.
imide. Other useful second-line agents include
Glioblastoma multiforme is a high-grade lesion,

Neurology
lamotrigine, topiramate, and zonisamide.
with extensive necrosis and hemorrhage within
the tumor. Such lesions have a poor prognosis,
32. The correct answer is C. A 4-year-old child is
with mean survival <1 year. Histopathology
able to hop and skip, draw crosses, count to 10,
shows poorly differentiated, pleomorphic astro-
and sing songs or recite poems from memory.
cytic cells with nuclear atypia. Necrosis along
with microvascular proliferation are often seen. Answer A is incorrect. A 2-year-old child can
walk up and down stairs, build a six-cube
Answer A is incorrect. Schwannomas arise
tower, and use two-word sentences, but he or
from the superior division of cranial nerve VIII,
she will not have achieved the milestones listed
are pathologically characterized by an Antoni
for 3-, 4-, and 5-year-old children.
A or B pattern, and present with such symp-
toms as hearing loss and dizziness. Answer B is incorrect. A 3-year-old child can
ride a tricycle, draw a circle, and use three-
Answer B is incorrect. Cavernous malforma-
word sentences, but he or she will not have
tions are one of a group of vascular malforma-
achieved the milestones listed for 4- and 5-year-
tions, and would not be characterized by neo-
old children.
plastic tissue.
Answer D is incorrect. A 5-year-old child will
Answer D is incorrect. Medulloblastomas are
be able to do these tasks, as well as jump over
tumors of childhood, and would not be seen
some obstacles, tie shoelaces, copy a square,
in this age group. Histopathology characteristi-
and print his or her name.
cally shows small round blue cells.
Answer E is incorrect. A 6-year-old child can
Answer E is incorrect. Meningiomas are the
ride a bicycle, knows his or her left hand from
most common low-grade intracranial tumors,
his or her right, can carry on a telephone con-
most often appearing in middle-aged women
versation, enjoys constructive and creative play,
and often presenting on the cerebral convexi-
and can read and write some words.
ties. Histopathology characteristically shows
plump pink cells with small amounts of hemo- 33. The correct answer is C. Carbon monoxide
siderin. Psammoma bodies may be seen. (CO) poisoning should be considered in any
case of headache with altered mental status.
31. The correct answer is B. Ethosuximide is the
The fathers headaches may be a coincidence,
rst-line agent and is effectively only against
but should increase the index of suspicion for
absence seizures. Seizures will eventually re-
CO exposure. Children are at especially high
solve spontaneously, but ethosuximide is po-
risk of CO poisoning due to their increased re-
tentially useful in minimizing symptoms and
spiratory rate and minute volume. Prolonged
associated learning decits and behavioral
exposure to even low levels of CO may pro-
problems during childhood.
HIGH-YIELD SYSTEMS
312 Section I: Organ Systems Answers

duce moderate to severe poisoning. Mild expo- orrhagic and ischemic stroke. An infused CT is
sure may result in headaches, somnolence, diz- contraindicated as an initial study because of
ziness, and nausea. Symptoms of moderate to the risk of further damage to the brain paren-
severe toxicity may include chest pain, dysp- chyma from extravasated contrast dye if the
nea, headache, ataxia, confusion, seizures, stroke is hemorrhagic.
coma, and eventually death. In severe poison-
Answer A is incorrect. If this patients stroke is
ing, tissue hypoxia may cause an increased an-
in fact related to his carotid disease, then a du-
ion gap metabolic acidosis secondary to in-
plex ultrasound may indeed help identify the
creased lactic acid production. His symptoms
underlying cause and direct subsequent ther-
(loss of consciousness and difculty walking)
apy (e.g., emergent carotid endarterectomy).
are concerning for more severe poisoning, and
While this would be an appropriate test follow-
an acidosis may be present.
ing a CT scan of the head showing a stroke in
Answer A is incorrect. Blood sugar is not af- the anterior circulation, it would be inappro-
fected by bloodstream CO levels. Hypoglyce- priate as an initial diagnostic study.
mia might cause headaches and confusion, but
Answer B is incorrect. Cerebral angiography is
Neurology

symptoms would not persist after a meal, as oc-


traditionally regarded as the gold standard for
curred in this case.
identifying vascular pathology (e.g., aneurysms
Answer B is incorrect. CO does not affect the or occlusion). However, because it is invasive
concentration of dissolved oxygen in serum. and carries the risk of substantial complica-
Since partial arterial oxygen pressure measures tions, it is rarely used without prior imaging
dissolved oxygen only, it is normal in cases of studies such as CT, MRI, and/or ultrasound.
CO poisoning.
Answer C is incorrect. Conventional radiogra-
Answer D is incorrect. Abnormalities may be phy has no role in the evaluation of patients with
seen on x-ray of the chest in cases of CO tox- acute neurologic injury unless cervical spine in-
icity and usually include pulmonary edema or jury is suspected, in which case an odontoid view
inltrates secondary to aspiration pneumonitis. and a lateral cervical spine lm should be ob-
However, in most cases of CO poisoning, x-ray tained. In this instance, it will offer no informa-
of the chest is normal. tion as to whether or not the patient has suffered
a stroke.
Answer E is incorrect. Conventional pulse
oximeters, which use two wavelengths of light, Answer E is incorrect. A properly performed
can only differentiate oxyhemoglobin from MRI of the brain has anatomic resolution that is
deoxyhemoglobin. Carboxyhemoglobin and superior to CT of the head in many ways. MRI
methemoglobin are misinterpreted as oxygen- is more sensitive in detecting early ischemic
ated hemoglobin; therefore results on pulse changes, imaging the brain stem/posterior fossa,
oximetry will most likely appear normal even and determining the date of injury. However,
in the presence of severe hypoxia. Methemo- because the image acquisition times are consid-
globinemia may be due to inherited defects erably longer for MRI, obtaining high-quality
in oxidative stress reduction pathways (such as images is dependent on the patients ability to
glucose-6-phosphate dehydrogenase deciency sit still. Due to this current limitation, MRI is
or pyruvate kinase deciency), certain medica- typically reserved for the subacute setting and
tions, or environmental exposures (particularly is often used to follow-up CT ndings in stroke
nitrites or nitrates). victims.

34. The correct answer is D. CT without contrast 35. The correct answer is B. This patient has a
is the imaging study of choice in this patient, normal blood pressure and heart rate, moist
chiey because of its speed, safety, and efcacy mucous membranes, and gives no history of re-
in distinguishing between the most common cent uid losses. She demonstrates euvolemic
causes of neurologic injury. When stroke is sus- hyponatremia without symptoms of acute wa-
pected, the scan is performed without contrast ter intoxication such as confusion, anorexia,
in order to aid in distinguishing between hem- nausea, vomiting, or coma. Common causes of
HIGH-YIELD SYSTEMS
Chapter 10: Neurology Answers 313

euvolemic hyponatremia include the syndrome decreased oncotic pressures and subsequent
of inappropriate secretion of ADH (SIADH), a loss across the capillary membranes.
reset osmostat, and isotonic salt losses with free
Answer E is incorrect. Other intracranial pro-
water repletion. In this case, the patient has ex-
cesses (e.g., SAH) are associated with a process
perienced recent onset of headaches and now a
known as cerebral salt wasting, thought to be
seizure, raising the possibility of a slowly pro-
due to increased levels of brain natriuretic pep-
gressing intracranial process. A hyponatremic
tide and concurrent aldosterone suppression.
seizure at a sodium concentration of 124
Despite hyponatremia and high plasma antidi-
mEq/L is unlikely given the clinical scenario of
uretic hormone, cerebral salt wasting can be
a chronic disorder. The history of recent-onset
differentiated from SIADH by signs and symp-
headaches and seizure should raise concern for
toms of hypovolemia.
intracranial malignancy, which is often associ-
ated with inappropriate ADH secretion from 36. The correct answer is E. This patient has open-
the pituitary gland. This is conrmed by ele- angle glaucoma (OAG). This condition is diag-
vated urine sodium levels in the setting of clini- nosed by an increased intraocular pressure, ab-

Neurology
cally normal volume status. Nonsteroidal anti- normal optic disk ndings, and typical visual
inammatory drugs potentiate the action of eld loss. The eye is not red or painful, the pu-
ADH and are also associated with the develop- pil and cornea appear normal, and it commonly
ment of SIADH. affects both eyes. OAG can be treated with a va-
Answer A is incorrect. Bartters syndrome is a riety of medications. Pilocarpine is a muscarinic
defect in sodium transport at the loop of Henle agonist that produces rapid miosis and contrac-
leading to hypokalemia, metabolic alkalosis, tion of the ciliary muscles. This can be used in
and hypercalciuria. It presents itself early in the acute treatment of OAG or closed-angle
life and is associated with growth defects, low glaucoma to open the trabecular meshwork
blood pressure, and mental retardation. It is around Schlemms canal, increasing drainage of
not associated with hyponatremia. aqueous humor and decreasing intraocular pres-
sure. Other agents that increase aqueous humor
Answer C is incorrect. Hyperglycemia in the
secretion include prostaglandins (latanoprost)
setting of diabetes (diabetic ketoacidosis or hy-
and epinephrine. OAG can also be treated with
perosmolar hyponatremic nonketosis) can lead
agents that decrease secretion, including
to hyperosmotic hyponatremia. High plasma
-blockers (timolol), carbonic anhydrase inhibi-
glucose levels pull water out of cells, result-
tors, and 2-agonists (brimonidine).
ing in a dilutional hyponatremia. At the same
time the increased osmotic pressure leads to an Answer A is incorrect. Corticosteroids are the
osmotic diuresis. The resultant hypovolemic, treatment of choice for a number of ophthal-
hyperosmolar hyponatremia must be corrected mic conditions including uveitis, retrobulbar
by volume replacement and insulin. Adjusted neuritis, optic neuritis, and allergic conjunc-
plasma sodium levels (2 mEq/L for every 100 tivitis. However, steroids are not indicated for
mg/dL of glucose >200 mg/dL) may actually the treatment of glaucoma.
show hypernatremia in these cases. Answer B is incorrect. Unlike OAG, narrow-
Answer D is incorrect. Cirrhosis-induced angle glaucoma is caused by the blockage of
hypoalbuminemia combined with periph- the aqueous outow passageway. Dilation of
eral vasodilation leads to free water retention the pupils may precipitate an attack since the
and hyponatremia. In early stages of cirrho- iris bunches up and narrows the angle. This
sis, transaminases and bilirubin are usually condition presents with intense pain and con-
elevated, but in late-stage liver disease, as the junctival injection. The patient may see halos
number of normal hepatic cells dwindles, around lights, may have decreased vision, and
transaminases may drop to near-normal levels. may present with nausea and vomiting. Treat-
These patients are hypervolemic, demonstrat- ment includes pilocarpine to constrict the
ing edema and ascites, but are intravascularly pupil, topical -blockers, carbonic anhydrase
volume depleted, with water losses caused by inhibitors, and 2-agonists. Patients should
HIGH-YIELD SYSTEMS
314 Section I: Organ Systems Answers

also be treated with intravenous mannitol, an concentrated in response to acute volume loss,
osmotic agent that draws water out of the eye and clinically he was severely hypovolemic.
and reduces pressure. Intravenous mannitol is However, it is now 2 days later and his initial
not used for the treatment of OAG. volume losses have been replaced by crystal-
line solutions and blood products.
Answer C is incorrect. Isoproterenol is a
-agonist. It has no use in the ophthalmic treat- Answer D is incorrect. This description is con-
ment of glaucoma. Glaucoma is treated with sistent with diabetes insipidus, not SIADH. In
topical -blockers. diabetes insipidus there is a loss of either cen-
tral production of ADH (or arginine vasopres-
Answer D is incorrect. Phenoxybenzamine is
sin) or renal sensitivity to ADH. Consequently,
a nonselective -blocker. It has no use in the
urine cannot be concentrated and is inappro-
ophthalmic treatment of glaucoma. Glaucoma
priately dilute. Plasma may be nearly isotonic
is treated with 2-agonists.
to severely hypertonic, depending on the pa-
37. The correct answer is A. This description is tients ability to drink large amounts of water.
consistent with a patient who is hyponatremic Similarly, patients may appear euvolemic to
Neurology

due to SIADH. ADH acts on the collecting tu- very dehydrated. While diabetes insipidus can
bules to increase the retention of free water. result from head trauma, this would not ac-
SIADH, therefore, is not caused by a lack of so- count for the patients low serum sodium level.
dium, but by excess free water, which explains Instead, he should be hypernatremic.
why this patient is hyponatremic in spite of the Answer E is incorrect. This description could
administration of isotonic sodium-containing suggest a patient with normal endocrine
uids. Plasma osmolality is decreased (<280 and renal function, leading one to suspect a
mOsm/kg) in SIADH due to free water reten- pseudohyponatremia due to hyperglycemia or
tion and urine is inappropriately concentrated glycerol/mannitol administration for intracra-
(>100 mOsm/kg water). Patients appear eu- nial hypertension. This patient is not described
volemic and signs such as ascites, peripheral as receiving either of these solutions. In ad-
edema, and heart failure are absent. This is be- dition, his change in mental status is best ex-
cause most of the free water concentrates intrac- plained by true hyponatremia due to SIADH,
ellularly, where it impairs cell function. In the not by pseudohyponatremia.
brain, this results in seizures, cerebral edema,
and brain stem herniation. Water restriction is 38. The correct answer is C. Emergency treat-
the major modality of therapy for SIADH, but ments in life-threatening situations do not re-
should be used very cautiously in patients with quire consent; consent is implied. Therefore
SIADH and SAH since they rely heavily on the most appropriate next step is to perform
maintaining blood pressure for continued cere- emergency evacuation of the hematoma before
bral perfusion. further neurologic decits develop. Should all
relevant parties be present, consent authority
Answer B is incorrect. This description is not
would serially fall upon (1) the patients
consistent with SIADH. Patients with SIADH
spouse; (2) an adult child of the patient who
have decreased plasma osmolality due to free
has the waiver and consent of all other quali-
water retention and inappropriately concen-
ed adult children of the patient to act as the
trated urine. In spite of the free water reten-
sole decision maker; (3) a majority of the pa-
tion, most of the free water concentrates intra-
tients reasonably available adult children; (4)
cellularly, and patients therefore do not appear
the patients parents; or (5) the individual
hypervolemic or display signs such as periph-
clearly identied to act for the patient before
eral edema, ascites, and heart failure.
the patient became incapacitated, the patients
Answer C is incorrect. This description may nearest living relative, or a member of the
have applied to the patient at the time of his clergy.
initial presentation: his plasma would have
been normal (isotonic), his urine would have
HIGH-YIELD SYSTEMS
Chapter 10: Neurology Answers 315

Answer A is incorrect. In the case of a life- 40. The correct answer is A. The stem describes
threatening condition, consent to treatment is the cherry-red retinal lesions characteristi-
implied. cally seen in infantile Tay-Sachs disease. Ab-
sence of hexosaminidase A in WBCs is diag-
Answer B is incorrect. While medical man-
nostic for Tay-Sachs disease. Tay-Sachs has an
agement to decrease ICP may be appropriate
autosomal recessive inheritance. It is most
in other types of intracranial hemorrhage, a
prevalent among Ashkenazi Jews (1:2530 are
progressing epidural hematoma must be surgi-
carriers). Infantile Tay-Sachs presents at 45
cally evacuated to relieve the pressure.
months of age with loss of motor skills, in-
Answer D is incorrect. Serial neurologic ex- creased startle reaction to noise, and retinal
aminations are appropriate for conscious pa- cherry-red spots. Seizures develop by 2 years of
tients while waiting for a neurosurgery consult, age, and children typically do not survive past
but the patient in the case was already uncon- 45 years of age.
scious, and the operation must be performed
Answer B is incorrect. Deciency of
immediately.
-glucocerebroside leads to accumulation of

Neurology
Answer E is incorrect. Consent to treatment is glucocerebroside in the reticuloendothelial
implied when emergency treatment for a life- system. Gauchers disease is diagnosed by the
threatening condition is required. This applies presence of Gaucher cells in the bone marrow.
whether or not the patient is a minor. Gaucher cells have a characteristic crinkled
paper appearance. Patients typically present
39. The correct answer is E. Thrombolytic ther- in adolescence with bone pain, hepatospleno-
apy is contraindicated in any patient with: megaly, and anemia. Thrombocytopenia often
stroke or head trauma in the past 3 months, re- causes easy bleeding and bruising. On x-ray, an
cent myocardial infarction, prior intracranial Erlenmeyer ask deformity of the distal femur
hemorrhage, or major surgery in the past 14 is commonly noted.
days. Other contraindications in the patients
history include gastrointestinal or urinary Answer C is incorrect. Presence of phenylke-
bleeding in the past 21 days, or seizures present tones in the urine is diagnostic for PKU. PKU
at the onset of a stroke. presents with fair skin, eczema, and musty body
odor. As the child ages, mental retardation, hy-
Answer A is incorrect. A bruise is not a con- peractivity with purposeless movements, rhyth-
traindication, assuming the platelet count, pro- mic rocking, and athetosis will become evident
thrombin time, and activated partial thrombo- if left untreated.
plastin time are normal.
Answer D is incorrect. Krabbes disease results
Answer B is incorrect. While carotid stenosis from the absence of -galactosidase and leads
can increase the risk of an ischemic stroke, it to accumulation of galactocerebroside. This
is not a contraindication to the administration disease presents during the rst year of life with
of tissue plasminogen activator because it does optic atrophy, seizures, and spasticity. Opistho-
not increase the risk of bleeding. tonos, or abnormal posturing characterized by
Answer C is incorrect. A myocardial infarction rigidity and arching of the back with the neck
10 years ago is not a contraindication. A more hyperextended, occurs, and the child usually
recent myocardial infarction, however, would dies by age 3 years.
be. Answer E is incorrect. Niemann-Pick disease
Answer D is incorrect. Only recent peptic ul- results from deciency of sphingomyelinase,
cers are a contraindication (within the past 21 leading to a build-up of sphingomyelin and cho-
days). lesterol. Type A presents with hepatosplenomeg-
aly, failure to thrive, and rapidly progressive neu-
rodevelopmental regression leading to spasticity
and rigidity in later stages. Type B has a more
variable course and pulmonary involvement.
HIGH-YIELD SYSTEMS
316 Section I: Organ Systems Answers

41. The correct answer is C. Intubation is the Answer A is incorrect. Although behavioral
most important rst step in cases in whom the therapies such as relaxation therapy and bio-
patient is not conscious, including apnea, de- feedback can be successful in the management
creased mental status, impending airway com- of headaches, it would not be the rst-line or
promise (e.g., burn injury or maxillofacial sole preventive measure for medication-induced
trauma), closed head injuries, and failed bag headaches.
ventilation. Intubation is also indicated in pa-
Answer C is incorrect. -Blockers have a
tients with a GCS score <8.
greater impact for prophylactic treatment of
Answer A is incorrect. Antiepileptics and man- migraine headaches than for headaches due to
nitol for volume status are indicated, but the chronic analgesic use.
patients airway should be secured rst.
Answer D is incorrect. Selective serotonin
Answer B is incorrect. Cervical spine evalua- reuptake inhibitors are a newer addition to the
tion is indicated in a patient who presents as a prophylactic treatment of headaches. Although
trauma victim, but unless this patient sustained useful, the rst step in management of medica-
an injury during the seizure there is no indica- tion overuse headaches is to control the offend-
Neurology

tion for cervical spine management in this pa- ing medication rather than start a new drug
tient. treatment.
Answer D is incorrect. Most post-ictal patients Answer E is incorrect. Tricyclic antidepres-
breathe spontaneously, even if they remain sants such as amitriptyline and nortriptyline
unconscious. However, delaying intubation are useful prophylactic agents in the long-term
would not be advisable in a patient with a mas- treatment and prevention of migraines and ten-
sive intracranial hemorrhage, since respiratory sion headaches. However, the rst step in man-
control could rapidly become compromised. agement of medication overuse headaches is to
control the offending medication, rather than
Answer E is incorrect. While a full trauma sur-
start a new drug.
vey would be appropriate if the patient came
into the emergency department seizing, airway
43. The correct answer is C. This patient exhibits
management is the rst step in this instance.
symptoms of elevated ICP. Possible physical
Once the patients airway is secure and she is
examination ndings include papilledema (as
not actively seizing, additional components of
shown in the image); a dilated ipsilateral pupil;
the trauma survey can be conducted as perti-
cranial nerve palsies of the third (most com-
nent to the case.
mon), fourth, and sixth nerves; and nuchal ri-
gidity. Hemiparesis, hypertonia, hyperreexia,
42. The correct answer is B. The patient most
or Cushings triad (systemic hypertension, bra-
likely suffers from chronic daily headaches in-
dycardia, and respiratory depression) are late
duced by analgesic medication overuse (e.g.,
signs. In a case of suspected elevated ICP, the
acetaminophen), also known as analgesic re-
patient should be stabilized and CT of the
bound headaches. They typically last at least 4
head without contrast should be performed. A
hours per day and occur 15 days per month
midline shift or effacement of the sulci or basi-
and are seen most commonly in women in
lar cisterns are possible ndings on CT that
their fties. The rst step in prevention would
conrm the diagnosis of elevated ICP. The eti-
be to withdraw the patient from the offending
ology of the elevated ICP, such as a mass or
medication. If there is another underlying
hemorrhage, may also be evident on CT.
headache syndrome, this will then reveal itself.
Other possible causes of elevated ICP are hy-
Another analgesic can be used instead, or the
drocephalus, vasculitis, infarcts, and CNS in-
same one can be later reintroduced, but with
fection.
the instruction to consume it less often and in
lower doses.
HIGH-YIELD SYSTEMS
Chapter 10: Neurology Answers 317

Answer A is incorrect. Mannitol is an osmotic Answer C is incorrect. Bleeding in the epidu-


agent that establishes a gradient between ral space is most likely to be an epidural he-
plasma and parenchyma, causing a diuresis matoma. This is usually the result of trauma.
and reduction in brain water content. It can be Prognosis is good if treated appropriately and
used to decrease ICP. In this patient, CT of the early.
head and other studies should be carried out to
Answer D is incorrect. Bleeding in the ventri-
establish intracranial hypertension before man-
cles is typically due to SAH in adults. Aneurys-
nitol is considered.
mal bleeding has an extremely poor prognosis
Answer B is incorrect. Primary infantile glau- if not caught in time, leading to an almost 40%
coma is a rare disease that begins within the mortality rate.
rst 3 years of life. It is caused by an anomaly
of the trabecular meshwork, leading to im- 45. The correct answer is E. The event described is
paired drainage of aqueous uid. It classically a classic complex partial seizure. This patient is
presents with tearing, photophobia, and eye- unlikely to be having a febrile seizure, which is
lid squeezing, and is treated with surgery. A dened as a seizure in a child <6 years associ-

Neurology
-blocker such as timolol can be used postop- ated with a temperature >38C (>100.4F) in
eratively to decrease aqueous production and the absence of a systemic infectious, inamma-
prevent or postpone the need for a second sur- tory, or metabolic underlying condition. Abnor-
gery. malities seen on EEG are an indication to treat
a rst nonfebrile seizure if the family so chooses.
Answer D is incorrect. LP can trigger her-
When there is an abnormal EEG, the risk of an-
niation in the setting of elevated ICP due to
other seizure during the next year increases
a mass. Therefore CT of the head should be
from 15%41%. A family history of epilepsy in-
obtained to rule out a mass before LP is per-
creases the risk of recurrent seizures. Phenobar-
formed. If CNS infection is strongly suspected,
bital is the rst-line anticonvulsant for children
antibiotic therapy should be initiated, even if
with partial seizures.
LP must be delayed.
Answer A is incorrect. Acetaminophen will
Answer E is incorrect. CT of the head without
control fever. Antipyretic therapy is appropri-
contrast is the initial imaging study of choice
ate therapy for a febrile seizure. However, this
in children with suspected intracranial hyper-
child is too old to have febrile seizures, which
tension. If the initial CT is normal or if nd-
occur in children between 6 months and 6
ings such as a neoplasm, hemorrhage, or unex-
years of age.
plained hydrocephalus are noted, MRI of the
brain can be considered for further evaluation. Answer B is incorrect. It is important to con-
trol the fever in febrile seizures. However, as-
44. The correct answer is A. The basal ganglia pirin should not be used because it can lead to
and thalamus are the classic sites of a hyperten- Reyes syndrome. Reyes syndrome results in
sive bleed. This is because the small perforator encephalopathy and fatty degeneration of the
arteries come directly off much larger vessels. liver. Most signicantly, this child is not having
This results in higher blood pressures than are a febrile seizure, and an anticonvulsant is more
usual in arteries of that size, and they are thus appropriate for her.
more prone to hypertensive bleeding.
Answer C is incorrect. This antibiotic combi-
Answer B is incorrect. Bleeding in the brain nation would be appropriate if the child was
stem is generally due to a hemorrhagic stroke. suspected of having meningitis. LP should be
This can be caused by chronic hypertension, performed on children who have focal neuro-
but bleeding is more likely to occur in the logic signs, nuchal rigidity, or are younger than
basal ganglia and thalamus during an acute hy- 6 months to evaluate for meningitis in the ab-
pertensive event. The prognosis for brain stem sence of contraindications. Some experts rec-
bleeds depends on the size of the hemorrhage ommend that all children <1 year have an LP
but is generally poor because the brain stem is
one of the eloquent areas of the brain.
HIGH-YIELD SYSTEMS
318 Section I: Organ Systems Answers

if they have any focal neurologic signs. This tion, and patients have been successfully resus-
child is old enough to exhibit the classic signs citated with neurologic recovery even after long
if meningitis were present and, therefore, her periods of cardiac arrest, if the patients core
low-grade temperature does not warrant an LP. body temperature was low at the time of arrest.
Other requirements include the absence of in-
Answer D is incorrect. Ethosuximide is the
toxication or poisoning, the absence of meta-
rst-line treatment for absence seizures. Ab-
bolic or endocrine derangements, evidence of a
sence seizures appear as staring spells lasting
catastrophic cerebral event, and the absence of
510 seconds. On EEG, they show classic
brain stem reexes, such as the dolls-eyes (oc-
three-per-second spike and wave discharges.
ulocephalic) reex and the vestibulo-ocular re-
Ethosuximide is inappropriate therapy for par-
ex (response to caloric stimulation).
tial seizures.
Answer A is incorrect. The presence of arti-
46. The correct answer is B. Confusion, getting cial ventilation does not hamper a diagnosis of
lost, and intact long-term memory in the pres- brain death.
ence of difculties with recent memories are all
Answer C is incorrect. Historically, cardiac
Neurology

suggestive of AD. Donepezil, rivastigmine, gal-


arrest was used to dene death. A brain death
antamine, and tacrine are cholinesterase inhibi-
denition is now used, and a heartbeat does
tors approved by the Food and Drug Administra-
not preclude brain death.
tion for treating dementia of the Alzheimers
type. While these drugs do not alter the underly- Answer D is incorrect. Because one point for
ing disease process, they can retard the cognitive each category denes the bottom of the GCS,
decline in some patients with mild to moderate it is not possible to score lower than 3. A pa-
AD. Of the three drugs, donepezil has the most tient scoring 3 of 15 is profoundly comatose,
favorable adverse-effect prole. and coma and/or absence of cerebral motor re-
sponses is required to diagnose brain death.
Answer A is incorrect. Bupropion is a nonsera-
tonergic antidepressant. Answer E is incorrect. Evidence of a devastat-
ing neurologic event is required to diagnose
Answer C is incorrect. Doxepin is a psycho-
brain death; the presence of a skull fracture
tropic drug with antidepressant and anxiolytic
does not prevent this diagnosis.
effects.
Answer D is incorrect. Gingko biloba has 48. The correct answer is B. Age-related macular
been studied as a supplement to cholines- degeneration (ARMD) is a degenerative dis-
terase inhibitors, but is not proven effective as ease of the macula (central portion of the ret-
a stand-alone drug. ina) that results in the loss of central vision.
ARMD is classied as wet (neovascular or exu-
Answer E is incorrect. Thiamine (vitamin
dative) or dry (atrophic) type. Risk increases as
B12) is used for thiamine deciency which
patients age, with smoking being a major mod-
causes apathy, irritability, depression, and se-
iable risk factor in this disease process. Laser
vere memory impairment if deciency is pro-
photocoagulation can reduce the risk of severe
longed. Thiamine deciency may be second-
visual loss in patients with treatable exudative
ary to familial poverty or caused by chronic
ARMD, typically with drusen accompanied by
alcohol abuse.
a choroidal neovascular membrane. The laser
Answer F is incorrect. Vitamin E has been stud- photocoagulation should be applied within 72
ied as a supplement to cholinesterase inhibitors, hours of the uorescein angiography.
but is not proven effective as a stand-alone drug.
Answer A is incorrect. Atropine is an antimus-
carinic agent that results in mydriasis and is not
47. The correct answer is B. A diagnosis of brain
an acceptable form of treatment for ARMD.
death requires a core body temperature of at
least 32.2C (90.0F). Lower temperatures ex- Answer C is incorrect. This patient has exu-
hibit a neuroprotective effect via enzyme inhibi- dative ARMD. Timolol is a nonselective
HIGH-YIELD SYSTEMS
Chapter 10: Neurology Answers 319

-antagonist that reduces the production of Headache and visual eld defects, classically
aqueous humor in the eye. It can be used topi- bitemporal hemianopsia, are characteristic of a
cally in treatment of open-angle glaucoma. prolactinoma. As a result, central imaging is
warranted. In fact, any elevation of prolactin
Answer D is incorrect. Topical steroids are not
that cannot be attributed to another cause (i.e.,
indicated for treatment of ARMD.
neuroleptic medication) warrants CNS imag-
Answer E is incorrect. Although trabeculec- ing to evaluate for the presence of a macroade-
tomy can provide curative treatment for glau- noma, microadenoma, or other central lesion.
coma, this patient has exudative ARMD. Tra-
Answer A is incorrect. Mammography is not
beculectomy is not indicated in this situation.
needed for this patient. Patients with prolacti-
noma can present with gynecomastia, and even
49. The correct answer is E. Due to the relent-
in rare cases, galactorrhea. However, neither is
lessly progressive nature of AD, it is important
mentioned in the stem.
for patients to have a living will or durable
power of attorney. However, this does not need Answer B is incorrect. -hCG levels are ele-
to occur before the onset of disease because vated in patients with choriocarcinoma (100%),

Neurology
the majority of patients still have the capacity embryonal carcinoma (40%60%), and semi-
for self-determination at the time of diagnosis. noma (5%--10%), so it is often included in the
Treatment decisions are likely to be more in initial screening panel for testicular cancer.
line with the patients desires if the patient is Testicular cancer is most common in men
involved early in deciding who should serve as from puberty to age 35, and clinical presenta-
a proxy. The physician should have a discus- tion can include a neck mass (supraclavicular
sion with the patient regarding disease course node), dyspnea (pulmonary metastases), back/
and prognosis as soon as possible. bone pain (spinal/skeletal metastases), CNS/
peripheral nervous system signs, leg swelling
Answer A is incorrect. Family members should
(inferior vena cava involvement), or gyneco-
be involved in the discussion about future care,
mastia (systemic endocrine effects). Isolated
but the patient should decide who will ulti-
erectile dysfunction is less common. -hCG is
mately make treatment decisions.
also a test that is used to determine pregnancy
Answer B is incorrect. Patients with early AD status. In a woman with hyperprolactinemia,
still retain executive function and self-identity, pregnancy must be considered as a cause of el-
and should be allowed to make health care de- evated hormone levels.
cisions and select a proxy.
Answer C is incorrect. Accepted management
Answer C is incorrect. Patients with early AD of elevated prolactin levels requires imaging
still retain executive function and self-identity, regardless of the degree of prolactin elevation.
and should be allowed to make health care de- Higher levels would suggest that the lesion was
cisions and select a proxy. indeed a macroadenoma. Simply monitoring
Answer D is incorrect. Substituted judgment the prolactin level, which should be <20 ng/
comes into play when a health care proxy is mL, is insufcient, and a high level is cause for
faced with a decision for an incompetent pa- immediate work-up.
tient when that patient did not make any spe- Answer E is incorrect. Ultrasound of the scro-
cic requests pertaining to the situation. Sub- tum is used in the evaluation of scrotal masses,
stituted judgment cannot be used to choose a and has no utility here. The man is not noted
health care proxy. to have any masses in his testicles or any tes-
ticular pain.
50. The correct answer is D. The presentation
and laboratory ndings suggest that the man
needs to be evaluated for a prolactinoma.
This page intentionally left blank
CHAPTER 11

Obstetrics

321
HIGH-YIELD SYSTEMS
322 Section I: Organ Systems Questions

Q U E ST I O N S

1. A 14-year-old G1P0 girl who is 29 weeks preg- (A) Administration of Rh0(D) immune globu-
nant with twins presents to the emergency de- lin
partment following a seizure. She was watch- (B) Disseminated intravascular coagulation
ing television and stood up to go to the panel
bathroom when she fell down and started (C) External fetal heart rate and uterine moni-
shaking. The patient has no history of seizures toring
and is otherwise healthy. She missed her last (D) Immediate cesarean delivery
obstetricians appointment, and her aunt states (E) Immediate vaginal delivery
that her niece has had a lot of headaches and (F) Internal fetal heart rate and uterine moni-
swelling over the past 2 days. On examination toring
the patient is somnolent and difcult to arouse,
and has edema of her hands and face. Her 3. A 19-year-old G1P0 woman presents to the
blood pressure is 205/120 mm Hg, pulse is 80/ emergency department in active labor and de-
Obstetrics

min, and respiratory rate is 16/min. The fetal livers a full-term male infant. The infant ap-
heart rate is 130/min. Which of the following pears healthy with the exception of jaundice
is the most correct advice for the patients (bilirubin 10 mg/dL [>95th percentile]). The
aunt? mother does not speak English, but a cousin
who does states that he has seen the mother
(A) Your niece has a life-threatening condi-
taking pills prescribed by her doctor, although
tion called eclampsia, and needs to be put
he does not know the reason she was taking
on strict bed rest and monitored until the
medication. Based on the newborns jaundice,
baby can be delivered at term
which drug was the mother most likely taking?
(B) Your niece has a life-threatening condi-
tion called eclampsia, and the baby needs (A) Angiotensin-converting enzyme inhibitor
to be delivered as soon as possible (B) Lithium
(C) Your niece has a life-threatening condi- (C) Phenytoin
tion called eclampsia, but this can be man- (D) Tretinoin
aged with antiseizure medications until (E) Trimethoprim-sulfamethoxazole
the baby can be delivered at term
(D) Your niece has a life-threatening condi- 4. A 16-year-old girl presents to the emergency
tion called preeclampsia, and needs to be department complaining of fever, chills, ab-
put on strict bed rest and monitored until dominal pain, and vaginal bleeding. She gives
the baby can be delivered at term a history of unprotected sexual activity with her
(E) Your niece has a life-threatening condi- 17-year-old boyfriend over the past several
tion called preeclampsia, and the baby months. Her last menstrual period was 8 weeks
needs to be delivered as soon as possible ago. She reports having a dilatation and curet-
tage procedure at an unlicensed abortion clinic
2. A 19-year-old woman at 32 weeks gestation recently to try to abort the pregnancy. Her tem-
was the driver in a front-end motor vehicle perature is 38.7C (101.7F), heart rate is 120/
crash. The air bags did not inate, and the pa- min, and blood pressure is 100/70 mm Hg. Pel-
tient sustained blunt trauma to the abdomen. vic examination reveals cervical motion tender-
The patient is taken to a nearby emergency de- ness, tissue in the internal os, and foul-smelling
partment in stable condition, where she notes vaginal discharge. Urine is positive for
a small amount of bright red blood on her un- -human chorionic gonadotropin. Which of
derwear. Maternal vital signs are signicant for the following is the most likely diagnosis?
a heart rate of 110/min and blood pressure of
110/55 mm Hg. What is most appropriate next
step in management?
HIGH-YIELD SYSTEMS
Chapter 11: Obstetrics Questions 323

(A) Ectopic pregnancy 6. A 30-year-old obese G3P2 woman is in active


(B) Pelvic abscess labor at 41 weeks gestation. She has no signi-
(C) Septic abortion cant past medical history, and had an uncom-
(D) Threatened abortion plicated pregnancy with appropriate prenatal
(E) Vaginal laceration evaluation. The patient ruptured membranes
spontaneously 30 minutes ago. Contractions
5. A 24-year-old woman presents to her primary occur regularly every 23 minutes. Early decel-
care physician with a complaint of 1 week of erations are noted on the fetal heart rate moni-
increased vaginal discharge with an unpleasant tor with each of the past ve contractions.
odor. She is sexually active with one partner Which is the most appropriate next step in
and uses oral contraception for birth control. A management?
pregnancy test is negative. Gynecologic exami-
(A) Change the maternal position
nation reveals a pink cervix and a thin white
(B) No further management is required
discharge. The discharge has a positive amine
(C) Place a fetal scalp probe
whiff test and a pH of 6. Wet saline mount
(D) Prepare for emergent cesarean delivery
microscopy yields the following image. Which

Obstetrics
(E) Start an amnioinfusion of saline
of the following is the most likely diagnosis?
7. A 25-year-old G2P1 woman who is 36 weeks
pregnant presents to her obstetrician complain-
ing of restlessness and weakness for the past
month. She says her boyfriend recently left her
and their 2-year-old son, and she feels over-
whelmed with this pregnancy. She denies feel-
ing depressed but does report that she has trou-
ble sleeping. She had an upper respiratory
infection last month, caught from my son,
and states she still has a sore throat. Laboratory
tests show:
WBC count: 8000/mm
Hemoglobin: 11.0 g/dL
Hematocrit: 40%
Platelet count: 250,000/mm
Thyroid-stimulating hormone: 0.5 U/mL
Reproduced, with permission, from Tintinalli JE, Kelen Free thyroxine: 4.0 ng/dL
GD, Stapczynski S, Ma OJ, Cline DM. Tintinallis Emer-
gency Medicine: A Comprehensive Study Guide, 6th edi- Which of the following is the best next step in
tion. New York: McGraw-Hill, 2004: Figure 108-1. management?
(A) Bacterial vaginosis (A) Measure postpartum thyroid hormone
(B) Neisseria gonorrhoeae cervicitis levels
(C) Trichomonas vaginalis (B) Perform partial thyroidectomy
(D) Vaginal candidiasis (C) Start levothyroxine therapy
(E) Vulvar candidiasis (D) Start propylthiouracil therapy
(E) Start radioiodine therapy
HIGH-YIELD SYSTEMS
324 Section I: Organ Systems Questions

8. A 19-year-old G0 woman presents to her family ceive. Her hypothyroidism is well controlled
physician complaining of dysmenorrhea for the and stable on thyroxine, and she has no other
past year. She reports severe right-sided pain medical conditions. She is healthy and does
that coincides with days 15 of her menstrual not smoke or drink alcohol. She would like to
cycle. Her menses occur regularly every 28 know if she should keep taking her thyroxine.
days, and she requires three to four pads per Which of the following is the best advice to
day for the rst 2 days of her bleeding and one give this patient?
to two pads per day for the remainder. She has
(A) No, but we would want to keep your thy-
never had surgery. She is not sexually active
roid levels balanced for the sake of your
and does not smoke. Her last menstrual period
baby, so you would be switched to meth-
was 1 week ago. Her temperature is 36.7C
imazole
(98.1F), blood pressure is 121/74 mm Hg,
(B) No, thyroxine is generally accepted as
heart rate is 80/min, and respiratory rate is 14/
safe during pregnancy, but if you are not
min. Physical examination reveals a thin,
comfortable taking it, there is no evidence
healthy-appearing young woman. Pelvic exami-
that being hypothyroid will affect your
nation reveals a normal sized uterus and no
Obstetrics

baby
cervical motion tenderness. Which of the fol-
(C) No, thyroxine is not safe when taken dur-
lowing is the most likely diagnosis?
ing pregnancy; it is better for both you and
(A) Ectopic pregnancy your baby for you to be hypothyroid
(B) Endometriosis (D) Yes, but we would likely decrease your
(C) Leiomyoma thyroxine during pregnancy because preg-
(D) Pelvic inammatory disease nancy is accompanied by mild physiologic
(E) Polycystic ovarian syndrome hyperthyroidism
(E) Yes, in fact we would likely need to in-
9. A 36-year-old G1P0 woman pregnant with crease your thyroxine during pregnancy to
twins presents to her obstetrician for her rou- avoid hypothyroidism, which may ad-
tine 32-week appointment. She has gained 5.4 versely affect your baby
kg (12 lb) in the past 2 weeks. When ques-
tioned about her weight gain, she states she has 11. A 23-year-old primigravid woman at 36 weeks
had headaches and some blurred vision for the gestation presents to the labor and delivery unit
past 2 weeks, which she thinks is secondary to with intense cramping that began several hours
dehydration. To circumvent this she has been ago. Her pregnancy has been uncomplicated.
drinking a lot of water, which she claims is Leopold maneuvers reveal a fetus appropriate
making me swell, even my hands. She also in size for gestational age in vertex presenta-
has had some epigastric pain for the past 2 tion. Tocometry reveals four irregular contrac-
weeks, which she attributes to all the water tions lasting >30 seconds over 30 minutes. The
Ive been drinking. Her blood pressure is fetal heart tracing is reassuring. The cervix is
142/90 mm Hg, pulse is 105/min, and respira- one ngerbreadth dilated on sterile vaginal ex-
tory rate is 18/min. Urinalysis reveals 1+ gluco- amination. After 6 hours the cervix is 1 cm di-
suria and 4+ proteinuria. Which of the follow- lated. A prostaglandin gel is administered, and
ing is the best next step in management? 2 hours later she is 2 cm dilated. At this time
oxytocin infusion is begun, and the cervix be-
(A) Administer magnesium sulfate only
gins to dilate and become effaced more rapidly.
(B) Administer oral antihypertensive therapy
Tocometry reveals powerful and regular con-
(C) Expectant management
tractions. After 4 hours of oxytocin infusion,
(D) Induce labor
she fails to progress. What curve in the image
(E) Platelet transfusion
best represents the labor curve of this patient,
10. A 24-year-old woman with chronic hypothy- and what is the most likely etiology?
roidism presents to her gynecologist for her an-
nual examination. She recently got married,
and she and her husband would like to con-
HIGH-YIELD SYSTEMS
Chapter 11: Obstetrics Questions 325

10
13. A 32-year-old G3P3 woman presents to her ob-
stetrician for help conceiving. She states her
Cervical dilat ion, cm

8 menstrual cycles have not been regular since


the birth of her third child 3 years ago. Fur-
Normal C
6 thermore, although she readily became preg-
nant with her other three children, she has
4
failed to become pregnant despite trying over
2 B the past 2 years. She has no signicant past
A medical history and takes only prenatal vita-
0 mins. Although she says she has not been ill
0 4 8 12 16 20 24
Hours in labor
lately, she reports feeling tired and cold all the
time. She also reports she has had trouble
Reproduced, with permission, from USMLERx.com.
sleeping over the past several months. Her
(A) Curve A; inadequate cervical ripening physical examination is normal. Laboratory
(B) Curve A; primigravid patient tests show:
(C) Curve B; inadequate force of uterine con-

Obstetrics
WBC count: 9000/mm
tractions
Hemoglobin: 8.0 g/dL
(D) Curve B; inadequate maternal sedation
Platelet count: 300,000/
(E) Curve C; cephalopelvic disproportion
Hematocrit: 40%
(F) Curve C; occiput anterior
Thyroid-stimulating hormone level: 0.5 U/mL
Free thyroxine: 2.0 ng/dL
12. A 32-year-old G3P3 woman is postoperative
Luteinizing hormone: 0.5 mU/mL
day 5 after an emergent cesarean section due
Follicle-stimulating hormone: 0.5 mU/mL
to fetal distress. The patient progressed rapidly
through passive labor without incident, but af- Which of the following will this woman likely
ter her membranes were ruptured manually, a need to take to conceive?
fetal scalp probe was placed in the active phase
(A) Clomiphene
secondary to several runs of mid-late decelera-
(B) Levothyroxine
tions. Cesarean section was ultimately per-
(C) Prednisone
formed after 2 hours of active labor secondary
(D) Progesterone
to fetal distress. The patient presents now with
(E) Propylthiouracil
a fever to 38.7C (101.7F) and uterine tender-
ness. Laboratory tests reveal a WBC count of
14,000/mm, with 70% neutrophils and 4%
bands. Which of the following is the most ap-
propriate treatment?
(A) Ampicillin and gentamicin
(B) Cefotaxime and levooxacin
(C) Clindamycin and gentamicin
(D) Imipenem
(E) Metronidazole and doxycycline
HIGH-YIELD SYSTEMS
326 Section I: Organ Systems Questions

14. A 22-year-old G1P0 woman who is 10 weeks 16. A 28-year-old G1P0 woman at 12 weeks gesta-
pregnant with twins presents to the emergency tion presents for routine follow-up with her ob-
department because of vomiting and dizziness. stetrician. She complains of mild nausea and
She has had morning sickness for the past occasional vomiting, but otherwise is doing
month and would vomit once or twice a day. well and reports no other symptoms or compli-
However, over the past week, she has been cations. Her physical examination is unremark-
vomiting multiple times a day, and she has able and fetal ultrasound is normal for gesta-
been unsuccessful at keeping anything down tional age. Laboratory tests show:
for the past 2 days. She denies fever or change
Free triiodothyronine: 180 ng/dL
in her bowel movements; her last bowel move-
Free thyroxine: 2.2 ng/dL
ment was that morning and was well formed.
Total thyroxine: 12 g/dL
She has otherwise been healthy. Physical ex-
Thyroid-stimulating hormone: 0.1 U/mL
amination reveals a tired-appearing, pale
(normal: 0.44 U/mL)
woman with poor skin turgor; otherwise her ex-
amination is unremarkable. Her blood pressure Results of a thyroid-stimulating hormone re-
is 110/75 mm Hg lying down and 90/45 mm ceptor antibody test are negative. Which of the
Obstetrics

Hg sitting up. Her pulse is 80/min lying down following best explains these ndings?
and 115/min sitting up. Her respiratory rate is
(A) Acute infectious thyroiditis
24/min, and her temperature is 37.2C
(B) Graves disease
(99.0F). Her original blood work results are:
(C) Hashimotos thyroiditis
WBC count: 14,000/mm3 (D) High serum estrogen concentration
Platelet count: 350,000/mm3 (E) High serum -human chorionic gonado-
Na+: 150 mEq/L tropin level
K+: 4 mEq/L
Cl: 88 mEq/L 17. A 23-year-old G1P0 woman at 37 weeks gesta-
HCO3: 26 mEq/L tion is seen in the antepartum care unit. She is
Hemoglobin: 15 g/dL having regular contractions and the cervix is
Hematocrit: 40% two ngerbreadths dilated on sterile pelvic ex-
Aspartate aminotransferase: 80 U/L amination. Results of electronic fetal heart
Alanine aminotransferase: 85 U/L monitoring are shown in the image. What
mechanism best explains the ndings seen on
What is this womans most likely diagnosis?
this tracing?
(A) Acute viral hepatitis A
(B) Food poisoning with Salmonella
(C) Hyperemesis gravidarum
(D) Preeclampsia 160
(E) Viral gastroenteritis
heart
Fet al

rat e

140

15. A 21-year-old woman presents to the clinic in 120


tears. She states that she recently found out she
was pregnant at 10 weeks gestation. She is a
recovering alcoholic but recently relapsed, Maximum
cont ract ion

consuming several drinks a day. She is nervous


Ut erine

about the effects of her drinking on her fetus.


For which of the following is the patient at
greatest risk? Minimum

(A) Eclampsia Time


(B) Hypoplastic lung Reproduced, with permission, from USMLERx.com.
(C) Macrosomia
(D) Microcephaly
(E) Polyhydramnios
HIGH-YIELD SYSTEMS
Chapter 11: Obstetrics Questions 327

(A) Artifact from strong uterine contractions 20. A 28-year-old G0 woman presents to the clinic
(B) Compression of the umbilical cord by complaining of inability to conceive and amen-
uterine contractions orrhea. She has been taking a low-dose oral
(C) Fetal arrhythmia contraceptive pill for the past 6 years, which
(D) Pressure on the fetal head coincident with she discontinued 3 months ago when she and
uterine contractions her husband decided they wanted to have chil-
(E) Reex response to fetal hypoxia from fetal dren. They have been sexually active with each
anemia other two to three times per week over the past
3 months, but the patient has not become
18. A 27-year-old G1P0 woman at 27 weeks gesta- pregnant. The patient denies a history of sexu-
tion presents to the emergency department after ally transmitted disease and states that until re-
a motor vehicle accident. The patient denies cently she has always had regular menstrual
any abdominal pain or cramping, contractions, cycles. She has not had a period since discon-
or vaginal bleeding. Examination reveals a tinuation of the oral contraceptive. Which of
gravid, nontender abdomen and a closed, non- the following is the most appropriate next step?
effaced cervix with no evidence of vaginal bleed-

Obstetrics
(A) Administer a progesterone challenge
ing. Fetal heart monitoring shows a fetal heart
(B) Check follicle-stimulating hormone and
rate of 145/min, with variable accelerations and
luteinizing hormone levels
no decelerations. The patient is Rh negative
(C) Observation
with no history of blood transfusion, while the
(D) Perform a hysterosalpingogram
father is of unknown Rh status and unavailable.
(E) Perform a pelvic ultrasound
The results of the Kleihauer-Betke test, in which
maternal blood is exposed to acid, shows a com-
21. A 31-year-old G3P2 woman at 37 weeks gesta-
bination of pale and stained RBCs. Which of
tion presents to the labor and delivery oor af-
the following is the best next step in manage-
ter 2 hours of contractions of increasing fre-
ment?
quency and intensity. An epidural anesthetic is
(A) Administer an appropriate dose of intra- requested on admission and placed. The pa-
muscular Rh0(D) immune globulin tient continues to have contractions for the
(B) Amniocentesis to measure the amniotic next 15 hours, during which time her mem-
uid bilirubin level branes rupture spontaneously. Vaginal exami-
(C) Emergent cesarean section nation at that time reveals a cervix that is soft,
(D) Induction of vaginal labor with prostaglan- 3 cm dilated, in an anterior position, and 80%
dins and oxytocin effaced. The fetal head is at the 1 station. Fe-
(E) Treatment with betamethasone tal heart tracings reveal a baseline heart rate of
156/min, with variable accelerations and no
19. A 27-year-old G1 woman is 20 weeks pregnant. signicant decelerations. Which of the follow-
She is currently in her third year of a family ing is the best next step in management?
practice residency and would like to travel to
(A) Apply intravaginal prostaglandin E2
Africa and Asia as part of an outreach mission
(B) Attempt forceps-facilitated delivery
with her program. She has received all of her
(C) Begin an infusion of oxytocin
childhood immunizations. She presents to the
(D) Increase the rate of intravenous uids to
obstetric clinic inquiring about the safety of
hydrate the patient
immunizations during pregnancy. Which of
(E) Proceed to cesarean section
the following vaccines is contraindicated in
pregnancy?
(A) Hepatitis A
(B) Hepatitis B
(C) Inuenza
(D) Tetanus
(E) Typhoid
(F) Varicella
HIGH-YIELD SYSTEMS
328 Section I: Organ Systems Questions

22. A 24-year-old G2P2 woman presents to the 24. A 30-year-old G3P2 woman at 25 weeks gesta-
emergency department complaining of vaginal tion has a history of gestational diabetes in her
bleeding and abdominal cramping. She is sex- previous pregnancy. Her fasting blood glucose
ually active in a monogamous relationship level at her initial 10-week screening visit was
with her husband. Her last menstrual period 110 mg/dL and urinalysis was negative for glu-
was 6 weeks ago. The patient is afebrile, and cose in the urine. The patient has not been
vital signs are within normal limits. Pelvic ex- taking her own blood sugars at home, but she
amination is notable for a dilated cervix, fetal has been adhering to a low-carbohydrate diet.
tissue in the vaginal vault, and no cervical mo- Over the past several weeks, she has noticed in-
tion tenderness. Which of the following is the creased fatigue and polyuria. Which of the fol-
most likely cause of this patients abortion? lowing is the next most appropriate step?
(A) Acute maternal infection (A) Administer a 3-hour glucose tolerance test
(B) Chromosomal abnormality (B) Administer a 50-g 1-hour glucose tolerance
(C) Maternal exposure to environmental test
chemicals (C) Begin insulin therapy
Obstetrics

(D) Maternal smoking (D) Check a urinalysis and start insulin if uri-
(E) Trauma nalysis reveals glucose in the urine
(E) Prescribe metformin to be taken daily
23. A 28-year-old G2P1 woman with hypertension
is in active labor at 42 weeks gestation. She 25. A 31-year-old woman with systemic lupus ery-
had an uncomplicated pregnancy and rup- thematosus who is 4 weeks pregnant presents
tured membranes spontaneously 2 hours ago. to her obstetrician for her rst prenatal visit.
Fetal heart tracings are reassuring, contractions She is very concerned that the lupus will affect
occur regularly every 23 minutes, and the pa- her baby. She was diagnosed with systemic lu-
tients blood pressure is 110/75 mm Hg. The pus erythematosus 5 years ago and her symp-
on-call physician is suddenly called into the toms have been well controlled with low-dose
patients room by the nurse because of the nd- prednisone. She has baseline renal insuf-
ings on the fetal heart monitor, as shown in the ciency, with a creatinine level of 1.3 mg/dL
image. Which of the following is the most that has been stable for the past 6 months. This
likely cause? is her rst pregnancy. For which of the follow-
ing is the baby at increased risk?
(A) Acute renal failure
(B) Chorioretinitis
Late (C) Complete heart block
160
heart
Fet al

rat e

140 (D) Ebsteins anomaly


120 (E) Rash
cont ract ion

26. A 30-year-old G0 woman with a past medical


Ut erine

Maximum
history of dysmenorrhea presents to an infertil-
ity clinic with her husband for a follow-up visit.
Minimum
The couple has been trying to get pregnant for
Time the past 3 years with no success. Their infertil-
Reproduced, with permission, from USMLERx.com. ity work-up thus far has included a semen anal-
ysis, hysterosalpingogram, and estrogen, pro-
(A) Fetal head compression
gesterone, and follicle-stimulating hormone
(B) Fetal myocardial depression
blood levels, all of which were normal. Cur-
(C) Fetal vagal stimulation
rently the woman feels well; her only com-
(D) Placenta previa
plaint is frustration regarding her inability to
(E) Placental abruption
conceive. A pelvic ultrasound done last week
demonstrated a 3-cm well-circumscribed mass
on the patients left ovary. Her last menstrual
HIGH-YIELD SYSTEMS
Chapter 11: Obstetrics Questions 329

period was 3 weeks ago. The ovarian mass most drochlorothiazide and lisinopril as an outpa-
likely represents which of the following? tient, but these drugs were discontinued when
she found out that she was pregnant. Her blood
(A) Corpus luteum cyst
pressure has been relatively well controlled in
(B) Ectopic pregnancy
the 120130 mm Hg systolic range without
(C) Endometrioma
medication, and urinalysis has consistently
(D) Leiomyoma
been negative for proteinuria at each of her
(E) Tubo-ovarian abscess
prenatal visits. She presents now to the obstet-
ric clinic with a blood pressure of 142/84 mm
27. A 34-year-old G1P0 woman at 29 weeks gesta-
Hg. A 24-hour urine specimen yields 0.35 g of
tion presents to the emergency department
proteinuria. Which of the following is the most
complaining of 2 hours of vaginal bleeding. The
appropriate next step?
bleeding recently stopped, but she was diag-
nosed earlier with placenta previa by ultra- (A) Administer oral furosemide
sound. She denies any abdominal pain, cramp- (B) Prepare for emergent delivery
ing, or contractions associated with the bleeding. (C) Restart the patients prepregnancy antihy-

Obstetrics
Her temperature is 36.8C (98.2F), blood pres- pertensive regimen
sure is 118/72 mm Hg, pulse is 75/min, and res- (D) Restricted activity and close monitoring as
piratory rate is 13/min. She reports she is Rh- an outpatient following initial inpatient
positive, her hemoglobin is 11.1 g/dL, and evaluation
coagulation tests, brinogen, and D-dimer levels (E) Start hydralazine
are all normal. On examination her gravid ab-
domen is nontender. Fetal heart monitoring is 29. A 32-year-old G2P1 woman at 35 weeks gesta-
reassuring, with a heart rate of 155/min, variable tion presents to her obstetrician for a routine
accelerations, and no decelerations. Two large- prenatal check-up. The mother has been previ-
bore peripheral intravenous lines are inserted ously diagnosed with mild preeclampsia, which
and two units of blood are typed and crossed. the obstetrician has chosen to manage expec-
What is the most appropriate next step in man- tantly. During the visit, a biophysical prole is
agement? performed and the amniotic uid index is
found to be <5 cm, indicating the develop-
(A) Admit to the antenatal unit for bed rest
ment of oligohydramnios. The biophysical pro-
and betamethasone
le is otherwise normal, with a total score of
(B) Admit to the antenatal unit for bed rest
8/10 and reassuring fetal heart tracings. How
and blood transfusion
should oligohydramnios be managed in this
(C) Admit to the antenatal unit for bed rest
patient?
and treatment with RhO(D) immune glob-
ulin (A) Administration of betamethasone, then ce-
(D) Emergent cesarean section sarean section in 24 hours
(E) Outpatient expectant management (B) Amnioinfusion with normal saline solution
(C) Biweekly fetal biophysical proles
28. A 32-year-old G3P2 woman at 35 weeks gesta- (D) Emergent cesarean section
tion has a past medical history signicant for (E) No change in management is necessary
hypertension. She was well-controlled on hy-
HIGH-YIELD SYSTEMS
330 Section I: Organ Systems Questions

30. At 38 weeks gestation, a 4030-g (8.9-lb) boy is abdominal pain and vaginal bleeding. The pa-
delivered by spontaneous vaginal delivery. tient states that she is sexually active with her
During the rst minute of life he is limp, cya- boyfriend and uses condoms basically all the
notic, lacks respiratory effort, has a heart rate of time. She states that her last menstrual period
95/min, exes his extremities, and grimaces to was 7 weeks ago and insists that her periods
nasal suctioning. By 5 minutes, he continues to have always been irregular, occurring every 3
grimace to nasal suctioning, has a weak cry, is to 4 months. She denies any past medical his-
well perfused with a heart rate of 160/min, and tory but states that she used to have a problem
is kicking both legs. Based on his Apgar scores, with excess facial hair prior to starting low-dose
when will the child need to be resuscitated? oral contraceptive pills. Which of the following
is the best next step in diagnosis?
(A) Indicated at 1 and 5 minutes
(B) Indicated at 1 minute and not at 5 minutes (A) Endometrial biopsy
(C) Indicated at 5 minutes and not at 1 minute (B) Measure prothrombin time/partial throm-
(D) Not enough information to determine boplastin time
(E) Not indicated at 1 or 5 minutes (C) Measure thyroid-stimulating hormone
Obstetrics

level
31. A 23-year-old G1P0 woman at 28 weeks gesta- (D) Measure urine -human chorionic gonad-
tion presents to her obstetrician for a prenatal otropin level
examination. She has received poor prenatal (E) Progesterone challenge
care up to this point, but is condent about (F) Ultrasound of the ovaries
dating the pregnancy. She denies use of alco-
hol and illicit drugs but has continued to 33. A full-term 2200-g (4.9-lb) boy was born to a
smoke during the pregnancy. The mother has 30-year-old G4P3 woman whose pregnancy
gained only 9 kg (20 lb) during the course of was complicated by a seizure disorder for
the pregnancy. The mothers temperature is which she inconsistently took carbamazepine.
36.8C (98.2F), pulse is 94/min, blood pres- The pregnancy was also notable for an abnor-
sure is 138/84 mm Hg, and respiratory rate is mal triple screen for which an amniocentesis
12/min. The fundal height is 23 cm above the was declined. His Apgar scores are 7 and 9 at 1
pubic symphysis. Further examination with ul- and 5 minutes, respectively. His temperature is
trasound reveals the fetus is <10% of the ex- 37.0C (98.6F), blood pressure is 65/45 mm
pected weight for the gestational age with sym- Hg, heart rate is 110/min, and respiratory rate
metric growth anomalies. What is the most is 30/min. His head circumference is <5th per-
likely cause for the intrauterine growth restric- centile. There is a small eshy sac protruding
tion of this fetus? from the sacral spine. His reexes are 2+
throughout, and his strength is 5/5 in all ex-
(A) In utero infection
tremities. His ngernails are very small. Which
(B) Inadequate maternal weight gain during
of the following is the most likely diagnosis?
pregnancy
(C) Maternal hypertension (A) Anoxia due to maternal seizing
(D) Maternal smoking (B) Fetal alcohol syndrome
(E) Singleton pregnancy (C) Perinatal exposure to carbamazepine
(D) Trisomy 18
32. A 22-year-old obese woman presents to the ob- (E) Trisomy 21
stetrics-gynecology clinic complaining of mild
HIGH-YIELD SYSTEMS
Chapter 11: Obstetrics Questions 331

34. A 24-year-old G1P0 woman at 31 weeks gesta- (A) Endometriosis


tion presents to the emergency department (B) Low sperm concentration
with a 4-hour history of abdominal cramping (C) Pelvic inammatory disease
and contractions. The contractions have been (D) Premature ovarian failure
regularly spaced at 10 minutes, but seem to be (E) Prior placement of an intrauterine device
increasing in intensity. She has had a small
amount of vaginal discharge, but is unable to E X T E N D E D M ATC H I N G
denitively say whether her water has broken.
She has not had any vaginal bleeding. Her
temperature is 36.8C (98.3F), blood pressure The response options for the next 2 items are
is 137/84 mm Hg, pulse is 87/min, and respira- the same. Select one answer for each item
tory rate is 12/min. Physical examination re- in the set.
veals a nontender abdomen with palpable con-
tractions every 8 minutes. Which of the For each child, identify the structure that is in-
following is the best next step in management? jured.

Obstetrics
(A) Cervical culture for Group B streptococci (A) Adrenal gland
(B) Digital cervical examination and assess- (B) Facial nerve
ment of dilation and effacement (C) Fifth and sixth cervical nerves
(C) Quantication of strength and timing of (D) Left clavicle
contractions with an external tocometer (E) Left humerus
(D) Speculum examination to rule out rupture (F) Left radius
of membranes and visually assess cervical (G) Liver
dilation and effacement (H) Phrenic nerve
(E) Ultrasound examination of the fetus (I) Right clavicle
(J) Seventh and eighth cervical nerve and rst
35. A 28-year-old woman and her husband present thoracic nerve
to her obstetrician. They have been married for (K) Spinal cord
7 years and have been trying to become preg- (L) Third and fourth cervical nerves
nant for the past 2 years. Prior to this the
woman used an intrauterine device for contra- 36. A child is delivered via vertex vaginal delivery.
ception, which she had in place for 5 years. On examination the Moro reex is asymmetri-
Both are healthy without any medical prob- cal (absent in the left upper extremity) and the
lems, and both deny a history of sexually trans- left sternocleidomastoid muscle is in spasm.
mitted diseases. The woman states that her
menstrual cycles have always been regular (ev- 37. Shortly after birth, a girl is examined by a neo-
ery 28 days, lasting for 5 days) since she was 14 natologist. Her right arm is adducted and inter-
years old. She also denies menorrhagia and nally rotated; the right palm is pronated. This
dysmenorrhea. Which of the following is the position does not change over several minutes.
most likely cause of this couples infertility?
HIGH-YIELD SYSTEMS
332 Section I: Organ Systems Questions

The response options for the next 3 items are 39. A 32-year-old G6P5 woman at 29 weeks gesta-
the same. Select one answer for each item tion with a history of four previous cesarean
in the set. sections presents to the emergency obstetrics
clinic with painless vaginal bleeding that began
For each patient with vaginal bleeding, select the this morning and ceased shortly thereafter. She
most appropriate diagnosis. reports normal fetal movement and denies vag-
inal loss of uid or uterine contractions. Her
(A) Abruptio placentae vital signs are normal and her uterine fundus is
(B) Breakthrough bleeding nontender and appropriately enlarged for ges-
(C) Cervical carcinoma tational age. There is no active bleeding from
(D) Ectopic pregnancy the vaginal canal. Fetal heart tracing is normal
(E) Endometrial cancer and transvaginal sonography reveals a viable fe-
(F) Foreign body tus and a low-lying placenta with its posterior
(G) Normal menses edge 1 cm from the internal cervical os.
(H) Placenta previa
(I) Preterm labor 40. An 18-year-old G1P0 woman at 35 weeks ges-
Obstetrics

(J) Threatened abortion tation presents to the emergency obstetrics


(K) Vulvar carcinoma clinic for evaluation. She reports a gushing loss
of uid from her vagina this morning with back
38. A 27-year-old G3P2 woman at 30 weeks gesta- pain, abdominal cramping, and vaginal spot-
tion presents to the emergency obstetrics clinic ting for the past several hours. The woman has
following a 35-mph rear-end collision with an- normal, stable vital signs and fetal heart trac-
other car. She has no previous medical history ing detects a fetal heart rate of 140/min with-
and her pregnancy has been uncomplicated. out decelerations. A tocometer detects uterine
She is anxious and complains of sudden onset contractions every 10 minutes lasting 30 sec-
of severe abdominal pain, dark vaginal bleed- onds each. Sterile vaginal examination reveals
ing, and uterine contractions. She is hemody- a cervical os that is dilated at two nger-
namically stable but tachycardic. There is hy- breadths.
pertonicity of the uterus and a nonreassuring
fetal heart tracing.
HIGH-YIELD SYSTEMS
Chapter 11: Obstetrics Answers 333

AN S W E R S

1. The correct answer is B. This patients seizure, complete abruption has occurred, as the pa-
hypertension, headache, and facial edema are tient is not frankly hypotensive and her bleed-
consistent with a diagnosis of eclampsia. Ec- ing was minimal. All women >24 weeks of ges-
lampsia is dened as seizure activity or coma tation subjected to abdominal trauma should
in an obstetric patient with preeclampsia. De- have continuous fetal and uterine monitoring
livery is an immediate necessity once the pa- with an external fetal heart rate to assess for
tient is stabilized to prevent maternal and fetal preterm labor and/or an abruption. Signs of fe-
mortality. tal compromise are associated with moderate
to severe abruption and would necessitate im-
Answer A is incorrect. Delaying delivery once
mediate delivery.
a seizure has occurred is contraindicated be-
cause eclampsia is life threatening and will Answer A is incorrect. Administration of
lead to multiorgan system failure. Rh0(D) immune globulin is appropriate in

Obstetrics
an unsensitized Rh-negative woman. This pa-
Answer C is incorrect. Preventing seizures is
tients blood type would need to be established
critical in eclampsia but will not prevent the
prior to administration of this drug.
multiorgan system failure that will occur if de-
livery is postponed. Answer B is incorrect. While patients with pla-
cental abruptions are at risk for developing dis-
Answer D is incorrect. Preeclampsia devel-
seminated intravascular coagulation, this patient
ops after 20 weeks gestation and is dened by
does not exhibit signs of a coagulopathy. A coag-
a blood pressure >140/90 mm Hg in a previ-
ulation panel should be included in routine lab-
ously normotensive patient, plus proteinuria.
oratory blood work on this patient; however, it is
Hypertension, headache, and edema are also
an inappropriate rst step prior to the establish-
symptoms of preeclampsia; however, once a
ment of external fetal and uterine monitoring.
patient has a seizure, she is classied as having
eclampsia. Answer D is incorrect. Although cesarean sec-
tion is the appropriate method of delivery in a
Answer E is incorrect. Preeclampsia does not
patient with a placental abruption, delivery is
require immediate delivery. Mild preeclampsia
only warranted when there are signs of fetal or
can be handled as an outpatient. Severe preec-
maternal compromise. This patient is at risk
lampsia can be monitored in an intensive care
for fetal distress due to uteroplacental insuf-
unit. Immediate delivery for severe preeclamp-
ciency, although further evaluation of the fetus
sia is only indicated for fetal age >30 weeks or
is necessary prior to delivery.
fetal distress.
Answer E is incorrect. Vaginal delivery is an
2. The correct answer is C. Abruptio placentae inappropriate treatment for a patient with a
refers to premature separation of a normally placental abruption. Patients with abruptions
implanted placenta after 20 weeks gestation, who demonstrate signs of maternal or fetal dis-
but prior to delivery of the infant. Since the de- tress and thus require emergent delivery should
tached portion of the placenta is unable to ex- be delivered by cesarean section.
change gases and nutrients, the fetus can be-
Answer F is incorrect. Internal monitoring re-
come compromised if the area of separation is
quires the rupture of membranes, which would
large. This patient is at risk for placental abrup-
be inappropriate management of this patient
tion secondary to compression-decompression
prior to delivery.
and acceleration-deceleration stresses of a mo-
tor vehicle crash. Vaginal bleeding in this set-
3. The correct answer is E. Sulfonamides, used
ting is concerning, as bleeding is one of the
to treat urinary tract infections, can displace
rst signs of abruption. It is unlikely that a
HIGH-YIELD SYSTEMS
334 Section I: Organ Systems Answers

bilirubin from albumin, leading to toxicity. cologic procedures, or incomplete spontane-


Newborns with hyperbilirubinemia are at risk ous abortions.
for neurotoxicity. At extremely elevated levels
Answer A is incorrect. While ectopic preg-
(i.e., >2530 mg/dL) these babies are at risk for
nancy may present with vaginal bleeding and
development of kernicterus. When to start pho-
abdominal or pelvic pain, the internal cervical
totherapy is controversial, though many physi-
os would be closed in an ectopic pregnancy,
cians would start phototherapy at bilirubin lev-
which contrasts with the patients presentation.
els of 1015 mg/dL.
Also, an ectopic pregnancy rarely presents with
Answer A is incorrect. Angiotensin-converting fevers and chills, symptoms more consistent
enzyme (ACE) inhibitors have been associ- with septic abortion.
ated with fetal renal defects, renal dysgenesis/
Answer B is incorrect. A pelvic abscess is a
dysplasia, renal failure, oligohydramnios, per-
complication of pelvic inammatory disease
sistent anuria following delivery, hypotension,
(PID) or other upper genital tract infection.
pulmonary hypoplasia, limb contractures sec-
While a pelvic abscess would produce high fe-
ondary to oligohydramnios, and stillbirth. ACE
vers and abdominal pain, it is unlikely to pro-
Obstetrics

inhibitors should be avoided during pregnancy,


duce vaginal bleeding. Additionally, the inter-
particularly in the second and third trimesters.
nal os would be closed.
ACE inhibitors have not been associated with
fetal jaundice. Answer D is incorrect. A threatened abor-
tion presents as pregnancy with painless vagi-
Answer B is incorrect. Lithium can cause car-
nal bleeding. The internal os is closed. Some
diac abnormalities, but has not been associated
of these women will ultimately lose the preg-
with fetal jaundice.
nancy, while a majority will continue to carry
Answer C is incorrect. Several anti-seizure the pregnancy successfully to full term.
medications are teratogenic: phenytoin can
Answer E is incorrect. A vaginal laceration
cause fetal hydantoin syndrome (growth re-
would produce vaginal bleeding and vaginal
tardation, underdeveloped nails, and mental
pain. It is unlikely to produce a fever unless it
deciency); carbamazepine is associated with
is accompanied by genital tract infection.
neural tube defects and other major malforma-
tions; valproate is associated with neural tube
5. The correct answer is A. Bacterial vaginosis is
defects; and phenobarbital can cause cardiac
caused by an imbalance in vaginal ora, with
defects and hemorrhagic disease of the new-
reduced numbers of lactobacilli and increased
born due to vitamin K depletion. However,
proportions of bacteria such as Gardnerella,
these medications have not been associated
Mobiluncus, or Peptostreptococcus species.
with jaundice.
Signs of bacterial vaginosis include a thin
Answer D is incorrect. Tretinoin (retinoic white vaginal discharge, vaginal pH >4.5, shy
acid) is a treatment for acne, and can cause odor on 10% potassium hydroxide whiff test,
severe fetal abnormalities and/or spontane- and clue cells on saline mount microscopy
ous abortion, but has not been associated with (visible in the center of the eld in the image).
jaundice. This diagnosis is established if three of these
four criteria are met.
4. The correct answer is C. This patient most
Answer B is incorrect. Patients suffering from
likely has a septic abortion. Common present-
Neisseria gonorrhoeae cervicitis often complain
ing symptoms include fever, malaise, chills,
of vaginal pruritus and discharge. However, up
abdominal or pelvic pain, and vaginal bleeding
to 50% of patients may not manifest any symp-
with or without retained products of concep-
toms at all. Diagnosis is established by identi-
tion. Septic abortions do not commonly com-
cation of N. gonorrhoeae on chocolate agar
plicate spontaneous abortions, but can occur
culture or by DNA probe testing. The potas-
as complications of illegally performed in-
sium hydroxide whiff test would be negative
duced abortions, foreign bodies, invasive gyne-
and no clue cells would be expected.
HIGH-YIELD SYSTEMS
Chapter 11: Obstetrics Answers 335

Answer C is incorrect. Similar to BV, Answer D is incorrect. Cesarean section is ap-


Trichomonas vaginalis infection may present propriate in cases of fetal distress. Early decel-
with malodorous vaginal discharge and el- erations are not a sign of fetal distress and thus
evated vaginal pH. However, Trichomonas in- normal labor and delivery should be allowed to
fection is distinguished from BV by the often continue.
yellow-green color of the vaginal discharge,
Answer E is incorrect. An amnioinfusion
and the visualization of the mobile protozoan
is used to prevent or relieve umbilical cord
with multiple agella on microscopy.
compression during labor. Cord compression,
Answer D is incorrect. Patients with can- which is manifested by repetitive variable de-
didiasis often complain of a thick vaginal dis- celerations, can compromise fetal blood ow
charge, vaginal pruritus, and/or dysuria. Signs and oxygenation.
of candidiasis include vaginal pH of 45, and
budding hyphae or spores when examined on 7. The correct answer is D. This woman is exhib-
a slide treated with 10% potassium hydroxide iting signs of subacute thyroiditis, consistent
preparation, which lyses the cells in the sample with a low thyroid-stimulating hormone (TSH)

Obstetrics
and makes the yeast easier to visualize. Clue and high thyroxine (T4). This commonly fol-
cells are not seen in candidiasis. lows a viral upper respiratory infection, and
pain from the thyroid can be referred to the
Answer E is incorrect. Vulvar candidiasis pre-
throat. Thyroid disorders are common during
sents with pruritus and redness of the exter-
pregnancy (0.5%-2% of pregnant women de-
nal genitalia. Vaginal discharge would not be
velop hyperthyroidism) and may be subtle and/
expected, vaginal pH would remain normal,
or present in the context of normal or near nor-
and clue cells would not be visualized on wet
mal thyroid hormone levels. Medical manage-
mount.
ment is the treatment of choice in pregnant
patients or in those with mild hyperthyroidism.
6. The correct answer is B. Early decelerations
are shallow, symmetric decelerations in which Answer A is incorrect. Although postpartum
the nadir of the deceleration is coincident with thyroid hormone levels will have to be ob-
the peak of the uterine contraction. They are tained, leaving this woman untreated puts her
mediated by vagal stimulation due to fetal head fetus at risk for intrauterine growth restriction
compression from the contracting uterus and (IUGR), craniosynostosis, and death.
thus indicate a normally functioning fetal auto-
Answer B is incorrect. Surgery is indicated for
nomic nervous system. They are not associated
pregnant women who cannot tolerate thioam-
with fetal hypoxia, acidosis, or poor neonatal
ides (e.g., propylthiouracil) or for those with
outcome. No further management is neces-
severe hyperthyroidism, such as those with
sary.
goiters large enough to obstruct the airway or
Answer A is incorrect. Altering the mothers esophagus, or those with a nonfunctional thy-
position (from back to side or side to back) is roid nodule that may be thyroid cancer. This
indicated to relieve cord compression. Variable patients symptoms are mild, and given the po-
decelerations, not early decelerations, are a tential risks associated with surgery, medical
sign of cord compression and thus a change in management is preferred.
this patients position is not necessary.
Answer C is incorrect. Levothyroxine is used
Answer C is incorrect. A fetal scalp probe is to treat hypothyroidism. Symptoms of hypo-
an invasive procedure in which an electrode thyroidism are weakness, fatigue, cold intoler-
is inserted transcervically into the fetuss scalp. ance, constipation, weight gain, depression,
It is indicated when external fetal heart rate menstrual irregularities, and hoarseness. On
monitoring is inadequate. Internal monitoring examination, these patients will have dry, cold,
provides an accurate beat-to-beat and baseline puffy skin; edema; thin eyebrows; bradycardia;
measurement of the fetal heart rate variability. and delayed relaxation of deep tendon reexes.
HIGH-YIELD SYSTEMS
336 Section I: Organ Systems Answers

Answer E is incorrect. Radioiodine treatment and infertility secondary to anovulation, hirsut-


leads to radioactive destruction of the majority ism, and obesity. While the ovarian cysts can
of the thyroid gland secondary to thyroid cells often cause unilateral pelvic pain, this patient
taking up the radioactive iodine. Although does not exhibit other classic signs of polycys-
commonly used for hyperthyroidism, radioiso- tic ovarian syndrome.
tope treatment is contraindicated during preg-
nancy due to the potential hazard to the fetus. 9. The correct answer is D. This woman presents
with preeclampsia, which is characterized by
8. The correct answer is B. Endometriosis is hypertension (140/90 mm Hg or greater) and
characterized by the presence of endometrial proteinuria. Nondependent edema, such as fa-
glandular and stromal tissue outside of the cial or hand edema, is usually present as well
uterine cavity or musculature. Although endo- but is not a necessary criterion. Proteinuria is
metriosis can be asymptomatic, patients com- dened as excretion of >300 mg of protein in
monly experience chronic pelvic pain that is 24 hours. The patient above is experiencing
often more severe during menses, dysmenor- both subjective as well as objective signs of se-
rhea, dyspareunia, abnormal menstrual bleed- vere disease, as evidenced by her headaches,
Obstetrics

ing, and infertility. A denitive diagnosis is visual changes, epigastric pain, and 4+ protei-
made by direct visualization of the ectopic en- nuria. The underlying pathophysiology of
dometrial tissue via laparoscopy or laparotomy, preeclampsia is vasospasm and leaky vessels,
although patients are often treated empirically but its origin is unclear. Vasospasm and en-
with oral contraceptive pills (OCPs) to avoid a dothelial leakage cause local hypoxemia of tis-
surgical procedure. sue, which can lead to hemolysis, necrosis, and
end-organ damage. It is cured only by termina-
Answer A is incorrect. An ectopic pregnancy is
tion of pregnancy and almost always resolves
a pregnancy that has implanted at a site other
after delivery. Thus, the management of the
than the endometrium of the uterine cavity.
disease will depend on the gestational age of
Pregnancy should always be considered in a
the fetus, together with the severity of the dis-
woman of reproductive age complaining of
ease. If the pregnancy is at term, then delivery
pelvic pain or vaginal bleeding. This patient,
is indicated. If the pregnancy is preterm, then
however, is not sexually active and reports a
severity of disease is assessed. If severe preec-
last menstrual period of 1 week ago.
lampsia is present, then delivery is usually indi-
Answer C is incorrect. Leiomyomata are be- cated regardless of gestational age. Administra-
nign neoplasms of smooth muscle origin that tion of magnesium sulfate serves as
commonly occur in the uterus, but can form anticonvulsant therapy and should be given
in the broad ligament. They typically occur in during labor to prevent eclampsia.
middle-aged women and cause menorrhagia,
Answer A is incorrect. Magnesium sulfate
often with resulting iron deciency anemia,
alone is not the proper course of action here
and infertility. They are generally not associ-
given the severity of preeclamptic disease. This
ated with pelvic pain or dysmenorrhea.
anticonvulsant is administered as an adjunct to
Answer D is incorrect. PID occurs as a result labor, in which it is used to prevent seizures in
of an ascending infection of the female geni- the setting of severe preeclampsia. However,
tal tract. It commonly occurs in young women it will not treat the underlying disease, which
and is caused by anaerobic and sexually trans- will progress and may result in life-threatening
mitted microorganisms. Clinical features in- complications if delivery is prolonged. Since
clude fever, chills, unilateral pelvic pain, and magnesium sulfate is excreted via the kidneys,
cervical motion tenderness. This patient re- it is vital to monitor renal function through
ports none of these symptoms. urine output, in addition to other potential
side effects or signs of toxicity such as respira-
Answer E is incorrect. Polycystic ovarian syn-
tory depression, dyspnea secondary to pulmo-
drome is a constellation of symptoms charac-
nary edema, and abolition of the deep tendon
terized by multiple ovarian cysts, amenorrhea
reexes.
HIGH-YIELD SYSTEMS
Chapter 11: Obstetrics Answers 337

Answer B is incorrect. Antihypertensive medi- Therefore it is important that hypothyroidism


cation is indicated in preeclampsia to treat the be well controlled during pregnancy.
hypertension, but it will not treat preeclamptic
Answer D is incorrect. Although pregnancy is
symptoms or prevent any feared complications
accompanied by a pseudohyperthyroidism (an
such as seizures.
increase in thyroid-binding globulin results in
Answer C is incorrect. Expectant manage- an elevated total triiodothyronine [T3] and T4),
ment is an option in premature pregnancies individuals who take T4 tend to need higher
with mild disease. These patients can then be levels of T4 during pregnancy.
monitored for worsening disease while pro-
longing the pregnancy to as close to term as 11. The correct answer is E. Curve C represents
possible to allow for fetal lung maturity. an arrest of active phase that best ts the case
presented here. It is dened as arrest of cervi-
Answer E is incorrect. Although complications
cal dilation when a patient has entered the ac-
of preeclampsia include placental abruption,
tive phase of labor and is depicted as a atten-
eclampsia, coagulopathies, hepatic subscapular
ing of the labor curve. Common causes
hematoma, hepatic rupture, renal failure, and

Obstetrics
include maternal factors (inadequate uterine
uteroplacental insufciency, there are no indi-
contractions, small pelvic diameter, or abnor-
cations present to warrant the administration
mally shaped pelvis) and fetal factors (malposi-
of a platelet transfusion. HELLP syndrome
tion and macrosomia).
(Hemolysis, Elevated Liver enzymes, and Low
Platelets) is a feared complication of severe Answer A is incorrect. Curve A represents
preeclampsia and may warrant a platelet trans- a prolonged latent phase. An unripe cervix is
fusion in addition to other therapies, of which not as easily softened and effaced. Thus, the
delivery is again the best treatment. presenting head may not progress despite ad-
equate uterine contractions.
10. The correct answer is E. During pregnancy
Answer B is incorrect. Curve A represents a
there is an increase in circulating thyroid-binding
prolonged latent phase. A primigravid patient
globulin as a result of estrogen stimulation of he-
is more likely to have a longer latent phase (av-
patic enzymes. As a result, pregnant women will
erage 6.4 hours, >20 hours abnormal) than a
often need to increase T4 supplementation dur-
multiparous woman (average 4.8 hours, >14
ing their pregnancy. Maternal thyroid hormones
hours abnormal).
are important for fetal neuron development and
studies have shown that children whose mothers Answer C is incorrect. Curve B represents a
had hypothyroidism during pregnancy had lower prolonged active phase. Inadequate uterine
IQs and other developmental problems. There- contraction is a common cause of a prolonged
fore it is important that hypothyroidism be well active phase, but in the case presented, there
controlled during pregnancy. is an arrest, rather than a prolongation, of the
active phase.
Answer A is incorrect. Methimazole is used in
treating hyperthyroidism and is not considered Answer D is incorrect. Curve B represents a
safe in pregnancy, as it causes fetal aplasia cutis prolonged active phase. A common cause of
congenita. a prolonged active phase is excessive maternal
sedation, not inadequate sedation.
Answer B is incorrect. Maternal thyroid hor-
mones are important for fetal neuron develop- Answer F is incorrect. Curve C represents an
ment. Therefore it is important that hypothy- arrest of active phase. Occiput anterior is the
roidism be well controlled during pregnancy. ideal position of the fetus in vertex presenta-
tion and would not be a cause of arrest of the
Answer C is incorrect. T4 is considered safe
active phase.
during pregnancy. Maternal thyroid hormones
are important for fetal neuron development. 12. The correct answer is C. This patient is suffer-
ing from postpartum endometritis, with risk fac-
HIGH-YIELD SYSTEMS
338 Section I: Organ Systems Answers

tors including nonspontaneous rupture of mem- thyroidism (low TSH and T4) and will likely
branes, internal fetal monitoring, and cesarean need levothyroxine treatment. However, she is
section. Postpartum endometritis is usually a likely not ovulating secondary to her pituitary
polymicrobial infection caused by anaerobes dysfunction (low follicle-stimulating hormone
and aerobes from the genital tract. Endometrial [FSH] and luteinizing hormone [LH]). In
cultures are generally not helpful in identifying cases such as these, ovulation often needs to be
organisms, as infection may result from organ- induced with an agent such as clomiphene.
isms that typically colonize the genital tract,
Answer B is incorrect. Levothyroxine should
such as group B streptococci (GBS), as well as
be part of this womans prescriptive regimen to
from organisms introduced via instrumentation.
combat her hypothyroidism. However, she will
Management of these patients involves sending
likely need direct ovulation stimulation to be-
blood cultures and subsequently treating with
come pregnant.
parenteral broad-spectrum antibiotics, including
coverage for -lactamase-producing microbes. Answer C is incorrect. Prednisone in addition
The gold standard treatment is intravenous clin- to the levothyroxine would be used to treat this
damycin and gentamicin. womans hypothyroidism during pregnancy,
Obstetrics

but it would not help her conceive.


Answer A is incorrect. This combination does
not provide adequate coverage for -lactamase- Answer D is incorrect. Progesterone is used
producing organisms. Ampicillin may be as an exogenously administered hormone to
combined with sulbactam and used as an al- maintain the corpus luteum and induce the
ternative to the clindamycin/gentamicin com- secretory phase of the endometrium for im-
bination. plantation, but it would not lead to conception
in a woman who is not ovulating.
Answer B is incorrect. This combination does
not provide adequate anaerobic coverage or Answer E is incorrect. Propylthiouracil is used
coverage of -lactamase-producing organisms. to medically manage hyperthyroidism, not hy-
pothyroidism.
Answer D is incorrect. Imipenem is active
against aerobes, anaerobes, and -lactamase-
14. The correct answer is C. Hyperemesis gravi-
producing organisms. However, it is not the ap- darum is dened as persistent severe vomiting
propriate rst-line drug for such an infection, during pregnancy and is principally a diagnosis
but may be used if the patient does not respond of exclusion. It can lead to severe dehydration,
clinically to less potent antibiotics. Additionally, hypochloremic alkalosis, hypokalemia, and a
imipenem enters breast milk and thus must be transient elevation in liver enzymes. It is more
used with caution in breast-feeding mothers. common in multiple pregnancies, as in this
Answer E is incorrect. Metronidazole may scenario, and is believed to be due to increas-
be used with ceftriaxone or levooxacin as an ing -human chorionic gonadotropin ( -hCG)
alternate therapy; however, it provides inap- levels.
propriate coverage when combined with doxy- Answer A is incorrect. Although viral hepati-
cycline. Furthermore, doxycycline is contrain- tis A does cause nausea and vomiting and an
dicated in nursing mothers because it causes increase in liver enzymes, it is also associated
tooth discoloration and enamel hypoplasia dur- with malaise, fatigue, low-grade fevers, and
ing tooth formation in infants. jaundice. Given this womans history of a mul-
tiple gestation pregnancy with accompanying
13. The correct answer is A. This patient is likely
morning sickness in the absence of any other
suffering from postpartum pituitary necrosis
signs or symptoms, the most likely diagnosis is
(Sheehans syndrome), which is believed to be
hyperemesis gravidarum.
a consequence of reduced blood ow to the pi-
tuitary during delivery or the postpartum pe- Answer B is incorrect. Salmonella food poi-
riod. This woman is exhibiting signs of hypo- soning usually causes vomiting in the context
of diarrhea and fever.
HIGH-YIELD SYSTEMS
Chapter 11: Obstetrics Answers 339

Answer D is incorrect. Although preeclamp- cessively large. Patients with FAS experience
sia is more common in multiple gestations, it IUGR.
is rare before 20 weeks gestation. In addition,
Answer E is incorrect. Excess amniotic uid
patients with preeclampsia rarely have isolated
may be a result of fetal anomalies including
acute nausea and vomiting. Other signs and
duodenal atresia, tracheoesophageal stula,
symptoms such as severe hypertension, edema,
or anencephaly. These anomalies prevent the
proteinuria, sudden weight gain, headache,
fetus from ingesting amniotic uid. FAS does
and vision changes usually accompany the gas-
not include these anomalies.
trointestinal distress.
Answer E is incorrect. Viral gastroenteritis can 16. The correct answer is E. The two most impor-
cause nausea and vomiting, but given a lack of tant factors that affect thyroid physiology in the
bowel changes or other symptoms, this wom- pregnant patient are estrogen and -hCG.
ans history is more consistent with hyperem- Early in pregnancy, when -hCG levels peak
esis gravidarum. at 1012 weeks, patients can have subclinical
hyperthyroidism because -hCG is a weak

Obstetrics
15. The correct answer is D. The prevalence of fe- stimulator of the TSH receptor and this stimu-
tal alcohol syndrome (FAS) among moderate lation causes excess production of thyroid hor-
to heavy drinkers and alcoholics is 10%50%. mones and a subsequent decline in TSH due
The diagnosis is based upon three criteria: to negative feedback on the pituitary, as is seen
IUGR (conrmed pre- or postnatal weight or in this patient. This transient subclinical hy-
height at or above 10th percentile), facial dys- perthyroidism occurs in 10%20% of patients
morphia (smooth philtrum, thin vermilion bor- and usually does not require therapy. Rarely,
der, short palpebral ssures, hypoplastic mid- high levels of -hCG can contribute to hyper-
face, and microcephaly), and central nervous emesis gravidarum, a syndrome consisting of
system abnormalities (structural or functional nausea, vomiting, and weight loss. In addition,
abnormalities, including mental restriction). patients with hydatidiform mole or choriocar-
FAS patients also have an increased risk of car- cinoma can have severe hyperthyroidism.
diac defects.
Answer A is incorrect. Acute infectious thyroidi-
Answer A is incorrect. Preeclampsia is dened tis presents with fever, chills, and acute onset of
as new-onset hypertension, proteinuria, and/or extreme pain in the thyroid. Many patients will
nondependent edema occurring at >20 weeks have a unilateral neck mass, which may be uc-
gestation. Eclampsia is seizures in a patient with tuant. Thyroid function is typically normal in
preeclampsia. Risk factors include nulliparity, patients with acute infectious thyroiditis.
black race, extremes of age, multiple gestation,
Answer B is incorrect. Graves disease is the
molar pregnancy, renal disease, family history of
most common cause of hyperthyroidism in
preeclampsia, and chronic hypertension. Preec-
pregnant as well as nonpregnant patients. Pa-
lampsia is not associated with FAS.
tients will present with signs and symptoms of
Answer B is incorrect. Pulmonary hypoplasia hyperthyroidism including fatigue, heat intol-
occurs secondary to severe reduction in amni- erance, anxiety, diarrhea, and palpitations. Pa-
otic uid and is often related to kidney dysgen- tients will typically have TSH levels <0.01 U/
esis or dysfunction, not FAS. mL and the serum will be positive for TSH-re-
ceptor antibodies.
Answer C is incorrect. Macrosomia is dened
as fetal weight >4.0 kg, or birth weight >90th Answer C is incorrect. Hashimotos disease is
percentile for gestational age. Maternal gesta- a common cause of hypothyroidism. These pa-
tional diabetes mellitus is a risk factor for mac- tients will have decreased levels of free T3 and
rosomia. It is thought that the fetal pancreas T4 and elevated TSH. Hypothyroidism is rare
produces excess insulin as a result of increased in pregnant patients because it can cause ano-
maternal blood glucose. This results in large vulation. In addition, hypothyroidism can cause
fat deposits which cause the fetus to grow ex- spontaneous abortion in the rst trimester.
HIGH-YIELD SYSTEMS
340 Section I: Organ Systems Answers

Answer D is incorrect. Elevated serum estro- testing, it is prudent to proceed as if the fetus
gen levels also cause a major change in thyroid were Rh positive. Thus, the mother should re-
function. Estrogen increases production as well ceive Rh0(D) immune globulin to prevent pos-
as decreases clearance of thyroid-binding glob- sible isoimmunization to the Rh antigen.
ulin. This causes an increase in total T4, but
Answer B is incorrect. Amniocentesis to mea-
the free T3 and T4 remain normal. The patient
sure bilirubin levels is a method of measuring
is clinically euthyroid and the TSH is normal.
the degree to which an Rh-positive fetus, car-
These abnormalities will typically be found in
ried by an Rh-sensitized mother, is affected by
pregnant patients. However, this patients nd-
hemolysis. Because the mother is not sensi-
ings of high free T3 and T4 as well as decreased
tized, any risk of isoimmunization would be to
TSH suggest a subclinical hyperthyroidism due
subsequent pregnancies, not the current one.
to the effects of -hCG.
Therefore, amniocentesis to ascertain the am-
niotic uid bilirubin level is not indicated.
17. The correct answer is D. A deceleration is a
slowing of the fetal heart rate, usually lasting Answer C is incorrect. Both the mother and
1015 seconds. Early decelerations begin and fetus are stable after the accident. At only 27
Obstetrics

end with contractions. They are a reassuring weeks, the fetus is still several weeks away from
nding and reect an increase in vagal tone as achieving full lung maturity, and an emergent
a result of uterine contractions exerting pres- cesarean section would put the infant at sig-
sure on the fetal head as it descends into the nicant risk for respiratory distress syndrome.
vaginal canal. In the absence of any maternal or fetal distress,
emergent cesarean section is not indicated at
Answer A is incorrect. The pattern shown on
this time.
the tracing is that of early decelerations and a
reassuring pattern and reects normal mater- Answer D is incorrect. There are no signs of
nal-fetal physiology, not artifact. maternal or fetal distress that would warrant
premature delivery of this infant. Thus, expec-
Answer B is incorrect. Compression of the
tant management for a term delivery with em-
umbilical cord leads to variable decelerations
phasis on prevention of Rh isoimmunization
and constitutes a nonreassuring tracing. This
of the mother is a more prudent course in this
tracing would show a sharp fall in fetal heart
clinical situation.
rate followed by a variable recovery time. They
do not display the consistent association with Answer E is incorrect. Both the mother and
contractions that are seen in the early decel- the fetus appear to be stable after the automo-
erations seen in the case above. bile accident, and there are no signs of im-
pending labor. Thus, the mother can be expec-
Answer C is incorrect. Fetal arrhythmia is not
tantly managed for a delivery at term. There is
associated with early decelerations.
no need to administer betamethasone to accel-
Answer E is incorrect. Fetal hypoxia from fe- erate maturation of the fetal lungs at this time.
tal anemia can result in a regular and smooth-
appearing sinusoidal wave pattern. This tracing 19. The correct answer is F. Varicella vaccine is a
is ominous and associated with high fetal mor- live attenuated vaccine. Women who have not
bidity and mortality. contracted chickenpox as children should be
vaccinated prior to considering pregnancy.
18. The correct answer is A. The mother has been
Answer A is incorrect. Two inactivated forms of
exposed to fetal blood cells, as evidenced by
the hepatitis A vaccine are available. Although
the Kleihauer-Betke test. This test uses the in-
limited data exist on the safety of these vaccines
creased resistance of fetal hemoglobin to elu-
during pregnancy, the vaccine is presumed to
tion by acid as compared to adult hemoglobin.
be safe like other inactivated vaccines. None-
Because the Rh antigen is inherited in an auto-
theless, it should only be administered to high-
somal dominant fashion, the fetus would be
risk women, such as those traveling to areas in
Rh negative if the father were also Rh negative.
which hepatitis A is endemic.
However, because the father is unavailable for
HIGH-YIELD SYSTEMS
Chapter 11: Obstetrics Answers 341

Answer B is incorrect. Hepatitis B vaccine is a rheic after two more months, she will require
recombinant vaccine and has not been shown a complete evaluation for amenorrhea. At this
to be harmful to the fetus or newborn. Hepa- time, however, she does not.
titis B vaccine is indicated in pregnant women
Answer D is incorrect. A hysterosalpingogram
who are completing an immunization series
is performed to determine the patency of the
that had begun prior to conception and to
fallopian tubes in a woman experiencing in-
nonimmunized women who are at high risk of
fertility. Women at high risk for tubal scar-
contracting hepatitis B.
ring include women with a history of sexually
Answer C is incorrect. The inuenza virus transmitted diseases (STDs), particularly prior
vaccine is an inactivated vaccine and should chlamydial infection. This patient does not
be administered to all pregnant women in Oc- need hysterosalpingography at this time, as she
tober to mid-November (prior to inuenza sea- does not meet the criteria for infertility (failure
son) regardless of gestational age of the fetus. to conceive after 12 months of frequent inter-
course without contraception) and she denies
Answer D is incorrect. The tetanus vaccine is
a history of STDs.
a modied bacterial toxoid; thus the toxoid is

Obstetrics
rendered inactive. It is safe during pregnancy Answer E is incorrect. A pelvic ultrasound is
and is indicated in any woman who has not useful in patients who are or may be pregnant,
completed her three-dose primary immuniza- or where there is concern for uterine or ovar-
tion or has not had a booster dose in 10 years. ian pathology (i.e., cysts or masses). It is not in-
dicated in this patient.
Answer E is incorrect. The typhoid vaccine
is a capsular polysaccharide vaccine that must
21. The correct answer is C. This patient is expe-
be administered parenterally. It is not a com-
riencing a prolonged latent phase of labor. Op-
monly administered vaccine but can be given
tions for managing a prolonged latent phase
to patients traveling to endemic areas.
include rest or augmentation of labor. Aug-
mentation of labor is often effected with oxyto-
20. The correct answer is C. Amenorrhea for 35
cin intravenous infusions and articial rupture
months following the discontinuation of OCPs
of membranes if they have not already sponta-
is common and should not cause concern. Pa-
neously ruptured. In this case of prolonged la-
tients whose menses do not return more than 5
bor in which the mother has a cervix favorable
months after discontinuation of OCPs, particu-
for delivery and has had spontaneous rupture
larly if they were taking low-dose OCPs, re-
of membranes, an oxytocin infusion is a rea-
quire a standard evaluation for amenorrhea.
sonable next step.
Answer A is incorrect. A progesterone chal-
Answer A is incorrect. Intravaginal prostaglan-
lenge is given to women with amenorrhea to
din E2 is used to induce labor when the patient
evaluate for ovarian estradiol production and
has a cervix that is unfavorable for oxytocin in-
the presence of a normal outow tract. This pa-
duction. Prostaglandin E2 is not used once la-
tient may require such a challenge if she does
bor is in progress, and carries a risk of uterine
not get her period within the next few months,
hyperstimulation, placental insufciency, and
but it is inappropriate at this point.
uterine rupture.
Answer B is incorrect. FSH and LH levels are
Answer B is incorrect. Forceps delivery may
checked as part of the evaluation of a woman
be used to facilitate delivery in cases in which
with amenorrhea. High FSH and LH levels
the power of uterine contractions alone is in-
indicate a lack of steroid hormone synthesis
sufcient to deliver the infant. However, pre-
or anatomic abnormality of the ovarian tissue,
requisites for forceps delivery include a fully
whereas low FSH and LH levels suggest pitu-
dilated cervix and fully engaged fetal head. For
itary dysfunction. If this patient is still amenor-
most forceps delivery techniques the fetal head
HIGH-YIELD SYSTEMS
342 Section I: Organ Systems Answers

station must be at least +2. This patient, who is Therefore, smoking is a possible cause of the
still in the latent rst stage of labor, has not yet abortion, but it is not the most likely cause.
progressed to a point where forceps are likely
Answer E is incorrect. The uterus at an early
to facilitate delivery of the infant.
gestational age is protected from blunt trauma
Answer D is incorrect. Hydration is often due to its small size. However, trauma from a
used to forestall premature labor. ADH is gynecologic or obstetric procedure can induce
manufactured in the hypothalamus along a miscarriage. This patient does not report any
with oxytocin, and is hypothesized to exhibit such history.
some cross-reactivity with oxytocin. Decrease
of ADH levels by hydration leads to lower lev- 23. The correct answer is E. Mild late decelera-
els of stimulation of oxytocin receptors, and tions are due to a central nervous system reex
thus may decrease the strength of contrac- response to fetal hypoxia. Hypoxia generally re-
tions. Therefore hydration would not facili- sults from uteroplacental insufciency, which
tate delivery, and may decrease the strength is commonly a sign of placental abruption or
of uterine contractions, prolonging the labor hypotension. Placental abruption tends to pre-
Obstetrics

process. sent with painful, dark vaginal bleeding that


does not cease spontaneously. In this case, with
Answer E is incorrect. While the progression
a history of maternal hypertension, the patient
to active labor has been prolonged, the fetal
is likely having a placental abruption and may
heart rate is reassuring of the fetus well-being.
require emergent cesarean section.
Thus, vaginal delivery may be allowed to pro-
ceed. If at any point the fetal heart rate pattern Answer A is incorrect. Fetal head compression
becomes nonreassuring and vaginal delivery is is associated with early decelerations, which
not imminent, a cesarean section would be in- begin and end at approximately the same time
dicated. as the maternal contraction. The tracing above
shows late decelerations.
22. The correct answer is B. Chromosomal abnor-
Answer B is incorrect. Fetal myocardial de-
malities, most commonly aneuploidy, account
pression is manifested by severe late decel-
for approximately 50% of all miscarriages, par-
erations, and usually with absent variability.
ticularly those occurring prior to 12 weeks ges-
Myocardial depression is associated with severe
tation.
acidosis and perinatal morbidity and mortality.
Answer A is incorrect. Acute maternal infec- The patient in this question is not exhibiting
tion, particularly toxoplasmosis, herpes sim- signs of severe fetal compromise.
plex, or rubella can increase the chances of
Answer C is incorrect. Early, not late, deceler-
early pregnancy loss. However, it is not the
ations are secondary to vagal stimulation due to
most common cause of miscarriage.
fetal head compression. In early decelerations,
Answer C is incorrect. Exposure to certain en- the nadir of the heart rate deceleration occurs
vironmental chemicals can also increase the simultaneously with the peak of the uterine
chances of pregnancy loss. However, this is not contraction.
the most common etiology.
Answer D is incorrect. Placenta previa predis-
Answer D is incorrect. Heavy maternal smok- poses a patient to peripartum hemorrhage and
ing is associated with an increase in pregnancy placental abruption, both of which may con-
loss; however, the rate of increase of sponta- tribute to maternal hypotension and fetal hy-
neous abortions associated with smoking (ap- poxia. In the absence of these clinical ndings,
proximately 24%) is much less signicant than placenta previa in and of itself does not cause
the number of spontaneous abortions associ- late decelerations or fetal distress. Placenta
ated with chromosomal abnormalities (>50%). previa presents with painless, bright red bleed-
ing that often ceases in 12 hours.
HIGH-YIELD SYSTEMS
Chapter 11: Obstetrics Answers 343

24. The correct answer is B. This woman has an Doppler echocardiography to screen for the
increased risk of developing gestational diabetes development of heart block.
in this pregnancy, given her history of gesta-
Answer A is incorrect. Babies born to mothers
tional diabetes in a previous pregnancy. All
with SLE are not at increased risk of develop-
women with increased risk should be screened
ing renal abnormalities or acute renal failure.
at 2428 weeks gestation with a random 1-hour
However, women with baseline lupus nephritis
50-g oral glucose challenge. If positive, screen-
are at increased risk of developing progressive
ing should follow with a 100-g 3-hour glucose
renal disease during pregnancy due to an in-
tolerance test. If this test is positive, the woman
creased burden placed on maternal kidneys.
is diagnosed with gestational diabetes. Treat-
ment should initially begin with a trial of diet Answer B is incorrect. Chorioretinitis is typi-
modication. If the patient fails to meet target cally seen in babies with congenital infections
blood sugar levels, insulin therapy should be such as toxoplasmosis and cytomegalovirus.
started. Toxoplasmosis also presents with hydrocepha-
lus and intracranial calcications. Cytomega-
Answer A is incorrect. A 3-hour glucose toler-
lovirus infection presents with microcephaly,

Obstetrics
ance test is indicated only after an initial 1-hour
hepatosplenomegaly, respiratory distress, and
50-g oral glucose challenge.
seizures.
Answer C is incorrect. Insulin therapy is not
Answer D is incorrect. Ebsteins anomaly is a
appropriate at this point. This patient does not
congenital cardiac defect consisting of malfor-
yet have a diagnosis of gestational diabetes.
mation of the tricuspid valve and right ventri-
Answer D is incorrect. While patients with ges- cle. It occurs more frequently in infants born to
tational diabetes will have glycosuria, initiation mothers who took lithium during pregnancy. It
of insulin therapy is not determined by the pres- is not associated with maternal SLE.
ence or absence of glucose in the urine. Insulin
Answer E is incorrect. Most babies with neo-
therapy is initiated in a patient with gestational
natal lupus have a rash, usually consisting of
diabetes who fails to meet target blood glucose
erythematous annular lesions on the scalp and
levels with diet and exercise alone.
periorbital area. However, the rash is typically
Answer E is incorrect. All of the oral hypogly- mild and resolves by 68 months of age.
cemic agents, including metformin, are con-
traindicated in pregnancy, as they can cause fe- 26. The correct answer is C. The patients infertil-
tal hypoglycemia. Only insulin therapy should ity and dysmenorrhea are most likely secondary
be administered to women with diabetes during to endometriosis, and the pelvic mass is most
pregnancy. likely an endometrioma, or chocolate cyst. Pa-
tients with endometriosis often experience
25. The correct answer is C. Neonatal lupus is chronic pelvic pain that is more severe during
caused by passive transfer of anti-Ro/SSA and/ menses, dysmenorrhea, dyspareunia, abnormal
or anti-La/SSB antibodies. The disorder can menstrual bleeding, and infertility. The optimal
occur in babies born to mothers with lupus, method of diagnosis is direct visualization via
Sjgrens syndrome, or in women with no prior laparoscopy or laparotomy, following initial im-
history of autoimmune disease. Not all women aging with ultrasound. On ultrasound, an endo-
with systemic lupus eythematosus (SLE) have metrioma appears as a complex mass, with vary-
these autoantibodies, so screening should be ing areas of hypodense and hyperdense lesions.
done at the beginning of prenatal care to deter-
Answer A is incorrect. A corpus luteum cyst
mine the risk of neonatal lupus and the need
is a functional ovarian cyst that occurs af-
for monitoring. The most serious complication
ter an egg has been released from a follicle.
of neonatal lupus is complete heart block.
Usually the follicle (now the corpus luteum)
Neonatal lupus is the most common cause of
breaks down if no pregnancy occurs, but it
congenital complete heart block. Fetuses at
can become lled with blood or uid, remain-
risk of neonatal lupus should have regular
ing in the ovary. This patients adnexal mass
HIGH-YIELD SYSTEMS
344 Section I: Organ Systems Answers

is unlikely to be a corpus luteum cyst, as the this patient blood transfusion is not indicated
ultrasound was performed at week two of her at this time.
menstrual cycle, around the time of ovulation.
Answer C is incorrect. The mother is Rh-
Corpus luteum cysts tend to appear toward the
positive. RhO(D) immune globulin is admin-
end of the menstrual cycle.
istered in the case of an Rh-negative mother
Answer B is incorrect. This patient does not and known or suspected Rh-positive fetus to
describe typical symptoms of ectopic preg- prevent isoimmunization and its complications
nancy, namely unilateral pelvic pain, vaginal such as fetal anemia and high-output cardiac
spotting, and amenorrhea. She also does not failure. Since the mother is Rh-positive, isoim-
report risk factors for ectopic pregnancy, such munization cannot occur.
as a history of STDs, prior pelvic surgery, or
Answer D is incorrect. Delivery in the case of
previous ectopic pregnancy.
placenta previa should be by cesarean section.
Answer D is incorrect. Leiomyomata are be- However, most sentinel bleeds resulting from
nign neoplasms of smooth muscle origin that placenta previa will cease spontaneously, and
commonly occur in the uterus, but can also thus are managed expectantly to allow for ad-
Obstetrics

form in the broad ligament. They generally ministration of betamethasone and further fe-
occur in women 3040 years old and are a tal development.
common cause of infertility. They tend not to
Answer E is incorrect. Rebleeding occurs in
cause pelvic pain or dysmenorrhea. While this
up to 70% of patients with placenta previa af-
patient could have a leiomyoma, given her his-
ter the sentinel bleed. Thus, the patient should
tory, her symptoms are more likely due to en-
be hospitalized on bed rest and preparations
dometriosis.
should be taken for life-threatening hemor-
Answer E is incorrect. A tubo-ovarian abscess is rhage and emergent cesarean section.
part of the spectrum of PID and occurs as a re-
sult of an ascending infection of the female gen- 28. The correct answer is D. Preeclampsia refers
ital tract. The most likely causes of tubo-ovarian to the new onset of hypertension and proteinu-
abscess are anaerobic and sexually transmitted ria (>0.3 g in a 24-hour urine specimen) after
microorganisms. Clinical features include fe- 20 weeks gestation in a previously normoten-
ver, chills, unilateral pelvic pain, and cervical sive woman. Eclampsia refers to the occur-
motion tenderness. This patient reports none of rence of one or more generalized convulsions
these symptoms. and/or coma in the setting of preeclampsia. In
mild preeclampsia, patients do not have any
27. The correct answer is A. Patients with pla- features of severe disease, such as central ner-
centa previa are often managed expectantly vous system symptoms, thrombocytopenia, or
unless there is fetal distress, persistent labor, or pulmonary edema. Many newly-diagnosed pa-
life-threatening hemorrhage. However, up to tients need close maternal monitoring to estab-
70% of patients experience a recurrence of lish disease severity and rate of progression,
bleeding and require delivery before fetal lung and to monitor for progression to eclampsia,
maturity has been achieved. Thus, preparation hypertensive crisis, abruptio placentae, or
for preterm delivery with betamethasone to HELLP syndrome (Hemolysis, Elevated Liver
promote fetal lung maturity is indicated. The enzymes, and Low Platelet count). However,
child should be delivered by cesarean section these complications are uncommon in women
at 36 weeks, or earlier if life-threatening hem- >32 weeks of gestation with mild hypertension
orrhage occurs. (<150/100 mm Hg), minimal proteinuria (e.g.,
<1 g/24 hours), and no other abnormalities.
Answer B is incorrect. At this time the bleed-
Therefore patients with these ndings may be
ing from the placenta previa has ceased, the
managed on an ambulatory basis after initial
mother has a hemoglobin level of 11.1 g/dL,
inpatient evaluation, with frequent maternal
and the fetus has a reassuring heart tone. In
and fetal evaluations. Restricted activity is typi-
HIGH-YIELD SYSTEMS
Chapter 11: Obstetrics Answers 345

cally recommended; there is no evidence that indicative of cord compression during labor.
complete bed rest has any benet. Antihyper- It is also used as a dilutional method in labors
tensive medications are only recommended in that are complicated by thick meconium-
patients with a systolic blood pressure >150 stained amniotic uid. It is not currently used
mm Hg or a diastolic blood pressure >100 mm in cases of oligohydramnios when there is no
Hg. evidence of fetal distress.
Answer A is incorrect. Diuretics are not rec- Answer D is incorrect. Oligohydramnios is
ommended during pregnancy due to their often caused by uteroplacental insufciency,
possible teratogenic effects. Furthermore, furo- placental abruption or infarction, congenital
semide is an ineffective antihypertensive agent abnormalities, IUGR, fetal demise, or ruptured
in patients who are not volume overloaded. membranes. While oligohydramnios may be
indicative of some underlying pathology, the
Answer B is incorrect. Delivery of the fetus is
oligohydramnios itself does not warrant emer-
indicated in a woman with severe preeclamp-
gent surgical delivery.
sia or eclampsia.
Answer E is incorrect. Since oligohydramnios

Obstetrics
Answer C is incorrect. ACE inhibitors are
in the third trimester is often associated with
contraindicated in pregnancy. They cause oli-
uteroplacental insufciency, it is prudent to
gohydramnios and renal damage to the fetus.
monitor the fetus at regular intervals for signs
Answer E is incorrect. Hydralazine and la- of distress. Biweekly biophysical prole testing
betalol are the two antihypertensive medica- is a prudent addition to this patients care.
tions recommended in pregnancy. Medical
therapy, however, is not indicated in patients 30. The correct answer is B. His Apgar scores are
with systolic blood pressures <150 mm Hg or 3 and 8 at 1 and 5 minutes, respectively. Apgar
diastolic blood pressures <100 mm Hg, as is scoring is a standardized method of summariz-
the case with this patient. ing an infants overall condition at birth and
predicting which infants are going to require
29. The correct answer is C. Oligohydramnios in resuscitation. Apgar scores of 0 to 3 indicate
this patient is most likely a reection of the the need for immediate resuscitation, either
uteroplacental insufciency caused by the bag-mask ventilation or intubation. Apgar
mild preeclampsia. While oligohydramnios it- scores of 810 indicate good cardiopulmonary
self poses some risk to the fetus through an in- adaptation. Apgar scores of 47 indicate possi-
crease in prevalence of cord compression, it ble need for resuscitation.
does not require immediate delivery if there is
Answer A is incorrect. An Apgar score of 8
no fetal or maternal distress. A prudent course
does not indicate a need for resuscitation.
of action would include regular monitoring
with biophysical prole testing to ensure fetal Answer C is incorrect. An Apgar score of 8
well-being and induction of labor when fetal does not indicate a need for resuscitation; how-
lung maturity has been achieved. ever, a score of 3 indicates the need for imme-
diate resuscitation.
Answer A is incorrect. In cases of oligohy-
dramnios without fetal distress, a trial of labor Answer D is incorrect. The information nec-
once fetal maturity has been achieved is not essary to determine Apgar scores and the need
contraindicated. While planned cesarean sec- for resuscitation are provided.
tion in this case would be much preferable to
Answer E is incorrect. An Apgar score of 3 in-
an emergent cesarean section, there is no need
dicates the need for immediate resuscitation.
to forego a trial of labor in this case.
Answer B is incorrect. Amnioinfusion is often 31. The correct answer is A. Causes of symmetric
used to treat fetuses that are experiencing se- IUGR include in utero infection, multiple
vere, repeated variable heart rate decelerations pregnancies, and maternal disease. Asymmetric
IUGR is more often due to uteroplacental in-
HIGH-YIELD SYSTEMS
346 Section I: Organ Systems Answers

sufciency from maternal hypertension, smok- uterine bleeding, endometrial carcinoma must
ing, and poor nutrition. Since this fetus dem- always be ruled out in that age group. This pa-
onstrates symmetric IUGR, the most likely tient is at a much lower risk for endometrial
cause of the IUGR is an early in-utero infec- cancer given her age and, therefore, an endo-
tion. metrial biopsy is inappropriate at this point.
Answer B is incorrect. Inadequate weight gain Answer B is incorrect. Coagulopathies are not
inhibits growth by cellular hyperplasia, and a common cause of uterine bleeding but can
causes asymmetric IUGR. However, because be considered after other more frequent possi-
nutritional deciency only affects cellular hy- bilities have been ruled out.
perplasia rather than division, it is more ame-
Answer C is incorrect. Hypothyroidism can
nable to reversal if the nutritional decit is cor-
cause heavy uterine bleeding and can be part
rected.
of the work-up; however, it is not the rst step
Answer C is incorrect. Maternal hypertension, in the diagnostic evaluation of this patient.
like smoking, causes IUGR through utero-
Answer E is incorrect. The most common
placental insufciency. This is likely to cause
Obstetrics

cause of vaginal bleeding in this patients age


asymmetric IUGR rather than the symmetric
group is anovulatory bleeding. The administra-
IUGR demonstrated in this child.
tion of a progesterone challenge is important in
Answer D is incorrect. Maternal smoking af- determining the etiology of anovulatory bleed-
fects fetal growth by causing a relative utero- ing because a negative result indicates insuf-
placental insufciency. The fetal nutritional cient estrogen production, whereas a positive
requirements exceed the placentas ability to result indicates adequate endogenous estrogen
provide nutrients, especially later in pregnancy. production. However, urine or serum -hCG
Thus smoking is more likely to cause asymmet- measurement is the rst step in evaluating this
ric IUGR. patient to rule out pregnancy.
Answer E is incorrect. Singleton pregnancy Answer F is incorrect. Given the history of obe-
has not been shown to be a risk factor for sity, hirsutism, and irregular menses, polycystic
IUGR. Rather, multiple gestations can lead to ovarian disease is high on the differential diag-
IUGR through decreased relative amount of nosis. An ultrasound of the ovaries is therefore
nutrients available to each fetus. appropriate to evaluate for ovarian cysts; how-
ever, it is not the rst step in evaluation of this
32. The correct answer is D. Pregnancy (intrauter- patient because pregnancy should be ruled out
ine or ectopic) must always be ruled out in a rst.
woman of childbearing age with abnormal
uterine bleeding. Ectopic pregnancy is a diag- 33. The correct answer is C. Carbamazepine is
nosis that cannot be missed because it can re- teratogenic, causing neural tube defects, n-
sult in maternal death, with shock secondary to gernail hypoplasia, microcephaly, and intra-
tubal rupture. Although some patients with ec- uterine growth retardation. The presentation
topic pregnancy present with hemodynamic of neural tube defects (spina bida and anen-
instability, more than 50% of patients are as- cephaly) varies based on the extent and loca-
ymptomatic before tubal rupture and do not tion of the lesion. This childs lesion is consis-
have an identiable risk factor for ectopic preg- tent with a sacral meningocele. Neural tube
nancy. Patients may simply present with abnor- defects cause an elevated -fetoprotein level
mal vaginal bleeding and abdominal pain. on triple screen.
Answer A is incorrect. Endometrial biopsy Answer A is incorrect. Anoxia secondary to
should be performed in all women older than maternal seizure is not teratogenic but may
35 years who have abnormal uterine bleeding. result in fetal demise, growth retardation, and
Although not a common cause of abnormal brain damage.
HIGH-YIELD SYSTEMS
Chapter 11: Obstetrics Answers 347

Answer B is incorrect. Symptoms of FAS are fa- ter has broken as a preventive measure against
cial abnormalities (macrognathia, thin upper lip, chorioamnionitis. In a patient who is unsure
short palpebral ssure, epicanthal fold, and thin of the status of her membranes at the time of
upper lip), poor growth (small head circumfer- presentation, it would be prudent to perform a
ence, short length, and low weight), cardiac de- speculum examination of the cervix.
fects, minor joint and limb abnormalities, and
Answer C is incorrect. External tocometers
developmental delay/mental retardation.
are useful for determining timing and dura-
Answer D is incorrect. Trisomy 18, or Ed- tion of contractions, but are unable to measure
wards syndrome, classically presents with the strength of uterine contractions. Therefore
rocker-bottom feet, low-set ears, micrognathia, they are of limited use in the diagnosis of pre-
prominent occiput, clenched hands, and con- term labor.
genital heart disease. This syndrome causes a
Answer E is incorrect. Ultrasound examina-
low maternal serum -fetoprotein, low estriol,
tion can determine fetal viability and gesta-
and low -hCG on triple screen.
tional age, and amniotic uid volume. It may
Answer E is incorrect. Trisomy 21, or Downs also be used to monitor patients for intrauter-

Obstetrics
syndrome, classically presents with a at facial ine bleeding. However, fetal ultrasound is
prole, prominent epicanthal folds, upslanting rarely useful as a diagnostic examination for
palpebral ssures, clinodactyly, simian crease, preterm labor.
and congenital heart disease. On the triple
screen, maternal serum -fetoprotein and es- 35. The correct answer is B. Infertility can be
triol levels are low, while the -hCG level is caused by male or female factors. One-third of
high. cases are exclusively due to female factors, 20%
are exclusively due to male factors, and an-
34. The correct answer is D. Serial assessment of other 20% are due to a combination of female
the cervix for changes in dilation and efface- and male factors. Given this wifes unremark-
ment is an important aid in the diagnosis of able past medical history, female factors are
preterm labor. Examination may be accom- less likely to be the cause of the couples infer-
plished by either gentle digital examination or tility. She has no symptoms of endometriosis,
by visual inspection with the aid of a specu- no history of PID or STDs, and appears to have
lum. Digital examination is contraindicated in properly functioning ovaries. Of the choices
preterm patients whose water has broken as a listed, a male factor is the most likely cause of
preventive measure against chorioamnionitis. this couples infertility. The most common
In a patient who is unsure of the status of her male factors include low semen volume, low
membranes at the time of presentation, it sperm concentration, decreased sperm motil-
would be prudent to perform a speculum ex- ity, and atypical sperm morphology.
amination of the cervix.
Answer A is incorrect. Endometriosis is a com-
Answer A is incorrect. GBS colonization is as- mon cause of female infertility, as a result of the
sociated with preterm labor, and may compli- pelvic adhesions that form from the ectopic en-
cate the neonatal course of children delivered dometrial tissue. However, this patient has none
to colonized mothers. GBS cultures are often of the classic symptoms, such as severe dysmen-
taken from women who are in preterm labor, orrhea and chronic pelvic pain that result from
and antibiotic treatment is often initiated until the adhesions. Hence, endometriosis is unlikely
cultures are determined to be negative. How- to be the cause of this couples infertility.
ever, a GBS test is not diagnostic for preterm
Answer C is incorrect. PID is caused by as-
labor since patients may be in false labor even
cending infection of the female genital tract,
if they are colonized with GBS.
and is commonly associated with Chlamydia
Answer B is incorrect. Digital examination is trachomatis and Neisseria gonorrhoeae. It is a
contraindicated in preterm patients whose wa- common cause of female infertility secondary
HIGH-YIELD SYSTEMS
348 Section I: Organ Systems Answers

to tubal occlusion. However, this patient has Answer A is incorrect. Although adrenal gland
neither a history of STDs nor symptoms such hemorrhage does occur in traumatic births, it
as lower abdominal pain, vaginal discharge, would not cause either of the clinical pictures
low back pain, fever, nausea, and vomiting. As described.
a result, PID is an unlikely cause of this cou-
Answer B is incorrect. Facial nerve palsy oc-
ples infertility.
curs after in utero compression and results in a
Answer D is incorrect. A patient with prema- accid half-face on the ipsilateral side.
ture ovarian failure would present with early
Answer E is incorrect. Fractures occasionally
amenorrhea (usually in her 30s) and prema-
occur to the extremities during delivery and
ture symptoms of menopause. Given the regu-
can cause an absent Moro reex of the affected
larity of this womans cycles, premature ovarian
side. A fracture of the humerus would not,
failure is unlikely.
however, cause spasm of the sternocleidomas-
Answer E is incorrect. Intrauterine devices toid muscle.
are associated with an increased risk of STDs,
Answer F is incorrect. Fractures occasionally
but there is no indication of prior STDs in this
Obstetrics

occur to the extremities during delivery and


woman. These devices are not associated with
can cause an absent Moro reex of the affected
subsequent infertility in the absence of prior
side. A fracture of the radius would not, how-
STD infection. This patient has no prior his-
ever, cause spasm of the sternocleidomastoid
tory of STDs or current symptoms, such as
muscle.
pelvic pain and abnormal vaginal discharge.
STDs can lead to infertility as a result of tubal Answer G is incorrect. Outside of the nervous
occlusion. system and the musculoskeletal system, the
liver is the most commonly injured organ dur-
ing delivery. Rupture of the organ may occur
Questions 36 and 37 and form a subcapsular hematoma. It should
36. The correct answer is D. The clavicle is the be investigated with ultrasound.
most commonly fractured bone during deliv- Answer H is incorrect. An injury to the
ery. Its fracture results in an absent Moro reex phrenic nerve would result in a decrease in
on the aficted side, along with spasm of the diaphragmatic contractility, and therefore pos-
sternocleidomastoid muscle on the affected sibly respiratory distress.
side. The child is generally unable to move the
Answer I is incorrect. A fracture of the right
ipsilateral arm, and crepitus or bony irregular-
clavicle would result in the mirror image of
ity may be palpated. The clavicle is especially
the situation described in the rst vignette: ab-
vulnerable during vertex presentations. Treat-
sence of the right Moro reex and spasm of the
ment is generally immobilization, although it
right sternocleidomastoid muscle.
is not always indicated. Prognosis is excellent.
Answer J is incorrect. An injury to the seventh
37. The correct answer is C. This child suffers and eighth cervical nerves and rst thoracic
from Erb-Duchenne paralysis, resulting from nerve results in an ipsilateral paralyzed hand
an injury to the fth and sixth cervical nerves. (Klumpkes paralysis) and often an ulnar nerve
The infant loses the power to abduct the arm distribution of numbness. T1 injury also typi-
from the shoulder, rotate the arm externally, cally results in Horners syndrome (ptosis, mio-
and supinate the forearm. There is often sen- sis, and anhidrosis).
sory loss in the outer aspect of the arm. This
Answer K is incorrect. Spinal cord injuries can
injury is relatively common among mac-
occur after longitudinal traction is exerted on
rosomic infants delivered via vertex delivery,
a child and can result in profound neurologic
secondary to lateral traction on the head or ex-
decits below the lesion, occasionally resulting
cessive traction on the shoulders. Treatment is
in death.
generally immobilization, and prognosis is al-
most always excellent.
HIGH-YIELD SYSTEMS
Chapter 11: Obstetrics Answers 349

Answer L is incorrect. Injury to the third and Answer B is incorrect. Breakthrough bleed-
fourth cervical nerves would not result in ei- ing is not a relevant term to describe any of the
ther of the injuries described in the vignettes. situations above. This term is generally used to
describe non-menses bleeding that occurs mid-
cycle in a nonpregnant woman on OCPs.
Questions 38, 39, and 40
Answer C is incorrect. Though it is possible
38. The correct answer is A. This is a classic pre- for women of these ages to have cervical car-
sentation for abruptio placentae, which refers cinoma, it would not be a likely cause of the
to the premature separation of an implanted vaginal bleeding in the above cases.
placenta from the myometrium. Trauma com-
monly causes abruptio placentae, as rapid ac- Answer D is incorrect. Ectopic pregnancy
celeration-deceleration can lead to shear stress refers to implantation of a fertilized ovum in
between the placenta and uterus. There is a an area other than the endometrial lining of
disruption of maternal vessels at the decidua the uterus, most commonly the ampulla. It
basalis that leads to rapid accumulation of is highly unlikely that an ectopic pregnancy
would survive and go unrecognized into the

Obstetrics
blood. The expanding hematoma causes dis-
section in a plane between the placenta and third trimester. All of the cases above are in the
the uterus. Classic presentation includes pain- third trimester.
ful dark vaginal bleeding, uterine contractions, Answer E is incorrect. Endometrial can-
and a nonreassuring fetal heart tracing. cer would be a concern in a postmenopausal
woman complaining of vaginal bleeding. It
39. The correct answer is H. Placenta previa re- would not be a primary concern in a pregnant
fers to an abnormal location of placental im- woman complaining of bleeding.
plantation that lies either directly over or adja-
cent to the internal os of the cervix. Risk factors Answer F is incorrect. None of the patients re-
include grand multiparity and multiple previ- ports any history of possible injury from a for-
ous cesarean sections. The classic presentation eign body.
is painless vaginal bleeding beyond 24 weeks Answer G is incorrect. Normal menses is an
gestation. Uterine contractions are typically ab- incorrect answer in all of the above because all
sent. The bleeding typically stops and unless three women have conrmed pregnancies and
there is hemodynamic instability, the well be- therefore should not be menstruating.
ing of the fetus will not be compromised.
Answer J is incorrect. Threatened abortion re-
Transvaginal sonography is the gold standard
fers to vaginal bleeding with a closed os in pre-
for diagnosing placenta previa.
viable pregnancies. All of the above cases are at
40. The correct answer is I. Onset of labor be- viability. At viability, what was once a threat-
tween 20 and 37 weeks gestation is known as ened abortion now becomes preterm labor.
preterm labor. A diagnosis is made when there Answer K is incorrect. Vulvar carcinoma is
are regular contractions (3 contractions every highly unlikely in the cases above. Vulvar car-
30 minutes lasting 30 seconds each) with an as- cinoma can generally be spotted on physical
sociated cervical change assessed by sterile examination and no vulvar abnormalities were
speculum examination. Patients typically pre- noted above. In addition, vulvar carcinoma
sent with abdominal, pelvic, and/or back dis- in most commonly seen in postmenopausal
comfort; menstrual-like cramps; and a change women in their 50s and 60s.
in vaginal discharge or bleeding.
This page intentionally left blank
CHAPTER 12

Gynecology

351
HIGH-YIELD SYSTEMS
352 Section I: Organ Systems Questions

Q U E ST I O N S

1. A woman brings her 15-year-old daughter to


her pediatrician for concerns about hair
growth. The child has always had a lot of body
hair and has been shaving her legs since she
was 12 years old. The mother reports that her
daughter has recently been noticing more hair,
especially along the upper lip and on the chest
and abdomen. The child is clearly distressed
about her appearance. Further questioning re-
veals that although the girl had her rst menses
at 11 years old, her menstrual cycles are irregu-
lar, and she sometimes skips cycles for months
Gynecology

at a time. Physical examination reveals a Image courtesy of the Centers for Disease Control and
young, heavy-set, olive-skinned teenager with Preventions Public Health Image Library.
moderate acne and dark hair growth along her (A) Treat her and her partner with oral uco-
upper lip, across her chest, and over her lower nazole and test for other sexually transmit-
abdomen. She exercises regularly. Which of ted diseases
the following is the most appropriate treatment (B) Treat her and her partner with oral met-
for this childs hirsutism? ronidazole and test for other sexually trans-
(A) Danazol mitted diseases
(B) Insulin (C) Treat her with oral uconazole and test for
(C) Levothyroxine other sexually transmitted diseases
(D) Oral contraceptives (D) Treat her with oral metronidazole and test
(E) Pergolide for other sexually transmitted diseases
(E) Treat her with vaginal uconazole and test
2. An 18-year-old college student presents to the for other sexually transmitted diseases
student health clinic with a complaint of copi- (F) Treat her with vaginal metronidazole and
ous yellow vaginal discharge. She has been sex- test for other sexually transmitted diseases
ually active with a new partner for the past
month, but she is unsure if her partner is mo- 3. A 28-year-old teacher presents to the clinic
nogamous. A speculum examination reveals complaining of 5 months of polyuria, polydip-
petechiae in the upper vagina and malodorous, sia, and weight loss. Additionally, her menses,
yellow-green discharge. A potassium hydroxide which have always been irregular, have stopped
preparation reveals no organisms, and results of altogether. She is concerned because both her
a Giemsa stain are shown in the image. Which mother and maternal aunt suffer from noninsu-
of the following is the most appropriate treat- lin-dependent diabetes, and they told her they
ment? had similar symptoms before they were diag-
nosed. Upon questioning she reveals that she is
in a committed relationship and has no desire
to have children, so she uses barrier protection
during intercourse. Physical examination re-
veals an obese woman with hirsutism currently
in no acute distress. Testing for -human chori-
onic gonadotropin level, random blood sugar
level, cholesterol panel, and a luteinizing hor-
mone/follicle-stimulating hormone ratio sug-
gests the patient has polycystic ovarian syn-
drome (PCOS). Although no one in her family
HIGH-YIELD SYSTEMS
Chapter 12: Gynecology Questions 353

has had cancer, she is concerned that her limits. She has never had a pelvic exam before,
symptoms are a harbinger of cancer or that she and asks why it is necessary for her to get a Pap
might be likely to suffer from cancer in the fu- smear. Which of the following statements
ture. This diagnosis would most raise her risk about cervical cancer screening is true?
for which kind of cancer?
(A) A patient whose cytology shows atypical
(A) Cervical cancer squamous cells of undetermined signi-
(B) Colon cancer cance and who is human papillomavirus
(C) Endometrial cancer negative should have repeat cytology in 12
(D) Lung cancer months
(E) Ovarian cancer (B) A patient whose cytology shows high-grade
squamous intraepithelial lesions should be
4. A 28-year-old G1A1 woman presents to a gyne- tested for human papillomavirus status
cology clinic with a chief complaint of reduced (C) Cervical cancer screening should be
menstrual ow for the past 6 months, espe- started at the time of rst reported sexual
cially last month. She denies any pain with activity, but no later than age 21 years

Gynecology
menstruation or irregularity in her cycle. She (D) Cervical cancer screening should be
says that she had an elective termination by di- started only after a patient rst reports sex-
lation and curettage approximately 9 months ual activity
ago. She is sexually active with one partner and (E) Women >30 years old who have had three
always uses condoms. Review of her records in- normal smears should be screened every 3
dicates a past history of abnormal Papanicolaou years indenitely
(Pap) smears, but she has not been followed re-
cently. She denies any history of irregular men- 6. A 42-year-old postmenopausal woman presents
ses, and says that age of menarche was 13 years. to the clinic complaining of vague abdominal
She takes no medications. Physical examina- pain, early satiety, and a 9-kg (20-lb) unin-
tion reveals a normally developed 68-kg (150- tended weight loss. She has a history of normal
lb) woman who is 183 cm (6') tall. She is in no Pap smears. On physical examination her ab-
acute distress. A -human chorionic gonado- domen is rm, with evidence of ascites and a
tropin test from her original visit 1 week ago is rm, irregular, and xed left adnexal mass pal-
negative. Which of the following is the most pated on vaginal examination. CT scan of the
likely diagnosis? abdomen and pelvis conrms the presence of
an ovarian mass that has features that are
(A) Ashermans syndrome
highly suspicious for cancer. What is the best
(B) Cervical stenosis
means to correctly diagnose and stage this
(C) Endometrial cancer
mass?
(D) Hypogonadotropic hypogonadism
(E) Kallmanns syndrome (A) Measurement of -fetoprotein, -human
(F) Pregnancy chorionic gonadotropin, and lactate dehy-
drogenase levels
5. A 21-year-old G0 woman presents for a well- (B) Measurement of cancer antigen 125 level
woman examination. Menarche began at age (C) MRI of the abdomen and pelvis
12 years, and her periods occur every 2630 (D) Percutaneous needle biopsy of the tumor
days and last 45 days. She has had two sexual for histopathologic staining
partners since becoming sexually active last (E) Surgical exploration with tumor debulking
year. She uses condoms for contraception and and nodal sampling
denies a history of sexually transmitted disease.
Her physical examination is within normal
HIGH-YIELD SYSTEMS
354 Section I: Organ Systems Questions

7. A 55-year-old woman is brought to the emer- 9. A 33-year-old G1P1 woman presents to her gy-
gency department by re and rescue personnel necologist for a Pap smear. It has been several
because of intractable back and thigh pain for years since she last saw a physician. She is not
the past 3 hours. Upon presentation she says currently sexually active, but takes oral contra-
that the pain is 9 of 10 in severity and localized ceptives. Her vaginal examination is normal,
to her lower back. She lives with her sister, and but her Pap smear shows moderate-grade cervi-
she has no primary care physician. She denies cal intraepithelial neoplasia. The patient un-
any complaints aside from fatigue, which she dergoes colposcopy and biopsies, which con-
attributes to her multiple jobs and caring for rm the diagnosis. What is the most
her sisters children. She has a pulse of 110/ appropriate management of this patient?
min, blood pressure of 140/88 mm Hg, respira-
(A) Continued annual Pap smears
tory rate of 20/min, and temperature of 37.8C
(B) Loop electrosurgical excision procedure
(100.1F). On physical examination she is ex-
(C) Radiation therapy
quisitely tender over the L23 area of the
(D) Serial colposcopies every 34 months
spine. She also has point tenderness over the
(E) Total abdominal hysterectomy
Gynecology

anterior right thigh. Sensation is intact over the


lower extremities bilaterally and she has 5/5
10. A 22-year-old G1P0 woman with excellent pre-
strength in the lower extremities bilaterally.
natal care presents at 36 weeks gestation for a
Breast examination reveals a retracted nipple
routine evaluation. Her pregnancy has been
and dimpling of the right breast. What will
uncomplicated to date. She had a cousin who
likely represent the mainstay of treatment for
developed a thyroid disorder during her preg-
this patients symptoms?
nancy and would like her thyroid function to
(A) Bone marrow transplant be screened. Which of the following sets of
(B) Chemotherapy laboratory values best reects thyroid function
(C) Hormone replacement therapy in pregnancy?
(D) Radiation therapy
(E) Surgery CHOICE TOTAL THYROXINE FREE THYROXINE

8. A 57-year-old G3P3 woman presents to her gy- A


necologist with complaints of vaginal pruritus
and increased vaginal discharge. The patient B no change
has no history of gynecologic surgery or sexu-
C no change
ally transmitted diseases; she is not currently
sexually active. A bimanual examination and
D no change no change
Pap smear are performed. The Pap smear is
positive for malignant squamous cells. Follow- E no change
up colposcopy shows no cervical lesions, but a
small lesion is noted on the lower vagina. Bi- F no change
opsy of this lesion conrms the diagnosis of
vaginal squamous cell cancer, while cross-sec- G
tional imaging excludes invasion of surround-
ing tissues. What is the most appropriate course
of treatment? (A) A
(B) B
(A) Chemotherapy (C) C
(B) Radiation therapy (D) D
(C) Surgical excision (E) E
(D) Surgical excision and chemotherapy (F) F
(E) Surgical excision and radiation therapy (G) G
HIGH-YIELD SYSTEMS
Chapter 12: Gynecology Questions 355

11. A 48-year-old woman presents to her gynecolo-


gist because of vaginal bleeding. She states that
after a year of hot ashes and irregular cycles,
she nally stopped menstruating 4 months ago.
Two days ago she began having some vaginal
bleeding that was very similar to her prior men-
ses. She is concerned because she heard that
the rst sign of endometrial cancer in post-
menopausal women is vaginal bleeding. She is
an otherwise healthy woman with no medical
problems. She exercises three times a week and
takes multivitamins. She had three children
when she was 2935 years old. She used oral
contraceptive pills for contraception from the
time she was 18 until she got married at the

Gynecology
age of 28. Which of the following is the most
appropriate next step in managing this wom- Reproduced, with permission, from Fauci AS, Braunwald
E, Kasper DL, Hauser SL, Longo DL, Jameson LJ,
ans vaginal bleeding? Loscalzo J, eds. Harrisons Online. New York: McGraw-
(A) Abdominal ultrasound Hill, 2008: Figure 178-1.
(B) Endometrial biopsy (A) Haemophilus ducreyi
(C) Follow-up examination in 6 months (B) Herpes simplex virus
(D) Measure serum level of follicle-stimulating (C) HIV
hormone (D) Human papillomavirus
(E) Prescription of testosterone cream (E) Treponema pallidum
12. A 19-year-old woman presents to her primary 13. A 35-year-old G4P4 obese woman is referred to
care physician with complaints of multiple her gynecology clinic by her primary care physi-
eshy growths around her vulva and vagina. cian for heavy menstruation and irregular cy-
She states that the growths are painless and cles. She has noticed these symptoms for several
have developed within the past several months. months. She reports being a late bloomer,
She is not currently sexually active, but had with onset of menses at age 13 years. She is sex-
been until 5 months ago with her previous boy- ually active and monogamous with her partner
friend. She denies any vaginal pruritus or dis- of 2 years. She is taking oral contraceptive pills
charge, abdominal pain, fevers, dysuria, or vag- and has a 5-year smoking history. An endome-
inal bleeding. Physical examination of her trial biopsy is read as endometrial hyperplasia,
genitalia reveals the lesions seen in the image. cannot rule out intraepithelial carcinoma.
Which of the following is the most likely cause -Human chorionic gonadotropin testing is neg-
of these lesions? ative. Which of the following most likely con-
tributed to this abnormality?
(A) Body habitus
(B) Late menarche
(C) Multiparity
(D) Sexual activity
(E) Smoking history
(F) Use of oral contraceptive pills
HIGH-YIELD SYSTEMS
356 Section I: Organ Systems Questions

14. An 18-year-old woman presents to the clinic 16. A 29-year-old African-American woman comes
because of 6 hours of severe abdominal pain, to the physician after discovering a mass on
nausea, and vomiting. She describes 6 days of breast self-examination. Her last menstrual pe-
mild lower abdominal pain, low-grade fever, riod was 2 weeks ago. She reports occasional
and abnormal vaginal discharge. She is sexu- bilateral gray nipple discharge that has not
ally active with two male partners, and her last changed since menarche. She has no signi-
menstrual period was 10 days ago. Her temper- cant past medical history and does not take any
ature is 39.5C (103.2F), blood pressure is medications. Examination reveals a 1.5-cm
100/60 mm Hg, heart rate is 110/min, and re- uctuant mass in the upper and outer quadrant
spiratory rate is 18/min. Physical examination of the left breast. Which of the following is the
reveals involuntary abdominal guarding. The best next step in management?
patient will not allow a pelvic examination.
(A) Cytological examination of the nipple dis-
Which of the following is the most appropriate
charge
management?
(B) Fine-needle aspiration
(A) Discharge home with oral antibiotics (C) Incisional biopsy
Gynecology

(B) Discharge home without antibiotics (D) Mammography


(C) Hospitalization for intravenous antibiotics (E) Reassurance and continued breast self-
and hydration examination
(D) Hospitalization for intravenous hydration
without antibiotics 17. A 26-year-old G0 woman is seen in her gyne-
(E) Hospitalization with oral antibiotics and cologists ofce for a routine examination. She
intravenous hydration reports that she has been sexually active with
four partners and has been treated for gonor-
15. A 65-year-old G2P2 postmenopausal woman rhea once in the past year. She has otherwise
presents to a gynecologist for the rst time in been healthy. Physical examination is unre-
many years complaining of vaginal bleeding, markable. Results of a Pap smear suggest a low-
pelvic pain, and increased urinary frequency. grade squamous intraepithelial lesion. What is
She reports she is sexually active with her hus- the most appropriate next step in manage-
band. After an appropriate work-up, a diagnosis ment?
of locally invasive squamous cell carcinoma of
(A) Instruct patient to return immediately for
the cervix is made. The tumor has extended
repeat Pap smear
approximately 9 mm into the cervical stroma,
(B) Reassure patient of results and instruct her
grading the cancer as stage IB. The patient is
to return to the ofce in 6 months
informed of the diagnosis and wishes to un-
(C) Refer immediately for colposcopy
dergo denitive therapy. What is the denitive
(D) Test for human papillomavirus types 6 and
therapy for this patients disease?
11
(A) Chemotherapy (E) Test for human papillomavirus types 16
(B) Cold knife cone excision and 18
(C) Loop electrosurgical excision procedure
(D) Radical hysterectomy
(E) Uterine artery embolization
HIGH-YIELD SYSTEMS
Chapter 12: Gynecology Questions 357

18. During routine breast examination of a 28-year- 20. A 32-year-old previously healthy woman pre-
old woman with no signicant family or past sents with a 1-week history of severe u-like ill-
medical history, a rm 2-cm mass is detected ness including malaise, fever, headache, an-
in the patients right breast. It is freely mobile orexia, sore throat, and severe myalgias. Her
and nontender. Ultrasound reveals a solid, last menstrual period was 3 weeks ago and nor-
well-circumscribed mass, which is later found mal. On physical examination she has subman-
to be benign by biopsy. Which of the following dibular and cervical lymphadenopathy, and a
statements is true? few discrete reddish-brown macules measuring
1 cm on her chest. Her throat is clear. Her
(A) There is an increased incidence of this tu-
pulse is 72/min, blood pressure is 112/74 mm
mor in Japanese women.
Hg, respiratory rate is 16/min, and temperature
(B) There is an increased incidence of this tu-
is 38.3C (100.9F). Results of her vaginal ex-
mor in women undergoing tamoxifen ther-
amination are shown in the image. She has
apy
multiple sexual partners and uses barrier pro-
(C) There is an increased risk of cancer in
tection most of the time. Without treatment,
both breasts with this condition

Gynecology
the next stage of this disease is clinically char-
(D) There is no increased risk for this tumor in
acterized by which of the following?
women with BRCA-1 mutations
(E) This tumor may increase in size during
pregnancy

19. A 52-year-old postmenopausal woman who was


diagnosed with advanced ovarian cancer pre-
sents to the clinic to discuss her treatment op-
tions. She has had a CT of the abdominal-
pelvic region that showed extensive disease
extending from her left ovary and involving her
uterus along with large pelvic nodes. What is
the best treatment for this patient?
(A) Chemotherapy and radiation therapy to
the pelvis followed by surgery
(B) Paclitaxel and cisplatin therapy followed
by CT surveillance
(C) Radiation therapy to the abdomen and pel-
vis
(D) Surgical debulking with a postsurgical
course of pacitaxel and cisplatin
(E) Tumor debulking alone

Image courtesy of the Centers for Disease Control and


Preventions Public Health Image Library.

(A) Condyloma latum


(B) Fibrotic bone lesions
(C) Macular rash over the hands and feet
(D) Painless ulcer
(E) Patchy alopecia
HIGH-YIELD SYSTEMS
358 Section I: Organ Systems Questions

21. An 18-year-old female college student presents 23. A 31-year-old African-American woman is diag-
to student health services with a complaint of a nosed with uterine broids. Which of the fol-
burning sensation while urinating and abdomi- lowing types of broids is most likely to inter-
nal pain. She denies urinary urgency or in- fere with conception and pregnancy?
creased frequency. She has no signicant past
(A) Intracavitary
medical history. She is currently sexually active
(B) Intramural
with a new partner. She does not use barrier
(C) Pedunculated
contraception. She denies any previous history
(D) Submucosal
of sexually transmitted diseases. On examina-
(E) Subserosal
tion she is afebrile, heart rate is 70/min, and
blood pressure is 120/60 mm Hg. Examination 24. A 20-year-old woman presents to her gynecolo-
reveals no peritoneal signs but there is tender- gist complaining of several days of vaginal itch-
ness to palpation over the suprapubic region. ing and increased vaginal secretions that have
On pelvic examination the cervix appears an unpleasant odor. She denies any recent fe-
edematous and friable with a small amount of ver, back pain, hematuria, or vaginal bleeding.
Gynecology

discharge from the os. A urine sample reveals She has been sexually active with multiple sex-
numerous WBCs but no organisms on Gram ual partners and rarely uses protection. On ex-
stain. A cervical swab is sent for Gram stain amination she has a moderate amount of frothy
and culture. Which of the following is the green discharge. Amine whiff test of the dis-
most likely explanation for these ndings? charge is negative, and the pH of the discharge
(A) Infection with Chlamydia trachomatis is 6. Multiagellated organisms are seen on mi-
(B) Infection with Escherichia coli croscopy. Which of the following is the most
(C) Infection with Neisseria gonorrhoeae likely diagnosis?
(D) Infection with Proteus mirabilis (A) Bacterial vaginosis
(E) Interstitial cystitis (B) Neisseria gonorrhoeae infection
(C) Syphilis
22. A 25-year-old woman who is about 5 months
(D) Trichomoniasis
pregnant with her rst child presents for the
(E) Vaginal candidiasis
rst time to an obstetrician. She has had no
prenatal care. When asked about her medical 25. Mifepristone is an effective abortifacient if
history, she states she sometimes takes medi- given within 72 hours of intercourse. Mifepris-
cine for depression, and she produces a pre- tone contains which of the following?
scription bottle with lithium tablets in it. She
is otherwise healthy and her pregnancy has (A) Estrogen and progestin
been uncomplicated to date. The fundus of (B) High-dose estrogen only
her uterus is 22 cm from the pubic symphysis, (C) Progesterone antagonist
fetal movement is felt, and fetal heart tones are (D) Progestin only
present at 130/min. Which of the following (E) Prostaglandin
tests should be advised given the patients lith-
ium ingestion?
(A) Chorionic villus sampling
(B) Fetal echocardiography
(C) Fetal renal ultrasound
(D) Maternal oral glucose tolerance test
(E) Measurement of -fetoprotein, -human
chorionic gonadotropin, and estriol levels
HIGH-YIELD SYSTEMS
Chapter 12: Gynecology Questions 359

26. A 45-year-old African-American woman who


was diagnosed with PCOS in her early twenties
presents to her gynecologist for her annual
visit. One of her close friends has recently been
diagnosed with ovarian cancer, so she is con-
cerned about her own cancer risk. Menarche
was at age 14 years, and she has yet to go
through menopause. She has a healthy 19-year-
old daughter. She has no family history of can-
cer. She does not smoke or drink and exercises
regularly. Aside from a diagnosis of PCOS, she
is otherwise in good health. Given her health
history, which of the following statements is
true?
(A) She should have annual mammograms, al-

Gynecology
though her risk of breast cancer is not
changed relative to women without PCOS
(B) She should have annual mammograms be-
cause she has an increased risk of develop-
ing breast cancer relative to women with- Reproduced, with permission, from Wolff K, Johnson RA,
out PCOS Surmond D. Fitzpatricks Color Atlas & Synopsis of Clini-
(C) She should have annual Pap smears, al- cal Dermatology, 5th edition. New York: McGraw-Hill,
though she has a decreased risk of develop- 2005: Figure 27-11.
ing cervical cancer relative to women (A) Efavirenz
without PCOS (B) Fluconazole
(D) She should have annual Pap smears be- (C) Metronidazole
cause she has an increased risk of develop- (D) Topical trichloroacetic acid
ing cervical cancer relative to women (E) Valacyclovir
without PCOS
(E) She should have annual Pap smears be-
cause she has an increased risk of develop-
ing ovarian cancer relative to women with-
out PCOS

27. A 19-year-old patient presents to her gynecolo-


gist complaining of a painful genital rash (see
image). The patient states that approximately 1
week ago she had unprotected sexual inter-
course with a new partner. She denies any vag-
inal pruritus or discharge, abdominal pain, fe-
vers, dysuria, or vaginal bleeding. In addition
to local care for the lesions, what is the most
appropriate treatment for this condition?
HIGH-YIELD SYSTEMS
360 Section I: Organ Systems Questions

E X T E N D E D M ATC H I N G Urinalysis is negative for RBCs, WBCs, and


bacteria. Her past medical history is signicant
for cervical cancer treated with surgery and ra-
The response options for the next 3 items are diation last year.
the same. Select one answer for each item
in the set. 29. A 25-year-old newlywed presents to the clinic
with increased urgency and severe dysuria over
For each of the following patients, select the most the past day. Her physical examination is nor-
likely cause of urinary incontinence. mal; blood pressure is 110/80 mm Hg, pulse is
(A) Acute cystitis 100/min, temperature is 37.2C (99.0F), and
(B) Acute urinary retention respiratory rate is 16/min. Urinalysis reveals 50
(C) Neurogenic bladder WBCs/hpf, 50 RBCs/hpf, and moderate bacte-
(D) Pelvic oor laxity riuria.
(E) Pyelonephritis
(F) Urethral stulization 30. A 37-year-old G5P5 woman comes to her ob-
Gynecology

(G) Urethral sphincter insufciency stetrician for a routine visit. She complains
(H) Urethral stricture about often leaking urine when she laughs or
sneezes. Urinalysis is negative for RBCs and
28. A 52-year-old woman presents to her gynecolo- WBCs. Her past medical history is signicant
gist with a 2-month history of incontinence. for breast cancer treated with radiation 3 years
She has been experiencing small amounts of ago.
urine leakage whenever her bladder is full.
HIGH-YIELD SYSTEMS
Chapter 12: Gynecology Answers 361

AN S W E R S

1. The correct answer is D. Given the history of other STDs because infection with one STD
irregular menses, acne, and excess weight, carries a higher risk of harboring another STD.
PCOS is the most likely cause of the patients
Answer A is incorrect. Oral uconazole is used
hirsutism. Oral contraceptive pills (OCPs) are
to treat fungal infections, including that of the
the rst-line therapy against hirsutism due to
mouth, throat, esophagus, lungs, and blood.
androgen excess, as seen in PCOS, as well as
for unknown causes. In patients with PCOS, Answer C is incorrect. Oral uconazole is
OCPs are also useful in establishing regular used to treat fungal infections, including that
menstrual cycles. of the mouth, throat, esophagus, lungs, and
blood.
Answer A is incorrect. Danazol is a synthetic
androgen used to treat endometriosis. It would Answer D is incorrect. Treatment must also
actually worsen the patients hirsutism. include treatment of the partner so the infec-

Gynecology
tion is not passed back and forth or to others.
Answer B is incorrect. Many patients with
PCOS are found to have high insulin levels, Answer E is incorrect. Vaginal uconazole is
which are believed to stimulate androgen pro- used to treat Candida albicans infection of the
duction in the ovaries, and are glucose intoler- vagina, not Trichomonas infection.
ant. As a result, antiglycemic medications have Answer F is incorrect. Oral treatment is supe-
been shown to alleviate some components of rior to vaginal treatment, so the urethra can be
the syndrome, such as hirsutism. treated to prevent self-reinfection. Treatment
Answer C is incorrect. Hypothyroidism is an- must also include treatment of the partner and
other cause of hirsutism. Patients would also testing for other sexually transmitted diseases
present with weight gain, cold intolerance, and (STDs).
constipation.
3. The correct answer is C. PCOS is dened by
Answer E is incorrect. Hyperprolactinemia, the presence of infrequent menses and high
most frequently caused by a pituitary adenoma, blood levels of androgens. In addition, patients
can present with hirsutism. Patients may also have high levels of circulating estrogens, par-
experience changes in vision, headaches, and tially due to chronic anovulation, and partially
galactorrhea. This patients symptoms, how- due to increased peripheral conversion of an-
ever, are most consistent with PCOS, not a drogens to estrone due to concomitant obesity.
prolactinoma. Endometrial cancer risk is increased in the set-
ting of unopposed estrogen exposure.
2. The correct answer is B. The wet prep re-
vealed infection with the motile protozoan Answer A is incorrect. The risk for cervical
Trichomonas vaginalis. Typical signs and symp- cancer is increased for patients with multiple
toms include a malodorous, green-yellow, sexual partners, early start of sexual activity,
frothy discharge, burning, pruritus, dysuria, fre- immunocompromised status, smoking history,
quency, dyspareunia, and punctate hemor- history of human papillomavirus (HPV) infec-
rhages on the vagina/cervix. Metronidazole is tion, and history of other STDs.
the treatment for this infection. Oral treatment Answer B is incorrect. There is no relationship
is preferred to vaginal treatment because it al- between sex hormones and the development of
lows for therapeutic levels in the urethra and colon cancer.
periurethral glands, which otherwise serve as a
source for endogenous recurrence. This is an Answer D is incorrect. Lung cancer is the
STD, so both she and her partner should be most common cause of cancer death in
treated. Additionally, she should be tested for women, but it is not related to PCOS.
HIGH-YIELD SYSTEMS
362 Section I: Organ Systems Answers

Answer E is incorrect. Risk factors for ovarian sent menstrual ow, but it is unlikely in the
cancer are family history of ovarian or breast presence of a negative -human chorionic gon-
cancer, and chronic uninterrupted ovulation. adotropin test.
The patient has no family history, and PCOS
is characterized by chronic anovulation. 5. The correct answer is A. Patients whose cytol-
ogy shows atypical squamous cells of unde-
4. The correct answer is A. Ashermans syndrome termined signicance (ASCUS) should be im-
is an etiology of secondary amenorrhea or hy- mediately tested for high-risk HPV infection.
pomenorrhea characterized by the presence of This helps to determine a patients risk for a
intrauterine synechiae. It is usually caused by high-risk lesion. If the patient tests positive for
instrumentation of the uterine cavity, as is the a high-grade HPV type, then she should pro-
case status-post dilation and curettage or an ceed to colposcopy and directed biopsy. How-
elective termination of pregnancy. Ashermans ever, if she tests negative for high-grade HPV,
syndrome, although uncommon, is the most she can be followed by another Pap smear in
likely diagnosis given the patients elective ter- 12 months.
Gynecology

mination. Also, the other choices are unlikely


Answer B is incorrect. Patients found to have
given the patients history. The diagnosis is
high-grade squamous intraepithelial lesions
conrmed by hysteroscopy.
on Pap smear should proceed directly to col-
Answer B is incorrect. Cervical stenosis is poscopy. As most high-grade lesions are associ-
a cause of secondary amenorrhea. It may be ated with a positive HPV status, HPV testing in
due to congenital, inammatory, or neoplastic these patients is not cost effective. During col-
causes, but the cause is usually surgical. Sur- poscopy a cervical biopsy may be performed to
gical operations such as electrocoagulation, better characterize the lesion.
cryotherapy, laser vaporization, conization, or
Answer C is incorrect. Cervical cancer screen-
cervical amputation are normally the culprits.
ing should be started approximately 3 years af-
Cervical stenosis can result in complete or
ter the onset of sexual activity, but no later than
partial obstruction of menstrual ow, causing
age 21 years. It is not necessary to start screen-
amenorrhea or hypomenorrhea. It is usually
ing at the time of rst sexual activity since cer-
diagnosed by pelvic ultrasound.
vical dysplasia takes many years to progress to
Answer C is incorrect. Endometrial cancer cervical cancer.
usually presents in older women as postmeno-
Answer D is incorrect. The upper age limit of
pausal vaginal bleeding.
21 years was recommended to ensure screen-
Answer D is incorrect. Hypogonadotropic hy- ing of women in whom a sexual history is dif-
pogonadism is a cause of hypomenorrhea and cult to obtain or who may be reluctant to dis-
is common in anorexic patients, athletes who close their sexual activity due to their status as
train excessively, and in genetic disorders such victims of sexual abuse or adolescents.
as Kallmans syndrome. This patients history
Answer E is incorrect. There is no consensus
does not suggest any of these causes.
regarding the age at which cervical cytology
Answer E is incorrect. Kallmanns syndrome testing may cease. But in women without risk
is a cause of hypogonadotropic hypogonadism factors for cervical cancer and who have had
due to insufcient secretion of gonadotropin- repeated negative cytologies, 6570 years may
releasing hormone by the hypothalamus. Di- be an acceptable age to stop screening. Women
agnosis is characterized by hypogonadism and >6575 years old who have risk factors for cer-
one or more nongonadal abnormalities includ- vical intraepithelial neoplasia (CIN), such as
ing anosmia, red-green color blindness, cleft exposure to diethylstilbestrol in utero or an im-
palate, urogenital abnormalities, or neurosen- munocompromised state, should continue to
sory hearing loss. be screened annually. Women who test positive
for HPV DNA or have new sexual partners may
Answer F is incorrect. Pregnancy should al-
also benet from continued screening.
ways be considered in cases of reduced or ab-
HIGH-YIELD SYSTEMS
Chapter 12: Gynecology Answers 363

6. The correct answer is E. To properly stage an have breast cancer is an ominous one and
ovarian mass that is highly suspicious for can- likely represents metastases to bone. With re-
cer, a full exploration and inspection of all pel- gard to therapy, most patients with metastatic
vic structures is required. There is no noninva- breast cancer do not recover with any means of
sive means to consistently diagnose and stage treatment. The decision to begin local therapy
ovarian cancer. Removal of the primary mass (surgery or radiation therapy) or systemic ther-
with histopathologic staining is required, and apy (endocrine medications or chemotherapy)
retroperitoneal exploration with pelvic and depends on the extent and localization of the
para-aortic nodal sampling is performed. disease. In this patient, with multiple symptom-
atic sites, a systemic treatment such as chemo-
Answer A is incorrect. Elevated -fetoprotein,
therapy is the treatment of choice.
-human chorionic gonadotropin, and lac-
tate dehydrogenase levels are associated with Answer A is incorrect. Bone marrow transplant
primary germ cell cancers. Measurements of is not part of the standard treatment for breast
these levels are performed in cases in which a cancer.
rm xed adnexal mass is palpated in a preme-

Gynecology
Answer C is incorrect. Hormone replacement
narchal or adolescent patient because primary
therapy is contraindicated in patients with
germ cell cancers are more prevalent in this age
breast cancer.
group.
Answer D is incorrect. Radiation therapy is
Answer B is incorrect. A work-up for ovarian
used as adjuvant therapy in breast cancer, ei-
cancer should include measuring the CA-125
ther after local excision for large tumors or af-
level because certain cancers can present with
ter lumpectomy with axillary node dissection.
an elevated level. In such instances, response
This patients tumor burden is likely to be in-
to chemotherapy can be monitored by the drop
operable.
in CA-125. However, cancers other than ovar-
ian cancer may elevate the CA-125 level (e.g., Answer E is incorrect. Surgery for metastatic
endometrial cancer, certain pancreatic can- breast cancer is considered a type of local treat-
cers). Benign conditions, such as endometriosis, ment. This modality might be recommended if
uterine leiomyoma, and pelvic inammatory there were only an isolated area of recurrence,
disease (PID), can elevate the level as well. CA- a single metastatic site, or concern for an im-
125 level assessment play no role in the staging pending fracture. Although difcult, a possibil-
of ovarian cancer as staging is performed surgi- ity of combination therapy with chemotherapy
cally. and surgery is could be considered.
Answer C is incorrect. Although MRI will 8. The correct answer is B. Small squamous cell
show the ovarian mass and nodes that are in- vaginal cancers in the upper portion of the va-
volved, MRI will not distinguish the type of
gina are often treated by surgical excision,
ovarian cancer. Tissues from the mass and
which consists of radical hysterectomy, upper
nodes are required to diagnose and fully stage
vaginectomy, and bilateral lymph node dissec-
the progression of cancer.
tion. However, resection of malignancies in the
Answer D is incorrect. Although a needle bi- lower portion of the vagina is difcult, and the
opsy will provide tissue to diagnose the mass, primary treatment is radiation therapy. Early
deep pelvic nodes (from which tissue is required stage cancers may be treated solely with
to properly stage the spread of disease) are not brachytherapy, but more advanced cancers are
reached by percutaneous needle biopsy. treated with external beam radiation.
Answer A is incorrect. Several chemothera-
7. The correct answer is B. This patient likely
peutic agents have been studied for use in ad-
has metastatic breast carcinoma. The nding
vanced vaginal cancer, but none have been
of a retracted nipple and dimpling of the breast
found to initiate a signicant therapeutic re-
in a woman with no regular health care is
sponse. Some experts advocate the use of che-
highly suggestive of breast carcinoma. The
motherapy in addition to radiation therapy,
complaint of bone pain in a patient likely to
HIGH-YIELD SYSTEMS
364 Section I: Organ Systems Answers

although randomized controlled trials inves- Answer E is incorrect. Since CIN II is a pre-
tigating this issue did not nd compelling evi- malignant lesion that has not yet spread be-
dence to support this position. yond the cervical epithelium, local excision is
adequate treatment. Total abdominal hyster-
Answer C is incorrect. Surgical resection of
ectomy is reserved for treating early invasive
cancer in the lower vagina is difcult, and the
disease that has not yet extended to the pelvic
primary modality of treatment is radiation ther-
wall. Early invasive disease includes lesions
apy.
with >3.0 mm invasion of the cervical stroma
Answer D is incorrect. Lower vaginal cancers and without involvement of either the pelvic
are not amenable to surgical resection, and side wall or the lower one-third of the vagina.
chemotherapeutic agents are of limited use in Treatment of CIN II with hysterectomy would
the treatment of vaginal cancer. be excessively morbid without providing any
additional benets over treatment with loop
Answer E is incorrect. The technical difculty
electrosurgical excision.
of resection of lower vaginal cancers precludes
surgical treatment for this disease.
Gynecology

10. The correct answer is F. High estrogen levels


during pregnancy lead to an almost twofold in-
9. The correct answer is B. This patient has CIN
crease in thyroid-binding globulin (TBG),
stage II. Dysplastic CIN II cells most often will
since estrogen increases TBG production and
undergo malignant transformation if not
TBG N-acetylation, which results in decreased
treated. The mean time to malignant transfor-
clearance of TBG. Due to the increased TBG,
mation is 4 years for CIN II, but occasionally
total and bound triiodothyronine (T3) and thy-
can be much quicker. While CIN I lesions can
roxine (T4) increase, but active free (unbound)
be monitored with colposcopy every 34
T3 and T4 hormone levels are unchanged.
months, the standard of care for CIN II and III
lesions is local excision by a loop electrosurgi- Answer A is incorrect. In a normal pregnant
cal excision procedure. This procedure can of- patient with no thyroid dysfunction, total
ten be performed in the gynecologists ofce. T4 will increase and free T4 will remain un-
changed.
Answer A is incorrect. These premalignant le-
sions need to be removed. Continued monitor- Answer B is incorrect. In a normal pregnant pa-
ing with annual Pap smears is insufcient man- tient with no thyroid dysfunction, total T4 will
agement of a patient with these lesions. increase due to increased TBG production.
Answer C is incorrect. Radiation therapy may Answer C is incorrect. In a normal pregnant
be used to treat early invasive disease and is patient with no thyroid dysfunction, total T4
the only effective therapy for advanced dis- will increase due to increased TBG produc-
ease that has extended into the parametria and tion, but free T4 will remain unchanged.
beyond (Stage IIb to IV). Advanced disease
Answer D is incorrect. In a normal pregnant
is usually treated with a combination of che-
patient with no thyroid dysfunction, total T4 will
motherapy and radiation. Given CIN II status
increase due to increased TBG production.
documented by coloposcopy, radiation therapy
would be an excessively morbid treatment for Answer E is incorrect. In a normal pregnant
a lesion amenable to simple local excision fol- patient with no thyroid dysfunction, total T4
lowed by surveillance. will increase due to increased TBG produc-
tion, but free T4 will remain unchanged.
Answer D is incorrect. Serial monitoring with
colposcopy every 34 months is appropriate Answer G is incorrect. In a normal pregnant
treatment for mild CIN (CIN I). However, for patient with no thyroid dysfunction, free T4
moderate to severe dysplasia (CIN II to III), will not increase secondary to increased TBG,
malignant transformation may occur rapidly, but will remain unchanged.
making local excision of the dysplastic tissue
necessary.
HIGH-YIELD SYSTEMS
Chapter 12: Gynecology Answers 365

11. The correct answer is C. Perimenopause is the is HPV, a DNA virus, a non-oncogenic strain.
period prior to the cessation of menses; it com- HPV is spread through direct skin-to-skin con-
monly occurs up to 2 years prior to the onset of tact during sexual intercourse. Lesions often
menopause. Menopause is dened as amenor- appear after a 612 week incubation period.
rhea for 1 years duration. This woman presents Certain high-risk strains of HPV (strains 16,
with symptoms of perimenopause (hot ashes 18, 45, and 56) are associated with an in-
and menstrual irregularities) of 1 year, and creased incidence of cervical cancer.
amenorrhea for 4 months. Because she has not
Answer A is incorrect. Haemophilus ducreyi
had amenorrhea for a full year, she is still con-
is the causative organism of chancroid. Chan-
sidered to be perimenopausal, and this episode
croid lesions are irregular, erythematous lesions
of bleeding likely represents an anovulatory
that have bloody or purulent secretions. These
menstrual cycle. She should continue to moni-
lesions arise 26 days after sexual contact, are
tor her vaginal bleeding and can be offered
quite painful, and are often associated with en-
medications to lessen her symptoms (i.e.,
larged tender lymph nodes. Treatment with a
OCPs).
single dose of azithromycin is often curative.

Gynecology
Answer A is incorrect. An abdominal ultra-
Answer B is incorrect. Herpes simplex virus
sound could be used to visualize the uterine
leads to painful vesicular lesions that often
cavity and would likely reveal a thickened en-
arise within days of sexual intercourse. The
dometrium in a menstruating woman. It would
lesions are at and regular, and last for 12
not be helpful in ruling out endometrial can-
weeks. These lesions would be inconsistent
cer. This would require a biopsy.
with the given history and examination nd-
Answer B is incorrect. Nulliparity and unop- ings.
posed estrogenic states are associated with in-
Answer C is incorrect. HIV is the organism
creased risk of endometrial cancer, neither of
that causes AIDS. While it places a woman at
which applies to this woman. Indeed, 10 years
a higher risk for getting HPV infection, HIV
on the OCP places this woman at a lower risk
does not cause genital warts.
for endometrial cancer. However, to rule out
endometrial cancer, the second most common Answer E is incorrect. Treponema pallidum
cancer of the female genital tract, an endome- is the infectious agent that causes syphilis. In
trial biopsy is the standard of care if vaginal primary syphilis the lesion is a chancre that
bleeding occurs >1 year after amenorrhea in a appears 1060 days after disease transmission.
postmenopausal woman. This lesion is painless, with indurated edges
and a punched out center. Six to eight weeks
Answer D is incorrect. Because this woman
later secondary syphilis may develop, with con-
is perimenopausal, luteinizing hormone and
stitutional symptoms and a maculopapular rash
follicle-stimulating hormone levels are likely to
that often occurs on the hands and feet. At this
be high and would not provide any further spe-
time condyloma lata may form, which have a
cic information.
regular, round, and at appearance that differ-
Answer E is incorrect. In menopausal women, entiates them from condyloma acuminata.
vaginal atrophy is a common cause of vaginal
bleeding, which can be treated with topical 13. The correct answer is A. A mechanism of en-
estrogen or testosterone creams. However, sub- dometrial carcinoma and hyperplasia is abnor-
stantial bleeding is less likely to result from vag- mal proliferation of glandular elements of the
inal atrophy than from an endometrial source. endometrium under the direction of high lev-
els of unopposed estrogen. Obesity leads to a
12. The correct answer is D. This history and im- state of high levels of estrone from the aromati-
age are consistent with condyloma acuminata, zation of androstenedione in peripheral fat.
commonly known as genital warts. These le- Other sources of estrogen include endogenous
sions are painless, raised, and irregular in sources such as hypersecretion from the ovaries
shape. The causative organism of these lesions and exogenous sources such as medications.
HIGH-YIELD SYSTEMS
366 Section I: Organ Systems Answers

Answer B is incorrect. Late menarche is pro- serious health and fertility implications. Rup-
tective while early menarche is a risk factor for ture and sepsis are possible risks and carry sig-
endometrial hyperplasia/carcinoma, as more nicant associated morbidity and mortality.
cycles of unopposed high estrogen increase
Answer D is incorrect. The standard of care
risk.
for management of tubo-ovarian abscess in-
Answer C is incorrect. Multiparity is protec- volves initiating broad-spectrum intravenous
tive, as the endometrium is not exposed to as antibiotics. Intravenous hydration alone is in-
many cycles of endometrial proliferation, while sufcient.
nulliparity is a risk factor.
Answer E is incorrect. Although PID can
Answer D is incorrect. Sexual activity is not sometimes be managed in the outpatient set-
known to be a risk factor for endometrial carci- ting with oral antibiotics, the diagnosis of tubo-
noma/hyperplasia. ovarian abscess requires intravenous antibiot-
ics. Oral antibiotic regimens and those lacking
Answer E is incorrect. Smoking has been
coverage for facultative and anaerobic patho-
found to be protective in large retrospective
Gynecology

gens are inadequate.


studies. It is thought to stimulate hepatic me-
tabolism of circulating estrogens. At the very
15. The correct answer is D. Patients with local-
least, it has not been found to be a risk factor.
ized cervical cancer, such as this patient, may
Answer F is incorrect. Use of OCPs has been be treated with radical hysterectomy (which in-
found to be protective for endometrial carci- volves the removal of the uterus and upper por-
noma/hyperplasia due to the content of proges- tion of the vagina) or radiation therapy plus
tins in most formulations. It is proliferation of chemotherapy. Both treatments have similar
the endometrial elements under the direction 5-year survival and complication rates. Patients
of unopposed estrogen that increases risk. undergoing surgery may retain ovarian func-
tion and have lower rates of vaginal stenosis.
14. The correct answer is C. This patient has More advanced disease or bulky early disease is
symptoms of PID, which, if untreated, can de- preferentially treated with radiation therapy
velop into a tubo-ovarian abscess. Treatment and adjuvant chemotherapy.
for tubo-ovarian abscess involves admission to
Answer A is incorrect. Chemotherapy has
the hospital for intravenous antibiotics, hydra-
been found to signicantly increase the sur-
tion, and possible surgical intervention. Pa-
vival rate of women when used as an adjuvant
tients with PID may give a 13 day history of
to radiation therapy. Chemotherapy is also
lower abdominal pain with or without nausea,
used as palliative therapy in women with recur-
vomiting, or fever. Tubo-ovarian abscess can
rent or advanced disease that cannot be treated
complicate PID in up to one-third of patients
locally. However, chemotherapy alone is not
admitted for management. It should be sus-
used as a curative therapy for cervical cancer.
pected in patients with severe pain, high fever,
nausea, vomiting, signs of sepsis, and perito- Answer B is incorrect. Cold knife cone exci-
neal signs. sion is most often used to treat patients who
have stage II or III CIN, a precursor of cervical
Answer A is incorrect. A patient with signs of
cancer. For this indication it has mainly been
peritonitis and possible sepsis is not a candidate
replaced by the loop electrosurgical excision
for outpatient management and should not be
procedure. It may also be used as denitive
discharged home.
treatment for young women with malignant in-
Answer B is incorrect. Tubo-ovarian abscess vasion <3 mm into the cervix who wish to pre-
is a serious microbial infection that must be serve their fertility.
treated with hospitalization, hydration, and in-
Answer C is incorrect. Loop electrosurgical
travenous antibiotics. Discharging the patient
excision procedure is used to excise cervical
home without appropriate treatments will have
HIGH-YIELD SYSTEMS
Chapter 12: Gynecology Answers 367

intraepithelial neoplasia II or III. It is not used gardless, tissue should be obtained from this
for resection of malignant lesions. palpable mass for pathologic examination.
Answer E is incorrect. Uterine artery embo- Answer E is incorrect. Breast cancer is a possi-
lization is a procedure performed by inter- bility in this 29-year-old woman, and therefore
ventional radiologists to treat uterine broids. reassurance and continued breast self-examina-
These metaplastic tumors are primarily fed by tion is not adequate management. Clinical ob-
the uterine artery, and embolization of their servation may be appropriate once it has been
vascular supply often results in reduction of determined that the lesion is not malignant.
bulk symptoms and menorrhagia. It is not used
to treat malignant cervical cancer. 17. The correct answer is C. With rare exceptions,
women with low-grade squamous intraepithe-
16. The correct answer is B. The lifetime inci- lial lesions on cervical cytology screening are
dence of breast cancer is 12% in women who referred for colposcopy because of the concern
live to age 90 years, and nearly 75% of patients for cervical cancer. Recall that cytology (Pap
have no known risk factors. Breast carcinoma is smear) is never diagnostic: pathologic examina-

Gynecology
not likely in a 29-year-old woman (only 2% of tion of a biopsy specimen (colposcopy and bi-
all breast cancers are diagnosed in women <30 opsy) is the only way to diagnose cervical can-
years old); however, with such a high inci- cer or precancerous lesions.
dence of breast cancer this is still a serious pos-
Answer A is incorrect. An immediate repeat
sibility, and the price of missing the diagnosis
Pap smear is not indicated in patients with
is too high. Whenever there is a palpable soli-
low-grade squamous intraepithelial lesions or
tary breast mass, tissue needs to be obtained for
high-grade squamous intraepithelial lesions.
pathologic examination. The best method is by
Pap tests have 70% sensitivity, so a second Pap
ne-needle aspiration, often guided by ultra-
test that was negative would not be a certain
sound; however, if the mass is readily palpable,
enough measure to make a denitive tissue di-
it may not be necessary to use radiographic
agnosis unnecessary.
guidance. Of note, one-half of all breast can-
cers develop in the upper outer quadrant of the Answer B is incorrect. It is inappropriate to de-
breast. lay testing that could lead to catching cervical
malignancy in an early stage.
Answer A is incorrect. The patient describes
physiologic nipple discharge that is unchanged. Answer D is incorrect. Because low-grade
Even if the patient were having abnormal dis- squamous intraepithelial lesions were detected
charge, cytologic examination of nipple dis- in this patient, HPV testing is not indicated.
charge is too frequently associated with false- Moreover, HPV types 6 and 11 are not the
negative results to be reliable. high-risk HPV types that are associated with
cervical cancer; these HPV serotypes are asso-
Answer C is incorrect. Incisional biopsy is too
ciated with benign condyloma.
aggressive to make the diagnosis when ne-
needle aspiration may sufce. Additionally, Answer E is incorrect. Because low-grade
if surgery is considered, removal of the entire squamous intraepithelial lesions were de-
mass is usually recommended. tected in this patient, HPV testing is not in-
dicated. Testing for the high-risk HPV types
Answer D is incorrect. Mammography is
is performed when cytology reveals atypical
not likely to be helpful in this case. Younger
squamous cells of undetermined signicance.
women have more brous breast tissue, which
If it is positive for the high-risk HPV types,
makes mammograms harder to interpret. El-
then colposcopy is recommended.
derly womens breasts can be more adequately
visualized by mammography because of the 18. The correct answer is E. Fibroadenomas are
fatty replacement of tissue that occurs with age. benign, slow-growing tumors, which are the
Breast ultrasound is the best option to evaluate
most common cause of a breast mass in
a breast mass in a woman <30 years old. Re-
women <35 years old. They commonly present
HIGH-YIELD SYSTEMS
368 Section I: Organ Systems Answers

as round, discrete, rubbery and mobile lesions ing alone will most likely leave residual disease
seen as solid on ultrasound. The hormonal or tumors that were too small to be visualized
changes experienced during pregnancy may during intra-abdominal exploration. Some
lead to enlargement of broadenomas. These forms of early resected ovarian cancer are
lesions should regress following delivery. treated with surgery followed only by observa-
tion. However, this patient has advanced dis-
Answer A is incorrect. Though there are no
ease, and thus chemotherapy is required.
clear ethnic predilections, broadenoma ap-
pears to be more common in African-Ameri-
20. The correct answer is B. Syphilis is a chronic
can women, not Japanese women.
disease produced by the spirochete Treponema
Answer B is incorrect. Tamoxifen therapy actu- pallidum. Clinically, syphilis is divided into
ally decreases the incidence of broadenomas, primary, secondary, and tertiary (or late) stages.
in much the same way that it works against This patient presents with multiple white-gray
breast cancer. However, tamoxifen therapy in- hypertrophic papules and nodules in the vulvar
creases the risk of endometrial malignancy. region. This is a characteristic skin nding of
Gynecology

secondary syphilis called condyloma latum.


Answer C is incorrect. This statement de-
This patient also presents with other features of
scribes lobular carcinoma in situ, not broad-
secondary syphilis, namely lymphadenopathy,
enoma, which is not associated with an in-
rash, and her symptoms of malaise, fever, head-
creased risk of malignancy.
ache, anorexia, sore throat, and severe myal-
Answer D is incorrect. Women with BRCA-1 gias. Thus, without treatment this patient may
mutations are more susceptible to both benign develop tertiary syphilis, the next stage of this
and malignant breast masses. disease. The manifestations of tertiary syphilis
include gummas (granulomatous lesions that
19. The correct answer is D. Tumor debulking is eventually necrose and become brotic) of the
indicated for any stage of ovarian cancer be- skin and bone, tabes dorsalis, and aortic aneu-
cause chemotherapy is more effective when the rysm.
tumor masses are each less than 1 cm in diame-
ter. Combined paclitaxel and cisplatin is the Answer A is incorrect. Condyloma latum is a
chemotherapy of choice for ovarian cancer. characteristic skin nding of secondary syphi-
lis. This painless, wart-like lesion often occurs
Answer A is incorrect. Optimal treatment for in warm, most sites and is highly contagious
advanced ovarian cancer is tumor debulking
followed by chemotherapy. Although neoadju- Answer C is incorrect. Rash is the most char-
vant chemotherapy can be used in patients in acteristic nding of secondary syphilis. The
whom surgery is considered excessively risky, rash consists of small red macular lesions sym-
it is not considered standard of care. More re- metrically distributed over the body, particu-
search is needed to elucidate the optimal use larly involving the palms, soles, and mucous
of neoadjuvant chemotherapy. membranes.

Answer B is incorrect. Chemotherapy is less Answer D is incorrect. In primary syphilis a


effective when the tumors are greater than 1 chancre develops after an incubation period
cm in diameter, and reducing the tumor size of 23 weeks. This is a painless ulcer with an
by debulking allows for more effective chemo- indurated base usually found on the vulva, va-
therapy. gina, or cervix, although it can occur in any
area that comes in contact with infectious le-
Answer C is incorrect. Because advanced ovar- sions. Not only is a chancre painless, but it
ian cancer often has peritoneal seeding and spontaneously heals in 36 weeks, which ex-
extensive volume of tumor, radiation is ineffec- plains why women may be unaware of the ini-
tive as the primary mode of treatment. tial infection.
Answer E is incorrect. Tumor debulking Answer E is incorrect. A patchy or so-called
alone is not curative or effective since debulk- moth-eaten alopecia is occasionally associ-
HIGH-YIELD SYSTEMS
Chapter 12: Gynecology Answers 369

ated with secondary syphilis. This condition is tation such as catheters, as well as those with
usually reversible with treatment. nosocomial infections. Proteus raises urine pH
and can be detected with urine Gram stain.
21. The correct answer is A. Chlamydia trachoma-
Answer E is incorrect. Interstitial cystitis is a
tis is an obligate intracellular organism and is
condition that results in recurring discomfort
one of the most common sexually transmitted
or pain in the bladder and the surrounding
organisms, causing urethritis, mucopurulent
pelvic region. Symptoms are characterized by
cervicitis, and late postpartum endometritis.
urinary urgency and increased frequency, but
Chlamydial infections are often asymptomatic,
not commonly dysuria. Although some symp-
but may also present with signs of a urinary
toms of interstitial cystitis resemble those of a
tract infection (nongonococcal urethritis) in-
bacterial infection, Gram stain or culture of
cluding dysuria and pyuria, but without any or-
the urine reveals no organisms. This condition
ganisms visible on Gram stain of a sterile urine
is one diagnosed primarily by exclusion.
sample. Direct uorescent antibody tests and
DNA detection tests using polymerase chain
22. The correct answer is B. Lithium ingestion is

Gynecology
reaction are highly sensitive and specic. Treat-
associated with Ebsteins anomaly of the heart,
ment consists of oral erythromycin, amoxicil-
which is downward displacement of the tricus-
lin, or azithromycin. Because gonorrhea often
pid valve resulting in tricuspid regurgitation.
coexists with chlamydia, therapy with an intra-
Fetal echocardiography can be used to visual-
muscular injection of ceftriaxone is also indi-
ize tricuspid regurgitation.
cated.
Answer A is incorrect. Chorionic villus sam-
Answer B is incorrect. Escherichia coli is re-
pling directly allows for evaluation of the chro-
sponsible for more than 90% of all uncompli-
mosomal composition of the fetus. Lithium is
cated urinary tract infections in women, which
not associated with fetal chromosomal abnor-
occur primarily via periurethral contamination
malities.
during sexual intercourse or via contamination
due to the proximity of the anus to the urethra. Answer C is incorrect. Angiotensin-converting
Symptoms include urinary frequency, urgency, enzyme inhibitors, not lithium, are associated
dysuria, and low-grade fever. Gram stain re- with fetal renal malformations.
veals gram-negative bacilli. Answer D is incorrect. A maternal oral glu-
Answer C is incorrect. Neisseria gonorrhoeae cose tolerance test is used to test for maternal
can cause inammation of mucous mem- diabetes, which is not a sequela of lithium use.
branes including the urethra, but is more In contrast, hypoglycemia sometimes occurs in
likely to be asymptomatic or minimally symp- pregnant women who take lithium.
tomatic. Gonococcal infection primarily mani- Answer E is incorrect. Abnormal -fetoprotein,
fests as male urethritis and female endocer- -human chorionic gonadotropin, and estriol
vicitis. In the female, the discharge is usually levels, commonly known as the triple screen,
described as thin, purulent, and mildly odor- may be indicative of neural tube defects and
ous. It is an organism that would be detected chromosomal abnormalities (trisomy 13, 18,
on urine Gram stain, showing 5 WBCs per or 21). They would not be inuenced by heart
oil-immersion eld with intracellular gram- defects seen with lithium use.
negative diplococci within leukocytes.
Answer D is incorrect. Proteus mirabilis is a 23. The correct answer is D. Fibroids (leiomyo-
gram-negative, facultatively anaerobic bacte- mata), benign tumors composed of smooth
rium commonly implicated in urinary tract in- muscle, are the most common gynecologic neo-
fections. However, Proteus infection is usually plasm. They are often multiple, and higher inci-
found in those with structural abnormalities of dence is associated with African-American race,
the urinary tract, those with ureteral instrumen- 2040 age range, and obesity. Fibroids contain
HIGH-YIELD SYSTEMS
370 Section I: Organ Systems Answers

hormone receptors, and thus may enlarge with Answer E is incorrect. Subserosal broids de-
increased estrogen states (i.e., obesity) and de- velop under the outer surface of the uterus.
crease in size when estrogen is suppressed (i.e., They are not commonly associated with men-
OCP use or menopause). Symptoms vary due to orrhagia or infertility. Symptoms of pelvic pain
size and location. Patients may be asymptomatic arise due to compression of surrounding pelvic
or experience severe pain, heavy and/or pro- structures.
longed menstrual bleeding, and mass effect.
Submucosal broids grow just beneath the sur- 24. The correct answer is D. Trichomonas vagina-
face of the uterine lining. They often cause lis is a protozoan with multiple agella that
menorrhagia. Although they are the least com- causes a sexually transmitted vaginitis. Most
mon type, they are most likely to interfere with commonly occurring symptoms include in-
conception and pregnancy. By creating a space- creased vaginal discharge with an unpleasant
occupying lesion, a submucosal broid may in- odor, dysuria, and vaginal pruritus. Infection
terfere with implantation of an embryo. Fibroids with Trichomonas causes an increase in vaginal
may enlarge during the rst trimester of preg- pH to >4.5 (normal vaginal pH is 3.84.2).
Gynecology

nancy. If enlarged to signicant sizes, they may Treatment with a single dose of 2 g of metro-
distort the uterine cavity and cause spontaneous nidazole is effective. The symptoms, increased
abortion, obstruct the birth canal, interfere with vaginal pH, and agellated organisms seen on
uterine contractions during labor, or cause post- wet mount establish the diagnosis.
partum hemorrhage. Treatment includes OCPs, Answer A is incorrect. Bacterial vaginosis is
uterine artery embolization, or myomectomy. caused by an imbalance in vaginal ora, with
Answer A is incorrect. Intracavitary submu- reduced numbers of lactobacilli and increased
cosal broids grow on a stalk arising from just proportions of bacteria such as Gardnerella, Mo-
under the uterine lining and protrude into the biluncus, or Peptostreptococcus species. Signs
uterine cavity. If large enough, they may cre- of infection include a thin, white vaginal dis-
ate a space-occupying lesion and impinge on charge, vaginal pH >4.5, shy odor on 10% po-
the growing fetus. Although the stalk is sub- tassium hydroxide whiff test, and clue cells on
mucosal, it does not occupy as much space as saline mount microscopy. Flagellated organisms
a nonpedunculated submucosal broid and is would not be seen in bacterial vaginosis.
thus less likely to interfere with implantation.
Answer B is incorrect. Patients suffering from
These broids may cause abdominal pain by
Neisseria gonorrhoeae cervicitis often complain
twisting and torsion. In rare cases they may
of vaginal pruritus and discharge. However, up
cause intermenstrual bleeding if they outgrow
to 50% of patients may not manifest any symp-
their blood supply and become necrotic. They
toms at all. Diagnosis is established by identi-
may also twist and compromise blood supply.
cation of N. gonorrhoeae on chocolate agar
Answer B is incorrect. Intramural broids are culture or by DNA probe testing. The potas-
the most common type of broids. They can sium hydroxide whiff test would be negative
become signicantly enlarged, at which point and no agellated organisms would be seen.
they may compress surrounding pelvic struc-
Answer C is incorrect. The rst manifestation
tures, resulting in urinary urgency, frequency,
of syphilis, caused by infection with Treponema
and back pain.
pallidum, is a painless primary chancre sore
Answer C is incorrect. A pedunculated subse- that appears 1060 days after the infecting
rosal broid is anchored just below the outer sexual contact. Several weeks later, symptoms
surface of the uterus and protrudes into the of secondary syphilis can occur, which include
pelvic cavity. It is unlikely to interfere with constitutional symptoms, a maculopapular
menstruation or fertility. Symptoms arise from rash on the hands and feet, and at, broad con-
compression of surrounding pelvic structures. dyloma lata. Vaginal symptoms do not result
from syphilis, and agellated organisms would
not be seen. Treponema obtained from syphi-
HIGH-YIELD SYSTEMS
Chapter 12: Gynecology Answers 371

litic lesions can be visualized with darkeld comparative studies show that mifepristone
microscopy. is as effective as levonorgestrel in preventing
pregnancy.
Answer E is incorrect. Patients with candidia-
sis often complain of a thick vaginal discharge, Answer E is incorrect. Prostaglandins, particu-
vaginal pruritus, or dysuria. Signs of candidia- larly prostaglandin E2, stimulate contraction
sis include vaginal pH in the range of 45, and of uterine smooth muscle and are thus used
budding hyphae or spores when examined on as abortifacients. They do not have a role in
a slide treated with 10% potassium hydroxide emergency contraception.
preparation, which lyses the cells in the sample
and makes the yeast easier to visualize. Clue 26. The correct answer is B. PCOS is associated
cells are not seen in candidiasis. with an increased risk of breast and endome-
trial cancer secondary to unopposed estrogen
25. The correct answer is C. Mifepristone is an secretion. It is a syndrome, not a disease, and
antiprogestin that has been shown to be 100% may present with hirsutism, menstrual abnor-
effective at preventing pregnancy when given malities, obesity, acanthosis nigricans, and pre-

Gynecology
within 72 hours of intercourse at a one-time cocious puberty. It is later associated with an
dose of 600 mg. Recent studies, however, have increased risk of infertility, metabolic syn-
demonstrated equal efcacy of use of a 10-mg drome, and type 2 diabetes mellitus.
dose of this drug, intended to minimize the ad-
Answer A is incorrect. Women with PCOS
verse effects and decrease disturbances of the
have an increased risk of breast and endome-
subsequent menstrual cycle. The high efcacy
trial cancer secondary to unopposed estrogen
of mifepristone compared to other regimens is
secretion.
likely secondary to its ability to inhibit implan-
tation as well as ovulation. Answer C is incorrect. An increased risk of
cervical cancer is associated with multiple or
Answer A is incorrect. The Yuzpe regimen,
high-risk sexual partners, STDs, smoking, and
which includes two doses of both estrogen and
multiparity.
progestin pills taken 12 hours apart, was the
rst available emergency contraceptive. Due Answer D is incorrect. An increased risk of
to the signicant nausea and vomiting associ- cervical cancer is associated with multiple or
ated with their administration, these medica- high-risk sexual partners, STDs, smoking, and
tions are disfavored. Progestin-only pills such multiparity, and is unrelated to PCOS.
as levonorgestrel, or antiprogestins such as
Answer E is incorrect. Risk factors associ-
mifepristone, are commonly used.
ated with ovarian cancer include nulliparity,
Answer B is incorrect. High-dose estrogen-only positive family history, early age of menarche
pills are equally effective when compared with or late age of menopause, and being of white
the progestin-only preparation. Most clinicians, race. Furthermore, Pap smears do not detect
however, prefer the latter, as the progestin-only ovarian cancer.
pills are administered in one day as compared
with the high-dose estrogen-only pills, which 27. The correct answer is E. The painful vesicular
require ve consecutive days of medication. nature of her lesions and the short incubation
time between exposure and symptoms indicate
Answer D is incorrect. Progestin-only pills
that this patient is suffering from genital her-
such as levonorgestrel are given in two doses
pes. Genital herpes is most often caused by
12 hours apart, within 72 hours of unprotected
herpes simplex virus-2. Valacyclovir is an anti-
intercourse. They are highly effective and have
viral medication that is used to treat primary
a side-effect prole that is much better toler-
and recurrent episodes of genital herpes, as
ated than the estrogen and progestin prepara- well as for suppression therapy to reduce the
tions (the Yuzpe regimen). Data from multiple frequency of genital herpes recurrences.
HIGH-YIELD SYSTEMS
372 Section I: Organ Systems Answers

Answer A is incorrect. Efavirenz is a non- 30. The correct answer is D. This history is one of
nucleoside reverse transcriptase inhibitor. It stress incontinence (urine leakage when intra-
prevents the reverse transcription of virus RNA abdominal pressure is increased during sneez-
into DNA. Thus it is effective treatment for a ing or coughing). The most likely cause of
retrovirus such as HIV. However, it is not ef- stress incontinence in a multiparous woman is
fective against a DNA virus such as herpes sim- sphincter insufciency as a consequence of
plex virus, that does not rely on reverse tran- pelvic oor laxity. Although abdominal or pel-
scriptase for replication. vic radiation therapy can be associated with
stricture formation and detrusor muscle dam-
Answer B is incorrect. Fluconazole is an an-
age, breast cancer radiation is unlikely to be
tifungal agent that inhibits fungal cell mem-
sufciently concentrated in the pelvis to cause
brane synthesis, and thus is not active against
such damage.
the herpes simplex virus. It would not be use-
ful in the treatment of genital herpes. Answer B is incorrect. Patients with acute
urinary retention complain about a sudden
Answer C is incorrect. Metronidazole is an
in ability to urinate. All three of these women
Gynecology

antibiotic that is effective in treating anaerobic


are complaining of incontinence, not reten-
bacteria and some protozoa. It is used to treat
tion. There are many causes of acute urinary
bacterial vaginosis and Trichomonas infections,
retention including alcohol, drugs, neurologic
but is ineffective against viruses such as herpes
damage, benign prostatic hyperplasia, kidney
simplex virus.
stones, and surgical complications. A catheter
Answer D is incorrect. Treatment with topi- can usually be placed through the urethra to
cal trichloroacetic acid is often used to treat the relieve the discomfort. The underlying cause
condyloma acuminata lesions caused by the then needs to be evaluated and appropriate
HPV infection. It is not used in the treatment of steps need to be taken to prevent further com-
the vesicular lesions of genital herpes. plications.
Answer C is incorrect. Patients with a neuro-
Questions 28, 29, and 30 genic bladder may complain of urinary incon-
tinence or urinary retention. Common causes
28. The correct answer is H. The history is most of neurogenic bladder include neuropathy,
consistent with overow incontinence. The trauma, tumors of the nervous system, and
most likely causes of overow incontinence are autoimmune diseases leading to neurological
outow obstruction (e.g., urethral stricture) or damage.
neurologic (e.g., sphincteric spasticity). Recent
surgery and pelvic radiation increase the likeli- Answer E is incorrect. Pyelonephritis is a kid-
hood of stricture formation being the cause of ney parenchymal infection most commonly
this womans incontinence. caused by gram-negative aerobic bacteria such
as Escherichia coli, the offending organism in
29. The correct answer is A. This woman presents about 80% of cases. The organisms generally
with classic signs of acute cystitis: urgency, ex- spread by ascending from the lower urinary
treme dysuria, and lack of fever. Acute cystitis tract. The patients symptoms are generally
is commonly caused by coliform bacteria as- characterized by dysuria, urgency, frequency,
cending along the urethra. It is commonly seen costovertebral tenderness, fever, chills, nausea,
in young women after sexual intercourse or and vomiting. Urinalysis will reveal pyuria and
urethral trauma. Urinalysis is usually positive bacteriuria, including a positive culture from a
for WBCs and RBCs, and the urine may con- clean mid-stream urine catch. Hospitalization,
tain frank blood. Pyelonephritis also will result intravenous uids, and parenteral antibiotics
in urinalysis results showing WBCs, RBCs, and are the mainstay of treatment. Failure to treat
bacteria, but patients also usually present with pyelonephritis early can result in acute paren-
high fevers, and can have costovertebral angle chymal injury, impaired renal tubular func-
tenderness. tion, bacteremia, or sepsis leading to death.
HIGH-YIELD SYSTEMS
Chapter 12: Gynecology Answers 373

Answer F is incorrect. A stula refers to any Answer G is incorrect. Urethral sphincter


abnormal connection. A urethral stula can insufciency generally results as a complica-
form between the urethra and the skin, the rec- tion of surgery to the prostate or pelvic region.
tum, or a blood vessel. It is usually a complica- There are two urethral sphincters: the internal
tion resulting from a ureteral stent placement sphincter is under involuntary control and the
or other surgery in the area. Recto-urethral external sphincter is under voluntary control.
stulas can also occur as a result of abnormal Neuronal or mechanical damage to the ner-
development in utero. vous supply or the sphincters themselves can
lead to incontinence.

Gynecology
This page intentionally left blank
CHAPTER 13

Psychiatry

375
HIGH-YIELD SYSTEMS
376 Section I: Organ Systems Questions

Q U E ST I O N S

1. A 79-year-old man is admitted to the hospital 3. A 14-month-old boy is brought to the clinic by
for an elective total knee replacement. He lives his mother because he has appeared drowsy for
by himself and performs all of his activities of the past 2 days. The child is unresponsive to
daily living. His medical history includes de- verbal stimuli, but on administering a sternal
generative joint disease, coronary heart disease, rub, the child exes his arms and legs. His
and hypertension. He has no history of psychi- blood pressure is 100/60 mm Hg, pulse is 60/
atric problems or alcohol and drug history. In min, respiratory rate is 8/min, and temperature
the evening, several hours after an uneventful is 35.6C (96.0F). On examination the child
surgical procedure, the patient becomes dia- has multiple circular scars over his forearms
phoretic and tachypneic. He is alert, but also and a number of contusions of varying ages
agitated and confused, and cannot give full at- over his lower legs. The mother reports that he
tention to the hospital staff and their questions. bruised his legs while walking around the
house. What additional physical examination
Psychiatry

He does remember his name, but does not be-


lieve that he is in a hospital. Which of the fol- nding would support the most likely diagno-
lowing is the most likely diagnosis? sis?
(A) Brief psychotic episode (A) Bilateral retinal hemorrhages
(B) Delirium (B) Blue sclerae
(C) Dementia (C) Cotton wool spots
(D) Normal aging (D) Positive fecal occult blood test
(E) Pseudodementia (E) Seborrheic keratoses on the head

2. A 32-year-old woman states that over the past 6 4. A 32-year-old man was apprehended by the po-
months she has felt constantly nervous. She lice 2 weeks ago for running across the freeway
adds that sometimes, I feel like my heart is go- without regard for his personal safety. The pa-
ing to burst. She also notes that her heart skips tient stated that he did this for the heck of it.
a beat from time to time, and that she is having The patient appeared distracted during the ini-
trouble sleeping. She denies feeling depressed. tial interview, spoke rapidly, and gave long and
The patient denies ight of ideas, pressured drawn out answers. When asked about his
speech, increased goal-directed activity, hallu- mood, the patient said that he is feeling okay,
cinations, or delusions. The patient also com- Im under the weather, if you get my drift. He
plains of increased bowel movements and was admitted to the inpatient unit for observa-
weight loss, along with signicant weakness tion. Since his admission, he has been saying
when she attempts to climb stairs or lift heavy that he will soon replace the president, that he
items. Her temperature is 37.8C (100.1F), is the next Alexander the Great, and that he
pulse is 102/min, blood pressure is 124/85 mm was told so by voices in his head. The patients
Hg, and respiratory rate is 18/min. Neurologi- admission laboratory values, including a drug
cal examination is signicant only for proximal screen for illicit substances, came back normal.
muscle weakness and hyperactive reexes. Which of the following is the most likely diag-
Which of the following is the best manage- nosis?
ment? (A) Bipolar disorder with psychotic features
(A) Alprazolam (B) Bipolar disorder without psychotic features
(B) Investigation of surreptitious laxative abuse (C) Depression with psychotic features
(C) Methimazole (D) Schizophrenia, disorganized type
(D) Sertraline (E) Schizophrenia, paranoid type
(E) Thyroid hormone replacement
HIGH-YIELD SYSTEMS
Chapter 13: Psychiatry Questions 377

5. A faculty member in physics at a major univer- 7. A 70-year-old schizophrenic patient with a


sity has Aspergers disorder. He appears to stare noted history of violence believes that he is the
through people when talking to them, as if Messiah and that his doctors are Roman in-
he were talking to someone far behind them. dels trying to destroy him. The patient is poorly
This professor has unusually poor social func- groomed, and he is currently yelling at medi-
tioning. Additionally, he appears quite clumsy, cal personnel, saying that in time people will
constantly dropping books and materials in- worship me as a god. Which of the following
volved in his experiments. Which of the fol- should this patient receive immediately?
lowing occurs most commonly in Aspergers
disorder? (A) Clozapine
(B) Electroconvulsive therapy
(A) Delayed language function (C) Haloperidol
(B) Impaired memory (D) Lithium
(C) Intact sense of humor (E) Olanzapine
(D) Low IQ
(E) Poor motor coordination 8. A 2-year-old boy being evaluated for an upper

Psychiatry
respiratory tract infection has very poor eye
6. A 28-year-old woman visits her physician be- contact. The child responds to smiles with dis-
cause she has been experiencing acute attacks interest. When the physician speaks with the
of headache, profuse sweating, and racing child, he continues to say stick while looking
heartbeat during the past 2 months. She also at the otoscope. The mother looks distressed,
has bouts of nausea, abdominal pain, and dysp- and says that this is one of the only words the
nea. She is being treated with clonidine for hy- child speaks. The mother states that her child
pertension without any response. Her blood has no interest in playing with other children.
pressure is 160/80 mm Hg and her heart rate is When the child plays with toys, he is only in-
normal. Laboratory studies show: terested in the individual parts of the toy, such
Na+: 140 mEq/L as the wheels, as opposed to the whole toy.
K+: 4.4 mEq/L Which of the following is the most likely diag-
Cl: 99 mEq/L nosis?
HCO3: 30 mEq/L (A) Age-appropriate behavior
Blood urea nitrogen: 10 mg/dL (B) Aspergers disorder
Creatinine: 0.4 mg/dL (C) Autistic disorder
Serum glucose: 170 mg/dL (D) Mental retardation
Total thyroxine: 8 mg/dL (E) Stranger anxiety
Thyroid-stimulating hormone: 3 mU/mL
Which of the following is the most likely diag-
nosis?
(A) Anxiety disorder
(B) Clonidine withdrawal
(C) Hyperthyroidism
(D) Pheochromocytoma
(E) Primary hyperaldosteronism
HIGH-YIELD SYSTEMS
378 Section I: Organ Systems Questions

9. A 47-year-old woman is brought to the emer- (D) Methamphetamine


gency department by the police after being (E) Phencyclidine
found wandering around a local park in a
nightgown. Her brother is called and he reports 11. A 42-year-old woman is brought to the emer-
a worsening in impulse control in his sister gency department by police after attempting to
over the past 2 years. During this time, she has rob a liquor store because it is what God
become more sexually aggressive, fondling wanted her to do. The patient reports that she
strangers in subways and disrobing in public. believes she is the Messiah and is carrying
Occasionally, she laughs at unknown stimuli the word of the Almighty. She reports she
and cannot be redirected during these emo- hears God talking to her often throughout the
tional outbursts. Over the past month she has day. She states she feels fantastic, and has
become a voracious eater and has gained 4.5 gone almost a week without sleeping. Her
kg (10 lb). She was red from her job as a speech is signicantly pressured, and she is eas-
postal worker 6 months ago for inappropriate ily distracted. Her medical record indicates she
behavior. Her blood pressure is 138/80 mm has had three prior episodes of psychotic symp-
Psychiatry

Hg; she appears older than her stated age, and toms associated with elevated mood. Which of
has a disheveled appearance. She is alert and the following is the most likely diagnosis?
oriented, but cannot recall why she was in the
(A) Bipolar I disorder
park. Physical examination reveals a mild in-
(B) Bipolar II disorder
tention tremor. Her Mini-Mental State Exami- (C) Cyclothymic disorder
nation score is 27, with decits in short-term (D) Schizoaffective disorder
recall. Which of the following is the most likely (E) Schizophrenia
diagnosis?
(A) Attention decit disorder 12. A 18-year-old single mother nds herself being
(B) Diffuse Lewy body dementia overprotective with her child. The mother
(C) Histrionic personality disorder would like to go back to school and pursue her
(D) Huntingtons disease passion for writing but is unable to afford day
(E) Klver-Bucy syndrome care. She often nds herself regretting her
pregnancy. The child is a product of an un-
10. A 19-year-old man is brought to the emergency planned pregnancy. Which of the following is
department after having attacked his next-door the most likely defense mechanism this patient
neighbor with a baseball bat. The police shot is using?
the suspect twice in the leg and once in the (A) Idealization
arm while attempting to subdue him, but six (B) Projection
police ofcers were still required to place him (C) Rationalization
in restraints. At one point during the struggle, (D) Reaction formation
the patient screamed out, I am the hand of (E) Sublimation
God; you cannot beat me! The patients com-
plete blood cell count and serum electrolyte 13. A 41-year-old man has suffered from a phobia
levels are within normal limits, but laboratory of ying on airplanes for as long as he can re-
studies show an elevated level of serum cre- member. He avoids ying when possible and
atine kinase. His urine is positive for myoglobi- worries that the plane will inevitably crash be-
nuria as well as for an illegal substance that is cause he is on it. His mother recently passed
known to block the ion-channel N-methyl-D- away, so he wants to y back home to attend
aspartate receptor complex. Which of the fol- the funeral. He has no history of substance
lowing is the most likely cause of this patients abuse and would like to try something to help
presentation? him cope with the upcoming ight. Which of
(A) Alcohol the following is the most appropriate manage-
(B) Cocaine ment at this time?
(C) Lysergic acid diethylamide
HIGH-YIELD SYSTEMS
Chapter 13: Psychiatry Questions 379

(A) Haloperidol (A) Buspirone


(B) Lorazepam (B) Diazepam
(C) Paroxetine (C) Haloperidol
(D) Phenobarbital (D) Sertraline
(E) Promethazine (E) Valproic acid

14. A 16-year-old girl is brought to the doctors of- 16. The family of a 70-year-old man believes he is
ce by her mother, who is extremely con- depressed. According to his family, the patient
cerned about her eating habits. The mother re- doesnt care about anything anymore. In-
ports that her daughter is becoming very stead, the patient sits in front of the television
preoccupied with her weight. She often refuses watching game shows all day. The family also
to eat, and when she does eat, she immediately states that the patient makes lewd comments to
goes to the bathroom to vomit. A diagnosis of female visitors. On mental status examination
bulimia nervosa is considered. Which of the the patient has unkempt hair, is uncooperative,
following ndings are most likely in this pa- and speaks very slowly. The patient denies trou-

Psychiatry
tient? ble sleeping or concentrating. The patient
looks surprised when asked about guilt and sui-
K+ CL HCO3 cidal ideation, and emphatically states, cer-
CHOICE (mEq/L) (mEq/L) (mEq/L) tainly not! His affect appears constricted. The
patient is oriented to person and place. He has
A 3.4 87 37 a marked impairment in attending to external
stimuli during Mini-Mental State Examina-
B 3.4 100 24 tion. The patient has poor inhibitory functions.
His language and abstraction are intact. MRI
C 4.4 101 12 of the brain shows frontal atrophy and ventric-
ular enlargement. Which of the following is
D 4.5 101 24
the likely cause of the patients current symp-
toms?
(A) A (A) Alzheimers dementia
(B) B (B) Depression
(C) C (C) Frontotemporal dementia
(D) D (D) Lewy body dementia
(E) Subcortical dementia
15. An 8-year-old girl is brought to her pediatrician
by her mother because of odd behavior. The 17. A 32-year-old woman with a history of major
mother has noticed that over the past year her depressive disorder is found lying on the oor
daughters arms and legs jerk wildly out of the in confusion, with muscles twitching, ushing,
blue. Lately, her one eye has started to blink and dilated pupils. On arrival to the emergency
out of sequence with the other and she began department she is found to have a widened
licking her lips so frequently that they are al- QRS complex. On which of the following
ways chapped. The mother states that as a tod- medications did she most likely overdose?
dler her daughter exhibited strange behavior,
like stacking a deck of playing cards on top of (A) Amitriptyline
itself repeatedly for hours at a time. Addition- (B) Bupropion
ally, the mother states that the child is very (C) Fluoxetine
pleasant and likable, with many friends. Which (D) Lithium
of the following is the most appropriate phar- (E) Phenelzine
macotherapy?
HIGH-YIELD SYSTEMS
380 Section I: Organ Systems Questions

18. The behavior of a 2-year-old is evaluated by 20. A 4-year-old child is brought to his pediatrician
a psychiatrist. An experiment is conducted by his mother and father. They are worried that
whereby the mother leaves the playroom. The their baby-sitter may be hurting their child.
child begins to cry. When the mother returns, They left the child with a baby-sitter for the
the child is extremely enthusiastic. Which of rst time 2 days ago. When the parents re-
the following behaviors is the child exhibiting? turned, the child was crying, and pleaded for
them to never leave again! When the pedia-
(A) Anxious attachment
trician questions the child alone, he denies any
(B) Attachment disorder
poor treatment by the baby-sitter or parents. He
(C) Secure attachment
simply says, mommy and daddy could get sick
(D) Stranger anxiety
and die when they are gone. The childs baby-
(E) Symbolism-based behavior
sitter, when questioned by phone, states that
19. A mother brings her 10-month-old daughter to the child wont stop crying when his parents
the pediatrician for a check-up. She is con- leave. When asked how he feels while his par-
cerned because the girl is not walking yet, un- ents are away, the child says, it makes me feel
Psychiatry

like her older brother was at that age. She can sick when they are gone. The child denies dis-
crawl and stand unassisted, uses a three- comfort when he is with his parents. On physi-
ngered pincer grasp when playing with her cal examination the child has no stigmata of
toys, babbles but does not say any recognizable abuse, such as scars, bruises, burns, or genital
words, and is able to wave bye-bye. Accord- lesions. His abdomen is soft, nontender, and
ing to the table below, which of the following nondistended. The child does not complain of
describes this childs development? bone pain. Which of the following is the most
appropriate next step?
(A) A
(B) B (A) Call child protective services
(C) C (B) Diagnose a likely separation anxiety disor-
(D) D der
(E) E (C) Diagnose the patient with a generalized
anxiety disorder
(D) Image the childs bones
(E) Recommend upper endoscopy

GROSS MOTOR FINE MOTOR LANGUAGE SOCIAL/COGNITIVE


CHOICE DEVELOPMENT DEVELOPMENT DEVELOPMENT DEVELOPMENT

A delayed normal normal delayed

B delayed normal normal normal

C normal delayed normal normal

D normal normal delayed delayed

E normal normal delayed normal


HIGH-YIELD SYSTEMS
Chapter 13: Psychiatry Questions 381

21. A 62-year-old man complains that he is feeling 23. A 12-year-old girl is recently diagnosed with at-
depressed. He says that over the past 2 years, tention decit/hyperactivity disorder and is
he has begun to have a vague feeling of sadness. started on a psychostimulant medication. She
In addition, he notes that he is extremely fa- has been previously healthy and has had nor-
tigued during the day, frequently nodding off to mal physical development prior to the diagno-
sleep at work. Before going to work, he fre- sis. Over the past month, however, she has ex-
quently takes ibuprofen to relieve nearly daily perienced nausea, nightmares, intermittent
morning headaches. When asked about changes bilateral calf pain, and irritability, and is cur-
in his life over the past 2 years, he reports that rently at the 35th percentile for height and
he has gained a signicant amount of weight weight for girls her age. At the time the medi-
due to job stress. Further questioning reveals cation was started 2 months ago, she was in the
that he does not feel guilty, has no suicidal ide- 50th percentile for height and weight. Her phy-
ation, and still enjoys his favorite pastime of sician recommends a drug holiday. Which of
reading mystery novels and holding a biweekly these signs and symptoms is likely to be amel-
book club with friends. The patients body mass iorated during a drug holiday?

Psychiatry
index is 42 kg/m. Results of physical examina-
(A) Calf pain
tion are within normal limits. Which of the fol-
(B) Decreased growth velocity
lowing is most appropriate for this patient?
(C) Irritability
(A) Methylphenidate (D) Nausea
(B) Modanil (E) Nightmares
(C) Sertraline
(D) Thyroid hormone replacement 24. A 42-year-old woman presents to an outpatient
(E) Weight loss and continuous positive airway psychiatry clinic. She has a 9-year history of de-
pressure pression and a 15-year history of substance
abuse, including cocaine, amphetamines, and
22. A 43-year-old woman with a history of bipolar opioids. She states that she has been feeling
disease is brought to the emergency depart- suicidal due to a greater depression than ever
ment after her husband found her unarousable before. She also admits to feeling fatigued and
at home. The staff discover that she ingested a having a ravenous appetite. When asked about
bottle of Tylenol and a bottle of alcohol before recent substance abuse, she gives vague an-
passing out. After being stabilized on the inpa- swers and states, I cant really remember now.
tient medical unit, she is transferred to an inpa- I dont want to talk about it. On mental status
tient psychiatry ward, where her psychiatrist examination she displays severe psychomotor
begins treatment for her bipolar disease. Two retardation. Her vital signs are stable. Which
days later she complains of a skin rash. Which of the following is the most likely cause of the
of the following drugs is most likely responsible patients symptoms?
for this rash?
(A) Cocaine intoxication
(A) Lamotrigine (B) Cocaine withdrawal
(B) Lithium (C) Opioid intoxication
(C) Olanzapine (D) Opioid withdrawal
(D) Paroxetine (E) Typical depression
(E) Valproic acid
HIGH-YIELD SYSTEMS
382 Section I: Organ Systems Questions

25. A 6-year-old girl is brought to the outpatient (A) One or more attempts to stop drinking
clinic for a check-up. She reached all develop- over the past 6 months
mental milestones until the age of 24 months. (B) The fact that he has been arrested twice in
Since then, she has become more distant and the past 6 months on a drunk and disor-
less socially engaging. She rarely initiates social derly charge
interactions and has a short temper. Verbal lan- (C) The fact that he has been red from three
guage consists mostly of grunting and loud jobs in the past year for coming to work
nonverbal yelling, a severe decline from her drunk
toddler years when she could speak in three- to (D) The fact that his girlfriend has been threat-
four-word sentences. She has never been com- ening to leave him because of his drinking
pletely toilet trained and has difculty follow- (E) The recurrent use of alcohol while driving
ing verbal commands from her caregivers. in the past 6 months
Which of the following is the most likely disor-
der in this child? 28. A 29-year-old woman is referred to a psychia-
trist by a primary care physician. The physician
(A) Aspergers disorder
Psychiatry

informs the psychiatrist that he believes the pa-


(B) Autism
tient is exhibiting a somatization disorder.
(C) Childhood disintegrative disorder
Which of the following behaviors and symp-
(D) Fragile X syndrome
toms will the psychiatrist note if this is the cor-
(E) Retts disorder
rect diagnosis?
26. A 29-year-old man presents to the psychia- (A) A variety of complaints, including at least
trist at the request of his wife. The patient has two abdominal and one neurologic
received psychiatric care throughout his life, (B) La belle indiffrence
due to his repeated ghts at school, bullying, (C) Physical symptoms that precisely mimic a
taunting, and aggressive disrespect for others textbook medical disorder
throughout his childhood and adolescence. (D) Pleasure and satisfaction on the part of the
The patient relates a history of lying to his wife patient if she is offered admission to the
and to other therapists, of which he seems hospital medical service
quite proud. He has cheated on his wife many (E) Relief on the part of the patient when she
times but appears to experience little remorse is informed that her problem is psychologi-
about his actions. His wife demanded that he cal in nature
seek therapy after she learned of one of his af-
fairs. What was the patients most likely diagno- 29. A 15-year-old boy is brought to the pediatrician
sis as a child? by his father because he has been acting like
a girl. When questioned alone, the patient
(A) Antisocial personality disorder
states that he is uncomfortable with his life. He
(B) Bipolar disorder
says he has always had more feminine interests
(C) Conduct disorder
than most of his same-gender friends. He tries
(D) Depression
to t in, but he simply feels more comfortable
(E) Substance abuse
when he is hanging out with women and play-
ing with dolls. The patient confesses that he
27. A 54-year-old man is brought to the emergency
has, for most of his life, believed that he should
department in an altered mental state. He is
have been born as a girl. Which of the follow-
exhibiting ataxia, nystagmus, and dysarthria
ing is the best initial treatment option for this
and appears very somnolent. After several
patient?
hours, he returns to his normal state of health.
When questioned about his presentation, he (A) Alprazolam
replies that he just had a few too many to (B) Hormonal therapy
drink. Which of the following best supports (C) Paroxetine
the theory that this patient has alcohol depen- (D) Psychological therapy to help the patient
dence? adjust to his biological gender
(E) Sex reassignment surgery
HIGH-YIELD SYSTEMS
Chapter 13: Psychiatry Questions 383

30. An 18-year-old woman is brought to the emer- physician about the verbal abuse that she suf-
gency department by her sister after she was fered during her childhood from a mother who
found lying asleep in bed, next to an empty gave her notably inconsistent parenting. She
bottle of acetaminophen. The sister thinks that says that she currently feels empty inside, like
something is up with the patients boyfriend she has no direction. She discusses the large
because she had returned home the previous number of tumultuous relationships that she
night horribly upset and in tears. When asked has had throughout her life, giving vague rea-
how many acetaminophen pills were in the sons as to why they have not worked out. When
bottle, the sister notes that the bottle was nearly the physician asks her to describe herself fur-
empty, and the patient could not have taken ther, she states that she is impulsive. When
any more than six pills. Which of the following the physician suggests that her personality
questions is the most helpful to ask the patient structure may be to blame, she screams,
during this interview? Youre just like the rest of them! and runs
out. Which of the following is the most likely
(A) Do you ever hear voices or see things that
diagnosis?
other people cant see?

Psychiatry
(B) Have you ever been hospitalized for psy- (A) Antisocial personality disorder
chiatric reasons before? (B) Avoidant personality disorder
(C) Tell me more about your relationship (C) Borderline personality disorder
with your boyfriend (D) Histrionic personality disorder
(D) There is no reason to elicit more informa- (E) Narcissistic personality disorder
tion; every suicidal patient should be hos-
pitalized 33. A 17-year-old high school student is brought to
(E) What recreational drugs do you use? the pediatrician by her mother for fatigue and
dizziness. She has no known medical problems.
31. A 12-year-old boy presented to a physician be- Her weight is within the normal range for her
cause of polyuria, polydipsia, nocturia, and de- height. Her respiratory rate is 10/min, heart rate
creased appetite for 2 months. The patient was is 58/min, and blood pressure lying down is
diagnosed with attention decit/hyperactivity 116/72 mm Hg and standing is 94/56 mm Hg.
disorder when he was 6 years old and is being Oxygen saturation is 99% on room air. On phys-
treated with a stimulant; 9 months ago he was ical examination her skin is dry and doughy, and
diagnosed with bipolar disorder and began her capillary rell is 3.5 seconds. Her cheeks are
treatment with valproic acid and olanzapine. swollen, her oropharynx is clear, and she has
Family history is signicant for bipolar disorder erosion of the enamel on her front teeth. She
and diabetes mellitus. His temperature is has calluses over the rst and second metacar-
36.6C (97.9F), pulse is 76/min, respiratory pophalangeal joints of her right hand. The re-
rate is 18/min, and blood pressure is 110/70 mainder of her physical examination is within
mm Hg. The patient has no abdominal pain, normal limits. Laboratory studies show:
nausea, or vomiting. His serum glucose level is
Complete blood cell count: Normal
300 mg/dL, and urine ketones are found in
HCO3: 32 mEq/L
trace amounts. Which of the following is the
Blood urea nitrogen: 44 mg/dL
most appropriate next step in management?
Creatinine: 0.6 mg/dL
(A) Discontinue olanzapine Serum amylase: 142 U/L
(B) Discontinue valproic acid Lipase: Normal
(C) Monitor blood sugar for the next 6 months
Which of the following is the most likely expla-
(D) Recommend diet and exercise
nation of these laboratory values?
(E) Start oral hypoglycemics
(A) Acute pancreatitis
32. A 24-year-old female psychiatry patient is ex- (B) Acute renal failure
tremely interested in everything the physician (C) Cardiac arrhythmia
says, exclaiming, Wow, doctor, you really un- (D) Respiratory acidosis
derstand me, youre the best! She tells the (E) Self-induced vomiting
HIGH-YIELD SYSTEMS
384 Section I: Organ Systems Questions

34. A man attends three sessions per week with his was found smoking with his friends at a neigh-
psychiatrist. During each session, he begins to bors house. Which of the following is the most
talk, saying whatever comes to his mind. Occa- likely diagnosis?
sionally, the patient relates dreams that he re-
(A) Antisocial personality disorder
members from the previous night. The psychi-
(B) Attention decit disorder
atrist remains silent for the majority of the
(C) Conduct disorder
session, asking only about details to clarify
(D) Normal preadolescent development
what the patient says. This is an example of
(E) Oppositional deant disorder
which of the following types of therapy?
(A) Behavioral therapy 37. A 38-year-old woman is brought to the physi-
(B) Cognitive therapy cian by her family because of severe sadness
(C) Interpersonal psychotherapy after the passing of her father two years prior.
(D) Psychoanalysis Since his death she has had a signicant
(E) Supportive psychotherapy weight loss, feels constantly fatigued, and is
having difculty sleeping. For the past couple
Psychiatry

35. A 63-year-old white woman with a 15-year his- months she has been feeling worse and has
tory of schizophrenia is admitted to the inpa- needed help from her family to take care of her
tient psychiatric ward after she was found in- children. She is convinced she is to blame for
side a department store wearing clothing that her fathers death; yet multiple family members
did not belong to her. On mental status exami- assure her and the doctor that her father passed
nation, the patient is sitting motionless in a away after a short battle with advanced cancer
chair. Her responses to questioning exhibit and the patient had nothing to do with his
paucity of content. The patient also displays death. Nonetheless, the patient reports that she
echopraxia while answering questions. On ex- hears her fathers voice several times a week
amination she displays increased tone in the telling her she could have done more to save
arms and legs that is proportional to the him. Which of the following is the most likely
amount of force used during the examination. diagnosis?
The patient also seems to leave her arms and
(A) Bipolar depression
legs in strange positions for extended periods.
(B) Cyclothymic disorder
Her temperature is 36.8C (98.2F), heart rate
(C) Dysthymic disorder
is 86/min, blood pressure is 123/76 mm Hg,
(D) Major depressive disorder
and respiratory rate is 16/min. Signicant med-
(E) Major depressive disorder with psychotic
ications include haloperidol 15 mg once daily.
features
Which of the following is the most likely diag-
nosis?
38. A 34-year-old man who was previously em-
(A) Catatonia ployed as a nurse presents to the emergency
(B) Dystonia department with dizziness and weakness. He
(C) Mental retardation states that for the past few weeks he has experi-
(D) Neuroleptic malignant syndrome enced a number of episodes of fainting spells,
(E) Serotonin syndrome weakness, and stomach aches. After a nger-
stick test reveals a serum glucose level of 24
36. A 12-year-old boy is referred by a teacher to the mg/dL, the patient is admitted to the medical
school psychologist because of his disruptive service, where his glucose level is stabilized.
behavior. The teacher states that he has been Subsequent laboratory studies show a low
disturbing the class for the past 8 months by C-peptide level, a high serum insulin level,
talking back to her and repeatedly calling out and positive titers for insulin antibodies. Which
during class without obeying the rules. His of the following is the best next step in man-
mother notes that he has been increasingly re- agement?
sentful and angry with her over mundane tasks,
(A) CT of the abdomen
such as cleaning his room. He has been caught
(B) Endoscopy
sneaking out through his window at night and
HIGH-YIELD SYSTEMS
Chapter 13: Psychiatry Questions 385

(C) Initiation of dialysis 41. A 67-year-old homeless man presents to the


(D) Institution of insulin therapy emergency department with hypertension, dia-
(E) Psychiatric consultation phoresis, tachycardia, and tremor. He is agi-
tated and confused, stating that he can feel
E X T E N D E D M ATC H I N G
rats crawling all over him. Physical examina-
tion reveals that he has ascites and a liver edge
that is palpable 6 cm below the costal margin.
The response options for the next 3 items are
the same. Select one answer for each item
in the set. The response options for the next 3 items are
the same. Select one answer for each item
For each of the following scenarios, choose the cor- in the set.
rect antidote or treatment.
Match each of the following adverse effects with
(A) Activated charcoal the drug with which it is most closely associated.
(B) Atropine
(A) Amitriptyline

Psychiatry
(C) Benzodiazepines
(D) Deferoxamine (B) Buspirone
(E) Dialysis (C) Clonazepam
(F) Dimercaprol (D) Clonidine
(G) Flumazenil (E) Clozapine
(H) Glucagon (F) Fluoxetine
(I) Haloperidol (G) Haloperidol
(J) Hyperbaric oxygen (H) Lamotrigine
(K) N-acetylcysteine (I) Lithium
(L) Naloxone (J) Olanzapine
(M)Vitamin K (K) Reserpine
(L) Risperidone
39. A 23-year-old woman is brought to the emer- (M)Sertraline
gency department by a group of friends who (N) Topiramate
leave immediately after dropping her off. The (O) Valproic acid
woman is unresponsive, with tightly constricted (P) Venlafaxine
pupils and a respiratory rate of 6/min.
42. Requires weekly complete blood count moni-
40. A 35-year-old businessman is brought to the toring for a dangerous adverse effect in 1%2%
emergency department by paramedics, who of patients.
were summoned by the police during a routine
trafc stop. The patient is jittery and exuber- 43. Is most associated with the metabolic syn-
ant. He is talking rapidly and is difcult to ex- drome of dyslipidemia, hyperglycemia, and
amine, but his pupils are large, and he is sweat- weight gain.
ing. Further examination reveals his heart rate
44. Is most associated with potentially life-threaten-
is 120/min and blood pressure is 150/100 mm
ing skin eruptions.
Hg. While in the emergency department he
becomes very agitated, struggling with the
nurses and stating that the doctors are out to
get me.
HIGH-YIELD SYSTEMS
386 Section I: Organ Systems Questions

The response options for the next 3 items are


wife is either extremely irritable and starting
the same. Select one answer for each item
ghts, or she is in bed depressed. She was
in the set
red from her job 3 weeks ago for inconsistent
performance.
Match each vignette with the most likely psychiat-
ric disorder. The response options for the next 3 items are
the same. Select one answer for each item
(A) Antisocial personality disorder
in the set
(B) Brief psychotic disorder
(C) Conversion disorder
Match each of the following clinical scenarios with
(D) Cyclothymia
the most appropriate description.
(E) Generalized anxiety disorder
(F) Narcolepsy (A) Agoraphobia
(G) Obsessive-compulsive disorder (B) Akathisia
(H) Oppositional deant disorder (C) Anhedonia
(I) Panic disorder (D) Ataxia
Psychiatry

(J) Posttraumatic stress disorder (E) Catatonia


(K) Retts syndrome (F) Compulsion
(L) Schizoaffective disorder (G) Delirium
(M)Schizophrenia, paranoid subtype (H) Delusion
(N) Social phobia (I) Dyskinesia
(O) Tourettes disorder (J) Dysthymia
(K) Echolalia
45. A 34-year-old man begins to sweat profusely ev- (L) Flight of ideas
ery time he has to give a presentation at the of- (M)Grandiosity
ce. The sweating causes such marked anxiety (N) Hypochondriasis
at each presentation that he soon develops pal- (O) Obsession
pitations, dizziness, and difculty breathing. (P) Somatization
He nds that if he has several alcoholic drinks
before the presentation, he feels calmer and 48. Several weeks after starting a new medication,
can complete the talk with less distress. a woman with a history of schizoaffective disor-
der returns to her psychiatrist complaining that
46. Two days after her husband was killed in a car she just cant sit still and dgets constantly.
accident, a 37-year-old woman became se-
verely agitated, yelling and screaming, using 49. Any time a young woman is aboard a bus or
expletives, and threatening one of her children subway, stands in line in a crowded supermar-
with a knife. Her son called emergency medi- ket, or crosses a bridge, she develops a rapid
cal services and she was taken to the nearest heartbeat, difculty breathing, sweating, nau-
emergency department, where she responded sea, and dizziness. She soon stops leaving the
to intramuscular haloperidol and was eventu- house for fear of developing the symptoms.
ally admitted. Four days later her agitation had
ceased, she was sincerely apologetic for her be- 50. A 27-year-old man has been hospitalized eight
havior, and she returned to her premorbid times for acute schizophrenic episodes. When
level of functioning. seen by his physician, he refuses to answer any
questions, merely repeating whatever has been
47. The husband of a 26-year-old woman consults said to him.
his lawyer for a divorce. For the past 2 years he
doesnt know whom hell come home to. His
HIGH-YIELD SYSTEMS
Chapter 13: Psychiatry Answers 387

AN S W E R S

1. The correct answer is B. The most likely diag- Answer A is incorrect. Anxiety disorder is cer-
nosis for this patient is delirium. Clinical fea- tainly in the differential for this patient. Ben-
tures typical of delirium as opposed to demen- zodiazepines such as alprazolam are the appro-
tia include a relatively rapid onset, a uctuating priate treatment for anxiety disorder. Medical
and cloudy level of consciousness, impaired causes of her anxiety, however, would have to
orientation, disordered thinking, impairment be ruled out rst before she is considered to
of recent memory only, and reversibility of the have an anxiety disorder.
condition.
Answer B is incorrect. The patient does com-
Answer A is incorrect. A brief psychotic epi- plain of weight loss, but does not have any
sode is characterized by schizophrenia-like complaints consistent with metabolic abnor-
symptoms that last less than a month. The malities (e.g., hyperphosphatemia), making
patient must experience delusions, hallucina- laxative abuse a less likely explanation. Also,

Psychiatry
tions, or disorganized speech in order to be di- her other symptoms and complaints can be ex-
agnosed with this condition. In some cases of plained well by the diagnosis of hyperthyroid-
delirium the patient may experience halluci- ism.
nations (especially visual); however, in contrast
Answer D is incorrect. The patient does not
to a brief psychotic episode, delirium is usually
describe any signicant signs of depression,
traceable to an underlying medical condition.
such as guilt, decreased interest in pleasurable
Answer C is incorrect. In contrast to delirium, activity, or decreased concentration. Thus, an
dementia is a chronic disease with an insidious antidepressant is not indicated.
onset and a progressive course. The patients
Answer E is incorrect. Hypothyroidism more
level of consciousness is generally not affected,
often presents as depression than as anxiety.
orientation is initially intact, recent and re-
Other signs of hypothyroidism include deepen-
mote memory are impaired, and the majority
ing of the voice, lack of energy, constipation,
of cases of dementia are not reversible.
abnormal sensitivity to cold temperatures,
Answer D is incorrect. Age-related cognitive weight gain (often in spite of a poor appetite),
decline (normal aging) involves a decreased and dry skin and hair.
ability to learn new information. It does not
include clouding of consciousness, change of 3. The correct answer is A. Based on the incon-
mental status, or problems with orientation. sistency between the mothers explanation of
the childs symptoms and the observed physical
Answer E is incorrect. Pseudodementia is the
ndings, the scenario is highly worrisome for
term given to patients that experience cognitive
child abuse. Given the childs altered mental
slippage in relation to depression. The patient in
status, it is likely that this toddler has also suf-
this case does not show evidence of depression.
fered diffuse brain injury. An ophthalmologic
examination would likely reveal retinal hemor-
2. The correct answer is C. In considering anxi-
rhages if this child sustained intracranial
ety disorders, one must rst always rule out hy-
trauma. Other indications of head trauma
perthyroidism. Typical physical manifestations
might include subdural hemorrhage(s) and/or
of hyperthyroidism include weight loss with
white matter changes visible on brain imaging
preserved appetite, heat intolerance, proximal
studies. All suspected cases of child abuse and
muscle weakness, and increased frequency of
neglect must be reported to the authorities.
bowel movements. Other general signs include
hyperactivity, tachycardia, palpitations, and hy- Answer B is incorrect. Blue sclerae are a nd-
perreexia. Psychiatric manifestations include ing in patients with osteogenesis imperfecta.
anxiety and insomnia. Methimazole, a thyroid This disease can be mistaken for child abuse
inhibitor, is appropriate therapy in this case. because the patients bones are frail and present
HIGH-YIELD SYSTEMS
388 Section I: Organ Systems Answers

with recurrent fractures at a young age. This pa- poor motivation, self-neglect, reduced emotion,
tient has no evidence of fractures and has an al- poor attention, constricted affect, and paucity of
tered mental status consistent with head injury. speech. Moreover, in the disorganized schizo-
phrenic patient, the thought process would be
Answer C is incorrect. Cotton wool spots are
far more difcult to understand due to loose
found in patients with a long-standing history
associations. The patient is delusional, but his
of hypertension and are generally not observed
thought process can be understood.
in otherwise healthy toddlers.
Answer E is incorrect. Although this patient
Answer D is incorrect. A positive fecal occult
has psychotic features that are seen in schizo-
blood test is a nonspecic test that would not
phrenia, he is not experiencing paranoid
specify a single diagnosis. Nevertheless, in cases
thoughts. His grandiosity and pressured speech
of child abuse, a positive fecal occult blood test
make him more likely to be suffering from bi-
may signal that the child has sustained intra-
polar disorder.
abdominal trauma and should prompt further
medical evaluation.
5. The correct answer is E. Core features of
Psychiatry

Answer E is incorrect. Seborrheic dermatitis Aspergers disorder include a lack of eye con-
is a common nding in infants and tends to tact, a paucity of facial and gestural expres-
be self-limited over time. The rash occurs in a sions, and a at and emotionless tone of voice.
cradle cap distribution on the scalp and pre- Other characteristics include pronounced likes
sents with a characteristic aky, scaly dermatitis and dislikes, repetitive routines or rituals, pecu-
with a stuck-on appearance. It is not indica- liarities in speech and language, the inability
tive of child abuse or neglect. to interact successfully with peers, absent sense
of humor, problems with nonverbal communi-
4. The correct answer is A. The patient exhibits cation, and clumsy and uncoordinated motor
grandiosity, pressured speech, distractibility, movements. There may, however, be special
and an increase in risk-taking behavior, all of abilities (rote memory) interwoven with dis-
which comprise a manic episode because they abilities and unusual interest in natural sci-
have lasted longer than 1 week. Thus, the diag- ences, complex calculations, or calendar cal-
nosis is bipolar disorder. Although schizo- culating.
phrenic patients may also engage in inappro-
Answer A is incorrect. Patients with Aspergers
priate actions such as running across the
disorder often have excellent language func-
freeway, they often relate poorly to the staff,
tion, although they have an impaired ability to
given their more bizarre delusions. Often, pa-
appreciate the meaning of the words they are
tients with schizophrenia also have attened
using.
affect, paucity of speech, and other negative
symptoms later in their illness, which decreases Answer B is incorrect. Patients with Aspergers
their ability to relate to staff. Because the pa- disorder often have tremendous rote memo-
tient is hearing voices, he certainly has psy- ries.
chotic features. Up to 50% of patients with bi-
Answer C is incorrect. Patients with Aspergers
polar disorder have psychotic features.
disorder often have little or no sense of humor.
Answer B is incorrect. Hearing voices is a psy-
Answer D is incorrect. Patients with Aspergers
chotic feature.
disorder often have normal to high IQs.
Answer C is incorrect. The patient states that
his mood is okay, although he throws in some 6. The correct answer is D. Pheochromocytoma
nonsensical pieces of speech after this. Addi- should be suspected in cases of difcult-to-
tionally, most psychotic content in depression control hypertension accompanied by various
is of a negative nature. adrenergic symptoms such as tachycardia,
sweating, nervousness, and orthostasis. These
Answer D is incorrect. This patient lacks the
symptoms are caused by excessive levels of nor-
negative symptoms of schizophrenia, such as
epinephrine or neuropeptide Y. Patients have a
HIGH-YIELD SYSTEMS
Chapter 13: Psychiatry Answers 389

tumor located in either or both adrenals or agitation. These typical neuroleptics have little
anywhere along the sympathetic nervous effect on negative symptoms. It is important to
chain. Additionally, patients may experience note that much of the aggression which occurs
attacks of nausea, abdominal pain, weakness, in schizophrenia is due to the patients poor un-
tremor, visual disturbance, chest pain, and dys- derstanding of reality, as opposed to a premedi-
pnea. Also, mild hyperglycemia is common in tated, instrumental aggression which occurs in
cases of pheochromocytoma. antisocial personality disorder.
Answer A is incorrect. Pheochromocytoma Answer A is incorrect. Atypical neuroleptics,
may be mistaken for an anxiety disorder such including clozapine, are thought to be effec-
as a panic attack given common physical symp- tive for both positive and negative symptoms of
toms of racing heart, sweating and nervous- schizophrenia. The use of clozapine is limited
ness, headache, muscle tension, chest pain, to refractive cases of schizophrenia due to the
and abdominal distress. However, patients with risk of agranulocytosis.
pheochromocytoma more often than not pre-
Answer B is incorrect. Electroconvulsive ther-
sent with hypertension (90%) which may be

Psychiatry
apy plays an important role in the therapy of
sustained or paroxysmal in nature.
treatment-resistant schizophrenia. It should be
Answer B is incorrect. Clonidine withdrawal considered after 34 drug trials, and is likely
syndrome can result from the abrupt with- most effective in patients with affective symp-
drawal or tapering of clonidine causing an toms. Electroconvulsive therapy plays no role in
hyperadrenergic state that mimics pheochro- the initial management of a patient with active
mocytoma. The syndrome consists of nausea, psychosis.
palpitation, anxiety, sweating, and headache,
Answer D is incorrect. Lithium is a mood stabi-
along with an elevation in blood pressure. In
lizing agent that is rst-line treatment for bipolar
this clinical scenario, the patient has remained
disorder. Approximately 80% of manic patients
on clonidine despite a poor response.
also respond to this treatment; however, a mea-
Answer C is incorrect. Tachycardia, heat in- surable response typically takes 12 weeks. As
tolerance, weight loss, and anxiety are also such, it is not able to control psychosis and agi-
features of hyperthyroidism. However, in this tation in the initial period of treatment, but may
clinical scenario, serum thyroxine and thyroid- be co-administered with another antipsychotic
stimulating hormone levels are within the nor- medication. Lithium is also recommended for
mal range and therefore rule out the diagnosis the treatment of schizophrenic patients who
of hyperthyroidism. have not achieved a signicant response to neu-
roleptics alone. Lithium may be an effective ad-
Answer E is incorrect. Patients with pri-
junctive treatment for patients with signicant
mary hyperaldosteronism typically have other
affective symptoms.
physical and biochemical ndings such as hy-
pokalemia (the patient in this clinical scenario Answer E is incorrect. Olanzapine is another
has normal potassium) and alkalosis. atypical antipsychotic that can be very effective
for treating patients with negative symptoms. It
7. The correct answer is C. Once medical causes would not be the treatment of choice for this
have been excluded, the determination of safety patient, as he exhibits positive symptoms.
is important in treating psychiatric illness. It is
necessary to maintain a controlled environment 8. The correct answer is C. Patients with autism
for the safety of both the patient and staff. Initial have impairment in three major areas. The rst
management of this patient includes an antipsy- area is impairment in social interaction. The
chotic, such as haloperidol, for psychosis and child will make little eye contact, has few peer
agitation. Therapy with typical neuroleptics, interactions, and will not engage in appropriate
such as haloperidol and chlorpromazine, is fre- interest-sharing activities, such as showing par-
quently successful in amelioration of positive ents toys they are interested in. The patient also
symptoms such as hallucinations, delusions, and has difculty with communication, learning and
HIGH-YIELD SYSTEMS
390 Section I: Organ Systems Answers

using few words. Words that are used are ste- 9. The correct answer is E. Klver-Bucy syndrome
reotyped and used incorrectly. Finally, patients is classied as a syndrome within the frontal
display restricted repetitive and stereotyped pat- lobe dementias. Classic symptoms include hy-
terns of behavior, interests, and activities, and perorality, hypersexuality, and perseverative
interest in parts of objects. The disorder is of- speech or behavior. Other symptoms include
ten conceptualized as missing the forest for apathy, personality changes, and amotivation.
the trees. Communication and social interac-
Answer A is incorrect. Attention decit disor-
tion may require a certain integrative function
der may present in adults, but it is character-
that autistic children lack. The pathogenesis of
ized by inability to focus on tasks, and difculty
the disorder, however, remains a mystery.
attending to details.
Answer A is incorrect. The fact that the pa-
Answer B is incorrect. Diffuse Lewy body dis-
tient does not respond to social cues such as
ease has prominent psychiatric signs such as
smiles is not normal at any age. Moreover, only
personality changes, depression, and halluci-
using one word at age 3 is not age-appropriate
nations. Patients often have weight loss, rather
behavior. By age 3, children should be putting
Psychiatry

than weight gain. Fluctuating mental status


together three-word sentences, along with rid-
and cognitive impairments are common. This
ing a tricycle and playing with other children.
patient, however, ts the classic presentation of
Answer B is incorrect. Differentiating Asperg- Klver-Bucy syndrome.
ers disorder from autism can be difcult. Chil-
Answer C is incorrect. Histrionic personal-
dren with Aspergers disorder often display
ity disorder is a persistent pattern of behaviors
similar poor social interaction and stereotyped
present throughout a patients life. Prominent
interests. What differentiates the two is that in
signs include inappropriate sexual behavior
Aspergers disorder, social communication is
and unstable relationships.
intact. In fact, speech in these patients often
develops early, and often has no prosody in it. Answer D is incorrect. Huntingtons disease is
These patients are often called little profes- a familial dementia with anticipatory expres-
sors for their poor social interaction, narrowed sion. It is caused by a triplet repeat and succes-
interests, and excellent speech. sive generations tend to express the mutation
earlier in age. Symptoms of the disease include
Answer D is incorrect. Mental retardation is
difculty in speaking and swallowing, involun-
dened as an IQ <70 with concurrent dysfunc-
tary movements, cognitive impairment, and
tion in adaptive behavior as expressed in con-
depression. Uncommonly, patients can present
ceptual (e.g., reading, writing, and money con-
with delusions, hallucinations, and obsessive
cepts), social (e.g., self-esteem and gullibility),
compulsive disorders. It is uniformly fatal.
and practical adaptive skills (e.g., personal ac-
tivities of daily living). This disorder can cause 10. The correct answer is E. This patient is suffer-
difculty in social and communication func- ing from a phencyclidine (PCP)-induced psy-
tions. The patients poor eye contact and social chosis. PCP is a dissociative drug that causes
relatedness point toward autism. It is often very users to feel detached from their bodies and
difcult to tell these two disorders apart. from their surroundings. Users generally feel
Answer E is incorrect. Peak stranger anxiety numb and may have a sense of invulnerability,
occurs at 810 months of age. The child has rendering them highly dangerous both to
integrated the idea that he or she is separate themselves and to others. PCP noncompeti-
from mother and is separate from the rest of tively binds to the N-methyl-D-aspartate recep-
the world. The child feels comfortable with tor complex and inactivates the channel, re-
the mother, but is not comfortable with strang- sulting in a blockade of the excitatory central
ers in the world. This fear, which occurs be- nervous system amino acids glutamate and as-
tween 6 months and 2 years of age, would not partate. The drug is excreted in the urine and
account for the behavior described in the stem. can be screened for in suspected patients who
present with psychotic symptoms. Treatment
HIGH-YIELD SYSTEMS
Chapter 13: Psychiatry Answers 391

consists mainly of benzodiazepines for seda- Answer B is incorrect. Bipolar II disorder in-
tion. Psychosis can be treated with an antipsy- volves episodes of hypomania rather than ma-
chotic with low anticholinergic activity (PCP nia. Hypomania can be thought of as a less se-
is an anticholinergic). For patients who ingest vere form of mania that does not signicantly
the drug, gastric lavage or charcoal can reduce impact social functioning. A hypomanic may
the absorption of the drug via the gastrointesti- engage in reckless or objectionable behaviors,
nal tract. but does not exhibit psychotic features such as
hallucinations or delusions.
Answer A is incorrect. Alcohol is a central ner-
vous system depressant that typically presents Answer C is incorrect. Cyclothymic disorder
with disinhibition, emotional lability, slurred involves hypomanic episodes interspersed be-
speech, ataxia, and somnolence. Frank psycho- tween several years of depressive symptoms.
sis is seen in the context of alcohol withdrawal, Hypomania can be thought of as a less severe
but only rarely is it seen with alcohol intoxica- form of mania that does not signicantly im-
tion. pact social functioning.

Psychiatry
Answer B is incorrect. Cocaine can cause Answer D is incorrect. For the diagnosis of
substance-related psychosis with hallucinations schizoaffective disorder to be made, the patient
and delusions. However, cocaine acts by pre- must have had psychotic symptoms in the ab-
venting the reuptake of dopamine molecules sence of mood symptoms at some point. Since
released in the synaptic clefts of nerves. she does not fulll this criterion, bipolar I is
the better diagnosis at this point. Psychosis is
Answer C is incorrect. This patients presenta-
dened as a psychiatric disorder in which con-
tion is unlikely to be due to lysergic acid dieth-
ceptions of reality are severely impaired.
ylamide (LSD), which typically causes power-
ful sensory disturbances such as the movement Answer E is incorrect. The patients psychotic
of shapes, light, and colors. LSD usually does symptoms are also associated with mood symp-
not produce hallucinations in the strict sense, toms. Hence the diagnosis of classic schizo-
but rather illusions and vivid daydream-like phrenia is inappropriate. Psychosis is dened
scenes. The drug acts on numerous receptors as a psychiatric disorder in which conceptions
in the central nervous system, including dopa- of reality are severely impaired.
mine receptors, adrenoreceptors, and serotonin
receptors. However, LSD does not block N- 12. The correct answer is D. With reaction forma-
methyl-D-aspartate receptors. tion the individual deals with intrapsychic con-
ict by behaviors, thoughts, or feelings that are
Answer D is incorrect. Amphetamines such
the opposite of her own, consciously unaccept-
as methamphetamine are stimulants that can
able thoughts. In this case, a mother subcon-
cause psychosis. However, amphetamines work
sciously wishes she did not have to care for her
by activating the release of norepinephrine and
child instead of pursuing a career. She reacts
dopamine at nerve endings.
against these feelings by acting in an overpro-
tective manner.
11. The correct answer is A. The patient is having
a manic episode with psychotic symptoms. Psy- Answer A is incorrect. In idealization, the indi-
chosis is dened as a psychiatric disorder in vidual deals with emotional conict and distress
which conceptions of reality are severely im- by attributing overly positive qualities to others.
paired. In her medical record, there is no inci-
Answer B is incorrect. When a person is pro-
dent reported in which the patient displayed
jecting they falsely attribute their own unac-
psychotic symptoms in the absence of signi-
ceptable feelings, impulses, or thoughts onto
cant mood changes, which would change the
another.
patients diagnosis to schizoaffective disorder,
thus the patient is given the diagnosis of bipo- Answer C is incorrect. The process of ratio-
lar I disorder. This can be a difcult distinction nalization involves the elaboration of ones
to make clinically. own thoughts, actions, and feelings by way of
HIGH-YIELD SYSTEMS
392 Section I: Organ Systems Answers

reassuring explanations that conceal their true Answer B is incorrect. This laboratory prole
meaning. represents a pure hypokalemia but no meta-
bolic alkalosis.
Answer E is incorrect. Sublimation involves
channeling potentially threatening and mal- Answer C is incorrect. This serum prole is
adaptive feelings and impulses into socially ac- of a patient experiencing normokalemic, nor-
ceptable outlets. mochloremic acidosis.
Answer D is incorrect. This is a normal serum
13. The correct answer is B. Lorazepam is a ben-
prole, which would be unlikely given this pa-
zodiazepine (GABAergic) anxiolytic used to
tients recurrent vomiting.
help patients cope with their phobias on a
short-term basis. When time is a factor in de-
15. The correct answer is C. The girl has
veloping a treatment plan, benzodiazepines are
Tourettes syndrome, which is characterized by
appropriate rst-line agents to produce an im-
motor and vocal tics. Tourettes syndrome is
mediate reduction in symptoms of anxiety.
also associated with obsessive-compulsive
Answer A is incorrect. Haloperidol is a typi- symptoms such as stacking a deck of cards re-
Psychiatry

cal antipsychotic that may be sedating, but peatedly. In terms of pharmacotherapy, the
it would not be used in this case. Rather, it is drugs of choice for Tourettes syndrome are
used to control psychotic symptoms such as haloperidol, pimozide, and clonidine; of these,
hallucinations and delusions. haloperidol has the most benign adverse effect
prole.
Answer C is incorrect. Paroxetine is an antide-
pressant in the selective serotonin reuptake in- Answer A is incorrect. Buspirone is used to
hibitor (SSRI) family. It is effective in treating treat anxiety disorders, not Tourettes syn-
specic phobias and might be helpful in long- drome.
term therapy, but because SSRIs typically re-
Answer B is incorrect. Diazepam is a benzo-
quire 46 weeks to exert therapeutic effects, a
diazepine, which is not used for medical man-
benzodiazepine would produce a much more
agement of Tourettes syndrome.
immediate response suitable for this particular
situation. Answer D is incorrect. Sertraline is an SSRI
used to treat depressive disorders, not Tourettes
Answer D is incorrect. Phenobarbital is a bar-
syndrome.
biturate that has anticonvulsant and sedative
properties. It is not indicated to treat phobias. Answer E is incorrect. Valproic acid can be
used for many disorders such as seizure disor-
Answer E is incorrect. Promethazine is an an-
ders and bipolar disorder. However, it is not
tipsychotic that is effective in treating paranoia
indicated for the treatment of Tourettes syn-
and hallucinations; however, it is not indicated
drome.
for the treatment of specic phobias.
16. The correct answer is C. The MRI demon-
14. The correct answer is A. Recurrent vomiting
strates the frontal atrophy often seen in fronto-
causes hypokalemic, hypochloremic, meta-
temporal dementia. Core features of frontal
bolic alkalosis. This is due to the loss of highly
lobe dementias include insidious onset, grad-
acidic uid from the stomach causing both a
ual progression, and an early decline in social
decrease in circulating blood volume (uid
and interpersonal conduct. Emotional blunt-
loss) and metabolic alkalosis (hydrochloric
ing and apathy also occur early without insight.
acid loss). The decrease in circulating blood
There is a marked decline in personal hygiene,
volume causes activation of the renin-
as well as signicant distractibility and motor
angiotensin-aldosterone system, thus causing
impersistence (failure to maintain a motor ac-
retention of sodium and water while potassium
tivity). In the types of frontal dementia associ-
is lost in the urine.
ated with aphasia, language is affected more
signicantly than personality. Frontal lobe de-
HIGH-YIELD SYSTEMS
Chapter 13: Psychiatry Answers 393

mentias may also cause patients to be apathetic function, recall abnormalities, and visuospa-
when medial frontal damage occurs and disin- tial disturbances. Subcortical dementias may
hibited when basal-frontal dysfunction pre- occur in Parkinsons disease, Huntingtons dis-
dominates. Social withdrawal and behavioral ease, and progressive supranuclear palsy, and
disinhibition may precede the onset of demen- in inammatory, infectious, vascular, and de-
tia by several years. In patients whose frontal myelinating illness. This patient does not have
lobe dementia primarily affects frontal lan- any of the prerequisite disorders for subcortical
guage, loss of spontaneity of speech is often the dementia.
rst noticeable symptom.
17. The correct answer is A. Amitriptyline is a tri-
Answer A is incorrect. In Alzheimers demen-
cyclic antidepressant used to treat major de-
tia, a subjective sense of memory loss appears
pression as well as in the treatment of neuro-
rst, followed by loss of memory detail and
pathic pain. It is associated with primarily
temporal relationships. All areas of memory
anticholinergic adverse effects, sedation, sexual
function deteriorate, including encoding,
dysfunction, and arrhythmias. It can be lethal
retrieval, and consolidation. Patients forget

Psychiatry
in overdose, causing coma, convulsions, and
landmarks in their lives less often than other
cardiac arrhythmias. Overdose is treated with
events. Agnosia (failure to recognize or iden-
intravenous uids for hypotension, sodium bi-
tify objects), aphasia (language disturbance),
carbonate if the QRS interval is >100 msec for
and apraxia occur later; however, a mild am-
cardioprotection, and -adrenergic vasopres-
nestic aphasia may be an early nding. Later,
sors if refractory.
patients with Alzheimers disease become pas-
sive, coarse, and less spontaneous. The patient Answer B is incorrect. Bupropion is a het-
does not seem to have memory problems as a erocyclic antidepressant used in treatment of
signicant aspect of his presentation, making depression as well as for smoking cessation.
this diagnosis less likely. Its most typical adverse effects include head-
ache, dry mouth, insomnia, and dizziness, but
Answer B is incorrect. It is often difcult to
it can also cause stimulant effects, aggravation
differentiate between the anhedonia seen in
of psychosis, and seizures. The medication is
depression and the apathy seen with degenera-
contraindicated in patients with known seizure
tion of the frontal lobe. Clues to the diagnosis
disorder or in those with anorexia nervosa or
of frontotemporal dementia are lack of depres-
bulimia nervosa (due to increase in seizures).
sive symptoms such as guilt and suicidal ide-
ation. Additionally, the patients signs of disin- Answer C is incorrect. Fluoxetine is an SSRI
hibition make the diagnosis of frontotemporal used in the treatment of depression, obsessive-
dementia more likely. compulsive disorder, bulimia nervosa, panic
disorder without agoraphobia, and premen-
Answer D is incorrect. Patients with Lewy
strual dysphoric disorder. Adverse effects in-
body disease present similarly to those with
clude decreased libido, insomnia, headache,
Alzheimers, but with more of a uctuating
anxiety, nervousness, nausea, diarrhea, an-
cognitive impairment that affects memory and
orexia, dry mouth, weakness, and tremor. It is
higher cortical functions. These patients may
relatively safe in overdose, but when used in
present with unexplained delirium. Associated
conjunction with monoamine oxidase inhibi-
features include visual or auditory hallucina-
tors can cause serotonin syndrome (fever, myo-
tions, mild extrapyramidal signs, or repeated
clonus, cardiovascular collapse, and mental
and unexplained falls.
status changes).
Answer E is incorrect. Patients with subcorti-
Answer D is incorrect. Lithium is used in the
cal dementias have a diagnosed disorder of
therapy of bipolar disorder, particularly treat-
deeper brain structures in the presence of a
ment and prophylaxis of bipolar disorder. Ad-
relatively unaffected cerebral cortex. The prin-
verse effects include electrolyte abnormalities,
cipal features of subcortical dementias include
diabetes insipidus, leukocytosis secondary to
slowed mentation, impairment of executive
demargination, and hypothyroidism. It has a
HIGH-YIELD SYSTEMS
394 Section I: Organ Systems Answers

narrow therapeutic range, so levels are checked Answer E is incorrect. Symbolism-based be-
often. In overdose, patients may have confu- havior occurs during Piagets preoperational
sion and seizures. stage of cognitive development. Examples of
this behavior include using a transitional ob-
Answer E is incorrect. Phenelzine is a mono-
ject, such as a blanket, to represent a parent.
amine oxidase inhibitor used in the treatment
The preoperational stage lasts from about 2 to
of depression, especially atypical depression.
7 years of age. The symbolic thinking permits
Common adverse effects include orthostasis
more exibility and planning in parent-child
and sexual dysfunction, but when combined
problem solving during this stage.
with tyramine-containing foods (in addition
to some over-the-counter cold medications),
19. The correct answer is E. This patient has
can cause hypertensive crisis. When used in
achieved normal developmental milestones in
conjunction with other antidepressant medica-
every category except language. By 10 months
tions (e.g., SSRIs), it can cause the serotonin
of age a child should be able to say mama
syndrome, which includes symptoms of confu-
and dada nonspecically, though this child is
sion, tremor, myoclonus, hyperthermia, sinus
unable to do so. The mother should not worry
Psychiatry

tachycardia (not a widened QRS complex),


that her daughter is not walking, however, as
and dilated pupils.
most children do not walk until about 12
months of age. By standing alone, she has
18. The correct answer is C. This child is showing
reached the appropriate developmental mile-
secure attachment style. The childs behavior,
stone for her age.
especially toward the parent upon reunion, is
indicative of the quality of the overall parent- Answer A is incorrect. This patient has normal
child attachment. Children who are distressed gross motor development and social cognitive
when the mother leaves and happy at her re- development, but delayed language develop-
turn are showing secure attachment. ment.
Answer A is incorrect. Children with anxious Answer B is incorrect. This patient has nor-
attachment are extremely depressed when their mal gross motor development and delayed lan-
mother departs. The child will be ambivalent guage development.
when she returns, seeking to remain close to
Answer C is incorrect. This patient does have
the mother but resentful, and resistant when
delayed language development.
the mother initiates attention.
Answer D is incorrect. This patient has nor-
Answer B is incorrect. Attachment disorder oc-
mal social/cognitive development.
curs in children who fail to develop secure at-
tachment to loving, protective caregivers. These 20. The correct answer is B. The child most likely
children are left without an important founda- has separation anxiety disorder, which is devel-
tion for healthy development and may develop opmentally inappropriate and excessive anxiety
emotional, behavioral, social, and developmen- concerning separation from home or from
tal problems. This child is displaying normal se- those to whom the individual is attached. This
cure attachment; therefore, the childs behavior can take the form of recurrent distress when
does not suggest an attachment disorder. separated, excessive worry about harm befall-
Answer D is incorrect. Stranger anxiety occurs ing attachment gures, and repeated night-
between 6 months and 2 years of age, and it is mares with themes of separation.
a normal part of development. It reects pref- Answer A is incorrect. There is no evidence
erential attachment to the mother over other that the baby-sitter is abusing this child. How-
possible attachment gures. Stranger anxiety ever, a high index of suspicion is always neces-
would be reected if the child began crying sary. Police data suggest that roughly 7000 to
when being handled by strangers and if he or 8000 baby-sitter offenses (the majority of which
she was then soothed by the mother. This anxi-
ety is not what is demonstrated.
HIGH-YIELD SYSTEMS
Chapter 13: Psychiatry Answers 395

are sex crimes) are reported to police over the impaired concentration, impotence, and per-
course of a year. sonality changes. Patients suffering from ob-
structive sleep apnea are at increased risk of
Answer C is incorrect. Generalized anxiety
pulmonary hypertension, right-sided heart fail-
disorder in children involves excessive anxi-
ure, stroke, myocardial infarction, and sudden
ety and worry about a number of events or
death. Primary treatment for patients with
activities such as school performance, social
sleep apnea is weight loss and nasal continuous
relations, or clothes, which causes signicant
positive airway pressure.
impairment or distress. It is manifested by so-
matic symptoms, self-consciousness, and social Answer A is incorrect. Narcolepsy, which may
inhibition. In this child, the distress seems to be treated with methylphenidate, is classically
revolve around separation, as opposed to other associated with uncontrollable sleep attacks in
aspects of life. which the patient abruptly falls asleep in inap-
propriate, embarrassing, and even dangerous
Answer D is incorrect. In 2001, child protec-
situations. Examples include falling asleep
tive service agencies investigated more than
while eating, driving, and having intercourse.
3.25 million reports of child abuse and neglect

Psychiatry
Most narcoleptics experience related symp-
in the United States. This is an increase of 2%
toms, including cataplexy (sudden loss of mus-
from the previous year. Teachers, law enforce-
cle tone), hypnagogic hallucinations (dream-
ment ofcers, social service workers, and phy-
like experiences while falling asleep but not
sicians made 56% of the reports. In this case,
yet asleep), and sleep paralysis (brief paralysis
however, there is no evidence that the child
associated with the onset of sleep or wakeful-
has been abused. Thus, further studies, such as
ness). However, only 10%15% of narcoleptic
bone imaging, to ascertain abuse are not nec-
patients will have all four classic symptoms.
essary.
Answer B is incorrect. Modanil, a non-
Answer E is incorrect. The child is likely ex-
amphetamine stimulant, is also used to treat
periencing symptoms of nausea caused by
narcolepsy. It would not be recommended in
the signicant distress of physical separation,
sleep apnea.
which is common in separation anxiety. This
conclusion can be drawn from the fact that the Answer C is incorrect. Although the patient
patient has no physical complaints when the does complain of vague depressive symptoms,
child is with his parents, and the child has no he states that he is not feeling guilty, is not
abdominal tenderness. If this child had physi- having suicidal ideation, and is still able to
cal symptoms, a more complete gastrointesti- enjoy his favorite hobby. Moreover, a patient
nal physical work-up would be indicated. with depression can present with insomnia or
hypersomnia, but this patient reports neither
21. The correct answer is E. Sleep apnea is char- symptom. Rather, he is having difculty stay-
acterized by interrupted breathing of 20 sec- ing awake during the day. This patient would
onds or longer during sleep, and it may be clas- not meet the Diagnostic and Statistical Man-
sied as central, obstructive, or mixed. The ual of Mental Disorders, Fourth Edition, Text
most common of these is obstructive sleep ap- Revision (DSM-IV-TR) criteria for depression,
nea, in which the patient has an intermittent which requires at least ve of the following
upper airway obstruction. Often this airway ob- symptoms to be present in any 2-week period:
struction is from the accumulation of fat on the depressed mood, anhedonia, change in ap-
sides of the upper airway, which causes the air- petite, insomnia or hypersomnia, psychomo-
way to become narrow and predisposed to clo- tor agitation or retardation, fatigue, feelings of
sure when the muscles relax. Obese patients worthlessness, diminished ability to think or
are more prone to this phenomenon. Patients concentrate, or recurrent thoughts of death/
may complain of snoring, morning headaches, suicidal ideation. Thus, an antidepressant
dry mouth on awakening, or gasping during would not be the rst treatment option.
sleep. They may also have depressed mood,
HIGH-YIELD SYSTEMS
396 Section I: Organ Systems Answers

Answer D is incorrect. Hypothyroidism can cents. Studies have shown that growth velocity
certainly cause weight gain and fatigue, along will normalize once the medication is stopped.
with depressed mood. Morning headache, For children whose attention decit disorder is
however, is more characteristic of obstructive well-controlled with psychostimulants, a drug
sleep apnea. Moreover, the patient does not holiday is recommended to allow for catch-
have other manifestations of hypothyroidism up growth.
such as muscle weakness, change in skin or
Answer A is incorrect. Benign musculoskeletal
hair, muscle cramps, or constipation.
pain syndromes are not uncommon in child-
hood and adolescence, a period when rapid
22. The correct answer is A. This patient pre-
anatomic and physiologic changes are taking
sented in the depression phase of her bipolar
place. Calf pain is not associated with psycho-
disease. Many of the commonly used agents for
stimulant medications and would not be re-
bipolar disease are most effective at treating
lieved by discontinuation of the medication.
and preventing manic episodes. Lamotrigine is
an anticonvulsant best known for effectively Answer C is incorrect. Irritability is a non-
treating the depressed phase of bipolar disease.
Psychiatry

specic symptom which may be related to the


However, caution must be taken to monitor for childs attention decit disorder, the psycho-
skin rashes that may present as a hypersensitiv- stimulant medication, expected mood swings
ity reaction or the more serious Stevens- of adolescence, or another cause.
Johnson syndrome.
Answer D is incorrect. Gastrointestinal dis-
Answer B is incorrect. Lithium is a common turbances are occasionally associated with psy-
treatment for bipolar disease and depression chostimulant use; however, such symptoms
with a narrow therapeutic index. The most are often mild and transient. Nausea alone is
common adverse effect of lithium toxicity is a a nonspecic symptom that may or may not be
tremor. directly caused by the medication.
Answer C is incorrect. Olanzapine is an atypi- Answer E is incorrect. Nightmares are not
cal antipsychotic best known for its effective- known to be related to psychostimulant medi-
ness in treating the negative symptoms of cations.
schizophrenia. The most common adverse ef-
fects of olanzapine are weight gain and somno- 24. The correct answer is B. In humans, discontin-
lence. uation of cocaine leads to dysphoria (a so-called
crash). Hypersomnolence and anergia are also
Answer D is incorrect. Paroxetine is a com-
common, along with increased appetite. Pa-
monly prescribed SSRI used in major depres-
tients are treated by allowing them to sleep and
sion. The use of antidepressants alone in bipo-
eat in a supportive environment. No medica-
lar disease increases the risk of manic episodes.
tions have been shown to be effective in treating
Nonetheless, common adverse effects of SSRIs
symptoms associated with cocaine withdrawal.
include sexual dysfunction, fatigue, and gastro-
intestinal upset. Answer A is incorrect. Cocaines physiologic
action is a blockade of norepinephrine and do-
Answer E is incorrect. Valproic acid is com-
pamine reuptake in the brain. Due to the nor-
monly used in bipolar disease to treat and pre-
adrenergic blockade, patients intoxicated with
vent manic episodes. Common adverse effects
cocaine will likely have tachycardia, hyperten-
include liver toxicity and pancreatitis. Impor-
sion, dilated pupils, and hyperthermia. The
tantly, when administered with lamotrigine,
dopaminergic reuptake blockade is responsible
valproic acid increases the risk of Stevens-
for hallucinations, particularly formication (the
Johnson syndrome by inhibiting the metabo-
feeling of insects crawling under the skin), and
lism of lamotrigine.
increased sexual arousal. The increased do-
pamine in the nigrostriatal pathway may also
23. The correct answer is B. Decreased growth ve-
cause stereotyped movements and an increase
locity is a well-known adverse effect of psycho-
in motor activity.
stimulant medications in children and adoles-
HIGH-YIELD SYSTEMS
Chapter 13: Psychiatry Answers 397

Answer C is incorrect. The characteristic Answer D is incorrect. Fragile X is a syndrome


pharmacologic action of opioids is analgesia. that occurs in male patients and is associated
Opioids, at lower doses, may have a behavior- with mental retardation, macroorchidism, and
ally disinhibiting effect, which presents as im- large ears.
paired judgment and social functioning. They
Answer E is incorrect. Retts disorder, which
are, however, sedating at higher dosages due
the patient does not have, is a genetic neuro-
to factors such as respiratory depression. Other
degenerative disease found only in female pa-
major features of intoxication are feelings of
tients. Patients have normal physical, mental,
euphoria or dysphoria, facial ushing, and pu-
and social development until about the age of
pil constriction.
5 months and then begin to regress in develop-
Answer D is incorrect. Opioid withdrawal may ment. Language and coordination are the most
cause depression; however, it also causes many common functions that are adversely affected.
other prominent symptoms that are not present
in this case. Autonomic symptoms are typically 26. The correct answer is C. The patient has anti-
characteristic of opioid withdrawal. These in- social personality disorder, which is character-

Psychiatry
clude goose esh, tachycardia, and increased ized by deceitfulness, aggressiveness, and irrita-
blood pressure. Musculoskeletal symptoms, bility, as exhibited by repeated physical ghts,
such as joint and muscle aches, are also ex- impulsiveness, and a reckless disregard for his
tremely characteristic of opioid withdrawal. own and others safety. However, to be diag-
nosed with antisocial personality disorder, the
Answer E is incorrect. Typical depression is
patient also has to have had conduct disorder,
usually characterized by a decrease in both ap-
which is the childhood precursor to antisocial
petite and sleep. Atypical depression, however,
personality disorder that occurs before age 15
may be characterized by an increase in these
years. Criteria for conduct disorder include
neurovegetative functions, as opposed to a de-
cruelty to people and animals, destruction of
crease.
property, deceitfulness or theft, and serious vio-
lation of parental rules.
25. The correct answer is C. Childhood disinte-
grative disorder is a disorder of early childhood Answer A is incorrect. The patient currently
and usually presents prior to age 10. Patients has antisocial personality disorder, but he
generally have normal psychosocial develop- would not have been diagnosed with this as a
ment during the rst 2 years of their lives and child because it is only diagnosed in adults. In
then begin to regress. Areas of dysfunction can many cases, children with conduct disorder go
include language, social play, bowel or bladder on to develop antisocial personality disorder.
control, and motor skills.
Answer B is incorrect. The patient is impul-
Answer A is incorrect. Aspergers disorder is sive, which could indicate manic episodes.
characterized by impaired social functioning, However, the patient has a signicant history
although it is not usually as severe as in autism. of repeated ghting as a child and adult, mak-
Patients can have stereotyped behaviors and ing conduct disorder and subsequent antisocial
may persevere in an activity for hours on end. personality disorder a more likely diagnosis.
Answer B is incorrect. Autism is characterized Answer D is incorrect. Although children with
by marked impairment in communication and depression may often be agitated and unhappy,
social interactions. Patients may also exhibit leading to an increased incidence of school
repetitive or stereotyped behavior, hobbies, or ghts, his current antisocial behavior makes
interests. Many patients also have mental retar- childhood depression a less likely diagnosis.
dation. Patients do not have a normal phase of
Answer E is incorrect. Children with conduct
development, as shown in this history.
disorder often abuse substances. However,
there is no evidence that this is the case.
HIGH-YIELD SYSTEMS
398 Section I: Organ Systems Answers

27. The correct answer is A. DSM-IV-TR denes disorder do not produce their symptoms inten-
dependence as a maladaptive pattern of sub- tionally and do not realize any gain from them.
stance use leading to clinically signicant im- Referral to a psychiatrist is an important com-
pairment or distress, as manifested by three or ponent of treatment, since behavioral and cog-
more of the following occurring at any time in nitive techniques can help such patients ex-
the same 12-month period: tolerance; with- press their emotional needs in a more
drawal; taking the substance in larger amounts constructive way. Once the disorder is identi-
or over a longer period than was intended, per- ed, it is important for the medical team to
sistent but unsuccessful efforts to cut down; minimize inappropriate medical interventions.
spending a great deal of time in obtaining, us-
Answer B is incorrect. La belle indiffrence
ing, or recovering from the substance; stopping
refers to the lack of concern that many patients
of important activities because of substance
with conversion disorder exhibit with regard to
use; or continuing the use despite knowledge
their condition. Conversion disorder is char-
of the problem. In this case, the presence of
acterized by the sudden loss of an aspect of
several attempts to stop would be indicative of
physical functioning, often secondary to acute
Psychiatry

dependence.
psychological distress. It is monosymptomatic
Answer B is incorrect. The DSM-IV-TR de- and almost always involves a single motor or
nes alcohol abuse as a maladaptive pattern of sensory symptom.
substance use leading to clinically signicant
Answer C is incorrect. A textbook presen-
impairment or distress, as manifested by one
tation points more to a factitious disorder
(or more) of the following, occurring within a
than to a somatization disorder, as the latter
12-month period: recurrent substance use re-
is often characterized by vague and indistinct
sulting in failure to fulll major role obligations;
complaints. Patients with factitious disorders
recurrent substance use in situations in which
consciously feign symptoms to be in the sick
it is physically hazardous; recurrent substance-
role. Although they consciously fake symp-
related legal problems; and/or continued sub-
toms, their motivation is unconscious. In fac-
stance use despite having persistent or recurrent
titious disorder, there is no obvious secondary
social or interpersonal problems. In this case,
gain such as money, drugs, or avoidance of
the use of alcohol in the face of legal ramica-
work. Patients with malingering are seeking
tions indicates alcohol abuse, not dependence.
secondary gains.
Answer C is incorrect. The failure to ful-
Answer D is incorrect. Pleasure and satisfac-
ll role obligations at work indicates alcohol
tion derived from an offer of a medical hospi-
abuse, not dependence.
talization points more toward a factitious disor-
Answer D is incorrect. The presence of so- der than to a somatization disorder. Factitious
cial/interpersonal problems indicates alcohol disorders result when a patient feigns or in-
abuse, not dependence. duces a medical condition in order to receive
some form of secondary gain.
Answer E is incorrect. The recurrent use of
alcohol while driving indicates abuse, not de- Answer E is incorrect. Patients with somatiza-
pendence. tion disorder often lack insight and are highly
offended if it is implied that their symptoms
28. The correct answer is A. Somatization disor- are psychological in nature.
der is a chronic psychiatric condition that typi-
cally presents with a long-standing history of 29. The correct answer is D. This patient has gen-
unsubstantiated medical complaints of pain in der identity disorder. Early psychological ther-
at least four sites, including two gastrointestinal apy to help the person focus on his or her bio-
symptoms, one sexual symptom, and one neu- logical gender role can result in less transsexual
rologic symptom. Patients with somatization behavior in the future. However, adults with
persistent gender identity disorder may seek sex
change surgery.
HIGH-YIELD SYSTEMS
Chapter 13: Psychiatry Answers 399

Answer A is incorrect. Alprazolam is a benzo- Answer E is incorrect. Although it is important


diazepine. Medication is not a recommended to address substance abuse as a contributor to
part of the treatment for gender identity disor- any suicide attempt, the details of the suicide
der. However, it may be used to treat comorbid and plan are more important in the acute set-
anxiety and depression. ting.
Answer B is incorrect. Hormone therapy
31. The correct answer is A. The patient presents
would be useful if the patient were going to
with symptoms of diabetes mellitus after treat-
have gender reassignment. It would not be ap-
ment with an atypical antipsychotic (olanzap-
propriate while a young patient is receiving
ine). Hyperglycemia, diabetes mellitus, and
psychotherapy.
acute-onset diabetic ketoacidosis have been re-
Answer C is incorrect. Paroxetine is an SSRI. ported with atypical antipsychotics, especially
Medication is not a recommended part of the olanzapine and clozapine. The most appropri-
treatment for gender identity disorder. How- ate next step in management of this patient
ever, it may be used to treat comorbid anxiety would be to discontinue olanzapine.

Psychiatry
and depression.
Answer B is incorrect. Valproic acid has many
Answer E is incorrect. Gender reassignment adverse effects including gastrointestinal ef-
may be appropriate later in life for persistent fects, tremors, weight gain, and hepatotoxicity.
gender identity disorder. It would not be ap- Diabetes has not been associated with valproic
propriate until psychotherapeutic prevention acid (although rare cases of pancreatitis have).
has been attempted.
Answer C is incorrect. Olanzapine has been
linked with acute-onset ketoacidosis, therefore
30. The correct answer is C. One of the most im-
immediate attention is required and monitor-
portant things to remember when interviewing
ing blood sugars would not be an appropriate
a suicidal patient is that patients may tend to
next step in management of this patient.
minimize their own symptoms, expressing a de-
nial of what they were really hoping to accom- Answer D is incorrect. Olanzapine has been
plish. However, all aspects of the patients sui- linked with acute-onset ketoacidosis, therefore
cide attempt, any previous suicide attempts, immediate attention is required and diet and
and the meaning behind the current crisis exercise alone would not be appropriate.
must all be explored. This question will enable
Answer E is incorrect. In this clinical sce-
the interviewer to learn more about the pa-
nario, the onset of diabetes is correlated with
tient, her feelings, and what is currently hap-
use of an atypical antipsychotic medication
pening in her life that led to this event.
and therefore discontinuation of the offending
Answer A is incorrect. If the patient is psy- agent altogether will most likely stop symptoms
chotic, then appropriate medications should of diabetes. Oral hypoglycemics would be indi-
be initiated; however, the details of her suicide cated for long-term therapy of non-drug associ-
attempt must be elicited rst. ated type 2 diabetes.
Answer B is incorrect. Although obtaining a
32. The correct answer is C. The patient displays
good psychiatric history is important, in the
many of the characteristics and feelings that
acute setting it is more important to address
are seen in borderline personality disorder.
the sincerity of the patients suicide attempt,
These patients often have childhoods that are
and evaluate whether she is still a danger to
characterized by inconsistent parenting and of-
herself.
ten a signicant amount of abuse. The parents
Answer D is incorrect. Any patient who is a may not reinforce the validity of the patients
danger to herself should be hospitalized, but to inner feelings. The patients tend to divide peo-
do so without an understanding of her actions ple into all good or all bad categories, a well-
would not be appropriate. known phenomenon termed splitting. They
HIGH-YIELD SYSTEMS
400 Section I: Organ Systems Answers

often have chronic feelings of emptiness and thereby reducing the likelihood of pancreatitis.
have many relationships that are labile in na- She does have an elevated amylase, but the li-
ture. Borderline patients have severe trouble pase is normal; this abnormality is most likely
regulating their emotions, as seen when the pa- secondary to chronic vomiting, not pancreati-
tient could not control her own anger at the tis.
therapist, and are therefore very impulsive.
Answer B is incorrect. Acute renal failure is
Answer A is incorrect. The patient does not dened as an abrupt decrease in renal func-
show a consistent disregard for the rules of so- tion leading to the retention of creatinine and
ciety and the rights of others, which is charac- blood urea nitrogen. Acute renal failure can
teristic of antisocial personality disorder. be caused by prerenal causes such as hypo-
volemia. Although the blood urea nitrogen
Answer B is incorrect. The patient does not
level in this case is elevated, the creatinine is
discuss being extremely afraid of situations that
normal. Thus, the etiology of these laboratory
may result in interpersonal rejection, a key
abnormalities is not acute renal failure.
characteristic of avoidant personality disorder.
Psychiatry

Answer C is incorrect. Although sustained


Answer D is incorrect. Although the patient
vomiting can lead to electrolyte abnormali-
displays shallow emotions, a characteristic of
ties such as hypokalemia, which can lead to
histrionic personality disorder, she also displays
arrhythmias, this patients dizziness is much
splitting, impulsivity, and volatile characteris-
more attributable to volume loss. It would not,
tics more indicative of a borderline personality
however, be unreasonable to obtain an ECG
disorder. Histrionic personality disorder is char-
to evaluate her cardiac function.
acterized by a pervasive pattern of excessive
emotionality and attention seeking. Answer D is incorrect. Respiratory acidosis
can be caused by carbon dioxide retention
Answer E is incorrect. The patient does not
secondary to airway obstruction or chronic ob-
exhibit grandiosity in her self-perception, mak-
structive pulmonary disease. This woman has
ing narcissistic personality disorder less likely.
an increased bicarbonate, which could be due
33. The correct answer is E. This patient is exhib- to renal compensation for a respiratory acido-
iting many signs of self-induced vomiting. The sis. However, based on her normal respiratory/
bicarbonate level is high, consistent with a pulmonary examination and normal oxygen
metabolic alkalosis. She has parotid gland saturation, there does not seem to be any pri-
swelling, dental pitting, and Russells sign (the mary pulmonary process. The increased bi-
metacarpophalangeal calluses caused by teeth carbonate level is most likely due to a primary
rubbing against the joint during vomiting). She metabolic alkalosis.
is orthostatic, and her blood urea nitrogen and
34. The correct answer is D. There are many dif-
creatinine levels are indicative of hypovolemia.
ferent types of psychotherapy, each of which
Increased serum amylase can be seen with pro-
has its unique advantages and disadvantages.
longed vomiting. Her fatigue and dizziness are
This therapy is psychoanalysis, in which the
most likely secondary to dehydration and the
psychiatrist plays a neutral role, listening for
possible electrolyte abnormalities (e.g., low po-
any connections in what the patient says. The
tassium) associated with chronic vomiting. The
patient speaks freely, a technique called free
mother is not aware of any abnormal eating
association, in which whatever is on the pa-
patterns or chronic vomiting, consistent with
tients mind is of psychoanalytic interest.
the patient concealing her behaviors, which is
Dreams, and their subsequent analysis, can
often seen in bulimia nervosa.
also play a large role in psychoanalysis, which
Answer A is incorrect. Acute pancreatitis is may give the therapist more clues to valuable
characterized by the abrupt onset of pain in connections in the patients life. Clarication,
the epigastric region radiating to the back, usu- confrontation, and interpretation are also used
ally caused by alcoholism or gallstones. This in this modality.
patient does not have any abdominal pain,
HIGH-YIELD SYSTEMS
Chapter 13: Psychiatry Answers 401

Answer A is incorrect. Behavioral therapy, phrenia, mania, depression, and anxiety, and as
sometimes closely linked to cognitive therapy part of certain neurologic disorders. Symptoms
in cognitive behavior therapy, is based on the include negativism (resistance to following di-
hypothesis that all human behavior grows out rections), mutism (paucity of speech), echop-
of conditioned reactions from childhood and raxia (repetition of movements made by an-
that all psychopathology is the result of inap- other person), waxy exibility (when the
propriate conditioning from childhood. Behav- patient can be moved and molded into strange
ior therapy is best suited for patients with a cir- body positions that he or she will maintain),
cumscribed disorder, such as anxiety, and not paratonia (involuntary resistance to passive
disorders that are long term or chronic. The movement), and stereotyped mannerisms.
focus of therapy is to lessen the anxiety associ-
Answer B is incorrect. Dystonias are invol-
ated with a particular situation or behavior and
untary movements of the agonist/antagonist
to gradually teach relaxation techniques when
muscles in a given area of the body. Examples
faced with the problematic environment.
include neck muscle dystonia (retrocollis/
Answer B is incorrect. Cognitive therapy, de- torticollis) and extraocular muscles (oculogyric

Psychiatry
veloped for mild to moderate depression and crisis). These movement disorders result from
dysthymia, relies on the theory that there is a dopamine antagonism in the nigrostriatal do-
close link between a persons habits or patterns pamine pathways of the basal ganglia. Dystonia
of conscious thought and that persons moods, is a known adverse effect of typical neurolep-
and that certain thoughts can cause and/or tics such as haloperidol. Although the patient
maintain a persons depressed mood. The is taking haloperidol, she does not exhibit dys-
goal of therapy is to have the patient actively tonic symptoms.
identify his or her negative thoughts and then
Answer C is incorrect. Mental retardation
change the content of those thoughts to reect
might include cognitive symptoms such as
a more positive view of him- or herself.
echolalia, but would not be likely to include
Answer C is incorrect. Interpersonal therapy the motor component mentioned.
was developed in response to the theory that
Answer D is incorrect. Neuroleptic malignant
depression is a defect in interpersonal rela-
syndrome is a rare, idiosyncratic but life-threat-
tions. The patient describes in detail his or her
ening reaction to antipsychotics that causes
relationships to other people in his or her life,
lead pipe rigidity; autonomic symptoms, in-
and deciencies in relationships are examined.
cluding diaphoresis, high fever, hypertension,
The patient is encouraged to become more
and increased heart rate; and neurologic dys-
aware of his or her own feelings toward others,
function.
and he or she becomes more able to express
feelings and communicate freely. Answer E is incorrect. Serotonin syndrome oc-
curs from hyperstimulation of 5-HT1A recep-
Answer E is incorrect. The goal of supportive
tors, often as a result of interaction between
therapy is to support the patient at his or her
monoamine oxidase inhibitors or typical neuro-
highest level of functioning. The therapist pro-
leptics and SSRIs. The classic triad of symptoms
vides a warm, empathetic environment that
that characterizes serotonin syndrome includes
should be seen as a reliable source of support
altered mental status with signicant restless-
and help. Supportive therapy is used in re-
ness, autonomic dysfunction, and neuromuscu-
sponse to an acute event (crisis intervention),
lar abnormalities. Autonomic symptoms include
in the case of a patient with long-term chronic
nausea, vomiting, and hyperthermia, while neu-
medical problems, or for chronic psychiatric
romuscular abnormalities include myoclonus,
patients (e.g., with schizophrenia) that cannot
nystagmus, hyperreexia, and lower extremity
make use of other types of therapies.
rigidity.
35. The correct answer is A. Catatonia is an altera-
36. The correct answer is E. Oppositional deant
tion in neuromuscular tone that occurs as part
disorder (ODD) is a persistent pattern of hos-
of many psychiatric disorders, including schizo-
HIGH-YIELD SYSTEMS
402 Section I: Organ Systems Answers

tile, deant behavior toward authorities that she is to blame for his death. These delusions
persists for >6 months. Symptoms are seen in and auditory hallucinations warrant a diagnosis
multiple settings, such as home and school. To of major depressive disorder with psychosis.
diagnose ODD, the behavior must be frequent
Answer A is incorrect. Bipolar depression oc-
and consistent when compared with other chil-
curs when a patient has a history of one or more
dren of the same age and development status,
manic episodes that alternate with periods of
and when the disruptive nature of the behavior
clinical depression. This patient does not de-
causes a signicant degree of impaired social
scribe any symptoms of mania (periods of ele-
and school functioning.
vated or expansive mood lasting at least 1 week).
Answer A is incorrect. Antisocial personality It seems that she has been consistently depressed
disorder is a persistent pattern of behavior with since the death of her father 2 years prior.
a central theme of disrespect for the rights of
Answer B is incorrect. Cyclothymic disorder
others. Patients may exhibit milder signs and/
involves periods of depressed mood for 2 or
or be diagnosed with conduct disorder during
more years that could be classied as dysthy-
adolescence but are diagnosed with this disor-
Psychiatry

mic disorder along with periods of hypomania.


der only after the age of 18 years.
Answer C is incorrect. Dysthymic disorder is
Answer B is incorrect. Attention decit disor-
diagnosed based on a depressed mood that lasts
der is a disorder of inattention and impaired
for 2 or more years. The patient must also have
ability to focus on given tasks. Patients can be
at least two of the following symptoms: poor ap-
hyperactive and become easily bored because
petite or overeating, insomnia or hypersomnia,
they are frustrated by their limitations. But this
low energy or fatigue, low self-esteem, poor
patient is predominantly negative and hostile
concentration, or feelings of hopelessness.
toward authority gures.
Answer D is incorrect. Major depressive disor-
Answer C is incorrect. Conduct disorder may
der is characterized by alterations in Sleep, loss
have a similar presentation to ODD; however,
of Interests or pleasures, feelings of Guilt, a de-
conduct disorder is characterized by violence
crease in Energy, decreased Concentration, al-
and a consistent disregard for the property and
terations in Appetite and Physical activity, and
well-being of others. Patients may have en-
thoughts about Suicide; these symptoms can
gaged in theft, assault, and injury of animals or
be remembered with the mnemonic SIGE-
small children.
CAPS. Not all patients will display every symp-
Answer D is incorrect. Normal adolescents tom of depression, but symptoms must persist
are prone to confrontation and arguments for >2 weeks before considering a diagnosis of
while trying to assert their independence. major depressive episode. Major depressive dis-
This patients behavior, however, is impairing order consists of one or more major depressive
his school and home functioning. He appears episodes that are not better characterized by
to have a disregard for authority and an inten- another mood disorder diagnosis.
tional deance of all rules and regulations.
38. The correct answer is E. It would appear that
37. The correct answer is E. This patient meets this patient is suffering from factitious disorder.
the criteria for major depression, with feelings The serologic prole of a patient injecting ex-
of sadness and anhedonia and disturbances in ogenous insulin is characterized by low C-pep-
both sleep and appetite. Her deterioration has tide levels because the production of the en-
prompted her family to seek professional help. dogenous insulin is suppressed along with
Her recent inability to take care of her kids C-peptide by-product levels. Antibodies are
may indicate worsening of her depression. Her also produced against the exogenous insulin.
delusion that she is responsible for her fathers Given that this patient has prior medical
death persists despite her familys attempts to knowledge, it must be assumed that he admin-
tell her otherwise. In addition she displays istered the medication to himself; this behavior
mood-congruent auditory hallucinations of her is consistent with factitious disorder. Patients
fathers voice that reinforce the delusion that with factitious disorder simulate physical or
HIGH-YIELD SYSTEMS
Chapter 13: Psychiatry Answers 403

psychiatric illness in order to gain attention tient. It should be noted that while the same
from medical personnel and often have an ex- symptoms can be seen in amphetamine intoxi-
tensive history of prior surgeries and hospital- cation, the treatment would be the same for
izations. A psychiatric consultation should be any severe agitation.
called before any other medical interventions
are initiated. 41. The correct answer is C. This patient is expe-
riencing delirium tremens, a consequence of
Answer A is incorrect. This patient has a psy-
alcohol withdrawal that usually appears 27
chiatric condition that warrants evaluation.
days after a patient stops drinking. Delirium
However, a CT scan would be helpful if an in-
tremens is characterized by tachycardia, hyper-
sulinoma was suspected. An insulinoma would
tension, diaphoresis, confusion, hallucinations
cause low blood glucose levels, but C-peptide
(especially tactile), and tremors. The main
would be high, as it is a by-product of endog-
treatment is with benzodiazepines.
enous insulin.
Answer A is incorrect. Activated charcoal is
Answer B is incorrect. Endoscopy is used to
a nonspecic binding substance that is used

Psychiatry
visualize the gastrointestinal tract in order to
to clear the gastrointestinal tract of substances
evaluate for pathology within the gastrointes-
that have not yet been absorbed. It would be
tinal system, including malignancy, ulcers, or
used in the setting of ingestion of salicylates,
inammatory conditions. This patients low
barbiturates, theophylline, and the like.
blood sugar is better explained by a psychiat-
ric condition, and thus psychiatric evaluation Answer B is incorrect. Atropine is the antidote
is the most appropriate step in management at for anticholinesterases and organophosphates
this time. such as pesticides.
Answer C is incorrect. Dialysis would be in- Answer D is incorrect. Deferoxamine is the
dicated if this patients low sugar was due to a treatment for an iron overdose.
metabolic abnormality such as acidosis (pH
Answer E is incorrect. Dialysis is used to clear
<7.1), abnormal electrolytes (potassium level
the blood of substances for which no other
>6.5 mEq/L or rapidly rising), intoxication
remedy is known (e.g., salicylates and barbitu-
(e.g., lithium or aspirin), volume overload, or
rates) and is used only as a last-line therapy.
uremia with related pericarditis, neuropathy, or
unexplained decline in mental status. Answer F is incorrect. Dimercaprol is used to
treat overdoses of heavy metals such as lead, ar-
Answer D is incorrect. Insulin therapy would
senic, mercury, and gold.
be appropriate in a patient with diabetes and
high blood sugar levels. However, this patient Answer G is incorrect. Flumazenil is used to
already has dangerously low blood sugar due to treat an overdose of benzodiazepines.
inappropriate use of insulin. Answer H is incorrect. Glucagon is used to
treat an overdose of -blockers.
Questions 39, 40, and 41 Answer J is incorrect. Hyperbaric oxygen treat-
ment is used for carbon monoxide exposure.
39. The correct answer is L. This patient is exhib-
iting classic signs of opiate overdose, which in- Answer K is incorrect. N-acetylcysteine is the
clude depressed respiration, constricted pupils, antidote for acetaminophen overdose.
somnolence, and eventually coma. Treatment
Answer M is incorrect. Vitamin K is used to
is with naloxone, an opiate antagonist.
reverse the effects of a warfarin overdose.
40. The correct answer is I. This patient is exhibit-
ing signs of cocaine intoxication, as shown by Questions 42, 43, and 44
his sympathetic arousal and exuberant mood.
Haloperidol is the appropriate treatment for an 42. The correct answer is E. Clozapine is an an-
agitated or psychotic substance-abusing pa- tipsychotic medication that improves the nega-
HIGH-YIELD SYSTEMS
404 Section I: Organ Systems Answers

tive symptoms of schizophrenia but can cause Answer F is incorrect. Fluoxetine is an SSRI
agranulocytosis in 1%2% of patients taking it. used in the treatment of depression, obsessive-
As a result, patients must agree to a weekly compulsive disorder, bulimia nervosa, panic
blood draw to check WBC count throughout disorder without agoraphobia, and premen-
therapy and for 4 weeks after stopping the drug. strual dysphoric disorder. Adverse effects in-
Agranulocytosis is an absolute contraindication clude decreased libido, insomnia, headache,
for continuing the drug. anxiety, nervousness, nausea, diarrhea, an-
orexia, dry mouth, weakness, and tremor.
43. The correct answer is J. Olanzapine is an atyp-
Answer G is incorrect. Haloperidol is a high-
ical antipsychotic associated with increased
potency typical antipsychotic medication that
triglycerides, somnolence, hyperglycemia, and
is associated with more extrapyramidal symp-
weight gain. The drug itself is particularly ef-
toms than the low-potency agents (e.g., chlo-
fective for negative symptoms of schizophrenia
rpromazine). Typical antipsychotics in general
and for agitation.
can cause extrapyramidal symptoms, hyperpro-
44. The correct answer is H. In approximately lactinemia, anticholinergic effects, and seda-
Psychiatry

10% of patients, lamotrigine causes a skin rash. tion, and are more associated with neuroleptic
Development of this potentially life-threaten- malignant syndrome than are atypical antipsy-
ing rash is associated with rapid increase in the chotics.
drug dosage. Development of any type of rash Answer I is incorrect. Lithium is used in the
is an absolute contraindication for continua- therapy of bipolar disorder, particularly treat-
tion of lamotrigine. ment and prophylaxis of bipolar disorder. Ad-
Answer A is incorrect. Amitriptyline is a tricy- verse effects include electrolyte abnormalities,
clic antidepressant used to treat major depres- diabetes insipidus, leukocytosis secondary to
sion and in the treatment of neuropathic pain. demargination, and hypothyroidism.
It is associated with primarily anticholinergic Answer K is incorrect. Reserpine is an antihy-
adverse effects, sedation, sexual dysfunction, and pertensive medication that is also used in the
arrhythmias. It can be lethal in overdose, caus- treatment of agitated psychotic states (e.g.,
ing coma, convulsions, and cardiac arrhythmias. schizophrenia) and in management of tardive
Answer B is incorrect. Buspirone is an effec- dyskinesia. It acts by depletion of sympathetic
tive anxiolytic that does not promote depen- biogenic amines (e.g., norepinephrine, do-
dence or tolerance. Its most common adverse pamine). Adverse effects include sedation and
effect is dizziness. dizziness.

Answer C is incorrect. Clonazepam is a ben- Answer L is incorrect. Risperidone is an atypi-


zodiazepine used in the treatment of anxiety cal antipsychotic medication. Adverse effects
(especially for the immediate relief of anxiety include extrapyramidal adverse effects, insom-
nia, agitation, anxiety, headache, and weight
symptoms) and panic disorder, and as an ad-
gain.
junct treatment of some seizures. Its adverse
effects include ataxia, confusion, memory dis- Answer M is incorrect. Sertraline is an SSRI
turbances, drowsiness, and somnolence. used in treating major depressive disorder. Ad-
verse effects include insomnia/somnolence,
Answer D is incorrect. Clonidine is a cen-
dizziness, headache, dry mouth, and sexual ad-
trally acting adrenergic agonist that inhibits
verse effects (e.g., ejaculatory disturbances).
the sympathetic nervous system via central
2-adrenergic receptors. In psychiatry, it can Answer N is incorrect. Topiramate is an anti-
be used in the treatment of attention decit convulsant and mood stabilizer. Adverse effects
hyperactivity disorder, and in targeting anxiety include weight loss, anorexia, psychomotor
and hyperarousal in posttraumatic stress disor- slowing, memory difculties, difculty con-
der. centrating, nausea, drop in serum bicarbonate,
and paresthesias.
HIGH-YIELD SYSTEMS
Chapter 13: Psychiatry Answers 405

Answer O is incorrect. Valproic acid is a mood Answer C is incorrect. Usually related to


stabilizer that can cause thrombocytopenia some psychosocial trauma, conversion disorder
and bleeding, in addition to hepatic failure, presents with a motor or sensory dysfunction
pancreatitis, and hyperammonemia; it is also (blindness, paralysis, seizure) that is incompat-
considered a teratogen. ible with any known medical diagnosis. Neuro-
logic symptoms often migrate and are transient
Answer P is incorrect. Venlafaxine is a serotonin
in nature. Patients tend to lack appropriate
norepinephrine reuptake inhibitor. Its adverse
concern for their symptoms.
effects include central nervous system effects
(headache, insomnia/somnolence, dizziness, Answer E is incorrect. The diagnosis of gener-
nervousness), gastrointestinal effects (nausea, alized anxiety disorder depends on a duration
dry mouth, constipation, anorexia), genitouri- of symptoms for >6 months and at least three
nary effects (abnormal ejaculation or orgasm), somatic symptoms for the same time period.
weakness, and diaphoresis and hypertension. It is not uncommon to see patients with gen-
eralized anxiety disorder self-medicate with
benzodiazepines or alcohol. Treatment is usu-

Psychiatry
Questions 45, 46, and 47 ally with an SSRI or other anxiolytic (e.g., bus-
45. The correct answer is N. Social phobia results pirone).
in a disabling sense of anxiety in situations in Answer F is incorrect. Narcolepsy is a disease
which the patient is expected to perform or will in which the patient falls asleep suddenly;
be scrutinized by others. It can severely impact these sleeping episodes (both normal and
the life of the patient, from ofce presentations narcoleptic) begin with rapid eye movement
to talking to unfamiliar people. It can be treated sleep. Treatment is usually with stimulants
with antidepressants and benzodiazepines; some (e.g., amphetamines).
patients respond to a presituational -blocker
aimed at minimizing somatic complaints. Answer G is incorrect. The diagnosis of obses-
sive-compulsive disorder requires the presence
46. The correct answer is B. Brief psychotic disor- of both obsessions and compulsions. Treat-
der is the development of psychotic symptoms ment is usually a combination of cognitive
(usually auditory hallucinations) occurring in behavioral therapy or psychodynamic therapy
the setting of a psychosocial stressor (e.g., the and either clomipramine or an SSRI such as
loss of a loved one). For the diagnosis to apply, uoxetine.
the episode must resolve within 1 month. In Answer H is incorrect. ODD is a pattern of
comparison to the other psychotic diagnoses, it negativistic, deant, disobedient, and hos-
has a much better prognosis. tile behavior toward authority gures for >6
months that is mostly diagnosed in adoles-
47. The correct answer is D. Patients with cyclo- cence. However, the patient does not violate
thymia alternate between hypomania and dys- social norms or the rights of others (as in anti-
thymia. The mood swings are much more fre- social personality disorder). Treatment involves
quent than in bipolar disorder, but the highs both individual and family therapy.
(hypomanic episodes) are not as pronounced,
and the lows (dysthymic episodes) are less Answer I is incorrect. Patients with panic dis-
debilitating. Many patients are aware of feeling order experience abrupt onset attacks, con-
chronically depressed, without recognizing sisting of several somatic symptoms and a feel-
their own hypomanic episodes. ing of impending doom. These attacks can be
brought on by particular environments (e.g.,
Answer A is incorrect. Patients with antisocial agoraphobia).
personality disorder have been termed socio-
paths; they violate the rights of others, the law, Answer J is incorrect. Posttraumatic stress dis-
and social norms. They are impulsive and they order is characterized by symptoms resulting
lack remorse for their behavior. from an extremely traumatic stressor (assault,
active combat, witnessing a violent event). The
patient relives the event via nightmares and
HIGH-YIELD SYSTEMS
406 Section I: Organ Systems Answers

ashbacks, experiences a state of increased can be treated with -blockade and by decreas-
arousal (hypervigilance, exaggerated startle), ing the dosage of the antipsychotic medication.
and tends to avoid stimuli associated with the Benzodiazepines and anticholinergic agents
event. The syndrome is also marked by psychic (as in the treatment of other extrapyramidal
numbing (detachment, social withdrawal, an- symptoms) may also help.
hedonia). In addition to supportive and cogni-
tive behavioral therapy, SSRIs can be helpful 49. The correct answer is A. Agoraphobia is de-
in the treatment of posttraumatic stress disor- ned by the DSM-IV-TR as anxiety about be-
der. ing in places or situations from which escape
might be difcult (or embarrassing) or in
Answer K is incorrect. Retts syndrome is a ge-
which help may not be available in the event
netic neurodegenerative disorder characterized
of having an unexpected or situationally predis-
by progressive developmental impairment after
posed panic attack or panic-like symptoms.
a period of normal development. It occurs al-
This young womans symptoms are consistent
most exclusively in girls.
with panic disorder with agoraphobia, which
Psychiatry

Answer L is incorrect. The most important occurs in 30%50% of all patients with panic
feature of schizoaffective disorder is the pres- disorder.
ence of mood symptoms (depression, mania,
or mixed) in conjunction with psychotic symp- 50. The correct answer is K. Echolalia is the im-
toms. However, the psychotic symptoms must mediate and involuntary repetition of words or
then continue for at least 2 weeks after the res- phrases just spoken by others. It can be seen in
olution of the mood symptoms. autism and many forms of schizophrenia.
Answer M is incorrect. Patients with schizo- Answer C is incorrect. Anhedonia is the com-
phrenia must have two or more of the follow- plete loss of pleasure in activities, people, or
ing for at least 6 months: hallucinations, delu- things that were previously pleasurable. For ex-
sions, disorganized speech, affective attening, ample, a person with major depression may no
alogia, and avolition. To be classied in the longer enjoy baseball games, which he or she
paranoid subtype, patients must have delusions had once followed religiously.
or hallucinations, but cognitive function and Answer D is incorrect. Ataxia is loss of coordi-
affect are relatively preserved. Of all the sub- nation, most commonly due to disturbance in
types of schizophrenia, this one has the best the cerebellum or neuronal pathways leading
prognosis. into and out of the cerebellum.
Answer O is incorrect. Tourettes disorder is Answer E is incorrect. Catatonic schizophre-
characterized by multiple vocal and motor nia is marked by at least two of the following:
tics occurring many times per day. It is more excessive motor activity, immobility, extreme
common in men, is known to have a genetic negativism, mutism, waxy exibility, echola-
predisposition, and usually begins in the teen- lia (repeating anothers words), or echopraxia
age years. Treatment is usually with haloperi- (mimicking anothers movements). Although
dol, pimozide, or clonidine, and psychological the patient in the second vignette displays
counseling can help with social adjustment. echolalia, he only has one symptom of catato-
nia and therefore could not be diagnosed with
Questions 48, 49, and 50 catatonic schizophrenia based on this presen-
tation.
48. The correct answer is B. Akathisia is a feeling
of restlessness (either subjective or objective), Answer F is incorrect. Compulsions are con-
or not being able to sit still. It is considered scious, repeated acts or behaviors that are un-
an extrapyramidal adverse effect and is associ- controllable. In obsessive-compulsive disorder,
ated with the use of many antipsychotic medi- compulsions are done to neutralize the anxiety
cations, especially the typical antipsychotics. It invoked by an obsession.
HIGH-YIELD SYSTEMS
Chapter 13: Psychiatry Answers 407

Answer G is incorrect. Delirium is dened Answer M is incorrect. Grandiosity is an in-


as a disturbance of consciousness or a waxing ated self-esteem and feeling of entitlement
and waning of normal levels of consciousness. that may be seen in mania. A person with bipo-
It can be experienced as confusion, disorienta- lar disorder may believe that nothing can stop
tion, cognitive impairment, or inability to con- him or her from getting what he or she wants,
centrate. Delirium can be due to many differ- and the person may become extremely agitated
ent etiologies, including structural damage to or irritated if obstructed.
the brain, infection, metabolic disorders, drugs,
Answer N is incorrect. Hypochondriasis is the
neoplasms, and autoimmune conditions.
preoccupation with or fear of having a serious
Answer H is incorrect. Delusions are disorders medical illness despite adequate medical reas-
of thought content. Subtypes of delusions in- surance.
clude paranoid, persecutory, bizarre, somatic,
Answer O is incorrect. Obsessions are persis-
grandiose, or referential. They are based on
tent, intrusive ideas, thoughts, impulses, or
incorrect perceptions that do not stem from a
images that cause anxiety or stress. Common
social or cultural belief system (e.g., religious

Psychiatry
obsessions are contamination and fear of harm
beliefs).
to oneself. The patient experiences obsessions
Answer I is incorrect. Dyskinesia is also known as inappropriate thoughts or impulses from his
as pseudoparkinsonism, or the shufing gait or her own mind (as opposed to thought inser-
and cogwheel rigidity seen in both parkin- tion, which is experienced as coming from the
sonism and as an adverse effect of many an- outside). In obsessive-compulsive disorder, ob-
tipsychotic medications (especially the typical sessions are neutralized or prevented by com-
antipsychotics). It can be remedied with anti- pulsions.
cholinergics (e.g., benztropine) or dopamine
Answer P is incorrect. Somatization is the
agonists (e.g., amantadine).
term applied to the experience of multiple, un-
Answer J is incorrect. Dysthymia is a milder explainable somatic symptoms that are not due
form of major depression, consisting of at least to any medical condition or to the effects of a
2 years of depressed mood with effects on eat- drug. The symptoms are not produced inten-
ing, sleeping, energy level, self-esteem, and tionally, and the patients are unaware of having
concentration and feelings of hopelessness. any psychiatric problems. Symptoms are often
related to pain at many different anatomic sites
Answer L is incorrect. Flight of ideas is a
(most commonly the back, the joints, and the
rapid ow of thought, manifested by acceler-
abdomen).
ated speech with abrupt changes from topic to
topic, often a characteristic of the thought pat-
tern of patients with mania.
This page intentionally left blank
CHAPTER 14

Pulmonary

409
HIGH-YIELD SYSTEMS
410 Section I: Organ Systems Questions

Q U E ST I O N S

1. A 78-year-old man is seen in the doctors ofce (E) Normal FEV1, decreased FVC, increased
for a nonproductive cough, 9-kg (20-lb) unin- FEV1:FVC ratio
tentional weight loss, and bilateral breast en-
largement, all occurring within the past 6 3. A 3-year-old boy presents to the emergency de-
months. He has smoked two packs per day for partment with a fever and difculty breathing.
the past 40 years. His past medical history is He is the product of a normal pregnancy and
otherwise unremarkable, and he takes no med- has been healthy since birth. His immuniza-
ications. His temperature is 36.7C (98.1F), tions are up to date. This morning he appeared
blood pressure is 125/85 mm Hg, pulse is 68/ to be in his usual state of health and was
min and regular, respiratory rate is 15/min, and dropped off at day care by his father. Later on,
oxygen saturation is 99% on room air. There his teacher noticed that he had suddenly be-
are crackles at the left lower lung eld and a come fussy and ushed and could not be con-
soled with toys, rocking, or hearing a story. He
Pulmonary

ridge of symmetric glandular tissue (1 cm in


diameter) around the nipple-areolar complexes also felt warm to the touch and was drooling
of both breasts. Complete blood cell count more than usual. When she took his tempera-
shows a WBC count of 6000/mm hemoglobin ture, it was 39C (102.2F). His parents were
of 14.7 g/dL, and platelet count of 210,000/ contacted immediately, and the patient was
mm. All other laboratory results are normal. brought to the emergency department. He ap-
X-ray of the chest shows a focal 5-cm mass le- pears toxic and anxious, and has loud labored
sion in the left lower lung corroborated by CT breathing. He is sitting upright, bracing him-
scan. Which of the following is most likely his- self on his arms, with his neck hyperextended
tologic type of lung cancer present in this pa- and mouth open. His temperature is 40C
tient? (104F), respiratory rate is 50/min, pulse is
140/min, blood pressure is 102/62 mm Hg, and
(A) Adenocarcinoma
oxygen saturation is 100% on room air. Lateral
(B) Bronchoalveolar cell carcinoma
x-ray of the neck is shown in the image. Laryn-
(C) Large cell carcinoma
goscopy reveals a large cherry-red epiglottitis.
(D) Small cell carcinoma
What is the most appropriate next step in man-
(E) Squamous cell carcinoma
agement?
2. A 30-year-old patient with a history of mild per-
sistent asthma (baseline peak expiratory ow
rate of 85%) presents to the emergency depart-
ment with shortness of breath and wheezing
that has not relieved by her albuterol inhaler
for the past 12 hours. She was able to tolerate
pulmonary function tests and a set was per-
formed. Which of the following is the most
likely test result?
(A) Decreased FEV1, normal/increased FVC,
decreased FEV1:FVC ratio, with post-
bronchodilator FEV1 increased by 13% Reproduced, with permission, from Stone CK,
(B) Decreased residual volume and total lung Humphries RL. Current Diagnosis & Treatment: Emer-
capacity gency Medicine, 6th edition. New York: McGraw-Hill,
2008: Figure 48-4.
(C) Increased FEV1, increased FVC, normal
FEV1:FVC ratio
(D) Increased residual volume, increased total
lung capacity, increased FEV1
HIGH-YIELD SYSTEMS
Chapter 14: Pulmonary Questions 411

(A) Antibiotic therapy reports feeling breathless when climbing the


(B) Corticosteroids stairs to her rst oor walk-up apartment, and
(C) Nasotracheal intubation has moderate difculty in providing her history
(D) Observation in complete sentences. X-ray of the chest shows
(E) Tracheostomy hyperinated lungs with attened diaphragms,
attenuated vascular markings, and a narrow
4. A 27-year-old woman is 7 months pregnant mediastinum. What agent(s) will provide the
with her rst child. Her pregnancy has been greatest relief of symptoms in the emergency
uncomplicated to date. She presents to the department?
emergency department complaining of sud-
(A) Albuterol and ipratropium bromide
den-onset, right-sided chest pain that is exacer-
(B) Antibiotics
bated with deep breathing and shortness of
(C) Magnesium sulfate
breath, which began 1 hour ago. She denies
(D) N-acetylcysteine
leg pain and notes that her legs began swelling
(E) Theophylline
during the sixth month of her pregnancy but

Pulmonary
the swelling has not worsened. Her tempera-
6. After an uncomplicated pregnancy and cesar-
ture is 37.9C (100.3F), blood pressure is
ean section for breech presentation, twins are
130/87 mm Hg, pulse is 107/min and regular,
born at 32 weeks gestation to a 24-year-old
respiratory rate is 24/min, and oxygen satura-
primigravida mother. Twin A weighs 1610 g
tion is 90% on room air, increasing to 98%
(3.5 lb) and has Apgar scores of 8 and 9 at 1
with 4 L oxygen via nasal cannula. Physical ex-
and 5 minutes, respectively. Twin B weighs
amination is signicant for crackles at the
1600 g (3.5 lb) and has Apgar scores of 7 and 8
lower right lung eld and a negative Homans
at 1 and 5 minutes, respectively. Within min-
sign bilaterally. X-ray of the chest appears nor-
utes of birth, twin B becomes mildly cyanotic
mal. The D-dimer level is elevated. ECG shows
and tachypneic with subcostal retractions, ex-
sinus tachycardia, right-axis deviation, S wave
piratory grunting, and nasal aring. Twin Bs
in lead I, Q wave in lead III, and an inverted T
blood pressure is 58/39 mm Hg, heart rate is
wave in lead III. Which of the following is the
130/min, respiratory rate is 100/min, and tem-
most appropriate next step in diagnosis?
perature is 37.0C (98.6F). Twin B is intu-
(A) Arterial blood gas analysis bated and given 70% fraction of inspired oxy-
(B) Doppler ultrasound of the lower extremity gen. Compared to twin A, what is twin B at
(C) MRI of the lower extremity greater risk of developing?
(D) Pulmonary angiography
(A) Apnea of prematurity
(E) Ventilation/perfusion scan
(B) Gastroesophageal reux disease
(C) Hyperbilirubinemia
5. A 65-year-old smoker previously diagnosed with
(D) No difference because they are both pre-
chronic obstructive pulmonary disease presents
mature
to the emergency department complaining of
(E) Retinopathy of prematurity
worsening cough and sputum production. She
HIGH-YIELD SYSTEMS
412 Section I: Organ Systems Questions

7. A 32-year-old white man with HIV and a re- 9. A 45-year-old Haitian immigrant presents to the
cent CD4+ cell count of 400/mm presents to emergency department with a chief complaint
the emergency department with a 3-day history of productive, blood-tinged cough for 2 months.
of fever, anorexia, cough, and night sweats. He He has been in the United States for 1 month.
recently returned from a camping vacation in His temperature is 40.1C (104.2F) and heart
Arizona, approximately 1 month prior to pre- rate is 105/min. On physical examination he ap-
sentation. He also describes diffuse joint pains. pears cachectic, and pulmonary rales are heard
His temperature is 38.9C (102F), oxygen sat- throughout his lung elds. X-ray of the chest re-
uration is 99% on room air, and there is a rash veals multiple bilateral upper lobe cavitary le-
on his arms and hands. There is dullness to sions with associated intrathoracic adenopathy.
percussion at the right lung base. X-ray of the Results of sputum culture are pending. Which
chest reveals a small right-sided inltrate and of the following tuberculosis medications can
hilar lymphadenopathy. Sputum analysis does potentially cause optic neuritis?
not reveal any organisms. He reportedly had a
(A) Ethambutol
negative puried protein derivative test 2
(B) Isoniazid
Pulmonary

months ago. Which of the following is the


(C) Levooxacin
most likely diagnosis?
(D) Pyrazinamide
(A) Coccidioidomycosis (E) Rifampin
(B) Histoplasmosis (F) Streptomycin
(C) Lung carcinoma
(D) Pneumocystis jiroveci pneumonia 10. A 30-year-old man has episodes of wheezing
(E) Sarcoidosis and shortness of breath two to three times per
(F) Tuberculosis week. Approximately every 2 weeks he awakens
at night due to cough and difculties breath-
8. A 55-year-old man was admitted to the hospital ing. He reports having similar symptoms since
2 weeks ago for rapid onset of cough, fatigue, he was a child, but believes that they are wors-
and pleuritic chest pain. He has worked as a ening somewhat now. His symptoms are wors-
sandblaster for the past year. When rst seen in ened by cold air and exercise and are improved
the hospital, he denied hemoptysis and smok- by rest. Which of the following is the most ap-
ing. Currently, the patient is intubated and on propriate treatment?
assist-control ventilation. His temperature is
(A) Daily high-dose inhaled corticosteroid and
36.7C (98F), pulse is 96/min, blood pressure
-agonist when needed
is 138/85 mm Hg, and respiratory rate is 18/
(B) Daily high-dose inhaled corticosteroid
min. A recent arterial blood gas study showed a
with oral steroids for exacerbations and
pH 7.42, arterial carbon dioxide pressure of 36
short-acting -agonist when needed
mm Hg, and arterial oxygen pressure of 110
(C) Daily low-dose inhaled corticosteroid and
mm Hg while on 100% oxygen. Physical exam-
short-acting -agonist when needed
ination is signicant for diffuse crackles
(D) Daily oral steroids and long-acting
throughout both lung elds, a loud pulmonic
-agonist
component of the second heart sound, and
(E) Short-acting -agonist when needed
jugular venous distention of 9 cm with a prom-
inent A wave, a left parasternal heave, and sym- 11. A 25-year-old man is recovering in the hospital
metric 3+ lower extremity pitting edema. from an open repair of his broken femur,
Which of the following is the most likely diag- which he suffered during an automobile acci-
nosis? dent. On postoperative day 3 he develops sud-
(A) Asbestosis den onset shortness of breath and vague chest
(B) Berylliosis discomfort. His temperature is 37.6C
(C) Byssinosis (99.6F), heart rate is 108/min, blood pressure
(D) Coal workers pneumoconiosis is 95/62 mm Hg, respiratory rate is 42/min, and
(E) Silicosis oxygen saturation is 89% on room air. Physical
HIGH-YIELD SYSTEMS
Chapter 14: Pulmonary Questions 413

examination is signicant for jugular venous inspiratory crackles in the lung bases. High-
distention to 9 cm and an accentuated pul- resolution CT shows patchy areas of ground
monic component of S2. A pulmonary angio- glass, reticular abnormalities, and traction
gram is shown in the image. Which of the fol- bronchiectasis. Laboratory ndings are normal
lowing is most likely to be decreased? except for an elevated erythrocyte sedimenta-
tion rate of 54 mm/hr. What is the recom-
mended treatment for this patients cough?
(A) Change the antihypertensive agent
(B) Prescribe amantadine
(C) Prescribe bronchodilators and long-term
domiciliary oxygen therapy
(D) Prescribe glucocorticoids
(E) Resect diseased lung elds

13. A 30-year-old woman presents to her physi-

Pulmonary
cians ofce because of 3 months of nonpro-
ductive cough, exertional dyspnea, fatigue,
Reprinted, with permission, from Brunicardi FC, Ander-
sen DK, Billiar TR, Dunn DL, Hunter JG, Matthews JB,
malaise, and blurred vision. She denies weight
Pollock RE, Schwartz SI. Schwartzs Principles of Surgery, loss, fever, chills, sweats, recent travel, or sick
8th edition. New York: McGraw-Hill, 2005: Figure 23-9. contacts. She works on the assembly line of an
electronics plant. Vital signs are unremarkable.
(A) Airway resistance Physical examination reveals she has tender
(B) Alveolar dead space red papules over her shins. The patient said
(C) Alveolar ventilation she rst noticed the bumps when she changed
(D) Pulmonary compliance oral contraceptive pills (her only medication),
(E) Pulmonary vascular resistance but assumed they would disappear. X-ray of the
chest shows bilateral hilar lymphadenopathy
12. A 51-year-old man presenting to the clinic for with pulmonary inltrates. Laboratory ndings
routine examination mentions that he has not are:
been able to get over the u. Further question-
ing claries that he has had a nonproductive WBC count: 5600/mm3
cough for the past 34 months and is unable to Hemoglobin: 14.3 g/dL
sustain his normal walking pace for prolonged Platelet count: 300,000/mm3
periods. He reports feeling more fatigued than Na+: 140 mEq/L
he recalls feeling last year. His medical history K+: 4.2 mEq/L
is signicant for hypertension, alcoholism, and Cl: 108 mEq/L
obesity. His blood pressure is well controlled Ca2+: 16 mg/dL
on losartan and hydrochlorothiazide. He has CO2: 24 mmol/L
no known drug allergies. His mother died of Blood urea nitrogen: 10 mg/dL
complications of type 2 diabetes mellitus. His Creatinine: 1.0 mg/dL
father had a fatal myocardial infarction at age Culture of bronchoalveolar lavage uid is neg-
56 years. The patient smokes half a pack of cig- ative. Which of the following is the most likely
arettes per day and has done so for the past 7 diagnosis?
years. He has worked all his adult life as an ac-
countant and has no known exposures to asbes- (A) Berylliosis
tos or organic dusts. His blood pressure is (B) Fungal infection
134/96 mm Hg, heart rate is 78/min, respira- (C) Lymphoma
tory rate is 16/min, temperature is 37C (D) Sarcoidosis
(98.6F), and oxygen saturation is 94% on (E) Tuberculosis
room air. Lung examination reveals bilateral
HIGH-YIELD SYSTEMS
414 Section I: Organ Systems Questions

14. A 30-year-old man presents to the resuscitation 16. A 5-year old girl is brought to the emergency
bay with gunshot wounds in the anterior and department in December by her mother, who
posterior left chest. Although in distress and complains that her daughter seems confused.
dyspneic, the patient is cooperative. He has a The mother reports that her daughter has com-
patent airway and is moving all extremities. His plained of intermittent headaches since the
pulse is 120/min, blood pressure is 120/90 mm two of them moved into the rst oor of an
Hg, and respiratory rate is 30/min. He has older apartment building 6 months ago. The
bounding distal pulses, and no other injuries mother has been at home with the daughter
are identied on secondary examination. X-ray for the past 24 hours and the girl appears le-
of the chest reveals uid in the pleural space, thargic and is complaining of joint aches, nau-
and a left chest tube thoracostomy yields 600 sea, and a headache. Her pulse is 120/min,
mL of bright red uid. Over the next hour 750 blood pressure is 130/85 mm Hg, respiratory
mL of blood is collected. What is the most ap- rate is 25/min, and oxygen saturation is 100%
propriate next step in management? on room air. The girls mother also notes hav-
ing a slight headache that started yesterday.
(A) Autotransfuse with the collected blood and
Pulmonary

Which of the following diagnostic tests should


continue to observe closely
be most rapidly pursued?
(B) Insert another chest tube
(C) Left thoracotomy (A) Arterial blood gas
(D) Remove the chest tube and suture the inci- (B) CT scan of the head
sion closed (C) Direct laryngoscopy
(E) Thoracentesis (D) ECG
(E) Toxicology screen
15. A 38-year-old man is being seen in his physi-
cians ofce after being involved in a car acci- 17. A 22-year-old college student presents to the
dent. He has a vague pain along his right ster- student health center because of a 10-day his-
nal border, where he crashed into the steering tory of headache, malaise, and low-grade fever.
wheel. His temperature is 36.6C (97.8F), He also complains of a nonproductive cough
pulse is 80/min, blood pressure is 123/75 mm for the past 7 days. X-ray of the chest shows
Hg, respiratory rate is 14/min, and oxygen satu- peribronchial inltrates. What is the most ap-
ration is 99% on room air. Physical examina- propriate empiric treatment for this patient?
tion is signicant for point tenderness over the
(A) Amoxicillin
right sternal border. X-ray of the chest shows
(B) Azithromycin
no broken ribs but a single, well-circumscribed
(C) Ceftriaxone
pulmonary nodule, 1.5 cm in diameter, lo-
(D) Penicillin
cated in the left lower lung eld. A search
through the patients electronic medical le re-
18. A 1-year-old child with cerebral palsy second-
veals that he had an x-ray of the chest taken 2
ary to perinatal asphyxia presents to her gen-
years ago. The radiology report from that time
eral pediatrician for a well-child visit. She was
reveals that the nodule was only 0.75 cm in di-
delivered at 37 weeks gestation by emergency
ameter. To characterize the lesion, CT of the
cesarean section for a tight nuchal chord. The
chest is performed and shows dense, occu-
patient has severe spastic quadriparesis that is
lated calcication within the lesion. Which of
limiting her movements. She also has mental
the following risk factors most increases the
retardation and is unable to speak. She has re-
chances of malignancy in this patient?
ceived physical and occupational therapy since
(A) Increased doubling time of tumor early infancy; however, her parents are con-
(B) Increased patient age cerned by her lack of improvement. Which of
(C) Nodule diameter of 1.5 cm or higher the following is the best choice for treatment of
(D) Presence of discrete border spasticity in this child?
(E) Presence of occulated calcication
HIGH-YIELD SYSTEMS
Chapter 14: Pulmonary Questions 415

(A) Baclofen
(B) Botulinum toxin
(C) Carbamazepine
(D) Discontinue physical therapy
(E) Hyperbaric oxygen

19. A 58-year-old man presents to the emergency


department complaining of fever and chills.
The fever started last night and has not sub-
sided, even though he took acetaminophen.
He had a successful appendectomy 3 days ago
and was discharged from the hospital 2 days
ago. His only medication is ibuprofen, which Reproduced, with permission, from Doherty GM, Way
is adequately controlling his pain. He is a LW. Current Surgical Diagnosis and Treatment, 12th edi-
tion. New York: McGraw-Hill, 2006: Figure 18-19.
30-pack-year smoker with a chronic cough pro-

Pulmonary
ductive of white sputum. He has noticed in- (A) Aspergillosis
creased sputum production, which has become (B) Klebsiella infection
yellowish-green. He denies dysuria, urgency, or (C) Lung cancer
frequency. His temperature is 38.4C (101.1F), (D) Sarcoidosis
heart rate is 88/min, respiratory rate is 16/min, (E) Tuberculosis
and blood pressure is 126/74 mm Hg. On exam-
ination he appears to be tired but not in acute 21. A 35-year-old HIV-positive man (CD4+ cell
distress. Pulmonary examination is limited be- count 150/mm) is seen in the emergency de-
cause deep inhalation causes coughing and partment with right-sided chest pain. The pa-
slight abdominal pain. There is no tactile fre- tient has become progressively dyspneic over
mitus or dullness to percussion. He has a the past few days. Suddenly, 30 minutes ago he
slightly erythematous, appropriately tender noticed a sharp pain in his chest associated
healing incision in the right lower quadrant with shortness of breath. His temperature is
without exudates and normal active bowel 37.7 (99.9F), blood pressure is 128/84 mm
sounds. Extremities are warm and well per- Hg, pulse is 102/min and regular, respiratory
fused without erythema or edema. Pulses are rate is 25/min, and oxygen saturation is 90% on
intact. Which of the following most likely room air. Physical examination reveals dimin-
could have prevented this condition? ished right-sided breath sounds and hyperreso-
nance. Jugular venous distention is 5 cm and
(A) Aggressive incentive spirometry
there is no tracheal deviation. ECG shows si-
(B) Early removal of the Foley catheter
nus tachycardia. X-ray of the chest shows a
(C) Early removal of the intravenous catheter
right-sided pneumothorax occupying approxi-
(D) Pre- and postoperative antibiotic prophy-
mately 10% of the right thoracic cavity. Which
laxis
of the following most likely caused this pa-
(E) Use of compression stockings and subcuta-
tients presentation?
neous heparin
(A) Intravenous drug use
20. A 21-year-old nonsmoking college student (B) Kaposis sarcoma
comes to the local emergency department be- (C) Mycobacterium tuberculosis
cause pf cough, weight loss, and low-grade fe- (D) Pneumocystis jiroveci pneumonia
ver. Occasionally his sputum is tinged with (E) Toxoplasmosis
blood. X-ray of the chest is shown in the image.
He reports traveling to Haiti on a medical mis-
sion trip several years ago. Which of the fol-
lowing is the most likely diagnosis?
HIGH-YIELD SYSTEMS
416 Section I: Organ Systems Questions

22. A 75-year-old man develops increased ventila- (E) Multiple pleural plaques with patchy pa-
tory requirements several days after requiring renchymal opacities
intubation for respiratory failure. X-ray of the
chest shows bilateral inltrates, and based on 25. A 23-year-old man is seen in the emergency
his ventilatory settings, the ratio of the partial department for sudden onset, right-sided pleu-
arterial pressure of oxygen to the fraction of in- ritic chest pain that developed 30 minutes ago
spired oxygen (PaO2:FiO2) is 190. Which of while he was watching television. The patient
the following is the most common underlying also complains of difculty breathing. He has
etiology of acute respiratory distress syndrome? no prior medical history, denies smoking and in-
travenous drug use, and does not take any medi-
(A) Aspiration of gastric contents
cations. His temperature is 37.3C (99.1F),
(B) Drug overdose
blood pressure is 130/82 mm Hg, pulse is 92/
(C) Lung or bone marrow transplantation
min and regular, respiratory rate is 20/min and
(D) Massive blood transfusion
shallow, and oxygen saturation is 98% on room
(E) Near-drowning
air. His body mass index is 18 kg/m. Dimin-
(F) Sepsis
Pulmonary

ished breath sounds, hyperresonance, and de-


creased tactile fremitus are prominent in the
23. A 31-year-old G4P3 woman gave birth via re-
right lung eld. The trachea is midline. X-ray of
peat cesarean section to a full-term, 3700-gm
the chest shows a 10% pneumothorax on the
(8.2-lb) baby girl. There were no complications
right. Which of the following is the most ap-
during the pregnancy or delivery. Two hours
propriate initial management?
after the birth the resident is called to evaluate
the baby girl. She is afebrile but is breathing (A) Needle decompression
rapidly with mild subcostal retractions. Breath (B) Observation with supplemental oxygen
sounds are equal and clear bilaterally. S1 and (C) Open thoracotomy with oversewing of the
S2 are normal and the point of maximal inten- pleural blebs and scarication of the
sity is not displaced. X-ray of the chest reveals pleura
attened diaphragms, prominent vascular (D) Thoracoscopy with stapling of blebs
markings, and uid lines in the ssures. Which (E) Tube thoracostomy with doxycycline pleu-
of the following is the most likely diagnosis? rodesis
(A) Diaphragmatic hernia
26. A 53-year-old man presents to the clinic with
(B) Neonatal respiratory distress syndrome
complaints of increasing shortness of breath, a
(C) Pulmonary hemorrhage
nagging cough, and weight loss over several
(D) Pulmonary interstitial emphysema
months. He reports no history of cigarette
(E) Transient tachypnea of the newborn
smoking but has worked underground in the
New York City subway system for the past 20
24. A 67-year-old man presents to his primary care
years. Spirometry demonstrates an FEV1:FVC
physician with complaints of dyspnea on exer-
ratio of 0.7 and an FEV1 value that is 60% of
tion over the past 6 months that has progres-
expected. The FEV1 improves to 70% of ex-
sively worsened to dyspnea at rest. He denies
pected with bronchodilator treatment. Which
cough and wheezing and has had no fevers,
of the following is the most likely diagnosis?
night sweats, or unintentional weight loss. The
man has never smoked and worked as a ship- (A) Asthma
builder for >30 years. Which of the following (B) Chronic aspiration
ndings on x-ray of the chest would conrm (C) Chronic obstructive pulmonary disease
the most likely diagnosis? (D) Histoplasmosis
(E) Tuberculosis
(A) Bilateral diffuse inltrates
(B) Bilateral hilar adenopathy
(C) Consolidation of lung tissue
(D) Focal mass with air bronchograms
HIGH-YIELD SYSTEMS
Chapter 14: Pulmonary Questions 417

27. A 22-year-old woman with mild persistent 29. A 55-year-old man presents to his physicians
asthma comes to the primary care clinic after ofce with increasing dyspnea on exertion. He
an emergency department visit 2 days ago for denies chest pain, diaphoresis, nausea, or vom-
an acute asthma exacerbation. She notes an in- iting. He has been involved in eight motor ve-
crease in frequency of wheezing and shortness hicle accidents in the past 3 years. Past medical
of breath for the past 4 months, with daily history is signicant for hypertension, for
symptoms, and has been symptomatic for at which he takes a diuretic. His temperature is
least 2 nights per week. She has also had three 37.2C (99.0F), blood pressure is 121/82 mm
emergency department visits during the same Hg, pulse is 85/min, respiratory rate is 14/min,
period. Her current asthma medications in- and oxygen saturation is 99% on room air.
clude montelukast (leukotriene inhibitor) daily Physical examination is signicant for a body
and an albuterol inhaler as needed. The pa- mass index of 35 kg/m, a diffuse and laterally
tients peak ow is 75% of predicted. Which of displaced point of maximal intensity, and an
the following is the most appropriate next step S3 gallop. Which of the following is the most
in management? appropriate next step in diagnosing his most

Pulmonary
likely underlying condition?
(A) Add a long-acting inhaled -adrenergic ag-
onist and low-dose inhaled steroid to the (A) Cardiac catheterization
regimen (B) Echocardiogram
(B) Add systemic steroids to the regimen (C) Exercise tolerance test
(C) Admit to the hospital for further pulmo- (D) Polysomnography
nary work-up (E) X-ray of the chest
(D) Discontinue the leukotriene inhibitor and
change the regimen to daily low-dose in- 30. A 74-year-old man presents to his primary care
haled steroids physician complaining of dyspnea and cough
(E) Start cromolyn sodium with blood-tinged sputum for the past several
weeks. He has diabetes and elevated choles-
28. A 78-year-old woman is seen in the emergency terol. Medications include a sulfonylurea and
department for difculty breathing and cough a statin. The patient has a 50-pack-year smok-
over the past 4 hours. She has a history of con- ing history and a family history of hyperten-
gestive heart failure for which she takes hydro- sion. His vital signs are within normal limits.
chlorothiazide, metoprolol, and enalapril. Her Physical examination reveals abdominal striae
oxygen saturation is 92% on room air. On ex- and moon facies, along with a truncal fat distri-
amination there is a high-pitched systolic cre- bution. X-ray of the chest reveals a single cen-
scendo-decrescendo murmur best heard at the tral nodule, and follow-up CT again demon-
right upper sternal border with radiation to the strates the nodule and multiple solid hepatic
carotids, and rales are present in both lung masses. Which of the following is the most
elds on inspiration. There is 2+ symmetrical likely diagnosis?
pitting edema bilaterally in the lower extremi-
(A) Adenocarcinoma of the lung
ties. X-ray of the chest shows an enlarged heart
(B) Carcinoma metastatic to the lung
and prominent pulmonary vasculature. Which
(C) Large cell carcinoma of the lung
of the following is the most likely cause of the
(D) Small cell carcinoma of the lung
patients pulmonary edema?
(E) Squamous cell carcinoma of the lung
(A) Decreased capillary uid oncotic pressure
(B) Decreased interstitial uid hydrostatic
pressure
(C) Increased capillary uid hydrostatic pres-
sure
(D) Increased capillary permeability
(E) Increased interstitial uid oncotic pressure
HIGH-YIELD SYSTEMS
418 Section I: Organ Systems Questions

31. A 35-year-old homeless man presents to the (C) Blastomyces dermatitidis


emergency department with chief complaints (D) Candida albicans
of a cough and fever. He is intoxicated. He ad- (E) Cryptococcus neoformans
mits to drinking about a fth of vodka every (F) Histoplasma capsulatum
day and conrms a history of delirium tremens (G) Mycobacterium tuberculosis
and blackouts. X-ray of the chest is signicant (H) Nocardia asteroides
for an air-uid level in the superior segment of (I) Sporothrix schenckii
the right lower lobe. Which of the following is
the most appropriate rst-line agent for treating 33. A 32-year-old farmer from Mississippi presents
this patients condition? to the physician because of fever and cough.
His symptoms are mild and developed gradu-
(A) Azithromycin
ally. X-ray of the chest shows pulmonary inl-
(B) Clindamycin
trates. Fungal culture on Sabouraud agar at
(C) Isoniazid
25C yielded nonencapsulated mycelial forms
(D) Moxioxacin
and spores. Blood agar culture at 37C yields
(E) Piperacillin-tazobactam
Pulmonary

yeast forms.
(F) Trimethoprim-sulfamethoxazole
34. A 32-year-old Hispanic immigrant presents to
32. A 5-month-old infant has failed to gain weight
the emergency department because of a non-
despite a good appetite. The childs mother re-
productive cough, fevers, night sweats, and an-
ports that the baby has up to eight bulky, foul-
orexia. X-ray of the chest shows cavitary pneu-
smelling, oily stools per day. A sweat chloride
monia and healed granulomas.
test reveals a chloride level of 78 mEq/L (nor-
mal: <60 mEq/L). Which of the following se-
quelae is most likely to occur as a result of this The response options for the next 3 items are
patients disease? the same. Select one answer for each item
in the set.
(A) Cirrhosis and subsequent hepatic failure
(B) Dehydration, electrolyte abnormalities, For each patient with pulmonary edema, select the
and acute hypotension most likely cause.
(C) Esophageal ulceration or strictures and up-
per gastrointestinal bleeding (A) Acute aortic insufciency
(D) Purple lines on the gums, red-brown dis- (B) Acute mitral regurgitation
coloration of the urine, and renal tubular (C) Acute respiratory distress syndrome
acidosis (D) Aspiration pneumonia
(E) Recurrent airway disease with eventual re- (E) Cardiac arrhythmia
spiratory insufciency associated with (F) Epidural hemorrhage
bronchiectasis (G) Epileptic seizures
(H) Myocardial infarction
(I) Rheumatic heart disease
E X T E N D E D M ATC H I N G
(J) Subarachnoid hemorrhage
(K) Subdural hemorrhage
The response options for the next 2 items are (L) Viral meningoencephalitis
the same. Select one answer for each item
in the set. 35. A 55-year-old man with a history of polycystic
kidney disease is on a plane from New York to
For each of the following patients, select the micro- Los Angeles when he develops a sudden onset,
organism that is most likely responsible for the in- intensely painful headache, neck stiffness, tac-
fection. tile fever, nausea, and vomiting. After the plane
(A) Actinomyces israelii lands the patient is rushed to the local emer-
(B) Aspergillus avus gency department, where he complains of
shortness of breath in addition to his persistent
HIGH-YIELD SYSTEMS
Chapter 14: Pulmonary Questions 419

headache. On examination there is no photo- (O) Squamous cell carcinoma


phobia, Brudzinskis and Kernigs signs are (P) Thymoma
both negative, and rales are auscultated on in-
spiration. X-ray of the chest shows a bat-wing 38. A 55-year-old man who was an unrestrained
appearance of the hilar shadows. driver in a motor vehicle accident was taken to
the emergency department and determined to
36. A 35-year-old woman presents to the emer- be uninjured. Three days later he presents to
gency department complaining of shortness of his physician because of tenderness over the
breath that began 3 hours ago while walking to right side of his chest. He has no other signi-
the store, but has still not subsided. She reports cant medical history and does not smoke. Vital
she had a severe bout of some throat infec- signs are normal. Physical examination reveals
tion as a child, for which she was not treated. bruising and tenderness over the eighth, ninth,
She denies a history of asthma or smoking. Her and tenth ribs on the right. Imaging studies are
vital signs are normal. On examination there is negative for fracture but do reveal a single 1.8-
a low-pitched diastolic rumble audible at the cm pulmonary nodule with a loculated pattern

Pulmonary
apex and an opening snap. of calcication in the apex of his right lung,
within 8 mm of the superior lobar bronchus.
37. A 22-year-old unresponsive man is transported
to the intensive care unit after being involved 39. A 43-year-old man from the central United
in a motor vehicle accident. The patient was States presents to his physician because of 6
unrestrained, and his car crashed head-rst months of productive cough, intermittent night
into a brick wall. He is on assist-control ventila- sweats, and a 6.8-kg (15-lb) unintentional weight
tion with a fraction of inspired oxygen of 0.85, loss. His past medical history is insignicant,
partial oxygen pressure of 90 mm Hg, and oxy- and he denies a history of smoking or recent
gen saturation of 94%. Vital signs are otherwise travel outside the United States. The patients
normal. X-ray of the chest shows interstitial last CD4+ cell count 2 weeks ago was 900/mm3.
pulmonary edema and bilateral perihilar alveo- His temperature is 37.8C (100.0F); other vital
lar edema, producing a characteristic butter- signs and physical examination ndings are un-
y pattern. Noncontrast CT of the head does remarkable. X-ray of the chest shows a 2.7-cm
not reveal intracranial hemorrhage. lesion in the right upper lung eld. Stain and
cultures are negative for acid-fast bacilli.
The response options for the next 3 items are
40. A 72-year-old man is seen in the doctors ofce
the same. Select one answer for each item
because of a nonproductive cough and 9-kg
in the set.
(20-lb) unintentional weight loss over the past
4 months. The patient has no signicant medi-
For each patient with a solitary pulmonary nodule,
select the most likely etiology. cal history, takes no medications, and denies
smoking. His temperature is 36.7C (98.1F),
(A) Adenocarcinoma blood pressure is 131/84 mm Hg, pulse is 75/
(B) Aspergilloma min and regular, and respiratory rate is 16/min.
(C) Bronchogenic cyst Physical examination is signicant for nger
(D) Carcinoid lung tumor clubbing. X-ray of the chest shows a focal 1.8-
(E) Cavitating abscess cm peripheral nodule in the left lower lobe.
(F) Coccidioidomycosis CT conrms the lesion and shows it does not
(G) Hamartoma invade the visceral pleura or lobar bronchus.
(H) Histoplasmosis
(I) Infection with Mycobacterium tuberculosis
(J) Lipoma
(K) Neurilemmoma
(L) Pulmonary arteriovenous malformation
(M)Rheumatoid nodule
(N) Sarcoidosis
HIGH-YIELD SYSTEMS
420 Section I: Organ Systems Answers

AN S W E R S

1. The correct answer is C. Primary lung cancer Answer E is incorrect. The paraneoplastic
is the leading cause of cancer death. The most syndrome associated with squamous cell carci-
signicant risk factor is smoking because it noma is hypercalcemia, due to the ectopic pro-
raises the lifetime relative risk of lung cancer duction of parathyroid hormone-related pep-
10- to 30-fold when compared with the lifetime tide (PTH-rP). Pathologic fractures and kidney
risk of nonsmokers. Patients with lung cancer stones are also commonly seen as a result of
can present with cough, hemoptysis, chest PTH-rP production.
pain, and unintentional weight loss. Physical
examination may reveal crackles or atelectasis. 2. The correct answer is A. Pulmonary function
Lung cancer is usually rst noted as a nodule tests measure several different lung volumes that
on x-ray of the chest but is best dened on CT. can differentiate obstructive and restrictive lung
Fine-needle aspiration under CT guidance or diseases, including FEV1, forced vital capacity
(FVC), residual volume (RV; the amount left in
Pulmonary

bronchoscopy with biopsy or brushings can


usually establish the histologic diagnosis. Gy- the lungs after forced exhalation), and total lung
necomastia, dened as the benign proliferation capacity (TLC; the total volume of the lungs af-
of glandular tissue in the male breast and seen ter inhalation). In obstructive lung diseases like
clinically as a rm mass extending concentri- asthma, lung volumes (TLC and RV) are typi-
cally from the nipple-areolar complex, is a fea- cally increased, while in restrictive lung diseases
ture of large cell carcinoma. Large cell carci- like sarcoidosis lung volumes are less than nor-
noma behaves similarly to poorly differentiated mal. In both obstructive and restrictive disease
non-small cell lung cancer and may produce FEV1 and FVC are reduced, but in obstructive
-human chorionic gonadotropin (-hCG), re- disease FEV1 is more dramatically reduced, re-
sulting in gynecomastia, milky nipple dis- sulting in a decreased FEV1:FVC ratio. In hy-
charge, and elevated serum concentrations of perresponsive airway obstruction like that seen
plasma -hCG. Large cell carcinoma of the in asthma, The FEV1 will improve with bron-
lung carries a poor prognosis (mean survival of chodilators.
719 months) and has often metastasized at Answer B is incorrect. RV and TLC are typi-
the time of diagnosis. cally normal or increased in asthma due to air-
Answer A is incorrect. Digital clubbing and hy- way obstruction that leads to air trapping and
pertrophic pulmonary osteoarthropathy can oc- hyperinated lungs.
cur with any histologic subtype of lung cancer, Answer C is incorrect. The FEV1 always de-
but they are most often associated with adeno- creases in asthma exacerbation.
carcinoma of the lung. Thrombophlebitis and
nonbacterial verrucous endocarditis are cardio- Answer D is incorrect. While increased RV
vascular complications of adenocarcinoma. and TLC are characteristic of asthma, in-
creased FEV1 is not.
Answer B is incorrect. Bronchoalveolar cell
carcinoma is a subtype of adenocarcinoma. It Answer E is incorrect. The FEV1 always de-
is most often associated with multiple nodules creases in asthma exacerbation.
on imaging studies, interstitial inltration, and
prolic sputum production. 3. The correct answer is C. Epiglottitis is a po-
tentially fatal condition that is characterized by
Answer D is incorrect. A variety of syndromes high fever, pharyngitis, dyspnea, and rapidly
associated with ectopic hormone production progressing respiratory obstruction. Staphylo-
are seen with small cell carcinoma, the subtype coccus aureus, Group A streptococci, Strepto-
of lung cancer highly associated with smoking. coccus pneumoniae, Haemophilus inuenzae
These include the syndrome of inappropriate type A and nontypeable strains, and Haemophi-
ADH secretion (SIADH) and the syndrome of
ectopic ACTH production.
HIGH-YIELD SYSTEMS
Chapter 14: Pulmonary Answers 421

lus parainuenzae are the most common and in pulmonary embolism (PE). Further-
pathogens. Haemophilus inuenzae type b is a more, tests for D-dimer are of limited diagnostic
less common cause of epiglottitis due to immu- utility in pregnancy because of the normal ele-
nization with vaccines. Epiglottitis is a pediat- vation of D-dimer in uncomplicated pregnancy.
ric emergency that often occurs in children X-ray of the chest may be normal initially but
27 years old. It typically manifests in a healthy may later show Westermark sign (dilatation of
child who has an abrupt onset of fever and sore pulmonary vessels and a sharp cutoff), atelecta-
throat, and then appears toxic with labored sis, small pleural effusion, and elevated dia-
breathing and difculty swallowing within sev- phragm. The ECG ndings described here are
eral hours. The child usually drools and keeps collectively nicknamed the S1Q3T3 pattern,
his or her neck hyperextended in an attempt to which is a nonspecic nding encountered in
maintain a patent airway. Laryngoscopy will re- any case of acute cor pulmonale (tension pneu-
veal a cherry-red, swollen epiglottis. Lateral ra- mothorax, bronchospasm, PE). An S1Q3T3 pat-
diographs of the upper airway show the thumb tern alone is not sufcient for diagnosis of PE. A
sign with a swollen epiglottis, as seen in the denitive diagnostic strategy is necessary in preg-

Pulmonary
image. Treatment requires creating an articial nant patients suspected of VTE. Ventilation/per-
airway by nasotracheal intubation. fusion lung scanning is currently considered the
diagnostic modality of choice in the pregnant
Answer A is incorrect. Appropriate antibiotic
population.
therapy should be started that provides cover-
age against Staphylococcus aureus, Group A Answer A is incorrect. Arterial blood gases are
streptococci, Streptococcus pneumoniae, and neither sensitive nor specic with regard to the
Haemophilus inuenzae. However, it is more diagnosis of PE. If an arterial blood gas analy-
important to secure the patients airway rst. sis were performed, respiratory alkalosis would
be a common nding because it is a feature of
Answer B is incorrect. Corticosteroids are not
both pregnancy and PE. Similar to the non-
indicated in the management of epiglottitis.
pregnant population, a normal partial oxygen
Answer D is incorrect. Epiglottitis is a true pressure, partial carbon dioxide pressure, and
pediatric emergency. The airway should be se- arterial-alveolar gradient is common with PE.
cured immediately if epiglottitis is suspected,
Answer B is incorrect. If the patient is stable
regardless of the degree of respiratory distress,
and has a nondiagnostic ventilation/perfusion
because 6% of affected children without an ar-
scan, lower extremity Doppler ultrasound is a
ticial airway die.
reasonable test to document deep vein throm-
Answer E is incorrect. Tracheostomy is reserved bosis (DVT). Because treatments for DVT and
for cases in which severe epiglottic edema pre- submassive PE are the same, identication of
vents intubation via the nasopharyngeal airway. DVT is sufcient to terminate the work-up of
PE.
4. The correct answer is E. Pregnancy is an inde-
pendent risk factor for the development of ve- Answer C is incorrect. MRI is a modality
nous thromboembolism (VTE). Studies have that can detect thigh and pelvic DVT in the
shown that the incidence of VTE during preg- nonpregnant population with a sensitivity ap-
nancy is approximately 1 in 1500, with the ma- proaching 100%. Comparable studies have not
jority of events occurring in the postpartum pe- been performed in the pregnant population.
riod and equal distribution of the remaining Furthermore, although no birth defects have
VTE events among the three trimesters. The been attributed to MRI, studies of safety have
clinical diagnosis of VTE during pregnancy is not been adequate to date. CT and ventilation/
insensitive and nonspecic because of normal perfusion scanning are superior to MRI in the
physiologic changes associated with pregnancy. evaluation of lung vasculature in cases of sus-
For example, lower extremity swelling and tac- pected PE.
hypnea are common in normal pregnancies Answer D is incorrect. Pulmonary angiogra-
phy is the most specic examination used to
HIGH-YIELD SYSTEMS
422 Section I: Organ Systems Answers

diagnose PE, and may be used in patients with the -agonist and anticholinergic agent to pro-
a high clinical pretest probability of PE, but a duce modest improvements in expiratory ow
low- or intermediate-probability ventilation/ rates, vital capacity, and arterial oxygen and
perfusion scan and negative lower extremity carbon dioxide levels in patients with moder-
ultrasound. Thus, pulmonary angiography ate to severe COPD. However, it would not
should be used only after these tests have been be used alone as a rst-line agent in an acute
tried. It can detect emboli as small as 12 mm. COPD exacerbation. In addition, serum levels
should be monitored in an attempt to mini-
5. The correct answer is A. This patient has mize toxicity. Nausea, tachycardia, and tremor
chronic obstructive pulmonary disease are reported side effects.
(COPD). Patients with COPD typically have
hyperinated lungs with attened diaphragms, 6. The correct answer is E. Given both the symp-
attenuated vascular markings, and a narrow toms and time course, twin B has developed
mediastinum. Exacerbations of COPD are of- neonatal respiratory distress syndrome (NRDS).
ten treated with an anticholinergic agent (ipra- NRDS and treatment with oxygen exposure
tropium bromide) plus a -agonist (a short-
Pulmonary

and mechanical ventilation predispose to reti-


acting one such as albuterol or a long acting nopathy of prematurity. NRDS and its treat-
-agonist such as salmeterol if administration ment also predispose to bronchopulmonary
of the short-acting agent is required too fre- dysplasia (also referred to as chronic lung dis-
quently). While asthma is a possible diagnosis, ease), persistent patent ductus arteriosus, inter-
this patients age, history of smoking, and previ- ventricular hemorrhage, and necrotizing en-
ous diagnosis of COPD make COPD exacer- terocolitis.
bation more likely. In asthma exacerbations,
Answer A is incorrect. Both twins are equally
intravenous steroids are rst-line therapy,
likely to develop apnea of prematurity because
whereas in COPD exacerbations, anticholin-
they are equally premature.
ergic agents are rst-line therapy.
Answer B is incorrect. Both twins are at
Answer B is incorrect. Infections are respon-
slightly increased risk of developing gastro-
sible for >50% of exacerbations, whether they
esophageal reux disease because they are pre-
be viral or bacterial, and in the setting of a
mature.
history of a change or increase in sputum, it
is recommended to treat empirically for bron- Answer C is incorrect. Both twins are at risk of
chitis/pneumonia even without an inltrate on hyperbilirubinemia because they are premature.
x-ray of the chest. However, in the emergency Answer D is incorrect. The development of
department setting, a -agonist, an anticholin- NRDS places the affected infant at a much
ergic agent, and supplemental oxygen would higher risk for developing certain complica-
provide faster symptomatic relief. tions specically associated with NRDS and its
Answer C is incorrect. Magnesium sulfate has treatments.
a bronchodilating effect that may be of benet
in asthma. Its use should be reserved for life- 7. The correct answer is A. Coccidioidomycosis,
threatening bronchospasm that is refractory to which is caused by a soil-dwelling fungus, is a
all other interventions. pulmonary infection endemic to the south-
western United States. The incubation period
Answer D is incorrect. N-acetylcysteine may can be from 14 weeks after exposure, but it
be used for its mucolytic/antioxidant proper- typically presents 1014 days after exposure.
ties, but in the acute setting, a combination of Patients with lymphoma or AIDS, patients of
a -agonist, an anticholinergic agent, and sup- Filipino, Native American, Mexican, or Afri-
plemental oxygen would provide faster symp- can-American descent, and pregnant women
tomatic relief. can often progress to the disseminated form
Answer E is incorrect. A methylxanthine such which can be fatal. Skin rashes can be part of
as theophylline may be added in addition to the presentation, including toxic erythema, er-
HIGH-YIELD SYSTEMS
Chapter 14: Pulmonary Answers 423

ythema nodosum, and erythema multiforme, sents with slowly progressive malaise, anorexia,
which appear to be part of a hypersensitivity re- weight loss, fever, and night sweats. Further-
action. Amphotericin B, ketoconazole, uco- more, acid-fast bacteria can be seen on sputum
nazole, and itraconazole can all be used as examination, although this is not 100% sensi-
therapies. tive for nding the bacteria.
Answer B is incorrect. Histoplasmosis is linked
8. The correct answer is E. Acute silicosis devel-
to bat exposure and bird droppings and is es-
ops in persons exposed to exceptionally high
pecially common in the Ohio and Mississippi
concentrations of silica. Silicosis can occur in
river valleys. Patients are usually asymptomatic
mining, quarrying, or construction that in-
unless they are immunocompromised. If dis-
volves drilling, cutting, grinding, blasting, or
seminated histoplasmosis is suspected, the diag-
crushing the earths crust. Acute silicosis symp-
nosis is made by culturing the affected organ,
toms develop suddenly and include weight
or by a urine polysaccharide antigen screen.
loss, fatigue, chest pain, and cough. On exami-
Answer C is incorrect. Lung cancer is not nation, patients have diffuse crackles, signs of

Pulmonary
likely in this young patient, even one who is cyanosis and cor pulmonale, and respiratory
immunocompromised. Also, the onset is fairly failure. Signs of cor pulmonale in this patient
acute to be a malignancy. include the loud pulmonic component of the
second heart sound, which indicates pulmo-
Answer D is incorrect. Although this patient
nary artery hypertension, and the prominent A
has risk factors for an opportunistic infection,
wave and left parasternal heave, which both in-
Pneumocystis jiroveci pneumonia usually pre-
dicate right ventricular hypertrophy. X-ray of
sents with a decreased oxygen saturation and
the chest usually shows silicoproteinosis, the
a history of shortness of breath, classically with
hallmark of acute silicosis. There is a charac-
diffuse ground-glass inltrates on x-ray. Fur-
teristic basilar alveolar pattern, without
thermore, P. jiroveci pneumonia is not com-
rounded opacities or calcications. For patients
monly seen until the CD4+ cell count falls
with acute silicosis, survival beyond 4 years af-
below 200/mm.
ter the onset of symptoms is rare. Patients are
Answer E is incorrect. Sarcoidosis is a disease encouraged to avoid further exposure and to
that is diagnosed by the ndings on x-ray of the quit smoking; however, no specic treatment
chest of bilateral hilar lymphadenopathy and cures or alters the course of the disease.
reticular opacities. Fifty percent of patients are
Answer A is incorrect. Asbestosis is most com-
asymptomatic and the ndings on x-ray of the
monly encountered by workers involved in the
chest are incidental. In those patients that are
mining, milling, and manufacturing of asbestos
symptomatic, the traditional restrictive lung
products, as well as those involved in the build-
disease symptoms of cough, dyspnea, and chest
ing trades of pipe tting and boiler making.
pain are the most common complaints. In the
The major health effects of asbestos, such as
patient described above, the onset of symp-
pulmonary brosis, lung cancer, and mesothe-
toms is unusually acute for sarcoidosis and the
lioma, do not present acutely.
temporal association with travel to a region en-
demic for coccidioidomycosis makes that the Answer B is incorrect. Berylliosis most com-
more likely cause. monly produces a chronic interstitial pneu-
monitis. Berylliosis is encountered in high
Answer F is incorrect. Tuberculosis (TB) is
technology industries, such as aerospace, nu-
not likely, as he had a negative puried pro-
clear, and electronics plants; ceramics plants;
tein derivative (PPD) test, so this option is less
plating facilities; and dental material and dye
likely than coccidioidomycosis. However, pa-
manufacturing.
tients with end-stage HIV (CD4+ cell count
<200/mm) can also have a falsely-negative Answer C is incorrect. Byssinosis is caused by
PPD test. Extrapulmonary TB is often seen inhalation of organic dust, specically cotton
in HIV patients with lymphadenitis and/or dust. After exposure for 10 years, workers with
miliary disease. A person with TB usually pre- recurrent symptoms such as chest tightness or
HIGH-YIELD SYSTEMS
424 Section I: Organ Systems Answers

a decrease in FEV1 may develop an obstructive 10. The correct answer is C. This patient has mild
pattern on pulmonary function testing. persistent asthma, dened as daytime symp-
toms two or more times per week and night-
Answer D is incorrect. Coal workers pneu-
time symptoms as often as once every 2 weeks.
moconiosis (CWP) has important medical
The most appropriate treatment choice for
signicance in nations where the coal min-
mild persistent asthma is a daily inhaled corti-
ing industry is important. CWP can cause
costeroid with a short-acting 2-agonist as
the development of chronic bronchitis and
needed, although recent studies show that the
obstructive lung disease. Furthermore, the
inhaled corticosteroid may be equally effective
chest radiographic ndings in CWP are starkly
if taken only in relation to symptoms.
different than those seen in acute silicosis. In
simple CWP, radiographic abnormalities con- Answer A is incorrect. This answer is an appro-
sist of small (1- to 5-mm in diameter), irregular priate treatment regimen for moderate persis-
opacities. In complicated CWP, nodules range tent asthma, depending on the degree of sever-
in size from 1 cm to involvement of an entire ity of symptoms. Moderate persistent asthma is
lobe and are generally conned to the upper dened as daytime symptoms daily and night-
Pulmonary

lung zones. time symptoms at least once each week in the


absence of treatment. There are asthma attacks
9. The correct answer is A. Combination therapy at least two times weekly.
with isoniazid, rifampin, ethambutol, and
Answer B is incorrect. This answer is an ap-
pyrazinamide is the standard rst-line therapy
propriate treatment regimen for moderate
for TB. They are all available in single daily
persistent asthma, depending on the degree
oral doses. The important adverse effect of
of severity of symptoms. Moderate persistent
ethambutol is optic neuritis, which is rare at
asthma is dened as daytime symptoms daily
usual therapeutic doses.
and nighttime symptoms at least once each
Answer B is incorrect. Hepatotoxicity is the week in the absence of treatment. There are
main concern with isoniazid. asthma attacks at least two times weekly.
Answer C is incorrect. Of the antituberculosis Answer D is incorrect. This is a treatment op-
drugs listed, levooxacin is the only one that is tion for severe persistent asthma, which typi-
not considered to be rst-line therapy but may cally requires multiple adjunctive treatment
be a reasonable choice as a second-line drug strategies. Severe asthma is dened as symp-
in the setting of resistance or toxicity to other toms constantly throughout the day in the
medications. Note that it is not approved by absence of treatment, and frequent day and
the US Food and Drug Administration for this nighttime attacks.
use.
Answer E is incorrect. This is the most appro-
Answer D is incorrect. As with isoniazid and priate treatment for mild intermittent asthma,
rifampin, hepatotoxicity of pyrazinamide is an dened as daytime symptoms no more than two
important concern. Gastrointestinal intoler- times per week, and nighttime symptoms no
ance is common. more than two times per month, which requires
Answer E is incorrect. Rifampin may cause an no daily treatment.
orange discoloration to bodily uids, including
11. The correct answer is D. The patients presen-
tears, sweat, and urine. A major adverse effect is
tation (history of stasis, tachypnea, dyspnea,
the induction of hepatic enzymes, which may
tachycardia, hypotension, low-grade fever, ele-
decrease the efcacy of other medications.
vated jugular venous distension, and accentu-
Answer F is incorrect. Streptomycin must ated pulmonic component of the second heart
be given parenterally, thus limiting its use. It sound) in combination with the lling defect
may still be used in the rst-line treatment of seen on CT angiography are diagnostic for PE.
TB. Ototoxicity is the major concern with this PE has many effects on pulmonary physiology.
drug. The compliance of the lungs, which is dened
HIGH-YIELD SYSTEMS
Chapter 14: Pulmonary Answers 425

as the volume change per unit pressure change, started, and if deterioration continues a second
decreases. The decrease occurs for a number of agent is added, such as cyclophosphamide or
reasons, such as lung edema, lung hemorrhage, azathioprine. Smoking cessation and supple-
and loss of surfactant. mental oxygen can also help. The outlook for
patients with idiopathic pulmonary brosis is
Answer A is incorrect. Airway resistance in-
bleak: the 5-year survival rate is 30%50%.
creases, rather than decreases, as a result of
PE. The increase in airway resistance is due to Answer A is incorrect. Angiotensin-converting
bronchoconstriction of unused/underperfused enzyme (ACE) inhibitors are associated with a
portions of lung. chronic nonproductive cough, which is attrib-
uted to the build-up of bradykinins that are typ-
Answer B is incorrect. Alveolar dead space is
ically broken down by the same enzyme. This
dened as an area of the lung that is ventilated
is frequently the cause of discontinuation of
but not perfused. Anatomically, this usually
ACE inhibitor therapy, and angiotensin recep-
represents the area at the top of the lung. In
tor blockers (ARBs) are often instituted instead.
PE, alveolar dead space increases, rather than
One percent of patients taking ARBs develop

Pulmonary
decreases, due to vascular obstruction.
cough as an adverse effect, compared with
Answer C is incorrect. In PE, alveolar venti- 5%20% of patients taking ACE inhibitors. Of
lation increases, rather than decreases, due to patients who developed a cough during treat-
stimulation of irritant receptors. Alveolar ven- ment with ACE inhibitors, approximately 80%
tilation is dened as the volume of fresh air have no cough with ARBs.
reaching the alveoli per minute. Mathemati-
Answer B is incorrect. Amantadine and riman-
cally it is expressed as: VA = (tidal volume
tadine both treat inuenza A and can decrease
anatomic dead space) respiratory rate. In
symptom duration by 50% if started within
PE, the increase in respiratory rate causes an
48 hours of onset of illness. Adverse effects of
increase in alveolar ventilation.
amantadine include anxiety, insomnia, livedo
Answer E is incorrect. Pulmonary vascular resis- reticularis, and ataxia. Rimantadine has fewer
tance is derived from Ohms law (R = V/I) and is central nervous system adverse effects and is
dened as: (input pressure output pressure) / equally effective.
blood ow. If an embolus blocks one or several
Answer C is incorrect. COPD typically devel-
lobes of the lung, then pulmonary vascular resis-
ops in those with more than a 20-pack-year ex-
tance rises. This rise, however, is less than what
posure to smoking. With less of a smoking his-
might be expected because pulmonary arterial
tory or in younger individuals, 1-antitrypsin
pressure rises, causing the pulmonary vascular
deciency should be suspected. This disease
resistance of nonembolized lung to decrease.
also presents with dyspnea, cough (although
generally productive), and weight loss. Physical
12. The correct answer is D. Idiopathic pulmonary
examination features of pursed-lip respirations,
brosis typically presents with dyspnea, dry
cough, and constitutional symptoms (weight mid-inspiratory crackles, and wheezing may be
loss, fatigue, and anorexia). The average age of present. X-ray of the chest typically shows hy-
presentation is 50 years, and often patients recall perinammation and emphysema. Bullae pres-
their symptoms as beginning with a viral respira- ent as local radiolucencies. Pulmonary func-
tory infection. CT ndings are typical of intersti- tion tests are a critical part of the diagnosis, with
tial lung disease (ILD) with patchy areas of a decreased vital capacity and an FEV1 that is
ground glass, reticular abnormalities, and trac- even further decreased, yielding a reduced ratio.
tion bronchiectasis. This patient has no history Treatment includes smoking cessation (prolongs
of an offending agent to cause ILD; however, in survival), bronchodilators, anticholinergics, and
patients whose ILD has a potential trigger, the theophylline derivatives (for symptom relief).
most important part of treatment is removal of Oxygen administration at home can improve
that exposure. Therapy in idiopathic pulmonary survival in patients who have stopped smoking
brosis is generally directed toward suppressing and are chronically hypoxemic (oxygen satura-
the inammatory process. Oral prednisone is tion 88% or lower).
HIGH-YIELD SYSTEMS
426 Section I: Organ Systems Answers

Answer E is incorrect. Lung volume reduc- Answer C is incorrect. X-ray of the chest may
tion surgery improves symptoms and survival be consistent with lymphoma, either Hodg-
in patients with upper lobe-predominant em- kins or non-Hodgkins. However, the patient
physema. lacks the symptoms and signs typically present
in patients with lymphoma: fever (temperature
13. The correct answer is D. The combination of >38.5C [101.3F]), night sweats, 10% weight
constitutional symptoms, respiratory com- loss over the preceding 6 months, and hepato-
plaints, erythema nodosum, blurred vision, and splenomegaly.
bilateral hilar lymphadenopathy in a young
Answer E is incorrect. While x-ray of the chest
adult strongly suggests the diagnosis of sarcoi-
can be indicative of pulmonary TB, it is difcult
dosis; however, a biopsy of the pulmonary
to explain the other ndings present in the case
lymphadenopathy, which showed noncaseating
by this diagnosis. TB does not cause hypercal-
granulomas, would be necessary to make a de-
cemia or erythema nodosum. Rather, ndings
nitive diagnosis. Bilateral hilar lymphadenop-
sometimes seen in TB, but not present in this
athy on x-ray of the chest is often seen in sar-
patient, are hyponatremia from SIADH, leuko-
Pulmonary

coidosis, but a similar pattern can be seen in


cytosis, fever, and chest pain.
lymphoma, TB, fungal infection, and brucello-
sis. Elevated serum ACE levels and hypercal-
14. The correct answer is C. Patients with a he-
cemia are other ndings characteristic of sar-
mothorax that is bleeding at a rate >200 mL/hr
coidosis. The Kveim-Siltzbach skin test, which
require an urgent thoracotomy to control the
entails an intradermal sarcoid protein injection
hemorrhage. It is likely that the internal tho-
and is positive in 70%80% of patients with
racic and/or intercostal arteries are lacerated.
sarcoidosis, is now less commonly used with
the advent of transbronchial biopsy of lung pa- Answer A is incorrect. Hemothoraces that are
renchyma. less severe (<200 mL/hr) can be treated with
autotransfusion of the collected blood.
Answer A is incorrect. Workers in high-tech-
nology elds such as aerospace, nuclear, and Answer B is incorrect. Inserting another chest
electronics plants, ceramics industries, plating tube will not help control the bleeding, which
facilities, dental material sites, and dye manu- is the priority.
facturing, can develop an ILD called beryl- Answer D is incorrect. Removing the chest
liosis. In berylliosis, x-ray of the chest shows tube will not correct the source of bleeding,
bilateral hilar lymphadenopathy with intersti- and the patient is at risk for exsanguinating.
tial inltrates, and a biopsy of the lesion shows
noncaseating granulomas. Both ndings are Answer E is incorrect. Thoracentesis is used
similar to those in sarcoidosis. Sarcoidosis can for diagnosis when the etiology of pleural uid
be distinguished from berylliosis by measuring is uncertain. In the setting of a gunshot wound
tissue levels of beryllium. While this patient and bright red uid in the pleural space, he-
does work in an electronics plant and may have mothorax is highly suspected, and urgent at-
a component of berylliosis, her overall clinical tention is needed due to the extent of bleeding.
picture is more consistent with sarcoidosis.
15. The correct answer is A. A solitary pulmonary
Answer B is incorrect. The distinction be- nodule is dened as a lung nodule, <3 cm in
tween pulmonary fungal infection and sarcoi- size that is discovered on x-ray or CT of the
dosis is important. After all, starting a patient chest. Solitary pulmonary nodules have a 40%
on systemic steroids for sarcoidosis can be di- chance of being malignant. Review of old ra-
sastrous if the patient actually has a fungal in- diographs is crucial because they can give an
fection. However, the clinical picture is more indication of the nodules doubling time. A
consistent with sarcoidosis, and the negative doubling time between 20 and 400 days is more
bronchoalveolar lavage culture argues against concerning for malignancy. For example, small
a fungal infection.
HIGH-YIELD SYSTEMS
Chapter 14: Pulmonary Answers 427

cell cancer doubles every 30 days, adenocarci- 16. The correct answer is A. It is critical to obtain
noma doubles every 180 days, and well- an arterial blood gas analysis, including mea-
differentiated cancers have an even longer dou- surements of carboxyhemoglobin and methe-
bling time. Benign processes generally have moglobin (which must be specically re-
doubling times that are <20 days (infection) or quested), to establish a diagnosis and assess
>450 days (old granulomas). The doubling hypoxemia in a patient who presents with a his-
time for this patient is not 2 years. On x-ray of tory and symptoms suggestive of carbon monox-
the chest a pulmonary nodule appears as a ide poisoning or toxic ingestion. Common
coin-shaped lesion, when in actuality it is best symptoms include dyspnea, headache, lethargy,
approximated as a three-dimensional sphere. and depressed mental status. Pulse oximetry
Thus, if a calculation is performed for the vol- cannot be used to rule out hypoxemia in the set-
ume of the nodule/sphere, we observe close to ting of carbon monoxide poisoning, since the
an eightfold increase in size. Thus, the lesion saturation recorded is the sum of oxygen- and
doubled in size three times during the 2-year carbon monoxide-bound hemoglobin; however,
period, meaning that the doubling time was a low saturation by pulse oximetry is an espe-

Pulmonary
2 years divided by 3, or every two-thirds of a cially ominous sign. Arterial blood gas analysis is
year (243.5 days). also important to assess for metabolic acidosis,
which could be secondary to hypoxia.
Answer B is incorrect. The likelihood of ma-
lignancy increases with age. In one study, the Answer B is incorrect. This is an impor-
chances of a nodule being malignant as a func- tant part of the work-up of carbon monoxide
tion of age were 3% in patients 3539 years old, poisoning and other causes of mental status
15% in those 4049 years old, 43% in those change in a child, but assessment for hypox-
5059 years old, and 50% in those >60 years emia is most urgent.
old. In another study, nodules in patients >50
Answer C is incorrect. Laryngoscopy and
years old had a 65% likelihood of malignancy,
bronchoscopy are important if smoke inhala-
whereas those in patients <50 years old had a
tion is suspected; look for singed nose hairs,
33% likelihood of being malignant. Given the
facial burns, hoarseness, wheezing, and car-
patients young age (38 years), doubling time is
bonaceous sputum. These patients require
a much more likely risk factor for malignancy.
early intubation since upper airway edema can
Answer C is incorrect. Nodule size is directly quickly become complete obstruction.
related to the likelihood of malignancy. Gener-
Answer D is incorrect. An ECG would be es-
ally, a nodule >3 cm in diameter signicantly
pecially important in the assessment of carbon
increases the likelihood of malignancy. In one
monoxide poisoning in elderly patients and
study, 80% of nodules >3 cm were malignant
other patients with a history of cardiac disease
compared with 20% in nodules <2 cm.
to evaluate the extent of ischemic damage sec-
Answer D is incorrect. Benign nodules tend ondary to the carbon monoxide ingestion. The
to have a smooth, discrete border, while malig- diagnosis of carbon monoxide poisoning and
nant lesions have an irregular border. degree of hypoxemia should be obtained rst in
this case.
Answer E is incorrect. The presence of certain
patterns of calcication (best seen on CT) favors Answer E is incorrect. A toxicology screen is
the lesion being benign. Such patterns include an important part of the differential diagnosis
popcorn calcication (as in this patient), dif- in this scenario, but is not the most urgent di-
fuse/homogenous calcication, central calci- agnostic, given the high suspicion of carbon
cation, and laminated/concentric calcication. monoxide poisoning based on the history and
However, the existence of areas of calcication the need to assess for hypoxemia.
is not associated with a benign lesion. An eccen-
tric lesion having an asymmetric area of calci- 17. The correct answer is B. Epidemiologically, a
cation should raise concern of malignancy. young patient with these symptoms and ndings
HIGH-YIELD SYSTEMS
428 Section I: Organ Systems Answers

most likely has atypical, or walking, pneumo- nia is the involuntary and sometimes painful
nia. The symptoms of atypical pneumonia, contractions of muscle tissue leading to jerking
which is caused by Chlamydia pneumoniae, and twisting movements of body parts.
Mycoplasma pneumoniae, and Legionella pneu-
Answer D is incorrect. Physical therapy is an
mophila, include fever, chills, myalgias, malaise,
established component of the management of
shortness of breath, and nonproductive cough.
cerebral palsy. It is started and continued at
M. pneumoniae often affects younger people,
home by the parents, with the intention of re-
including school-aged children, military re-
ducing muscle tone.
cruits, and college students. Radiographically,
several patterns can be seen on x-ray of the Answer E is incorrect. Hyperbaric oxygen does
chest, but there is no pathognomonic appear- not improve spasticity in children with cerebral
ance for atypical pneumonia. Peribronchial palsy.
pneumonia is the most common. The mac-
rolide antibiotics are the preferred therapy for 19. The correct answer is A. The most likely cause
atypical pneumonia. Azithromycin is favored of this patients fever is pneumonia. The mne-
Pulmonary

because of its high activity, low resistance, and monic for postoperative fever is the 6 Ws:
decreased gastrointestinal adverse effects as Wind (pneumonia, atelectasis), Water (urinary
compared to erythromycin. tract infection), Wound, Wonder drugs (i.e.,
erythromycin, isoniazid, penicillin, captopril, as-
Answer A is incorrect. Amoxicillin is not an ef- pirin, allopurinol, and heparin), Walking
fective treatment for atypical pneumonia. (DVT), and What happened (medical inter-
Answer C is incorrect. Ceftriaxone is not an ventions such as blood transfusions or intrave-
effective treatment for atypical pneumonia. nous lines). Immediately (approximately within
the rst 24 hours), the most common causes are
Answer D is incorrect. Penicillin is not an ef-
postoperative inammation, streptococcal or
fective treatment for atypical pneumonia.
clostridial wound infections, or transfusion reac-
tion. In the acute postoperative period (approxi-
18. The correct answer is A. Cerebral palsy is a
mately 37 days), the most common cause of fe-
group of clinical syndromes characterized by
ver is pneumonia, which is often associated with
motor and postural dysfunction due to disor-
intraoperative ventilator use, or previously ac-
ders of early brain development. Cerebral palsy
quired infections. Other causes at this time in-
ranges in severity and can result from isch-
clude urinary tract infection, intravenous site in-
emic, metabolic, genetic, infectious, develop-
fection/thrombosis, or noninfectious wound
mental, and acquired etiologies. It is often as-
complication, such as hematoma or foreign body
sociated with epilepsy and visual, speech, and
reaction. Incentive spirometry encourages air
intellectual abnormalities. Some patients may
movement and clearance of secretions, and has
exhibit spasticity, or strong sustained contrac-
been shown to decrease the occurrence of post-
tions and hyperreexivity. Baclofen is a muscle
operative pneumonia. In a patient with underly-
relaxant that reduces the involuntary, abnor-
ing lung disease, the risk of acquiring a respira-
mal movements and posturing affecting these
tory infection following surgery is increased.
patients by inhibiting reex pathways in the
spinal cord. Answer B is incorrect. Although a urinary
tract infection is in the differential diagnosis,
Answer B is incorrect. Botulinum toxin is be-
it is much less likely. There are no urinary
ing studied for the treatment of spasticity in
symptoms, and there is no history of prolonged
specic muscle groups. It is not recommended
catheterization. The risk of acquiring a urinary
for children <18 years old and is used primarily
tract infection postoperatively increases with
in calf muscles. However, it may be considered
duration of catheterization.
for focal aspects of generalized spasticity.
Answer C is incorrect. Findings suggestive of
Answer C is incorrect. Carbamazepine is an
catheter infection, such as localized erythema,
antiepileptic that has been used in children
tenderness, and swelling, are not seen in this
who have cerebral palsy with dystonia. Dysto-
case.
HIGH-YIELD SYSTEMS
Chapter 14: Pulmonary Answers 429

Answer D is incorrect. A wound infection Answer C is incorrect. Lung cancer can cause
(nonclostridial) is the most common cause of wasting, low-grade fevers, cough, and night
fever in the subacute postoperative period (ap- sweats. It is certainly a possible cause of pul-
proximately 1 week to 1 month). The timing monary nodules; however, while it may be
(3 days postoperation) and benign surgical site hard to immediately discount the possibility of
described in the case make this diagnosis less its occurrence, the constellation of symptoms,
likely. including the lack of a smoking history, and
history of international travel should make one
Answer E is incorrect. DVT typically presents
lean more heavily towards TB.
in 46 days; however, no risk factors (e.g., pro-
longed immobilization or hypercoagulable Answer D is incorrect. Sarcoidosis is an idio-
state) are mentioned, and the patient is not pathic disorder that typically presents in Afri-
exhibiting symptoms of calf tenderness or leg can-Americans in their 20s to 30s. About 50%
swelling to suggest this diagnosis. of patients realize that they have sarcoidosis
by an x-ray of the chest, which reveals bilat-
20. The correct answer is E. The classic symp- eral mediastinal adenopathy. Sarcoidosis can

Pulmonary
toms of cough, low-grade fevers, and wasting mimic TB, but is a diagnosis of exclusion, and
should raise the suspicion of pulmonary TB, TB should be ruled out before this diagnosis is
given the patients age, clinical presentation, entertained.
and history of previous travel to an area with a
high incidence of TB. The addition of ab- 21. The correct answer is D. Pneumothorax is an
scesses and uid levels on x-ray demonstrate uncommon but potentially fatal complication
that the pneumonia is cavitary. TB is one of of HIV believed to occur in 2%6% of HIV pa-
the more prominent cavitary pneumonias. tients at some point during their infection.
Blood streaking of the sputum is frequently More than 80% of the cases of pneumothorax
documented as in this patient. Some other risk in HIV patients occur in conjunction with
factors for TB include HIV-positivity, imprison- Pneumocystis jiroveci pneumonia. Extensive tis-
ment, homelessness, and malnourishment. A sue invasion within the alveolar interstitium is
PPD test can be used to screen for a latent in- common in severe cases, and may be an im-
fection, but will tell you nothing in a patient portant factor in causing necrosis and subse-
with active TB. The rst-line treatment for TB quent pneumothorax. Patients with pneu-
is a combination of four drugs: isoniazid, ri- mothorax present with the sudden onset of
fampin, ethambutol, and pyrazinamide for a unilateral pleuritic chest pain and dyspnea.
minimum of 6 months. Physical examination can reveal decreased or
absent breath sounds, hyperresonance, and de-
Answer A is incorrect. Aspergillus infection
creased tactile fremitus on the affected side.
in the lung can cause invasive aspergillosis,
aspergilloma, chronic necrotizing Aspergillus Answer A is incorrect. Pneumothorax may re-
pneumonia, and allergic bronchopulmonary sult from unsuccessful attempts to inject drugs
aspergillosis, none of which will offer a more into the central circulation via the subclavian
appropriate diagnosis than TB. Aspergillus is and jugular veins (pocket shots). However,
often seen in neutropenic patients and is seen there is no evidence that this patient uses in-
as a fungus ball with cavitation on x-ray of jection drugs.
the chest. The treatment for Aspergillus infec-
Answer B is incorrect. Kaposis sarcoma is a vas-
tion is amphotericin B or itraconazole.
cular tumor arising from infection with human
Answer B is incorrect. Klebsiella produces lo- herpesvirus 8. Kaposis sarcoma is the most com-
bar pneumonias that more classically involves mon tumor arising in patients with AIDS and
currant jelly sputum. Klebsiella pneumonia is 20,000 times more common in patients with
is often seen in alcoholics and diabetics, and is AIDS than in the general population. While Ka-
often antibiotic-resistant. posis sarcoma does occur in conjunction with
pneumothorax in patients with AIDS, it is not as
frequent as Pneumocystis jiroveci pneumonia.
HIGH-YIELD SYSTEMS
430 Section I: Organ Systems Answers

Answer C is incorrect. Mycobacterium tuber- born (TTN). TTN is caused by a temporary


culosis is a mycobacterial opportunistic infec- pulmonary edema that results from delayed
tion encountered in AIDS. M. tuberculosis has clearance of fetal lung uid. TTN typically
no association with pneumothorax in patients presents with mild to moderate respiratory dis-
with AIDS. tress within the rst 6 hours of life, manifested
by tachypnea, cyanosis, subcostal retractions,
Answer E is incorrect. Toxoplasmosis is a pro-
nasal aring, and grunting. In mild TTN respi-
tozoan opportunistic infection encountered in
ratory distress persists for 1224 hours, but can
AIDS. Toxoplasmosis has no association with
last up to 4872 hours in more severe cases.
pneumothorax in patients with HIV.
Breath sounds are usually clear but can be
coarse. Risk factors for TTN include prema-
22. The correct answer is F. Acute respiratory dis-
ture birth, precipitous birth, and cesarean sec-
tress syndrome (ARDS) is the presence of pul-
tion. These are thought to contribute to im-
monary edema in the absence of volume over-
load or depressed left ventricular function. paired clearance of fetal lung uid due to the
Early disease may manifest with only mild tac- absence of the thoracic squeeze the baby
Pulmonary

hypnea. Sepsis accounts for the majority of normally experiences during passage through
cases of ARDS and is an especially important the birth canal. However, regardless of gesta-
cause of ARDS in patients with a history of al- tional age and method of delivery, all infants
coholism. The PaO2:FiO2 ratio is used to as- are at risk for developing TTN.
sess respiratory status. Values <300 in the set- Answer A is incorrect. X-ray of the chest in an
ting of bilateral x-ray of the chest inltrates and infant with a diaphragmatic hernia would re-
a pulmonary capillary wedge pressure of <18 veal air-lled bowel in the chest. The physical
mm Hg dene acute lung injury (ALI), a pul- exam of this patient is not consistent with a dia-
monary syndrome characterized by noncardio- phragmatic hernia, in which one would expect
genic pulmonary edema. ARDS is a more se- absent or decreased breath sounds on the side
vere form of ALI with a PaO2:FiO2 ratio <200. of the hernia and heart sounds displaced to the
Answer A is incorrect. Aspiration of gastric opposite side of the thorax. Clinically, patients
contents is a less common cause of ARDS. with diaphragmatic hernias are usually in se-
vere respiratory distress.
Answer B is incorrect. Drug overdose of medi-
cations such as aspirin, opioids, tricyclic anti- Answer B is incorrect. In NRDS, x-ray of the
depressants, and cocaine may result in ARDS, chest reveals a ne reticular granular pattern
but it is not the most common cause. and air bronchograms. Although this neonates
history is consistent with NRDS, more acute
Answer C is incorrect. Lung or bone mar- respiratory distress would be expected. Also,
row transplantation is a cause of ARDS, but is this infant is at low risk of developing NRDS
much less common than sepsis in causing this because she is full-term.
clinical syndrome.
Answer C is incorrect. On x-ray of the chest,
Answer D is incorrect. Massive blood transfu- diffuse opacication of one or both lungs and
sion is a less common cause of ARDS. air bronchograms would be seen in a case of
Answer E is incorrect. Near-drowning is a less pulmonary hemorrhage.
common cause of ARDS. Answer D is incorrect. On x-ray of the chest,
pulmonary interstitial emphysema (PIE) ap-
23. The correct answer is E. X-ray of the chest pears as linear lucencies radiating from the hi-
demonstrated uid in the ssures, prominent lum or as large cyst-like blebs. This neonates
pulmonary vascular markings, and overexpan- time course is consistent with PIE, in which
sion (as evidenced by attening of the dia- infants develop respiratory distress within 48
phragm). The history and imaging ndings are hours of birth. However, neonates with PIE are
consistent with transient tachypnea of the new- typically more ill and have hypotension, bra-
HIGH-YIELD SYSTEMS
Chapter 14: Pulmonary Answers 431

dycardia, and hypoxia. PIE usually affects ex- Answer A is incorrect. The initial treatment for
tremely preterm infants with NRDS or sepsis. patients with pneumothoraces occupying >15%
of the hemithorax is aspiration of the pleural
24. The correct answer is E. This patient most space. Usually an 8-Fr catheter is threaded over
likely suffers from asbestosis, a condition typi- an 18-gauge needle. Once in proper position,
cally found in shipyard and textile workers. the needle is removed and the air in the pleu-
X-ray of the chest often demonstrates small bi- ral space is manually evacuated. However, if the
lateral parenchymal opacities, and 50% of pa- patient is unstable, then aspiration should be
tients with asbestosis will have multiple pleural bypassed in favor of tube thoracostomy.
plaques.
Answer C is incorrect. Open thoracotomy has
Answer A is incorrect. Bilateral diffuse inl- been largely replaced by video-assisted thora-
trates are seen in several conditions, including coscopy because hospitalization time is shorter
atypical pneumonia. A patient might present and postoperative pain is less. Currently, tho-
with nonproductive cough, dyspnea, and fever. racotomy is indicated after thoracoscopy has
failed or if thoracoscopy is unavailable. If tho-

Pulmonary
Answer B is incorrect. Bilateral hilar adenopa-
racotomy is undertaken, the pleural blebs are
thy is characteristic of sarcoidosis. Sarcoidosis
oversewn and the pleura scarred.
occurs most often in African-American women
and causes non-caseating granulomas in vari- Answer D is incorrect. An alternative to tube
ous organ systems. thoracostomy is video-assisted thoracoscopy.
With this procedure, the bullae are treated
Answer C is incorrect. Consolidation of lung
with wedge resection via an endoscopic sta-
tissue suggests a bacterial pneumonia. Look for
pler. The indications for thoracoscopy include
a patient with an acute onset of fever, produc-
failure of aspiration treatment, failure of lung
tive cough, and decreased breath sounds over
re-expansion after 3 days of tube thoracostomy,
the consolidation.
persistence of bronchopleural stula after 3
Answer D is incorrect. A focal mass would sug- days, and recurrent pneumothorax after pleu-
gest primary lung carcinoma, but this patient rodesis. Video-assisted thoracostomy is a safe
has no smoking history and no constitutional and effective management option for the treat-
symptoms, making this a much less likely diag- ment and prevention of pneumothorax.
nosis. Patients with lung carcinoma might pre-
Answer E is incorrect. A patient with a pri-
sent with chronic cough, dyspnea, weight loss,
mary spontaneous pneumothorax that fails
hemoptysis, and paraneoplastic syndromes.
aspiration should be treated with tube thora-
25. The correct answer is B. Primary spontaneous costomy. For a patient being managed with a
pneumothorax is most commonly due to a rup- chest tube, the tube can be clamped when no
ture of subpleural apical blebs, which usually air has been seen emanating from the thoracic
occurs in tall, thin young men. The history and cavity for 12 hours. The chest tube may be re-
physical examination presented are classic for moved after there has been no evidence (ra-
pneumothorax: sudden-onset unilateral pleuritic diographic or clinical) of pneumothorax for 24
chest pain and dyspnea. X-ray of the chest is the hours. Instillation of a pleurodesis agent such
diagnostic modality of choice because it shows a as doxycycline decreases the recurrence rate of
visceral pleural line and/or lung retraction from pneumothorax.
the chest wall when pneumothorax is present.
26. The correct answer is C. This mans age,
Observation with supplemental oxygen is the
symptoms, and history suggest a diagnosis of
best approach for pneumothoraces occupying
COPD. The diagnosis is supported by a de-
less than 15% of the hemithorax because these
creased FEV1:FVC ratio of 0.7 and a FEV1
often resolve spontaneously. The rate of reab-
sorption is 1.25% of the volume of the hemitho- value that is 60% of expected. Although the ob-
rax per 24 hours. The rate is increased with the structed airways are not completely reversible,
administration of supplemental oxygen.
HIGH-YIELD SYSTEMS
432 Section I: Organ Systems Answers

which differentiates it from asthma, it is com- Answer B is incorrect. This is appropriate for a
mon to see up to a 15% improvement in FEV1 severe asthma exacerbation requiring hospital-
after bronchodilator therapy, even in patients ization, not as a control treatment for moderate
with COPD. persistent asthma. Signs of respiratory distress
indicating that hospitalization is needed include
Answer A is incorrect. Asthma is largely due to
dyspnea at rest, retractions, paradoxical breath-
hyperreactive airways, which become inamed
ing, and use of accessory muscles of respiration.
and constrict in response to a variety of triggers
(cold, infection, exercise, or allergens). The Answer C is incorrect. The patients symptoms
airway constriction, however, is largely revers- have resolved with treatment in the emergency
ible with bronchodilator treatment. department and she does not require hospital-
ization at this point.
Answer B is incorrect. Chronic aspiration typi-
cally occurs in one of two settings. First, in the Answer D is incorrect. This is appropriate for
setting of neurologic impairment due to demen- mild persistent asthma (>2 episodes per week
tia or following a stroke, aspiration may be sec- but <1 episode per day).
Pulmonary

ondary to a loss of cough and swallow reexes.


Answer E is incorrect. Cromolyn sodium is
Second, a pharyngeal or esophageal disorder,
more useful in the pediatric population or pa-
such as laryngopharyngeal or gastroesophageal
tients with mild persistent asthma. Cromolyn
reux, cricopharyngeal spasm, strictures, Zenker
is an inhaled nonsteroidal anti-inammatory
diverticulum, achalasia, or changes postradia-
drug used for asthma prophylaxis. The drug
tion or postsurgery for neoplastic processes, may
stabilizes mast cells, leading to decreased pul-
also result in aspiration. There is no history that
monary inammation.
any of these risk factors is present.
Answer D is incorrect. Chronic histoplasmosis 28. The correct answer is C. The patient is suffer-
would be associated with a restrictive lung dis- ing from aortic stenosis, which is causing pul-
ease pattern, which would result in normal or monary edema. The classic triad of symptoms
increased FEV1:FVC ratio. Also, the subway of aortic stenosis include congestive heart fail-
worker does not directly work with soil, where ure, syncope, and angina. Aortic stenosis
the histoplasmosis fungus resides. In addition, causes cardiogenic pulmonary edema by mak-
histoplasmosis is most often found in the Mid- ing it harder for the left ventricle to move
western and southeastern states and along the blood forward, resulting in increased lling
Ohio and Mississippi river valleys. pressures in the pulmonary capillaries. Patients
with pulmonary edema complain of dyspnea,
Answer E is incorrect. No history of prior posi-
orthopnea, and paroxysmal nocturnal dyspnea,
tive PPD test, immunocompromised status,
and rales will classically be heard with auscul-
travel history, or sick contacts are present that
tation. X-ray of the chest often shows an en-
would suggest a diagnosis of TB. Also, no his-
larged heart and prominent pulmonary vascu-
tory of chills or sweats is given and the spirom-
lature. The primary pathogenic mechanism in
etry results indicate that a minimally reversible
cardiogenic pulmonary edema is an increase in
obstructive lung process is at work.
the capillary uid hydraulic pressure due to
left ventricular systolic or diastolic dysfunction.
27. The correct answer is A. This patient now has
moderate persistent asthma, with daily symp- Answer A is incorrect. Decreased capillary
toms and more than one night per week of uid oncotic pressure, which occurs in states
symptoms, along with worsening end-expiratory of hypoalbuminemia, could precipitate pulmo-
ow rate. The recommended treatment for nary edema.
moderate persistent asthma is low- to medium-
Answer B is incorrect. Decreased capillary
dose inhaled steroids along with a long-acting
interstitial pressure causes pulmonary edema.
bronchodilator, either a -adrenergic agonist or
This condition arises from the rapid removal of
theophylline.
pleural effusion.
HIGH-YIELD SYSTEMS
Chapter 14: Pulmonary Answers 433

Answer D is incorrect. Increased capillary per- acute coronary syndrome. At this time, how-
meability can cause pulmonary edema. Sepsis, ever, the patient lacks symptoms that would
radiation, oxygen toxicity, and the acute respi- suggest CAD, namely substernal chest pain
ratory distress syndrome are possible causes of provoked by exertion and relieved by rest or ni-
increased capillary permeability. troglycerin. Furthermore, an exercise tolerance
test would not be expected to help conrm the
Answer E is incorrect. Pulmonary edema can
diagnosis of OSA.
be caused by an increase in interstitial oncotic
pressure, which would drive uid from the Answer E is incorrect. An x-ray of the chest may
capillaries into the interstitium. The interstitial show an enlarged heart, but it would not be ex-
oncotic pressure is derived from ltered plasma pected to help conrm the diagnosis of OSA.
proteins and proteoglycans in the interstitium.
AS, however, would not cause the interstitial 30. The correct answer is D. Key features include
oncotic pressure to change. the central location, history of smoking (small
cell carcinomas are closely associated with a
29. The correct answer is D. The patients history history of smoking), and the presence of a para-

Pulmonary
and physical examination are highly suspicious neoplastic syndrome, in this case, Cushings
for sleep apnea. Obstructive sleep apnea (OSA) syndrome from tumor elaboration of ACTH
is most prevalent in men 3060 years old who outside hypothalamic-pituitary-adrenal axis
present with a history of snoring, excessive day- regulation. SIADH and Eaton-Lambert syn-
time sleepiness, nocturnal choking or gasping, drome are other paraneoplastic syndromes as-
witnessed apneic episodes, moderate obesity, sociated with small cell lung carcinoma. Me-
and hypertension. The patients dyspnea on ex- tastases are commonly present at the time of
ertion, diffuse and laterally displaced point of diagnosis. Favored sites are brain, liver, and
maximal intensity, and S3 gallop are symptoms bone.
and signs of left heart failure, a manifestation
Answer A is incorrect. Adenocarcinoma typi-
of OSA. It is believed that left heart failure
cally presents peripherally and is not associated
arises from repetitive episodes of nocturnal as-
with Cushings syndrome.
phyxia and concomitant negative intrathoracic
pressure. The negative intrathoracic pressure Answer B is incorrect. In this patient there is
increases cardiac afterload, which manifests a single lesion in the lung and multiple lesions
over time as left heart failure. The denitive di- in the liver. The site of multiple solid masses
agnostic modality for OSA is a sleep study is more likely the destination of the metastasis,
(polysomnography). Polysomnography docu- while the solitary nodule in the lung is likely
ments arousals, obstructions, episodes of hy- to be the primary.
poxemia, and the various stages of sleep. A
Answer C is incorrect. Large cell carcinoma
sleep study would be useful in this case be-
typically present on the periphery, are not as-
cause it would conrm the diagnosis and estab-
sociated with Cushings syndrome, and are the
lish the need for therapeutic intervention with
least frequent type of lung cancer.
continuous positive airway pressure.
Answer E is incorrect. Although squamous
Answer A is incorrect. Cardiac catheterization
cell lung carcinoma presents centrally and is
would not be useful in the diagnosis of OSA.
clearly linked to smoking, a paraneoplastic
Answer B is incorrect. Echocardiography may syndrome marked by excess ACTH would not
be helpful in detecting wall motion abnormali- be expected. Excess parathyroid hormone and
ties and roughly gauging cardiac function, but it resultant hypercalcemia, however, is associated
would not help conrm the diagnosis of OSA. with squamous cell cancers.
Answer C is incorrect. Exercise tolerance test-
31. The correct answer is B. The patients presen-
ing is used to diagnose coronary artery disease
tation is classic for aspiration pneumonia. Spe-
(CAD), evaluate patients with known CAD,
cically, the history of alcoholism and black-
localize ischemia, and risk-stratify patients after
HIGH-YIELD SYSTEMS
434 Section I: Organ Systems Answers

outs makes this patient much more prone to nas or Burkholderia cepacia, infect the bron-
aspiration pneumonia. The x-ray ndings of a chial tree, provoking an intense inammatory
necrotizing process in the superior segment of response. Infection stimulates mucus produc-
the right lower lobe is also consistent with aspi- tion, and damage to the wall of the airway with
ration pneumonia. Clindamycin is the recom- resultant bronchiectasis ensues.
mended rst-line therapy for the anaerobic in-
Answer A is incorrect. Cirrhosis and hepatic
fection seen in aspiration pneumonia.
failure are possible complications of hepatitis.
Answer A is incorrect. Azithromycin is a mac-
Answer B is incorrect. The salt-wasting forms
rolide commonly used to treat gram-positive
of congenital adrenal hyperplasia, a disorder
infections seen in community-acquired pneu-
of defective steroidogenic pathways, are associ-
monia. However, it is not an effective agent for
ated with these complications.
treating atypical infections or anaerobic infec-
tions commonly seen in aspiration pneumo- Answer C is incorrect. Esophageal ulceration
nia. or strictures and upper gastrointestinal bleed-
ing are possible complications of gastroesopha-
Pulmonary

Answer C is incorrect. Isoniazid is one of the


geal reux disease.
rst-line agents used for the prevention and
treatment of TB. Isoniazid is never used alone Answer D is incorrect. This answer describes
as a rst-line agent. possible sequelae of lead poisoning.
Answer D is incorrect. Moxioxacin is a uo-
roquinolone commonly used to treat pneu- Questions 33 and 34
monia, specically community-acquired
pneumonia. However, moxioxacin is used 33. The correct answer is F. This patients fungal
predominantly to treat gram-positive infections. culture ndings are typical for Histoplasma
Although it has good in vitro activity against capsulatum. Pulmonary histoplasmosis should
anaerobes, it is much less effective in the treat- always be included in the differential diagnosis
ment of in vivo anaerobic infections. if sarcoidosis, TB, or malignancy is suspected.
Cases are most common in the Ohio and Mis-
Answer E is incorrect. Piperacillin-tazobactam sissippi River valleys after exposure to bird or
is an extended-spectrum -lactam antibiotic bat droppings. In immunocompetent patients,
with adequate gram-positive, gram-negative, >95% will have asymptomatic disease, but se-
and anaerobic coverage. It is effective in the vere illness can occur. The vast majority of
treatment of pseudomonal infections. How- cases are self-limiting and do not require treat-
ever, it is not rst-line therapy for aspiration ment, but itraconazole or amphotericin B can
pneumonia. be given in severe cases.
Answer F is incorrect. Trimethoprim-sulfame-
thoxazole is used to treat many types of infec- 34. The correct answer is G. This patients symp-
tions and is also commonly used to treat Pneu- toms and x-ray ndings are concerning for reac-
mocystis jirovecii pneumonia. However, it is tivation of TB. Denitive diagnosis can be made
not used for the treatment of aspiration pneu- by identifying Mycobacterium tuberculosis on
monia. sputum smear or culture. Skin testing is often
done when TB is expected, but is neither sensi-
32. The correct answer is E. This patient has cys- tive nor specic for identifying active TB. Ther-
tic brosis (CF), which may be diagnosed in apy for TB requires appropriate doses of multi-
childhood with failure to thrive and failure to ple drugs to which the organism is susceptible,
gain weight despite a good appetite. Children taken regularly for a sufcient amount of time.
often have bulky, foul-smelling stools as a re- Antitubercular drugs include isoniazid, ri-
sult of pancreatic insufciency. CF disease in fampin, streptomycin or amikacin, ethambutol,
the lung accounts for most of the morbidity and pyrazinamide as well as some quinolones
and mortality from this disease. Bacteria in the (i.e., levooxacin or moxioxacin). Treatment
lungs of patients with CF, typically Pseudomo- regimens typically have an initial phase of ther-
HIGH-YIELD SYSTEMS
Chapter 14: Pulmonary Answers 435

apy followed by a continuation phase. For ex- enous spread to extrapulmonary sites is com-
ample, the RIPE regimen consists of 2 months mon. Blastomyces can be identied by fungal
of rifampin, isoniazid, pyrazinamide, and culture growth at 37C revealing broad-based
ethambutol followed by 7 additional months budding yeast. Unlike histoplasmosis, blastomy-
with rifampin and isoniazid. cosis should be treated with amphotericin B or
an azole (ketoconazole or uconazole) if there
Answer A is incorrect. Actinomyces israelii
is extrapulmonary disease. The one exception
is an anaerobic gram-positive bacterium of
is mild disease in an immunocompetent host,
the family Actinomycetaceae and infection
which can be followed by observation.
typically manifests clinically as draining sinus
tracts. Involvement of the lungs is rare. Ac- Answer D is incorrect. Pneumonia due to
tinomyces are constituents of normal oral ora Candida is rare unless it is part of systemic ill-
and humans are the only reservoir. The typical ness in immunocompromised hosts. It is dif-
patient is a male (male:female ratio 2:1) with cult to identify candidal pneumonia since
poor oral hygiene. Actinomyces is best grown positive cultures cannot distinguish between
on brain-heart infusion broth at 37C with oropharyngeal contamination (normal coloni-

Pulmonary
6%10% carbon dioxide. The organism is char- zation) or a pathologic state. Fungal culture
acterized by sulfur granules in pus (not actu- reveals yeast and pseudohyphae. Unless there
ally sulfur, but they appear yellow). Treatment is involvement of other organs, treatment is not
is with penicillins or tetracyclines. indicated. When other organs are involved,
treatment with amphotericin B and uco-
Answer B is incorrect. Aspergillus can manifest
nazole is initiated.
as pulmonary disease as either allergic bron-
chopulmonary aspergillosis (ABPA) or as asper- Answer E is incorrect. Most people have
gilloma. ABPA most often occurs in patients been exposed to Cryptococcus and are asymp-
with CF or asthma who present with bronchial tomatic. However, some immunocompetent
obstruction, fever, malaise, and expectoration people can still develop symptomatic disease,
of brownish mucous plugs. X-ray of the chest which characterized by cough, chest pain, fe-
would show upper lobe inltrates and atelecta- ver, weight loss, or hemoptysis. Diagnosis is
sis from mucoid impaction. ABPA is diagnosed made by culturing the organism from sputum
by skin test reactivity as well as serum IgG and (characteristic encapsulated yeast forms). The
IgE to Aspergillus. Aspergillomas usually arise most common nding on x-ray of the chest is
in previous lung cavities due to TB, sarcoi- noncalcied pulmonary nodules; cavitation
dosis, or other fungal infections. Pulmonary is rare. Patients with symptoms can be treated
infection can be asymptomatic or manifest as with uconazole or amphotericin B.
hemoptysis. Diagnosis is usually by x-ray of
Answer H is incorrect. Nocardia is an uncom-
the chest showing a solid mass surrounded by
mon aerobic gram-positive bacterium of the
a radiolucent crescent. Culture from sputum
family Nocardiaceae and typically is respon-
(without radiologic ndings) cannot be used in
sible for opportunistic infections, but infection
diagnosis due to colonization of Aspergillus in
can occur in the immunocompetent host. No-
patients with any type of lung disease. Medical
cardia species are endemic to soil and aquatic
treatment is rarely helpful. Patients are treated
environments and are acquired by inhalation.
by surgical resection of the aspergilloma or
Nocardia infection most commonly occurs in
lobectomy.
patients who use steroids (e.g., asthma, COPD,
Answer C is incorrect. Blastomyces is endemic and transplant recipients) or patients with solid
to the Ohio and Mississippi River basins (simi- or hematologic malignancies; however, it can
lar to histoplasmosis) as well as the Midwest rarely occur in patients with AIDS who are
and along the Great Lakes, and occurs most receiving Pneumocystis jiroveci pneumonia
often in those with signicant exposure to soil. prophylaxis with trimethoprim-sulfamethox-
Blastomycosis can be asymptomatic or mani- azole. Isolation of Nocardia from sputum in-
fest as acute or chronic pneumonia. Hematog- dicates infection. The microorganism appears
HIGH-YIELD SYSTEMS
436 Section I: Organ Systems Answers

as lamentous gram-positive branching rods 36. The correct answer is I. Rheumatic heart dis-
on Gram stain and are differentiated from Ac- ease can cause a variety of valvular disorders:
tinomyces by acid-fast staining. Specimens for aortic stenosis, aortic insufciency, mitral re-
culture should be grown on buffered charcoal gurgitation, and mitral stenosis. Pulmonary
yeast extract (the same as that used for Legion- edema and dyspnea are clinical manifestations
ella) or Thayer-Martin agar. MRI of the brain of mitral stenosis. The edema and dyspnea can
should be performed due to the propensity of be precipitated by tachycardia (as during exer-
Nocardia for central nervous system involve- cise), volume overload, and atrial brillation.
ment. Treatment has not been studied in for- Recurrent bronchitis (seen in this patient) is a
mal trials, but sulfonamides are typically given. frequent pulmonary symptom of mitral stenosis
due to congestion. Classically, physical exami-
Answer I is incorrect. Sporothrix is not highly
nation of mitral stenosis reveals a low-pitched
virulent and disease is limited to cutaneous
diastolic rumble audible at the apex and an
and lymphatic structures. Expect Sporothrix in-
opening snap.
fection in a rose gardener who has pricked his
or her ngers with a thorn. Involvement of the
Pulmonary

37. The correct answer is C. The patient is suffer-


respiratory system is rare but can occur with
ing from ARDS, which is dened by noncar-
inhalation of conidia in patients with COPD,
diogenic pulmonary edema. There are many
AIDS, or alcoholism. The typical patient with
direct and indirect etiologies of ARDS. Direct
pulmonary disease is a middle-aged male alco-
causes include aspiration, pneumonia, inhala-
holic who presents with symptoms and an x-ray
tion injury, and pulmonary contusion. Indirect
of the chest that is concerning for TB. Growth
causes include sepsis, shock, disseminated in-
of Sporothrix on Sabouraud agar at 2527C
travascular coagulation, and pancreatitis. Four
reveals thin, septate hyphae with branching
criteria dene ARDS without biopsy, all of
and stranding; conidia arrange themselves
which are present in this patient. The four cri-
around the hyphae in a bouquet-like fashion.
teria include acute onset, bilateral patchy air-
Pulmonary infection is deadly if untreated and
space disease, pulmonary capillary wedge pres-
should be treated with amphotericin B or itra-
sure <18 mm Hg, and an PaO2:FiO2 ratio of
conazole.
200 or less.
Answer A is incorrect. Acute aortic insuf-
Questions 35, 36, and 37 ciency (regurgitation) is a serious disease with
35. The correct answer is J. The patient shows high mortality. It is characterized by acute car-
symptoms classic for a subarachnoid hemor- diovascular collapse and may present as severe
rhage. This is typically caused by a ruptured dyspnea, hypotension, or angina. Causes in-
aneurysm, arteriovenous malformation, or clude trauma, aortic dissection, and infections
trauma to the circle of Willis. Berry aneurysms (rheumatic fever, endocarditis).
are associated with polycystic kidney disease Answer B is incorrect. Acute mitral regurgita-
and coarctation of the aorta. Patients with sub- tion is a life-threatening complication of pap-
arachnoid hemorrhage experience a sudden illary muscle rupture following myocardial
onset, intensely painful headache; neck stiff- infarction. It leads to massive pulmonary con-
ness; tactile fever; nausea and vomiting; and a gestion and, frequently, death.
uctuating level of consciousness. Subarach-
noid hemorrhage can cause pulmonary edema Answer D is incorrect. Aspiration pneumonia
and its effects are believed to be due to massive occurs when a patient inhales material from the
sympathetic discharge. Other causes of neuro- oropharynx that is colonized by upper airway
genic pulmonary edema include epileptic sei- ora. It is common in people with impaired
zure and head injury. Subdural and epidural consciousness or impaired ability to swallow
hematomas are not known to cause pulmonary (i.e., intubated patients, coma). In most cases
edema. the organisms isolated include Staphylococ-
cus aureus, Streptococcus pneumoniae, Enter-
obacteriaceae, and Haemophilus inuenzae.
HIGH-YIELD SYSTEMS
Chapter 14: Pulmonary Answers 437

In alcoholics, Klebsiella is classic. Intubated Questions 38, 39, and 40


patients are at risk of Pseudomonas aeruginosa
38. The correct answer is G. Hamartomas are the
infection.
most common cause of benign tumors of the
Answer E is incorrect. Cardiac arrhythmias lung. The lesions are composed of cartilage
may be life threatening if they lead to ventricu- surrounded by connective tissue and fat. In the
lar brillation. The patients described here do United States, hamartomas represent 5%8%
not have stories suggestive of arrhythmia. of all solitary pulmonary nodules and 0.25% of
all autopsy cases. Most pulmonary hamartomas
Answer F is incorrect. Epidural hematoma is
are asymptomatic and rst noticed as an inci-
usually the result of blunt trauma to the cra-
dental nding on x-ray of the chest. The typi-
nium and frequently involves laceration of
cal radiographic appearance is a solitary pul-
branches of the middle meningeal artery. It is
monary nodule with a popcorn calcication.
a high-pressure bleed that produces a biconvex
The rib pain in this case is due to bruising and
(lens-shaped) hyperdensity on CT of the head.
not fractures or the hamartoma.
Epidural hematomas have the unique property

Pulmonary
of being limited by suture lines (because the
39. The correct answer is H. Histoplasmosis is en-
dura is attached rmly to the skull at the su-
demic in the central United States, specically
ture lines).
around the Mississippi River valley. Risk factors
Answer G is incorrect. Epilepsy is character- include AIDS, spelunking, and exposure to bat
ized by a pattern of recurrent seizures. The sei- or bird excrement. Chronic infection is associ-
zures may be either partial (focal) or general- ated with low-grade fever, anorexia, weight loss,
ized. Treatment is often lifelong and aimed at night sweats, and productive cough. X-ray of the
improving quality of life. chest may reveal hilar or mediastinal adenopa-
thy, in addition to a solitary pulmonary nodule
Answer H is incorrect. Patients with myocar-
or a focal inltrate. Complement xation anti-
dial infarction classically present with subster-
body titers of 1:16 or 1:32 are presumptive for a
nal chest pain that radiates to the left arm or
diagnosis of histoplasmosis. The presence of or-
shoulder. Patients are often diaphoretic and
ganisms by silver stain on biopsy (bone marrow,
tend to describe their discomfort as intense
lymph node, or liver) or bronchoalveolar lavage
pressure: It feels like there is an elephant sit-
is diagnostic. TB and fungi (both histoplasmosis
ting on my chest.
and coccidioidomycosis) are the most frequent
Answer K is incorrect. Subdural hemorrhage causes of infectious granulomas that present as
is commonly due to tearing of the bridging solitary pulmonary nodules. TB is less likely
veins that extend from the cortex into the ve- given the negative smear and culture (which
nous sinuses. It is typically a low-pressure bleed can take weeks to complete). Coccidioidomyco-
and appears on noncontrast-enhanced CT of sis should be suspected, rather than histoplas-
the head as a hyperdense crescentic mass along mosis, if the patient is in the southwestern
the cerebral convexity. United States.
Answer L is incorrect. Inammation of the
40. The correct answer is A. Patients with lung can-
leptomeninges (pia and arachnoid mater) is
cer typically present with a history of cough, he-
one of the most common manifestations of
moptysis, weight loss, and chest pain. Adenocar-
viral infections of the central nervous system.
cinoma is the most common non-small cell type
The term meningitis implies lack of cerebral
of lung cancer, representing 35%40% of all
(encephalitis) and spinal cord (myelitis) in-
lung cancers. Adenocarcinoma is most often lo-
volvement, but some pathogens may cause a
cated peripherally and is the subtype most com-
combination of signs and symptoms consistent
monly found in nonsmoking patients. Further-
with meningoencephalitis or encephalomyeli-
more, digital clubbing can occur with any type
tis. The term aseptic is often used to indicate
of lung cancer but is most frequently associated
a viral etiology.
with adenocarcinoma. Although all subtypes of
HIGH-YIELD SYSTEMS
438 Section I: Organ Systems Answers

lung cancer can present as a solitary pulmo- that present as solitary pulmonary nodules. TB
nary nodule, this presentation is most fre- is less likely given the negative smear and cul-
quently associated with adenocarcinoma and ture.
large cell carcinoma.
Answer J is incorrect. A pulmonary lipoma
Answer B is incorrect. Aspergilloma may occurs in the pulmonary parenchyma. Lipo-
manifest as an asymptomatic radiographic ab- mas are asymptomatic and are usually rst
normality in a patient with preexisting cavitary observed on x-ray or CT of the chest. Unlike
lung disease due to sarcoidosis, TB, or other hamartomas, lipomas have a uniform fatty den-
necrotizing pulmonary processes. It is caused sity with soft tissue strands due to their brous
by the fungus Aspergillus and is far more prev- stroma.
alent in immunocompromised hosts. It is un-
Answer K is incorrect. Neurilemmoma is a
likely here due to the lack of history of preex-
neurogenic tumor that arises from the cells of
isting cavitary disease.
the nerve sheath and the paraganglionic and
Answer C is incorrect. Bronchogenic cysts are autonomic ganglia. They comprise approxi-
Pulmonary

rare, congenital lesions that are often identi- mately 21% of all adult mediastinal tumors and
ed secondary to infection. More than half are believed to be Schwann cells derived from
of all patients with bronchogenic cysts are as- the neural crest. These masses usually arise
ymptomatic. The most common symptoms are from the side of a nerve, are well encapsulated,
chest pain and dysphagia due to compression and have a unique histologic pattern. The be-
of mediastinal structures. This diagnosis is un- nign lesion essentially manifests with cosmetic
likely due to its extremely low incidence. deformity, a palpable mass, and/or symptoms
similar to a compressive neuropathy. Neuro-
Answer D is incorrect. One percent to 6% of
logic symptoms tend to present late. Symptoms
all lung tumors are carcinoid tumors. Although
can be vague, with an average interval of up to
these neoplasms are capable of producing a
5 years before the diagnosis is established.
variety of substances, including biologically
active peptides and hormones, most are inac- Answer L is incorrect. The clinical presenta-
tive. Carcinoids developing within large airway tion of pulmonary arteriovenous malformation
structures grow slowly; can become quite large; may vary from no symptoms to severe illness.
and can cause persistent atelectasis, recurrent Symptoms generally develop between the
pneumonia, pulmonary abscess, and bron- fourth and fth decades of life. The most com-
chiectasis. Although peripheral carcinoid lung mon complaints are epistaxis, dyspnea, platyp-
tumors are in the differential for a solitary pul- nea, and hemoptysis. Pulmonary arteriovenous
monary nodule, they are relatively rare. malformation may be suspected when a soli-
tary pulmonary nodule is found incidentally
Answer E is incorrect. Cavitating abscesses on
on x-ray of the chest; however, its extremely
x-ray of the chest may appear as an area of thick
low incidence (approximately 1:40,000) makes
pneumonic consolidation, depending on the
it unlikely.
stage of the abscess and preceding the emer-
gence of the typical cavitary air-uid form. Answer M is incorrect. Extra-articular mani-
festations of rheumatoid arthritis are more
Answer F is incorrect. Coccidioidomycosis
likely to be present in patients with severe dis-
would be suspected, rather than histoplasmo-
ease. The absence of rheumatoid arthritis in
sis, in a patient with a recent travel history to
these three cases makes a rheumatoid nodule
the southwestern United States, Mexico, or
an unlikely diagnosis. In fact, pleural effusion
Central and South America. It is caused by
is the most common manifestation of rheuma-
breathing in spores of a fungus found in desert
toid arthritis in the chest.
regions.
Answer N is incorrect. A solitary lung mass is
Answer I is incorrect. TB and fungi (both his-
a rare thoracic presentation of sarcoidosis, mak-
toplasmosis and coccidioidomycosis) are the
ing this diagnosis unlikely. Also, it is usually as-
most frequent causes of infectious granulomas
ymptomatic, although it can be accompanied
HIGH-YIELD SYSTEMS
Chapter 14: Pulmonary Answers 439

by a dry cough that persists, excessive tearing, cemia due to parathyroid-like hormone pro-
or skin rash. A diagnosis of pulmonary sarcoi- duction.
dosis can be conrmed only after other diag-
Answer P is incorrect. Thymoma is the most
noses have been ruled out and when there is
common neoplasm of the anterior mediasti-
histologic evidence of noncaseating granuloma
num and accounts for 50% of anterior medi-
from a thoracic biopsy. The cause of sarcoido-
astinal masses. Peak incidence is in the fourth
sis is unknown, and it can appear and disap-
to fth decades of life. Presenting symptoms
pear suddenly.
include cough, chest pain, superior vena cava
Answer O is incorrect. Squamous cell carci- syndrome, dysphagia, and hoarseness, and do
noma accounts for 25%30% of all lung can- not typically include night sweats, phlegm pro-
cers. The classic manifestation is a cavitary le- duction, or weight loss. On x-ray of the chest
sion in a proximal bronchus. It has a distinct the lesion typically appears as a smooth mass
dose-response relationship to tobacco smoking. in the upper half of the chest, overlying the su-
It is more likely to be associated with hypercal- perior portion of the cardiac shadow near the
junction of the heart and great vessels.

Pulmonary
This page intentionally left blank
CHAPTER 15

Renal/Genitourinary

441
HIGH-YIELD SYSTEMS
442 Section I: Organ Systems Questions

Q U E ST I O N S

1. A 65-year-old man with chronic obstructive (E) Hyperglycemia


pulmonary disease who requires home oxygen (F) Streptococcal infection
at night and has cor pulmonale presents to the
emergency department with worsening short- 3. A 75-year-old man with chronic obstructive
ness of breath. His respiratory rate is 22/min pulmonary disease and coronary artery disease
and heart rate is 104/min. He has distant presents to the emergency department com-
breath sounds and is using accessory muscles plaining of blood in his urine for the past sev-
of breathing. In addition to his baseline eral days. He denies any difculty passing his
chronic respiratory acidosis, the man is found urine or any frequency, urgency, or ank pain.
to have a metabolic acidosis on arterial blood He smokes about 1 pack of cigarettes a day and
Renal/Genitourinary

gas analysis. Laboratory tests show: has done so for the past 50 years. His tempera-
ture is 36.6C (97.9F), heart rate is 60/min,
Na+: 138 mEq/L
and blood pressure is 128/85 mm Hg. Digital
K+: 3.6 mEq/L
rectal examination reveals a normal prostate
Cl: 118 mEq/L
with no nodules. Urinalysis shows <10 RBCs/
HCO3: 16 mEq/L
hpf and 4 WBCs/hpf. There are no casts or
Phosphate: 2.0 mg/dL
crystals. His hemoglobin is 12 g/dL and WBC
Glucose: 98 mg/dL
count is 9800/mm. Which of the following is
Blood urea nitrogen (BUN): 10 mg/dL
the most likely diagnosis?
Creatinine: 0.8 mg/dL
(A) Acute bacterial prostatitis
Urinalysis is positive for glucose. Which of the
(B) Adenocarcinoma of the bladder
following diagnoses best explains this patients
(C) Adenocarcinoma of the prostate
laboratory ndings?
(D) Benign prostatic hyperplasia
(A) Right heart failure (E) Interstitial cystitis
(B) Steroid-induced glucosuria (F) Transitional cell carcinoma of the bladder
(C) Type I renal tubular acidosis
(D) Type II renal tubular acidosis 4. An 83-year-old woman who lives alone is ad-
(E) Type IV renal tubular acidosis mitted because of confusion. She is accompa-
nied to the hospital by her neighbor, who states
2. A 37-year-old man presents to the emergency that as recently as 3 months ago the patient was
department complaining of swelling of his legs, able to perform basic and independent activi-
hands, and face for 4 days. On examination he ties of daily living, and enjoyed gardening in
is afebrile with generalized edema of his upper her yard. However, after her husbands death 6
and lower extremities as well as his face. His weeks ago the patient has not been seen out-
examination is otherwise notable only for scars side her home. When the neighbor went to
in a linear pattern on his middle to lower arms check on the patient today, she found her to be
bilaterally. Basic metabolic panel and com- awake and alert but confused. Her house was
plete blood cell count are normal, but liver uncharacteristically unkempt, and a few empty
function tests show a total protein level of 5.4 TV dinners were strewn around the living
mg/dL and albumin of 2.8 mg/dL. Urinalysis room. Her blood pressure is 106/66 mm Hg,
shows 3+ protein without signicant WBCs or heart rate is 68/min, and temperature is 37.3C
RBCs. Which of the following is the most (98.1F). The remainder of her examination is
likely etiology of this patients disease? unremarkable, except for marked temporal
wasting. Laboratory tests show:
(A) Antibody deposition
(B) Cardiomyopathy Na+: 137 mEq/L
(C) Cocaine abuse K+: 3.5 mEq/L
(D) Heroin abuse Ca2+: 6.9 mg/dL
HIGH-YIELD SYSTEMS
Chapter 15: Renal/Genitourinary Questions 443

Mg2+: 1.5 mEq/L Creatine kinase-myocardial band: 1 IU/L


BUN: 8 mg/dL Troponin-I: <0.3 IU/L
Creatinine: 0.5 mg/dL
Which of the following is the most appropriate
Inorganic phosphate: 3.1 mg/dL
next step in management?
Albumin: 1.5 g/dL
What is the best next step in managing this pa-
tients low calcium levels?
(A) This patient has low BUN levels; there-
fore, her hypocalcemia likely does not rep-
resent a decrease in active calcium. She
Reproduced, with permission, from Fuster V, ORourke
does not need calcium repletion at this RA, Walsh RA, Poole-Wilson P, eds. Hursts The Heart,
time 12th edition. New York: McGraw-Hill, 2008: Figure

Renal/Genitourinary
(B) This patient has low creatinine levels; 13-13.
therefore, her hypocalcemia is likely more
severe than measured. She needs aggres- (A) 2 L Normal saline at 225 mL/hr
sive calcium repletion (B) Hemodialysis
(C) This patient has low creatinine levels; (C) Intravenous pyelogram
therefore, her hypocalcemia likely does (D) Percutaneous angioplasty
not represent a decrease in active calcium. (E) Pericardiocentesis
She does not need calcium repletion at
this time 6. A 47-year-old man presents to his primary care
(D) This patient is hypoalbuminemic; there- physician complaining of urinary frequency,
fore, her hypocalcemia is likely more se- urgency, and pain with urination for the past 2
vere than measured. She needs aggressive days. On further questioning the patient re-
calcium repletion ports feeling feverish and having a vague back-
(E) This patient is hypoalbuminemic; there- ache. He has no past medical history but has a
fore, her hypocalcemia likely does not rep- brother who was diagnosed with prostate can-
resent a decrease in active calcium. She cer 2 years ago at age 60 years. His temperature
does not need calcium repletion at this is 38.5C (101.2l F) and blood pressure is
time 128/80 mm Hg. Digital rectal examination re-
veals an enlarged prostate that is tender with ir-
5. A 64-year-old man with a history of diabetes regular contour. Urinalysis reveals 510 WBCs/
and colon polyps is admitted for episodic chest hpf and 35 RBCs/hpf. A complete blood cell
pain. The pain started 2 days ago and is sharp, count is notable only for a WBC count of
nonradiating, and intermittent. It is worse with 15,000/mm. Prostate-specic antigen level
recumbency and with deep inspiration. His was tested at the patients request and was ele-
blood pressure is 118/68 mm Hg and drops to vated at 4.8 ng/dL. What additional test would
116/64 mm Hg with inspiration, pulse is 80/ best conrm the patients diagnosis?
min, and temperature is 37.3C (99.2F). On (A) Blood culture
physical examination the patient is in signi- (B) Serial serum prostate-specic antigen
cant distress. Further examination reveals the (C) Transrectal ultrasound
absence of jugular venous distension, presence (D) Transrectal ultrasound and transrectal
of a cardiac friction rub, and normal pulmo- prostate biopsy
nary examination. Extremities are warm and (E) Urine culture (with or without prostate
well perfused. An ECG is shown in the image. massage)
Laboratory tests show:
BUN: 73 mg/dL
Creatinine: 6.3 mg/dL
Glucose: 150 mg/dL
HIGH-YIELD SYSTEMS
444 Section I: Organ Systems Questions

7. A 35-year-old man with appendiceal carci- sedimentation rate and markedly elevated anti-
noma presents to the urgent care clinic 1 week double-stranded DNA titers. At her rheumatol-
after receiving chemotherapy with oxaliplatin, ogists ofce, it is discovered that she has traces
5-uorouracil, and leucovorin. He complains of blood in her urine and proteinuria. Renal
of severe fatigue progressively worsening over biopsy is scheduled, and she is started on a
the past 4 days, with decreased oral intake of 2-month course of prednisone. Over the course
food and liquids. Review of systems reveals of her therapy, which drug-related adverse ef-
back pain that he has had for several months. fect is this patient most likely to experience?
Physical examination shows a lethargic, ill-
(A) Hair loss
appearing man with normal vital signs. He has
(B) Hypertension
a draining enterocutaneous stula with an os-
(C) Hypoglycemia
tomy bag but no other abnormalities. Com-
(D) Leukopenia
plete blood cell count shows mild anemia and
Renal/Genitourinary

(E) Weight loss


leukocytosis. An electrolyte panel shows nor-
mal sodium and potassium levels with BUN of
10. A 3-year-old girl with a past history of recurrent
76 mg/dL and creatinine of 3.3 mg/dL. Results
cystitis is brought to the emergency department
of urinalysis are essentially normal. He has
because of 24 hours of decreased oral intake
been taking aspirin for the pain. Which of the
and an oral temperature of 38.8C (101.9F).
following is the most likely cause of renal fail-
Her blood pressure is 80/40 mm Hg, heart rate
ure in this patient?
is 120/min, and respiratory rate is 28/min. The
(A) Acute interstitial nephritis secondary to as- child appears well developed with marked cos-
pirin tovertebral angle tenderness on the left. She
(B) Direct renal toxicity of oxaliplatin does not have a sacral tuft of hair or sacral dim-
(C) Hypovolemia secondary to chemotherapy ple. Her urine culture grows >100,000 colony-
and inadequate oral intake of uids forming units of Escherichia coli. An ultrasound
(D) Nephrolithiasis is done that does not show any hydronephrosis,
(E) Obstruction of the ureter by the tumor but does show blunted calyces on the left kid-
ney. Following administration of intravenous an-
8. A 50-year-old man complains to his physician tibiotics and resolution of her acute illness, what
of difculty with erections. Over the past sev- test is the most appropriate?
eral months, his erections have been very pain-
(A) CT scan without contrast
ful. He denies any genital pain when his penis
(B) Cystoscopy
is not erect. His penis has also acquired a
(C) Intravenous pyelogram
prominent curve when it is erect. His blood
(D) MRI of the spine
pressure is 130/78 mm Hg and heart rate is 79/
(E) Voiding cystourethrogram
min. Genital examination reveals his penis is
atraumatic and has a palpable dense plaque on 11. A 43-year-old woman with familial cystinuria
the dorsal aspect of the shaft. There is no dis- seeks evaluation for intermittent ank pain.
charge from the meatus. What is the best next She has a history of kidney stones but has been
step in managing the condition that is causing episode free since beginning pharmacologic
the patients erectile dysfunction? management and aggressive hydration 2 years
(A) Ascorbic acid ago. She admits to getting lazy with her uid
(B) Immediate decompression of the corpora intake over the past several months but insists
cavernosa she is continuing her medication, although she
(C) Observation and emotional support cannot remember its name. Initial x-ray of the
(D) Penile prosthesis abdomen reveals a small opacity within the
(E) Surgical treatment proximal right ureter. Urinalysis demonstrates
a pH of 7.8, no trace of cystine crystals, and el-
9. A 23-year-old woman was recently diagnosed evated calcium excretion of 325 mg/24 hr.
with systemic lupus erythematosus. At the time Which of the following medications taken to
of diagnosis she had an elevated erythrocyte
HIGH-YIELD SYSTEMS
Chapter 15: Renal/Genitourinary Questions 445

prevent cystine stones most likely contributed


to this stone formation?
(A) Allopurinol
(B) Enalapril
(C) Furosemide
(D) Hydrochlorothiazide Reproduced, with permission, from Fuster V, ORourke
(E) Potassium bicarbonate RA, Walsh RA, Poole-Wilson P, eds. Hursts The Heart,
12th edition. New York: McGraw-Hill, 2008: Figure
12. A 60-year-old man with celiac sprue presents to 13-36.
the doctor for an annual check-up. He is in rel- (A) Administration of 20 mEq/hr of potassium
ative good health and has a family history of chloride
coronary artery disease; his brother had a myo- (B) Continuation of her home dose of al-
cardial infarction at age 55 years. He has a

Renal/Genitourinary
buterol
blood pressure of 128/78 mm Hg and heart (C) Epinephrine bolus
rate of 70/min. He mentions that his friend has (D) Phosphate repletion
just been diagnosed with prostate cancer and (E) Using 1.8% saline instead of 0.45% saline
wants to know what screening test would be ap- for uid replacement
propriate for him. What is the recommended
screening test for prostate cancer in this pa- 14. A 62-year-old police ofcer is brought to the
tient? emergency department after having a seizure
(A) Cystoscopy that began spontaneously while he was sitting at
(B) Digital rectal examination and prostate- his desk. He has no history of seizures or neuro-
specic antigen logic disorders. His temperature is 37.3 C
(C) Prostate-specic antigen alone (99.2F), blood pressure is 110/90 mm Hg, and
(D) Transrectal needle biopsy heart rate is 100/min. He localizes pain on deep
(E) Yearly history and physical examination palpation of the nail beds and sternal rub but is
still in a state of altered consciousness. Labora-
13. A 67-year-old woman with type 2 diabetes mel- tory tests show:
litus, asthma, coronary artery disease, and atrial Na+: 120 mEq/L
brillation was admitted to the hospital for K+: 4.5 mEq/L
uid repletion 3 days ago after a 1-week history Cl: 94 mEq/L
of diarrhea. On admission her serum glucose CO2: 24 mmol/L
level was 400 mg/dL, sodium was 156 mEq/L, BUN: 20 mg/dL
BUN was 30 mg/dL, and creatinine was 0.9 Creatine: 1.0 mg/dL
mg/dL. Examination revealed dry mucous Glucose: 88 mg/dL
membranes and tenting of the skin on her
hands. She was given insulin and promptly A urine specimen obtained by Foley catheter
started on 0.45% saline at 200 mL/hr. On her shows a urine osmolality of 300 mOsm/kg and
second hospital day her glucose level had nor- urine sodium level of 40 mEq/L. After treat-
malized, she produced 100 mL/hr of urine, ment of the acute hyponatremia with slow ad-
and her sodium level was within normal limits. ministration of hypertonic saline, an extensive
Later in the day she began to complain of fa- work-up reveals a neoplasm. Which of the fol-
tigue and weakness. Today, hospital day 3, she lowing is the most likely neoplasm based on
has difculty moving her limbs and is com- the patients hyponatremia?
plaining of severe constipation. An ECG from (A) Insulinoma
her second hospital day is shown in the image. (B) Multiple myeloma
Which of the following would have prevented (C) Small cell lung cancer
her most recent symptoms? (D) Testicular embryonal tumor
(E) Thymic carcinoid
HIGH-YIELD SYSTEMS
446 Section I: Organ Systems Questions

15. A 48-year-old man is brought to the emergency mg/dL, and protein C level of 2.1 g/mL (nor-
department confused and disoriented. He re- mal: >4 g/mL). Urinalysis reveals heavy pro-
ports recent onset of nausea and has had several teinuria. Which of the following disorders is
episodes of emesis in the past 4 days. On further most often associated with this presentation?
questioning he also notes a metallic taste in his
(A) Alcohol consumption
mouth, frequent hiccups, and pruritus. On
(B) Asthma
physical examination there is a rough, Velcro-
(C) Diabetes mellitus
like sound heard across his precordium. Which
(D) Hypertension
of the following is the most likely diagnosis?
(E) Minimal change disease
(A) Addisonian crisis
(B) Fulminant hepatic failure 18. A 23-year-old construction worker is admitted
(C) Heroin withdrawal after his right leg became trapped under a con-
crete block at a construction site. After removal
Renal/Genitourinary

(D) Renal insufciency


(E) Vitamin B12 deciency of the block, there were visible signs of crush
injury to his thigh, and he was airlifted to a lo-
16. A 60-year-old man presents to the urologist cal hospital. In transit he received several bags
complaining of difculty urinating. He states of intravenous uids and analgesics. He is alert
he frequently gets out of bed in the middle of and oriented on arrival at the hospital. His
the night to go to the bathroom. Once he gets blood pressure is 140/80 mm Hg, pulse is 70/
to the bathroom he cant urinate easily and min, and respiratory rate is 12/min. Physical
must bear down to do so. He denies any his- examination is signicant for multiple contu-
tory of sexually transmitted disease, trauma to sions and abrasions on the anterior portion of
the genitourinary tract, or prior genitourinary his right thigh. Distal pulses are palpable, and
instrumentation. On rectal examination the sensation in the affected extremity is normal.
patient has an enlarged prostate and one 1-cm Laboratory studies show a potassium level of
area of induration located on the middle poste- 6.1 mEq/L, phosphate of 8.2 mg/dL, calcium
rior aspect of the prostate. He has a prostate- of 6.3 mg/dL, and uric acid of 20 mg/dL. What
specic antigen level of 6 ng/mL (normal: 04 additional laboratory nding is most consistent
ng/mL), BUN of 20 mg/L, and creatinine of with his most likely diagnosis?
1.6 mg/L. The patient undergoes a transrectal
(A) Creatine kinase levels <1000 IU/L
prostate biopsy, and no dysplasia or atypia is
(B) Decreased serum lactic acid dehydroge-
present. Which of the following is the most ap-
nase
propriate treatment?
(C) Elevated homocysteine levels
(A) Brachytherapy (D) Elevated troponin I levels
(B) Finasteride (E) Positive urine dipstick for blood without
(C) Radical retropubic prostatectomy RBCs on microscopic examination
(D) Transurethral resection of the prostate
(E) Watchful waiting 19. The radiograph shown in the image was taken
in a 36-year-old man who presented to the
17. A 45-year-old woman presents to her physician emergency department with severe left ank
complaining of lower extremity edema. Her pain and one episode of hematuria. Which of
temperature is 36.7C (98.1F), blood pressure the following measures could cause another
is 150/90 mm Hg, heart rate is 80/min, and re- episode of this patients symptoms?
spiratory rate is 14/min. In addition to the nd-
(A) Calcium restriction
ing of 1+ pitting edema of her ankles bilater-
(B) Increased uid intake (>2 L/d)
ally, she has diffuse crackles on pulmonary
(C) Protein restriction
examination. She denies shortness of breath,
(D) Sodium restriction
chest pain, or orthopnea. Initial laboratory
(E) Thiazide diuretic
evaluation reveals an albumin level of 1.8 g/dL
(normal: 3.55.5 g/dL), total cholesterol of 200
HIGH-YIELD SYSTEMS
Chapter 15: Renal/Genitourinary Questions 447

21. A 37-year-old man with a history of alcoholism


and ulcerative colitis is being evaluated for an
elevated BUN level. He was rst admitted after
complaining of weakness and lightheadedness
several weeks ago following an episode of
bloody diarrhea. Initial work-up revealed
heme-positive stool and hemoglobin of 7.1 mg/
dL. A colonoscopy was performed revealing se-
vere colitis. He was transfused with 2 U of
packed RBCs. Ten days later, he began to com-
plain of abdominal pain and was febrile. Cur-
rently his blood pressure is 100/60 mm Hg and
pulse is 110/min. Physical examination shows

Renal/Genitourinary
abdominal tenseness with rebound and pale
extremities. Emergent colectomy is performed,
and broad spectrum antibiotics are started. In
recovery, laboratory values demonstrate an ele-
vation in his BUN to 82 mg/dL. Urinalysis is
heme negative and does not reveal any casts.
What is most likely to have contributed to this
Reproduced, with permission, from Tanagho EA, McAn- patients azotemia?
inch JW. Smiths General Urology, 17th edition. New
York: McGraw-Hill, 2008: Figure 16-15. (A) Alcoholism
(B) Blood transfusion
20. A 42-year-old woman presents to the emer- (C) Broad spectrum antibiotics
gency department complaining of 3 days of fe- (D) Gastrointestinal bleeding
ver, severe right-sided abdominal pain, and dy- (E) Glomerulonephritis
suria. She has had similar symptoms twice in
the past year and was treated for urinary tract 22. A 5-year-old girl with acute lymphocytic leuke-
infection. Past medical history is also signi- mia has been hospitalized on the pediatric he-
cant for hypertension diagnosed 2 years earlier, matology-oncology service for more than 2
for which she takes a diuretic and an angio- months. A physician has been assigned to take
tensin-converting enzyme inhibitor. Her tem- care of her on his rst day. During her hospi-
perature is 38.6C (101.4F), pulse is 82/min, talization the patient has had many infections,
respiratory rate is 18/min, and blood pressure is including Candida septicemia. For the past 2
160/100 mm Hg. On physical examination the weeks, her main problem has been persistent
patient appears acutely distressed. The abdomi- hypokalemia, despite daily repletion. What
nal examination is limited by voluntary guard- laboratory value should be measured in this pa-
ing. CT of the abdomen shows enlarged kid- tient, and is it expected to be high or low?
neys with multiple cysts. Urinalysis reveals (A) Ca2+, high
many WBCs and RBCs as well as 1+ protein (B) Ca2+, low
and positive leukocyte esterase. What factor is (C) Mg2+, high
best correlated with the penetrance of this pa- (D) Mg2+, low
tients disease? (E) Na+, high
(A) Age (F) Na+, low
(B) Diet high in protein
(C) Frequency of urinary tract infections
(D) Presence of hepatic cysts
(E) Spontaneous mutation
HIGH-YIELD SYSTEMS
448 Section I: Organ Systems Questions

23. A 50-year-old man presents to his primary care 25. A 40-year-old woman presents to the urologist
physician with the chief complaint of inability for evaluation of a renal mass discovered inci-
to achieve an erection. The patient has type 2 dentally on CT of the abdomen. The mass was
diabetes mellitus that is well controlled with 5 cm in diameter, originating from the upper
diet and exercise, atrial brillation, seasonal al- pole of the right kidney. The center of the mass
lergies, and a past surgical history of a bilateral contains an area of calcication. The patient
varicocelectomy. He currently takes warfarin has a distant history of acute pancreatitis second-
and intranasal steroids. The man states that his ary to a scorpion bite and a 20-pack-year smok-
problem with achieving an erection has been ing history. She does not drink alcohol. The de-
going on for the past 3 weeks. He admits to cision was made to proceed with surgical
having masturbated about a week ago. He did removal of the mass, and a partial nephrectomy
this because he noticed that he was able to was performed. The patient did well postopera-
have a normal erection when the woman with tively, and her Foley catheter was removed on
Renal/Genitourinary

whom he is having an affair left for the morn- postoperative day 2. That evening she had sub-
ing. He states that it took him about 3 minutes stantial output into a Jackson-Pratt drain that the
to reach climax and ejaculate. He denies any surgeons left in place. The output of the drain
curvature of the penis with erection or when has been 30 cc/hr for the past few hours, and
accid. The man is afebrile, has a heart rate of the uid is light pink to clear. The patient is ex-
110/min that is regularly irregular, and a blood amined and states that she is not in pain. Her
pressure of 130/85 mm Hg. His abdomen is temperature is 37.7C (99.8F), heart rate is 80/
soft, with no palpable masses. The man has an min, and blood pressure is 110/70 mm Hg.
uncircumcised penis that is normally devel- What test or procedure would be appropriate to
oped and two testicles that are normal in size conrm the suspected diagnosis?
and texture. What is the most likely cause of
(A) Exploratory laparotomy
the mans erectile dysfunction?
(B) Measurement of amylase level in Jackson-
(A) Diabetes mellitus Pratt drain
(B) Medication (C) Measurement of creatinine level in Jackson-
(C) Peyronies disease Pratt drain
(D) Prior genitourinary surgery (D) Measurement of serum amylase level
(E) Psychogenic (E) Measurement of serum hemoglobin level

24. A 38-year-old man was thrown from his motor- 26. A 70-year-old woman with a history of renal ar-
cycle and brought to the emergency department tery stenosis presents to the emergency depart-
by emergency medical services. The paramedics ment with decreased urine output. She was re-
report his initial blood pressure was 83/52 mm cently started on a new medication by a
Hg, pulse was 128/min, and respiratory rate was physician, and a few days later noticed that she
33/min. He was given 2 L of lactated Ringers was producing less urine. Otherwise she feels
solution en route, and medical antishock trou- well and has no complaints. She is afebrile and
sers were applied. On arrival to the trauma bay her blood pressure is 156/88 mm Hg. Labora-
his blood pressure is 108/76 mm Hg, pulse is tory tests show:
105/min, and respiratory rate is 28/min. On ex-
Na+: 139 mEq/L
amination pelvic compression elicits severe
K+: 4.1 mEq/L
pain, and blood is noted at the urethral meatus.
HCO3: 24 mEq/L
Which of the following is the most appropriate
BUN: 41 mg/dL
next step in evaluating the blood at the urethral
Creatinine: 1.8 mg/dL
meatus?
Urine Na+: 6 mEq/L
(A) Abdominal ultrasound Urine creatinine: 11 mg/dL
(B) Digital rectal examination
Which of the following medications did she
(C) Foley catheter insertion and urinalysis
most likely start recently?
(D) Intravenous pyelogram
(E) Retrograde urethrogram
HIGH-YIELD SYSTEMS
Chapter 15: Renal/Genitourinary Questions 449

(A) Amikacin (A) Bilateral anorchia


(B) Benztropine (B) Congenital adrenal hyperplasia
(C) Cimetidine (C) Cryptorchidism
(D) Enalapril (D) Retractile testis (pseudocryptorchidism)
(E) Hydroxyurea (E) Strangulated inguinal hernia

27. A 52-year-old man is recovering in the surgical 29. A 65-year-old patient with a history of bipolar
intensive care unit from a total colectomy for disorder, well-controlled with lithium, is being
colorectal adenocarcinoma 1 day earlier. He evaluated for hypernatremia. Her only com-
has one peripheral intravenous line that is be- plaint is 4 months of polyuria and thirst. Her
ing used to run a patient-controlled analgesia blood pressure is 106/68 mm Hg and pulse is
pump, 0.5 normal saline at 86 mL/hr, and ce- 102/min. Physical examination reveals her mu-
fazolin. He has no complaints, appears well, cous membranes are dry, and skin turgor is

Renal/Genitourinary
and is conversational. Relevant laboratory nd- normal. The remainder of the physical exami-
ings are a serum sodium level of 110 mEq/L; nation is unremarkable. Laboratory tests show:
his sodium level was 137 mEq/L 1 day earlier.
Na+: 147 mEq/L
What is the best next step in the management
K+: 4.7 mEq/L
of this patient?
Cl: 110 mEq/L
(A) Discontinue cefazolin HCO3: 24 mEq/L
(B) Discontinue patient-controlled analgesia BUN: 12 mg/dL
pump Creatinine: 1.1 mg/dL
(C) Draw blood for testing from the other arm Plasma osmolality: 305 mOsm/kg
(D) Restrict uid intake Urine osmolality: 200 mOsm/kg
(E) Switch to hypertonic saline infusion
Which of the following is most likely to resolve
this patients electrolyte imbalance?
28. A 4-month-old boy born at 29 weeks gestation
is brought to the pediatrician for a well-baby (A) Exogenous ADH
examination. During the pregnancy, an amnio- (B) Fluid restriction
centesis was performed out of concern for ad- (C) Intravenous uids
vanced maternal age (40 years). Results were (D) Potassium restriction
normal, and the karyotype was normal, 46 XY. (E) Thiazide diuretic
The child has been feeding well, and interacts
with his mother and father by tracking their
faces and vocalizing when excited. His temper-
ature is 36.4C (97.5F) and heart rate is 120/
min. Genitourinary examination shows a well-
formed penis with no evidence of hypospadias
or epispadias and a well-formed scrotum with
no palpable testes. On abdominal examination
there is a mass palpable in the left lower quad-
rant. The mass is 1 cm in diameter and found
midway between the pubic symphysis and the
anterior superior iliac crest. This mass does not
move on gentle manipulation. Which of the
following is the most likely diagnosis?
HIGH-YIELD SYSTEMS
450 Section I: Organ Systems Questions

30. A 47-year-old woman comes to the emergency given that fungi can be difcult to isolate and
department with the chief complaint of cold the man continues to show signs of infection.
ngers. She has had intermittent episodes of For which of the following does amphotericin
this condition for several years, but today the put the patient at greatest risk?
pain was unbearable. On review of systems she
(A) Leukocytoclastic vasculitis
mentions chronic symptoms of reux and pro-
(B) Nephrogenic diabetes insipidus
gressive difculty moving her ngers. When
(C) Type I distal renal tubular acidosis
the physician examines her, he notes the ap-
(D) Type II proximal renal tubular acidosis
pearance of her ngers (see image), and he
(E) Type IV distal renal tubular acidosis
also sees multiple telangiectasias on her skin
and hard nodules on the extensor surfaces of 32. A 29-year-old man with no past medical history
her forearms. If the electrolyte abnormality as- presents to a urologist after 2 years of unsuc-
sociated with this patients most likely disease is cessful attempts at conceiving a child. The
Renal/Genitourinary

found, what is the rst step in correcting it? man states that his wife is 24 years old and has
no medical problems. She was evaluated for
infertility by a gynecologist, and no abnormali-
ties were found. The man has no history of sex-
ually transmitted disease or urologic diseases.
Physical examination reveals a tall man with
long legs who appears younger than his stated
age. He has minimal facial hair and a slight
fullness to his breasts bilaterally. The patients
testicles are 2.2 cm long and rm. A semen
sample is obtained, which shows no sperm. For
what disease is this man at increased risk?
Reproduced, with permission, from Wolff K, Johnson RA,
(A) Germ cell tumor
Surmond D. Fitzpatricks Color Atlas & Synopsis of Clini-
cal Dermatology, 5th edition. New York: McGraw-Hill,
(B) Paraphimosis
2005: Figure 14-32. (C) Peyronies disease
(D) Renal cell carcinoma
(A) 1 ampule of intravenous glucose (E) Transitional cell carcinoma
(B) Insulin and bicarbonate
(C) Intravenous calcium gluconate 33. A pediatrician notes bilateral abdominal masses
(D) Intravenous normal saline in a 4-month-old girl. The girls mother reports
(E) Oral calcium channel blocker that the child has been very irritable and is not
interested in eating, and has noticeably de-
31. A 43-year-old man with hypertension and acute creased formula intake. Her temperature is
myelogenous leukemia is admitted to the hospi- 36.6C (97.8F), blood pressure is 140/90 mm
tal for fever and chills. He is currently receiving Hg, respiratory rate is 30/min, and heart rate is
chemotherapy, but he does not know which 120/min. Abdominal examination reveals
kind. He is febrile to 38.5C (101.3F) and has marked upper abdominal fullness and irregu-
a respiratory rate of 12/min. His blood work is larly contoured bilateral masses, but no appar-
notable for a WBC count of 10,000/mm and ent tenderness to palpation. There are in-
an absolute neutrophil count of 900/mm. The creased tortuous makings surrounding the
patient undergoes blood cultures, urine cul- childs umbilicus. Normal bowel sounds are
tures, sputum cultures, and x-ray of the chest, all heard. Laboratory tests show:
of which are negative. He is started on empiric
broad-spectrum antibiotics. The patient contin- WBC count: 6000/mm
ues to spike temperatures to 38.5C (101.0 F). Creatinine: 1.0 mg/dL
The decision is made to start the patient on em- Aspartate aminotransferase: 8 U/L
piric antifungal therapy with amphotericin B, Alanine aminotransferase: 12 U/L
HIGH-YIELD SYSTEMS
Chapter 15: Renal/Genitourinary Questions 451

CT with intravenous contrast reveals the kid- and a temperature of 37.6C (99.6F). The re-
neys are 10 cm in the vertical dimension (10 mainder of the physical examination is unre-
12 cm is normal size in full-grown adults) with markable. He also has a past medical history
cortical cystic lesions, and they exhibit poor signicant for untreated benign prostatic hy-
uptake of the contrast. Which of the following perplasia. What intervention is most likely to
is the most likely diagnosis? prevent the recurrence of these symptoms?
(A) Autosomal dominant polycystic kidney dis- (A) Increase uid intake
ease (B) Lithotripsy
(B) Autosomal recessive polycystic kidney dis- (C) Surgical intervention to remove an ob-
ease structing renal calculus
(C) Bilateral megaureters (D) Treatment for benign prostatic hyperplasia
(D) Chronic viral hepatitis (E) Treatment with antibiotics

Renal/Genitourinary
(E) Medullary cystic disease
36. A 33-year-old African-American man presents
34. A 56-year-old man presents to the emergency to a rural clinic with a 2-day history of painful
department because of severe pain in his right genital sores. The man admits to sexual con-
ankle. The pain began 4 days ago, and he de- tact with multiple partners and has never been
nies any history of trauma. His only medical tested for sexually transmitted diseases, includ-
problem is recent diagnosis of hypertension, ing HIV. On physical examination there are
for which he takes a mild antihypertensive. His two tender sores with sharp edges and a yellow-
temperature is 37.2C (98.9F), blood pressure ish exudate on his penis. Tender unilateral in-
is 138/68 mm Hg, pulse is 80/min, and respira- guinal adenopathy is also noted. Which of the
tory rate is 14/min. On examination his ankle following is most likely to conrm the diagno-
is swollen, red, and diffusely tender. Joint uid sis?
aspirate reveals needle-shaped negatively bire-
(A) Fluorescent treponemal antibody absorp-
fringent crystals under polarized light micros-
tion test
copy. What medication most likely contributed
(B) HIV viral load
to his current presentation?
(C) Polymerase chain reaction for Haemophi-
(A) Atenolol lus ducreyi
(B) Diltiazem (D) Serology for L1, L2, and L3 serotypes of
(C) Enalapril Chlamydia trachomatis
(D) Hydralazine (E) Tissue biopsy
(E) Hydrochlorothiazide (F) Tzanck smear

35. A 55-year-old man with diabetes presents with


fever, malaise, and dysuria. On physical exami-
nation he has costovertebral angle tenderness
HIGH-YIELD SYSTEMS
452 Section I: Organ Systems Questions

37. A 78-year-old man is admitted to the medical 38. A 10-year-old boy is brought to his pediatrician
intensive care unit with aspiration pneumonia because his legs and arms have looked puffy
and sepsis. Before admission, he had been treated for the past 3 days. The pediatrician notes that
at his nursing home with sulfamethoxazole/ he saw this child in the clinic 2 weeks earlier
trimethoprim for a urinary tract infection. In with the chief complaint of a sore throat and
the hospital he is treated with additional anti- prescribed antibiotics. On examination the boy
biotics for pneumonia and large volumes of has diffuse upper and lower extremity edema,
uid. He becomes hemodynamically stable and his blood pressure is 168/110 mm Hg. Uri-
with treatment, but his fever persists, and he nalysis shows microscopic hematuria and 1+
develops a maculopapular rash on his chest, proteinuria.
abdomen, and thighs. His WBC count is
12,000/mm (66% neutrophils, 23% lympho- 39. A 23-year-old man calls his primary care physi-
cytes, and 9% eosinophils), BUN is 34 mg/dL, cian in a panic because he noticed that the toi-
Renal/Genitourinary

and creatinine is 2.1 mg/dL. Cultures are nega- let was full of bright red blood after urinating.
tive. Which of the following is the most likely A review of systems is positive for a recent bad
diagnosis? cold. Based on a urinalysis showing gross he-
maturia and 1+ protein, the physician recom-
(A) Acute interstitial nephritis
mends a renal biopsy, which shows diffuse me-
(B) Glomerulonephritis
sangial IgA deposits.
(C) Hypovolemic renal failure
(D) Nephrolithiasis 40. An 8-year-old girl is brought to her pediatrician
(E) Pyelonephritis after complaining to her father that her ballet
outt, purchased 1 month earlier, feels too
E X T E N D E D M ATC H I N G tight. In the review of systems, her father notes
that she has not been to her biweekly ballet
The response options for the next 3 items are
class very often recently because she has been
the same. Select one answer for each item
sick repeatedly with mild colds. On examina-
in the set.
tion she has 2+ edema to the knees and elbows
bilaterally. Laboratory testing shows albumin
For each patient with renal failure, select the most level of 2.4 g/dL and total protein of 5.3 g/dL,
likely etiology. and urinalysis shows 4+ protein. Her symptoms
and laboratory studies improve dramatically af-
(A) Alports syndrome ter 4 weeks of prednisone.
(B) Diabetic nephropathy
(C) Focal segmental glomerulosclerosis
(D) Goodpastures syndrome
(E) IgA nephropathy
(F) Lupus nephritis
(G) Membranoproliferative nephropathy
(H) Membranous nephropathy
(I) Minimal change disease
(J) Poststreptococcal glomerulonephritis
(K) Renal amyloidosis
(L) Wegeners granulomatosis
HIGH-YIELD SYSTEMS
Chapter 15: Renal/Genitourinary Answers 453

AN S W E R S

1. The correct answer is D. The clinical scenario Answer A is incorrect. Antibody deposition in
is consistent with a generalized proximal renal the glomerular basement membrane is seen in
tubular dysfunction known as Fanconis syn- Goodpastures syndrome, a type of nephritic
drome. The patients Fanconis syndrome is syndrome that presents with hemoptysis and
manifest as evidence of a non-anion gap meta- respiratory symptoms in addition to renal nd-
bolic acidosis accompanied by hypophos- ings.
phatemia and glycosuria (with normal blood
Answer B is incorrect. Cocaine abusers can
glucose levels).
develop dilated cardiomyopathy, which can
Answer A is incorrect. Right heart failure is lead to congestive heart failure. Heart failure
also known as cor pulmonale. This classically can cause edema, but the edema would likely

Renal/Genitourinary
occurs secondary to the hypoxia and pulmo- be conned to the lower extremities, and other
nary hypertension that result from chronic ob- ndings on examination would include gallops
structive pulmonary disease. This is not related or murmurs, elevated jugular venous pressure,
to the patients laboratory ndings. and signs of pulmonary vascular congestion.
Answer B is incorrect. Steroids may cause hy- Answer C is incorrect. Cocaine may be in-
perglycemia. However, the glycosuria that is jected and leave marks such as those seen on
seen with steroids should be accompanied by the arms of this patient. Also, cocaine may
hyperglycemia, not a blood glucose level of 98 cause acute renal failure secondary to rhab-
mg/dL. domyolysis, but red urine and myoglobuline-
Answer C is incorrect. Type I renal tubular mia would also be seen.
acidosis (RTA) is a distal process. It occurs Answer E is incorrect. Diabetic patients with
from a renal distal tubular insult, which causes diabetic nephropathy can present with protei-
a defect in urinary ammonium production nuria in the nephrotic range. Diabetic neph-
and urine acidication. Type I RTA would not ropathy is particularly common in diabetics
cause urine phosphate and glucose wasting, with poor glycemic control. However, there is
which are more indicative of proximal tubule no evidence that this patient has diabetes; he
dysfunction. does not complain of polyuria or polydipsia
Answer E is incorrect. The most common type and he has no glucose or ketones reported in
of RTA is type IV. This type is characterized by his urine.
hyperkalemic, hyperchloremic metabolic aci- Answer F is incorrect. Streptococcal infection
dosis. can cause postinfectious glomerulonephritis, a
type of nephritic syndrome that presents with
2. The correct answer is D. This patient presents hypertension, hematuria, and mild proteinuria,
with generalized edema, hypoalbuminemia, usually several weeks after an upper respiratory
and marked proteinuria, ndings strongly sug- infection. The patient presented here has a
gestive of a nephrotic syndrome. The track nephrotic, not nephritic, picture.
marks on his arms are highly suggestive of
drug abuse. Putting this picture together, one 3. The correct answer is F. The patient has tran-
can deduce that the patient has focal segmen- sitional cell carcinoma (TCC). Most bladder
tal glomerulosclerosis, a nonspecic nephrotic cancers are TCCs. The disease is more com-
syndrome that may be secondary to heroin mon in men (2.7:1 male-female ratio), and the
abuse. Other causes of secondary focal seg- mean age of diagnosis is 65 years. Smoking and
mental glomerulosclerosis include lithium and exposure to industrial solvents and dyes such as
malignancy, particularly lymphoma. aniline are other risk factors for TCC. Painless
hematuria is the presenting symptom in about
HIGH-YIELD SYSTEMS
454 Section I: Organ Systems Answers

90% of bladder carcinomas. Irritative voiding to albumin is also low, and calcium measure-
symptoms can also occur, depending on the lo- ments will be demonstrably low. Indeed, for
cation and size of the lesion. each 1 g/dL reduction in albumin levels, the
total calcium concentration will be lowered by
Answer A is incorrect. Acute bacterial prosta-
0.8 mg/dL. This reduction does not represent
titis usually presents with perineal, suprapubic,
a reduction in ionized calcium, and thus will
or sacral pain. Patients may complain of ir-
not cause clinical hypocalcemia.
ritative voiding symptoms. Depending on the
severity, pain, fever, and urinary obstruction Answer A is incorrect. BUN levels correlate
with retention can occur. Prostate examination to dietary intake and hydration status. In an
would be contraindicated in such men because undernourished elderly woman, it is not sur-
it may result in sepsis. prising that as protein intake decreased, BUN
dropped slightly. BUN, however, plays no role
Answer B is incorrect. Primary adenocarcino-
Renal/Genitourinary

in the measurement of total or ionized calcium


mas of the bladder are rare. Adenocarcinoma
levels.
accounts for approximately 2% of bladder tu-
mors in the United States. TCC constitutes the Answer B is incorrect. Creatinine plays no
vast majority of bladder neoplasms. role in determining calcium levels.
Answer C is incorrect. Prostate cancer may Answer C is incorrect. Creatinine levels cor-
present as a focal nodule within the pros- relate to lean muscle mass levels. In an un-
tate during digital rectal examination (DRE). dernourished and elderly woman, it is not sur-
Many prostate cancers are only detected by bi- prising that creatinine levels are subnormal.
opsy prompted by an elevated prostate-specic Creatinine levels, however, play no role in the
antigen (PSA) level. measurement of total or ionized calcium lev-
els.
Answer D is incorrect. The patient is not pre-
senting with obstructive symptoms that would Answer D is incorrect. Decreased serum albu-
be consistent with benign prostatic hyperpla- min levels lead to an underestimation of clini-
sia (BPH). BPH may also result in an enlarged cally important calcium levels. Aggressive cal-
prostate on DRE. cium repletion is not warranted.
Answer E is incorrect. Interstitial cystitis
5. The correct answer is B. With signicant
presents with pain during periods of bladder
azotemia, friction rub, and presentation typical
distention. This pain is often relieved by void-
of pericardial irritation, this patient is likely suf-
ing. Frequency, urgency, and nocturia are also
fering from uremic pericarditis. In addition, his
common. The diagnosis is often one of exclu-
ECG shows ST segment elevations, which are
sion, although cystoscopy can aid in the diag-
characteristic of the early stages of pericarditis.
nosis.
His stable blood pressure, with absent pulsus
paradoxus, and general appearance suggest
4. The correct answer is E. This patient is under-
pericarditis without tamponade physiology.
nourished and in a neglected state of health.
This is an important distinction to make be-
Often, the only manifestation of poor nutrition
cause it greatly inuences the initial manage-
in an elderly patient is subtle changes in rou-
ment of uremic pericarditis. In the presence of
tine laboratory values including creatinine, a
tamponade, drainage of the effusion is of para-
reection of decreased muscle mass; albumin,
mount importance before other measures are
a reection of poor protein intake; and cal-
undertaken to reverse the cause of the disease.
cium. Because active, ionized calcium is only a
Because this patient likely does not have tam-
fraction (45%) of what is measured in total cal-
ponade, initial management consists of emer-
cium, there are some settings in which a low
gent hemodialysis. As uremic patients are co-
total calcium measurement does not reect
agulopathic, heparin should not be used in the
overall calcium depletion. Specically, when
extracorporeal circuit during hemodialysis.
circulating albumin levels are decreased, the
40% of total calcium that is normally bound
HIGH-YIELD SYSTEMS
Chapter 15: Renal/Genitourinary Answers 455

Answer A is incorrect. This patient is likely Answer A is incorrect. A blood culture is un-
suffering from chronic or acute chronic renal necessary. A source of infection is apparent,
failure and subsequent uremia. These patients and urine culture will likely identify the organ-
are likely to be volume overloaded because ism. Urosepsis is not suspected because the pa-
decreased renal function leads to accumula- tient appears well with normal blood pressure.
tion of ingested uids. Administration of intra-
Answer B is incorrect. In prostatitis, elevated
venous uids without instituting hemodialysis
levels of PSA are often observed. Serially, mea-
will not address the pericardial process and
suring the PSA is not useful in the diagno-
may put additional strain on the heart.
sis or treatment of prostatitis. PSA is used in
Answer C is incorrect. Intravenous pyelo- the screening of prostate cancer. A physician
gram (IVP) is a diagnostic study that allows for should, however, wait until the prostatitis has
examination of both the upper and the lower resolved before carrying out such screening.

Renal/Genitourinary
urinary tract. It is employed in renal failure pa- In patients with prostate cancer, PSA levels
tients to evaluate for reversible causes of renal can be used postoperatively to assess for com-
failure such as pyelonephritis, renal calculi, or pleteness of the proctectomy and disease recur-
obstruction. IVP cannot reverse this patients rence.
uremic pericarditis.
Answer C is incorrect. Transrectal ultrasound
Answer D is incorrect. Heart catheterization is is not necessary for the diagnosis of prostatitis.
an unnecessary procedure for this patient be- Transrectal ultrasound would only be indicated
cause his process is not ischemic in nature. if the patient did not respond to conventional
antibiotic therapy or if prostatic abscess was
Answer E is incorrect. Pericardial uid drain-
suspected. In addition, ultrasound of the blad-
age should take precedence over dialysis only
der and kidneys to assess for substantial post-
if tamponade physiology is present.
void residual or hydronephrosis from urinary
6. The correct answer is E. The diagnosis is acute obstruction may also be indicated. CT scan
bacterial prostatitis. Acute bacterial prostati- may also be useful if an abscess was suspected.
tis is the most commonly diagnosed urologic Answer D is incorrect. Transrectal ultrasound
disorder in men <50 years old. If prostatitis is with transrectal prostate biopsy is used in eval-
suspected, the clinician should send a urine uating men suspected of having prostatic can-
culture, and some clinicians suggest doing this cer. The elevated PSA level in this case can be
after gentle prostatic massage. The prostatic ex- explained by the patients probable infection,
pressate can also be cultured and/or analyzed and thus no testing for prostate cancer is nec-
microscopically. Although prostatic massage essary at this point.
is the most complete way to document the di-
agnosis, care should be taken during the ma- 7. The correct answer is C. This patient pre-
nipulation of the prostate. If the patient does sents with prerenal failure, as evidenced by the
have acute bacterial prostatitis, vigorous mas- BUN:creatinine (Cr) ratio >20:1. His fatigue
saging can lead to bacteremia. In addition, this and malaise is likely secondary to renal failure.
maneuver can be extremely painful for the pa- Oxaliplatin is associated with dehydration in
tient. The diagnosis is generally a clinical one some patients, and patients are encouraged to
in combination with a positive urine culture. keep up their oral uid intake after treatment.
The presentation of urgency, frequency, and/or This patient also has a potentially high-output
retention in addition to systemic signs such as stula, which can contribute to hypovolemia if
fever would be enough to make the presump- he fails to keep up his oral uid intake.
tive diagnosis. Furthermore, if the patient is
Answer A is incorrect. Nonsteroidal anti-
in retention and the clinician is suspicious of
inammatory drug-induced acute interstitial
prostatitis, the placement of a suprapubic tube
nephritis (AIN) would present with an intrin-
would be safer than risking sepsis by placing a
conventional Foley catheter.
HIGH-YIELD SYSTEMS
456 Section I: Organ Systems Answers

sic renal failure picture, with the BUN:Cr ratio Answer A is incorrect. Ascorbic acid or vi-
<20:1 and WBCs with casts on urinalysis. tamin C is not used in the treatment of Pey-
ronies disease. However, there is some sup-
Answer B is incorrect. Oxaliplatin may in-
port for treating this disorder with vitamin E
crease serum creatinine but does not have
and aminobenzoic acid, which are thought to
acute renal failure as a major adverse effect.
break down the brous connective tissue of the
Additionally, drug-induced acute renal failure
plaque.
would be expected to show an intrinsic renal
failure picture, with a BUN:Cr ratio <20:1. Answer B is incorrect. Priapism, or prolonged
This patient has a prerenal failure prole. painful erection, is treated with decompression
of the corpora cavernosa.
Answer D is incorrect. The patient does com-
plain of back pain, which might suggest neph- Answer D is incorrect. After failed medical
rolithiasis. Nephrolithiasis may cause ureteral therapy, if the patient remains impotent, a pe-
Renal/Genitourinary

obstruction, but it would be highly unlikely nile prosthesis can be inserted as an alternative
to have bilateral obstruction causing postrenal to surgical management.
failure. Kidney stones may also be associated
Answer E is incorrect. The Nesbit procedure
with acute tubular necrosis, but this would
is successfully used in patients with Peyronies
present with an intrinsic renal failure, includ-
disease that can achieve an erection but have
ing BUN:Cr <20:1 and granular or renal tubu-
failed conservative management. The proce-
lar casts on urinalysis.
dure involves making an incision in the tunica
Answer E is incorrect. Obstruction distal to albuginea and placing a plicating stitch on the
the kidneys can cause postrenal azotemia, convex side of the penis across from the point
which would present with a similar clinical of greatest curvature, thus straightening the
picture. However, bilateral ureteral obstruction penis. Plaque removal and grafting have also
is highly unlikely in this patient, and his tumor been utilized.
is likely too superior to cause urethral obstruc-
tion. 9. The correct answer is B. Corticosteroids are
a very commonly used class of medication. In
8. The correct answer is C. Peyronies disease this case, steroids are used to control the renal
is a clinical problem affecting middle-aged manifestations of her systemic lupus erythema-
and older men that is caused by the develop- tosus. However, they can lead to multiple lo-
ment of a brous plaque involving the tunica cal and systemic adverse effects that a clinician
albuginea. Patients present with a curvature of must be able to recognize. Commonly, these
the penis, pain with erection, and poor erec- include acne, hirsutism, hypertrichosis, and
tion distal to the area of induration. In about weight gain. Corticosteroid use is less com-
50% of cases, the induration spontaneously monly associated with diabetes, hypertension,
resolves. Because often no medical treatment arrhythmias, and osteoporosis. Other important
is needed, observation and emotional support adverse effects include reduced defense against
are advised as the rst line of therapy. If the in- infection and several psychiatric disturbances,
duration does not resolve, medical treatment including euphoria, psychosis, and depression.
can be attempted. Medical treatment includes
Answer A is incorrect. Patients taking cortico-
vitamin E, calcium channel blockers, and lo-
steroids are likely to experience hypertrichosis,
cally injected steroids to aid in the breakdown
not hair loss.
of connective tissue. Surgical treatment in-
volves excision of the plaque and skin graft. Answer C is incorrect. Corticosteroids typi-
Complications of surgery include loss of sexual cally cause hyperglycemia, not hypoglycemia,
function and damage to the urethra. Medical by increasing hepatic gluconeogenesis and by
treatment has limited success, but should be inhibiting peripheral utilization and glucose
attempted prior to surgical correction. uptake.
HIGH-YIELD SYSTEMS
Chapter 15: Renal/Genitourinary Answers 457

Answer D is incorrect. Corticosteroids pro- Answer D is incorrect. An MRI of the spine is


duce a leukocytosis, not leucopenia, by caus- a useful test to evaluate children suspected of
ing an increase in absolute neutrophil counts, having spinal cord abnormalities. The physical
or neutrophilia. This is caused by decreased examination ndings of a tuft of hair or a sacral
neutrophil adhesion and increased release of dimple might suggest that this test is the appro-
stores of the bone marrow. priate imaging modality. In the absence of such
ndings or other neurologic decits, MRI is not
Answer E is incorrect. Patients taking cortico-
the most appropriate imaging study at this time.
steroids are likely to experience weight gain,
not weight loss.
11. The correct answer is E. Cystinuria is typically
caused by a genetic defect in the tubular amino
10. The correct answer is E. The girl has pyelo-
acid transport system. This disorder is difcult to
nephritis and a scar from a previous insult. A
adequately treat. There are no medications that

Renal/Genitourinary
voiding cystourethrogram, carried out after the
can reverse the defect in cystine transport, and
acute infection is resolved, will help determine
thus most therapies target increasing the solu-
if the patient has vesicoureteral reux, a condi-
bility of the molecule in urine. As with calcium
tion that results from an abnormal connection
stones, increasing the urine volume and thus de-
of the ureters and bladder and is associated
creasing the cystine concentration by adequate
with urinary tract infections (UTIs) and renal
hydration helps prevent crystallization. Urine al-
damage. Vesicoureteral reux is diagnosed if
kalization with potassium bicarbonate also helps
radiopaque contrast material lls the bladder
increase the solubility of cystine. However, el-
and ascends into the ureters while the child is
evation of the urine pH can increase the risk of
voiding. Reux should be sought in children
calcium phosphate stone formation, especially
with unexplained failure of renal growth, renal
in the setting of low urine volume. The patients
scarring, or febrile UTI.
new stone is likely to be a calcium stone be-
Answer A is incorrect. A CT scan without con- cause it is radio-opaque and urinary calcium is
trast is helpful in the evaluation of stones. This elevated.
patient does not exhibit symptoms of renal
Answer A is incorrect. Allopurinol is a xan-
colic, so stones are unlikely, and thus a non-
thine oxidase inhibitor used for the treatment
contrast CT has little utility.
of hyperuricemia and uric acid stones associ-
Answer B is incorrect. Cystoscopy allows di- ated with gout. It effectively lowers urinary uric
rect visualization of the bladder. This modality acid levels and is helpful in preventing gouty
can help dene some pediatric urologic issues ares. It plays no role in the treatment of stones
such as ureteroceles, foreign objects, tumors, due to cystinuria.
posterior urethral valves in boys, and vesicoen-
Answer B is incorrect. Enalapril is an angio-
teric stula, but does not allow investigation of
tensin-converting enzyme inhibitor that works
the upper tract. The patient does not have any
to decrease the glomerular hydrostatic pressure
indication for cystoscopy.
and relieve hypertension. It plays no role in
Answer C is incorrect. An IVP is a very use- the treatment of cystine stone formation. Cap-
ful test to assess structural or anatomic defects topril, another drug in the same class as enal-
within the genitourinary tract. The contrast dye april, has been used in patients with cystine
is injected intravenously and its excretion into stones, with limited success. Its mechanism re-
the collecting system is captured with x-ray im- lies on the binding of a sulfhydryl group on the
aging. This modality can identify reux in some drug to cystine residues.
cases. However, the test does not necessarily
Answer C is incorrect. Loop diuretics increase
capture images of the patient voiding. The void-
the level of calcium in the urine and thus can
ing process produces pressure that might cause
contribute to the formation of calcium stones.
the urine (or contrast) to reux into the upper
There is no role, however, for loop diuretics in
tract, which would not be recognized if the
the treatment of cystinuria.
bladder was not contracting.
HIGH-YIELD SYSTEMS
458 Section I: Organ Systems Answers

Answer D is incorrect. Thiazide diuretics de- combined with poor uid intake. Initial man-
crease the level of calcium in the urine and are agement of the hyperglycemia includes insu-
often given to patients with recurrent calcium lin administration and volume repletion. With
nephrolithiasis. There is no role, however, for insulin administration, however, most patients
thiazides in the treatment of stone disease from should receive potassium, as insulin drives po-
cystinuria. tassium intracellularly. Water loss also depletes
body stores of potassium because of aldoster-
12. The correct answer is B. The most sensitive one-mediated sodium retention (and potassium
method for screening men for prostate cancer excretion). This patient is manifesting classic
employs both PSA and DRE. The use of PSA symptoms of moderate to severe hypokalemia.
helps detect cancer earlier, and its use is likely The ECG shows attened T waves, which is
responsible for the increase in detection of commonly seen in hypokalemia (as opposed
prostate cancer. DRE will help identify cancer to peaked T waves in hyperkalemia). In severe
Renal/Genitourinary

in men that might not have a PSA elevation. cases, one would also expect to see accid pa-
Screening beginning at age 50 years is consid- ralysis, hypercapnia, and rhabdomyolysis.
ered appropriate for most men. However, risk
Answer B is incorrect. In addition to insu-
factors such as having a rst-degree relative
lin, -agonists drive potassium intracellularly.
with prostate cancer or being African-Ameri-
Continuation of her home dose of albuterol
can may necessitate earlier screening begin-
would worsen, not lessen, her symptoms of hy-
ning at age 40 years.
pokalemia.
Answer A is incorrect. Cystoscopy is used in
Answer C is incorrect. Epinephrine, through
surveillance of bladder cancer and may be a
its -agonism, also drives potassium intracel-
part of a hematuria work-up. It does not have
lularly. This would worsen, not lessen, her hy-
a role in the screening or management of pros-
pokalemic symptoms.
tate cancer.
Answer D is incorrect. Although insulin drives
Answer C is incorrect. If the practitioner had
phosphate intracellularly, this patient shows no
only one test to screen for prostate cancer, PSA
signs of acute hypophosphatemia (hemolytic
would be the rst choice. PSA has relatively
anemia, increased susceptibility to infection, hy-
low specicity, as it can be elevated in settings
percapnia, encephalopathy, and heart failure).
other than prostate cancer. However, a combi-
nation of PSA and DRE provides a more thor- Answer E is incorrect. This patient is hyperna-
ough screening program. tremic in addition to being hypokalemic. Vol-
ume correction with hypertonic saline is not
Answer D is incorrect. Biopsy is the only way
indicated in this setting. Hypotonic saline, or in
to conrm prostate cancer, but is not an ap-
more severe cases 5% dextrose in water, are both
propriate screening test. Biopsy of the prostate
reasonable approaches to volume replacement.
should be undertaken if the PSA level is >4.0
ng/mL, if there is a signicant rise in the PSA,
14. The correct answer is C. Syndrome of inap-
or the DRE is abnormal.
propriate ADH secretion (SIADH) occurs in
Answer E is incorrect. A yearly history and about 50% of patients with small cell lung can-
physical examination would not be sufcient cer. This inappropriate production of vasopres-
for the screening of prostate cancer. The man sin does not always cause the overt symptoms
should, however, have a yearly history and phys- of hyponatremia that this scenario depicts. The
ical as part of his regular health maintenance. patient may compensate for the hyponatremia
by decreasing water intake, and thus increas-
13. The correct answer is A. This patient was ad- ing production of atrial natriuretic peptide.
mitted with hyperglycemia and dehydration. Tumors that secrete ADH include those with
This is a common presentation for diabetics neuroendocrine features, such as carcinoids,
with poor glycemic control, especially when non-small cell lung cancer, central nervous
HIGH-YIELD SYSTEMS
Chapter 15: Renal/Genitourinary Answers 459

system neoplasms, and cancers of the head and caused by hypovolemia (prerenal disease) or can
neck and genitourinary and gastrointestinal tracts. cause hypervolemic states (oliguria or anuria).
Answer A is incorrect. Insulinomas need to be Answer A is incorrect. The most common
considered when working up hypoglycemia. manifestation of acute adrenal insufciency
The hypoglycemia often occurs during fasting. is shock, but nonspecic symptoms may pre-
The patients normal glucose level makes this dominate in the initial manifestations of the
unlikely. Insulinomas are unrelated to hypona- syndrome. These include nausea and vomit-
tremia. ing, weakness, and abdominal pain. Changes
in arousal may also occur, ranging from fatigue
Answer B is incorrect. Multiple myeloma is
and lethargy to coma.
an aberrant proliferation of plasma cells in the
bone marrow, resulting in the clonal produc- Answer B is incorrect. Symptoms of hepatic
tion of a monoclonal immunoglobulin. Inva- failure include nausea, anorexia, jaundice, and

Renal/Genitourinary
sion of the bone can lead to osteolytic lesions, abdominal pain. Hepatic encephalopathy, if
osteopenia, and pathologic fractures. Multiple present, can lead to asterixis, confusion, and
myeloma is frequently accompanied with ane- even coma.
mia, hypercalcemia, and renal insufciency.
Answer C is incorrect. Signs and symptoms
This patient does not have any of these nd-
of opiate withdrawal are opposite to the effects
ings. Furthermore, multiple myeloma is not
often associated with opiate use and abuse.
associated with SIADH or hyponatremia.
These include anxiety, tremor, and u-like
Answer D is incorrect. Testicular embryonal symptoms such as rhinorrhea, sneezing, nau-
tumors can cause a paraneoplastic syndrome. sea, and vomiting. Other common symptoms
However, they do not cause SIADH. These include papillary dilation, lacrimation, yawn-
tumors can be a source of intact human chori- ing, and diarrhea.
onic gonadotropin, which can lead to elevated
Answer E is incorrect. Vitamin B12 deciency
steroidogenesis and aromatase activity. Elevated
is most often due to pernicious anemia, which
human chorionic gonadotropin levels can lead
causes macrocytic anemia. Vitamin B12 de-
to gynecomastia in men, while women are usu-
ciency may also lead to signs and symptoms
ally asymptomatic.
discernible on examination. Classically, mental
Answer E is incorrect. Thymic carcinoma sluggishness, a shiny tongue, and a shufing,
is not frequently associated with SIADH but broad-based gait are associated with pernicious
is the second most common cause of ectopic anemia-induced vitamin B12 deciency. Pa-
ACTH production. Fifteen percent of cases tients may also exhibit personality changes and
of ectopic ACTH production are attributed to memory impairment. Other nonspecic neu-
thymic carcinoma, while more than 50% of rologic ndings due to damage to the posterior
cases are attributed to small cell lung cancer. and lateral spinal columns include loss of dex-
terity, paresthesias, and weakness.
15. The correct answer is D. Perhaps because of the
crucial role of kidneys in ltering toxins from the 16. The correct answer is D. The patient has
plasma, renal dysfunction often presents with a BPH, which may produce obstructive (hesi-
wide range of signs and symptoms. As uremic tancy, weakened and intermittent urinary
toxins increase, patients will complain of pru- stream, urinary retention) or irritative (urge
ritus, nausea and vomiting, hiccups, and a me- incontinence, nocturia) symptoms. While
tallic taste in the mouth. Other signs include biopsy is not usually warranted in cases of
uremic fetor (urine-like odor to breath), altered BPH, DRE in this patient revealed a suspi-
sensorium, malaise, asterixis, and pericardial cious nodule requiring further evaluation. In
friction rub in cases of uremic pericarditis (the most cases of BPH, DRE reveals a uniformly
sound heard on physical examination in this enlarged, rubbery prostate without areas of in-
case). Signs and symptoms of altered volume sta- duration. The patient in this case had an area
tus may also be present, as renal failure can be
HIGH-YIELD SYSTEMS
460 Section I: Organ Systems Answers

of induration, suggestive of cancer. Addition- Answer A is incorrect. Acute alcohol con-


ally, while BPH may cause mildly elevated sumption is associated with signicant electro-
levels of PSA, prostate cancer can also result lyte disturbances such as hyponatremia and hy-
in elevated PSA levels. Therefore, biopsy is pokalemia. Of those alcoholics that progress to
warranted to distinguish between benign and cirrhosis, some can suffer from hepatic-induced
malignant disease. This patients biopsy re- renal failure (hepatorenal syndrome); however,
vealed a benign process. BPH can be man- there is no known association between alcohol
aged expectantly with medication, or with sur- consumption and nephrotic syndrome.
gery, depending on the severity of symptoms
Answer B is incorrect. Asthma is not associated
and associated ndings. This patient reported
with the development of nephrotic syndrome.
moderate to severe symptoms. Additionally,
his elevated creatinine level suggests that he Answer D is incorrect. Hypertensive nephro-
is in renal failure secondary to obstruction of sclerosis is a well-known phenomenon that
Renal/Genitourinary

the prostatic urethra. Therefore, transurethral typically leads to progressive renal impairment
resection of the prostate is the best treatment. and mild proteinuria. It is not associated with
frank nephrotic syndrome.
Answer A is incorrect. Brachytherapy would
be a reasonable option had the biopsy shown Answer E is incorrect. Minimal change dis-
more dysplasia and irregularity consistent with ease is by far the most common cause of neph-
carcinoma. Brachytherapy involves placing ra- rotic syndrome in children, but is actually less
dioactive seeds within the tumor, allowing for common in adults, in whom it is often more
localized radiation therapy. severe. Nephrotic syndrome is a triad of pro-
teinuria (>3 g/24 hr), hypoalbuminemia (<3.0
Answer B is incorrect. Finasteride is a
g/dL), and peripheral edema. Hyperlipidemia
5-reductase inhibitor that can be used to treat
and thrombotic disease are also seen fre-
BPH. The presence of renal disease (i.e., ele-
quently.
vated creatinine level) would encourage more
aggressive steps to prevent further renal dam- 18. The correct answer is E. This patient has suf-
age. fered rhabdomyolysis secondary to a crush injury
Answer C is incorrect. Radical retropubic and is at risk for pigment (myoglobin)-induced
prostatectomy would be a reasonable choice renal failure. Even without overt renal failure,
if the biopsy had shown prostatic carcinoma. myoglobinuria is common after crush injuries
However, this procedure is much too aggres- and is manifested by the nding of blood on
sive an approach for BPH. urine dipstick but the absence of RBCs on mic-
roscopic examination. The dipstick assay reacts
Answer E is incorrect. Watchful waiting would
with both hemoglobin and myoglobin, and thus
be acceptable if the man were less symptom-
will indicate the presence of blood in either case.
atic and did not have an elevated creatinine
But because the glomeruli are intact, no blood
level.
passes into the urine. Intracellular contents of
muscle cells leak out as circulation returns to
17. The correct answer is C. This patient exhib-
the damaged tissue. Lactate dehydrogenase, uric
its pedal and pulmonary edema, hypoalbu-
acid, phosphate, and potassium levels can in-
minemia, hyperlipidemia, decreased levels of
crease rapidly, and patients must be monitored
protein C, and proteinuria. This constellation
for signs of cardiac and systemic irregularities,
of ndings is typical of nephrotic syndrome.
such as peaked T waves and/or muscle weakness.
Other important abnormalities seen in this
In severe cases, dialysis is used to manage severe
disorder include hypercoagulability (second-
hyperkalemia.
ary to the decreased anticoagulant effect of
protein C), milky-appearing serum, and frothy Answer A is incorrect. A hallmark of rhab-
urine. The most common cause of nephrotic domyolysis is the elevation of creatine kinase
syndrome in adults is diabetes and ensuing dia- levels, which can climb >100,000 IU/L. In
betic nephropathy. the case of severe crush injuries, high levels
HIGH-YIELD SYSTEMS
Chapter 15: Renal/Genitourinary Answers 461

are all the more likely. Treatment strategies for sorption. The resulting decrease in calcium
acute rhabdomyolysis usually involve following excretion prevents stone formation.
plasma creatine kinase levels, aggressive hydra-
Answer E is incorrect. Thiazide diuretics
tion, and urine alkalinization.
lower calcium excretion in patients with hyper-
Answer B is incorrect. Serum lactate dehydro- calciuria, and there is some evidence that they
genase levels increase during rhabdomyolysis. are benecial in patients without identied hy-
percalciuria as well.
Answer C is incorrect. Homocysteine levels can
increase with end-stage renal disease (ESRD),
20. The correct answer is A. This patient has au-
most likely secondary to decreased glomerular
tosomal dominant polycystic kidney disease
ltration. This poses a potential risk to ESRD
(PCKD), characterized by the presence of en-
patients who are already at increased risk for
larged, often asymmetric kidneys with multiple
cardiovascular events because elevated homo-
cysts, due to defective polycystin proteins which

Renal/Genitourinary
cysteine levels are associated with increased risk
impair epithelial cell maturation. The mean
of coronary events. Although this patient may
age at diagnosis is 43 years and penetrance is
have some renal impairment, no evidence is
thought to be close to 100% if patients live long
provided that he suffers from ESRD.
enough (making age the factor that best corre-
Answer D is incorrect. Troponin I is specic lates with penetrance). Hypertension develops
to cardiac tissue and levels are unlikely to be prior to renal failure, which is usually present by
elevated in a crush injury that does not involve age 60 years. Other complications include in-
direct damage to the myocardium. fection, cyst hemorrhage and rupture, renal cal-
culi, and cerebral berry aneurysms (the cause of
19. The correct answer is A. This patient has a left death in approximately 10%).
ureteral stone. Eighty percent of patients with
Answer B is incorrect. A high-protein diet is
renal calculi form calcium stones, most often
not associated with increased penetrance of the
calcium oxalate. Various forms of diet modica-
autosomal dominant PCKD gene mutation;
tion can help prevent calculus formation. Even
however, after the diagnosis has been made, a
though hypercalciuria is commonly found in
low-protein diet may help to delay or limit pro-
patients who form stones, decreased calcium
gression of renal disease.
intake actually increases absorption and excre-
tion of oxalate, thereby enhancing stone forma- Answer C is incorrect. Frequent UTIs may
tion. Calcium restriction is therefore not recom- accelerate the decline in renal function; how-
mended for prevention of stones. ever, they are also not associated with the
penetrance of the gene mutation in PCKD.
Answer B is incorrect. Increased uid intake
Frequent infections are a consequence of pre-
increases the urine ow rate and decreases
existing cysts.
urine solute concentration, leading to de-
creased stone formation. Answer D is incorrect. Hepatic cysts in pa-
tients with PCKD are most often asymptom-
Answer C is incorrect. High protein intake
atic. In addition, there is no association be-
causes increases in urinary calcium, uric acid,
tween the presence of hepatic cysts and the
and citrate excretion. Although a low-protein
severity of kidney disease. Hepatic cysts may
diet has not been proven to prevent stone for-
often be found incidentally at autopsy.
mation, a high-protein diet is a risk factor for
stone formation in men. Answer E is incorrect. Though approximately
half of patients do not report a family history of
Answer D is incorrect. Reabsorption of so-
PCKD, the presence of a spontaneous (as op-
dium and water in the proximal tubule creates
posed to an inherited) mutation has no bearing
a gradient for calcium reabsorption. Limiting
on disease penetrance. In fact, the three spe-
sodium intake causes increased sodium reab-
cic mutations known to result in PCKD are
sorption and hence increased calcium reab-
phenotypically indistinguishable.
HIGH-YIELD SYSTEMS
462 Section I: Organ Systems Answers

21. The correct answer is D. Retention of BUN, Answer E is incorrect. Hypernatremia will not
or azotemia, is a laboratory nding with mul- prevent the correction of hypokalemia.
tiple etiologies. Azotemia is most often a mani-
Answer F is incorrect. Hyponatremia will not
festation of volume status (as in the case of
prevent the correction of hypokalemia.
contraction azotemia) and prerenal failure. As
blood volume or effective volume decreases,
23. The correct answer is E. This man is likely
proximal tubular reabsorption of sodium is ac-
having situational difculty with erections.
companied by reabsorption of urea such that
This illustrates the importance of a good doc-
the BUN:Cr ratio is often >20:1. There are, tor-patient relationship, allowing personal is-
however, several other important causes of el- sues to be explored. This man is having an
evation in BUN. These include processes that extramarital affair. New sexual partners, extra-
result in increased circulating amino acid lev- marital sexual relations, conicts with a part-
els such as gastrointestinal bleeding (from di- ner, questions about sexual identity, childhood
Renal/Genitourinary

rect digestion of blood protein). abuse, and fear of disease or pregnancy are all
Answer A is incorrect. Although alcoholism possible contributors to the situational erectile
can lead to poor nutrition and hydration, lead- dysfunction (ED) that this case illustrates. Psy-
ing to prerenal azotemia, the most likely cause chological factors may often coexist with or-
is volume depletion due to gastrointestinal ganic causes.
bleeding. Answer A is incorrect. Diabetes is one of many
Answer B is incorrect. Blood transfusion diseases that can cause or contribute to dif-
should actually act to decrease BUN by in- culty with erections. Other risk factors for ED
creasing intravascular volume, thus increasing include coronary artery disease, alcoholism,
the glomerular ltration rate. peripheral vascular disease, neurologic disor-
ders, smoking, hyperlipidemia, and low high-
Answer C is incorrect. Broad spectrum antibi-
density lipoprotein cholesterol. However, this
otics do not typically lead to azotemia. Some
man is capable of achieving an erection, thus
antibiotics, such as aminoglycosides, cause ne-
there is no reason to suspect diabetic neuropa-
phritis.
thy of the penis.
Answer E is incorrect. Urinalysis was heme
Answer B is incorrect. Medications are often
negative and did not reveal any casts; therefore,
implicated in men with ED. The history, how-
the patient most likely does not have a glo-
ever, states that the man was able to achieve an
merulonephropathy. With glomerulonephritis,
erection with self-stimulation. The difference
hematuria and RBC casts are usually seen.
in ability to achieve and maintain an erection
in different situations is not explained by medi-
22. The correct answer is D. When repleting a
cation.
patients potassium, always remember to re-
place his or her magnesium as well. Hypo- Answer C is incorrect. Peyronies disease is the
magnesemia can prevent correction of hy- development of a brous area within the cor-
pokalemia, even if the potassium is repleted pora cavernosa. This disease can contribute to
appropriately. The other electrolytes may also or cause ED, and occurs most often in men
be abnormal in this very ill child, but they are 4560 years old. The history specically states
not the most pertinent to make note of when that the man does not notice any curvature of
treating hypokalemia. the penis, and physical examination does not
mention a plaque, making this a very unlikely
Answer A is incorrect. Hypercalcemia will not
diagnosis.
prevent the correction of hypokalemia.
Answer D is incorrect. Prior surgeries should
Answer B is incorrect. Hypocalcemia will not
always be considered when evaluating a man
prevent the correction of hypokalemia.
for ED. Surgeries that may cause ED include
Answer C is incorrect. Hypermagnesemia will procedures on the prostate, penis, abdomen,
not prevent the correction of hypocalcemia. and pelvic vasculature. The varicocelectomy is
HIGH-YIELD SYSTEMS
Chapter 15: Renal/Genitourinary Answers 463

not likely to cause ED unless there was a bilat- had a low resistance path to follow and the
eral insult in the blood supply to the testicles, Jackson-Pratt drain was relatively urine free.
causing an anorchid state. The physical exami-
Answer A is incorrect. An exploratory laparo-
nation is not consistent with bilateral testicular
tomy is not needed in this situation. Further-
atrophy.
more, a urine leak such as this can usually be
managed expectantly. Drainage of the bladder
24. The correct answer is B. Blood at the urethral
with a catheter may aid the healing of the iat-
meatus suggests urethral injury. A rectal exam-
rogenic injury.
ination can evaluate for a high-riding, ballota-
ble prostate, which further suggests urethral Answer B is incorrect. Measuring the urinary
injury. After DRE, a more denitive test such amylase level would be useful if uid of pan-
as a urethrogram can be performed. creatic origin was suspected. The patient does
not have pancreatitis, or more specically, a

Renal/Genitourinary
Answer A is incorrect. Although an abdominal
pseudocyst, making the diagnosis of pancreatic
ultrasound may be used in the initial evalu-
uid unlikely. Also, given that the surgical pro-
ation of trauma, it will not provide any infor-
cedure was urologic in nature, a urine leak is
mation about either the pelvic fracture or the
more likely.
potential for urethral injury.
Answer D is incorrect. A serum amylase test
Answer C is incorrect. A Foley catheter should
has no role here. Serum amylase is used in di-
never be placed in a patient with suspected
agnosing pancreatitis. The patient has no epi-
urethral damage because the catheter could
gastric pain, and there is no reason to think
increase the urethral injury.
that an acute attack of pancreatitis would cause
Answer D is incorrect. An IVP does not ade- an increase in drain output.
quately assess urethral injury in the setting of
Answer E is incorrect. Serum hemoglobin
trauma and takes too long to obtain to be of
should be checked postoperatively if bleeding
use in emergent situations.
is suspected. However, the uid is described as
Answer E is incorrect. A urethrogram is the de- clear and not grossly bloody.
nitive study for suspected urethral injury. How-
ever, a rectal examination should be done rst 26. The correct answer is D. This woman pres-
as part of the rapid trauma survey to assess the ents with hypertension and decreased urine
position of the prostate, as well as to assess rectal output several days after starting a new medi-
tone and determine the presence of gross blood. cation. Her elevated creatinine makes the
diagnosis of acute renal failure likely. Two
25. The correct answer is C. The uid in the other factors in the stem point to a probable
Jackson-Pratt drain is urine. This is secondary etiology. Her BUN:Cr ratio is >20:1, and her
to a leak in the resected kidney. A urine leak fractional excretion of sodium (FENa) is <1%.
such as this should be considered when injury [FENa is calculated as follows: (urine sodium
to the ureter, kidney, or bladder is possible plasma creatinine) / (plasma sodium urine
(i.e., iatrogenically in obstetric/gynecologic or creatinine) 100.] These two indices sug-
urologic surgery). The suspicion that uid con- gest a prerenal cause of her renal failure. Of
tains urine can be easily conrmed by sending the medications listed, only enalapril is com-
the uid for a creatinine level. When this level monly associated with causing prerenal failure.
is higher than expected in the patients blood Enalapril, an angiotensin converting enzyme
(there is no reason to suspect an elevated se- inhibitor, inhibits angiotensin II. Angiotensin
rum creatinine in this patient), the suspicion II selectively constricts the renal efferent arte-
is conrmed. The timing also suggests that the riole. By inhibiting angiotensin II production,
patient has a urine leak because the Foley cath- enalapril leads to relative dilation of the effer-
eter has just been removed. The catheter had ent arteriole, resulting in a decrease in the glo-
been keeping the bladder empty so the urine merular ltration rate. Additionally, renal artery
HIGH-YIELD SYSTEMS
464 Section I: Organ Systems Answers

stenosis is a relative contraindication for taking hyponatremia (sodium <120 mEq/L). How-
angiotensin-converting enzyme inhibitors, as it ever, it is important to ensure the hypona-
causes hypoperfusion of the kidney, leading to tremia is real before initiating treatment.
a higher sensitivity to the decrease in glomeru-
Answer E is incorrect. Hypertonic saline can
lar ltration rate that these medications cause.
be used for the treatment of severe, symptom-
Answer A is incorrect. Aminoglycosides affect atic hyponatremia. In severe cases (seizures),
tubular cell function and have a high risk of correction should not exceed 1.52 mEq/hr,
causing nephrotoxicity. Often presenting with especially if hyponatremia has been long stand-
casts and proteinuria, aminoglycosides are most ing. Too-rapid correction of the hyponatremia
often associated with acute tubular necrosis. can result in central pontine myelinolysis. This
patient does not have true hyponatremia, so
Answer B is incorrect. Benztropine is an an-
correction is not warranted.
ticholinergic agent used commonly to combat
Renal/Genitourinary

some of the adverse effects of neuroleptic med-


28. The correct answer is C. The mass that is felt
ication. As an anticholinergic agent, it often
is likely the testis trapped in the inguinal canal.
causes urinary retention, and in severe cases
The testes are characteristically unable to be
can lead to obstructive (postrenal) uropathy.
manipulated into the scrotum, as seen in this
Answer C is incorrect. Although cimetidine is case. This disorder is also more likely to occur
commonly associated with elevations in serum in premature infants. The testes have a good
creatinine because it blocks tubular secretion chance of descending by 1 year of age. Thus,
of creatinine, most cimetidine-related renal dis- at this point, no correction is needed. If the
ease occurs because of interstitial nephritis. testes are not appreciated on physical examina-
tion, imaging with ultrasound or MRI can be
Answer E is incorrect. Hydroxyurea is a uric
useful. If MRI is not feasible, the presence of
acid derivative commonly used as maintenance
testicular tissue can be established by adminis-
chemotherapy in patients with sickle cell dis-
tering 1500 U human chorionic gonadotropin
ease. In patients taking hydroxyurea, there is
intramuscularly for 3 days. If testicular tissue
an increased risk of uric acid crystal formation
is present, there should be a signicant rise in
in the renal tubules, leading to an intrarenal
testosterone.
obstruction. However, this medication is not
known to cause prerenal renal failure. Answer A is incorrect. Bilateral anorchia
should be in the differential for nonpalpable
27. The correct answer is C. This patient has a gonads in this child. The mass that is felt, how-
dangerously low serum sodium value but no ever, is likely one of the testicles. If bilateral
apparent symptoms of hyponatremia (confu- anorchia existed, it would be manifest with de-
sion, stupor, seizures). Therefore, it is likely creased testosterone and increased gonadotro-
that this value is spurious. This is a common pin.
occurrence in patients who are phlebotomized
Answer B is incorrect. Virilizing forms of con-
in the same arm as the intravenous infusion,
genital adrenal hyperplasia can give females
proximal to the catheter site.
the phenotypic appearance of a man due to
Answer A is incorrect. There is no association fusion of the labial scrotal folds. The clitoris
between intravenous cefazolin and hypona- may also overdevelop, giving the appearance of
tremia. a penis. The karyotype excludes this as a diag-
Answer B is incorrect. Patient-controlled anal- nosis because these female pseudohermaphro-
gesia pumps are not a signicant source of free dites are 46 XX.
water and are unlikely to contribute to hypona- Answer D is incorrect. Retractile testis
tremia. (pseudocryptorchidism) can mimic true cryp-
Answer D is incorrect. Fluid restriction is the torchidism. This occurs from a hyperactive
appropriate treatment for mild, asymptomatic cremasteric reex that draws the testicle into
HIGH-YIELD SYSTEMS
Chapter 15: Renal/Genitourinary Answers 465

the inguinal canal. These testicles should be the management of acute dehydration. This
able to be pulled back into the scrotum. Cold patient is likely to be dehydrated, but simply
room temperature, genital manipulation, and administering uids will have little overall ef-
fear can cause the reex to occur. The typical fect on her condition.
age for this to occur is older than the clinical
Answer D is incorrect. The use of diuret-
scenario (56 years old).
ics actually reduces serum potassium levels.
Answer E is incorrect. Cryptorchid males are Therefore, potassium restriction in not rec-
more likely to have inguinal hernias. In fact, ommended when using diuretics. In fact, a
about 90% of these cryptorchid boys have her- potassium-rich diet or a second diuretic such
nias on the ipsilateral side due to the persis- as spironolactone or amiloride is often used
tently open processus vaginalis. The examina- in these patients to maintain serum potassium
tion, however, would not be consistent with a levels.

Renal/Genitourinary
strangulated inguinal hernia (i.e., the child is
feeding well and not in any obvious distress). 30. The correct answer is D. This patient has
CREST, which consists of Calcinosis, Ray-
29. The correct answer is E. This patient is suf- nauds phenomenon (presenting complaint),
fering from lithium-induced nephrogenic dia- Esophageal dysmotility, Sclerodactyly, and Te-
betes insipidus (DI) and demonstrates typical langiectasias. CREST is a type of scleroderma
elevations in plasma osmolality without com- that falls under the limited cutaneous systemic
pensatory elevations in urine osmolality. Indeed, sclerosis designation. The prognosis is depen-
normal renal function would yield a urine os- dent on the extent of skin involvement. The
molality closer to 700 mOsm/kg or higher in initial treatment of hypercalcemia is always
this situation. In the setting of prolonged lith- hydration, which can normalize the calcium
ium exposure, there is a 20% risk of permanent level without further treatment. The empiric
insensitivity to ADH. In most cases, however, use of acid-reducing agents, particularly proton
normal renal function returns with cessation of pump inhibitors, is generally recommended in
lithium therapy. Appropriate treatment of lith- order to prevent the development of esopha-
ium-induced DI includes the administration of geal strictures. Calcium channel blockers are
a thiazide diuretic, which decreases the delivery useful in decreasing the frequency of attacks of
of ltrate to the distal tubule and limits urine Raynauds phenomenon. The telangiectasias
volume. Other therapeutic options include non- create a primarily cosmetic problem that can
steroidal anti-inammatory drugs (to decrease be improved with green foundation make-up
ltration at the glomerulus), amiloride (to pre- or laser therapy for particularly large lesions.
vent accumulation of lithium in the collecting
Answer A is incorrect. Intravenous glucose am-
duct cells), and, in cases of known partial DI,
pules can be given for severe, symptomatic hy-
DDAVP (synthetic ADH).
poglycemia. Also, in the case of hyperkalemia,
Answer A is incorrect. ADH is effective for the glucose promotes insulin secretion, which sub-
diagnosis and treatment of central DI. How- sequently promotes potassium uptake. Glucose
ever, this patient has nephrogenic resistance to will not have an effect on hypercalcemia.
ADH, and thus is unlikely to respond to exog-
Answer B is incorrect. These are treatments
enous ADH administration.
for hyperkalemia. Insulin promotes cellular
Answer B is incorrect. Temporary uid restric- uptake of potassium, and bicarbonate increases
tion is a useful tool in the evaluation of polyu- the pH, which also shifts potassium into the
ria and hyponatremia. In the setting of elevated cell.
plasma sodium levels and likely volume deple-
Answer C is incorrect. This is a treatment for
tion, however, uid restriction is inappropriate
hyperkalemia. Calcium gluconate does not
and may worsen the hypernatremia.
correct the hyperkalemia; however, it is the
Answer C is incorrect. Intravenous uids are most expedient method of stabilizing the myo-
a necessary step in the resuscitation phase for cardium.
HIGH-YIELD SYSTEMS
466 Section I: Organ Systems Answers

Answer E is incorrect. Oral calcium channel RTA are typically hyporeninemic hypoaldoster-
blockers are used in the long-term manage- onism with diabetes mellitus and chronic in-
ment of Raynauds phenomenon to prevent re- terstitial nephritis. Treatment of hyperkalemia
current attacks. Oral calcium channel blockers with furosemide or potassium-binding agents
have no role in the acute treatment of hyper- will often restore ammonium production and
calcemia in this scenario. urine acidication.

31. The correct answer is C. Amphotericin B, a 32. The correct answer is A. The man described
broad-spectrum antifungal agent, is a drug that has Klinefelters syndrome. The classic triad
is infamous for numerous unpleasant side ef- is small, rm testes, azoospermia, and gyneco-
fects including type I (distal) RTA. There are mastia. The syndrome (47,XXY) is associated
newer (liposomal) preparations that are bet- with an increased likelihood of germ cell tu-
ter tolerated, but these newer preparations are mors in extragonadal sites. Breast cancer is also
Renal/Genitourinary

often very costly. The mnemonic that can be much more likely in these patients.
used to recall the most common adverse effects
Answer B is incorrect. Paraphimosis describes
of amphotericin, fever and chills, is shake and
a retracted foreskin that cannot be replaced to
bake. Type I RTA is a defect in distal hydro-
its normal position.
gen intercalated cells. Urinary pH is usually
>5.5, and serum potassium levels can be high Answer C is incorrect. Peyronies disease is a
or low. Other classic causes of type I RTA are disease in which a hard plaque develops on the
collagen vascular disease (Sjgrens syndrome), penis resulting in painful erection, curvature
cirrhosis, and nephrocalcinosis. of the penis, and poor erection quality. It is not
associated with Klinefelters syndrome.
Answer A is incorrect. Leukocytoclastic vascu-
litis is an inammatory reaction in small ves- Answer D is incorrect. Renal cell carcinoma is
sels in such organ systems as the skin (painless, not associated with Klinefelters syndrome.
nonblanching lesions) and kidney (glomeru- Answer E is incorrect. The risk factors for
lonephritis). Such skin physical examination transitional cell carcinoma include exposure to
ndings are not mentioned in the question dyes and smoking. There is no known relation-
stem. ship to Klinefelters syndrome.
Answer B is incorrect. Nephrogenic DI is a
disorder that can be caused by drugs that dam- 33. The correct answer is B. This child has auto-
age the concentrating ability of the renal col- somal recessive PCKD, also referred to as in-
lecting ducts. The usual offending agents are fantile polycystic kidney disease. The gene re-
lithium and demeclocycline. sponsible for this disease has been localized to
chromosome 6p21, though the exact mecha-
Answer D is incorrect. Type II RTA is a defect nism that causes this diseases clinical manifes-
of proximal bicarbonate reabsorption. This can tations is unknown. The kidneys in autosomal
lead to acidemia and increased bone turnover, recessive PCKD are very large and have a ra-
causing rickets and osteomalacia. The causes dial spoke pattern in the cortex seen on radiog-
of this disorder include hereditary disorders raphy. These cysts arise from the distal tubules
(e.g., cystinosis, tyrosinemia, glycogen storage and collecting ducts. Autosomal recessive
disease type 1, and Wilsons disease), Fanconis PCKD occurs in 1 in 20,000 births. Most cases
syndrome, and treatment with carbonic anhy- are diagnosed in the rst year of life. In addi-
drase inhibitors. tion to renal failure she also has caput medusa,
Answer E is incorrect. Type IV RTA is classi- a sign of the portal hypertension often caused
cally due to a deciency of or resistance to al- by the disease. Of note, this patients creati-
dosterone, which results in hyperkalemia and nine is 1.0 mg/dL. This might be normal in an
a consequent decrease in ammonium produc- adult, but in a patient with a smaller muscle
tion and urine acidication. The causes of this mass such as an infant, the creatinine is abnor-
mally high.
HIGH-YIELD SYSTEMS
Chapter 15: Renal/Genitourinary Answers 467

Answer A is incorrect. Autosomal dominant birefringent under polarized light. In patients


PCKD is the more common type of PCKD. prone to the disease, thiazide diuretics can lead
However, patients are usually asymptomatic to elevations in plasma urate levels and conse-
until the third decade of life. This child is too quent crystal precipitation in dependent joints.
young for the typical presentation of the auto- Hydrochlorothiazide increases plasma urate
somal dominant form. Autosomal dominant levels by blocking the secretion of urate at the
PCKD is known to cause hypertension and re- distal tubule.
nal failure, but portal hypertension is not regu-
Answer A is incorrect. -Blockers are associ-
larly seen.
ated with sexual dysfunction and bradycardia.
Answer C is incorrect. Bilateral megaureter Less commonly, they can cause hyperkalemia
would not cause the radial cystic pattern that is and increased airway reactivity. There is no as-
seen in autosomal recessive PCKD. A megau- sociation with hyperuricemia or gout.

Renal/Genitourinary
reter is dened as a dilated ureter with or with-
Answer B is incorrect. Some calcium channel
out dilation of the renal pelvices and calyces.
blockers have been associated with increased
Given the poor excretion of contrast in the
mortality when used immediately after acute
CT scan, the ureters are difcult to assess, but
myocardial infarction, although this associa-
there is no indication that this is the diagnosis.
tion is controversial. More commonly, these
Answer D is incorrect. Chronic hepatitis can medications are associated with headache, diz-
lead to scarring of the liver (cirrhosis) and ziness, and peripheral edema. There is no as-
eventually portal hypertension. Cirrhosis might sociation with hyperuricemia or gout.
explain the signs of portal hypertension with
Answer C is incorrect. Angiotensin-converting
normal liver function tests. At this late stage
enzyme inhibitors are important for the hyper-
the liver function test results are not elevated
tensive patient. Although they are not associ-
because the hepatocytes are damaged to the
ated with increased plasma urate levels, they
point where they cannot produce the enzymes
can lead to hyperkalemia and angioedema.
that when present are indicative of acute cell
damage. However, there is no direct relation- Answer D is incorrect. Hydralazine acts on
ship between chronic hepatitis and bilaterally smooth muscle cGMP to cause relaxation and
enlarged kidneys with cysts. vasodilatation. It has no direct action on urate.
Answer E is incorrect. Medullary cystic dis-
35. The correct answer is D. The patient has BPH
ease is an inherited disease that leads to small
and diabetes, which put him at an increased
kidneys with medullary cysts, not large kidneys.
risk for developing complicated UTIs. BPH
The lesions in medullary cystic kidney disease
is a common disorder that usually occurs in
are usually not found in the cortex, but rather
men older than 50 years and manifests with in-
appear in the medulla as the name suggests.
creased frequency of urination, nocturia, hesi-
The inheritance is in an autosomal dominant
tancy, urgency, and weak urinary stream. Many
pattern. Additionally, this disease typically pre-
men with BPH are asymptomatic or have only
sents in the third or fourth decade of life. The
mild symptoms, and may not require therapy.
symptoms are polyuria, anemia, and progres-
Additionally, many patients improve or stabi-
sive renal insufciency. There is no cure for
lize without therapy. Some patients with BPH
this disease, and unfortunately patients prog-
can develop acute urinary retention, UTIs due
ress to ESRD.
to chronic retention, hydronephrosis, and even
renal failure. Medical therapy includes (-adren-
34. The correct answer is E. This patient is suffer-
ergic antagonists and 5-reductase inhibitors.
ing from an acute attack of gout, characterized
Men who develop upper tract injury (e.g., hy-
by an exquisitely painful joint and signs of local
dronephrosis or renal dysfunction), or lower
inammation. Gout is caused by a precipita-
tract injury (e.g., urinary retention, recurrent
tion of uric acid crystals, which are negatively
infection, or bladder decompensation) may re-
HIGH-YIELD SYSTEMS
468 Section I: Organ Systems Answers

quire invasive therapy with transurethral resec- Answer A is incorrect. The primary chancre of
tion of the prostate. Untreated BPH can cause syphilis is usually a single painless papule that
acute urinary retention, recurrent UTIs, hydro- appears on the genitals. Inguinal lymphade-
nephrosis, and even renal failure. nopathy can be present but is generally bilat-
eral and painless. The treatment of choice is
Answer A is incorrect. Increased uid intake
penicillin.
is often recommended for patients with recur-
rent kidney stones; however, the patient has an Answer B is incorrect. While this patient is at
underlying anatomic abnormality (BPH) that high risk for HIV, the lesion is not consistent
puts him at greater risk for recurrent kidney with this diagnosis.
stones and pyelonephritis, so his BPH must be
Answer D is incorrect. Lymphogranuloma ve-
addressed in order to lower this risk.
nereum is caused by certain serotypes of Chla-
Answer B is incorrect. Lithotripsy would be mydia trachomatis. The primary genital lesion
Renal/Genitourinary

indicated if a renal calculus was indicated by is a small, painless vesicle that appears from
the diagnostic work-up. Shock wave lithotripsy days to weeks after infection that often goes un-
employs high-energy shock waves produced noticed. The lesion resolves in a few days, but
by an electrical discharge. The shock waves most presentations include fever, chills, myal-
are transmitted through water and directly fo- gias, and painful inguinal lymphadenopathy
cused onto a renal/ureteral stone. The change (two thirds are unilateral). The Frei test is now
in tissue density between the soft renal tissue obsolete, and a complement xation test as
and the hard stone causes a release of energy at well as a microimmunouorescence test have
the stones surface. This energy fragments the replaced it.
stone.
Answer E is incorrect. Biopsy can be useful for
Answer C is incorrect. Surgical intervention the diagnosis of human papillomavirus infec-
to remove a renal calculus would be indicated tion when it is not clinically obvious, and also
if there was a suspicion of an obstructing stone; for evaluating malignancies.
however, extracorporeal shock wave lithotripsy
Answer F is incorrect. The Tzanck smear is
is typically the treatment of choice for stones
used to detect the presence of multinucleated gi-
that do not pass on their own, although other
ant cells indicative of herpes simplex virus infec-
options include pyelolithotomy and percutane-
tion. Herpes simplex virus is the leading cause
ous nephrolithotomy.
of painful genital ulcers, but the lesions are usu-
Answer E is incorrect. Treatment with antibi- ally multiple small vesicles. Genital herpes may
otics would treat this episode, but would not be associated with reactive lymphadenopathy.
prevent future episodes from occurring.
37. The correct answer is A. This patient has in-
36. The correct answer is C. The lesion described trinsic renal failure due to AIN, most likely
is consistent with chancroid caused by Haemo- caused by the antibiotics he has received. Peni-
philus ducreyi, a deep painful lesion with an cillins, cephalosporins, and sulfonamides are
exudate. There are often multiple lesions and among the causes of AIN, and drugs account
it is associated with inguinal adenopathy in for 70% of cases of AIN. AIN is an inamma-
about half of patients. Chancroid ranks third tory process of the renal tubules similar to an
behind herpes simplex virus and syphilis as a allergic reaction. Findings include rash, fever,
cause of genital ulcers in the United States, and eosinophilia, as well as WBCs and WBC
but it should always be kept in the differential casts on urinalysis.
diagnosis. The diagnosis is usually made on
Answer B is incorrect. Glomerulonephritis
clinical evidence, but polymerase chain reac-
can present with laboratory values consistent
tion testing for the causative organism shows
with intrinsic renal failure; however, the uri-
promise. Treatment is with a single 1-g dose of
nalysis will show hematuria and RBC casts.
azithromycin given orally.
HIGH-YIELD SYSTEMS
Chapter 15: Renal/Genitourinary Answers 469

Glomerulonephritis is not associated with eo- 40. The correct answer is I. Minimal change dis-
sinophilia. ease is a type of nephrotic syndrome seen most
commonly in children, often after viral upper
Answer C is incorrect. This patient, who was
respiratory tract infection. Symptoms include
previously septic, has been adequately volume
edema and increased susceptibility to infec-
resuscitated and is hemodynamically stable, so
tion, and laboratory testing shows characteristic
there is no reason to suspect that he has con-
decreases in serum albumin and total protein,
tinuing renal failure secondary to hypovolemia.
as well as a high degree of proteinuria. Re-
Furthermore, hypovolemic renal failure would
sponse to steroids among children is excellent
have a prerenal picture with a BUN:Cr ratio
and can be diagnostic of the condition.
>20:1.
Answer A is incorrect. Alports syndrome is a
Answer D is incorrect. There is no evidence
hereditary nephritic syndrome that presents
that this patient has urinary obstruction with

Renal/Genitourinary
in males 520 years old. It is associated with
postrenal failure secondary to kidney stones.
painless hematuria and nerve deafness. On
Answer E is incorrect. Pyelonephritis can electron microscopy, the glomerular basement
present with fever, as well as WBCs and WBC membrane is split.
casts, on urinalysis, especially in a patient
Answer B is incorrect. Diabetic nephropathy
with recurrent UTI. However, this patient has
can present with either diffuse hyalinization
negative urine cultures and no left shift on
or nodular glomerulosclerosis (Kimmelstiel-
complete blood count. Additionally, rash and
Wilson lesions). It is the result of long-stand-
eosinophilia would not be explained by pyelo-
ing, poorly controlled diabetes mellitus, and
nephritis.
presents with gross proteinuria and renal fail-
ure, as well as increased mesangial matrix on
Questions 38, 39, and 40 light microscopy.

38. The correct answer is J. Postinfectious glomer- Answer C is incorrect. Focal segmental glo-
ulonephritis most often occurs 13 weeks after merulosclerosis causes a nephrotic syndrome
group A -hemolytic streptococcal infection that is usually idiopathic but that can be re-
and presents with a nephritic picture, includ- lated to heroin abuse and HIV infection.
ing edema, hypertension, and oliguria. Urinal- Answer D is incorrect. Goodpastures syn-
ysis shows red cells and nonnephrotic range drome is a nephritic disease that presents
proteinuria. Treatment: symptomatic therapy. mainly in men in their mid-20s. It presents
The major goal is to control edema and blood with hemoptysis and hematuria as a result of
pressure, so water and salt might be restricted. antibodies against type IV collagen in the base-
Dialysis would only be used in the event of ment membranes, which stains in a linear
life-threatening hyperkalemia and clinical fashion on immunouorescence.
manifestations of uremia.
Answer F is incorrect. Lupus nephritis can
39. The correct answer is E. IgA nephropathy is present with either a nephritic or a nephrotic
most commonly seen in young men and chil- picture in the setting of systemic lupus ery-
dren, and is often associated with upper respi- thematous. Subendothelial deposits are seen
ratory infection, gastrointestinal symptoms, on light microscopy.
or u-like illness. The presenting symptom is Answer G is incorrect. Membranoprolifera-
red- or cola-colored urine, and the characteris- tive nephropathy has a distinctive tram-track,
tic biopsy, which is the standard for diagnosis, double-layered basement membrane. It is a
shows focal glomerulonephritis with mesangial slowly progressive nephritic syndrome that is
IgA deposition. Again, aggressive treatment of usually idiopathic.
blood pressure is key, often with use of an an-
giotensin-converting enzyme inhibitor. Answer H is incorrect. Membranous nephrop-
athy is the most common cause of a nephritic
syndrome. It is an immune complex disease
HIGH-YIELD SYSTEMS
470 Section I: Organ Systems Answers

that presents with a spike and dome appear- inammatory disease. Amyloid deposits are
ance of IgG and C3 in the glomerular base- seen in a Congo red or apple green stain in ab-
ment membrane. Its causes are unknown, but dominal fat, the glomerulus, and many other
it has been shown to be associated with ma- tissues.
laria, hepatitis B virus, syphilis, and gold treat-
Answer L is incorrect. Wegeners granuloma-
ment.
tosis is a vasculitis that affects mainly the lungs
Answer K is incorrect. Renal amyloidosis is and kidneys. It presents with hemoptysis, he-
a disease that is secondary to the deposition maturia, a nephritic picture, and positive tests
of amyloid in the glomerulus. It is most fre- for circulating anti-neutrophilic cytoplasmic
quently seen in the context of malignancy or antibodies.
Renal/Genitourinary
SECTION II

Full-Length
Examinations

Test Block 1

Test Block 2

Test Block 3

Test Block 4

Test Block 5

Test Block 6

Test Block 7

Test Block 8

471
This page intentionally left blank
Test Block 1

473
474 Section II: Full-Length Examinations Questions

Q U E ST I O N S

1. A 60-year-old white man presents to his pri- and is likely to regress with residual scar-
mary care physician for an annual examina- ring if untreated
tion. He has a history of hypertension that is
controlled with diet and medication. He has 2. A 41-year-old man presents to his physician be-
felt well. He has, however, noticed for the past cause of recurrent suicidal thoughts. He has
10 months a nonresolving small bump on his been depressed for >6 months and can nd no
face that occasionally bleeds when he scratches interest in life anymore. He has difculty sleep-
it. He is a banker and does not spend much ing and constantly feels fatigued. He feels
time outdoors, but had frequent sunburns as a guilty about his role as a father and husband
boy. Examination reveals a 0.6-cm esh-col- and feels things might be better off without
ored papule with telangiectatic vessels, a rolled me around. Which of the following is the phy-
edge, and slight central crusting located on the sicians most appropriate response?
cheek (see image). Which of the following is (A) Do you think you can do it?
the expected progression of this lesion? (B) Everyone feels this way sometimes.
(C) Have you ever killed anyone before?
(D) Have you thought of a plan, or a way to
commit suicide?
(E) How would you feel if your wife commit-
ted suicide?
(F) Why do you want to die?

3. A patient comes to the emergency department


complaining of acute-onset tearing pain in his
back. X-ray of the chest reveals a widened me-
diastinum. Which of the following is the most
FULL-LENGTH EXAMS

common etiology of this condition?


(A) Atherosclerosis
(B) Hypertension
(C) Protein C deciency
Reproduced, with permission, from Wolff K, Johnson RA, (D) Rheumatic heart disease
Surmond D. Fitzpatricks Color Atlas & Synopsis of Clini- (E) Trauma
cal Dermatology, 5th edition. New York: McGraw-Hill,
2005: Figure 11-17. 4. A 12-year-old boy is referred to his physician
(A) An initial radial growth phase that is fol- because of blinking too much, which his
lowed by a vertical growth phase; prognosis mother thinks she rst noted >1 year ago. The
is signicantly related to depth of the le- boy states that he does not notice that it is oc-
sion at time of excision curring unless someone tells him about it.
(B) Good prognosis if excised early, but a pro- When he is nervous, it seems to worsen. He is
pensity for lymphatogenous metastasis embarrassed about it and wants to control it.
mandates frequent follow-up after excision What class of medications has been shown to
(C) Occurs exclusively within the epidermis worsen this condition?
and has no malignant potential (A) Antipsychotic
Test Block 1

(D) Slow growth with destruction of local tis- (B) Depressants


sue that may extend along nerves to pene- (C) Hallucinogens
trate into the central nervous system (D) Selective serotonin reuptake inhibitors
(E) Will remain stable for an indenite period, (E) Stimulants
Test Block 1 Questions 475

5. A 42-year-old white mother of two presents to vers that occur in the evenings and resolve by
the emergency department with severe right morning for several weeks. Physical examina-
upper quadrant (RUQ) abdominal pain. She tion reveals painless cervical lymphadenopa-
says she has had episodes like this before, but thy. After the patient is stabilized, what is the
has never sought medical attention, and this most appropriate next step in management?
episode is more severe than usual. She also has
(A) Bronchoscopy
nausea and vomiting. On physical examination
(B) HIV testing
she is febrile to 38.2C (100.8F), obese, and
(C) PPD testing
has tenderness to palpation in the RUQ. A
(D) Selective serotonin reuptake inhibitor
RUQ ultrasound reveals stones in her gallblad-
therapy
der. Laboratory test results are:
(E) X-ray of the chest
WBC count: 13,000/mm3
Hematocrit: 36% 7. A 57-year-old man who underwent kidney
Na+: 142 mEq/L transplantation 1 month earlier presents to the
K+: 4.5 mEq/L emergency department with a 2-day history of
Cl: 103 mEq/L fever and abdominal pain. He has a fever of
Ca2+: 9.4 mg/dL 38.5C (101.3F). He complains of rebound
CO2: 24 mmol/L tenderness during his abdominal examination.
Blood urea nitrogen: 12 mg/dL Which of the following is the most likely caus-
Creatinine: 0.7 mg/dL ative organism?
Glucose: 100 mg/dL (A) Enterococci
Total bilirubin: 3.8 mg/dL (B) Pneumocystis jiroveci
Total protein: 7.7 g/dL (C) Pseudomonas aeruginosa
Albumin: 4.5 g/dL (D) Staphylococcus aureus
Aspartate aminotransferase: 130 U/L (E) Streptococcus pneumoniae
Alanine aminotransferase: 150 U/L
Alkaline phosphatase: 300 U/L 8. Some vaccines cannot be administered to preg-
Amylase: <30 U/L nant women or immunosuppressed patients

FULL-LENGTH EXAMS
Lipase: 8 U/L because they contain live virus. Several vac-
Which of the following is most likely responsi- cines used today are safe for this population be-
ble for her symptoms and laboratory values? cause they contain inactivated virus. Which of
the following vaccines contains inactivated vi-
(A) A stone obstructing her common bile duct
rus?
(B) A stone obstructing her common hepatic
duct (A) Intramuscular inuenza vaccine
(C) A stone obstructing her cystic duct (B) Mumps-measles-rubella vaccine
(D) A stone obstructing her hepatoduodenal (C) Oral polio virus vaccine
ampulla (of Vater) (D) Rotavirus vaccine
(E) Stones in her gallbladder (E) Varicella vaccine

6. A 23-year-old female nursing student presents


to the emergency department with shortness of
breath, racing heart, and a feeling that
something terrible is happening. She states
that this episode began 30 minutes earlier
when she awoke from sleep drenched in sweat.
Test Block 1

She has had several similar episodes in the


past, but they resolved without intervention.
She states that she has been very anxious about
her health lately, and has had intermittent fe-
476 Section II: Full-Length Examinations Questions

9. A 65-year-old man recently started taking an hematuria, or vaginal bleeding. She has been
antihyperglycemic medication because of ele- sexually active with one partner and uses bar-
vated blood glucose and glycosylated hemoglo- rier protection most of the time. A urine
bin levels. During his next ofce visit, he is -human chorionic gonadotropin screen is
back under good glycemic control but now negative. On examination she has a moderate
complains of nausea and mild abdominal pain amount of yellow discharge. Results of Giemsa
since he started the medication. His blood staining of McCoy cells are shown in the im-
pressure is 102/76 mm Hg, heart rate is 112/ age. Which of the following is the most effec-
min, and respiratory rate is 24/min. Laboratory tive oral pharmacotherapy for this disorder?
studies show:
Na+: 134 mEq/L
K+: 4.7 mEq/L
Cl: 101 mEq/L
HCO3: 7.1 mEq/L
Blood urea nitrogen: 16 mg/dL
Creatinine: 1.4 mg/dL
Partial carbon dioxide pressure: 16 mm Hg
pH: 7.24
Which antihyperglycemic medication most
likely caused this patients acute presentation?
(A) Acarbose
(B) Glipizide
(C) Metformin
(D) Repaglinide Reproduced, with permission, from Tintinalli JE, Kelen
(E) Rosiglitazone GD, Stapczynski S, Ma OJ, Cline DM. Tintinallis Emer-
gency Medicine: A Comprehensive Study Guide, 6th edi-
tion. New York: McGraw-Hill, 2004: Figure 108-3.
10. A 17-year-old boy with mild hemophilia type A
FULL-LENGTH EXAMS

presents to the emergency department with (A) Azithromycin


right lower quadrant abdominal pain. He also (B) Fluconazole
reports nausea, vomiting, and anorexia. On (C) Metronidazole
physical examination the patient has positive (D) Penicillin
psoas and obturator signs. Ultrasound reveals a (E) Vancomycin
nonperforated appendix with a wall thickness
of 5 mm. The patient is diagnosed with acute 12. A 55-year-old woman with a medical history of
appendicitis. Work-up shows his coagulation a prior myocardial infarction and hypertension
factor VIII level is slightly lower than normal. is suffering from unilateral throbbing head-
Which of the following should this patient re- aches preceded by a 10-minute visual aura.
ceive prior to undergoing an appendectomy to The headaches last between 10 and 24 hours
reduce the risk of hemorrhage? and are accompanied by photophobia. She has
tried nonsteroidal anti-inammatory drugs to
(A) Aspirin stop the headaches once they occur, but they
(B) Desmopressin are not effective. Which medication is the next
(C) Packed RBCs logical abortive therapy to try in this patient?
(D) Platelets
(E) Whole blood (A) -Blockers
Test Block 1

(B) Acetaminophen
11. An 18-year-old G2P2 woman presents to her (C) Ergotamine
gynecologists ofce complaining of several (D) Triptans
days of vaginal itching and discomfort on uri- (E) Valproate
nation. She denies any recent fever, back pain,
Test Block 1 Questions 477

13. A 63-year-old man is brought into the emer- studies are performed, including a complete
gency department by his wife after he com- blood cell count, liver enzyme measurements,
plains of acute-onset chest pain, like an ele- and hepatitis and syphilis serologic studies, and
phant is sitting on my chest. After the patient PPD testing. The patients CD4+ cell count is
is stabilized, his electrocardiogram is noted to 322/mm. Three days later the clinic nurse notes
have ST-segment elevations in leads II, III, and an area of slight induration measuring 7 mm in
aVF. This patient is most likely suffering an diameter at the site of PPD testing. Which of the
acute myocardial infarction affecting which following is the best course of action?
part of the heart?
(A) Admit to the hospital in respiratory isola-
(A) Anterior tion; culture sputum for acid-fast bacilli
(B) Indeterminate (B) Continue to monitor serial PPD tests; do
(C) Inferior not administer treatment at this time
(D) Lateral (C) Obtain an x-ray of the chest and do not ad-
(E) Posterior minister treatment if the radiograph is
clear
14. A 34-year-old man with Crohns disease pre- (D) Obtain an x-ray of the chest and prescribe
sents to the emergency department because of a 9-month course of isoniazid if the radio-
2 days of increasing abdominal pain. The pain graph is clear
is diffuse, although most intense in the peri- (E) Start oral trimethoprim-sulfamethoxazole
umbilical region. On questioning he says he
has not had a bowel movement since the onset 16. A 77-year-old white man is brought to the
of the pain, nor has he vomited. On initial clinic by his daughter, who says that her father
physical examination his abdomen is very dis- is having trouble remembering important fam-
tended and tender to palpation. Thirty minutes ily occasions and difculty managing his -
later his clinical condition has worsened. nances. These behaviors have progressively
Which of the following additional ndings worsened in the past year, but her father denies
would best indicate the need for emergent sur- that anything is wrong. He has no history of de-
gery? pression, anxiety, or head trauma. Neurologic

FULL-LENGTH EXAMS
examination is normal. His medical history is
(A) Absence of bowel sounds
signicant for type 2 diabetes mellitus diag-
(B) Absence of atus
nosed 20 years ago and atrial brillation. An
(C) Bilious vomiting
uncle was diagnosed with Parkinsons disease at
(D) Diffuse rebound tenderness
the age of 60 years. Which of the following is
(E) Metabolic alkalosis
the most likely diagnosis?
15. A 35-year-old man who was recently diagnosed (A) Alzheimers disease
with HIV infection presents for his initial evalu- (B) Delirium
ation at an HIV clinic. The patient is afebrile (C) Dementia with Lewy body
and his vital signs are within normal limits. He (D) Progressive supranuclear palsy
has no complaints and his physical examination (E) Vascular dementia
is unremarkable. A series of baseline laboratory
Test Block 1
478 Section II: Full-Length Examinations Questions

17. A 56-year-old woman with type 1 diabetes mel- previously been counseled to quit smoking and
litus presents to the emergency department has successfully done so, but asks about other
with right-sided ank pain that radiates to the treatment options to relieve his symptoms. The
lower abdomen and groin. The patient notes rst-line treatment for his leg pain works to al-
chills, nausea, and one episode of vomiting this leviate symptoms by which of the following
morning, as well as increased frequency of uri- mechanisms?
nation. Her temperature is 38.8C (101.8F),
(A) Developing collateral circulation in ex-
blood pressure is 130/82 mm Hg, heart rate is
tremities
92/min, respiratory rate is 14/min, and blood
(B) Dissolving thrombus
glucose is 147 mg/dL. There is right-sided cos-
(C) Inhibiting -adrenergic receptors
tovertebral angle tenderness. Urinalysis is nota-
(D) Inhibiting HMG-CoA reductase
ble for pyuria and bacteriuria. Which of the
(E) Irreversibly inhibiting cyclooxygenase
following is the most likely pathogen involved
in this patients diagnosis?
20. Two months after undergoing a pneumonec-
(A) Enterobacter species tomy for a lung nodule, a 54-year-old male
(B) Escherichia coli smoker presents to his physician complaining
(C) Klebsiella species of productive cough, a fever of 38.2C
(D) Proteus species (100.8F), and chills for 2 days. A thorough re-
(E) Pseudomonas species view of systems reveals a 2.7-kg (6-lb) weight
(F) Staphylococcus species loss over the past 2 months and intermittent
hip pain. Basic laboratory studies show:
18. A 6-month-old girl is brought to her pediatri-
Na+: 120 mEq/L
cian because of increasing frequency of stools
K+: 4.2 mEq/L
over the past 2 weeks. Her mother describes
Cl: 104 mEq/L
smelly, loose stools that oat in water and re-
HCO3: 25 mEq/L
ports her daughter has a history of atulence
Blood urea nitrogen: 24 mg/dL
and fussiness after feeding. Birth history is sig-
Creatinine: .09 mg/dL
nicant for failure to pass meconium in the
WBC count: 14,400/mm3
FULL-LENGTH EXAMS

rst 24 hours after birth. She is feeding with


Hemoglobin: 35.6 g/dL
formula and occasional rice cereal. Physical
Hematocrit: 14.3%
examination is signicant for weight below the
Platelet count: 256,000/mm3
fth percentile. Stool is guaiac negative, and
cultures are negative. Which of the following What is the most likely cause of this patients
would conrm the most likely diagnosis? hyponatremia?
(A) Endoscopic biopsy of the small intestine (A) Pneumonia
(B) Stool qualitative fat analysis (B) Postsurgical atelectasis
(C) Sweat chloride test (C) Recent head trauma
(D) Ultrasound of the pancreas (D) Recurrence of small cell lung carcinoma
(E) X-ray of the abdomen and pelvis with con- (E) Renal failure
trast
21. A 38-year-old African-American woman pre-
19. An 80-year-old man presents to his primary sents to her primary physician because of
care physician with recurrent complaints of cough and difculty breathing. Physical exami-
calf pain. He says the pain worsens with ambu- nation is signicant for hepatomegaly and dif-
lation and improves on rest. The patient has a fuse lymphadenopathy. X-ray of the chest is
Test Block 1

past medical history of hyperlipidemia, angina, shown in the image. Which of the following is
and hypertension, treated with statins, nitro- the most appropriate next step for conrming
glycerin as needed for chest pain, and lisino- the diagnosis?
pril, respectively. On physical examination the
patient has 1+ distal pulses, no edema, and an
ankle-brachial index of 0.70. The patient had
Test Block 1 Questions 479

(A) Call a family meeting to see what the pa-


tients two daughters want to do; try to
reach a consensus about whether to extu-
bate
(B) Call the ethics committee to help decide
how to treat the patient from this point for-
ward
(C) Explain that there is little hope for recov-
ery given the patients multiple medical
problems and difcult surgery; therefore,
the team can decide to extubate the pa-
tient and withdraw care
(D) Explain to the wife that although they un-
derstand how difcult it is for her, her hus-
bands wishes as stated in his living will
Reproduced, with permission, from USMLERx.com.
must be respected, despite the fact that he
(A) Liver biopsy cannot state his wishes himself at this point
(B) Open lung biopsy (E) Extubate the patient without any further
(C) Scalene node biopsy discussion
(D) Serum angiotensin-converting enzyme
level measurements 23. A 60-year-old man presents to the emergency
(E) Transbronchial lung biopsy department with severe headache and diapho-
resis that has been occurring intermittently for
22. A 75-year-old man with a history of coronary the previous 24 hours. He also notes occasional
artery disease, congestive heart failure, type 2 palpitations. The patients blood pressure is
diabetes mellitus, and chronic obstructive pul- 220/110 mm Hg, and he has papilledema on
monary disease who is status post-myocardial funduscopic examination. Intravenous phen-
infarction is scheduled for thoracotomy for tolamine is effective at improving his blood
non-small cell lung cancer. Before his surgery, pressure. Subsequent measurement of 24-hour

FULL-LENGTH EXAMS
he has a discussion with his long-time physi- urinary catecholamine metabolites yields the
cian and his wife about the risks involved in following results:
the procedure, given his extensive comorbidi-
Urine catecholamines: 260 g
ties. He decides that he does not want to live
Norepinephrine: 225 g
with lung cancer and wants to proceed with Epinephrine: 1.6 g
the surgery, but he states that if the surgery is Metanephrine: 3.0 mg
complicated and he cannot be extubated, he Vanillylmandelic acid: 12 mg
wants to have care withdrawn and die with dig-
nity after a week. He signs a living will to this Which of the following is most likely to also be
effect. His surgery is, in fact, complicated with found in this patient?
major episodes of hypoxia and hypotension, (A) Insulinoma
and he remains intubated 1 week post surgery. (B) Medullary thyroid cancer
At this point his wife, unwilling to let go, ap- (C) Pancreatic adenocarcinoma
proaches the team and says, I know he will (D) Papillary thyroid carcinoma
pull through, please do not give up on him (E) Pituitary adenoma
yet. What is the most appropriate course of ac-
tion for the physicians?
Test Block 1
480 Section II: Full-Length Examinations Questions

24. A 48-year-old man presents to the emergency (A) Attributable risk


department with a 4-hour history of new-onset (B) Causation
shortness of breath, difculty breathing while (C) Incidence
supine, and nonproductive cough. Medical (D) Prevalence
history reveals he had rheumatic fever as a (E) Relative risk
child. Vital signs are within normal limits, and
oxygen saturation is 97% on room air. On ex- 27. A 15-year-old girl presents to a urologist for a
amination there is a low-pitched diastolic rum- follow-up visit. As a young child, she had fre-
ble as well as an opening snap, both best aus- quent urinary tract infections and an episode
cultated at the apex. Rales and musical rhonchi of pyelonephritis. The subsequent workup
are auscultated on inspiration in both lung leads to the diagnosis of bilateral grade 4 vesi-
elds. ECG shows an irregularly irregular coureteral reux (VUR). The patient has been
rhythm. Creatine kinase-myocardial bound on and off antibiotic prophylaxis for several
fraction, troponin T, and troponin I tests 6 years, with her last breakthrough UTI when
hours after the onset of symptoms are negative. she was 13 years old. She notes that she has not
The thyroid-stimulating hormone level is nor- had any leaking or loss of urine during the day
mal. Which of the following is the most likely or at night. She has just become sexually active
cause of the patients pulmonary edema? with her boyfriend and states that she has been
using condoms. Her temperature is 36.6C
(A) Acute aortic insufciency
(97.8F) and blood pressure is 120/85 mm Hg.
(B) Acute mitral regurgitation
Her abdomen is soft and there is no tenderness
(C) Mitral stenosis
to palpation. The patient is a well-developed,
(D) Myocardial ischemia
Tanner stage IV girl. Her blood urea nitrogen
(E) Thyroid disease
is 15 mg/dL and creatinine is 1.5 mg/dL, and
are unchanged from the past two visits. Today,
25. A 29-year-old man is brought into the emer-
a voiding cystourethrogram shows left-sided
gency department by his sister, who indicates
grade 3 VUR and right-sided grade 4 VUR,
that the patient has been extremely agitated
with no residual contrast postvoid. At this
and has not moved his head for nearly an hour.
point, what is the most appropriate step in
FULL-LENGTH EXAMS

She notes that he currently lives at home with


management?
her, after a month-long stay in a psychiatry fa-
cility for schizophrenia. She provides a list of (A) 99mTechnetium dimercaptosuccinic acid
his medications, which she updated this morn- scan
ing after his psychiatrist increased the dosage (B) Bilateral ureteral reimplant
of one of his medications. Which of the follow- (C) Continue antibiotic prophylaxis
ing medications is the most appropriate man- (D) Repeat voiding cystourethrogram yearly
agement at this point? (E) Video urodynamics
(A) Alprazolam
28. A child is brought to his pediatrician for evalu-
(B) Diphenhydramine
ation of a rash (see image). The child has had
(C) Haloperidol
recurrent episodes of this skin irritation, which
(D) Muscle relaxants
his mother notes to be nonpainful and nonpru-
(E) Sertraline
ritic. On physical examination the lesions are
weepy and ooze uid that forms golden-
26. The state universitys center for biological re-
colored crusts over the affected skin. The pa-
search is conducting a cross-sectional survey to
tient has mild local lymphadenopathy but no
determine the relationship between fruit con-
constitutional symptoms or rashes elsewhere
Test Block 1

sumption and colorectal cancer. Participants


on his body. Which is the most appropriate
are selected randomly and contacted by tele-
treatment for this patient?
phone. Which of the following parameters can
be determined from this type of study?
Test Block 1 Questions 481

30. A 42-year-old woman comes to her physician


because she has been having stomachaches
and headaches nearly every day for the past 10
months, as well as occasional dry throat and
palpitations. She has been feeling restless, has
difculty concentrating at work, and worries
constantly about her relationships, health, and
nances, despite having no signicant prob-
lems in these areas. She denies drug or alcohol
abuse. Her temperature is 36.9C (98.5F),
blood pressure is 132/80 mm Hg, pulse is 106/
min, and respiratory rate 16/min. Examination
indicates an alert and oriented patient with no
memory decits and an unremarkable physical
examination. Thyroid-stimulating hormone
Reproduced, with permission, from Wolff K, Johnson RA, levels and urine toxicology indicate no abnor-
Surmond D. Fitzpatricks Color Atlas & Synopsis of Clini- malities, and an ECG shows sinus tachycardia.
cal Dermatology, 5th edition. New York: McGraw-Hill, Which of the following is the most likely diag-
2005: Figure 22-10. nosis?
(A) Acyclovir (A) Acute stress disorder
(B) Change facial soaps (B) Adjustment disorder with anxiety
(C) Hydrocortisone cream (C) Generalized anxiety disorder
(D) Intravenous vancomycin (D) Hypochondriasis
(E) Topical mupirocin (E) Malingering
(F) Tretinoin cream
31. An unconscious 24-year-old man is brought to
29. A 39-year-old obese African-American man a rural urgent care clinic by his family. The pa-
presents with several episodes of weakness and tient is cyanotic and has clearly swollen lips

FULL-LENGTH EXAMS
tingling in his right forearm. This morning he and tongue. His mother states that he is ex-
became extremely concerned when he was un- tremely allergic to peanuts and the problem
able to grip his coffee cup and could not open developed suddenly at a restaurant next door.
the car door. He is an electrician and has un- The clinic has no supplies to intubate the pa-
able to go to work for the past week as a result tient. Which of the following medications is
of these episodes. He notes that he has also contraindicated for this patient?
been awakened at night with numbness and
tingling in the same arm. He has a history of (A) Albuterol
diabetes and has smoked for 25 years. He re- (B) Diphenhydramine
ports that his father also has diabetes and re- (C) Epinephrine
cently had coronary artery bypass surgery. Phys- (D) Labetalol
ical examination reveals weak grip of the right (E) Methylprednisolone
hand compared to the left with signs of thenar (F) Ranitidine
atrophy. Which of the following is the most ap-
propriate next diagnostic test?
(A) Cerebral angiogram
(B) CT of the head
(C) Electroencephalogram
Test Block 1

(D) Electromyography
(E) MRI of the head
482 Section II: Full-Length Examinations Questions

32. A 59-year-old man presents to the clinic with a (A) Coronary artery aneurysm
broad-based unsteady gait. He is unable to walk (B) Dresslers syndrome
in a straight line and moves from side to side as (C) Myocardial infarction
he walks. His upper limb coordination is in- (D) Ventricular aneurysm
tact. He has no history of vertigo, and no nys- (E) Ventricular free wall rupture
tagmus is noted on physical examination.
Which of the following is the most likely cause 35. A city health ofcial wishes to test the popula-
of this patients symptoms? tion for a new outbreak of a highly virulent
coronavirus, for which a number of potential
(A) Cerebellar hemisphere infarction
screening tests have been developed. Due to
(B) Cerebellar vermis lesion
the acuity of the outbreak and limited re-
(C) Hydrocephalus
sources, just one test has to be selected. Which
(D) Menieres disease
of the following characteristics is most desir-
(E) Vestibular neuronitis
able in a screening test for a disease?
33. A 17-year-old basketball player pivots as his (A) Based on blood tests
exed knee is straightening. He notices instant (B) Behavioral counseling opportunities
pain in the knee but continues to play and n- (C) High sensitivity and low specicity
ishes the game. The next day he is limping in (D) Low cost
practice, and he tells the coach that his knee (E) Low prevalence of disease
has been catching, locking, and giving way.
The athletic trainer examines the knee, nding 36. A 33-year-old woman presents to the emer-
joint effusion and medial joint line tenderness. gency department complaining of intermittent
Which of the following is the most likely diag- palpitations, sweating, and headaches for 6
nosis? months. She has also noticed that her clothes
seem looser over the past few months. She de-
(A) Chondromalacia patella
nies any psychiatric disorders or signicant past
(B) Medial collateral ligament tear
medical history and is not taking any medica-
(C) Medial meniscus tear
tions. The patients pulse is 90/min, blood pres-
(D) Pes anserine bursitis
FULL-LENGTH EXAMS

sure is 170/93 mm Hg, respiratory rate is 21/


(E) Quadriceps tendonitis
min, and oxygen saturation is 100% on room
air. She is a thin, diaphoretic woman, with an
34. A 69-year-old man arrives to the emergency de-
otherwise unremarkable physical examination.
partment complaining of severe substernal
The patient is admitted to the hospital, and 24-
chest pain with radiation to his left arm. Serum
hour urine metanephrine excretion is 2 mg/
troponin levels and creatine kinase-myocardial
day. Other signicant laboratory values include
bound levels are elevated, and ECG reveals
an elevated erythrocyte sedimentation rate and
ST-segment elevation in leads V2 through V4.
elevated plasma total catecholamines. Which
He is treated with tissue plasminogen activator
of the following is the most appropriate phar-
and admitted to the hospital. He does well and
macotherapy?
is discharged home in stable condition. Three
weeks later, he develops a fever and sharp chest (A) Immediately start intravenous nitroprus-
pain and returns to the emergency department. side and titrate to control blood pressure
At this time his temperature is 38.78C (B) No pharmacotherapy is indicated, so pro-
(101.8F), heart rate is 104/min, blood pres- ceed immediately to surgery
sure is 128/84 mm Hg, and respiratory rate is (C) Start hydrochlorothiazide 12.5 mg orally
22/min. On examination he has a cardiac rub, four times a day and titrate to control
Test Block 1

and chest examination is limited by patients blood pressure


discomfort. ECG shows diffuse ST-segment el- (D) Start propranolol 40 mg orally twice a day;
evations in both limb and precordial leads. titrate dosage to control blood pressure and
Which of the following is the most likely diag- symptoms
nosis?
Test Block 1 Questions 483

(E) Start with phenoxybenzamine 10 mg 38. A 24-year-old G1P0 woman at 32 weeks gesta-
orally four times a day; titrate dosage to tion presents to the emergency department af-
control blood pressure and symptoms ter experiencing regular contractions approxi-
mately 10 minutes apart for 3 hours. Her
37. A 33-year-old woman presents to the emer- temperature is 37.7C (99.8F), blood pressure
gency department with abdominal pain, nau- is 136/87 mm Hg, pulse is 95/min, and respira-
sea; and emesis that began this morning. The tory rate is 18/min. While in the emergency
abdominal pain is crampy and diffuse through- department her membranes rupture spontane-
out her abdomen, and she also describes her ously, and she progresses through labor and de-
abdomen as looking slightly enlarged. She has livers a premature infant 6 hours later. Which
a history of chronic pancreatitis as well as a of the following most likely caused premature
cholecystectomy and two cesarean sections. labor and delivery in this patient?
The patient states she has had atus but no
(A) Antihypertensive treatment with nifedipine
bowel movements since the pain began. Her
(B) Excessive water intake
temperature is 37.1C (98.7F), blood pressure
(C) Untreated Chlamydia cervicitis
is 130/80 mm Hg, heart rate is 96/min, and re-
(D) Untreated preeclampsia
spiratory rate is 15/min. On physical examina-
(E) Use of nonsteroidal anti-inammatory
tion there is diffuse abdominal tenderness with
drugs
distention and high-pitched bowel sounds
without rebound tenderness or guarding pres-
39. Which of the following medications is most as-
ent. The remainder of the physical examina-
sociated with QT prolongation?
tion is not contributory. X-ray of the abdomen
is shown in the image. Which of the following (A) Chlorpromazine
is the most likely diagnosis? (B) Clozapine
(C) Haloperidol
(D) Olanzapine
(E) Quetiapine
(F) Ziprasidone

FULL-LENGTH EXAMS
40. A boy is seen at a rural health clinic in Bolivia.
He demonstrates signicant mental retarda-
tion, motor spasticity, and an abnormal gait.
The child is also deaf and mute. The patient is
of appropriate height and sexual development,
and he has no macroglossia or goiter. His
mother experienced considerable fatigue and
cold intolerance throughout her pregnancy.
Thyroid testing is obtained and thyroxine and
thyroid-stimulating hormone levels are found
Reprinted, with permission, from Brunicardi FC, Ander- to be normal. Which of the following is the
sen DK, Billiar TR, Dunn DL, Hunter JG, Matthews JB, most likely cause of this patients condition?
Pollock RE, Schwartz SI. Schwartzs Principles of Surgery,
(A) Autoimmune thyroiditis
8th edition. New York: McGraw-Hill, 2005: Figure 27-14.
(B) Current iodine deciency
(A) Colon cancer (C) Dyshormonogenesis
(B) Mesenteric ischemia (D) Hypopituitarism
(C) Pancreatitis (E) Iodine deciency late in pregnancy and af-
Test Block 1

(D) Perforated gastric ulcer ter birth


(E) Small bowel obstruction (F) Maternal iodine deciency during early
pregnancy
(G) Thyroid dysgenesis
484 Section II: Full-Length Examinations Questions

E X T E N D E D M ATC H I N G The response options for the next 2 items are


the same. Select one answer for each item
The response options for the next 2 items are in the set.
the same. Select one answer for each item
in the set. For each of the patients with a urinary complaint,
select the test that would most likely lead to the
For each of the following patients, select the most correct diagnosis.
likely diagnosis. (A) -Human chorionic gonadotropin and
(A) Burkitts lymphoma -fetoprotein
(B) Acute lymphocytic leukemia (B) Bladder biopsy
(C) Acute myelocytic leukemia (C) Complete blood cell count
(D) Chronic lymphocytic leukemia, blast crisis (D) CT scan of the pelvis
(E) Chronic myelogenous leukemia (E) Cystoscopy
(F) Ewings sarcoma (F) Digital rectal examination
(G) Follicular lymphoma (G) Fasting blood glucose
(H) Mantle cell lymphoma (H) Papanicolaou smear
(I) Mucosa-associated lymphoid tissue lym- (I) Prostate-specic antigen
phoma (J) Salt-load response
(J) Neuroblastoma (K) Urinalysis and urine culture
(K) Pheochromocytoma (L) Water deprivation test
(L) Primary cutaneous anaplastic large cell (M)Wet prep
lymphoma
(M)Szary syndrome 43. A 15-year-old boy presents to his physician
complaining of polyuria and polydipsia for the
41. A 55-year-old man with a history of atopic der- past few weeks. He also admits to wetting the
matitis presents to the emergency department bed twice last week. He is sexually active and
with diffusely erythematous skin, fever, and denies dysuria and urethral discharge. On
general malaise. Peripheral blood smear shows physical examination the boy is very thin, and
FULL-LENGTH EXAMS

cerebriform nuclei and cells with scant cyto- review of medical records indicates a 6.8-kg
plasm. (15-lb) weight loss over the past year. His tem-
perature is 37.2C (98.9F), heart rate is 84/
42. A 5-year-old boy from Kenya is brought to the min, respiratory rate is 14/min, and blood pres-
pediatrician after his mother palpates an ab- sure is 120/80 mm Hg. He has a normal ab-
dominal mass while bathing him. Biopsy of the dominal examination with no palpable masses.
mass shows numerous pale macrophages
among lymphoid cells with a high mitotic in- 44. An 18-year old woman presents to her physi-
dex. cian complaining of urinary urgency, fre-
quency, and slight burning and discomfort
when she voids for the past 2 days. She denies
fever or chills, but did note a spot of blood on
the toilet paper yesterday. She has no past med-
ical history. She takes oral contraceptives, and
her last menstrual period was 9 days ago. She
had intercourse for the rst time 4 days ago
and used a barrier contraceptive. She has a
Test Block 1

heart rate of 80/min, respiratory rate of 12/min,


and temperature of 37.2C (98.9F). On exam-
ination she has mild discomfort on deep palpa-
tion of the lower abdomen.
Test Block 1 Questions 485

The response options for the next 2 items are


45. A 29-year-old G2P1, blood type A, Rh-positive
the same. Select one answer for each item
woman delivers a 3620-g (8-lb) girl. The neo-
in the set.
nates examination is notable for frontal boss-
ing and extreme pallor. The cord blood is
For each neonate with jaundice, select the most Coombs positive. At 3 hours of age, her hemo-
likely diagnosis. globin is 6 mg/dL, total bilirubin is 13 mg/dL,
conjugated bilirubin is 1 mg/dL, and reticulo-
(A) 1-Antitrypsin deciency cyte count is 4/dL.
(B) ABO incompatibility
(C) Biliary atresia 46. A 3100-g (6.8-lb) boy is born to a blood type B,
(D) Breast-feeding Rh-negative, 23-year-old primigravida at 38
(E) Cephalohematoma weeks gestation via spontaneous vaginal deliv-
(F) Crigler-Najjar syndrome ery. He is being exclusively breast-fed but is
(G) Gilberts syndrome only latching on to the breast for short periods.
(H) Hereditary spherocytosis On the third day of life he appears jaundiced.
(I) Neonatal hepatitis His total bilirubin level is 15 mg/dL, his conju-
(J) Physiologic gated bilirubin is 0.8 mg/dL, and his liver func-
(K) Rh incompatibility tion tests are normal.
(L) Sepsis
(M)Toxoplasmosis infection

FULL-LENGTH EXAMS
Test Block 1
486 Section II: Full-Length Examinations Answers

AN S W E R S

1. The correct answer is D. This is a classic de- SCC but typically appears suddenly and grows
scription of nodular basal cell carcinoma rapidly to approximately 12 cm within a cou-
(BCC). BCC is usually described as a pearly, ple of weeks. The lesion is a dome-shaped pap-
telangiectatic nodular lesion with rolled bor- ule that may crust, and typically remains stable
ders. As the tumor outgrows its blood supply it for months to years before regressing. Since
can develop central crusting and ulceration. scarring is typical, these are usually excised.
BCC is the most common tumor of the skin,
and is referred to as an epithelioma due to the 2. The correct answer is D. This patient is con-
low metastatic potential. Major risk factors are templating suicide and is in great danger to
childhood sun exposure, fair skin, chronic der- himself. Questions should be nonjudgmental,
matitis, and xeroderma pigmentosum. BCC is yet direct. It is important to elucidate details of
usually very slow-growing, but can result in ex- the plan and preparations that have been made
tensive destruction of local tissues and exten- to implement the plan.
sion into the central nervous system if un- Answer A is incorrect. A willingness to die is
treated. Basal cell nevus syndrome, a genetic not helpful information in determining sui-
form of BCC, is associated with a mutation in cidality. Patients may feel too depressed to re-
the PTCH gene. alistically determine whether they want to die
Answer A is incorrect. This description char- or not.
acterizes malignant melanoma. Though pig- Answer B is incorrect. Normalizing the situa-
mented BCC can resemble malignant mela- tion may make the patient feel disregarded. Ask
noma, the lesion described here is classic about specic details regarding the plan.
nodular BCC and should not be confused with
malignant melanoma. Answer C is incorrect. Asking about homicide
does not help gather information about the
Answer B is incorrect. This course is typical of current suicidality.
squamous cell carcinoma (SCC). SCC is clas-
FULL-LENGTH EXAMS

sically a scaly, erythematous lesion that may Answer E is incorrect. Using guilt or blame is
ulcerate. Three percent of cutaneous SCCs not a useful tool when discussing suicide with
metastasize, typically to adjacent lymph nodes. a patient.
Risk factors for metastatic lesions include lo- Answer F is incorrect. Patients may not have
cation in scar tissue, immunodeciency, ex- a specic reason for contemplating suicide. It
posure to x-rays and ultraviolet radiation, and is more helpful to discuss how close they are to
location on the head or neck. Prognosis is ex- actually attempting suicide.
cellent if completely excised (95% cure rate),
but survival is only 50% at 5 years if metastatic. 3. The correct answer is B. Aortic dissection, es-
Answer C is incorrect. This course is typical pecially in the ascending portion, is most com-
of seborrheic keratosis, which can mimic ma- monly associated with hypertension. Acute
lignant melanoma, SCC, or BCC, but is a be- management of this potentially life-threatening
nign lesion. Horny cysts embedded in the sur- condition involves blood pressure control and
face of the lesion, when present, support this prompt evaluation for surgical repair.
diagnosis. Abrupt eruption of multiple pruritic Answer A is incorrect. Aortic aneurysms, es-
seborrheic keratomas may occur as a paraneo- pecially in the abdomen, are most commonly
plastic syndrome (the sign of Leser-Trlat, most associated with atherosclerosis. Until they rup-
Test Block 1

commonly seen with gastrointestinal adenocar- ture, these generally present no distress to the
cinomas), but this association is controversial. patient and may be discovered incidentally.
Answer E is incorrect. This describes the Aortic aneurysms larger than 5 cm are gener-
course of keratoacanthoma, which can mimic ally treated surgically.
Test Block 1 Answers 487

Answer C is incorrect. Protein C is an antico- indicates biliary obstruction. The lack of an in-
agulant, therefore a hereditary protein C de- crease in amylase and lipase makes obstruction
ciency leads to a hypercoagulable state. This is at the ampulla less likely (such an obstruction
not, however, an etiology of aortic dissection. would cause pancreatic inammation). A pa-
tient with choledocholithiasis should undergo
Answer D is incorrect. Rheumatic heart dis-
extraction of stones from the common bile duct.
ease is a childhood sequela of bacterial infec-
This may be achieved endoscopically, via endo-
tion that is not commonly seen in patients who
scopic retrograde cholangiopancreatography
were born in the era of antibiotics. When it is
and sphincterotomy, or operatively.
seen, it commonly leads to mitral, and less fre-
quently aortic, valvular disease. It is not, how- Answer B is incorrect. The common hepatic
ever, an etiology of aortic dissection. duct is formed by the convergence of the right
and left hepatic ducts. The cystic duct joins the
Answer E is incorrect. Trauma is a possible
common hepatic duct to form the common
etiology of aortic dissection, and this must be
bile duct. Therefore, gallstones are not found
ruled out in incidents of trauma, but hyperten-
in the common hepatic duct.
sion is still the most common etiology of aortic
dissection. Answer C is incorrect. A stone obstructing her
cystic duct would not result in an elevation of
4. The correct answer is E. Stimulants or do- her total bilirubin, aspartate aminotransferase,
pamine agonists can worsen or even precipitate alanine aminotransferase, or alkaline phos-
tics in patients. This patient likely has a motor phatase. However, it would cause the pain of
tic disorder that can be treated with haloperi- biliary colic observed in this patient. These
dol, pimozide, or clonidine. symptomatic patients should be treated with
cholecystectomy.
Answer A is incorrect. Antipsychotics can be
used to lessen the severity of these involuntary Answer D is incorrect. If a stone was obstruct-
movements. ing the ampulla of Vater, some pancreatic in-
ammation would be expected from the con-
Answer B is incorrect. Depressants are not
comitant obstruction of the common bile duct
known to produce or worsen tics.

FULL-LENGTH EXAMS
and the pancreatic duct, with a resulting in-
Answer C is incorrect. Hallucinogens may crease in amylase and lipase. Denitive treat-
produce bizarre behavior but not perseverative ment includes cholecystectomy with cholangi-
or stereotyped behaviors such as in this case. ography and removal of visualized stones from
Answer D is incorrect. Selective serotonin the common bile duct.
reuptake inhibitors have not been shown to in- Answer E is incorrect. The stones observed in
crease the prevalence of tics in patients. her gallbladder are not those responsible for
her symptoms and laboratory values. They are
5. The correct answer is A. This patient is present- not obstructing and are not causing a back-up
ing with signs and symptoms that are highly sus- of bile from ductal obstruction. However, the
picious for biliary colic with obstruction or in- stone which is not obstructing the common
ammation of the biliary tree. Ultrasound shows bile duct was likely formed in the gallbladder.
stones in the gallbladder, but those stones are Asymptomatic gallstones require no treatment.
not necessarily responsible for her current pain. However, approximately 1%2% of people per
The laboratory ndings can help pinpoint the year with this condition develop symptoms of
site of obstruction by observing its effects. In this gallstone disease. These symptomatic patients
case, a stone appears to be obstructing the com- should be treated with cholecystectomy.
Test Block 1

mon bile duct. The elevated total bilirubin im-


plies obstruction at this site, with consequent 6. The correct answer is E. The patient is de-
back-up of bile into the liver, causing some he- scribing symptoms that are consistent with
patocellular damage (i.e., elevated aspartate Hodgkins disease, including night sweats and
aminotransferase and alanine aminotransferase). Pel-Ebstein (cyclic) fevers. Her shortness of
Additionally, the elevated alkaline phosphatase breath may be consistent with a mediastinal
488 Section II: Full-Length Examinations Answers

mass caused by adenopathy. The patient may carry an increased risk for this patient as op-
also be suffering from other chest pathology, in- posed to other immunosuppressed patients.
cluding pulmonary tuberculosis (TB), asthma,
Answer E is incorrect. Patients who have re-
and cardiomyopathy, and an x-ray of the chest
ceived a bone marrow transplant or who are
would be useful in the diagnosis or exclusion of
functionally asplenic are at an increased risk
any of these conditions.
of infection with Streptococcus pneumoniae
Answer A is incorrect. A bronchoscopy may be and other encapsulated organisms. The great-
warranted in the future but is not indicated in est risk of infection with these organisms in the
the absence of known lung pathology. transplant patient is in the late posttransplanta-
tion period, or >100 days posttransplant.
Answer B is incorrect. HIV testing might be
prudent in this patient given her night sweats
8. The correct answer is A. Intramuscular inu-
and recurrent fevers; however, this is not the
enza vaccines contain inactivated viruses that
most important step in diagnosing this patient.
reect the strains anticipated to circulate dur-
Answer C is incorrect. Performing a PPD test ing the upcoming winter. Current guidelines
might help in the diagnosis of pulmonary TB; are that children 6 months to 5 years old
however, this diagnosis will take 2448 hours. should receive the vaccine, along with women
X-ray of the chest is a more appropriate next in the second or third trimester of pregnancy,
step in this patients care. adults >50 years old, and adults at high risk be-
cause of underlying conditions such as diabe-
Answer D is incorrect. Although the patients
tes, heart or lung disease, or immunocompro-
symptoms are consistent with a panic attack,
mise. Vaccination is also recommended to
the presence of night sweats and intermittent
health care workers and a patients household
fevers warrants further diagnostic evaluation
members who may transmit the virus to at-risk
and testing before empirically treating the pa-
groups. The vaccine is usually administered in
tient for panic disorder with a selective sero-
the fall and early winter.
tonin reuptake inhibitor.
Answer B is incorrect. MMR, or measles-
7. The correct answer is C. Patients with a recent mumps-rubella vaccine, contains live virus and
FULL-LENGTH EXAMS

history of kidney transplant are at an increased therefore should not be given to immunocom-
risk of bacterial infections. Septicemia and promised patients or pregnant women. It is
peritonitis in these patients is often caused by usually given to children at 1215 months and
Pseudomonas aeruginosa. Patients with cystic then again at 46 years of age, but the second
brosis and patients with HIV can also be at an dose can be given as soon as 4 weeks after the
increased risk of infection with this organism. rst dose.
Answer A is incorrect. Patients who have re- Answer C is incorrect. Oral polio virus is the
ceived liver transplants, not kidney transplants, oral polio vaccine still administered in certain
are at an increased risk for infection with en- parts of the world and contains live virus. In
terococci. the United States the primary polio vaccine is
the inactivated form, inactivated poliovirus vac-
Answer B is incorrect. Patients with HIV and
cine, which is administered to children in four
low CD4+ cell counts, as well as infants 28
doses, at 2 months, 4 months, 618 months,
months old, have the most increased risk for
and 46 years.
infection with Pneumocystis jiroveci. Rather
than peritonitis and septicemia, P. jiroveci usu- Answer D is incorrect. The rotavirus vaccine
ally causes pneumonia with characteristically is a mixture of four rotavirus types and is used
Test Block 1

low oxygen saturation. to immunize infants against rotaviral gastroen-


teritis, the most common cause of diarrhea in
Answer D is incorrect. Staphylococcus aureus
young children. It is an attenuated live virus
is a common pathogen, but not one that would
vaccine.
Test Block 1 Answers 489

Answer E is incorrect. The varicella vaccine mechanism, the main adverse effect for pa-
contains live virus. It is recommended to be tients is weight gain. Rarely, these medications
administered to children at 1218 months of can also exacerbate congestive heart failure.
age. In older children and adults two doses
must be given, with the second dose at least 4 10. The correct answer is B. Desmopressin is a va-
weeks after the rst dose. sopressin analog that promotes the release of
von Willebrands factor and factor VIII from
9. The correct answer is C. This patient was tissue stores. Administration of desmopressin
started on an antihyperglycemic agent, and will lead to a rapid increase in a patients factor
quickly developed gastrointestinal symptoms VIII level and is therefore useful in patients
and an anion gap metabolic acidosis with com- with mild hemophilia type A (with a deciency
pensatory respiratory alkalosis. This patient was in coagulation factor VIII) to decrease bleed-
likely taking metformin, a biguanide that low- ing during surgery. In patients with mild he-
ers blood glucose levels by decreasing hepatic mophilia, desmopressin is preferable to re-
glucose production and increasing insulin sen- placement therapy with coagulation factor
sitization. Metformin is a known cause of lactic VIII, which should be used for severe hemo-
acidosis, which occurs extremely rarely in philia A or active bleeding because it is not as-
healthy individuals (5 cases per 100,000), but sociated with a risk of viral transmission (HIV
more commonly in patients with renal, heart, or hepatitis) and achieves the same ultimate
or liver disease. The mortality rate of met- goal of increasing the coagulation factor con-
formin-treated patients with lactic acidosis is centrations. Because of past experience with
close to 50%. Neither arterial lactate levels nor viral transmission, cryoprecipitate should not
plasma metformin concentrations are good be used to treat hemophilia A.
predictors of mortality; death more closely cor-
Answer A is incorrect. Nonsteroidal anti-
relates with underlying comorbid conditions.
inammatory drugs (NSAIDs) such as aspirin
Supportive care is given to patients with mild
are associated with an increased risk of bleed-
to moderate lactic acidosis. For patients with
ing. These medications should not be used in
profound metformin-induced lactic acidosis
patients who are prone to bleeding, including
(pH <7.10), use of hemodialysis or sodium bi-

FULL-LENGTH EXAMS
hemophiliacs or patients with von Willebrands
carbonate infusion is recommended.
disease. Administration of aspirin is also con-
Answer A is incorrect. An -glucosidase in- traindicated in this patient, especially prior to
hibitor, acarbose inhibits intestinal absorption surgery.
of carbohydrates. Because of this mechanism
Answer C is incorrect. Packed RBC transfu-
of action, these drugs have a high rate of un-
sions do not include platelets or clotting fac-
pleasant adverse effects, including atulence
tors. Because this patient is decient in clotting
and diarrhea.
factor VIII, there is no utility in transfusing
Answer B is incorrect. Sulfonylureas achieve packed RBCs to this patient preoperatively.
glycemic control by increasing -cell secretion
Answer D is incorrect. This patient has hemo-
of insulin. The main adverse effect is hypogly-
philia type A. This is a hereditary bleeding dis-
cemia. Rarely, sulfonylureas have been known
order in which there is a deciency of coagula-
to cause nausea and photosensitivity.
tion factor VIII. This disorder is not associated
Answer D is incorrect. Meglitinides act in a with abnormal platelet function or quantitative
fashion similar to the sulfonylureas by increas- platelet deciencies. Therefore, there is no
ing -cell insulin secretion. Their main adverse utility in transfusing platelets into this patient
effect is hypoglycemia. preoperatively.
Test Block 1

Answer E is incorrect. Thiazolidinediones Answer E is incorrect. Administration of whole


increase insulin sensitivity at the muscles and blood will replenish RBCs and some clotting
liver by increasing the utilization, and decreas- factors; however, it is decient in platelets and
ing the production, of glucose. Because of this clotting factors V, VIII, and XI. Because this
490 Section II: Full-Length Examinations Answers

patient is decient in clotting factor VIII, pro- Answer A is incorrect. -Blockers are used as
viding whole blood to this patient preopera- prophylactic therapy for migraine headaches,
tively has little if any utility. In addition, whole preventing migraines from occurring. They do
blood transfusions are associated with a small not help end a headache once it has occurred.
yet quantiable risk of transmission of viral in-
Answer C is incorrect. Ergotamines should be
fections (HIV and hepatitis).
avoided in patients with coronary artery disease
and hypertension because they cause coronary
11. The correct answer is C. The constellation of
artery vasoconstriction and hypertension. Er-
symptoms and the presence of Trichomonas
gotamine is contraindicated in women who are
vaginalis protozoa on Giemsa staining estab-
or may become pregnant, since the drugs may
lish the diagnosis of trichomoniasis vaginitis.
cause fetal harm. Ergotamine is also contrain-
The treatment of choice for this condition is
dicated in patients with peripheral vascular
one 2-mg dose of metronidazole.
disease, coronary heart disease, uncontrolled
Answer A is incorrect. Azithromycin is a mac- hypertension, stroke, impaired hepatic or renal
rolide antibiotic that inhibits RNA synthesis by function, and sepsis.
binding to the 50S ribosomal RNA subunit. It
Answer D is incorrect. Triptans can be help-
is used to treat chlamydial infections but is not
ful, but have been linked to coronary artery
therapeutic for trichomoniasis.
constriction and should be avoided in patients
Answer B is incorrect. Fluconazole is an anti- with cardiovascular risk factors.
fungal medication that inhibits cell membrane
Answer E is incorrect. Valproate is an anti-
synthesis of various fungi. It is often used to
convulsive medication that is most effective as
treat vaginal candidiasis, but is has no antipro-
prophylactic treatment of migraine headaches
tozoal activity and thus is ineffective against
because it helps prevent migraines, but is not
Trichomonas vaginalis infections.
helpful in stopping an attack once it has oc-
Answer D is incorrect. Penicillin is an antibi- curred.
otic whose mechanism of action involves in-
terruption of cell wall synthesis. Its spectrum 13. The correct answer is C. In a standard 12-lead
FULL-LENGTH EXAMS

of activity includes most gram-positive cocci, ECG, ST-segment elevations in leads II, III,
most anaerobes, and a limited number of and aVF indicate an evolving myocardial in-
gram-negative bacteria. It is not active against farction (MI) in the inferior portion of the
Trichomonas vaginalis. heart. The posterior descending artery (usually
a branch of the right coronary artery) supplies
Answer E is incorrect. Vancomycin is an anti-
the inferior portion of the ventricles
biotic that is effective against numerous gram-
positive organisms, including methicillin- Answer A is incorrect. ST-segment elevations
resistant Staphylococcus aureus, and which is in leads V1, V2, V3, and V4 indicate an evolv-
almost always administered as an intravenous ing MI in the anterior portion of the heart.
infusion. Vancomycin is not active against The left anterior descending artery supplies the
Trichomonas vaginalis. anterior portion of the left ventricle.
Answer B is incorrect. One of the primary pur-
12. The correct answer is B. If NSAIDs are not ef-
poses of performing an ECG is for more accu-
fective, other analgesics such as acetamino-
rate assessment of the location of ischemic por-
phen, and acetaminophen/caffeine combina-
tions of the heart. Therefore, the presence of
tion medications (such as Excedrin and other
ST-segment elevations in different leads is very
over-the-counter migraine analgesics) can be
helpful in determining which portions of the
Test Block 1

used to abort a migraine attack. NSAIDs allevi-


heart might be experiencing infarction.
ate pain associated with an attack and the caf-
feine dilates coronary vessels and breaks the
headache.
Test Block 1 Answers 491

Answer D is incorrect. ST-segment elevations 15. The correct answer is D. Mycobacterium tuber-
in leads V5 and V6 indicate an evolving MI in culosis has become much more prevalent in the
the lateral portion of the heart. The left cir- United States as a result of the HIV epidemic.
cumex artery supplies the lateral portion of In fact, infection with HIV may increase a pa-
the left ventricle. tients risk of developing active TB by a factor of
>15. All HIV-infected individuals should be
Answer E is incorrect. ST-segment depressions
screened for latent TB infection by means of a
in V1 and V2 would suggest ischemia in the
PPD skin test. HIV-infected individuals who
posterior portion of the heart. The posterior
have a skin test reaction of >5 mm or who have
descending artery supplies the posterior por-
close household contacts with persons with ac-
tion of the heart.
tive TB should automatically receive prophylac-
14. The correct answer is D. Diffuse rebound ten- tic treatment for TB.
derness most likely indicates generalized peri- Answer A is incorrect. Although respiratory
tonitis, which, combined with signs and symp- isolation in the ward would be required if
toms of small bowel obstruction (SBO), most this patient were detectably ill, there is no in-
likely indicates rupture of inamed, obstructed dication that he needs to be hospitalized. A
small bowel. This is a surgical emergency re- 9-month course of isoniazid is indicated only
quiring an immediate operation. The other because of the patients high risk of developing
symptoms that are worrisome in a patient be- active TB.
lieved to have an SBO are tachycardia, hy-
Answer B is incorrect. Monitoring serial PPD
potension, and metabolic acidosis.
tests is a proper course of action for any HIV-
Answer A is incorrect. Bowel sounds can be positive individual with a PPD <5 mm; how-
variable in SBO, although the classic presen- ever, this patient has a PPD of >5 mm and
tation is high-pitched tinkles and peristaltic should be treated accordingly.
rushes. Although the absence of bowel sounds
Answer C is incorrect. Even if x-ray of the
indicates a progression, it does not necessarily
chest is clear, current guidelines indicate that
warrant a surgical emergency.
all HIV-positive patients with a PPD test >5

FULL-LENGTH EXAMS
Answer B is incorrect. The absence of atus mm should receive prophylactic anti-TB therapy.
indicates a complete obstruction (obstipation).
Answer E is incorrect. Trimethoprim-sulfame-
This is worrisome for a complete obstruction
thoxazole is not active against Mycobacterium
but does not warrant a surgical emergency.
tuberculosis. The patients CD4+ cell count is
Answer C is incorrect. Bilious vomiting is >200/mm, so prophylaxis against Toxoplasma
commonly seen in SBO, and the more proxi- gondii and Pneumocystis jiroveci pneumonia is
mal the obstruction, the more common is the therefore not indicated.
bilious vomiting (although it will not be seen if
the obstruction is proximal to the second por- 16. The correct answer is A. Alzheimers disease is
tion of the duodenum, where the common bile manifested by a progressive dementia with
duct empties into the duodenum). Although memory loss, general cognitive dysfunction,
SBO is an indication for surgery within 24 and functional impairments. Although patients
hours, additional diagnostic tests would need with Alzheimers disease can have imaging
to be performed to conrm this diagnosis. ndings suggestive of increased cerebral atro-
phy, it is not uncommon for CT studies to be
Answer E is incorrect. Metabolic alkalosis is
normal and age appropriate.
commonly seen in patients with SBO, particu-
larly those who have experienced vomiting (this Answer B is incorrect. Delirium has a more
Test Block 1

is from the loss of acidic gastric contents). On the acute onset of days to weeks, not months to
other hand, metabolic acidosis would be signi- years. Patients generally have a more clouded
cantly more concerning because it implies likely sensorium, with uctuations in level of con-
strangulation and necrosis of the intestines, and sciousness. It is usually reversible.
would require an immediate operation.
492 Section II: Full-Length Examinations Answers

Answer C is incorrect. Dementia with Lewy Answer C is incorrect. UTIs due to Klebsiella
bodies is classically seen in patients with Par- are seen among hospitalized patients rather
kinsons disease or Parkinsonian features. The than in the ambulatory population.
dementia is progressive, with episodes of wax-
Answer D is incorrect. In cases of UTIs due to
ing and waning of cognitive function, and is
Proteus, the alkaline urine promotes the devel-
set apart from other dementia syndromes by
opment of a struvite stone that may form stag-
the following key symptoms: increased day-
horn calculi in the urinary tract.
time drowsiness and sleep, episodes of staring
into space, disorganized ow of ideas, and vi- Answer E is incorrect. UTIs due to Pseudomo-
sual hallucinations. Although this patient has nas are seen among hospitalized patients rather
a family history of Parkinsons disease, he does than in the ambulatory population.
not show any symptoms such as tremor, brady-
Answer F is incorrect. Although staphylococci
kinesia, or cogwheel rigidity.
may account for 5%20% of UTIs, E. coli has
Answer D is incorrect. Progressive supranuclear been associated with 70%95% of UTIs and
palsy is a dementia syndrome characterized by therefore is more likely to be involved in the
vertical supranuclear palsy with downward gaze pathogenesis described. UTIs due to Staphylo-
disturbance. Other characteristics include pos- coccus infections, although relatively uncom-
tural instability and personality changes such as mon, occur more frequently in young, ambu-
apathy, anxiety, and disinhibition. latory women.
Answer E is incorrect. What differentiates vas-
18. The correct answer is C. Cystic brosis (CF)
cular dementia from other types of dementia is
often presents primarily with gastrointestinal
the observed onset of cognitive symptoms im-
symptoms in infancy, most commonly a failure
mediately following a stroke, with infarcts seen
to thrive and malabsorption. These effects are
on imaging and ndings on physical examina-
due to thick pancreatic secretions secondary to
tion consistent with a prior stroke. The onset of
ineffective chloride transport, which leads to
cognitive dysfunction is very abrupt rather than
plugging of the pancreatic acini. The inability
progressive, and then followed by a continued
of pancreatic enzymes to enter the gastrointes-
deterioration. Although atrial brillation in-
FULL-LENGTH EXAMS

tinal tract leads to malabsorption of fats. Par-


creases this mans risk for a stroke, there is no
ents of children with CF will typically report
history of a stroke and no focal neurologic de-
foul-smelling, oily stools and difculty feeding.
cits.
Seven to ten percent of patients with CF will
have a history of meconium ileus at birth. Di-
17. The correct answer is B. This patient has a clin-
agnosis can be conrmed by a sweat chloride
ical presentation most consistent with acute, un-
test result >60 mEq/L.
complicated pyelonephritis. She presents with
fever, chills, nausea, vomiting, costovertebral Answer A is incorrect. A biopsy may be useful
tenderness, increased urinary frequency, and a in diagnosing malabsorptive disorders due to
urinalysis that is consistent with an infectious mucosal abnormalities such as celiac disease.
process. Escherichia coli is the causative patho- However, since there is nothing in her diet yet
gen most commonly seen in urinary tract infec- that would expose her to gluten, celiac disease
tions (UTIs) (in both uncomplicated upper and is not a likely diagnosis.
lower tract infections). This organism accounts
Answer B is incorrect. Stool qualitative fat
for 70%95% of UTIs. Because this patient does
analysis would only conrm a malabsorptive
not have an indwelling urinary catheter in place
process, which is already suggested by the pa-
and was not hospitalized at the time of presenta-
tients presentation. It would not help in diag-
Test Block 1

tion, other pathogens are even less likely to be


nosing the etiology of that process.
causative in this case.
Answer D is incorrect. Pancreatitis may result
Answer A is incorrect. UTIs due to Enter-
from plugging of acini with thick secretions,
obacter are seen among hospitalized patients
leading to autodigestion of the pancreatic tis-
rather than in the ambulatory population.
sue. Ultrasound of the pancreas may show this
Test Block 1 Answers 493

autodigestion, but it will not make the etiology secretion (SIADH) due to ectopic production
clear, and will therefore not help in the diag- of ADH by a bronchogenic small cell lung car-
nosis of CF. cinoma. ADH acts on the collecting tubules to
increase the retention of free water. SIADH,
Answer E is incorrect. X-ray of the abdomen
therefore, is not caused by a lack of sodium but
and pelvis could show anatomic abnormalities
by excess free water. However, the Na level will
and intestinal obstruction, but has no role in
be decreased in the laboratory values because
diagnosing CF.
of the increased water uptake. Plasma osmolal-
ity decreases due to free water retention and
19. The correct answer is A. This patients clinical
inability to diurese and dilute the urine. There-
presentation is classic for claudication (repro-
fore, urine electrolyte levels will be relatively
ducible leg pain caused by atherosclerosis and
elevated. Treatment is free water restriction
reduced blood ow to the extremities that oc-
and diuresis if the patient is stable, and infu-
curs with walking and is relieved with rest), es-
sion of isotonic saline if neurologic symptoms
pecially given his history of cardiovascular risk
appear. This carcinoma is strongly correlated
factors. Exercise, the rst-line treatment of
with smoking history and is a rapidly progres-
claudication, promotes the development of col-
sive tumor. Weight loss is typical of the presen-
lateral circulation in extremities and alleviates
tation of this carcinoma, and hip pain suggests
symptoms of poor perfusion. Ankle-branchial
bony metastasis.
index is used as an objective measure of ath-
erosclerotic occlusion. Measurements of >0.90 Answer A is incorrect. This patient most likely
are normal, those <0.90 are associated with pe- has a pneumonia, which can cause SIADH in
ripheral vascular disease, and those <0.40 are some cases. Small cell lung carcinoma, how-
associated with rest pain. ever, is a more likely cause of SIADH in this
patient given his smoking history, history of
Answer B is incorrect. Fibrinolytic agents are
lung nodule, weight loss, and bone pain.
often used to dissolve coronary artery or central
nervous system clots if caught within a speci- Answer B is incorrect. Postsurgical atelectasis
ed time frame. Claudication is caused by sta- can cause SIADH but would not be commonly
ble atherosclerotic plaques, not clots. seen 2 months after surgery.

FULL-LENGTH EXAMS
Answer C is incorrect. -Blockers work by in- Answer C is incorrect. Head trauma can cause
hibiting the -adrenergic stimulation of cardiac SIADH; however, this patient has no history of
muscle and the smooth muscle of blood ves- trauma.
sels. They are not rst-line treatment for clau-
Answer E is incorrect. Renal failure can pre-
dication and, in fact, may worsen symptoms of
sent with hyponatremia; however, this patient
claudication.
has no evidence of renal failure given his nor-
Answer D is incorrect. The statins as a class of mal blood urea nitrogen and creatinine.
lipid-lowering agents work by inhibiting HMG-
CoA reductase. Although hyperlipidemia is of- 21. The correct answer is E. Sarcoidosis is a mul-
ten an underlying condition in patients who tisystem granulomatous disease of unknown
suffer from claudication, reducing lipid levels etiology, which has a higher prevalence in Afri-
has not been shown to alleviate the symptoms can-American women. Although many clinical
of claudication. manifestations can be seen, sarcoidosis most
frequently involves the lungs and therefore
Answer E is incorrect. Irreversible inhibition
commonly presents with symptoms of cough,
of cyclooxygenase enzyme is the mechanism
dyspnea, and chest pain. This patients x-ray of
by which aspirin exerts its effect. Aspirin is rec-
Test Block 1

the chest shows bilateral hilar lymphadenopa-


ommended for prevention of coronary artery
thy as well as interstitial inltrates. Physical ex-
disease, not for peripheral vascular disease.
amination reveals diffuse lymphadenopathy,
also consistent with a diagnosis of sarcoidosis.
20. The correct answer is D. This patient most
likely has the syndrome of inappropriate ADH However, to obtain a denitive diagnosis in
this patients case, a transbronchial lung biopsy
494 Section II: Full-Length Examinations Answers

showing a mononuclear cell granulomatous in- Answer E is incorrect. It is appropriate to ad-


ammatory process is needed. dress both the concerns and the needs of the
patients family, in this case his wife, before
Answer A is incorrect. Liver biopsies in pa-
extubating the patient, even though ultimately
tients with sarcoidosis are often positive. How-
this is the nal outcome.
ever, noncaseating granulomas are so common
in the liver that this method is not considered
23. The correct answer is B. The patient has a
an acceptable diagnostic procedure.
pheochromocytoma, which is associated with
Answer B is incorrect. An open lung biopsy multiple endocrine neoplasia (MEN) types IIA
would certainly provide a denitive diagnosis, and IIB. MEN IIA includes medullary thyroid
but a less invasive approach would be more carcinoma, pheochromocytoma, and parathy-
prudent. However, an open lung biopsy may roid hyperplasia. MEN IIB includes medullary
be required if less invasive tests are unable to thyroid carcinoma, pheochromocytoma, and
provide a diagnosis, despite a very high clinical mucosal/gastrointestinal neuromas. Other neo-
suspicion. plastic diseases associated with pheochromocy-
toma include von Hippel-Lindau syndrome
Answer C is incorrect. A scalene node biopsy
and neurobromatosis.
is most likely to yield a positive result, but it
may provide an unacceptable number of false- Answer A is incorrect. Insulinoma is part of
positive results and therefore is not considered MEN type I and is not typically associated with
an appropriate diagnostic procedure. pheochromocytoma.
Answer D is incorrect. Angiotensin-converting Answer C is incorrect. Pancreatic adenocar-
enzyme levels are elevated in approximately cinoma is associated with Peutz-Jeghers syn-
60% of untreated patients with sarcoidosis. Al- drome (36% lifetime risk) and hereditary pan-
though measurements of plasma angiotensin- creatitis (40% lifetime risk). It is not typically
converting enzyme levels can be valuable in associated with pheochromocytoma.
assessing the course of the disease, they are not
Answer D is incorrect. Papillary thyroid cancer
helpful in making the initial diagnosis.
typically affects women (2.5:1, male:female)
FULL-LENGTH EXAMS

in their 30s and 40s, contains calcied psam-


22. The correct answer is D. A living will speci-
moma bodies, and is not typically associated
cally states the wishes of a patient and is meant
with pheochromocytoma.
to be used in lieu of a direct discussion with
the patient if the patient is unable to state his Answer E is incorrect. Pituitary adenoma is
wishes at any point. Because this patient made not typically associated with pheochromocy-
his wishes very clear, both verbally and in writ- toma. It is part of MEN type I, along with pan-
ing, his wife cannot override his living will now creatic neuroendocrine tumors and parathy-
that he cannot speak for himself. roid hyperplasia.
Answer A is incorrect. There is no need for a
24. The correct answer is C. The history of rheu-
family meeting to decide whether to extubate
matic fever should raise suspicion of mitral
the patient because the living will clearly states
stenosis, caused by the fusion of the commis-
the patients wishes.
sures, producing the typical sh mouth valve.
Answer B is incorrect. Because the patient has The clinical manifestations of mitral stenosis
a living will, there is no need for an ethics con- include dyspnea, pulmonary edema, and atrial
sult in this case to help mediate decision mak- brillation. Other ndings consistent with mi-
ing between the medical team and patients tral stenosis are a low-pitched diastolic rumble
Test Block 1

family. at the apex, an opening snap, and x-ray of the


chest showing a dilated left atrium (straighten-
Answer C is incorrect. Luckily, in this case,
ing of the left heart border and elevation of the
the patient has stated his wishes clearly before
left main stem bronchus). Mitral stenosis
undergoing surgery, so there is no need for the
team to decide how to proceed.
Test Block 1 Answers 495

causes an increase in the pulmonary capillary decompensation and the development of pul-
hydraulic pressure, resulting in a net increase monary edema. A normal thyroid-stimulating
in uid transported into the interstitium. This hormone level, however, argues against thyroid
patient should be treated with warfarin for his disease.
atrial brillation, diuretics for his pulmonary
congestion, and a -blocker to allow increased 25. The correct answer is B. Contraction of the
ventricular lling time. neck muscles in an unnatural position is
known as torticollis; in this case, the patient is
Answer A is incorrect. Acute aortic insuf-
experiencing an acute dystonic reaction as an
ciency causes an increase in left ventricular
adverse effect of one of his antipsychotic medi-
volume, with elevation of left ventricular end-
cations, most likely a high-potency typical an-
diastolic and left atrial pressure, resulting in
tipsychotic such as haloperidol, droperidol, u-
pulmonary edema. Aortic insufciency, how-
phenazine, or thiothixene. Treatment of acute
ever, typically presents with a diastolic decre-
dystonia is with an anticholinergic agents such
scendo murmur at the left upper sternal border
as benztropine or with diphenhydramine; the
and a wide pulse pressure (>60 mm Hg).
patient will literally loosen up within a mat-
Answer B is incorrect. Pulmonary edema ter of seconds. Prophylaxis for acute dystonic
can also arise from acute mitral insufciency, reactions can be provided with benztropine.
which results from papillary muscle dysfunc-
Answer A is incorrect. Alprazolam is a short-
tion or chordae tendineae rupture following
acting benzodiazepine that is used to treat
MI. With acute mitral insufciency, the abil-
anxiety. It has no role in the treatment of acute
ity of the left atrium to ll the left ventricle is
dystonia.
reduced, resulting in pulmonary edema. How-
ever, the patients presentation and physical Answer C is incorrect. Haloperidol and other
ndings are not consistent with mitral insuf- typical antipsychotic medications are respon-
ciency. While these patients do present with sible for causing acute dystonias such as in
pulmonary edema, there is a high-pitched, this patient. Initiating haloperidol would only
blowing, holosystolic murmur at the apex, lat- worsen the patients torticollis.
erally displaced point of maximal intensity,

FULL-LENGTH EXAMS
Answer D is incorrect. Although it would
and history of severe substernal chest pain (if appear that the neck muscles are in spasm,
infarction is the cause of the patients acute mi- muscle relaxants are not indicated; an acute
tral insufciency). dystonic reaction warrants administration of di-
Answer D is incorrect. Acute onset of severe phenhydramine.
myocardial ischemia can lead to a sudden loss Answer E is incorrect. Sertraline is a selective
of systolic and diastolic function, resulting serotonin reuptake inhibitor used primarily to
in decreased cardiac output and pulmonary treat depression and anxiety disorders. It has no
edema. The presentation, however, is not typi- role in the treatment of acute dystonia.
cal for myocardial ischemia. The patient lacks
angina with or without radiation of pain to the 26. The correct answer is D. Prevalence is the
neck, jaw, shoulders, or arms for >30 minutes. number of cases of a disease or condition in the
However, 23% of MIs are atypical or silent. If population at a given point in time. Cross-
this were an atypical MI, the cardiac enzymes sectional studies collect information on disease
would still be elevated. In fact, the sensitivity presence and on risk factors for its develop-
of creatine kinase-myocardial bound, troponin ment simultaneously.
T, and troponin I >6 hours after the onset of
symptoms is 91%, 94%, and 100%, respec- Answer A is incorrect. Attributable risk is cal-
Test Block 1

tively. culated in cohort studies and is the difference


in incidence rate between study and control
Answer E is incorrect. In the presence of dia- groups. Cross-sectional studies do not produce
stolic dysfunction, fever, sepsis, and anemia the data required for this calculation.
thyroid disease can precipitate hemodynamic
496 Section II: Full-Length Examinations Answers

Answer B is incorrect. Causation cannot be de- and risks associated with antibiotics, antibiotics
termined from cross-sectional studies because should only be used as a temporizing therapy
information on exposure to potential risk factors (i.e., until elective surgery can be performed).
and information on presence or absence of dis-
Answer D is incorrect. There is little utility in
ease is collected at the same point in time. Chi-
continuing to repeat the voiding cystourethro-
square analysis may reveal signicant associa-
gram (VCUG) in this patient. She is already
tions based on data gathered in a cross-sectional
developed (Tanner stage IV) and voiding well
survey, which in turn can suggest causal rela-
(no residual urine on VCUG, no inconti-
tionships. However, only prospective studies can
nence, and no other urinary complaints). If
actually prove causation.
the patient was prepubescent or had voiding
Answer C is incorrect. Incidence refers to the dysfunction, such as problems with lling and
number of new cases of a disease or condition emptying the bladder, yearly VCUG would be
in a population per unit of time. more appropriate.
Answer E is incorrect. Relative risk reects the Answer E is incorrect. Video urodynamics
strength of association between exposure to a are not necessary in this case. In general, uro-
risk factor and development of a disease or con- dynamics are the study of storage and voiding
dition. It is calculated in cohort studies. function of the lower urinary tract. Video uro-
dynamics are a way to gather data on voiding
27. The correct answer is B. The patient has high- and lling pressures and a way to capture radio-
grade reux that has persisted past puberty. Al- graphic evidence of function. This test is more
though there is much debate over the manage- useful in situations where voiding dysfunction
ment of VUR in younger patients, the situation is suspected. This patient does not have dif-
in this case is much clearer. High-grade reux culty voiding. The test is more often used in
in females that persists into adulthood with no the peri-potty training period, or if other signs
clear secondary cause should be managed sur- or symptoms suggest voiding abnormalities.
gically. Surgical correction in this population
is favored because of the increased likelihood 28. The correct answer is E. Impetigo is a bacte-
of the complications of UTIs during preg- rial skin infection commonly caused by either
FULL-LENGTH EXAMS

nancy. During pregnancy, physiologic relax- Staphylococcus aureus or group A Streptococ-


ation of the ureter allows for more urinary sta- cus. It characteristically results in crusting skin
sis and makes women with bacteriuria much lesions that are often nonpainful and nonpru-
more susceptible to developing pyelonephritis. ritic. Characteristically it starts with a tiny vesi-
Untreated upper tract infections are associated cle or pustule that rapidly develops into a
with morbidity for both the mother and the fe- honey-colored crusted plaque. The infection
tus, including premature labor, low birth may be spread to other parts of the body by
weight, preeclampsia, and maternal anemia. contact with ngers, clothing, towels, or other
This patient is also a candidate because she is materials that come into contact with the im-
already exhibiting the feared complication of petiginous lesions. For patients with a limited
untreated VUR, which is renal failure (creati- number of lesions, treatment with topical anti-
nine level 1.5 mg/dL). biotic (mupirocin) is preferred; for more wide-
spread, severe cases, oral antibiotics may be
Answer A is incorrect. Dimercaptosuccinic
used (erythromycin, cephalexin, or dicloxacillin).
acid scans are used to assess the extent of renal
scarring that can result from episodes of pyelo- Answer A is incorrect. Acyclovir is not indi-
nephritis. The test is useful in situations where cated for the treatment of bacterial skin in-
reux surgery is more questionable or where fections. Acyclovir is commonly used to treat
Test Block 1

evidence of organ damage is required before common viral infections such as herpes sim-
proceeding to surgical management. plex, varicella zoster virus (chickenpox), or
herpes zoster.
Answer C is incorrect. Given this patients ele-
vated creatinine, infection despite prophylaxis,
Test Block 1 Answers 497

Answer B is incorrect. Changing facial soaps Answer C is incorrect. An electroencephalo-


will not treat a bacterial skin infection. Al- gram would be useful if there were reason to
though milder facial soaps may reduce subse- suspect that the mans symptoms were due to a
quent irritation of the skin, antibiotic therapy seizure disorder. However, there is no history of
is still required for cases of impetigo. uncontrollable movements, loss of conscious-
ness, or sensory disturbance other than the par-
Answer C is incorrect. Topical steroids, such
esthesias indicative of carpal tunnel syndrome.
as hydrocortisone, are not routinely indicated
It would be very unusual for a focal simple sei-
for the treatment of bacterial skin infections.
zure to present in this manner, moving it far
Topical steroids are more commonly used for
lower on the differential than carpal tunnel or
conditions such as contact dermatitis, sebor-
stroke.
rhea dermatitis, or atopic dermatitis (eczema).
Answer E is incorrect. MRI of the head would
Answer D is incorrect. Intravenous vancomy-
be one modality to look for a stroke, and is
cin is not indicated for the treatment of impe-
more sensitive than CT in identifying early
tigo. For severe, widespread impetigo, a course
ischemic strokes. However, there is nothing in
of oral antibiotics is generally sufcient.
the stem to suggest an acute pathology, and in
Answer F is incorrect. Tretinoin cream is not this case the test would be a waste of resources.
indicated for the treatment of bacterial skin in- More importantly, the clinical diagnosis of car-
fections; it is typically used in the treatment of pal tunnel is most likely, and an MRI of the
acne vulgaris. head would not be indicated.

29. The correct answer is D. The man has signs and 30. The correct answer is C. The patient presents
symptoms of carpal tunnel syndrome, entrap- with typical features of generalized anxiety dis-
ment of the median nerve characterized by pain order. The diagnostic criteria include excessive
and paresthesias in the medial portion of the anxiety and worry occurring more days than
palm. Carpal tunnel is often a clinical diagnosis, not for at least 6 months about a number of
although several objective examination ndings events or activities. Additionally, at least three
and studies can be of use, including nerve con- of the following six symptoms must be present:

FULL-LENGTH EXAMS
duction studies by electromyography. restlessness, fatigue, difculty concentrating, ir-
Answer A is incorrect. Angiography is used in ritability, muscle tension (e.g., headaches), and
the setting of an acute ischemic stroke, when sleep disturbance. The symptoms typically
identifying the vascular anatomy would change cause clinically signicant distress or impair-
medical or surgical management. This invasive ment in social, occupational, or other areas of
maneuver is far too risky in a patient whose functioning.
symptoms are not very likely to be attributable Answer A is incorrect. Acute stress disorder is
to stroke; in such a patient if stroke were more characterized as excessive and persistent anxi-
likely than carpal tunnel syndrome (the correct ety, nightmares, agitation, and sometimes de-
diagnosis), then a CT scan of the head would pression following an extraordinary life event
be a better initial study. (war, natural disaster). It is similar to posttrau-
Answer B is incorrect. CT of the head would matic stress disorder but is less severe and in-
be useful to determine if the patients symp- volves a shorter time frame.
toms had a cerebrovascular etiology. However, Answer B is incorrect. This patient has no ex-
the fact that he reports chronic intermittent posure to an obvious stressor; therefore, the di-
numbness and tingling that occasionally wake agnosis of adjustment disorder with anxiety is
him at night, and the fact that he has weakness not the most appropriate one.
Test Block 1

and atrophy in the distribution of the median


Answer D is incorrect. Although the patient
nerve distal to the carpal tunnel are highly sug-
presents with several physical complaints (e.g.,
gestive of carpal tunnel syndrome. While this
headaches, stomachaches, palpitations), her
man clearly has risk factors for stroke, the more
anxiety is not conned to having a serious ill-
likely diagnosis is carpal tunnel syndrome.
ness, as in hypochondriasis.
498 Section II: Full-Length Examinations Answers

Answer E is incorrect. There is no evidence Answer A is incorrect. A cerebellar hemi-


that the patient is malingering. Malingering sphere lesion would present with ipsilateral
involves the voluntary production of psycho- limb ataxia. Hemispheric lesions are character-
logical and physical symptoms to accomplish a ized by decomposition of movement, dysme-
specic goal (receive compensation, legitimize tria and rebound, intention tremor, or kinetic
taking a sick day from work). In this situation, tremors (presenting in motion). Hemispheric
the patient enjoys her work and does not ap- cerebellar dysfunction is best ascertained clini-
pear to have any ulterior motives when discuss- cally via the nger-to-nose and heel-to-knee tests.
ing her symptoms.
Answer C is incorrect. The classic triad of
clinical features in normal pressure hydro-
31. The correct answer is D. The patient is suffer-
cephalus includes gait disturbance, urinary
ing from allergen-induced anaphylaxis, a type I
incontinence, and cognitive disturbance.
hypersensitivity reaction due to diffuse release of
These features are believed to be derived from
vasoactive amines (i.e., histamine). A -blocker
dysfunction of the periventricular white mat-
would play no role in resuscitating this patient,
ter tracts, especially of the frontal lobe con-
and would worsen the patients hypotension and
nections. A patient with hydrocephalus would
bronchospasm. In fact, patients chronically on
more likely present with a frontal ataxia, also
-blocker therapy undergoing an anaphylactic
known as a magnetic gait or gait apraxia with
reaction may be refractory to treatment with epi-
impaired gait initiation. There is the appear-
nephrine, and in these patients glucagon should
ance of the patients feet being stuck to the
be administered, as it has inotropic and chrono-
oor, with short steps, decreased stride length
tropic effects not mediated by -receptors.
and height, a broadened base, and outwardly
Answer A is incorrect. Albuterol (a rotated feet.
-adrenergic agonist) can assist in relaxing the
Answer D is incorrect. Menieres disease is as-
airway and improving ventilation.
sociated with episodic vertigo, sensorineural
Answer B is incorrect. Diphenhydramine (pri- hearing loss, an aura of fullness or pressure in
marily an H1 histamine blocker) is useful to the ear or side of the head, and tinnitus. The
counteract the effects of histamine. episodic vertigo can be followed by a period of
FULL-LENGTH EXAMS

nausea and unsteadiness for several hours.


Answer C is incorrect. Epinephrine is the
treatment of choice for severe anaphylactic Answer E is incorrect. Patients with vestibular
shock because it reverses the severe hypoten- neuronitis present with rapid onset of a severe
sion and bronchospasm. vertigo along with nausea, vomiting, and gait
instability. The physical examination is signi-
Answer E is incorrect. Methylprednisolone (a
cant for a spontaneous vestibular nystagmus that
potent corticosteroid) is useful to limit the in-
is unilateral, horizontal, or horizontal-torsional,
ammatory response in severe allergic reactions.
and that is suppressed with visual xation and
Answer F is incorrect. Ranitidine, an antago- does not change direction with gaze. The beat-
nist of the H2 histamine receptor, can be used ing of the fast phase is away from the affected
to counteract the effects of histamine on myo- side. This patient does not exhibit nystagmus.
cardial and peripheral vascular tissue in pa-
tients experiencing anaphylactic shock. 33. The correct answer is C. This case describes
the usual mechanism of injury for a meniscal
32. The correct answer is B. The cerebellum inte- tear. Pain, catching, locking, and giving way
grates and coordinates sensory perception and are common symptoms of a torn meniscus.
motor output. Therefore, cerebellar lesions do The joint line tenderness and effusion further
Test Block 1

not cause paralysis, but typically cause ne mo- suggest meniscus tear as the cause of the pa-
tor movement disorders, as well as decits in tients symptoms.
equilibrium, posture and motor learning. The
patient presents with gait ataxia, which is typi-
cally of a vermis lesion.
Test Block 1 Answers 499

Answer A is incorrect. A chondromalacia pa- carditis. Ventricular aneurysms place patients


tella is a chronic overuse injury with an insidi- at risk for clot and resulting stroke, as well as
ous onset that causes anterior knee pain. It is rupture and resulting tamponade.
related to problems with patellar tracking.
Answer E is incorrect. Ventricular free wall
Answer B is incorrect. A medial collateral rupture is a fatal complication of MI that gen-
ligament injury is possible with the action de- erally arises within the rst week. Rupture
scribed and can present with joint line tender- leads to rapid tamponade, circulatory collapse,
ness. However, the mechanical symptoms of and death.
catching and locking are more consistent with
meniscal injury. 35. The correct answer is D. The best screening
tests are inexpensive, easy to administer, and
Answer D is incorrect. Pes anserine bursitis
safe. Additionally, they have a high sensitivity
can cause pain on the medial aspect of the
and specicity, a high positive predictive value,
proximal tibia. This is more likely to develop
and treatment after screening is more effective
insidiously and would not cause joint line ten-
than subsequent treatment without screening
derness.
(i.e., screening leads to an efcacious interven-
Answer E is incorrect. Quadriceps tendonitis tion).
can cause giving way, but not catching or lock-
Answer A is incorrect. For example, while im-
ing, which implies an intra-articular pathology.
munologic assays for HIV based on blood sam-
It would also not be associated with joint line
ples have a higher specicity than those based
tenderness.
on cheek swabs, such a relationship is not nec-
essarily generalizable to all diseases.
34. The correct answer is B. Dresslers syndrome
is an immunologically mediated late complica- Answer B is incorrect. Behavioral counseling
tion of MI that is characterized by fever, pleu- is an important component of prevention, but
ritis, and pericarditis. It generally arises weeks this is a separate endeavor from screening. Ap-
after an MI and resolves on its own. A remit- proaches to prevention include immunization,
ting and relapsing course is not uncommon. It chemoprevention, behavioral counseling, and

FULL-LENGTH EXAMS
may be treated with NSAIDs or corticosteroids. screening.
Notably, the ST-segment elevation observed in
Answer C is incorrect. Sensitivity is the prob-
pericarditis is more diffuse than in MI, involv-
ability of a person with disease testing positive,
ing limb and precordial leads and there is a
while specicity is the probability of a person
noted diffuse PR segment depression. Also, the
without disease testing negative. While sensi-
ST-segment elevation is concave upward and is
tivity is often discussed as the more important
associated with upright T waves.
characteristic of a screening test (because a
Answer A is incorrect. Coronary artery aneu- high sensitivity is a rule-out for disease), a
rysm is a complication of Kawasakis disease, a high specicity will also make the screening
systemic vasculitis that is more commonly en- tool more precise. The best screening tools will
countered in the pediatric population. have both.
Answer C is incorrect. Although ST-segment Answer E is incorrect. A low prevalence would
changes are observed in this patient, they are lead to a relatively lower positive predictive
diffuse, involving both limb and precordial value, the probability that someone with a posi-
leads. In MI, ST-segment elevation occurs in tive test actually has the disease. Higher disease
physiologically related leads, representing vas- prevalence leads to a higher positive predictive
cular territories of the corresponding coronary value.
Test Block 1

arteries.
36. The correct answer is E. This woman has a
Answer D is incorrect. Ventricular aneurysm
clinical picture and laboratory ndings consis-
is a potential complication of MI but does not
tent with pheochromocytoma. The classic triad
tend to present with fever, pleuritis, and peri-
of symptoms in a patient with a pheochromo-
500 Section II: Full-Length Examinations Answers

cytoma includes episodic headache, sweating, typically presents with nausea and/or vomiting,
and tachycardia. In addition, one expects ele- and patients may have a distended abdomen
vated urine and plasma catecholamines. Prior along with high-pitched bowel sounds. Supine
to surgical removal of tumor, pharmacotherapy and upright x-rays of the abdomen are useful
is used to better control the patients blood in making the diagnosis because dilated loops
pressure. Because a pheochromocytoma se- of small bowel can be seen, along with air-uid
cretes catecholamines, it is recommended that levels and lack of gas in the colon. Some pa-
an -adrenergic blocker such as phenoxyben- tients with partial SBO can initially be treated
zamine is started. After approximately 2 weeks conservatively with close observation, intrave-
of therapy, the patient should be ready for sur- nous uids, and a nasogastric tube for proximal
gery. A -blocker should be given only after decompression. However, patients with perito-
phenoxybenzamine. neal signs should be taken to the operating
room for surgical decompression and repair.
Answer A is incorrect. Nitroprusside is indi-
cated in hypertensive emergencies. There is no Answer A is incorrect. The acute onset of dif-
indication that this woman is having a hyper- fuse abdominal pain and emesis in a patient
tensive emergency, so nitroprusside would not with a history of previous abdominal surgeries
be our rst-line treatment. supports a diagnosis of SBO more than colon
cancer. Although tumors and cancers of the
Answer B is incorrect. Although surgery to
small bowel can cause obstruction, it is un-
remove the tumor is indicated, it is recom-
likely that colon cancer would produce the
mended that the patient is rst medically man-
history, physical examination ndings, and ra-
aged using an -adrenergic blocker to control
diographic ndings noted in this case.
blood pressure.
Answer B is incorrect. Mesenteric ischemia
Answer C is incorrect. Thiazide diuretics,
may present with abdominal pain, nausea, and
such as hydrochlorothiazide, are a good rst-
vomiting; however, the radiographic ndings
line treatment in patients with essential hyper-
are not typically seen in a patient with mes-
tension. However, this patient has a pheochro-
enteric ischemia. In addition, the majority of
mocytoma, a cause of secondary hypertension,
cases of mesenteric ischemia occur in older
FULL-LENGTH EXAMS

so hydrochlorothiazide would not be the most


patients who have cardiac abnormalities or ath-
appropriate treatment.
erosclerosis.
Answer D is incorrect. A -blocker, such as pro-
Answer C is incorrect. Patients with a diagno-
pranolol, should never be the rst antihyperten-
sis of pancreatitis may present with diffuse ab-
sive started in patients with a suspected diagnosis
dominal pain, nausea, and vomiting; however,
of pheochromocytoma. Peripheral -adrenergic
the radiographic ndings are not typically seen
receptors cause vasodilatation, and blocking
in a patient with pancreatitis.
these receptors with unopposed -adrenergic
receptor stimulation can lead to vasoconstric- Answer D is incorrect. A patient with a perfo-
tion, or increased blood pressure in the setting rated gastric ulcer typically presents with acute,
of decreased heart contractility and heart rate severe abdominal pain and free intraperitoneal
due to the 1-adrenergic effects on the heart. air. This radiograph does not show free intra-
This would essentially mean that a weaker heart peritoneal air but rather shows dilated loops of
would be pumping against a higher afterload. small bowel, air-uid levels, and absence of gas
However, once an -blocker such as phenoxy- in the colon. These signs/symptoms are consis-
benzamine has been started, a -blocker may be tent with a diagnosis of SBO rather than a per-
added to decrease tachycardia. forated gastric ulcer.
Test Block 1

37. The correct answer is E. The most common 38. The correct answer is C. Chorioamnionitis is
causes of SBO include peritoneal adhesions a condition that can affect pregnant women in
(from previous abdominal surgery), hernias,
and cancer or tumors. The abdominal pain
Test Block 1 Answers 501

which the chorion and amnion (the mem- effects and a large likelihood of causing severe
branes that surround the fetus) and the amni- orthostatic hypotension, with a decreased risk of
otic uid (in which the fetus oats) become extrapyramidal symptoms.
infected. Genital tract infections have been
Answer B is incorrect. Clozapine is an anti-
implicated as a cause of increased rates of pre-
psychotic medication that improves the nega-
term labor. In addition, Chlamydia infection
tive symptoms of schizophrenia but can cause
increases the rate of chorioamnionitis, which
agranulocytosis in 1% to 2% of patients taking
also causes a rise in the rate of preterm labor. It
it. As a result, patients must agree to a weekly
is postulated that the inammatory response of
blood draw to check WBC count throughout
the mother or fetus is the mechanism of pre-
therapy and for 4 weeks after stopping the drug.
term labor induced by infectious causes.
Agranulocytosis is an absolute contraindication
Answer A is incorrect. Nifedipine is also an for continuing the drug.
effective tocolytic. The mechanism of action
Answer C is incorrect. Haloperidol is a high-
involves blockage of inux of calcium into the
potency typical antipsychotic medication that
intracellular space through the cell membrane
is associated with more extrapyramidal symp-
and sarcoplasmic reticulum.
toms than the low-potency agents (e.g., chlo-
Answer B is incorrect. Hydration is often used rpromazine). Typical antipsychotics can cause
as a method to forestall premature labor. ADH extrapyramidal symptoms, hyperprolactinemia,
is manufactured in the hypothalamus along anticholinergic effects, and sedation, and are
with oxytocin and is hypothesized to exhibit more associated with neuroleptic malignant
some cross-reactivity with oxytocin receptors. syndrome than are atypical antipsychotics.
Thus, hydrating a patient in preterm labor may
Answer D is incorrect. Olanzapine is an atypi-
decrease ADH levels and may delay cervical
cal antipsychotic associated with increased
change and decrease contractions.
triglycerides, somnolence, hyperglycemia, and
Answer D is incorrect. Complications of weight gain. The drug itself is particularly ef-
preeclampsia include fetal distress or stillbirth, fective for negative symptoms of schizophrenia
intrauterine growth restriction, oligohydram- and for agitation.

FULL-LENGTH EXAMS
nios, or progression to eclampsia. Delivery is
Answer E is incorrect. The adverse effects of
the denitive treatment for preeclampsia, and
quetiapine include somnolence, orthostatic
therapeutic delivery necessitated before ma-
hypotension, and dizziness. It may increase risk
turity is reached can lead to complications of
of cataracts, and frequent eye examinations are
prematurity in the infant. However, premature
recommended with its use. It also has rare ad-
labor is not caused by preeclampsia.
verse effect of QT prolongation, but this effect
Answer E is incorrect. NSAIDs have been is more commonly seen with ziprasidone.
clinically demonstrated to be effective tocolyt-
ics with minimal maternal adverse effects. The 40. The correct answer is F. This patient most
mechanism of action is decreased production likely has neurologic endemic cretinism,
of prostaglandins, which leads to decreased in- caused by maternal iodine deciency during
tracellular levels of calcium and inhibition of early pregnancy and characterized by mental
uterine contractions. However, use of NSAIDs retardation, spasticity, abnormal gait, and deaf-
may cause multiple fetal complications, in- ness. Many patients have complete deafness,
cluding premature closure of the ductus arte- and these individuals are typically mute as
riosus and oligohydramnios. well. Goiter is uncommon, and patients typi-
cally are euthyroid. This disorder is most com-
Test Block 1

39. The correct answer is F. Ziprasidone is the monly found in India, China, Indonesia, Bo-
atypical antipsychotic that is most associated livia, Ecuador, and Peru.
with QT prolongation.
Answer A is incorrect. Autoimmune thyroiditis
Answer A is incorrect. Chlorpromazine is a typ- is a highly uncommon cause of congenital hy-
ical antipsychotic with severe sedative adverse
502 Section II: Full-Length Examinations Answers

pothyroidism and can cause transient hypothy- lymphoma (CTCL). This phase of disease usu-
roidism in infants. It is the cause of Hashimo- ally occurs after the more localized stages
tos thyroiditis in adults. (patch, plaque, and tumor) have persisted for
several years. Unfortunately, these stages may
Answer B is incorrect. While iodine deciency
go undiagnosed or be misdiagnosed as atopic
is a possible cause of hypothyroidism in chil-
dermatitis or other skin disease and may not be
dren, this patient would likely have an elevated
recognized as malignancy. Szarys syndrome
thyroid-stimulating hormone, low thyroxine, a
is a term used to describe a systemic form of
goiter, and many of the signs and symptoms of
CTCL, during which Szary cells are visible
hypothyroidism (including fatigue, cold intol-
in the peripheral blood and the entire skin sur-
erance, coarse skin, and edema).
face is affected. Szary cells are recognizable
Answer C is incorrect. Congenital errors in for their convoluted or cerebriform nuclei and
production of thyroid hormone, thyroid-binding scant cytoplasm.
globulin, and iodide transport molecules ac-
count for approximately 10% of cases of con- 42. The correct answer is A. Palpable abdominal
genital hypothyroidism. Patients would not be masses may represent several forms of malig-
expected to have normal thyroxine and thyroid- nancy in children, including neuroblastoma,
stimulating hormone levels. Wilms tumor, and Burkitts lymphoma. Chil-
dren from Africa, where infection with Epstein-
Answer D is incorrect. Hypopituitarism can
Barr virus may lead to an endemic form of
cause hypothyroidism due to low levels of
Burkitts lymphoma, are particularly prone to
thyroid-stimulating hormone release. Patients
this disease. The numerous benign macrophages
often demonstrate other manifestations of hy-
with abundant clear cytoplasm distributed
popituitarism, such as adrenal insufciency or
among tumor cells give the tissue biopsy a
diabetes insipidus.
starry sky appearance on low-power light mi-
Answer E is incorrect. Iodine deciency late croscopy. Burkitts lymphoma is associated with
in pregnancy and after birth is considered the the t(8:14) translocations of the c myc gene.
cause of myxedematous endemic cretinism.
Answer B is incorrect. Although acute lym-
Both neurologic and myxedematous cretinism
FULL-LENGTH EXAMS

phocytic leukemia occurs primarily in chil-


are part of the spectrum of endemic cretinism,
dren, it also accounts for 20% of adult leuke-
which causes irreversible changes in mental
mias. Patients may complain of malaise, weight
development in areas of endemic hypothy-
loss, bone pain, and hemorrhage. Clinical ex-
roidism. Myxedematous endemic cretinism is
amination may reveal lymphadenopathy and/
found predominantly in the Congo region of
or hepatosplenomegaly in half of the patients.
Africa. Disorders include mental retardation,
Biopsy in L1 type would be expected to re-
short stature, and hypothyroidism.
veal small lymphoblasts with scant cytoplasm,
Answer G is incorrect. Thyroid dysgenesis is condensed nuclear chromatin, and indistinct
the most common cause of congenital hypo- nuclei. Biopsy in L2 type would be expected
thyroidism, which is in turn the most common to reveal larger lymphoblasts with a moderate
treatable cause of mental retardation. Neo- amount of cytoplasm, dispersed chromatin,
nates with thyroid dysgenesis often are not af- and multiple nucleoli. Biopsy in L3 type would
fected by hypothyroidism due to maternal thy- be expected to reveal lymphoblasts with deep
roxine that crossed the placenta. The patient in cytoplasmic basophilia with prominent cyto-
the question would be hypothyroid if this were plasmic vacuolation.
his diagnosis.
Answer C is incorrect. Acute myelocytic leu-
Test Block 1

kemia is more common in adults, presenting


Questions 41 and 42 with fatigue, hemorrhage, bruising, lymphade-
nopathy, and hepatosplenomegaly. This condi-
41. The correct answer is M. This patients pre- tion is associated with leukocytosis, blasts in pe-
sentation is consistent with Szarys syndrome, ripheral blood, and the presence of Auer rods.
the most advanced stage of cutaneous T-cell
Test Block 1 Answers 503

Answer D is incorrect. The median age for pre- Answer H is incorrect. NHLs commonly pres-
sentation of chronic lymphocytic leukemia is 60 ent with hepatosplenomegaly, widespread pal-
years. When symptomatic, this condition clas- pable adenopathy, and abdominal masses. Man-
sically presents with lymphadenopathy, hepato- tle cell lymphoma is an unusual type of NHL
splenomegaly, and lymphocytosis. Lymph node predominantly described in older men, present-
biopsy would be expected to show diffusely ef- ing with generalized lymphadenopathy, spleno-
faced nodal architecture with occasional resid- megaly, and peripheral blood involvement. The
ual naked germinal centers. The nodal inltrate t(11:14) translocation is usually present involv-
is usually composed of mature-appearing, small ing the cyclin D1 gene (bcl-1). Histology would
lymphocytes, with an admixture of prolympho- be expected to show lymphoid cells with slightly
cytes and paraimmunoblasts. irregular or cleaved nuclei.
Answer E is incorrect. Chronic myelogenous Answer I is incorrect. NHLs commonly present
leukemia (CML) typically affects patients older with hepatosplenomegaly, widespread palpable
than 50 years. CML is a myeloproliferative dis- adenopathy, and abdominal masses. Mucosal
order that is often discovered on routine blood associated lymphoid tissue lymphoma most
tests, where leukocytosis and thrombocytosis commonly involves the stomach. Histology
are discovered. When symptomatic, fatigue, would be expected to show dense, monoto-
weight loss, malaise, abdominal discomfort, nous, lymphoid inltrate in the lamina propria,
and bleeding due to platelet dysfunction are and pale-staining cells that represent marginal
common. The t(9;22) translocation creates the zone B lymphocytes.
Philadelphia chromosome, which is often seen
Answer J is incorrect. Primary neuroblastoma
in CML. The blast crisis refers to the prolifera-
most commonly presents as a painless abdomi-
tion of blast cells. Bone marrow biopsy in the
nal mass, hypertension, respiratory distress,
blast crisis of this condition would be expected
Horners syndrome, or cord compression. Of-
to reveal large foci or clusters of blasts.
ten, however, neuroblastoma is not diagnosed
Answer F is incorrect. Ewings sarcoma may until after it has metastasized. Low-grade fe-
develop in any bone or soft tissue but is most ver, fatigue, and failure to thrive (which is evi-
common in the at and long bones. This con- denced by his small size) occur with metastatic

FULL-LENGTH EXAMS
dition is characterized by localized pain and disease. Urinary catecholamines are elevated
swelling rather than abdominal or cutaneous in neuroblastoma, but denitive diagnosis re-
involvement. Histology reveals monotonous quires biopsy from two separate sites, which
sheets of small round blue cells with hyper- would be expected to show neuroblasts and
chromatic nuclei. Schwannian cells, as well as rosettes, or clumps
of tumor cells with indistinct borders.
Answer G is incorrect. Non-Hodgkins lym-
phomas (NHLs) commonly present with hep- Answer K is incorrect. Pheochromocytoma is a
atosplenomegaly, widespread palpable ad- catecholamine-secreting tumor that classically
enopathy, and abdominal masses. Follicular presents with the triad of episodic headache,
lymphoma is one of the more common NHLs. tachycardia, and sweating. Half of patients
Because follicular lymphoma is generally slow have episodic hypertension. Biopsy should be
growing or indolent, patients tend to present avoided when this diagnosis is suspected be-
with generalized disease. Follicular lymphoma cause uncontrolled tumor manipulation may
typically presents in older patients. Histology of result in a surge of catecholamine release.
affected lymph nodes reveals relatively small,
Answer L is incorrect. Anaplastic large cell
well-differentiated cells organized in a nodular
lymphoma (ALCL) is divided into two types
pattern. The classic genetic analysis demon-
Test Block 1

of disease: primary cutaneous and primary sys-


strates the t(14:18) translocation with overex-
temic. Primary cutaneous ALCL presents in
pression of bcl-2. Follicular lymphoma does
the skin without evidence of extracutaneous
not classically present with atopic dermatitis.
disease at the time of diagnosis. Cutaneous in-
504 Section II: Full-Length Examinations Answers

ltrates involve both upper and deep dermis, trapelvic pathology. While both cases present
with small, reactive lymphocytes at the periph- with urinary complaints, both are explained
ery of the lesions. by diseases better evaluated by tests other than
CT. In addition, radiation should be avoided in
pediatric patients when possible.
Questions 43 and 44
Answer E is incorrect. Cystoscopy is rarely
43. The correct answer is G. The boy should be indicated for evaluation of UTI in women,
suspected to have type 1 diabetes mellitus. Pre- even in patients with chronic recurrent UTIs.
sentation often includes urinary symptoms of Women with a history of childhood UTI,
polyuria and polydipsia because of osmotically stones, or painless hematuria should be evalu-
active glucose that spills into the urine. Chil- ated with cystoscopy. Most men diagnosed
dren will often complain of wetting the bed at with UTI without an obvious source should
night. Human leukocyte antigen-DR3 is associ- undergo urologic evaluation.
ated with type 1 diabetes. Measuring the fast-
ing blood glucose level would conrm the di- Answer F is incorrect. Digital rectal exami-
agnosis. nation, along with prostate-specic antigen,
is used as a screening tool for prostate cancer.
44. The correct answer is K. This description is Screening is recommended for men >50 years
classic for uncomplicated lower UTI. She is old.
not exhibiting fevers, chills, or ank pain that Answer H is incorrect. Pap smear is used to
would suggest an upper tract infection, namely screen for cervical cancer and precancerous
pyelonephritis. The urinalysis will screen for lesions. Screening should be done annually in
the diagnosis, and the culture will conrm and all sexually active women or those at least 21
give bacterial sensitivities. years old.
Answer A is incorrect. -Human chorionic Answer I is incorrect. Prostate-specic antigen,
gonadotropin and -fetoprotein are appropriate along with digital rectal examination, is used
in the evaluation and diagnosis of malignant as a screening tool for prostate cancer. The boy
germ cell tumors. In women, germ cell tumors in this case is too young to be considered for a
FULL-LENGTH EXAMS

most often present with abdominal or pelvic diagnosis of prostate cancer, which usually oc-
pain. The girl in this case does not have such curs in men >50 years old.
symptoms, making malignant germ cell tumor
an unlikely diagnosis. Answer J is incorrect. Salt-load response, also
known as an aldosterone suppression test, is
Answer B is incorrect. If bladder cancer is used to diagnose primary hyperaldosteronism.
suspected, a bladder biopsy may be part of the Patients usually present with hypertension and
diagnostic evaluation; however, more appro- may have muscle weakness secondary to hy-
priate initial testing includes ultrasound, CT pokalemia. In this test, the urine aldosterone
scan, and cystoscopy for direct visualization level is measured after a odium load; if sodium
of the mass. Gross hematuria is a presenting levels fail to be suppressed, primary hyperal-
symptom in 80%90% of patients with bladder dosteronism can be diagnosed.
cancer.
Answer L is incorrect. The effect of water
Answer C is incorrect. A complete blood cell deprivation on urine osmolality is used to dis-
count may reveal an increased WBC count tinguish primary polydipsia from diabetes in-
in the case of UTI but is nonspecic for the sipidus in the case of polyuria. Both of these
source of infection. A complete blood cell patients have a more likely reason for increased
Test Block 1

count is not essential in a new presentation of urination and should be worked up accord-
diabetes and would not be of value in the diag- ingly.
nostic work-up.
Answer M is incorrect. Bacterial vaginosis of-
Answer D is incorrect. CT scan of the pelvis ten presents with increased vaginal discharge
should be done in the case of suspected in-
Test Block 1 Answers 505

and pruritus. It is diagnosed by microscopic Answer C is incorrect. Biliary atresia is a con-


evaluation of a wet mount preparation of vagi- genital defect that causes an obstructive jaun-
nal specimen, which reveals clue cells. dice, resulting in acholic stools.
Answer E is incorrect. Cephalohematomas are
Questions 45 and 46 subperiosteal collections of blood that are pres-
ent in 1%2% of newborns. They can cause
45. The correct answer is K. This neonate has jaundice as they decompose.
jaundice secondary to Rh incompatibility. The
mother was exposed to Rh antigen either via Answer F is incorrect. Crigler-Najjar syn-
her prior pregnancy or blood transfusions, and drome results in a severe unconjugated biliru-
she developed antibodies to Rh factor. Mater- binemia that is accompanied by normal stool
nal Rh antibody crosses the placenta and de- color.
stroys fetal RBCs. The hemolysis results in un- Answer G is incorrect. Gilberts syndrome is
conjugated hyperbilirubinemia, anemia, and generally diagnosed in adolescents that present
increased hematopoiesis. The diagnosis is sup- with mild unconjugated jaundice.
ported by a positive Coombs test. Giving Rh-
Answer H is incorrect. Hereditary spherocy-
negative women Rh0(D) immune globulin
tosis can cause an unconjugated jaundice sec-
when they are exposed to Rh-positive blood
ondary to hemolysis.
can prevent Rh isoimmune hemolytic disease
of the newborn. Answer I is incorrect. Neonatal hepatitis is a
cause of jaundice that is generally maternally
46. The correct answer is D. Breast-feeding jaun- transmitted and thus the mother should be se-
dice is the most common cause of nonphysio- ropositive and may have passed IgG antibodies
logic jaundice. It occurs in neonates who are to the neonate.
being exclusively breast-fed, usually because of
Answer J is incorrect. Physiologic jaundice
poor intake. Decreased milk intake leads to in-
generally occurs in premature infants, is mild,
creased enterohepatic circulation and abnor-
and can be overcome with phototherapy.
mally high reabsorption of bilirubin normally

FULL-LENGTH EXAMS
excreted in the feces. Jaundice is usually appar- Answer L is incorrect. Sepsis can cause neo-
ent on day 3 of life. natal jaundice secondary to hemolysis from
Answer A is incorrect. 1-Antitrypsin de- marked inammation.
ciency can cause neonatal hepatitis, in addi- Answer M is incorrect. Toxoplasmosis can
tion to the pulmonary manifestations. cause a severe congenital infection but is not
Answer B is incorrect. ABO incompatibility generally associated with jaundice.
can cause a generally milder neonatal jaundice
that is triggered almost exclusively in type A or
B neonates born to type O mothers.

Test Block 1
This page intentionally left blank
Test Block 2

507
508 Section II: Full-Length Examinations Questions

Q U E ST I O N S

1. An emergency cesarean section due to persis- (A) Amiodarone


tent late decelerations is performed on an afe- (B) Atorvastatin
brile, Group B Streptococcus-negative, 17-year- (C) Digoxin
old girl at 42 weeks gestation. A 4600-g (D) Metoprolol
(10.1-lb) girl is delivered. The amniotic uid is (E) Nitroglycerin
meconium stained. The neonates blood pres-
sure is 50/30 mm Hg and heart rate is 100/min, 4. A 37-year-old G5P5 woman with diabetes is
and the infant has not yet taken a breath. She immediately postpartum from delivery of a 4.5-
is cyanotic and limp on examination. What is kg (10-lb) baby boy at 37 weeks gestation.
the most appropriate next step in manage- Time elapsed from the onset of labor to deliv-
ment? ery of the placenta was 4.5 hours. Delivery was
complicated by a second-degree perineal tear.
(A) Give ampicillin and gentamicin
Postpartum vital signs are within normal limits
(B) Obtain x-ray of the chest
when the patient begins to hemorrhage vagi-
(C) Provide articial ventilation with bag-mask
nally. Estimated blood loss is 200 mL so far.
(D) Provide supplemental oxygen
Bimanual examination reveals a soft, enlarged,
(E) Suction trachea
boggy uterus. Which of the following is the
most appropriate rst step in treatment?
2. Pain disorder is a psychological syndrome that
is considered part of the family of somatoform (A) Bimanual uterine massage
disorders. Which of the following statements (B) Hysterectomy
regarding pain disorder is true? (C) Internal uterine massage
(D) Intravenous methylergonovine
(A) It occurs more often in males than in fe-
(E) Intravenous oxytocin infusion
males
(F) Thorough speculum examination to evalu-
(B) It often gives the patient some form of sec-
ate for vaginal wall lacerations
ondary gain
FULL-LENGTH EXAMS

(C) Patients are often very in touch with their


5. A 27-year-old man requests an HIV test at his
emotions
local hospital. The patient is a poor historian
(D) Patients often have good insight into their
but complains of a mild nonproductive cough.
disorder
He is breathing rapidly. HIV test is positive and
(E) The diagnosis requires pain or dysfunction
his CD4+ cell count is 186/mm. X-ray of the
at four distinct sites, at least two of which
chest is shown in the image. Which of the fol-
are gastrointestinal, one is neurologic, and
lowing is the most appropriate next diagnostic
one is sexual
step?
3. A 68-year-old man with no previous medical
problems presents to the clinic with shortness
of breath and swelling of his ankles. Symptoms
have progressed over the past few months. The
patient has a regular heart rate and rhythm, an
S4 is heard on cardiac auscultation, and in-
spiratory rales are heard throughout the lower
half of the lung elds. Also, 2+ pitting edema
Test Block 2

is evident over the tibial surfaces. Echocardiog-


raphy reveals an ejection fraction of 60%.
Which of the following is most likely to im-
prove this patients condition?
Test Block 2 Questions 509

(A) FEV1 <40% predicted


(B) Partial arterial oxygen pressure >50
mm Hg
(C) Partial carbon dioxide pressure >50
mm Hg
(D) Respiratory rate of 25/min
(E) The patient is anxious and complains of
feeling like they are gasping for air

8. A 67-year-old man comes into the emergency


department with a 20-minute episode of sub-
sternal chest pain. The man appears to be in
moderate discomfort and indicates that the
pain is in the middle of his chest. Upon ques-
tioning the man says this pain is not new to
him, as he has been suffering this type of pain
Reproduced, with permission, from Hall JB, Schmidt
for the past 3 years and it usually occurs after
GA, Wood LDH. Principles of Critical Care, 3rd edition. meals or at nighttime. He also complains of an
New York: McGraw-Hill, 2005: Figure 48-1. acid taste in his mouth that accompanies this
pain. Which of the following treatments is
(A) Bronchoalveolar lavage and culture most appropriate for this patient?
(B) Bronchoalveolar lavage and serology
(C) Bronchoalveolar lavage uid stained with (A) -Blocker
methamine silver/Giemsa staining (B) Aspirin
(D) Intraoperative biopsy (C) Nissen fundoplication
(E) Intravenous blood draw and serology (D) Nitroglycerin
(E) Omeprazole
6. A 46-year-old Japanese-American woman pre-
sents to her primary care physicians with com- 9. An 83-year-old woman presents to the clinic

FULL-LENGTH EXAMS
plaints of increasing shortness of breath, fatigue, with complaint of a dull lower back pain that is
chest pain, abdominal distension, and a fainting not affected by positional changes. It has been
spell. She has a history of amyloidosis, for which gradually worsening over the past several
she has been repeatedly hospitalized. Her ECG months. Her blood pressure is 161/84 mm Hg
shows low-voltage QRS complexes. Her vital and pulse is 75/min. Cardiac examination is
signs are stable, and her laboratory values are notable for S4. Abdominal examination reveals
normal. Which of the following would be most a pulsatile mass approximately 5.5 cm in diam-
helpful in establishing the correct diagnosis? eter and palpable in the epigastric area. Periph-
eral pulses are normal. Results of sonography
(A) Cardiac enzymes are pending. Which of the following is the
(B) CT scan of the abdomen most likely diagnosis?
(C) Echocardiogram
(D) Recheck potassium (A) Abdominal aortic aneurysm
(E) X-ray of the chest (B) Colonic obstruction
(C) Intestinal arteriovenous malformation
7. A 68-year-old man recently diagnosed with (D) Pancreatitis with pseudocyst
mild chronic obstructive pulmonary disease (E) Peptic ulcer disease
presents to the emergency department in the
Test Block 2

throes of a new exacerbation of symptoms.


Which of the following is a reason to intubate?
510 Section II: Full-Length Examinations Questions

10. A 55-year-old man with a history of diabetes,


hypertension, and atrial brillation presents
with a newly diagnosed aphasia. His word rep-
etition ability and language comprehension are
both impaired, but his uency is preserved.
Where is the lesion responsible for this kind of
aphasia located?
(A) Infarct in the territory of the left posterior
cerebral artery
(B) Stroke in the arcuate fasciculus
(C) Stroke in the lenticulostriate branches of
the middle cerebral artery Reproduced, with permission, from Lichtman MA, Beut-
(D) Stroke involving the inferior division of the ler E, Kipps TJ, Seligsohn U, Kaushansky K, Prchal JT.
middle cerebral artery Williams Hematology, 7th edition. New York: McGraw-
(E) Stroke involving the superior division of Hill, 2006: Plate XXV-32.
the middle cerebral artery (A) Folic acid
(B) Iron
11. An 86-year-old man is brought to the emer-
(C) Vitamin B12
gency department by his daughter because of
(D) Vitamin C
increased bruising of recent onset. The daugh-
(E) Vitamin K
ter has noticed multiple bruises on her fathers
arms and legs over the past few months. The 12. A 26-year-old woman and her husband, her
patient lives by himself and gives no history of only sexual partner, present to her obstetrician
trauma. The patient does complain of pain and seeking help after their third miscarriage in 2
bleeding in his gums as well as increased joint years. She is healthy and does not smoke, drink
pain. He has also noticed a rash on his but- alcohol, or use recreational drugs. She is not
tocks (see image). Vital signs are stable. Labo- currently taking any medications and denies
ratory studies show: any history of sexually transmitted diseases. She
FULL-LENGTH EXAMS

WBC count: 4000/mm is the oldest of seven children, and there is no


Hemoglobin: 10.3 g/dL record of infertility or miscarriage in her fam-
Platelet count: 235,000/mm ily. She has been pregnant three times, losing
Activated partial thromboplastin time: the pregnancies after 5, 6, or 7 weeks. What
27 seconds would be the most appropriate rst step in de-
Prothrombin time: 12 seconds termining the cause of her miscarriages?
Which of the following is the best treatment (A) A 3-month trial of an oral contraceptive
for this patient? pill
(B) A chromosome analysis
(C) A hysterosalpingogram
(D) An exploratory laparotomy
(E) Vaginal and cervical cultures

13. A 24-year-old woman comes to a plastic sur-


geon requesting rhinoplasty. She has had ve
such surgeries in the past 3 years but still be-
lieves that her nose looks hideous. She pro-
Test Block 2

duces photographs of herself that predate her


rst surgery and shows them to the surgeon as
proof that her nose was monstrous. The pa-
tients nose appears totally normal to the sur-
Test Block 2 Questions 511

geon, both in the preoperative images and at 15. A 46-year-old woman presents to the emer-
presentation, and he states this. The patient gency department with acute onset of severe
becomes agitated and demands to have an op- right upper quadrant abdominal pain that radi-
eration, stating that she cannot live with a de- ates to the infrascapular region. Her medical
formed face. Which of the following is the history is signicant for obesity, hypertension,
best next step in management? obstructive sleep apnea, and gastric bypass sur-
gery 2 years ago after which she lost 79 kg (150
(A) Perform the surgery as requested
lb). The patient complains of nausea and vom-
(B) Prescribe a selective serotonin reuptake in-
iting that accompanies the pain. Her tempera-
hibitor
ture is 38.9C (101.2F), blood pressure is
(C) Prescribe valium for agitation
144/88 mm Hg, heart rate is 76/min, and respi-
(D) Reassure the patient that her nose is nor-
ratory rate is 14/min. Abdominal examination
mal and gently refuse to perform the oper-
is signicant for right upper quadrant tender-
ation
ness along with guarding and cessation of in-
(E) Refer the patient for psychotherapy
spired breath on deep palpation of the right
14. A 67-year-old woman visits her physician be- upper quadrant. Which test should be ordered
cause of severe left leg pain, worse with stand- rst for this patient?
ing and walking, starting insidiously 3 weeks (A) Abdominal ultrasound
ago. There has been no trauma to the area. (B) CT scan of the abdomen
The patient also notes that over the past 6 (C) Hepato-iminodiacetic acid scan
months, the tips of her ngers have become in- (D) MRI of the abdomen
creasingly thick, she has lost 2.3 kg (5 lb) unin- (E) X-ray lm of the abdomen
tentionally, and she has been suffering from a
persistent, nonproductive cough. She has a 16. A 32-year-old G1P0 woman who is 26 weeks
60-pack-year smoking history and continues to pregnant presents to her obstetrician for a rou-
smoke two packs per day. Her temperature is tine prenatal visit. The pregnancy has been un-
36.7C (98.1F), blood pressure is 132/81 mm complicated, and the patients weight gain has
Hg, pulse is 75/min and regular, and respira- been appropriate. Her blood pressure has

FULL-LENGTH EXAMS
tory rate is 15/min. Physical examination is sig- ranged from 105/90 to 120/85 mm Hg. Her
nicant for clubbed ngers and pain on palpa- urine has not shown any proteinuria and has
tion of the distal left tibia. X-ray lm of the been contained only traces of glucose for the
chest shows a focal 5-cm mass lesion in the left past two visits. During this visit, urinalysis re-
lower lung that is highly suspicious for bron- veals 1+ glucosuria. The patient undertakes
chogenic carcinoma. Results of bronchoscopy her scheduled 50-g oral glucose tolerance test,
with biopsy are pending. Which of the follow- which shows a serum glucose level of 135 mg/
ing is the most likely histologic type of lung dL. Which of the following is the most appro-
cancer present in this patient? priate next step in managing this womans dia-
betes?
(A) Adenocarcinoma
(B) Bronchoalveolar cell carcinoma (A) Dietary restrictions
(C) Large cell carcinoma (B) Exercise regimen
(D) Small cell carcinoma (C) Insulin injections
(E) Squamous cell carcinoma (D) Oral metformin
(E) Oral sulfonylurea
Test Block 2
512 Section II: Full-Length Examinations Questions

17. A 63-year-old African-American woman with strength in her right arm and leg. MRI of the
osteoarthritis is seen in the ophthalmology head shows a right parietal infarct. In review-
clinic for routine examination. Her last ocular ing her records, it is determined that her new-
examination was 5 years ago. She reports no born screen for inborn errors of metabolism is
problems with her vision since her last exami- abnormal. Which of the following most likely
nation. Her visual acuity is 20/40 in both eyes; led to the patients cerebrovascular accident?
intraocular pressure is 29 mm Hg in the right
(A) Fabrys disease
eye and 31 mm Hg in the left eye. Gonioscopic
(B) Homocystinuria
evaluation shows open anterior-chamber angle
(C) Krabbes disease
and no peripheral anterior synechiae. Results
(D) Niemann-Pick disease
of a funduscopic examination are shown in the
(E) Phenylketonuria
image. Which of the following would be the
most appropriate next step in management of
19. A 42-year-old man presents to his physician
the patient?
with complaints of weight gain circumferen-
tially around his abdomen and a hump on
his neck. Physical examination shows a
rounded face and purple stretch marks on the
abdomen. On CT a solitary pituitary adenoma
is noted. Unfortunately, he has contraindica-
tions to surgical resection, and alternative
means of treatment must be considered.
Which of the following treatment strategies is
most appropriate for this patient?
(A) Glucocorticoids
(B) Insulin
Reproduced, with permission, from Fauci AS, Braunwald (C) Ketoconazole
E, Kasper DL, Hauser SL, Longo DL, Jameson LJ, (D) Observation
Loscalzo J, eds. Harrisons Online. New York: McGraw- (E) Pituitary irradiation
FULL-LENGTH EXAMS

Hill, 2008: Figure 29-15.

(A) Atropine 20. A 17-year-old G0 girl with dysmenorrhea sec-


(B) Observation with follow-up in 6 weeks ondary to moderate endometriosis presents
(C) Pilocarpine with her mother to the gynecologist for follow-
(D) Timolol up. She was started on a combination oral con-
(E) Trabeculectomy traceptive pill 10 months ago but experienced
only mild relief of her symptoms. She returns
18. A 5-month-old girl is rushed to the hospital by to the gynecologists ofce today to discuss the
her parents because she is not moving her left next line of therapy. What is the most appropri-
side. Her mother reports there were no prob- ate next step in treating her endometriosis?
lems when she was breast-fed 4 hours earlier. (A) Dexamethasone
Prior to this she has been healthy. She was (B) Electrocautery, endocoagulation, or laser
born full-term by spontaneous vaginal delivery resection
after an uncomplicated pregnancy. Her Apgar (C) Leuprolide
scores were 9 at both 1 and 5 minutes. There is (D) Progestin only therapy
no family history of stroke, deep vein thrombo- (E) Unopposed estrogens
sis, or pulmonary embolism. In the emergency
Test Block 2

department her temperature is 36.5C 21. A 43-year-old man develops fever, headache,
(97.7F), blood pressure is 76/45 mm Hg, re- and altered mental status. His past medical his-
spiratory rate is 26/min, and heart rate is 96/ tory is notable only for a motor vehicle acci-
min. Her physical examination is notable for dent 2 years prior, during which he sustained a
1/5 strength in her left arm and leg and 5/5 splenic laceration requiring splenectomy.
Test Block 2 Questions 513

Which of the following is most likely causing (A) Aortic stenosis


this patients symptoms? (B) Patent ductus arteriosus
(C) Tetralogy of Fallot
(A) Cryptococcus neoformans and Listeria
(D) Transposition of the great arteries
monocytogenes
(E) Ventricular septal defect
(B) Cryptococcus neoformans and Streptococ-
cus pneumoniae
24. A 52-year-old man develops a pruritic, scaling
(C) Haemophilus inuenzae and Staphylococ-
area on the skin of his chest. On examination
cus aureus
an erythematous rash is noted, with plaque-like
(D) Neisseria meningitides and Staphylococcus
lesions measuring 57 cm. He is treated with a
aureus
short course of oral and topical prednisone for
(E) Neisseria meningitides and Streptococcus
an assumed allergic dermatitis, which brings
pneumoniae
some resolution. The patient is then lost to fol-
low-up. He presents 7 years later with diffuse
22. A 6-year-old boy presents to the pediatricians
erythroderma of his entire body for 2 weeks.
ofce with 4 days of productive cough with
He states he has been feeling run down for the
whitish sputum and shortness of breath. His
past 6 months and has lost 8.2 kg (18 lb). On
mother reports that he has had multiple epi-
examination he is cachectic. His WBC count
sodes of upper respiratory infections in the
is 6000/mm, hemoglobin is 9.5 g/dL, and
past, and recently a few classmates at school
platelet count is 105,000/mm. A differential
were sent home because of cough and fever.
includes 21% atypical lymphocytes. A diagno-
The boy has had no fever or rhinorrhea, no
sis of mycosis fungoides is made. Which of the
gastrointestinal symptoms, and no ear pain. On
following treatments will most likely be recom-
examination he is afebrile but tachypneic and
mended to the patient?
mildly tachycardic. There is end-expiratory
wheezing bilaterally. Which of the following is (A) High-dose acyclovir
the most appropriate initial step in diagnosis? (B) Immediate intravenous ampicillin
(C) Intravenous amphotericin B
(A) Complete blood cell count with differen-
(D) Topical nitrogen mustard
tial

FULL-LENGTH EXAMS
(E) Total skin electron beam radiation therapy
(B) Measure peak expiratory ow rate
(C) Methacholine challenge
25. A 17-year-old girl was diagnosed with type 1 di-
(D) Sputum smear
abetes mellitus 7 years earlier. She recently
(E) X-ray of the chest
started college and forgot to take her insulin
and went to a party where she consumed sev-
23. A 15-day-old full-term boy is brought to the
eral beers. She presents to the emergency de-
clinic by his parents because of shortness of
partment with rapid, deep respirations and a
breath, failure to tolerate feeding, and inade-
solvent-like odor on her breath. What treat-
quate weight gain. On physical examination
ment should be administered rst?
there is no cyanosis evident on mucosal mem-
branes. Pulmonary rales are heard bilaterally (A) Bicarbonate repletion
with an active precordium and a soft 2/6 holo- (B) Glucose
systolic murmur most prominent at the left (C) Intravenous uids
lower sternal border. ECG is signicant for (D) Potassium repletion
biventricular hypertrophy. X-ray of the chest (E) Urinalysis
shows increased pulmonary vascularity. Which
of the following is the most likely etiology?
Test Block 2
514 Section II: Full-Length Examinations Questions

26. A 70-year-old woman was walking across her episode of profound hypotension to 60/30 mm
kitchen when she tripped over a throw rug and Hg that required 2 L of normal saline and 6 U
fell onto her outstretched right hand. She did of packed RBCs. The patient was nally stabi-
not hit her head or lose consciousness. She had lized after emergent uterine artery emboliza-
immediate pain and swelling in her right wrist tion. Her hospital stay was further complicated
and was brought to the emergency department by acute renal failure, presumed secondary to
by her daughter. X-ray of the right wrist reveals her hypotensive event. She is now awaiting dis-
a dorsally displaced, dorsally angulated fracture charge. Blood tests performed several weeks af-
of the distal radius with a displaced intra- ter discharge would most likely reveal which of
articular fragment in the radiocarpal joint. the following?
There was no break in the skin. The patient is
otherwise healthy, and her past medical history
is signicant only for a T11 compression frac-
THYROID-
ture 2 years ago. Which of the following is the
most appropriate treatment? STIMULATING
CHOICE PROLACTIN HORMONE
(A) Closed reduction and arm sling
(B) Closed reduction and long arm cast A
(C) Ice and compressive bandage
(D) Open reduction and internal xation B normal
(E) Physical therapy and night splint
C
27. A 76-year-old man with multiple medical prob-
lems, including diabetes and coronary artery D
disease, is admitted to the hospital for coronary
E
artery bypass grafting. On the second postoper-
ative day, he develops acute shortness of breath
and tachycardia, and is rushed to CT scan to (A) A
rule out pulmonary embolism. That night, his (B) B
FULL-LENGTH EXAMS

nurse notices that his urine output is low de- (C) C


spite the intravenous uids he is receiving, and (D) D
morning laboratory values show a blood urea (E) E
nitrogen level of 45 mg/dL and creatinine of
2.3 mg/dL. This pathology would have been 29. A 65-year-old previously healthy man with be-
resistant to which of the following prophylactic nign prostatic hyperplasia presents with fever,
measures? malaise, and dysuria. On physical examination
(A) Acetylcysteine administration before CT he has costovertebral angle tenderness and a
scan temperature of 37.3C (99.9F). The remain-
(B) Aggressive intravenous saline hydration be- der of the physical examination is unremark-
fore CT scan able. Which of the following is the most appro-
(C) Better chronic control of diabetes priate therapy?
(D) Prophylactic hemodialysis (A) 7-day outpatient course of trimethoprim
(E) Use of MRI with gadolinium, instead of sulfa
contrast CT (B) 10-day outpatient course of ciprooxacin
(C) 10-day outpatient course of moxioxacin
28. A 34-year-old G3P3 woman is postpartum day (D) Inpatient treatment with intravenous cef-
Test Block 2

14 after a vaginal delivery that was complicated triaxone


by a 1500-mL hemorrhage secondary to uter-
ine atony. The patient did not respond to oxy- 30. Fine-needle aspiration biopsy has high sensitiv-
tocin or methylergonovine and experienced an ity and moderate specicity to diagnose malig-
Test Block 2 Questions 515

nancy in a thyroid nodule. Which of the fol- (D) Switch to an angiotensin-converting


lowing statements is true about the use of enzyme inhibitor
ne-needle aspiration biopsy in this case? (E) Switch to hydrochlorothiazide
(A) A positive result is a strong conrmation of
33. A 34-year-old woman presents to her primary
disease
care physician complaining of 2 weeks of
(B) It is a strong indicator of those who may
cough and shortness of breath. A review of sys-
develop the disease later on, and a moder-
tems reveals that she has had two to three epi-
ate indicator of those who currently have
sodes of coughing up blood but no fever, chills,
the disease
or weight loss. Her examination is notable for
(C) The test has a strong ability to rule out dis-
decreased breath sounds at both lung bases.
ease in healthy individuals
Laboratory values include blood urea nitrogen
(D) There is a moderately high rate of false-
and creatinine levels of 42 mg/dL and 2.3 mg/
negatives
dL (previous laboratory values were within nor-
(E) This is a good test to use if malignancy is
mal limits), respectively, and urinalysis shows
considered very likely in a given patient
2+ proteinuria, microscopic hematuria, and
RBC and granular casts. X-ray of the chest
31. A population of 200 medical students under-
shows blunting of the costodiaphragmatic an-
goes testing for disease X. Of the 100 students
gles bilaterally and a small right lower lung in-
with this disease, only 90 receive positive test
ltrate. Which of the following is the most
results. Meanwhile, 25 students without dis-
likely cause of this patients renal failure?
ease X receive positive test results. Given this
information, what is the probability that a stu- (A) Antibodies directed against the glomerular
dent has disease X given that he or she tests basement membrane
positive for the disease? (B) Hepatitis C-associated cryoglobulinemia
(C) IgA deposition in the glomerular mesan-
(A) 75 / (10 + 75)
gium
(B) 75 / (25 + 75)
(D) Postinfectious reaction
(C) 90 / (90 + 10)
(E) Small vessel inammation
(D) 90 / (90 + 25)

FULL-LENGTH EXAMS
(E) Not enough information is provided
34. A 27-year-old G2P1 woman at 37 weeks gesta-
tion presents to the perinatal evaluation unit
32. A 36-year-old G2P1 woman at about 4
after noticing leakage of uid from her vagina.
months of gestation presents to her obstetri-
The external fetal monitor reveals a reactive fe-
cian for her rst prenatal visit. She has been
tal heart tracing and no contractions. On ster-
feeling well, but has not been to see a physi-
ile speculum examination the cervix appears
cian yet because of a death in the family. Her
closed, and there is minimal uid in the vagi-
past medical history is signicant for chronic
nal vault. What results from the nitrazine test
hypertension, which is currently well con-
would suggest that premature rupture of mem-
trolled on a calcium channel blocker, and for
branes has occurred?
Raynauds syndrome, also well controlled. Her
blood pressure is 125/85 mm Hg, heart rate is (A) 1+ Protein
95/min, and respiratory rate is 18/min. Which (B) 3++ Protein
is the most appropriate next step in managing (C) Negative ferning test
this patients hypertension? (D) pH <7
(E) pH >7
(A) Maintain the current dose of her calcium
(F) Presence of small blood
channel blocker
Test Block 2

(B) Switch to a -adrenergic blocker


(C) Switch to an angiotensin II receptor
blocker
516 Section II: Full-Length Examinations Questions

35. An 80-year-old woman presents to her primary


care physician with signs of thyrotoxicosis.
Blood tests reveal a low thyroid-stimulating
hormone level, markedly increased triiodothy-
ronine, and only moderately increased thyrox-
ine. A thyroid scan shows a solitary, small hot
nodule with complete suppression of the unaf-
fected lobe. Which of the following is the best
initial treatment?
(A) -Blocker
(B) Carbimazole
(C) Propylthiouracil
(D) Radioactive iodine
(E) Thyroidectomy

36. A 2-year-old African-American boy is brought


to the pediatrician by his mother because of a
4-day history of fever, irritability, and wheez-
ing. He has no past medical history, no known
allergies, and no family history of asthma or
other respiratory conditions. His mother can-
not specify the time when the wheezing began, Reprinted, with permission, from Brunicardi FC, Ander-
but it seems to have been present ever since he sen DK, Billiar TR, Dunn DL, Hunter JG, Matthews JB,
had a particularly violent coughing t at his Pollock RE, Schwartz SI. Schwartzs Principles of Surgery,
cousins house a few days earlier. On physical 8th edition. New York: McGraw-Hill, 2005: Figure 38-3.
examination the child has wheezing in the left (A) Albuterol metered-dose inhaler treatment
upper lung elds and decreased breath sounds (B) Bronchoscopy
throughout all left lung elds. X-ray of his (C) Helical CT scan of the chest
FULL-LENGTH EXAMS

chest is shown in the image. Which of the fol- (D) Left chest tube insertion
lowing is the most appropriate next step in pa- (E) MRI of the chest
tient care?
37. A 33-year-old man comes to the clinic with a
rapidly expanding target-shaped rash on his
shoulder in the same spot where he removed a
tick 3 days ago. The physician suspects Lyme
disease. Which of the following is the next step
in diagnosis?
(A) Biopsy rash and culture for Borrelia burg-
dorferi
(B) Collect plasma and culture for Borrelia
burgdorferi
(C) Collect plasma and test for Lyme antigen
(D) Collect plasma for enzyme-linked immu-
nosorbent assay and Western blot analysis
(E) Collect urine and test for Lyme antigen
Test Block 2

38. An 80-year-old man presents to the emergency


department complaining of severe lower ab-
dominal pain for 12 hours. History reveals that
he has not urinated in at least 18 hours and has
Test Block 2 Questions 517

experienced dull back pain for about 1 week. from his blood because they were slowing
He has not had regular medical care over the down his brain. During the admission inter-
past 20 years. He is afebrile with blood pressure view, his wife reported that he had not gone to
of 150/95 mm Hg, pulse of 88/min, and respi- work in >2 weeks, no longer showed interest in
ratory rate of 16/min. Physical examination re- his hobbies, and refused to eat even his favorite
veals a large palpable suprapubic mass that is foods. She had awakened many times over the
tender. Digital rectal examination shows an past month to see him lying in bed and staring
asymmetrically enlarged, rm, nodular pros- at the ceiling. During the mental status exami-
tate. Laboratory tests show mild anemia as well nation, the patient moved very slowly and de-
as a blood urea nitrogen level of 46 mg/dL and liberately, scarcely making eye contact. How-
creatinine of 2.8 mg/dL. Urinalysis after supra- ever, during his 3-week admission, the
pubic catheterization is unremarkable. Which centipedes gradually disappeared from his
of the following is the most likely cause of his blood, he no longer believed his mind was be-
acute renal failure? ing slowed, and his mood improved. By the
time of his discharge, his wife was thrilled to
(A) Glomerular Ig deposition
see that he had returned to his baseline. Which
(B) Hypercalcemia
of the following is the most likely diagnosis?
(C) Neurogenic bladder
(D) Obstructive nephropathy (A) Brief psychotic episode
(E) Renal tubular inammation (B) Depression with psychotic features
(F) Staghorn calculus (C) Schizoaffective disorder
(D) Schizophrenia
39. A 2-year-old girl is brought to the pediatrician (E) Specic phobia
by her mother, who is concerned because her
daughter has had difculty walking. She had a 41. A 75-year-old man presents to the emergency
normal birth history as well as normal develop- department complaining of a rash. He says he
ment until the age of 1 year. Her mother states noticed its onset 3 days prior and that it has
that her other three children all could walk by been worsening since then. His review of sys-
15 months of age, and is concerned that this tems is notable only for feeling tired lately. He

FULL-LENGTH EXAMS
child continues to crawl. On physical examina- has no known allergies. Physical examination
tion the girl appears to have an enlarged fore- reveals erythematous lesions covering his trunk
head as well as lateral bowing of the forearms and extremities that are most severe on his legs
and legs. Laboratory tests show a calcium level and buttocks. He denies pruritus. The lesions
of 7.4 mg/dL, phosphate of 1.9 mg/dL, and do not blanch under pressure. Massive spleno-
parathyroid hormone of 160 pg/mL. Addition- megaly is observed. His WBC count is 80,000/
ally, while the 25(OH) vitamin D level is nor- mm, hemoglobin is 8.8 g/dL, and platelet
mal, the 1,25(OH)2D level is low. This pa- count is 8000/mm. The patients infectious
tients condition is most likely due to an work-up is negative, but his bone marrow aspi-
abnormality in which organ? rate shows cells resembling mature lympho-
cytes packing the bone marrow. The patient is
(A) Gut
admitted to the hospital for further evaluation
(B) Kidney
and supportive care. Which of the following
(C) Liver
potential therapies is the most appropriate for
(D) Parathyroid
rapid and long-lasting correction of this pa-
(E) Skin
tients thrombocytopenia?
40. A 52-year-old man with a history of major de- (A) High-dose corticosteroids
Test Block 2

pressive episodes was admitted to an inpatient (B) Platelet transfusion


psychiatric hospital because of a suicide at- (C) Splenectomy
tempt using a kitchen knife. In the patients (D) Therapy with 5-uorouracil
words, he was trying to release the centipedes (E) Therapy with granulocyte colony-
stimulating factor
518 Section II: Full-Length Examinations Questions

42. The physician is consenting for a patient to be (E) Gastroparesis


enrolled in a randomized, double-blind, pla- (F) Hepatitis
cebo-controlled study. She asks about the ad- (G) Inammatory bowel disease
vantage of the study being randomized. Ran- (H) Intracranial hemorrhage
domization reduces which form of bias? (I) Labyrinthitis
(J) Meningitis
(A) Enrollment bias
(K) Migraine
(B) Lead time bias
(L) Myocardial infarction
(C) Measurement bias
(M)Pancreatic carcinoma
(D) Observer bias
(N) Pancreatitis
(E) Recall bias
(O) Sigmoid volvulus
(P) Small bowel obstruction
43. A 17-year-old boy is brought to the emergency
department by paramedics. He was found un-
44. A 55-year-old African-American man presents
conscious after a motor vehicle accident. Upon
to the urgent care center because of nausea
presentation the patient is noted to have multi-
and vomiting. He reports that this is the third
ple mandibular and maxillary fractures as well
such episode in the past week, and is accompa-
as an open right-sided tibial bular fracture.
nied by decreased appetite. He also notes that
The patient remains unconscious with an oxy-
the walls and lights seem to have a yellowish-
gen saturation of 78% on 2 L oxygen via face
green tint. His wife complains that he has not
mask with a pulse of 146/min and blood pres-
been acting himself, and she says he was last
sure of 60/20 mm Hg. Which of the following
in the hospital for an irregular heart beat.
is the rst step in management?
(A) Administer uids 45. An 85-year-old woman is brought to the emer-
(B) Continue with face mask ventilation and gency department by her daughter because of
proceed with the rest of the primary nausea, vomiting, abdominal pain, distention,
trauma survey and constipation. Abdominal examination re-
(C) Proceed with a cricothyroidotomy veals a tympanic abdomen. X-ray of the abdo-
(D) Proceed with nasopharyngeal intubation men shows an inverted U-shaped loop lacking
FULL-LENGTH EXAMS

(E) Proceed with oropharyngeal intubation haustration and overlapping the liver, the di-
lated descending colon, and the left side of the
E X T E N D E D M ATC H I N G
pelvis. The ratio of air to uid is >2:1.

46. A 64-year-old woman presents to her physician


The response options for the next 3 items are because of nausea and vomiting. She reports
the same. Select one answer for each item that although the vomiting can happen at any
in the set. time, it usually occurs a few hours after meals.
More recently she has begun to lose her appe-
For each of the following patients, select the most tite and notes a feeling of fullness after only a
likely diagnosis. few bites of food. Her husband remarks that
(A) Bulimia nervosa she has been belching a lot and seems to have
(B) Digoxin toxicity lost weight in the preceding months. Past med-
(C) Gastric outlet obstruction ical history is signicant for gastroesophageal
(D) Gastroenteritis reux disease and longstanding type 2 diabetes
mellitus.
Test Block 2
Test Block 2 Answers 519

AN S W E R S

1. The correct answer is E. This neonate is at high press their emotions and needs through physi-
risk for meconium aspiration syndrome because cal means.
there is thick meconium in the amniotic uid. Answer D is incorrect. Patients with pain dis-
To minimize this risk, the oropharynx should be order often lack insight. They are not invent-
suctioned as soon as the head is delivered and ing their symptoms, and they are unaware that
then again after delivery. The trachea should be they are causing the pain themselves. If they
suctioned through an endotracheal tube prior to were aware that this was the case, the diagnosis
respiration being initiated. This practice mini- would be factitious disorder.
mizes meconium aspiration into the smaller air-
ways. Postterm infants are at increased risk for Answer E is incorrect. Pain disorder is charac-
passing meconium in utero, likely secondary to terized by pain at one or more sites that results
placental insufciency. in signicant impairment of function and can-
not be better ascribed to another source. The
Answer A is incorrect. Ampicillin and gen- specic list of symptoms given here (pain or
tamicin would be good treatments for Group dysfunction at four distinct sites, at least two of
B streptococcus but not meconium aspiration which are gastrointestinal, one is neurologic,
pneumonia. The history is negative for major and one is sexual) are criteria for the diagnosis
risk factors for bacterial pneumonia because of somatization disorder, not pain disorder.
her mother is afebrile and negative for Group
B streptococci. 3. The correct answer is D. Systemic hyperten-
Answer B is incorrect. The infant should be sion and left ventricular hypertrophy is often
stabilized before diagnostic studies are per- one of the main etiologies of diastolic dysfunc-
formed. Clinically, a diagnosis can be made tion and therapy centers around aggressive
without an x-ray of the chest. However, if one treatment including -blockers. Lowering heart
were obtained it would show patchy inltrates, rate also allows more ventricular lling time.
coarse streaking, and overexpansion with at- Tachycardia should be avoided in the setting of

FULL-LENGTH EXAMS
tening of the diaphragm. diastolic failure.
Answer C is incorrect. Articial ventilation Answer A is incorrect. In the absence of an
is important in patients in respiratory distress. appropriate arrhythmia, amiodarone would be
However, in cases of meconium aspiration, the unlikely to improve this patients condition.
trachea should be suctioned rst. Answer B is incorrect. Atorvastatin is a very ef-
Answer D is incorrect. Supplemental oxygen fective statin that can reduce cholesterol levels.
is helpful when infants have respiratory depres- There is no indication that cardiac ischemia
sion. However, the trachea needs to be suc- was the cause of this patients diastolic dysfunc-
tioned rst in patients with meconium aspira- tion.
tion. Answer C is incorrect. Recognize that this pa-
tient has diastolic dysfunction by the normal
2. The correct answer is B. Patients with pain ejection fraction in the setting of clear heart
disorder often realize a secondary gain from failure. In theory, digoxin is of little value in
their symptoms. This gain may take many the setting of diastolic dysfunction. It may be
forms, such as time off from work or sympathy useful in the setting of atrial brillation, but
from family members and friends. not in this patients management.
Test Block 2

Answer A is incorrect. Pain disorder is more Answer E is incorrect. Nitroglycerin is a po-


common in females than in males. tent venodilator and arterial dilator used to
Answer C is incorrect. Patients with pain dis- relieve symptoms of myocardial ischemia and
order are often emotionally unsophisticated, reduce preload. In the setting of diastolic dys-
and this is thought to be the reason they ex- function, the heart relies on signicant pre-
520 Section II: Full-Length Examinations Answers

load to maintain cardiac output. Nitroglycerin 5. The correct answer is C. This patient has
should be used judiciously to avoid potential Pneumocystis jiroveci pneumonia. It can pre-
hypotension. sent with dyspnea on exertion, fever, nonpro-
ductive cough, weight loss, fatigue, and im-
4. The correct answer is A. In a normal postpar- paired oxygenation. Risk factors include
tum uterus, continued contractions help com- impaired cellular immunity and AIDS (CD4+
press the vessels and stop bleeding. This patient cell count <200/mm). X-ray of the chest can
is experiencing postpartum hemorrhage sec- show diffuse bilateral interstitial inltrates with
ondary to uterine atony. Risk factors include a ground glass appearance, but infection does
uterine overdistention secondary to diabetes not always present in that manner. Diagnosis is
and macrosomia, and myometrial exhaustion made based on cytology from induced sputum
secondary to a prolonged labor (especially third or silver staining of a bronchoscopy specimen.
stage). The most appropriate rst step in treat-
Answer A is incorrect. While culture is im-
ing uterine atony is bimanual uterine massage.
portant in the diagnosis of many infectious
This technique is usually successful in control-
diseases, culture is not available to diagnose P.
ling bleeding without the need for medication
jiroveci pneumonia.
infusions.
Answer B is incorrect. Serology on bronchos-
Answer B is incorrect. Hysterectomy is ap-
copy specimen is not clinically useful for iden-
propriate in the management of postpartum
tifying P. jiroveci pneumonia, but is important
hemorrhage only as a last resort when all other
in the diagnosis of other opportunistic infec-
interventions have failed. It should never be
tions seen in patients with AIDS, such as his-
considered an initial treatment option.
toplasmosis.
Answer C is incorrect. Uterine massage
Answer D is incorrect. Open lung biopsies are
should be performed bimanually, with one
used when abnormalities are found on CT or
hand placed over the uterus externally and one
x-ray of the chest and other diagnostic tests are
hand placed per vagina.
inconclusive. It can lead to the diagnosis of can-
Answer D is incorrect. Like oxytocin, methyl- cer, benign tumors, sarcoidosis, Wegeners gran-
FULL-LENGTH EXAMS

ergonovine can be used to help control bleed- ulomatosis, rheumatoid lung disease, or certain
ing. However, it cannot be given to hyper- infections such as aspergillosis, tuberculosis, vi-
tensive patients and should only be used as a ral pneumonia, and coccidioidomycosis.
second- or third-line agent. Additionally, meth-
Answer E is incorrect. Serum serologic testing
ylergonovine is always given intramuscularly,
is not clinically useful for diagnosing P. jiroveci
not intravenously.
pneumonia, but urine and serum polysac-
Answer E is incorrect. Oxytocin infusion is charide antigen testing is important in other
generally used to help control bleeding sec- opportunistic infections seen in patients with
ondary to uterine atony. It should, however, be AIDS, such as histoplasmosis.
administered after bimanual uterine massage
has begun. 6. The correct answer is C. This woman has re-
strictive cardiomyopathy. This condition is often
Answer F is incorrect. A thorough speculum
brought on by inltrative diseases, such as amy-
examination, often under anesthesia, to evalu-
loidosis, sarcoidosis, and carcinoid syndrome,
ate for vaginal or cervical lacerations is an ap-
and may lead to extensive cardiac damage. Due
propriate second step in management of post-
to increased myocardial stiffness, ventricular l-
partum hemorrhage. Bimanual examination in
ing is impaired, leading to the manifestations of
this patient strongly suggests uterine atony as
Test Block 2

right heart failure. To identify this condition, it


the source for her blood loss and thus evaluat-
is helpful to obtain an echocardiogram to distin-
ing for and repairing vaginal or cervical lacera-
guish it from other possible causes (valvular and
tions is an inappropriate rst step in treatment.
other cardiomyopathies). A cardiac catheteriza-
tion could also be done.
Test Block 2 Answers 521

Answer A is incorrect. This test would be ideal Answer E is incorrect. Increased somnolence
if myocardial infarction (MI) were a concern. or mental status changes may be due to hyper-
In this case, the patients history does not t a capnia, in which case the patient should be in-
classic description, and she has little reason to tubated. As long as a patient is awake enough
present in an atypical fashion. If the echocar- to be anxious, they are probably not hypercar-
diogram did not yield any results, enzymes bic.
might help but are not necessary.
8. The correct answer is E. The history is highly
Answer B is incorrect. The abdominal disten-
suggestive of gastroesophageal reux disease
sion this patient is experiencing is secondary to
(GERD), which presents with substernal chest
her right-sided heart failure, not any specic
pain worse after meals or when recumbent. Pa-
abdominal process. Therefore, this examina-
tients commonly also complain of an acid taste
tion is unwarranted in this case.
in the mouth from the reux. Mild to moder-
Answer D is incorrect. Rechecking the potas- ate GERD is treated initially with lifestyle
sium is necessary if the signs and symptoms modications including weight loss and antac-
point to potassium as being the etiology. It is ids. Pharmacologic treatment follows in pa-
most often done with laboratory error. Her tients with severe or refractory disease, with
ECG results do not indicate peak T waves, histamine-receptor antagonists (ranitidine or
and the patient does not exhibit other signs or cimetidine) or proton pump inhibitors
symptoms, so this test would not be pertinent (omeprazole or lansoprazole). This patient has
at this time. been experiencing symptoms for several years
and likely has moderate to severe disease.
Answer E is incorrect. Although X-ray of the
chest would help rule out other conditions, it Answer A is incorrect. A -blocker is impor-
would not allow denitive diagnosis of restric- tant in the treatment of MI and has proven to
tive cardiomyopathy. decrease mortality in this setting. MI presents
with crushing substernal pain, diaphoresis,
7. The correct answer is C. Partial carbon diox- nausea/vomiting, dyspnea, and bradycardia or
ide pressure >50 mm Hg indicates severe hy- tachycardia. There is no evidence that this pa-

FULL-LENGTH EXAMS
percarbia in the setting of respiratory distress tient is presenting with an acute cardiac event.
and is a reason to intubate. -Blockade is also indicated in the chronic
treatment of stable and unstable angina, as it
Answer A is incorrect. Spirometry revealing
decreases myocardial oxygen demand.
an FEV1:FVC ratio <0.7 accompanied by an
FEV1 <40% predicted is indicative of severe Answer B is incorrect. Aspirin is important
chronic obstructive pulmonary disease that is in the treatment of MI and has proven to de-
GOLD (Global Initiative for Chronic Obstruc- crease mortality in this setting. Aspirin is also
tive Lung Disease) stage III. An FEV1 <40% indicated in the chronic treatment of stable
of predicted is not, however, used to evaluate and unstable angina, as it prevents platelet ag-
whether a patient needs to be intubated or not. gregation in atherosclerotic plaques.
Answer B is incorrect. Partial arterial oxygen Answer C is incorrect. This is the surgical
pressure (PaO2) <50 mm Hg indicates severe treatment of refractory or severe GERD. It
hypoxemia in the setting of respiratory distress, would not be indicated as primary treatment,
and is a reason to intubate. PaO2 >50 mm Hg, as pharmacologic management would be initi-
however, is not a reason to intubate. ated rst.
Answer D is incorrect. While a respiratory rate Answer D is incorrect. Nitroglycerin is indi-
Test Block 2

of 25/min qualies as tachypnea, a rapid respi- cated for the acute treatment of angina, as it
ratory rate by itself is not a good reason to in- increases oxygen delivery to the myocardium.
tubate. Fatigue caused by tachypnea is a valid It is also used in the treatment of MI as a pre-
reason to intubate. load and afterload reducing agent.
522 Section II: Full-Length Examinations Answers

9. The correct answer is A. Abdominal aortic an- Answer A is incorrect. An infarct in the terri-
eurysms (AAAs) are typically asymptomatic un- tory of the left posterior cerebral artery would
til rupture but can occasionally cause a dull render a transcortical sensory aphasia.
lower back or ank pain. The aneurysms are
Answer B is incorrect. A stroke in the arcuate
predominantly caused by atherosclerosis, and
fasciculus would likely result in conduction
more than 90% occur below the renal arteries.
aphasia, which is characterized by problems
Real-time ultrasonography is the gold standard
with repeating what is said, but preserved u-
for screening for AAA because sensitivity with
ency and comprehension.
this imaging method approaches 100%. Rou-
tine sonographic evaluation involves measur- Answer C is incorrect. A stroke in the lenticu-
ing the longitudinal, anteroposterior, and trans- lostriate branches of the middle cerebral ar-
verse dimensions of the aorta. The normal tery would affect the subcortical regions of the
diameter of the aorta is approximately 2 cm, brain, such as the internal capsule, pons, and
and surgery is generally indicated above 4.5 basal ganglia, and thus would more likely pre-
cm. The nding of a pulsatile epigastric mass sent with motor and sensory symptoms instead
is highly suggestive of AAA. of with aphasia, which is a manifestation of
cortical damage.
Answer B is incorrect. Colonic obstruction
does not t with the gradual nature of the com- Answer E is incorrect. Brocas aphasia is char-
plaint. Colonic obstruction would present with acterized by preserved comprehension, but
abdominal distention and pain in a more acute problems with language production. The cause
fashion. is most likely a lesion in Brocas area, usually
caused by a large stroke in the territory of the
Answer C is incorrect. An intestinal arterio-
superior division of the middle cerebral artery,
venous malformation would rarely be large
and can present with associated right-sided
enough to cause somatic symptoms without
hemiparesis.
also causing hemodynamic complications.
Answer D is incorrect. Although the present- 11. The correct answer is D. Vitamin C (ascorbic
ing complaint could be interpreted as an epi- acid) deciency or scurvy typically presents in
FULL-LENGTH EXAMS

sode of chronic pancreatitis, which can also elderly, malnourished patients as fatigue, ane-
present with a pseudocyst, the mass would not mia, gingivitis, and arthralgias. A rash like the
be pulsatile. Furthermore, one would expect one seen in the image represents the purpuric,
pancreatitis to occur in alcoholics or patients perifollicular rash that may be seen on the legs
with gallstones, neither of which is indicated or buttocks of patients with ascorbic acid de-
in this patient. ciency. This patients clinical presentation is
consistent with scurvy, which is treated with re-
Answer E is incorrect. Clinically signicant
placement of ascorbic acid.
peptic ulcer disease would be unlikely in a pa-
tient with the nding of a pulsatile epigastric Answer A is incorrect. Folate deciency is
mass. Also, the pain of peptic ulcer disease the most common vitamin deciency in the
does not characteristically localize to the back. United States. It causes a megaloblastic, mac-
rocytic anemia. Folate deciency can be differ-
10. The correct answer is D. This patient presents entiated from vitamin B12 deciency because
with Wernickes aphasia, featuring problems it lacks neurologic symptoms. This patients
with language comprehension and impairment clinical presentation is more consistent with
of both language input and output. Usually the scurvy than folate deciency.
speech is uent but does not make sense. Some-
Answer B is incorrect. Patients with iron de-
Test Block 2

times Wernickes aphasia is accompanied by a


ciency will have a microcytic anemia. Gingivi-
contralateral superior homonymous quadran-
tis, arthralgias, and rash, however, are not seen
tanopia. The lesion is located in Wernickes
in iron deciency anemia, so it is unlikely that
area, and is most likely caused by a stroke in the
iron supplementation will correct this patients
inferior division of the middle cerebral artery.
anemia.
Test Block 2 Answers 523

Answer C is incorrect. Vitamin B12 (cobala- Answer E is incorrect. Pelvic inammatory dis-
min) deciency is a common cause of anemia ease is a common cause of infertility and can
in strict vegetarians, chronic alcoholics, and lead to spontaneous miscarriage. Pelvic inam-
patients who lack intrinsic factor. Cobalamin matory disease is less likely to be the cause of
deciency is characterized by megaloblastic, repeated spontaneous abortions in this woman,
macrocytic anemia, and neurologic symptoms. given the frequency of her miscarriages. This
The anemia in this patient is more likely due test would be very low-yield in this patient.
to vitamin C deciency, given the rest of her
clinical presentation. 13. The correct answer is E. This patient is suffer-
ing from body dysmorphic disorder (BDD),
Answer E is incorrect. Vitamin K deciency
which is dened as preoccupation with some
can result in easy bleeding and bruising, as it
perceived physical abnormality to the point of
adversely affects vitamin K-dependent clotting
experiencing profound distress and impair-
factors. Patients with decient vitamin K will
ment. A referral for psychotherapy is the most
have an elevated prothrombin time and acti-
appropriate next step.
vated partial thromboplastin time with a nor-
mal bleeding time. Answer A is incorrect. The nature of BDD is
such that performing surgery is not likely to re-
12. The correct answer is C. Approximately 20% lieve the patients suffering. Patients with this
of repeated pregnancy losses are secondary to disorder are usually as certain of their abnor-
uterine pathologies. Visualizing the uterus mality after surgery as they were before. In fact,
with a hysterosalpingogram is the rst step to they are more likely to sue a surgeon for per-
rule out physical uterine pathology. forming a botched procedure, since they are
not able to perceive any improvement in their
Answer A is incorrect. Oral contraceptive pills
appearance.
(OCPs) are often used for women with endo-
metriosis. Endometriosis is a disorder charac- Answer B is incorrect. Although selective se-
terized by growth of endometrium outside of rotonin reuptake inhibitors have been shown
the uterus. OCPs can be used to minimize the to be effective in treating some patients with
growth of this ectopic endometrial stroma and BDD, such patients should be evaluated by a

FULL-LENGTH EXAMS
thus decrease symptoms. OCPs would help to psychiatrist before being placed on psychotro-
rule out uterine abnormalities as a cause for pic medication.
her miscarriages.
Answer C is incorrect. This patient does not
Answer B is incorrect. In women with an iso- require sedation; she requires psychotherapy to
lated spontaneous abortion prior to 11 weeks address the underlying issues that drive her ob-
gestation, approximately 50% are due to chro- session.
mosome abnormalities. However, in women
Answer D is incorrect. While reassurance
who have repeated pregnancy losses, this is
along with a refusal to perform surgery is an
not the case. Only 2%4% of these women
appropriate measure, it is not an adequate one
are found to have non-viable karyotypes. As a
without a referral to psychotherapy for deni-
result, chromosomal analysis is extremely low
tive management.
yield and often the last test to be done on these
women.
14. The correct answer is A. Patients with a his-
Answer D is incorrect. An exploratory laparo- tory of cough, unintentional weight loss, and
tomy can be used to evaluate infertility, espe- signicant smoking history should be worked
cially if endometriosis is suspected, since the up for lung cancer. Adenocarcinoma is the
Test Block 2

procedure can help to restore fertility. How- most common lung cancer, occurring most of-
ever, the suspicion for endometriosis in this pa- ten peripherally. Hypertrophic pulmonary os-
tient is low. In addition, exploratory laparotomy teoarthropathy (HPOA) is common and is
is highly invasive and should be reserved until characterized by the proliferation of soft and
absolutely necessary. osseous tissue at the distal portions of extremi-
524 Section II: Full-Length Examinations Answers

ties. Among lung cancer patients, hypertrophic some that patients undergoing gastric bypass or
pulmonary osteoarthropathy is most frequently other bariatric surgery should have a prophy-
associated with adenocarcinoma and least fre- lactic cholecystectomy. A presentation suspi-
quently with small cell carcinoma. HPOA typi- cious for acute cholecystitis should be worked
cally presents with digital clubbing, periostosis up initially with an ultrasound, as this is an in-
of long bones, and synovial effusions of the expensive test that can be done at the bedside
larger joints. Periostosis is often accompanied rapidly.
by pain on palpation of the affected area.
Answer B is incorrect. Although CT scan of
When HPOA is suspected, the chest should be
the abdomen may show an edematous gall-
examined closely because lung neoplasm is the
bladder wall with uid surrounding the gall-
most frequent cause of acute HPOA.
bladder, this test is much more expensive than
Answer B is incorrect. Bronchoalveolar type an ultrasound and cannot be performed at the
of adenocarcinoma is associated with multiple bedside, and therefore it is not the rst test that
nodules, interstitial inltration, and prolic you would order when working up a patient
sputum production. Its appearance on x-ray with a presentation suspicious for acute chole-
of the chest mimics interstitial pneumonia. It cystitis.
is less predictably associated with digital club-
Answer C is incorrect. A hepato-iminodiacetic
bing or HPOA.
acid (HIDA) scan may reveal a lling defect of
Answer C is incorrect. Large cell carcinoma is the common bile duct, but is not useful for vi-
a relatively uncommon form of bronchogenic sualizing the gallbladder. In addition, this test
carcinoma that may produce -human chori- is not performed at the bedside and is more ex-
onic gonadotropin, resulting in gynecomastia, pensive than an ultrasound. A HIDA scan may
milky nipple discharge, and elevated serum be useful as a second-line approach when diag-
levels of this hormone. nosis via ultrasound is indeterminate.
Answer D is incorrect. Small cell carcinoma Answer D is incorrect. MRI of the abdomen is
is strongly related to cigarette exposure and as- not indicated in the work-up of a patient with
sociated with Cushings syndrome, syndrome acute cholecystitis. This test is very expensive,
FULL-LENGTH EXAMS

of inappropriate ADH secretion, peripheral cannot be performed at the bedside, and it


neuropathy, subacute cerebellar degeneration, takes longer to obtain the results when com-
and Eaton-Lambert syndrome. Small cell car- pared with abdominal ultrasound.
cinoma has the worst prognosis of the broncho-
Answer E is incorrect. Plain abdominal lms
genic carcinomas and has metastasized at the
have limited utility in the diagnosis of acute
time of diagnosis in two-thirds of patients.
cholecystitis, as they will visualize <15% of
Answer E is incorrect. Squamous cell carci- calculi. Right upper quadrant ultrasound has
noma is most often located centrally and as- greater sensitivity and specicity than x-ray
sociated with hypercalcemia via production of lms of the abdomen in the diagnosis of acute
parathyroid hormone-related peptide. It is also cholecystitis.
associated with pathologic fractures and kidney
stones. 16. The correct answer is A. This woman has mild
gestational diabetes, as a serum glucose con-
15. The correct answer is A. This patient presents centration >130 mg/dL after 50-g oral glucose
with acute cholecystitis. This diagnosis is sup- tolerance test is abnormal. The rst line of
ported by right upper quadrant abdominal pain treatment to control blood glucose is dietary
that is accompanied by nausea, vomiting, fever, restriction. Patients are placed on the Ameri-
Test Block 2

guarding, and a positive Murphys sign. Al- can Dietetic Association diet, which restricts
though obesity increases ones risk of develop- calories and adheres to the conventional food
ing gallbladder disease, this risk is even greater pyramid.
following rapid weight loss (such as after gastric
bypass surgery). It has even been suggested by
Test Block 2 Answers 525

Answer B is incorrect. For gestational diabetes, used in the acute treatment of OAG or closed-
an American Dietetic Association diet is the angle glaucoma to open the trabecular mesh-
rst line of treatment. However, diet in combi- work around Schlemms canal, increasing
nation with exercise is recommended because drainage of aqueous humor and decreasing in-
it is more efcient than diet alone at control- traocular pressure. However, it is not typically
ling diabetes. a rst-line therapy for OAG.
Answer C is incorrect. Diet control is the rst Answer E is incorrect. Although trabeculec-
line of treatment for gestational diabetes, fol- tomy can provide curative treatment of closed-
lowed by insulin injection. Therefore, this angle glaucoma, it is not rst-line treatment for
woman will only be treated with insulin if di- OAG. A patient with closed-angle glaucoma
etary restrictions alone cannot control her dia- usually presents with headache, malaise, and
betes. general distress, with loss of visual acuity as in-
traocular pressure increases.
Answer D is incorrect. Oral hypoglycemic
agents can cause hypoglycemia in the fetus and
18. The correct answer is B. Homocystinuria re-
therefore should not be used in pregnancy.
sults from a defect in cystathionine or methio-
Answer E is incorrect. Oral hypoglycemic nine synthesis, leading to accumulation of ho-
agents can cause hypoglycemia in the fetus and mocysteine. It can cause failure to thrive,
therefore should not be used in pregnancy. mental retardation, osteoporosis, megaloblastic
anemia, lens dislocation, and most importantly,
17. The correct answer is D. The patient has pri- thromboembolic events (deep vein thrombo-
mary open-angle glaucoma (OAG), the most ses, pulmonary emboli, and cerebrovascular
common type of glaucoma. The eyes undergo accidents). Other heritable causes of hyperco-
a progressive loss of peripheral vision and then agulable states include factor V Leiden muta-
central vision. On funduscopic examination, tion, protein C deciency, protein S deciency,
the optic discs hollowed-out appearance is re- antithrombin III deciency, and defects in -
ferred to as cupping. There are no other brinolysis.
symptoms, which highlights the importance of
Answer A is incorrect. Fabrys disease results

FULL-LENGTH EXAMS
measuring intraocular pressure to screen for
from a deciency of -galactosidase A and
this disease. Timolol is a nonselective
leads to an accumulation of ceramide trihexo-
-antagonist that reduces the production of
side. It presents with renal failure. It does not
aqueous humor in the eye. It can be used topi-
cause cerebrovascular accidents.
cally to treat OAG. It is important to check for
a history of cardiac and pulmonary disease Answer C is incorrect. Krabbes disease results
prior to prescribing. from the absence of -galactosidase and leads
to accumulation of galactocerebroside. This
Answer A is incorrect. Atropine is an antimus-
disease presents during the rst year of life with
carinic agent that results in mydriasis and is not
optic atrophy, seizures, and spasticity. It does
an acceptable form of treatment for glaucoma.
not cause cerebrovascular accidents.
Answer B is incorrect. This patient has pri-
Answer D is incorrect. Niemann-Pick disease
mary OAG with visible cupping of the optic
results from deciency of sphingomyelinase,
nerve, retinal nerve ber layer defects, and in-
leading to a build-up of sphingomyelin and
creased intraocular pressure. It is necessary to
cholesterol. The infantile form presents with
provide the patient with a form of treatment
hepatosplenomegaly, failure to thrive, and rap-
because the pathology will likely progress with-
idly progressive neurodevelopmental regres-
out appropriate treatment. In addition, the vi-
Test Block 2

sion. It does not cause cerebrovascular acci-


sion loss is irreversible.
dents.
Answer C is incorrect. Pilocarpine is a mus-
Answer E is incorrect. Phenylketonuria results
carinic agonist that produces rapid miosis and
from an enzyme or cofactor deciency that
contraction of the ciliary muscles. This can be
526 Section II: Full-Length Examinations Answers

prevents phenylalanine from being converted maturation causes a decreased level of circulat-
to tyrosine. It classically presents with fair skin, ing estrogen, which in turn decreases the prolif-
eczema, and musty body odor. As the child eration of endometrial tissue and minimizes the
ages, mental retardation will develop if left un- pain associated with ectopic endometrium. Leu-
treated. Phenylketonuria does not cause cere- prolide is an appropriate medical treatment
brovascular accidents. choice for endometriosis that has failed treat-
ment of combination OCPs.
19. The correct answer is C. Ketoconazole, an an-
Answer A is incorrect. Dexamethasone is not a
tifungal medication that can be given topically
treatment for endometriosis.
or systemically, also has the effect of decreasing
adrenal steroidogenesis. Adrenal insufciency Answer B is incorrect. Surgical treatment is
is a possible but rare adverse effect that can be generally reserved for patients with severe dis-
treated with steroid replacement therapy. ease, those who fail hormonal therapy, or the
older infertile patient.
Answer A is incorrect. Steroid replacement
therapy would exacerbate the patients Cush- Answer D is incorrect. Progestin therapy can
ings syndrome symptoms, the opposite of the be used to treat endometriosis. However, due
desired effect. Steroid replacement therapy to its mixed results and the fact that continuous
may be necessary after the patient has received therapy can lead to breakthrough bleeding, de-
ketoconazole and possibly develops adrenal in- pression, bloating, and weight gain, and is used
sufciency. in patients who do not desire fertility, it is not
the most appropriate choice for this teenager.
Answer B is incorrect. One of the possible
effects of Cushings syndrome is the develop- Answer E is incorrect. Unopposed estrogen
ment of the symptoms of diabetes mellitus, therapy would likely exacerbate symptoms of
secondary to glucose intolerance. Although in- endometriosis. In addition, such therapy is a
sulin may be effective in treating those symp- risk factor for acquiring endometrial cancer.
toms (polydipsia, polyuria, polyphagia), it will
not decrease adrenal steroidogenesis. 21. The correct answer is E. Patients who have
had their spleens removed or are functionally
FULL-LENGTH EXAMS

Answer D is incorrect. The patient is symp-


asplenic, such as patients with sickle cell dis-
tomatic, and therefore warrants some sort of
ease, can be susceptible to meningitis caused
treatment, even though it will not be as effec-
by Streptococcus pneumoniae and Neisseria
tive as surgical resection. Ketoconazole would
meningitidis. Any encapsulated organism such
be most appropriate, given its effect of decreas-
as S. pneumoniae, N. meningitides, or Haemo-
ing adrenal steroidogenesis.
philus inuenzae may cause infection in as-
Answer E is incorrect. Pituitary irradiation has plenic patients, as they are unable to manufac-
a long lag time between treatment and remis- ture a new antibody immune response to these
sion, and remission only occurs in <50% of pa- organisms after the spleen is removed. This is
tients. It is more commonly used for postopera- the reason for preoperative inoculation against
tive recurrences. these organisms in scheduled splenectomy.
HIV infection and deciency of opsonizing an-
20. The correct answer is C. Leuprolide is a gon- tibodies can also both leave the patient suscep-
adotropin-releasing hormone (GnRH) analog tible to meningitis caused by S. pneumoniae.
that is commonly used to help control the symp-
Answer A is incorrect. Cryptococcus and Lis-
toms of endometriosis. Release of pituitary folli-
teria are both important causes of meningitis
cle-stimulating hormone (FSH) and luteinizing
in patients with advanced HIV disease, but not
Test Block 2

hormone (LH) requires pulsatile secretion of


in patients with asplenia.
GnRH. Constant pituitary stimulation with leu-
prolide, or other GnRH analogs, inhibits FSH/ Answer B is incorrect. While Streptococcus
LH secretion, preventing normal folliculogene- pneumoniae is an important cause of menin-
sis and ovulation. Impaired follicle growth and
Test Block 2 Answers 527

gitis in asplenic patients, Cryptococcus neofor- Furthermore, subcutaneous emphysema may


mans is not a common cause of meningitis in also be seen.
this patient population. C. neoformans is an
important cause of meningitis in patients with 23. The correct answer is E. When evaluating
diseases that expose patients to high levels of heart defects, it is important to consider the
corticosteroid hormones, and in patients with presence or absence of cyanosis, nature of any
compromised immune systems due to HIV dis- murmur, and age at presentation. Ventricular
ease. septal defects (VSDs) are the most common
congenital heart defects and account for most
Answer C is incorrect. Staphylococcus aureus
hospital admissions for heart defects in infants
is not more likely to cause meningitis in peo-
1428 days old. The nature of symptoms is
ple with asplenia.
highly dependent on the size of the defect.
Answer D is incorrect. While meningococcus The ECG ndings and the quiet murmur in
is an important cause of meningitis in patients this child are consistent with a large VSD. Ra-
with asplenia, Staphylococcus aureus is not diographic ndings commonly include in-
likely to cause meningitis in this patient popu- creased pulmonary vascular markings. In more
lation. severe VSDs, enlargement of the left atrium
and left ventricle as well as the pulmonary ar-
22. The correct answer is B. Peak expiratory ow tery can also be seen on x-ray of the chest.
rate is the maximum ow rate achieved during
Answer A is incorrect. Aortic stenosis is usually
the forced vital capacity maneuver, beginning
the result of a bicuspid or single commissure
after full inspiration and ending at maximal ex-
valve and symptoms are largely dependent on
piration. This is signicantly decreased during
the degree of stenosis. ECG would likely only
an asthmatic episode and is the simplest and
show left ventricular hypertrophy. Most impor-
fastest diagnostic step in the primary care set-
tantly, VSDs would be more common at this
ting.
age.
Answer A is incorrect. Blood eosinophilia is
Answer B is incorrect. Patent ductus arteriosus
common and may be useful if one suspects ei-
is often seen in premature infants and carries a

FULL-LENGTH EXAMS
ther allergy or asthma, but eosinophilia is not
female predominance. The characteristic mur-
sensitive or specic enough to be a main diag-
mur is often described as machine-like and in-
nostic criterion for asthma. Therefore, a com-
volves both systole and diastole.
plete blood cell count is not indicated in the
initial work-up. Answer C is incorrect. Tetralogy of Fallot is
the second most common heart defect in in-
Answer C is incorrect. Methacholine chal-
fants 1428 days old, but would likely present
lenge is a test for bronchial hyperresponsive-
with cyanosis. Additionally, pulmonary blood
ness when the patient is not acutely ill and is
ow would be decreased in this condition.
not specic for asthma.
Remember, the tetralogy includes right ven-
Answer D is incorrect. Sputum specimen tricular outow obstruction, right ventricular
from an asthmatic may reveal Charcot-Leyden hypertrophy, overriding aorta, and subaortic
crystals and Curschmann spirals that can be VSD. Radiographic ndings typically include
diagnostic for asthma, are but not required for a boot-shaped heart due to hypertrophy of the
diagnosis. right ventricle.
Answer E is incorrect. In mild to moderate Answer D is incorrect. Transposition of the
asthma without adventitious sounds other than great arteries is also a common heart defect,
Test Block 2

wheezing, x-ray of the chest is not needed. but cyanosis would likely be present from
X-ray of the chest is indicated if the patient birth. Other features are variable but increased
has severe asthma and/or exacerbation requir- pulmonary vascular ndings are typical.
ing hospitalization. In such a case, x-ray would
show barrel chest and increased lung volumes.
528 Section II: Full-Length Examinations Answers

24. The correct answer is E. The patient is pre- venous uids and insulin. Patients often pres-
senting with end-stage mycosis fungoides, a ent with signicant dehydration that must be
variant of cutaneous T lymphocyte lymphoma. reversed as quickly as possible. Excessive alco-
This disease is generally indolent, and patients hol intake, as in this patient, can lead to poor
may have several years of eczematous or der- diabetes control (e.g., problems with medica-
matitic skin lesions before the diagnosis is es- tion/blood sugar balance), which can contrib-
tablished. Skin lesions begin as patches and ute to DKA. Also, while not directly causative,
plaques, and may progress to tumors. In other in some people with diabetes excess alcohol
cases, such as this one, Szarys syndrome de- can worsen dehydration and thereby contrib-
velops, which manifests as diffuse erythro- ute to the development of DKA.
derma and circulating tumor cells (Szary
Answer A is incorrect. Bicarbonate repletion
cells). Total skin electron beam therapy is of-
is most effective for lactic acidosis, but patients
ten recommended for patients with Szary syn-
should be watched carefully for uid overload
drome because the entire body can be treated
and lactic acidosis, which can occur secondary
without the morbidity that would result from
to the amount necessary to replete. It is not in-
conventional radiotherapy. Electron beam
dicated in treating DKA.
therapy offers the ability to treat supercial skin
lesions with little penetration into deeper tis- Answer B is incorrect. Glucose administration
sues. Unfortunately, the disease is usually not is not indicated early in the treatment of DKA,
curable in late stages. since the patient already has extremely high se-
rum glucose levels that she cannot utilize due
Answer A is incorrect. There is no indication
to her lack of insulin. Once uids and insulin
that the patient is suffering from a viral illness
have been started, glucose could be adminis-
that would respond to acyclovir.
tered to meet the childs increased metabolic
Answer B is incorrect. Although the patients needs and prevent hypoglycemia.
condition may mimic staphylococcal scalded
Answer D is incorrect. Potassium repletion is
skin syndrome, the timing of his presentation
not indicated as an initial step in the treatment
is not as acute as it would be if he were suffer-
of DKA. Serum potassium is usually elevated
ing from a bacterial infection.
FULL-LENGTH EXAMS

in ketoacidosis secondary to hyperosmolality


Answer C is incorrect. Despite sounding like and insulin deciency, which instigates potas-
a fungal disease, mycosis fungoides is a cutane- sium movement out of cells. However, with the
ous T-lymphocyte lymphoma and the patient administration of insulin there is often a rapid
would not benet from antifungal therapy. fall in plasma potassium concentration, which
can be repleted with potassium chloride.
Answer D is incorrect. Topical nitrogen mus-
tards are also used to treat localized disease but Answer E is incorrect. Urinalysis should be
cannot realistically be effectively applied to the obtained to conrm the diagnosis but is not
entire body. necessary to begin uid treatment.

25. The correct answer is C. Diabetic ketoacidosis 26. The correct answer is D. This patient has a
(DKA) is one consequence of untreated diabe- Colles fracture. In general, operative manage-
tes mellitus (chronic high blood sugar or hy- ment is required if a fracture is intra-articular
perglycemia), and is linked to an impaired glu- and/or open. Percutaneous pin xation, open
cose cycle. In a diabetic patient, DKA begins reduction and internal xation, and external
with deciency in insulin. This is most com- xation are all possible ways to manage this
monly due to undiagnosed diabetes mellitus, fracture, which has a displaced intra-articular
Test Block 2

or in patients who have been diagnosed with fragment.


diabetes, failure to take prescribed insulin.
Answer A is incorrect. Closed reduction is the
DKA has a 100% mortality rate if left un-
treatment of choice if the fracture is displaced,
treated. Treatment consists of aggressive intra-
closed, and extra-articular. This fracture is
Test Block 2 Answers 529

intra-articular. A sling would not adequately 28. The correct answer is A. This woman is at risk
maintain the reduction and thus would not be for Sheehans syndrome, or postpartum pituitary
used. necrosis, secondary to her severe obstetric hem-
orrhage and subsequent profound hypotension.
Answer B is incorrect. The fracture had a dis-
Patients with Sheehans syndrome have de-
placed intra-articular piece, so operative man-
creased levels of all hormones produced by the
agement is needed. A long arm cast would
anterior pituitary (growth hormone, thyroid-
appropriately maintain reduction, but closed
stimulating hormone, ACTH, prolactin, LH,
reduction cannot properly realign the joint sur-
FSH), with only modest if any decrease in hor-
face.
mones released by the posterior pituitary (ADH,
Answer C is incorrect. Ice and compression oxytocin). Therefore, blood tests will likely re-
are appropriate with isolated soft tissue injury. veal a decrease in levels of both prolactin and
However, immobilization is required for frac- thyroid-stimulating hormone.
tures.
Answer B is incorrect. Both prolactin and thy-
Answer E is incorrect. Although physical ther- roid-stimulating hormone would be decreased.
apy may ultimately be needed after immobili- Although normal levels of these hormones are
zation, initial therapy must focus on regaining possible in a patient suffering from Sheehans
proper bone and joint alignment. syndrome, this is very unlikely. Some patients
do demonstrate normal hormone levels for
27. The correct answer is D. This patient has con- several months postpartum and only become
trast-induced nephropathy due to the kidneys aware of pituitary dysfunction when they are
being responsible for getting rid of the dye and evaluated for amenorrhea. Sheehans results in
it causing a renal azotemia. The patient may panhypopituitarism, as opposed to this answers
also have some underlying renal insufciency values. Most patients, however, present to their
due to diabetes. Of the answer choices given, physicians within weeks of delivery, usually
prophylactic hemodialysis to remove contrast secondary to an inability to lactate.
dye from the circulation is the only treatment
Answer C is incorrect. Thyroid-stimulating
that has not been shown to prevent acute renal
hormone will be decreased, not increased.

FULL-LENGTH EXAMS
failure due to contrast nephropathy.
Answer D is incorrect. Prolactin will be de-
Answer A is incorrect. Acetylcysteine may re-
creased, not increased.
duce the risk of contrast-induced nephropathy
by an unknown mechanism. Answer E is incorrect. Both prolactin and thy-
Answer B is incorrect. Intravenous hydration roid-stimulating hormone are decreased.
with saline before and after administration of
29. The correct answer is B. Acute pyelonephritis
contrast has been shown to reduce the occur-
is suggested by ank pain, nausea/vomiting,
rence of acute renal failure, especially in pa-
and fever (>38C [>100.4F]) with or without
tients with some underlying chronic renal in-
costovertebral tenderness, and may occur in
sufciency.
the absence or presence of cystitis symptoms.
Answer C is incorrect. Underlying diabetic Ciprooxacin is an oral uoroquinolone rec-
nephropathy increases the risk of contrast- ommended for initial empiric treatment of in-
induced nephropathy. Patients with better gly- fection caused by gram-negative bacilli. Es-
cemic control are less likely to have underlying cherichia coli accounts for 70%95% of acute,
diabetic nephropathy and renal insufciency. uncomplicated upper and lower urinary tract
Answer E is incorrect. Gadolinium contrast infections.
Test Block 2

used with MRI is not associated with contrast- Answer A is incorrect. This treatment is inad-
induced nephropathy; therefore, MRI with equate because -lactam regimens shorter than
contrast should be considered as an alterna- 14 days have been associated with unaccept-
tive to CT in patients at high risk for contrast- ably high failure rates.
induced nephropathy.
530 Section II: Full-Length Examinations Answers

Answer C is incorrect. Moxioxacin, a newer one must divide the number of true positives by
uoroquinolone, should be avoided in pyelo- the total number of people who tested positive
nephritis because it may not achieve adequate for the disease. In this case, the number of true
concentrations in the urine to be effective. positives (90) is divided by the number of people
who tested positive (90 + 25).
Answer D is incorrect. This patient can re-
ceive adequate treatment as an outpatient. Answer A is incorrect. This fraction (true-
Indications for admission and intravenous negatives / total negatives) would be used to cal-
therapy include inability to maintain oral hy- culate the negative predictive value of the test.
dration or take oral medications, uncertainty
Answer B is incorrect. This fraction (true-
regarding the diagnosis, or severe illness with
negatives / total without disease) gives the spec-
high fevers and intense pain. This patient is a
icity of the test.
relatively healthy man with no signicant past
medical history and a likely uncomplicated Answer C is incorrect. This fraction (true-
case of acute pyelonephritis. positives / total diseased) gives the sensitivity of
the test.
30. The correct answer is C. Remember
Answer E is incorrect. This question provides
SNOUT and SPIN: SeNsitivity rules OUT
us with the information necessary to calculate
and SPecicity rules IN. High sensitivity rules
the positive predictive value.
out disease in those who test negative, while
high specicity conrms disease in those who 32. The correct answer is A. Calcium channel
test positive. A highly sensitive test is good for blockers are not contraindicated during preg-
screening because it will not miss many indi- nancy, and are used to treat Raynauds syn-
viduals with disease. drome. Given that this womans hypertension
Answer A is incorrect. A positive result from a is controlled on a calcium channel blocker,
test with high specicity is needed to conrm there is no reason to switch her to a
disease. In the case of high sensitivity, as in this -adrenergic blocker.
case, a negative test rules out or conrms lack Answer B is incorrect. As this womans hyper-
of disease.
FULL-LENGTH EXAMS

tension is controlled on a calcium channel


Answer B is incorrect. Sensitivity and specic- blocker and they are not contraindicated dur-
ity are not predictors of who may or may not ing pregnancy, there is no reason to switch her
develop a disease. to a -adrenergic blocker. Although they are
generally thought to be safe in pregnancy, they
Answer D is incorrect. A highly sensitive test
are class D and may be associated with brady-
may have a high false-negative ratio, but it will
cardia, intrauterine growth retardation, and
also have a low false-positive ratio. These pa-
neonatal hypoglycemia.
rameters make this a good test to screen for a
condition. However, this answer choice is in- Answer C is incorrect. Angiotensin II receptor
complete because a high false-negative ratio blockers are contraindicated during pregnancy,
on its own is not sufcient to screen effectively. as they may cause renal and pulmonary com-
plications.
Answer E is incorrect. Because tests with high
specicity are used to conrm disease, they are Answer D is incorrect. Angiotensin-converting
more appropriate in a setting where the suspi- enzyme inhibitors are considered teratogenic.
cion of malignancy is already very high. In this They have been associated with renal defects,
case, ne-needle aspiration biopsy has a high renal dysgenesis/dysplasia, renal failure, oligo-
sensitivity and is used to rule out disease in hydramnios, persistent anuria following deliv-
Test Block 2

healthy individuals as a screening test. ery, hypotension, pulmonary hypoplasia, limb


contractures secondary to oligohydramnios,
31. The correct answer is D. This question requires and stillbirth. Angiotensin-converting enzyme
one to determine the positive predictive value of inhibitors should be avoided during pregnancy,
the testing method employed. To calculate it, particularly in the second and third trimesters.
Test Block 2 Answers 531

Answer E is incorrect. Thiazides are preg- Answer A is incorrect. 1+ Protein is not a re-
nancy category D drugs, and have been associ- sult of the nitrazine test. Urine can contain var-
ated with electrolyte disorders and thrombocy- ied amounts of protein in kidney disease. The
topenia in newborns. obstetric complication most commonly associ-
ated with protein in the urine is preeclampsia.
33. The correct answer is A. This patient has The presence of protein does not diagnose rup-
Goodpastures syndrome characterized by de- ture of membranes.
position of antibodies to the basement mem-
Answer B is incorrect. The nitrazine test does
brane in glomeruli and alveoli. This leads to
not measure protein. 3++ Protein in urine is
glomerulonephritis with symptoms of renal
suggestive of protein loss via the kidneys, per-
failure, hematuria, and nephritic range prote-
haps associated with preeclampsia. The pres-
inuria, as well as pulmonary hemorrhage caus-
ence of protein does not diagnose rupture of
ing cough and occasionally hemoptysis and
membranes.
bloody effusions at the lung bases. Immunou-
orescence staining of a renal biopsy sample Answer C is incorrect. The ferning pattern, or
shows the pathognomonic linear deposition of arborization, on microscope slide is a separate
the antiglomerular basement membrane anti- test for amniotic sac rupture. Some uid from
body. the posterior vaginal fornix is placed on a mi-
croscope slide and is left to dry for at least 10
Answer B is incorrect. Cryoglobulinemia, of-
minutes. Once dry, the slide is examined for
ten associated with hepatitis B or C infection,
a delicate ferning pattern. A positive ferning
causes glomerular disease associated with ne-
test indicates a possible rupture.
crotizing skin lesions, arthralgias, fevers, and
hepatosplenomegaly. Answer D is incorrect. A pH <7 demonstrates
an acidic environment, which is noted when
Answer C is incorrect. IgA deposition in the
testing urine or vaginal secretions, but not am-
glomerular mesangium is seen in IgA neph-
niotic uid.
ropathy, or Bergers disease. It presents with
gross hematuria and nephrotic syndrome but Answer F is incorrect. Presence of small blood
does not cause pulmonary symptoms. is not diagnostic for rupture of membranes.

FULL-LENGTH EXAMS
The presence of blood, however, can result in
Answer D is incorrect. Postinfectious glo-
a false-positive result for the nitrazine test.
merulonephritis following group A streptococ-
cal infection causes edema, oliguria, and hy-
35. The correct answer is D. Radioactive iodine is
pertension, and does not cause concomitant
the treatment of choice in this patient, who has
pulmonary symptoms such as those seen in
a solitary toxic adenoma. It will shrink her thy-
this patient.
roid gland, and the patient will likely become
Answer E is incorrect. Inammation of small euthyroid in 13 months. Ultimately, 80% of pa-
vessels, or small vessel vasculitides, such as We- tients will become hypothyroid and will require
geners granulomatosis, Churg-Strauss disease, thyroid hormone supplementation (levothyrox-
and microscopic polyangiitis, cause pauciim- ine). Thyroid lobectomy, which would remove
mune glomerulonephritis characterized by re- the tumor without eliminating thyroid function,
nal failure with lack of antibodies on immuno- could also be considered.
uorescence staining.
Answer A is incorrect. -Blockers are com-
monly given at the beginning of thiocarbamide
34. The correct answer is E. The nitrazine test
treatment in order to avoid symptoms of thyro-
measures pH. Amniotic uid is alkaline; there-
toxicosis. It is not given, however, in a similar
Test Block 2

fore, the expected pH would be >7. It is impor-


manner with radioactive iodine treatment.
tant to remember that false positives can occur
in the presence of semen, cervical mucus,
trichomonas infection, blood, and unusually
basic urine.
532 Section II: Full-Length Examinations Answers

Answer B is incorrect. Carbimazole, like pro- suggestive of foreign body aspiration and not
pylthiouracil, is a member of the thiocarba- an asthma attack, intervention via bronchos-
mide class of drugs, and is not appropriate for copy would be the best next step.
toxic adenoma.
Answer C is incorrect. Helical CT scan of the
Answer C is incorrect. Propylthiouracil is a chest will have greater contrast resolution than
member of the thiocarbamide class of drugs, plain radiograph and thus be able to demon-
which block thyroid hormone synthesis by in- strate some airway foreign bodies that are other-
hibiting thyroid peroxidase. While a good treat- wise radiolucent. Also, CT scan can depict the
ment for other causes of hyperthyroidism, it is foreign body within the lumen of the tracheo-
not the treatment of choice for toxic adenoma. bronchial tree and its three-dimensional posi-
tion within the thorax. However, this patients
Answer E is incorrect. Thyroidectomy could
history, examination, and x-ray of the chest are
be an appropriate treatment of toxic adenoma
already highly suggestive of foreign body aspi-
when the nodule is large or refractory to medi-
ration, and intervention via endoscopy would
cal management. Hot nodules (as in this pa-
be the best next step.
tient) have a low likelihood of cancer, thus
medical treatment would be the rst-line treat- Answer D is incorrect. A chest tube should be
ment before surgical intervention. placed when there is a need to drain the con-
tents of the pleural space, whether it be air in
36. The correct answer is B. Partial airway ob- the case of a pneumothorax, blood in the set-
struction occurs when the patients upper air- ting of a hemothorax, pus caused by an empy-
way is partially occluded or if obstruction oc- ema, or uid in the case of an effusion. In this
curs distal to the carina. Patients with partial case the child has a hyperlucent and hyperex-
airway obstructions may present days to weeks panded left lung due to foreign body aspira-
after a foreign body aspiration, and the condi- tion. There is no need for chest tube drainage.
tion may be diagnosed because of sequelae
Answer E is incorrect. MRI of the chest would
such as recurrent pneumonia, persistent
be noninvasive and without radiation exposure,
cough, hemoptysis, atelectasis, or wheezing, as
while displaying high-resolution multiplanar
in this case. The patients chest radiograph
FULL-LENGTH EXAMS

images of soft tissue. Of note, MRI has been


demonstrates collapse of the left lung, hyperlu-
particularly successful in identifying aspirated
cency and hyperexpansion of the left hemitho-
peanuts when depicting the high signal inten-
rax, and a mild deviation of the mediastinum
sity of their fat content in comparison to low-
toward the right chest, all of which suggest the
intensity lung tissue. However, this patients
presence of a foreign body in the left tracheo-
history, examination, and x-ray of the chest are
bronchial tree. Once diagnosed by using radio-
already highly suggestive of foreign body aspi-
graphic ndings or high clinical suspicion, an
ration, and intervention via endoscopy would
aspirated foreign body should be removed via
be the best next step.
rigid bronchoscopy in the controlled setting of
an operating room by personnel trained in air-
37. The correct answer is A. This patients rash,
way foreign body removal.
erythema chronicum migrans, is suggestive of
Answer A is incorrect. Albuterol metered-dose Lyme disease, caused by the spirochete Borre-
inhaler treatment is used in cases of asthma lia burgdorferi, which is transmitted by the bite
exacerbation. Asthma is a hyperreactive airway of Ixodes ticks. It is endemic in the northeast-
disease in which chronic inammation and ex- ern United States. Treatment is oral doxycy-
acerbations of this state can lead to episodes of cline, amoxicillin, or cefuroxime axetil. Pa-
respiratory distress with wheezing, chest tight- tients with certain neurologic or cardiac forms
Test Block 2

ness, breathlessness, coughing, and even air- of the illness may require intravenous treat-
way obstruction. X-ray of the chest ndings are ment with drugs such as ceftriaxone or penicil-
usually normal or show some bilateral hyperin- lin. Cultures have only been shown to be use-
ation. However, because this patients history, ful in diagnosing Lyme disease when taken
examination, and x-ray of the chest are highly early in the course of the disease by biopsying
Test Block 2 Answers 533

the characteristic rash. It is less useful in the Answer C is incorrect. Neurogenic bladder is
case of later disease, or when used on plasma a cause of urinary retention, but is more likely
or cerebrospinal uid specimens. to present with other signs and symptoms of
neurologic disease and additional focal neuro-
Answer B is incorrect. Culturing plasma is less
logic ndings on examination.
sensitive than culturing tissue taken from the
erythema chronicum migrans rash characteris- Answer E is incorrect. Renal tubular inam-
tic of Lyme disease. mation may be seen with a number of causes
of renal failure, such as acute interstitial ne-
Answer C is incorrect. The Lyme antigen test
phritis. However, such processes would not
was designed to be used on urine, and does not
present with urinary retention, and would most
yield reliable results.
likely present with casts or WBCs on urinaly-
Answer D is incorrect. This early in the course sis.
of the disease, ELISA and Western blot may be
Answer F is incorrect. A staghorn renal cal-
falsely negative, due to the delay in seroconver-
culus can cause urinary obstruction, although
sion after the initial exposure. Within 1 week
it is unlikely to be bilateral or to cause acute
of exposure, sensitivity of ELISA and Western
urethral obstruction and postrenal acute renal
blot is only 32%. After several weeks, sensitivity
failure. Additionally, staghorn calculi are as-
rises to 87%.
sociated with chronic urinary tract infection,
Answer E is incorrect. The Lyme antigen test which is not seen in this patients presentation.
is unreliable, and should not be used to sup-
port the diagnosis of Lyme disease. 39. The correct answer is B. This girl has signs
consistent with rickets, a disorder of bone min-
38. The correct answer is D. This is an example of eralization due to hypocalcemia or hypophos-
postrenal, or obstructive, acute renal failure. phatemia. Children may present with dental
The patient most likely has prostate carcinoma enamel hypoplasia, delay in motor milestones,
given the asymmetric, rm, nodular prostate and frequent infectious diseases. Clinical pre-
palpable on physical examination. Back pain sentation may also include tetany, convulsions,
may be evidence of bone metastases. A benign alopecia, and skeletal abnormalities. Skeletal

FULL-LENGTH EXAMS
or malignant enlarged prostate can obstruct the ndings include widened growth plates, frontal
urinary outow tract by blocking the urethra, bossing, enlargement of the wrists, bowing of
leading to acute urinary retention. It may pre- the distal forearm, lateral bowing of the femur
sent with an acute renal failure picture such as and tibia, and delay in closure of the fonta-
this one, in which the patient is anuric and nelles. Hypocalcemic rickets is typically due to
shows laboratory evidence of renal insufciency. a deciency or resistance to vitamin D. This
patient has ndings that suggest vitamin D de-
Answer A is incorrect. Glomerular Ig depo-
pendent rickets type I, which is caused by a
sition is seen in some forms of glomerular
mutation in the gene that encodes the
disease, which may present with renal insuf-
1--hydroxylation enzyme in the kidney. These
ciency. However, glomerular disease is un-
patients will have normal or elevated levels of
likely to present with acute urinary retention,
the precursor, 25(OH)D, but will have signi-
and would present with edema, proteinuria or
cantly low levels of the active metabolite
hematuria, hypertension, and oliguria.
1,25(OH)2D.
Answer B is incorrect. Hypercalcemia is as-
sociated with calcium phosphate renal calculi, Answer A is incorrect. Disorders of the gut
such as inammatory bowel disease, celiac
which can cause acute ureteral obstruction but
disease, cystic brosis, and extensive bowel
Test Block 2

only rarely would cause bilateral obstruction


surgery may cause malabsorption of vitamin
leading to an acute renal failure picture. Ad-
D, which can lead to rickets in children or os-
ditionally, this patients physical examination
teomalacia in adults. These patients will have
is not consistent with nephrolithiasis, which
hypocalcemia and hypophosphatemia, but
would present with ank pain and tenderness.
534 Section II: Full-Length Examinations Answers

they will have decreased levels of both 25(OH) Answer D is incorrect. The diagnosis of
D and 1,25(OH)2D. schizophrenia requires there to be continuous
signs and symptoms of impaired reality test-
Answer C is incorrect. The liver is responsible
ing for at least 6 months in the absence of any
for 25-hydoxylation of vitamin D into 25(OH)
mood symptoms. This patients symptoms have
D. A deciency in this enzyme would result in
only been present for several weeks. In addi-
low levels of both 25(OH)D and 1,25(OH)2D.
tion, peak onset in men is between the ages
The clinical presentation would be very similar
of 18 and 25, although schizophrenia can cer-
to the patient presented here.
tainly be diagnosed in older populations.
Answer D is incorrect. Hypoparathyroidism is
Answer E is incorrect. The Diagnostic and
a cause of hypocalcemia; however, these pa-
Statistical Manual of Mental Disorders, Fourth
tients will have hyper-, not hypophosphatemia.
Edition, Text Revision criteria for specic pho-
In addition, these patients will not have ele-
bia include a marked and persistent fear that
vated 25(OH)D or decreased 1,25(OH)2D.
is excessive and unreasonable and not better
Answer E is incorrect. Vitamin D is synthe- accounted for by some other mental disease.
sized in the skin in response to ultraviolet ra- This patients perception of centipedes is much
diation. Elderly patients, particularly in the better included in the diagnosis of depression
winter, or other people with little sun exposure with psychotic features.
may develop vitamin D deciency. These pa-
tients will also have hypocalcemia and hypo- 41. The correct answer is C. The patients condi-
phosphatemia, but they will have decreased tion is most consistent with a diagnosis of
levels of both 25(OH)D and 1,25(OH)2D. chronic lymphocytic leukemia. Although most
patients with this disease are diagnosed inciden-
40. The correct answer is B. Depression can pre- tally or with complaints of fatigue, 10%20% of
sent with additional symptoms. In this case, the patients present with massive splenomegaly and
patient presented with the symptoms of major resultant anemia and thrombocytopenia. This
depressive disorder (namely, decreased activity, patient has symptoms of both, and his physical
anhedonia, decreased food intake, decreased examination reveals classic petechiae, likely due
FULL-LENGTH EXAMS

sleep, and psychomotor retardation) but has to the low platelet count. Splenectomy in pa-
additional delusions. The key to diagnosis is tients with chronic lymphocytic leukemia in-
that the psychotic features end with the resolu- duced thrombocytopenia corrects the abnormal-
tion of the depressive episode. Depression with ity in 70%85% of cases.
psychotic features often requires special treat-
Answer A is incorrect. High-dose corticoster-
ment with antipsychotic medications and elec-
oids are used as therapy for thrombocytopenia
troconvulsive therapy.
in patients diagnosed with idiopathic throm-
Answer A is incorrect. In a brief psychotic epi- bocytopenic purpura, an antibody-mediated
sode, the psychotic behavior is present for >1 illness causing marked thrombocytopenia.
day but for <1 month (as in this case). However, Corticosteroids may also be used as part of a
the psychotic symptoms must exist in the ab- chemotherapeutic regimen to treat acute lym-
sence of mood symptoms. Brief psychotic disor- phocytic leukemia. They are generally not ef-
der usually carries a much better prognosis. fective in chronic leukemias and should never
be used alone in these illnesses because they
Answer C is incorrect. Schizoaffective disor-
may produce an incomplete remission, after
der is a condition in which the patients psychi-
which the patient has a high risk of relapse
atric symptoms are present during an episode
with refractory disease.
of mood disorder, but then continue for at least
Test Block 2

2 weeks after the resolution of the mood disor- Answer B is incorrect. Platelet transfusion may
der. The centipedes, in this case, disappeared improve this patients thrombocytopenia ini-
with the patients depression, thereby prevent- tially; however, platelets will become seques-
ing a diagnosis of schizoaffective disorder. tered in the patients spleen rapidly. This seques-
Test Block 2 Answers 535

tration would lead to further splenomegaly and Answer C is incorrect. Measurement bias is
thrombocytopenia. when the process of collecting data affects
the outcome. For example, when asked about
Answer D is incorrect. 5-Fluorouracil is a che-
weekly alcohol consumption, people tend to
motherapeutic agent used to treat many types
give answers that are less than the true value.
of cancer. Although the end result of its use
Such a bias is an example of measurement
may be normalization of blood counts as ma-
bias, which is not affected by randomized de-
lignant cells are killed, the immediate response
sign.
is a further decrease in counts from its antimet-
abolic action. Therefore, the initial blood work Answer D is incorrect. Observer bias is when
would reveal a decrease in platelet counts. an observers evaluation of a patients response
to treatment is affected by knowing which
Answer E is incorrect. Granulocyte colony-
study group the patient is in. This is avoided by
stimulating factor is used to stimulate the bone
double-blind design; randomization has no ef-
marrow to produce WBCs following adminis-
fect on observer bias.
tration of chemotherapy. It would not be effec-
tive in treating this patients thrombocytopenia Answer E is incorrect. Recall bias can be
for two reasons. Platelets are not derived from found in retrospective trials and involves a
granulocytes, the site of action of granulocyte patients possible exaggeration of risk factors
colony-stimulating factor. The patients bone or exposures if it is known that the patient de-
marrow is packed with leukemic cells; this veloped the disease putatively associated with
is the reason that his platelet count is so low, them. This is not affected by randomized de-
and any amount of bone marrow stimulation sign.
will be ineffective in producing normal, viable
cells. 43. The correct answer is C. Cricothyroidotomy is
indicated in patients who cannot be intubated
42. The correct answer is A. Enrollment bias is or who have sustained signicant maxillofacial
the aw that occurs when patients are assigned trauma, such as this patient who has multiple
to different study groups in a nonrandom fash- maxillary and mandibular fractures. Carbon di-
ion. When a study is randomized, it ensures oxide retention is a common problem with cri-

FULL-LENGTH EXAMS
that there is not a bias in the way that patients cothyroidotomies so routine arterial blood
were divided into the two study groups, there- gases should be checked on these patients.
fore ensuring that any difference in outcome Currently, cricothyroidotomy and percutane-
between the two groups is due to the interven- ous transtracheal ventilation are preferred over
tion being studied, and not because of differ- tracheostomy in most traumas because they are
ences between the two groups. relatively safe and simple. However, a disad-
vantage of a cricothyroidotomy is the inability
Answer B is incorrect. Lead time bias is when
to use a tube larger than 6 mm because of the
a diagnosis is made earlier than it was before,
limited aperture of the cricothyroid space. Cri-
but no change is made in the true survival.
cothyroidotomy is also contraindicated in pa-
Lead time bias is primarily a concern for stud-
tients <12 years old because of the risk of dam-
ies that seek to measure the survival benet of
age to the cricoid cartilage and the subsequent
disease screening programs. Because screening
risk of subglottic stenosis.
tests may allow for an earlier diagnosis of a dis-
ease (e.g., lung cancer) than would otherwise Answer A is incorrect. The patient is exhibit-
have occurred, patients in the screening groups ing classic signs of shock (hypotension, tachy-
may have a longer period of survival following cardia, mental status changes, tachypnea, dia-
diagnosis than those in the unscreened group, phoresis, and pallor) and circulation is one of
Test Block 2

even if the course of their disease is unchanged the rst things to evaluate in a trauma. How-
by medical interventions. Lead time bias refers ever, an airway is always the primary concern,
to the mistaken interpretation of these data as and in this young patient it is likely that he
demonstrating a survival benet. This is not af- will maintain circulation until an airway is se-
fected by randomized design. cured.
536 Section II: Full-Length Examinations Answers

Answer B is incorrect. Face mask ventilation Answer A is incorrect. Bulimia may present
is not adequate in this patient given the oxygen with vomiting, but it would be induced. Pa-
saturation level and airway patency, and ad- tients with bulimia nervosa have episodes of
equacy of ventilation should take precedence binge eating followed by a compensatory activ-
over other treatment. ity (such as self-induced vomiting or excessive
exercise).
Answer D is incorrect. Nasopharyngeal intu-
bation and oropharyngeal intubation are con- Answer C is incorrect. Early satiety, bloating,
traindicated in patients who have sustained and vomiting are common manifestations of
signicant maxillofacial trauma or who have gastric outlet obstruction. This diagnosis needs
unstable facial fractures. to be conrmed with gastric emptying studies
and is most commonly a complication of pep-
Answer E is incorrect. Nasopharyngeal intu-
tic ulcer disease; without a history of peptic
bation and oropharyngeal intubation are con-
ulcer disease, gastric outlet obstruction is un-
traindicated in patients who have sustained
likely.
signicant maxillofacial trauma or who have
unstable facial fractures. Answer D is incorrect. Vomiting, diarrhea, and
abdominal discomfort are common symptoms
of gastroenteritis. In gastroenteritis there is an in-
Questions 44, 45, and 46 fectious etiology (bacterial or viral). Neither sick
44. The correct answer is B. Digoxin has a low contacts nor foreign travel are mentioned in the
therapeutic index and toxicity is common, espe- vignettes, making gastroenteritis less likely.
cially in the elderly. Toxicity is often signaled by Answer F is incorrect. Hepatitis (acute or
nausea, vomiting, and anorexia. Heart palpita- chronic) can be a cause of abdominal dis-
tions may also be present. Disturbances of color comfort along with fatigue and vomiting, but
vision, with a tendency toward yellow-green col- it will typically present with high aminotrans-
oring, can occur. The patients wifes complaint ferase levels and hyperbilirubinemia. Acutely,
that he has not been acting himself might be patients often appear jaundiced and have a
a sign of mental status changes, which is also a tender right upper quadrant, none of which is
FULL-LENGTH EXAMS

common sign of digitalis toxicity. present in the patient described.

45. The correct answer is O. Sigmoid volvulus is Answer G is incorrect. Inammatory bowel
the most common form of volvulus of the gas- disease is often associated with abdominal pain
trointestinal tract. It is particularly common in and diarrhea, and is a chronic condition with
elderly persons. Patients present with abdomi- a waxing and waning course. A patient with
nal pain, distension, and constipation. Nausea Crohns disease may have small intestine in-
and vomiting may present late. Predisposing volvement causing constipation, but this would
factors include chronic constipation, institu- not be the most likely diagnosis.
tionalization, and an excessively mobile sig- Answer H is incorrect. Nausea and vomiting
moid colon. Findings on x-ray of the abdomen are common with an intracranial hemorrhage,
are usually diagnostic. but headache is equally common, and there is
often an abnormality on neurologic examina-
46. The correct answer is E. Gastroparesis, or de- tion. Also, there is often an underlying reason
layed gastric emptying, describes a paralysis of for the hemorrhage that has not been elicited
the stomach. The most common cause of gas- in any of these patients, such as trauma, bleed-
troparesis is diabetes mellitus. The primary ing diathesis, drug abuse, alcoholism, or hyper-
symptoms are nausea and vomiting, usually af- tension.
Test Block 2

ter meals. Anorexia, weight loss, and early sati-


ety are also common. Heartburn, epigastric Answer I is incorrect. Labyrinthitis (inamma-
pain, and frequent belching also support the tion in the vestibular labyrinth), is acute in on-
diagnosis of gastroparesis. Diabetes affects set, and vertigo, nausea, and vomiting are usu-
phase 3 of the migrating motor complex. ally tied to movement.
Test Block 2 Answers 537

Answer J is incorrect. Meningitis can present Answer N is incorrect. Pancreatitis, an acute


with nausea and vomiting, but it would most inammatory process of the pancreas, presents
likely be accompanied by neurologic symp- with severe abdominal pain and nausea. Lab
toms such as headache and a stiff neck. studies would show elevated lipase and amylase
levels. The two most common risk factors are
Answer K is incorrect. Migraine headaches
alcoholism and gallstones; however, there are
typically involve nausea and vomiting, but
other causes such as hypertriglyceridemia. With-
unilateral or bilateral headache is a prominent
out an appropriate history, pancreatitis is not the
symptom. Migraines also often have neuro-
most likely diagnosis in any of these patients.
logic symptoms (such as scotomas), and may
have a prodrome. Answer P is incorrect. Small bowel obstruc-
tions are common causes of abdominal pain,
Answer L is incorrect. Myocardial infarcts may
nausea, and vomiting. A small bowel obstruc-
atypically present (particularly in women) with
tion most commonly results from postsurgical
nausea and vomiting. But without identiable
adhesions or hernias, and tumors will rarely be
risk factors (smoking, hypertension, elevated
the underlying cause. Small bowel obstruction
LDL cholesterol level, diabetes, or family his-
is a surgical emergency and need to be treated
tory), an MI is not the most likely diagnosis.
before the development of peritoneal signs that
Answer M is incorrect. Pancreatic carcinomas suggest small bowel rupture. Radiographic nd-
often present with an obstruction, as they are ings would include air- and uid-lled loops of
clinically silent until there is a mass effect. They small bowel, and eventually free air under the
may obstruct the small bowel, producing nausea diaphragm if the bowel were to rupture.
and vomiting, or they may obstruct the common
bile duct, producing painless jaundice.

FULL-LENGTH EXAMS
Test Block 2
This page intentionally left blank
Test Block 3

539
540 Section II: Full-Length Examinations Questions

Q U E ST I O N S

1. A 34-year-old woman presents to her primary (C) The varicella virus is teratogenic; there-
care physician complaining of pufness in her fore your newborn may have some mild
legs, arms, and face for 3 days. She often feels birth defects
a little bloated during her menstrual period (D) Your baby must be treated soon after birth
but is currently in the middle of her cycle, and because chickenpox is a serious illness in
states that she has not been drinking more than newborns
usual or eating too much salt. Examination re- (E) Your baby will be protected because of
veals generalized edema but is otherwise unre- your antibodies to the virus
markable. Her physician draws blood for com-
plete blood cell count, electrolytes, and thyroid 3. A 70-year-old smoker with a 50-pack-year smok-
function tests, and does a dipstick urinalysis in ing history is currently using a daily disk in-
his ofce which shows 2+ proteinuria. He de- haler that delivers salmeterol and uticasone.
cides to hospitalize her and perform a renal bi- In addition, she has been recently discharged
opsy to determine the etiology of her nephrotic from the hospital for a chronic obstructive pul-
syndrome. Biopsy shows a thickened basement monary disease exacerbation and is being ta-
membrane and immunouorescence reveals pered off her 2-week course of oral prednisone.
IgG and C3 deposits in a spike and dome At one point, theophylline was added to her
pattern in the basement membrane. Which of regimen during the inpatient stay and she was
the following is the most likely cause of this pa- sent home with a prescription for supplemen-
tients nephropathy? tal oxygen to be delivered for 12 hours during
the night. Which of the agents that this patient
(A) Anti-glomerular basement membrane anti-
is currently taking has been shown to decrease
bodies
overall mortality in patients with chronic ob-
(B) Hepatitis B virus
structive pulmonary disease?
(C) Idiopathic
(D) Penicillamine (A) Inhaled glucocorticoids
FULL-LENGTH EXAMS

(E) Systemic lupus erythematosus (B) Salmeterol


(C) Supplemental oxygen
2. A 26-year-old woman who is 38 weeks pregnant (D) Theophylline
presents to the emergency department in ac-
tive labor. She has felt ill for the past 2 days 4. A full-term girl is born to a 38-year-old white
and has been taking acetaminophen for fevers. woman who received no prenatal care. Length
Last night she broke out in an itchy rash that and weight are in the 50th percentile; head cir-
has spread over her arms and torso. She is a cumference is <5th percentile. She has ve
day care teacher, and 2 weeks earlier one of digits on each hand and foot, as shown in the
the children in her class was diagnosed with image. Her ears are low-set with at pinnae
chickenpox. When questioned, she says she and a pointed upper helix. She also has a III/
does not remember if she had chickenpox as a VI systolic ejection murmur that is loudest at
child. The patient wants to know what this the left sternal border. The karyotype for this
means for her baby. Which of the following is child would most likely show which of the fol-
the best advice to give this woman? lowing?
(A) Nothing needs to be done; chickenpox in
children and newborns is usually a mild,
Test Block 3

self-limiting illness
(B) The chance of transmitting the virus to
your baby is low, so we will monitor the
baby and treat if symptoms develop
Test Block 3 Questions 541

(A) Hepatitis A series


(B) Hepatitis B series
(C) Pneumococcal vaccine
(D) Tetanus booster
(E) Varicella vaccine

7. A 34-year-old man presents with fever and


night sweats for 3 weeks and productive cough.
A recent HIV test was negative. A PPD test per-
formed on admission shows 15 mm induration.
His erythrocyte sedimentation rate is 97 mm/
hr. Past medical history is signicant for a re-
lapse of alcoholism. Review of systems reveals
generalized fatigue over the past month and a
Reproduced, with permission, from Fuster V, ORourke 3.2-kg (7-lb) weight loss. His temperature is
RA, Walsh RA, Poole-Wilson P, eds. Hursts The Heart, 39.6C (103.3F), respiratory rate is 25/min,
12th edition. New York: McGraw-Hill, 2004: Figure and oxygen saturation is 86% on room air. Bi-
12-13. lateral pulmonary rales are noted on physical
(A) Normal 46,XY examination and moderate sternal retractions
(B) One X chromosome and no Y chromo- are present. X-ray of the chest reveals reticu-
some lonodular inltrates spread evenly throughout
(C) Three copies of chromosome 13 both lung elds. Which of the following is the
(D) Three copies of chromosome 18 most likely diagnosis?
(E) Three copies of chromosome 21 (A) Latent tuberculosis
(B) Miliary tuberculosis
5. An 8-year-old boy presents to his pediatrician (C) Pneumocystis jiroveci pneumonia
with recent onset of daily nosebleeds that have (D) Primary tuberculosis
been occurring over the past 2 weeks. He has (E) Reactivation pulmonary tuberculosis

FULL-LENGTH EXAMS
bruises on his legs and arms that appeared after
playing on the playground at school. The pa- 8. A 68-year-old man with coronary artery disease,
tient has a history of easy bruising since early atherosclerosis, and atrial brillation is brought
childhood. He denies painful, tender, or swol- to the emergency department with sudden on-
len joints. The boys mother reports that she set of severe, crampy abdominal pain. The
has a history of von Willebrands disease. A pain began after a massive bowel movement
work-up for von Willebrands disease would and was accompanied by nausea and vomiting.
most likely reveal which of the following? His temperature is 36.8C (98.2F), blood
(A) Increased bleeding time pressure is 136/80 mm Hg, heart rate is 76/
(B) Normal coagulation factor concentrations min, and respiratory rate is 14/min. The pa-
(C) Prolonged prothrombin time tient is writhing in pain without evidence of
(D) Shortened partial thromboplastin time guarding or rebound tenderness. In addition,
(E) Thrombocytopenia the patient states his most recent bowel move-
ment had some blood mixed in with the stool.
6. A 33-year-old man recently diagnosed with Which test is the gold standard for diagnosing
HIV comes to the clinic to reestablish care af- the presenting condition?
ter not seeing a physician for >10 years. Which (A) Angiogram
of the following immunizations is contraindi-
Test Block 3

(B) Barium swallow


cated in this patient at this time? (C) Hepato-iminodiacetic acid scan
(D) Ultrasound of the abdomen
(E) X-ray of the abdomen
542 Section II: Full-Length Examinations Questions

9. A 20-year-old man presents to the clinic with a patient is married with three children ages 3,
painful right testicle for the past month. He 5, and 9 years. All are healthy. Which of the
says the pain has increased in intensity over the following is the most appropriate next step in
past 2 weeks. He denies any penile discharge. management?
He is sexually active and has had several part-
(A) The patient should undergo genetic test-
ners over the past few months. His temperature
ing for Osler-Weber-Rendu syndrome
is 37.2C (98.9F). On examination the right
(B) The patient should undergo genetic test-
testicle is swollen but no obvious mass is palpa-
ing for tuberous sclerosis
ble. He is given the presumptive diagnosis of
(C) The patient should undergo genetic test-
epididymitis and sent home on antibiotics. Af-
ing for von Hippel-Lindau disease
ter completing the course of antibiotics, the
(D) There is no need for the patient or his chil-
patient returns to the clinic still complaining
dren to undergo genetic testing
of swelling and a dull discomfort. What is the
(E) You should proceed directly to testing the
most appropriate imaging study at this point?
patients children
(A) CT of the pelvis
(B) Cystoscopy 12. A 26-year-old man was recently promoted to a
(C) Intravenous pyelogram new position in his corporation, one that re-
(D) MRI of the testicles quires a substantial amount of international
(E) No imaging is necessary, try a second travel. Prior to accepting the position, he had
course of antibiotics not own in 7 years because his father was
(F) Testicular ultrasound killed in an airplane crash. During his rst
transatlantic ight, he developed extreme anxi-
10. A 65-year-old man in the hospital receiving spi- ety, marked by palpitations, chest pain, and dif-
nal anesthesia for crush injuries to his lower culty breathing. He was eventually demoted
extremities starts to notice labored breathing. because of his continued refusal to attend
He is afebrile but develops nausea and vomit- meetings in Europe. Which of the following is
ing, and last had a bowel movement yesterday. this patients likely diagnosis?
His abdomen is distended and tympanic to per-
(A) Angina pectoris
FULL-LENGTH EXAMS

cussion, although there are scattered bowel


(B) Generalized anxiety disorder
sounds. A plain upright abdominal lm reveals
(C) Hyperthyroidism
a largely dilated colon extending from the ce-
(D) Social phobia
cum to the splenic exure. Water-soluble en-
(E) Specic phobia
ema fails to reveal mechanical obstruction.
Which of the following is the most likely diag- 13. A 39-year-old white man is referred to the gas-
nosis? trointestinal clinic for further work-up of long-
(A) Acute colonic pseudo-obstruction standing inammatory bowel disease. He has
(B) Diverticulitis had 1 week of worsening symptoms including
(C) Intussusception frequent, loose, bloody stools and abdominal
(D) Left-sided colonic adenocarcinoma cramping. He is currently taking high-dose oral
(E) Rectal squamous cell carcinoma corticosteroids and cyclosporine, as his disease
has been refractory to mesalamine, sulfasala-
11. A patient with a history of visceral cysts and he- zine, and lower doses of corticosteroids in the
mangioblastomas has recently been diagnosed past. His laboratory tests thus far are negative
with renal cell carcinoma at age 43 years. His for Clostridium difcile, ova, and parasites. Sur-
family history is positive for a maternal grand- gical resection is performed and a specimen is
Test Block 3

father and an uncle, both of whom died in shown in the image. Which of the following
their 50s from renal cell carcinoma. His ndings is characteristic of this patients condi-
mother died of a brain tumor shortly after tion?
giving birth to him. He has no siblings. The
Test Block 3 Questions 543

15. A 36-year-old woman presents to an infertility


clinic with her husband. They married 1 year
ago and have been trying to conceive ever
since. She states she is in good health, al-
though she complains of irregular menses over
the past year, with feelings of inadequacy, dys-
pareunia, difculty sleeping, and episodes of
warmth and sweating at night. Her history re-
veals no history of thyroid disease, no galactor-
rhea, and a body mass index of 30 kg/m. She
uses no medications or street drugs, but notes a
Reprinted, with permission, from PEIR Digital Library 15-pack-year smoking history. She has never
(http://peir.net). been pregnant, nor has she ever had any sexu-
ally transmitted diseases. She has not menstru-
(A) Absence of ganglion cells on full-thickness
ated in 4 months, but pregnancy tests have
rectal biopsy
been repeatedly negative. Her husband, who is
(B) An enterovesicular stula
also 36 years old, is also in good health and has
(C) Cobblestoning
two children from a previous marriage. Which
(D) Crypt abscesses
of the following is the best next step in man-
(E) Transmural inammation
agement?
14. A 35-year-old African-American woman pre- (A) Measure follicle-stimulating hormone and
sents to the emergency department complain- luteinizing hormone levels
ing of fever and cough. She says that the cough (B) Obtain a hysterosalpingogram
is productive and the sputum looks like dark (C) Obtain a postcoital test
blood. On questioning she admits that she (D) Perform a progestin challenge
sometimes experiences shortness of breath and (E) Perform MRI of the brain
right-sided chest pain. She also admits binge
drinking to the point of passing out several 16. A 40-year-old man is brought to the emergency

FULL-LENGTH EXAMS
times a week. Her temperature is 39.4C department after being involved in a motor ve-
(103F). A chest radiograph shows consolida- hicle crash. He is hemodynamically stable. Pri-
tion of the upper and right middle lobes with mary and secondary surveys are performed,
loss of the right heart border. Which of the fol- and his injuries seem to consist of a right hip
lowing is the most likely pathogen? dislocation and a small laceration on his fore-
head. His right lower extremity is shortened,
(A) Klebsiella pneumoniae
exed, adducted, and internally rotated at the
(B) Legionella pneumophilia
hip. His right lower extremity was warm and
(C) Pneumocystis jiroveci pneumonia
well perfused with distal pulses intact. An or-
(D) Pseudomonas aeruginosa pneumonia
thopedic surgeon is called because of a sus-
(E) Streptococcus pneumoniae
pected closed hip dislocation. Prompt treat-
ment will signicantly reduce the incidence of
what complication?
(A) Avascular necrosis
(B) Femoral nerve damage
(C) Myositis ossicans
(D) Posttraumatic degenerative arthritis
Test Block 3

(E) Recurrent dislocation


544 Section II: Full-Length Examinations Questions

17. A 78-year-old man with a 9-year history of (A) Cardiac catheterization


chronic myelogenous leukemia treated with (B) Clinical judgment based on physical
imatinib presents with weight loss and early sa- examination
tiety. Splenomegaly is noted on examination, (C) CT scan of the chest
and routine laboratory evaluation reveals a (D) Echocardiography
WBC count of 11,000/mm, hemoglobin of (E) Electrocardiogram
7.5 g/dL, and platelet count of 460,000/mm. (F) MRI of the heart
An attempted bone marrow aspiration is unsuc- (G) X-ray of the chest
cessful. Bone marrow biopsy is then performed
and yields the image shown. Which of the fol- 19. A 22-month-old boy presents to the pediatri-
lowing is the most likely diagnosis? cian because he has not yet begun to walk. He
was able to hold his head steady at 5 months,
sit unsupported at 8 months, and crawl at 12
months. When he crawls, he drags his legs. He
takes no medications and has no known aller-
gies. His family history is noncontributory. On
examination his legs are hypertonic bilaterally
with brisk patellar reexes; he has ankle clonus
and upgoing toes bilaterally. Strength is 4 of 5
in his legs bilaterally and 5 of 5 in his arms bi-
laterally. What is the most appropriate therapy?
(A) ACTH and clonazepam
(B) Benztropine
Reprinted, with permission, from Lichtman MA, Beutler (C) Diazepam and physical therapy
E, Kipps TJ, Seligsohn U, Kaushansky K, Prchal JT. Wil- (D) Levodopa and carbidopa
liams Hematology, 7th edition. New York: McGraw-Hill, (E) Pimozide
2006: Plate XIV-11.

(A) Bernard-Soulier syndrome 20. Which of the following is an appropriate


FULL-LENGTH EXAMS

(B) Idiopathic sideroblastic anemia choice in the management of a patient with a


(C) Myelodysplastic syndrome heroin overdose?
(D) Myelobrosis (A) Administer 4 mg of intravenous naloxone
(E) Progression of chronic myelogenous leuke- (B) Assess the patients kidney function
mia to blast phase (C) Check the patients respiratory function
(F) Second malignant neoplasm resulting and establish airway access if necessary
from exposure to alkylating agents (D) Monitor the patient for potentially life-
threatening withdrawal symptoms
18. A 71-year-old woman presents to an ambula- (E) Treat withdrawal symptoms with benzodi-
tory clinic with a chief complaint of dyspnea azepines
upon exertion. Over the past few weeks she has
had a chronic cough and shortness of breath 21. A 45-year-old HIV-positive man with a CD4+
when walking more than two city blocks. She cell count of 63/mm presents to the emer-
has a long history of hypertension that has been gency department with a headache and fever.
poorly controlled in recent years. On physical Treatment for bacterial meningitis is initiated
examination she has an elevated jugular ve- while the physician runs additional laboratory
nous pulse and rales are evident on lung exam- tests. However, his symptoms fail to improve
Test Block 3

ination. Cardiac enzyme tests are negative. with treatment and all bacterial cultures return
What is the most appropriate next step in dis- negative. Lumbar puncture plus which of the
tinguishing systolic from diastolic heart failure? following are most likely to conrm the diag-
nosis?
Test Block 3 Questions 545

(A) Cryptococcal antigen test (A) Over the dorsum of the hand
(B) Culture (B) Over the medial side of the palm
(C) India ink stain (C) Over the palmar aspect of the third digit
(D) Potassium hydroxide preparation (D) Over the palmar aspect of the thumb
(E) Silver stain (E) Over the superior lateral upper arm

22. A 27-year-old G1P1 woman presents to her ob- 24. The Framingham Heart Study began in 1950
stetrician 3 weeks after an uncomplicated labor with the intent of collecting data on risk factors
and delivery of a healthy, full-term, 3.2-kg (7-lb) and following patients over time to assess the
baby boy. The patient states that she has been effect of these factors on the development of
nursing her son without trouble since delivery disease. This is an example of which type of
until several days ago, when she noticed that her study design?
right nipple was red, dry, and cracked. Her right
(A) Case-control
breast has become increasingly sensitive, yet she
(B) Meta-analysis
has continued breast-feeding her son despite the
(C) Prospective cohort
discomfort. Her temperature is 38C (100.4F).
(D) Randomized controlled trial
Physical examination reveals a warm, well-cir-
(E) Retrospective cohort
cumscribed, 5-cm erythematous patch on the
right breast that is tender to light touch. No pus 25. A 54-year-old African-American man presents
can be expressed from the right nipple. Which to his physician complaining of difculty uri-
of the following is the most appropriate initial nating. He urinates more often during the day
therapy? and often wakes at night to urinate. He also has
(A) Incision and drainage difculty starting and maintaining a stream of
(B) Intravenous oxacillin urine. Digital rectal examination reveals a dif-
(C) Oral cephalexin fusely large, rubbery prostate. Urinalysis and
(D) Oral doxycycline urine culture reveal no hematuria or signs of
(E) Warm compresses infection. What is the best next step in diagno-
sis?

FULL-LENGTH EXAMS
23. A 16-year-old football quarterback is in the
(A) Cystoscopy
midst of throwing a football with his shoulder
(B) Measure creatinine levels
in an abducted and externally rotated position.
(C) Measure prostate-specic antigen level
His throwing arm is suddenly hit by an oncom-
(D) Ultrasound-guided transrectal prostate
ing defender, and he senses an immediate pop
biopsy
and pain in his shoulder. He is unable to move
(E) X-ray of the chest and bone scan
his shoulder without intense discomfort and
maintains it in an adducted, externally rotated
position. He states that his whole arm hurts
and feels heavy. A lump is palpated anteriorly
at his shoulder, and a defect is noted below his
acromioclavicular joint. The athletic trainer
fears an associated nerve injury. Where should
the trainer test light touch to assess the integ-
rity of this nerve?
Test Block 3
546 Section II: Full-Length Examinations Questions

26. A 42-year-old woman with stage IV breast can- (E) Nitroglycerin 0.3 mg sublingually as
cer presents to her oncologist with a complaint needed
of worsening chest pain over the past 2 weeks.
The pain is retrosternal, worsens when she lies 27. A 21-year-old college student is brought to the
down, and improves when she leans forward. emergency department after having attended a
She carries a diagnosis of neoplastic pericardi- party at which he was offered a substance blot-
tis and has had similar symptoms in the past. ted on a small sheet of paper. A few minutes af-
She has tried nonsteroidal anti-inammatory ter placing the substance on his tongue, the
drugs, colchicine, and even steroids, but these student noticed that the edges of the wall had
medications have only given her temporary re- begun to look like cylindrical bars, with ash-
lief and her chest pain soon returns. She has ing lights alternating in bright colors alongside
no other medical problems. She takes tamox- the bars. At one point he felt that the walls of
ifen and bisphosphonates. On physical exami- the room were shifting horizontally and spin-
nation she is tearful and appears to be in pain. ning. This effect lasted for about an hour and
She has a regular rate and rhythm, normal S1 then dissipated. Which of the following is the
and S2, and a pericardial friction rub. Her most likely cause of this patients symptoms?
lungs are clear to auscultation bilaterally. The (A) Cocaine
remainder of her physical examination is be- (B) Ecstasy
nign. Her temperature is 37.5C (99.5F), (C) Heroin
heart rate is 80/min, blood pressure is 132/78 (D) Lysergic acid diethylamide
mm Hg, respiratory rate is 16/min, and oxygen (E) Marijuana
saturation is 99% on room air. Her ECG shows
diffuse ST-segment elevation. X-ray of the chest 28. A 70-year-old man presents with a painless
is shown in the image. Which of the following swelling of the right supraclavicular area. He
is the best way to manage this patients chest states he has lost 11.3 kg (25 lb) over the past 3
pain? months and describes generalized pruritus. On
examination he is cachectic and his spleen is
palpable 2 cm below the costal margin. A mat-
FULL-LENGTH EXAMS

ted mass is palpable in the right supraclavicu-


lar area. X-ray of the chest reveals mediastinal
lymphadenopathy. The supraclavicular mass is
biopsied and reveals many Reed-Sternberg
cells with few lymphocytes. The man is treated
with chemotherapy and radiation therapy but
shows little response. He dies from respiratory
failure 6 months later. Which of the following
is the most likely diagnosis?
(A) Burkitts lymphoma
(B) Lymphocyte-depleted Hodgkins lym-
phoma
(C) Lymphocyte-predominant Hodgkins lym-
phoma
(D) Nodular sclerosing lymphoma
Reprinted, with permission, from Chen MYM, Pope TL, (E) Non-Hodgkins lymphoma
Ott DJ. Basic Radiology. New York: McGraw-Hill, 2004:
Figure 4-52.
(F) T lymphocyte leukemia
Test Block 3

(A) Balloon pericardiotomy 29. A 15-year-old girl presents to the outpatient


(B) Emergent pericardiocentesis clinic complaining of progressive pain around
(C) Ibuprofen 600 mg orally three times a day her left knee that has become intractable. She
(D) Morphine patient-controlled analgesia states the pain is worst at night and does not re-
spond to nonsteroidal anti-inammatory medi-
Test Block 3 Questions 547

cations. She also admits to having intermittent from the ear. There is redness around the base
fever and gradual weight loss over the past 3 of the ear, and MRI shows bone involvement.
months. On examination the left thigh is swol- The patient denies having a fever, but states
len, tense, and tender. Laboratory tests show: that he occasionally sweats at night and has a
headache. The patients temperature is 37.9C
Hemoglobin: 12.4 g/dL
(100.2F). Which of the following is the most
WBC count: 10,600/mm
appropriate pharmacotherapy?
Platelet count: 380,000/mm
Erythrocyte sedimentation rate: 18 mm/hr (A) A uoroquinolone plus a -lactam
Alkaline phosphatase: 320 U/L (B) Amoxicillin
(C) Amphotericin B
X-ray studies are shown in the image. Which of
(D) Methicillin
the following is the most likely diagnosis?
(E) Topical antibiotics

31. A 72-year-old woman presents to the clinic


with a chief complaint of weakness and fatigue.
She has had difculty breathing over the past
several weeks and now must sleep on two pil-
lows at night to get comfortable. In addition,
she reports an episode of syncope last week.
Cardiac biopsy reveals amyloid deposits.
Which of these ndings is most consistent with
the biopsy results?
(A) Ejection fraction of 55% on echocardiogra-
phy
(B) Harsh diastolic murmur
(C) Right lower quadrant abdominal mass
(D) U waves on ECG
(E) Upper extremity edema

FULL-LENGTH EXAMS
(F) Wheezing on lung examination

32. A 68-year-old woman with a central retinal


vein occlusion presents to the ophthalmology
Reproduced, with permission, from Doherty GM, Way
clinic for an annual examination. She had a
LW. Current Surgical Diagnosis and Treatment, 12th edi-
tion. New York: McGraw-Hill, 2006: Figure 42-49.
cerebrovascular accident 2 years ago. She notes
that for the past 3 months, she has been experi-
(A) Fibrous dysplasia encing periorbital pain around her right eye,
(B) Nonossifying broma associated with an ipsilateral headache. She
(C) Osteoid osteoma also complains of intermittent blurry vision,
(D) Osteomyelitis and seeing halos around objects. Which of
(E) Osteosarcoma the following is the most appropriate curative
treatment for this patient?
30. A 45-year-old African-American man with dia-
betes reports to his primary care physician (A) Acetazolamide
complaining of unbearable pain in his left ear. (B) Atropine
The patient states the pain is worse with chew- (C) Laser iridotomy
ing and any movement of his head or ear lobe. (D) Pilocarpine
Test Block 3

The primary care physician notes pus draining (E) Timolol


548 Section II: Full-Length Examinations Questions

33. A 57-year-old man presents to the physician (A) Inadequate nutrition from formula
with a 4-month history of difculty swallowing (B) Maternal alcohol use during pregnancy
food, which has worsened slightly during that (C) Maternal cocaine use during pregnancy
period. Upon further questioning the physician (D) Maternal use of antithyroid medication
discovers that he takes a proton pump inhibitor (E) Poorly controlled maternal diabetes
for a long-standing history of gastroesophageal
reux disease (GERD). Because of his GERD, 36. A 67-year-old woman recently started a new
he carefully watches what he eats, and he does medication to treat her essential hypertension.
not drink any alcohol, nor does he smoke. He At a previous primary care visit her blood pres-
had a normal upper endoscopy approximately sure was 145/88 mm Hg, so she was sent home
2 years ago. Which of the following is the most with instructions to adopt a low-salt diet. On
likely explanation for his current symptoms? returning to the clinic her blood pressure was
148/90 mm Hg, and she was started on an anti-
(A) Achalasia
hypertensive. Two weeks into this course she
(B) Barretts esophagus
began to complain of muscle weakness and dif-
(C) Benign esophageal stricture
culty breathing, so she returns to her physi-
(D) Diffuse esophageal spasm
cian. Suspecting a dysrhythmia, the physician
(E) Esophageal cancer
orders an ECG (see image). Which of the fol-
34. A 40-year-old G3P3 woman presents to her gy- lowing medications could have led to this ad-
necologist for urine leakage. She has had dia- verse effect?
betes for many years and it has been poorly
controlled. When asked further about her
urine leakage, she reports it mainly occurs if
she coughs or sneezes when she has a full blad-
der. Which of the following is most likely to re-
solve this womans incontinence?
(A) Anticholinergic medication
(B) Intermittent catheterization
FULL-LENGTH EXAMS

(C) Sacral nerve stimulation


(D) Surgery Reproduced, with permission, from Fuster V, ORourke RA,
(E) Vaginal estrogen cream Walsh RA, Poole-Wilson P, eds. Hursts The Heart, 12th edi-
tion. New York: McGraw-Hill, 2008: Figure 13-35.
35. A 1-week-old term infant is brought to the pe- (A) Acetazolamide
diatrician for her rst visit following a home (B) Furosemide
birth. Her father is concerned that she sleeps (C) Hydralazine
too much and is only having bowel movements (D) Hydrochlorothiazide
every 34 days. She passed meconium on the (E) Losartan
rst day of life. He is worried that his daughter
is sick because his wife partied heavily dur- 37. An otherwise healthy 25-year-old medical stu-
ing the rst 2 months of pregnancy before they dent presents to the emergency department
realized she was pregnant. The maternal medi- with a 2-week history of chills, fever, and mal-
cal history is also signicant for an overactive aise. He has a fever of 38.3C (101F) and sta-
neck gland for which she takes medicine. The ble vital signs with no other abnormal ndings.
infants height, weight, and head circumfer- Urine, blood, and sputum cultures are taken
ence are all in the 50th percentile. Her tem- and all are negative for growth. The patient is
Test Block 3

perature is 35C (95F). She has an enlarged admitted for observation and further testing,
anterior fontanelle, enlarged tongue, poor yet after a week no explanation is found.
muscle tone, and dry skin. Which of the fol- Which of the following is the most likely cause
lowing is the most likely etiology for the in- of fever in this patient?
fants condition?
Test Block 3 Questions 549

(A) Collagen vascular disease spouse says the patient complained of shaki-
(B) Granulomatous disease ness and a racing heartbeat. The patient has
(C) Infection type 2 diabetes mellitus and recently started
(D) Neoplasm sulfonylurea therapy. The patient also began
(E) Pulmonary emboli taking 81 mg of aspirin at the time of her dia-
betes diagnosis. There is a family history of
38. A 54-year-old man visits his physician for a reg- heart disease. On examination the patient is
ular check-up. The man has a history of diver- oriented to person only, appears lethargic, and
ticular disease and diabetes and has no current is diaphoretic. Her temperature is 36.9C
medical complaints. He denies tobacco use or (98.4F), blood pressure is 155/91 mm Hg,
any family history of cardiac disease. On exami- pulse is 112/min, and respiratory rate is 18/
nation, the patient is afebrile and normoten- min. Laboratory studies are pending. Which of
sive. His lipid prole shows a total cholesterol the following is most likely to be used in the
level of 230 mg/dL, LDL cholesterol of 135 management of this patient?
mg/dL, and HDL cholesterol of 47 mg/dL. In
(A) Epinephrine administration
terms of managing this mans hyperlipidemia,
(B) Insulin administration
what is the best next step?
(C) Naloxone administration
(A) Lower LDL cholesterol to <100 mg/dL (D) Normalization of glucose levels
(B) Lower LDL cholesterol to <130 mg/dL (E) Tissue plasminogen activator administra-
(C) Raise HDL cholesterol to >50 mg/dL tion
(D) Recommend a high-ber and low-fat diet
(E) Recommend regular exercise 41. A 37-year-old man presents to his new primary
care physician with a small, symmetric, mono-
39. A 26-year-old woman, 14 weeks pregnant with chromatic brown macule with regular borders
her rst child, presents to the emergency de- that has been on his forearm for many years.
partment complaining of light vaginal bleeding After complaining that none of my doctors
and mild cramping that began the previous have taken me seriously, he states that he be-
day. Her blood pressure is 112/80 mm Hg, lieves he is dying of melanoma. He adds that

FULL-LENGTH EXAMS
heart rate is 90/min, respiratory rate is 16/min, he has seen several specialists over the past
and temperature is 37C (98.6F). Pelvic ex- year, and while all of them have assured him
amination reveals some blood in the vaginal that his lesion is just a mole, he is certain
vault without evidence of clots or tissue, and that this is not the case. The patient has no
the cervical os is closed. Fetal heart tones are other complaints, no systemic symptoms, and
present at 135/min. What is the most appropri- no medical or surgical history. Which of the
ate next step in management? following is the most likely diagnosis?
(A) Betamethasone administration (A) Basal cell carcinoma
(B) Cervical cerclage (B) Body dysmorphic disorder
(C) Dilation and curettage (C) Hypochondriasis
(D) Magnesium sulfate administration (D) Somatization disorder
(E) Observation (E) Squamous cell carcinoma

40. A 56-year-old woman is brought to the emer-


gency department by her spouse for evaluation
of obtundation. Earlier in the evening, the
Test Block 3
550 Section II: Full-Length Examinations Questions

42. A 72-year-old woman undergoes three-vessel 44. On routine evaluation, a 52-year-old woman is
coronary artery bypass graft for coronary artery found to have a serum calcium level of 12.2
disease. Her postoperative course is compli- mg/dL and an elevated serum parathyroid hor-
cated by wound infection leading to sepsis with mone level. On further questioning she admits
multidrug-resistant Klebsiella. Her sepsis is dif- to daily muscle pain that has been increasing
cult to manage, and she is on pressors, uids, in intensity. Which of the following is the most
and antibiotics for several days before becom- likely cause of her condition?
ing afebrile and hemodynamically stable. Her
(A) Ectopic parathyroid tissue
creatinine rises postoperatively, and on postop-
(B) Multiple adenoma
erative day 4 electrolytes show blood urea ni-
(C) Parathyroid carcinoma
trogen (BUN) of 34 mg/dL and creatinine of
(D) Parathyroid hyperplasia
2.6 mg/dL. Urine sodium level is 46 mEq/L,
(E) Single adenoma
fractional excretion of sodium is calculated as
2.1%, and urine osmolarity is 310 mOsm. Uri-
nalysis shows muddy brown granular casts. E X T E N D E D M ATC H I N G
Over the next week her creatinine level contin-
ues to rise despite aggressive uid manage- The response options for the next 2 items are
ment. However, she continues to make small the same. Select one answer for each item
amounts of urine, and BUN and creatinine in the set.
levels gradually return to normal over the next
few weeks. Which of the following is the most For each patients infection, select the most likely
likely pathogenesis of this womans acute renal etiologic agent.
failure?
(A) Bacillus cereus
(A) Allergic reaction to dobutamine (B) Campylobacter jejuni
(B) Autoimmune glomerular destruction (C) Clostridium difcile
(C) Hypovolemic shock (D) Enteric adenovirus
(D) Ischemic tubular injury (E) Escherichia coli
(E) Klebsiella infection of the kidney (F) Giardia lamblia
FULL-LENGTH EXAMS

(G) Rotavirus
43. A 22-year-old G2P1 woman at 38 weeks gesta- (H) Salmonella
tion presents to the labor and delivery suite in (I) Shigella
labor after a pregnancy with poor prenatal (J) Staphylococcus aureus
care. After several hours of labor, her tempera- (K) Vibrio cholerae
ture is 39.1C (102.4) and she has signicant (L) Vibrio parahaemolyticus
abdominal tenderness. Her WBC count is
18,400/mm. The child is delivered with Apgar 45. The parents of an otherwise healthy 13-month-
scores of 8 at 1 and 5 minutes, but is lethargic old boy present to the emergency department
the day after delivery, with a temperature of in the winter concerned about their sons re-
34.8C (94.6F), respiratory rate of 24/min, cent diarrhea and vomiting. They report that
heart rate of 186/min, and mean blood pres- he developed a mild fever approximately 2 days
sure of 32 mm Hg. Which of the following is ago along with nonbloody, nonbilious vomit-
the most likely cause of this neonates condi- ing. They took him to the pediatrician at that
tion? time, and he was diagnosed with gastroenteri-
(A) Chlamydia trachomatis infection tis. His parents were told to monitor his uids
(B) Group B streptococci infection closely and return if his condition changed. He
has now stopped vomiting but has developed
Test Block 3

(C) HIV infection


(D) Toxoplasmosis watery stools that are passed approximately ev-
(E) Treponema palladium infection ery 12 hours. His parents report that he at-
tends day care, and recently there have been
Test Block 3 Questions 551

numerous children out of the program because the past 48 hours includes cereals, fruits, egg
of similar illness. His heart rate is 122/min, salad sandwiches, steamed vegetables, ham-
blood pressure is 80/50 mm Hg, and respira- burger, grilled chicken, Christmas cookies,
tory rate is 28/min. His physical examination is and homemade eggnog. At the time of evalua-
notable for dry mucous membranes. tion, her vital signs are stable but she has a
mild fever of 38.4C (101.1F). She is fatigued
46. An otherwise healthy 9-year-old girl is brought but alert. Her abdominal examination is nota-
to the emergency department after the acute ble for periumbilical tenderness, but no guard-
development of fever (38.8C [101.8F]), nau- ing or rebound tenderness is noted, and no
sea, vomiting, diarrhea, and crampy periumbil- masses are palpated.
ical abdominal pain. A review of her diet over

FULL-LENGTH EXAMS
Test Block 3
552 Section II: Full-Length Examinations Answers

AN S W E R S

1. The correct answer is C. Nephrotic syndrome with mothers infected within 2 weeks of deliv-
with the histology pattern of membranous ery are at greatest risk for complications, as ma-
nephropathy is the most common cause of ternal antibodies may not have been available
nephropathy among white adults and is most to confer passive immunity to the infant. Pa-
frequently idiopathic. Up to one-third of pa- tients should be treated with varicella-zoster
tients undergo spontaneous remission, one- immune globulin.
third continue to have proteinuria but maintain
Answer A is incorrect. If untreated, chicken-
stable renal function, and the last third progress
pox infection in newborns carries an approxi-
to end-stage renal failure at 510 years. Treat-
mately 30% mortality rate.
ment is controversial, as corticosteroids alone
as a primary therapy have not been proven to Answer B is incorrect. If untreated, chickenpox
be effective and the risks of cytotoxic therapy infection in newborns carries an approximately
do not justify their use when spontaneous par- 30% mortality rate. Treatment with varicella-
tial to complete remission may occur in up to zoster immune globulin is started before symp-
40% of patients. Currently, cytotoxic therapy toms develop for maximum efcacy.
should only be offered to patients who are con- Answer C is incorrect. Varicella infection dur-
sidered to be at high risk for progression based ing the rst trimester can result in limb and
on clinical risk factors. digit defects, but the risk is small (approxi-
Answer A is incorrect. Anti-glomerular base- mately 1%).
ment membrane antibodies are seen in Good- Answer E is incorrect. If a woman develops
pastures syndrome, which presents with ne- symptoms from varicella virus (as this woman
phritic syndrome and signs of pulmonary has), she will not have adequate antibodies
hemorrhage such as hemoptysis. Renal biopsy against the virus in time to pass them on to the
in this disorder shows linear deposits of the an- infant before delivery.
tibody, which is different from the histologic
FULL-LENGTH EXAMS

pattern seen in membranous glomerulone- 3. The correct answer is C. Supplemental oxy-


phropathies. gen is the only therapy demonstrated to de-
Answer B is incorrect. The membranous crease mortality in patients with chronic ob-
nephropathy histology may also be seen in pa- structive pulmonary disease (COPD). For
tients with hepatitis B, but this is less common patients with resting hypoxemia (resting arte-
than the idiopathic form. rial oxygen saturation <88%, or <90% with
signs of pulmonary hypertension or right heart
Answer D is incorrect. The membranous failure), the use of oxygen has been demon-
nephropathy histology may be seen in renal strated to have a signicant impact on mortal-
disease associated with the drugs penicillamine ity. The Medical Research Council Trial dem-
and gold, which may be used to treat rheuma- onstrated that 12 hours per day was superior to
toid arthritis. This patient, however, has no his- no oxygen supplementation, but continuous
tory of rheumatologic disease. oxygen was even better.
Answer E is incorrect. The membranous ne- Answer A is incorrect. Several recent trials
phropathy histology may also be seen in pa- have failed to nd a benecial effect for the
tients with systemic lupus erythematosus, but regular use of inhaled glucocorticoids on the
this is less common than the idiopathic form. rate of decline of lung function, as assessed by
Test Block 3

FEV1.
2. The correct answer is D. Varicella is a highly
contagious virus that causes a potentially life- Answer B is incorrect. Although COPD is
threatening illness in newborns. The presenta- characterized by airway inammation/obstruc-
tion is variable but can include fever, vesicular tion that is largely irreversible, salmeterol given
lesions, pneumonia, or meningitis. Newborns in the acute setting can be used to improve any
Test Block 3 Answers 553

component of airway constriction that is revers- abnormalities in the coagulation pathway. Be-
ible. No studies have shown that it affects over- cause bleeding time correlates with platelet
all mortality in COPD. function, vWD is associated with decreased
platelet interaction with the vessel endothe-
Answer D is incorrect. Theophylline has not
lium and subsequent increased bleeding time.
been shown to decrease mortality in COPD.
Answer B is incorrect. vWF normally stabi-
4. The correct answer is D. The infant has microg- lizes factor VIII in the circulation. Therefore,
nathia, low-set/malformed ears, and abnormal deciency of vWF may lead to factor VIII pro-
clenched ngers, which are all signs of trisomy teolysis with reduced levels of factor VIII in the
18, or Edwards syndrome. Other common nd- circulation.
ings include rocker-bottom feet, a prominent oc-
Answer C is incorrect. Because vWF stabi-
ciput, and heart murmurs consistent with con-
lizes factor VIII of the coagulation pathway,
genital heart disease. Risk factors include
increased partial thromboplastin time (PTT)
maternal age >35 years.
(due to decreased factor VIII concentration)
Answer A is incorrect. Karyotypes are useful may occur in a patient with abnormal vWF.
in detecting large translocations and deletions, Factor VIII is part of the intrinsic coagulation
aneuploidy, and polyploidy. There are many pathway, which corresponds to PTT, not pro-
congenital defects resulting from single gene thrombin time (PT). Therefore, decreased lev-
disorders, microdeletions, small translocations, els of factor VIII will not lead to prolonged PT,
uniparental disomy, and environmental factors only prolonged PTT.
that are undetectable by karyotype analysis.
Answer D is incorrect. vWF normally stabi-
Edwards syndrome, however, is detectable by
lizes factor VIII in the circulation. Abnormali-
karyotype analysis.
ties of vWF, such as those seen in vWD, are as-
Answer B is incorrect. Turners syndrome, sociated with a prolonged (not shortened) PTT
chromosomal complement XO, presents at due to increased proteolysis of factor VIII and
birth with webbing of the neck and coarctation decreased factor VIII concentrations.
of the aorta. Often it is not detected until pu-
Answer E is incorrect. Patients with vWD have

FULL-LENGTH EXAMS
berty, when these females present with primary
normal platelet counts, but abnormal platelet
amenorrhea secondary to ovarian dysgenesis.
function, resulting in abnormal interactions
Short stature is another common nding in
between platelets and the vessel endothelium.
Turners syndrome.
Because there is no indication of decreased
Answer C is incorrect. Trisomy 13, or Pa- platelet formation or increased platelet destruc-
taus syndrome, classically presents with mi- tion in a patient with vWD, the patient would
crophthalmia, microcephaly, cleft lip and/or not have disease-related thrombocytopenia, ex-
cleft palate, polydactyly, and congenital heart cept in the 2N variant. The 2N variant is very
disease. rare, and patients will present with hematuria,
Answer E is incorrect. Trisomy 21, or Downs joint and soft tissue bleeding, unlike the other
syndrome, classically presents with at facial variants of vWD that present mostly with mu-
prole, prominent epicanthal folds, simian cocutaneous bleeds.
creases, congenital heart disease, and mental
6. The correct answer is E. Live vaccines are not
retardation.
recommended for immunocompromised indi-
5. The correct answer is A. Von Willebrands fac- viduals. Live attenuated viral vaccines include
tor (vWF) forms a bridge between platelets and oral poliovirus, measles, varicella, mumps, and
Test Block 3

vessel endothelium and acts as a carrier protein yellow fever vaccines.


for factor VIII of the coagulation pathway. Pa-
tients with von Willebrands disease (vWD;
qualitative or quantitative defects in vWF)
therefore have defective platelet function and
554 Section II: Full-Length Examinations Answers

Answer A is incorrect. The hepatitis A series is sidered in an elderly patient that presents with
recommended for all HIV-positive patients. It acute, intense abdominal pain that is accompa-
can be administered with the hepatitis B vac- nied by diarrhea, a large bowel movement,
cine. and/or nausea and vomiting. This is a diagnosis
that requires immediate surgical attention and
Answer B is incorrect. The hepatitis B series is
has a mortality rate of about 80%. It is very im-
recommended for all HIV-positive patients. It
portant to recognize the risk factors and exami-
can be administered with the hepatitis A vac-
nation ndings so that a rapid diagnosis can be
cine.
made. This condition requires emergent surgi-
Answer C is incorrect. The pneumococcal cal intervention. Classically, patients with a di-
vaccine is recommended for all HIV-positive agnosis of acute mesenteric ischemia present
patients every 35 years. with pain out of proportion to physical exami-
Answer D is incorrect. A tetanus booster is rec- nation ndings. The heart is the most common
ommended for all patients every 10 years. source of an embolus that lodges in the supe-
rior mesenteric artery, leading to acute mesen-
7. The correct answer is B. Miliary tuberculosis teric ischemia. An angiogram remains the gold
(TB) refers to the hematogenous spread of My- standard for diagnosing mesenteric ischemia. If
cobacterium tuberculosis following reactivation an embolus is found by angiogram, the patient
of a latent infection or progressive primary in- must be taken to the operating room immedi-
fection. The clinical presentation of miliary ately for surgical embolectomy and possible
TB varies from fever of unknown origin to mul- ischemic bowel resection.
tisystem organ failure. The important informa- Answer B is incorrect. Barium swallow is not
tion here is the classic miliary pattern on x-ray indicated in the diagnosis of acute mesenteric
of the chest indicative of hematogenous ischemia.
spread.
Answer C is incorrect. A hepato-iminodiacetic
Answer A is incorrect. Latent TB would not acid (HIDA) scan is not indicated in the di-
be symptomatic, but may be accompanied by agnosis of acute mesenteric ischemia. HIDA
radiographic abnormalities. scans are a useful adjunct to ultrasound in the
FULL-LENGTH EXAMS

Answer C is incorrect. Pneumocystis jiroveci diagnosis of acute cholecystitis.


pneumonia would be more likely in the setting Answer D is incorrect. Ultrasound of the ab-
of HIV infection, hematologic malignancy, or domen is not the gold standard for diagnosing
other immunocompromised. acute mesenteric ischemia.
Answer D is incorrect. While miliary TB can Answer E is incorrect. Although x-ray lms
be a result of progressive primary infection, this of the abdomen are often ordered for patients
is less common in the era of antibiotics. The who present with acute abdominal pain, this
most common symptom of primary infection is is not the gold standard for diagnosing acute
fever, with <25% of patients having pulmonary mesenteric ischemia.
complaints. The most common radiographic
nding is hilar adenopathy, with focal inl- 9. The correct answer is F. Testicular malignancy
trates being less common. can present with testicular discomfort and
Answer E is incorrect. Most cases of symp- swelling, which may lead a clinician to diag-
tomatic TB are due to reactivation of a latent nose the patient with orchitis or epididymitis.
infection conned to the lungs. The typical The initial trial of antibiotics is reasonable, but
radiograph ndings might reveal more discrete given the persistent swelling and discomfort,
Test Block 3

inltrates or cavitary lesions involving the api- an ultrasound is warranted. Despite this pa-
cal-posterior segments of the upper lobes. tients presentation, the pathognomonic pre-
sentation for testicular cancer is a painless tes-
8. The correct answer is A. Embolization leading ticular mass.
to acute mesenteric ischemia should be con-
Test Block 3 Answers 555

Answer A is incorrect. CT scan of the pelvis Answer C is incorrect. Intussusception is most


is a reasonable way to assess if there are large commonly associated with infants and a sau-
nodes or other signs of metastatic disease. The sage-type mass felt in the left lower quadrant,
cost and exposure to radiation is not appropri- as well as having currant-jelly stool.
ate in this situation. Ultrasound is the next best
Answer D is incorrect. Left-sided colonic ad-
step.
enocarcinoma produces colonic distention by
Answer B is incorrect. Cystoscopy is a useful mechanical obstruction with an apple-core
tool for evaluation of the bladder. The evalua- lesion. The enema in this situation ruled out
tion of the testicle is not aided by cystoscopy. mechanical obstruction.
Answer C is incorrect. Intravenous pyelogram Answer E is incorrect. Squamous cell carci-
is a test that helps clinicians assess the urinary noma of the rectum would not present with
tract. The test would not add any information the symptoms from which this patient is suffer-
to the evaluation of the testicle. ing.
Answer D is incorrect. MRI is not necessary
11. The correct answer is C. This patient has
at this point. This method of imaging would
symptoms and family history suggesting von
identify a lesion and its extent, but it is costly
Hippel-Lindau (VHL) disease, an autosomal
and unnecessary.
dominant disorder characterized by hemangio-
Answer E is incorrect. The persistence of blastomas, retinal angiomas, pheochromocy-
symptoms should lead to imaging. Another at- toma, and renal cell carcinoma. Various mani-
tempt at antibiotics would probably not delay festations of this disease have affected the
treatment considerably, but imaging is more previous two generations of this patients fam-
appropriate. ily, suggesting that his own symptoms are due
to germline mutations. Therefore he should
10. The correct answer is A. This patient has a undergo testing for VHL because early diagno-
typical presentation for Ogilvies syndrome. Pa- sis would markedly improve subsequent medi-
tients typically present with nausea, vomiting, cal care and survival rates. Individuals known
constipation, diarrhea, abdominal distention, a to be carriers of the VHL gene should undergo

FULL-LENGTH EXAMS
tympanic abdomen, and positive bowel sounds, annual ophthalmic and neurologic examina-
and with hypokalemia, hypocalcemia, and hy- tions, periodic imaging studies, and surveil-
pomagnesemia. X-rays show dilation of the lance of renal function.
right large intestine. Bowel distension may
Answer A is incorrect. Osler-Weber-Rendu
make breathing labored. Before making this di-
syndrome, also known as hereditary hemor-
agnosis, mechanical obstruction must be ruled
rhagic telangiectasia, is an autosomal domi-
out, and toxic megacolon excluded on the ba-
nant disease in which vascular lesions (telangi-
sis of the clinical picture. Toxic megacolon
ectasias, arteriovenous malformations, and
presents with colonic dilation and fever, tachy-
aneurysms) are found throughout the body,
cardia, leukocytosis, and anemia. The actual
particularly in the lungs, brain, and gastrointes-
etiology of Ogilvies syndrome is unknown.
tinal tract.
Treatment includes nasogastric and rectal tube
placement, frequent turning, and neostigmine Answer B is incorrect. Tuberous sclerosis is
to promote decompression of the colon. Serial characterized by benign tumors of the brain,
abdominal x-rays help to monitor cecal size eyes, skin, and kidneys that cause symptoms of
and determine the aggressiveness of therapy. mental retardation and seizures beginning in
the rst year of life.
Answer B is incorrect. Diverticulitis would
Test Block 3

most commonly present as localized abdomi- Answer D is incorrect. Because early detection
nal pain with possible signs of peritonitis and of VHL disease and subsequent renal cell car-
fever. This patients presentation is not typical cinoma improves survival rates in patients, it is
of diverticulitis. imperative that the patient undergoes screen-
556 Section II: Full-Length Examinations Answers

ing. If he is positive, then his children should is much more likely that the patient is experi-
undergo testing as well. encing a specic phobia to ying.
Answer E is incorrect. The diagnosis of VHL Answer D is incorrect. Social phobia results
disease in an individual who has or plans to in a disabling sense of anxiety during situations
have children is potentially important because where the patient is expected to perform or will
the prognosis of patients with renal cell carci- be scrutinized by others (e.g., giving a presenta-
noma, the most lethal lesion of this disease, is tion, performing in front of a large audience, or
greatly improved with early detection. Early public speaking). It can severely impact the life
detection of retinal lesions may also allow for of the patient, from ofce presentations to talk-
more timely laser treatment to preserve vision. ing to unfamiliar people. It can be treated with
The proper sequence of genetic testing is to antidepressants and benzodiazepines; some
conrm the suspected trait in the patient him- patients respond to a presituational -blocker
self and then proceed to testing his offspring. aimed at minimizing somatic complaints.

12. The correct answer is E. According to the Di- 13. The correct answer is D. Ulcerative colitis
agnostic and Statistical Manual of Mental Dis- (UC) is an idiopathic autoinammatory disor-
order, Fourth Edition, Text Revision (DSM-IV- der of the colon that always involves the rec-
TR), a specic fear is marked by a persistent tum. In fact, 40%50% of cases are limited to
fear that is excessive or unreasonable, cued by the rectum or rectosigmoid. Lesions may also
the presence or anticipation of a specic object spread proximally in a continuous manner to
or situation. The fear may take on the quali- involve the entire colon. Findings include
ties of a panic attack. In many cases, the feared crypt abscesses with numerous polymorphonu-
situation may have personal meaning for the clear leukocytes, friable mucosal patches that
patient, as in this case. The fear is so intense bleed easily, and pseudopolyps; the diagnosis is
that the activity (fear of ying) is avoided, re- made on colonoscopy with biopsy showing
sulting in functional decline. In other cases, these ndings. Symptoms include bloody diar-
the fear is of an object or situation that can be rhea and colicky abdominal pain. UC is ini-
avoided due to limited exposure (e.g., the med- tially treated with sulfasalazine or mesalamine.
FULL-LENGTH EXAMS

ical student that is afraid of snakes). The treat- Refractory disease is managed with corticoster-
ment of specic phobia usually involves sys- oids and cyclosporine; however, total colec-
temic desensitization, in which the patient is tomy is curative for long-standing disease or
progressively exposed to the activity or object fulminant colitis. This patients surgical speci-
that is feared, or ooding, in which the pa- men shows a pan-ulcerative colitis. The mu-
tient is exposed directly to the feared object or cosa has a lumpy, bumpy appearance because
activity until the symptoms of anxiety lessen. of areas of inamed but intact mucosa are sep-
arated by ulcerated areas. UC carries a mark-
Answer A is incorrect. Although this indi-
edly increased risk of colorectal cancer in long-
vidual is experiencing chest pain, the story is
standing cases.
much more consistent with one of psychiatric
disease, given that the symptoms occur only in Answer A is incorrect. Hirschsprungs disease
the context of ying. is caused by absence of autonomic innerva-
tion of the bowel wall, leading to inadequate
Answer B is incorrect. The diagnosis of gener-
relaxation and peristalsis resulting in intestinal
alized anxiety disorder depends on duration of
obstruction. Patients present with abdominal
symptoms for >6 months and three or more so-
distention, bilious vomiting, and failure to pass
matic symptoms for the same time period. How-
meconium in the rst 24 hours of life. Diag-
ever, because this patients symptoms are in re-
Test Block 3

nosis is conrmed by rectal biopsy showing an


sponse to a single stimulus (ying), the diagnosis
absence of ganglion cells. Treatment is via co-
of generalized anxiety disorder does not apply.
lostomy.
Answer C is incorrect. Although hyperthyroid-
Answer B is incorrect. Crohns disease involves
ism can produce some of the symptoms (palpi-
transmural lesions that can involve the entire
tations, chest pain, and difculty breathing), it
Test Block 3 Answers 557

bowel wall thickness. As a result, these patients Answer D is incorrect. Pseudomonas aerugi-
often develop enterovesical and enterocutane- nosa is a gram-negative, aerobic, rod-shaped
ous stulas. Physical examination may reveal bacterium which can produce a greenish color
perianal ssures or stulas. and grape-like odor in culture. It is a rare cause
of community-acquired pneumonia, but rather
Answer C is incorrect. Crohns disease is an
is more associated with pneumonia of immu-
inammatory disease of the gastrointestinal
nocompromised hosts such as those on ventila-
tract that may be infectious in nature. It can af-
tors and those with cystic brosis.
fect any part of the digestive tract, but typically
involves the ileocecal region. Symptoms in- Answer E is incorrect. Streptococcus pneumo-
clude abdominal pain, watery diarrhea, fever, niae is a spherical, gram-positive, encapsulated,
and weight loss. Diagnosis is based on biopsy -hemolytic bacterium responsible for causing
of the affected area showing transmural inam- a wide variety of infections including otitis me-
mation, noncaseating granulomas, cobblestone dia, cellulitis, meningitis, and pneumonia. It
mucosal morphology, and skip lesions. Creep- is the most common cause of pneumonia in
ing fat on gross dissection is pathognomonic. adults, is lobar in nature, and often is associ-
Crohns disease is managed with sulfasalazine, ated with high fevers.
and then corticosteroids and immunosuppres-
sion if refractory. The newest treatment is the 15. The correct answer is A. It is estimated that
anti-tumor necrosis factor antibody, iniximab. approximately 90% of normal couples should
Surgical resection may be necessary for perfo- be able to conceive after 12 months of attempt-
ration. ing to achieve pregnancy. However, approxi-
mately 10%15% of couples in the reproduc-
Answer E is incorrect. The inammation seen
tive age group are affected by infertility. There
in Crohns disease is transmural on biopsy.
are ve main causes of infertility: ovulatory
dysfunction, uterine and tubal disorders, male
14. The correct answer is A. Klebsiella pneumo-
factor, cervical factor, and peritoneal factor.
niae is a gram-negative, nonmotile, encapsu-
The patient reports a history of irregular men-
lated, facultatively anaerobic, rod-shaped bac-
ses in the setting of perimenopausal symptoms,
terium which is part of the normal ora of the

FULL-LENGTH EXAMS
including irregular cycles due to anovulatory
mouth, skin, and intestines. However, it can
cycles, vasomotor symptoms such as hot
cause urinary tract infections, wound infec-
ashes, and vaginal dryness caused by de-
tions, and pneumonia in certain populations.
creased estrogen levels. Decreased estrogen
Pneumonia from this organism is typically seen
levels cause a dramatic rise in follicle-stimulat-
in alcoholics due to aspiration of vomit. Like
ing hormone and luteinizing hormone levels,
other aspiration pneumonias, the middle and
which are helpful in diagnosing ovarian fail-
upper lobe are often the primary areas in-
ure.
volved, as they are the lowest points when lying
down. Klebsiella pneumonia is characterized Answer B is incorrect. A hysterosalpingogram
by a cough productive of currant jelly sputum. is a radiologic study in which dye is placed
into the uterine cavity via a transcervical
Answer B is incorrect. Legionella pneumo-
catheter and is the initial test for intrauterine
philia is a gram-negative, intracellular bacteria
shape and tubal patency. It is best performed
which can cause a pneumonia via transmission
between days 6 and 10 of the menstrual cycle.
from aerosolized droplets. Common sources
Any uterine abnormality should be conrmed
include hot water systems, ventilation ducts, or
with a hysteroscopy, while tubal abnormalities
other areas with standing water.
are conrmed with laparoscopy, the gold stan-
Test Block 3

Answer C is incorrect. Pneumocystis jiroveci dard for diagnosing tubal and/or peritoneal dis-
pneumonia is an opportunistic infection seen ease. Although uterine or tubal abnormalities
in immunosuppressed patients and those with can account for some barriers to conception,
AIDS. X-ray of the chest can show diffuse bi- the patients history is negative for congenital
lateral interstitial inltrates with a ground-glass uterine or tubal disorders or sexually transmit-
appearance.
558 Section II: Full-Length Examinations Answers

ted diseases, which makes a structural cause hip, the course of these vessels can be altered
unlikely and would not explain her associated and blood ow hindered.
symptoms.
Answer B is incorrect. The femoral artery,
Answer C is incorrect. A postcoital test evalu- vein, and nerve can be injured in an anterior
ates the cervical factor of infertility by assessing dislocation. However, this patient suffered a
cervical mucus for motile sperm after inter- posterior dislocation based on mechanism and
course. However, the patient related no history clinical appearance.
of a cone biopsy or cryotherapy to the cervix
Answer C is incorrect. Myositis ossicans can
that may account for a negative postcoital test.
occur following hip dislocation, but it is related
Answer D is incorrect. A progesterone chal- to muscle damage and hematoma formation
lenge consists of administering this hormone rather than the timing of treatment.
to the patient for 710 days to see if it triggers
Answer D is incorrect. Posttraumatic degen-
menstruation. If a period does occur as a result
erative arthritis commonly develops after poste-
of taking progestin, it means that the uterus
rior dislocations of the hip. This complication
and vagina were primed properly under the
seems to be related to the severity of the initial
presence of estrogen, which causes the uterine
trauma rather than delay of treatment.
lining to thicken. This conrms that estrogen
is present and that the ovaries are capable of Answer E is incorrect. Although recurrent dis-
ovulating but lack proper cycling. However, if location is a rare complication to traumatic hip
no menstruation takes place after the progestin dislocation, its incidence is not associated with
challenge and the anatomy of the reproductive time to relocation as is avascular necrosis.
tract is found to be normal, then a lack of es-
trogen is the most likely cause of anovulation. 17. The correct answer is D. Myelobrosis refers
Further hormone testing can then be used to to the conversion of bone marrow to a solid
make a diagnosis. This patients presentation is collection of brous tissue. It is a late manifes-
most consistent with premature ovarian failure, tation of chronic myelogenous leukemia that is
which can be conrmed with serum follicle- of unknown etiology and uniformly fatal. As
stimulating hormone and luteinizing hormone the bone marrow ceases to produce RBCs, ex-
FULL-LENGTH EXAMS

levels. tramedullary hematopoiesis ensues, beginning


in the spleen and leading to massive spleno-
Answer E is incorrect. A space-occupying pro- megaly that may progress to splenic infarction.
lactinoma may cause secondary amenorrhea, Megakaryocytes and platelets persist in the
which is dened as the absence of menstrual marrow, and large platelets are present periph-
periods for 6 months in a woman who had erally. Diagnosis generally requires bone mar-
previously been regular, or for 12 months in a row biopsy because aspiration of the solid -
woman who had irregular periods. Anatomic brous tissue is not possible.
compression or high prolactin levels can in-
hibit proper pituitary function, resulting in Answer A is incorrect. Bernard-Soulier syn-
hypogonadism and amenorrhea. Obtaining a drome is an autosomal dominant disorder in
serum prolactin level can help make the diag- which platelets lack the membrane glycopro-
nosis. However, in the absence of galactorrhea, tein necessary for binding vWF to the plate-
this diagnosis is less likely. let surface. It is inherited and is not related to
chronic myelogenous leukemia. Patients af-
16. The correct answer is A. Avascular necrosis is fected by this disorder may experience sponta-
a complication of hip dislocations and occurs neous bruising, epistaxis, and petechiae. This
in approximately 15% of cases. The incidence patients presentation is not consistent with this
Test Block 3

of avascular necrosis may be diminished signif- diagnosis.


icantly by prompt reduction of a hip disloca- Answer B is incorrect. Sideroblastic anemia
tion. The arterial supply to the femoral head is caused by defective iron incorporation into
runs along the femoral neck. In a dislocated the heme molecule, leading to deposition of
Test Block 3 Answers 559

excess iron in the bone marrow and liver. Idio- tinguishing systolic and diastolic failure, it can
pathic sideroblastic anemia occurs most often conrm elevated left atrial pressure, but would
in persons who are elderly and is classied as not be indicated unless myocardial ischemia
a myelodysplastic syndrome. It is characterized was suspected.
by macrocytic anemia and varying degrees of
Answer B is incorrect. Many of the signs and
leukopenia and thrombocytopenia. Bone mar-
symptoms of heart failure including dyspnea,
row aspiration demonstrates hypercellular mar-
reduced exercise tolerance, edema, and the
row. Again, this is not consistent with the bone
neurohormonal response are similar in both
marrow of this patient.
forms of heart failure. Therefore it is often dif-
Answer C is incorrect. The myelodysplas- cult to distinguish the two clinically.
tic syndromes are premalignant conditions
Answer C is incorrect. CT scan of the chest is
in which a single clonal immature blood cell
expensive and would add little information.
proliferates in a disordered manner, causing
cytopenias, increased bone marrow cellularity, Answer E is incorrect. While ECG can sug-
and malfunctioning peripheral cells. Although gest one diagnosis over the other, it is generally
these conditions often progress to leukemias if unreliable.
not treated, they are not known to be caused Answer F is incorrect. MRI of the heart is very
by chronic myelogenous leukemia. Addition- expensive and would not be very helpful.
ally, the bone marrow biopsy from this patient
reveals decreased cellularity. In myelodysplas- Answer G is incorrect. X-ray of the chest in
tic syndromes, an increase would be expected. both cases may show pulmonary congestion.
Systolic failure might be more associated with
Answer E is incorrect. Progression to blast cardiomegaly, but this is not very reliable.
phase is one rapidly fatal outcome of indolent
chronic myelogenous leukemia. It develops 19. The correct answer is C. This child has spastic
following asymptomatic disease, and results in cerebral palsy (CP) affecting his lower limbs.
splenomegaly, anemia, thrombocytopenia, and CP is a general term that describes a group of
replacement of peripheral lymphocytes with disorders that appear in the rst few years of
nucleated blast cells. Bone marrow may be dif-

FULL-LENGTH EXAMS
life and affect a childs ability to coordinate
cult to aspirate, but biopsy reveals hypercellu- body movements. These disorders are caused
larity. Although this outcome is possible for the by damage to a childs brain during fetal devel-
patient in this case, the bone marrow biopsy is opment, birth, or the rst few months after
not consistent with this diagnosis. birth. CP is the most common movement dis-
Answer F is incorrect. Exposure to alkylating order in children. Pyramidal CP, the most
agents has been implicated in the pathogenesis common type of CP, is characterized by spas-
of myelogenous leukemias. Imatinib, however, ticity, hyperreexia, slow and effortful volun-
is an Abl inhibitor and is not an alkylating tary movements, and impaired ne motor
agent. function, and can affect all of the limbs. De-
layed developmental milestones, persistence of
18. The correct answer is D. Systolic dysfunction infantile reexes (e.g., Babinskis reex), con-
and diastolic function differ in regard to their tractures, and weakness or underdevelopment
pathophysiology and management. However, of affected limbs are common. Affected chil-
they can appear similar clinically. Echocardio- dren classically walk on their toes and have a
gram is the best (inexpensive, noninvasive, and scissor gait. Spastic CP is treated with muscle
portable) method of evaluation. It allows mea- relaxants (e.g., diazepam, dantrolene, ba-
surement of ejection fraction, cavity size, and clofen), physical and occupational therapy, as-
Test Block 3

wall thickness, as well as a view of any valvular sist devices, and surgical release of contractures
abnormalities. when necessary.
Answer A is incorrect. Cardiac catheterization Answer A is incorrect. This is the appropriate
is invasive and carries with it cost and morbid- therapy for infantile spasms (Wests syndrome),
ity (including contrast load). In terms of dis- which is characterized by clusters of general-
560 Section II: Full-Length Examinations Answers

ized tonic-clonic seizures, very high amplitude Kidney function, however, is not usually com-
slow waves on electroencephalogram called promised in heroin abusers.
hypsarrhythmia, and arrest of psychomotor de-
Answer D is incorrect. Although patients in
velopment at the age when the seizures began.
heroin withdrawal have symptoms that are very
Seizures usually begin between 312 months
intense and unpleasant, those symptoms are
of age. This child is not having seizures.
not life-threatening. Withdrawal symptoms in-
Answer B is incorrect. Benztropine is an an- clude dysphoria, nausea, rhinorrhea, lacrima-
ticholinergic that treats dystonia, which occurs tion, piloerection, muscle spasms (commonly
in Parkinsons disease and as an adverse effect in the legs; hence the phrase kicking the
associated with typical antipsychotics. This habit), sweating, yawning, diarrhea, fever, and
child does not have Parkinsons disease and is insomnia.
not taking any antipsychotic medication.
Answer E is incorrect. To minimize with-
Answer D is incorrect. Levodopa and carbi- drawal symptoms, patients can be treated with
dopa treats Parkinsons disease, which is char- clonidine or methadone. Clonidine works to
acterized by resting tremor, bradykinesia, rigid- suppress the rebound hyperactivity of the sym-
ity, postural instability, masked facies, memory pathetic nervous system, and methadone acts
loss, and micrographia. Parkinsonism affects as a cross-tolerant drug, since it is also an opi-
individuals who are in middle age and who are oid but has a much longer half-life than her-
elderly, not children. oin. Benzodiazepines are not used in the treat-
ment of heroin addiction.
Answer E is incorrect. Pimozide is used to
treat tics. Tics are involuntary rapid move-
21. The correct answer is A. Cryptococcal menin-
ments, gestures, or utterances, of which this
gitis is an opportunistic infection seen in AIDS
child has none. Tics are most commonly seen
and other immunocompromised patients. It
in children with Tourettes syndrome.
can present as a headache, fever, and/or im-
paired mentation. Diagnosis is by lumbar
20. The correct answer is C. Patients who over-
puncture, which will show decreased glucose,
dose on heroin can present with respiratory
increased protein, and increased leukocyte
FULL-LENGTH EXAMS

compromise and may stop breathing alto-


count with monocytic predominance. Cere-
gether, as all opiates strongly depress respira-
brospinal uid (CSF) cryptococcal antigen test
tory drive. Therefore, any patient who presents
is 95% sensitive in patients with the disease and
to the emergency department with the hall-
more sensitive than microscopy and culture.
marks of an opioid overdose (such as pinpoint
pupils, needle-track marks, and respiratory Answer B is incorrect. While lumbar punc-
compromise) should be monitored and an ture and culture may yield a diagnosis, it often
emergent airway established if necessary. takes several days to several weeks for results
and larger volumes of CSF are often required.
Answer A is incorrect. Naloxone works as an
CSF cryptococcal antigen test is much more
opioid antagonist, binding strongly to the opi-
sensitive and can be performed in a relatively
oid receptors and reversing the effects of an
short amount of time on a smaller sample size.
opiate overdose. The reversal works immedi-
ately, and most patients recover consciousness Answer C is incorrect. India ink stain on the
with intravenous (IV) administration. Nalox- CSF can be used in diagnosis of cryptococcal
one works so well that soon after its adminis- meningitis, but is only 50% sensitive in pa-
tration, patients may start to experience with- tients with disease. The question asks for the
drawal symptoms. Administration of 0.4 mg, most likely way to conrm the diagnosis of
Test Block 3

however, is an appropriate initial dose. Cryptococcus, and therefore the more sensitive
and specic test, lumbar puncture and serol-
Answer B is incorrect. Once IV heroin users
ogy, would be the better answer.
are stable, it is important to assess them for
HIV, skin abscesses, and bacterial endocarditis. Answer D is incorrect. Potassium hydroxide
preparation is used to assess for the presence of
Test Block 3 Answers 561

Candida albicans, the organism responsible for coloration and enamel hypoplasia in the babys
thrush in immunosuppressed patients. Potas- developing teeth.
sium hydroxide or Gram stain will show bud-
Answer E is incorrect. Although warm com-
ding yeast and/or pseudohyphae if Candida is
presses are an appropriate adjuvant to antibi-
present.
otic therapy, they are not the appropriate initial
Answer E is incorrect. Silver stain is not used treatment for mastitis.
in the diagnosis of cryptococcal meningitis. It
can be used for diagnosis of other opportunis- 23. The correct answer is E. Anterior dislocations
tic infections including Histoplasma and Blas- of the shoulder can be associated with axillary
tomyces. artery and nerve damage. Before closed reduc-
tion and immobilization are performed, it is
22. The correct answer is C. Mastitis is a cellulitis important to assess for neurovascular injury.
of the periglandular breast tissue that fre- The axillary nerve ends as the superior lateral
quently occurs within the rst 3 months post- cutaneous nerve supplying the area over the
partum. Mastitis results from breast-feeding- deltoid muscle.
related nipple trauma and is most commonly
Answer A is incorrect. This area is supplied by
caused by Staphylococcus aureus. Initial treat-
the radial nerve, which is not typically injured
ment consists of an oral antibiotic effective
in an anterior shoulder dislocation. Radial
against penicillin-resistant staphylococci, such
nerve injury can be associated with a humeral
as dicloxacillin or a cephalosporin, although
shaft fracture and presents as a wrist drop.
antibiotic coverage against oral ora from the
suckling baby should be considered in a pa- Answer B is incorrect. This area is supplied by
tient not responding to antistaphylococcal ther- the ulnar nerve, which is not typically injured
apy. in an anterior shoulder dislocation. Ulnar
nerve injury can be associated with a fracture
Answer A is incorrect. A breast abscess can
of the medial condyle of the humerus, and
present with similar features as in mastitis,
these patients can present with weakness of
such as breast pain, systemic symptoms (e.g.,
the intrinsic muscles of the hand. They exhibit
fever, vomiting), and a palpable uctuant

FULL-LENGTH EXAMS
decreased ability to hold a card between the
mass. Spontaneous drainage from the mass or
thumb and index nger, and may show claw-
the nipple can also occur. Because there are
ing (exion) of the fourth and fth digits.
no signs of pus, incision and drainage are not
indicated. Answer C is incorrect. This area is supplied
by the median nerve, and this area is also of-
Answer B is incorrect. Although IV oxacillin
ten used to test the C7 dermatome. Neither is
would provide adequate antistaphylococcal
typically injured in an anterior shoulder dislo-
coverage, an IV antibiotic is generally not nec-
cation.
essary for the treatment of mastitis. This patient
is exhibiting the typical signs of a local celluli- Answer D is incorrect. This area is supplied
tis that can be treated with oral antibiotics. IV by the median nerve, which is not typically in-
therapy could be considered if the infection jured in an anterior shoulder dislocation. Me-
had invaded more subcutaneous tissue or if dian nerve injury can be associated with a su-
the infection did not respond to oral antibiotics pracondylar humerus fracture and presents as
after several days of therapy. decreased thumb function.
Answer D is incorrect. Doxycycline is effective 24. The correct answer is C. This is an example of
predominantly against intracellular organisms a prospective cohort study because patients
Test Block 3

and does not have adequate antistaphylococcal were assessed for presence or absence of risk
coverage. Additionally, doxycycline is an abso- factors and followed over time for development
lute contraindication in actively breast-feeding of disease.
mothers because tetracycline causes tooth dis-
562 Section II: Full-Length Examinations Answers

Answer A is incorrect. Case-control studies as- controversial because of the overlap of PSA
semble case groups with a disease and matched levels seen in BPH and prostate cancer.
control groups without disease, and then col-
Answer D is incorrect. This step would be ap-
lect retrospective data on exposure to risk fac-
propriate if prostate cancer were suspected. Al-
tors.
though irritative urinary symptoms can also be
Answer B is incorrect. Meta-analysis involves seen with prostate cancer, in such cases urinal-
a statistical combination of data from several ysis usually reveals hematuria. Furthermore,
studies, often via a literature search. DRE in this patient revealed no abnormalities,
decreasing the suspicion for prostate cancer
Answer D is incorrect. Randomized controlled
and thus not warranting invasive diagnostic
trials are experimental, prospective studies that
testing.
are designed, for example, to test therapeutic
interventions. Answer E is incorrect. These tests are appro-
priate to evaluate for metastasis if prostate can-
Answer E is incorrect. Although the study is
cer is strongly suspected.
a cohort study, it is not retrospective. A retro-
spective cohort study will assemble a cohort
26. The correct answer is A. This patient has neo-
based on history of risk factors, and examine
plastic pericarditis with pericardial effusion,
outcomes and past exposures at the time of
which is a uid collection in the pericardial
data collection.
space, as seen in the image. Pericarditis is an
inammation of the pericardial space and is a
25. The correct answer is B. This patient most
common cause of pericardial effusion. Because
likely has benign prostatic hyperplasia (BPH),
this patients symptoms are bothersome, treat-
which occurs normally with aging and is seen
ment is indicated. Balloon pericardiotomy, pro-
in >50% of men 5160 years old. Urinary ob-
longed pericardial catheter drainage, surgical
structive symptoms (hesitancy, weak stream,
pericardiectomy, and intrapericardial scleros-
intermittent stream, incomplete emptying, uri-
ing therapy are all appropriate methods of
nary retention, bladder fullness) and irritative
treatment. Balloon pericardiotomy is per-
symptoms (nocturia, daytime frequency, urge
formed by inserting an uninated balloon into
FULL-LENGTH EXAMS

incontinence, and opening hematuria) can be


the pericardial space, inating the balloon, and
seen in patients aficted with BPH. Digital
pulling it through the pericardium to create a
rectal exam (DRE) often reveals a smooth,
hole through which uid can drain into the
rm, rubbery prostate. Induration or masses
pleural or peritoneal space.
should raise suspicion of cancer. Urinalysis
should always be performed to exclude infec- Answer B is incorrect. Emergent pericardio-
tion, as should a serum creatinine measure- centesis is indicated in unstable patients with
ment to assess for renal insufciency (caused cardiac tamponade. Although pericardiocente-
by obstructive uropathy), which occurs in 10% sis would relieve this patients chest pain, the
of patients with prostatism. Some physicians relief would only be temporary because more
may also chose to measure the serum prostate- uid would soon collect in the pericardial
specic antigen (PSA) level to establish a base- space. Therefore, pericardiocentesis would not
line level and monitor for prostate cancer, es- be a good long-term solution to her pain.
pecially given the patients risk factor of
Answer C is incorrect. Nonsteroidal anti-
ethnicity.
inammatory drugs (NSAIDs) can be used
Answer A is incorrect. Cystoscopy is not rec- in the treatment of pericarditis; however, an
ommended for longitudinal BPH monitoring. NSAID will only temporarily relieve this pa-
Test Block 3

tients symptoms and thus is not a good long-


Answer C is incorrect. Serum PSA measure-
term solution.
ment is considered optional, although most
physicians include it in the initial evaluation of Answer D is incorrect. A morphine patient-
patients with suspected BPH. Its use remains controlled analgesia would help control this
patients pain, but it would not treat the under-
Test Block 3 Answers 563

lying problem of pericarditis. Thus, morphine hallucinogenic drug. Unlike LSD, it can be
is not a good long-term solution. abused, and users can become psychologically
dependent on it.
Answer E is incorrect. Nitroglycerin is used
to treat chest pain in patients having angina
28. The correct answer is B. This patient has pre-
or acute myocardial infarction. It would have
sented with a rare form of Hodgkins disease,
little effect on pain secondary to pericarditis, as
the lymphocyte-depleted type. Although mod-
seen in this patient.
ern therapies have made the classic distinctions
of Hodgkins disease less important in terms of
27. The correct answer is D. This patients presen-
prognosis, lymphocyte-depleted disease re-
tation is likely due to lysergic acid diethylam-
mains quite aggressive clinically. It accounts
ide (LSD), which typically causes powerful
for only 2% of Hodgkins disease pathologies
sensory disturbances such as the movement of
and is seen most often in older men. Patients
shapes, light, and colors. LSD usually does not
often present with advanced disease and signi-
produce hallucinations in the strict sense, but
cant systemic symptoms. Biopsy reveals abun-
rather illusions and vivid daydream-like scenes.
dant Reed-Sternberg cells with a paucity of
The drugs action on numerous receptors, in-
lymphocytes compared to other forms of the
cluding dopamine receptors, adrenoreceptors,
disease.
and serotonin receptors, may also contribute to
the various physiologic reactions that users may Answer A is incorrect. Burkitts lymphoma is
experience, such as pupillary dilation, tachy- a non-Hodgkins lymphoma characterized on
cardia, salivation, uterine contraction, and in- biopsy by a starry sky appearance due to the in-
creased body temperature. Patients rarely get gestion of tumor cells by macrophages.
addicted or dependent on the drug because its
Answer C is incorrect. Lymphocyte-predom-
rapid tolerance prevents regular use.
inant Hodgkins lymphoma is an uncommon
Answer A is incorrect. Cocaine is not the form of the disease, accounting for 6% of cases.
most likely cause of this patients sensory dis- It carries an excellent prognosis. On biopsy,
turbances, although it is important to note that lymph nodes reveal diffuse lymphocytic inl-
cocaine can cause hallucinations and psycho- trate with few Reed-Sternberg cells. This form

FULL-LENGTH EXAMS
sis when taken in large amounts. Cocaine can of the disease would not be expected to be as
be snorted, sniffed, injected, or smoked, but it aggressive as this patients illness.
is not usually taken orally on blotted paper.
Answer D is incorrect. Nodular sclerosing
Answer B is incorrect. Ecstasy is the street Hodgkins disease is the most common form
name of 3,4-methylenedioxymethamphet- (70% of cases) and is most commonly diag-
amine (MDMA), an illicit drug that is used nosed in young women. Histology reveals -
recreationally. Users of ecstasy experience so- brosis with Reed-Sternberg and lymphoid cells.
cial disinhibition, euphoria, and heightened This form of the disease is less aggressive than
self-awareness; however, hallucinations are un- this patients and is uncommon in men who
common. are elderly.
Answer C is incorrect. Heroin is unlikely to Answer E is incorrect. The Reed-Sternberg
have caused this patients condition, as it is cell is pathognomonic for Hodgkins disease,
usually snorted, smoked, or injected. This drug effectively ruling out non-Hodgkins lym-
causes an intense euphoric feeling but gener- phoma.
ally does not cause sensory disturbances. It is
Answer F is incorrect. The presence of the
highly addictive, and withdrawal symptoms
Reed-Sternberg cell excludes a diagnosis of
Test Block 3

may develop within a day of the last heroin in-


leukemia. In addition, leukemia would not be
take.
expected to cause such severe lymphadenopathy.
Answer E is incorrect. Marijuana is unlikely
to be responsible for this patients presentation, 29. The correct answer is E. This is a classic pre-
as it is usually smoked and is not known as a sentation of osteosarcoma. In general, osteosar-
564 Section II: Full-Length Examinations Answers

coma will present with progressive pain that is ylococcus aureus is necessary. The excellent an-
worse at night, constitutional symptoms, and tipseudomonal activity of the uoroquinolones
swelling/erythema over the site of the tumor. Al- has generally made them the treatment of
kaline phosphatase is elevated in more than half choice for necrotizing otitis externa, although
of all patients with osteosarcoma. The x-ray a combination of a -lactam antibiotic and
demonstrates the sunburst appearance of neo- aminoglycoside is also effective. Treatment
plastic bone formation. Of all osteosarcomas, should also include surgical dbridement of
60% are seen in children and adolescents, with any granulation or osteitic bone.
males being affected 1.52 times more often
Answer B is incorrect. Amoxicillin will not
than females. This tumor has a tendency to af-
cover Pseudomonas aeruginosa, one of the
fect the metaphyses of long bones, particularly
likely infecting organisms.
the distal femur, proximal tibia, and proximal
humerus. Treatment includes chemotherapy, Answer C is incorrect. Amphotericin B covers
and if possible, limb-sparing surgical resection, fungi and would be ineffective in this case.
though amputation may be necessary in some Answer D is incorrect. Methicillin will not
cases. Long-term survival is 60%80%, with the cover Pseudomonas aeruginosa, one of the
best survival occurring in those patients who are likely infecting organisms.
responsive to chemotherapy.
Answer E is incorrect. Topical treatment will
Answer A is incorrect. Fibrous dysplasia also be insufcient to deal with this degree of infec-
has many different appearances on x-ray lms tion.
and can mimic many other tumors. However,
it is a benign lesion that would not present as 31. The correct answer is A. The biopsy results
aggressively as the lesion in this case. should point to amyloidosis causing a restric-
Answer B is incorrect. Nonossifying broma tive cardiomyopathy. The hallmark of restric-
is a benign lesion and so should not result in tive cardiomyopathies is diastolic dysfunction.
weight loss and fever. Also, it would appear ra- This can be seen in situations in which systolic
diolucent on x-ray lms and not radiodense as function is within normal limits and the main
in this case. etiology is the inability of the ventricles to ll
FULL-LENGTH EXAMS

properly. An ejection fraction of 55% would be


Answer C is incorrect. The classic presenta- consistent with diastolic dysfunction.
tion of osteoid osteoma is night pain that re-
sponds to NSAIDs, which was not the case in Answer B is incorrect. A harsh diastolic mur-
this patient. mur would not be consistent with cardiac
amyloidosis. Diastolic murmurs are classied
Answer D is incorrect. Infection can take on as early, mid-, or late and can be caused by a
many different appearances radiographically variety of pathologies. This includes aortic or
and must always be considered with tumor. pulmonary regurgitation, mitral or tricuspid
However, the normal complete blood cell stenosis, myxoma, heart block, and left-to-right
count and erythrocyte sedimentation rate help shunts. None of these conditions would be
to rule out this diagnostic possibility. denitively associated with a diagnosis of amy-
loid.
30. The correct answer is A. Otitis externa is gen-
erally treated with topical medication; how- Answer C is incorrect. Patients with diastolic
ever, the rarer form of it, malignant otitis ex- dysfunction due to restrictive cardiomyopathies
terna or necrotizing otitis externa, is essentially will often have an enlarged, painful, and pul-
an osteomyelitis of the temporal bone. Risk fac- satile liver. There is no relationship between a
Test Block 3

tors include diabetes and immunocompro- right lower quadrant mass and amyloidosis.
mised status. The mortality rate can be as high Answer D is incorrect. U waves are often seen
as 53%. Imaging studies include CT, MRI, and with hypokalemia.
a combination technetium and gallium scan
can also be used. Aggressive parenteral antibi- Answer E is incorrect. Edema is another sign
otic therapy aimed at Pseudomonas and Staph- of persistently elevated venous pressure and
Test Block 3 Answers 565

would be consistent with cardiac amyloidosis. diagnosis of an esophageal stricture, although


However, this edema is generally found in the all strictures should ultimately be examined
lower extremities. endoscopically and biopsied to rule out Bar-
retts esophagus and/or esophageal cancer.
Answer F is incorrect. Crackles upon pul-
Treatment of most cases of stricture involves
monary auscultation can be indicative of pul-
maximizing antacid therapy and endoscopi-
monary edema due to left-sided heart failure.
cally dilating the stricture using bougies.
However, wheezing is not consistent with the
biopsy results. Answer A is incorrect. Achalasia is an esopha-
geal motility disorder that often presents with
32. The correct answer is C. This patient has dysphagia, retention of food in the esophagus,
symptoms of subacute closed-angle glaucoma. and regurgitation, all of which are secondary
While medical treatment can provide symp- to an absence of nerve ganglion cells in the
tomatic relief, only laser iridotomy provides cu- esophagus wall. The patients history of GERD
rative treatment. Laser iridotomy forms a per- without symptoms of regurgitation makes a
manent connection between the anterior and stricture more likely.
posterior chambers and prevents recurrence.
Answer B is incorrect. Barretts esophagus
In some cases, the fellow eye should undergo
involves columnar metaplasia of the distal
prophylactic laser iridotomy.
esophagus and can result from long-standing
Answer A is incorrect. Acetazolamide is car- GERD. While Barretts esophagus itself does
bonic anhydrase inhibitor that can decrease not usually cause dysphagia, patients with Bar-
aqueous humor secretion and reduce intraocu- retts esophagus do have a higher incidence of
lar pressure. However, it only provides symp- benign esophageal stricture that can result in
tomatic treatment and is not curative. symptoms of dysphagia.
Answer B is incorrect. Atropine is an antimus- Answer D is incorrect. Diffuse esophageal
carinic agent that results in mydriasis, which spasm is an esophageal motility disorder that
will further exacerbate her symptoms of closed- often presents with intermittent dysphagia and
angle glaucoma. substernal chest pain. The patients history of

FULL-LENGTH EXAMS
GERD and symptoms of constant dysphagia
Answer D is incorrect. Pilocarpine is a mus-
make a stricture more likely.
carinic agonist that produces rapid miosis and
contraction of the ciliary muscles. This can Answer E is incorrect. Esophageal cancer of-
be used in the acute treatment of closed- or ten causes progressive dysphagia and should al-
open-angle glaucoma to open the trabecular ways be considered in the context of dysphagia.
meshwork around Schlemms canal, increas- However, this patient has no history of Barretts
ing drainage of aqueous humor and decreasing esophagus (which progresses to cancer in 1%
intraocular pressure. However, it only provides of cases), nor does he have signicant risk fac-
symptomatic treatment and is not curative. tors for esophageal cancer (i.e., tobacco or
alcohol use). The patients history of GERD
Answer E is incorrect. Timolol is a nonselec-
and symptoms of relatively constant dysphagia
tive -antagonist that reduces the production
make a stricture more likely.
of aqueous humor in the eye. It can be used
topically in treatment of chronic open-angle
34. The correct answer is D. This woman gives a
glaucoma, but does not provide curative treat-
classic history of stress incontinence, that is,
ment for chronic closed-angle glaucoma.
urinary leak with increases in abdominal pres-
sure (e.g., cough, sneeze, physical exertion).
33. The correct answer is C. Benign esophageal
Test Block 3

Her stress incontinence is likely due to pelvic


stricture is associated with inammatory edema
oor laxity secondary to pregnancy and child-
in the lower esophagus secondary to long-
birth. Treatment is surgical correction.
standing erosive reux and can result in dys-
phagia. A barium swallow study can aid in the
566 Section II: Full-Length Examinations Answers

Answer A is incorrect. Anticholinergic medi- output, and sunken fontanelles are signs of in-
cation such as oxybutynin can be used to treat adequate nutrition.
urge incontinence.
Answer B is incorrect. Signs of fetal alcohol
Answer B is incorrect. Intermittent catheter- syndrome are facial abnormalities (macrog-
ization can be used to treat overow inconti- nathia, thin upper lip, short palpebral ssure,
nence. In patients with overow incontinence, and epicanthal fold), poor growth (small head
there is continuous leaking of small volumes circumference, short stature, and low weight),
of urine due to incomplete emptying of the cardiac defects, minor joint and limb abnor-
bladder secondary to obstruction or failure of malities, and developmental delay/mental re-
the detrusor muscle to contract. Diabetic neu- tardation.
ropathy can also lead to overow incontinence;
Answer C is incorrect. Fetal cocaine exposure
however, this womans symptoms are most con-
can cause intrauterine growth retardation, mi-
sistent with stress incontinence.
crocephaly, and bowel atresia. It is not associ-
Answer C is incorrect. Sacral nerve stimula- ated with transient congenital hypothyroidism.
tion can be used to treat urge incontinence,
Answer E is incorrect. Fetal complications of
of which this patient does not report symp-
gestational diabetes mellitus include macroso-
toms. Urge incontinence manifests by a sud-
mia; intrauterine growth retardation; cardiac,
den strong desire to urinate that is difcult to
renal, and neural tube defects; hypocalcemia;
suppress and may result in involuntary passing
hyperbilirubinemia; polycythemia; hypogly-
of urine if the patient does not make it to the
cemia; and neonatal respiratory distress syn-
toilet in time. It is due to detrusor instability
drome. Gestational diabetes is not associated
and can be secondary to upper motor neuron
with transient congenital hypothyroidism.
lesions, infection, neoplasms, or stones.
Answer E is incorrect. Vaginal estrogen cream 36. The correct answer is E. This patient is mani-
can be used to treat incontinence secondary to festing classic signs of hyperkalemia, including
hypoestrogenism of the vaginal wall, as seen in weakness and typical ECG ndings such as
older postmenopausal women. peaked T waves and widened wave complexes.
FULL-LENGTH EXAMS

There are a few antihypertensive medications


35. The correct answer is D. The infant has tran- that are commonly associated with hyper-
sient congenital hypothyroidism, which causes kalemia. These include angiotensin-converting
lethargy, slow movement, hoarse cry, feeding enzyme inhibitors, angiotensin receptor block-
problems, constipation, macroglossia, umbili- ers, and potassium-sparing diuretics. Losartan
cal hernia, large fontanelles, hypotonia, dry is an angiotensin receptor blocker. The other
skin, hypothermia, and prolonged jaundice. antihypertensives listed are either associated
Congenital hypothyroidism is usually idio- with hypokalemia or have no effect on potas-
pathic. However, some cases may result from sium balance at all.
fetal exposure to antithyroid medications (i.e.,
Answer A is incorrect. Carbonic anhydrase in-
propylthiouracil or methimazole), which can
hibitors such as acetazolamide can have a sim-
suppress the production of fetal thyroid hor-
ilar but less pronounced effect on potassium
mone. These medications are cleared from the
balance as thiazide diuretics. They can lead to
circulation within days, and by several weeks
increased distal sodium delivery and high al-
after delivery, most infants will become euthy-
dosterone levels.
roid.
Answer B is incorrect. Loop diuretics such as
Answer A is incorrect. Formula is designed to
furosemide and bumetanide commonly lead
Test Block 3

provide adequate nutrition. However, infants


to hypokalemia through a similar mechanism
can be malnourished if the formula is mixed
as the thiazide diuretics.
incorrectly, if an insufcient quantity is given,
or under the rare circumstance that the infant Answer C is incorrect. Hydralazine, which up-
is intolerant to a component of the formula. regulates cGMP leading to smooth muscle cell
Failure to thrive, hard stools, decreased urine relaxation, commonly leads to reex tachycar-
Test Block 3 Answers 567

dia or drug-induced reaction. There is no asso- Answer C is incorrect. For those with diabetes
ciation with potassium abnormalities. mellitus, an HDL cholesterol >45 mg/dL is
desirable. This patients HDL cholesterol level
Answer D is incorrect. Thiazide diuretics,
is >45 mg/dL and therefore needs no interven-
such as hydrochlorothiazide, can lead to hy-
tion.
pokalemia by increasing distal tubular delivery
in the setting of high aldosterone. Answer D is incorrect. A high-ber low-fat diet
is ideal, but this patient needs more immediate
37. The correct answer is C. Infection is the most attention for his high LDL cholesterol level.
common cause of fevers of unknown origin,
Answer E is incorrect. Obesity is also a risk
dened as a fever >38.3C (>101F) lasting >3
factor, but it will not lower his hyperlipidemia
weeks that remains undiagnosed after three
as quickly as will more direct therapy to lower
outpatient visits or 3 days of hospitalization.
his LDL cholesterol (such as drug therapy).
Answer A is incorrect. Collagen vascular dis-
ease and granulomatous diseases combined ac- 39. The correct answer is E. This patient is expe-
count for >15% of fevers of unknown origin. riencing a threatened abortion, which is differ-
entiated from a spontaneous abortion by the
Answer B is incorrect. Collagen vascular dis-
cervical os being closed. The fetus is still via-
ease and granulomatous diseases combined ac-
ble, though observation is necessary, as threat-
count for >15% of fevers of unknown origin.
ened abortions may become spontaneous abor-
Answer D is incorrect. Neoplasm is a com- tions. Currently there is no evidence that bed
mon cause of fever of unknown origin, but less rest is helpful for preventing the opening of the
common than either autoimmune disease or os and loss of the pregnancy.
infection.
Answer A is incorrect. Betamethasone is used
Answer E is incorrect. Pulmonary emboli are to mature the fetal lungs in preterm infants. At
a relatively rare cause of fevers of unknown ori- this gestational age, the fetus is not viable.
gin.
Answer B is incorrect. Cervical cerclage, or
suturing the cervix closed, is a procedure that

FULL-LENGTH EXAMS
38. The correct answer is A. The National Cho-
is performed if the cervix is incompetent. Pa-
lesterol Education Program (NCEP) Adult
tients with cervical incompetence have dila-
Treatment Panel III has issued guidelines that
tion and effacement of the cervix, usually dur-
recommend that any person with cardiovascu-
ing the second trimester. Occasionally this will
lar disease or diabetes mellitus achieve an LDL
lead to loss of the fetus. There is no evidence
cholesterol of <100 mg/dL. However, some re-
to suggest that this woman has an incompetent
cent evidence suggests that for patients with
cervix.
cardiovascular disease and diabetes (very high-
risk patients), an LDL cholesterol goal of <70 Answer C is incorrect. If the cervical os were
mg/dL may be more optimal. open with bleeding at this gestational age, a
spontaneous abortion would be inevitable. An
Answer B is incorrect. The NCEP guidelines
recommend that those with at least two risk emergent dilation and curettage would then be
factors (including tobacco use, hypertension, necessary to minimize the chance of retention
high LDL cholesterol, low HDL cholesterol, of fetal products and subsequent infection.
diabetes mellitus, and obesity) achieve an LDL Answer D is incorrect. Magnesium sulfate is
cholesterol goal of <130 mg/dL. An LDL cho- a tocolytic that is used to slow down uterine
lesterol goal of <160 mg/dL is recommended contractions in preterm labor. However, this
Test Block 3

for those without cardiovascular disease or di- woman is only 14 weeks along. Given that she
abetes mellitus and who have only one or no is so far from viability we would not start mag-
risk factors. nesium sulfate. It is not a long-term treatment
option.
568 Section II: Full-Length Examinations Answers

40. The correct answer is D. The patient is hypo- Color variegation (this lesion is monochro-
glycemic, and normalization of the blood glu- matic and brown), and a Diameter of 6 mm.
cose level is likely to alleviate her symptoms. Basal cell carcinoma typically presents with a
Sulfonylureas act primarily as insulin secret- slow-growing skin mass that is ulcerated, with
agogues, but also limit hepatic glucose produc- or without pigmentation. There is a very low
tion, decrease lipolysis, and decrease hepatic risk that this patients lesion is cancerous.
clearance of serum insulin. When initiating
Answer B is incorrect. Patients with body dys-
these medications, it is important to use the
morphic disorder are preoccupied with some
lowest possible dose and to increase slowly to
perceived physical abnormality to the point
avoid overshoot hypoglycemia.
of experiencing profound distress and impair-
Answer A is incorrect. The patient already ex- ment. This patient does not indicate that he
hibits symptoms of adrenergic excess (tachycar- feels his condition is abnormal, only that he
dia, tremulousness, palpitations, and diaphore- is convinced he has a specic disorder. Hypo-
sis) as would be expected with hypoglycemia. chondriasis is thus a more likely diagnosis.
Answer B is incorrect. Administering insulin Answer D is incorrect. Somatization disorder
would further decrease the blood glucose level is a chronic psychiatric condition that typically
and exacerbate the patients symptoms. presents with a long-standing history of unsub-
stantiated medical complaints. The DSM-IV-
Answer C is incorrect. It is unlikely that the
TR criteria for a somatization disorder require
patient has overdosed on opiates given her lack
that the patient have a history of pain in at least
of respiratory depression.
four sites, including two gastrointestinal symp-
Answer E is incorrect. It is unlikely that the toms, one sexual symptom, and one neurologic
patient is suffering from ischemic stroke or symptom. This patient has only one complaint
myocardial infarction. Although she is lethar- and no medical history, making somatization
gic and obtunded, she has no focal neurologic disorder an unlikely diagnosis.
signs. Hypotension would be expected if the
Answer E is incorrect. When assessing the
cause of her mental status changes was cardiac
risk of a pigmented lesion, the ABCD criteria
failure. If the patient does not improve after
FULL-LENGTH EXAMS

apply: Asymmetry (this lesion is symmetric),


glucose administration, further evaluation of
Border irregularity (the borders here are regu-
her cardiac and neurologic status would be
lar), Color variegation (this lesion is mono-
warranted.
chromatic and brown), and a Diameter of 6
41. The correct answer is C. This patient is exhibit- mm. Squamous cell carcinoma typically pre-
ing the symptoms of hypochondriasis, which in- sents with a raised, pigmented skin lesion that
volves the persistent fear and conviction that one may be ulcerated, exudative, scabbed, or itchy.
There is a very low risk that this patients lesion
is very ill, combined with the misinterpretation
is cancerous.
of normal bodily symptoms. The DSM-IV-TR
criteria for hypochondriasis require that the pa-
42. The correct answer is D. This woman most
tient have at least a 6-month history of persistent
likely has acute tubular necrosis (ATN), which
and debilitating fear that is not diminished by
presents with a picture of acute renal failure,
appropriate medical evaluation and reassurance.
often following sepsis, hypotension, or other
Men and woman are equally affected, and the
ischemic insult. Although prerenal failure can
disorder usually presents in middle age. Treat-
also result from these, it resolves more rapidly
ment consists of reassurance and a referral for
with uid administration, and presents with a
psychotherapy.
prerenal picture, including BUN:creatinine ra-
Test Block 3

Answer A is incorrect. When assessing the risk tio >20:1 and fractional sodium excretion <1%.
of a pigmented lesion, the ABCD criteria ap- ATN, in contrast, lasts longer, with creatinine
ply: Asymmetry (this lesion is symmetric), Bor- elevation often persisting for several weeks, and
der irregularity (the borders here are regular), presents with signs of renal failure such as ele-
Test Block 3 Answers 569

vated BUN and creatinine, fractional sodium Answer A is incorrect. Chlamydia trachoma-
excretion >2%, and low urine osmolarity due tis is a sexually transmitted disease that causes
to impaired urine concentrating ability despite cervicitis and pelvic inammatory disease. In-
hypovolemia. Muddy brown casts on urinalysis fection with chlamydia during pregnancy does
are characteristic of ATN. increase the risk of chorioamnionitis. However,
conjunctivitis and pneumonia are the most
Answer A is incorrect. Dobutamine is a com-
common manifestations of chlamydia in new-
monly used pressor, which could potentially
borns, not sepsis.
have been administered in this patient. Certain
drugs such as aminoglycosides and NSAIDs Answer C is incorrect. HIV may be transmit-
can cause acute interstitial nephritis; however, ted vertically from mother to child late in preg-
this has not been described with dobutamine. nancy or during labor. Infection with HIV does
Furthermore, in acute interstitial nephritis, not cause chorioamnionitis or neonatal sepsis.
urinalysis classically shows WBCs and WBC Answer D is incorrect. Primary infection of a
casts, as well as eosinophils. pregnant woman with toxoplasmosis can cause
Answer B is incorrect. Autoimmune glomeru- congenital toxoplasmosis in the newborn child.
lonephritis may be seen in conditions such as Complications of congenital toxoplasmosis in-
lupus and Goodpastures syndrome. However, clude chorioretinitis, hydrocephalus, seizures,
these causes of glomerular disease present with and mental retardation. The infection is often
nephritic or nephrotic syndrome with hyper- subclinical in infants with disease manifesta-
tension, edema, and proteinuria, and are not tions appearing later in life. Infection in adults
consistent with this patients presentation. is usually asymptomatic.
Answer C is incorrect. Hypovolemia can cause Answer E is incorrect. Treponema palladium
prerenal failure; however, in this condition, is the causative organism in syphilis, which can
one would expect a higher BUN:creatinine cause congenital defects such as liver failure
ratio, lower fractional sodium excretion, and and skin lesions. However, it is not implicated
higher urine osmolarity. Furthermore, hypo- in chorioamnionitis or neonatal sepsis.
volemic renal failure should be responsive to

FULL-LENGTH EXAMS
uids over 2472 hours and would not take 44. The correct answer is E. Because the parathy-
weeks to resolve. roid hormone level and serum calcium con-
centration is high, the patient most likely has
Answer E is incorrect. Pyelonephritis does not
primary hyperparathyroidism. Single adenoma
commonly cause renal failure. Furthermore,
is the most frequent cause of primary hyper-
in kidney infection, one would expect ank
parathyroidism, accounting for 80% of cases.
pain, dysuria, and WBCs or WBC casts in the
urinalysis, none of which are described in this Answer A is incorrect. Ectopic parathyroid tis-
patient. sue accounting for hyperparathyroidism is very
rare.
43. The correct answer is B. Group B streptococci
Answer B is incorrect. Multiple adenomas are
(GBS) commonly colonize the gastrointestinal
much less common than a single adenoma.
or genital tracts of adults. In pregnant females,
colonization with GBS can lead to chorioam- Answer C is incorrect. Parathyroid carcinoma
nionitis and sepsis. In newborn infants, group is an exceedingly rare cause of primary hyper-
B streptococcal infection can lead to bactere- parathyroidism.
mia, sepsis, and meningitis. Screening for GBS Answer D is incorrect. Parathyroid hyperpla-
colonization is commonly done in pregnant sia is the second most likely cause of primary
Test Block 3

women between 3537 weeks gestation. hyperparathyroidism, accounting for 15% of


Women who are GBS positive then receive IV cases.
penicillin in labor to protect the infant from
getting the infection during delivery.
570 Section II: Full-Length Examinations Answers

Questions 45 and 46 Answer B is incorrect. Campylobacter jejuni


is a gram-negative agellated rod that, along
45. The correct answer is G. Rotavirus is the most
with rotavirus and enterotoxigenic Escherichia
important cause of severe dehydrating diarrhea
coli, is one of the three most common causes
in early childhood. It is most commonly en-
of diarrhea in the world. Domestic animals
countered during the winter months and is
and poultry are a reservoir for these bacteria,
spread easily along the fecal-oral route. Child
which are passed by the fecal-oral route and
care centers and childrens hospitals are com-
unpasteurized milk. Infection presents with a
mon sites of encounter with the infectious
prodrome of fever, headaches, and malaise, fol-
agents. The virions of the rotaviruses are highly
lowed by bloody, loose diarrhea. Treatment is
contagious and shed in high concentrations in
with a uoroquinolone and uid resuscitation.
the stool before and during the clinical mani-
festations. Patients typically present after an in- Answer C is incorrect. Clostridium difcile is
cubation period of <48 hours with low-grade to the pathogen of antibiotic (usually clindamy-
moderate-grade fever and simultaneous vomit- cin or ampicillin)-associated colitis. Symptoms
ing. Watery diarrhea is a later manifestation, include diarrhea, fever, and abdominal cramp-
often occurring for 57 days. Patients must be ing. C. difcile toxins can be tested for in stool
monitored closely for deteriorating clinical samples. Treatment is usually with metronida-
condition secondary to dehydration, which if zole and discontinuation of antibiotics.
left untreated can lead to eventual death.
Answer D is incorrect. Enteric adenovirus is
Treatment is aggressive hydration and antibi-
another common viral etiology of diarrhea in
otic therapy is not recommended.
infants and children. However, this virus is
most prevalent in the spring and summer sea-
46. The correct answer is H. Acute enteritis is the
sons. Infection is often accompanied by fever,
most common manifestation of enteric salmo-
rhinorrhea, cough, and acute respiratory dis-
nellosis. Patients infected with nontyphoidal
ease. No antivirals or vaccines are appropriate
Salmonella experience an acute onset of nau-
for the treatment of adenovirus, and aggressive
sea, vomiting, and abdominal cramping, fol-
hydration is the treatment of choice.
lowed by mild to severe watery diarrhea that
FULL-LENGTH EXAMS

may contain blood and mucus. The incuba- Answer E is incorrect. Escherichia coli bacte-
tion period for infection is typically 672 hours rial colitis is caused by several subtypes of the
with a mean incubation period of 24 hours. E. coli bacteria. Enterotoxigenic E. coli causes
The major source of nontyphoidal Salmonella a rice water-type stool and is often seen in trav-
is constituted by animals, specically poultry elers to Third World countries. Enterohemor-
and poultry products. In this case, the patients rhagic E. coli causes a hemorrhagic colitis, and
clinical presentation is consistent with salmo- the specic subtype E. coli O157:H7 results in
nellosis, and the recent inclusion of egg salad, hemolytic uremic syndrome. This syndrome is
chicken, and homemade eggnog in her diet characterized by thrombocytopenia and renal
should lead to high suspicion of the diagnosis. failure. Enteroinvasive E. coli results in colitis
Therapy is aggressive hydration alone; in fact, along with fever, and WBCs can be found in
antibiotics are detrimental in this situation and the stool. Treatment is aggressive oral hydra-
should never be prescribed. tion without antibiotic therapy.
Answer A is incorrect. Bacillus cereus is a Answer F is incorrect. Giardia lamblia is a
gram-positive spore-forming rod that causes agellated protozoan that infects the duode-
food poisoning when the spores land on food num and small intestine in affected patients.
and germinate, creating a toxin. Ingestion of Infection is often asymptomatic, but in patients
Test Block 3

the toxin will lead to sudden onset nausea, manifesting symptoms, watery diarrhea is the
vomiting, and diarrhea. Because the reaction most common presentation. Other symptoms
is due to preformed toxins, antibiotics are not include malaise and weakness, atulence, ab-
useful, and treatment is supportive. dominal distention, foul-smelling greasy stools,
weight loss, and anorexia. G. lamblia has an
Test Block 3 Answers 571

incubation period of approximately 12 weeks, Answer K is incorrect. Vibrio cholerae is a


and diarrhea can be prolonged, lasting in some gram-negative agellated rod that is passed by
patients for several weeks. the fecal-oral route, usually through drinking
contaminated water. It presents with sudden-
Answer I is incorrect. Shigella is a nonagel-
onset, voluminous rice water diarrhea. Treat-
lated gram-negative rod. It is found only in hu-
ment is primarily with uid resuscitation and
mans and passed by the fecal-oral route, and
oral rehydration as patients can lose up to 1
is commonly found in communal living situ-
L of uid in an hour, and death is commonly
ations or day care. Infection presents with fe-
from dehydration. Denitive treatment is with
ver and abdominal pain, along with blood and
antibioticseither doxycycline or a uoroqui-
pus-speckled diarrhea. Treatment is with uo-
nolone.
roquinolones and uid resuscitation.
Answer L is incorrect. Vibrio parahaemo-
Answer J is incorrect. Staphylococcus aureus
lyticus is a gram-negative rod that is found in
is a pathogen implicated in multiple cases of
marine water and is a common cause of diar-
food-borne illness each year. The mechanism of
rhea in Japan, where raw seafood is commonly
infection is based on preformed toxins that are
consumed. Infection presents similarly to V.
found in infected foods and are able to elicit the
cholerae with rice water diarrhea; however,
clinical symptoms described. The incubation
V. parahaemolyticus typically resolves within 3
period for S. aureus food poisoning is within
days without need for treatment.
hours after ingestion, and emesis is commonly
the rst and most signicant symptom.

FULL-LENGTH EXAMS
Test Block 3
This page intentionally left blank
Test Block 4

573
574 Section II: Full-Length Examinations Questions

Q U E ST I O N S

1. A 51-year-old woman presents to her primary 3. A 30-year-old man is brought by ambulance to


care physician complaining of headache for the emergency department after a major motor
the past 2 weeks. Further questioning reveals vehicle crash. He is lethargic but conscious
weakness over the past several months that has and after the primary survey he is found to
led to gradual impairment of physical activi- have signicant hypotension and tachycardia.
ties. The patient has no other complaints, takes Secondary survey reveals an unstable pelvic
no medications other than a multivitamin, and girdle and what appears to be a developing pel-
smokes one pack of cigarettes a day. Her blood vic hematoma. There is no blood at the ure-
pressure is 150/94 mm Hg. Laboratory studies thral meatus. Anteroposterior x-ray of the pelvis
reveal a potassium level of 3.1 mEq/L and so- is shown in the image. Which of the following
dium of 143 mEq/L. Imaging reveals bilateral is contraindicated in this patient?
adrenal hyperplasia. Which of the following
would be most appropriate for this patient?
(A) Clonidine
(B) Hydrochlorothiazide
(C) Metoprolol
(D) Spironolactone
(E) Sumatriptan

2. The parents of a 4-week-old white boy bring


their son to the pediatric emergency depart-
ment concerned about their sons feeding pat-
terns. His mother reports he was feeding well
for the rst several days but developed nonbil-
Reprinted, with permission, from Brunicardi FC, Ander-
ious emesis toward the end of his second week
sen DK, Billiar TR, Dunn DL, Hunter JG, Matthews JB,
FULL-LENGTH EXAMS

of life. The episodes of emesis have worsened Pollock RE, Schwartz SI. Schwartzs Principles of Surgery,
and now occur after every feeding, with some 8th edition. New York: McGraw-Hill, 2005: Fig. 42-41.
episodes being projectile in nature. Immedi-
ately after vomiting, the newborn is hungry and (A) Emergent pelvic angiography
ready to feed again. He has a small, mobile (B) Exploration of the hematoma
nontender epigastric mass on physical exami- (C) Manual prostate examination
nation. Heart rate is 128/min, blood pressure is (D) Orthopedic consultation for xation
80/52 mm Hg, respiratory rate is 37/min, and (E) Retrograde urethrogram
temperature is 37.5C (99.5F). What acid- (F) Volume resuscitation
base disorder is the patient most at risk of de-
veloping? 4. A 56-year-old man with diabetes and hyperten-
sion presents to the ophthalmology clinic with
(A) Hyperchloremic-hypokalemic metabolic a history of a blind spot in his right eye. He re-
acidosis ports that it is painless, occurred suddenly, and
(B) Hyperchloremic-hypokalemic metabolic is only in his right eye. He has a visual eld
alkalosis loss in the right superior hemield. Examina-
(C) Hypochloremic-hyperkalemic metabolic tion of the retina shows peripheral hemor-
alkalosis rhages and disc congestion. Which of the fol-
Test Block 4

(D) Hypochloremic-hypokalemic metabolic lowing is the most likely diagnosis?


acidosis
(E) Hypochloremic-hypokalemic metabolic (A) Acute ophthalmic artery occlusion
alkalosis (B) Branch retinal vein occlusion
(C) Central retinal vein occlusion
Test Block 4 Questions 575

(D) Diabetic retinopathy admits to current cocaine and alcohol use. He


(E) Hypertensive retinopathy is hypertensive to 162/100 mm Hg and diapho-
retic. His amylase and lipase levels conrm
5. A 25-year-old African-American woman devel- that he has acute pancreatitis. He is placed on
ops a nonproductive cough and dyspnea on ex- empiric antibiotics, aggressive intravenous u-
ertion that progressively worsens over several ids, and is made nothing by mouth status.
months. X-ray of the chest demonstrates bilat- His blood pressure remains elevated. What
eral hilar adenopathy with interstitial inltrates agent should the physician use to lower his
bilaterally. Biopsy of a waxy nodule on the blood pressure?
womans arm reveals noncaseating granuloma
(A) Alprenolol
formation. Which of the following symptoms is
(B) Atenolol
most likely to be seen in patients with this dis-
(C) Lorazepam
ease?
(D) Metoprolol
(A) Alopecia (E) Verapamil
(B) Ataxia and dysmetria
(C) Distal extremity weakness 8. A 5-year-old boy presents with fever, pallor, and
(D) Keratoconjunctivitis bony pain and is diagnosed with acute lym-
(E) Koilonychia phoid leukemia following extensive laboratory
work-up and bone marrow biopsy. His labora-
6. A 28-year-old woman presents to the emer- tory studies revealed a WBC count of 240,000/
gency department complaining of sharp ab- mm, hemoglobin of 8.1 g/dL, and platelet
dominal pain localized to the left lower quad- count of 79,000/mm. Treatment with chemo-
rant. She reports feeling perfectly well until therapy is initiated emergently. Twelve hours
half an hour prior to her arrival. When asked if after his therapy is begun he develops acute
she could be pregnant, the patient responds ventricular arrhythmias. Review of his bedside
that her last menstrual period was 2 weeks ago, chart reveals anuria over the past 8 hours, de-
and that she has not been sexually active dur- spite vigorous provision of intravenous uids.
ing the past month. The patients temperature Laboratory evaluation reveals a blood urea ni-

FULL-LENGTH EXAMS
is 36.9C (98.4F) and blood pressure is trogen level of 150 mg/dL, sodium of 2.5
110/70 mm Hg. Abdominal examination elicits mEq/L, and potassium of 8.9 mEq/L. Urinaly-
guarding and rebound tenderness. Pelvic ex- sis reveals uric acid crystals in his urine. Which
amination is normal, and urine -human cho- element of his disease and/or therapy is most
rionic gonadotropin is negative. Which of the likely to be accountable for his condition?
following is the most likely cause of this pa-
(A) Cardiac damage due to doxorubicin ther-
tients abdominal pain?
apy
(A) Acute appendicitis (B) Incorrect dosing of the anesthetic given
(B) Acute gastritis during the bone marrow biopsy
(C) Ectopic pregnancy (C) Leukemic inltrate of the kidney leading
(D) Mesenteric ischemia to decreased renal function
(E) Ruptured corpus luteum cyst (D) Overhydration leading to acute renal fail-
ure
7. A 48-year-old man presents to the emergency (E) Rapid destruction of leukocytes by chemo-
department complaining of stabbing abdomi- therapeutic agents
nal pain that increases when he tries to eat. He
Test Block 4
576 Section II: Full-Length Examinations Questions

9. A 58-year-old long-time alcoholic is brought to temporarily relieved by acetaminophen. He


the emergency department after vomiting has had two episodes of pneumonia in the last
blood. After a comprehensive work-up, the 5 months, and a maternal uncle who died at
bleeding is determined to be due to ruptured an early age. On examination he is a thin child
esophageal varices. The patient has been who is breathing rapidly. Auscultation of the
treated for this condition several times before. chest shows decreased breath sounds in both
He is admitted and a management plan is de- bases. X-ray of the chest shows consolidation of
termined. Four days following a new interven- the left and right lower lobes. Since the patient
tion, the patient becomes confused, agitated, cannot produce a good sputum sample, bron-
and lethargic. On examination he demon- choscopy is performed and a lavage sample is
strates asterixis and hyperreexia. Given the sent. Preliminary Gram stain results are as fol-
onset of symptoms following treatment, what lows:
was the most likely new intervention?
Color: Yellow
(A) -Blocker therapy Consistency: Thick
(B) Balloon tamponade Neutrophils: Many
(C) Endoscopic band ligation Gram stain: Numerous gram-negative rods
(D) Endoscopic sclerotherapy
Which of the following causative organisms is
(E) Portal systemic shunt
most likely involved?
10. A delirious 82-year-old man with terminal pan- (A) Klebsiella pneumoniae
creatic cancer is having episodes of hypoten- (B) Listeria monocytogenes
sion due to a systemic infection. The intensive (C) Pseudomonas aeruginosa
care unit team is considering use of potent in- (D) Staphylococcus aureus
travenous antibiotics to treat the infection.
Treatment is begun, despite a signed living will 12. An 82-year-old man with hypertension, hyper-
in the form of a Do Not Resuscitate (DNR) cholesterolemia, and chronic obstructive pul-
order. Did the team make the wrong manage- monary disease presents to the emergency de-
ment decision? partment with severe back pain. He also feels
FULL-LENGTH EXAMS

diaphoretic and nauseous. The patient states


(A) No, a DNR order should not inuence ac-
he is usually compliant with his medications
tion against treatable conditions
but sometimes forgets to take them. He denies
(B) No, but if a durable power of attorney was
any alcohol use or illicit drug use. His temper-
in the le, that typically supersedes an ad-
ature is 37.1C (98.7F), blood pressure is
vance directive provided by the patient
165/104 mm Hg, pulse is 112/min, respiratory
(C) No, but the DNR order dictates that intra-
rate is 18/min, and oxygen saturation is 94%.
venous antibiotics should be stopped if the
On physical examination there is no jugular
patient has adverse effects from the treat-
venous distension, and there are diminished
ment
but equal pulses bilaterally in the lower ex-
(D) Yes, a health care proxy should have been
tremities. Cardiovascular examination reveals
consulted before treatment was begun
regular rate and rhythm without murmurs,
(E) Yes, a signed DNR order indicates that the
rubs, or gallops. CT of the chest is shown in
patient did not want to prolong his suffer-
the image. Which of the following is the most
ing, so no treatment should have been
likely diagnosis?
given

11. A 6-year-old boy presents to the clinic with dys-


Test Block 4

pnea at rest and cough productive of thick mu-


coid sputum over the last 5 days. He has a tem-
perature of 39.4C (103F), which is only
Test Block 4 Questions 577

room very slowly and refuses to make eye con-


tact with the physician. He has lost nearly 6.8
kg (15 lb) in 1 year. When directly questioned
about suicidal ideation, he indicates that he of-
ten thinks about dying and has thought about
jumping out the window of his ofce on the
27th oor. He also states that he has never ex-
perienced anything like this before in his life.
Which of the following is most appropriate for
this patient?
Reprinted, with permission, from Brunicardi FC, Ander-
sen DK, Billiar TR, Dunn DL, Hunter JG, Matthews JB, (A) Amitriptyline
Pollock RE, Schwartz SI. Schwartzs Principles of Surgery, (B) Benztropine
8th edition. New York: McGraw-Hill, 2005: Figure 21-12. (C) Citalopram
(D) Phenelzine
(A) Aortic dissection (E) Risperidone
(B) Lumbar disc herniation
(C) Pancreatic cancer 15. A 58-year-old man presents to the emergency
(D) Perforated peptic ulcer department with 1 day of high fever to 39.5C
(E) Ruptured aortic aneurysm (103.1C) along with muscle aches and mal-
aise. During the initial interview, he remem-
13. A 20-year-old recent immigrant presents to his bers his name but is not oriented to place or
primary care physician with complaints of time, and he cannot recall the names of his
jaundice, discoloration of his eyes, tremor, dys- four children. He has no known medical prob-
phagia, and depressed mood. Examination re- lems. His wife claims that he is in perfect
veals jaundice, green-brown deposits in the health and reports that he was out playing in
cornea, hepatomegaly, tremor, and rigidity of the yard with their grandchildren 3 days ago.
the extremities. The physician suspects the pa- They have not been out of the country in >2
tient suffers from a specic liver metabolic ab- years and have no animals at home. A periph-
normality and wants to conrm his diagnosis

FULL-LENGTH EXAMS
eral blood smear demonstrates morulae within
with further evaluation. Which of the follow- several monocytes. Laboratory studies show:
ing would be the most appropriate next step in
diagnosis? Platelet count: 84,000/mm
WBC count: 2500/mm
(A) Check the levels of serum iron, ferritin, Hemoglobin: 14.2 g/dL
and transferrin, and the percentage satura- Serum transaminases: Mildly elevated
tion of iron
(B) Check the serum level of ceruloplasmin Which of the following is most likely to have
(C) Order an abdomen CT scan with contrast caused this patients disease?
(D) Order hepatitis screening panel (A) Babesia microti
(E) Perform ultrasound-guided paracentesis (B) Borrelia burgdorferi
(C) Ehrlichia chaffeensis
14. A 38-year-old man is brought to his doctor by (D) HIV
his wife, who describes a distinct change in his (E) Leishmania donovani
personality over the past several weeks. Al- (F) Rickettsia rickettsii
though he holds a successful job, he has not (G) Trypanosoma brucei
gone to work for the past week because he
doesnt feel like it. He typically sits in bed
Test Block 4

and stares blankly at the television for hours at


a time. Occasionally she nds him crying for
no apparent reason. He enters the examination
578 Section II: Full-Length Examinations Questions

16. A 25-year-old woman presents to her primary the following is the most appropriate next step
care physician with fatigue, malaise, swollen in evaluation?
feet, and a weight gain of >4.5 kg (>10 lb) in (A) Blood cultures
the past 2 weeks. She also describes a rash that (B) MRI of the abdomen
is distributed primarily over her face, neck, and (C) Percutaneous drainage
distal arms. On questioning, she recalls that (D) Ultrasound
she has also had sore knees, wrists, and ankles (E) Urine culture
on and off for the past few months. Her face is
quite red, exhibiting an erythematous eruption 18. An 18-year-old woman presents to her primary
over her cheeks and the bridge of her nose, care doctor for a follow-up appointment subse-
and she has small, pinpoint lesions on her buc- quent to a diagnosis of infectious mononucleo-
cal mucosa and hard palate. She has 2+ pitting sis. One month ago her WBC count was
edema up to her knees bilaterally. The remain- 12,000/mm, hemoglobin was 13.7 g/dL, and
der of her physical examination is within nor- platelet count was 183,000/mm. At the cur-
mal limits. Her urine dipstick shows 3+ protei- rent follow-up visit, WBC count is 7800/mm,
nuria and microscopic hematuria. Which of hemoglobin is 14.2 g/dL, and platelet count is
the following autoantibodies would be ex- 78,000/mm. The patient denies any increased
pected to correlate with disease activity in this bleeding or spontaneous bleeding. Additional
patient? laboratory tests are notable for a bleeding time
(A) Anticentromere antibodies of 9 min, an activated partial thromboplastin
(B) Anti-double-stranded DNA antibodies time of 27 sec, a prothrombin time of 12 sec,
(C) Anti-Jo-1 antibodies and a normal clotting time. What is the best
(D) Anti-Scl-70 antibodies next step in management?
(E) La antibodies (A) Conservative management with follow-up
(F) Ro antibodies and repeat complete blood cell count
(B) Initiate steroids
17. A 65-year-old woman with a history of coronary (C) Splenectomy
artery disease presents to the emergency de- (D) Transfuse fresh frozen plasma
FULL-LENGTH EXAMS

partment complaining of severe abdominal (E) Transfuse packed RBCs


pain for the past 2 days, rst beginning as dull (F) Transfuse platelets
pain near the umbilicus but now localized to
the right lower quadrant (RLQ). The patient 19. A 52-year-old woman with ovarian cancer has
reports that just prior to examination by the been in the hospital for more than 4 months
physician, she experienced a sudden decrease and is currently without decision-making ca-
in intensity of pain, but she still feels very un- pacity. On the most recent CT of the chest, ab-
comfortable and must remain still on the domen, and pelvis, bulky disease was evident
stretcher. On examination the patient appears throughout. She has already been through two
in distress, and she is hypotensive with systolic sessions of chemotherapy, the most recent of
blood pressure at 85 mm Hg (systolic blood which provided minimal disease reduction.
pressure at triage was 110 mm Hg), and her Her chosen health care proxy, in agreement
temperature is 40C (104F). She has a dif- with the wishes of the rest of the family, has re-
fusely tender abdomen with point tenderness quested a third round of chemotherapy. Her
in her RLQ, accompanied by guarding and re- physician refuses to administer another round
bound. Laboratory values showed a leukocyto- of chemotherapy. What is the most appropriate
sis of 20,000/mm with 95% polymorphonu- response of the health care proxy?
clear lymphocytes. Her ECG shows
Test Block 4

nonspecic ST-T changes from a baseline (A) Because the patient chose the health care
ECG taken 6 months ago. CT of the abdomen proxy when she had capacity, the physician
shows a loculated mass in the RLQ. Which of must follow her instructions
Test Block 4 Questions 579

(B) Because this treatment was partially suc- (D) She most likely has an upper respiratory
cessful the rst time it was administered, tract infection that will resolve on its own
the physician is ethically required to ad- (E) The physician are obligated to contact
minister the chemotherapy child protective services before he can dis-
(C) Physician is legally and ethically required cuss her condition further
to administer the chemotherapy
(D) Physician is legally, but not ethically, re- 21. A 25-year-old white man presents to the emer-
quired to administer the chemotherapy gency department with diarrhea, weight loss of
(E) Physician is neither legally nor ethically re- 4.5 kg (10.0 lb) over the preceding 2 months,
quired to administer the chemotherapy and RLQ pain. His temperature is 38.2C
(100.7F), heart rate is 82/min, and blood pres-
20. An 11-month-old child is brought to the physi- sure is 110/70 mm Hg. Abdominal examina-
cian by her parents because they are concerned tion reveals mild tenderness with no rebound
about seizures of increasing frequency and tenderness, and a mass is palpated in the RLQ.
severity during which the child ails her upper His stool tests positive for occult blood. Subse-
extremities. Physical examination is normal ex- quent colonoscopy reveals erythematous, fria-
cept for psychomotor delay. On questioning, ble mucosa with ulcerations in a longitudinal
her parents mention that she is not speaking at distribution. The ulcerations are segmental
all, she is not yet walking, and in fact only re- and interspersed with intervening normal mu-
cently has been able to sit on her own. Electro- cosa. Colonic biopsies show a dense inamma-
encephalography shows very-high-voltage, ran- tory inltrate with neutrophils and mononu-
dom, slow waves and spikes in all cortical clear cells. The patient should be informed of
areas. When her parents inquire about her which of the following?
prognosis, what are they told?
(A) Antibiotics have been shown to benet pa-
(A) Although her spasms will eventually re- tients with his condition
gress, she is likely to be neurologically im- (B) Appendectomy should be scheduled
paired (C) Curative proctocolectomy with ileoanal
(B) Her condition is due to an infectious agent anastomoses is indicated at this time

FULL-LENGTH EXAMS
and her symptoms will most likely resolve (D) Toxic megacolon is a frequent complica-
upon treatment tion of his disease
(C) Her condition is entirely benign and re- (E) Trial gluten-free diet is indicated
quires no treatment

Test Block 4
580 Section II: Full-Length Examinations Questions

22. A postmenopausal 50-year-old woman with ciably enlarged. The skin shows no rashes.
cryptogenic liver cirrhosis and coagulopathy is Laboratory studies show a WBC count of
admitted to the hospital because of severe vagi- 56,000/mm (with 59% neutrophils), a hemo-
nal bleeding secondary to uterine broids. The globin level of 8.5 g/dL, and a platelet count of
patient is deemed to be a poor surgical candi- 21,000/mm ECG shows normal sinus rhythm
date because of her coagulopathy, and she is with no ST-segment changes. Results of a bone
transferred to the medicine service for preoper- marrow biopsy are shown in the image. The
ative management, including packed RBCs cells stain positively with myeloperoxidase.
and fresh frozen plasma transfusions. Although Which of the following is the most likely diag-
the patient initially gave her informed consent nosis?
to receive transfusions, after 2 units of packed
RBCs she refuses further transfusions, stating,
I want a more denitive treatment than just
transfusions! The patients hematocrit is
steadily decreasing and she continues to experi-
ence vaginal bleeding. Although the internist
thoroughly explains the nature, indications,
risks, benets, and alternatives regarding blood
product transfusions, and the need for transfu-
sions prior to denitive surgery, the patient
continues her refusal to receive blood products.
The patient denies feeling depressed and is
alert and oriented to time, place, and person.
There are no signs of depression or suicidality.
Which of the following is the most appropriate Reproduced, with permission, from Lichtman MA, Beut-
next step in patient care? ler E, Kipps TJ, Seligsohn U, Kaushansky K, Prchal JT.
Williams Hematology, 7th edition. New York: McGraw-
(A) Call for psychiatry consultation to deter- Hill, 2006: Plate XIV-9.
mine competence
FULL-LENGTH EXAMS

(B) Call the ethics committee and seek ap- (A) Acute myelogenous leukemia
proval to override the patients decision (B) Anemia of chronic disease
(C) Hold transfusions and have a discussion (C) Chronic myelogenous leukemia
with the gynecology consult service regard- (D) Multiple myeloma
ing alternatives to surgery, such as inter- (E) Thrombotic thrombocytopenia purpura
ventional radiology procedures
(D) Since the patient had given consent for 24. A 46-year-old man with no past medical history
earlier transfusions, the internist should comes to the emergency department complain-
continue the transfusions and reverse the ing of an inability to move his left leg. He takes
coagulopathy to prepare the patient for no medications or herbal compounds and de-
surgery nies having any allergies. The patient currently
(E) Talk to patients husband and seek his con- lives with a male companion, with whom he
sent for the patients transfusions has been monogamous for 18 months. He de-
nies any tobacco, ethanol, or intravenous drug
23. A 63-year-old man presents to his physician use and has never had an HIV test. Neurologic
with fatigue and frequent nosebleeds that have examination reveals no cranial nerve decits,
developed gradually over 4 months. He says he and strength, sensation, and tone are normal in
feels abdominal fullness and becomes full eas- the upper extremities. The man has substan-
Test Block 4

ily when eating. On physical examination he is tially increased tone in both legs, left more
afebrile but shows pallor of the conjunctivae than right, and strength is 3/5 in the right hip
and some dried blood around the nares. Sple- exors and knee extensors and 0/5 on the left.
nomegaly is present, but the liver is not appre-
Test Block 4 Questions 581

His reexes are increased bilaterally, left more terstitial markings. Which of the following
so than the right, and four beats of clonus are laboratory abnormalities is most likely to be
detected on the left. His toes are upgoing bilat- seen?
erally. Laboratory tests are remarkable for a
(A) Calcium level of 11.3 mEq/L
WBC count of 3000/mm; a lumbar puncture
(B) Glucose level of 210 mg/dL
is unremarkable. Follow-up HIV testing is posi-
(C) Hemoglobin of 10.5 g/dL
tive, and his CD4+ cell count is 28/mm. MRI
(D) Sodium level of 151 mEq/L
of the brain shows several nonenhancing white
(E) WBC count of 11,600/mm with 80% lym-
matter lesions; no mass effect is evident. Which
phocytes
of the following is the most likely mechanism
for this patients symptoms? 27. A 56-year-old woman presents to the surgery
(A) Demyelination of the central nervous clinic. A suspicious lesion was found on rou-
system tine mammography 1 week earlier. The patient
(B) Infection with Candida albicans denies any pain at the site of the lesion, but
(C) Infection with herpes simplex virus type 1 over the past month she has experienced in-
(D) Infection with JC virus creased fatigue. Past gynecologic history is no-
(E) Infection with Toxoplasma gondii table for nulliparity, onset of menarche at the
age of 9 years, and age at menopause of 50
25. A 55-year-old man with diabetes, hypertension, years. Her pulse is 76/min, blood pressure is
and a history of pneumonia treated as an out- 123/76 mm Hg, and temperature is 37.8C
patient 20 years earlier presents to the clinic (100.1F). Breast examination is signicant for
during early fall for a blood pressure check. He a xed, nontender 2-cm lump in the upper
currently works as a respiratory therapist at a lo- outer quadrant of the left breast. Pelvic exami-
cal hospital. The physician suggests that he re- nation is within normal limits. Mammography
ceive a pneumococcus vaccination. What is his revealed an irregular 2-cm lump corresponding
primary indication for receiving the vaccine? with the physical ndings. A breast biopsy is
performed, which shows ductal carcinoma in
(A) Age
situ. Which aspect of this patients history,
(B) Diabetes

FULL-LENGTH EXAMS
physical examination, and laboratory ndings
(C) Health care worker
is the most inuential in determining her prog-
(D) Hypertension
nosis?
(E) Pneumonia
(A) Age of menarche
26. A 36-year-old African-American woman pre- (B) Histology of the tumor
sents to the physicians ofce complaining of (C) Presence of fatigue and fever
increasing dyspnea on exertion and fatigue. (D) Shape of the tumor
She has also noticed that she has been urinat- (E) Size of the tumor
ing more frequently, even though she has not
been drinking more uids, including caffein-
ated beverages. She has never traveled outside
the United States. On review of systems she
also reports some blurry vision, night sweats,
muscle weakness, constipation, and a 2.3-kg (5-
lb) weight loss. X-ray of the chest reveals bilat-
eral hilar lymphadenopathy and increased in-
Test Block 4
582 Section II: Full-Length Examinations Questions

28. A 48-year-old man is rushed by ambulance to (A) Hospice


the emergency department after collapsing at (B) Intravesicular bacille Calmette-Gurin
home. According to his wife, his medical his- and systemic chemotherapy
tory is signicant for mild asthma and recently (C) Pelvic exenteration and systemic chemo-
diagnosed hypertension, for which he was pre- therapy
scribed 25 mg atenolol twice a day. His wife (D) Systemic chemotherapy
states that his blood pressure was elevated to- (E) Transurethral resection of the bladder tu-
day when he checked it at the supermarket and mor, pelvic radiation, and systemic chemo-
that he had taken several additional pills to at- therapy
tempt to control it. His heart rate is 45/min,
blood pressure is 95/60 mm Hg, and respira- 30. A 32-year-old woman with long-standing
tory rate is 20/min. His oxygen saturation is schizophrenia presents with muscle cramps,
98% on 2 L of oxygen. He is obtunded. An nausea, and polyuria. Her lungs are clear to
ECG reveals complete heart block. Atropine auscultation, and the physician does not nd
and isoproterenol are administered. What addi- any signs of edema. Skin turgor appears nor-
tional intervention should be considered at this mal. Laboratory tests show:
time? Na+: 129 mEq/L
(A) Amiodarone K+: 3.5 mEq/L
(B) Cardiac pacing Cl: 100 mEq/L
(C) Diphenhydramine Blood urea nitrogen: 15 mg/dL
(D) Glucagon Creatinine: 1.0 mg/dL
(E) Nebulized albuterol Glucose: 99 mg/dL
Urine osmolality: 105 mOsm/kg
29. A 60-year-old man presents to his primary care Urine Na+: 21 mmol/L
physician with the chief complaints of fatigue Which of the following is the most likely cause
and unintentional weight loss over the past few of her electrolyte abnormality?
months. He has noted blood in his urine, but
never brought it to a physicians attention be- (A) Adrenal insufciency
FULL-LENGTH EXAMS

cause it was never painful. Over the past 2 (B) Dehydration


weeks, he has also developed a nonproductive (C) Psychogenic polydipsia
cough. On physical examination the man is (D) Renal failure
cachectic with temporal wasting. He has a pal- (E) Syndrome of inappropriate ADH
pable, irregularly shaped mass that is located
just above the pubic symphysis to the left of the 31. A 32-year-old man with myasthenia gravis pre-
midline. Digital rectal examination reveals a sents to the emergency department complain-
normal prostate and guaiac-negative stool. Uri- ing of difculty breathing. On examination he
nalysis is positive for blood. CT scans of the has marked ptosis, is dyspneic, and is having
chest, abdomen, and pelvis show a large tumor difculty speaking and swallowing. Serial mea-
in the bladder and several suspicious-looking surements of the patients vital capacity are
pelvic and cervical lymph nodes. X-ray of the 700, 620, and 580 mL. X-ray of the chest re-
chest reveals four 1-cm lesions in the mans left veals a right middle lobe inltrate. Which of
lung. He undergoes a cystoscopy and a 4-cm the following is the best next step in manage-
mass is seen on the left superior aspect of the ment?
bladder. A biopsy reveals transitional cell carci- (A) Albuterol nebulizer treatments
noma. Which treatment is most appropriate for (B) High-dose edrophonium
this mans disease?
Test Block 4

(C) Immediate endotracheal intubation and


plasmapheresis
(D) Inhaled racemic epinephrine
(E) Intravenous antibiotics
Test Block 4 Questions 583

32. A 34-year-old man who lives in a homeless 33. A 2-month-old, full-term infant is brought to
shelter is frequently seen by the physician who his pediatrician because of a rash on his scalp.
volunteers at the shelters clinic. Over time he The rash has been present for several days and
builds a relationship with the physician, and does not seem to bother him. The mothers
the physician learns that he is homosexual, al- pregnancy and delivery were uncomplicated,
though this is not widely known. She suggests and the patient has been healthy and growing
that he be tested for HIV. When he returns to well at his neonatal visits. There are no sick
the clinic for results of the test (which came contacts or pets at home. On examination, the
back positive), she notices that he has several superior scalp is erythematous and slightly
crusted linear wounds on his upper arms and greasy, with a yellowish scale and plaque for-
chest. The patient tells her that he had a ght mation. Which of the following is the best
with his partner, who stabbed him several management for this patient?
times with a kitchen knife. He also says he will
(A) 1% Hydrocortisone cream
not tell his partner of his HIV status. In which
(B) Oral griseofulvin therapy
of the following circumstances will the physi-
(C) Selenium sulde shampoo
cian be required to override patient condenti-
(D) Topical emollients
ality?
(E) Topical ketoconazole cream
(A) If the physician suspects abuse, she must
inform the shelter 34. A 64-year-old man with a history of hyperten-
(B) Inform only the health department that sion and carcinoid tumor of the appendix pre-
the patient is HIV positive sents to the emergency department complain-
(C) Inform only the patients partner that the ing of palpitations, trouble breathing, and
patient is HIV positive weakness. It has been getting worse over the
(D) The physician must inform both the health past day, and this morning he nearly passed
department and the patients partner of his out. His blood pressure is 148/90 mm Hg,
HIV status pulse is 104/min, respiratory rate is 22/min,
(E) There is no reason to break patient con- and temperature is 37.1C (98.9F). Physical
dentiality examination reveals an S3 gallop, a prominent

FULL-LENGTH EXAMS
S4, jugular venous distension, ascites, bipedal
edema, a point of maximal impulse that is
shifted down and to the left, and hepatomeg-
aly. ECG shows QRS complexes that are low
voltage, and echocardiogram establishes that
left and right atria are enlarged with a thick
right ventricle. A pressure tracing shows a dip/
plateau sign. Which of the following is the
most likely diagnosis?
(A) Cerebrovascular accident
(B) Coarctation of the aorta
(C) Mitral stenosis
(D) Prinzmetals angina
(E) Restrictive cardiomyopathy
Test Block 4
584 Section II: Full-Length Examinations Questions

35. A 32-year-old G2P1 woman at 32 weeks gesta- (A) Avoiding the use of COX-selective agents
tion presents to the obstetrics and gynecology reduces this patients stroke risk
clinic for a follow-up visit. She complains of fa- (B) If a patient is exposed to a COX-selective
tigue, polyuria, and lower back pain. Her pre- agent, his or her risk of suffering a stroke is
vious pregnancy was without complications, increased by a factor of 1.2
and the child weighed 3.5 kg (7 lb 10 oz) at de- (C) Patients on COX-selective agents have 1.2
livery. The patient is in no acute distress. Physi- times as many strokes as patients not on
cal examination is notable for a fundal height such medications
of 33 cm. Extremities reveal some mild edema (D) Patients with rheumatoid arthritis have 1.2
of the feet. A glucose loading test at 26 weeks times as many strokes as patients without
gestation showed a blood glucose level of 145 the disease
mg/dL. Results of a subsequent glucose toler- (E) The odds that people who suffered a stroke
ance test are as follows: had previously used COX-2 inhibitors are
1.2 times greater than in people who did
Fasting: 105 mg/dL
not suffer a stroke
1 hour: 210 mg/dL
2 hours: 185 mg/dL
37. A 59-year-old man presents for evaluation prior
3 hours: 190 mg/dL
to placement of a pacemaker-debrillator. His
Results of other prenatal tests have been reas- past medical history is signicant for heart fail-
suring. She denies any contractions or loss of ure secondary to a myocardial infarction 15
uid. Ultrasound reveals a fetus that is large for years earlier, hypercholesterolemia, hypothy-
gestational age, with an estimated fetal weight roidism, and a basal cell carcinoma which was
of >4 kg (>8.8 lb). What is the most appropri- surgically excised 6 months ago. He currently
ate next step in management? takes atorvastatin, lisinopril, carvedilol, clopi-
dogrel, aspirin, levothyroxine, and a multivita-
(A) Amniocentesis
min. Laboratory tests show:
(B) Chorionic villus sampling
(C) Measurement of a cortisol level WBC count: 6000/mm
(D) Measurement of hemoglobin A1c level Hemoglobin: 12 g/dL
FULL-LENGTH EXAMS

(E) Measurement of random blood glucose Hematocrit: 38%


levels Platelet count: 200,000/mm
Thyroid-stimulating hormone: 12 U/L
36. A 48-year-old woman with rheumatoid arthritis
What should be done next to appropriately
presents to her rheumatologist for follow-up.
manage this patients thyroid disorder?
She was diagnosed 3 years ago, has been main-
tained on disease-modifying agents and non- (A) Decrease the patients levothyroxine dos-
steroidal anti-inammatory drugs, and has age, delay surgery by 35 days, and re-
been doing well. She has no complaints at check thyroid function then
present but is concerned about her current (B) Increase the patients levothyroxine dos-
anti-inammatory medication and recent me- age, delay surgery by 35 days, and re-
dia reports of stroke risk demonstrated with check thyroid function then
other drugs in the same class. Recent studies (C) Increase the patients levothyroxine dos-
examining the relationship between stroke and age, delay surgery by 46 weeks, and re-
use of cyclooxygenase-2 (COX-2) inhibitors in check thyroid function then
arthritis revealed an odds ratio of 1.2. What in- (D) Perform surgery as scheduled, decrease the
formation should the patient receive about patients levothyroxine dosage, and re-
these studies? check thyroid function in 46 weeks
Test Block 4

(E) Perform surgery as scheduled, increase the


patients levothyroxine dosage, and re-
check thyroid function in 46 weeks
Test Block 4 Questions 585

38. A 50-year-old woman recently diagnosed with (A) Extracorporeal shock wave lithotripsy
amyotrophic lateral sclerosis presents to the (B) Hydration and alkalinization of the urine
physician with new-onset drooling that is inter- (C) Low-sodium diet
fering with her ability to work. She is unmar- (D) Penicillamine
ried with no children and wishes to remain at (E) Thiazide diuretics
her job. Which of the following is the best
management? 40. A 21-year-old woman presents to the emer-
gency department because of severe abdominal
(A) Administer glycopyrrolate, an anticholin- pain for the past 6 hours. She is a college stu-
ergic, 0.10.2 mg intramuscularly two to dent and her boyfriend was recently diagnosed
three times a day with bacterial meningitis. After his diagnosis,
(B) Baclofen, a -aminobutyric acid analogue the patients boyfriend was treated with oral rif-
muscle relaxant, 510 mg two to three ampin, and she was given prophylactic rif-
times a day ampin as well. The patient has a history of
(C) Carbamazepine, an antiepileptic, 200 mg asthma, uses an albuterol inhaler and oral con-
twice daily traceptives, and has no allergies. On examina-
(D) Consult psychiatry to rule out depression tion the patient is in obvious discomfort, with
(E) Diphenhydramine, an antihistamine, 50 pain on palpation of the lower left quadrant.
100 mg at night Her serum -human chorionic gonadotropin
level is 954 mg/dL, and transvaginal ultrasound
39. A 45-year-old woman with a history of un-
reveals free uid in the cul-de-sac. Laparoscopy
treated gout presents to the emergency depart-
reveals a ruptured ectopic pregnancy. What is
ment complaining of severe ank pain for 1
the most likely reason for the failure of the pa-
day. CT of the abdomen is shown in the im-
tients oral contraceptive?
age. Which of the following is the best way to
treat this patients condition? (A) Albuterol use
(B) Competition for binding sites
(C) Inactivation by a cellular pump
(D) Patient error

FULL-LENGTH EXAMS
(E) Rifampin use

Reprinted, with permission, from Brunicardi FC, Ander-


sen DK, Billiar TR, Dunn DL, Hunter JG, Matthews JB,
Pollock RE, Schwartz SI. Schwartzs Principles of Surgery,
8th edition. New York: McGraw-Hill, 2005: Figure 39-7.
Test Block 4
586 Section II: Full-Length Examinations Questions

41. A 42-year-old man with recently diagnosed hy- 43. A 42-year-old man presents for his annual phys-
pertension presents to the emergency depart- ical examination. His blood pressure is 130/80
ment with ank pain of a few hours duration. mm Hg and pulse is 75/min. His heart rate is
His blood pressure is 152/92 mm Hg, heart rate regular with no murmurs, rubs, or gallops.
is 78/min, and temperature is 36.9C (98.4F). Lungs are clear to auscultation bilaterally and
He has a II/VI systolic murmur at the lower the abdomen is benign. No subclavicular
sternal border with a midsystolic click, positive lymphadenopathy is appreciated, but there is a
bowel sounds, bilateral palpable kidneys, and a xed palpable rm nodule midline that does
liver that is 10 cm in size. No spleen tip is pal- not shift with swallowing. A ne needle biopsy
pable. Urinalysis is positive for RBCs and is ordered, which is read as indeterminate for
WBCs, but negative for bacteria. Laboratory follicular neoplasm. Which of the following is
tests show: the best next step in management?
WBC count: 7200/mm (A) High-resolution ultrasonography
Hemoglobin: 14.2 g/dL (B) Observation with follow-up every 4 months
Platelet count: 250,000/mm (C) Radionucleotide scan
Na+: 145 mEq/L (D) Thyroxine therapy with close follow-up
Blood urea nitrogen: 20 mg/dL (E) Total thyroidectomy
Creatinine: 2.2 mg/dL
44. A 52-year-old man presents for an annual phys-
On further questioning the man admits to hav-
ical examination. He is generally in good
ing had several urinary tract infections over the
health but complains of recent difculty with
past few months and notes that his doctor told
urination. He reports that he has a weak, inter-
him that he had some sort of cardiac valve
rupted stream with dribbling. He denies noctu-
prolapse. Which of the following is the most
ria, urgency, bladder fullness, daytime fre-
likely diagnosis?
quency, hematuria, fevers, chills, or family
(A) Autosomal dominant polycystic kidney history of prostate cancer. On examination he
disease is afebrile and has no costovertebral angle ten-
(B) Autosomal recessive polycystic kidney derness. His abdomen is soft and nontender.
FULL-LENGTH EXAMS

disease Which of the following would be the rst ap-


(C) Medullary cystic disease propriate screening test?
(D) Medullary sponge kidney
(A) Cytoscopy
(E) Nephrolithiasis
(B) Digital rectal examination
(F) Pyelonephritis
(C) Prostate biopsy
(D) Prostate-specic antigen level
42. A 41-year-old obese woman presents to the
(E) Urinalysis and urine culture
emergency department with her three chil-
dren. She is complaining of right upper quad-
rant (RUQ) pain that is worse when she eats
(particularly when she eats fatty foods). On ex-
amination, she ceases inspiration on palpation
of her RUQ. The physician obtains an abdomi-
nal ultrasound, but the results are equivocal.
Which of the following is the most appropriate
next test?
(A) CT of the abdomen
Test Block 4

(B) Hepatic iminodiacetic acid scan


(C) MRI of the abdomen
(D) Sestamibi scan
(E) X-ray of the abdomen
Test Block 4 Questions 587

E X T E N D E D M ATC H I N G 46. A 17-year-old boy presents to the emergency


department complaining of recent onset vomit-
ing and diarrhea. Shortly after eating his home-
The response options for the next 2 items are packed lunch of ham and cheese sandwiches,
the same. Select one answer for each item chips, apple, and cookies, he developed severe
in the set. nausea and nonbilious, nonbloody emesis.
Within 2 hours he also developed nonbloody
For each of the following patients, select the most diarrhea. His heart rate is 88/min, blood pres-
likely etiologic agent. sure is 106/65 mm Hg, and respiratory rate is
(A) Bacillus cereus 14/min. Abdominal examination reveals no
(B) Campylobacter jejuni tenderness, distention, or masses. Fecal occult
(C) Clostridium difcile blood test results are negative.
(D) Enteric adenovirus
(E) Escherichia coli
(F) Giardia lamblia
(G) Salmonella enteritidis
(H) Shigella
(I) Staphylococcus aureus
(J) Vibrio cholerae
(K) Vibrio parahaemolyticus

45. A previously healthy 7-year-old girl is brought


to the emergency department because of pro-
longed nonbloody, watery diarrhea. The epi-
sodes began approximately 5 days ago, when
she simultaneously developed a low-grade fe-
ver to 38.2C (100.8F). Her travel history is
notable for a week-long camping trip she took

FULL-LENGTH EXAMS
with her father to Mexico approximately 10
days ago, where she went swimming. Her heart
rate is 88/min, blood pressure is 102/65 mm
Hg, and respiratory rate is 16/min. She appears
tired, but physical examination reveals no ab-
dominal distention, tenderness, or masses. Fe-
cal occult blood test results are negative.

Test Block 4
588 Section II: Full-Length Examinations Answers

AN S W E R S

1. The correct answer is D. The patient is suffer- rstborn children. Nonbilious, nonbloody, pro-
ing from primary hyperaldosteronism, and jectile vomiting after feeding develops 36
from the effects of elevated aldosterone: hyper- weeks after birth. Infants with pyloric stenosis
tension, hypokalemia, and metabolic alkalosis. are often described as hungry vomiters, given
Symptoms could be alleviated by reversing or their desire to feed immediately after episodes
blocking the effects of aldosterone. Aldosterone of emesis. As a result of the malnourishment
increases sodium reabsorption and increases and dehydration, infants with pyloric stenosis
potassium secretion and hydrogen ion secre- develop hypochloremic, hypokalemic, meta-
tion. Spironolactone antagonizes aldosterone bolic alkalosis. This metabolic acid-base disor-
at its receptor in the distal convoluted tubule der is caused by the massive loss of gastric hy-
and would be likely be benecial in this pa- drochloric acid. As the patient enters a state of
tient. volume depletion and loses hydrochloric acid,
the kidney responds to the metabolic alkalosis
Answer A is incorrect. Clonidine is a centrally
by excreting bicarbonate in the urine. This re-
acting -adrenergic agonist that might aid in
sults in obligate potassium and sodium loss. Hy-
treating the patients hypertension, but not the
pochloremia also induces the renin-angiotensin-
electrolyte imbalances caused by the patients
aldosterone axis, which induces increased
hyperaldosteronism. This class of antihyperten-
aldosterone production and maintains the hy-
sives also could potentially contribute to the
pokalemic alkalosis. In addition, the alkalotic
patients complaints of weakness and fatigue.
state induces hydrogen ions to move out of the
Answer B is incorrect. Hydrochlorothiazide intracellular space, where they are replaced by
is a commonly used diuretic that inhibits re- potassium ions moving into the cells, furthering
sorption of sodium and chloride at the distal hypokalemia.
convoluted tubule. It would be an appropriate
Answer A is incorrect. The loss of gastric hy-
choice if this patient were presenting with es-
drochloric acid results in a metabolic alkalosis.
sential hypertension, but it is inadequate in the
FULL-LENGTH EXAMS

Additionally, as the gastric acid is lost, chloride


context of hypertension secondary to hyperal-
ions are also lost, creating a hypochloremic
dosteronism.
and not a hyperchloremic state.
Answer C is incorrect. Metoprolol could re-
Answer B is incorrect. The loss of gastric hy-
solve the patients hypertension by antagoniz-
drochloric acid causes a hypochloremic and
ing -adrenergic receptors, but would not ad-
not a hyperchloremic state.
dress the electrolyte imbalances caused by the
patients hyperaldosteronism. Moreover, fa- Answer C is incorrect. Because of the mecha-
tigue is a common adverse effect of -blockers, nism by which potassium is lost in an obliga-
which could potentially exacerbate the pa- tory manner secondary to the loss of bicarbon-
tients complaint of fatigue. ate, the metabolic alkalosis that is produced in
pyloric stenosis is of a hypokalemic and not a
Answer E is incorrect. Selective serotonin ago-
hyperkalemic nature.
nists such as sumatriptan are rst-line agents
for treatment of acute migraine headaches. Answer D is incorrect. Loss of hydrochlo-
Though this patient does complain of head- ric acid from the episodes of emesis creates a
ache, the history is not suggestive of migraines, metabolic alkalosis and not an acidosis distur-
and this agent would not address the patients bance. The hypochloremic and hypokalemic
Test Block 4

underlying problem of hyperaldosteronism. descriptors are correct, however, and describe


the electrolyte imbalance that occurs second-
2. The correct answer is E. The patient has pylo- ary to pyloric stenosis.
ric stenosis. Pyloric stenosis has a multifaceted
etiology, including genetic and environmental 3. The correct answer is B. This patient has a
factors. It is more common in boys (46:1) and pelvic fracture with resultant hemodynamic in-
Test Block 4 Answers 589

stability. In such a patient, one must never ex- eral vision loss. It occurs in patients who are el-
plore a suspected pelvic or retroperitoneal he- derly and is often idiopathic. The examination
matoma. Rather, follow the patient with serial typically shows diffuse retinal hemorrhages in
hematocrit and hemoglobin measurements to all four quadrants with dilated, tortuous retinal
monitor for sufcient intravenous resuscitation. veins. There may also be cotton-wool spots,
This patient should not be subjected to major disc edema, and neovascularization of the disc
pelvic surgery at this point because these he- or retina.
matomas often are a collection of blood from
Answer D is incorrect. Diabetic retinopathy
many vessels, most of which are not amenable
usually presents with bilateral visual defects
to direct surgical control. The best chance to
that can lead to blindness because the retina
curb the patients bleeding is external xation.
is very sensitive to hyperglycemic effects, lead-
Answer A is incorrect. Emergency pelvic an- ing to loss of retinal pericytes and microvascu-
giography with embolization is a treatment op- lar endothelial cells and thickening of retinal
tion in the hemodynamically unstable patient. basement membrane. Funduscopic examina-
tion typically shows dot-and-blot hemorrhages
Answer C is incorrect. Prostate examina-
and microaneurysms that extend across the
tion may demonstrate a high-riding ballotable
horizontal raphe.
prostate or lack of a palpable prostate, both of
which are suggestive of urethral injury. Answer E is incorrect. Hypertensive retinopa-
thy usually presents with bilateral visual de-
Answer D is incorrect. Orthopedic consulta-
fects. Hemorrhages are not conned to a sector
tion is mandatory in this case.
of the retina and usually cross the horizontal
Answer E is incorrect. Due to the association raphe. There are two contributing pathologies
between pelvic and urethral injury, a retro- to this disease, including arteriolar thickening
grade urethrogram must be ordered before Fo- and acute vascular injury.
ley catheter placement is considered.
5. The correct answer is D. Ophthalmologic
Answer F is incorrect. Rapid volume resusci-
complications occur in roughly 20% of patients
tation is critical in the hemodynamically un-
with sarcoidosis, most commonly in the form

FULL-LENGTH EXAMS
stable patient with a pelvic fracture.
of keratoconjunctivitis or chorioconjunctivitis.
4. The correct answer is B. The differential diag- Answer A is incorrect. Causes of alopecia in-
nosis of painless unilateral vision loss can in- clude bullous disease, severe folliculitis, and
clude several possibilities such as central and chemical reactions, but it is not a symptom of
peripheral vein occlusion, diabetic retinopathy, sarcoidosis.
and hypertensive retinopathy. The patient re-
Answer B is incorrect. Ataxia and dysmetria
ports a history of hypertension and diabetes,
are not among the neurologic complications of
both of which are risk factors for branch retinal
sarcoidosis.
vein occlusion. Additionally, funduscopic ex-
amination shows supercial hemorrhages in a Answer C is incorrect. Although neurologic
sector of the retinal along a retinal vein. The involvement occurs in a small percentage of
hemorrhages usually do not cross the horizon- patients with sarcoidosis, it typically manifests
tal raphe (midline). as cranial nerve palsies, hydrocephalus, or
lymphocytic meningitis, and not as extremity
Answer A is incorrect. In acute ophthalmic
weakness.
artery occlusion, the entire retina typically ap-
pears whitened. Such disease occurs in the set- Answer E is incorrect. Koilonychia, or spoon-
Test Block 4

ting of carotid artery occlusive disease or other ing of the nails, is typically a sign of iron-
ischemic disease such as embolic disease and deciency anemia, and is not typically seen in
coagulopathy. sarcoidosis.
Answer C is incorrect. Central retinal vein oc-
6. The correct answer is E. The sudden onset of
clusion can also present with painless unilat-
this patients abdominal pain, the presence of
590 Section II: Full-Length Examinations Answers

guarding and rebound tenderness, and the fact an ECG should also be ordered to rule out
this patients last menstrual cycle was 2 weeks ischemic injury secondary to cocaine use,
ago, all suggest the possibility of a ruptured which may be masked by the abdominal pain.
corpus luteum cyst. Normally, a mature follicle
Answer A is incorrect. Pure -blockers are
transforms into a corpus luteum after ovula-
contraindicated in the setting of cocaine abuse
tion, only to atrophy into a corpus albicans
because unopposed a activity may increase hy-
around the time of the next ovulation. Occa-
pertension.
sionally, a corpus luteum will fail to degener-
ate and result in the formation of a cyst; when Answer B is incorrect. Pure -blockers are
these rupture, the patient experiences acute contraindicated in the setting of cocaine abuse
abdominal pain. It can be difcult to distin- because unopposed a activity may increase hy-
guish clinically between this pathology and ad- pertension.
nexal torsion, perforated bowel, or ectopic Answer D is incorrect. Metoprolol is another
pregnancy. pure -blocker that is contraindicated in the
Answer A is incorrect. Acute appendicitis is setting of cocaine abuse.
a possibility; however, the pain of appendici- Answer E is incorrect. Verapamil is a calcium
tis is usually a dull, vague pain originating at channel blocker that may be used as an anti-
the umbilicus and migrating to the RLQ. The arrhythmic or antihypertensive. Verapamil will
rapid onset and sharp nature of this patients not treat the underlying cause of this patients
pain is more suggestive of a ruptured cyst. hypertension and will not prevent additional
Answer B is incorrect. Acute gastritis describes withdrawal complications.
an inammation of the stomach lining. The
onset of gastritis is unlikely to occur over half 8. The correct answer is E. This patient is expe-
an hour, and would not localize to the left riencing acute tumor lysis syndrome. This situ-
lower quadrant. ation occurs when large numbers of rapidly
proliferating cells (as are present in acute lym-
Answer C is incorrect. Ectopic pregnancy is phoid leukemia) are exposed to chemothera-
effectively ruled out by a negative -human peutic agents. Cellular destruction results in
FULL-LENGTH EXAMS

chorionic gonadotropin test and normal pelvic release of intracellular contents into the blood-
examination. stream, leading to hyperuricemia and hyper-
Answer D is incorrect. Mesenteric ischemia kalemia. Uric acid precipitates in the renal tu-
may be either acute or chronic. The chronic bules, causing oliguric/anuric acute renal
variety leads to pain associated with meals, is failure, and cardiac arrhythmias may occur due
linked to vascular comorbidities, and would to high potassium levels. This syndrome is
not likely be a consideration in a patient of this largely preventable with administration of al-
age group. The pain of acute mesenteric ische- lopurinol, aggressive hydration, and close mon-
mia may be sudden in onset, but it is visceral itoring of electrolytes; however, patients with
in nature and likely to be poorly localized. acute lymphoid leukemia who have very high
Like chronic mesenteric ischemia, it generally WBC values at diagnosis, as well as patients
affects patients >60 years old. with Burkitts lymphoma, remain at risk.
Answer A is incorrect. Doxorubicin is a che-
7. The correct answer is C. Given this patients
motherapeutic agent that is known to cause
history, his persistent hypertension is likely to
long-term cardiac damage. This damage does
be a withdrawal symptom. Although not ordi-
not occur in the acute setting, however.
narily used as antihypertensives, benzodiaz-
Test Block 4

epines are indicated in the supportive treat- Answer B is incorrect. Incorrect dosing of an-
ment of cocaine/amphetamine and alcohol esthesia might cause somnolence or hyperac-
withdrawal, and will treat the underlying cause tivity, but would not be expected to cause the
of this patients hypertension. In this instance, reported electrolyte and urinary abnormalities.
Test Block 4 Answers 591

Answer C is incorrect. Leukemic inltrate of 10. The correct answer is A. Although this patient
the kidneys is extremely rare, and would not be suffers from a terminal condition and has indi-
expected to cause such acute renal failure. cated through his DNR order that resuscitation
should not be performed if he suffers from car-
Answer D is incorrect. Overhydration would
diac or respiratory arrest, the advance directive
not be expected to cause these symptoms in a
does not apply to action against treatable con-
patient with well-functioning kidneys.
ditions. A living will in the form of a DNR or-
der is an advance directive provided by the pa-
9. The correct answer is E. The treatment algo-
tient that indicates the desire to prevent
rithm for variceal hemorrhage includes soma-
life-sustaining treatment in the event of a ter-
tostatin therapy or an analog and endoscopic
minal disease or a persistent vegetative state.
therapy, including band ligation and sclero-
However, feeding, intravenous uids, treat-
therapy. If this is unsuccessful or bleeding re-
ment of infections or other treatable condi-
curs continually, then balloon tamponade or
tions, and pain management are not restricted
portal systemic shunt placement should be
under a DNR order.
considered. Although portal systemic shunt
placement is an excellent means to decom- Answer B is incorrect. A durable power of at-
press the portal circulation, the main negative torney is a legally designated health care proxy
consequence is due to the diversion of blood who serves as a decision maker if the patient
away from the hepatic ltering mechanism. In lacks decision-making capacity. Although the
other words, toxin-lled blood circumvents the health care proxy can make decisions for a pa-
liver and goes directly to the systemic circula- tient if his or her wishes are unclear, any ad-
tion, thus putting the patient at risk for hepatic vance directive created by the patient when he
encephalopathy. This patient, a recurrent or she had capacity would supersede the deci-
bleeder, likely underwent shunt placement sions made by the surrogate.
and is now suffering from encephalopathy.
Answer C is incorrect. A DNR order does not
Answer A is incorrect. -Blocker therapy is address adverse effects of appropriate treatment
used to medically treat portal hypertension due measures, only resuscitation efforts in the case
to its effect on the systemic and splanchnic cir- of cardiac or respiratory arrest.

FULL-LENGTH EXAMS
culations. This therapy is a commonly used as
Answer D is incorrect. Although the health
a preventative measure, but not in the case of
care proxy can make decisions for a patient if
acute bleeding, and it does not lead to hepatic
his or her wishes are unclear, any advance di-
encephalopathy.
rective created by the patient when he or she
Answer B is incorrect. Balloon tamponade had capacity would supersede the decisions
would be an appropriate measure for bleed- made by the surrogate.
ing varices that have not been controlled with
Answer E is incorrect. Although this patient
other measures. However, this would not push
suffers from a terminal condition, his DNR or-
the patient into hepatic encephalopathy.
der indicates only that resuscitation should not
Answer C is incorrect. Endoscopic band liga- be performed. The advance directive does not
tion is commonly performed simultaneously apply to action against treatable conditions.
with sclerotherapy.
11. The correct answer is C. In any child present-
Answer D is incorrect. Endoscopic sclerother-
ing with recurrent pneumonia and poor growth,
apy is an excellent intervention both to treat
cystic brosis should be considered. Since the
ongoing variceal bleeding and as a secondary
pathophysiology results in decreased bacterial
preventative measure for individuals with var-
Test Block 4

clearance from the lungs secondary to impaired


ices at risk of rebleeding. However, sclero-
ciliary function, these patients are susceptible to
therapy does not put patients at risk for hepatic
a range of pulmonary infections including
encephalopathy and therefore is likely not the
Staphylococcus aureus, Burkholderia cepacia,
intervention used in this case.
Stenotrophomonas maltophilia, and Haemophi-
592 Section II: Full-Length Examinations Answers

lus inuenzae. However, Pseudomonas infection Answer C is incorrect. Pancreatic cancer is a


in patients with typical histories for cystic bro- chronic process that does not typically present
sis virtually assures the diagnosis in the absence with acute onset tearing back pain and nor-
of conrmatory sweat chloride or genetic test- mal laboratory values. Pancreatic cancer in the
ing. head of the pancreas is often associated with
painless jaundice, weight loss, pruritus, acholic
Answer A is incorrect. Klebsiella pneumonia
stools, dark urine, and abdominal pain. This
occurs in the elderly, the hospitalized, and in
patients history, physical examination, labo-
the setting of chronic alcoholism, usually sec-
ratory ndings, and radiographic ndings are
ondary to aspiration.
more suggestive of an aortic dissection than
Answer B is incorrect. Listeria monocytogenes pancreatic cancer.
is a gram-positive rod that causes pneumonia
Answer D is incorrect. Perforated peptic ulcer
in the very young and the elderly due to their
typically presents with sudden onset abdomi-
relatively immunocompromised state, but
nal pain; however, it is not associated with de-
would not contribute to recurrent pneumonia
creased pulses in the lower extremities bilater-
or poor growth.
ally. Although patients with perforated peptic
Answer D is incorrect. Staphylococcal infec- ulcers may present with back pain, the history,
tions occur commonly in cystic brosis pa- laboratory ndings, physical examination, and
tients, but can also be seen in other immuno- radiographic ndings are more supportive of a
compromised states. Staphylococcus aureus diagnosis of aortic dissection than of perforated
appears as clustered gram-positive cocci. peptic ulcer.
Answer E is incorrect. Abdominal aortic an-
12. The correct answer is A. An aortic dissection
eurysms (AAAs) may be asymptomatic or may
occurs when there is a tear in the aortic intima
present with back or abdominal pain. Risk fac-
that forms a false lumen for blood to ow
tors for AAA include hypertension, atheroscle-
through. Patients typically present with acute
rosis, male gender, advanced age, and smok-
onset chest pain and/or back pain that is char-
ing. A ruptured AAA typically presents with a
acteristically tearing in nature. The pain may
triad of abdominal pain, hypotension, and a
FULL-LENGTH EXAMS

radiate as the dissection enlarges. Risk factors


pulsatile abdominal mass. Because this patient
for aortic dissection include hypertension, as
is hypertensive and does not show any indica-
seen in this patient who may not be compliant
tion of an AAA or rupture (retroperitoneal he-
with his medications. Although the gold stan-
matoma), this diagnosis is less likely than a di-
dard for diagnosis of aortic dissection is an an-
agnosis of aortic dissection.
giogram, CT with intravenous contrast, MRI,
or transesophageal echocardiography may show
13. The correct answer is B. Decreased cerulo-
a pseudolumen that aids in the diagnosis. In
plasmin, along with Kayser-Fleischer rings and
the image the pseudolumen can be seen in the
neurologic abnormalities, is characteristic of
descending portion of the aortic arch.
Wilsons disease. Patients may also present with
Answer B is incorrect. Lumbar disc herniation jaundice, hematemesis, depression, dysphagia,
involves extrusion of the intervertebral disc and speech impairment, and abdominal distention
compression of the nerve roots with resultant with hepatomegaly and/or splenomegaly.
lower back and lower extremity pain. These pa-
Answer A is incorrect. Elevated serum iron,
tients may have decreased lower extremity re-
elevated percentage saturation of iron, and el-
exes and weakness of their lower extremities.
evated ferritin with decreased serum transferrin
However, a diagnosis of lumbar disc herniation
are characteristic of hemochromatosis. Symp-
Test Block 4

is not likely in this patient because it would not


toms usually do not appear until patients are 40
explain the diminished pulses in the lower ex-
years old and may include joint pain (the most
tremities. Aortic dissection is more likely than
common complaint), bronze skin, fatigue, loss
lumbar disc herniation.
of sex drive, diabetes, heart problems, abdomi-
Test Block 4 Answers 593

nal pain, and jaundice. Corneal deposits are tipsychotic medications. As this patient is not
unlikely as is rigidity with a tremor. currently taking any antipsychotics, prophylaxis
against such symptoms is not indicated.
Answer C is incorrect. Abdominal CT would
be appropriate for diagnosing structural liver Answer D is incorrect. Phenelzine is a mono-
abnormalities such as hepatocellular carci- amine oxidase inhibitor that can cause a hyper-
noma and sclerosing cholangitis. Both of these tensive crisis in combination with sympathomi-
entities are unlikely to present with corneal de- metic amines (found in some decongestants)
posits, but may present with corneal injection and in tyramine-containing foods (e.g., aged
and jaundice. Both may also present with hep- wines and cheese). Hypertensive crises can be
atomegaly. Tremor and rigid extremities are fatal; therefore the potential for lethal interac-
uncharacteristic. tions is reason enough to avoid monoamine
oxidase inhibitors in this patient.
Answer D is incorrect. Hepatitis screening
panel will determine whether the patient has Answer E is incorrect. Risperidone is an atypi-
been infected with any of the hepatitis viruses, cal antipsychotic used to treat impulsivity, la-
but will not determine any metabolic abnor- bility, agitation, and both the positive and neg-
malities in the liver. Hepatitis may present with ative symptoms of schizophrenia. This patient
jaundice and hepatomegaly, but not corneal does not have any of these, nor does he have
deposits and a tremor. any impairment in his reality testing; thus an
antipsychotic is not indicated.
Answer E is incorrect. The patient does not
have evidence of ascites, and paracentesis is in-
15. The correct answer is C. Ehrlichia chaffeensis
appropriate.
is an intracellular bacteria that causes human
monocytic ehrlichiosis, carried by the Ixodes
14. The correct answer is C. This man is suffering
tick. The disease can present with fever, in ad-
from a major depressive episode, characterized
dition to nonspecic symptoms such as mal-
by weight loss, anhedonia, tearfulness, and psy-
aise, myalgias, headache, and chills, in addi-
chomotor retardation. Feelings of inadequacy
tion to nausea and vomiting. Mental status
and excessive guilt pervade his thoughts. Anti-
changes, such as the confusion exhibited by

FULL-LENGTH EXAMS
depressant medication is indicated. In addition,
this patient, are common. Laboratory abnor-
his extensive suicidal ideation put him at in-
malities include leukopenia, thrombocytope-
creased for suicide. Citalopram is a selective
nia, and mildly elevated transaminases. Exami-
serotonin reuptake inhibitor; these drugs have
nation of the buffy coat of a peripheral blood
minimal adverse effects in comparison to other
smear can sometimes demonstrate morulae in
antidepressants, such as tricyclic antidepres-
leukocytes but is rarely seen. Diagnosis can de-
sants and monoamine oxidase inhibitors, both
nitively be made on serologies; detection of a
of which can be lethal in overdose or with cer-
four-fold increase in E. chaffeensis antibody ti-
tain foods or other medications. Other exam-
ter (to at least 1:64) by indirect immunouores-
ples of commonly used selective serotonin
cence in paired serum samples obtained ap-
reuptake inhibitors include uoxetine, sertra-
proximately 3 weeks apart are sufcient for
line, and escitalopram.
diagnosis. Treatment is with doxycycline.
Answer A is incorrect. Amitriptyline is a tricy-
Answer A is incorrect. Babesiosis is a proto-
clic antidepressant that can cause signicant
zoan disease that is transmitted by ticks, with
anticholinergic adverse effects, orthostatic hy-
most cases found in the same geographic dis-
pertension, and third-degree heart block, es-
tribution as Lyme disease. The clinical pre-
pecially in patients with second-degree heart
sentation can be similar to that of ehrlichiosis,
Test Block 4

block. These drugs can be lethal in overdose.


but Babesia microti is an intraerythrocytic, not
Answer B is incorrect. Benztropine is an an- intraneutrophilic, parasite. It is usually treated
ticholinergic that is used in psychiatry as pro- with a combination of quinine sulfate and clin-
phylaxis for extrapyramidal symptoms to an- damycin.
594 Section II: Full-Length Examinations Answers

Answer B is incorrect. Borrelia burgdorferi is known as CREST syndrome, which is charac-


the spirochete that causes Lyme disease, and terized by Calcinosis, Raynauds phenomenon,
like Ehrlichia is carried in the Ixodes tick. It Esophageal dysmotility, Sclerodactyly, and
can present with many of the same symptoms Telangiectasias. This womans presentation is
as ehrlichiosis; however, in this case, the pres- not consistent with CREST syndrome, so she
ence of intraleukocyte morulae is much more would not be expected to have anticentromere
indicative of ehrlichiosis. Note, however, that antibodies.
both Ehrlichia chaffeensis and Borrelia burg-
Answer C is incorrect. Anti-Jo-1 is an antibody
dorferi can be present in the same Ixodes tick
to the tRNA synthetase that is seen in polymyo-
and therefore co-infect.
sitis and dermatomyositis. These patients tend
Answer D is incorrect. Acute HIV infection to have interstitial lung disease, mechanics
can be associated with a u-like syndrome hands (dry, cracked skin), Raynauds phenom-
during which the antibody test is negative but enon, and arthritis, and they tend to have treat-
the viral load is extremely high. HIV can also ment-refractory disease.
cause leukopenia due to decreases in CD4+
Answer D is incorrect. Anti-Scl-70 is an anti-
T lymphocytes, but no intracellular bodies are
body to DNA topoisomerase I that is seen in
visible on peripheral blood smear.
diffuse scleroderma. This patients presenta-
Answer E is incorrect. Leishmania donovani, tion is not consistent with scleroderma, which
a parasite found in Bangladesh, northeastern is characterized by thickening and tightening
India, Nepal, Sudan, and Brazil, that typically of the skin of the ngers and the skin proximal
causes visceral leishmaniasis (also known as to the wrists and ankles. Other minor criteria
kala-azar), and has a wide range of symptoms for the diagnosis of scleroderma include sclero-
that may include splenomegaly, peripheral dactyly (skin tightening limited only to the n-
lymphadenopathy, pancytopenia, hypergam- gers), digital pitting scars resulting from ische-
maglobulinemia, and hypoalbuminemia. The mia, and bibasilar pulmonary brosis.
infection can be diagnosed with demonstration
Answer E is incorrect. La (anti-SSB) antibod-
of the parasite in slides or in cultures of tissue
ies are seen in approximately 10%15% of pa-
aspirate or biopsy. This patients presentation is
FULL-LENGTH EXAMS

tients with SLE, but their levels do not corre-


not consistent with that of leishmaniasis.
late with disease activity.
Answer F is incorrect. Rickettsia rickettsii is
Answer F is incorrect. Ro (anti-SSA) antibod-
the pathogen that causes Rocky Mountain
ies are seen in approximately 30%45% of
spotted fever. It would not present with neuro-
patients with SLE, but their levels do not cor-
logic symptoms.
relate with disease activity. The presence of
Answer G is incorrect. Trypanosoma brucei is Ro antibodies is associated with dry eyes and
the agent of African sleeping sickness, which is mouth, subacute cutaneous lupus erythemato-
transmitted to humans by the tsetse y. Its diag- sus, neonatal lupus, and photosensitivity.
nosis requires detection of the parasite in blood
or lymph node aspirate. 17. The correct answer is C. In patients suspected
of perforated appendix, percutaneous drainage
16. The correct answer is B. This patient most with interval appendectomy may be an appro-
likely has systemic lupus erythematosus (SLE) priate alternative if the perforation has led to a
because she ts 5 of the 11 SLE criteria: pho- well-loculated abscess and the patient is not
tosensitivity, malar rash, arthralgias, renal in- septic. Given the complex history of this pa-
volvement, and oral lesions. Although several tient, if the appendix is indeed perforated and
Test Block 4

of the antibodies listed might be seen in this an abscess has formed, then percutaneous
patient, only anti-double-stranded DNA corre- drainage can be a viable alternative to perform-
lates with glomerulonephritis in SLE. ing emergency appendectomy.
Answer A is incorrect. Anticentromere anti- Answer A is incorrect. While hypotension and
bodies are seen in limited scleroderma, also fever can be the presenting symptoms for sep-
Test Block 4 Answers 595

sis, this patient has symptoms of an acute abdo- thrombocytopenia and/or evidence of spon-
men, which can only be diagnosed via radio- taneous bleeding may warrant transfusion
logic evaluation. of platelets. Packed RBCs lack platelets and
therefore have no use when managing a pa-
Answer B is incorrect. MRI is appropriate for
tient with ITP.
patients with contraindications for CT scan,
such as metal implants or allergy to intrave- Answer F is incorrect. There is no indication
nous contrast. The patient in this question does for platelet transfusion. Platelet counts are
not have such contraindications and a CT scan high enough to inhibit spontaneous hemor-
with contrast provides the most informative rhage. In addition, this patient has no evidence
evaluation of the abdominal symptoms. of serious bleeding and therefore would not be
a candidate for a platelet transfusion. Finally,
Answer D is incorrect. Abdominal ultrasound
transfused platelets are as rapidly destroyed as
is the standard diagnostic tool for acute chole-
endogenous platelets in ITP and would not
cystitis, not for appendicitis.
improve her platelet count.
Answer E is incorrect. This patient has no uri-
nary symptoms such as dysuria or hematuria, 19. The correct answer is E. This case centers on
and her symptoms are more indicative of a gas- the concept of medical futility. Treatment is fu-
trointestinal ailment. tile when there is no pathophysiologic ratio-
nale for treatment, when a maximal interven-
18. The correct answer is A. Postinfectious idio- tion is currently failing, when a given
pathic thrombocytopenic purpura (ITP) is a intervention has already failed, or when a par-
self-limited disease process; up to 80% of cases ticular treatment would not achieve the goals
will resolve within 6 months. Because this pa- of care. If these criteria are met, the physician
tient has no clinical evidence of spontaneous is neither ethically nor legally required to pro-
bleeding or symptoms due to her low platelets, vide treatment, even if requested by a health
no additional medical procedures are war- care proxy or other family member. In this
ranted. If the patient developed spontaneous case, the woman has advanced metastatic ovar-
bleeding as a result of the ITP, further mea- ian cancer and most recently failed the treat-

FULL-LENGTH EXAMS
sures to control the symptoms would be indi- ment being requested.
cated. Spontaneous bleeding is not likely to oc-
Answer A is incorrect. If a treatment has been
cur with platelet counts >20,000/mm, and
shown to be medically futile, the physician is
major spontaneous bleeding is not likely to oc-
neither ethically nor legally required to provide
cur with platelet counts >10,000/mm.
treatment, even if requested by a health care
Answer B is incorrect. Postinfectious ITP is a proxy or other family member.
self-limited disease process that resolves with-
Answer B is incorrect. This woman has re-
out additional therapy. Steroids may be useful
cently failed the treatment being requested. It
in severe cases of ITP but would not be indi-
is therefore medically futile to continue, and
cated for this patient because she is asymptom-
the physician is neither ethically nor legally re-
atic.
quired to provide treatment.
Answer C is incorrect. Splenectomy is an op-
Answer C is incorrect. If a treatment has been
tion for treating refractory ITP that does not re-
shown to be medically futile, the physician is
solve with conservative therapy or steroids.
neither ethically nor legally required to provide
Answer D is incorrect. Fresh frozen plasma re- treatment, even if requested by a health care
places clotting factors but lacks RBCs, WBCs, proxy or other family member.
Test Block 4

or platelets. It has no utility in the treatment of


Answer D is incorrect. If a treatment has been
ITP.
shown to be medically futile, the physician is
Answer E is incorrect. Packed RBCs do not neither ethically nor legally required to provide
contain platelets or clotting factors. Although treatment, even if requested by a health care
postinfectious ITP is self-limited, symptomatic proxy or other family member.
596 Section II: Full-Length Examinations Answers

20. The correct answer is A. The child in question rooxacin, have been shown to benet patients
has the classic triad of Wests syndrome: infantile with CD, especially in conjunction with im-
spasms, regression of psychomotor development, munosuppression and the rst-line therapy of
and hypsarrhythmia on electroencephalography. mesalamine products.
This disease typically progresses through three
Answer B is incorrect. Although the RLQ pain
stages, beginning with mild and infrequent
and fever raise concerns for appendicitis, this
spasms that become more frequent and severe
patients history and physical ndings suggest
during the second stage before resolving during
inammatory bowel disease.
the nal stage. They are often followed by the ap-
pearance of other seizure disorders and/or neuro- Answer C is incorrect. A proctocolectomy
logic impairment. would be curative for ulcerative colitis, but it
is only indicated for specic complications of
Answer B is incorrect. Febrile seizures in in-
CD in severe, medically refractory disease, or
fants can occur secondary to disseminated
for cancer prophylaxis in selected high-risk pa-
or localized infectious agents of virtually any
tients.
form, some of which are treatable. However,
the child in question has infantile spasms, Answer D is incorrect. Toxic megacolon is
which do not have an infectious etiology. sometimes seen in patients with ulcerative coli-
tis but rarely in those with CD.
Answer C is incorrect. Many benign condi-
tions (e.g., exaggerated Moro reexes) or ste- Answer E is incorrect. A gluten-free diet is in-
reotyped movements can be mistaken for sei- dicated for patients with celiac sprue. Although
zures in children and infants. Unfortunately, patients with celiac disease may present with
the child in question has an abnormal electro- diarrhea and weight loss, the results of colonos-
encephalogram and psychomotor delay, which copy and colonic biopsy in this patient are in-
is indicative of more serious pathology. dicative of CD.
Answer D is incorrect. Upper respiratory tract 22. The correct answer is C. Although the patient
infections are the most common pathology in has consented to earlier transfusions, she has
children and can even result in fever and fe- the right to change her mind at any time.
FULL-LENGTH EXAMS

brile seizures. A very sick 3-month-old child Therefore the most appropriate next step is to
may even appear to have psychomotor delay, withhold further transfusions and discuss the
since there are so few milestones to observe alternative plans with the care team.
at this age. However, the electroencephalo-
graphic changes seen in this child would not Answer A is incorrect. The patient does not
be expected for a febrile seizure. show any signs or symptoms of depression and/
or suicidality. While psychiatry consultation
Answer E is incorrect. Certain patterns of pa- may be appropriate as part of the patients care
thology associated with child abuse (e.g., reti- plan, the patients condition requires urgent
nal hemorrhages and shaken baby syndrome) transfusions and health care decisions need to
should prompt immediate notication of child be made immediately. Gynecological consul-
protective services. This childs symptoms to tation takes precedence.
not readily conform to any well-documented
pattern of abuse. Answer B is incorrect. The patient appears
competent and is aware of the consequences
21. The correct answer is A. This patient has of not receiving blood products. Therefore the
Crohns disease (CD). Among the most com- ethics committee cannot override the patients
mon symptoms of CD are abdominal pain, of- decision despite probable poor outcome.
Test Block 4

ten in the RLQ, and diarrhea. Rectal bleeding,


Answer D is incorrect. The patient has the
weight loss, and fever may also occur. The seg-
right to change her mind despite earlier in-
mental distribution of the lesions and cob- formed consent.
blestoning of the mucosa in this patient indi-
cate that this is CD and not ulcerative colitis. Answer E is incorrect. The patient is not
Antibiotics, including metronidazole and cip- deemed incapacitated, and so her husband
Test Block 4 Answers 597

cannot serve as surrogate and make any health 24. The correct answer is D. Although uncom-
care decisions on her behalf. mon, the HIV-positive patient can initially
present with full-blown AIDS, which is dened
23. The correct answer is A. Acute myelogenous as a CD4+ cell count <200/mm and/or the
leukemia is the clonal proliferation of blasts presence of an AIDS-dening illness. This pa-
that are unable to differentiate into mature my- tients presentation and symptoms are consis-
eloid cells. The median age at presentation is tent with progressive multifocal leukoenceph-
65 years and this form accounts for >80% of alopathy, which is caused by the JC virus.
leukemias in adults. Common symptoms at Polymerase chain reaction assays of cerebrospi-
presentation are those resulting from bone nal uid for the JC virus are 80% sensitive and
marrow failure, including fatigue, dizziness, or 95% specic, but no other CSF changes are
dyspnea on exertion. Infection is common due notable. Median survival is approximately 6
to low levels of mature neutrophils, and there months after diagnosis.
are often signs of organ inltration by blasts,
Answer A is incorrect. Demyelination of the
usually the spleen, liver, and gums. Complete
central nervous system is the mechanism that
blood cell count reveals a greatly elevated
underlies multiple sclerosis. The characteris-
WBC count and thrombocytopenia, while the
tic presentation of multiple sclerosis includes
peripheral smear shows a predominance of im-
many neurologic complaints separated in space
mature myeloid cells, which stain positively
and time that cannot be explained by a single
with myeloperoxidase.
lesion. An MRI demonstrates multiple periven-
Answer B is incorrect. Anemia of chronic dis- tricular lesions in the white matter. Given this
ease is often associated with renal failure and patients immunodeciency, however, an infec-
results from decreased erythropoietin produc- tious cause is much more likely.
tion. Anemia develops from the lack of stimu-
Answer B is incorrect. Infection with Candida
lus driving red blood cell maturation. The
albicans can cause esophagitis, oral thrush,
peripheral smear usually is normocytic with
and vaginitis but generally does not affect the
occasional presence of burr cells.
central nervous system. Treatment is with 13
Answer C is incorrect. Chronic myelogenous weeks of antifungal therapy.

FULL-LENGTH EXAMS
leukemia (CML) also involves the myeloid lin-
Answer C is incorrect. Infection with herpes
eage of cells, but in CML they retain the abil-
simplex virus type 1 (HSV-1) can cause HSV
ity to differentiate. As a result, peripheral smear
encephalitis, which typically presents with
should show an abundance of myeloid cells in
headache, fever, and nuchal rigidity as well as
all stages of differentiation, including blasts.
with mental status changes. Focal neurologic
Most patients present in the chronic phase of
signs and seizures may also be present. Lum-
the disease. Cytology in the vast majority of
bar puncture demonstrates mononuclear cells
cases shows the presence of the Philadelphia
and RBCs. The suspicion of HSV encephalitis
chromosome.
requires prompt therapy with intravenous acy-
Answer D is incorrect. Multiple myeloma can clovir. This patients presentation is not char-
produce anemia and recurrent infections due acteristic of HSV encephalitis.
to bone marrow failure. However, renal disease
Answer E is incorrect. Infection with Toxo-
is much more common, and bone pain due to
plasma gondii results in encephalitis that can
increased osteoclast activity and lytic lesions is
present with focal neurologic decits. MRI of
seen.
the brain will demonstrate ring-enhancing le-
Answer E is incorrect. Thrombotic throm- sions. Treatment consists of pyrimethamine,
Test Block 4

bocytopenia purpura is characterized by mi- leucovorin, and sulfadiazine, to which most pa-
croangiopathic hemolytic anemia and throm- tients respond within 1 week. More than 90%
bocytopenia. Peripheral smear should show of patients will test positive for anti-Toxoplasma
schistocytes without increased blasts. gondii IgG.
598 Section II: Full-Length Examinations Answers

25. The correct answer is B. Diabetes is an indica- and weight loss, it should not cause lymphade-
tion for the vaccine given that diabetic patients nopathy or increased interstitial markings.
are considered to be immunocompromised.
Answer C is incorrect. Anemia can be associ-
Diabetics, especially poorly controlled diabetic
ated with fatigue and dyspnea on exertion, but
patients, are vulnerable to infection. The
this patient has many other symptoms.
chronic hyperglycemia of diabetes leads to ab-
normalities in cell-mediated immunity and Answer D is incorrect. Diabetes insipidus
phagocyte function. Also, the vascular impair- can cause polyuria, fatigue, and weight loss; it
ment of diabetes prevents the immune system should not cause lymphadenopathy, blurry vi-
from responding appropriately to an infectious sion, or increased interstitial markings.
challenge. Answer E is incorrect. Lymphocytosis can be
Answer A is incorrect. Age >65 years old is an due to either infection or malignancy. Patients
indication for the pneumococcal vaccine, as with tuberculosis can have a variety of presen-
the elderly and very young are especially sus- tations; however, the patient does not report
ceptible to developing pneumococcal pneu- risk factors for tuberculosis (e.g., international
monia. travel, health care profession), making this di-
agnosis much less likely. Leukemia would not
Answer C is incorrect. Health care workers are
present with interstitial lung disease.
recommended to receive pneumococcal vac-
cine due to their exposure in the work setting. 27. The correct answer is E. The staging of breast
However, for this patient, his immunocom- cancer is predicated largely upon the size of
promised state secondary to diabetes is a more the tumor and whether there is nodal involve-
pressing reason to administer the vaccine. ment. Stage is the critical factor in prognosis
Answer D is incorrect. Hypertension is not it- for breast cancer.
self an indication for the vaccine although sig- Answer A is incorrect. Early age of menarche
nicant heart disease would indicate that the increased the risk of developing breast cancer
patient should receive the vaccine. in this patient, but it does not change the prog-
Answer E is incorrect. Current respiratory in- nosis.
FULL-LENGTH EXAMS

fection is a contraindication for the vaccine, Answer B is incorrect. The histology of the tu-
which is the case prior to the administration mor does not contribute as much to the prog-
of any vaccine. The patient also has a history nosis of breast cancer as does tumor size. The
of one episode of community-acquired pneu- diagnosis of lobular carcinoma in situ does,
monia that was treated as an outpatient. For however, increase the probability of invasive
patients with this history but no further im- carcinoma of both breasts.
munocompromised state, such as diabetes,
the pneumococcal vaccine would not be indi- Answer C is incorrect. Presence of fatigue and
cated. fever are not involved in the staging of breast
carcinoma.
26. The correct answer is A. This patient has sar- Answer D is incorrect. While irregular masses
coidosis. X-ray of chest shows bilateral hilar on mammography are more likely to be malig-
lymphadenopathy and increased interstitial nant, shape does not contribute to formal stag-
markings, which t criteria for stage II sarcoi- ing and prognosis.
dosis. Activated sarcoid macrophages secrete
calcitonin, which can lead to hypercalcemia. 28. The correct answer is D. The patient is suffer-
Evidence of hypercalcemia in this patient in- ing from bradycardia and hypotension second-
Test Block 4

cludes fatigue, muscle weakness, constipation, ary to an overdose of -blockers. Glucagon is


and polyuria. an appropriate treatment. Glucagon receptor-
Answer B is incorrect. Although diabetes mel- mediated effects on many tissues are similar to
litus can cause polyuria, fatigue, blurry vision,
Test Block 4 Answers 599

the effects of -adrenergic stimulation and in- tion that it would be warranted in some pa-
clude increased intracellular levels of cAMP, tients, but given the extent of spread, this man
leading to relaxation of bronchial, vascular, would probably not stand to benet.
and gastrointestinal smooth muscle, as well as
Answer A is incorrect. Supportive or palliative
increased calcium levels in cardiac muscle,
care is indeed warranted in some cases if the
which increases contractility. Isoproterenol and
patient is competent, informed about options,
atropine are rst-line agents to treat heart block
and does not desire treatment. At this point the
and bradycardia in -blocker toxicity but in re-
patient has not voiced any objection to treat-
fractory or severe cases glucagon should be ad-
ment. Therefore, the standard of care should
ministered.
be offered rst.
Answer A is incorrect. Amiodarone is a class
Answer B is incorrect. This combination is not
III antiarrhythmic agent that acts on the myo-
regularly used in the management of bladder
cardium to increase the time of the action po-
cancer. Intravesicular bacille Calmette-Gurin
tential. It also acts as a vasodilator. It is used in
is one of the agents used to treat supercial
treatment of severe ventricular arrhythmias, es-
transitional cell carcinoma without the use of
pecially supraventricular arrhythmias.
systemic chemotherapy.
Answer B is incorrect. Although cardiac pac-
Answer C is incorrect. Extensive pelvic exen-
ing may be helpful for stimulating contractility
teration (en bloc resection of all pelvic organs)
in -blocker-induced bradycardia, this is more
and systemic chemotherapy would not be the
commonly used in patients with torsades de
standard treatment for a patient with metastatic
pointes due to sotalol toxicity.
disease. There is some evidence that a metas-
Answer C is incorrect. Dihydraphenamine tectomy and chemotherapy in addition to local
is an effective H1-blocker antihistamine. This bladder and pelvic dissection may improve sur-
may be used prophylactically to avoid poten- vival; however, this is not the standard of care,
tial anaphylaxis, including respiratory distress, and given the numerous lung lesions, such an
in patients known to have risk factors for drug aggressive approach is of limited benet.
interactions.
Answer E is incorrect. Transurethral resection

FULL-LENGTH EXAMS
Answer E is incorrect. Albuterol is a selective of bladder tumor, pelvic radiation, and sys-
2-agonist and is indicated in exacerbations of temic chemotherapy is not warranted for meta-
restrictive airway disease. Although -blocker static bladder cancer. The use of radiation and
overdose could contribute to bronchospasm, aggressive surgery does have a role in invasive
this patient is not in apparent respiratory dis- cancer without evidence of metastases, but this
tress. In albuterol toxicity, -blockers are often patient has metastatic disease. Transurethral
used to decrease heart rate. resection with intravesicular chemotherapy is
used in supercial cancers.
29. The correct answer is D. Fatigue and weight
loss are suspicious for metastatic disease or a 30. The correct answer is C. This patient has eu-
metabolically active tumor. The work-up has volemic hyponatremia. Her urine osmolality
led to the diagnosis of metastatic transitional and sodium level are low, which indicates that
cell carcinoma. The treatment of this terrible she is appropriately producing dilute urine.
disease is systemic chemotherapy. With todays The etiology of her disorder is the intake of too
therapies, patients with metastatic transitional much water, probably related to her psychiatric
cell carcinoma have a median survival of about condition. There can be a relationship be-
12 months and a 3-year survival rate of about tween chronic schizophrenia and psychogenic
Test Block 4

15%20%. MVAC (methotrexate, vinblastine, polydipsia. This can be treated with clozapine.
doxorubicin [Adriamycin], and cisplatin) is the
Answer A is incorrect. Adrenal insufciency
standard chemotherapeutic regimen, but other
presents as euvolemic hyponatremia, but se-
less toxic combination therapies are used. The
rum potassium levels would be high.
utility of surgery in patients with metastatic dis-
ease is not well dened. There is some sugges-
600 Section II: Full-Length Examinations Answers

Answer B is incorrect. Dehydration presents neuromuscular defect. Although the right mid-
as hypovolemic hyponatremia. Urine osmolal- dle lobe inltrate is suggestive of pneumonia,
ity and urine sodium levels would be high. the patients presentation and declining vital
capacity is compatible with a myasthenic cri-
Answer D is incorrect. Renal failure presents
sis that has been precipitated by the infection.
as hypervolemic hyponatremia. This patients
Clearly, intravenous antibiotics would be indi-
blood urea nitrogen and creatinine levels are
cated but only after emergent intubation.
normal, which indicates that she is not in renal
failure.
32. The correct answer is D. Physicians are ethi-
Answer E is incorrect. Syndrome of inappro- cally and legally obligated to override patient
priate ADH secretion presents as euvolemic condentiality in the following cases: (1) sui-
hyponatremia, but urine osmolality is usually cidal patients, (2) gunshot and knife wounds,
xed and elevated. (3) infectious diseases (health authorities and
at-risk third parties must be informed), (4)
31. The correct answer is C. Myasthenia gravis is child and elder abuse, and (5) impaired auto-
an autoimmune disease caused by antibodies mobile drivers. The physician should inform
against postsynaptic acetylcholine receptors. the patient that she is obligated to breech usual
Treatment is typically aimed at downregulating condentiality in the case of HIV status. She
the immune response or increasing the con- should encourage the patient to disclose his
centration of acetylcholine within the synapse. HIV status to his partner and other sexual con-
In emergency situations such as during myas- tacts, but she must do so if he will not.
thenia crisis with impending respiratory failure,
Answer A is incorrect. Physicians must main-
plasmapheresis and intravenous immunoglob-
tain condentiality in cases of abuse between
ulin after intubation provide a means of tem-
competent adults because this is not included
porarily relieving symptoms. Indicators of re-
in the list of activities in which condentiality
spiratory compromise include loss of upper
may be breached.
airway integrity and decreasing vital capacity or
negative inspiratory force. The arterial blood Answer B is incorrect. Health authorities and
gas is not a good indicator of the need for intu- at-risk third parties must both be informed of
FULL-LENGTH EXAMS

bation in these patients. The lifetime risk of a HIV status.


myasthenic crisis is 15%20% and is often pre-
Answer C is incorrect. Health authorities and
cipitated by infection, metabolic stress, or non-
at-risk third parties must both be informed of
depolarizing anesthetic agents.
HIV status.
Answer A is incorrect. Albuterol is helpful as a
Answer E is incorrect. The physician should
treatment in asthmatic and chronic obstructive
inform the patient that she is obligated to
pulmonary disease treatments with obstructive
breech usual condentiality in the case of HIV
defects. This patient has a neuromuscular dis-
status. She should encourage him to disclose
order leading to restrictive defect.
his HIV status to his partner and other sexual
Answer B is incorrect. Edrophonium is useful contacts, but she must do so if he will not.
in diagnosing myasthenia, although it is rarely
used as treatment. 33. The correct answer is C. This infant has seb-
orrheic dermatitis (SD) of the scalp, or cradle
Answer D is incorrect. Inhaled racemic epi-
cap. These lesions are erythematous and char-
nephrine can be a useful bronchodilator in pe-
acterized by greasy, yellow plaque and scale
diatric patients with croup. However, it is not
formation. SD is frequently associated with
indicated in adults with restrictive defects.
Test Block 4

Malassezia furfur colonization, although cau-


Answer E is incorrect. Intravenous antibiotics sality has not been proven. Treatment is with
would be useful in a patient whose shortness selenium sulde, tar, or zinc pyrithione sham-
of breath was caused by pneumonia but not a poos.
Test Block 4 Answers 601

Answer A is incorrect. SD elsewhere on the Answer C is incorrect. Mitral stenosis is almost


body is often successfully treated with topi- always associated with rheumatic heart disease
cal corticosteroids or antifungals; however, in adults. It can lead to pulmonary congestion
the scalp can usually be managed easily with due to increased left atrial pressure. Left-sided
shampoos. heart failure may result. An opening snap, dia-
stolic rumble, and echocardiogram visualiza-
Answer B is incorrect. Oral griseofulvin is
tion of the valve leaets are helpful in making
used to treat tinea capitis, which is character-
this diagnosis.
ized by erythema and scaling of the scalp, ac-
companied by alopecia. Plaque formation is Answer D is incorrect. Prinzmetals, or atypi-
atypical of tinea capitis. cal, angina occurs at rest and is associated with
ST elevation. It is due to ischemia and would
Answer D is incorrect. Topical emollients are
not present with these symptoms.
often effective treatments for scalp eczema
(atopic dermatitis). Eczematous lesions are typ-
35. The correct answer is A. This patient is suffer-
ically erythematous and scaling which results
ing from gestational diabetes. Infants of mothers
in the presence of intense itching and dryness,
with gestational diabetes have an increased inci-
thus distinguishing eczema from SD.
dence of macrosomia (an infant who weighs at
Answer E is incorrect. This is an appropriate least 4.0 kg for diabetic mothers, or at least 4.5
treatment for nonscalp SD. The antifungal kg for nondiabetic mothers). In diabetic patients
most commonly used for nonscalp SD is keto- with poor maternal glucose control, preeclamp-
conazole. sia, macrosomia, or evidence of fetal lung matu-
rity, early delivery should be considered. Mac-
34. The correct answer is E. Carcinoid syndrome rosomia may lead to longer labor, shoulder
is due to metastasis of tumors that secrete an dystocia, and birth trauma. If early delivery is
abnormally high amount of serotonin, which considered, then the lecithin:sphingomyelin
can eventually lead to damage to the lining of (L:S) ratio and phosphatidylglycerol level
the heart. Restrictive cardiomyopathy may re- should be obtained to assess fetal lung maturity.
sult. This condition is commonly associated An L:S ratio >2 indicates pulmonary maturity,

FULL-LENGTH EXAMS
with inltrative diseases such as amyloidosis, and an L:S ratio <2 indicates pulmonary imma-
sarcoidosis, or hemochromatosis. Congestive turity. If, however, the L:S ratio is <2 and phos-
heart failure symptoms (right greater than left), phatidylglycerol is present, then the chance of
as described in this question, may result when neonatal respiratory distress syndrome is <5%.
left ventricle lling is restricted, with resultant The test performed to assess these levels is am-
decreased output and compliance, and in- niocentesis.
creased lling pressure. The echocardiogram
Answer B is incorrect. Chorionic villus sam-
and ECG help pinpoint this as the cause of the
pling is performed at 1012 weeks gestation,
patients problem. Treatment for this condition
for the purpose of early assessment of fetal
is to control the underlying cause (e.g., iron
karyotype. It would not be appropriate in this
chelation for hemochromatosis), diuretics, an-
patient. Amniocentesis is indicated in the
giotensin-converting enzyme inhibitors, and
clinical situation described above, as well as
nitrates.
in mothers 35 years old or older, as the risk for
Answer A is incorrect. There is no indication congenital abnormalities such as Downs syn-
that the patient has undergone neurologic drome increases with advanced maternal age.
damage and therefore stroke would not be high
Answer C is incorrect. If delivery is necessary,
on the differential.
corticosteroids may be given to the mother to
Test Block 4

Answer B is incorrect. Coarctation of the aorta hasten fetal lung maturity. However, a cortisol
may lead to hypertension in adults and is char- level has no role in prenatal screening.
acterized by a midsystolic murmur and notch-
ing of the ribs.
602 Section II: Full-Length Examinations Answers

Answer D is incorrect. Hemoglobin A1c levels matoid arthritis as the risk factor, rather than
should be measured in patients with known NSAID use in the treatment of rheumatoid ar-
diabetes at the rst prenatal visit. If it is >8.5%, thritis.
the mother should be counseled regarding con-
genital abnormalities and an ultrasound done 37. The correct answer is E. This patients thyroid-
at about 15 weeks gestation; the fetus should stimulating hormone (TSH) level is slightly
also be aggressively assessed for congenital ab- high, suggesting that his dose of levothyroxine
normalities. (synthetic thyroxine) needs to be raised, usu-
ally by 2550 g at a given time. He is also
Answer E is incorrect. A random blood glu-
mildly anemic, which may be related to his hy-
cose level is not indicated here. This patient
pothyroidism or to chronic disease more gener-
has failed the 3-hour glucose tolerance test,
ally. Neither of these conditions, however, are
indicating a diagnosis of gestational diabetes.
serious enough to warrant postponing surgery.
Management for her diabetes should begin
Thyroid function should be rechecked in 46
with regular exercise, an American Diabetes
weeks, as it takes this long for exogenous thy-
Association diet with at least 1800 calories/day,
roid hormone to provide negative feedback to
and frequent ngerstick glucose readings at
the pituitary gland, leading to a drop in TSH
home.
production. This patients dosage can be read-
justed at this time.
36. The correct answer is E. Odds ratios are used
in retrospective case-control studies to analyze Answer A is incorrect. The patients TSH
the relationship between an exposure and dis- level is slightly high, suggesting that his dose
ease or condition of interest. The starting point of levothyroxine (synthetic thyroxine) needs to
is the presence or absence of the disease, and be raised, not lowered. In addition, it is neither
among patients in those two categories, an necessary nor effective to delay surgery by 35
odds ratio describes the likelihood that each days. While the patient may (or may not) note
had the exposure of interest. a response to the change in medication after a
few days, a complete response, including feed-
Answer A is incorrect. Although the study
back on the pituitary gland leading to lower
demonstrates a correlation between nonsteroi-
FULL-LENGTH EXAMS

TSH levels, will not be apparent for 46 weeks.


dal anti-inammatory drugs (NSAIDs) use by
patients with rheumatoid arthritis and subse- Answer B is incorrect. The dosage of levothy-
quent stroke risk, it does not prove causation. roxine does need to be increased, given the
The increase in stroke incidence may be due patients elevated TSH; however, delaying
to a feature of the disease and not a sequelae of surgery is unnecessary and rechecking thyroid
NSAID use, in which case not using NSAIDs function in 1 week is redundant. It takes 35
in the treatment of arthritis might not reduce days for the patient to begin to notice a physi-
stroke risk. ologic change in response to a change in thy-
roid hormone dosage, but requires 46 weeks
Answer B is incorrect. This answer describes
for a complete response and for TSH levels to
relative risk, which is used in prospective co-
drop appropriately.
hort studies and starts with the absence of dis-
ease. Relative risk describes the likelihood of Answer C is incorrect. In this case it is appro-
developing a particular disease or condition af- priate to raise the dosage of levothyroxine and
ter a given exposure. to recheck thyroid function in 46 weeks, as
it will take this long for feedback changes in
Answer C is incorrect. Attributable risk is the
TSH levels to become apparent on laboratory
difference in incidence rate between exposed
testing. It is not necessary, however, to delay
Test Block 4

and unexposed patients and is used in prospec-


surgery by 46 weeks in order to make small
tive studies.
adjustments in the patients dosage.
Answer D is incorrect. This answer confuses
Answer D is incorrect. This management ap-
the putative underlying relationship and as-
proach would be appropriate if the patients
signs the increased risk for stroke to rheu-
TSH level was low, indicating a slightly hyper-
Test Block 4 Answers 603

thyroid state and the need for a lower dose of Answer A is incorrect. Alkalinization and hy-
levothyroxine. It is appropriate to proceed with dration alone are highly effective in dissolving
surgery under these conditions. It is also neces- uric acid stones, and extracorporeal shock wave
sary to wait 46 weeks before the physiologic lithotripsy is therefore unnecessary in these pa-
response to any change in thyroid hormone tients.
dosage is complete.
Answer C is incorrect. A low-sodium diet is
effective in treating calcium stones, but is not
38. The correct answer is A. Amyotrophic lateral
part of the management of uric acid stones.
sclerosis is a slowly progressive, generalized
motor muscle paralysis involving both lower Answer D is incorrect. Penicillamine is an ef-
and upper neurons that typically progresses to fective treatment for cystine stones, but has not
death in 35 years. Drooling is a common been used in treatment of uric acid stones.
symptom and is due to facial muscle weakness
Answer E is incorrect. Thiazide diuretics can
and reduced swallowing ability rather than in-
be used to treat hypercalciuria in patients with
creased secretions. However, anticholinergic
calcium stones, but are not used in patients
drugs such as glycopyrrolate can be useful in
with uric acid stones.
decreasing secretions in order to manage these
symptoms.
40. The correct answer is E. Rifampin is an in-
Answer B is incorrect. Baclofen is potentially ducer of the P450 enzymes. That is, it causes
useful for treating muscle spasticity in amyo- the enzymes to metabolize the drugs that they
trophic lateral sclerosis. act on at a faster rate, decreasing blood con-
centrations of these drugs, which are oral con-
Answer C is incorrect. Carbamazepine is po-
traceptive pills in this case.
tentially useful for treating muscle spasms and
cramps that accompany amyotrophic lateral Answer A is incorrect. Albuterol would not in-
sclerosis. duce the P450 enzymes. Therefore, it would
not increase the metabolism of the oral contra-
Answer D is incorrect. Although depression,
ceptive pills.
pseudobulbar symptoms (outbursts of laughter

FULL-LENGTH EXAMS
or tearfulness), and cognitive loss are symp- Answer B is incorrect. Rifampin does not
toms in amyotrophic lateral sclerosis, this pa- share binding sites with estrogen or progeste-
tients current problem is medical and does not rone, the components of combined oral con-
require psychiatric evaluation. traceptive medications.
Answer E is incorrect. Diphenhydramine is a Answer C is incorrect. Sex hormones are not
sedative and is potentially a useful treatment pumped into or out of cells but rather diffuse
for insomnia, which is very common in pa- across the lipid bilayer.
tients with amyotrophic lateral sclerosis. Seda-
Answer D is incorrect. Patient error is possible
tives should be used sparingly in these patients.
but is not likely in the presence of a known
mechanism for drug interaction.
39. The correct answer is B. This patient has uric
acid renal calculi, which are nonopaque on
41. The correct answer is A. Autosomal dominant
plain lm but detectable by CT scan. Uric
polycystic kidney disease (ADPCKD) is the
acid stones are common in patients with gout
more common type of polycystic kidney dis-
who are not receiving effective treatment for
ease. Symptoms usually begin in the third to
hyperuricemia. There are three mainstays of
fourth decade of life. Renal cyst formation,
treatment for this condition: hydration, which
chronic interstitial inammation, and recur-
Test Block 4

reduces the concentration of uric acid in the


rent urinary tract infections (UTIs) lead to
urine; alkalinization of the urine, which de-
chronic kidney disease manifest as hyperten-
creases the precipitation of uric acid; and al-
sion and an elevated creatinine. Other compli-
lopurinol, which lowers uric acid production
cations of ADPCKD include intracranial berry
and excretion.
604 Section II: Full-Length Examinations Answers

aneurysms, mitral valve prolapse, and UTIs. chia coli, enter the upper tract. The patient
Physical examination may reveal palpable and has a history of UTIs; however, pyelonephritis
tender kidneys. Rupture of renal cysts can would be an unusual cause of hypertension
cause ank pain and hematuria. and bilateral palpable kidneys. Furthermore,
urinalysis showed no bacteria, making infec-
Answer B is incorrect. Autosomal recessive
tion unlikely.
polycystic kidney disease (ARPCKD), with an
incidence of 1:20,000 births, is less common
42. The correct answer is B. In a hepatic iminodi-
than the autosomal dominant type (1:400 to
acetic acid scan, iminodiacetic acid is absorbed
2000 births). This disease affects infants and
from the bloodstream and secreted in the bile
children, with a majority of cases being diag-
ducts. If there is cholecystitis, then the gall-
nosed in the rst year of life. The gene respon-
bladder will not be visualized. This is the next
sible for ARPCKD has been located on chro-
appropriate test after an equivocal abdominal
mosome 6p21. The cysts are formed from the
ultrasound.
distal tubules and are typically arranged in a
radial fashion. In addition to renal failure and Answer A is incorrect. CT is not an appropri-
loss of urine-concentrating ability, these pa- ate test when evaluating RUQ pain because it
tients may develop portal hypertension, likely is unable to detect most gallstones.
secondary to hepatic brosis.
Answer C is incorrect. MRI is not an appro-
Answer C is incorrect. Medullary cystic dis- priate test when evaluating RUQ pain because
ease is an autosomal dominant disease that it is too costly and time consuming.
leads to small kidneys with medullary cysts,
Answer D is incorrect. A sestamibi scan is
generally sparing the cortex and papillae. Cysts
useful in the evaluation of parathyroid dis-
range from 1 to 10 mm in size and can origi-
ease. Radiolabeled sestamibi is injected into
nate from various parts of the nephron includ-
the bloodstream and becomes concentrated
ing the loop of Henle, distal convoluted tu-
in overactive parathyroid adenomas, allowing
bule, and collecting duct. This disease presents
them to be visualized.
in third or fourth decade of life. The symptoms
are polyuria, anemia, and progressive renal in- Answer E is incorrect. X-ray of the abdomen
FULL-LENGTH EXAMS

sufciency. There is no cure for this disease. is not very helpful in the evaluation of biliary
colic or cholecystitis because only 10% of
Answer D is incorrect. Medullary sponge kid-
stones are radiopaque. X-ray of the abdomen
ney is usually asymptomatic, but can present
can be useful in the evaluation of gallstone
with hematuria, ank pain, or dysuria. The
ileus, which will demonstrate air in the gall-
disease is characterized by renal tubular ecta-
bladder (pneumobilia) and radiodensity near
sia and enlargement of medullary collecting
the ileocecal valve.
ducts, papillae, and calyces. Although patients
are at risk for nephrolithiasis, urine-concentrat- 43. The correct answer is C. If a radionucleotide
ing defects, and UTIs, patients have an excel- scan shows a hot nodule, the patient should be
lent renal prognosis and a normal life expec- evaluated for hyperthyroidism and observed
tancy. carefully. If the scan indicates a cold nodule,
Answer E is incorrect. Kidney stones can the mass is more likely to be malignant and
cause ank pain and hematuria. However, surgery should be the next option. Most nod-
acute nephrolithiasis rarely if ever causes pal- ules (70%90%) are cold, and most of these
pable kidneys or renal failure. Additionally, this are benign. Therefore if a nodule is scanned as
patients clinical presentation necessitates the warm the chance of malignancy is signicantly
Test Block 4

provisional diagnosis of simultaneous bilateral reduced.


stones, which would be unusual. Answer A is incorrect. High-resolution ultra-
Answer F is incorrect. Pyelonephritis is an sonography offers better anatomic denition
infection of the kidney parenchyma. Bacteria than scintigraphy but is not a cost-effective ini-
from the lower tract, most commonly Escheri-
Test Block 4 Answers 605

tial test. Scintigraphy is also the only test that Questions 45 and 46
can elucidate the functional status of a nodule.
45. The correct answer is F. Giardia lamblia is a
Answer B is incorrect. Observation alone is agellated protozoan that infects the duode-
not sufcient in this instance, as the patients num and small intestine in infected patients.
age, gender, and nature of the mass suggest a Infection is often asymptomatic but in patients
possible malignancy. who manifest symptoms, watery diarrhea is the
most common presentation. Other symptoms
Answer D is incorrect. Thyroxine therapy with
include malaise and weakness, atulence, ab-
close follow up may be appropriate depending
dominal distention, foul-smelling greasy stools,
on the results of the radionucleotide scan and
weight loss, and anorexia. The key to this pa-
thyroid function tests.
tients history is found in the timetable of her
Answer E is incorrect. Surgery is not appro- illness, in addition to her recent travel and ex-
priate immediately; further studies should be posure. G. lamblia has an incubation period of
done to evaluate the nature of the nodule. approximately 12 weeks and diarrhea can be
prolonged, lasting in some patients for several
44. The correct answer is B. An annual digital rec- weeks. The organism has been associated with
tal examination after reaching 50 years of age waterborne outbreaks, and the childs recent
is the most universally recommended screen- swimming in natural water sources with nearby
ing tool for prostate cancer, and is a mandatory wildlife is the likely area where she was in-
part of the physical examination for a middle- fected.
aged man with recent-onset urinary com-
plaints. 46. The correct answer is I. Staphylococcus aureus
Answer A is incorrect. Cytoscopy is not recom- is implicated in multiple cases of food-borne
mended for screening or longitudinal benign illness each year. The mechanism of infection
prostatic hypertrophy monitoring. This would is based on preformed toxins that are found in
be an appropriate test in a younger man with infected foods and are able to illicit the clinical
recurrent UTIs. symptoms described. The incubation period
for S. aureus food poisoning is within hours af-

FULL-LENGTH EXAMS
Answer C is incorrect. Ultrasound-guided bi- ter ingestion, and emesis is commonly the rst
opsy of the prostate is used to provide the de- and most signicant symptom. Precooked, pre-
nitive diagnosis of suspected prostate cancer packaged meats with high salt contents (i.e.,
but would not be used as an initial test. the ham from the patients sandwiches) are
Answer D is incorrect. Measuring the prostate- commonly implicated in cases of S. aureus
specic antigen level is recommended yearly food poisoning.
for screening patients at high risk or >50 years Answer A is incorrect. Bacillus cereus is a
old; start screening at age 40 years for African- gram-positive, spore-forming rod that causes
American men. With a level >4.1 ng/mL, tran- food poisoning when the spores land on food
srectal ultrasonography with biopsy of suspicious and germinate, creating a toxin. Ingestion of
areas is recommended. However, an elevated the toxin will lead to sudden onset nausea,
prostate-specic antigen level may be due to be- vomiting, and diarrhea. Because the reaction
nign prostatic hypertrophy, prostatitis, UTI, pan- is due to preformed toxins, antibiotics are not
creatic trauma, or carcinoma. useful and treatment is supportive.
Answer E is incorrect. Urinalysis and urine Answer B is incorrect. Campylobacter jejuni
culture would be appropriate to rule out in- is a gram-negative agellated rod that, along
fection and hematuria; however, the patient with rotavirus and enterotoxigenic Escherichia
Test Block 4

denies fever, chills, urgency, frequency, costo- coli, is one of the three most common causes
vertebral angle tenderness, or bladder fullness. of diarrhea in the world. Domestic animals
UTIs are uncommon in men of this age. and poultry are a reservoir for these bacteria,
which are passed by the fecal-oral route and
unpasteurized milk. Infection presents with a
606 Section II: Full-Length Examinations Answers

prodrome of fever, headaches, and malaise, fol- pain, and watery diarrhea. The diarrhea can be
lowed by bloody, loose diarrhea. Treatment is positive for heme, and fever may or may not be
with a uoroquinolone and uid resuscitation. present. Salmonella infections occur as a re-
sult of the contamination of food with animal
Answer C is incorrect. Clostridium difcile is
feces. Common animal carriers are turtles and
the pathogen of antibiotic (usually clindamy-
chickens. Treatment is aggressive hydration
cin or ampicillin)-associated colitis. Symptoms
without antibiotic therapy.
include diarrhea, fever, and abdominal cramp-
ing. C. difcile toxins can be tested for in stool Answer H is incorrect. Shigella is a nonag-
samples. Treatment is usually with metronida- ellated gram-negative rod. It is found only in
zole and discontinuation of antibiotics. humans and passed by the fecal-oral route, and
is often present in communal living situations
Answer D is incorrect. Enteric adenovirus is
or day care. Infection presents with fever and
a common viral etiology of diarrhea in infants
abdominal pain, along with blood and pus-
and children. Infection with this virus is most
speckled diarrhea. Treatment is with uoro-
prevalent in the spring and summer seasons.
quinolones and uid resuscitation.
Infection is often accompanied by fever, rhi-
norrhea, cough, and acute respiratory disease. Answer J is incorrect. Vibrio cholerae is a
No antiviral agents or vaccines are appropriate gram-negative agellated rod that is passed by
for the treatment of adenovirus, and aggressive the fecal-oral route, usually through drinking
hydration is the treatment of choice. contaminated water. It presents with sudden
onset voluminous rice water diarrhea. Treat-
Answer E is incorrect. Escherichia coli coli-
ment is primarily with uid resuscitation and
tis is caused by several subtypes of the E. coli
oral rehydration because patients can lose up
bacterium. Enterotoxigenic E. coli causes a
to 1 L of uid in an hour, and death is com-
rice water-type stool and is often seen in trav-
monly due to dehydration. Denitive treat-
elers to Third World countries. Enterohemor-
ment is with antibiotics, either doxycycline or
rhagic E. coli causes a hemorrhagic colitis, and
a uoroquinolone.
the specic subtype E. coli O157:H7 results in
hemolytic uremic syndrome. This syndrome is Answer K is incorrect. Vibrio parahaemolyticus
FULL-LENGTH EXAMS

characterized by thrombocytopenia and renal is a gram-negative rod found in marine water


failure. Enteroinvasive E. coli results in colitis and is a common cause of diarrhea in Japan,
along with fever, and WBCs can be found in where raw seafood is commonly consumed.
the stool. Treatment is aggressive oral hydra- Infection presents similarly to Vibrio cholerae
tion without antibiotic therapy. with rice water diarrhea; however, V. para-
haemolyticus typically resolves within 3 days
Answer G is incorrect. Salmonella enteritidis
without the need for treatment.
causes less than 1 week of nausea, abdominal
Test Block 4
Test Block 5

607
608 Section II: Full-Length Examinations Questions

Q U E ST I O N S

1. A 54-year-old woman with a history of rheuma- sis best explains the patients current presenta-
toid arthritis (RA) presents to the rheumatology tion?
clinic complaining of swelling in her ankles
(A) Conversion disorder
and puffy eyes present for the past week. She
(B) Depression
has had RA for the past 15 years and is cur-
(C) Hypochondriasis
rently being managed with several medica-
(D) Multiple sclerosis
tions. She has no other signicant past medical
(E) Somatization disorder
history and has had no recent change in her
diet. She does not smoke or drink alcohol. She 3. A 62-year-old woman presents to the emer-
has a temperature of 36.9C (98.4F), heart gency department after a sudden syncopal epi-
rate of 110/min, and blood pressure of 144/92 sode. She has no chronic medical conditions
mm Hg. Her face appears puffy, with signi-
and takes no medications. However, during the
cant bilateral periorbital edema. Examination
past 36 hours she has felt acutely ill, with ab-
of her heart and lungs is normal. Her abdomen
dominal pain and repeated episodes of diar-
is soft and nondistended with no hepatospleno-
rhea and vomiting. Her blood pressure is
megaly. She has 2+ pitting edema in both legs.
144/85 mm Hg, pulse is 70/min, respiratory
Urinalysis shows 3+ protein. Renal biopsy is
rate is 10/min, and temperature is 37.9C
read as renal disease consistent with membra-
(100.3F). Laboratory studies show:
nous nephropathy. Which of the following
medications most likely caused her acute pre- Na+: 155 mEq/L
sentation? K+: 2.1 mEq/L
Cl: 105 mEq/L
(A) Aspirin
HCO3: 36 mEq/L
(B) Corticosteroids
(C) Cyclophosphamide An ECG demonstrates attened T waves and
(D) Methotrexate prominent U waves that normalize after potas-
FULL-LENGTH EXAMS

(E) Penicillamine sium repletion. Further tests reveal normal 24-


hour urine-free cortisol levels and persistently
2. A 29-year-old woman presents to the emer- diminished plasma renin activity. Which of the
gency department concerned that she is having following is the most likely diagnosis?
a stroke. Her husband states, my wife just
(A) 21-Hydroxylase deciency
cant seem to walk right. When questioned,
(B) Addisons disease
she meekly says that for the past month her
(C) Conns syndrome
legs just wont work, as if she has no strength in (D) Cushings disease
them. She denies numbness or tingling, and (E) Cushings syndrome
otherwise feels well. During the interview, the
husband repeatedly answers questions for his 4. A 60-year-old man with idiopathic dilated car-
wife. He notes that the weakness began just af- diomyopathy presents for a routine visit. Over
ter the patients mother died 1 week ago. Tear- the past few years his symptoms have been sta-
fully, the patient states that she did not visit her ble. He has signicant fatigue and dyspnea
mother before her death because my mom upon mild to moderate exertion with marked
and my husband didnt get along. The pa- limitation of daily activity. His past medical
tients cranial nerves II-XII are grossly intact; history is negative for hypertension, diabetes,
motor activity is 5/5 in the upper extremities smoking, or coronary artery disease. Echocar-
Test Block 5

and 1/5 in the lower extremities; sensation is diogram reveals four-chambered dilatation and
intact to pinprick and vibration; the patient is global hypokinesis of the myocardium. Which
unable to ambulate without signicant assis- of the following therapies may worsen the pa-
tance; and her cerebellar function is intact. tients condition?
Results of an MRI are normal. Which diagno-
Test Block 5 Questions 609

(A) Adult-dose aspirin jects who do not have breast cancer, as shown
(B) Carvedilol on mammography. Data are collected about
(C) Enalapril each patients use of red wine. Which of the
(D) Exercise training following data analyses is most suitable for this
(E) Spironolactone study?
(A) Attributable risk
5. A 44-year-old man presents to the emergency
(B) Number needed to treat
department complaining of 3 hours of severe
(C) Odds ratio
epigastric pain that radiates to his back. He de-
(D) Prevalence
nies experiencing similar episodes in the past
(E) Relative risk
but remarks that he has been diagnosed previ-
ously with gallstones but elected to not have a
7. A 4-year-old Asian girl is brought to the emer-
cholecystectomy. Examination and laboratory
gency department because of a persistent fever
ndings are consistent with acute pancreatitis,
to 38.3C (101F) for the past 5 days that was
and he is admitted to the hospital. Two months
difcult to manage with acetaminophen. She
following discharge, he continues to have ab-
is also complaining of neck pain. The child is
dominal pain and elevated amylase levels. CT
febrile (38.7C [101.6F]) with otherwise sta-
of the abdomen is shown in the image. Which
ble vital signs. She has bilateral conjunctivitis,
of the following is the most likely diagnosis?
a reddened tongue, a scarlatiniform rash on
her trunk, and a single palpable 2.5-cm lymph
node of her right anterior cervical chain. If left
untreated, what condition is this patient at
highest risk for developing?
(A) Cardiomyopathy
(B) Coronary artery aneurysms
(C) Encephalitis
(D) Epidermal necrolysis
(E) Retropharyngeal abscess

FULL-LENGTH EXAMS
(F) Stroke

Reproduced, with permission, from Chen MYM, Pope 8. A 1-month-old baby presents to the emergency
TL, Ott DJ. Basic Radiology. New York: McGraw-Hill, department with poor feeding and seizure ac-
2004: Figure 11-60. tivity. The mother reports that the baby fell on
(A) Aortic dissection the oor while she was feeding him. Fundu-
(B) Cholecystitis scopic examination revealed retinal hemor-
(C) Pancreatic pseudocyst rhages in the right eye. Which of the following
(D) Perforated duodenal ulcer is the most likely diagnosis?
(E) Recurrent acute pancreatitis (A) Group B streptococcal meningitis
(B) Obstructive hydrocephalus
6. A hypothetical study is performed that exam- (C) Shaken baby syndrome
ines the link between breast cancer and red (D) Type 1 diabetes mellitus
wine consumption. Patients who have been di- (E) von Willebrands disease
agnosed with breast cancer are paired with sub-
Test Block 5
610 Section II: Full-Length Examinations Questions

9. A 26-year-old man with HIV and a history of tion reveals frontal bossing and bilateral bow-
intravenous drug use presents to the emer- ing of her tibias. Laboratory tests are ordered.
gency department with 5 days of low-grade fe- Which of the following laboratory abnormali-
ver, nonproductive cough, and dyspnea on ex- ties is most likely to be associated with this dis-
ertion that progressed to dyspnea at rest. His order?
oxygen saturation is 88% on room air. Arterial
(A) Decreased alkaline phosphatase levels
blood gas analysis shows an arterial-alveolar
(B) Elevated alkaline phosphatase levels
gradient of 45 and arterial oxygen pressure of
(C) Elevated serum calcium
65 mm Hg. X-ray of the chest reveals diffuse
(D) Elevated serum phosphate
bilateral inltrates. Which of the following is
(E) Positive rheumatoid factor
the most appropriate treatment?
(A) Ampicillin + gentamicin 12. The concerned parents of a 3-month-old boy
(B) Intravenous clindamycin present to the emergency department because
(C) Isoniazid + pyrazinamide + rifampin + of their sons behavior during feedings. His
ethambutol mother reports that he often chokes and gags
(D) Third-generation cephalosporin + during feedings and spits up signicant
doxycycline amounts of material after such episodes. Addi-
(E) Trimethoprim-sulfamethoxazole + tionally, he is often irritable and has been seen
oral prednisone to arch his back when seated. His pediatrician
has advised close monitoring of his weight be-
10. An 18-year-old man is brought to the emer- cause he has moved from the 60th percentile
gency department following a high-speed mo- to the 45th percentile since birth. His birth,
tor vehicle crash. He is in stable condition and family, and social histories are unremarkable.
reports no loss of consciousness following the Vital signs are stable and there are no remark-
accident. His temperature is 36.8C (98.3F), able ndings. Relevant laboratory ndings are:
heart rate is 86/min, blood pressure is 135/84
Na+: 141 mEq/L
mm Hg, and respiratory rate is 15/min. The pa-
Cl: 105 mEq/L
tient describes chest pain that worsens with in-
Mg2+: 1.5 mg/dL
FULL-LENGTH EXAMS

halation and with abduction of his right arm.


K+: 4.2 mEq/L
X-ray of the chest shows a widened mediasti-
Ca2+: 9.8 mg/dL
num, along with loss of the aortic knob. In ad-
HCO3: 23 mEq/L
dition, rst and second rib fractures are noted
PO4: 2.1 mg/dL
on the x-ray lm. Which test should be com-
Glucose: 96 mg/dL
pleted next?
Creatinine: 0.5 mg/dL
(A) Aortogram Blood urea nitrogen: 15 mg/dL
(B) MRI scan
Which of the following is the best course of ac-
(C) Transthoracic echocardiography
tion for the pediatrician to take?
(D) Upper endoscopy
(E) Ventilation-perfusion scan (A) Conservative management with parental
reassurance
11. A 44-year-old woman presents to her primary (B) Initiation of daily antacid treatment
care physician for a routine check-up. She (C) Prescription of a histamine-2 receptor
states that over the past year she has become antagonist
more bowlegged, and her hats are now too (D) Prescription of a prokinetic agent
small to t her comfortably. She denies any fo- (E) Prescription of a proton pump inhibitor
Test Block 5

cal pain or tenderness, but does admit to occa-


sional dull aching in her lower legs that is ex- 13. A 16-year-old girl comes to see her pediatrician
acerbated by exercise. Her temperature is alone, complaining of itchy, greenish vaginal
36.9C (98.4F), pulse is 76/min, and blood discharge for 4 days. When questioned about
pressure is 119/83 mm Hg. Physical examina- her sexual practices, the patient blushes and
Test Block 5 Questions 611

admits that although she is usually careful tion, and bowel sounds are absent. She does
about using condoms, she was at a party re- not have rebound tenderness. Complete blood
cently where she drank too much and forgot to cell count, urinalysis, and basic metabolic
insist that her boyfriend use a condom. After panel with extended tests were unremarkable.
disclosing this information, she suddenly looks Flat and upright abdominal lms reveal gen-
alarmed and says, Youre not going to tell my eral distension of the small intestine and colon
parents this, are you? They dont know that my with air present throughout the bowel. Which
boyfriend and I have sex, and they would kill of the following is the most likely diagnosis?
me. In addition to gently reminding the pa-
(A) Acute appendicitis
tient about safe sex, how should the pediatri-
(B) Acute cholecystitis
cian respond?
(C) Complete bowel obstruction
(A) Although I would encourage you to be (D) Paralytic ileus
open and honest with your parents, your (E) Partial small bowel obstruction
health is most important, and everything
you tell me about your sexual practices is 15. A 52-year-old high school teacher is seen in the
completely condential clinic with a chief complaint of reduced exer-
(B) Although this is not generally my prac- cise tolerance. An avid soccer player, she has
tice, I will make an exception for you be- had to sit out portions of league games due to
cause you are a long-standing patient. shortness of breath, and reports having to pause
However, as your physician I do not ap- to rest when bringing in groceries. She worked
prove of your refusal to tell your parents as a welder 20 years ago, prior to earning her
about your habits degree in education. Medications include
(C) Im sorry, but because you are under 18 combined estrogen and progesterone tablets
years old I will have to get your mothers for menopausal symptoms and occasional ibu-
consent before treating you for vaginitis profen for aches and pains. X-ray of the chest
(D) I really will need to tell your parents be- reveals honeycombing, and bronchoalveolar
cause now that you have had unprotected lavage is positive for two asbestos bodies per
sex I need their consent to do an HIV test milliliter. Which of the following ndings on

FULL-LENGTH EXAMS
(E) Unless you agree to tell your parents pulmonary function testing is most likely?
about your sexual activity, I cannot treat
CHOICE FEV1 FVC FEV1/FVC RATIO
you for your vaginal infection
A normal
14. A 40-year-old white woman presents to the
B normal normal
emergency department with 2 days of constant
moderate abdominal pain accompanied by C
nausea and vomiting. She has not had a bowel
D normal normal normal
movement or passed atus in 4 days. Her medi-
cal history is signicant for a chronic pain syn- E normal
drome for which she takes opiates regularly.
Her vital signs are normal. On examination, (A) A
she is in mild distress, her abdomen is dis- (B) B
tended with mild diffuse tenderness to palpa- (C) C
(D) D
(E) E
Test Block 5
612 Section II: Full-Length Examinations Questions

16. A 58-year-old man is brought to the trauma bay (F) 20p12


unresponsive and intubated. He was an unre- (G) 22q11
strained driver in a high-speed motor vehicle
crash. Medical transporters report that the cars 18. A 78-year-old man presents to the clinic with
windshield was broken. His pulse is 58/min fatigue over the past few weeks. He lives in a
and regular, blood pressure is 80/50 mm Hg, second-story apartment and has increasing dif-
and respiratory rate is 15/min. Chest ausculta- culty walking up the stairs. Vital signs are sig-
tion reveals good air ow bilaterally and no ad- nicant for a blood pressure of 165/86 mm Hg.
ventitial lung sounds. He opens his eyes to The height of the QRS waves in leads V1 and
command and his pupils are equally round V5 on the 12-lead ECG are >35 mm. Echocar-
and reactive to light. He is unable to move his diography reveals increased wall thickness, left
extremities or withdraw to pain. His extremities atrial dilatation, and an ejection fraction of
are pink and warm. Which of the following is 60%. Which of the following is the most likely
the most likely diagnosis? cause of this patients heart failure?
(A) Addisonian crisis (A) Diastolic dysfunction caused by constric-
(B) Anaphylaxis tive pericarditis
(C) Atrial brillation (B) Diastolic dysfunction caused by hyperten-
(D) Bacteremia sive cardiomyopathy
(E) Congestive heart failure (C) Diastolic dysfunction caused by restrictive
(F) Inadequate uid repletion cardiomyopathy
(G) Spinal cord injury (D) Systolic dysfunction caused by diastolic
cardiomyopathy
17. A 2-year-old girl has had repeated infections (E) Systolic dysfunction caused by hypertro-
with opportunistic pathogens, including Pneu- phic cardiomyopathy
mocystis jiroveci. She also has had a right-sided (F) Systolic dysfunction caused by ischemic
aortic arch with congenital ventricular septal heart disease
defect and an underdeveloped thymus. She has
a short philtrum of the upper lip with cleft pal- 19. A G1P1 woman presents with her 5-month-old
FULL-LENGTH EXAMS

ate, down-slanting palpebral ssures, and a daughter to the pediatricians ofce. The infant
poplastic mandible. Relevant laboratory ndings was delivered by cesarean section at 37 weeks
are: gestation because of breech presentation.
There were no complications during delivery.
Na+: 140 mEq/L
The infant has reached all developmental
Cl: 104 mEq/L
milestones as expected and is up to date with
Mg2+: 1.1 mg/dL
immunizations. However, on physical exami-
K+: 4.0 mEq/L
nation the pediatrician senses a palpable
Ca2+: 7.0 mg/dL
click about the infants hip joint when he
HCO3: 24 mEq/L
gently adducts the hip and directs a posterior
PO4: 2.0 mg/dL
force. What is the most appropriate treatment
Glucose: 97 mg/dL
at this time?
Creatinine: 0.6 mg/dL
Blood urea nitrogen: 13 mg/dL (A) A exion-abduction orthosis
(B) A removable extension splint
What chromosomal segment has been impli-
(C) Closed reduction and spica cast
cated in the constellation of these signs and
(D) No treatment is necessary
symptoms?
(E) Surgical correction
Test Block 5

(A) 7q11
(B) 8q24 20. After losing his wife to ovarian cancer 5 weeks
(C) 11p13 ago, a 58-year-old man with no psychiatric his-
(D) 15q11 tory presents to his primary care physician with
(E) 17p13 a 2.3-kg (5-lb) weight loss. He complains of not
Test Block 5 Questions 613

eating or sleeping well since his wife was ad- (A) Captopril
mitted to inpatient hospice. He reports hearing (B) Cocaine
the voice of his wife as he lies in bed attempt- (C) Diazepam
ing to fall asleep. His physical examination is (D) Lithium
unremarkable, but he appears tearful through- (E) Verapamil
out the encounter. Which of the following is
the best next step in management? 23. A 36-year-old woman presents at the primary
care clinic for a routine health check-up. She
(A) Administer 10 cycles of electroconvulsive
has no complaints aside from occasional low-
therapy
back pain and states that she has been gener-
(B) Offer supportive reassurance
ally healthy. She smokes cigarettes on occasion
(C) Prescribe olanzapine
and drinks 45 glasses of wine each week. She
(D) Prescribe zolpidem
is currently on no medications. Her family his-
(E) Refer to an inpatient psychiatric facility
tory is signicant for type 2 diabetes mellitus
and endometrial cancer at age 46 years in her
21. The mother of a 7-year-old girl brings her to
paternal grandmother, as well as colon cancer
the pediatrician because she is concerned that
in her father at 50 years of age and in her half-
her daughter has already begun to develop
brother when he was 44. Her physical exami-
breasts. She has been an otherwise healthy
nation, including digital rectal examination, is
child with no signicant illnesses or surgeries.
normal. Which of the following recommenda-
The mother states that she went through me-
tion is most appropriate at this time?
narche at age 14 years, and her other two
daughters began puberty at around age 12 (A) Colonoscopy beginning at age 50 years,
years. On physical examination the pediatri- and every 5 years after that
cian notes the girl has Tanner stage III breasts, (B) Immediate colonoscopy
but lacks pubic hair. Laboratory evaluation re- (C) Immediate CT scan of the abdomen
veals elevated gonadotropin-releasing hor- (D) Immediate double-contrast barium enema
mone, luteinizing hormone, and follicle-stimu- (E) Yearly digital rectal examinations and sig-
lating hormone levels. Which of the following moidoscopy every 23 years, beginning at

FULL-LENGTH EXAMS
is the most likely diagnosis? age 44 years
(A) Adrenal tumor
24. A 35-year-old homeless woman is brought to
(B) Central precocious puberty
the emergency department after being found
(C) McCune-Albright syndrome
unconscious in the street. Her pupils are 2 mm
(D) Ovarian tumor
dilated, respiratory rate is 6/min, and she has
(E) Primary hypothyroidism
extensive scarring across her forearms. It is as-
sumed that she has taken a drug of some type,
22. A 65-year-old man is brought to the emergency
and a urine toxicology screen is ordered.
department in a confused state following a sei-
Which of the following statements is true of
zure in public. Witnesses report the seizure as
this womans condition?
occurring suddenly and lasting at least 23
minutes. A call to his family reveals that he has (A) Lorazepam is the treatment of choice for
no major medical problems other than hyper- this womans condition
tension and high anxiety, and that both are (B) The fatal consequence of this condition is
well controlled by his medications. On exami- myocardial infarction
nation the patients tongue is unharmed, al- (C) The substance that caused her condition
though his pants are covered in fresh urine and only binds effectively to receptors
Test Block 5

feces. He is oriented to person, but not place (D) Tolerance to the euphoric effects of this
or time, and has no recollection of what oc- substance develops slowly
curred. Physical examination is otherwise un- (E) Withdrawal from this substance is life-
remarkable. Withdrawal from which drug is threatening
most likely responsible for this patients sei-
zure?
614 Section II: Full-Length Examinations Questions

25. A 5-year-old boy with an itchy rash is brought tion greatly reduced the incidence of this con-
to the pediatrician by his mother. She states dition?
that the rash rst appeared 3 weeks ago. He
(A) Diphtheria-tetanus-pertussis
had a similar rash on his face when he was
(B) Haemophilus inuenzae type B
about a year old, but that resolved. This rash
(C) Measles-mumps-rubella
rst appeared on his left arm and is now on the
(D) Pneumococcus
backs of his knees. He has otherwise been well.
(E) Varicella
She thought he was allergic to their new laun-
dry detergent, but the rash has failed to resolve 27. An 83-year-old woman with a history of hyper-
despite changing back to their previous deter- tension, emphysema, and a 90-pack-year smok-
gent. He had bronchiolitis as an infant, but has ing history presents to a gastroenterologist for a
otherwise been healthy. On examination the follow-up appointment. Six weeks ago she was
lesions shown in the image are also present on seen in the emergency department because of
the exor surfaces of both proximal upper ex- acute severe lower abdominal pain that began
tremities and the popliteal fossae bilaterally. earlier that morning. The patient recalled hav-
The diaper area is spared. The plaques are ery- ing had several episodes of painless bright red
thematous and dry, with scattered papules and blood per rectum during the weeks preceding
excoriations. Which of the following is the her visit. The patient complained of fever,
most likely diagnosis? chills, nausea, and emesis accompanying the
pain. CT scans of the abdomen and pelvis re-
vealed an abscess, and the patient was subse-
quently begun on oral antibiotics. Which of
the following most likely could have reduced
the likelihood of this condition?
(A) Blood pressure control
(B) Exercise
(C) High-ber diet
(D) Reduction of caffeine intake
FULL-LENGTH EXAMS

(E) Smoking cessation

Reproduced, with permission, from Wolff K, Johnson RA, 28. A 58-year-old woman presents to her primary
Surmond D. Fitzpatricks Color Atlas & Synopsis of Clini- care physician for her annual health mainte-
cal Dermatology, 5th edition. New York: McGraw-Hill, nance examination. Overall she feels well, but
2005: Figure 2e-12. she does complain of pain and stiffness in her
(A) Allergic contact dermatitis hands that has progressively worsened over the
(B) Atopic dermatitis past year. She notes the pain is worst at the end
(C) Pityriasis rosea of the day. She has had some relief with ibu-
(D) Psoriasis profen. Her ngers are tender to palpation at
(E) Seborrheic dermatitis the proximal and distal interphalangeal joints
bilaterally. She also has limited exion at these
26. A 2-year-old boy is brought to the emergency joints. Which of the following is the most likely
department by his parents. His mother reports diagnosis?
that he developed a fever to 38.8C (101.9F) (A) Calcium pyrophosphate deposition disease
today and began to complain of throat pain (B) Gout
while eating. Shortly after dinner, he began to (C) Osteoarthritis
Test Block 5

drool and had noisy breathing. The patient is (D) Psoriatic arthritis
sitting on the examining table with his neck (E) Rheumatoid arthritis
hyperextended and chin protruding. His lung
examination is normal without rales, rhonchi, 29. A 26-year-old G1P0 woman at 37 weeks gesta-
or wheezing. The advent of what immuniza- tion presents to her obstetricians ofce for a
Test Block 5 Questions 615

routine prenatal check-up. The fetal head is measured prior to extubation was collected from
palpated by the obstetrician in the upper abdo- charts, and was analyzed to see which data dif-
men with Leopold maneuvers, and a breech fered between the successfully and unsuccess-
presentation is suspected. Cervical examina- fully extubated patients. What type of study is
tion reveals a cervix that is posterior, moderate described?
in consistency, and 20% effaced. The fetus is
(A) Case-control
not yet engaged in the pelvis, and the amniotic
(B) Clinical trial
sac is intact. Breech presentation is then con-
(C) Correlation
rmed by ultrasound. A biophysical prole of
(D) Prospective cohort
the fetus is performed, with a reactive nonstress
(E) Retrospective cohort
test and an overall score of 10/10. Which of the
following is the best next step in management?
32. A 24-year-old man is brought to the emergency
(A) Elective cesarean section delivery department after suffering blunt trauma to his
(B) Emergent cesarean section delivery chest in a motor vehicle accident. His respira-
(C) Expectant management for vaginal breech tory rate is 32/min, heart rate is 125/min, and
delivery blood pressure is 80/45 mm Hg, with a de-
(D) Induction of labor with intravaginal prosta- crease to 65/40 mm Hg on inspiration. Physi-
glandin E2 for a vaginal delivery cal examination reveals decreased heart sounds
(E) Offer the patient an external cephalic ver- and a pericardial friction rub. Which other
sion symptom would the physician expect to see as
part of the patients presentation?
30. A 10-month-old boy is brought to the clinic
(A) Bounding pulse
with cough, fever, and wheezing. The parents
(B) Bradycardia
report that he has had recurrent episodes of
(C) Holosystolic murmur
pneumonia, otitis media, and sinusitis during
(D) Hypertension
the past 4 months. His temperature is 39C
(E) Jugular venous distention
(102.2F), heart rate is 95/min, respiratory rate
is 30/min, blood pressure is 110/65 mm Hg,
33. A 68-year-old African-American woman pre-

FULL-LENGTH EXAMS
and crackles are auscultated on the left chest.
sents to her physician with palpitations, breath-
On physical examination no tonsillar tissue is lessness, and fatigue of 2 months duration.
seen, and no lymph nodes are palpated. X-ray She also reports an unintentional 6.8-kg (15.0-
of the chest shows left lower lobe consolida- lb) weight loss during this period, which she
tion. Further work-up yields a normal absolute ascribes to stress at her workplace. The patient
lymphocyte count and low levels of IgG, IgA, is currently taking captopril for high blood
IgM, and IgE, and ow cytometry demon- pressure, amiodarone for arrhythmias, and
strates the absence of circulating B lympho- omeprazole for gastroesophageal reux disease.
cytes. Which of the following is the most likely She is a nonsmoker. Her temperature is 36.8C
underlying immune deciency in this patient? (98.2F), blood pressure is 125/80 mm Hg, and
(A) Brutons agammaglobulinemia heart rate is 86/min and regular. Which of the
(B) Common variable immunodeciency following is the best next step in management?
(C) DiGeorges syndrome (A) Check thyroid-stimulating hormone and
(D) IgA deciency thyroid hormone levels
(E) Severe combined immunodeciency (B) Provide reassurance and follow-up in
6 months
31. In a study of predictive factors for failing extuba-
(C) Refer the patient to a psychiatrist for possi-
Test Block 5

tion in the intensive care unit, patients who


ble anxiety disorder
failed extubation (and consequently had to be
(D) Substitute nifedipine for captopril
reintubated) were matched with intensive care
(E) Substitute procainamide for amiodarone
unit patients who were successfully extubated.
For each patient, the physiologic data routinely
616 Section II: Full-Length Examinations Questions

34. A 28-year-old man visits an infectious disease


specialist after being told he is HIV-positive.
His CD4+ cell count is 400/mm and his HIV
viral load is 126,000 copies/mL. He believes he
was infected 2 months earlier while sharing a
needle to inject drugs. He reports no symp-
toms, denying lymphadenopathy, fevers, chills,
and night sweats; he says he feels ne. Which
of the following is the most appropriate next
step?
(A) Begin highly active antiretroviral therapy
immediately because his CD4+ cell count Reproduced, with permission, from Lichtman MA, Beut-
is falling rapidly and is unlikely to rebound ler E, Kipps TJ, Seligsohn U, Kaushansky K, Prchal JT.
(B) Begin trimethoprim-sulfamethoxazole as Williams Hematology, 7th edition. New York: McGraw-
Hill, 2006: Figure XXV-41.
prophylaxis against Pneumocystis jiroveci
pneumonia (A) Chronic obstructive pulmonary disease
(C) Retest for HIV; the patient may have (B) Depression
cleared the infection (C) Gastrointestinal bleeding
(D) Retest for HIV; the patient may have had a (D) Intracranial aneurysm
false-positive result (E) New onset coronary artery disease
(E) Schedule a follow-up appointment in
1 month to retest CD4+ cell count and vi- 36. The on-call intern in the newborn nursery is
ral load, and instruct the patient to call if paged because a baby girl born at 34 weeks
he experiences any symptoms gestation is experiencing bilious vomiting. The
newborn was reportedly doing well since deliv-
35. A 68-year-old patient presents to her primary ery 16 hours ago and had been tolerating oral
care practice complaining of easy fatigue and feeding without complication. The pregnancy
shortness of breath over the past year. Six was complicated by mild polyhydramnios.
FULL-LENGTH EXAMS

months ago, her identical twin died of an in- There is no parental family history of gastroin-
tracranial hemorrhage. Her history is notable testinal abnormalities. The newborn has a
only for recurrent epistaxis. She is postmeno- heart rate of 117/min, blood pressure of 65/48
pausal and takes an iron supplement. She mm Hg, respiratory rate of 42/min, and tem-
claims she is eating well, sleeping well, and is perature of 37.0C (98.6F). Physical examina-
at peace with her sisters passing. Physical ex- tion is unremarkable except for mild jaundice.
amination reveals the lesions shown in the im- The abdomen is not distended. A plain lm of
age and pale conjunctiva. Which of the follow- the abdomen shows two air-lled structures in
ing is the most likely cause of this womans the upper abdomen, both showing air and
symptoms? uid. The rst step in management of this pa-
tient should involve which of the following
treatment strategies?
(A) Echocardiogram
(B) Gastrostomy tube placement
(C) Immediate surgical correction
(D) Nasogastric decompression
(E) Radiologic studies of the spine and chest
Test Block 5

37. A 28-year-old man is brought to the emergency


department by his friend after having a con-
frontation with his girlfriend. He has a stab
wound in the anterior neck just lateral to the
Test Block 5 Questions 617

left lateral border of the thyroid cartilage. His 39. A 38-year-old woman presents for a routine
past medical history is unremarkable. His pulse health maintenance examination. She has no
is 88/min, blood pressure is 138/85 mm Hg, complaints and physical examination is within
and respiratory rate is 20/min. On examination normal limits. Relevant laboratory ndings in-
it is difcult to assess whether the platysma has clude a calcium level of 11.8 mg/dL, magne-
been penetrated. Subcutaneous air is palpated. sium level of 2.9 mg/dL, and intact parathyroid
The trachea is midline. What is the most ap- hormone level of 75 pg/mL (normal: 1165
propriate management for this injury? mg/mL). On further screening her mother,
brother, and 10-year-old son also have elevated
(A) Arteriography
serum calcium levels. The patient has no per-
(B) Esophagoscopy
sonal or family history of malignancy. Which
(C) Oral and intravenous contrast CT of the
of the following is the most likely diagnosis in
neck
this patient?
(D) Surgical exploration
(E) Tracheobronchoscopy (A) Familial hypocalciuric hypercalcemia
(B) Jansens disease
38. A 55-year-old Asian male philosophy professor (C) Primary hyperparathyroidism
presents to his primary care physician with (D) Type I multiple endocrine neoplasia
months of intermittent watery diarrhea and ep- (E) Type IIB multiple endocrine neoplasia
isodes of wheezing and facial ushing some-
times precipitated by consuming alcohol. He 40. A middle-aged woman drives herself to the
has no past medical history or recent travel his- emergency department. When she is seen by
tory. His temperature is 37.5C (99.5F), blood the physician, she tells him that her chest pain
pressure 130/80 mm Hg, pulse 80/min, and re- has been getting worse over the past day. She
spiratory rate 22/min. Physical examination re- says she is starting to get this chest pain at rest
veals some facial ushing, moderate elevation and it is not responding to the sublingual nitro-
in jugular venous pressure, II/VI systolic ejec- glycerin like it used to. Recognizing that this
tion murmur at the left upper sternal border, could be an emergent situation, the physician
wheezes in the lungs, and mild lower extremity orders an immediate ECG. What ECG

FULL-LENGTH EXAMS
edema. Complete blood cell count and basic changes would require sending this patient im-
metabolic panel with extended panel were un- mediately to the revascularization laboratory?
remarkable. CT of the abdomen demonstrated
(A) Delta waves
a thickened desmoplastic reaction in the mes-
(B) Flipped T waves
entery surrounding an area of terminal ileum
(C) Low-voltage ECG
and three possible lesions in the liver. Which
(D) New left bundle-branch block
of the following is most likely to conrm the
(E) ST depressions
diagnosis?
(A) Serum antigliadin antibody titer
(B) Serum cortisol level
(C) Serum gastrin level
(D) Urine 5-hydroxyindoleacetic acid level
(E) Urine vanillylmandelic acid level
Test Block 5
618 Section II: Full-Length Examinations Questions

E X T E N D E D M ATC H I N G a blood urea nitrogen level of 17 mg/dL and


creatinine of 1.8 mg/dL. Urinalysis shows 4+
The response options for the next 2 items are blood and 2+ protein.
the same. Select one answer for each item
in the set. The response options for the next 2 items are
the same. Select one answer for each item
For each patient with glomerular disease, select the in the set.
most likely diagnosis.
(A) Alports syndrome For each patient with nipple discharge, select the
(B) Amyloidosis most likely diagnosis.
(C) Cryoglobulinemia (A) Fibroadenoma
(D) Focal glomerular sclerosis (B) Fibrocystic changes
(E) Goodpastures syndrome (C) Galactocele
(F) IgA nephropathy (D) Intraductal papilloma
(G) Lupus nephritis (E) Invasive ductal carcinoma
(H) Membranoproliferative glomerulonephri- (F) Invasive lobular carcinoma
tis (G) Invasive papillary carcinoma
(I) Minimal change disease (H) Pagets disease of the breast
(J) Polyarteritis nodosa (I) Pregnancy
(K) Postinfectious glomerulonephritis (J) Prolactinoma
(L) Wegeners granulomatosis
43. A 44-year-old woman presents to the emer-
41. A 46-year-old man presents to the emergency gency department because of serosanguinous
department complaining of leg swelling, pink- discharge from her right nipple. She noticed a
colored urine, and headache. He was in good discharge this morning, but it was clear at that
health until 3 months ago, when he noticed point and not bloody. When she removed her
several days of pink-colored urine following an clothes this evening, however, her undercloth-
upper respiratory tract infection. Both the in- ing had some blood on it. When she pressed
FULL-LENGTH EXAMS

fection and the urine discoloration resolved on the right breast, there was more bloody dis-
spontaneously, but after about 6 weeks his uri- charge. At that point she became panicked and
nary symptoms recurred, this time accompa- rushed to the emergency department. The pa-
nied by ank pain. His symptoms again re- tient denies breast pain or tenderness. She re-
solved, and he was asymptomatic until today. ports a past obstetric history of two uncompli-
His blood pressure is 160/94 mm Hg and he cated pregnancies with spontaneous vaginal
has 2+ edema up to his shins bilaterally. Uri- delivery, and an age of 12 years at menarche.
nalysis shows 20 RBCs/hpf and 2+ protein. An- Her last period was 3 months ago and she be-
tinuclear antibody and antiglomerular base- lieves she is experiencing menopause. Family
ment membrane antibody titers are negative, history is negative for neoplasm of any kind.
and serum complement is normal. There are no palpable masses on breast exami-
nation. However, when pressed, serosan-
42. A 32-year-old man is being evaluated for new- guinous discharge is expressed from the right
onset hypertension. His only other medical nipple.
problem is difculty with hearing, which has
been worsening since childhood. Family his- 44. A 44-year-old woman presents to the clinic be-
tory is signicant for an uncle with kidney cause of bloody discharge from her right nip-
problems, but the specics are not known. His
Test Block 5

ple. She says that at rst it was clear, but after a


blood pressure is 158/94 mg Hg and heart rate couple of weeks it seemed pink, then frankly
is 64/min. On physical examination the patient bloody. The bleeding resolved, but this symp-
is in no acute distress. He has diminished audi- tom prompted her to examine her breasts, and
tory acuity bilaterally. Laboratory studies show
Test Block 5 Questions 619

at that point she noticed a lump in her breast, 45. A 72-year-old woman is brought to the emer-
close to the axilla. Her family history is signi- gency department because of altered mental sta-
cant for a mother and aunt with ovarian can- tus and combative behavior. Two weeks ago the
cer. On physical examination she is in no acute patient was admitted to a nursing home after
distress. She has a 1-cm lump in the right up- sustaining a hip fracture; history also reveals
per outer quadrant. The lump is hard and im- hypertension and recent urinary tract infec-
mobile, although it is regular in shape. Palpa- tions. Although normally calm and loving
tion of the lump elicits no discharge from the per her familys report, the patient has grown
nipple. restless and combative with staff and residents
in past days.
The response options for the next 2 items are
46. A 38-year-old woman with bipolar disorder is
the same. Select one answer for each item
brought to the emergency department because
in the set.
of confusion. She appears to be apathetic with
For each of the clinical scenarios listed, select the a at affect and highly distractible; she grows
most likely diagnosis. lethargic, hypotensive, and unarousable during
her examination. Her lithium level is >3.5
(A) Brief psychotic disorder mEq/L (normal: 0.40.8 mEq/L).
(B) Creutzfeldt-Jakob disease
(C) Delirium due to a general medical condi-
tion
(D) Dementia, Alzheimers type
(E) Depression
(F) Irritable mania
(G) Normal age-associated cognitive decline
(H) Parkinsons disease
(I) Pseudodementia
(J) Psychotic disorder resulting from a general
medical condition

FULL-LENGTH EXAMS
(K) Substance intoxication delirium
(L) Vascular dementia

Test Block 5
620 Section II: Full-Length Examinations Answers

AN S W E R S

1. The correct answer is E. This patient has had clusion, it seems to best t the clinical picture
the onset of nephrotic syndrome. Patients with described above. The Diagnostic and Statisti-
RA are at risk for multiple types of renal dis- cal Manual of Mental Disorders, Fourth Edi-
ease, due both to their underlying disease as tion, Text Revision criteria for conversion dis-
well as the therapies used to manage RA. In order are as follows: (1) neurologic function is
this patient with membranous nephropathy, lost; (2) psychological factors are associated
the most likely etiologic agent is penicillamine. with the symptom or decit because the initia-
Up to 7% of patients treated with penicillamine tion or exacerbation of the symptom or decit
will develop membranous nephropathy, and is preceded by conicts or other stressors. In
proteinuria usually develops within 612 the question stem, the patient feels conict be-
months. Cessation of therapy will usually result cause she was unable to visit her mother be-
in resolution of proteinuria. Gold, another dis- cause of her husbands controlling nature.
ease-modifying antirheumatic drug, can also When her mother died, the patient became
cause membranous nephropathy. unable to walk. This allows the patient to ex-
press her resentment of her controlling hus-
Answer A is incorrect. Aspirin or nonsteroi-
band by making herself unable to cooperate
dal anti-inammatory drugs (NSAIDs) are very
with him.
commonly used in the management of patients
with RA and may cause renal disease. NSAIDs Answer B is incorrect. The patient may very
can cause nephrotic syndrome due to mini- well be depressed. One could even argue that
mal change disease. Another NSAID-induced her conversion disorder has occurred as a way
renal complication is acute interstitial nephri- for her mind to alleviate her depression. How-
tis. Chronic aspirin use may increase the risk of ever, her current presentation is best explained
chronic renal failure, causing analgesic nephrop- by conversion disorder.
athy.
Answer C is incorrect. Hypochondriasis is de-
Answer B is incorrect. Corticosteroids are ned by a preoccupation with fears of having,
FULL-LENGTH EXAMS

also used in the management of RA, but do or by the idea that one has a serious disease.
not cause nephrotic syndrome. Corticoster- These feelings are based on the persons misin-
oids may actually be used to treat nephrotic terpretation of bodily symptoms.
syndrome or other renal pathology, including
Answer D is incorrect. Multiple sclerosis is
rheumatoid vasculitis.
characterized by multiple neurologic events
Answer C is incorrect. Cyclophosphamide is (e.g., blurry vision, bladder incontinence, and
an alkylating agent and a very potent immuno- neurologic leg pain) which can be linked to
suppressive drug. An important genitourinary white matter tracts in the brain. Moreover,
complication of cyclophosphamide therapy these events must occur in different parts of
is acute hemorrhagic cystitis, which presents the body at multiple points in time. MRIs in
with hematuria. patients with multiple sclerosis usually show
multiple hyperintense white matter lesions.
Answer D is incorrect. Methotrexate is another
Thus, this would be an unlikely diagnosis.
disease-modifying antirheumatic drug that is
commonly used to treat patients with RA. How- Answer E is incorrect. Somatization disorder
ever, it is not typically associated with renal presents with a history of many physical com-
toxicity. Its primary adverse effects include gas- plaints beginning before age 30 years, which
Test Block 5

trointestinal upset, oral ulcerations, and dose- occur over a period of several years. The pa-
dependent liver function abnormalities. tient must have at least four pain symptoms in-
volving multiple sites, at least two unexplained
2. The correct answer is A. Although the diagno- gastrointestinal symptoms such as nausea and
sis of conversion disorder is a diagnosis of ex- indigestion, at least one sexual complaint
Test Block 5 Answers 621

and/or menstrual complaint, and at least one athy, or dilated cardiomyopathy (systolic dys-
pseudoneurologic symptom, such as blindness function), as described in this case. NSAIDs
or inability to walk, speak, or move. They must have not been proven to improve mortality in
result in the patient seeking frequent medical patients with heart failure. In contrast, there are
treatment, or in signicant impairment in so- several mechanisms by which they may exacer-
cial, occupational, or other important areas of bate the condition and would be contraindi-
functioning. cated in this patient. NSAIDs may worsen after-
load by inhibiting prostaglandin synthesis
3. The correct answer is C. Conns syndrome, or (eliminating their effects on vasodilation) and by
primary hyperaldosteronism, results from an counteracting the benets of angiotensin-con-
adrenal adenoma. It is manifested by hyperten- verting enzyme inhibitors.
sion, hypokalemia, hypernatremia, low plasma
Answer B is incorrect. A number of -blockers,
renin, and increased plasma aldosterone. In
including carvedilol and metoprolol, have
this case, an episode of viral gastroenteritis
been proven to improve survival in patients
likely resulted in gastrointestinal loss of potas-
with heart failure and without other contrain-
sium, aggravating the chronic hypokalemia
dications (including bradycardia, hypotension,
seen with this disorder. The patients syncopal
atrioventricular conduction abnormalities, and
episode could have been caused by a hy-
asthma).
pokalemia-induced ventricular arrhythmia.
Answer C is incorrect. Angiotensin-converting
Answer A is incorrect. 21-Hydroxylase de-
enzyme inhibitors are the most effective drugs
ciency is a congenital salt-wasting syndrome.
for improving survival and slowing the progres-
Hypertension and hypernatremia would not be
sion of heart failure due to systolic dysfunction
seen.
regardless of severity.
Answer B is incorrect. Addisons disease is in-
Answer D is incorrect. Exercise training is as-
correct because this entails adrenal insufciency
sociated with both improved morbidity and
and thus the electrolyte disturbances (hypona-
mortality in patients with heart failure.
tremia with hyperkalemia) would be the oppo-
site of those seen here. Hyperpigmentation of Answer E is incorrect. Spironolactone is a

FULL-LENGTH EXAMS
the skin, weight loss, weakness, and eosinophilia competitive inhibitor of aldosterone at the min-
are often seen in adrenal insufciency. eralocorticoid receptor that has been shown to
improve survival in patients with symptomatic
Answer D is incorrect. Cushings disease refers
heart failure. The mechanism may be by in-
to those cases of Cushings syndrome caused by
creased potassium levels or by inhibiting the
a pituitary adenoma. It would be characterized
effects of aldosterone on receptors in the heart.
by increased cortisol.
Answer E is incorrect. Twenty-four-hour urine 5. The correct answer is C. A pseudocyst is a col-
free cortisol levels would be expected to be lection of pancreatic secretions with high con-
elevated in endogenous causes of Cushings centrations of digestive enzymes and a wall
syndrome, and we have no history to suggest consisting of brous/granulation tissue. Unlike
that this patient has taken exogenous glucocor- a true pancreatic cyst, a pseudocyst does not
ticoids (the most common cause of Cushings have an epithelial lining. Pseudocysts occur af-
syndrome). ter an acute attack of pancreatitis in approxi-
mately 10% of patients. The differential diag-
4. The correct answer is A. Based on the patients nosis includes a variety of other pancreatic
symptoms (comfortable at rest but symptoms cystic neoplasms. The diagnosis is usually
Test Block 5

with less-than-normal activity), he can be classi- made by ultrasound or CT. The image shows a
ed as having New York Heart Association class large pancreatic pseudocyst abutting the stom-
III heart failure. Heart failure can be due to re- ach and likely causing abdominal pain. Pan-
strictive cardiomyopathy (sarcoid and amyloid), creatic pseudocysts are typically managed by
hypertrophic (diastolic dysfunction) cardiomyop-
622 Section II: Full-Length Examinations Answers

observation for 6 weeks prior to surgical inter- reduction, cannot be calculated using a case-
vention because spontaneous resolution is control study without additional data about the
most likely to occur in the rst 6 weeks. Be- underlying population.
yond the 6-week period, the likelihood of spon-
Answer D is incorrect. Prevalence cannot be
taneous resolution diminishes, and the rate of
calculated using a case-control study because
complications increases.
it is a property of the underlying population.
Answer A is incorrect. This patients presenta-
Answer E is incorrect. The relative risk, or the
tion and CT ndings are inconsistent with aor-
risk of developing the disease in those exposed
tic dissection. These patients typically present
versus those not exposed to a risk factor, can-
with abrupt onset of severe, tearing back pain
not be calculated in case-control studies. Be-
(in dissection distal to the left subclavian) or
cause the outcome has occurred with a case-
anterior chest pain (in ascending aortic dissec-
control study, one cannot measure the risk
tion).
for developing the outcome directly because
Answer B is incorrect. Cholecystitis tends to the prevalence of a risk factor in the underly-
present with intermittent right upper quadrant ing population is not known. However, when
pain, often following fatty meals. Also, uncom- the incidence of the disease in the underlying
plicated cholecystitis would not raise the serum population is low, the odds ratio approximates
amylase levels or have CT ndings like those the relative risk.
shown in the image.
7. The correct answer is B. The patient de-
Answer D is incorrect. This patients presen-
scribed in this vignette has a classical presenta-
tation is not consistent with duodenal ulcer
tion of Kawasaki disease. To diagnose Kawasa-
perforation. Duodenal ulcer perforation most
kis disease, a patient must have fever for >5
frequently occurs in patients with a history of
days in addition to at least four of the following
peptic ulcer symptoms and is characterized by
criteria: bilateral nonexudative conjunctivitis,
acute onset of severe, diffuse abdominal pain.
mucosa changes of the oropharynx, changes of
Free air would have been seen on CT.
the peripheral extremities such as edema and/
Answer E is incorrect. This patient is present- or erythema of the hands or feet, rash, and cer-
FULL-LENGTH EXAMS

ing with persistent abdominal pain and CT vical lymphadenopathy (unilateral with at least
ndings suggestive of pseudocyst, not pancrea- one node >1.5 cm). Patients with Kawasakis
titis. disease are treated with intravenous immuno-
globulin and aspirin. Before discharge, all pa-
6. The correct answer is C. The study described tients with Kawasakis disease should undergo
is a case-control study in which cases (patients an echocardiogram to examine for the pres-
with disease) are paired with controls (without ence of coronary artery aneurysms, a complica-
disease) and information is retrospectively col- tion affecting up to 40% of untreated patients.
lected about past exposure to possible etiologic
Answer A is incorrect. Although patients with
factors. The odds ratio is determined as the ra-
Kawasakis disease may develop coronary artery
tio of the odds of exposure in those with disease
aneurysms, pericarditis, myocarditis, or valvu-
to the odds of exposure in those without dis-
lar regurgitation, they have not been shown to
ease in case-control studies. This statistic is
be at increased risk for the development of any
mathematically identical to the ratio of odds of
form of cardiomyopathy.
disease in exposed persons to the odds of dis-
ease in unexposed persons. Answer C is incorrect. Aseptic meningitis can
be associated with Kawasakis disease, but en-
Answer A is incorrect. Attributable risk, also
Test Block 5

cephalitis is not.
called the risk difference, cannot be calculated
using a case-control study. Answer D is incorrect. Epidermal necrolysis is
a feature of toxic epidermal necrolysis, which
Answer B is incorrect. Number needed to
is related to Stevens-Johnson syndrome, where
treat, which is the inverse of the absolute risk
a skin rash results from an adverse drug reac-
Test Block 5 Answers 623

tion and progresses to skin sloughing. By de- 9. The correct answer is E. This patient has a
nition, this involves the mucous membranes. classic presentation of Pneumocystis jiroveci
Although Kawasakis disease also involves the pneumonia, characterized by low-grade fever,
oral mucosa and can be associated with peel- progressive dyspnea, hypoxia, increased arte-
ing of the ngertips, there is no skin sloughing. rial-alveolar gradient, and x-ray of the chest
with diffuse bilateral inltrates. First-line ther-
Answer E is incorrect. Although mucous
apy for is trimethoprim-sulfamethoxazole. If
membrane involvement may be severe in pa-
the patients arterial oxygen pressure is <70
tients with Kawasakis disease, the injected
mm Hg, oral prednisone should be added. The
pharynx and strawberry tongue are not linked
other regimens would not be effective against
to an infectious etiology; therefore, patients are
P. jiroveci pneumonia.
not at increased risk for retropharyngeal ab-
scess development. Answer A is incorrect. Ampicillin + gentami-
cin is a possible treatment choice for entero-
Answer F is incorrect. Kawasakis disease rep-
coccal septicemia, depending on sensitivity,
resents a multisystem acute vasculitis with
which would not present with cough, but
potential risk of developing cardiac compli-
rather with fever and low blood pressure.
cations. However, there is no evidence that
children with Kawasakis disease are at an in- Answer B is incorrect. Intravenous clindamy-
creased risk of stroke or other neurologic com- cin has been used in combination with prim-
plications. aquine for treatment of P. jiroveci pneumonia,
but it is not the rst-line therapy.
8. The correct answer is C. Shaken baby syn-
Answer C is incorrect. Isoniazid + pyrazin-
drome tends to present with a suspicious-
amide + rifampin + ethambutol is a possible
sounding history involving nonaccidental
regimen for treatment of Mycobacterium tu-
trauma. The baby will have retinal hemor-
berculosis, which can also present with a fever
rhages, subdural hematomas on CT, and white
and nonproductive cough. However, the most
matter change may be seen on MRI. When
common abnormality on x-ray of the chest is
abuse is suspected, a full skeletal survey must
hilar adenopathy.
be performed to look for fractures, and child

FULL-LENGTH EXAMS
protective services must be notied. Answer D is incorrect. A third-generation cepha-
losporin + doxycycline is a possible treatment for
Answer A is incorrect. A patient with Group
non P. jiroveci community-acquired pneumonia,
B streptococcal meningitis would present with
which would most likely present with a produc-
fever and irritability and would have a positive
tive cough and lobar opacity on x-ray of the chest.
lumbar puncture.
Answer B is incorrect. In obstructive hydro- 10. The correct answer is A. Patients who present
cephalus, funduscopy would reveal a bulging after a high-speed motor vehicle crash or rapid
disc with blurry margins and the absence of deceleration injury must be considered for aor-
subdural hematomas. tic disruption. The diagnosis of aortic disruption
is supported in this case by the radiographic
Answer D is incorrect. Type 1 diabetes mel-
ndings, including rib fractures, which are often
litus does not typically present in a newborn.
seen along with aortic disruption. Aortography is
However, patients with long-standing diabetes
the gold standard for diagnosis of aortic disrup-
can develop retinal hemorrhages and white
tion and is used to conrm a suspected diagno-
exudates.
sis. In experienced hands, a sensitivity of 67%
Answer E is incorrect. von Willebrands dis- 100% and a specicity of 98%100% have been
Test Block 5

ease can lead to subdural hematomas; how- achieved using aortography to diagnose an aor-
ever, the patient will most likely have a history tic disruption. Although less invasive imaging
of nosebleeds or increased bruising. This sce- techniques are currently being studied (includ-
nario is more suspicious of child abuse. ing CT scan and transesophageal echocardiog-
624 Section II: Full-Length Examinations Answers

raphy), angiography remains the most reliable adenomas, malignancies such as multiple my-
means of diagnosis. eloma and lung cancer, granulomatous dis-
ease, and vitamin D intoxication.
Answer B is incorrect. MRI is not indicated
for the diagnosis of aortic disruption because Answer D is incorrect. Elevated phosphate
of the lengthy time involved in acquiring the levels are not seen in Pagets disease. The most
image, as well as its limitations in patients with frequent cause of hyperphosphatemia is renal
pacemakers and other metallic implants. failure.
Answer C is incorrect. Transthoracic echocar- Answer E is incorrect. Positive rheumatoid
diography is a noninvasive method for evalu- factor is not commonly associated with Pagets
ating cardiac anatomy and function. This im- disease. It is found in patients with RA.
aging modality is limited, however, because
thoracic structures such as ribs and lungs do 12. The correct answer is A. Gastroesophageal
not allow for full evaluation of cardiac anatomy reux disease (GERD) is a common disorder
and the aorta. Due to its limitations, transtho- that generally becomes evident during the
racic echocardiography would not be the best rst few months of life, peaking at about 4
test to work up a diagnosis of aortic disruption. months of age; most patients will have resolu-
tion of symptoms by 1224 months of age.
Answer D is incorrect. Upper endoscopy is
Common clinical manifestations of GERD
useful for visualizing the upper gastrointestinal
include postprandial regurgitation with signs
tract; however, it has no utility in the diagno-
of esophagitis (irritability, arching, gagging,
sis of an aortic disruption. This patient shows
choking) and resultant failure to thrive. Be-
no clinical signs of acute upper gastrointestinal
cause most cases of GERD will completely
tract injury (i.e., bleeding and/or mediastinal
resolve by 24 months of age, management is
emphysema) and thus does not require endos-
aimed at conservative therapy with parental
copy.
reassurance. Small, frequent feedings; thick-
Answer E is incorrect. A ventilation-perfusion ened feedings; and keeping the infant upright,
scan would be helpful in the diagnosis of a pul- or elevating the head of the bed after feedings,
monary embolism, but it has no utility in diag- are often sufcient to provide signicant im-
FULL-LENGTH EXAMS

nosing an aortic disruption. This patient shows provements in symptoms. Note: the lab nd-
no clinical signs of pulmonary embolism (i.e., ings are within normal limits.
hypoxia, dyspnea, tachycardia, and/or tachy-
Answer B is incorrect. Although antacids are
pnea) and thus does not require a ventilation-
a commonly used antireux medication, they
perfusion scan.
are not routinely recommended for chronic
use because of the common adverse effects of
11. The correct answer is B. Pagets disease is a
diarrhea and constipation.
disorder of bone remodeling. It is initiated by a
pathologic increase in osteoclastic bone resorp- Answer C is incorrect. Histamine-2 receptor
tion, followed by a compensatory increase in antagonists, such as cimetidine, famotidine,
osteoblastic bone production. Serum calcium nizatidine, and ranitidine, act to selectively in-
and phosphate levels are generally normal in hibit histamine receptors on the gastric parietal
Pagets disease; however, elevated alkaline cells, thereby reducing acid secretion. They
phosphatase levels are a common nding in have a favorable safety prole but are not of-
this disorder. ten used as rst-line therapy in infant GERD.
They are usually prescribed after conservative
Answer A is incorrect. Alkaline phosphatase
measures such as altering feeding techniques
should be elevated, reecting increased bone
Test Block 5

and infant positioning have failed.


formation.
Answer D is incorrect. Prokinetic agents, such
Answer C is incorrect. Hypercalcemia is not
as metoclopramide, bethanechol, and erythro-
seen in Pagets disease. Causes of elevated cal-
mycin, have varying mechanisms of action, but
cium include parathyroid hormone-secreting
Test Block 5 Answers 625

controlled trials have not shown signicant ef- tend to localize to McBurneys point in the
cacy for treating GERD. lower right quadrant over 46 hours, where fo-
cal rebound tenderness can develop. It is also
Answer E is incorrect. Proton pump inhibitors
associated with a leukocytosis and fever.
include omeprazole, lansoprazole, pantopra-
zole, rabeprazole, and esomeprazole. They act Answer B is incorrect. In acute cholecystitis,
by blocking the hydrogen-potassium ATPase the pain would be more localized in the right
channels involved in gastric acid secretion. upper quadrant (positive Murphys sign), and
Although effective, they are not generally initi- after 3 hours to 3 days, can develop rebound
ated as rst-line therapy in childhood GERD. and guarding. The pain is often related to food
intake. Patients often have a low-grade fever.
13. The correct answer is A. Minors requesting Also, no distension or air-uid levels would be
treatment for sexually transmitted diseases are present on the abdominal lms.
entitled to complete condentiality.
Answer C is incorrect. Complete bowel ob-
Answer B is incorrect. The physician must re- struction would present with crampy intermit-
spect the condentiality rights of all minors, tent abdominal pain, increased or high-pitched
and it would be inappropriate to imply that an bowel sounds, and obstipation. The abdominal
exception is being made for this particular pa- lms would show air-uid levels proximal to
tient. the obstruction and the absence of air distal to
Answer C is incorrect. Minors have the right the obstruction.
to condential treatment and screening for Answer E is incorrect. Partial bowel obstruc-
sexually transmitted diseases. Parental consent tion would present with crampy intermittent
is not necessary for minors to receive this treat- abdominal pain, increased or high-pitched
ment. bowel sounds, and passage of atus without
Answer D is incorrect. Minors have the right stool. The abdominal lms would also likely
to condential treatment and screening for sex- not show diffuse intestinal distension with air-
ually transmitted diseases. Parental consent is uid levels proximal to the obstruction and
not necessary for minors to receive this screen- some air distal to the obstruction.

FULL-LENGTH EXAMS
ing.
15. The correct answer is A. The patient has as-
Answer E is incorrect. Minors requesting bestos, as evidenced by the nding of asbestos
treatment for sexually transmitted diseases are bodies on bronchoalveolar lavage and sug-
entitled to complete condentiality. Physicians gested by the x-ray of the chest and symptoms
are legally and ethically prohibited from with- in light of a history of possible exposure. A la-
holding treatment. tency period of 20 years or longer between ex-
posure and symptomatic disease in asbestosis is
14. The correct answer is D. This patient is pre- not uncommon. Asbestosis is one of the pneu-
senting with a clinical scenario and radiologic moconioses and, as a disease of the intersti-
ndings consistent with a paralytic ileus. Clini- tium, is an example of restrictive lung disease.
cally, she is taking regular doses of a medica- The pulmonary function test pattern seen in
tion that slows bowel motility (opiates), has restrictive disease is for FEV1 and FVC to both
constant abdominal pain, is obstipated, has ab- decrease preserving a normal FEV1:FVC ratio.
sent bowel sounds, and is not focally tender
anywhere with no signs of peritoneal irritation. Answer B is incorrect. In some instances of
Radiologically, she has air throughout the co- obstructive lung disease, the FEV1 decreases,
lon and small intestine, both of which are dis- while FVC remains normal. The resultant ra-
Test Block 5

tio of FEV1:FVC still decreases.


tended; this is more consistent with an ileus
than a small bowel obstruction. Answer C is incorrect. In lung disease with air-
ow obstruction, pulmonary function tests can
Answer A is incorrect. Acute appendicitis can
begin with diffuse abdominal pain, but will reveal decreased FEV1 and increased FVC,
lowering the FEV1:FVC ratio.
626 Section II: Full-Length Examinations Answers

Answer D is incorrect. This patient has a sig- and clammy skin, jugular venous distension,
nicant ber burden, is symptomatic, and has and rapid, weak, and thready pulses. Often in
evidence of interstitial disease on x-ray of the cases of congestive heart failure the pulmonary
chest. It is thus very unlikely that her pulmo- examination will reveal congestion.
nary function tests would be normal.
Answer F is incorrect. Inadequate uid reple-
Answer E is incorrect. This pattern is the op- tion would cause hypovolemic shock, and re-
posite of what is seen in restrictive lung disease exive tachycardia to compensate for loss of
and is not noted in obstructive illness either. volume would be expected. In this case, you
would expect to see signs of decreased periph-
16. The correct answer is G. The patient likely eral perfusion such as decreased capillary rell,
suffered a cervical spine injury that resulted in weakened peripheral pulses, or cool/clammy/
neurogenic shock. This spinal cord injury is as- cyanotic extremities.
sociated with a sudden loss of sympathetic in-
put to the smooth muscles in vessels. This loss 17. The correct answer is G. The patient described
causes a sudden decrease in peripheral vascu- has DiGeorges syndrome. The translocation re-
lar resistance and thus a decrease in blood sponsible and the signs of the disease can best
pressure and bradycardia. Unlike other causes be remembered with the mnemonic CATCH
of shock, neurogenic shock is associated with 22. The Cardiac anomalies, Abnormal facies,
warm, dry extremities. Thymic hypoplasia, Cleft palate, and Hypocal-
cemia, are all consistent with a broad spectrum
Answer A is incorrect. Addisonian crisis can
of conditions associated with 22q11.2 deletions,
occur in a patient with adrenal insufciency
including DiGeorges syndrome, velocardiofa-
who has had an acute stress or with bilateral
cial syndrome, and conotruncal anomaly face
adrenal hemorrhage or infarction. Patients of-
syndrome.
ten complain of nonspecic symptoms, such
as anorexia, nausea, vomiting, abdominal pain, Answer A is incorrect. Williams syndrome
weakness, fatigue, and lethargy. Bradycardia is is associated with a deletion at 7q11.23 and
not expected. manifests clinically as a round face with full
cheeks and lips, strabismus, supravalvular aor-
FULL-LENGTH EXAMS

Answer B is incorrect. In anaphylactic shock,


tic stenosis and other cardiac malformations,
peripheral vascular resistance is reduced so
variable degrees of mental retardation, and an
there is a reexive tachycardia in an attempt to
extremely friendly personality.
maintain adequate perfusion. In this case, you
would expect hypotension, respiratory distress, Answer B is incorrect. Mild mental retarda-
urticaria, ushing, and angioedema. tion with abnormal facies, sparse hair, multiple
cone-shaped epiphyses, and multiple carti-
Answer C is incorrect. Atrial brillation is a
laginous exostoses is characteristic of Langer-
cause of cardiogenic shock. However, the heart
Giedion (trichorhinophalangeal, type II) syn-
rate is typically fast and irregular. In cardio-
drome. This disorder is caused by an 8q24
genic shock, physical examination will reveal
microdeletion.
hypotension, cool, clammy skin, jugular ve-
nous distension, and rapid, weak, and thready Answer C is incorrect. WAGR syndrome is
pulses. caused by microdeletions of 11p13. The syn-
drome presents with abnormal facies, hyper-
Answer D is incorrect. Bacteremia is a cause
nephroma (Wilms tumor), Aniridia, and vary-
of septic shock. The patient would present with
ing degrees of male Genital hypoplasia and
fever, tachycardia, and leukocytosis.
mental Retardation.
Test Block 5

Answer E is incorrect. Congestive heart fail-


Answer D is incorrect. Microdeletions to
ure is a cause of cardiogenic shock. Cardiac
15q11 are responsible for Prader-Willi and An-
output is reduced and the heart compensates
gelman syndromes. Both syndromes are char-
with tachycardia. In cardiogenic shock, physi-
acterized by mental retardation and hypotonia.
cal examination will reveal hypotension, cool
Prader-Willi syndrome also includes obesity
Test Block 5 Answers 627

with short stature, small hands and feet, and Echocardiography would show pericardial
hypogonadism. Angelman syndrome presents thickening.
with ataxic movements and seizures, uncon-
Answer C is incorrect. Restrictive cardiomyop-
trollable episode of laughter, and midface hy-
athy (seen in various inltrative diseases) is a
poplasia.
less common cause of diastolic dysfunction.
Answer E is incorrect. 17p13 Microdeletions Again, the evidence of hypertrophy makes this
result in Miller-Dieker syndrome, presenting even less likely in this patient.
clinically as microcephaly with severe mental
Answer D is incorrect. The patients imaging
retardation, seizures, growth failure, hypoplas-
results point to diastolic dysfunction, not sys-
tic male genitalia, and pachygyria (decreased
tolic dysfunction, due to increased wall thick-
development of brain cortex gyri and sulci).
ening and normal ejection fraction.
Answer F is incorrect. Microdeletions affect-
Answer E is incorrect. The patients imaging
ing chromosome 20p12 give rise to the Alagille
results point to diastolic dysfunction, not sys-
syndrome. This syndrome manifests with skel-
tolic dysfunction, due to increased wall thick-
etal defects, including buttery vertebrae, bile
ening and normal ejection fraction.
duct paucity with cholestasis, heart defects (es-
pecially pulmonary artery stenosis), and ocular Answer F is incorrect. The patients imaging
abnormalities. results point to diastolic dysfunction, not sys-
tolic dysfunction, due to increased wall thick-
18. The correct answer is B. Given the patients ening and normal ejection fraction.
history of hypertension and hypertrophy on
ECG, this is the most likely cause of his dia- 19. The correct answer is A. The infant has devel-
stolic dysfunction. Diastolic dysfunction occurs opmental dysplasia of the hip. If left untreated,
when there is abnormal relaxation of the left the infant is at risk for persistent dysplasia, os-
ventricle, which impairs lling and results in teoarthritis, and avascular necrosis of the femo-
elevated left ventricular, left atrial, and pulmo- ral head. The appropriate treatment for infants
nary venous pressures. Causes of diastolic dys- younger than 6 month is a exion-abduction
function include acute ischemia, chronic hy- orthosis known as a Pavlik harness, which is de-

FULL-LENGTH EXAMS
pertension, severe aortic stenosis, inltrative signed to stabilize the affected hip in a reduced
cardiomyopathy (e.g., amyloid), and hypertro- position.
phic cardiomyopathy. Systolic dysfunction re-
Answer B is incorrect. The affected hip in a
fers to the inability for the ventricle to properly
patient with developmental dysplasia of the
squeeze. This results in a decrease in stroke
hip must be kept in a exed, abducted position
volume and a compensatory rise in preload in
to maintain reduction. Extension does not en-
order to maintain end-organ perfusion by ad-
gage the femoral head in the acetabulum and
vancing further to the right on the Frank- will not stabilize the dislocatable hip.
Starling curve.
Answer C is incorrect. Once the newborn pe-
Answer A is incorrect. It is important to di- riod is complete (after 6 months of age), closed
agnose constrictive pericarditis as a cause of reduction with application of a spica cast is the
diastolic dysfunction, as this is one of the more treatment of choice.
treatable etiologies. Other ndings point to a
hypertrophic cause and moreover, echocar- Answer D is incorrect. This infant presented
diography would likely reveal a constrictive with a dislocatable hip. Without treatment,
pericarditis if it existed. Symptoms of constric- this may lead to signicant deformity from
tive pericarditis include leg edema, abdominal developmental dysplasia of the hip. The long-
Test Block 5

fullness, and pain secondary to hepatic con- term complications of developmental dysplasia
gestion, and these may progress to exertional of the hip include leg-length discrepancy and
dyspnea, cough, and orthopnea. Signs include osteoarthritis.
Kussmauls sign, pericardial knock, and oth-
ers consistent with right-sided heart failure.
628 Section II: Full-Length Examinations Answers

Answer E is incorrect. The Pavlik harness is Answer E is incorrect. This patient is experi-
the treatment of choice for a newborn with a encing a normal part of the grieving process
dislocated or subluxed hip. Once the child is and therefore does not require admission to an
older (1524 months old), open reduction is inpatient psychiatric facility.
indicated when an attempt at closed reduction
has failed. 21. The correct answer is B. This patient is pre-
senting with precocious puberty, which is gen-
20. The correct answer is B. This man is likely erally dened as sexual development at <8
suffering from normal bereavement, which is years of age for girls and <9 years of age for
part of the grieving process. Loss of a loved one boys. In central, or true, precocious puberty,
can have a serious impact on health status. there is early activation of the normal hypotha-
The symptoms of bereavement, including lamic-pituitary axis. The majority of these cases
weight loss, decreased appetite, and decreased occur in girls and are idiopathic; however, cer-
sleep, can mimic those of major depression. tain central nervous system lesions, like hamar-
Searching behaviors, such as auditory or vi- tomas or other tumors, can cause central pre-
sual hallucinations, can also be seen. If these cocious puberty. Because the disorder is caused
symptoms resolve within 8 weeks, they are nor- by early activation of the normal signaling
mal. The proper approach with this patient is pathway, the patients tend to have normal se-
to listen, support, reassure, and follow up quential pubertal development. However,
closely to watch for development of a more se- many patients do not have adrenarche at pre-
rious disorder. A diagnosis of major depression sentation because the adrenals are under sepa-
would require persistence of these symptoms, rate control. Laboratory evaluation will reveal
in addition to severe feelings of guilt, helpless- elevated levels of gonadotropin-releasing hor-
ness, hopelessness, and/or suicidal ideation. mone (GnRH) as well as luteinizing hormone
(LH), follicle-stimulating hormone (FSH), and
Answer A is incorrect. Electroconvulsive
estrogen.
therapy induces seizures in an attempt to treat
severe psychoses, such as catatonic schizophre- Answer A is incorrect. Adrenal tumors can
nia, major depression with psychotic features, secrete estrogen and cause early sexual devel-
FULL-LENGTH EXAMS

and refractory obsessive-compulsive disorder. opment. However, these patients will have
This patient does not t into any of these cat- decreased GnRH, LH, and FSH levels due to
egories and would hence not achieve any ben- negative feedback by estrogen on the hypothal-
et. The adverse effects of headache and retro- amus and pituitary.
grade/anterograde memory loss are additional
Answer C is incorrect. McCune-Albright syn-
reasons not to use electroconvulsive therapy to
drome is a cause of gonadotropin-independent
treat this patient.
precocious puberty. In addition to the dysfunc-
Answer C is incorrect. Olanzapine is an atypi- tions caused by autonomous activity of other
cal antipsychotic that is used to treat depres- endocrine glands, early sexual development
sion with psychotic features, schizophrenia, occurs due to autonomous ovarian function,
and other psychotic disorders. Although this usually from a dominant ovarian cyst that in-
patient is experiencing some auditory hallu- dependently produces estrogen. These patients
cinations, such symptoms are consistent with will have decreased LH, FSH, and GnRH due
searching behaviors and are related to the to negative feedback by estrogen on the hypo-
grieving process. thalamus and pituitary. It is also characterized
by polyostotic brous dysplasia and caf au lait
Answer D is incorrect. Zolpidem is a sleep aid
spots.
with a very low addictive potential. Although
Test Block 5

it would likely enable this patient to get more Answer D is incorrect. An ovarian tumor that
sleep, it would not combat his other bereave- secretes estrogen is another cause of gonadotro-
ment symptoms, and thus it is not the next best pin-independent precocious puberty. These pa-
treatment at this time. tients will have early sexual development, but
GnRH, LH, and FSH levels will be decreased
Test Block 5 Answers 629

due to negative feedback of estrogen on the hy- 23. The correct answer is B. This patient should
pothalamus and pituitary. undergo colonoscopy as soon as possible. Her
history is consistent with hereditary nonpolypo-
Answer E is incorrect. Patients with chronic
sis colorectal cancer (HNPCC), the most com-
hypothyroidism may experience early sexual
mon hereditary cause of colon cancer. HN-
development. Because thyroid-stimulating hor-
PCC is inherited in an autosomal dominant
mone (TSH) and FSH are structurally similar,
fashion, and patients with this syndrome tend
high levels of TSH in primary hypothyroidism
to develop colon cancer at an average age of 44
can directly activate the FSH receptor. These
years. This woman meets the Amsterdam II cri-
patients would likely have other signs of hypo-
teria for HNPCC, which includes the follow-
thyroidism including weight gain, fatigue, dry
ing: (1) three or more relatives with HNPCC-
skin, constipation, and cold intolerance. In ad-
associated cancer (including cancer of the
dition, GnRH levels would not be expected to
colon, endometrium, small bowel, kidney, and
be elevated in hypothyroidism.
ureter), two of whom are rst-degree relatives,
22. The correct answer is C. Benzodiazepines are (2) two or more successive generations af-
sedatives that bind at specic receptors and act fected, and (3) one or more relatives diagnosed
to enhance the inhibitory tone of GABA recep- at <50 years of age. Patients with a positive
tors in the central nervous system. Some com- family history of HNPCC should undergo
mon indications include induction of anesthe- colonoscopy every 12 years after age 25 years
sia, alcohol withdrawal, insomnia, epilepsy, (or when 10 years younger than the youngest
and generalized anxiety. Patients who have de- affected relative), and every year after age 40
veloped benzodiazepine dependency are at risk years. This patient is already 36 years old, and
for potentially fatal complications from with- should undergo immediate colonoscopy.
drawal, including seizures. Benzodiazepine Answer A is incorrect. Colonoscopy in a pa-
withdrawal should be suspected in any patient tient with a positive family history for colon
taking a drug from this class who suffers a sei- cancer should begin before 50 years of age.
zure.
Answer C is incorrect. A CT scan of the abdo-
Answer A is incorrect. Angiotensin-converting men is useful in assessing patients with colon

FULL-LENGTH EXAMS
enzyme inhibitors would be a logical choice cancer that has metastasized to lymph nodes
for treating this patients hypertension, al- and the liver, as well as in diagnosing recur-
though this class is known to cause severe with- rence of resected or metastatic colon cancer. A
drawal symptoms or seizures. scan at this time would not be the appropriate
Answer B is incorrect. Cocaine is among next step, and would not negate the need for a
many common illegal drugs known to cause colonoscopy.
seizures. However, there is no reason to suspect Answer D is incorrect. A double-contrast bar-
that this man uses cocaine. Furthermore, with- ium enema can be used to examine the entire
drawal from cocaine is far less likely to cause rectum and colon and carries less risk than
seizures than its use or abuse. colonoscopy. However, the sensitivity of this
Answer D is incorrect. Lithium is known to method for detecting large polyps and cancers
cause seizures; however, there is no evidence is less than that of colonoscopy, and the proce-
of the patient taking this substance. Further- dure does not permit removal of polyps or bi-
more, withdrawal from lithium is far less likely opsy of cancers; thus follow-up colonoscopy is
to cause seizures than their use or abuse. needed if any abnormalities are found. A dou-
ble-contrast barium enema is also more likely
Answer E is incorrect. Calcium channel than colonoscopy to identify artifacts and other
Test Block 5

blockers would be a logical choice for treating ndings (such as stool) as polyps. Although use-
this patients hypertension, although this class ful in patients with an average risk of develop-
is known to cause severe withdrawal symptoms ing colorectal cancer, double-contrast barium
or seizures. enema would not be appropriate in this high-
risk patient.
630 Section II: Full-Length Examinations Answers

Answer E is incorrect. While digital rectal to the constipation opiates induce is slower to
examinations are useful for detecting rectal or develop.
prostate cancer, they are clearly not sufcient
Answer E is incorrect. Patients in heroin with-
in screening for colon cancer, though there is
drawal have symptoms that are very intense
a role for fecal occult blood testing performed
and unpleasant, but such symptoms are not
three times annually. Flexible sigmoidoscopy
life-threatening.
can detect lesions 60 cm from the anus, but
will not detect lesions in the proximal colon. 25. The correct answer is B. Atopic dermatitis (ec-
Yearly digital rectal exam and exible sigmoi- zema) typically appears before the age of 7
doscopy every 23 years would not be sufcient years and manifests as itchy, often dry, scaling
in this high-risk patient. erythematous plaques with excoriations. In
older children (as in this case) and adults the
24. The correct answer is C. This woman is suffer-
exural areas (though not the groin) are most
ing from a heroin overdose. Heroin is a highly
commonly involved. In infants and very young
addictive member of the opioid family, which
children the extensor surfaces, posterior scalp,
includes morphine and codeine. These drugs
and face are typically involved, while the dia-
activate opioid receptors found in both the cen-
per region is spared. Chronic atopic dermatitis
tral nervous system and the gastrointestinal sys-
manifests with thickened skin and excoriated
tem. There are three major opioid receptor sub-
papules.
types: , , and . Activation of receptors in
the central nervous system and gastrointestinal Answer A is incorrect. The history mentions a
tract result in heroins euphoric effects as well as possible allergen exposure (detergent), but this
to its adverse effects: constipation (due to de- is extremely unusual. Furthermore, a dermati-
creased bowel motility), sedation, pupillary con- tis resulting from detergent exposure would be
striction, and decreased respiration. expected to predominantly manifest in areas in
contact with clothing; the initial involvement
Answer A is incorrect. Naloxone is an opioid
of the face in this case would be difcult to as-
antagonist that binds strongly to opioid recep-
sociate with detergent.
tors. It acts quickly to reverse the symptoms of
FULL-LENGTH EXAMS

opiate overdose, and most patients recover con- Answer C is incorrect. Pityriasis rosea is typi-
sciousness with intravenous administration. Pa- cally seen in older children and adults. A her-
tients must be closely monitored when nalox- ald patch is followed by erythematous, oval,
one is administered, however, as its half-life is scaly papules on the trunk and proximal ex-
shorter than that of most recreational opiates, tremities that spread from the neck down. In-
and patients can re-experience respiratory com- volvement of the face is unusual.
promise once its effects wear off. Lorazepam Answer D is incorrect. Lesions typical of
has no use in the setting of heroin intoxication. chronic plaque psoriasis are well-demarcated
Answer B is incorrect. Patients who overdose erythematous papules and plaques with silvery
on heroin usually present with respiratory com- scales with variable pruritus. While lesions are
promise and may stop breathing altogether. For usually found on extensor surfaces (elbows and
this reason, patients who present to the emer- knees), as in this case, the age of onset is typi-
gency department with the hallmarks of an cally much later in childhood. Nail ndings
opioid overdose (such as pinpoint pupils, nee- (the oil drop sign, local brownish discolor-
dle-track marks, and respiratory compromise) ation of the nail, or pitting) may also be seen
should be closely monitored and an emergency in psoriasis, but are not associated with atopic
airway secured if necessary. Myocardial infarc- dermatitis.
Test Block 5

tion due to illicit drug use is usually associated Answer E is incorrect. Seborrheic dermatitis is
with cocaine overdose. an erythematous, scaling rash associated with
Answer D is incorrect. Tolerance to the eu- mild pruritus. In contrast to atopic dermatitis,
phoria, sedation, and decreased respiration seborrheic dermatitis is most common in areas
caused by opiates develops rapidly. Tolerance rich in sebaceous glands (forehead, scalp, nasola-
Test Block 5 Answers 631

bial folds, nasal bridge, chest, axilla, and groin), due to its narrow radius (Laplaces law: pressure
and is not usually associated with dry skin. = wall tension/radius). Diverticulitis is an infec-
tion of a diverticulum that may result in an ab-
26. The correct answer is B. Epiglottitis is a rap- scess or pericolic infection and may present
idly progressive infection of the supraglottic with abdominal pain, peritoneal signs, leukocy-
structures that can lead to complete airway ob- tosis, fever, chills, nausea, vomiting, change in
struction. Patients typically present with rapid- bowel habits, and/or dysuria. Diverticuli are also
onset fever with dysphagia, drooling, mufed the number one cause of lower gastrointestinal
voice, inspiratory retractions, cyanosis, and soft bleeding in the elderly. Patients who consume
stridor. The presentation described is referred low-ber diets or have chronic constipation may
to as the snifng position with hyperexten- be at increased risk for developing diverticuli.
sion of the neck and protrusion of the chin.
Answer A is incorrect. Increased intraluminal
The most common cause of this condition was
colonic pressure leads to the development of
Haemophilus inuenzae type B before immu-
diverticula. Although it is a good idea to treat
nizations with the Hib vaccine became avail-
hypertension for the sake of preventing further
able. The primary pathogens of epiglottitis now
medical complications, treating hypertension
include Streptococcus species, nontypable H.
has no impact on preventing diverticuli forma-
inuenzae, and viral pathogens.
tion.
Answer A is incorrect. Diphtheria-tetanus-
Answer B is incorrect. There have not been
pertussis (DTaP) vaccination is responsible for
any conclusive studies that show a relationship
reducing the occurrence of diphtheria, tetanus,
between exercise and diverticuli formation.
and pertussis. It has not been shown to play a
One study indicated that physical activity may
role in reducing the incidence of epiglottitis,
reduce diverticulosis; however, this was not sta-
since the implicated pathogens are different
tistically conrmed.
from those targeted by the DTaP vaccine.
Answer D is incorrect. There have not been
Answer C is incorrect. Measles-mumps-
any conclusive studies that show a relationship
rubella vaccination has signicantly reduced
between caffeine and diverticuli formation.
the occurrence of measles, mumps, and ru-

FULL-LENGTH EXAMS
bella. However, these viral antigens have not Answer E is incorrect. Although smoking ces-
been shown to be implicated in the etiology of sation should be encouraged in all populations
epiglottitis. because of the health risks that smoking pre-
sents, this would not have had any impact on
Answer D is incorrect. The vaccine for pneu-
reducing the likelihood of diverticuli forma-
mococcal species has aided in reducing the
tion.
incidence of ear infections, meningitis, and
pneumonia.
28. The correct answer is C. Osteoarthritis is a
Answer E is incorrect. The introduction of the common disorder in which articular cartilage
varicella vaccine has greatly reduced the inci- is damaged as a result of multiple factors in-
dence of chickenpox, but has not impacted the cluding aging, mechanical trauma, genetics,
occurrence of epiglottitis. and biochemistry. Loss of articular cartilage
causes narrowing of the joint spaces. Second-
27. The correct answer is C. The patient pre- ary bone remodeling leads to osteophyte devel-
sented to the emergency department with an opment. Pain due to osteoarthritis is described
episode of diverticulitis. A diverticulum is a as dull and aching, which gets worse with ac-
herniation of the colonic mucosa that occurs as tivity and may resolve with rest, but as the dis-
Test Block 5

a result of increased pressure in the colon. Di- ease progresses, pain may also be present at
verticuli are often seen in the sigmoid colon rest. Patients may also complain of stiffness and
because this area of the colon is subjected to limited mobility of the affected joints. Multiple
the greatest amount of intraluminal pressure joints are typically involved, with the hands be-
632 Section II: Full-Length Examinations Answers

ing a common site of disease. In the hand, the 29. The correct answer is E. External cephalic
proximal and distal interphalangeal (PIP, DIP) version is a series of maneuvers performed via
joints are classically affected, while the meta- maternal abdomen to convert from breech to
carpophalangeal joints are spared. Enlarge- cephalic presentation. External cephalic ver-
ment of the DIP and PIP joints results in He- sion has been shown to signicantly reduce the
berdens and Bouchards nodes, respectively. attendant risks of both vaginal breech delivery
and cesarean section. External cephalic ver-
Answer A is incorrect. Calcium pyrophosphate
sion cannot be performed once labor is initi-
deposition disease, a spectrum of disease that
ated or the fetus is engaged in the pelvis, with-
includes the clinical entity known as pseudog-
out adequate amniotic uid, or when fetal
out, may resemble osteoarthritis, presenting
heart tracings are nonreassuring. The risks of
as progressive joint degeneration. Physical ex-
performing external cephalic version include
amination ndings as well as radiographs may
placental abruption and cord prolapse, which
resemble osteoarthritis. However, pseudogout
would require emergent cesarean section.
most commonly affects the knees.
Answer A is incorrect. Elective cesarean deliv-
Answer B is incorrect. Acute gouty arthritis
ery is recommended for persistent breech pre-
is typically monarticular, and like acute infec-
sentation. However, external cephalic version
tious arthritis, presents with redness, swelling,
with subsequent vaginal delivery can eliminate
and warmth of the affected joint. Patients with
some of the risks of cesarean delivery, and in
long-standing gout can develop chronic topha-
this case should be attempted rst.
ceous gout, in which urate crystals deposit
in connective tissue. On x-ray lms, chronic Answer B is incorrect. Cesarean delivery is
tophaceous gout appears as extensive soft tissue the preferred method of delivery of fetuses
swelling with calcications adjacent to joints, with persistent breech presentation. However,
and extensive erosion of the bones. breech presentation is not an indication for an
emergent delivery. Operative delivery may take
Answer D is incorrect. Psoriatic arthritis is a
place on a planned basis to eliminate the risks
disorder associated with psoriasis. It is an in-
associated with emergent surgery.
ammatory arthritis, so the pattern of pain re-
FULL-LENGTH EXAMS

sembles that of RA, not osteoarthritis. Patients Answer C is incorrect. Cesarean delivery of
complain of pain and stiffness that is worst in fetuses in breech presentation has been found
the morning, often in the DIP joints. Although in several studies to reduce the rate of perinatal
this patient has DIP joint involvement, by de- morbidity and mortality, and is associated with
nition, patients with psoriatic arthritis must also higher Apgar scores after delivery. Therefore
have psoriasis. Most patients have evidence of planned cesarean delivery of fetuses in persis-
psoriatic skin and/or nail disease prior to pre- tent breech presentation is recommended.
senting with arthritis.
Answer D is incorrect. External cephalic ver-
Answer E is incorrect. RA in the hands typi- sion followed by vaginal delivery or planned
cally affects the metacarpophalangeal and PIP cesarean delivery are the preferred methods of
joints, with sparing of the DIP joints and ab- delivering breech presentation infants. Vaginal
sence of Heberdens nodes. Patients with RA delivery of a child in breech position is not rec-
have pain and stiffness in the hands that is most ommended, and external cephalic version can-
severe in the morning and after a long period not be attempted once labor has begun. Thus,
of inactivity. On examination patients with RA it is inadvisable to induce labor with a fetus in
will typically have swollen joints that are warm, breech presentation.
red, and tender. Radiographs in RA show ero-
Test Block 5

sion of the bone, as opposed to the joint space


narrowing, sclerosis, and osteophytes seen in
osteoarthritis.
Test Block 5 Answers 633

30. The correct answer is A. The patient described thrive, diarrhea, severe bacterial infections, op-
has Brutons (X-linked) agammaglobulinemia. portunistic infections, and chronic candidiasis.
Patients are typically well until 69 months of The absolute lymphocyte count is, by deni-
age, at which point maternal IgG antibodies tion, decreased.
become inactive. Then patients acquire infec-
tions with extracellular pyogenic organisms, in- 31. The correct answer is A. This study is a case-
cluding Streptococcus pneumoniae and Haemo- control study, where those who failed extubation
philus inuenzae, and present with pneumonia, are the cases (disease present) and those who
otitis media, and sinusitis. Low levels of IgG, were successfully extubated are the controls (dis-
IgA, IgM, and IgE are found on laboratory ease not present). Valid matching is facilitated
evaluation, and ow cytometry demonstrates in this case by a well-dened population (intu-
the absence of B lymphocytes. Because the bated patients in the intensive care unit) from
gene that is mutated in this disease, Brutons which to draw both cases and controls.
tyrosine kinase, is on the X chromosome, the
Answer B is incorrect. Clinical trials are pro-
disease is overwhelmingly more common in
spective studies that examine random use of an
male patients. B lymphocyte development can
intervention and its effect on outcome.
be arrested by mutations in at least four other
genes that are inherited in an autosomal reces- Answer C is incorrect. Correlation studies are
sive manner, resulting in the same clinical used to compare disease frequencies between
phenotype of agammaglobulinemia. Inheri- entire populations (as opposed to individuals).
tance of two mutant alleles of these genes, Answer D is incorrect. Prospective cohort
however, is extraordinarily rare, and the vast studies take a sample group (a cohort) with a
majority of agammaglobulinemic patients have risk factor and match them to a control with-
the X-linked variety. out the risk factor. The groups are then studies
Answer B is incorrect. Common variable im- over time to assess development of disease.
munodeciency is a syndrome characterized Answer E is incorrect. Retrospective cohort
by hypogammaglobulinemia and impaired studies assemble a cohort based on retrospec-
specic antibody production, with circulating tive identication of risk factors, and then

FULL-LENGTH EXAMS
B lymphocytes. compare current outcome data with that of a
Answer C is incorrect. DiGeorges syndrome, matched control group.
also know as velocardiofacial syndrome, is
largely a disorder of T lymphocytes that results 32. The correct answer is E. Cardiac tamponade
from inappropriate thymic development. In classically presents with Becks triad: hypoten-
>80% of cases, it is caused by a microdeletion sion, mufed heart sounds, and jugular venous
of chromosome 22. It often presents with hy- distention. Cardiac tamponade results from
pocalcemia in the rst few days of life that can rapid engorgement of the pericardial space
be severe enough to result in tetany or seizures with blood, and often results from penetrating
if untreated. trauma to a ventricle or aortic dissection.
Answer D is incorrect. IgA deciency is the Answer A is incorrect. In addition to Becks
most common primary immunodeciency triad, cardiac tamponade can present with a
and can be asymptomatic or present with re- narrowed pulse pressure. A bounding pulse is
current upper respiratory infections. However, usually associated with aortic insufciency.
the other immunoglobulins would be found at Answer B is incorrect. In cardiac tamponade,
normal levels. In the setting of blood transfu- impaired diastolic lling leads to a drop in
sions it is associated with anaphylaxis.
Test Block 5

stroke volume, resulting in hypotension. The


Answer E is incorrect. Severe combined im- body attempts to respond by increasing heart
munodeciency is a life-threatening condition rate, thus tamponade usually presents with
affecting T and, in many cases, B lymphocytes. tachycardia, not bradycardia.
Patients present commonly with failure to
634 Section II: Full-Length Examinations Answers

Answer C is incorrect. Rupture of a papil- Answer A is incorrect. Patients often will expe-
lary muscle may result in mitral regurgitation, rience a transient increase in viral load and dip
which can present with a new onset holosys- in CD4+ cell counts at the time of serocon-
tolic murmur. This is a common presentation version. In the majority of patients, the viral
of acute myocardial infarction, not tamponade. load will fall to a set-point, and the CD4+ cell
count will rebound before falling again over
Answer D is incorrect. Cardiac tamponade
several years as the disease progresses.
usually presents with hypotension, not hyper-
tension. Answer B is incorrect. The patients CD4+
cell count is not yet in the range that requires
33. The correct answer is A. Heart palpitations, prophylaxis for Pneumocystis jiroveci pneumo-
breathlessness, and fatigue (the latter often sec- nia. Prophylaxis starts when CD4+ cell counts
ondary to insomnia) are all signs of hyperthy- dip below 200 cells/mm. It is expected that
roidism. This patients weight loss is also consis- his CD4+ cell count will rebound to near-
tent with a hypermetabolic state. Amiodarone normal levels, but rechecking within a month
has been known to cause both hyper- and hypo- is prudent, and if necessary, prophylaxis can be
thyroidism in previously euthyroid individuals. started at that time.
Thyroid hormone levels should be assessed be-
Answer C is incorrect. There are no reported
fore any medication adjustments are attempted.
cases of patients clearing HIV infection. The
Answer B is incorrect. Although providing re- HIV viral load shows that the patient has ongo-
assurance is often helpful, this patients presen- ing viral replication.
tation requires further investigation. Instruct-
Answer D is incorrect. The patients HIV vi-
ing her to return in 6 months would not be
ral load and decreased CD4+ cell count shows
appropriate.
that the patient has ongoing HIV viral replica-
Answer C is incorrect. Although this patients tion, and that the test was not falsely positive.
symptoms may be linked to an anxiety disor-
der, there is no strong evidence to suggest this 35. The correct answer is C. Osler-Weber-Rendu
type of disorder to be the cause. syndrome, also known as hereditary hemor-
FULL-LENGTH EXAMS

rhagic telangiectasia, is an autosomal dominant


Answer D is incorrect. Captopril is generally
condition in which vascular lesions (telangiecta-
well tolerated; a persistent, dry cough is the
sias, arteriovenous malformations, and aneu-
most common adverse effect. Thus, it is un-
rysms) are found throughout the body, particu-
likely to be contributing to the patients com-
larly in the lungs, brain, and gastrointestinal
plaints.
tract. This patient has lesions on her oral mu-
Answer E is incorrect. Substitution of procain- cosa as shown and pale conjunctiva suggesting
amide for amiodarone would not be appropri- anemia. The most likely explanation for her
ate before initial laboratory tests have been per- symptoms is that she has mucosal telangiectasias
formed. in her gastrointestinal tract that have begun to
bleed and are making her anemic.
34. The correct answer is E. The patient appears
to be in the acute seroconversion stage of HIV Answer A is incorrect. Although chronic ob-
infection. During this stage, the HIV viral load structive pulmonary disease can cause easy fa-
will peak and the CD4+ cell count will fall be- tigue and dyspnea on exertion, this patient has
fore both establish a set-point, usually within no prior history, and the physical examination
the rst 34 months of infection. Treatment is ndings are more suggestive of anemia.
only recommended for this stage of infection if Answer B is incorrect. Although stressful life
Test Block 5

the CD4+ cell count is <200/mm or if the pa- events such as the recent death of a loved one
tient is experiencing symptoms of acute viral can often lead to depression, this patient is
syndrome, characterized by u-like symptoms, complaining of isolated decreased exercise tol-
including fever, chills, night sweats, and erance. The fact that she feels ne and is eat-
lymphadenopathy. ing and sleeping well point away from depres-
Test Block 5 Answers 635

sion and toward a physiologic cause such as tially required. The usual surgical repair for
anemia. duodenal atresia is duodenoduodenostomy,
but such repair is performed after nasogastric
Answer D is incorrect. Although this patient
tube placement and intravenous uid replace-
may have intracranial arteriovenous malforma-
ment.
tions and/or aneurysms, they are unlikely to be
responsible for her chronic fatigue and short- Answer E is incorrect. Radiologic studies of
ness of breath. the spine and chest should be performed to
evaluate for associated anomalies, but such
Answer E is incorrect. Although coronary ar-
evaluations are postponed until after initial
tery disease can cause easy fatigue and dysp-
uid replacement and gastric decompression.
nea on exertion, this patient has no prior his-
tory, and the physical examination ndings are
37. The correct answer is D. This is a zone II pen-
more suggestive of anemia.
etrating neck injury. Zone II is dened as the
area between the angle of the mandible and
36. The correct answer is D. This case, in combi-
the cricoid cartilage. It contains the internal
nation with the abdominal lm (demonstrating
and external carotid arteries, jugular veins,
the classic double-bubble sign), is characteris-
pharynx, larynx, esophagus, recurrent laryngeal
tic for duodenal atresia. The condition is be-
nerve, spinal cord, trachea, esophagus, thoracic
lieved to be caused by failure to recanalize the
duct, and thymus. Surgical exploration is indi-
intestinal lumen after the solid phase of intesti-
cated if the platysma is penetrated, if there is
nal development during gestation. Most patients
subcutaneous air, or if there is an expanding
present with bilious vomiting within the rst day
hematoma. Of note, the threshold for intuba-
of life and are noted to have nondistended abdo-
tion should be very low in these patients.
mens. Half of patients with duodenal atresia are
born prematurely, often following pregnancies Answer A is incorrect. Arteriography is use-
complicated by polyhydramnios. Initial treat- ful as the initial test for zone I (between the
ment of patients with duodenal atresia is aimed clavicle/suprasternal notch and the cricoid car-
at reducing gastric pressure by placement of a tilage) and zone III (between the angle of the
nasogastric or orogastric tube, in addition to pro- mandible and the base of the skull) injuries

FULL-LENGTH EXAMS
viding IV uid replacement. because the clinical examination is less reli-
able due to the deeper location of the vascular
Answer A is incorrect. Congenital heart dis-
structures.
ease is present in approximately 10% of pa-
tients with duodenal atresia. As a result of this Answer B is incorrect. Esophagoscopy is use-
high incidence of heart disease, echocardio- ful for zone I and III injuries if there is clinical
grams should be performed on patients with evidence for esophageal or pharyngeal injury.
duodenal atresia to rule out associated cardiac Signs include dysphagia, bloody saliva, sucking
anomalies. Cardiac studies should be under- neck wound, blood nasogastric aspirate, crepi-
taken after uid replacement and nasogastric tus, and bleeding from the mouth. It is im-
decompression in cases of hemodynamically portant to keep in mind that there is a risk of
stable patients. esophageal perforation during esophagoscopy,
so the procedure should only be performed
Answer B is incorrect. Gastrostomy tube
when necessary.
placement is helpful for draining the stomach
and providing protection of the airway. Such Answer C is incorrect. CT of the neck should
management is undertaken after the patient not delay surgical exploration. It is particularly
is stabilized and has undergone uid replace- useful in diagnosing laryngeal injury, but it
Test Block 5

ment and gastric decompression. does not replace esophagoscopy if esophageal


injury is suspected.
Answer C is incorrect. In hemodynamically
stable patients, surgical intervention is not ini- Answer E is incorrect. Tracheobronchoscopy
is useful for zone I and III injuries if there is
636 Section II: Full-Length Examinations Answers

suspicion of larynx or tracheal injury. Signs due to inappropriate sensing of serum calcium
include hoarseness, stridor, respiratory dis- by receptors in the parathyroid gland and the
tress, subcutaneous emphysema, drooling, kidney. This leads to increased secretion of
and hemoptysis. Although this patients crepi- parathyroid hormone (PTH) and increased re-
tus is suggestive of injury to the aerodigestive nal tubular reabsorption of calcium, despite
tract, surgical exploration is needed because of high serum levels. FHH can be differentiated
the large number of vulnerable structures in from primary hyperparathyroidism by the fol-
zone II. lowing: PTH may be elevated in FHH, but not
to the same degree as in patients with primary
38. The correct answer is D. Elevated urine hyperparathyroidism; the presence of hypercal-
5-hydroxyindoleacetic acid (a breakdown prod- cemia in multiple family members, especially
uct of serotonin) levels would conrm the diag- young children; >99% of ltered calcium is re-
nosis of carcinoid syndrome, in this case second- absorbed in FHH, while <99% is reabsorbed in
ary to a small bowel carcinoid tumor with primary hyperparathyroidism; and absence of
metastases to the liver. This syndrome can result abnormal imaging of the parathyroid gland.
from carcinoid tumors producing excess sero-
Answer B is incorrect. Jansens disease is an-
tonin at sites without venous drainage into the
other inherited disorder of hypercalcemia. It is
portal system (e.g., rectal or bronchial) or from
due to excessive activity of the PTH receptor
carcinoid metastases to the liver. It manifests as
in target tissues. Serum PTH levels will be very
episodic watery diarrhea, cutaneous ushing,
low. Patients typically have extensive bone dis-
vasomotor instability, wheezing and bronchial
orders such as short-limbed dwarsm and cys-
spasm, and right-sided valvular lesions which
tic bone disease. Bone disease is rare in FHH.
can cause right-sided heart failure (peripheral
edema). These tumors precipitate a vigorous Answer C is incorrect. In most patients with
desmoplastic reaction in the mesentery resulting primary hyperparathyroidism, PTH levels
in small bowel obstruction (the most common would be expected to be much higher than the
presentation of a carcinoid tumor). mild elevation seen in this patient, although
approximately 10% of patients with primary
Answer A is incorrect. Serum antigliadin an-
hyperparathyroidism can have normal PTH
FULL-LENGTH EXAMS

tibodies are present in the malabsorption syn-


levels. Primary hyperparathyroidism is rarely
drome called celiac disease.
genetic (unless inherited as part of a multiple
Answer B is incorrect. Serum cortisol levels endocrine neoplasia syndrome), so the nd-
play no role in the diagnosis of carcinoid tu- ing of hypercalcemia in multiple family mem-
mors, but are used in the diagnosis of Cush- bers, especially in this patients 10-year-old son,
ings syndrome. makes this diagnosis less likely.
Answer C is incorrect. Serum gastrin levels are Answer D is incorrect. Multiple endocrine
used in the diagnosis of the Zollinger-Ellison neoplasia type I is an autosomal dominant dis-
syndrome. order causing primary hyperparathyroidism,
Answer E is incorrect. Urine vanillylmandelic pituitary tumors, and pancreatic tumors. Pri-
acid levels play no role in the diagnosis of car- mary hyperparathyroidism would usually be
cinoid tumors, but can conrm the elevation expected to have more signicantly elevated
of catecholamine levels when a pheochromo- PTH. The absence of malignancy in the pa-
cytoma is present. tient or the family makes this diagnosis less
likely.
39. The correct answer is A. This family has famil- Answer E is incorrect. Multiple endocrine
Test Block 5

ial hypocalciuric hypercalcemia (FHH), an au- neoplasia type IIB consists of medullary thy-
tosomal dominant inherited disorder causing roid cancer, pheochromocytoma, and mucosal
hypercalcemia, hypocalciuria, and mild hyper- neuromas. Hyperparathyroidism and disorders
magnesemia. Although heterozygotes have a of calcium homeostasis are not components of
benign hypercalcemia, homozygotes can have this syndrome.
severe neonatal hypercalcemia. The disorder is
Test Block 5 Answers 637

40. The correct answer is D. Bundle-branch block in adults. Recurrent hematuria with or without
occurs when there is abnormal conduction ank pain is a common presentation. Occa-
through one of the conductive branches that sup- sionally, however, IgA nephropathy can present
plies the left or right ventricle. These blocks oc- as acute renal failure with headache, edema,
cur when the heart is injured, such as during and decreased urine output, in addition to he-
myocardial infarctions. Signs of acute myocardial maturia and ank pain. An immune complex
infarction necessitate immediate revasculariza- glomerulonephritis, IgA nephropathy, can be
tion, and include new left bundle-branch block differentiated from other immune-mediated
or ST-segment elevations. These signs would dis- nephritides (lupus and postinfectious nephri-
tinguish unstable angina (which may have ST tis) by normal complement levels. Final diag-
depression) from a current myocardial infarction. nosis of IgA nephropathy is made by renal bi-
opsy, which shows mesangial IgA deposits on
Answer A is incorrect. Delta waves are associ-
immunouorescence.
ated with Wolff-Parkinson-White syndrome and
are seen on ECG as an initial slowed upstroke
42. The correct answer is A. Also known as heredi-
of the QRS complex, resulting in its widening.
tary nephritis, Alports syndrome is an X-linked
The delta wave occurs because an accessory
syndrome consisting of progressive glomerular
pathway exists in the heart, exciting the ven-
disease (nephritis), sensorineural hearing loss,
tricles directly from the atria, as opposed to the
and ocular ndings (anterior lenticonus). It man-
normal condition of having the impulse travel
ifests itself in childhood (when detected) as as-
solely through the atrioventricular node. Be-
ymptomatic microhematuria and, in most cases,
cause this conduction pathway does not have
progresses to renal failure by 35 years of age.
the normal delay at the atrioventricular node,
This patient manifests the rst overt signs of re-
the ventricles are excited earlier, resulting in
nal failure with elevated creatinine and hyper-
a delta wave. Treatments for Wolff-Parkinson-
tension.
White syndrome include antiarrhythmics or
ablation of the accessory pathway, depending Answer B is incorrect. Amyloidosis is a dis-
on the patients symptoms. order caused by extracellular deposition of
insoluble abnormal brils that injure tissue.
Answer B is incorrect. Flipped T waves are

FULL-LENGTH EXAMS
Roughly two dozen different unrelated pro-
nonspecic and are not an indication to bring a
teins are known to form amyloid brils in vivo.
patient to the revascularization laboratory unless
When amyloidosis affects the kidney, bright
they occur with other signs that point to current
green uorescence is observed under polarized
myocardial infarction.
light after Congo red staining.
Answer C is incorrect. MI would not cause a
Answer C is incorrect. Cryoglobulins are im-
low-voltage ECG. Low-voltage ECGs imply
munoglobulins that undergo reversible pre-
that the heart is far from the electrodes and
cipitation at low temperatures. Cryoglobuline-
may be seen in conditions like emphysema,
mia may result in systemic inammation via
pericarditis, and in obese patients.
cryoglobulin-containing immune complexes.
Answer E is incorrect. ST-segment depressions When the kidney is involved, cryoglobulins are
are consistent with unstable angina or non-ST- seen in the lumen.
segment elevation myocardial infarction and it
Answer D is incorrect. Focal segmental glo-
would be left to the discretion of the treating
merulosclerosis is the second most common
physician to assess the patients level of risk and
cause of nephrotic syndrome in children. A
to pursue an aggressive/invasive or conservative
more aggressive form of this disease is associated
course.
with HIV infection. Light microscopy typically
Test Block 5

reveals segmental sclerosis or hyalinosis, and


Questions 41 and 42 electron microscopy can show podocyte efface-
ment. The microscopic ndings, however, are
41. The correct answer is F. IgA nephropathy is inconsistent with this patients biopsy results.
the most common cause of glomerulonephritis
638 Section II: Full-Length Examinations Answers

Answer E is incorrect. Goodpastures syn- evated, serum C3 is decreased, and care is sup-
drome is a disease characterized by autoanti- portive because almost all patients have a com-
bodies directed against an antigen intrinsic to plete recovery (children more than adults).
the glomerular basement membrane, result-
Answer L is incorrect. Wegeners granuloma-
ing in acute glomerulonephritis and crescent
tosis is a cause of nephritic syndrome charac-
formation. Classically, it may also present
terized by granulomatous inammation of the
with pulmonary hemorrhage and hemoptysis.
lung and kidney, leading to hemoptysis, sinus
Goodpastures syndrome has a predilection for
symptoms, and nephritis. It is classically C-
men in their mid-20s. Plasma exchange and
ANCA positive, and patients respond to high-
pulsed steroids are the treatment of choice.
dose steroids, albeit with frequent relapses.
Despite treatment, it may progress to end-stage
renal disease.
Answer G is incorrect. Lupus nephritis is a Questions 43 and 44
common complication of systemic lupus erythe- 43. The correct answer is D. Invasive ductal papil-
matosus, affecting 40%85% of patients. In fact, lary carcinoma, invasive ductal carcinoma, in-
the severity of renal disease often determines the vasive lobular carcinoma, and intraductal pap-
overall prognosis for patients with lupus. Renal illoma can cause nipple discharge. However,
involvement can range from mild abnormalities the most common cause of bloody nipple dis-
to full-blown nephritic or nephrotic syndrome. charge is intraductal papilloma; it often occurs
Lupus nephritis is mediated by immune depo- without evidence of a palpable breast mass. In-
sition in the glomerulus. Treatment includes traductal papillomas are more likely to develop
prednisone and cytotoxic therapy. in menopausal or perimenopausal women.
Answer H is incorrect. Membranoprolifera- The discharge may be bloody, serous, or tur-
tive glomerulonephritis is a hard-to-treat pro- bid.
gressive nephritic syndrome often associated
with hepatitic C infection. Light microscopy 44. The correct answer is E. This is most likely in-
demonstrates the classic tram track double- vasive ductal carcinoma. Although intraductal
layered basement membrane. Despite treat- papilloma is the most common cause of a
FULL-LENGTH EXAMS

ment with corticosteroids, the disease usually bloody nipple discharge, bloody nipple dis-
progresses slowly to renal failure. charge is still suspicious for carcinoma. Since
80% of all breast cancers are invasive ductal
Answer I is incorrect. Minimal change dis- carcinomas, this is the most likely diagnosis.
ease is the most common cause of nephrotic Typical clinical features of breast carcinoma
syndrome in children and is characterized by are hard, irregular, painless, and immobile
minimal histologic change by light microscopy. masses located in the right outer quadrant.
There is an increased risk of thrombotic events
in minimal change disease due to low serum Answer A is incorrect. Fibroadenoma is a be-
levels of antithrombin III, tissue plasminogen nign, slow-growing breast tumor, and is the
activator, and other derangements in the co- most common breast tumor in women <30
agulation cascade secondary to heavy proteinu- years old. It usually presents as a rm but mo-
ria. Treatment is with steroids, and prognosis is bile mass.
excellent. Answer B is incorrect. Fibrocysitic changes
Answer J is incorrect. Polyarteritis nodosa is are associated with a bloody discharge in about
a form of systemic necrotizing vasculitis that one-third of cases, but usually the discharge
classically affects small to medium-size vessels. is tinged yellow. Fibrocystic changes occur
Test Block 5

It is classically P-ANCA positive. along a spectrum of clinical ndings that may


include cystic change, nodularity, stromal pro-
Answer K is incorrect. Postinfectious glo- liferation, and epithelial hyperplasia. An exag-
merulonephritis is associated with group A gerated response of breast tissue to hormones
-hemolytic streptococcal infection within the and growth factors causes cyclic, premenstrual,
past 2 weeks. The antistreptolysin O titer is el- bilateral breast pain, and tenderness.
Test Block 5 Answers 639

Answer C is incorrect. Galactocele is a cys- Answer A is incorrect. Brief psychotic disorder


tic dilation of a duct that is lled with a thick, involves at least one of the following: delusions,
milky uid. It is not associated with a bloody hallucinations, or disorganized speech. In addi-
discharge, but there may be a yellow-tinged tion, the symptoms cannot be accounted for by
discharge. a general medical condition. Both of the pa-
tients described have underlying medical con-
Answer F is incorrect. Invasive lobular carci-
ditions.
noma is not as common as invasive ductal car-
cinoma, and thus is not the most likely answer. Answer B is incorrect. Creutzfeldt-Jakob dis-
ease is a rare disease of rapid neurodegenera-
Answer G is incorrect. Invasive papillary carci-
tion that is caused by prions. More than 90% of
noma is not as common as invasive ductal car-
patients experience myoclonus in addition to
cinoma. It can then be inferred that in a pre-
progressive dementia, and death occurs within
sentation highly suspicious for any carcinoma,
1 year of disease onset.
it is not the most likely diagnosis.
Answer D is incorrect. Alzheimers disease is
Answer H is incorrect. Pagets disease of the
the most common cause of dementia. It typi-
breast is an adenocarcinoma that presents with
cally presents in people 5060 years old, is
characteristic eczematous nipple pathology.
slowly progressive, and does not affect a per-
Answer I is incorrect. Although pregnancy can sons level of consciousness. Conversely, delir-
cause nipple discharge, it is unlikely to present ium has an acute onset, typically lasting from
as seen in either of these cases. days to weeks, and impairs a persons level of
consciousness.
Answer J is incorrect. Prolactinoma does
cause nipple discharge; however, it would not Answer E is incorrect. Depression is not gen-
account for the breast mass seen in these cases. erally associated with true delirium, although
very depressed patients may appear delirious
or demented. The patients in these scenarios
Questions 45 and 46 do not present with signs or symptoms of de-
45. The correct answer is C. Delirium is dened pression such as sadness, tearfulness, weight

FULL-LENGTH EXAMS
as impairment in consciousness, usually ac- change, problems with sleep, or loss of interest
companied by cognitive decits, emotional la- in previous activities.
bility, hallucinations, impulsivity, or violent be- Answer F is incorrect. Mania involves erratic
havior. Delirium is generally considered to be and disinhibited behavior, low frustration tol-
an acute reversible disorder. Delirium is classi- erance with irritability and violent behavior,
ed according to etiology: delirium due to a and vegetative signs such as excessive energy,
medical condition, substance-intoxication de- insomnia, or weight loss. The patient in the
lirium, substance-withdrawal delirium, and de- rst vignette exhibits an acute onset of irritabil-
lirium NOS (not otherwise specied). In this ity coupled with cognitive decline as a result of
clinical scenario the delirium appears to be her medical condition.
brought on by the patients medical condition
(e.g., postsurgery or urinary tract infection). Answer G is incorrect. Normal aging is as-
Delirium is classic for waxing and waning of sociated with a decreased ability to learn new
consciousness and rapid onset; in comparison, material and a slowing of cognition; however,
dementia is characterized by a gradual de- it does not show a progressive deteriorating
crease in cognition (not consciousness). course.
Answer H is incorrect. Parkinsons disorder is
46. The correct answer is K. This patients acute
Test Block 5

a movement disorder; in addition, 40%80% of


impairment in consciousness ts the denition patients become demented.
of delirium. In this case, the precipitating
cause is lithium toxicity. Answer I is incorrect. Pseudodementia occurs
when depression in the elderly presents as symp-
640 Section II: Full-Length Examinations Answers

toms of cognitive impairment. The patient in ther of the patients described exhibit psychotic
this clinical scenario does not present with signs symptoms such as hallucinations or delusions.
or symptoms of depression such as sadness, tear-
Answer L is incorrect. Vascular dementia is
fulness, weight change, problems with sleep, or
the second most common cause of dementia.
loss of interest in previous activities.
The disorder progresses in a stepwise fashion
Answer J is incorrect. It may be difcult to dis- due to cerebrovascular disease and intermittent
tinguish the diagnosis of delirium due to a gen- vascular events (i.e., strokes). Neurologic signs
eral medical condition from a psychotic disor- are common. Physical ndings include carotid
der resulting from a general medical condition; bruits and funduscopic abnormalities. Cogni-
however, the latter refers to hallucinations or tive impairment may be patchy, with cognitive
delusions that result from medical illness (e.g., function intact in some areas.
temporal lobe epilepsy or meningitis). Nei-
FULL-LENGTH EXAMS
Test Block 5
Test Block 6

641
642 Section II: Full-Length Examinations Questions

Q U E ST I O N S

1. A 76-year-old man returns to his primary care ratory rate is 26/min. An ECG shows normal
physician for follow-up of a known medical con- sinus rhythm with peaked T waves. Urinalysis
dition. He has no complaints and his physical shows a urine pH of 5.0; it is negative for ke-
examination is unremarkable. Laboratory stud- tones, WBC esterase, and nitrite, but is positive
ies reveal a calcium level of 6.1 mg/dL, a phos- for protein. Arterial blood gas analysis and lab-
phate level of 6.2 mg/dL, and a parathyroid hor- oratory tests show:
mone level of 150 pg/mL (normal: 1115 pg/
pH: 7.3
mL). Which of the following is the patients
Partial pressure of carbon dioxide: 32 mm Hg
most likely underlying medical condition?
Partial pressure of oxygen: 98 mm Hg
(A) End-stage renal disease Oxygen saturation: 99% on room air
(B) Hyperthyroidism Na+: 140 mEq/L
(C) Lung cancer K+: 6.0 mEq/L
(D) Parathyroid adenoma Cl: 120 mEq/L
(E) Sarcoidosis HCO3: 14 mEq/L
Blood urea nitrogen: 16 mg/L
2. A 57-year-old African-American man presents Creatinine: 1.1 mg/L
to his physician with gradual onset of pain in
Which of the following is the most likely diag-
the right leg over 3 months, and acute onset of
nosis?
back pain. He reports that the pain is right on
the bone and becomes worse with walking. He (A) Hyperkalemia
does not recall any inciting trauma to the leg. (B) Hyperventilation
He has no chronic illnesses but reports two epi- (C) Ketoacidosis
sodes of pyelonephritis over the last 18 months. (D) Lactic acidosis
Review of systems is signicant for mild fa- (E) Renal tubular acidosis
tigue. Physical examination shows tenderness
FULL-LENGTH EXAMS

along the right femur with no pain on straight 4. A 31-year-old primigravid woman presents to
leg raise. The legs are warm to the touch with the perinatal evaluation unit complaining of
good pulses and no sensory or motor decit. painful contractions and her water break-
No abdominal bruit is heard. Laboratory re- ing approximately 1 hour before presentation.
sults are signicant for hypercalcemia, elevated She is admitted to the labor oor after an ini-
creatinine, and a low hemoglobin level. X-ray tial evaluation. During the course of her labor,
of the spine reveals compression fractures. the obstetrician becomes increasingly worried
Which of the following additional tests would about cephalopelvic disproportion. During
conrm the most likely diagnosis? which stage of labor would the doctor be able
to make this observation and diagnosis?
(A) Abdominal ultrasound showing stenosis of
the right iliac artery (A) First stage, active phase
(B) Bone densitometry with diminished (B) First stage, latent phase
T-score (C) Second stage
(C) Bone marrow biopsy showing plasmacyto- (D) Third stage
sis (E) Fourth stage
(D) Decreased serum vitamin D level
(E) Nerve conduction studies 5. A 7-month-old boy is brought to the pediatri-
Test Block 6

cian by his parents, who are concerned about


3. A 65-year-old man with type 2 diabetes melli- his recent change in behavior. He was healthy
tus and eczema presents to his physician for a until 3 days earlier, when he developed severe
routine ofce visit. He is currently taking met- diarrhea and was diagnosed with gastroenteri-
formin and atorvastatin. His blood pressure is tis. His parents now describe new-onset vomit-
140/86 mm Hg, heart rate is 75/min, and respi- ing with pallor and lethargy. Current heart rate
Test Block 6 Questions 643

is 109/min, blood pressure is 102/68 mm Hg, Urinalysis reveals pink urine, with a pH of 5.5,
respiratory rate is 20/min, and temperature is 110 RBCs/hpf, and 8 WBCs/hpf. There is 1+
38.0C (100.4F). The infant is pale but he- protein and 3+ heme, but no glucose, nitrite,
modynamically stable. He is crying and in sig- or leukocyte esterase. What is the best next step
nicant discomfort, particularly on abdominal in diagnosis?
palpation. He has bloody mucus in his diaper.
(A) Contrast-enhanced CT of the head
Which is the most appropriate next step to
(B) Lumbar puncture
both diagnose and potentially treat this condi-
(C) MRI with gadolinium contrast
tion?
(D) Non-contrast-enhanced CT of the head
(A) Air contrast or barium enema (E) Referral for ophthalmologic exam
(B) Colonoscopy (F) Tonometry
(C) Endoscopy
(D) Surgery 7. A 10-year-old boy is brought to the clinic be-
(E) Watchful waiting cause of a sore throat, rhinorrhea, and rash that
began 2 days earlier. His parents note that to-
6. A 72-year-old woman is brought to the emer- day his throat is so sore he has been unable to
gency department by her daughter complain- swallow. Yesterday his mother noticed he was
ing of 2 days of headache. The patient de- developing a diffuse rash (see image) that was
scribes her headache as a dull, bilateral ache. nonpainful and nonpruritic. On physical ex-
She has no nausea, vomiting, or visual changes. amination his oropharynx is markedly inamed
The patient has lived independently in the with tonsillar exudates. A course of antibiotics
same house for 45 years. The daughter last saw is prescribed. Which of the following is the
her mother 2 days ago, when she seemed ne. most likely diagnosis?
The patient says that her doctor treats her for
high cholesterol and high blood pressure with
simvastatin, hydrochlorothiazide, metoprolol,
lisinopril, and calcium supplements with vita-
min D. At her last visit, however, the daugther

FULL-LENGTH EXAMS
noticed several of the pills were running low.
Her temperature is 37.5C (100F), heart rate
is 95/min, blood pressure is 215/105 mm Hg,
respiratory rate is 19/min, and oxygen satura-
tion is 97% on 2-L nasal cannula. Physical ex-
amination nds a pleasant elderly woman in
no acute distress. Fundoscopic examination is
notable for papilledema and several retinal
hemorrhages. Neurologic, cardiac, chest, and
abdominal examinations are unremarkable.
Laboratory tests show:
Reproduced, with permission, from Wolff K, Johnson RA,
Na+: 144 mEq/L Surmond D. Fitzpatricks Color Atlas & Synopsis of Clini-
K+: 4.8 mEq/L cal Dermatology, 5th edition. New York: McGraw-Hill,
Cl: 110 mEq/L 2005: Figure 22-35.
HCO3: 23 mEq/L (A) Fifth disease
Blood urea nitrogen: 25 mg/dL (B) Impetigo
Creatine: 1.4 mg/dL (C) Kawasakis disease
Test Block 6

(D) Measles
(E) Mononucleosis
(F) Scarlet fever
644 Section II: Full-Length Examinations Questions

8. A 40-year-old African-American woman pre- 10. A 55-year-old otherwise healthy man presents
sents to the emergency department complain- to the clinic because of a cough productive of
ing of confusion, colicky abdominal pain, vom- green sputum, fever, chills, and pleuritic chest
iting, spiking fevers, and chills. She has noted pain over the past 4 days. Physical examination
her stools over the past 2 days to be lighter col- reveals egophony and increased tactile fremi-
ored than usual. Her temperature is 39.6C tus in the left lower lobe. Complete blood cell
(103.3F), pulse is 120/min, blood pressure is count shows a WBC count of 20,000/mm
85/55 mm Hg, and respiratory rate is 25/min. with 15% bands. X-ray of the chest is shown in
On examination she is in obvious distress, the image. Which of the following is the most
sclera are icteric, bowel sounds are normal, likely pathogen causing this mans illness?
and abdomen is tender to palpation in the
right upper quadrant without rebound. Labora-
tory values reveal a WBC count of 18,000, to-
tal bilirubin of 4.5, aspartate aminotransferase
of 90 I/U, alanine aminotransferase of 100 I/U,
and alkaline phosphatase of 250 I/U. CT re-
veals dilated intrahepatic bile ducts with stones
in the gallbladder and pneumobilia. Which of
the following has rst priority in the manage-
ment of this patient?
(A) Establish vascular access and give intrave-
nous uids
(B) Further image the biliary system to pin-
point the obstruction
(C) Perform emergent decompression of bil-
iary system Reproduced, with permission, from Chen MYM, Pope
TL, Ott DJ. Basic Radiology. New York: McGraw-Hill,
(D) Place nasogastric tube
2004: Figure 4-19.
(E) Prescribe empiric oral antibiotics
FULL-LENGTH EXAMS

(A) Escherichia coli


9. A 20-year-old white man presents to the emer- (B) Klebsiella
gency department with a mild cough, weight (C) Mycoplasma pneumoniae
loss, low-grade fevers, and night sweats. He (D) Pseudomonas
lives in Tennessee and spends his weekends ex- (E) Staphylococcus aureus
ploring caves. X-ray of the chest shows small (F) Streptococcus pneumoniae
apical cavitary lesions bilaterally. The physi-
cian diagnosis a cavitary pneumonia. Puried 11. A mother and father bring their 7-year-old son
protein derivative test is negative. He denies to the pediatrician because he is developing fa-
any history of asthma, allergies, smoking, and cial hair. The mother states he has always been
immunosuppression. Which of the following is healthy and had been developing normally un-
the most appropriate next step(s) in manage- til his voice began squeaking about 1 month
ment? ago. Last week while her son was getting out of
the shower, she noticed some chest hair. The
(A) Ceftriaxone
father states that he went through puberty at a
(B) Itraconazole and possibly amphotericin B
young age and was shaving by age 14 years. On
(C) Nystatin swish and swallow
physical examination the boy is a thin, well-
(D) Pyrazinamide
appearing child. His examination is remark-
Test Block 6

(E) Trimethoprim-sulfamethoxazole
able only for some hair growth on his upper lip
and sparse chest hair. Levels of which of the
following would most likely be low in this pa-
tient?
Test Block 6 Questions 645

(A) 5-Reductase 14. A 14-year-old girl is brought to the emergency


(B) 17-Hydroxyprogesterone department by paramedics while seizing. Her
(C) Androstenedione parents, who report she has a history of sei-
(D) Cortisol zures, found her in her bedroom and were un-
(E) Dihydrotestosterone sure of how long she had been seizing. On ar-
rival the patients pulse is 125/min, blood
12. A 39-year-old man with hypertension and dia- pressure is 160/100 mm Hg, temperature is
betes presents to the emergency department 38.9C (102F), and respiratory rate is 22/min.
with acute onset painless loss of vision in his On examination the patient has her arms ex-
left eye. He reports that he has had eeting epi- tended rigidly at her sides and she is arching
sodes of vision loss in the past. His blood pres- her back rhythmically and appears to be aspi-
sure is 158/88 mm Hg, pulse is 80/min, and re- rating. After several failed attempts at nasopha-
spiratory rate is 16/min. Ocular examination ryngeal intubation, rapid sequence induction
shows decreased visual acuity in the left eye allowed placement of a nasopharyngeal airway.
and a relative afferent pupillary defect. Fundu- Several infusions of intravenous benzodiaz-
scopic examination shows a cherry red spot epines, phenytoin, and phenobarbital produce
with surrounding edema. Laboratory values re- little effect. Which of the following statements
veal a WBC count of 10,600/mm, a glucose regarding this patients condition is true?
level of 120 mg/dL, and an erythrocyte sedi-
(A) Irreversible neuronal injury usually occurs
mentation rate of 10 mm/hr. Which of the fol-
after 10 minutes
lowing is the most likely diagnosis?
(B) Management options are strongly inu-
(A) Acute ophthalmic artery occlusion enced by the cause of the seizure
(B) Central retinal artery occlusion (C) Mortality following status epilepticus is
(C) Central retinal vein occlusion >50%
(D) Giant cell arteritis (D) Patients with minimal myoclonic activity
(E) Tay-Sachs disease are not at risk for serious complications
(E) Rhabdomyolysis is a potential life-
13. A 14-month-old boy is playing in the living threatening complication

FULL-LENGTH EXAMS
room with his 3.5-year-old sister. Their mother
leaves briey to answer the phone. On return- 15. A 22-year-old G0 woman presents to the clinic
ing to the living room, the mother notices that seeking a rell of her oral contraceptive pills.
the children are still playing together with no She was started on a combination pill 2 years
noticeable change in behavior. Later in the ago for menorrhagia and has had no complica-
day, the boy refuses feeding and seems to have tions. She has no other past medical or surgical
excessive salivation and a dry cough. The history and denies smoking. She is not cur-
mother is concerned and decides to have her rently sexually active. Which of the following
son evaluated at the local emergency depart- would constitute an absolute contraindication
ment. Which of the following is the most logi- to the use of oral contraceptive pills?
cal next step in evaluating this child based
(A) Active liver disease
solely on the history presented?
(B) History of atherosclerosis
(A) Barium contrast radiologic evaluation (C) Lactation
(B) Complete blood cell count with differen- (D) Occasional smoking (three to four ciga-
tial rettes per week)
(C) Endoscopic evaluation (E) Uterine broids
(D) Parental reassurance that the child is
Test Block 6

healthy
(E) Radiologic studies of the neck, chest, and
abdomen
646 Section II: Full-Length Examinations Questions

16. A 22-year-old woman is brought to the hospital 18. A 47-year-old man presents to his physician
by her family because of progressive weight loss with mouth pain for the past 2 weeks. It seems
over the past 4 months, fatigue, depression, to be getting worse and is making eating un-
and refusal to eat. Her blood pressure is 95/65 comfortable. He rst noticed a small blister on
mm Hg and pulse is 120/min. Her body mass the inside of his cheek that popped immedi-
index is 16 kg/m. The patient is hospitalized ately after forming and has been very painful.
and undergoes a strict refeeding protocol, in Two more oral ulcers and four ulcerated le-
addition to psychotherapy. After 3 weeks of sions on his back have developed since then
treatment she is started on a pharmacologic and are not healing well. He denies fevers,
agent to treat her persistent symptoms of de- chills, diarrhea, constipation, or abdominal
pression. The following morning, the patient pain, and has no signicant past medical his-
experiences a generalized tonic-clonic seizure. tory. On examination there are three shallow
Which medication is the most likely cause of ulcerations, 5 mm to 1 cm in diameter, and a
this patients seizure? accid blister on the oral mucosa and tongue.
The lower gingiva are eroded in a 1-cm area,
(A) Aripiprazole
and an ulceration can be visualized on the pos-
(B) Bupropion
terior oropharynx. Which of the following is
(C) Escitalopram
the most likely cause of these symptoms?
(D) Fluoxetine
(E) Imipramine (A) Antibodies against (intraepidermal) des-
moglein proteins
17. A 24-year-old woman presents to the emergency (B) Antibodies against (subepidermal) hemi-
department after being involved in a motor ve- desmosome glycoproteins
hicle accident. She is moving all extremities and (C) Antibodies against basement membrane
is conversant. She indicates that while driving at (D) Aphthous ulcers
30 mph, she veered off the road to avoid an on- (E) Vesicle formation due to herpes virus in-
coming car and crashed into a guard fence. Her fection
airbag deployed, and she denies losing con-
sciousness. She was placed in a stabilization col- 19. A 60-year-old man with a newly diagnosed fo-
FULL-LENGTH EXAMS

lar in the eld. In the emergency department cal brain lesion presents with aphasia. His abil-
she complains of some minor cuts on her face ity to repeat words and his uency are both im-
and arms as well as neck pain. Her temperature paired, and his language comprehension is also
is 37.2C (99.0F), blood pressure is 134/80 mm diminished. He has no other symptoms, and
Hg, pulse is 90/min, and respiratory rate is 12/ the remainder of his neurologic examination is
min. On examination she is alert and oriented, unremarkable. From what kind of aphasia is he
and extraocular movements are intact. Her pu- most likely suffering?
pils are equally round and reactive to light. She
(A) Brocas aphasia
has full strength and sensation in her face and
(B) Conduction aphasia
extremities. Neck examination reveals diffuse
(C) Global aphasia
soreness posteriorly but no midline tenderness.
(D) Transcortical motor aphasia
There are no dislocations or step-offs. Breath-
(E) Wernickes aphasia
alyzer test and urine toxicology screen are nega-
tive. What is the most appropriate next step in
management?
(A) Halo placement
(B) High-dose corticosteroids
Test Block 6

(C) Lateral x-ray of the cervical spine with ex-


ion and extension
(D) MRI of the cervical spine
(E) Remove collar and allow patient to slowly
ex, extend, and rotate neck
Test Block 6 Questions 647

20. A 60-year-old man presents to a urologist com- 22. A 3000-g (6.6-lb) girl is born at 37 weeks gesta-
plaining of difculty having an erection. The tion by spontaneous vaginal delivery to a
man says the quality of his erections has been 26-year-old mother of Chinese origin. Preg-
getting worse over the past year. He has a his- nancy and birth were uncomplicated, with Ap-
tory of hyperlipidemia, for which he is non- gar scores of 9 and 10 at 1 and 5 minutes, re-
compliant with his medications, and has spectively. Physical examination and newborn
smoked one pack per day for the past 40 years. laboratory results are normal. The infant is ex-
On review of systems, he notes that his but- clusively breast-fed and is latching on and feed-
tocks and lower back hurt when he walks. His ing well with adequate urine output. She pre-
heart rate is 70/min and blood pressure is sents on her fth day of life because of
130/80 mm Hg. His examination is notable for increasing jaundice and scleral icterus. She is
distant breath sounds bilaterally and global at- afebrile with stable vital signs. Laboratory stud-
rophy of the lower extremities. Which of the ies show:
following would the physician expect to nd?
Total bilirubin: 22 mg/dL
ANKLE BRACHIAL Unconjugated bilirubin: 19.7 mg/dL
ANKLE BRACHIAL INDEX Conjugated bilirubin: 0.3 mg/dL
CHOICE FEMORAL PULSES INDEX (at rest)* (after exercise) WBC count: 12,000/mm
A absent 0.3 0.3 What is the best next step in treatment?
B weak 0.3 0.6
(A) Abdominal ultrasound
C weak 0.6 0.3 (B) Discharge the patient on formula and with
a beroptic bilirubin blanket
D increased 0.2 0.2
(C) Erythromycin
E increased 0.6 0.3 (D) Exchange transfusion
*Normal 0.91.3
(E) Phototherapy

23. A 52-year-old woman comes to the clinic to be


(A) A evaluated for hypertension. On her previous

FULL-LENGTH EXAMS
(B) B visit her blood pressure was 132/96 mm Hg.
(C) C On this visit her blood pressure is 130/98 mm
(D) D Hg. She has never taken medication or been
(E) E previously diagnosed with hypertension. Physi-
cal examination shows that her point of maxi-
21. A 68-year-old man presents to his primary care mal impulse is displaced to the sixth intercostal
physician with a persistent cough over the past space and she has no signs of congestive heart
2 months, with dyspnea on exertion. Physical failure or edema. Laboratory tests show a so-
examination reveals decreased breath sounds dium level of 148 mEq/L, potassium of 2.4
in all lung elds and a prolonged expiratory mEq/L, chloride of 111 mEq/L, and bicarbon-
phase. Pulmonary function tests reveal a de- ate of 27 mEq/L. What is the best rst step in
creased FEV1, decreased FEV1:FVC ratio, and diagnosis?
increased residual volume. What would this
patients x-ray of the chest most likely show? (A) Measure 24-hour urine cortisol excretions
(B) Measure plasma renin and aldosterone
(A) Apical cavitary lesion levels
(B) Diffuse bilateral inltrates (C) Measure urinary metanephrine excretion
(C) Hyperination with diaphragmatic atten- (D) Renal angiography
Test Block 6

ing and increased anteroposterior diameter (E) Repeat blood pressure measurement
(D) Lobar consolidation
(E) Mass lesion with mediastinal adenopathy
648 Section II: Full-Length Examinations Questions

24. A 67-year-old man comes to the emergency de- (A) Allopurinol


partment complaining of a red rash (see im- (B) Colchicine
age) and fever for >3 days. His past medical (C) Indomethacin
history is signicant for hypertension, diabetes, (D) Intra-articular corticosteroid injections
obesity, and calcic aortic stenosis. His temper- (E) Niacin
ature is 39.2C (102.6F), blood pressure is
145/86 mm Hg, and heart rate is 110/min. He 26. A young woman who is 5 months pregnant vis-
has a III/VI systolic murmur at his left sternal its her primary care physician. She has had a
border not documented on previous examina- headache and myalgia for 2 days, and she now
tions and inspiratory crackles at both lung has a fever to 38.3C (101F). Today she also
bases. He has 2+ pitting edema bilaterally in began to develop a maculopapular and pete-
her ankles to the knees. Which of the follow- chial rash on her ankles and wrists that is
ing is the most appropriate initial pharmaco- spreading centrally and to her palms and soles.
therapy? She has had no sick contacts. Five days ago she
spent the weekend with her family at their
cabin in the North Carolina woods. Given the
most likely diagnosis, what is the most appro-
priate treatment for this womans illness?
(A) Chloramphenicol
(B) Ciprooxacin
(C) Doxycycline
(D) Penicillin
(E) Trimethoprim-sulfamethoxazole

27. A 38-year-old G5P4 woman at 37 weeks gesta-


tion has a history of preeclampsia in previous
Reproduced, with permission, from Wolff K, Johnson RA,
pregnancies. She presented to the obstetric
Surmond D. Fitzpatricks Color Atlas & Synopsis of Clini-
cal Dermatology, 5th edition. New York: McGraw-Hill, clinic for her initial screening at 10 weeks ges-
FULL-LENGTH EXAMS

2005: Figure 22-38. tation but was subsequently lost to follow-up.


At her initial visit the patient had a blood pres-
(A) Gentamicin only sure of 140/70 mm Hg and no proteinuria.
(B) Hydrochlorothiazide Since then, the patient reports experiencing
(C) Metoprolol and enalapril headaches, some visual disturbances, and light-
(D) Penicillin and gentamicin headedness but has not sought health care for
(E) Penicillin only any of her symptoms. The patient now presents
to the emergency department complaining of a
25. A 77-year-old woman presents to her primary headache. Her blood pressure is 162/90 mm
care physician for evaluation. The patient Hg. An ultrasound of the uterus is likely to re-
states that other than several episodes of gout veal which of the following?
in the past 4 years, she has been quite healthy.
She currently has no complaints but wants to (A) Intrauterine growth restriction
discuss treatment and/or prophylaxis options to (B) Macrosomia
help protect her against recurrent episodes of (C) Placenta previa
gout. The patients musculoskeletal examina- (D) Polyhydramnios
tion is within normal limits, with full range of (E) Rocker-bottom feet
motion and no focal tenderness in any of her
Test Block 6

joints. First-line prophylactic therapy for


chronic gout involves which treatment?
Test Block 6 Questions 649

28. A mother was recently bathing her 3-year-old uids, but the patients condition continues to
girl when she felt a bulge in her childs left decline. Which of the following is the most ap-
lower ank. Chart review indicates the child has propriate next step in management?
had exomphalos, macroglossia, and gigantism
(A) Clonidine
since birth. On physical examination the tod-
(B) Continued use of cooling blankets and in-
dler appears alert and playful, but her blood
travenous uids
pressure is slightly elevated. She has a 7- 4-cm
(C) Dantrolene
palpable, immobile mass in her left lower abdo-
(D) Ice packs to the axillae and groin
men. Results of ultrasound-guided biopsy of the
(E) Lorazepam
mass are pending. Which of the following is the
most likely pathologic nding on biopsy?
31. A 74-year-old woman presents to the clinic be-
(A) Epithelial, blastic, and stromal elements cause of a chronic cough. She also reports oc-
(B) Serous uid casional hemoptysis and shortness of breath in
(C) Small blue cells addition to fatigue and low mood. She has
(D) Teeth and cartilage smoked 2 packs per day for 40 years. She is also
(E) Urine troubled by constipation and lack of appetite.
Four months ago she passed a kidney stone.
29. A 47-year-old man involved in a motor vehicle X-ray of the chest reveals a 4-cm lung mass.
accident is unresponsive and unarousable. His Work-up for metastatic disease is negative. Lab-
respiratory rate is 8/min, heart rate is 120/min, oratory tests show a sodium level of 136 mEq/L,
blood pressure is 90/65 mm Hg, and tempera- potassium of 4.3 mEq/L, and calcium of 12.9
ture is 37.1C (98.8F). He does not open his mEq/L. What is the most likely histologic type
eyes, nor does he vocalize spontaneously or in of lung cancer?
response to painful stimuli such as a sternal
(A) Adenocarcinoma
rub or nail bed stimulation. He makes no spon-
(B) Bronchoalveolar carcinoma
taneous movements, but his muscles do ex in
(C) Large cell carcinoma
response to painful stimuli. Which of the fol-
(D) Small cell lung cancer
lowing is the most appropriate next step?
(E) Squamous cell lung cancer

FULL-LENGTH EXAMS
(A) Administer intravenous mannitol
(B) CT of the head 32. A 66-year-old man presents to his primary care
(C) Intubate physician complaining of abdominal pain and
(D) Measure blood glucose yellowing of the skin and eyes. The pain has
(E) Push tissue plasminogen activator been getting progressively worse over the last
few months, radiates to the back, and is associ-
30. Approximately 8 hours after receiving an injec- ated with nausea, weakness, and a 6.8-kg (15-
tion of haloperidol, an extremely agitated man lb) weight loss. Physical examination reveals a
in the emergency department begins com- cachectic, icteric man in no acute distress. His
plaining that his room is too hot. The nurse temperature is 36C (96.8F), heart rate is 85/
subsequently reports a temperature of 40.2C min, and blood pressure is 130/85 mm Hg. Ab-
(104.4F), pulse of 114/min, and blood pres- dominal examination is notable for a palpable,
sure of 182/115 mm Hg. An hour later the pa- nontender gallbladder. What are the chances
tient is stiff and sweating profusely. His WBC that his condition will be curable with surgery?
count is 27,000/mm and his creatinine kinase
(A) 1%5%
level is 52,140 U/L. Over the next 24 hours he
(B) 10%20%
becomes minimally responsive to verbal stim-
(C) 30%40%
Test Block 6

uli. The patient is placed on a cardiac monitor


(D) 50%60%
for constant observation of vital signs, and at-
(E) 95%99%
tempts are made to decrease the patients tem-
perature with cooling blankets and intravenous
650 Section II: Full-Length Examinations Questions

33. A 5-month-old girl presents to the hospital with (B) Admit to hospital, order nothing by
signs of severe sepsis and pneumonia. At ad- mouth, place nasogastric tube, and give
mission she is hemodynamically unstable with intravenous uids
a heart rate of 78/min, blood pressure of 55/38 (C) Admit to hospital, place rectal tube, ad-
mm Hg, and respiratory rate of 40/min. She is minister enemas, and disimpact bowels
directly admitted to the pediatric intensive care (D) Emergency laparotomy
unit, where she abruptly decompensates and (E) Order nothing by mouth and observe in
attempts at resuscitation are unsuccessful. Post- emergency department
mortem examination of the child demonstrates
a child who is small for her age, with a small 35. Following presentation to his primary care doc-
thymus (0.86 g) that was located within the an- tor, a 59-year-old man was diagnosed with mul-
terior neck. Pathologic examination of the thy- tiple myeloma. The patient has been partici-
mic sample demonstrates few thymocytes and pating in a clinical trial for a chemotherapeutic
a lack of corticomedullary differentiation. Ad- agent and has begun to show early signs of dis-
ditional ndings at autopsy included absent ease remission. He has been immobile for 3
tonsils, adenoids, and Peyers patches. What weeks due to fatigue and bone tenderness. He
test, if performed, would have diagnosed this now presents to the emergency department
patients condition and possibly saved her life? with new-onset erythema, swelling, and tender-
ness of the right lower extremity. The patient
(A) Blood cultures with antibiotic sensitivities
states he has never had anything like this in the
at time of presentation
past. His temperature is 37.3C (99.2F), blood
(B) Echocardiography at time of presentation
pressure is 136/84 mm Hg, pulse is 74/min,
(C) Routine neonatal screening at time of
and respiratory rate is 13/min. What is the rst
birth
step in management of this patient?
(D) Serum uric acid level performed anytime
(E) WBC count with differential at time of (A) Cessation of chemotherapy
birth (B) Inferior vena cava lter
(C) Intravenous heparin
34. A 40-year-old woman presents to the emergency (D) Ventilation-perfusion scan
FULL-LENGTH EXAMS

department because of 3 hours of cramping ab- (E) Warfarin


dominal pain and multiple episodes of vomiting
after which the pain was temporarily relieved. 36. A 57-year-old white woman presents to her pri-
She says the vomit is greenish. Her last bowel mary care physician complaining of pain in
movement was 2 days prior to presentation, but her shoulders and hips. She reports progressive
she continues to pass atus. Her vital signs are difculty putting her bra and stockings on in
normal except for a pulse of 115/min. On physi- the morning. She reports recent fevers, mal-
cal examination she is in mild distress, and her aise, and a 2.7-kg (6-lb) unintentional weight
abdomen is somewhat distended and diffusely loss over the past 3 months. She reports no vi-
tender with high-pitched bowel sounds and sual disturbances, headache, or jaw claudica-
multiple old surgical scars. She does not have tion. Her temperature is 37.9C (100.3F),
rebound tenderness. Complete blood cell blood pressure is 137/84 mm Hg, heart rate is
count, urinalysis, and electrolyte levels are unre- 86/min, and respiratory rate is 12/min. Physical
markable. Flat and upright x-rays of the abdo- examination reveals decreased active range of
men reveal multiple air-uid levels and residual motion of the shoulders and hips bilaterally
air in the colon. Which of the following is the but normal muscle strength throughout. Re-
most appropriate management? exes are 2+ bilaterally. Laboratory studies
show an erythrocyte sedimentation rate of 76
Test Block 6

(A) Admit to hospital, administer intravenous


mm/hr, negative rheumatoid factor, and nor-
uids, and perform endoscopic retrograde
mal creatine kinase and aldolase levels. Radio-
cholangiopancreatography
graphs of the shoulders and hips reveal no joint
abnormalities. What is the most appropriate
treatment for this patients condition?
Test Block 6 Questions 651

(A) Amitriptyline 39. A 6-year-old boy presents to the clinic with un-
(B) High-dose prednisone relenting fever for >10 days with headache. Ac-
(C) Iniximab etaminophen has not improved the fever. His
(D) Low-dose prednisone mother says that he sounds stuffed up and
(E) Methotrexate that he has developed a cough over the last 2
days. He had a similar episode 2 months ago
37. An 84-year-old woman suffering from terminal and was treated with amoxicillin with resolu-
breast cancer is hospitalized and intubated due tion of symptoms. On physical examination the
to pneumonia. Her clinical condition worsens, child has a widened nose bridge and nasal pol-
and 1 week later her health care proxy decides yps, with purulent discharge from the ostiae bi-
to withdraw care and offer comfort measures laterally. X-ray of the chest is shown in the im-
only. Her family requests that she be given age. Which of the following is the most likely
high doses of morphine to ensure that she will diagnosis?
not be in pain. Her physicians agree to this,
even though they know that opioids may fur-
ther depress her respirations. The physicians
willingness to treat this patient with potentially
fatal drugs for palliative purposes is an example
of which principle?
(A) Autonomy
(B) Dereliction
(C) Double effect
(D) Malpractice
(E) Physician-assisted suicide
Reproduced, with permission, from USMLERx.com.
38. A 65-year-old woman with a history of myocar-
dial infarction, hypertension, and asthma pre- (A) Allergic rhinitis
sents with new-onset hallucinations. She can (B) Cystic brosis

FULL-LENGTH EXAMS
no longer sleep at night because she sees small (C) Kartageners syndrome
children and cats in her apartment. She thinks (D) Nasal foreign body
she must be going crazy and is too frightened (E) X-linked severe combined immunode-
to explain the symptoms to her husband. She ciency
has no prior psychiatric history. Her blood pres-
sure is 115/80 mm Hg supine and 90/60 mm
Hg standing. Physical examination reveals an
alert, oriented elderly woman with a slight rest-
ing tremor and mild rigidity in her upper and
lower extremities, but no cogwheeling. Mini-
Mental State Examination reveals decits in
long-term recall. What abnormal neuronal
nding is expected?
(A) Birefringent crystals
(B) Dark pigmentation in the substantia nigra
(C) Eosinophilic cytoplasmic inclusions
(D) Hypersegmented nuclei
Test Block 6

(E) Neurobrillary tangles and webs


652 Section II: Full-Length Examinations Questions

40. A 75-year-old cognitively intact woman with a 42. A 25-year-old woman is recovering from an un-
history of diabetes, coronary artery disease, hy- complicated open appendectomy for acute ap-
percholesterolemia, and peripheral vascular pendicitis. She has no signicant past medical
disease is diagnosed with glioblastoma multi- history and is not taking any medications. Her
forme. She is referred to a neurologic oncolo- temperature is 36.7C (98.1F), blood pressure
gist. The treatment for glioblastoma multi- is 118/78 mm Hg, pulse is 72/min and regular,
forme that has been shown to improve respiratory rate is 14/min, and oxygen satura-
mortality is resection followed by radiation tion is 99% on room air. Physical examination
therapy. Given her multiple cardiac risk fac- is signicant for a 5-cm (2-in) incision in the
tors, however, this patient is a poor surgical right lower quadrant, which is clean, dry, and
candidate. The oncologist is concerned that intact. Which of the following should be rec-
the risks of resection may outweigh the benets ommended to prevent the development of ve-
and it is not clear which treatment option nous thromboembolism?
would provide the best quality-of-life outcome
(A) Early ambulation
for the patient. Which of the following is the
(B) Intermittent pneumatic compression with
best way to approach this issue with the pa-
or without elastic stockings
tient?
(C) Low-molecular-weight heparin
(A) Asking the patient to take time to digest (D) Subcutaneous heparin
what has been said and not ask questions (E) Warfarin plus elastic stockings
(B) Detailing the intricate specics of numer-
ous studies investigating the treatment op- 43. An 8-year-old boy is brought to the physician
tions so as to completely inform the patient because of refusal to walk for 3 days. On ques-
(C) Discussing the possible treatment modali- tioning he states that both his legs hurt. His
ties while disclosing the relative morbidi- mother states he has not been himself for the
ties and mortality associated with each past month and has been sleeping much
treatment option more than usual. His temperature is 38.4C
(D) Insist that the patient discuss her medical (101.1F), heart rate is 164/min, respiratory
options with her family members present rate is 42/min, and blood pressure is 90/54 mm
FULL-LENGTH EXAMS

(E) Presenting the possible treatment modali- Hg. The child is pale and listless. Severe hep-
ties with the highest chance of improving atomegaly is noted. There are no obvious de-
mortality formities of the lower extremities or hip, knee,
or ankle joints. Laboratory tests show:
41. A 72-year-old African-American woman with a
WBC count: 98,000/mm
long history of gastroesophageal reux disease
Hemoglobin: 8.9 g/dL
and heavy tobacco use presents to her primary
Platelet count: 75,000/mm
care physician complaining of weight loss and
Na+: 135 mEq/L
difculty swallowing that has progressed over
K+: 4.0 mEq/L
the past few years such that solids are nearly
Cl: 104 mEq/L
impossible to get down. She also complains of
Blood urea nitrogen: 8 mg/dL
a constant cough and a funny sound when
Creatinine: 0.6 mg/dL
she breathes. Her vital signs are stable. The
physician orders an endoscopy and nds a sam- What is the best next step in management?
ple of columnar epithelium on biopsy. Which (A) Bone marrow biopsy
of the following is the most likely diagnosis? (B) Delivery of emergent chemotherapy
(A) Achalasia (C) Platelet transfusion
Test Block 6

(B) Adenocarcinoma of the esophagus (D) Typing of his brothers bone marrow to as-
(C) Esophageal varices sess the potential for a matched related
(D) Gastric cancer bone marrow transplant
(E) Hiatal hernia (E) X-ray of the chest
Test Block 6 Questions 653

E X T E N D E D M ATC H I N G dominal masses and no nodules on prostate ex-


amination. Relevant laboratory ndings are a
WBC count of 5000/mm3 and a creatinine
The response options for the next 3 items are
level of 0.9 mg/dL.
the same. Select one answer for each item
in the set.
45. A 40-year-old man comes to the clinic because
of hematuria and occasional ank pain. He has
For each patient with hematuria, select the most
a family history of kidney disease; his mother
likely diagnosis.
and her two sisters had kidney disease and all
(A) Bladder cancer three were ne until their 40s. His temperature
(B) Coagulation disorder is 36.9C (98.4F) and blood pressure is
(C) Exercise-induced hematuria 130/80 mm Hg. His serum creatinine level is
(D) Myoglobinuria 1.5 mg/dL.
(E) Nephrolithiasis
(F) Polycystic kidney disease 46. A 57-year-old man presents to the physician be-
(G) Prostate cancer cause of hematuria. He denies pain on urina-
(H) Pyelonephritis tion, but reports ank pain that has been rela-
(I) Renal cell carcinoma tively constant over the past month. He also
(J) Urinary tract infection notes intermittent fever and weight loss over
the past 6 weeks. His temperature is 37.7C
44. A 70-year-old man presents to the physicians (99.9F). He has no costovertebral angle ten-
ofce with painless hematuria. He reports a derness. He has a WBC count of 5200/mm3
50-pack-year smoking history. His temperature and a hematocrit of 58%.
is 37.4C (99.3F). Examination reveals no ab-

FULL-LENGTH EXAMS
Test Block 6
654 Section II: Full-Length Examinations Answers

AN S W E R S

1. The correct answer is A. The patient has hy- bone pain that is worse with movement, lytic
pocalcemia, hyperphosphatemia, and hyper- bone lesions, compression fractures, and hy-
parathyroidism. These ndings are consistent percalcemia due to increased osteoclast activ-
with secondary hyperparathyroidism, which is ity. Renal failure is also common due to depo-
most commonly due to end-stage renal disease. sition of antibody light chains and associated
Hyperphosphatemia is due to impaired secre- renal amyloidosis. Diagnosis is aided by a bone
tion of phosphate in the proximal tubule. This marrow biopsy showing a clonal plasma cell
contributes to hypocalcemia by binding of population of >10% of the marrow, monoclo-
phosphate to calcium. Hypocalcemia is also nal immunoglobulin (M component) in the
due to decient production of 1,25-dihy- serum or the urine, and other features like lytic
droxyvitamin D in the kidney, which leads to bone lesions, anemia, and elevated serum cal-
decreased absorption of ingested calcium. cium.
Parathyroid hormone levels increase to com-
Answer A is incorrect. Abdominal ultrasound
pensate for the hypocalcemia and hyperphos-
that showed stenosis of the iliac artery would
phatemia.
be evidence of vascular insufciency. Periph-
Answer B is incorrect. Hyperthyroidism can eral vascular disease would present as claudica-
cause hypercalcemia, not hypocalcemia. The tion (leg pain worse with exertion and relieved
mechanism is via increased bone turnover, by rest). It would not, however, account for the
with bone resorption exceeding new bone for- punched-out lesions seen on x-ray.
mation.
Answer B is incorrect. A densitometry scan
Answer C is incorrect. Lung cancer, particu- would be useful to rule out osteoporosis, which
larly squamous cell cancer, is a common cause is unlikely given the patients age and gender,
of hypercalcemia due to production of parathy- and is also unlikely given the patients x-ray
roid hormone-related protein. Calcium will be ndings.
elevated, phosphate will be decreased, and para-
FULL-LENGTH EXAMS

Answer D is incorrect. Decreased serum vita-


thyroid hormone will be low or normal.
min D would be seen in osteomalacia, which
Answer D is incorrect. A parathyroid adenoma often presents with pathologic fractures, not
is the most common cause of primary hyper- lytic lesions on x-ray.
parathyroidism. Although this condition will
Answer E is incorrect. Nerve entrapment or
present with elevated levels of parathyroid hor-
disease can be diagnosed with nerve conduc-
mone, the patient will have hypercalcemia and
tion studies, and can be the cause of leg pain.
hypophosphatemia.
This diagnosis, however, is less likely given the
Answer E is incorrect. Sarcoidosis is a cause of lack of pain on the straight leg raise, as well as
hypercalcemia, not hypocalcemia. The mecha- the radiologic ndings.
nism of hypercalcemia is increased production
of 1,25-dihydroxyvitamin D by macrophages in 3. The correct answer is E. The clinical scenario
the granulomas. Parathyroid hormone will not represents type IV renal tubular acidosis (RTA),
be elevated. the most common type of RTA. The distur-
bance can be thought of as relative aldosterone
2. The correct answer is C. Multiple myeloma is deciency or resistance, which is most com-
a malignant proliferation of plasma cells, monly found in diabetic nephropathy. A hy-
Test Block 6

which causes them to overproduce monoclo- poaldosterone state leads to hyperkalemia, re-
nal immunoglobulin. Due to the defective im- sulting in decreased ammonium production
mune system, patients will often have recurrent and urine acidication. The patients arterial
infections, of which pneumonia and pyelone- blood gas shows a process consistent with a
phritis are the most common. Patients develop metabolic acidosis with respiratory compensa-
Test Block 6 Answers 655

tion, while his electrolyte panel shows that the Answer C is incorrect. The second stage of
acidosis is hyperchloremic in nature. The labor is the time starting with complete cervi-
ECG ndings reect the changes found in hy- cal dilatation and ending with delivery of the
perkalemia. Type IV RTA can also be seen infant. Cephalopelvic disproportion would be
with tubulointerstitial renal diseases, hyperten- observed prior to this stage.
sive nephrosclerosis, and HIV nephropathy.
Answer D is incorrect. The third stage of labor
Drugs such as angiotensin-converting enzyme
starts from the time of delivery of the infant to
inhibitors, nonsteroidal anti-inammatory
the delivery of the placenta. Because this stage
drugs, and trimethoprim can all cause hyper-
occurs after the fetus is delivered, it is not re-
kalemia leading to type IV RTA. Treatment is
lated to the diagnosis of cephalopelvic dispro-
furosemide and potassium binding agents such
portion.
as sodium polystyrene sulfonate.
Answer E is incorrect. The fourth stage of la-
Answer A is incorrect. The hyperkalemia that
bor is the immediate postpartum period after
is present is secondary to relative aldosterone
delivery of the placenta. During this stage,
deciency. Hyperkalemia is not the precipitat-
which lasts approximately 2 hours, marked ma-
ing event.
ternal physiologic changes occur.
Answer B is incorrect. Hyperventilation may
cause a primary respiratory alkalosis. This pa- 5. The correct answer is A. This case presents a
tient has a primary metabolic acidosis with re- classic picture of intussusception, which is the
spiratory compensation. most common cause of bowel obstruction in
the rst 2 years of life. Its etiology is unknown,
Answer C is incorrect. Ketoacidosis occurs
but risk factors include male gender, Meckels
almost exclusively in type 1 diabetes mellitus.
diverticulum, intestinal lymphoma, Henoch-
Furthermore, the urinalysis was negative for
Schnlein purpura, parasites, polyps, adenovi-
ketones, and there is not an anion gap, making
rus infection and other upper respiratory infec-
overt ketoacidosis unlikely.
tions, celiac disease, and cystic brosis (CF).
Answer D is incorrect. Metformin can cause In a hemodynamically stable patient without
lactic acidosis, which should cause an anion signs of bowel perforation, air contrast or bar-

FULL-LENGTH EXAMS
gap metabolic acidosis. Based on the labora- ium enema is the appropriate rst step to both
tory values, the anion gap is normal with a diagnose the intussusception and potentially
value of 6. reduce it. Barium enemas will demonstrate a
lling defect or cupping in the head of barium,
4. The correct answer is A. Cephalopelvic dis- where the advancement is impeded by the in-
proportion indicates that there is a relative size tussusception. An alternative to barium is an
difference between the size of the fetus head air enema under uoroscopic guidance, reduc-
and the mothers pelvis, meaning that the head ing the risk of barium spillage. Additionally,
would unlikely t through the pelvic outlet such interventions are often therapeutic by way
during labor in its current position. The diag- of pressure reduction of the intussusception.
nosis is made during the active phase of the
Answer B is incorrect. Colonoscopy has been
rst stage of labor, which is slowed or arrested.
used to diagnose and treat intussusception in
This diagnosis is usually related to fetal mal-
older children and adults. However, in this in-
presentation or malposition.
fant, a barium or air contrast enema should be
Answer B is incorrect. The latent phase of the rst-line treatment, followed by surgery if such
rst stage of labor is characterized by more an intervention fails.
mild, irregular, widely spaced contractions
Test Block 6

Answer C is incorrect. The most common site


with concomitant cervical change and efface-
of intussusception is at the ileocecal junction,
ment. This phase lasts between 4 and 6 hours,
far too distal to be visualized in an upper gas-
depending on the parity of the woman in labor.
trointestinal endoscopy.
Cephalopelvic disproportion would not classi-
cally be clinically diagnosed in this phase.
656 Section II: Full-Length Examinations Answers

Answer D is incorrect. Surgical reduction, the Answer A is incorrect. Contrast in the CT ap-
denitive treatment of this disorder, should pears bright white, the same color as an acute
only be done as rst-line treatment in patients bleed, so when the concern is for intracranial
with signs of bowel perforation or bleeding, hemorrhage, a non-contrast CT scan is pre-
or after the more conservative approach of air ferred.
contrast or barium enema fails. Surgery has the
Answer B is incorrect. Lumbar puncture
additional advantage of allowing the surgeon to
should never be performed when signs of ele-
remove the lead point of the intussusception,
vated intracranial pressure are present because
such as a Meckels diverticulum, reducing the
of the risk of brain stem herniation and death.
chances of recurrence.
Answer C is incorrect. MRI is both slower and
Answer E is incorrect. Intussusception must be
often less available than CT. In a hyperten-
diagnosed and treated at the rst clinical suspi-
sive emergency, time is of the essence, so CT
cion. Although the child is currently hemody-
is preferred. Furthermore, CT is more sensi-
namically stable, the presence of bloody mucus
tive for the detection of intracranial bleeding.
(currant jelly stools) should alert the physician
MRI would be the test of choice to look for an
to the need for further work-up. Prolonged in-
intracranial mass, but this patients headache
tussusception can lead to bowel ischemia, per-
has been present for only 2 days. The classic
foration, and peritonitis. Prognosis is directly
history associated with intracranial mass is a
related to the duration of the intussusception
headache that worsens over weeks to months
before reduction.
accompanied by focal neurologic decits.
6. The correct answer is D. This woman is having Answer E is incorrect. Retinal hemorrhages
a hypertensive emergency that requires immedi- can be due to primary eye pathology, most
ate risk stratication and treatment. Hyperten- commonly diabetic retinopathy. However, this
sive urgency is dened as a systolic blood pres- woman has signs and symptoms indicating a
sure >180 mm Hg and/or a diastolic blood systemic etiology.
pressure >110 mm Hg. Hypertensive emergency
Answer F is incorrect. Acute angle-closure
is distinguished from urgency by the presence of
glaucoma can manifest as headache, but other
FULL-LENGTH EXAMS

end-organ damage: encephalopathy, intracranial


signs would be expected to be present as well.
hemorrhage, acute left ventricular failure with
Acutely elevated intraocular pressure can cause
pulmonary edema, unstable angina, acute myo-
the eye to feel hard to palpation, conjunctival
cardial infarction, dissecting aortic aneurysm,
injection, nonreactive pupil, and blurred vi-
rising serum creatinine level, hematuria, retinal
sion.
hemorrhage, and papilledema. The empty pill
bottles suggest that the patient may have run out
7. The correct answer is F. Scarlet fever consists
of her antihypertensive medications. Further-
of an upper respiratory tract infection that is as-
more, her markedly elevated blood pressure,
sociated with a characteristic rash. It is caused
papilledema, and hematuria clearly point to a
by group A Streptococcus. The rash often ap-
diagnosis of hypertensive emergency. One must
pears within 2448 hours after symptom onset.
also assess for complications other than renal
It usually spares the face, instead affecting the
failure. The nding of papilledema is concern-
neck, trunk, and extremities. The rash is a dif-
ing for an elevated intracranial pressure that can
fuse, erythematous, papular eruption (sandpa-
be caused by a hypertensive intracranial hemor-
per rash) that is often bright red in color and
rhage. Non-contrast CT is the preferred imag-
blanches on pressure. Antibiotics, such as peni-
ing modality to look for hemorrhage. Therapy
cillin, are commonly used in treatment to
for hypertensive emergency focuses on pharma-
Test Block 6

shorten the clinical course of disease and to


cologic lowering of blood pressure. However,
prevent rheumatic fever.
the mean arterial blood pressure should be low-
ered by no more than 25% in the rst hour to Answer A is incorrect. This presentation is not
prevent an iatrogenic ischemic stroke. consistent with fth disease (parvovirus B19),
which typically presents without a prodrome
Test Block 6 Answers 657

and whose major manifestation is a erythema- more important for management than her high
tous slapped cheek rash that is pruritic and WBC count and dilated intrahepatic ducts
maculopapular in nature, and spreads from the with pneumobilia (indicative of suppurative as-
arms to the trunk and legs. Parvovirus B19 may cending cholangitis) is her hemodynamic in-
precipitate an anaplastic crisis in susceptible stability and evidence of hypoperfusion to vital
patient populations. organs (i.e., brain), in other words, her devel-
oping sepsis. Stabilizing her requires large-bore
Answer B is incorrect. This presentation is
vascular access and uid resuscitation to bol-
not consistent with impetigo (Staphylococcus
ster her blood pressure. This takes precedence
aureus or group A Streptococcus), which typi-
over the decompression of her biliary tree; she
cally presents without any constitutional symp-
would not be able to tolerate anesthesia (most
toms and is manifest by supercial bacterial
of which would cause further hypotension)
infection of the skin. The lesions are vesiculo-
without it.
pustular in nature and often rupture (weepy
lesions), leaking exudate that forms a charac- Answer B is incorrect. Adequate imaging has
teristic golden crust. already been performed, and there are more
pressing issues to manage initially.
Answer C is incorrect. This presentation is not
consistent with Kawasakis disease, which is a Answer C is incorrect. Emergent decom-
vasculitic disease that has strict diagnostic cri- pression of the biliary tree is the appropriate
teria, including high-grade fever (>5 days) plus management for acute cholangitis that does
four of the following: bilateral conjunctivitis, not respond to intravenous uid resuscitation,
polymorphous rash, cervical lymphadenopa- nothing-by-mouth (NPO) status, and appropri-
thy, diffuse mucous membrane erythema, and/ ate broad-spectrum empiric intravenous anti-
or erythema of the palms/soles or indurative biotics. The development of sepsis and hemo-
edema of the hands/feet. dynamic instability takes precedence over the
decompression of the biliary tree. Additionally,
Answer D is incorrect. This presentation is not
>70% of patients with acute cholangitis will
consistent with classic measles (a paramyxovi-
respond to this regimen and will not require
rus), which typically presents with a prodrome
decompression. If decompression is eventu-

FULL-LENGTH EXAMS
of the 3 Cs (Cough, Conjunctivitis, and Co-
ally required, endoscopic sphincterotomy with
ryza, with the appearance of Kopliks spots on
stone extraction and/or stent insertion is the
the buccal mucosa. The viral exanthem (be-
treatment of choice for establishing biliary
ginning 12 days after the prodrome) is char-
drainage in acute cholangitis. Open surgery
acterized by an erythematous, maculopapular
or a direct percutaneous interventional radiol-
rash spreading from the head toward the feet.
ogy procedure is an acceptable consideration,
Answer E is incorrect. This presentation is not although there are increased rates of morbidity
consistent with mononucleosis (Epstein-Barr and mortality with open surgery.
virus), which typically presents with pharyngi-
Answer D is incorrect. Nasogastric suction
tis, fever, lymphadenopathy, and fatigue, but
may be required if the patient is vomiting and
without a characteristic rash. When patients
has severely impaired mental status, putting
with mononucleosis are treated with ampicil-
her at high risk for aspiration. This may be
lin, they may develop a diffuse rash resembling
necessary; however, intravascular resuscitation
that of measles; however, this child had no his-
in this case is more important due to the devel-
tory of recent antibiotic use.
oping sepsis.
8. The correct answer is A. This patient is pre- Answer E is incorrect. Oral antibiotics are not
Test Block 6

senting with characteristic signs and symptoms appropriate as a treatment for acute cholan-
of acute suppurative cholangitis with develop- gitis, nor does the administration of antibiotic
ing sepsis known as Reynoldss pentad (fever, therapy take priority over uid resuscitation in
jaundice, right upper quadrant pain, hypoten- a septic patient.
sion, and altered mental status). However,
658 Section II: Full-Length Examinations Answers

9. The correct answer is B. The patient is most munocompromise increases the likelihood for
likely infected with Histoplasma capsulatum, a symptomatic, disseminated disease. His puri-
fungus endemic to the Ohio and Mississippi ed protein derivative test is negative, and in
river valleys. The infection causes a mild feb- the absence of severe immunocompromise, it
rile syndrome in most and a self-limited pneu- makes tuberculosis less likely.
monia in some. Patients complain of chest
Answer E is incorrect. Trimethoprim-sulfame-
pain or cough. Physical examination is gener-
thoxazole is useful as an antibiotic and in the
ally unremarkable despite radiographic nd-
treatment of parasitic diseases, notably pneu-
ings of inltrates and mediastinal and hilar
monia due to Pneumocystis jiroveci, a fungus
lymphadenopathy. A complement xation titer
causing pneumonia in those with weakened
of at least 1:32 or a fourfold increase in titer is
immune systems, including the young, elderly,
suggestive of acute histoplasmosis. Dissemi-
and malnourished, and those with AIDS; and
nated disease with chronic pneumonia reects
toxoplasmosis, a parasitic disease caused by the
uncontained or poorly contained primary dis-
protozoan Toxoplasma gondii. The most com-
ease, or reactivation of a primary disease in the
mon method of acquisition by humans is con-
context of HIV infection. Chronic dissemi-
sumption or handling of raw or undercooked
nated disease in a young healthy patient may
meat, though handling of cat feces is also a
not always require amphotericin B, but this an-
classic risk factor. In healthy adults infection
swer option describes the best drug regimen.
is generally asymptomatic, but in the immu-
Treatment with amphotericin B, or with itra-
nodecient the disease may prove fatal, with
conazole in patients without meningeal dis-
encephalitis, chorioretinitis, and other cardiac
ease, is generally curative.
and hepatic manifestations. It is not effective
Answer A is incorrect. Histoplasma capsula- in the treatment of histoplasmosis.
tum is a fungus and antibiotics are less effec-
tive than antifungals in treating this class of 10. The correct answer is F. In an outpatient pop-
pathogens. ulation of otherwise healthy individuals <60
years old, one of the most likely pathogens is
Answer C is incorrect. Nystatin swish and
Streptococcus pneumoniae. The consolidated
swallow is predominantly used for oral can-
FULL-LENGTH EXAMS

lobar pneumonia seen in the radiograph is typ-


didal infections. Candida is a fungus character-
ical of pneumococcal disease.
istically infecting the oral mucosa and tongue
of those on broad-spectrum antibiotics, cortico- Answer A is incorrect. Escherichia coli most
steroids, or those with impaired cell-mediated commonly causes pneumonia in patients >60
immunity (i.e., patients with AIDS). Without years old or with comorbidities, including
mention of antibiotic course, steroids, or com- chronic obstructive pulmonary disease, heart
promised immunity, in the context of signs and failure, diabetes, liver disease, and alcohol
symptoms inconsistent with oral candidiasis, abuse. E. coli is a rare cause of pneumonia,
nystatin is a poor choice for treatment. and when it is isolated it is typically a hospital-
acquired, not community-acquired, case.
Answer D is incorrect. Pyrazinamide is used
to treat tuberculosis in combination with iso- Answer B is incorrect. Klebsiella most com-
niazid and rifampin. Tuberculosis is caused by monly causes pneumonia in patients >60 years
Mycobacterium tuberculosis and is character- old or with comorbidities, including chronic
ized by the formation of tubercles on the lungs obstructive pulmonary disease, heart failure,
and other tissues of the body. Patients tend to diabetes, liver disease, and alcohol abuse. Clas-
present coughing up sputum and blood, with sically, Klebsiella pneumonia presents with
fever, weight loss, and chest pain. Infectious currant-jelly sputum, and is often due to aspi-
Test Block 6

particles are spread via respiratory droplets. ration (as in an alcoholic).


Risk factors for acquisition of the disease in-
Answer C is incorrect. Mycoplasma is a com-
clude close contact with infected individuals
mon cause of community-acquired pneumonia
generally in the context of military barracks,
in healthy persons. However, the radiographic
prisons, and endemic areas of the world. Im-
Test Block 6 Answers 659

appearance of mycoplasmal pneumonia is usu- Answer C is incorrect. Androstenedione is a


ally the atypical pattern of diffuse inltrate precursor to testosterone, and therefore would
throughout the lungs, without lobar consolida- not be expected to be decreased in a case of
tion. virilization.
Answer D is incorrect. Pseudomonas should Answer E is incorrect. Dihydrotestosterone
be considered in patients with CF and in those is the active form of testosterone. To develop
with nosocomial pneumonia, neutropenia, or male-pattern hair, testosterone must be con-
late-stage HIV infection. It rarely causes com- verted to dihydrotestosterone by 5-reductase
munity-acquired pneumonia in healthy pa- found in the skin. Levels of dihydrotestoster-
tients. one would not be decreased in a patient with
virilization.
Answer E is incorrect. Staphylococcus aureus
most commonly causes pneumonia in patients
12. The correct answer is B. The differential diag-
>60 years old or with comorbidities, including
nosis of painless unilateral vision loss can in-
chronic obstructive pulmonary disease, heart
clude several possibilities. The patient reports
failure, diabetes, liver disease, and alcohol
a history of transient ischemic attacks and has
abuse. It is the most common cause of ventila-
a history of hypertension and diabetes, all of
tor-associated pneumonia.
which are risk factors for central retinal artery
occlusion. Additionally, funduscopic examina-
11. The correct answer is D. This boy has prema-
tion shows a cherry red spot and surrounding
ture virilization secondary to increased andro-
retinal edema (yellow). Results of treatment re-
gens. Of the answers listed here, all should be
main unsatisfactory; however, the goal is to re-
increased in the case of premature virilization
store blood ow as soon as possible. This is ac-
except cortisol. Given that his father also went
complished by decreasing intraocular pressure
through puberty at a very young age, this boys
to promote retinal perfusion and to potentially
premature virilization is more likely to be
dislodge emboli (intravenous acetazolamide,
caused by an inherited condition than an an-
topical -blockers, and ocular massage).
drogen-secreting tumor. A deciency in
21-hydroxylase would lead to shunting of the Answer A is incorrect. In acute ophthalmic

FULL-LENGTH EXAMS
17-hydroxyprogesterone from the glucocorti- artery occlusion, there is usually no cherry red
coid pathway into the androgen pathway. This spot in the foveola. The entire retina typically
is a benign condition in children, and the par- appears whitened.
ents should be reassured. The child should be
Answer C is incorrect. Central retinal vein oc-
followed when he goes through true puberty in
clusion can also present with painless unilat-
the event that his premature virilization is a
eral vision loss. It occurs in patients who are el-
precursor to other conditions, such as type 2
derly and is often idiopathic. The examination
diabetes mellitus.
typically shows diffuse retinal hemorrhages in
Answer A is incorrect. 5-Reductase is an en- all four quadrants with dilated, tortuous, reti-
zyme found in the skin that converts testoster- nal veins. There may also be cotton-wool spots,
one to the active dihydrotestosterone, which is disc edema, and neovascularization of the disc
responsible for male-pattern hair growth. Lev- or retina.
els of 5-reductase would not be decreased in
Answer D is incorrect. Giant cell arteritis typi-
a patient with virilization.
cally occurs in patients older than 50 years with
Answer B is incorrect. 17-Hydroxyprogesterone a history of temporal headaches, scalp tender-
is the branch point for the glucocorticoid and ness, jaw claudication, muscle pains, weakness,
Test Block 6

androgen pathways (using 21-hydroxylase and or weight loss. Erythrocyte sedimentation rate
17-hydroxylase, respectively), and is increased is also typically elevated.
in cases of 21-hydroxylase deciency. Levels
of 17-hydroxyprogesterone would not be ex-
pected to be low in a patient with virilization.
660 Section II: Full-Length Examinations Answers

Answer E is incorrect. A cherry red spot is seen impaction, mucosal erosion, abrasion, local
in Tay-Sachs disease, but it typically presents scarring, migration, peritonitis, mediastinitis,
early in life, occurs bilaterally, and is accompa- pneumomediastinum, pneumonia, or even
nied by other systemic manifestations. aortoenteric stula.

13. The correct answer is E. The story described 14. The correct answer is E. Status epilepticus re-
is classic for a foreign body ingestion. Eighty fers to a continuous state of seizure activity or a
percent of foreign body ingestions occur in series of seizures during which there is no return
children. Most such episodes occur in children to consciousness in the inter-ictal period. The
6 months to 3 years old. Older children are of- minimal duration of seizure activity in status ep-
ten the culprits for the introduction of the for- ilepticus has traditionally been cited as 1530
eign body to the younger patient. Initially, an minutes. Practically speaking, anyone who is
episode of choking, gagging, and coughing brought to an emergency department who has
may be followed by dysphagia, increased saliva- been seizing for >5 minutes will be treated as
tion, food refusal, emesis, or pain in the neck, having status epilepticus. This condition is a
throat, or sternal notch region. The rst step in medical emergency with a wide range of poten-
evaluation of suspected foreign body ingestion tially lethal complications. Prolonged intracta-
involves plain anteroposterior radiographs of ble muscle contraction secondary to seizures
the neck, chest, and abdomen, with lateral can lead to respiratory compromise, rhabdomy-
views of the neck and chest. olysis, hyperthermia, and lactic acidosis.
Answer A is incorrect. Barium contrast radio- Answer A is incorrect. Irreversible neuronal
logic evaluation may be helpful in asymptom- injury (seen as cortical laminar necrosis) is
atic patients with negative plain lms, but this thought to occur after as few as 3060 minutes
study is discouraged because of the risk of as- of continuous seizure activity.
piration. Additionally, the use of contrast can
Answer B is incorrect. The initial management
make subsequent visualization of the object
of patients in status epilepticus is aimed at se-
and removal of it more difcult.
curing the patients airway and administering
Answer B is incorrect. There is no evidence agents to cause cessation of seizure activity and
FULL-LENGTH EXAMS

in the history that this patient has an infectious is the same regardless of the underlying cause.
etiology to his symptoms and presentation.
Answer C is incorrect. The mortality of patients
A complete blood cell count with differential
in status epilepticus is approximately 20%.
would not aid in the diagnosis of foreign body
ingestion. Answer D is incorrect. After 45 minutes of
continuous seizure activity, symptoms may be-
Answer C is incorrect. Endoscopic evaluation
come subtle as myoclonic activity decreases.
is the management strategy for symptomatic
Such patients are often still seizing within the
patients with negative plain lms or patients
central nervous system and remain at risk for
with respiratory distress or other evidence of a
serious neuronal damage.
life-threatening condition. Endoscopic evalua-
tion in a stable patient without prior radiologic 15. The correct answer is A. Active liver disease is
studies would not be recommended. an absolute contraindication to oral contracep-
Answer D is incorrect. Although this patient is tive pills (OCPs) for two reasons. First, orally
not experiencing respiratory distress or showing administered steroid hormones are metabo-
evidence of impending distress, a full work-up lized in the liver and thus will not be appropri-
for suspected foreign body ingestion should be ately cleared in patients with liver disease. Sec-
Test Block 6

initiated. Given the history, foreign body inges- ond, studies have suggested a theoretical
tion is a likely cause for this patients presen- increase in the risk of liver disease in users of
tation, so parental reassurance and discharge high-dose OCPs.
from the emergency department without a Answer B is incorrect. OCPs are absolutely
work-up is not a good option. The potential contraindicated in a patient with a history of
sequelae of foreign body ingestion include
Test Block 6 Answers 661

thromboembolic disease. A history of arterial Answer D is incorrect. Fluoxetine is a rst-line


disease such as atherosclerosis, however, is not agent used in treating depression in patients
a contraindication. with eating disorders. It is a selective serotonin
reuptake inhibitor, and common adverse ef-
Answer C is incorrect. Pregnancy, not lacta-
fects include nausea, irritability, and sexual
tion, is a contraindication to OCP use. Al-
dysfunction.
though OCP use in early pregnancy has not
generally been associated with an increased Answer E is incorrect. Imipramine is a tricy-
risk of congenital abnormalities, studies sug- clic antidepressant used in the treatment of
gest there may be an increased risk of congeni- depression. It has not been shown to be associ-
tal urinary tract defects. ated with a decreased seizure threshold.
Answer D is incorrect. Active heavy smoking
17. The correct answer is E. This patient presents
(>15 cigarettes/day) in women >35 years old is
with neck soreness in the setting of a moderate-
an absolute contraindication to OCP use. Oc-
speed motor vehicle accident. There are no fo-
casional smoking, particularly in a woman <35
cal neurologic signs, no other distracting inju-
years old, is a relative risk but not an absolute
ries (painful injuries that can mask other
contraindication.
concerning injuries), and no signs of intoxica-
Answer E is incorrect. OCPs are commonly tion. Neither has there been a loss of con-
prescribed to treat hemorrhage associated with sciousness. It is therefore reasonable to trust
uterine broids. The presence of broids is the patients symptoms and signs as an indica-
thus not a contraindication to OCP use. How- tion of the severity of her neck injury. In this
ever, undiagnosed abnormal uterine bleeding case, because midline tenderness, obvious de-
is a contraindication, and the source must be formities, and focal neurologic ndings are ab-
identied before OCPs can be used. sent, radiographic studies are not warranted. A
supervised trial without the collar is appropri-
16. The correct answer is B. Bupropion is an atyp- ate.
ical antidepressant (blocking norepinephrine
Answer A is incorrect. Halo placement is used
and dopamine reuptake) that was found, for
to stabilize and immobilize cervical fractures.

FULL-LENGTH EXAMS
unclear reasons, to increase the risk of seizures
in patients suffering from anorexia by decreas- Answer B is incorrect. Steroids are used to re-
ing the seizure threshold. Bupropion is also duce edema and the possibility of permanent
contraindicated in patients undergoing an damage when frank spinal cord damage is sus-
abrupt withdrawal from alcohol or benzodiaz- pected either clinically or radiographically.
epines and should be used with caution in pa-
Answer C is incorrect. Flexion/extension
tients with seizure disorders. Clinicians should
lms are often used to clear a patient of pos-
use this medication cautiously in patients with
sible neck cervical spinal injuries when the
eating disorders and only if no other options
criteria for collar removal have not been met.
are available.
If the patient has focal neurologic symptoms,
Answer A is incorrect. Aripiprazole is an atypi- painful distracting injuries, posterior midline
cal antipsychotic agent and is not a rst-line tenderness, any evidence of intoxication, or an
agent in the treatment of depression. Adverse altered level of alertness, these studies should
effects include dry mouth, constipation, and be obtained before clearing a patients neck in-
anxiety. juries.
Answer C is incorrect. Escitalopram is a selec- Answer D is incorrect. Cervical MRI scan is
tive serotonin reuptake inhibitor used for the an appropriate intervention to demonstrate the
Test Block 6

treatment of depression. Its adverse effects in- extent of soft tissue injury in the event of neck
clude restlessness, nausea, blurred vision, dif- trauma. It should be used only when there is
culty sleeping, and dry mouth. reason to suspect spinal cord injury or vertebral
fracture.
662 Section II: Full-Length Examinations Answers

18. The correct answer is A. The lesions in pem- 19. The correct answer is C. The patient suffers
phigus vulgaris (PV) are a result of antibodies from a global aphasia, characterized by a global
against the intraepidermal desmogleins 1 and impairment in language comprehension, u-
3. Almost invariably, PV begins with accid le- ency, and word repetition. This occurs when
sions on the oromucosa that rupture easily. It there are both anterior and posterior lesions.
can progress to involve other mucosal areas
Answer A is incorrect. Brocas aphasia is char-
(pharynx, larynx, conjunctiva, rectal mucosa).
acterized by problems with language produc-
Areas of denuded skin can be quite painful.
tion, but preserved comprehension. The lesion
This clinical picture suggests that antidesmog-
that causes Brocas aphasia affects the third
lein 3 antibodies can be found in isolation.
frontal convolution of the left frontal lobe. It
The lesions are painful and may cause dif-
corresponds to Brodmanns areas 44 and 45.
culty eating. Other common areas of skin in-
volvement are the scalp, groin, axillae, and Answer B is incorrect. Conduction aphasia is
face. The diagnosis of PV requires skin biopsy characterized by problems with repeating what
showing intraepithelial acantholysis with an in- is said, but preserved uency and comprehen-
tact basement membrane. sion. It results from damage to the arcuate fas-
ciculus, a band of nerve bers that connects
Answer B is incorrect. Bullous pemphigoid
Brocas and Wernickes areas.
(BP) is caused by antihemidesmosomal anti-
bodies, resulting in disruption subepidermally, Answer D is incorrect. This type of aphasia
at the basement membrane. The predomi- is characterized by impaired uency, but pre-
nant areas affected are exural surfaces, in- served repetition and comprehension. Lesions
cluding the groin and axillae. Oral mucosa is that cause transcortical motor aphasia are
involved in one-third of all cases. Blisters are typically smaller than those that cause Brocas
tenser, with thicker rims than in PV, but may aphasia and are superior to and often anterior
leave erosions and crusting when unroofed. to Brocas area (Brocas area is not affected).
BP typically occurs in adults >60 years old ver- Answer E is incorrect. Wernickes aphasia rep-
sus a middle-age prevalence in PV. Biopsy is resents a problem with language comprehen-
required for the diagnosis of BP, which shows sion, which affects both language output and
FULL-LENGTH EXAMS

subepithelial blister formation. input. Wernickes area is the posterior region


Answer C is incorrect. Anti-basement mem- of the left superior temporal gyrus or the rst
brane antibodies are typically found in Good- gyrus of the temporal lobe. This area corre-
pastures syndrome, a pulmonary-renal disorder sponds to Brodmanns area 21 and 42.
that does not involve the skin.
20. The correct answer is C. The man has aortoil-
Answer D is incorrect. Multiple painful oral iac occlusive disease which is causing his con-
ulcers are often the earliest manifestation stellation of symptoms including impotence,
of Behets disease and may also be seen in claudication of the buttocks, and global lower
Crohns disease. However, these ulcers are not extremity atrophy. This classic presentation is
preceded by vesicles or bullae and do not oc- called Leriches syndrome. Vascular disease
cur outside the oral mucosa. Instead, they are must be considered when patients complain of
frequently associated with manifestations of the impotence. Thirty to fty percent of men with
underlying disease process. aortoiliac occlusive disease have impotence. In
Answer E is incorrect. The oral lesions in her- aortoiliac occlusive disease, you should expect
pes simplex virus infection can be very painful to see a weak or absent femoral pulse, an ankle-
but are more typically vesicular with evolution brachial index (ABI) which is slightly sub-
Test Block 6

to a crusting stage. Recurrent lesions primarily normal to normal at rest (this patient has no rest
occur on nonkeratinized mucosa such as the pain), and an ABI that is decreased with exer-
lips or gums. cise. ABI = Ratio of blood pressure in the ankle
to blood pressure in the arm. Normal is >1.0,
with claudication (pain with exercise) <0.7, and
with rest pain <0.4.
Test Block 6 Answers 663

Answer A is incorrect. An ABI at rest of 0.3 22. The correct answer is E. This infant is at risk of
would likely cause rest pain. developing hyperbilirubinemia because of sev-
eral factors: she is breast-fed, was carried nearly
Answer B is incorrect. This answer is incor-
to full term, and is of east Asian descent. Mater-
rect because an ABI at rest of 0.3 would likely
nal age >35 years is also a risk factor. It is physi-
be causing pain at rest, which this patient does
ologic for bilirubin to increase after birth, peak-
not have. Also, if occlusive disease is suspected,
ing at postpartum days 57. However, levels this
the ABI should not increase with exercise.
high are abnormal. Bilirubin can be neurotoxic,
Answer D is incorrect. One would not expect so it is important to reduce bilirubin levels in
the femoral pulse to be increased in the setting the neonate to prevent kernicterus (also known
of aortoiliac occlusive disease. The ABI is also as bilirubin encephalopathy). Phototherapy is
too low not to be associated with rest pain. the principal treatment for unconjugated hyper-
Answer E is incorrect. This answer is incor- bilirubinemia, as it converts unconjugated bili-
rect because the femoral pulse should not be rubin into water-soluble compounds that can be
increased in this patient. excreted in the urine.
Answer A is incorrect. In conjugated hyper-
21. The correct answer is C. This patient most bilirubinemia, abdominal ultrasound is usually
likely suffers from chronic obstructive pulmo- the initial test to evaluate for structural abnor-
nary disease, as evidenced by his physical ex- malities of the biliary tree.
amination and pulmonary function tests indic-
ative of obstructive disease. Destruction of lung Answer B is incorrect. Fiberoptic bilirubin
parenchyma leads to abnormally enlarged air- blankets are only appropriate for term infants
spaces, as suggested by the increased residual with nonhemolytic disease and mild hyper-
volume and hyperination on x-ray of the bilirubinemia. This neonates bilirubin level
chest. is signicantly elevated and requires close ob-
servation rather than discharge. Breast milk
Answer A is incorrect. An apical cavitary le- and poor hydrational status can lead to an
sion is characteristic of tuberculosis, which increased bilirubin level, so supplementing
would not typically result in prolonged expira- breast milk with formula or temporarily switch-

FULL-LENGTH EXAMS
tion or decreased FEV1:FVC ratio. ing to formula might be advisable.
Answer B is incorrect. Diffuse bilateral in- Answer C is incorrect. Jaundice can occur in
ltrates often occur in infectious processes, the setting of sepsis, in which case it is impor-
which would not likely present as insidiously as tant to treat both the jaundice and the underly-
this patient or cause increased residual volume ing cause (the infection). Sepsis is less likely in
on pulmonary function tests. Congestive heart this infant because of her normal WBC count
failure, another possible cause of this picture, and lack of risk factors. If she were to be treated
would not produce major abnormalities on for sepsis, ampicillin and gentamicin are the
pulmonary function testing. most common empiric treatment.
Answer D is incorrect. Lobar consolidation Answer D is incorrect. Exchange transfusions
most commonly occurs in infectious processes, are rarely performed. This is reserved for neo-
which would not likely present as insidiously as nates with signs of bilirubin neurotoxicity (sei-
this patient or cause increased residual volume zures, high-pitched cry, lethargy, poor feeding,
on pulmonary function tests. and hypertonicity) or when bilirubin levels are
Answer E is incorrect. Mass lesion with medi- exceptionally high. The specic bilirubin level
astinal adenopathy may be seen in lung carci- that requires a transfusion is based on an algo-
Test Block 6

noma, which would not likely cause increased rithm that takes into account the infants age,
residual volume (although changes associated gestational age, failure of prior attempts at pho-
with emphysema, the correct answer, are not totherapy, presence of isoimmune hemolytic
uncommon in patients with lung cancer for
obvious reasons).
664 Section II: Full-Length Examinations Answers

disease, sepsis, glucose-6-phosphate dehydro- 24. The correct answer is D. Endocarditis is diag-
genase deciency, asphyxia, temperature in- nosed by persistently positive blood cultures for
stability, and acidosis. viridans streptococci, Streptococcus bovis, Staph-
ylococcus aureus, enterococci, or HACEK group
23. The correct answer is B. This patients dia- organisms. HACEK stands for Haemophilus
stolic hypertension and associated hypokalemia spp., Actinobacillus actinomycetemcomitans,
can be explained by excessive aldosterone se- Cardiobacterium hominis, Eikenella corrodens,
cretion. She has no edema and is not taking and Kingella kingae. Other ndings include new
any diuretics. Aldosterone functions by distal murmur or cardiac vegetation seen on echocar-
tubular exchange of sodium for potassium. Hy- diography, fever, petechiae, splinter hemorrhages
persecretion of aldosterone increases this ex- (linear red-brown lesions in the nail beds),
change to the point of progressive depletion of Janeway lesions (painless lesions on palms/soles),
body potassium stores. Along with the in- Oslers nodes (painful nodules in pulp of ngers
creased sodium comes an increase in water, in- and toes), and Roth spots (retinal hemorrhages).
creasing the intravascular volume and causing Diagnosis is based on Duke criteria. Endocardi-
hypertension. The increased volume in the tis occurs most commonly in patients with valvu-
heart displaces the point of maximal impulse lar heart disease, congenital heart disease, pros-
to the sixth intercostal space. Very low renin thetic valves, or intravenous drug abusers. Other
levels along with high aldosterone levels point risk factors include diabetes, advanced age, and
to primary hyperaldosteronism or Conns syn- the use of anticoagulants or steroids. The pa-
drome. However, to diagnose Conns syn- tients presentation with fever, new murmur, and
drome, lack of suppression of aldosterone is petechiae are highly suggestive of infective endo-
also necessary by saline loading. carditis, and immediate medical management
Answer A is incorrect. A 24-hour urine cortisol should be started with a -lactam and an amino-
measurement is used in the work-up of Cush- glycoside, pending blood cultures.
ings syndrome, a syndrome of excessive corti- Answer A is incorrect. Gentamicin alone is
sol secretion. Patients present with weight gain, not indicated. It should be combined with a
purple striae, moon facies, sweating, and -lactam to cover resistant streptococci and en-
FULL-LENGTH EXAMS

thickening of the skin, none of which is pres- terococci.


ent in this patient.
Answer B is incorrect. A diuretic may be indi-
Answer C is incorrect. Urinary metanephrine cated to treat the patients systolic dysfunction,
excretion is a helpful screening test for diagnos- but antibiotic therapy should be initiated rst
ing pheochromocytoma. Patients with pheo- for treatment of infective endocarditis.
chromocytoma present with episodic hyperten-
Answer C is incorrect. Metoprolol and enal-
sion along with orthostatic changes in blood
april are indicated in the treatment of conges-
pressure and symptoms of headache, diaphore-
tive heart failure. The patients basilar crackles
sis, headache, and anxiety.
and edema could be indicative of systolic dys-
Answer D is incorrect. Renal angiography can function, but this is presumably due to endo-
be useful to diagnose renal artery stenosis. Re- carditis, which should be treated rst. Conges-
nal artery stenosis decreases the ow of blood tive heart failure in the absence of infective
to the kidneys, thus causing hypertension by endocarditis rarely manifests with fever and
activating the renin-aldosterone axis. However, petechiae. The new-onset murmur also sug-
a renal angiogram is not the rst step in diag- gests an acute process.
nosis.
Answer E is incorrect. Penicillin alone is not
Test Block 6

Answer E is incorrect. This patients abnormal indicated when the causative organism is un-
laboratory ndings point away from essential known, or when there could be an extracardiac
hypertension. Blood pressure can be measured source of infection. Penicillin (-lactam) and
again, but this is not the rst step in diagnosis. gentamicin (aminoglycoside) together cover
for the most common microorganisms in in-
Test Block 6 Answers 665

fective endocarditis (streptococci and entero- as hiking, camping, or brush-clearing, usually in


cocci). Penicillin will cover Streptococcus bovis the summer months. The preferred treatment
and viridans streptococci. Penicillin alone will for RMSF in both adults and children is doxycy-
not cover enterococci and certain streptococ- cline. Tetracyclines can cause dental staining in
cal strains (C and G). children, but this risk is minimal in short courses
and must be weighed against the benets of
25. The correct answer is A. Allopurinol, a xan- treatment. Doxycycline can also cause dental
thine oxidase inhibitor, assists in lowering staining in developing fetuses and may cause
plasma urate and urinary uric acid concentra- enamel hypoplasia and depression of fetal bone
tions. This effect aids in tophus mobilization growth. For this reason, chloramphenicol is rec-
and is the rst-line therapy for prophylaxis of ommended for treatment of pregnant women
recurrent gout. Allopurinol will transiently with RMSF. Care should be used in administer-
raise uric acid levels. ing chloramphenicol during the third trimester,
however, due to the risk of grey baby syndrome.
Answer B is incorrect. Colchicine can be used
for treatment of chronic gout (as well as acute Answer B is incorrect. Ciprooxacin is not ef-
gout ares) but is less preferred due to its ad- fective for the treatment of RMSF.
verse effect prole. Up to 80% of colchicine
Answer C is incorrect. Doxycycline and other
users suffer gastrointestinal adverse effects, in-
tetracycline antibiotics can cause dental stain-
cluding cramping, nausea, and diarrhea.
ing in developing fetuses and also cause enamel
Answer C is incorrect. In cases of acute gout, hypoplasia and depression of fetal bone growth.
the pharmacologic treatment of choice is a Therefore its use is discouraged in pregnant
nonsteroidal anti-inammatory drug, often in- women.
domethacin.
Answer D is incorrect. Penicillin is not effec-
Answer D is incorrect. Use of corticosteroids is tive for the treatment of RMSF.
mainly reserved for treatment of acute gout in
Answer E is incorrect. Trimethoprim-sulfame-
patients who cannot tolerate nonsteroidal anti-
thoxazole is not effective for the treatment of
inammatory drugs.
RMSF.

FULL-LENGTH EXAMS
Answer E is incorrect. Niacin use can be asso-
ciated with acquired hyperuricemia and would 27. The correct answer is A. Tight blood pressure
potentiate, not prevent, recurrent episodes of control during pregnancy is important to pre-
gout. vent complications, such as preeclampsia or
eclampsia, placental abruption, intrauterine
26. The correct answer is A. This woman has signs growth restriction with oligohydramnios, or
and symptoms of Rocky Mountain spotted fever preterm birth. Intrauterine growth restriction is
(RMSF), a tick-borne illness caused by Rickettsia presumed secondary to uteroplacental insuf-
rickettsii and the most common rickettsial infec- ciency due to partial placental abruption or va-
tion in the United States. RMSF is most com- soconstriction of the placental vessels.
mon in the southeastern and south central states,
Answer B is incorrect. Macrosomia, or fetal
and North Carolina accounts for 35% of re-
growth in excess of 5000 g, is a frequent com-
ported cases. The early phase of the illness devel-
plication of poorly controlled gestational diabe-
ops within 214 days of being bitten by an in-
tes, not hypertension.
fected tick. It is typically characterized by fever,
headache, malaise, myalgias, arthralgias, and Answer C is incorrect. Placenta previa is not
nausea. Most patients with RMSF also develop a associated with hypertension during pregnancy.
Test Block 6

maculopapular and petechial rash beginning on Placental abruption is a frequent complication


the ankles and wrists and spreading to the palms of hypertension.
and soles as well as centrally. The diagnosis of
Answer D is incorrect. Oligohydramnios, not
RMSF requires a high index of suspicion and an
polyhydramnios, is associated with hyperten-
awareness of risk factors for tick exposure, such
sion during pregnancy.
666 Section II: Full-Length Examinations Answers

Answer E is incorrect. Rocker-bottom feet are intoxication or hyperglycemic coma. Anyone


associated with trisomy 13 and 18. with a Glasgow Coma Scale score <8 requires
immediate intubation due to the likelihood of
28. The correct answer is A. This child has Beck- increased intracranial pressure and consequent
with-Wiedemann syndrome, which is character- respiratory depression.
ized by congenital exomphalos, macroglossia,
Answer A is incorrect. Mannitol can be used
and gigantism; organomegaly and hemihyper-
to temporarily reduce brain swelling secondary
trophy are common as well. The tumor most
to trauma, and it may be useful in this patient
commonly found in patients with Beckwith-
if signs of increased intracranial pressure ap-
Wiedemann syndrome is Wilms tumor. Wilms
pear. However, to stabilize the patients airway
tumor is classically detected by a mother who is
and breathing in the event of increased intra-
bathing her child and encounters an abdominal
cranial pressure, intubation is a higher priority.
or ank mass. Sixty percent of patients have sec-
ondary hypertension from renal ischemia. A Answer B is incorrect. CT of the head is an
Wilms tumor may be cystic, but this is very important step in this patients work-up be-
rare. The most common form of Wilms tumor cause a brain injury is likely, but the patients
is triphasic (it has epithelial, blastic, and breathing must be secured rst.
stromal elements).
Answer D is incorrect. An important cause of
Answer B is incorrect. These ndings may be coma that must be ruled out is hyperglycemia.
seen in a multilocular cyst or other renal cyst, However, a Glasgow Coma Scale score <8 in
but this child has Beckwith-Wiedmann syn- this trauma patient indicates severe brain dam-
drome, which is associated with Wilms tumor. age, and measuring blood glucose is secondary
to stabilizing the patients respiration.
Answer C is incorrect. Small blue cells are
characteristic of neuroblastoma. A neuroblas- Answer E is incorrect. It is possible that this
toma is usually part of the adrenal gland, while patient has had a severe embolic stroke. How-
a Wilms tumor is part of the renal paren- ever, given that the internal bleeding of this
chyma. Abdominal imaging is usually used to trauma patient has not yet been assessed, tissue
distinguish the two. Neuroblastoma is not asso- plasminogen activator is not appropriate at this
FULL-LENGTH EXAMS

ciated with Beckwith-Wiedemann syndrome. time. After stabilizing the patient and imaging
studies, this option may be reconsidered.
Answer D is incorrect. Teeth and cartilage are
the classic nding in a mature teratoma, which
30. The correct answer is C. Neuroleptic malig-
is unlikely to be found in this patient.
nant syndrome (NMS) is a life-threatening re-
Answer E is incorrect. Urine may be aspirated action to antipsychotic medications. Symptoms
if the needle is improperly placed or if the pa- of NMS include sudden altered mental status,
tient has a urinoma (a cyst containing urine). stiffness or tremor, and hyperthermia. Auto-
This would be unlikely in this child, who is nomic instability (in this case, tachycardia and
presenting with a congenital syndrome com- hypertension) is often associated. Mortality can
monly associated with Wilms tumor. Risk be as high as 10%, and rhabdomyolysis, myo-
factors for urinoma include urinary surgery or globinuria, and acute renal failure can occur,
trauma (the number one cause), urinary ob- in addition to myocardial infarction, respiratory
struction, neurogenic bladder, and urethral di- failure, and seizures. Rapid cooling (with blan-
verticulum. kets and ice) plus intravenous access for uid
administration are necessary. Dantrolene (a di-
29. The correct answer is C. The patient is coma- rect acting muscle relaxant) and bromocriptine
Test Block 6

tose, characterized by unresponsiveness to (a dopamine agonist) are two agents that can
painful stimuli, including a sternal rub or nail be used to treat NMS.
bed stimulation. He has a Glasgow Coma
Answer A is incorrect. Clonidine is a centrally
Scale score of 5 (no eye or verbal response, and
acting adrenergic agonist that inhibits the sym-
decorticate posturing to pain). Furthermore,
pathetic nervous system via central 2-adrenergic
the toxicology screen and glucose test rule out
Test Block 6 Answers 667

receptors. While clonidine might work to lower bophlebitis, nonbacterial verrucous endocardi-
this patients hypertension, it will not combat the tis, and hypertrophic pulmonary osteoarthropa-
central disease process (NMS). thy are associated with adenocarcinoma.
Answer B is incorrect. The use of cooling Answer B is incorrect. Bronchioalveolar car-
blankets and intravenous uids are methods cinoma is a subtype of adenocarcinoma that
of rapid cooling that may be benecial to the tends to be associated with multiple nodules.
patient, but they will not reverse the effects
Answer C is incorrect. Large cell carcinoma
of central dopaminergic blockade from the
represents approximately 10% of lung cancers
neuroleptics. Dantrolene will directly relax
and is typically peripherally located. Gyneco-
muscles and bromocriptine will serve as a do-
mastia is associated with large cell carcinoma.
pamine agonist.
Answer D is incorrect. Small cell carcinomas
Answer D is incorrect. Ice packs placed on
are of neuroendocrine origin and tend to be
the patients axillae and groin are an effective
centrally located. They metastasize early, com-
method of rapid cooling. So too is evaporative
monly to bone, liver, or brain. They may be as-
cooling using water spray with fans. However,
sociated with multiple paraneoplastic syndromes
these methods do not reverse the effects of
such as Cushings syndrome and the syndrome
central dopaminergic blockade from the neu-
of inappropriate ADH secretion. However, small
roleptics. Dantrolene will directly relax muscle
cell carcinomas do not produce parathyroid-
and bromocriptine will serve as a dopamine
related peptide. Eaton-Lambert myasthenic syn-
agonist.
drome, subacute cerebellar degeneration, and
Answer E is incorrect. Lorazepam is a benzo- peripheral neuropathy are other paraneoplastic
diazepine anxiolytic that is relatively quickly syndromes attributed to small cell carcinoma.
absorbed. The drug is used in panic attack
for treatment of anxiety and agitation, and for 32. The correct answer is B. This patient likely
prevention of alcohol and other sedative with- has pancreatic cancer, a common and usu-
drawal. It would not be effective in the treat- ally fatal disease. Symptoms consistent with
ment of NMS. pancreatic adenocarcinoma include jaun-

FULL-LENGTH EXAMS
dice, persistent abdominal pain, weight loss,
31. The correct answer is E. Lung cancer causes and nausea. Presence of Courvoisiers sign
more cancer-related deaths in the United (the palpable nontender gallbladder) suggests
States than colorectal, breast, and prostate can- the diagnosis. Only 10%20% of patients who
cers combined. This patient has an extensive present with pancreatic cancer are found to
smoking history and presents with lung mass have disease that has not yet metastasized or
and hypercalcemia. She does not have meta- invaded local vessels. These patients can un-
static disease to bone. She has a paraneoplastic dergo a Whipple procedure (pancreaticoduo-
syndrome with secretion of parathyroid-related denectomy), while the great majority are
peptide. These clues lead to a diagnosis of treated with palliative care strategies.
squamous cell lung cancer. Squamous cell car-
Answer A is incorrect. While pancreatic can-
cinomas are centrally located lesions that grow
cer is usually fatal, 10%20% of patients are
intraluminally and spread by direct extension.
lucky to have resectable disease. These patients
They are the least likely to metastasize. They
undergo pancreaticoduodenectomy and can
secrete parathyroid hormone-like peptide, which
be cured.
causes signicant hypercalcemia.
Answer C is incorrect. Pancreatic cancer can
Answer A is incorrect. Adenocarcinoma is the
grow silently and begin to metastasize before it
Test Block 6

most common lung cancer, accounting for


begins to impinge on the bile ducts and cause
40% of cases. There are many subtypes (e.g.,
pain or jaundice. Thus, only 10%20% of cases
acinar, bronchioalveolar, and papillary), but
are amenable to surgical cure.
they tend to be peripherally located. Throm-
668 Section II: Full-Length Examinations Answers

Answer D is incorrect. Because of the location provided signicant information for diagnosing
of most pancreatic masses (the head) and their this patients condition.
propensity to metastasize silently, only 10%
Answer C is incorrect. Routine screening at
20% are curable with surgery. Even 20% of pa-
time of birth is performed for all infants born
tients who are thought to have resectable dis-
in hospital settings and was therefore most
ease are found to be unresectable at the time
likely performed for this child. Screening tests
of laparotomy.
are done for a variety of conditions, including
Answer E is incorrect. Pancreatic cancer is congenital hypothyroidism, CF, sickle cell dis-
rarely caught early enough to respond to sur- ease, and multiple metabolic disorders, such as
gery. An example of a cancer with a 99% surgi- phenylketonuria, galactosemia, and homocysti-
cal cure rate is squamous cell skin cancer. nuria, to name a few. Presently, SCID is not a
routine neonatal screen.
33. The correct answer is E. Severe combined im-
Answer D is incorrect. Serum uric acid can be
munodeciency (SCID) is a group of genetic
used as a screening test for deciency of purine
syndromes marked by absent adaptive immune
nucleoside phosphorylase, which results in a
function caused by a failure of lymphocytes to
mild form of SCID. In purine nucleoside phos-
proliferate. All cases have T-lymphocyte de-
phorylase deciency, the level of uric acid will
fects, although some will also have B-lympho-
be extremely low. This variant almost never
cyte and/or natural killer cell problems. Pa-
causes immunodeciency at this young age.
tients affected with SCID generally present
Furthermore, this deciency is extremely rare,
within the rst few months of life with failure
and only about 50 cases have been described
to thrive, diarrhea, recurrent infections, and
in the medical literature. For these reasons, a
opportunistic infections caused by Candida al-
WBC count is a much more appropriate gen-
bicans, Pneumocystis jiroveci, measles, vari-
eral screening test for SCID.
cella, cytomegalovirus, Epstein-Barr virus, ade-
novirus, and other such organisms. Patients
34. The correct answer is B. This patient is pre-
will have small thymuses that are often <1 g
senting with clinical and radiologic evidence
and can fail to descend from the neck, as well
of a partial small bowel obstruction. She does
FULL-LENGTH EXAMS

as absence or underdevelopment of other lym-


not appear to have a complete obstruction be-
phoid tissues (tonsils, adenoids, lymph nodes,
cause she is still passing atus and has air in
and Peyers patches). Infants with SCID are, by
her colon on lms. Additionally, she has no ev-
denition, lymphopenic (<3000/mm) at birth,
idence of necrotic bowel or peritoneal signs on
and, therefore, all cases could be identied
examination, so she is not likely to require
with a complete blood cell count with differen-
emergent surgical intervention. The appropri-
tial at the time of birth, a simple screening that
ate management of a partial small bowel ob-
can save these childrens lives.
struction is to admit to the hospital for observa-
Answer A is incorrect. Blood cultures with tion, decompress the bowel proximal to the
sensitivities performed at time of presentation obstruction with nasogastric suction, place the
would have aided in identifying the etiology of patient on NPO status, and replace ongoing
sepsis in this patient but would not have helped uid losses from vomiting/nasogastric suction
in diagnosing SCID. Given the acuity of her to avoid electrolyte abnormalities.
presentation, blood culture results would not
Answer A is incorrect. Although intravenous
have been available in time to be denitive in
uids are important to rehydrate a patient
identifying the opportunistic pathogens likely
who has been vomiting, the bilious color of
implicated in her infections.
her vomitus suggests bowel obstruction, not
Test Block 6

Answer B is incorrect. Although cardiac fail- pancreatitis, making endoscopic retrograde


ure eventually developed in this patient as a cholangiopancreatography unnecessary. It is
result of her overwhelming infections, cardiac important to make the patient NPO status and
function is not affected in patients with SCID. place a nasogastric tube to eliminate bilious
Therefore, an echocardiogram would not have uid from the stomach.
Test Block 6 Answers 669

Answer C is incorrect. A rectal tube can be a patients with contraindications to medical an-
therapeutic intervention for a large bowel ob- ticoagulants, a history of a recurrent throm-
struction but plays no role in small bowel ob- boembolism with adequate anticoagulation,
struction. pulmonary hypertension with recurrent em-
boli, or for those patients undergoing surgi-
Answer D is incorrect. Emergency laparo-
cal pulmonary embolectomy or thromboend-
tomy is not necessary in cases of partial bowel
arterectomy. Filter placement is not considered
obstruction. However, peritonitis due to bowel
a rst-line treatment in a patient with a DVT.
obstruction is a surgical emergency. In such
This patient should be started on heparin im-
cases the clinical, radiologic, and electrolyte
mediately. Placement of an inferior vena cava
picture would have to point toward peritonitis,
lter may be considered at a later time if this
ischemic bowel, and no air distal to the ob-
patient does not respond to medical manage-
struction.
ment.
Answer E is incorrect. Observation in the
Answer D is incorrect. The patient is not pre-
emergency department is not appropriate, es-
senting with any signs of PE (dyspnea, pleuritic
pecially if one has not taken steps to manage
chest pain, cough, tachypnea, and tachycardia)
the obstruction indicated by the lms and clin-
and does not have physical exam ndings sug-
ical picture (i.e., nasogastric tube and intrave-
gestive of a PE. Although DVT in the lower ex-
nous uid replacement).
tremities accounts for about 95% of PEs, there
35. The correct answer is C. This patient has mul- is no indication to work up the diagnosis of a
PE in this patient at this time.
tiple myeloma, a malignancy of the plasma
cells that is associated with hypercoagulability. Answer E is incorrect. Patients with a history
It is not uncommon for these patients to de- of DVT are maintained on warfarin as outpa-
velop deep venous thrombosis (DVT). This pa- tients in order to prevent the development of
tient also has an increased risk of DVTs be- additional thromboses. Warfarin, however, is
cause he has been immobile for several weeks. not begun immediately in these patients be-
Full anticoagulation must be initiated in order cause it is associated with a transient hyperco-
to prevent the development of additional agulable state due to inhibition of protein C

FULL-LENGTH EXAMS
thrombi and progression of the DVT. Since and protein S.
50% of patients with symptomatic DVTs will
develop pulmonary emboli (PE) if left un- 36. The correct answer is D. Given this patients
treated, it is recommended that anticoagula- description of shoulder and pelvic girdle pain,
tion therapy be started in this patient. Antico- constitutional symptoms, elevated erythrocyte
agulation in patients with an existing DVT is sedimentation rate, and normal radiograph,
achieved by either intravenous heparin or low- she is suffering from polymyalgia rheumatica.
molecular-weight heparin. This is a rheumatic condition that typically af-
fects women of European descent between 50
Answer A is incorrect. Patients with malig-
and 70 years old. Typical presentation includes
nancy (including multiple myeloma) may
pain and stiffness of the shoulder and pelvic
have an acquired hypercoagulable state. The
girdle and possibly fever, malaise, and weight
treatment in these patients includes traditional
loss. Erythrocyte sedimentation rates are typi-
treatment for the thrombosis as well as treat-
cally elevated and can reach levels >100 mm/
ment of the underlying disorder. Since this
hr. Treatment for this condition includes low-
patient has begun to show signs of disease re-
dose steroids followed by tapering 24 weeks
mission, chemotherapy should be continued at
after symptoms remit.
this time.
Test Block 6

Answer A is incorrect. Amitriptyline and other


Answer B is incorrect. Inferior vena cava l-
tricyclic antidepressants are used to treat bro-
ters are indicated in the treatment of DVT in
670 Section II: Full-Length Examinations Answers

myalgia. Most patients with bromyalgia are Answer A is incorrect. Autonomy is the prin-
<50 years old, have widespread musculoskel- ciple stating that physicians must respect the
etal pain with no constitutional symptoms, and individual rights and opinions of their patients.
have a normal erythrocyte sedimentation rate. Although this applies in all cases, this case is
not specically a case of autonomy.
Answer B is incorrect. High-dose prednisone
is used to treat polymyositis, a progressive sys- Answer B is incorrect. Dereliction is one of
temic connective tissue disease characterized the four elements of a malpractice suit. The
by immune-mediated striated muscle inam- physicians in this case are not committing
mation. Patients with this condition have simi- malpractice. After discussion with the patients
lar symptoms as in this patient, but typically family and consideration of the patients rights
have elevated creatine-kinase and aldolase and wishes, the physicians are acting in accor-
levels. High-dose prednisone is used to treat gi- dance with sound ethical principles.
ant cell arteritis, a condition on the same dis-
Answer D is incorrect. The physicians in this
ease spectrum as polymyalgia rheumatica and
case are not committing malpractice; after dis-
characterized by chronic vasculitis of large and
cussion with the patients family and consid-
medium-sized vessels that often results in head-
eration of the patients rights and wishes, the
ache, jaw claudication, or vision loss.
physicians are acting in accordance with sound
Answer C is incorrect. Iniximab is a tumor ethical principles.
necrosis factor- inhibitor used to treat a va-
Answer E is incorrect. The physicians in this
riety of conditions including rheumatoid ar-
case are offering opioids as a palliative mea-
thritis, inammatory bowel disease, and anky-
sure, not a treatment intended to hasten the
losing spondylitis. Ankylosing spondylitis is a
patients death. This case, in which the patient
chronic inammatory disease of the spine that
lacks capacity and her family and physicians
typically affects men in their 20s and shows ra-
decide to withdraw care, is not an example of
diographic evidence of joint disease, especially
suicide.
at the sacroiliac joints. The patient in this case
does not t the typical description of ankylos- 38. The correct answer is C. Eosinophilic cyto-
ing spondylitis and is most likely to have poly-
FULL-LENGTH EXAMS

plasmic inclusions are also known as Lewy


myalgia rheumatica. bodies and are prevalent neuronal ndings in
Answer E is incorrect. Methotrexate is a fo- patients with this form of dementia. Lewy body
lic acid competitive inhibitor used as rst-line dementia is characterized by new onset hallu-
treatment for rheumatoid arthritis and falls un- cinations, delusions, extrapyramidal signs, and/
der the category of disease-modifying antirheu- or repeated loss of balance. Patients may expe-
matic drugs. Seventy-ve percent of patients rience periods of lucidity and may undergo ex-
with rheumatoid arthritis have a positive rheu- tensive delirium evaluations.
matoid factor and radiologic evidence of joint
Answer A is incorrect. Birefringent crystals are
disease. In rare cases methotrexate can be used
found in joint aspirate of patients with gout.
as steroid-sparing agent in polymyalgia rheu-
matica. Answer B is incorrect. Lewy body disease
causes neuronal loss in the frontal lobes, locus
37. The correct answer is C. Double effect is ceruleus, and substantia nigra. However, darkly
the ethical principle that states that a palliating pigmented neurons are normal neuronal nd-
treatment is acceptable even if it may hasten ings in the substantia nigra, so this answer does
death because the intended effect is to relieve not reect expected abnormal ndings.
the patients suffering. In this case, opioids may
Test Block 6

Answer D is incorrect. Hypersegmented nu-


cause respiratory depression and hasten the pa-
clei are features of polymorphonuclear neutro-
tients death after extubation; however, this is
phils, not neurons in Lewy body disease.
ethically acceptable because of the importance
of providing palliative comfort measures for the Answer E is incorrect. Neurobrillary tangles
patient. and webs can be seen in other forms of demen-
Test Block 6 Answers 671

tia but are not specic for Lewy body disease. fections and failure to thrive, generally present-
Neurobrillary tangles may also be found in ing early in life.
Alzheimers disease and Lewy bodies of Parkin-
sons disease. 40. The correct answer is C. The patient has a
right to know and understand the risks and
39. The correct answer is B. CF commonly pres- benets associated with each treatment option.
ents as sinonasal disease in children, often as Although the physician should help guide the
recurrent sinusitis. Typical symptoms of sinus- patients decision, he should have a frank dis-
itis include headache, fever, and cough due to cussion of the morbidity and mortality, includ-
postnasal drip. However, any child with recur- ing the life expectancy and possibility of de-
rent sinusitis should be evaluated for other creased quality of life, associated with each
structural or immunologic contributors. In CF, treatment.
the frequency of nasal polyposis is quite vari-
Answer A is incorrect. Requesting that the pa-
able, but it is rare in individuals without CF.
tient not ask questions will not help ensure that
CT often shows signs of chronic sinusitis, and
the patient understands her options. Asking the
x-ray of the chest shows multiple cystic spaces
patient to summarize what has been said is a
within the lungs with soft tissue densities ll-
good way for the physician to ensure that the
ing the mid-distal airways. CF is conrmed by
patient understands each option. This ensures
a chloride sweat test >60 mEq/L.
that the patient is making a truly informed de-
Answer A is incorrect. Allergic rhinitis can cision about her care.
cause nasal polyps due to chronic inamma-
Answer B is incorrect. Although the physician
tion of the nasal mucosa. However, the disease
should discuss treatment options and their rela-
shows denite seasonal variability, and while
tive risks and benets, it would be too confus-
it may cause headache and congestion, it also
ing for the patient to discuss the details of all
would not cause fever or purulent discharge
relevant clinical trials. The physician should
from the nose unless severe enough to cause
not simply act as an information repository; he
acute sinusitis.
or she must synthesize the clinical knowledge
Answer C is incorrect. Patients with ciliary about the subject and present it in a format

FULL-LENGTH EXAMS
dyskinesias, of which Kartageners is one cause, that the lay person can understand.
have structural defects in ciliary function, as
Answer D is incorrect. By omitting family
opposed to CF, in which there is a mechanical
members from such discussions, the physician
obstruction. The symptoms can appear similar,
is protecting the patient from potentially coer-
in that patients with Kartageners syndrome
cive inuences and ensuring that the patient is
can present with recurrent sinusitis as well as
making her own decision. The patient can also
bronchopulmonary infections. Sterility and si-
decide which information to disclose to family
tus inversus are classically present.
members. However, such decisions are not un-
Answer D is incorrect. Nasal foreign bodies, der the discretion of the physician, and if the
which are common in young children, can patient does want her family present, the physi-
cause recurrent sinusitis due to obstruction of cian should support the decision.
the nasal ostiae. However, sinusitis is usually
Answer E is incorrect. The physician is obli-
unilateral unless both nostrils are involved.
gated to disclose to the patient all treatment
Answer E is incorrect. Immunodeciency options for her condition, not just those that
should be a concern in patients with recur- the physician thinks will work best.
rent sinopulmonary or cutaneous infections.
Test Block 6

X-linked SCID involves mutations in the com-


mon ( chain of interleukin receptors, which
leads to severe T- and B-lymphocyte dysfunc-
tion. Clinically, the patients have recurrent in-
672 Section II: Full-Length Examinations Answers

41. The correct answer is B. Most adenocarcino- etry should also be recommended because it
mas arise from Barretts esophagus, which is decreases the incidence of postoperative pneu-
characterized by columnar epithelium replac- monia and atelectasis.
ing normal squamous tissue. This change is
Answer B is incorrect. Treatment with pneu-
largely precipitated by constant inammation
matic compression and elastic stockings is gen-
due to reux from the stomach and is strongly
erally reserved for patients with a moderate risk
associated with malignancy, increasing the risk
of DVT/PE, patients >40 years old, and those
nearly 40 times. Due to the malignancy, pa-
with a history of recent general surgery, myo-
tients often complain of a slow history of dys-
cardial infarction, cerebrovascular accident,
phagia, especially of solids, and resulting
bed rest, or chronic illnesses such as cancer,
weight loss. Stridor and nocturnal cough are
inhibitor deciency states, or antiphospholipid
also prominent features. Endoscopy is the diag-
syndrome.
nostic procedure of choice.
Answer C is incorrect. Patients with a high
Answer A is incorrect. Achalasia is due to an
risk of DVT/PE have a history of recent ortho-
unknown etiology that affects the neuronal
pedic surgery, trauma, or DVT. These patients
control of the esophagus and affects solids and
have a 10%20% risk of DVT/PE and should
liquids. It is characterized by a birds beak
have prophylaxis consisting of elastic stockings
appearance on barium swallow.
plus low-molecular-weight heparin using ei-
Answer C is incorrect. Esophageal varices are ther enoxaparin, 30 mg subcutaneously twice a
nearly always due to an increase in portal pres- day; dalteparin, 2500 IU subcutaneously once
sure and may present with upper gastrointes- daily; or warfarin.
tinal bleeding. Symptoms of liver disease will
Answer D is incorrect. Patients with a moder-
usually be present (e.g., jaundice, pruritus, ab-
ate risk of DVT/PE are typically >40 years and
dominal pain).
have a history of recent general surgery, myo-
Answer D is incorrect. Gastric cancer in the cardial infarction, cerebrovascular accident,
United States continues to decrease. Family bed rest, or chronic illnesses such as cancer,
history, tobacco, vitamin C deciency, and inhibitor deciency states, or antiphospholipid
FULL-LENGTH EXAMS

preserved foods are risk factors (there has been syndrome. These patients have a 2%4% risk
a rise of gastric cancer in Japan and Eastern of DVT/PE and should have prophylaxis con-
Europe possibly for this reason). It is character- sisting of heparin, 5000 units subcutaneously
ized by weight loss and gastric pain. twice a day or intermittent pneumatic com-
pression with or without elastic stockings.
Answer E is incorrect. Hiatal hernias are as-
ymptomatic but may lead to reux. Unless they Answer E is incorrect. Patients with a high risk
become entangled, they would not present as of DVT/PE have a history of recent orthopedic
in this question. surgery, trauma, or DVT. These patients have
a 10%20% risk of DVT/PE and should have
42. The correct answer is A. Thromboprophylaxis prophylaxis consisting of elastic stockings plus
is an important part of postsurgical care. The low-molecular-weight heparin using either
risk of DVT or PE after surgery can be classi- enoxaparin, 30 mg subcutaneously twice a day;
ed as low, moderate, and high. Patients who dalteparin, 2500 IU subcutaneously once daily,
are <40 years old and undergo an uncompli- or warfarin.
cated surgery, such as the one referred to here,
have only a 0.4% risk of DVT/PE. In this pa- 43. The correct answer is A. This patient is pre-
tient population, early ambulation is the pro- senting with a classic case of acute lymphocytic
Test Block 6

phylaxis of choice. Although not an answer leukemia. Although this disease may be dif-
choice, pulmonary toilet and incentive spirom- cult to differentiate from acute myelogenous
leukemia, the next step in treatment is the
Test Block 6 Answers 673

same for both illnesses. Patient evaluation prior 45. The correct answer is F. Patients with polycys-
to beginning therapy requires a complete blood tic kidney disease often present in their 40s
cell count, chemistry studies assessing major with some degree of renal failure. This mans
organ function, a bone marrow biopsy, and a disease has been inherited in an autosomal
lumbar puncture to rule out occult central ner- dominant fashion. The hematuria and ank
vous system involvement. The latter two proce- pain may be caused by the cysts and cyst rup-
dures are generally performed together so that ture, which can cause bleeding.
conscious sedation is necessary only once.
46. The correct answer is I. Renal cell carcinoma
Answer B is incorrect. Although chemother-
is the most likely diagnosis here. The disease
apy should be initiated without delay in a pa-
can cause hematuria without dysuria. The pain
tient such as this, lumbar puncture should be
described here is not an acute process, but
performed rst if possible.
rather a deep, subacute pain. Furthermore, the
Answer C is incorrect. A patient with a plate- man has experienced weight loss and fevers,
let count of 75,000/mm is not considered at both evidence of a neoplastic process. The -
risk for spontaneous bleeding. An emergent nal piece of data that should clinch the diag-
platelet transfusion would be considered for nosis is the elevated hematocrit. Polycythemia
a patient with a platelet count of 25,000/mm is often seen in renal cell carcinomas that se-
and would be indicated in one with a platelet crete erythropoietin.
count 10,000/mm or lower.
Answer B is incorrect. While hematuria is a
Answer D is incorrect. For the most part, bone rare symptom of some coagulation disorders,
marrow transplant is not used as a rst ap- none of these patients has other stigmata of co-
proach to curing childhood acute leukemia. agulopathy such as bruising, epistaxis, or other
The one exception to this is the acute my- mucosal bleeding. In addition, ank pain is not
elogenous leukemia patient with a matched associated with coagulation disorders.
sibling donor, for whom bone marrow trans-
Answer C is incorrect. Exercise-induced he-
plant would be considered rst-line therapy.
maturia may be gross or microscopic, begins
This is never the case for an acute lymphocytic
after strenuous exercise, and remits within sev-

FULL-LENGTH EXAMS
leukemia patient. Even when sibling typing is
eral days of the exercise. Exercise-induced he-
required, it can be carried out after initial che-
maturia is a diagnosis of exclusion and more
motherapy has been initiated. Lumbar punc-
serious causes must be ruled out.
ture should be done beforehand.
Answer D is incorrect. Myoglobinuria causes
Answer E is incorrect. In the absence of evi-
a red discoloration of urine similar to hemoglo-
dence of chest pathology on history or physical
binuria. The presence of urine myoglobin can
examination, x-ray of the chest is generally not
be determined by a urine dipstick positive for
required as part of the initial work-up of pediat-
blood, but an absence of RBCs on microscopic
ric patients with acute leukemia.
evaluation. Approximately 200 g of muscle
must be destroyed before urine is colored.
Questions 44, 45, and 46 None of these patients has a history suggestive
of muscle destruction.
44. The correct answer is A. In an older man,
painless hematuria is classic for bladder can- Answer E is incorrect. Nephrolithiasis may
cer. Smoking, as well as aniline dye exposure, present with hematuria and ank pain and is
are risk factors for transitional cell carcinoma, known to run in families. The ank pain is
the most common form of bladder cancer. generally severe and persistent until a stone is
Test Block 6

passed. Treatment is supportive, as stones will


pass spontaneously. Nephrolithiasis does not
cause systemic symptoms.
674 Section II: Full-Length Examinations Answers

Answer G is incorrect. Hematuria is rarely the creased WBC count. Hematuria is a rare nd-
presenting symptom of prostate cancer, unless ing in pyelonephritis.
the tumor has invaded the bladder. At this lo-
Answer J is incorrect. Hematuria may be a
cally advanced stage, symptoms such as uri-
presenting symptom of urinary tract infection,
nary frequency, retention, or dysuria would be
and one would also expect dysuria, frequency,
expected.
increased WBC count, and positive urine cul-
Answer H is incorrect. Patients with pyelone- ture. Urinary tract infection is more common
phritis present with dysuria, urinary frequency, in women and is rarely a cause of increased
fevers, costovertebral angle tenderness, and in- creatinine levels.
FULL-LENGTH EXAMS
Test Block 6
Test Block 7

675
676 Section II: Full-Length Examinations Questions

Q U E ST I O N S

1. A 45-year-old white mother of three presents to Which of the following tests would have pre-
the emergency department complaining of in- dicted the increased risk of recurrent spontane-
termittent epigastric pain that has awakened ous abortions in this patient?
her from sleep. The pain is severe, lasts for a
(A) CT of the pelvis
couple of hours, and is sometimes accompa-
(B) Lupus anticoagulant
nied by vomiting. The pain sometimes occurs
(C) Pituitary function
after large meals. Her bowel movements have
(D) Potassium hydroxide whiff test
been normal. Her temperature is 38.0C
(E) Prothrombin time
(100.4F), pulse is 100/min, blood pressure is
125/60 mm Hg, and respiratory rate is 20/min.
3. A homeless 57-year-old African-American vet-
On physical examination the patient is obese,
eran, well known to the emergency depart-
her sclerae are anicteric, and her bowel sounds
ment, is brought in by police. He became
are normal with a positive Murphys sign. Labo-
combative when the police tried to arrest him
ratory tests reveal no abnormalities on presenta-
for harassing a Vietnamese restaurant owner.
tion. Which of the following is most likely re-
The police state that the patient insists he has
sponsible for the patients signs and symptoms?
been seeing those Viet Cong bastards again.
(A) Adhesions in her abdominal cavity He is a known abuser of alcohol and opioids.
(B) Cancer in the head of her pancreas On mental status examination, the patient con-
(C) Gallstones in her common bile duct tinues to insist that anyone with a gun is one
(D) Gallstones intermittently obstructing her of the enemy. The patient, when asked about
cystic duct the incident with the store owner, states that he
(E) Inammation of her appendix usually tries to avoid anything that reminds
him of the Vietnamese. When he is faced with
2. A 36-year-old woman presents to her gynecolo- such reminders, he becomes extremely agi-
gist because of recurrent spontaneous abor- tated and behaves in ways he wishes he
FULL-LENGTH EXAMS

tions. She has been pregnant three times in the didnt. A toxicology screen is negative. His
past. However, two of the pregnancies ended in temperature is 37.3C (99.1F), blood pressure
miscarriages prior to week 10 of gestation, and is 140/82 mm Hg, heart rate is 71/min, and re-
the most recent pregnancy resulted in unex- spiratory rate is 18/min. Which of the follow-
plained death of the normal fetus at 13 weeks ing is the best pharmacotherapy for this pa-
gestation. No chromosomal abnormalities were tient?
detected on karyotypes of both parents and the
(A) Chlordiazepoxide
13-week-old fetus. The patient then underwent
(B) Chlorpromazine
a hysterosalpingogram and vaginal ultrasound,
(C) Methadone
which revealed normal maternal anatomy. The
(D) Sertraline
patient denies any history of sexually transmit-
ted infections or a change in vaginal discharge,
4. A 23-year-old man presents to the emergency
although both rapid plasma reagin and Vene-
department with a chief complaint of fever, fa-
real Disease Research Laboratory tests are posi-
cial pain, and epistaxis developing over the
tive. She denies any abnormalities in her men-
past 36 hours. His past medical history is signif-
strual cycle and states she gets her period every
icant for focal segmental glomerulosclerosis
28 days, during which time she uses three to
that required renal transplant 4 months ago.
four pads per day. The patient is a thin, pale
Test Block 7

He had previously been relatively stable on his


woman who avoids the sun because of a facial
immunosuppressive regimen. His temperature
rash that develops with sun exposure. Physical
is 39.1C (102.4F) and his other vital signs
examination is notable for arthritis in the hands
are within normal limits. The physician notices
and a rash on the lower extremities bilaterally.
that there is blood-tinged purulent nasal dis-
Test Block 7 Questions 677

charge, and there are necrotic areas in the na- 6. A 7-year-old boy diagnosed last year with sickle
sal septum. He was brought to the hospital by cell disease after a pain crisis comes to the of-
his parents, who were concerned that he wasnt ce for a routine check-up. He has been well
acting normally. Which of the following is on hydroxyurea with no pain crises or acute
the most appropriate therapy? chest episodes since starting the medication a
year ago. He has had no infections over the last
(A) Intravenous amphotericin B
year and today has no fevers or evidence of in-
(B) Intravenous vancomycin
creasing splenomegaly. Which of the following
(C) Oral azithromycin
is an appropriate preventive measure for this
(D) Oral ketoconazole
patient?
(E) Topical corticosteroid
(A) Monthly RBC transfusion
5. One evening, a nurse gives medication to a (B) Pneumococcal vaccine
62-year-old man who is 1 day status posthemi- (C) Splenectomy
colectomy. The nurse hands the patient two (D) Treatment with deferoxamine
pills, and the patient notices that one does not (E) Weekly intramuscular benzathine penicil-
look familiar to him. He asks the nurse what it lin
is called and what it is for. She explains that it
is propranolol prescribed for high blood pres- 7. A 5-year-old girl with no signicant past medi-
sure. The patient insists that he has never had cal history is brought to the emergency depart-
high blood pressure and refuses to take the ment because of a change in mental status. For
medication. The nurse contacts the night oat the past 4 days she has had rhinorrhea, a dry
intern, who reviews the mans chart and con- cough, and a low-grade fever. One hour prior
cludes that the propranolol was ordered by to presentation she had a 3-minute episode
mistake. The order should have been written during which her extremities shook rhythmi-
for the patient in the next room. What is the cally, and she did not respond to her name.
best next step in management? Following the event she is sleepy but arousable.
She denies having a headache. There is no his-
(A) Ask the nurse to remind the patient to al-
tory of head trauma or family history of sei-
ways be alert to the medications he re-

FULL-LENGTH EXAMS
zures. Her temperature is 39C (102.2F) but
ceives
other vital signs are normal. Physical examina-
(B) Do not dwell on it because the error was
tion reveals a drowsy but otherwise well-
discovered before any harm was done
appearing girl. Her neural examination is non-
(C) Reprimand the nurse for not double-
focal. Serum electrolytes, leukocyte count, and
checking the patients medications before
hemoglobin are all within normal limits.
bringing them to the bedside
Which of the following is the most likely cause
(D) Sign out to the team in the morning the
of this seizure?
details of the medication error
(E) Visit the patients bedside and explain that (A) Arteriovenous malformation
an error was made in writing the medica- (B) Elevated core temperature
tion order, but thankfully the patient no- (C) Intracerebral tumor
ticed the error before taking the wrong (D) Meningitis
medication (E) Subdural hematoma
Test Block 7
678 Section II: Full-Length Examinations Questions

8. An 87-year-old woman with hypertension and is 18/min. On physical examination the abdo-
osteoporosis trips and falls over her sleeping men is very tender, rigid, and distended, with
poodle. She does not lose consciousness, and increased tympany. Bowel sounds could not be
head and neck imaging rules out cervical frac- appreciated. The remainder of the physical ex-
ture and intracranial bleed. Pelvic lms dem- amination is noncontributory. X-ray of the ab-
onstrate a nondisplaced fracture of the pubic domen shows free intraperitoneal air. Relevant
ramus and rule out concomitant fractures of laboratory ndings are as follows:
the femur or acetabulum. Which of the follow-
WBC count: 14,000/mm3
ing is the most appropriate management?
Hemoglobin: 13.2 g/dL
(A) Avoid weight bearing prior to follow-up ra- Platelet count: 303,000/mm3
diography in 1 week; further recommenda- Direct bilirubin: 0.2 mg/dL
tions are dependent on radiographic evi- Total bilirubin: 0.9 mg/dL
dence of healing Amylase: 210 U/L
(B) Mandatory 2 weeks of bedrest, followed by Lipase: 20 U/L
weight bearing as tolerated Aspartate aminotransferase: 33 U/L
(C) Pain management and weight bearing as Alanine aminotransferase: 30 U/L
tolerated Alkaline phosphate: 45 U/L
(D) Pelvic stabilization with an external xa-
Which of the following is the most likely diag-
tion device
nosis?
(E) Surgical reduction of the fracture
(A) Cholecystitis
9. A 65-year-old man with uncontrolled hyperten- (B) Hepatitis
sion presents to the emergency department (C) Pancreatitis
with altered mental status. His blood pressure (D) Perforated peptic ulcer
is 210/130 mm Hg, pulse is 110/min, and oxy- (E) Small bowel obstruction
gen saturation is 99% on room air. CT of the
head reveals a posterior fossa bleed. One hour 11. A 27-year-old man presents to the clinic be-
later the patient goes into respiratory failure. cause of dyspnea and fatigue for the past 4
FULL-LENGTH EXAMS

Which of the following is the most likely etiol- weeks that are worsened by exertion. On ques-
ogy? tioning he also complains of malaise, myalgia,
arthralgias, and headache. He has no signi-
(A) Central herniation
cant past medical history. Temperature is
(B) Cerebellar tonsillar herniation into the fo-
37.5C (99.5F), blood pressure is 130/80 mm
ramen magnum
Hg, heart rate is 90/min, respiratory rate is 14/
(C) Cingulate herniation under the falx cere-
min, and oxygen saturation is 99% on room air.
bri
Physical examination reveals 3+ pitting edema
(D) Epidural hemorrhage
of both lower extremities and a jugular venous
(E) Uncal herniation
pressure of 14 cm H2O that increases with in-
spiration. The point of maximal impulse is
10. A 66-year-old man with hypertension, peptic
poorly assessed, but S1 and S2 are discerned.
ulcer disease, and melanoma presents to the
Chest is clear to auscultation bilaterally. His
emergency department with sudden onset of
liver is enlarged with a positive uid wave.
severe (9/10) abdominal pain in the left upper
ECG demonstrated nonspecic low-voltage
quadrant. The patient denies nausea or vomit-
QRS complexes. Echocardiogram shows nor-
ing, and says he has no appetite despite his last
mal ventricular wall thickness. CT shows a
meal being more than 6 hours ago. The pain
thickened pericardium. Catheterization reveals
Test Block 7

radiates to the left shoulder and back and has


prominent x and y descents with left ventricu-
been getting worse with time. He is lying still
lar end-diastolic pressure equal to right ventric-
on the bed with his knees drawn up to his chest
ular end-diastolic pressure. Which of the fol-
and is taking shallow breaths. His temperature
lowing is the most appropriate treatment?
is 37C (100.2F), blood pressure is 152/88
mm Hg, pulse is 110/min, and respiratory rate
Test Block 7 Questions 679

(A) Diuresis (D) The team should give the dose of antibiot-
(B) Ibuprofen ics necessary to cure the current infection
(C) Pericardiocentesis (E) The team should withhold any additional
(D) Pericardiotomy treatment
(E) Prednisolone
14. An HIV-positive 33-year-old man who refuses
12. A 34-year-old man with a history of HIV infec- antiretroviral therapy presents to the emer-
tion presents to his primary care physician be- gency department complaining of a 6-week
cause of a 4-day history of purulent yellow- history of fever, cough, and night sweats. His
green sputum, shortness of breath, and chest current CD4+ cell count is 46/mm and his vi-
pain that is worse with inspiration. His temper- ral load is 54,000/mL. X-ray of the chest reveals
ature is 38.8C (101.8F), blood pressure is hilar lymphadenopathy and bilateral inltrates.
116/78 mm Hg, respiratory rate is 22/min, and A complement xation test reveals a titer of
heart rate is 88/min. Physical examination re- 1:128. His physician suspects coccidioidomy-
veals decreased breath sounds and dullness to cosis. Which of the following would be an ap-
percussion in the right lower lobe. X-ray of the propriate treatment?
chest and blood cultures are obtained, and re-
(A) Intravenous amphotericin B followed by
sults are pending. Which of the following is
2 months of oral itraconazole
most likely causing the patients underlying
(B) Intravenous vancomycin for 14 days
condition?
(C) Intravenous vancomycin for 2 months
(A) Group B streptococcus (D) Oral itraconazole for 2 weeks
(B) Haemophilus inuenzae (E) Supportive care only; the disease is self-
(C) Pneumocystis jiroveci limited
(D) Respiratory syncytial virus
(E) Streptococcus pneumoniae 15. A 22-year-old woman presents to the emer-
gency department with a chief complaint of
13. An 84-year-old woman with chronic obstruc- sharp chest pain for the past day. The pain ra-
tive pulmonary disease has been on a ventilator diates to her shoulders and worsens when she

FULL-LENGTH EXAMS
for 3 weeks in the intensive care unit, despite lies down. In addition, she complains of being
multiple spontaneous breathing trials. When short of breath and having a mild cough. She
she was admitted to the hospital 4 weeks ago has no signicant past medical history and
for pneumonia, she signed a Do Not Resusci- takes no medications except for oral contracep-
tate form and gave her niece durable power of tive pills. On physical examination the patient
attorney in writing. One day the patients tem- is crying and leaning forward. Her lungs are
perature spikes to 40C (104F), and the team clear to auscultation bilaterally. Her cardiac ex-
tells the family that intravenous antibiotics amination reveals a regular rate and rhythm,
would be a necessary step to keep the patient normal S1 and S2, and a soft friction rub. Her
alive. The niece instructs the medical team to temperature is 38.8C (101.8F), pulse is 100/
withhold any additional life-sustaining treat- min, blood pressure is 102/73 mm Hg, respira-
ment. How should the medical team proceed? tory rate is 23/min, and oxygen saturation is
100% on room air. An ECG is performed and
(A) All treatments should be given until the
showed diffuse ST-segment elevation. X-ray of
patient goes into cardiac or respiratory ar-
her chest shows an enlarged, ask-shaped car-
rest
diac silhouette. Which of the following is the
(B) The team should call the hospital ethics
most likely etiology of this patients condition?
committee before further decisions are
Test Block 7

made (A) Aortic dissection


(C) The team should discuss the matter with (B) Myocardial ischemia
the patients daughter because she is the (C) Thromboembolism
closest family member and therefore (D) Thyroid storm
should be the surrogate decision maker (E) Viral infection
680 Section II: Full-Length Examinations Questions

16. A 19-year-old woman undergoes a bone mar- medication, the name of which he cannot re-
row transplant from her human leukocyte anti- call. He was in good health until mid-day,
gen-matched sister to treat refractory acute when he went to a picnic with friends. He re-
lymphocytic leukemia. In addition to antibiotic ports having eaten aged cheddar cheese, olives,
prophylaxis, her oncologist recognizes the need and a few glasses of red wine. The patient is di-
for vaccination against likely pathogens. What aphoretic with a blood pressure of 200/130 mm
is the appropriate vaccination schedule for pro- Hg and a heart rate of 120/min. ECG reveals
tection against Streptococcus pneumoniae, Hae- sinus tachycardia and a urine toxicology test is
mophilus inuenzae, and Neisseria meningiti- negative. Which of the following has the pa-
dis? tient most likely recently started taking?
(A) Advise the patient and all close contacts to (A) Fluphenazine
avoid vaccination to avoid the risk of live (B) Lamotrigine
viral reactivation (C) Methylphenidate
(B) Forgo vaccination of the patient because of (D) Tranylcypromine
the risk of pathogen reactivation in the im- (E) Triazolam
munosuppressed host, but vaccinate all
household members 19. A 13-year-old white girl has had a recurrent
(C) Vaccinate 12 and 24 months after trans- cough that is productive of white sputum for
plantation and conrm that all household the past 6 months. She is the product of a nor-
members have been vaccinated mal pregnancy, delivered at 37 weeks gesta-
(D) Vaccinate immediately after giving donor tion. She also had recurrent upper respiratory
bone marrow to condition the marrow as it infections as a child. She is at the 10th percen-
engrafts tile for weight and height, even though her par-
(E) Vaccinate the patient prior to beginning ents and brother are at the 50th percentile for
the immunosuppressive transplant proto- weight and height. She is unhappy with her
col weight and stature and reports minimal weight
gain despite having a large appetite and eating
17. A 70-year-old man with a history of well- large meals. She also reports chronic large,
FULL-LENGTH EXAMS

controlled hypertension has an asymptomatic foul-smelling, oating stools. She had her rst
carotid bruit during a routine physical exami- period 2 years ago but has not had her period
nation. An ultrasound of the patients neck re- in the past 6 months. Her blood pressure is
veals 80% stenosis of the right carotid artery. 117/81 mm Hg, pulse is 60/min, respiratory
The current recommended management in rate is 20/min, and temperature is 38.3C
this situation includes which of the following? (101F). She appears small for her stated age
and has digital clubbing. The rest of the physi-
(A) Carotid angioplasty
cal examination is unremarkable. X-ray of the
(B) Carotid endarterectomy
chest shows hyperinated lungs. Which of the
(C) Medical management with aspirin 325 mg
following is the most appropriate next step in
every day
managing this patients nutritional status?
(D) Medical management with warfarin
titrated to International Normalized Ratio (A) Administer laxatives
of 23 (B) Dietary supplements alone
(E) Medical risk factor management followed (C) Encourage a lower caloric intake
by carotid endarterectomy with the onset (D) Pancreatic enzyme replacement and dietary
of neurologic symptoms supplements
(E) Withhold fat from her diet
Test Block 7

18. A 45-year-old man with treatment-refractory


depression presents to the emergency depart- 20. A 58-year-old man presents to his primary care
ment complaining of throbbing headache, physician for follow-up. Five years ago he was
nausea, and vomiting. Over the past several diagnosed with type 2 diabetes mellitus, and 9
years he has tried numerous antidepressants years ago he was diagnosed with hypertension.
without success and recently started a new
Test Block 7 Questions 681

At present his medications include metformin (A) CT scan with contrast


and hydrochlorothiazide. There is a family his- (B) Dilation and curettage
tory of cerebrovascular accident. On examina- (C) Follow weekly -hCG levels
tion there is no evidence of retinopathy or neu- (D) Laparoscopy
ropathy, and his blood pressure is 126/72 mm (E) Repeat transvaginal ultrasound
Hg. Laboratory studies indicate a hemoglobin
A1c of 6.8% and trace albumin in the urine. 22. A 64-year-old woman with Graves disease pre-
Which of the following additions to the pa- sents to the emergency department with sev-
tients medication regimen is most appropriate eral hours of vomiting and diarrhea. She is ac-
at this time? companied by her husband, who says that she
has been very agitated and does not remember
(A) Furosemide
her name or where she is. One week ago she
(B) Gembrozil
was diagnosed with viral pneumonia. Her tem-
(C) Labetalol
perature is 40C (104F), heart rate is 145/
(D) Lisinopril
min, and blood pressure is 145/92 mm Hg. Her
(E) Lispro insulin
skin is jaundiced. On cardiac examination she
is tachycardic but has a regular rhythm with a
21. A 31-year-old G1P1 woman returns to the
normal S1 and S2; no murmurs are heard. Pul-
emergency department complaining of worsen-
monary examination is signicant for diffusely
ing lower abdominal pain and vaginal bleed-
coarse breath sounds. Her abdomen is soft and
ing. She presented to the emergency depart-
nondistended with no masses. On neurologic
ment 3 days ago with the same complaints.
examination she has a coarse tremor in both
Serum -human chorionic gonadotropin
hands and 3+ reexes bilaterally in the biceps,
(-hCG) at that time was 1250 mIU/mL, and
knees, and ankles. Which of the following is
transvaginal ultrasound was negative for an in-
the best agent for initial management?
trauterine or extrauterine gestational sac and
did not show an adnexal mass or echogenic or (A) Dobutamine
large volume cul-de-sac uid. She is sexually (B) Enalapril
active with her husband, and she uses a dia- (C) Levothyroxine

FULL-LENGTH EXAMS
phragm for contraception. She had a chlamyd- (D) Methimazole
ial infection 7 years ago but otherwise her past (E) Propranolol
medical history is unremarkable. Her last men-
strual period was 6 weeks ago. She is afebrile 23. Why are PPD tests used throughout the United
with normal vital signs. Laboratory studies States as a screening test for tuberculosis?
show: (A) The test has a low negative predictive
WBC count: 5400/mm value
Hemoglobin: 13.9 g/dL (B) The test has a low positive predictive value
Platelet count: 200,000/mm (C) The test is dependent on disease incidence
Quantitative serum -hCG level: 1100 mIU/mL (D) The test is dependent on disease preva-
lence
Which of the following is the best next step in
(E) The test is highly sensitive
diagnosis?
Test Block 7
682 Section II: Full-Length Examinations Questions

24. A 27-year-old heroin addict presents to the 26. A 43-year-old woman who works as a nurse has
emergency department with mental status a 10-year-old son who was diagnosed with insu-
changes and severe headache. Physical exami- lin-dependent diabetes at age 7 years. It is later
nation reveals substantial meningismus, a posi- discovered that she has been injecting her son
tive Brudzinski sign, and track marks on both with glucagon to simulate her childs illness.
arms. In the absence of focal neurologic signs, Which of the following is most likely true
a lumbar puncture is performed, which shows: about the mother?
WBC count: 257,000/mm, 62% lymphocytes (A) She has a history of investigation by child
RBC count: 4000/mm protective services
Glucose: 81 mg/dL (B) She has an abnormal psychological prole
Total protein: 40 mg/dL (C) She has used one doctor for most of her
Gram stain: No organisms seen life
(D) She will appear to those who know her to
Blood cultures are negative. Given the patients
be a poor mother in the past
known risk factor of intravenous drug use, a
(E) She will not stop her behavior even after
rapid HIV antibody test is also performed, and
being found out
the results are indeterminate. Titers for herpes
simplex virus type 1 are sent as well, along with
27. A 72-year-old woman presents to the emer-
other viral tests; acyclovir is started prophylacti-
gency department because of headache. She
cally. Which of the following measures is most
says that the pain, which has been present for
immediately appropriate?
the past week, is constant, rates 7 of 10 in se-
(A) Determine the HIV viral load verity, and localized to the left side of her fore-
(B) Obtain an echocardiogram head and above her left ear. She also notes
(C) Repeat the lumbar puncture pain in her jaw when she chews. She also com-
(D) Start highly active antiretroviral therapy plained of blurry vision. On review of systems
(E) Start intravenous ceftriaxone and vanco- she notes a 4.5-kg (10-lb) weight loss over the
mycin past month. She denies chest pain, shortness of
breath, abdominal pain, change in bowel
FULL-LENGTH EXAMS

25. A 45-year-old obese white man comes to his movements, or weakness or numbness in her
physician for a routine visit. He complains of extremities. She has a temperature of 37.2C
burning chest pain for the past month that oc- (99F), heart rate of 86/min, and blood pres-
curs with meals and sometimes awakens him sure of 116/78 mm Hg. She has tenderness to
from sleep. He smokes one pack of cigarettes palpation of the left temporal area. On neuro-
and drinks two beers daily. His temperature is logic examination cranial nerves II through XII
36.5C (97.7F), heart rate is 65/min, blood are intact, she has normal sensation to light
pressure is 137/78 mm Hg, and respiratory rate touch and vibration bilaterally, and strength is
is 9/min. Which of the following would be the 5/5 in all four extremities. Laboratory tests
most appropriate initial treatment for his symp- show:
toms?
WBC count: 9800/mm
(A) Avoiding spicy foods Hemoglobin: 10.1 g/dL
(B) Histamine2-receptor blockers Platelet count: 560,000/mm
(C) Lifestyle modication including weight Erythrocyte sedimentation rate: 110 mm/hr
loss, cessation of smoking, and avoiding al- C-reactive protein: 3.2 ng/mL (normal 0.7
cohol 1.89 ng/mL)
(D) Nissen fundoplication Alanine aminotransferase: 38 U/L
Test Block 7

(E) Proton pump inhibitors Aspartate aminotransferase: 62 U/L


Alkaline phosphatase: 410 U/L
Which of the following is the best rst step in
management?
Test Block 7 Questions 683

(A) Antineutrophil cytoplasmic antibody titer 29. A 42-year-old homeless man is brought to the
(B) Initiation of glucocorticoid therapy emergency department. He is obtunded and
(C) Initiation of therapy with nonsteroidal anti- there is vomit on his clothing. His breathing is
inammatory drugs rapid and deep. He has a glucose level of 430
(D) Magnetic resonance angiography of the mg/dL and a blood urea nitrogen:creatinine ra-
brain tio of 28. The presence of which of the follow-
(E) Temporal artery biopsy ing is important in distinguishing nonketotic
hyperglycemia from diabetic ketoacidosis?
28. A nurse in the newborn care nursery is con-
(A) Dehydration
cerned because a white male newborn has had
(B) Hyperglycemia
one episode of emesis and has failed to pass
(C) Kussmaul breathing
meconium. His heart rate is 125/min, blood
(D) Mental status change
pressure is 75/50 mm Hg, respiratory rate is 40/
(E) Vomiting
min, and temperature is 37.1C (98.7F).
Physical examination is unremarkable except
30. A 35-year-old homosexual man presents to his
for a noticeably distended abdomen. A radio-
physician expressing concern that he might
graph after water-soluble air contrast enema is
have contracted a sexually transmitted disease
shown in the image. Which of the following is
through several instances of unprotected sex
the most likely underlying cause of this condi-
with anonymous partners over the past several
tion?
years. He requests testing and his Western blot
test is read as positive for HIV. He asks what
the chances are that the result is wrong. What
is the correct reply?
(A) This is unlikely, as the test has 100% valid-
ity
(B) This is possible, as the test has low positive
predictive value in this circumstance
(C) This is unlikely, as the test is highly sensi-

FULL-LENGTH EXAMS
tive
(D) This is unlikely, as the test is highly spe-
cic

31. A 36-year-old G1P0 woman presents for her


rst prenatal visit late in her rst trimester of
Reproduced, with permission, from Tintinalli JE, Kelen pregnancy complaining of persistent vaginal
GD, Stapczynski S, Ma OJ, Cline DM. Tintinallis Emer- bleeding, nausea, and pelvic pain. Physical ex-
gency Medicine: A Comprehensive Study Guide, 6th edi-
amination is notable for a gravid uterus larger
tion. New York: McGraw-Hill, 2004: Figure 127-1.
than expected for gestational age. Fetal heart
(A) Cystic brosis tones are absent. Which of the following is
(B) Edwards syndrome most likely to be true?
(C) Meckels diverticulum
(A) -Human chorionic gonadotropin level
(D) Trisomy 21
will be higher than normal
(E) Williams syndrome
(B) Hematocrit will be increased
(C) Laboratory evaluation will reveal an in-
crease in thyroid-stimulating hormone
Test Block 7

(D) No theca lutein cysts will be visualized on


pelvic ultrasound
(E) Pelvic sonogram will show a normal size
uterus
684 Section II: Full-Length Examinations Questions

32. A 60-year-old man visits a health clinic for the Which of the following is the best next step in
third time since moving to the United States management?
from Ireland about a year ago. On the rst two
visits, his blood pressure was 140/90 and
142/86 mm Hg. As recommended by the clinic
physician, he eats a healthy, low-salt diet in-
cluding plenty of fruits and vegetables, and ex-
ercises regularly. Today he notes that for about
the past 6 months, he has had difculty urinat-
ing. He stated that it often takes him a while to
start his stream, and the stream is not as force-
ful as it used to be. Today he has a blood pres-
sure of 142/88 mm Hg and heart rate of 70/
min. Physical examination shows his prostate is
rm and large without nodularity. Urinalysis is
normal, and his serum creatinine is 0.8 g/dL.
ECG shows normal sinus rhythm. What medi-
cation is most appropriate to start in this pa-
tient?
(A) Finasteride Reproduced, with permission, from Chen MYM, Pope
(B) Lisinopril TL, Ott DJ. Basic Radiology. New York: McGraw-Hill,
(C) No medication is needed 2004: Figure 4-52.
(D) Terazosin
(A) Furosemide
(E) Thiazide diuretic
(B) Ibuprofen
(C) Intravenous ceftriaxone
33. A 46-year-old man presents to the clinic be-
(D) No treatment is indicated
cause over the past several years his shoe size
(E) Thoracentesis
has increased and his wedding ring feels
FULL-LENGTH EXAMS

tighter. He states that his wife thinks his fore-


35. A 47-year-old man with hypertension is pre-
head is getting bigger. Laboratory studies show
scribed hydrochlorothiazide by his primary
elevated serum levels of insulin-like growth fac-
care physician. Which of the following is a po-
tor-1. Which of the following is the most ap-
tential effect of this medication?
propriate initial therapy for this patient?
(A) Hyperkalemia
(A) Cortisol
(B) Hyperlipidemia
(B) Growth hormone
(C) Hypernatremia
(C) Metyrapone
(D) Hypocalcemia
(D) Octreotide
(E) Hypouricemia
(E) Thyroid hormone
36. A 35-year-old woman returns to her primary
34. A 64-year-old woman presents to the clinic
care physician for follow-up 2 weeks after pass-
with difculty breathing and mild chest pain.
ing a kidney stone. This is the third stone that
The chest pain is not exertional or positional
the patient has passed in the past year. The
and is not relieved by rest. She has no cough or
stones were evaluated, and all three were com-
fever, and no other symptoms. Review of sys-
posed of calcium phosphate. She has no other
tems is signicant for morning stiffness of the
past medical history. She mentions that she has
Test Block 7

knees and hands. Physical examination shows


not been taking any medications but just
some decreased breath sounds and dullness to
started using over-the-counter eye drops for her
percussion in both lung bases. There are nor-
dry eyes. Her temperature is 37.1C (98.8F),
mal heart sounds and no evidence of cardio-
heart rate is 80/min, blood pressure is 120/80
megaly or jugular venous distension. A decubi-
mm Hg, and respiratory rate is 18/min. On ex-
tus x-ray of the chest is shown in the image.
Test Block 7 Questions 685

amination her mucous membranes appear dry. tient, and inform them of the patients spe-
Her lungs are clear to auscultation, and her ab- cic threats against the young woman
domen is soft and nontender. A chart review (C) Contact the student who is threatened di-
reveals that her urine pH since she passed the rectly and inform her of the situation, al-
rst stone has been consistently >7.0. The phy- lowing her to contact the police herself;
sician decides to give her oral challenge with the physicians obligation is fullled at this
1.9 mmol/kg of ammonium chloride and re- point
peats a measurement of her urine pH. The pH (D) Do nothing because it is a violation of pa-
after this challenge is 6.8. The physician de- tient condentiality to disclose the details
sires to continue the work-up after this nding. of his conversation with the patient to the
What antibodies test(s) would most likely yield authorities
the diagnosis? (E) Do nothing because the patient has made
similar threats during previous manic epi-
(A) Antiglomerular basement membrane
sodes and has not carried them out
(B) Antihistone
(C) Anti-Ro/SSA and anti-La/SSB
39. A 56-year-old man with hypertension presents
(D) Circulating anti-neutrophilic cytoplasmic
to the clinic with progressive confusion over 2
autoantibodies
days. He also has a rash on his hands. He has
(E) Perinuclear anti-neutrophilic cytoplasmic
no history of trauma to the head, no preceding
autoantibody
headache or neck stiffness, and does not drink
alcohol or use any illicit drugs. His only medi-
37. An 85-year-old white man with hypertension
cation is a thiazide diuretic. On examination
and cerebrovascular disease presents to his phy-
he is afebrile with intact pupillary reexes and
sicians ofce for a routine check-up. Review of
negative Brudzinskis sign. Laboratory studies
systems is positive only for periodic ringing in
are as follows:
the ears. Physical examination reveals left fa-
cial paralysis and poor hearing on the left side. Na+: 138 mEq/L
A stroke in which vascular territory is most K+: 4.3 mEq/L
likely? Blood urea nitrogen: 17 mg/dL

FULL-LENGTH EXAMS
Creatinine: 1.1 mg/dL
(A) Anterior cerebral artery
Glucose: 93 mg/dL
(B) Anterior inferior cerebellar artery
Total bilirubin: 3.2 mg/dL
(C) Middle cerebral artery
Direct bilirubin: 0.6 mg/dL
(D) Posterior cerebral artery
Albumin: 3.6 g/dL
(E) Posterior inferior cerebellar artery
Aspartate aminotransferase: 15 U/L
Alanine aminotransferase: 12 U/L
38. A 23-year-old man with major depression with
WBC count: 8600/m3
psychotic features fails to rell his medication
Hemoglobin: 8.5 g/dL
prescriptions for several months. He calls his
Platelet count: 90,000/m3
psychiatrist and states that he is very upset be-
Reticulocytes: 3.2%
cause a fellow student who is clearly in love
Salicylate level: undetectable
with him has not returned his calls. While
speaking to the psychiatrist, he sounds agitated, Results of a urine toxicity screen are negative,
displays ight of ideas and clanging speech pat- and CT of the head is negative for intracranial
terns, and begins making threats against the hemorrhage and mass lesions. Which of the
fellow student. What is the most appropriate following is the most likely diagnosis?
next step for the physician to take?
(A) Diuretic overdose
Test Block 7

(A) Call the patients roommate and ask him (B) Drug-induced hemolytic anemia
to keep an eye on the patient until he (C) Liver failure
can safely be brought to the hospital (D) Thrombotic thrombocytopenic purpura
(B) Call the police and ask them to help emer- (E) Viral meningoencephalitis
gency medical services apprehend the pa-
686 Section II: Full-Length Examinations Questions

40. A 32-year-old diabetic woman at 43 weeks ges- (A) Dobutamine


tation has an elective cesarean delivery of a (B) Epinephrine
4-kg (9-lb) baby boy. On postoperative day 1 (C) Isoproterenol
she develops a fever of 38.7C (101.7F) in the (D) Labetalol
setting of vague uterine tenderness. She is (E) Norepinephrine
treated with intravenous clindamycin and gen-
tamicin for endometritis. The patient contin- 42. A 63-year-old woman presents to psychiatry
ues to spike fevers over the next few days and clinic for an initial visit. She states that her
complains of worsening abdominal pain, with husband has recently fallen ill, and that he
extension to the right ank and lower back. On may die in the next month. She spends most of
the morning of postoperative day 4, the pa- her time at the local church, praying that her
tients fever spikes to 39.9C (103.8F). Pelvic husband will be cured. Even though her hus-
examination is unremarkable, and an MRI band is currently in a coma, she hears him talk
shows a thrombosed left ovarian vein. Which to her at the church. She also sees his image at
of the following is the most appropriate treat- times when she walks along the street. The pa-
ment strategy? tients preoccupation with her husbands illness
has kept her from spending any time with her
(A) Add additional -lactamase coverage to the
other family or friends, and it has also kept her
current antibiotic regimen, without hepa-
from paying her bills. The patient denies any
rin administration
psychiatric history previous to her husband fall-
(B) Change the antibiotic regimen without
ing ill. She further denies changes in energy,
heparin administration
sleep, or appetite. Which of the following is
(C) Continue the current antibiotic regimen
the most appropriate diagnosis at this point?
and add heparin
(D) Discontinue the antibiotic treatment and (A) Adjustment disorder
administer heparin alone (B) Major depressive disorder
(E) Surgically remove the clot (C) Major depressive episode
(D) Normal grief
41. A 70-year-old man is brought to the emergency (E) Psychotic disorder not otherwise specied
FULL-LENGTH EXAMS

department after his family found him to be


extremely pale and acting weak and confused. 43. A 60-year-old man with a history of congestive
On arrival the patient is noncommunicative heart failure presents to his physician with an
but arousable and has a fever of 38.6C 8-month history of exertional fatigue and exces-
(101.5F), blood pressure of 85/50 mm Hg, sive daytime sleepiness. His wife has told him
and pulse of 95/min. Physical examination that he has episodes of choking in his sleep.
demonstrates clear lung elds and normal car- He has been taking enalapril, metoprolol, and
diac and abdominal examinations, but the pa- hydrochlorothiazide and is compliant. He de-
tient is clearly pallid. Laboratory studies reveal nies chest pain, palpitations, or swelling in his
a WBC count of 17,000/mm with 94% neu- legs. He is afebrile and vital signs are normal.
trophils, and urinalysis reveals >100,000 bacte- His oxygen saturation is 99% on room air and
ria/mL with increased leukocyte esterase, ele- his body mass index is 30.2 kg/m. On cardiac
vated nitrites, and a WBC count >5/hpf. The examination his apical impulse is diffuse and
patient is given aggressive uid resuscitation shifted 2 cm to the left of the midclavicular
and central venous access is established. Al- line. Heart sounds are normal. Jugular venous
though his central venous pressure has risen to distention and peripheral edema are absent. X-
above an appropriate level of 8 mm Hg, his ray of the chest shows no active disease and
mean arterial pressure remains <65 mm Hg. ECG is signicant for left ventricular hypertro-
Test Block 7

Which of the following is the most appropriate phy. Which of the following is the most likely
subsequent medication? cause of the patients exertional fatigue?
Test Block 7 Questions 687

(A) Decreased left ventricular afterload (C) Benign prostatic hyperplasia


(B) Decreased right ventricular afterload (D) Central diabetes insipidus
(C) Hypertension (E) Detrusor instability
(D) Increased negative intrathoracic pressure (F) Diabetes mellitus
(E) Pulmonary vasodilation (G) Gonorrhea
(H) Interstitial cystitis
44. A 40-year-old man is playing basketball at his (I) Nephrogenic diabetes insipidus
companys annual picnic when he falls directly (J) Neurogenic bladder
onto his left elbow. He has instant pain and is (K) Prostatitis
unable to straighten his elbow. In obvious dis- (L) Psychogenic polydipsia
tress, he is taken immediately to the emergency (M)Urethral stricture
department by his wife. He has a bag of ice on
his left elbow, which is very swollen. On exam- 45. A 70-year-old man presents to the physician be-
ination he is unable to sense light touch and cause of urinary frequency for several weeks.
two-point discrimination in his left fth digit He states that he was urinating well after a
and the medial aspect of his left hand. All other transurethral resection of the prostate 5 months
areas are neurologically intact. What motor ago, but his urinary stream decreased again
test will be most useful to assess the integrity of over the past 3 weeks. He denies pain or blood
the nerve most likely to be damaged? in his urine. He is afebrile and has a palpable
bladder dome above the pubic symphysis. On
(A) Abduction of the second and third digits
digital rectal examination the prostatic fossa is
(B) Extension of the fth digit
empty. Laboratory tests show a WBC count of
(C) Extension of the wrist
4000/mm3, creatinine of 1.0 mg/dL, and potas-
(D) Flexion of the thumb
sium of 5.0 mEq/L.
(E) Flexion of the wrist
46. A 40-year-old woman with a history of multiple
E X T E N D E D M ATC H I N G sclerosis presents to her physician for routine
follow-up. She has been utilizing clean inter-
mittent self-catheterization for the past 3

FULL-LENGTH EXAMS
The response options for the next 2 items are
the same. Select one answer for each item months. She denies any abdominal or ank
in the set. pain. She has a temperature of 37C (98.6F)
and blood pressure of 120/80 mm Hg. Her
For each patient with urinary frequency, select the WBC count is 4000/mm3. A urine culture is
most likely diagnosis. positive for Escherichia coli.

(A) Acute cystitis


(B) Asymptomatic bacteriuria

Test Block 7
688 Section II: Full-Length Examinations Answers

AN S W E R S

1. The correct answer is D. This patient is pre- is associated with an elevation in WBC count
senting with characteristic signs and symptoms and temperature, along with nausea, vomit-
of cholelithiasis with intermittent obstruction of ing, and anorexia. On physical examination,
the cystic duct. This accounts for the duration one would see pain at McBurneys point, and
of her pain, association with large meals, vomit- sometimes a Rovsings, psoas, or obturator sign.
ing, and the presence of a Murphys sign (pain It also is much more likely to occur in young
upon deep inspiration during palpation of the people rather than middle-aged women. The
right upper quadrant). Her personal characteris- treatment of appendicitis entails surgical re-
tics should have also pointed towards chole- moval of the appendix.
lithiasis. The 4 Fs that put a person at higher
risk for stones are Fat, Forty, Female, and Fer- 2. The correct answer is B. Antiphospholipid an-
tile. Cholecystectomy is recommended if pa- tibody syndrome (APLS) is characterized by a
tients have recurrent symptoms, or have signs of hypercoagulable state with circulating antibod-
complications such as acute cholecystitis (fever, ies that leads to recurrent fetal loss and/or arte-
right upper quadrant pain, and leukocytosis). rial or venous thrombosis as well as thrombocy-
topenia. Thrombus formation and infarction
Answer A is incorrect. Adhesions in the abdo- may account for the pregnancy morbidity ob-
men would likely be the cause of a partial or served in APLS. APLS may occur as a primary
complete small bowel obstruction (SBO), not disorder or in association with an underlying
biliary colic. SBO would present with crampy disorder, most commonly systemic lupus ery-
abdominal pain, possibly with changes in thematosus, as in the case described, but also
stools, and would not have pain localizing to with infection, neurologic disease, or the use of
the right upper quadrant with a positive Mur- specic drugs (procainamide, chlorpromazine,
phys sign. Surgical laparotomy would be nec- phenytoin, and quinidine, among others).
essary if SBO were suspected. False-positive syphilis serology may occur in
Answer B is incorrect. Cancer in the head of patients with APLS because the anticardiolipin
FULL-LENGTH EXAMS

the pancreas commonly presents with painless antibodies in the patients serum cross-react
jaundice and is not often accompanied by in- with the syphilis antigen used in the test. It
termittent postprandial epigastric pain. Find- should be noted that although rapid plasma re-
ings on palpation of the right upper quadrant agin and/or Venereal Disease Research Labora-
would likely not be tenderness and a Murphys tory tests may be falsely positive, anti-trepone-
sign, but could include a painless palpable mal antigen assays and uorescent treponemal
gallbladder (Courvoisiers sign). If localized, antibody assay will remain specic for syphilis.
pancreatic cancer may be amenable to surgi- APLS should be considered when at least one
cal resection. clinical criterion (vascular thrombosis or preg-
nancy morbidity) and one laboratory criterion
Answer C is incorrect. Stones in the common
(positive anticardiolipin antibody or lupus anti-
bile duct (choledocholithiasis) could present
coagulant assay) are met.
with similar epigastric pain, but would most
likely be more constant. As well, one would Answer A is incorrect. Anatomic abnormalities
expect signs of an increased bilirubin level on may contribute to recurrent spontaneous abor-
laboratory tests, and physical examination nd- tions. Although hysterosalpingograms have a
ings (jaundice and scleral icterus). Treatment diagnostic accuracy of only 55% when used
is via endoscopic retrograde cholangiopancre- alone, this imaging modality used in combina-
Test Block 7

atography and endoscopic sphincterotomy. tion with ultrasound examination improves the
diagnostic accuracy to 90%. CT scan, on the
Answer E is incorrect. Appendicitis would
other hand, has limited utility in diagnosing
present with diffuse abdominal pain which
uterine anatomic abnormalities and would be
later localizes to the right lower quadrant. It
less likely to show an underlying abnormality
Test Block 7 Answers 689

that could contribute to recurrent spontaneous cits in PTSD include symptoms of constant
abortions. Evidence of systemic lupus erythe- re-experiencing of traumatic events, avoid-
matosus in the context of recurrent spontane- ance of stimuli associated with the traumatic
ous abortions is more likely due to APLS than event, and persistent symptoms of increased
to an anatomic abnormality. arousal. The re-experiencing of traumatic
events can often be misinterpreted as halluci-
Answer C is incorrect. Pituitary disorders may
nations. Moreover, efforts to avoid reminders
cause alterations in hormone levels that lead to
of the traumatic events can make the patient
infertility and absence of ovulation with oligo-
appear delusional. Patients who are suffering
or amenorrhea. One study showed that women
from PTSD benet from cognitive psycho-
with hypopituitarism have a miscarriage rate
therapy oriented toward PTSD and selective
of 28% and a mid-trimester fetal death rate of
serotonin reuptake inhibitors.
11%. Although the patient may have a pitu-
itary disorder leading to the recurrent sponta- Answer A is incorrect. Chlordiazepoxide may
neous abortions, this diagnosis is less likely, be used to treat alcohol withdrawal or mild
as she has been able to conceive and has had anxiety. However, both diagnoses are unlikely
normal menstrual cycles without amenorrhea given the patients normal vital signs and ex-
or oligomenorrhea. Evidence of systemic lu- tremely agitated behavior on certain cues.
pus erythematosus in the context of recurrent
Answer B is incorrect. It is possible that the
spontaneous abortions with normal menstrual
patient has schizophrenia and needs antipsy-
history is more likely to be due to APLS than
chotics. However, his hypervigilance, as well as
to a deciency in pituitary function.
the content of his hallucinations and thoughts,
Answer D is incorrect. Cervical cultures are make it more likely that the disorder is the re-
useful for diagnosing sexually transmitted in- sult of trauma and stress.
fections as well as bacterial, trichomonal, and
Answer C is incorrect. The patient is display-
fungal infections of the female reproductive
ing agitation, which would not typically be
tract. The potassium hydroxide whiff test is
seen in someone suffering from opiate intoxi-
useful to support a diagnosis of bacterial vagi-
cation. Thus, PTSD is more likely than opiate
nosis. Bacterial vaginosis is the most common

FULL-LENGTH EXAMS
withdrawal.
cause of vaginitis in women. Although this di-
agnosis may be associated with increased inci- 4. The correct answer is A. In the setting of an
dence of preterm delivery, postpartum fever, immunocompromised host with sinus symp-
and cervical intraepithelial neoplasia, it has toms, a high degree of suspicion should be
not been shown to be associated with recurrent maintained for invasive fungal sinusitis. This
spontaneous abortions. patients symptoms developed over a short time
Answer E is incorrect. Prothrombin time is used frame, and he presented with systemic symp-
to assess coagulation factors involved in the ex- toms, indicative of acute fungal sinusitis. Diag-
trinsic pathway of the clotting cascade. Although nosis depends on pathologic demonstration of
lupus anticoagulants may cause prolongation the organism from a biopsy specimen, but the
of the activated partial thromboplastin time be- usual fungi causing acute sinusitis are zygomy-
cause the prothrombinase complex cannot as- cetes and Aspergillus. While these infections
semble properly, lupus anticoagulants have no are difcult to treat, especially in the presence
direct effect on the prothrombin time. An abnor- of central nervous system invasion, intravenous
mal prothrombin time therefore would not have amphotericin B would be the antimicrobial of
predicted a diagnosis of APLS as the underlying choice. Emergent surgical biopsy and dbride-
cause of recurrent spontaneous abortions. ment would also be indicated.
Test Block 7

Answer B is incorrect. The clinical picture


3. The correct answer is D. Patients with post-
of rapid-onset nasal septum necrosis and sys-
traumatic stress disorder (PTSD), especially
temic symptoms is more suggestive of an in-
African-American combat veterans, are often
vasive fungal infection, rather than a bacterial
misdiagnosed with schizophrenia. Core de-
etiology. While a bacterial source is possible,
690 Section II: Full-Length Examinations Answers

this patients immunocompromised state sig- sponsibility to discuss the situation with the pa-
nicantly raises the possibility of fungal infec- tient and not to wait for the next team to make
tion. Furthermore, if this patient had bacterial a decision about disclosure. Although the
sinusitis, the most common bacterial agents person who wrote the order should be made
would be Streptococcus pneumoniae and Hae- aware of his or her mistake, this answer does
mophilus inuenzae, and vancomycin would not go far enough. The patient must also be in-
be an inappropriate antibiotic choice for these formed.
bacteria.
6. The correct answer is B. Sickle cell patients
Answer C is incorrect. While antibacterials
generally have splenic dysfunction due to red
may play a role in this patients care, the like-
blood cell sickling within splenic microvascu-
lihood of fungal disease makes an intravenous
lature. The subsequent repeated splenic infarc-
antifungal a more appropriate therapy.
tions lead to functional asplenism or autosple-
Answer D is incorrect. The fungi typically re- nectomy by adulthood. This places them at
sponsible for invasive fungal sinusitis (Aspergil- increased risk for infection by encapsulated or-
lus, Fusarium, and the zygomycetes) are typi- ganisms, which include Streptococcus pneumo-
cally unresponsive to ketoconazole. niae and Haemophilus inuenzae. Recom-
mended vaccinations include those against
Answer E is incorrect. Topical corticosteroids
pneumococcus, H. inuenzae, hepatitis B, and
would not play a role in the treatment of inva-
inuenza virus.
sive fungal sinusitis.
Answer A is incorrect. Chronic RBC transfu-
5. The correct answer is E. Physicians are obli- sion can decrease the percentage of hemoglo-
gated to disclose to patients errors made in bin S by three mechanisms: (1) dilution, (2)
their medical care. In this case, the error was suppression of endogenous erythropoietin pro-
harmless because the patient discovered the duction due to increased hematocrit, and (3)
mistake before taking the wrong medication, longer half-life of normal hemoglobin as com-
but he nonetheless has a right to know about pared to hemoglobin S. However, transfusion is
the error. Furthermore, knowing about the er- only indicated during aplastic crises and splenic
FULL-LENGTH EXAMS

ror could encourage him to continue being an sequestration, to prevent worsening of acute
active and conscientious participant in his own chest syndrome, and to prevent initial or recur-
medical care. rent stroke due to intracerebral thrombosis.
Answer A is incorrect. Although this is gener- Answer C is incorrect. Splenectomy is indi-
ally a good practice, and it is worthwhile to re- cated after the rst episode of a splenic seques-
mind patients to be alert and active participants tration crisis.
in their own medical care, the interns rst pri-
Answer D is incorrect. Deferoxamine is an
ority should be disclosure of the error.
iron chelating agent, and is used with chronic
Answer B is incorrect. Physicians are obligated transfusion to prevent iron overload syndrome.
to disclose to patients errors that are made in This is particularly useful in -thalassemia.
their medical care.
Answer E is incorrect. Intramuscular benza-
Answer C is incorrect. This error was not the thine penicillin injections are the treatment of
fault of the nurse, who followed orders as writ- choice for syphilis infection and have no place
ten and distributed medication she believed in preventive measures for sickle cell patients.
was correct for the patient. Blaming the situa- However, daily oral penicillin is a good pro-
tion on the nurse is an inappropriate response phylactic measure against pneumococcal in-
Test Block 7

and ignores the physicians true responsibilities. fections, and is recommended for all children
with sickle cell disease up to the age of 5 years.
Answer D is incorrect. Having discovered the
However, children older than age 5 who have
error himself, the intern should do his best
never had a severe pneumococcal infection or
to fully disclose the error to the patient. Even
splenectomy may discontinue penicillin pro-
though the error was not his fault, it is his re-
phylaxis without increase in infection rate.
Test Block 7 Answers 691

7. The correct answer is B. A seizure in a febrile low-energy fall. After a diagnosis of a low-
child that occurs without another precipitating energy pubic ramus fracture is conrmed, and
cause can be attributed to the fever. Febrile sei- additional fractures of the pelvis and femur are
zures are the most common cause of seizure ruled out, treatment is generally nonsurgical
activity in children from the ages of 6 months and aimed at pain reduction and regaining
to 5 years, and approximately 1 in every 25 function, with an emphasis on mobilization as
children will have a febrile seizure during soon as tolerated.
childhood. This is a benign condition and
Answer A is incorrect. Repeat radiographic
>95% of children who experience a febrile sei-
examination is important, and should be done
zure will not develop epilepsy later in life. The
12 weeks following mobilization of the pa-
work-up of a suspected febrile seizure should
tient. However, weight bearing should not be
consist of ruling out most serious conditions
delayed.
that can present with seizure activity, such as
meningitis, electrolyte imbalance, or trauma. Answer B is incorrect. No clearly dened
Treatment is supportive: the fever should be amount of bedrest is indicated for this injury.
controlled and the child protected from injury The goal in this patients treatment should be
should the seizure recur. pain management and a return to her previous
level of functioning.
Answer A is incorrect. Arteriovenous malfor-
mations are abnormal connections of arteries Answer D is incorrect. An operative approach
and veins. They can bleed and cause seizures, is indicated for acetabular and pelvic fractures,
usually generalized tonic-clonic seizures. They but would not be necessary in this patient.
are not associated with fever or upper respira- Answer E is incorrect. An operative approach
tory infections. is indicated for acetabular and some other
Answer C is incorrect. Brain tumors can pre- pelvic fractures, but is not necessary for a low-
sent with seizures, but they usually present energy fracture of the pubic ramus.
with other symptoms such as headache, nau-
sea, vomiting, low-grade fever, lethargy, and 9. The correct answer is B. Herniation of the cer-
weight loss. They are not associated with high- ebellar tonsils downward can cause respiratory

FULL-LENGTH EXAMS
grade fever or upper respiratory infections. depression and death by compressing the brain
stem/medulla. In fact, posterior fossa lesions, as
Answer D is incorrect. Meningitis can cause in this case, can also cause upward herniation
seizures, and fever is a sign of meningitis. of the cerebellum. The most likely etiology of
However, the patient does not have nuchal ri- this respiratory failure is compression of the re-
gidity or an elevated WBC count, which are spiratory center in the brain stem by herniation
classically seen in patients with meningitis. Al- through the foramen magnum.
though nuchal rigidity is typically not seen in
very young patients, this child is old enough to Answer A is incorrect. The history and loca-
demonstrate this cardinal sign of meningitis. tion of the bleed is more consistent with a
tonsillar herniation. Central herniation is also
Answer E is incorrect. Subdural hemato- known as transtentorial herniation, in which
mas result from rupture of the cerebral bridg- the diencephalon and parts of the temporal
ing veins and can cause seizures, headache, lobes are forced through a notch in the tento-
changes in mental status, and contralat- rial membrane. Decreased consciousness oc-
eral hemiparesis. This usually happens after curs early in this type of herniation. Central
trauma, and is more unlikely if headache and diabetes insipidus may also result from com-
history of trauma are absent. pression of the pituitary stalk.
Test Block 7

8. The correct answer is C. This patient has suf- Answer C is incorrect. Cingulate gyrus hernia-
fered a low-energy pelvic fracture, which is the tion typically presents by compressing the an-
most common type of fracture in the elderly, terior cerebral artery, and can cause infarction.
and is more common in women than men. This may manifest as a stroke of the frontal
Most patients with this injury have sustained a lobe.
692 Section II: Full-Length Examinations Answers

Answer D is incorrect. Epidural hemorrhage Answer C is incorrect. Pancreatitis is not as-


was not present on the CT. However, if the sociated with free intraperitoneal air as seen in
bleed was large enough it could cause a her- this patients x-ray of the abdomen. In addition,
niation. patients with pancreatitis often present with el-
evated amylase and lipase, as well as epigastric
Answer E is incorrect. Uncal herniation can
pain, nausea, and vomiting. The history, physi-
compress the third cranial nerve and classically
cal, laboratory, and radiographic ndings are
results in increased intracranial pressure caus-
more supportive of a diagnosis of perforated
ing hypertension, bradycardia, and altered re-
peptic ulcer than pancreatitis.
spiratory pattern.
Answer E is incorrect. SBO inhibits the pas-
10. The correct answer is D. This patient has a sage of intestinal contents and is therefore of-
history of peptic ulcer disease and now presents ten seen with nausea and emesis. In addition,
to the emergency department with a perforated these patients present with crampy abdominal
peptic ulcer. The anterior gastric or duodenal discomfort, distention, and high-pitched bowel
walls are the most common site of such perfo- sounds. It is common to see distended loops of
ration. Patients with an anterior wall perfora- small bowel and air-uid levels radiographi-
tion do not present with acute bleeding be- cally; however, free intraperitoneal air is not
cause of the lack of major blood vessels on the seen in SBO. The history, physical, and labora-
anterior gastric surface. Instead, patients may tory ndings are more supportive of a diagnosis
present with acute upper abdominal pain that of perforated peptic ulcer than SBO.
may radiate to the shoulder or back and a
mildly elevated WBC count and amylase level. 11. The correct answer is D. This patient has con-
Peptic ulcer perforation frequently occurs sev- strictive pericarditis, a chronic process of repair
eral hours after a meal. In addition, an upright after a pericardial injury that leads to brous
x-ray of the abdomen will show free intraperi- thickening of the pericardium and constricts
toneal air, which may lead to increased tym- normal lling of the chambers. The most com-
pany and abdominal distention. Physical exam- mon causes include postviral, radiation, ure-
ination often reveals abdominal rigidity and mia, tuberculosis, postcardiac surgery, and id-
FULL-LENGTH EXAMS

inaudible bowel sounds. iopathic conditions. Clinical manifestations


include right-sided greater than left-sided heart
Answer A is incorrect. The patient in this case
failure as evidenced usually by clear lungs, in-
presents with left upper abdominal pain that
creased jugular venous pressure with Kuss-
began 6 hours after his last meal. Patients with
mauls sign, hepatic congestion, ascites, and
cholecystitis characteristically present with
peripheral edema. This patients history is con-
right upper quadrant pain that is accompanied
cerning for a viral cause, given his fever and ar-
by nausea and vomiting. In addition, cholecys-
thralgia. Moreover, the echocardiographic,
titis is associated with increased bilirubin, alka-
CT, and catheterization results are diagnostic.
line phosphatase, and possibly amylase. Chole-
The echocardiogram shows normal wall thick-
cystitis is often diagnosed with ultrasound and
ness, whereas restrictive pericarditis has in-
is not associated with the presence of free in-
creased wall thickness. CT shows thickened
traperitoneal air.
pericardium. Cardiac catheterization shows
Answer B is incorrect. Hepatitis is not asso- equalization of diastolic pressures of both
ciated with free intraperitoneal air but is as- chambers along with prominent x descents and
sociated with abnormal liver function tests, y descents, both reections of the constrictive
including elevated aspartate transaminase and pressure imposed by the rigid pericardium.
alkaline phosphatase and right upper quadrant The treatment in this situation is pericar-
Test Block 7

pain. The history, physical, laboratory, and ra- diotomy, a surgical intervention.
diographic ndings are more supportive of a
diagnosis of perforated peptic ulcer than hepa-
titis.
Test Block 7 Answers 693

Answer A is incorrect. Diuresis has no role in Answer D is incorrect. Immunocompro-


treating constrictive pericarditis. mised patients have an increased risk of viral
infections such as respiratory syncytial virus.
Answer B is incorrect. Ibuprofen is appropri-
However, overall, the number one cause of
ate for treating pericarditis without a constric-
pneumonia in these patients is Streptococcus
tive process.
pneumoniae.
Answer C is incorrect. Pericardiocentesis is
performed in cases of pericardial tamponade 13. The correct answer is E. On admission, this
with the exception of aortic or myocardial rup- patient took all necessary steps to make sure
ture. Of note is Becks triad: low arterial blood that resuscitation did not take place and that a
pressure, increased central venous pressure, surrogate decision maker was chosen to act if
and distant heart sounds. the patient no longer had decision-making ca-
pacity. The niece, the chosen health care
Answer E is incorrect. Prednisolone is appro-
proxy, requests that life-sustaining care be with-
priate for treating pericarditis without a con-
held. This is ethically the same as a withdrawal
strictive process that does not resolve with con-
of care, a right that all patients and their deci-
servative anti-inammatory drugs.
sion makers can exercise. Examples of life-
sustaining interventions that can be withheld
12. The correct answer is E. This patient is pre-
or withdrawn include ventilation, uids, nutri-
senting with lobar pneumonia, an infection of
tion, and medications, such as antibiotics.
the lung parenchyma characterized by fever,
productive cough, dyspnea, night sweats, and Answer A is incorrect. Although the Do Not
pleuritic chest pain. Physical examination may Resuscitate order is a form of living will cre-
reveal decreased or bronchial breath sounds, ated by the patient and should be abided by if
rales, wheezing, dullness to percussion, egoph- cardiac or respiratory arrest occurs, withdrawal
ony, and tactile fremitus. Although immuno- or withholding of care is another right that pa-
compromised patients are subject to infections tients or elected decision makers can exercise
from fungi, viruses, and Pneumocystis jiroveci, in situations such as the one presented.
the number one cause of pneumonia in these
Answer B is incorrect. In this case, the hospital

FULL-LENGTH EXAMS
patients is Streptococcus pneumoniae (just as in
ethics committee does not need to be involved
immunocompetent patients).
because a clear surrogate decision maker was
Answer A is incorrect. Pneumonia caused by chosen when the patient had capacity.
Group B streptococcus is common among neo-
Answer C is incorrect. Decisions should
nates, not immunocompromised patients.
be made by close family members (e.g., the
Answer B is incorrect. Patients with chronic daughter), friends, or a personal physician if a
obstructive pulmonary disease have an in- living will or durable power of attorney is not
creased risk of pneumonias caused by Haemo- on le. However, the patient chose the niece
philus inuenzae, Moraxella catarrhalis, and as her legal health care proxy, and this super-
Streptococcus pneumoniae. Increased risk of H. sedes the requests of another family member
inuenzae infection is not associated with im- not designated by the patient.
munocompromise.
Answer D is incorrect. Because the patients
Answer C is incorrect. Immunocompromised chosen surrogate health care decision maker
patients have an increased risk of fungal infec- has requested that additional life-sustaining
tions such as Pneumocystis jiroveci. However, treatment, such as antibiotics, be withheld, the
overall, the number one cause of pneumonia team should act in accordance to the nieces
Test Block 7

in these patients is Streptococcus pneumoniae. wishes.


694 Section II: Full-Length Examinations Answers

14. The correct answer is A. In immunocompe- Answer B is incorrect. Myocardial ischemia


tent patients, isolated pulmonary infection with can cause chest pain, but classically presents as
coccidioidomycosis does not require treatment a dull pain radiating from the patients chest to
beyond supportive care. In immunocompro- the left arm. It can also be associated with dys-
mised patients or patients with disseminated pnea and palpitations. Diagnostic tests include
disease, the recommended rst-line therapy is ECG, cardiac enzyme levels, and, ultimately,
intravenous amphotericin B, with or without angiography.
several months of oral itraconazole following
Answer C is incorrect. This patient has peri-
the initial intravenous therapy. Sometimes itra-
carditis, not a pulmonary embolism. Although
conazole is given for many years for long-term
this patient has risk factors (family history,
suppression of the infection in immunocom-
smoking, and use of oral contraceptives) and
promised hosts.
symptoms (dyspnea, chest pain, and cough)
Answer B is incorrect. Vancomycin is not an that could be consistent with pulmonary em-
antifungal. bolism, the positional nature of her pain, her
friction rub, her slight cardiomegaly on x-ray of
Answer C is incorrect. Vancomycin is not an
the chest, and her ECG ndings suggest that
antifungal.
she has pericarditis. Thromboembolism is not
Answer D is incorrect. In this severely immu- a possible etiology of pericarditis, so it is not
nocompromised patient, oral itraconazole for the correct answer.
only 2 weeks is not effective to prevent recur-
Answer D is incorrect. Symptoms consistent
rence.
with thyroid storm include tachycardia, sys-
Answer E is incorrect. In this severely immu- tolic hypertension, delirium, and fever. This
nocompromised patient, the coccidioidomy- patients clinical picture does not suggest this
cosis infection must be treated. His immune diagnosis.
system is not capable of clearing the infection
without antifungal therapy. 16. The correct answer is C. This is the current
protocol for posttransplant vaccination. In
15. The correct answer is E. This patient has acute bone marrow transplant patients, optimal re-
FULL-LENGTH EXAMS

pericarditis, an inammatory process of the peri- sponses to vaccines cannot be reached until
cardial space. Pericarditis often presents with the immune system is reconstituted, a process
pleuritic chest pain radiating to the back, dysp- that may take up to 2 years. In the case of an
nea, pericardial friction rub, and occasionally allogeneic transplant, memory cells are elimi-
fever. Common causes of pericarditis include nated during the pretransplant conditioning
infectious, autoimmune, traumatic, and neo- protocol, and so the recipient requires a new
plastic processes. Nine of 10 cases of pericarditis primary and booster series against common
are believed to be viral or idiopathic, so it is pathogens. Immunization of close contacts is
quite likely that this patient has a viral etiology. given to prevent local spread via direct contact
Diagnostic tests included ECG, echocardiogra- and secretions.
phy, and x-ray of the chest. An enlarged, ask-
Answer A is incorrect. The vaccines in ques-
shaped cardiac silhouette is often apparent on
tion do not confer risk of reactivation infection
x-ray of the chest. Treatment is through non-
to the patient. The risk of reactivation of the
steroidal ant-inammatory drugs and, if symp-
live polio vaccine is high enough that patients
toms of tamponade appear, pericardiocentesis.
and their family members should avoid receipt
Answer A is incorrect. Although aortic dissec- of the vaccine for 24 months posttransplant.
tion can cause chest pain, this pain typically The measles-mumps-rubella vaccine confers a
Test Block 7

presents as a sudden, intense, tearing chest lower risk to transplant patients when given to
pain that may radiate to the neck and jaw. Im- family members, but should still be given with
aging is typically used to make the diagnosis caution.
and usually involves either CT or transesopha-
geal echocardiography.
Test Block 7 Answers 695

Answer B is incorrect. Reactivation concerns Answer E is incorrect. Surgery is indicated in


may be appropriate when the live polio and this patient due to the extent of the stenosis
measles-mumps-rubella vaccines are consid- despite being asymptomatic. Medical risk fac-
ered. For this reason, these vaccines should not tor management followed by surgery may be a
be given to bone marrow transplant patients rational therapeutic plan in a patient with high
for 24 months posttransplant. The vaccines in surgical risk and is therefore deemed unsafe for
question, however, are all killed or partial or- elective surgery. However, in a patient with mi-
ganisms, so reactivation is not a concern. nor comorbidities, such as this patient, surgical
risk would be low.
Answer D is incorrect. Vaccinating the patient
immediately after transplant will not offer pro-
18. The correct answer is D. Tranylcypromine is a
tection, as her immune system will not be able
monoamine oxidase (MAO) inhibitor used to
to mount an appropriate response to the vaccine.
treat refractory depression. Hypertensive crisis
Answer E is incorrect. Because the pretrans- may occur when MAO inhibitors are taken
plant protocol consists of eliminating the pa- with tyramine-rich foods such as aged cheese,
tients own marrow, including memory T and fermented meats, wine, and beer. The metabo-
B lymphocytes, vaccinating prior to the trans- lism of tyramine-containing foods is blocked by
plant will likely not confer protection posttrans- the inhibition of MAO, and the subsequent in-
plant. Additionally, cancer patients undergoing crease in circulating tyramine leads to a synap-
chemotherapy do not respond adequately to tic release of norepinephrine, resulting in a hy-
vaccinations. peradrenergic state.
Answer A is incorrect. Fluphenazine is a typi-
17. The correct answer is B. Recent double-blind,
cal antipsychotic agent and is not used to treat
randomized controlled clinical trials have
depression. It does not cause hypertensive cri-
demonstrated that patients with asymptomatic
sis when ingested with tyramine-rich foods.
carotid stenosis measuring 60% or greater who
were healthy enough to undergo surgery were Answer B is incorrect. Lamotrigine is a mood
at a reduced risk of ipsilateral cerebrovascular stabilizer and is not used to treat depression.
accident if they underwent carotid endarterec- It does not cause hypertensive crisis when in-

FULL-LENGTH EXAMS
tomy versus medical management with aspirin gested with tyramine-rich foods.
alone. Therefore, given the patients stable
Answer C is incorrect. Methylphenidate is a
condition and the nding of an asymptomatic
stimulant. It does not cause hypertensive crisis
80% carotid stenosis, this patient would most
when ingested with tyramine-rich foods.
likely benet the most from surgery.
Answer E is incorrect. Triazolam is a short-
Answer A is incorrect. Recent studies indicate
acting benzodiazepine and is not used to treat
that carotid angioplasty with stenting may play a
depression. It does not cause hypertensive cri-
role similar to carotid endarterectomy, but with
sis when ingested with tyramine-rich foods.
fewer adverse effects and less morbidity. How-
ever, angioplasty on its own has not yet been
19. The correct answer is D. Cystic brosis (CF)
shown to be superior to carotid endarterectomy.
is an autosomal recessive disease that is more
Answer C is incorrect. Medical management common in the white population. Life expec-
with aspirin has been shown to be inferior to tancy for patients with CF is 30 years, although
surgery in treating asymptomatic carotid steno- patients are now living longer due to recent ad-
sis of 60% or greater. vances in treatment. The hallmark of CF is an
abnormal exocrine gland function which in-
Answer D is incorrect. Warfarin has not been
Test Block 7

volves multiple organ systems and leads to


shown to have therapeutic value in carotid dis-
chronic respiratory infections, pancreatic insuf-
ease. In fact, it actually may be of some detri-
ciency, and musculoskeletal manifestations.
ment due the profound anticoagulation, vigi-
A defect in the CFTR (cystic brosis trans-
lant monitoring, and possible adverse effects
that accompany treatment.
696 Section II: Full-Length Examinations Answers

membrane regulator) gene results in decreased Answer C is incorrect. No further antihyper-


secretion of chloride, resulting in thicker, vis- tensive therapy is needed at this time.
cid secretions, making them hard to clear and
Answer E is incorrect. The patients hemo-
a more favorable environment for bacterial
globin A1c indicates good glycemic control on
growth. Pancreatic insufciency leads to poor
metformin alone.
weight gain in association with frequent stools
that are malodorous, greasy, and associated 21. The correct answer is C. -hCG can be de-
with atulence and colicky pain after feeding. tected in the serum 8 days after the luteinizing
Treatment of pancreatic insufciency involves hormone surge, assuming a pregnancy has oc-
the replacement of pancreatic enzymes and curred. The -hCG concentration in a normal
eating a high-calorie diet. Dietary supple- intrauterine pregnancy rises steadily until 41
ments, including fat-soluble vitamins, should days of gestation, at which time it plateaus at
also be administered to replace lost nutrients. approximately 100,000 mIU/L. The average
Answer A is incorrect. Laxatives may be ad- doubling time for -hCG levels is 2 days. A
ministered to treat intestinal obstruction in -hCG level that does not double within 2
older patients, and would be inappropriate in days suggests an at-risk pregnancy, and a de-
this case, as the patient is not obstructed. crease in -hCG levels indicates a failed preg-
nancy, whether intrauterine or extrauterine. In
Answer B is incorrect. Because pancreatic
such an event, patients should be monitored
insufciency results in poor absorption of fat-
weekly until the pregnancy has passed.
soluble vitamins A, D, E, and K, they must be
replaced with dietary supplements. However, Answer A is incorrect. A CT scan with or with-
pancreatic enzyme replacement is also neces- out contrast is not an appropriate diagnostic
sary to aid in digestion, which is required for tool in assessing the presence of an intrauter-
adequate nutrient absorption in the gut. ine or extrauterine pregnancy.
Answer C is incorrect. Patients with pancre- Answer B is incorrect. Dilation and curet-
atic insufciency are encouraged to maintain tage is not used to investigate the presence of
a higher-than-normal-calorie diet, not lower, an ectopic pregnancy, due to the potential for
FULL-LENGTH EXAMS

with unrestricted fat intake due to their pro- disruption of a viable intrauterine pregnancy.
pensity for malabsorption and failure to thrive. In this patient with a failed pregnancy, dilation
and curettage may be indicated in the future if
Answer E is incorrect. Even when a patient
she fails to pass the products of conception. At
has signicant steatorrhea, fat is not withheld
this point, however, it is inappropriate.
from the diet. They are actually encouraged to
have an unrestricted fat intake once pancreatic Answer D is incorrect. Laparoscopy or laparo-
enzyme supplementation has been instituted. tomy is not indicated in this patient, as there is
evidence that her pregnancy is failing. Surgical
20. The correct answer is D. Lisinopril is an ap- interventions are indicated in a patient with a
propriate addition at this time given the pa- rising -hCG level and a documented adnexal
tients microalbuminuria and history of both mass on transvaginal ultrasound.
hypertension and type 2 diabetes mellitus. An-
Answer E is incorrect. The gestational sac is
giotensin-converting enzyme inhibitors have
generally evident on transvaginal ultrasound
been demonstrated to delay nephropathic pro-
in a viable pregnancy when the serum -hCG
gression in patients with diabetes and microal-
level is >1500 mIU/mL (4.55 weeks of gesta-
buminuria.
tion). A negative transvaginal ultrasound in a
Answer A is incorrect. Adding an additional patient with a -hCG level <1500 mU/mL is
Test Block 7

diuretic is not necessary given the degree of nondiagnostic and should be repeated after
hypertensive control his current regimen has 72 hours as long as the serum -hCG level
achieved. continues to rise. A repeat transvaginal ultra-
sound, however, is not indicated when the se-
Answer B is incorrect. There is no information
given regarding the patients lipid status.
Test Block 7 Answers 697

rum -hCG level is falling, as this is a sign of a Answer A is incorrect. A low negative predic-
failed pregnancy. tive value would indicate that a negative result
did not often indicate that the disease could be
22. The correct answer is E. The patient is pre- ruled out in the patient and would suggest the
senting with thyrotoxic shock, or thyroid storm. test was not as useful.
This is a complication of thyrotoxicosis that is
Answer B is incorrect. A low positive predic-
usually preceded by infection, trauma, surgery,
tive value would indicate that a positive result
or diabetic ketoacidosis. Patients typically pre-
did not necessarily indicate that the patient
sent with jaundice, nausea and vomiting, diar-
had the disease and would suggest the test was
rhea, seizure, delirium, fever, tachycardia, and
not as useful.
hypertension. If untreated, thyrotoxic shock
can be rapidly fatal, and even with treatment, Answer C is incorrect. The test is independent
mortality is as high as 30%. Mortality is usually of disease incidence, which is the occurrence
due to cardiac failure, arrhythmias, and hyper- of new cases in a dened interval of time. For
thermia. -Blockers such as propranolol are screening purposes, a highly sensitive test is
mainstays in the initial management of pa- preferred for the purpose of ruling out disease.
tients with thyrotoxic shock.
Answer D is incorrect. The test is independent
Answer A is incorrect. Dobutamine is a of disease prevalence, which is the occurrence
-agonist that would worsen this patients con- of a disorder or trait in a given population. For
dition. It would further increase her heart rate screening purposes, a highly sensitive test is
and increase her risk of developing high-output preferred for the purpose of ruling out disease.
cardiac failure. -Antagonists are the preferred
therapy. 24. The correct answer is A. This patients symp-
toms are consistent with aseptic meningitis,
Answer B is incorrect. Enalapril is an angio-
which can be caused by any number of viruses.
tensin-converting enzyme inhibitor that is used
However, given this patients HIV risk factor
in the management of hypertension, renal dis-
(intravenous drug use), the diagnosis of acute
ease, and congestive heart failure. It is not used
HIV syndrome should be high on the differen-
in acutely managing thyroid storm.

FULL-LENGTH EXAMS
tial. Acute HIV syndrome usually occurs 23
Answer C is incorrect. Levothyroxine would weeks after the initial exposure. Given the in-
worsen this patients condition signicantly. All determinate antibody test (which is possible
attempts should be made to reduce levels of because it takes many weeks to mount an anti-
circulating thyroid hormone. body response to the virus), a viral load would
be diagnostic. In acute HIV infection, the viral
Answer D is incorrect. Methimazole is a thi-
load is extremely high (up to 100,000 RNA
onamide that decreases synthesis of thyroid
copies/mL).
hormone. Although methimazole helps to de-
crease production of new thyroid hormone, Answer B is incorrect. An echocardiogram
it has no effect on the release of already syn- would be helpful in diagnosing valvular dis-
thesized hormone or the level of circulating ease, wall motion abnormalities, endocarditis,
hormone. A better choice is propylthiouracil, or thrombi in one of the chambers of the heart.
another thionamide that in addition to the ac- However, because there are no focal neuro-
tions of methimazole, also blocks the periph- logic signs in this patient, stroke is not high on
eral conversion of thyroxine to triiodothyro- the differential. Therefore, there is no need to
nine. look for a source of emboli.
Answer C is incorrect. Given the low RBC
Test Block 7

23. The correct answer is E. Highly sensitive tests


count in the cerebrospinal uid from the origi-
are used to rule out disease and have a low
nal lumbar puncture, this was not a traumatic
false-negative rate, so if the patient has a nega-
tap; there is no reason to believe that the rst
tive result one can rule out disease in the pa-
lumbar puncture was inaccurate.
tient. A negative PPD test is highly sensitive for
ruling out tuberculosis.
698 Section II: Full-Length Examinations Answers

Answer D is incorrect. Starting an HIV- 26. The correct answer is E. This case is an exam-
positive patient on antiretroviral therapy is ple of factitious disorder by proxy, more com-
decided based on the CD4+ cell count, viral monly known as Munchausens syndrome by
load, and positive symptoms of the disease. Pa- proxy. In this disorder, caregivers deliberately
tients should begin a regimen if they have a exaggerate, fabricate, and/or induce physical
CD4+ cell count <200/mm, high viral load and psychological problems in others. In most
(>100,000/mL), fall in CD4+ cell count >100/ cases, the perpetrators of Munchausens by
mm per year, or symptoms such as thrush, proxy will continue their behavior even after
wasting, or an AIDS diagnosis. they have been discovered. Factitious disorder
must be differentiated from malingering be-
Answer E is incorrect. Intravenous ceftriaxone
cause factitious disorder has a goal of primary,
and vancomycin are the prophylactic treat-
or psychological gain. Malingering means the
ments of choice for bacterial meningitis. Since
perpetrator has a secondary goal (e.g., food,
there was a lymphocytic predominance and no
shelter, money).
organisms were seen on Gram stain, a bacte-
rial meningitis is unlikely. Therefore, empiric Answer A is incorrect. In many cases of
antibiotics are not indicated in this case. Munchausen syndrome by proxy, there is no
history of investigation by child protective ser-
25. The correct answer is C. The patient likely has vices.
gastroesophageal reux disease, which often pres-
Answer B is incorrect. Unfortunately, many
ents with heartburn, regurgitation, and dysphagia
perpetrators of Munchausens syndrome by
that is typically worse with meals and when lying
proxy appear psychologically healthy.
supine. Mild symptomatic disease can usually be
managed empirically; lifestyle and dietary modi- Answer C is incorrect. Many perpetrators of
cations along with antacids and nonprescription Munchausens syndrome by proxy doctor
H2-receptor antagonists are usually sufcient. shop, or move from doctor to doctor, when
Lifestyle modications include cessation of smok- physicians become suspicious.
ing, sleeping with the head of bed elevated, avoid-
Answer D is incorrect. Mothers who are later
ing excessive alcohol, and avoidance of late
diagnosed with Munchausens syndrome by
FULL-LENGTH EXAMS

meals. Patients with debilitating symptoms usu-


proxy often give the semblance of being the
ally require more pharmacologic acid-suppressive
ideal mother. It often appears that they are
therapy or antireux surgery.
taking excellent, dutiful care of their sick child,
Answer A is incorrect. Avoiding spicy foods when it is they who are inducing the illness.
has not been shown to be effective in treating
gastroesophageal reux disease. 27. The correct answer is B. This patient presents
with signs and symptoms suggestive of tempo-
Answer B is incorrect. H2-receptor blockers,
ral, or giant cell, arteritis. Patients are usually
while providing symptomatic relief, may be
at least 50 years old. Clinical presentation of-
unnecessary given the probable improvement
ten includes headache that is unilateral and lo-
in symptoms with lifestyle modications. How-
calized to the temporal or frontal regions. The
ever, they can be effective as supplementary
headache of giant cell arteritis has no pathog-
therapy to lifestyle modication.
nomonic features, but typically the headache is
Answer D is incorrect. This surgical proce- either new in a patient without a history of
dure has been shown to prevent active reux headaches or of a new type in a patient with a
and further damage to the esophagus; however, history of chronic headaches. The temporal ar-
it is reserved for cases that have been refractory tery may appear enlarged and tender to palpa-
Test Block 7

to maximal medical management, which this tion. A bruit may be auscultated over the tem-
patient has not attempted yet. poral or carotid artery. Patients may complain
of jaw or arm claudication and visual changes.
Answer E is incorrect. Proton pump inhibitors
Without appropriate therapy, temporal arteritis
should be used in cases that are refractory to
can lead to ischemic optic neuropathy and
lifestyle modications.
blindness. Patients can also have systemic
Test Block 7 Answers 699

symptoms including fever, weight loss, and fa- a granular ground-glass material in the lower
tigue. A key laboratory nding is an elevated central abdomen. Meconium ileus occurs pri-
erythrocyte sedimentation rate, often >100 marily in newborn infants with CF, occurring
mm/hr. The diagnosis of temporal arteritis in approximately 10% of infants with the con-
should be considered in patients at 50 years old dition. Up to 90% of infants presenting with
or older presenting with fever, anemia, and el- meconium ileus have CF, and it is considered
evated erythrocyte sedimentation rate. The di- pathognomonic of CF unless proven other-
agnosis is conrmed by biopsy of the temporal wise.
artery as soon as possible. However, initiation
Answer B is incorrect. Edwards syndrome
of therapy with glucocorticoids should not be
is a chromosomal trisomy caused by meiotic
delayed while biopsy results are pending, as the
nondisjunction of chromosome 18. The char-
risk of permanent vision loss is high.
acteristic clinical ndings in Edwards syn-
Answer A is incorrect. Antineutrophil cyto- drome include low birth weight, short sternum,
plasmic antibody is found in patients with We- rocker-bottom feet, microcephaly with promi-
geners granulomatosis, another type of vasculi- nent occiput, micrognathia, mental retarda-
tis that affects medium- and small-sized vessels, tion, cardiac and renal anomalies, and a distin-
among other disorders. Wegeners classically af- guishing hand positioning of closed sts with
fects the lungs and kidneys. Pulmonary symp- the index nger overlapping the third digit and
toms include cough, dyspnea, and hemoptysis. the fth digit overlapping the fourth. Gastroin-
Renal involvement is usually a pauci-immune testinal abnormalities, specically meconium
glomerulonephritis. ileus, are not commonly associated with Ed-
wards syndrome.
Answer C is incorrect. Nonsteroidal anti-in-
ammatory drugs may be used to abort migraine Answer C is incorrect. A Meckels diverticu-
or tension headaches. Migraines may be unilat- lum is a true diverticulum. It is dened as
eral and cause visual disturbances. However, in a remnant of the embryonic yolk sac and is
a patient 50 years old or older presenting with the most frequent congenital gastrointestinal
temporal tenderness, jaw claudication, consti- anomaly. Symptoms usually present during the
tutional symptoms, and a signicantly elevated rst 2 years of life, but primary symptoms gen-

FULL-LENGTH EXAMS
erythrocyte sedimentation rate, temporal arteri- erally manifest within the rst decade. Most
tis should be highest on the differential. symptomatic patients have diverticuli that are
lined by an ectopic mucosa that is acid secret-
Answer D is incorrect. Magnetic resonance an-
ing and can cause intermittent painless rectal
giography may show large-artery involvement,
bleeding by way of ulceration of the adjacent
but temporal artery biopsy is considered the
normal ileal mucosa. Episodes of bowel ob-
gold standard for diagnosing temporal arteritis.
struction with Meckels diverticulum have
Answer E is incorrect. Temporal artery biopsy been described but are relatively rare. Meco-
is the gold standard for diagnosing temporal ar- nium ileus is not associated with Meckels di-
teritis. However, sight-preserving management verticulum.
should not be delayed in patients who present
Answer D is incorrect. Trisomy 21 (Downs
with the clinical signs and symptoms of tempo-
syndrome) is caused by meiotic nondisjunction
ral arteritis. Following steroid induction, tem-
or a translocation of chromosome 21. It most
poral artery biopsy would be the next step in
commonly presents clinically with hypotonia,
management.
at face, upward and slanted palpebral s-
sures and epicanthic folds, mental retardation,
28. The correct answer is A. Meconium ileus, as
growth failure of varying degrees, pelvic dys-
Test Block 7

described in this vignette, often presents in the


plasia, cardiac malformations, speckled irises
rst 2448 hours of life as abdominal disten-
(Brusheld spots), and simian crease of the
tion, emesis, and failure to pass meconium
palms. Intestinal atresia can be seen in patients
(the rst stool after birth). Abdominal radio-
with Downs syndrome, but meconium ileus is
graphs typically show loops of dilated bowel
uncommon.
with air-uid levels and frequently demonstrate
700 Section II: Full-Length Examinations Answers

Answer E is incorrect. Williams syndrome Answer B is incorrect. Positive predictive value


is caused by a microdeletion of 7q11.23 with is dependent on the sensitivity and specicity
clinical manifestations, including round face of the test, as well as the prevalence of the dis-
with full lips and cheeks; iris with stellate pat- ease in the population being tested. The pair
tern; strabismus; variable degrees of mental of tests used for HIV testing collectively have
retardation; cardiac malformations, especially very high sensitivity and specicity. Because
supravalvular aortic stenosis; and an extremely this patient falls into a high-risk population for
friendly personality. Gastrointestinal com- HIV infection, the positive predictive value of
plaints, specically meconium ileus, are not the test in this circumstance is higher than it
associated with Williams syndrome. would be for a lower-risk subject.
Answer C is incorrect. High sensitivity in-
29. The correct answer is C. Kussmaul breathing
creases the negative predictive value of a test,
is the term for the deep, rapid breathing often
allowing one to rule out a disease if the result
seen in diabetic ketoacidosis (DKA). It is the
is negative. This would be relevant if the pa-
physiologic response to extreme acidosis as the
tient was asking about a negative test result and
body tries to eliminate excess acid in the form
whether he might still have the disease. The
of expired carbon dioxide. Because nonketotic
HIV screening test (enzyme-linked immuno-
hyperglycemia (NKH) does not result in the
sorbent assay) has high sensitivity for infection
formation of ketones or the onset of acidosis,
of more than a months duration.
Kussmaul breathing is not normally seen in pa-
tients suffering from NKH. Fruity, acetone
31. The correct answer is A. This patient is pre-
breath is the result of the acidosis and is an-
senting with the classic signs of a gestational
other distinguishing characteristic of DKA.
trophoblastic tumor from either a complete or
Answer A is incorrect. Dehydration often pre- partial hydatidiform mole. Gestational tropho-
cipitates both DKA and NKH. Because glucose blastic disease is a rare, highly treatable condi-
is an osmotic diuretic, patients with extreme tion in which cancer cells grow in the tropho-
hyperglycemia will be dehydrated regardless of blast (tissue formed immediately after
the cause. fertilization that eventually forms the pla-
FULL-LENGTH EXAMS

centa). Typically, this disease affects primi-


Answer B is incorrect. Hyperglycemia is a hall-
gravid patients at the extremes of maternal age
mark of both DKA and NKH, although DKA
and presents with vaginal bleeding as the tu-
has a concomitant acidosis, whereas NKH does
mor separates from the underlying decidua.
not.
With continued growth of the tumor, pressure
Answer D is incorrect. Mental status changes, on the uterus results in pelvic pain. In addi-
to varying degrees, are commonly seen in both tion, the uterus will be larger than expected
DKA and NKH. due to the growth of the tumor. Regardless of
Answer E is incorrect. Vomiting is a common whether this mole is partial or complete,
symptom seen in both DKA and NKH. -hCG levels will be higher than normal, re-
sulting in extreme nausea. Serial -hCG mea-
30. The correct answer is D. Specicity allows surements must be monitored after treatment
one to rule in a disease with condence. Speci- to ensure all parts of the hydatidiform mole
city is dened as: true-negatives / (true-nega- have been removed.
tives + false-positives). The conrmatory step Answer B is incorrect. Secondary to continu-
of the HIV test, a Western blot test for viral pro- ous vaginal bleeding, patients presenting with
teins, is known to have high specicity, and molar pregnancies are typically anemic.
Test Block 7

thus a low chance of being incorrect.


Answer C is incorrect. The -subunit of
Answer A is incorrect. Other diseases can hCG is the same as the -subunit for thyroid-
cause false-positive HIV tests, decreasing the stimulating hormone. As a result, the elevated
validity of the test.
Test Block 7 Answers 701

-hCG stimulates the thyroid gland to produce Answer B is incorrect. Lisinopril is an angio-
a hyperthyroid state. Therefore, thyroid-stimu- tensin-converting enzyme inhibitor that is
lating hormone levels will be decreased. used to treat hypertension. This class of drug
is thought to prevent progression of microal-
Answer D is incorrect. Ovarian cysts arising
buminuria to proteinuria in patients with dia-
in the normal process of ovulation are called
betes. However, an angiotensin-converting
functional or theca-lutein cysts. These cysts
enzyme inhibitor would not help the mans
can be stimulated by gonadotropins, includ-
urinary symptoms.
ing follicle-stimulating hormone and -hCG.
Because of higher than normal -hCG levels, Answer C is incorrect. The man has hyperten-
multilocular theca lutein cysts are classically sion and BPH, and he needs treatment, as long
seen in patients with gestational trophoblastic as there are no contraindications to starting
disease. therapy.
Answer E is incorrect. Because the tumor is Answer E is incorrect. Thiazides are popular
located within the uterus, a pelvic ultrasound as rst-line agents in treating hypertension, but
will show an enlarged uterus. An ultrasound in this case terazosin is preferred because it can
will also show a lack of fetal tissue and amni- simultaneously treat the patients BPH.
otic uid.
33. The correct answer is D. Acromegaly is caused
32. The correct answer is D. Terazosin, an by excessive release of growth hormone, most
-blocker, should be the rst-line treatment in commonly due to an adenoma in the anterior
this patient. Based on his symptoms and physi- pituitary. Because growth hormone is secreted
cal exam, he has benign prostatic hyperplasia in a pulsatile fashion, measuring serum levels
(BPH). He also has hypertension that has been is not a reliable method of diagnosing acro-
noted on three separate occasions despite a megaly. Instead, serum insulin-like growth fac-
healthy low-salt diet and exercise. At this point, tor-1, a growth factor secreted by the liver un-
terazosin monotherapy, which can address der growth hormone stimulation, is measured
both his hypertension and BPH, is indicated. to conrm the diagnosis. Octreotide is a soma-
The prostate and base of the bladder contain tostatin analogue that counteracts growth hor-

FULL-LENGTH EXAMS
1-adrenoreceptors. The prostate contracts mone hypersecretion. It is often used for im-
when these receptors are stimulated by an ago- mediate relief of symptoms or for preoperatively
nist. Terazosin blocks effects of the 1a subtype shrinking large macroadenomas.
in the prostate, improving the symptoms and
Answer A is incorrect. Cortisol is used to treat
signs of BPH. These receptors are the same as
patients with glucocorticoid deciency due to
those found in the vascular system, where the
adrenal insufciency or pituitary deciency
postsynaptic 1-adrenergic receptors cause pe-
of ACTH. Although a pituitary adenoma can
ripheral constriction when stimulated. Thus,
decrease secretion of ACTH, the patient is not
pharmacologic blockade causes a decrease in
currently presenting with ndings consistent
systemic blood pressure. Because terazosin is a
with low cortisol. Therefore cortisol would not
selective blocker and does not have any ef-
be the primary therapy.
fect on presynaptic 2-receptors, it will not
cause reex tachycardia. Answer B is incorrect. Acromegaly is due to
excessive secretion of growth hormone, not its
Answer A is incorrect. Finasteride is a medica-
deciency. Therefore administration of growth
tion that has a role in treating BPH. However,
hormone would worsen his signs and symp-
it has no role in the treatment of hypertension;
toms.
terazosin is a better choice. Finasteride works
Test Block 7

by inhibiting the enzyme 5-reductase, which Answer C is incorrect. Metyrapone is an an-


converts testosterone to dihydrotesterone. tagonist of cortisol production that is used to
treat patients with Cushings disease due to an
ACTH-producing pituitary adenoma.
702 Section II: Full-Length Examinations Answers

Answer E is incorrect. Thyroid hormone is 35. The correct answer is B. Thiazides can cause
used to treat patients with hypothyroidism due hyperlipidemia with an increase in both total
to intrinsic thyroid disorders or hypopituitar- and LDL cholesterol.
ism. Although a pituitary adenoma can sup-
Answer A is incorrect. Hypokalemia (not
press secretion of other hormones in the an-
hyperkalemia) with thiazide diuretics is a
terior pituitary, including thyroid-stimulating
common early complication of treatment.
hormone, the patient is not currently present-
Increased sodium and water delivery to the
ing with signs of hypothyroidism. His imme-
aldosterone-sensitive potassium secretory site
diate problem is the excessive level of growth
in the collecting tubules, as well as increased
hormone, so thyroid hormone would not be
aldosterone secretion in response to volume
considered primary therapy. Thyroid levels
depletion, leads to increased potassium loss.
should be checked, however.
Answer C is incorrect. Hyponatremia (not hy-
34. The correct answer is E. Patients with long- pernatremia) is a rare but potentially fatal com-
standing rheumatoid arthritis can develop lung plication of several types of diuretic therapy,
manifestations of the disease, including pleural but most severe cases are secondary to thiazide
diseases such as effusions, empyemas, necrosis, diuretic use.
or cavitations. This decubitus x-ray of the chest
Answer D is incorrect. Thiazide diuretics
shows a pleural effusion with layering of the
sometimes result in hypercalcemia secondary
uid. Effusions greater than 1 cm in the decu-
to active calcium resorption in the distal tu-
bitus view are usually large enough to be safely
bule.
sampled. While it is not necessarily an indica-
tion to absolutely do a thoracentesis, in this Answer E is incorrect. Hyperuricemia (not
case thoracentesis is both diagnostic and thera- hypouricemia) is a relatively common compli-
peutic. To aid in determining the cause of the cation of thiazide diuretics, which cause de-
effusion, one can use the Lights criteria to dis- creased excretion by increasing net reabsorp-
tinguish between a transudate and an exudate. tion of urate.
Answer A is incorrect. Furosemide would be
36. The correct answer is C. The patient has a
FULL-LENGTH EXAMS

indicated for congestive heart failure contribut-


type I renal tubular acidosis (RTA) that was
ing to pleural effusion. However, the patients
conrmed by challenging her with NH4Cl. Pa-
symptoms are not consistent with congestive
tients with a distal RTA cannot make the urine
heart failure.
pH <5.5. As a result, they are at risk for cal-
Answer B is incorrect. A likely cause of this cium phosphate stones. The antibodies that
patients effusion is rheumatoid arthritis, which would reveal a cause of this distal RTA would
is suggested by her morning stiffness. However, be anti-Ro/SSA and anti-La/SSB. These anti-
further work-up is indicated before that diagno- bodies are found in high frequency in patients
sis can be conrmed and treatment started. with Sjgrens syndrome. The patients dry mu-
cous membranes and use of eye drops could
Answer C is incorrect. Parapneumonic effu-
also suggest this as a diagnosis and cause of the
sions occur with bacterial pneumonias; how-
RTA. Patients with Sjgrens may present with
ever, these often do not resolve with antibiot-
a distal RTA and no other clinical ndings.
ics alone and require drainage via a chest tube.
Additionally, this patient has no evidence of in- Answer A is incorrect. Antiglomerular base-
fection to make her a candidate for antibiotics. ment membrane antibody is associated with
Goodpastures syndrome. The effect on the
Answer D is incorrect. Any new effusion with
kidney is a rapidly progressive glomerulone-
Test Block 7

no clear etiology (e.g., congestive heart failure


phritis, which can cause renal failure. The pa-
or pneumonia) should be tapped and analyzed
tient is not described as having renal failure,
for possible oncologic or rheumatologic etiolo-
nor does this disease classically present with
gies.
kidney stones.
Test Block 7 Answers 703

Answer B is incorrect. Antihistone antibody tralateral disturbance of pain/temperature,


is seen in drug-induced systemic lupus ery- dysphagia/dysarthria/hiccup, and ipsilateral
thematosus. Because the patient is not taking Horners syndrome.
any medication, this diagnosis is unlikely. The
drugs that are commonly implicated include 38. The correct answer is B. This patient is manic
procainamide, hydralazine, phenytoin, sulfon- and has a history of psychosis. The Tarasoff de-
amides, and isoniazid. cision states that if a patient presents a serious,
credible danger to another person, physicians
Answer D is incorrect. Circulating anti-neu-
have an obligation to protect the threatened
trophilic cytoplasmic autoantibodies are found
party. In this case it would be reasonable to dis-
in patients with Wegners granulomatosis. This
close to police that the patient is psychotic and
disorder is more common in men. These pa-
has made specic threats to harm the young
tients may present with fever, fatigue, and
woman. Until the patient is hospitalized and
malaise. The upper airway is also involved, in-
treated for mania, he may pose a serious threat
cluding nasal and sinus discomfort and upper
to her.
respiratory infections.
Answer A is incorrect. The physician must
Answer E is incorrect. Perinuclear anti-neutro-
take reasonable steps to protect the threatened
philic cytoplasmic autoantibodies antibody are
party; asking the patients roommate to control
found in patients with microscopic polyangi-
a potentially manic and psychotic person is
itis. This condition is not associated with distal
not a reasonable measure under these circum-
RTA.
stances.
37. The correct answer is B. Although there are Answer C is incorrect. The physician must
other possible causes for this mans symptoms, take a more active role than that described in
an ischemic stroke of the anterior inferior cere- this answer choice. He may contact the threat-
bellar artery is most likely. Strokes in this distri- ened party to inform them of the patients
bution typically present with gaze palsy, deaf- statements, but he must also contact emer-
ness, tinnitus, and ipsilateral facial weakness. gency medical services to take the patient into
Unlike strokes in the posterior inferior cerebel- custody. As a physician, his opinion would ex-

FULL-LENGTH EXAMS
lar artery, there is no Horners syndrome, dys- pedite the apprehension of the patient.
phagia, or dysarthria. The anterior inferior cer-
Answer D is incorrect. The Tarasoff decision
ebellar artery is a branch of the basilar artery.
states that it is ethically sound and appropriate
Answer A is incorrect. Anterior cerebral artery to override patient condentiality in cases of
strokes present with leg paresis, amnesia, per- potential harm to third parties.
sonality changes, foot drop, gait dysfunction, or
Answer E is incorrect. The physician cannot
cognitive changes.
rely on previous behavior or actions of the pa-
Answer C is incorrect. Middle cerebral artery tient; he must act on the current information
strokes typically present with aphasia (domi- and take steps to protect the threatened party.
nant hemisphere), neglect (nondominant
hemisphere) contralateral hemiparesis, gaze 39. The correct answer is D. For the patient with
preference, or homonymous hemianopsia. mental status changes, it is important to rule
out life-threatening causes such as intracranial
Answer D is incorrect. Posterior cerebral ar-
hemorrhage and meningoencephalitis. In this
tery strokes present with homonymous hemi-
case, there is a low suspicion of both due to the
anopsia, memory decits, or dyslexia/alexia.
history negative for trauma, the negative CT,
Test Block 7

Answer E is incorrect. Posterior inferior cer- and a lack of signs of infection. Mass lesions
ebellar artery strokes present with sudden onset causing herniation are also ruled out by CT.
nausea/vomiting, vertigo, hoarseness, ataxia, Metabolic causes of mental status change
ipsilateral palate and tongue weakness, con- should always be considered. Besides a low
704 Section II: Full-Length Examinations Answers

platelet count and hemolytic anemia, throm- aerobic coverage of organisms that typically
botic thrombocytopenic purpura can also pres- colonize the pelvis and genital tract and should
ent with change in mental status, which can not be changed. The patients persistent fever
range from mild confusion to coma. The etiol- spikes are not a sign of antibiotic failure, but
ogy of the mental status change is thrombotic rather are due to the production of clot, which
microvascular occlusion within the brain pa- necessitates heparin administration.
renchyma. Although thrombotic thrombocy-
Answer D is incorrect. Heparin alone will not
topenic purpura can be associated with medi-
adequately treat ovarian vein thrombosis be-
cations (e.g., cyclosporine or antiplatelet
cause the ovarian vein and clot are generally
drugs), most cases are idiopathic.
infected.
Answer A is incorrect. Diuretics are not asso-
Answer E is incorrect. Surgery is not needed
ciated with mental status changes. Even if the
at this time. Initial therapy is broad-spectrum
patient had taken enough to cause volume de-
antibiotics and heparin. A rapid response to
pletion and hypotension, there would be other
treatment is not expected and may take up to
electrolyte disturbances.
7 days. Thromboses generally resolve with an-
Answer B is incorrect. Hemolytic anemia can tibiotics and heparin. Surgery is only rarely
be precipitated by medications such as penicil- indicated in cases of inadequate response to
lin or quinidine but would not be associated therapy.
with mental status changes.
41. The correct answer is E. This patient is in
Answer C is incorrect. Liver failure may lead
shock due to urosepsis. As aggressive uid re-
to uremia and encephalopathy (the hepato-
suscitation has failed to bring the mean arterial
renal syndrome). However, in the absence of
pressure up to acceptable levels, and because
elevated aspartate aminotransferase and ala-
the administration of further uid infusions
nine aminotransferase and with a normal albu-
would render the patient vulnerable to pulmo-
min level, liver failure is unlikely.
nary edema, a pressor would be the next treat-
Answer E is incorrect. Viral meningoencepha- ment. Of the medications noted, norepineph-
litis is unlikely due to the absence of a fever, rine is the appropriate choice because it
FULL-LENGTH EXAMS

normal WBC count, and absence of menin- specically targets the cause of the low pres-
geal signs. sure, namely the relaxation of peripheral blood
vessels due to sepsis. It is a predominantly
40. The correct answer is C. Septic ovarian vein -adrenergic agonist, and as such will constrict
thrombosis is a rare complication of delivery, peripheral blood vessels.
occurring in <1 in 500 deliveries. It generally
Answer A is incorrect. Dobutamine, which
occurs as a consequence of a pelvic infection
is predominantly a positive inotropic agent,
and is the result of intimal injury in the setting
would not be as useful as norepinephrine, as
of a hypercoagulable state. Treatment includes
it does not address the major cause of the hy-
antibiotics and heparin. The current antibiotic
potension. However, if the patient achieves an
regimen is appropriate and does not need to be
appropriate mean arterial pressure but cannot
changed. Typically, patients retain normal
maintain an appropriate cardiac output, dobu-
function in the affected ovary. Patients should
tamine is the subsequent medication of choice.
be counseled that the recurrence of septic
ovarian vein thrombosis is rare. Answer B is incorrect. Epinephrine, which
has predominantly either - or -adrenergic ef-
Answer A is incorrect. No further -lactamase
fects, typically has no role as a pressor in sep-
coverage is needed. The combination of clin-
Test Block 7

tic shock, although it may be used in patients


damycin and gentamicin provide adequate
unresponsive to other agents. Its use in septic
coverage.
shock may cause the patient to experience
Answer B is incorrect. The current antibiotic myocardial ischemia, arrhythmias, and de-
regimen provides appropriate aerobic and an- creased splanchnic blood ow.
Test Block 7 Answers 705

Answer C is incorrect. Isoproterenol, a nonse- Without a further history of psychotic symptoms,


lective -agonist, could actually worsen the re- this diagnosis would be difcult to support.
laxation of the peripheral vasculature due to its
-adrenergic effects. 43. The correct answer is D. The patients presen-
tation is consistent with obstructive sleep apnea
Answer D is incorrect. Labetalol is a com-
(OSA) superimposed on a known diagnosis of
bined and sympatholytic and is used in the
congestive heart failure. While polysomnogra-
treatment of hypertensive urgency.
phy is necessary for denitive diagnosis of OSA,
the ndings in this case are still typical of OSA:
42. The correct answer is A. Adjustment disorder
a man 3060 years old, with a history of noctur-
is the development of emotional or behavioral
nal choking, daytime sleepiness, witnessed ap-
symptoms in response to an identiable
neic episodes, mild hypertension, and moderate
stressor(s) occurring within 3 months of the
obesity. OSA can adversely affect left ventricular
onset of the stressor(s), and disappearing within
(LV) function in patients with congestive heart
6 months of the resolution of the stressor.
failure. A compromised LV decreases cardiac
These symptoms or behaviors are clinically sig-
output and results in exertional fatigue and
nicant as evidenced by marked distress that is
weakness. This complication results from a
in excess of what would be expected from ex-
number of causes. During each obstructive ap-
posure to the stressor or signicant impairment
neic event, negative intrathoracic pressure in-
in social or occupational functioning. The pa-
creases, thereby increasing LV afterload and ad-
tient meets this description.
versely affecting LV function. In addition,
Answer B is incorrect. Major depressive dis- recurrent hypoxia and chronically elevated sym-
order requires that a patient has two major pathoadrenal activity in patients with OSA can
depressive episodes, separated by at least 2 also precipitate LV dysfunction in patients with
months, within a 1-year period. This patient congestive heart failure. Treatment of OSA in
does not meet criteria for major depressive epi- these patients often results in improvement of
sodes. LV function.
Answer C is incorrect. A major depressive epi- Answer A is incorrect. LV afterload (the load

FULL-LENGTH EXAMS
sode requires ve of the following: insomnia, against which the LV ejects after the opening
decreased interest in pleasurable activities, in- of the aortic valve) increases, not decreases,
creased guilt, decreased energy, decreased con- during apneic events. The increase in afterload
centration, decreased appetite, psychomotor is a result of an increase in negative intratho-
retardation, and suicidal ideation. Also, these racic pressure, which provides further resis-
symptoms must last for at least 2 weeks. This tance against which the heart must pump.
patient does not meet ve of these criteria.
Answer B is incorrect. Right ventricular after-
Moreover, her symptoms are better accounted
load increases, not decreases, during apneic
for by an inability to adjust to her husbands ill-
events. The increase in afterload is a result of
ness.
an increase in negative intrathoracic pressure
Answer D is incorrect. Normal grief does not and pulmonary vasoconstriction.
include being preoccupied to the point that
Answer C is incorrect. While hypertension is a
one withdraws from all contact with loved ones
cause of systolic dysfunction, it is not the most
and friends, and that one is unable to tend to
likely cause in this patient. Given his normal
issues such as bills. These characteristics are
blood pressure with medications, this patients
what separates adjustment disorder from nor-
hypertension is under control.
mal grief.
Test Block 7

Answer E is incorrect. During apneic events,


Answer E is incorrect. During an adjustment
there is a decrease in the amount of oxygen
disorder or grief, it is common for patients to
reaching the lungs. Physiologically, pulmo-
see and talk to deceased or sick loved ones.
nary vasoconstriction, not vasodilation, occurs.
Clinically, pulmonary hypertension and right
706 Section II: Full-Length Examinations Answers

heart failure are consequences of prolonged his transurethral resection of the prostate was
pulmonary vasoconstriction. At that point, the successful, but during the procedure he may
patient might display symptoms and signs such have sustained an injury to his urethra. A stric-
as shortness of breath (nonexertional), jugular ture developed, causing the man to have dif-
venous distension, and peripheral edema. culty voiding completely. This residual urine
makes the man feel like he has to urinate
44. The correct answer is A. The medial aspect of shortly after the last urination.
the forth and fth digits, as well as medial as-
pect of the hand (both the volar and dorsal sur- 46. The correct answer is J. Multiple sclerosis is
face), is supplied by the ulnar nerve. This an autoimmune disorder resulting in waxing
nerve can be injured in a fracture of the me- and waning neurologic abnormalities. Plaques
dial condyle of the humerus. These patients in the sacral spinal cord can result in loss of
can present with weakness of the hand intrin- bladder tone, causing urinary retention and
sic muscles because the ulnar nerve supplies overow incontinence. Incontinence can be
many of these muscles. For example, testing prevented by the practice of intermittent self-
abduction of the second and third digits against catheterization to empty the bladder. However,
resistance tests the dorsal interossei, which are catheter insertion can introduce bacteria into a
supplied by the ulnar nerve. normally sterile environment. In this case,
however, the patients bacteriuria is asymptom-
Answer B is incorrect. Extension of the fth
atic and not responsible for an increase in uri-
digit is supplied by the radial nerve and one of
nary frequency.
its terminal branches, the posterior interosseus
nerve. These nerves do not supply the area in- Answer A is incorrect. Acute cystitis would
volved in this case. present with symptoms of pain or burning on
urination in addition to urinary frequency. Pa-
Answer C is incorrect. Wrist extension is sup-
tients are likely to be febrile and have an ele-
plied by the radial nerve, which can be dam-
vated WBC count.
aged in a humeral shaft fracture. These pa-
tients would have decreased sensation on the Answer B is incorrect. Patients with neuro-
dorsal surface of their hand. genic bladders can practice clean, intermit-
FULL-LENGTH EXAMS

tent self-catheterization to prevent overow


Answer D is incorrect. The thumb is mostly
incontinence. However, catheter placement
supplied by the median nerve, which can be
may result in the introduction of bacteria into
injured in a supracondylar humerus fracture.
a sterile environment. In this case, the patient
One would also expect sensation changes over
is colonized with bacteria but is asymptomatic.
the palmar aspect of the rst, second, and third
Therefore, overow incontinence secondary
digits, and the lateral palm.
to a neurogenic bladder, and not asymptom-
Answer E is incorrect. Wrist exion is also atic bacteriuria, is the cause of her urinary fre-
supplied by the median nerve. An injury to the quency.
median nerve would produce sensory decits
Answer C is incorrect. BPH occurs in 90%
over the palmar aspect of the rst three digits
of men >80 years old. Symptoms include uri-
and the lateral palm, none of which are pres-
nary frequency, hesitancy, urgency, straining,
ent in this patient.
decreased force of urinary stream, and incom-
plete bladder emptying with a large postvoid
Questions 45 and 46 residual volume. Transurethral resection of
the prostate is often performed to treat BPH.
45. The correct answer is M. Given the history of Pharmacologic interventions that can prevent
Test Block 7

instrumentation and the timing of the new the development of a urethral stricture include
symptoms, a urethral stricture is the likely 1-adrenergic blockers (such as terazosin) and
cause of the mans symptoms. It is likely that 5-reductase inhibitors (such as nasteride).
Test Block 7 Answers 707

Complications of untreated BPH include uri- symptoms, such as fever, pelvic pain, nausea,
nary retention, bladder calculi, recurrent infec- and vomiting, indicate disease due to other
tions, and renal insufciency. disorders such as pelvic inammatory disease,
salpingitis, endometritis, and/or tubo-ovarian
Answer D is incorrect. Central diabetes in-
abscesses. Men with gonococcal urethritis
sipidus (DI) presents with extreme thirst and
complain of urethral discomfort, dysuria, and
urinary frequency due to decreased secretion
purulent discharge. Unilateral scrotal pain and
of ADH (vasopressin) by the posterior pituitary
swelling suggests epididymitis. In a patient >35
gland or hypothalamus. Decreased ADH re-
years old, epididymitis is most likely due to N.
sults in an inability to effectively concentrate
gonorrhoeae or Chlamydia trachomatis. Patients
urine. It may be idiopathic, or patients may
should be treated with a single dose of intra-
report recent head trauma, cranial surgery, or
muscular ceftriaxone and oral azithromycin for
symptoms consistent with a brain neoplasm. In
gonorrhea and chlamydia, respectively, as the
central DI, serum ADH levels are low and the
two infections often coexist. More intensive an-
water deprivation (Miller-Moses) test fails to
tibiotic regimens are required for more exten-
produce more concentrated urine.
sive infection. Disseminated disease (including
Answer E is incorrect. Detrusor instability a migratory polyarthritis, petechial rash, endo-
causes uninhibited bladder contractions and carditis, or meningitis) occurs in 1% of patients.
leads to symptoms of urge incontinence (in-
Answer H is incorrect. Interstitial cystitis is
ability to hold urination until the appropriate
characterized by urgency and chronic pel-
facilities can be reached, once an urge is felt).
vic pain in addition to urinary frequency. Ap-
In addition to urgency, patients complain of
proximately 90% of patients are female, 94%
frequency and nocturia. Elderly patients may
are white, and the median age of diagnosis is
not perceive an urge and may simply experi-
40 years. The etiology is poorly understood
ence large-volume urinary loss; however, the
and thought to involve bladder hypersensitiv-
mechanism is the same. A causative urinary
ity, although there appears to be an associa-
tract infection should be ruled out before de-
tion with autoimmune diseases, irritable bowel
trusor instability itself is treated with anticho-
syndrome, bromyalgia, and atopy. Cystoscopy
linergic agents (such as tolterodine) or tricyclic

FULL-LENGTH EXAMS
may be normal or reveal ulceration, especially
antidepressants.
on overdistension. Potassium sensitivity testing
Answer F is incorrect. Patients with undiag- (infusion of potassium chloride solution via
nosed or poorly controlled diabetes mellitus Foley catheter) is sometimes used in diagnosis.
may present with polydipsia and polyuria, No denitive treatment exists, though analge-
caused by osmotic diuresis secondary to hy- sics, anti-inammatory drugs, antidepressants,
perglycemia. Urinalysis will show glycosuria. sodium pentosan polysulfate (thought to in-
Patients with type 1 diabetes are typically <30 crease production of the protective glycosamin-
years old and may report fatigue, malaise, and oglycan layer in the bladder wall), electrical
weight loss despite adequate food consump- stimulation, and acupuncture may provide
tion. Patients with type 2 disease tend to be some relief.
older and obese and may also suffer from hy-
Answer I is incorrect. Nephrogenic DI pre-
pertension, hyperlipidemia, cardiovascular dis-
sents with symptoms similar to those of central
ease, or polycystic ovarian syndrome.
DI; however, it is due to renal resistance to the
Answer G is incorrect. Infection with Neis- effects of ADH. Patients do not concentrate
seria gonorrhoeae most commonly causes cervi- their urine in response to exogenous ADH.
citis in women and urethritis in men. Women Causes of nephrogenic DI include chronic re-
Test Block 7

typically complain of purulent, yellow-green nal insufciency and drugs such as lithium.
vaginal discharge and occasionally have dys-
Answer K is incorrect. Patients with prostatitis
uria if urethritis is also present; however, they
suffer from obstructive urinary symptoms similar
may be completely asymptomatic. More severe
to those associated with BPH or urethral stric-
708 Section II: Full-Length Examinations Answers

tures; however, these patients also report fever, Answer L is incorrect. Patients with psycho-
chills (in acute bacterial cases, more common genic polydipsia drink excessively and thus uri-
in men >35 years old), malaise, urethral dis- nate excessively. They may develop symptoms
charge, and pain in the lower back, abdomen, of hyponatremia secondary to water intoxica-
pelvis, and scrotum. On rectal examination the tion. A water deprivation test will exclude DI
prostate is tender and boggy. Evidence of in- and lead to the diagnosis.
fection may be found on complete blood cell
count, urinalysis, and urine culture.
FULL-LENGTH EXAMS
Test Block 7
Test Block 8

709
710 Section II: Full-Length Examinations Questions

Q U E ST I O N S

1. A 5-year-old boy is brought to the pediatrician (C) Radiation and/or chemotherapy


because of persistent hypercalciuria discovered (D) Radical hysterectomy
on routine urinalysis. The boy has a history of (E) Repeat colposcopy in 3 months
renal stones and Ehlers-Danlos syndrome. His
family history is notable for a sister, 22 years 3. A 2-year-old boy is brought to the clinic for
old, who was diagnosed with Ehlers-Danlos evaluation after 2 days of low-grade tactile fe-
syndrome as a child. The boy is examined and ver, irritability, and decreased activity. His
is in no distress. He is in the 5th percentile for mother reports that on waking him this morn-
height and the 10th percentile for weight. His ing, his pillow was stained with a small amount
respiratory rate is 30/min, heart rate is 100/min, of yellow uid. His heart rate is 92/min, blood
and blood pressure is 110/70 mm Hg. The boy pressure is 104/70 mm Hg, respiratory rate is
has a soft late-systolic murmur that is heard 22/min, and temperature is 38.8C (101F).
best at the apex. His lungs are clear and abdo- His external ear canal is normal, but otoscopic
men is soft. His lower extremities are notable examination shows the tympanic membrane
for laxity of his knee joint. Laboratory studies appears dull, erythematous, and bulging.
show: Which of the following is the most likely bacte-
rial pathogen implicated in this condition?
Na+: 140 mEq/L
K+: 2.6 mEq/L (A) Haemophilus inuenzae type B
Cl: 119 mEq/L (B) Neisseria meningitidis
HCO3: 9 mEq/L (C) Pseudomonas aeruginosa
Blood urea nitrogen: 8 mg/L (D) Staphylococcus aureus
Creatinine: 0.5 mg/L (E) Streptococcus pneumoniae
Arterial pH: 7.2
4. A 24-year-old man presents to his physician be-
What is the most appropriate treatment for this
cause of neck pain that has become more se-
boys renal disorder?
FULL-LENGTH EXAMS

vere over the past several months. He describes


(A) Acetazolamide the pain as dull and says he feels very stiff in
(B) Ammonium chloride the morning, but the stiffness improves with
(C) Potassium citrate activity. He also notes the pain often awakens
(D) Sodium polystyrene sulfonate him during the night. On physical examina-
(E) Vitamin D tion his lungs are clear to auscultation bilater-
ally, but he seems to have diminished air entry
2. A 57-year-old woman is referred for colposcopy due to impaired chest wall expansion. Palpa-
following an abnormal Papanicolaou (Pap) tion of the cervical spine causes pain. X-ray of
smear nding of high-grade squamous intraepi- the neck is shown in the image. Which of the
thelial lesion. Her last Pap smear 3 years ago following is the patient at risk for developing?
was normal and she has never had an abnor-
mal Pap smear until now. She denies weight
loss, abdominal pain, vaginal bleeding, or dys-
pareunia. Colposcopy identies two lesions
near the squamocolumnar junction. Which of
the following is the best next step in manage-
ment?
Test Block 8

(A) Endocervical curettage and directed bi-


opsy of the lesions
(B) Observe with Pap smears every 4 months
for 1 year
Test Block 8 Questions 711

6. A problem-based learning case being discussed


in class regards an immunodeciency disorder
involving a 12-year-old boy. Shortly after he be-
gan walking, he developed worsening instability
and imbalance until he was eventually conned
to a wheelchair at 9 years of age. Additionally,
he developed oculocutaneous telangiectasias
beginning at approximately 3 years of age. Dur-
ing the discussion of this patients case, it is
added that such patients commonly have
chronic sinopulmonary disease and also have a
high incidence of malignancy, particularly lym-
Reproduced, with permission, from Chen MYM, Pope phoreticular malignancies. Which of the follow-
TL, Ott DJ. Basic Radiology. New York: McGraw-Hill,
ing is the mechanism of action responsible for
2004: Figure 7-38.
this patients condition?
(A) Anterior uveitis
(A) Absent respiratory burst
(B) Gastroesophageal reux disease
(B) Blocked lysosomal trafcking
(C) Malar rash
(C) Defective DNA repair
(D) Mitral stenosis
(D) Defects in peroxisome function
(E) Pleuritis
(E) Impaired toll-like receptor signaling
5. A 46-year-old African-American mother of three
7. A 46-year-old African-American woman with
presents to the emergency department com-
diabetes visits her primary care physician be-
plaining of colicky abdominal pain, vomiting,
cause of an ulcer on her toe. The ulceration
fevers, and chills. She has noted her stools over
has a darkened keratinous base with central ne-
the past 2 days to be lighter-colored than usual.
crosis involving skin, fat, muscle, and bone.
She has had similar pain before, but it resolved
There is also a thick purulent discharge pres-
with time and was never this bad. Her tempera-
ent. She denies any pain, tenderness, or dis-
ture is 39.6C (103.3F), pulse is 115/min,

FULL-LENGTH EXAMS
comfort, but states that sensation in her feet is
blood pressure is 95/60 mm Hg, and respiratory
diminished. Her temperature is mildly elevated
rate is 22/min. On examination she is in obvious
at 38C (100F). Which of the following is the
distress and the sclerae are icteric. Bowel sounds
most appropriate initial treatment?
are normal and the abdomen is tender to palpa-
tion in the right upper quadrant without re- (A) Clindamycin
bound. Laboratory studies show a WBC count (B) Doxycycline
of 15,000/mm, total bilirubin of 4.0 mg/dL, (C) Metronidazole
and elevated levels of alanine aminotransferase, (D) Penicillin G
aspartate aminotransferase, and alkaline phos- (E) Piperacillin and gentamicin
phatase. Ultrasound reveals dilated intrahepatic
bile ducts with a few stones in the gallbladder.
Which of the following is the most likely diag-
nosis?
(A) Acute cholangitis
(B) Acute cholecystitis
(C) Complete small bowel obstruction
(D) Hepatitis
Test Block 8

(E) Pancreatic cancer


712 Section II: Full-Length Examinations Questions

8. A 34-year-old woman presents to her gynecolo- dL, and conjugated bilirubin of 1.8 mg/dL. His
gist complaining of incontinence. She states hematocrit is 50%. Coombs test is negative.
that for the past year, ever since the birth of her What is the most likely etiology of the childs
third child, she has been leaking small jaundice?
amounts of urine such that she needs to wear
(A) Bile duct obstruction
liners in her underwear to prevent her clothes
(B) Breast-feeding
from being wet. She states the leakage occurs
(C) Hemolytic disease
all the time but is worse when she drinks a lot
(D) Physiologic
of coffee or water. Physical examination re-
(E) Sepsis
vealed peritoneal skin irritation and steady
leakage of urine. Which of the following is
11. A 25-year-old university student presents to the
most likely to indicate the correct diagnosis?
campus health clinic after detecting a mass in
(A) Bladder capacity measurement his right testicle during self-examination. He
(B) Catheterization has no signicant medical history and no his-
(C) Cystometry with bethanechol chloride tory of cancer in his family. Physical examina-
(D) Sphincteric function analysis tion reveals a rm mass in his right testicle that
(E) Voiding cystourethrography does not transilluminate. Transscrotal ultra-
sound of the right testicle conrms the pres-
9. In a nationwide study, 10% of the screening ence of a solid mass, and serum tumor markers
tests performed on patients with disease X re- are signicant for an elevated -fetoprotein
sulted in false-negative outcomes, while 25% level. What is the best next step in manage-
of the tests falsely identied healthy patients as ment?
having the disease. To determine the probabil-
(A) Biopsy of the testicular mass
ity that a patient who tests positive for disease X
(B) Platinum-based chemotherapy
actually does have the disease, what additional
(C) Radiation therapy
information is necessary?
(D) Radiation therapy followed by radical
(A) No additional information is necessary orchiectomy
(B) The incidence of the disease must be (E) Radical orchiectomy
FULL-LENGTH EXAMS

known
(C) The prevalence of the disease must be 12. Which of the laboratory ndings in the follow-
known ing table are characteristic of immune throm-
(D) The sensitivity of the test must be known bocytopenic purpura? PT refers to prothrom-
(E) The specicity of the test must be known bin time and aPTT refers to activated partial
thromboplastin time.
10. A 23-year-old G2P1 Asian woman delivers a
3100-g (6.8-lb) boy at term by spontaneous vag-
inal delivery after a pregnancy without compli-
cation. Her prenatal laboratory results are no-
table for a positive Group B Streptococcus
screen and an A-negative blood type. The in-
fants Apgar scores were 8 and 9 at 1 and 5 min-
utes, respectively, and his blood type is AB pos-
itive. He is being exclusively breast-fed, but the
mother notes that he is not feeding well. On
his discharge at 48 hours of life he is in no dis-
Test Block 8

tress but is jaundiced with a bilirubin level of


15 mg/dL, unconjugated bilirubin of 13.2 mg/
Test Block 8 Questions 713

BLEEDING TIME PLATELET COUNT


CHOICE (min) PT (sec) APTT (sec) (/mm3) FACTOR IX FACTOR VIII

A 3 17 27 202,000 abnormal normal

B 4 12 42 190,000 abnormal normal

C 4 12 45 230,000 normal abnormal

D 8 13 27 58,000 normal normal

E 9 20 28 143,000 normal abnormal

F 9 20 29 67,000 normal normal

(A) A 14. An otherwise healthy 27-year-old woman pre-


(B) B sents to the emergency department with lower
(C) C abdominal pain, nausea, vomiting, anorexia,
(D) D and fever that began yesterday morning. The
(E) E patient states that the pain is present bilater-
(F) F ally; however, it is more severe in the right
lower quadrant of the abdomen than the left.
13. An 18-year-old woman comes to her pediatri- The patient has had ve sexual partners in the
cian for a precollege medical evaluation. She past year with whom she sometimes used con-
got straight As throughout high school, and she doms and sometimes had unprotected inter-
recently performed in her dance troupes course. Her menstrual cycles have been regu-
spring ballet. She has no physical complaints lar with a moderate amount of bleeding. Her
but mentions in passing that she wants to lose temperature is 38.2C (100.8F), blood pres-
more weight so she can be skinny like the rest sure is 112/72 mm Hg, pulse is 72/min, and re-
of the girls. She is 168 cm (5'6") tall and spiratory rate is 13/min. On physical examina-

FULL-LENGTH EXAMS
weighs 46 kg (101 lb); her body mass index is tion there is bilateral lower quadrant abdominal
16.3 kg/m. Her pulse is 55/min and blood pain, along with cervical discharge and cervi-
pressure is 95/50 mm Hg. The physician notes cal motion tenderness. A urine -human cho-
dry, yellow-tinged skin and soft, ne hair on rionic gonadotropin test is negative. Her WBC
her arms and trunk. She is not currently sexu- count is 13,500/mm, hemoglobin is 13.8 g/
ally active and denies drinking alcohol and do- dL, and platelet count is 315,000/mm. What
ing drugs, but she does smoke two to three cig- is the best next step in management?
arettes a day. Her chart indicates that she
(A) Abdominal plain lms
reached menarche at age 13 years. Which of
(B) Begin oral contraceptives
the following is most likely to be diagnosed in
(C) Laparotomy
this patient?
(D) Potassium hydroxide whiff test
(A) Hirsutism (E) Ultrasound
(B) Hypothyroidism
(C) Pituitary adenoma
(D) Secondary amenorrhea
(E) Small bowel obstruction
Test Block 8
714 Section II: Full-Length Examinations Questions

15. A 56-year-old woman with congestive heart (D) Colonoscopy every 23 years with yearly
failure, managed with diuretics and an angio- sigmoidoscopy starting at age 30 years
tensin-converting enzyme inhibitor, presents to (E) Colonoscopy every 23 years with yearly
her physician with progressive difculty breath- sigmoidoscopy starting at age 40 years
ing over the past week. She states that it is dif-
cult for her to take a full breath, and occasion- 17. A 53-year-old white woman presents to her pri-
ally she will develop pain over the right lateral mary physician complaining of u-like symp-
chest that becomes worse with inspiration. The toms, including muscle and joint pains, which
patient quit alcohol use after her heart attack have developed gradually over the course of a
but has continued to smoke two packs of ciga- month. The woman also reports a low-grade fe-
rettes a day, and has done for more than 25 ver during this time period. Her past medical
years. Review of systems is signicant for brief history is signicant for hypertension, for
episodes of fever and occasional blood-tinged which she takes clonidine and captopril. She
sputum over the past 5 months. On examina- also takes procainamide for arrhythmia, methi-
tion she is afebrile with a oxygen saturation of mazole for hyperthyroidism, and amitriptyline
99% on room air and a respiratory rate of 17/ for depression. The patients temperature is
min. She has decreased breath sounds at the 37.7C (99.8F), blood pressure is 132/86 mm
right base with dullness to percussion; the left Hg, pulse is 82/min and regular, and respira-
lung eld is clear. There is no jugular venous tory rate is 18/min. Physical examination is
distension. An ECG shows no acute changes. normal. Laboratory studies reveal anti-histone
X-ray of the chest shows a uid collection in antibodies and the presence of anti-ss DNA.
the right base and a spiculated mass at the right C3 and C4 complement levels are within nor-
hilum with a normal cardiac silhouette. Re- mal limits. Resolution of this patients symp-
sults of thoracentesis reveal: toms will most likely occur following discon-
tinuation of which of the following drugs?
Appearance: Turbid
WBCs: 43,000/mm lymphocyte predominant (A) Amitriptyline
RBCs: 11,000/mm (B) Captopril
pH: 7.32 (C) Clonidine
FULL-LENGTH EXAMS

Glucose: 87 mg/dL (D) Methimazole


(E) Procainamide
Which of the following is the most likely etiol-
ogy of the patients effusion?
18. A 4-year-old girl is brought into the emergency
(A) Alcoholic cirrhosis department by her mother, who reports that
(B) Collagen vascular disease her daughter has been wetting the bed for the
(C) Exacerbation of congestive heart failure past week despite previously being potty-
(D) Malignancy trained. She believes this may be due to her
(E) Pneumococcal pneumonia daughter drinking more uids recently: she
previously drank about 67 cups of water or
16. A 55-year-old man recently diagnosed with co- juice daily, but she now drinks 1415 cups of
lon cancer and familial adenomatous polyposis uids per day. Physical examination reveals hy-
sees his doctor with regard to the care of his potension, tachycardia, and a fruity odor to the
5-year-old son. The son is genetically tested patients breath. The physician suspects a new
and found to have a mutated APC gene. What diagnosis of type 1 diabetes mellitus. Labora-
should the physician recommend as colon can- tory tests show a glucose level of 538 mg/dL,
cer screening for the 5-year-old son? blood urea nitrogen of 27 mg/dL, and creati-
nine of 1.5 mg/dL. Further tests reveal an an-
Test Block 8

(A) Digital rectal examinations only starting at


ion gap metabolic acidosis (gap: 26, pH 7.18)
age 10 years
and ketonuria (4+). The physician diagnoses
(B) Digital rectal examinations only starting at
diabetic ketoacidosis and starts intravenous
age 40 years
uid hydration with normal saline with potas-
(C) Colonoscopy every 23 years with yearly
sium supplementation, and insulin infusion.
sigmoidoscopy starting at age 10 years
Test Block 8 Questions 715

TIME SINCE BLOOD PRESSURE NA+ K+ CI HCO3 GLUCOSE


ARRIVAL (hrs) (mm Hg) (mEq/L) (mEq/L) (mEq/L) (mEq/L) (mq/dL)

0:00 80/41 140 3.5 103 11 538

1:00 86/49 139 3.7 105 12 433

2:00 92/57 141 4.0 101 14 346

3:00 93/59 141 4.1 100 16 272

4:00 100/65 140 4.3 104 18 241

5:00 121/82 141 4.2 103 21 217

Results of hourly serum chemistry tests are he reports a headache immediately after the in-
shown in the table above. Her laboratory tests jury, he is otherwise neurologically intact.
normalize nicely, but 5 hours later she starts Three hours later, his friends notice that he
complaining of a headache. Which of the fol- has slumped over in his chair and does not re-
lowing is the best next step in management? spond to their questions. On evaluation in the
emergency department, he has a xed, dilated
(A) Administer mannitol
pupil. What is most likely to be found on a CT
(B) Increase rate of insulin infusion
scan of this patient?
(C) Increase rate of intravenous uid rehydra-
tion (A) Blood in the subarachnoid space
(D) Obtain an ECG (B) Crescent-shaped, concave hyperdensity
(E) Perform a lumbar puncture that does not cross midline
(C) Irregularly shaped hyperdensity within the

FULL-LENGTH EXAMS
19. A 28-year-old woman presents to her primary parenchyma
care physician with complaints of vaginal itch- (D) Lens-shaped, convex hyperdensity limited
ing and discomfort for the past day, as well as by the sutures
white cottage cheese discharge for the past 3 (E) Normal CT scan
days. The patients medical history is signi-
cant for epilepsy well controlled on phenytoin. 21. An 11-day-old baby is diagnosed with tuberous
Pelvic examination reveals genital erythema sclerosis following the discovery of a heart
and excoriation, and a potassium hydroxide rhabdomyoma and an intraventricular hamar-
preparation reveals pseudohyphae. Which of toma in the region of the foramen of Monroe.
the following is the most appropriate denitive In addition, the baby also has bilateral kidney
treatment? masses, but as of yet does not have obvious skin
lesions. Which of the following is a major com-
(A) Clindamycin
plication of tuberous sclerosis?
(B) Doxycycline
(C) Metronidazole (A) Glioblastoma multiforme
(D) Oral uconazole (B) Liver adenocarcinoma
(E) Topical miconazole (C) Pancreatic adenocarcinoma
(D) Seizures
Test Block 8

20. A young man with no history of neurologic dis-


order is playing basketball when he falls on the
court and hits the side of his head. Although
716 Section II: Full-Length Examinations Questions

22. A 19-year-old college sophomore presents to bated, epinephrine is administered, and the pa-
the university health clinic because of a 2-day tient is again shocked without response. Which
history of burning upon urination. Her vital of the following would be the best choice for
signs are within normal limits and physical ex- the patients present state?
amination reveals moderate suprapubic pain.
(A) Amiodarone
Physical examination is also negative for vagi-
(B) Digoxin
nal discharge and cervical motion tenderness.
(C) Diltiazem
She has been sexually active for the past 15
(D) Metoprolol
months with four different partners. She admits
(E) Procainamide
to infrequent condom use but has been taking
(F) Tocainide
oral contraceptive pills. Which of the following
is most likely to be causing her symptoms?
25. A physician gets paged to the emergency de-
(A) Chlamydia trachomatis partment to evaluate a young, well-muscled
(B) Enterococcus faecalis man weighing 102 kg (225 lb) complaining of
(C) Klebsiella pneumoniae weakness. While his history is taken, the physi-
(D) Neisseria gonorrhoeae cian notices that he is using accessory neck
(E) Pseudomonas aeruginosa muscles to breathe. The patient is a nurse who
(F) Staphylococcus saprophyticus recently received the inuenza vaccine. The
doctor places him on oxygen by nasal cannula
23. A 16-month-old, full-term boy is brought to the and orders pulmonary function tests that show
emergency department with burns on his chest a vital capacity of 1.5 L. The physician decides
and left arm. His mother states that she was to intubate due to impending respiratory fail-
boiling water and accidentally splashed some ure. Cerebrospinal uid analysis shows an ele-
onto him when he startled her. She applied ice vated protein level and normal WBC count.
at home and immediately brought him to the Results of electromyographic study are pend-
emergency department for evaluation. His ing. First-line treatment of this disorder consists
temperature is 37C (98.6F), pulse is 150/ of which of the following?
min, and respiratory rate is 22/min. He cries
(A) Anti-thymic immunoglobulin
FULL-LENGTH EXAMS

during the examination, but touching the le-


(B) Corticosteroids
sions lightly does not cause increased distress.
(C) Cyclosporine and tacrolimus
The burns are brightly erythematous in some
(D) Plasmapheresis and intravenous immune
areas, with patchy whitish, waxy spots. Several
globulin
large, thin blisters have unroofed. The burns
(E) Tumor necrosis factor- receptor blockers
do not blanch with pressure, although this ma-
neuver does elicit pain. What is the depth of 26. A patient presents to the clinic with complaints
the described burns? of difculty swallowing and says, the food
(A) Deep partial-thickness burn seems to get stuck in my throat when I try to
(B) Full-thickness/third-degree burn swallow. A barium swallow shows smooth nar-
(C) Supercial partial-thickness burn rowing at the lower esophageal sphincter
(D) Supercial/rst-degree burn (LES). Which of the ndings in the table are
most commonly seen in this condition on ma-
24. An obese 75-year-old man with a history of cor- nometry?
onary artery disease and prior three-vessel by-
pass graft is brought to the emergency depart-
ment with the complaint of chest pain. Upon
Test Block 8

entering the emergency department the pa-


tient becomes unresponsive and stops breath-
ing. He is pulseless and found to be in ventric-
ular tachycardia, and is subsequently shocked
with no response. The patient is then intu-
Test Block 8 Questions 717

29. A 30-year-old man with type 2 diabetes melli-


LES
tus presents to the ofce complaining of noc-
CHOICE LES TONE RELAXATION PERISTALSIS
turnal sweating and headaches. He states that
A normal aperistalsis since his insulin regimen was changed 1
month ago, he has been waking up at night
B normal hyperperistalsis with these symptoms. He also notes that his
morning glucose levels via ngersticks have
C normal aperistalsis
been 250300 mg/dL. Last night when this oc-
D impaired aperistalsis
curred, he checked his blood sugar and it was
50 mg/dL. He currently takes 30 U of neutral
E impaired hyperperistalsis protamine Hagedorn every morning and 15 U
every night. Which of the following is the most
appropriate next step in patient care?
(A) A
(B) B (A) Decrease the evening dose of insulin
(C) C (B) Decrease the morning dose of insulin
(D) D (C) Increase the evening dose of insulin
(E) E (D) Increase the morning dose of insulin
(E) Monitor plasma glucose levels in the
27. Which of the following is associated with the morning more closely
development of primary central nervous system
lymphoma? 30. A 45-year-old man with no known medical his-
tory underwent a total knee replacement 3
(A) Epstein-Barr virus
days ago. Postoperatively the man had nausea
(B) JC virus
that necessitated the placement of a nasogas-
(C) Mycobacterium avium complex
tric tube. On postoperative day 2 the patients
(D) Previous radiation exposure
nasogastric tube was discontinued. On postop-
(E) Toxoplasma gondii
erative day 3 laboratory tests showed:
28. A 27-year-old woman presents to her psychia- Na+: 145 mEq/L

FULL-LENGTH EXAMS
trist with a 6-week history of 6.8-kg (15-lb) K+: 7.2 mEq/L
weight gain, increased sleep, generalized Cl: 110 mEq/L
moodiness, and generalized fatigue. She HCO3: 24 mEq/L
notes that she has been eating much more than Blood urea nitrogen: 9 mg/dL
usual, especially sugary and carbohydrate-rich Creatinine: 0.8 mg/dL
foods. Approximately 2 months ago, her anc
ECG is within normal limits. The house of-
ended their engagement. She had several epi-
cer examines the man, who has a heart rate of
sodes of this same feeling while in college, dur-
89/min, blood pressure of 136/86 mm Hg, re-
ing which time she initially sought psychiatric
spiratory rate of 12/min, and temperature of
help. She denies any suicidal or homicidal ide-
36.9C (98.4F). The man states that he feels
ation. Which of the following is the most ap-
ne except for incisional pain (4 of 10 in sever-
propriate treatment?
ity) and mild nasal congestion. Which of the
(A) Desipramine following is the most likely cause of the mans
(B) Lithium hyperkalemia?
(C) Olanzapine
(A) Accidental insulin injection
(D) Sertraline
(B) Addisons disease
(E) Zolpidem
Test Block 8

(C) Hemolysis (in collected blood sample)


(D) Nasogastric suction
(E) Renal failure
(F) Rhabdomyolysis
718 Section II: Full-Length Examinations Questions

31. A 67-year-old woman with a family history of cannot be contacted. The neighbors who
heart disease is seen in the clinic for follow-up called 911 claim they saw the boy seizing in
of an elevated total cholesterol level. Dietary the backyard and were not sure how long he
modication and initiation of an exercise pro- had been there, but state that it took the ambu-
gram have not yielded change in her lipid pro- lance at least 10 minutes to arrive. They are
le, so she began taking atorvastatin. Which aware that the child has a long-standing seizure
complaint from the patient regarding her new disorder of some kind and add that they believe
medication would be most concerning? he often abuses cocaine. On arrival the pa-
tients pulse is 125/min, blood pressure is
(A) Aching limbs
160/100 mm Hg, temperature is 38.9C
(B) Cognitive decline
(102F), and respiratory rate is 22/min. On ex-
(C) Headache
amination the patient has his arms extended
(D) Malar rash
rigidly at his side and is arching his back rhyth-
(E) Shortness of breath
mically and appears to be aspirating blood.
32. A 66-year-old man presents to the emergency The patient is covered in his own urine and
department with epigastric pain after being dis- blood is coming from his mouth. Nasopharyn-
charged 1 week ago for an episode of acute geal intubation is unsuccessful after several at-
tempts. Which of the following is the most ap-
pancreatitis. CT of the abdomen is shown in
propriate next step in management?
the image. Which of the following statements
about this diagnosis is true? (A) Intravenous glucose, thiamine, and nalox-
one
(B) Intravenous lorazepam and phenytoin
(C) Intravenous phenobarbital
(D) Placement of an electroencephalography
monitor
(E) Rapid sequence intubation

34. A 68-year-old man with hypertension, coronary


FULL-LENGTH EXAMS

artery disease, and previous inferior myocardial


infarction presents to the emergency depart-
ment with sudden onset of severe tearing
chest pain that radiates to the back. The pain is
Reproduced, with permission, from Chen MYM, Pope accompanied by diaphoresis, nausea, and light-
TL, Ott DJ. Basic Radiology. New York: McGraw-Hill, headedness. The patient denies loss of con-
2004: Figure 26-3. sciousness or evidence of syncope. Administra-
(A) The mature form of this structure will con- tion of sublingual nitroglycerine has not
tain blood, necrotic debris, and WBCs provided any relief. Blood pressure in the right
(B) This structure cannot be caused by pancre- arm is 80/40 mm Hg, while that in the left arm
atic trauma is 190/110 mm Hg. A diastolic murmur is
(C) This structure communicates with the exo- heard at the left sternal border, which is consis-
crine ductal system tent with aortic insufciency. An ECG is ob-
(D) This structure is at risk for rupture and tained that shows no acute ischemic ST seg-
bacterial superinfection ment changes. X-ray of the chest reveals a
(E) This structure is best diagnosed with MRI widened mediastinum. Which of the following
is the most appropriate next step in diagnosis?
33. A 16-year-old boy is brought to the emergency
Test Block 8

(A) Transesophageal echocardiogram


department by paramedics while seizing. The (B) Transthoracic echocardiogram
patients parents are known drug addicts and (C) Upper endoscopy
(D) Ventilation-perfusion scan
(E) X-ray of the abdomen
Test Block 8 Questions 719

35. A 15-year-old boy presents to the physicians of- (D) He should refer patients only to hospital B
ce complaining of fatigue, headache, myalgias, so as not to take advantage of patients for
and sore throat that have been continuous for his own economic benet
1.5 weeks. The patient has generalized lymph- (E) The physician should not be involved in
adenopathy of the cervical, submandibular, and commercial interests as long as he is in-
epitrochlear nodes, in addition to splenomegaly. volved in patient care in the same medical
His tonsils are signicantly enlarged with exu- eld
date. Peripheral blood smear shows 65% lym-
phocytes, with 15% atypical lymphocytes. Het- 37. A patient presents to her primary care physi-
erophile antibody test is positive. Which is the cian with vague complaints of epigastric pain.
most likely cause of his syndrome? X-ray of the abdomen is suggestive of a type II
(paraesophageal) hiatal hernia, and this is later
(A) Cytomegalovirus
conrmed on barium swallow. The patient is
(B) Epstein-Barr virus
recommended to undergo surgical repair. The
(C) HIV-1
goal of surgery is to prevent which complica-
(D) Human herpes virus-6
tion?
(E) Infection with Toxoplasma
(A) Barretts esophagus
36. After years of hard work, a cardiologist and re- (B) Gastric cancer
searcher creates a new, more exible coronary (C) Gastric ulcer
artery stent that is easier to manipulate during (D) Gastric volvulus/incarceration
cardiac catheterization. The initial response to (E) Gastroesophageal reux disease
this invention is so positive that he decides to
take a business partner. He begins a small com- 38. A 68-year-old woman presents to the emer-
pany that manufactures and sells these stents. gency department with acute onset of a frontal
The physician knows that the interventional headache, blurry vision with halos around
cardiologists at hospital A purchase and use his lights, and pain in the right eye. Her past medi-
stents, while those at the hospital downtown, cal history is notable for hypertension and to-
Hospital B, do not use his stents. He also knows bacco use. She was recently seen by her oph-

FULL-LENGTH EXAMS
that outcomes at the two hospitals are similar, thalmologist for treatment of anterior uveitis.
although no data have yet emerged about Ocular examination shows mid-dilated pupils,
which stent is better. What is the most appro- corneal edema, and conjunctival hyperemia.
priate way for this physician to approach car- Tonometry reveals intraocular pressure of her
diac catheterization for his private patients? right eye to be 55 mm Hg. Which of the fol-
lowing is the most appropriate initial step in
(A) He may refer patients to either hospital
management?
and does not need to disclose his eco-
nomic interests in hospital A; the patients (A) Laser iridotomy
can ultimately decide to which hospital (B) Oral pilocarpine
they want to go (C) Topical acetazolamide (eye drops)
(B) He may refer patients to either hospital but (D) Topical atropine (eye drops)
must inform them of his economic inter- (E) Topical prednisone (eye drops)
ests in hospital A
(C) He should refer patients only to hospital A
so they may benet from his superior tech-
nology
Test Block 8
720 Section II: Full-Length Examinations Questions

39. A 42-year-old African-American man presents 41. A 32-year-old man visits his doctor because of
to his physician for a check-up. His father, who an extremely itchy rash on his left arm for the
is 65 years old, recently started dialysis, and the past week. The lesion is oval, papular, and ery-
patient wants to make sure he is healthy. A re- thematous, with central clearing and a dry
view of systems is completely normal and he scale at the periphery. He recently spent time
has no physical complaints. The patients with his 7-year-old nephew who is being
blood pressure is 164/95 mm Hg, heart rate is treated by the pediatrician for a scaly, red, pru-
82/min, and he is afebrile. Chart review indi- ritic rash on his chest.
cates his blood pressure 2 years ago was 158/93
mm Hg. Results of fundoscopic and neurologic The response options for the next 3 items are
examinations are normal. The remainder of the same. Select one answer for each item
the physical examination is unremarkable, as in the set.
are laboratory test results. What is the most ap-
propriate therapy at this time? For each patient with vaginal bleeding during preg-
(A) Diet and exercise nancy, select the most likely diagnosis.
(B) Diltiazem and lisinopril (A) Abruptio placentae
(C) Hydrochlorothiazide alone (B) Bacterial vaginitis
(D) Hydrochlorothiazide and lisinopril (C) Candidal vaginitis
(E) Lisinopril alone (D) Cervical carcinoma
(E) Ectopic pregnancy
E X T E N D E D M ATC H I N G (F) Endometrial cancer
(G) Foreign body
(H) Gonococcal cervicitis
The response options for the next 2 items are
(I) Molar pregnancy
the same. Select one answer for each item
(J) Normal labor
in the set.
(K) Normal menses
For each patient with rash, select the most likely (L) Placenta previa
(M)Threatened abortion
FULL-LENGTH EXAMS

diagnosis.
(N) Trichomoniasis
(A) Drug reaction (O) Vesicovaginal stula
(B) Erythema migrans (P) Vulvar carcinoma
(C) Guttate psoriasis
(D) HIV seroconversion 42. A 36-year-old G2P1 woman at 35 weeks gesta-
(E) Nummular eczema tion presents to the emergency department
(F) Pityriasis rosacea complaining of abdominal pain and vaginal
(G) Secondary syphilis bleeding. She has had an uneventful preg-
(H) Tinea corporis nancy until this point with regular prenatal
care. Her temperature is 36.9C (98.4F),
40. A 45-year-old woman presents to the clinic blood pressure is 168/102 mm Hg, pulse is 74/
with a lesion on her posterior right thigh, just min, and respiratory rate is 14/min. Pelvic ex-
above the knee. The lesion is erythematous amination reveals blood pooled in the posterior
with an area of central clearing. It appeared 2 fornix and pain on palpation of the uterus. To-
days ago and has grown larger since then. She cometry reveals irregular contractions, and fe-
also complains of fever, occasional chills, head- tal heart tones are 180190/min.
ache, and achy muscles.
Test Block 8
Test Block 8 Questions 721

43. A 27-year-old G1P0 woman at 29 weeks gesta- (A) Acute lymphocytic leukemia
tion presents to her obstetrician with bright red (B) Acute myelogenous leukemia
vaginal bleeding that began 2 hours ago. To- (C) Burkitts lymphoma
cometry reveals a quiescent uterus; fetal heart (D) Chronic lymphocytic leukemia
tones are in the 150s and reactive. (E) Chronic myelogenous leukemia
(F) Common variable immunodeciency
44. A 29-year-old G2P1 woman at 37 weeks gesta- (G) Hodgkins lymphoma
tion presents to her obstetrician with a single- (H) Hyper IgE syndrome
ton fetus who is moving regularly. The mother (I) Hyper IgM syndrome
noted abdominal pain and a small amount of (J) Multiple myeloma
vaginal bleeding mixed with mucous vaginal (K) Nodular sclerosing lymphoma
discharge. Tocometry demonstrates regular (L) Osteomyelitis
contractions; fetal heart tones are in the 140s (M)Posttransplant lymphoproliferative disease
and reactive.
45. A 10-year-old boy presents with pallor, fatigue,
and hepatosplenomegaly. Bone marrow aspi-
The response options for the next 2 items are
rate reveals eosinophilic needle-like inclusions.
the same. Select one answer for each item
in the set.
46. A 55-year-old woman presents with fatigue and
frequent infections. Radiographic imaging of
For each of the following patients, choose the most
the bone marrow reveals lytic lesions.
likely diagnosis.

FULL-LENGTH EXAMS
Test Block 8
722 Section II: Full-Length Examinations Answers

AN S W E R S

1. The correct answer is C. This patient has type search for carcinoma is the appropriate evalua-
I renal tubular acidosis (RTA). His laboratory tion following a nding of high-grade
tests show a non-gap acidosis. This renal disor- squamous intraepithelial lesion, because of the
der is also known to cause nephrolithiasis, and much higher risk of nding precancerous le-
is associated with collagen vascular diseases sions such as cervical intraepithelial neoplasia
such as Ehlers-Danlos syndrome (EDS). The II or III, or invasive cancer. Biopsy specimens
defect in type I RTA is a defect in hydrogen ex- will be considered satisfactory if the entire le-
cretion. Treatment of both type I and type II sion and the boundaries of the transformation
RTA involves potassium citrate. Potassium cit- zone (squamocolumnar junction) are in-
rate is preferred over bicarbonate salts in this cluded.
case because it will help with the boys hy- Answer B is incorrect. Observation with Pap
pokalemia and requires less frequent dosing. smears, a screening test, is not acceptable for
Treating this boys acidemia will reduce his an initial Pap smear that shows a high-grade in-
chances of forming stones. traepithelial lesion. According to the Bethesda
Answer A is incorrect. Acetazolamide is not 2001 guidelines, observation is acceptable only
used in the treatment of type I RTA. The drug for low-grade intraepithelial lesions and only
can actually be a cause of type II RTA. The in certain circumstances (e.g., reliable patients
drug inhibits carbonic anhydrase activity, and that are also immunocompetent).
thus causes an acidosis because of decreased Answer C is incorrect. Radiation and chemo-
bicarbonate production. therapy are modalities reserved for invasive
Answer B is incorrect. Ammonium chloride carcinoma.
is not used to treat RTA. This is an acid that Answer D is incorrect. Radical hysterectomy is
can be used to help diagnose defects in urine part of the treatment for biopsy-proven invasive
acidication. It is given to cause acidemia, af- carcinoma.
ter which the urine pH is followed. If the pa-
FULL-LENGTH EXAMS

tient cannot adequately acidify the urine, the Answer E is incorrect. Repeat colposcopy is
diagnosis of RTA is made. not part of the acceptable management of a
patient with a high-grade intraepithelial lesion.
Answer D is incorrect. Sodium polystyrene The lesions observed via colposcopy following
sulfonate (Kayexalate) would not be recom- the abnormal Pap smear must be addressed
mended for treatment of type I or type II RTA. with biopsy to evaluate for carcinoma.
This medication helps lower serum potassium,
and the boy has a serum potassium that is al- 3. The correct answer is E. The patient de-
ready below normal. Sodium polystyrene sul- scribed is typical for a young child with acute
fonate is useful in treating patients with type otitis media (AOM). Although patients with
IV RTA that frequently have an elevated serum AOM can be asymptomatic, many present with
potassium secondary to decreased aldosterone myriad symptoms, including ear pulling, ear
levels or decreased aldosterone effectiveness. pain, ear drainage, fever, lethargy, irritability,
Answer E is incorrect. Vitamin D is not used decreased activity, and decreased appetite, to
to treat type I RTA. Chronic acidosis does lower name a few. The image shown illustrates an
tubular reabsorption of calcium, but correcting otoscopic view of an erythematous, bulging
the acidosis should result in less loss of calcium. tympanic membrane, which is typical of AOM.
Combining the history and the physical exami-
Test Block 8

2. The correct answer is A. The majority of cervi- nation, this patient is most likely to have a bac-
cal dysplasia and cancers arise adjacent to the terial AOM. The most common pathogen
squamocolumnar junction of the cervix. Thus, causing about 40% of AOM episodes is Strep-
endocervical curettage and directed biopsy to tococcus pneumoniae.
Test Block 8 Answers 723

Answer A is incorrect. Nontypeable Haemo- Answer B is incorrect. AS is not associated with


philus inuenzae, not the type B form, may an increased risk of gastroesophageal reux dis-
cause 25%30% of episodes of AOM and is the ease. Patients with AS are at increased risk of
second most common cause of this frequent developing ulcerations in the ileum and colon,
pediatric ailment. H. inuenza type B does which are typically asymptomatic. However,
not usually cause AOM. Although it was once some patients with AS have endoscopic and
a major cause of meningitis and epiglottitis, clinical ndings of inammatory bowel disease.
it has almost completely disappeared due to
Answer C is incorrect. Malar rash is a cuta-
widespread inoculation with the Hib vaccine
neous nding in patients with systemic lupus
in infancy.
erythematosus, and is not typically found in
Answer B is incorrect. Neisseria meningitidis patients with AS.
is not an important cause of AOM or otitis ex-
Answer D is incorrect. Patients with AS are at
terna.
risk for cardiac complications, including aortic
Answer C is incorrect. Pseudomonas aeruginosa insufciency, mitral valve prolapse, and third-
is the most common cause of otitis externa but degree heart block. AS is not associated with
is not a major cause of AOM and is therefore an increased risk of developing mitral stenosis.
not the correct answer for this question.
Answer E is incorrect. Pleuritis is an extra-
Answer D is incorrect. Studies have impli- articular nding in patients with rheumatoid
cated Staphylococcus aureus in about 5% of arthritis, and may also be seen in patients with
episodes of AOM, making it a relatively rare systemic lupus erythematosus. It is not typically
pathogen for inner ear infections. found in patients with AS. Patients with AS may
have abnormal pulmonary function due to im-
4. The correct answer is A. This patient is pre- paired chest wall expansion, and some patients
senting with signs and symptoms consistent are found to have pulmonary apical brosis.
with ankylosing spondylitis (AS), a chronic in-
ammatory disease of the axial skeleton. AS is 5. The correct answer is A. This patient is pre-
most common in white men, with onset of senting with characteristic signs and symptoms

FULL-LENGTH EXAMS
symptoms at age 2030 years. There is an asso- of acute cholangitis known as Charcots triad
ciation between AS and the human leukocyte (fever, jaundice, and right upper quadrant
antigen HLA-B27. The criteria for diagnosing pain). She also has hypotension, one of the two
AS include a history of inammatory back additional indicators of poor prognosis in Reyn-
pain, limitation of both exion/extension and olds pentad (the other being altered mental
lateral exion of the spine, impaired chest wall status). Moreover, she is in a high-risk group
expansion, and radiographic ndings of sacro- for gallstones, and likely has had episodes of
iliitis, including blurring of the cortical mar- undiagnosed biliary colic in the past. Her lab
gins of subchondral bone followed by erosions values are also consistent with an infectious
and sclerosis. Inammatory back pain is fur- process involving the biliary system and caus-
ther dened as insidious in onset and lasting ing some mild inammation of the liver. Ultra-
>3 months, with morning stiffness and im- sound indicated stones in the gallbladder and,
provement of pain with activity. Patients with more importantly, dilated intrahepatic ducts
AS are at risk for a variety of extra-articular from a biliary obstruction.
manifestations, one of the most common of
Answer B is incorrect. Acute cholecystitis
which is acute anterior uveitis. This complica-
would present with colicky abdominal pain
tion occurs in 25%40% of patients with AS,
and tenderness to palpation in the right upper
and presents with unilateral eye pain, blurred
Test Block 8

quadrant, but the patient would not likely pre-


vision, and photophobia. The disorder is
sent with such overt signs of a serious infection
treated with local steroids and atropine, and of-
(hypotension, high-grade fevers, and tachycar-
ten recurs. AS is also associated with fracture of
dia/tachypnea). Acute cholecystitis can cause
the spine, aortic regurgitation, IgA nephropa-
mild jaundice, but the magnitude of this pa-
thy, and bowel ulcerations.
724 Section II: Full-Length Examinations Answers

tients problem is too great for that diagnosis. childhood with recurrent bacterial and fungal
Although the ultrasound indicated stones in infections and granuloma formation in the skin,
the gallbladder, the major problem is biliary gastrointestinal, and genitourinary tracts. Move-
obstruction (most likely from choledocholithia- ment function is unaffected in these patients.
sis) and the resulting infection (shown by dila-
Answer B is incorrect. Blocked lysosomal traf-
tion of intrahepatic ducts).
cking is seen in mucolipidosis III, a rare lyso-
Answer C is incorrect. The patient is still pass- somal storage disease leading to accumulated
ing stool, although it is light colored. The pa- glycoproteins and glycolipids. In this autosomal
tient has normal bowel sounds and no disten- recessive disorder (also called pseudo-Hurlers
sion of the abdomen and has focal rather than syndrome), symptoms develop at 35 years of
diffuse tenderness. Also, jaundice or elevated age, and include skeletal and joint abnormali-
liver enzyme levels would not be seen. ties, coarse facial features, and corneal cloud-
ing. Ataxia is not a feature of this disease.
Answer D is incorrect. Hepatitis has a more
subacute onset and does not have many of the Answer D is incorrect. Defects in peroxisome
signs of acute bacterial infection on its way to function are seen in many rare disorders of in-
septic shock, which are presented in this case. born errors of metabolism, the most common
The jaundice and right upper quadrant tender- of which is adrenoleukodystrophy (ALD). In
ness often follow a viral prodrome that includes this disease, very long-chain fatty acids accu-
fever, nausea, vomiting, and upper respiratory mulate in cells and lead to dysfunctional my-
tract symptoms. Lab tests would reveal a nor- elination. Usually, ALD presents in childhood
mal WBC count and markedly elevated liver with seizures, ataxia, adrenal insufciency, and
function tests. loss of sight and hearing. Telangiectasias are
not seen with this disorder.
Answer E is incorrect. Cancer in the head of
the pancreas commonly presents with painless Answer E is incorrect. Although genetic de-
jaundice and is not often accompanied by such fects in toll-like receptor signaling have been
obvious signs of infection as spiking fevers and identied, they are very uncommon and do not
elevated WBC count. The right upper quad- cause clinically signicant immunodeciency.
FULL-LENGTH EXAMS

rant would likely not be tender, but a painless


palpable gallbladder (Courvoisiers sign) could 7. The correct answer is E. Patients with diabetes
be present. Evidence of impending sepsis (hy- are at increased risk of developing nonhealing
potension, tachycardia, and tachypnea) is not ulcers secondary to distal symmetric polyneu-
often a presenting feature. ropathy, peripheral arterial insufciency, lim-
ited joint mobility and bony deformities, obe-
6. The correct answer is C. Ataxia-telangiectasia sity, and chronic hyperglycemia. Patients may
is a syndrome with cutaneous, immunologic, present with lesions ranging from a small blis-
neurologic, and endocrinologic abnormalities. ter or cellulitis to a ulcer involving the bone as
Patients have oculocutaneous telangiectasias in the case described in the stem. The best
with progressive cerebellar ataxia that eventu- treatment is tight glucose control combined
ally leads to connement to a wheelchair, of- with patient education about daily foot inspec-
ten by the age of 1012 years. Additional nd- tion, appropriate footwear, drying and nail
ings often include a selective IgA absence with trimming, and referral to a podiatrist when
low concentrations of IgE; IgM is often present necessary. Treatment of ulcers may include de-
in the low-molecular-weight form. Ataxia- brdement, growth-stimulating factors, reduc-
telangiectasia is an autosomal recessive disor- tion of weight-bearing, and improvement in ar-
der that is due to defective DNA repair and re- terial supply. Penetrating infections are due to
Test Block 8

sults from a mutation in the ATM gene. secondary infection with anaerobes and or
other bacilli, most commonly Pseudomonas
Answer A is incorrect. Absent respiratory burst
aeruginosa. The correct answer includes a
is the biochemical hallmark of chronic granu-
broad-spectrum penicillin and an aminoglyco-
lomatous disease. These patients present in
side that are able to combat such organisms.
Test Block 8 Answers 725

Answer A is incorrect. Clindamycin, a lincos- Answer B is incorrect. Catheterization is both


amide antibiotic, treats anaerobic infections in- therapeutic and diagnostic for overow in-
cluding those caused by gram-negative bacilli. continence (secondary to outow inhibition).
This medication may be used as second-line In this scenario, catheterization would likely
treatment of such an infection when paired demonstrate a normal or low-normal bladder
with ciprooxacin, but it is considered less suit- volume.
able than piperacillin and gentamicin, second-
Answer C is incorrect. Bethanechol chloride
ary to increased side effects (including eradica-
is a parasympathomimetic drug that can be
tion of commensals) and higher cost.
used to test whether incontinence is due to a
Answer B is incorrect. Doxycycline is a tetracy- lower motor neuron lesion or a muscle lesion.
cline. It is commonly used to treat Haemophilus Lack of a detrusor contraction to bethanechol
inuenzae, Streptococcus pneumoniae, Myco- chloride administration suggests muscle dam-
plasma pneumoniae, Chlamydia psittaci, Chla- age, and an increased response suggests lower
mydia trachomatis, and Neisseria gonorrhoeae. motor neuron involvement. In this womans
It will not combat Pseudomonas aeruginosa and case, her response should be normal.
other anaerobes that inhabit diabetic ulcers.
Answer D is incorrect. A common cause of
Answer C is incorrect. Metronidazole is com- incontinence in multiparous women is pel-
monly used against agellated protozoa, and vic oor laxity resulting in sphincteric insuf-
works well against anaerobic bacteria, among ciency. However, this usually presents as stress
others; however, it is not very effective against incontinence, which is urine leakage in con-
Pseudomonas aeruginosa, a gram-negative bacil- ditions of increased abdominal pressure (e.g.,
lus commonly found in invasive diabetic ulcers. sneezing or coughing). This womans descrip-
tion of total incontinence is more consistent
Answer D is incorrect. Penicillin G is primar-
with a vesicovaginal stula.
ily used to treat staphylococcal and streptococ-
cal infections. It is not very effective against
9. The correct answer is C. To determine the
Pseudomonas aeruginosa, a gram-negative ba-
probability that a patient who tests positive for
cillus commonly found in penetrating diabetic
disease X actually does have the disease (the

FULL-LENGTH EXAMS
ulcers. Broad-spectrum penicillins like pipera-
positive predictive value of the test), one must
cillin are more effective.
know both the sensitivity of the test and the test
specicity. Both values can be derived from in-
8. The correct answer is E. Given that the urine
formation provided. However, to determine the
leakage is constant, worse when urine produc-
positive predictive value, one must also know
tion is increased (by caffeine or volume), and
the prevalence of the disease so the absolute
began after childbirth, the most likely cause of
number of true and false positives can be cal-
this womans incontinence is a vesicovaginal
culated: positive predictive value = (true posi-
stula. Fistulas can develop following trauma
tives) / (true + false positives). Lower disease
(as during childbirth) and require surgical cor-
prevalence results in a lower positive predictive
rection. A stula can sometimes be directly vi-
value and results in a less reliable positive test.
sualized on physical examination, but a void-
ing cystourethrogram has more sensitivity. Answer A is incorrect. The positive predictive
value of a test depends on the prevalence of
Answer A is incorrect. Increased bladder ca-
the disease within a specic population, but no
pacity can be seen in cases of bladder outlet
population-specic information is given.
obstruction (prostatic hypertrophy or stricture)
but is usually associated with overow inconti- Answer B is incorrect. The incidence of a dis-
Test Block 8

nence (which is seen when the bladder is full). ease is the number of new disease cases per
In this patient, her bladder capacity should population at risk over a specied time inter-
be normal or even low, based on the fact that val. This information is not necessary in deter-
urine continuously leaks out and does not al- mining the positive predictive value of a test.
low the bladder to reach full capacity.
726 Section II: Full-Length Examinations Answers

Answer D is incorrect. The false-positive ratio, Answer E is incorrect. Sepsis causes a conju-
listed as 10%, is equal to 1 sensitivity. There- gated hyperbilirubinemia. The mothers group
fore, we know the test has 90% sensitivity. B Streptococcus status puts this boy at increased
risk for neonatal sepsis. However, aside from his
Answer E is incorrect. Specicity can be cal-
jaundice, this child is otherwise well appearing,
culated as 1 false-positive rate. From the in-
which would not be the case if he was septic.
formation given, we know that the specicity is
75%.
11. The correct answer is E. Any sold, rm, testic-
ular mass should be considered testicular can-
10. The correct answer is B. Breast-feeding jaun-
cer unless proven otherwise. The use of trans-
dice is the most common cause of nonphysio-
scrotal ultrasound demonstrating a solid mass
logic jaundice. It occurs in neonates who are
is enough to conrm the diagnosis. Treatment
being exclusively breast-fed, usually because of
for testicular cancer always involves radical or-
poor intake. Decreased milk intake leads to in-
chiectomy of the aficted testicle.
creased enterohepatic circulation and abnor-
mally high reabsorption of bilirubin that is nor- Answer A is incorrect. Conrmation of a solid
mally excreted in the feces. Jaundice is usually mass on transscrotal ultrasound is enough to
apparent on the third day of life and peaks at conrm the diagnosis of testicular cancer. In
>12 mg/dL between days 3 and 6. In contrast general, biopsy of a testicular mass is never
with physiologic jaundice, total and conju- done for the diagnosis of cancer.
gated bilirubin levels are higher.
Answer B is incorrect. Although nonsemi-
Answer A is incorrect. Obstructive jaundice nomatous testicular cancer is generally sensi-
in a newborn is most often caused by biliary tive to platinum-based chemotherapy, this mo-
atresia. It is usually diagnosed when jaundice dality is generally reserved for advanced disease
persists for >2 weeks and is accompanied by or for after orchiectomy, and not as neoadju-
acholic stools and/or dark urine. However, at vant therapy before orchiectomy.
this age (2 days old) breast-feeding jaundice is
Answer C is incorrect. The -fetoprotein level
much more likely.
is elevated, suggesting nonseminomatous germ
FULL-LENGTH EXAMS

Answer C is incorrect. Rh isoimmune hemo- cell testicular cancer. In contrast to semino-


lytic disease of the newborn causes immediate mas, these cancers are not radiosensitive.
severe unconjugated hyperbilirubinemia, ane-
Answer D is incorrect. The -fetoprotein level
mia, and eventually increased hematopoiesis
is elevated, suggesting a nonseminomatous
(including frontal bossing). This diagnosis is
germ cell testicular cancer. In contrast to semi-
supported by a positive Coombs test. This oc-
nomas, these cancers are not radiosensitive.
curs when Rh-negative women have previously
been exposed to Rh factor and subsequently 12. The correct answer is D. Idiopathic thrombo-
make Rh antibodies that cross the placenta cytopenic purpura (ITP) is associated with iso-
and destroy fetal RBCs. Giving Rh-negative lated thrombocytopenia that results from pri-
women Rh0(D) immune globulin when they mary immune-mediated platelet destruction.
are exposed to Rh-positive blood can prevent This may cause prolonged bleeding time, but
Rh isoimmune hemolytic disease of the new- it would have no impact on the PT or aPTT
born. and is not associated with abnormalities of co-
Answer D is incorrect. Physiologic jaundice agulation factors.
results in a mild unconjugated hyperbiliru- Answer A is incorrect. ITP is not associated
binemia after the third day of life, and resolves with abnormalities of coagulation factor IX
Test Block 8

by 1 week in full-term neonates and 2 weeks


and is associated with decreased platelet levels
in preterm neonates. Notably, the conjugated
not seen in these laboratory values. These lab-
bilirubin is always normal, and the total bili-
oratory values also display an increased PT and
rubin level is always <14 g/dL in physiologic
aPTT. A deciency in factor IX would only
jaundice.
lead to a prolonged PTT, not PT.
Test Block 8 Answers 727

Answer B is incorrect. ITP is not associated consistent with sick euthyroid syndrome, or
with abnormalities of coagulation factor IX. low triiodothyronine and thyroxine concentra-
Due to thrombocytopenia seen in ITP, there tions accompanied by low thyroid-stimulating
will be a prolonged bleeding time and de- hormone levels and high reverse triiodothyro-
creased platelet count. Factor IX is part of the nine levels.
intrinsic coagulation pathway and would in-
Answer C is incorrect. A pituitary adenoma
deed lead to increased PTT and a diagnosis of
can present with a variety of symptoms, in-
hemophilia B. Hemophilia B is associated with
cluding eye problems (bitemporal hemiano-
normal platelet counts but abnormal aPTT.
psia), acromegaly, and hyperprolactinemia.
Answer C is incorrect. These laboratory val- Although high levels of prolactin can lead to
ues are signicant for a prolonged aPTT and amenorrhea, this case is much more suggestive
abnormal levels of factor VII. Factor VII is a of amenorrhea secondary to anorexia nervosa.
coagulation factor in the extrinsic pathway.
Answer E is incorrect. Small bowel obstruc-
Therefore, abnormal levels of factor VII should
tion is usually associated with severe cramping,
lead to increased PT, not aPTT. A deciency
abdominal pain, nausea, and vomiting. The
of coagulation factors is not seen in ITP.
patient has no abdominal pain, cramping, nau-
Answer E is incorrect. ITP is not associated sea, or vomiting, and because this patient has
with abnormalities of coagulation factor VII. no history of previous surgeries, small bowel
These values show a prolonged PT and abnor- obstruction is highly unlikely.
mal levels of factor VII. Factor VII is part of the
extrinsic pathway, and decreased levels should 14. The correct answer is E. This patients history,
lead to prolonged PT. However, ITP is associ- physical examination ndings, and laboratory
ated with decreased platelets and is not associ- ndings are suggestive of pelvic inammatory
ated with abnormal coagulation factors. disease (PID). This sexually active woman has a
history of multiple sexual partners and does not
Answer F is incorrect. These laboratory values
use condoms consistently. In addition, abnormal
are signicant for thrombocytopenia and pro-
vaginal discharge may indicate a history of gon-
longed bleeding time and PT. ITP causes only
orrhea, chlamydia, or other sexually transmitted

FULL-LENGTH EXAMS
a prolonged bleeding time because of the de-
infections. Minimal criteria for a diagnosis of
crease in platelet number. Thrombocytopenia
PID include abdominal pain, cervical motion
has no effect on the coagulation cascade and
tenderness, and/or purulent cervical discharge.
therefore should not prolong PT or aPTT.
Because the differential diagnosis also includes
appendicitis, ectopic pregnancy, adnexal torsion,
13. The correct answer is D. Anorexia nervosa is a
diverticulitis, ileitis, and ulcerative colitis, further
serious psychiatric disorder characterized by
tests should be conducted to work up this pa-
refusal to maintain a normal body weight, dis-
tient. An ultrasound may be used to rule out an
torted body image, and amenorrhea for at least
abscess resulting from this patients PID and can
3 months. Secondary amenorrhea is seen in
also be used to rule out many of the other differ-
>90% of women with anorexia nervosa.
ential diagnoses.
Answer A is incorrect. The patients ne hairs
Answer A is incorrect. X-rays of the abdomen
are most consistent with lanugo, often seen in
have limited utility in diagnosing PID. Al-
patients with anorexia nervosa and other condi-
though an x-ray of the abdomen may be useful
tions of starvation. Lanugo is the bodys attempt
for ruling out some of the potential diagnoses,
to insulate itself as the percentage of body fat
an ultrasound has greater utility in this case
decreases. Hirsutism is the abnormal growth of
because the history, physical examination, and
Test Block 8

dark thick hair in women in areas where hair


laboratory ndings suggest PID.
growth is usually absent or minimal.
Answer B is incorrect. Patients with anorexia
nervosa can present with abnormal thyroid
tests. However, these laboratory values are most
728 Section II: Full-Length Examinations Answers

Answer B is incorrect. This patients presenta- cirrhosis, such as jaundice, hepatomegaly, or


tion is consistent with PID. Although she may caput medusa. Effusions secondary to cirrhosis
be interested in starting oral contraceptives are similar in character to those produced by
because she is sexually active, this would not congestive heart failure, and clinically they are
be an appropriate next step in a patient with usually unilateral and on the right side.
gynecologic pathology. This patient must be
Answer B is incorrect. Collagen vascular dis-
diagnosed and treated appropriately for her
ease such as rheumatoid arthritis and lupus
condition to reduce the risk of long-term com-
can cause exudative effusions, often with low
plications.
pH and decreased glucose concentration. In
Answer C is incorrect. Laparoscopy provides this patient, however, there are none of the
a denitive diagnosis in the case of PID. How- classic symptoms of rheumatic disease.
ever, laparotomy is not indicated in the diag-
Answer C is incorrect. Although the patient
nostic work-up of a patient with suspected PID.
has a history of congestive heart failure, the
Because the patient is otherwise healthy, non-
presentation of this effusion is atypical for
invasive diagnostic studies such as ultrasound
those caused by heart failure. When pulmo-
and subsequent treatment with antibiotics are
nary vascular pressures rise secondary to left
indicated before considering invasive proce-
ventricular dysfunction, uid shifts across the
dures.
vascular endothelium into both lung elds in
Answer D is incorrect. A potassium hydrox- 80% of cases. The effusion is commonly clear
ide whiff test is useful in making a diagnosis of or straw colored with few WBCs. X-ray of the
bacterial vaginosis. These patients usually pres- chest often shows evidence of cardiomegaly.
ent with vaginal discharge and vulvovaginal
Answer E is incorrect. Parapneumonic effu-
pruritus. The presentation and history in this
sions are caused by local disruption of pulmo-
case is not consistent with a diagnosis of bacte-
nary vasculature, often secondary to a bacterial
rial vaginosis and, therefore, the whiff test has
pneumonia. The effusion is commonly puru-
limited utility.
lent with a predominance of polymorphonu-
clear cells.
15. The correct answer is D. Pleural effusion sec-
FULL-LENGTH EXAMS

ondary to malignancy is most commonly due


16. The correct answer is C. The 5-year-old son
to primary lung cancer, although other sources
should start undergoing screening and colonos-
such as breast and lymphoma are seen. The fe-
copy starting at 10 years old according to re-
ver and blood-tinged sputum are some mani-
cent guidelines. Familial adenomatous polypo-
festations of the likely endobronchial location
sis (FAP), caused by mutation of the APC gene,
of the mass, and her long history of smoking is
can lead to the development of hundreds to
the major risk factor. The development of pleu-
thousands of colonic polyps capable of malig-
ral effusion represents regional spread of the
nant transformation (risk approaches 100% by
malignancy. Thoracentesis often reveals a tur-
age 45 years). These polyps tend to develop in
bid or bloody uid with a lymphocytic pre-
the second and third decades of life with a
dominance and a normal pH and glucose con-
mean age of 16 years, but they can appear in
centration. However, it is important to
those as young as 8 years old. All patients with
remember that up to 30% of patients can have
FAP will require colectomy because of the ex-
a low pH and up to 20% a low glucose because
tremely high risk of cancer, but often this is de-
low pH and low glucose correlate with a higher
layed as long as possible due to the psychologi-
burden of tumor cells in the uid. Therefore,
cal consequences. Colonoscopy is needed for
if an exudate has a low pH or glucose, malig-
surveillance prior to the surgical removal to
Test Block 8

nancy is still on the differential, although it is


monitor for multiple large (>1 cm) adenomas
less likely.
or adenomas with villous histology, and/or
Answer A is incorrect. Although the patient high-grade dysplasia, any of which would ne-
has a history of alcohol use, she does not have cessitate the removal of the colon earlier than
any other symptoms or signs associated with planned.
Test Block 8 Answers 729

Answer A is incorrect. A digital rectal exami- low as compared to procainamide. The most
nation would be insufcient to detect early common adverse effects of clonidine are drows-
polyps that may be in the colon. iness or dizziness.
Answer B is incorrect. Age 40 years is too late Answer D is incorrect. This patient is taking
to start looking for signs of colorectal cancer in methimazole for hyperthyroidism, but it is not
someone with a rst-degree relative with FAP. a drug reported to cause DILE. However, pro-
Also, a digital rectal examination would be in- pylthiouracil (another antithyroidal) has been
sufcient to detect early polyps that may be in associated with a low risk of causing DILE.
the colon. Methimazole infrequently causes a number of
nonspecic adverse effects, such as gastrointes-
Answer D is incorrect. Age 30 years is too late
tinal complaints and a rash.
to start screening in someone with a family his-
tory of FAP.
18. The correct answer is A. This patient is mani-
Answer E is incorrect. Age 40 years is when a festing signs of cerebral edema, which can oc-
person with a rst-degree relative with colorec- cur with treatment and management of dia-
tal cancer should begin sigmoidoscopic screen- betic ketoacidosis (DKA). Risk factors include
ing (or 10 years before the earliest age of onset younger age, new diagnosis of type 1 diabetes
of colorectal cancer in the family). Again, this mellitus, severe acidemia at initial presenta-
age would be too late to screen somebody with tion, elevated blood urea nitrogen at presenta-
a positive family history of FAP. tion, failure of serum sodium to rise with insu-
lin and uid therapy, and use of bicarbonate
17. The correct answer is E. This patient exhibits therapy in treatment. DKA therapy likely exac-
signs of drug-induced lupus erythematosus erbates but does not initiate the pathologic pro-
(DILE), a milder version of systemic lupus ery- cesses leading to cerebral edema. Patients nor-
thematosus that generally resolves within weeks mally initially present with symptoms of
to months after discontinuation of the offending headache that begin 412 hours after begin-
medication. Severe but noninammatory joint ning treatment for DKA. Later signs include
pain is common in patients with DILE, as is hypertension, bradycardia, pupil abnormalities,

FULL-LENGTH EXAMS
myalgia. Patients may also experience a low- and decerebrate or decorticate posturing. The
grade fever. The usual absence of central ner- time course and symptoms reported are consis-
vous system (CNS) and renal involvement in tent with this diagnosis. Management of cere-
DILE helps to distinguish it from systemic lupus bral edema includes decreasing the rate of ad-
erythematosus. While DILE may be induced by ministration of intravenous uids, and starting
a number of medications, the two most com- the administration of intravenous mannitol. If
mon culprits are procainamide and hydralazine. there is no response, a second dose can be
given in 2 hours. Based on trending of the pa-
Answer A is incorrect. Use of amitriptyline
tients laboratory tests, other steps in manage-
is not a known risk factor for DILE. Amitrip-
ment (less urgent) include adding dextrose to
tyline most commonly causes anticholinergic
intravenous uids (when glucose is <250300
adverse effects (blurred vision, constipation,
mg/dL to prevent hypoglycemia), and switch-
dry mouth, light-headedness, confusion, and
loss of urinary control). ing the insulin infusion to a subcutaneous in-
sulin regimen (upon closure of the anion gap,
Answer B is incorrect. While captopril has serum bicarbonate >16 mEq/L, or pH >7.30).
been reported to cause DILE, the risk is very
low as compared to procainamide. Four to seven Answer B is incorrect. Increasing the rate of
insulin infusion is not necessary in this patient,
percent of patients report an urticarial or macu-
Test Block 8

whose hyperglycemia is rapidly resolving. An


lopapular rash on captopril, and <2% report a
increase in insulin administration will further
cough or dysgeusia (diminished sense of taste).
lower serum glucose and plasma osmolality,
Answer C is incorrect. While clonidine has promoting additional water movement in the
been reported to cause DILE, the risk is very brain. Furthermore, since this patients bi-
730 Section II: Full-Length Examinations Answers

carbonate level is >16 mEq/L, she should be elevated intracranial pressure. Cerebral edema
transitioned to subcutaneous insulin. If she is treated by a decrease in administration of in-
is started on a lispro regimen, the rst dose travenous uids and the administration of in-
should be given 15 minutes before discontinu- travenous mannitol.
ation of insulin infusion. If she is started on
regular insulin, the rst dose should be given 19. The correct answer is E. The patients com-
an hour before discontinuation of insulin in- plaints, as well as the potassium hydroxide
fusion. While adjustment of the insulin regi- preparation, are suggestive of vaginal candidia-
men is important, in this patient with cerebral sis. Miconazole and uconazole both treat
edema, administration of mannitol is the most candidiasis; however, uconazole is an inhibi-
critical next step in management. tor of the cytochrome P450 enzyme system
and would lead to increased serum concentra-
Answer C is incorrect. In this patient with
tions of phenytoin, possibly leading to toxicity.
signs of cerebral edema, it is necessary to de-
Topical miconazole avoids this risk.
crease the rate of intravenous uid adminis-
tration instead of increasing it. Increasing the Answer A is incorrect. Clindamycin could be
rate of uid administration will increase intrac- used to treat bacterial vaginosis. Bacterial vagi-
ranial pressure and worsen cerebral edema. nosis is caused by Gardnerella vaginalis. It is
Also, in initial management of DKA, it is im- associated with an increased amount of a thin,
portant to use isotonic uids (Ringers lactate gray, liquid, homogenous vaginal discharge
or normal saline) instead of hypotonic uids and an often shy odor that is especially recog-
to reduce chances of cerebral edema. Fluid nizable after sexual intercourse.
repletion should proceed at a carefully calcu- Answer B is incorrect. Doxycycline is appro-
lated rate to replete uid status over 48 hours, priate treatment for Chlamydia infection and
but not any faster. The rate over the rst 24 may lead to candidal vulvovaginitis. In acute
hours should not exceed 1.52 times the main- vulvovaginal candidiasis, vulvar pruritus and
tenance uid rate. Higher rates increase the burning are the main symptoms with erythema
chance of cerebral edema. and edema of the vestibule and labia majora
Answer D is incorrect. ECGs are important and minora. Chronic candidiasis includes
FULL-LENGTH EXAMS

and necessary in the management of patients edema and lichenication of the vulva, often
with DKA who are hypokalemic or hyper- with a grayish sheen. This patient is typically
kalemic, to monitor for cardiac arrhythmias. older, obese, and often has long-standing dia-
However, this patients potassium is in the nor- betes mellitus.
mal range. Initial serum potassium was in the
Answer C is incorrect. Metronidazole would
low-normal range, but with potassium supple-
not treat Candida but could be used to treat
mentation in intravenous uids, it quickly
Trichomonas infection. Trichomonas presents
reached a normal stable level. It would not be
with a yellow discharge, dyspareunia, and vul-
wrong to obtain an ECG if there were suspi-
var itching.
cion for electrolyte abnormalities, but it should
not be the next step in this patient with cere- Answer D is incorrect. Oral uconazole is in-
bral edema. correct because it would increase serum levels
of phenytoin through P450 metabolism inter-
Answer E is incorrect. In this patient, there is action. Topical miconazole is the safer choice
no indication for a lumbar puncture. A lum- in a patient on phenytoin.
bar puncture may be useful to evaluate men-
ingitis as a cause of headache, but this patient 20. The correct answer is D. This patient has the
is exhibiting signs and symptoms of cerebral
Test Block 8

classic presentation of an epidural hematoma.


edema as a complication of DKA treatment. If Trauma to the side of the skull (most com-
a lumbar puncture were necessary for further monly affecting the middle meningeal artery)
evaluation, a head CT to evaluate for possibil- is immediately followed by a lucid interval. As
ity of herniation must always be obtained prior blood accumulates in the hematoma and com-
to lumbar puncture in a patient with suspected
Test Block 8 Answers 731

presses the brain, progressive obtundation is the hemorrhage. However, this would not be
seen, often accompanied by a xed, dilated, the case because of the relatively short period
blown pupil (60% of patients with epidural between injury and treatment in the emer-
hematoma present with a blown pupil, 85% of gency department.
which are ipsilateral). Because the blood is in
the epidural space, it is bound by the sutures, 21. The correct answer is D. The classic triad of
and therefore the classic nding is a lens- tuberous sclerosis is mental retardation, seizure
shaped, convex hyperdensity. activity, and facial hamartoma, in addition to
the aforementioned heart rhabdomyoma, intra-
Answer A is incorrect. In a subarachnoid hem-
ventricular hamartoma, and angiomyolipoma
orrhage, blood is seen in the subarachnoid
of the kidney. Other dermatologic lesions in-
space. The common presentation of a sub-
clude: ash leaf spots (hypopigmented macules),
arachnoid hemorrhage is acute onset headache
and shagreen spots (leathery cutaneous thick-
(the worst headache of my life), often accom-
ening). Tuberous sclerosis may occur sporadi-
panied by neck stiffness. Although it can occur
cally or with familiar autosomal dominant in-
secondary to trauma, the onset is much more
heritance. Most children present with seizures.
likely to be immediate and acute in onset, in-
stead of delayed and more progressive, as is the Answer A is incorrect. There is no known as-
case in epidural hematomas. sociation between glioblastoma and tuberous
sclerosis. Glioblastoma multiforme is a brain
Answer B is incorrect. A crescent-shaped,
tumor that commonly presents with nonspe-
concave hyperdensity that does not cross the
cic complaints and increased intracranial
midline is seen in a subdural hematoma. This
pressure. It is a rapidly progressive cancer, and
is caused by tearing of the bridging veins that
the most common primary brain cancer.
penetrate the dura and perfuse the brain pa-
renchyma. Although this is a possibility in this Answer B is incorrect. There is no known as-
patient, it is much more likely in persons who sociation between liver adenocarcinoma and
are elderly and/or alcoholics. Therefore, this tuberous sclerosis. Hepatocellular carcinoma
patients demographics, as well as the lucid in- is the most common primary liver malignancy.
terval between injury and onset of symptoms, It is associated with cirrhosis, hepatitis B virus,

FULL-LENGTH EXAMS
make it much more likely to be an epidural hepatitis C virus, alcoholism, hemochromato-
hematoma. sis, Wilsons disease, and 1-antitrypsin de-
ciency.
Answer C is incorrect. An irregularly shaped
hyperdensity within the parenchyma is the Answer C is incorrect. There is no known as-
most likely radiologic nding in a parenchymal sociation between pancreatic adenocarcinoma
hemorrhage. This scenario is less likely than and tuberous sclerosis. Most pancreatic cancers
an epidural hematoma in this patient because are adenocarcinoma located in the head of the
parenchymal hemorrhage is more commonly pancreas. Pancreatic cancer is more often seen
associated with hypertension, tumors, vascu- in African-Americans, cigarette smokers, males,
lar malformations, and amyloid angiopathy. and people with diabetes or chronic pancreati-
Although a parenchymal bleed may occur in tis. Pancreatitis presents with abdominal pain,
severe head trauma, the lucid interval would weight loss, jaundice, and occasionally migra-
not have been observed. tory thrombophlebitis (Trousseaus sign).
Answer E is incorrect. Given that this patient
22. The correct answer is F. The most likely
does not have a history of neurologic disorder
pathogen for urinary tract infection (UTI) in
and given the temporal relationship between
this age group is Escherichia coli. Staphylococ-
Test Block 8

his injury and the onset of symptoms, it is


cus saprophyticus is the cause of approximately
highly unlikely that he would have a normal
10% of uncomplicated UTIs in sexually active
CT scan. The blood of intracranial hemor-
college-age women.
rhage is resorbed, so it is possible that days af-
ter the injury there is no radiologic evidence of
732 Section II: Full-Length Examinations Answers

Answer A is incorrect. Although the patient because the capillary supply is intact. The epi-
may be at risk for sexually transmitted disease, dermis and less than half of the dermis are in-
the symptoms are more consistent with a lower volved in supercial second-degree burns.
UTI. Nevertheless, urinary symptoms in sexu-
Answer D is incorrect. Supercial, or rst-
ally active young adults should never be as-
degree, burns are quite painful, erythematous,
sumed to be due to a UTI unless cervicitis and
and blanch with pressure. Because only the
PID have been considered.
epidermis is involved, these burns are dry and
Answer B is incorrect. While most UTIs are do not blister.
caused by gram-negative bacteria, Enterococ-
cus faecalis is one of the gram-positive organ- 24. The correct answer is A. At this point in the
isms that can be involved, often in the setting Advanced Cardiac Life Support algorithm for
of nosocomial infections. pulseless ventricular tachycardia, amiodarone
should be administered. If amiodarone is un-
Answer C is incorrect. Klebsiella is a common
available, lidocaine or magnesium sulfate are
cause of UTIs but is not the most likely patho-
acceptable alternatives.
gen listed.
Answer B is incorrect. Digoxin is not indicated
Answer D is incorrect. Gonococcal urethritis/
for ventricular tachycardia. It is primarily used
cervicitis would likely have a different clinical
for supraventricular arrhythmias.
presentation. Nevertheless, urinary symptoms
in sexually active young adults should never be Answer C is incorrect. Diltiazem is not indi-
assumed to be due to a UTI unless cervicitis/ cated for ventricular tachycardia. It is useful for
PID have been considered. supraventricular arrhythmias.
Answer E is incorrect. Pseudomonas is com- Answer D is incorrect. Metoprolol is not indi-
monly associated with nosocomial infections cated in pulseless ventricular tachycardia, but
of the bladder and would be unlikely given the it is useful for polymorphic stable ventricular
clinical scenario. tachycardia.
Answer E is incorrect. Procainamide would be
23. The correct answer is A. This is a classic de-
FULL-LENGTH EXAMS

indicated for a stable ventricular tachycardia,


scription of a deep partial-thickness or deep
but not pulseless ventricular tachycardia.
second-degree burn. The epidermis and most
of the dermis are involved. There is some loss Answer F is incorrect. Tocainide is often used
of pain sensation, but pain with deep pressure for long-term management of ventricular ar-
is still experienced. The capillary supply is rhythmias, as it is available in an oral formula-
somewhat compromised in these deeper burns, tion.
and blanching may not occur when pressure is
applied. These burns can be difcult to distin- 25. The correct answer is D. The diagnosis is
guish from full-thickness burns, and careful Guillain-Barr syndrome, suggested by the pa-
observation may lead to reclassication as the tients weakness and recent vaccination. It is an
lesions evolve. inammatory demyelinating polyneuropathy
characterized by the acute onset of muscle
Answer B is incorrect. Full-thickness, or third- weakness, areexia, and eventually accid pa-
degree, burns involve both the epidermis and ralysis, commonly occurring after a viral infec-
the entire dermis. The area is typically pain- tion of the upper respiratory or gastrointestinal
less and white-grey to black, dry, and does not tract. The ascending paralysis of Guillain-Barr
blanch. syndrome is somewhat unpredictable and can
Test Block 8

Answer C is incorrect. Supercial partial- progress rapidly (over hours) to compromise re-
thickness or supercial second-degree burns spiratory function. First-line treatment consists
are typically quite painful and involve blister- of plasmapheresis and intravenous immune
ing. Blanching occurs when pressure is applied globulin. The presumed mechanism of action
Test Block 8 Answers 733

of both is inhibition of antibody-mediated au- in the general population. Symptoms of CNS


toimmune demyelination, although the details lymphoma can be focal or nonfocal, and the
remain obscure. development of the disease is usually associ-
ated with a very low CD4+ count (i.e., <50/
Answer A is incorrect. Antithymic immuno-
mm).
globulin has not been found to be benecial
in Guillain-Barr syndrome. Answer B is incorrect. The JC virus is not as-
sociated with CNS lymphoma. The disease
Answer B is incorrect. Corticosteroids have
caused by the JC virus is known as progres-
not been found to be benecial in Guillain-
sive multifocal leukoencephalopathy, which
Barr syndrome.
can present with focal or nonfocal neurologic
Answer C is incorrect. Cyclosporine and tac- symptoms over a short period of time. An MRI
rolimus have not been found to be benecial of the brain shows nonenhancing white mat-
in Guillain-Barr syndrome. ter lesions. Polymerase chain reaction assays
for the JC virus are 80% sensitive and 95%
Answer E is incorrect. Tumor necrosis factor-
specic. The median duration of survival is ap-
receptor blockers have not been found to be
proximately 16 months after diagnosis; there
benecial in Guillain-Barr syndrome.
is no specic treatment.
26. The correct answer is D. Achalasia is a condi- Answer C is incorrect. Mycobacterium avium
tion that commonly leads to dysphagia, or dif- complex can infect the lung and the gastroin-
culty swallowing. Manometry is an excellent testinal tract and can become disseminated,
test to assess for achalasia, and classically shows but it is not associated with primary CNS lym-
increased LES tone, impaired LES relaxation, phoma.
and aperistalsis. Besides the birds beak ap-
Answer D is incorrect. Previous radiation ex-
pearance on barium swallow, the esophagus it-
posure is associated with the development of
self will commonly be dilated.
many other types of cancers, including leu-
Answer A is incorrect. Decreased LES tone kemia (e.g., chronic myelogenous leukemia),
would be expected to lead to symptoms of gas- thyroid cancer, and multiple myeloma, but it

FULL-LENGTH EXAMS
troesophageal reux, the opposite result from is not specically linked to the development of
the dysphagia that is associated with achalasia. CNS lymphoma.
Answer B is incorrect. In addition to demon- Answer E is incorrect. Infection with Toxo-
strating on manometry increased LES tone and plasma gondii leads to encephalitis that can
impaired LES relaxation, achalasia is marked present with focal neurologic decits. MRI of
by aperistalsis. the brain will demonstrate ring-enhancing le-
Answer C is incorrect. One of the fundamen- sions. More than 90% of patients will test posi-
tal elements of achalasia is impaired LES re- tive for anti-Toxoplasma gondii IgG. Treatment
laxation that does not allow for substances to is with pyrimethamine, leucovorin, and sulfa-
easily pass the LES. Therefore, normal LES re- diazine, and most patients respond within 1
laxation would not be expected to create symp- week. T. gondii is not associated with the devel-
toms of dysphagia. opment of CNS lymphoma, but if such a re-
sponse does not occur, an alternative diagnosis
Answer E is incorrect. One of the hallmarks of (e.g., CNS lymphoma) should be considered.
achalasia is aperistalsis, not hyperperistalsis.
28. The correct answer is D. This patients symp-
27. The correct answer is A. Virtually all cases of toms of hyperphagia, weight gain, and hyper-
Test Block 8

primary CNS lymphoma in patients with HIV somnolence, are consistent with atypical de-
are associated with Epstein-Barr virus. Before pression. Sertraline is a selective serotonin
the use of highly active antiretroviral therapy, reuptake inhibitor (SSRI), which, like the
the frequency of CNS lymphoma was 1000 monoamine oxidase (MAO) inhibitors has a
times higher among HIV-positive patients than 55%75% success rate in the treatment of atyp-
734 Section II: Full-Length Examinations Answers

ical depression. Because adverse effects and 2 and 4 A.M. until between 10 and 11 A.M.
drug interactions are drawbacks to treatment Thus, as the sun rises (dawn phenomenon),
with MAO inhibitors, most patients with atypi- so does the blood glucose level.
cal depression are treated rst with an SSRI.
Answer B is incorrect. Decreasing the morn-
Adverse effects of MAO inhibitors include
ing insulin dose would exacerbate morning hy-
weight gain, dry mouth, and hypertensive crisis
perglycemia.
if the patient does not avoid tyramine-rich
foods, such as cheese. Other examples of com- Answer C is incorrect. Increasing the evening
monly used SSRIs include uoxetine, citalo- dose of insulin would exacerbate nocturnal
pram, and escitalopram. hypoglycemia and would therefore worsen re-
bound hyperglycemia.
Answer A is incorrect. Desipramine is a tricyclic
antidepressant. Although these drugs have been Answer D is incorrect. Increasing the morning
shown to be effective in the treatment of major dose of insulin treats the patients symptoms but
depression, atypical depression responds more not the cause of the problem. Although it would
preferentially to SSRIs and MAO inhibitors. decrease morning hyperglycemia, it would do
nothing to affect nocturnal hypoglycemia.
Answer B is incorrect. Lithium is a mood sta-
bilizer often used in the treatment of bipolar Answer E is incorrect. This patients symptoms
disorder. As this woman has no history of ma- are due to his insulin regimen, which must be
nia, a diagnosis of bipolar disorder is unlikely; adjusted to prevent continued morbidity.
therefore lithium would not be helpful.
30. The correct answer is C. Pseudohyperkalemia
Answer C is incorrect. Olanzapine is an atypi-
should always be considered when evaluating
cal antipsychotic medication used to treat agi-
hyperkalemia. Poor phlebotomy technique can
tation and the positive and negative symptoms
cause RBCs to hemolyze, which allows intra-
of schizophrenia. Two major adverse effects in-
cellular potassium to leak into the sample be-
clude somnolence and weight gain.
ing tested. The house ofcer, however, was
Answer E is incorrect. Zolpidem is a nonben- correct in checking the patient. If the clinical
zodiazepine used to treat insomnia. This pa- picture were suspicious, emergent measures to
FULL-LENGTH EXAMS

tient has no symptoms consistent with insom- protect the cardiac membrane from depolariz-
nia; therefore zolpidem is not indicated. ing and to lower serum potassium would be
necessary.
29. The correct answer is A. This patient is experi-
Answer A is incorrect. Insulin causes potas-
encing the Somogyi effect, which is episodic
sium to shift intracellularly. Administration of
nocturnal hypoglycemia followed by morning
insulin in an otherwise healthy person would
rebound hyperglycemia. Hypoglycemia is
cause hypokalemia.
caused by an excess of insulin at night, often as
a result of a change in the insulin regimen. Answer B is incorrect. Addisons disease re-
This leads to a hyperadrenergic state that con- fers to primary adrenal insufciency. This can
sists of increased gluconeogenesis and glyco- cause hyperkalemia. Addisons disease also
genolysis and decreased peripheral glucose up- causes hyponatremia, hypotension, fatigue,
take, all of which contribute to rebound anorexia, nausea, and vomiting. The patients
hyperglycemia. The appropriate management blood pressure and sodium level are normal,
is to decrease the nighttime dose of insulin to and his nausea is easily attributable to other
avoid hypoglycemia. This is in contrast to the causes (e.g., recent anesthetics or narcotics).
dawn phenomenon, which is morning hyper-
Answer D is incorrect. Gastric contents are
Test Block 8

glycemia that occurs in type 1 and type 2 dia-


acidic. Their removal, either by nasogastric
betes mellitus and in nondiabetics. Growth
tube or vomiting, causes alkalemia and vol-
hormone, an insulin antagonist, as well as cor-
ume depletion. Volume depletion results in
tisol, glucagons, and adrenaline, rise between
increased aldosterone levels, and alkalemia
Test Block 8 Answers 735

causes forced natriuresis due to spilling bicar- Answer D is incorrect. A lupus-like syndrome
bonate into the urine (sodium delivery to the has been reported as a potential adverse effect
distal nephron is enhanced due to obligatory of statin therapy but would not require imme-
bicarbonate loss in the urine). The combina- diate evaluation.
tion of distal sodium delivery and high aldos-
Answer E is incorrect. Myopathy from statin
terone levels leads to urine potassium excretion
initiation can sometimes be accompanied by
at the collecting duct. As a result, nasogastric
shortness of breath without identiable etiol-
suction is a cause of hypokalemia, not hyper-
ogy on cardiopulmonary work-up. This symp-
kalemia.
tom is reversible with statin discontinuation,
Answer E is incorrect. Renal failure may also unlike the potential for kidney damage with
cause hyperkalemia. However, this patient has widespread rhabdomyolysis.
a normal creatinine level, making renal failure
unlikely. 32. The correct answer is D. This is a pancreatic
pseudocyst, which is so named because the
Answer F is incorrect. Though orthopedic sur-
wall does not have an epithelial lining. Instead,
gery is the most common iatrogenic cause of
the wall is a brous capsule, formed by the
rhabdomyolysis, the clinical presentation is not
body to wall off the hemorrhage and fat necro-
suggestive of rhabdomyolysis, which is a cause
sis from pancreatitis or pancreatic trauma. This
of hyperkalemia. If the laboratory tests were re-
structure is not a true cyst and therefore does
peated and conrmed the diagnosis of hyper-
not communicate with the exocrine ductal sys-
kalemia, an elevated creatine phosphokinase
tem. They form as a result of pancreatic trauma
level would help diagnose muscle breakdown.
or pancreatitis, and are best visualized with CT
or ultrasound. If the cysts are small, they may
31. The correct answer is A. Patients who are new
be reabsorbed by the body and can therefore
to taking statin medications, as well as those
be managed medically. Larger cysts will need
whose doses have recently been changed, are
either internal (pancreaticogastrostomy) or ex-
at risk for rhabdomyolysis and renal damage
ternal (percutaneous) drainage. Internal drain-
from accumulation of creatinine. Serum cre-
age is favored if possible, with possible stula
atine kinase levels can be measured in patients

FULL-LENGTH EXAMS
formation if an external drainage procedure is
suffering from muscle pain to determine
used. Bacterial superinfection and rupture
whether tissue breakdown is occurring. Pa-
(leading to peritonitis) are two of the complica-
tients on statins can experience muscle pain
tions of pancreatic pseudocysts.
without creatine kinase elevation; one study
suggested that this may be due to reversible mi- Answer A is incorrect. New pancreatic pseudo-
tochondrial myopathy. Because of the potential cysts contain blood, necrotic debris (fat necro-
for progression to renal failure, a complaint sis and pancreatic autodigestion), and WBCs.
that might herald the onset of rhabdomyolysis, After a period of time the contents degenerate
such as limb aches or other musculoskeletal into a straw-colored, serous uid.
pain, should be taken seriously.
Answer B is incorrect. This patient had a pre-
Answer B is incorrect. Randomized trials have vious episode of pancreatitis, which can lead to
shown decline in memory and cognitive func- pseudocyst formation. Pancreatic trauma, such
tion with statin use, and it appears that the as impact of the steering wheel on the abdo-
elderly are at increased risk for these adverse men, can also lead to pseudocysts.
effects. Although troublesome and especially
Answer C is incorrect. This structure is not a
concerning from a quality-of-life standpoint,
true cyst and therefore does not communicate
this would not be an urgent issue.
Test Block 8

with the exocrine ductal system.


Answer C is incorrect. Headache is a nonspe-
Answer E is incorrect. Pancreatic pseudocysts
cic adverse effect of many types of medica-
are best visualized with abdominal ultrasound
tions, as well as being a common complaint in
or CT.
patients not on medications. Unless it has un-
usual features, it is not of particular concern.
736 Section II: Full-Length Examinations Answers

33. The correct answer is E. Status epilepticus re- agnosis, are best deferred until the patient has
fers to a continuous state of seizure activity or a been stabilized.
series of seizures during which there is no re-
turn to consciousness in the inter-ictal period. 34. The correct answer is A. The patient presents
The minimal duration of seizure activity in sta- with a very high suspicion for a thoracic aortic
tus epilepticus has traditionally been cited as dissection. This diagnosis is supported by a pre-
1530 minutes. Practically speaking, anyone sentation of severe, tearing chest pain that radi-
who is brought to an emergency department ates to the back, pressure differences between
who has been seizing for >5 minutes will be the right and left upper extremities, and a wid-
treated as having status epilepticus. It is a med- ened mediastinum on x-ray of the chest. The
ical emergency with a wide range of potentially transesophageal echocardiogram (TEE) is be-
lethal complications and 20% mortality. The coming the gold standard for diagnosing aortic
most important initial step in management, dissection. The information obtained from
however, is simply to attend to the ABCs: TEE is frequently used to guide patient man-
Airway, Breathing, and Circulation. Due to agement. TEE is a portable procedure that can
the inaccessibility of the oropharynx in such be performed at the bedside and can yield di-
patients, this is often best accomplished with agnostic information in just a few minutes. It
nasopharyngeal airway placement or even intu- allows for direct visualization of the thoracic
bation. If intubation is unsuccessful in a pa- aorta and determination of disease extent. TEE
tient who is in danger of aspirating, rapid se- can assess the ascending arch, aortic insuf-
quence induction must be employed to secure ciency, and proximal coronary arteries. With a
the patients airway. sensitivity of 97%99% and a specicity of al-
most 100% with the addition of M-mode imag-
Answer A is incorrect. At many centers intra-
ing, TEE is the best test for further work-up of
venous glucose, thiamine, and naloxone, and
an aortic dissection and for guiding manage-
oxygen by facemask are given presumptively
ment.
to all unconscious patients that arrive in the
emergency department. While these therapies Answer B is incorrect. Transthoracic echocar-
have little risk and may be appropriate in this diography (TTE) has limited use in the evalu-
FULL-LENGTH EXAMS

patient, the next logical step in a patient with ation of a thoracic aortic dissection. This
presumed status epilepticus would be securing imaging modality has a lower sensitivity and
an airway via intubation and the administra- specicity when compared to TEE, CT scan,
tion of medication capable of ending seizure or MRI. In addition, TTE is most frequently
activity. used to assess the proximal aortic root and to
conrm aortic insufciency, but it is limited in
Answer B is incorrect. Following a basic pri-
that it is difcult to visualize the distal ascend-
mary survey (airway, breathing, and circula-
ing, transverse, and descending aorta. Because
tion), the next step in management is the ad-
patient management often depends on the site
ministration of a benzodiazepine and a loading
of dissection entry and the extent of the aortic
dose. If intravenous benzodiazepines and phe-
dissection, TTE would not be the rst-line di-
nytoin are ineffective, phenobarbital may be
agnostic test ordered.
given intravenously. However, the patients air-
way is the more pressing issue. Answer C is incorrect. Upper endoscopy is not
indicated in diagnosing an aortic dissection or
Answer C is incorrect. The primary objective in
in determining specics about the dissection
this clinical scenario is to address airway, breath-
such as its extent. Although upper endoscopy
ing, and circulation. Since this patient is unable
may be useful when visualizing the esophagus
to protect his airway, it is imperative to intubate
Test Block 8

and stomach in the case of peptic ulcer disease


him prior to any other medical intervention.
or other upper gastrointestinal abnormality,
Answer D is incorrect. Complex studies such this patient most likely presents with an aortic
as electroencephalography and brain imaging, dissection and would benet little from upper
although they may be useful in establishing di- endoscopy.
Test Block 8 Answers 737

Answer D is incorrect. A ventilation-perfusion Answer D is incorrect. Human herpes virus


scan is used to diagnose or rule out a pulmo- rarely causes a heterophile antibody-negative
nary embolism and has no potential for diag- infectious mononucleosis. The syndrome is
nosis in cases of aortic dissection. usually quite mild.
Answer E is incorrect. X-ray of the abdomen is Answer E is incorrect. Toxoplasmosis is usu-
not indicated in the work-up of a thoracic aor- ally asymptomatic, although it is estimated to
tic dissection. Although further work-up may cause 1% of acute mononucleosis syndromes.
be indicated if the dissection is believed to ex- Heterophile antibody test would be negative.
tend to the abdominal aorta, this would not be It rarely causes pharyngitis, and rarely would
the most appropriate next step in diagnosis. In a peripheral blood smear show >10% atypical
addition, TEE and CT scan are more sensitive lymphocytes.
and specic for determining the extent of a dis-
section than an x-ray of the abdomen. 36. The correct answer is B. This case is one of a
conict of interest; that is, the physician has
35. The correct answer is B. The patient de- both a nancial interest and a patient care in-
scribed has infectious mononucleosis, the most terest. Patients have a right to full disclosure of
common cause (90% of cases) of which is Ep- these interests and of the physicians nancial
stein-Barr virus (EBV). Symptoms typically gain if they choose hospital A.
consist of malaise, fatigue, fever, headache,
Answer A is incorrect. This answer would be
sore throat, nausea, abdominal pain, and myal-
appropriate if the physician had evidence that
gias. Generalized lymphadenopathy is com-
his stent is better; however, given the current
mon (90% of cases), and epitrochlear lymph
lack of evidence and his own knowledge that
node enlargement is particularly suggestive of
outcomes at the two hospitals are similar, this
infectious mononucleosis. Splenomegaly is
is not a justication to refer solely to one hos-
found in 50% of cases. The diagnosis of infec-
pital.
tious mononucleosis is made by the character-
istic peripheral blood smear, with 60%70% Answer C is incorrect. This answer would be
lymphocytes and >10% atypical forms. The appropriate if the physician had evidence that

FULL-LENGTH EXAMS
heterophile antibody test is a sensitive, specic his stent is better; however, given the current
test that conrms EBV as the infectious agent. lack of evidence and his own knowledge that
outcomes at the two hospitals are similar, this
Answer A is incorrect. Cytomegalovirus is a
is not a justication to refer solely to one hos-
herpes virus that generally causes latent, as-
pital.
ymptomatic infection in immunocompetent
hosts. It is the second most common cause of Answer D is incorrect. Because outcomes at
infectious mononucleosis. Patients with cyto- the two hospitals are similar, the physician is
megalovirus mononucleosis will have a nega- justied in referring patients to either hospital.
tive heterophile antibody test. Although the They will receive good care at either one, and
clinical presentation can be similar, there he will have fullled his obligation in patient
are some differences: cytomegalovirus mono- care.
nucleosis rarely causes the lymphadenopathy, Answer E is incorrect. Although there is a con-
splenomegaly, and tonsillar exudates so often ict of interest inherent the physicians busi-
seen in EBV infection. ness and medical practice, he is allowed to do
Answer C is incorrect. Primary HIV infection both as long as he fully discloses his interests to
can cause a generalized viral syndrome similar his patients.
to EBV mononucleosis. However, the hetero-
Test Block 8

phile antibody test would be negative. Symp- 37. The correct answer is D. Gastric volvulus or
toms seen in acute HIV infection but not in incarceration is the feared complication of type
EBV include mucocutaneous ulceration, rash, II (paraesophageal) hiatal hernia, and while
and diarrhea. surgical repair of the asymptomatic patient is
still the standard of care, there are increasing
738 Section II: Full-Length Examinations Answers

data to suggest that nonoperative management Answer B is incorrect. Oral pilocarpine is used
might be preferable, as the incidence of gastric for the treatment of xerostomia caused by sali-
volvulus is lower than historically believed. vary gland hypofunction resulting from radio-
therapy for cancer of the head and neck, or for
Answer A is incorrect. Barretts esophagus, or
Sjgrens syndrome. Topical pilocarpine eye-
columnar metaplasia of the distal esophagus, is
drops, however, would be effective in lowering
a complication of long-standing gastroesopha-
the intraocular pressure, as it facilitates uid
geal reux disease, and can potentially lead to
drainage through the canal of Schlemm.
esophageal adenocarcinoma. As gastroesopha-
geal reux disease is not commonly seen in Answer D is incorrect. Atropine is helpful in
type II hiatal hernia, Barretts esophagus is not treating iritis and uveitis, not glaucoma. Atro-
a common complication of this type of hernia. pine is an antimuscarinic agent that results in
It is, however, commonly seen as a complica- mydriasis, which will further exacerbate her
tion of type I hiatal hernia. symptoms of closed-angle glaucoma (which
typically presents as this patient did) because
Answer B is incorrect. Gastric cancer has two
of its effects on the closure of the trabecular
forms, intestinal type and diffuse type. Intes-
meshwork around Schlemms canal, which
tinal type is associated with diet, Helicobacter
acts to drain the aqueous humor from the an-
pylori colonization, and chronic gastritis. The
terior chamber. Prevention of attacks with eye-
risk factors for diffuse type are unknown. There
drops is essential for the treatment of closed-
is no known association between gastric cancer
angle glaucoma. This drug should not be used
and type II (paraesophageal) hiatal hernia.
in patients with open-angle glaucoma either.
Answer C is incorrect. While there is a slightly Patients with open-angle glaucoma tend to
increased risk of gastric ulcer in patients suffer- have few symptoms, but may begin to notice
ing from type II (paraesophageal) hiatal hernia, blind spots in their eld of vision, especially in
the greatest risk is of gastric volvulus. the periphery. Although open-angle glaucoma
Answer E is incorrect. Gastroesophageal re- cannot be cured, patients can generally prevent
ux disease is a common complication of type the progression of symptoms with eyedrops.
I (sliding) hiatal hernia, but not of type II her- Answer E is incorrect. While steroids are in-
FULL-LENGTH EXAMS

nia. dicated for treatment of her anterior uveitis, it


also elevates intraocular pressure and will ex-
38. The correct answer is C. Immediate reduction acerbate her symptoms. She requires emergent
of intraocular pressure in acute (also called an- treatment for closed-angle glaucoma to prevent
gle-closure or narrow-angle) glaucoma is a permanent vision loss.
medical emergency in order to prevent blind-
ness. Reduction can be achieved by using a 39. The correct answer is D. By denition the pa-
carbonic anhydrase inhibitor such as acetazol- tient has stage 2 hypertension, with repeated
amide, which acts to decrease the production blood pressure measurements >160 mm Hg
of aqueous humor. In the past these agents systolic or >100 mm Hg diastolic. According to
were given by mouth as pills, but now they are the Joint National Committee on Prevention,
available as eyedrops. Other agents that also Detection, Evaluation, and Treatment of High
decrease aqueous humor inow are Blood Pressure, two-drug therapy is indicated.
-adrenergic blockers (timolol, betaxolol, and Diuretics are particularly effective in African-
levobunolol) and -agonists (apraclonidine and American patients and are therefore appropri-
brimonidine). ate in combination with an angiotensin-
converting enzyme inhibitor, angiotensin
Answer A is incorrect. This treatment is indi-
Test Block 8

receptor blocker, -blocker, or calcium chan-


cated only after medical reduction of intraocu-
nel blocker. Contraindications or compelling
lar pressure. An iridotomy is a procedure that
indications also guide drug choice.
uses lasers to open a new channel in the iris.
This new channel then relieves pressure and Answer A is incorrect. Lifestyle modication is
prevents another attack. appropriate for patients who are prehyperten-
Test Block 8 Answers 739

sive, with a systolic blood pressure between 120 HIV syndrome varies but may include fevers,
and 139 mm Hg and a diastolic blood pressure lymphadenopathy, arthralgias, nausea, vomit-
between 80 and 89 mm Hg. ing, encephalitis, and an erythematous macu-
lopapular rash.
Answer B is incorrect. This combination of
medications does not include a diuretic, which Answer E is incorrect. The rash of nummular
are particularly effective in African-American eczema appears as closely-grouped vesicles and
patients. papules that coalesce into large plaques. The
plaques are often crusted and excoriated sec-
Answer C is incorrect. Hydrochlorothiazide
ondary to scratching. Nummular eczema is a
is an appropriate choice for this patient. How-
chronic inammatory dermatitis often seen on
ever, given the severity of his hypertension, two
the lower legs during cold, dry seasons.
drugs are recommended.
Answer F is incorrect. The rash of pityriasis
Answer E is incorrect. Lisinopril in isolation is
rosea is erythematous and pruritic with central
inadequate treatment of stage 2 hypertension.
clearing and peripheral scaling. The distribu-
tion is typically on the trunk and begins with a
Questions 40 and 41 herald patch and then spreads downward and
peripherally but rarely spreads as far as the dis-
40. The correct answer is B. Erythema migrans is tal extremities.
usually due to Lyme disease. It is classically de-
scribed as an erythematous oval lesion with Answer G is incorrect. Secondary syphilis af-
central clearing, which may be confused with fects the palms and soles and is associated with
the herald patch of pityriasis rosea. The classic systemic symptoms.
bulls-eye occurs in only a minority of cases.
Erythema migrans may be pruritic. The lesion
Questions 42, 43, and 44
typically occurs near the tick bite in a warm
area of the body. Systemic symptoms may in- 42. The correct answer is A. Placental abruption
clude fever and chills in approximately two- classically presents with bright red vaginal
thirds of patients, as well as headache, arthral- bleeding and uterine pain and typically occurs

FULL-LENGTH EXAMS
gias, myalgias, diffuse lymphadenopathy, and during the third trimester. If the abruption is
meningitis. signicant, there may also be fetal distress as
indicated by the fetal tachycardia and hyper-
41. The correct answer is H. Tinea corporis, or tension as in this scenario.
ringworm, is a dermatophyte infection com-
monly seen in children or their close contacts. 43. The correct answer is L. In contrast to abrup-
The lesion is erythematous, scaled, and papu- tion, placenta previa presents with painless
lar. As it expands there is central clearing. vaginal bleeding without notable uterine con-
Tinea corporis is often very pruritic. tractions or irritability.
Answer A is incorrect. Cutaneous drug reac-
44. The correct answer is J. Normal labor can oc-
tions vary in appearance but most often mani-
cur spontaneously between 37 and 42 weeks
fest as a diffuse morbilliform or urticarial rash.
gestational age and has as its hallmark regular
The rash remits when the offending drug is dis-
uterine contractions with progressive cervical
continued.
effacement and dilatation. The loss of the cer-
Answer C is incorrect. Guttate psoriasis causes vical mucus plug may be accompanied by a
a scaling rash with multiple small well-dened small amount of vaginal bleeding.
papules with a tendency to coalesce.
Test Block 8

Answer B is incorrect. Bacterial vaginitis pre-


Answer D is incorrect. Approximately 50% sents with thin white or gray vaginal discharge
70% of HIV-infected patients experience that may have an unpleasant odor. Vaginal
acute HIV syndrome, which is associated with pain or vulvar irritation is atypical.
a rapid increase in plasma viremia. Acute
740 Section II: Full-Length Examinations Answers

Answer C is incorrect. Thick, odorless, Answer N is incorrect. Trichomonas infection


clumpy white (cottage cheese) discharge can be asymptomatic, or patients may present
along with pruritus that may be intense are with yellow or green vaginal discharge that is
commonly seen with vaginal candidiasis. usually profuse.
Answer D is incorrect. With routine screen- Answer O is incorrect. Patients with vesicova-
ing, cervical cancer is commonly found in ginal stula present with leakage of urine from
its asymptomatic stages via an abnormal Pap the vagina that may be continuous or intermit-
smear result. If advanced, disease may present tent. The patient may complain of increased
with abnormal vaginal bleeding, vaginal pain, vaginal discharge following pelvic radiation or
dyspareunia, and abnormal vaginal discharge. pelvic surgery.
Answer E is incorrect. Unilateral pelvic pain, Answer P is incorrect. Vulvar carcinoma may
vaginal bleeding, and a low rate of increase in present as a mass or, more commonly, as vul-
the -human chorionic gonadotropin level for var pruritus. Advanced vulvar cancer may be
stage of pregnancy are seen in ectopic preg- accompanied by bleeding, pain, and discharge.
nancy.
Answer F is incorrect. Most patients with en- Questions 45 and 46
dometrial carcinoma are postmenopausal,
and postmenopausal bleeding is the herald- 45. The correct answer is B. The bone marrow as-
ing symptom. In perimenopausal patients, ir- pirate is pathognomonic for acute myeloge-
regular or abnormally heavy bleeding patterns nous leukemia (AML) due to the presence of
should raise suspicion for endometrial carci- Auer rods. Auer rods, seen in the cytoplasm of
noma. leukemic cells, are eosinophilic needle-like in-
clusions that are not always seen in AML but
Answer G is incorrect. The most common that are pathognomonic for myeloid disease.
symptoms of a vaginal foreign body are mal- Other characteristics of AML cells include a
odorous discharge and vaginal bleeding. Vagi- high nuclear:cytoplasmic ratio and open nu-
nal pain or dysuria may also be present. clear chromatin, as seen in the bone marrow
FULL-LENGTH EXAMS

Answer H is incorrect. Many women with aspirate. AML accounts for approximately 20%
gonococcal infection are asymptomatic. Pa- of childhood leukemias. The presentation can
tients with symptoms may present with vaginal be similar to that of acute lymphocytic leuke-
discharge, dyspareunia, and pelvic pain. mia, and bone marrow aspirate may be re-
quired for diagnosis. When a diagnosis of AML
Answer I is incorrect. Symptoms of mo- is suspected, histochemical stains demonstrat-
lar pregnancy may include size greater than ing myeloid enzymes (e.g., peroxidase) may be
dates, vaginal bleeding, and an unusually helpful for diagnosis.
high -human chorionic gonadotropin level
for stage of pregnancy. Snowstorm on ultra- 46. The correct answer is J. Multiple myeloma is
sound and the vaginal passage of small bits of a malignancy of plasma cells. As the atypical
hydatidiform tissue may also be seen. malignant cells ll the bone marrow, normal
Answer K is incorrect. In addition to regular plasma cells and other cell lines are not pro-
bleeding, normal menses may present with duced adequately. The result is an increased
mild to moderate lower abdominal pain, mild susceptibility to infection, anemia, and easy
nausea, loss of appetite, bloating, and mood bruising. Most patients with newly diagnosed
symptoms. multiple myeloma are >50 years old but may
be as young as 25 years.
Test Block 8

Answer M is incorrect. Vaginal bleeding or


spotting in the presence of pregnancy and a Answer A is incorrect. Acute lymphocytic leu-
closed cervix is seen with threatened abortion. kemia is characterized by proliferation of im-
mature lymphocytes and is common in chil-
dren.
Test Block 8 Answers 741

Answer C is incorrect. Burkitts lymphoma is Answer H is incorrect. Hyper IgE syndrome is


caused by an 8:14 translocation and is a B lym- an immunodeciency characterized by aller-
phocyte lymphoproliferative disorder. gies and increased susceptibility to staphylo-
coccal infection.
Answer D is incorrect. Chronic lymphocytic
leukemia is characterized by proliferation of Answer I is incorrect. Hyper IgM syndrome is
nearly mature lymphocytes and is common in caused by a deciency in B lymphocyte class
persons who are elderly. switching due to the loss of CD40/CD40L in-
teractions.
Answer E is incorrect. Chronic myelogenous
leukemia is characterized by proliferation of Answer K is incorrect. Nodular sclerosing lym-
nearly mature myeloid cells without eosino- phoma is a brosing form of Hodgkins lym-
philic inclusions and is common in persons phoma.
who are elderly.
Answer L is incorrect. Osteomyelitis is caused
Answer F is incorrect. Common variable im- by bacteremia seeding to the bones and may
munodeciency is an immunodeciency of cause lytic lesions.
humoral immunity that presents insidiously in
Answer M is incorrect. Posttransplant lym-
persons who are elderly.
phoproliferative disease occurs in transplant
Answer G is incorrect. Hodgkins lymphoma recipients, likely secondary to immunosuppres-
is characterized by Reed-Sternberg cells and sion.
mediastinal lymphadenopathy.

FULL-LENGTH EXAMS
Test Block 8
This page intentionally left blank
ABOUT THE SENIOR EDITORS

Tao Le, MD, MHS Kristen Vierregger, MD

Tao Le, MD, MHS Tao has been a well-recognized gure in medical education for the past 15 years. As senior editor, he
led the expansion of First Aid into a global educational series. In addition, he is the founder and
editor-in-chief of the USMLERx online test bank series as well as a cofounder of the Underground
Clinical Vignettes series. As a medical student, he was editor-in-chief of the University of California,
San Francisco Synapse, a university newspaper with a weekly circulation of 9000. Tao earned his
medical degree from the University of California, San Francisco, in 1996 and completed his residency
training in internal medicine at Yale University and fellowship training at Johns Hopkins University in
allergy and immunology. In addition, he earned a Master of Health Science degree at the Johns
Hopkins Bloomberg School of Public Health. At Yale, he was a regular guest lecturer on the USMLE
review courses and an adviser to the Yale University School of Medicine curriculum committee. He is
currently chief of allergy and clinical immunology in the Department of Medicine at the University of
Louisville.
Kristen Vierregger, MD Kristen is a resident in pathology at the University of California, Irvine. She graduated from medical
school at the University of Pennsylvania and started working for First Aid as an author during her sec-
ond year there. She became an editor for USMLERx Step 2 CK the following year, and has now en-
joyed the position of senior editor for the project. If Kristen werent a doctor, she would be a profes-
sional poker player.

ABOUT THE EDITORS

Herman Singh Bagga, MD Herman is a resident in urology at the University of California, San Francisco. He hails from New York
City (which is where he gets his rough and tough exterior); however, he was raised mostly in the
small city of Erie, Pennsylvania (where he gained his soft, sensitive side). He began working with the
First Aid and USMLERx teams during his third year at Johns Hopkins University School of Medicine
and wishes all students great success as they utilize these materials to advance in their medical ca-
reers.
Thomas L.H. Hocker, MD Thomas graduated with honors from Harvard Medical School and served his transitional internship
year at Santa Clara Valley Medical Center in the beautiful California Bay Area. He is now starting his
dermatology residency at the Mayo Clinic. He has had the honor of contributing as both an author
and editor to the USMLERx Step 2 CK website. Thomas's hobbies include martial arts, running, and
socializing with people from all walks of life. He intends to pursue a research career in dermatology
and hopes to continue working on medical educational tools.
Christopher R. Kinsella, Jr., MD Topher was born and raised in St. Louis and graduated from Drexel University College of Medicine in
2009 after spending 2 years in the Caribbean at St. George's University School of Medicine. He is currently
a postdoctoral associate in the division of plastic surgery at the University of Pittsburgh School of Medicine.
Current nonacademic goals include visiting all seven continents (Australia and Antarctica to go), learning
how to hockey stop, and driving a stick shift. He is forever grateful for the support of his family and friends,
considers it an honor to work with the First Aid team, and hopes it helps.
Matthew O'Rourke, MD Matthew is a resident in pediatrics at the Morgan Stanley Children's Hospital of New York-Presbyte-
rian. When not working for First Aid and USMLERx, Matt enjoys running and reading National Geo-
graphic.
John Rhyner, MD John is an internal medicine resident at the University of Michigan. He graduated from Duke Univer-
sity School of Medicine in 2008, where he was a Eugene A. Stead scholar and received honors awards
for his research in the genomics of cardiothoracic anesthesia and cardiac electrophysiology. He earned
a Bachelor of Arts degree with honors in genetics and molecular biology from Northwestern Univer-
sity. John enjoys reading, cooking, and the visual arts. He plans to pursue a cardiology fellowship and
academic practice.
Sade Clarke Udoetuk, MD Sade is a resident at the Baylor College of Medicine psychiatry program. She has worked with both
the USMLERx Step 1 and Step 2 CK projects as an author and an editor. Sade is very interested in aca-
demic medicine and hopes to pursue a fellowship in psycho-oncology. Her husband, Joshua, is also
an author for USMLERx Step 2 CK, and she strongly suspects their daughter, Jaclyn, may contribute to
the project at a future date as well.

ABOUT THE AUTHORS


Cynthia Adams, MD
Cynthia is a resident at the Boston Combined Residency Program in Pediatrics at Children's Hospital Boston. She began working on USMLERx while she was a fourth-year medical
student at the University of Pennsylvania. In her free time Cynthia enjoys being outdoors, exercising, exploring New England, and spending time with her family.
Clarissa Barnes, MD
Clarissa is a resident in internal medicine at Johns Hopkins University School of Medicine. She is from South Dakota and is proud to bring the "midwest work ethic" to the East
Coast. She began working on USMLERx while she was a fourth-year medical student at Johns Hopkins. During her limited free time she enjoys cooking with her husband for her
family and friends.
Rachel Bortnick, MPhil
Rachel is a fth-year student in the Medical Scientist Training Program at Harvard Medical School. She graduated from the University of Chicago with Bachelor of Arts degrees in bi-
ology and philosophy and from the University of Cambridge with a Master of Philosophy degree in neuroscience. Her current research focuses on neurodevelopment, and in the
future she hopes to combine her interests in neuroscience, medicine, writing, and global health.
Christopher Chapman, MD
Chris is a resident in internal medicine at the University of Chicago. He was born and raised in a small northern suburb of the Windy City known as Lake Villa. Upon realizing his
dream of a baseball career would not pan out, he decided to pursue medicine at Johns Hopkins School of Medicine. It was there as a fourth-year medical student that his work
with USMLERx began. In his spare time Chris enjoys spending time with his wife, eating deep-dish pizza, and watching baseball on the south side.
Lia Clattenburg, MD, MPH
Lia is a resident in anesthesiology at John Hopkins School of Medicine. During her fourth year of medical school at Johns Hopkins, she pursued many adventures (gardening, cy-
cling, hiking, travel), including being an author for USMLERx. She also enjoys maintaining her rooftop garden.
Gillian Diercks
Gillian is nishing her nal year of medical school at Columbia College of Physicians and Surgeons and will be applying for residency in otolaryngologyhead and neck surgery. She
began working on USMLERx while taking a year off as a Doris Duke Clinical Research Fellow.
Vahid Entezari, MD
Vahid is a research fellow in the orthopedic biomechanics laboratory of the Beth Israel Deaconess Medical Center in Boston. He joined the USMLERx team in 2007 and learned a
lot about answering questions from writing many. His career goal is to become an orthopedic surgeon and practice as a clinician scientist in an academic institute. He, his wife Sara
(who is also a doctor), and their daughter enjoy spending their free time together.
Erica Y. Fan
Erica is a fourth-year medical student at St. Louis University School of Medicine. She began writing for USMLERx during her third year of medical school. Erica earned her Bachelor
of Arts degree in biological anthropology from Northwestern University, followed by a Master of Science degree in forensic molecular biology from George Washington University.
She enjoys running, shopping, amateur photography, traveling, and Photo Hunt, and intends to pursue a career in pathology.
Carl Erik Fisher
Carl is a fourth-year student at Columbia University College of Physicians and Surgeons. In 20072008 he was a Doris Duke Clinical Research Fellow in the psychiatry department
at Columbia University/New York State Psychiatric Institute, where he studied brain stimulation, neuromodulation, and neuroethics. Carl has been involved in academic medicine
as well, having served as part of the national leadership of the American Medical Student Association and on the board of directors of the National Residency Matching Program. At
Columbia he has been a student coinstructor of a number of peer-led courses.
Sattar Gojraty, MD
Sattar is a resident in internal medicine at the Hospital of the University of Pennsylvania. He is originally from Wisconsin and visits there every chance he gets. When not writing
questions for USMLERx, Sattar can be found training for his rst marathon, visiting local coffee shops, and "slaving" in the hospital.
Nilay Kavathia, MD
Nilay is an internal medicine resident at Thomas Jefferson University Hospital. He is originally from the great state of Arizona. He graduated from the University of Arizona College
of Medicine, where he began working on USMLERx as a fourth-year student. Nilay is a big fan of anything that is funny, and hopes to bring some of that ridiculousness to Jefferson
as an intern. One of his goals for residency is to write a history and physical in iambic pentameter. He has many other nonmedical interests such as sports, politics, and, of course,
comedy. Nilay wishes the best for future medical students.
Laura Meints
Laura is a fth-year student in a combined MD/MBA program at Vanderbilt University School of Medicine and will soon be applying for a residency in obstetrics and gynecology.
She began working on USMLERx projects when she was a third-year medical student. In her free time Laura enjoys art and photography, volunteer activities in Nashville, hiking the
nearby hills and back areas of middle Tennessee, and cooking.
Robert Micheletti, MD
Robert is a resident in the dual internal medicine and dermatology program at the Hospital of the University of Pennsylvania. He graduated from Duke University School of Medi-
cine in 2008, where he won the Cavanagh Award for academic excellence. During medical school he presented research on atherosclerosis in HIV-infected patients. In his spare
time Robert enjoys cycling, running, camping, and spending time with his wife.
Daniel L. Miller
Daniel is pursuing a Master of Public Heath degree with a concentration in epidemiologic and biostatistical methods for public health and clinical research at Johns Hopkins
Bloomberg School of Public Health. He is originally from Parker, South Dakota, and began working on USMLERx while a fourth-year medical student at Johns Hopkins. During his
free time he enjoys snowboarding as well as running 10K and marathon road races.
Deepika Nemani
Deepika is a fourth-year medical student at the University of Pennsylvania School of Medicine. She began working on USMLERx last year. Deepika is currently enjoying her "free
time" before starting internship next year.
Timothy Niessen
Originally from rural Pennsylvania, Tim is currently pursuing a Master of Public Health degree in biostatistics and epidemiology at Johns Hopkins Bloomberg School of Public Health
with a focus on preventable medical errors. He started writing questions for USMLERx during his fourth year of medical school. In his free time Tim tends his garden and enjoys
running and playing with his daughter Zoe. He is looking forward to a residency in internal medicine.
Howard ORourke, MD
Howard is a resident in diagnostic radiology at the University of Pittsburgh Medical Center. He graduated from the University of Rochester School of Medicine & Dentistry in 2006.
Rohith Piyaratna, MD
Rohith grew up in Sri Lanka and moved to the United States when he was 18 years old to attend college. He completed his undergraduate education at Macalester College in St.
Paul, Minnesota. Rohith earned his medical degree from Vanderbilt University School of Medicine. Currently he is a resident in the Department of Anesthesiology at Stanford Uni-
versity. In his spare time he enjoys weight lifting, reading, and falling asleep on the couch when not working.
Anthony Prince, MD
Anthony is a surgical resident in the Department of Otolaryngology at the Mount Sinai Medical Center in New York City. He began working on First Aid and USMLERx while in his
nal year at the University of Pennsylvania School of Medicine. In his free time Anthony enjoys long-distance running, golng, spending time with friends and family, and traveling
outside the United States.
Fiorella Saponara, MD
Fiorella is an intern at Maimonides Medical Center in New York City, and next year will be heading to the University of Michigan to begin her residency in ophthalmology. She be-
gan working on First Aid and USMLERx projects during her fourth year of medical school at the University of Pennsylvania School of Medicine. In her free time Fiorella enjoys travel-
ing abroad, cooking, spending time outdoors, and visiting with her family.
Asha Jayendrakumar Shah, MD
Asha is a resident in the department of internal medicine at Emory University Medical Center and plans to pursue a career in primary care medicine. She began working on
USMLERx while she was a fourth-year medical student at the University of Pennsylvania School of Medicine. In her spare time she enjoys traveling, reading, listening to pop music,
watching movies, salsa dancing, baking, and spending time with friends and family.
Monica E. Shukla
Monica is a fourth-year medical student at Vanderbilt University School of Medicine. She rst contributed to USMLERx Step 2 CK as an author during her third year in medical
school. She is currently working on USMLERx Step 1 as an editor, heading up a team of authors at Vanderbilt. Monica hopes to continue her involvement in medical education dur-
ing her residency training. In her free time she enjoys cooking, taking long walks around Nashville, hanging out with family and friends, and planning her next vacation to . . . some-
where tropical.
Meghan Sise
Meghan is a fourth-year medical student at Columbia University College of Physicians and Surgeons. She is originally from Loudonville, New York, and a proud graduate of Shaker
High School and the University of Notre Dame. She began working for USMLERx during a year as a Doris Duke Clinical Research Fellow and plans on pursuing residency in internal
medicine.
D'Mitri Soanos, MD
D'Mitri is a resident in the orthopedic surgery program at the University of Utah. He began working on USMLERx while a fourth-year medical student at Vanderbilt University School
of Medicine, where he was a tutor for undergraduates interested in a career in medicine. His research interests include perioperative infection in orthopedic surgery and the role of
statins in sepsis. Also interested in international health care, he has traveled the remote outback of Australia studying the deadly infection melioidosis. D'Mitri is looking forward to
enjoying the great outdoors of Utah as soon as he gets a day off.
Anna E. Teeter
Anna is a fourth-year medical student at Duke University School of Medicine. She was an author for USMLERx during her third year of medical school. A varsity swimmer in college,
Anna still enjoys swimming as part of the Duke Club Swimming Team. She also enjoys working with Habitat for Humanity in Durham, North Carolina, and generally being outdoors
as much as the steamy North Carolina weather allows. She plans to pursue a residency in urology.
Joshua D. Udoetuk, MD
Joshua is a transitional resident at the University of Texas-Houston Medical School, after which he will be entering an ophthalmology residency at Baylor College of Medicine. He
and his wife Sade began working on USMLERx projects while they were medical students at the University of Pennsylvania. In his free time Joshua enjoys spending time with his
family (especially their daughter Jaclyn), photography, and playing sports.
Brant W. Ullery, MD
Brant recently began his general surgery residency at the Hospital of the University of Pennsylvania. He has thoroughly enjoyed contributing to both the USMLERx Step 1 and the
Step 2 CK projects during his medical school career at Weill Medical College of Cornell University. His hobbies include cooking, wine tasting, working out, and mindless strolls
throughout Center City. Brant intends to pursue a fellowship in cardiothoracic surgery.
Kelly Vranas, MD
Kelly is an intern in internal medicine at the Hospital of the University of Pennsylvania. She began working for USMLERx as a third-year medical student at Weill Medical College of
Cornell University. She also participated in a medical elective at the Hospital of the University of Vienna during her fourth year of medical school. Kelly plans on going into either
gastroenterology or critical care medicine. She enjoys running, playing volleyball and softball, reading mystery novels, cooking, and taking bike rides along the banks of the
Schuylkill River.
David Wei
David is in his last year of medical school at Columbia University College of Physicians and Surgeons. He started working on USMLERx when he began a one-year research fellow-
ship between his third and fourth years of medical school. He will be applying for residency in orthopedics.
Zachary Zavodni
Zack is a fourth-year medical student at Duke University School of Medicine. He began working with USMLERx during his third year of medical school. Zack has received awards
from the Research to Prevent Blindness organization and the National Eye Institute for his research on age-related macular degeneration. He graduated summa cum laude from
Northwestern University, where he earned degrees in chemistry and economics. Zack enjoys playing tennis, traveling, and spending time outdoors. He plans to pursue a career in
ophthalmology.
ABOUT THE ASSOCIATE AUTHORS
Marina Frimer, MD
Marina is a resident in obstetrics and gynecology at the Albert Einstein College of Medicine. She contributed to both the USMLERx Step 1 and Step 2 CK projects. Marina graduated
from Stony Brook School of Medicine in 2007. She plans to go into academic medicine while working to improve the health of women. In her spare time Marina enjoys traveling
and spending time with family.
Mark J. Mann, MD
Mark is a resident in urology at the State University of New York Upstate Medical University in Syracuse, now in his rst postgraduate year of training. He rst became involved in
USMLERx as a fth-year medical student, after taking a year off to complete a Doris Duke Clinical Research Fellowship. Mark is an avid New York sports fan and enjoys running
when he isn't spending time with family and friends.
Parin J. Patel, MD
Parin is an intern at the Hospital of the University of Pennsylvania, where he is training in internal medicine. Long interested in medical education, he was a peer tutor at Harvard
Medical School during his rst two years, and conducted various research studies on standardized testing and markers of progression in medical school during his fourth year. He
began working with USMLERx in his last year of medical school. Also interested in international health care, Parin has studied medicine in Oxford, UK, and at La Universidad Fran-
cisco Marroquin in Guatemala. Parin enjoys the slightly-warmer-than-Boston weather in Philadelphia by going for long bike rides and playing any sport that he meets, including
lawn bowling.
Sunil Sheth
Sunil is a fourth-year student at Harvard Medical School and plans to apply for a residency in neurology. Originally from Texas, he has spent the past nine years in school on the
East Coast. When he is not in the hospital or the lab, Sunil enjoys playing and watching soccer, and occasionally travels the world to do so. He intends to pursue a career combin-
ing research and clinical medicine.
Tian Zhang
Tian is a fourth-year medical student in the Health Sciences and Technology division of Harvard Medical School; she plans to apply for residency in internal medicine and to spe-
cialize in medical oncology. Tian contributed to USMLERx during her third year of medical school. In her spare time she enjoys reading and cooking, as well as running and biking
with her anc along the Charles River.
-

Вам также может понравиться